You are on page 1of 1415

Introduction

This E-Book of CDS Previous Year Questions is designed by the Experts from

BYJU’S Exam Prep for all the candidates to practice who are appearing for

the CDS Exam. You will have to look no further to find the Practice Questions

for your upcoming CDS Exam as this E-book contains all in one, with

detailed solutions. BYJU’s Exam Prep’s Previous Year Questions E-Book is

designed keeping in mind the difficulty level of the CDS Exam and the

Importance played by the Previous Year Questions during the Preparation

of the Exam. The Previous Year Papers also helps the candidate in

familiarizing with the Exam Pattern and also understand Various Regions

of the Syllabus. This E-book Offers You Previous Year Question Papers with

Answer Key and their Detailed Solution. Not only this, but you can also

improve your Question solving time by practicing the questions given in this

book. The Experts have kept in mind the importance of the Previous Year

Questions for the Candidates while preparing this E-book. So, Start

Practicing and ACE your CDS Exam with BYJU’s Exam Prep.

#DreamStriveSucceed
Table of Contents
1. CDS II 2021 4-37
✓ Answer Key 38-39
✓ Solution 40-104
2. CDS I 2021 105-139
✓ Answer Key 140-141
✓ Solution 142-203
3. CDS II 2020 204-237
✓ Answer Key 238-239
✓ Solution 241-312
4. CDS I 2020 313-350
✓ Answer Key 351-352
✓ Solution 353-434
5. CDS II 2019 435-473
✓ Answer Key 474-475
✓ Solution 476-549
6. CDS I 2019 550-591
✓ Answer Key 592-593
✓ Solution 594-659
7. CDS II 2018 660-703
✓ Answer Key 704-705
✓ Solution 707-783
8. CDS I 2018 784-825
✓ Answer Key 826-827
✓ Solution 828-899
9. CDS II 2017 900-950
✓ Answer Key 951-952
✓ Solution 953-1026
10. CDS I 2017 1027-1071
✓ Answer Key 1072-1073
✓ Solution 1074-1156
11. CDS II 2016 1157-1197
✓ Answer Key 1198-1199
✓ Solution 1201-1283
12. CDS I 2016 1284-1329
✓ Answer Key 1330-1331
✓ Solution 1332-1414
www.byjusexamprep.com

CDS II 2021
ENGLISH
Direction: Each of the following sentences in 7. bags. It is _________ duty of the
this section has a blank space with four words Government to look into this matter;
or group of words given. Select whichever word
A. but also B. still
or group of words you consider the most
appropriate for the blank space and indicate C. the D. yet
your response on the answer sheet accordingly. 8. however, it is also the duty of each
1. The polythene bags _________ non- individual to ____________
biodegradable, i.e., they cannot
A. get it
A. are B. become
B. do it
C. is D. were
C. convene it
2. be decomposed ___________ micro-
organisms into manure. They D. see to it

A. by B. through 9. that we ____________ use polythene bags.


By using these we will
C. into D. in
A. didn't B. weren’t
3. remain as _________ even after years.
Animals that started C. don’t D. aren’t
A. they are B. it is 10. not only harm the environment but
C. even D. after ourselves as well. Thus, say ‘No’ to plastic
bags and contribute ___________ society.
4. eating waste food ___________ with these
polythene bags, ultimately A. by B. to

A. besides B. beside C. into D. in


C. thorough D. along Direction: Each of the following sentences in this
section has a blank space with four options.
5. started dying __________ their internal
Select whichever preposition or determiner you
system was getting blocked.
consider the most appropriate for the blank
A. of B. though space and indicate your response on the answer
C. because D. for sheet accordingly.

6. The Government had no alternative 11. He succeeded _________ dint of


___________ to ban these polythene perseverance and hard work.

A. yet B. but A. by B. for

C. so D. because C. on D. upon

PAGE 4
www.byjusexamprep.com

12. Due to his illness, he could not finish the Direction: The following items have one part of
work __________ time. the sentence followed by four alternatives.
Complete the sentences by choosing the correct
A. by B. for
alternative.
C. on D. upon
21. I asked two people the way to the station
13. There is no meaning ____________ what
A. but neither of them could help me
you say.
B. however none of them could show me
A. by B. for
the way
C. in D. on C. yet they had doubts about the station
14. These are good rules _______________ by. D. nevertheless they couldn’t guide me
A. with B. to 22. The room was very warm
C. in D. on A. because the AC was set to moderate
15. He is the man I have been looking temperature
__________. B. though the AC was set to very cold
A. at B. for C. since the AC was not very warm in its
C. in D. on temperature setting

16. Don’t loiter ___________ the street. D. since the AC was in a very low
temperature setting
A. near B. around
23. We could leave today, or we could leave
C. in D. on tomorrow
17. Sit here ___________ me. A. preferring what you want
A. by B. beside B. as you prefer one of the two
C. in D. on C. depending on what you prefer
18. We mustn’t shy _______________ entry- D. whichever you prefer
level or freelance jobs as they help us gain
an insight into the context in which a 24. Mohan and I couldn’t get into the house
company operates. because

A. in B. with A. neither of us had the keys


B. both of us do not have the keys
C. away from D. upon
C. either of us did not have the keys
19. Civil society and media have a major role in
making the coalition work ___________ the D. neither of us did have the keys
people.
25. There are many good hotels, you can
A. for B. about choose to stay in
C. with D. over A. many of them
20. He travelled ____________ Mr. Joshi’s car. B. any one of them
A. by B. for C. either of them
C. in D. on D. all of them

PAGE 5
www.byjusexamprep.com

26. The bus service is very good; there is a bus previous winter. With human beings the matter
is otherwise. I doubt whether the percentage of
A. after ten minutes
birds that will have died of hunger during the
B. in ten minutes present winter (1946-47) is as great as the
percentage of human beings that will have died
C. before ten minutes
from this cause in India and central Europe
D. every ten minutes during the same period. But every human
27. We live near a busy airport; the planes fly death by starvation is preceded by a long period
of anxiety and surrounded by the corresponding
A. near our house anxiety of neighbours. We suffer not only the
B. by our house evils that actually befall us, but all those that our
intelligence tells us we have reason to fear. The
C. over our house curbing of impulses to which we are led by
D. around our house forethought averts physical disaster at the cost
of worry, and general lack of joy. I do not think
28. Dan was very quiet. He didn't say a word that the learned men of my acquaintance, even
A. all the evening when they enjoy a secure income, are as happy
as the mice that eat the crumbs from their tables
B. the entire evening while the erudite gentlemen snooze. In this
C. all the entire evening respect, therefore, I am not convinced that there
has been any progress at all.
D. entire evening
31. The birds die of hunger in winter because
29. I don't like stories
A. they do not move to warmer places
A. which can have unhappy endings
B. people do not feed them
B. which had sad endings
C. they do not get the food of their choice
C. that have unhappy endings
D. they are too young to get the food
D. which are unhappy endings
32. The birds do not foresee the catastrophe
30. Not everything
because they
A. that happened was my fault
A. cannot predict an accident
B. which happen was my fault
B. overlook a difficult situation
C. what happened was my fault
C. cannot expect a sudden disaster
D. whatever happened was my fault
D. ignore the problems
Direction: In this section you have two short
passages. After each passage, you will find some Direction: In this section you have two short
items based on the passage. Read the passage passages. After each passage, you will find some
and answer the items based on it. items based on the passage. Read the passage
and answer the items based on it.
Passage - I
Passage - I
As to happiness, I am not so sure. Birds, it is true,
die of hunger in large numbers during the winter, As to happiness, I am not so sure. Birds, it is true,
if they are not birds of passage. But during the die of hunger in large numbers during the winter,
summer they do not foresee this catastrophe or if they are not birds of passage. But during the
remember how nearly it befell them in the summer they do not foresee this catastrophe or

PAGE 6
www.byjusexamprep.com

remember how nearly it befell them in the Direction: More than eight months after the
previous winter. With human beings the matter national lockdown was announced in late March,
is otherwise. I doubt whether the percentage of urban India is learning to live with the Covid-19
birds that will have died of hunger during the pandemic. In fact, indicating a positive outlook
present winter (1946-47) is as great as the for the future, many survey respondents in a
percentage of human beings that will have died recent survey say they plan to return to pre-
from this cause in India and central Europe lockdown levels of shopping, personal grooming,
during the same period. But every human death going to cinemas and socializing as pandemic
by starvation is preceded by a long period of fears continues to recede.
anxiety and surrounded by the corresponding
The survey findings highlight that the
anxiety of neighbours. We suffer not only the
suppression of consumer demand because of
evils that actually befall us, but all those that our
fears of job losses and salary cuts could be
intelligence tells us we have reason to fear. The
coming to an end. Increasingly, urban Indians are
curbing of impulses to which we are led by
showing increased confidence about the future
forethought averts physical disaster at the cost
of the economy. This could be an indicator of the
of worry, and general lack of joy. I do not think
possible 'pent-up demand' that several
that the learned men of my acquaintance, even
economists have been talking about - a demand
when they enjoy a secure income, are as happy
that could be unleashed once a vaccine is
as the mice that eat the crumbs from their tables
developed and distributed, or when there are
while the erudite gentlemen snooze. In this
signs of the pandemic's spread reducing to
respect, therefore, I am not convinced that there
negligible levels or vanishing totally. Around 65
has been any progress at all.
per cent of respondents said they had settled
33. Human beings cannot be happy because into new routines, or that they saw signs of the
they situation improving, or they had come to terms
with the pandemic and were moving on with
A. do not get time to enjoy
their lives. And since the survey was conducted
B. worry too much about their work before the news of the successful trials of the
Pfizer vaccine for the coronavirus was
C. are not healthy
announced, it is likely that the consumers are
D. worry too much about future now even more positive in their outlook about
the future.
34. Which one of the following is the antonym
of the word 'erudite' in the passage? 36. The general tone of the passage is that of
A. Qualified A. Optimism B. Pessimism
B. III-educated C. Fatalism D. Defeatism.
C. Logical 37. The willingness of consumers to go back to
normal lifestyle indicates their
D. Learned
A. Casual attitude
35. Which one of the following is the central
theme of the passage? B. Change of moods
A. Life of the birds and the mice C. Desire for future plans
B. Starvation in India and central Europe D. Sense of economic security.
C. Progress of mankind 38. Which statement in the passage/phrase
indicates that 'Life must go on'?
D. Disasters in 1946-47

PAGE 7
www.byjusexamprep.com

A. They settled into their new, routines 44. The film I saw was hilarious.
B. Salary cuts could be coming to an end A. tragic B. serious
C. People are moving on with their lives C. uproarious D. jovial.
D. 'Pent-up demand' 45. On that day, pandemonium reigned in the
hall.
39. What does the author mean by 'pent-up
demand'? A. hullaballoo B. uproar
A. Desire (suppressed) to spend money C. peace D. accolade
once pandemic is controlled
46. The police detained me today amidst busy
B. Economic normalcy of consumers traffic.
C. Flamboyant shopping by consumers A. impeded B. released
D. Criticism of the public on the pay-cuts C. confined D. held.
40. What, according to the author, is the reason 47. In my state of despair, I confessed
behind suppression of consumer demands? everything.
A. The lack of vaccine till date A. despondency B. determination
B. Professional insecurity C. dependant D. elation
C. Motive to save up some money 48. He was accused by the entire community
D. Careful and calculative attitude of the after he failed in the mission.
general public A. vindicated B. incriminated
Direction: Each item in this section consists of C. indicted D. arraigned
sentences with an underlined word followed by
four words or group of words. Select the option 49. There was much to boast about the quality
that is opposite in meaning to the underlined of his work.
word and mark your response on the answer A. bluster B. brag
sheet accordingly.
C. deprecate D. flaunt
41. I am very particular about it.
50. He delivered an eccentric speech.
A. quiet B. vague
A. an odd B. a peculiar
C. precise D. minute
C. a normal D. an idiosyncratic
42. He has become paunchy.
Direction: Each of the following sentences
A. stout B. slim has a word or phrase underlined. Read the
C. plump D. fat sentences carefully and find which part of
speech the underlined word belongs to.
43. This river originates from the Ganges. Indicate your response on the answer sheet
A. inaugurates. accordingly.

B. culminates 51. They wandered around aimlessly.

C. initiates A. Verb B. Adjective

D. emanates C. Intensifier D. Noun

PAGE 8
www.byjusexamprep.com

52. We went away after they had left. Direction: Each item in this section has a
sentence with three underlined parts labelled as
A. Pronoun B. Adjective
(a), (b), (c). Read each sentence to find out
C. Intensifier D. Conjunction whether there is any error in any underlined part
and indicate your response on the answer sheet
53. Public culture is with extremely new civil
against the corresponding letter i.e., (a) or (b) or
societies.
(c). If you find no error, your response should be
A. Adverb B. Intensifier indicated as (d).
C. Adjective D. Noun Clause 61. Everything is going well; (a)/ we didn't have
54. You are paying less attention to your (b)/ any problem (c)/ No error (d).
studies these days. A. (a) B. (b)
A. Adverb B. Adjective C. (c) D. (d)
C. Intensifier D. Noun 62. Lavanya hasn't gone (a)/ to work (b)
55. Why, is it really Sujata on the phone? yesterday.(c)/ No error.(d)

A. Interjection B. Adjective A. (a) B. (b)

C. Intensifier D. Noun C. (c) D. (d)

56. Sit down and rest a while. 63. Look! The boy over there (a)/ wears the
same sweater (b)/ as you.(c)/ No error.(d)
A. Adverb B. Adjective
A. (a) B. (b)
C. Intensifier D. Noun
C. (c) D. (d)
57. Rakesh is too old run fast.
64. It begins (a)/ to turn dark: (b)/ shall I switch
A. Adverb B. Conjunction
on the lights? (c)/ No error.(d)
C. Intensifier D. Noun
A. (a) B. (b)
58. For the next generation of interior
C. (c) D. (d)
architects an design graduates, work
opportunities are immense. 65. Raman and Mitali (a)/ have been married
(b)/ for twenty years. (c)/ No error.(d)
A. Adverb B. Adjective
C. Noun D. Pronoun A. (a) B. (b)

59. Life is a solo fight, and each person makes C. (c) D. (d)
his or her own journey. 66. I have played (a)/ basketball (b)/ for the
A. Noun B. Adjective past three hours.(c)/ No error.(d)

C. Intensifier D. Adverb A. (a) B. (b)

60. The cat loves comfort. C. (c) D. (d)

A. Indefinite article 67. Jamila had a book (a)/ in front of her, (b)/
but she didn't read it.(c)/ No error. (d)
B. Definite article
A. (a) B. (b)
C. Intensifier
C. (c) D. (d)
D. Subject

PAGE 9
www.byjusexamprep.com

68. When she heard the news (a)/ she hasn't A. R S Q P B. Q P R S


been (b)/ very pleased.(c)/ No error.(d)
C. P R S Q D. Q P S R
A. (a) B. (b)
74. P. and garbage
C. (c) D. (d)
Q. today our environment
69. Where are you coming from? (a)/ Are you
(b)/ an American? (c)/ No error.(d) R. filth and squalor

A. (a) B. (b) S. is in constant grip of

C. (c) D. (d) A. R P S Q B. R S P Q

70. I went to Canada (a)/ a few years ago (b)/ C. Q S P R D. Q S R P


for a holiday. (c)/ No error. (d) 75. P. of corruption everywhere
A. (a) B. (b) Q. to ease out
C. (c) D. (d) R. the monster
Direction: Each of the following items in this S. it is the duty of the government
section consists of a sentence, parts of which
have been jumbled. These parts have been A. S Q R P B. Q R S P
labelled as P, Q, R and S. Given below each
C. P R S Q D. Q R P S
sentence are four sequences, namely (a), (b), (c)
and (d). You are required to rearrange the 76. P. made the movement
jumbled parts of the sentence and mark your
Q. the poor visibility
response accordingly.
R. of the vehicles
71. P. from Europe to America
S. quite difficult
Q. in 1992
A. Q R P S B. R S P Q
R. his first voyage
S. Columbus made C. Q R S P D. Q P R S

A. S P R Q B. S R P Q 77. P. Mohan clearly

C. P R S Q D. Q R S P Q. I couldn’t see

72. P. scientists have R. only a few yards ahead

Q. the effects of S. though he was

R. warned us about A. R Q S P

S. climate change B. R S P Q

A. R P S Q B. R S P Q C. P R S Q
C. P R Q S D. P Q R S D. S R Q P
73. P. speed is a potent cause 78. P. both winter and summer
Q. industrial growth at a terrific Q. extreme
R. in a big city R. are truly
S. of pollution S. here in India

PAGE 10
www.byjusexamprep.com

A. R P S Q B. S R Q P 85. They fought _________ the last man in the


army.
C. S P R Q D. P S R Q
A. on B. from
79. P. get crystallised
C. with D. to
Q. the self and world view and
86. Keep him _________ arm’s length.
R. physical space where basic ideas of
A. at B. by
S. Public culture is a mental
C. for D. off
A. R Q P S B. S R Q P
87. He succeeded ____________.
C. S R P Q D. Q R S P
80. P. that he ordered A. on himself

Q. the post was B. by himself

R. the first job he performed on reaching C. in the long run

S. My release from jail D. on the long run

A. R P S Q B. R S P Q 88. Have you ever tried ___________ a coconut


tree?
C. P R S Q D. R Q P S
A. climbing
Direction: Each of the following sentences in
this section has a blank space and four words or B. to climbing
group of words are given after the sentence. C. going on
Select the most appropriate word or group of
words for the blank space and indicate your D. going up
response in the answer sheet accordingly.
89. He is a man _______ means.
81. Honesty is __________ on his face.
A. for B. in
A. wrote B. written
C. of D. about
C. writing D. writes
90. He dislikes ____________ punish his
82. He lives ________ his pen. friends.
A. by B. with A. have to B. having to
C. on D. off C. for D. regarding
83. Much water has run __________ the bridge Direction: Each item in this section consists of a
since then. sentence with an underlined word followed by
four words. Select the option that is nearest in
A. near B. about
meaning to the underlined word and mark your
C. under D. in response on the answer sheet accordingly.
84. The gun _________ with a loud noise. 91. He drowned in the flood water last year.
A. went on B. went off A. swarmed B. swam
C. went about D. went around C. submerged D. floated

PAGE 11
www.byjusexamprep.com

92. Autumn is rather a dry season. Direction: Given below are some
idioms/phrases followed by four alternative
A. arid B. humid
meanings to each. Choose the most appropriate
C. rainy D. moist response from among (a), (b), (c) or (d), and
mark your response in the answer sheet
93. Our differences are growing day by day.
accordingly.
A. confabulations
101. Forty winks
B. interferences
A. Winking forty times
C. disagreements
B. Sleep disorders.
D. discrepancies
C. Long sleeping hours
94. The big tree hindered access of sunlight into
D. A short sleep during the day
the house.
102. Life in the raw
A. impeded B. shaded
C. facilitated D. poured A. Life in its natural, unembellished state

95. The complexity of the issue baffled B. Life at its easiest


everyone. C. Life that is complex,
A. conclusion B. intricacy D. A daredevil's life
C. grievance D. complacency 103. A paper tiger
96. He drank excessive amount of liquor. A. A person or thing that appears
A. inculpable B. inordinate threatening but is ineffectual

C. unreasonable D. innocuous B. To threaten someone but do no harm

97. He initiated the dialogue. C. Environmental protection paperwork

A. ordered B. interfered D. To speak to people in a roaring voice

C. began D. planned 104. Penny-wise and pound-foolish

98. She looked pale after recovering from A. Wise to spend each penny carefully
Covid-19. B. Careful about small amounts but
A. sallow B. ruddy careless about large amounts

C. glowing D. radiant C. People who don't understand the


importance of each penny
99. I overrule your proposal.
D. Careful about large amounts but
A. wan B. veto careless about small amounts
C. weaken D. supersede 105. Pin back your ears
100. His behaviour shocked me. A. To listen carefully to something.
A. entreated B. pacified
B. Person or organisation that pins
C. appalled D. scintillated important issues

PAGE 12
www.byjusexamprep.com

C. To keep yourself away from hearing bad Direction: In this section each item consists of six
stuff sentences of a passage. The first and sixth
sentences are given in the beginning as S1 and
D. To clean your ears with a pin
S6. The middle four sentences in each have been
106. Turncoat jumbled up and labelled as P, Q, R and S. You are
required to find the proper sequence of the four
A. Expert at altering coats sentences and mark your response accordingly
B. Someone who deserts one group to join on the answer sheet.
another 111. S1: An ideal citizen is one who establishes
C. A truly dishonest person. his standard in everything.
S6: He can be called a thorough gentleman.
D. Going round and round in a court of law
P: Because he is a true patriot.
107. Say your piece
Q: Also, he can lay down his life for the
A. Speak but don't listen honour of his country.
B. Speak in a talkative manner R: That he can make any sacrifice for his
C. Make your argument piece by piece motherland.

D. Express your opinion S: He loves his country so much so.


The correct sequence should be
108. Top-notch
A. S R Q P B. R PQ S
A. The highest marking on a tree or a
building C. P Q R S D. Q R S P
B. Person or organization that attracts 112. S1: There are multiple legal avenues for
only the top patients to indict doctors, but rarely is it the
other way round.
C. Of highest possible quality
S6: It is clear that patients have multiple
D. Person of integrity avenues to pursue, should they feel an
109. Under the table injustice has been perpetrated against
them.
A. Work under difficult circumstances
P: An FIR can be lodged against them under
B. Working undercover the section 304A of the Indian Penal Code.
C. Working under furniture Q: A compensation case can be filed in a
consumer forum.
D. Making or receiving payments
surreptitiously R: For one occurrence, there are multiple
forums where doctors have to defend
110. At the drop of a hat themselves.
A. Clumsy person who drops hats S: For Instance, a complaint can be made to
everywhere their employer.
B. Suddenly and without much thought The correct sequence should be
C. Do something without much pressure A. PQ R S B. P S R Q
D. A happy and easy-going man C. R S Q P D. S R Q P

PAGE 13
www.byjusexamprep.com

113. S1: Paragraphs are the building blocks of disaster management system add another
any write-up. layer of damages during the disasters.
S6: Understanding of these makes one a Q: Major interventions act as force
good writer of paragraphs. multipliers during such disasters.
P: A paragraph need not be pages together R: The violation of legal and other prudent
in length. norms, further increases the damages.
Q: But actually a paragraph is a group of at S: Climate change is increasing these
least five sentences on the lower limit. vulnerabilities.
R: In reality, unity and coherence, not The correct sequence should be
length, constitute a real good paragraph.
A. S Q R P B. P Q S R
S: Many people define paragraphs in terms
of their length. C. Q S R P D. R P S Q

The correct sequence should be 116. S1: Now only fifteen minutes were left.

A. Q P R S B. R Q P S S6: Our performance was excellent.

C. S Q P R D. Q R S P P: Thank God, we rubbed the word 'defeat'


writ large on our forehead.
114. S1: Then what is necessary with regard to
taming science and technology is: Q: It was a game of life and death.

S6: Then why do we not tame atomic R: Each one had concluded that we were
energy to peaceful purposes ? going to lose.
P: We must have the basic knowledge of the S: It was a matter of minutes.
two.
The correct sequence should be
Q: It is said, 'Science is a useful servant but
A. P S Q R B. P Q S R
destructive master'.
C. R P S Q D. R S Q P
R: By the misuse of science, mankind will
meet its doom quite soon. 117. S1: The river Ganga is very important for us
S: If we do not harness it for the welfare of culturally, spiritually, ecologically as well as
mankind, I am afraid, a total annihilation is economically.
a must. S6: Journey in the right direction is firmly
The correct sequence should be making progress, gathering momentum
and becoming a people's movement.
A. S R Q P B. P Q S R
P: It is rich in cultural heritage, natural
C. R Q S P D. Q R S P splendour and biodiversity.
115. S1: Uttarakhand is vulnerable to disasters. Q: People have used its water since the
S6: The lack of ability to learn lessons from beginning of civilisation for different
disasters, and the lack of any accountability, purposes.
ensure the perpetuation of the situation. R: It has most dense cultivation in its basin
P: The absence of necessary monitoring, and is critical for ensuring food and water
early warning systems and the overall security.

PAGE 14
www.byjusexamprep.com

S: We need to utilise its water for different P: Thus, they say, as many people, so many
purposes while ensuring that its natural hobbies.
ecology is protected, aquatic life thrives,
Q: In due course, that becomes your hobby.
and forests along the river remain
rejuvenated. R: Whatever leisure time you get, you use
The correct sequence should be that in creative activity.

A. Q R S P B. Q S R P S: Hobby is a leisure time activity.

C. R S Q P D. P R Q S The correct sequence should be


118. S1: The Republic Day in India falls on 26th A. P Q R S B. P Q S R
January.
C. S R Q P D. R S Q P
S6: He attends the march past.
120. S1: Life in a village is ideal.
P: The President of India takes the salute.
S6: They can no longer be considered
Q: It is organized at the India Gate. backward.
R: People from far and wide come to
P: Our villages are no longer dirty.
witness it.
Q: Moreover, they are disease-free.
S: It is celebrated with great pomp and
show. R: Because the village people are well-
The correct sequence should be educated now.

A. P R S Q B. P Q S R S: Rather, they are highly clean and tidy.

C. S R Q P D. R S Q P The correct sequence should be


119. S1: Different people have different hobbies. A. S R Q P B. P S Q R
S6: With hobbies, you spend time in a C. S R P Q D. R S Q P
gainful way.

PAGE 15
www.byjusexamprep.com

General Knowledge
1. Who among the following was chosen for 7. What is the range capability of Agni-P
the kuvempu award 2020? Ballistic Missile?
A. Dr. Rajendra Kishore Panda A. 1,000 - 2,000 km
B. Amitav Ghosh B. 2,000 - 3,000 km
C. Vikram Seth C. 3,000 - 4,000 km
D. Chetan Bhagat D. 4,000 - 5,000 km
2. Avani Lek-hara became the first Indian 8. Which one of the following Indian States
woman to win two Paralympics medals at has recently declared itself as the first
the Tokyo Paralympics 2020 . Which one of rabies-free State?
the following was her discipline?
A. Punjab
A. Badminton B. Shooting
B. Himachal Pradesh
C. Table Tennis D. Archery
C. Tamil Nadu
3. Bagram Air Base is located in
D. Goa
A. Turkey B. Pakistan
9. Department of Public Enterprises is a part
C. Afghanistan D. Israel of
4. The Panchmuli Lake is situated near A. Ministry of Finance.
A. Statue of Equality B. Ministry of Heavy Industries.
B. Thiruvalluvar Statue C. PMO.
C. Dhyan Buddha Statue D. Ministry of Commerce and Industry.
D. Statue of Unity 10. Chronologically arrange the following
5. Pinaka, developed in India, is a Indian-origin astronauts on the basis of
their flying into space starting with the first:
A. battle tank.
1) Sunita Williams
B. multi-barrel rocket launcher.
2) Rakesh Sharma
C. anti-tank guided missile.
3) Sirisha Bandla
D. armoured utility vehicle.
4) Kalpana Chawla
6. What is itat e-dwar?
Select the correct answer using the code
A. An Urdu magazine published from given below:
Lucknow
A. 4-3-2-1
B. An e-filing portal of Income Tax
Appellate Tribunal B. 2-4-1-3

C. A religious monument C. 2-1-4-3

D. Name of a UNESCO world heritage site D. 1-4-3-2

PAGE 16
www.byjusexamprep.com

11. President Jovenel Moise, who was 16. Why was Dholavira in the news recently?
assassinated recently, was the President of
A. Skeletons discovered at this site
which one of the following countries?
B. For its conservation effects
A. Dominican Republic
B. Haiti C. For the discovery of gold and precious
stones at this site
C. Guatemala
D. It received the UNESCO World Heritage
D. Cuba Tag
12. Recently, which one among the following 17. Which one of the following is the oldest
Ministries was formed under the football tournament in Asia?
Government of India?
A. Federation Cup
A. Ministry of New and Renewable Energy
B. Santosh Trophy
B. Ministry of Development of North-
Eastern Region C. Durand Cup

C. Ministry of Cooperation D. Rovers Cup

D. Ministry of Jal Shakti 18. Who among the following is the first Indian
woman to win a medal in the Olympics?
13. Indian Naval Ship INS Tabar had recently
participated in a two-day naval exercise A. Karnam Malleswari
with the Italian Navy. The exercise was
B. Mirabai Chanu
conducted
C. P.T. Usha
A. Ionian Sea.
D. P.V. Sindhu
B. Adriatic Sea.
19. The Russian Federation participated in the
C. Tyrrhenian Sea.
Tokyo Olympics, 2020 under which of the
D. Mediterranean Sea. following names?
14. Which one of the following countries has A. Russian Olympic Committee
recently been awarded a malaria-free
certification by the WHO? B. Russian Federal Committee

A. Pakistan C. Russian Sports Committee

B. Bhutan D. Russian Republican Committee

C. China 20. Which of the following States/Union


territories/Region are in special focus in the
D. Nepal National Mission on Edible Oils – Oil Palm?
15. Who amongst the following was honoured A. Himachal Pradesh and Andaman and
with Gandhi Peace Prize for the year 2020? Nicobar Islands
A. Qaboos bin Said Al Said B. North-East Region and Andaman and
B. Sheikh Mujibur Rahman Nicobar Islands

C. Ekal Abhiyan Trust C. North-East Region and Gujarat


D. Sulabh International D. Uttar Pradesh and Gujarat

PAGE 17
www.byjusexamprep.com

21. The town of Chisht, from which the Sufi 26. Which one of the following States is not
Chishti Silsila derives its name, is located in prominent for plantation agriculture?
A. Western Turkey A. Rajasthan
B. Central Afghanistan B. Assam
C. Eastern Iran C. Nagaland
D. Eastern Iraq D. Kerala
22. The rules for congregational worship 27. The distance between which two cities have
(Sangat) involving collective recitation were been reduced by the Atal Tunnel?
organized
A. Bagdogra to Gangtok
A. Guru Nanak
B. Jammu to Srinagar
B. Guru Angad
C. Guru Arjan C. Manali to Leh

D. Guru Govind Singh D. Itanagar to Tawang

23. Who amongst the following will be at more 28. Which one of the following biosphere
risk with respect to the earthquake hazard reserves comprises islands with estuaries,
zone specified by the Geological Survey of beaches, coral reefs, salt marshes and
India? mangroves?

A. Shahji at Secunderabad A. Sunderban biosphere reserve


B. Ranbir at Indore B. Gulf of Mannar biosphere reserve
C. Malti at Pithoragarh C. Nilgiri biosphere reserve
D. Maitri at Bhubaneswar D. Nandadevi biosphere reserve
24. The Government wants to prepare a plan 29. Which one of the following is not a correct
for drought-prone areas of the country. statement with reference to the
Which one of the following regions will be Constitution of India?
predominantly focused on?
A. The Constitution (Eighty-Fifth)
A. Coastal Andhra Pradesh Amendment Act has inserted the ‘Right
B. Ganga plains to Education’ under Article 21A

C. North-eastern region B. Article 22 talks about preventive


detention.
D. Kutch region
C. Right to Constitutional Remedies is in
25. Four persons are travelling to the different
Part-III of the Constitution.
states of India. To whom will you advise to
protect oneself from blizzards? D. The writ jurisdiction of High Courts is
wider than the Supreme Court of India.
A. The person travelling to Arunachal
Pradesh 30. Which one of the following statements
B. The person travelling to Assam about the Cripps Mission is not correct?

C. The person travelling to Tripura A. It was sent to India by the British


Government in March 1942
D. The person travelling to Odisha

PAGE 18
www.byjusexamprep.com

B. It proposed that the Constitution of 34. Domingo Paes, the medieval traveller, has!
India was to be framed by an elected described the city of
Constituent Assembly of the Indian
people. A. Madurai B. Vijayanagara

C. Any province which was not prepared to C. Arikamedu D. Gingee


accept the Constitution would be free 35. The extent of damage caused by an
to retain its constitutional provisions earthquake is not influenced by which one
existing at that time. of the following?
D. The British Government could enter
A. Strength of earthquake
into a separate constitutional
arrangement with any of the acceding B. Population density
provinces.
C. Type of building
31. Which of the following statements
regarding constitutional amendment is/are D. The climate of the area
correct? 36. An avalanche is a type of which one of the
1. The procedure for an amendment to the following disasters?
Constitution is provided in Article 368
A. Atmospheric B. Terrestrial
2. A Bill to amend the Constitution can be
C. Aquatic D. Biological
introduced in either House of the
Parliament. 37. An infection constantly maintained at a
3. The special procedure in Article 368 baseline level in a geographic area without
vests constituent powers upon the external input is known as
ordinary legislation. A. Endemic B. Pandemic
Select the correct answer using the code C. Epidemic D. Outbreak
given below:
38. Streams and rivers coming from the
A. 1 only B. 1, 2 and 3
mountains deposit heavy materials of rocks
C. 2 and 3 only D. 1 and 2 only and boulders in
32. Which one of the following is not a A. Khadar B. Bhangar
circumstance for the proclamation of
Emergency by the President of India under C. Bhabar D. Terai
Article 352 of the Constitution of India? 39. Which of the following statements
A. War regarding the electoral rolls is/are correct?

B. External aggression 1. There is one general electoral roll for


every territorial constituency.
C. Internal disturbance
2. No person is ineligible for inclusion in
D. Armed rebellion
the electoral roll on grounds of religion,
33. Akbar issued a Farman in 1598 permitting in race, caste, sex, or any of them.
the city of Cambay (Khambat), Gujarat, the
construction of a 3. A citizen of India, not less than 18 years
of age, can cast his/her vote unless
A. Temple B. Church disqualified under a law of an
C. Synagogue D. Jain Upashraya appropriate legislature.

PAGE 19
www.byjusexamprep.com

Select the correct answer using the code 44. Which among the following is not a
given below: condition for the disqualification of a
Member of Parliament?
A. 1 only B. 1, 2 and 3
A. Voluntary acquisition of citizenship of a
C. 2 and 3 only D. 1 and 2 only
foreign country
40. Which one of the following statements is
B. Holding the office of the Chairperson of
not correct about ‘Totalitarianism’?
the National Commission for Women
A. It is not akin to autocracy and
C. The Member abstains from voting in the
authoritarianism.
House without prior permission
B. It usurps the political freedom of the
D. The Member holds the office of the
individuals, but it doesn’t usurp
Chairman of the Board of Directors of
personal freedoms.
the National Coal Development
C. It implies the abolition of civil society. Corporation Ltd.
D. It is usually identified with a one-party 45. Who among the following considered the
state. Directive Principles of State Policy as aiming
at ‘furthering the goals of social exclusion’?
41. The federation of which one of the
following States has been described as “an A. B .N. Rau B. Granville Austin
indestructible union of indestructible
states”? C. K. C Wheare D. Rajni Kothari

A. India B. U.S.A 46. D performs her train journey by the


shortest route from Bengaluru to New
C. Canada D. Australia Delhi. Which one of the following rivers will
42. Who among the following is the author of she not cross while performing the
the famous essay, “The End of History and journey?
the Last Man”? A. Narmada B. Yamuna
A. Francis Fukuyama C. Godavari D. Krishna
B. Daniel Bell 47. Identify the State on the basis of the
C. Abraham Lincoln following characteristics:

D. Anthony Giddens 1. Tropic of Cancer passes through the


State.
43. Which one of the following statements is
not correct? 2. The State has more north-south
extensions.
A. Allahabad High Court has a Bench at
Lucknow. 3. The State has an international border
with Bangladesh and Myanmar.
B. Madhya Pradesh High Court has a
Bench at Gwalior. Select the correct answer using the code
given below:
C. Rajasthan High Court is located at
Jodhpur. A. Tripura B. Mizoram

D. Guwahati High Court has no Bench. C. Nagaland D. Manipur

PAGE 20
www.byjusexamprep.com

48. M wants to visit a place in a Union Territory, B. Civil Disobedience Movement.


which is located at 34° N and 77° E . Which
C. Quit India Movement.
one of the following Union Territories must
he have planned to visit? D. Protest against the partition of Bengal.
A. Andaman and Nicobar Islands 53. Which of the following statements is/are
correct with respect to Time Zone in India?
B. Lakshadweep
1. There is one standard time for the whole
C. Puducherry
country.
D. Ladakh
2. Andaman and Nicobar Islands and
49. Consider the following statements with Lakshadweep Islands have different Time
respect to the adolescent population (age Zones.
group of 10 to 19 years) of India, according
3. Indian Standard Time (IST) is five and a
to the 2011 Census:
half hours behind GMT.
1. The adolescent population comprises
Select the correct answer using the code
half the population of the country.
given below:
2. The group is regarded as a youthful
A. 1 only B. 2 only
population with high potential.
C. 1 and 2 only D. 1 and 3 only
3. In this group, there is a greater number
of females than males. 54. D had recently visited Khardung La, Nubra
Valley and several Buddhist monasteries.
Which of the statements given above is/are
Which one of the following States /UT had
correct?
she visited?
A. 1 only B. 2 only
A. Himachal Pradesh
C. 1 and 2 only D. 2 and 3 only
B. Uttarakhand
50. In a discussion, M from Arunachal Pradesh,
C. Sikkim
J from Assam, N from Meghalaya and S
from Nagaland are claiming that as per D. Ladakh
Census 2011, their State has the maximum
55. Which one of the following States receives
density of population. Identify the person
rainfall from both the Arabian Sea branch
making the correct claim.
and the Bay of Bengal branch of Monsoon?
A. M B. J
A. Punjab B. Maharashtra
C. N D. S
C. Meghalaya D. Tamil Nadu
51. Swami Dayanand Saraswati took inspiration
56. Inceptisols, Entisols, Vertisols and Mollisols
from
are orders of which one of the following?
A. Puranas B. Vedas
A. Sugarcane B. Soil
C. Medieval saints D. Sufism
C. Coal D. Copper
52. The call for renunciation of (all) voluntary
57. Shad witnessed contour bunding and
associations with the (British) Government’
contour ploughing while visiting one of the
was given during
States in India. Identify the State from the
A. Non-Cooperation Movement. options given below.

PAGE 21
www.byjusexamprep.com

A. Punjab 61. Following the Constitution (One Hundred


and First Amendment) Act, 2016, the
B. Haryana
Parliament of India enacted quite a few GST
C. Himachal Pradesh Acts in the year 2017 . Which one of the
D. Rajasthan following does not fall in this category?

58. Which one of the following target groups is A. The Central Goods and Services Tax Act
the beneficiary of “Garib Kalyan Rojgar B. The Integrated Goods and Services Tax
Abhiyan scheme? Act
A. All rural women C. The Goods and Services Tax
B. All returning migrants (Compensation to States) Acts
C. All below poverty line households D. The State Goods and Services Tax Act
D. All new migrants to the town 62. Which one of the following is not correct in
respect of the Directorate of Enforcement?
59. Match List I with List II and select the
correct answer using the code given below A. It is a specialized financial investigation
the lists: agency under the Department of
Revenue, Ministry of Finance.
List I List II
(Curve) (Indication) B. It enforces the Foreign Exchange
(a) Lorenz 1. Inflation and Management Act, 1999
curve employment C. It enforces the Prevention of Money
(b) Phillips 2. Tax rates and tax Laundering Act, 2002
curve revenue D. It enforces the Prohibition of Benami
(c) Engel 3. Inequality in the Property Transaction Act, 1988
curve distribution of
63. Which one of the following is not correct?
income or wealth
(d) Laffer 4. Income and A. Taxes on agricultural income is subject
curve proportion of to the State List.
expenditure on food B. Price control is a subject under the
Concurrent List.
A. (a)-3 (b)-4 (c)-1 (d)-2
C. Insurance does not come under the
B. (a)-2 (b)-1 (c)-4 (d)-3 Union List.
C. (a)-3 (b)-1 (c)-4 (d)-2 D. Forests’ is a subject under the
D. (a)-2 (b)-4 (c)-1 (d)-3 Concurrent List.

60. Which one of the following is a typical 64. Which one of the following statements is
example of monopolistic competition? not correct about the nature of India’s
federal system?
A. Retail vegetable markets
A. There is no equality of representation of
B. Market for soaps
States in the Council of States.
C. Indian Railways
B. Sikkim was not initially included in the
D. The labour market for software Union as a full-fledged State.
engineers

PAGE 22
www.byjusexamprep.com

C. Special provisions have been laid down 68. Which of the following ideas was preached
in the Constitution of India for Andhra by the Kherwar or Sapha Har movement of
Pradesh under Article 371 D the 1870s?
D. The Constitution of India enables the A. Acceptance of the Hindu pantheonic
federal government to acquire the order
strength of a unitary system during
B. Monotheism and internal social reform
emergencies.
C. Philosophy of Yoga and Mimansa
65. The Constitution of India guarantees
freedom of speech and expression. But the D. Polytheism
freedom is subject to certain reasonable
69. In which one of the following years was the
restrictions imposed by the State. These
Poona Sarvajanik Sabha established?
restrictions may relate to which of the
following? A. 1884 B. 1876
1) Defamation C. 1869 D. 1870
2) Decency or morality 70. Which one of the following was an
important Pandya port, celebrated for its
3) Incitement to an offence
pearls in Sangam poems and Greek
Select the correct answer using the code accounts?
given below:
A. Muchiri B. Korkai
A. 1 only
C. Puhar D. Arikamedu
B. 2 and 3 only
71. Who among the following ancient Indian
C. 1 and 3 only kings was praised in glowing terms in the
Prayaga Prashasti?
D. 1, 2 and 3
A. Ashoka B. Harshavardhana
66. Which one of the following amendments in
the Constitution of India made a C. Samudragupta D. Bindusara
Proclamation of Emergency immune from
72. The Self-Respect Movement was initiated
judicial review?
by
A. 39th Amendment
A. B .R. Ambedkar
B. 40th Amendment
B. Jyotiba Phule
C. 38th Amendment
C. E .V. Ramaswamy Naicker
D. 44th Amendment
D. Jawaharlal Nehru
67. Which one of the following British officials
73. The location of the ancient city of Taxila
was elevated to the position of Governor-
(Takshshila), mentioned in ancient Indian
General after the Regulating Act of 1773?
texts, was identified by
A. Warren Hastings
A. Alexander Cunnigham
B. Lord North
B. R.D . Banerji
C. Mount Stuart Elphinstone
C. John Marshall
D. Philip Francis
D. Daya Ram Sahni

PAGE 23
www.byjusexamprep.com

74. Who among the following was not a Jain C. Value of all opportunities forgone
Acharya?
D. Value of next best alternative that is
A. Bhadrabahu given up
B. Khema 79. Which one of the following statements in
the context of social sector spending in
C. Haribhadra
India during 2014 – 19 (both States and the
D. Siddhasena Divakara Union Government together) is true?
75. In India, the first major public appearance A. Expenditure on education was 5% of
of Mahatma Gandhi was in GDP.
A. Champaran (1917) B. Expenditure on health was 4% of the
social services expenditure.
B. Kheda (1918)
C. There was a stagnation in the spending
C. Inauguration of Banaras Hindu
on education as a per cent of GDP.
University (1916)
D. Health sector spending amounted to
D. Rowlatt Satyagraha (1919)
80. According to UNDP’s Human Development
76. Consider the following statements about
Report – 2020, in which of the HDI
Patanjali’s Mahabhashya:
components has India improved in recent
1. It makes a mention of Kautilya. years?
2. It is a book on grammar and refers to 1. Life expectancy at birth
historical personalities only incidentally.
2. Expected years of schooling
Which of the statements given above is/are
3. GNI per capita
correct?
4. Mean years of schooling
A. 1 only B. 2 only
Select the correct answer using the code
C. Both 1 and 2 D. Neither 1 nor 2
given below:
77. Which one of the following may lead to a
A. 1 and 2 only
movement along the demand curve of a
commodity? B. 2 and 3 only
A. Change in its price C. 1 and 3 only
B. Change in price of the other D. 1, 2, 3 and 4
commodities
81. Which one of the following is true of a pure
C. Change in the income of the consumer voluntary exchange between two parties A
and B?
D. Change in tastes and preferences of
consumers A. A can exploit B or vice versa
78. Which one of the following is the B. Both gain; it is a win-win situation
opportunity cost of a chosen activity?
C. If A makes a profit, it must be at the cost
A. Out of pocket cost of B
B. Out of pocket cost plus cost incurred by D. Both can lose
the Government

PAGE 24
www.byjusexamprep.com

82. The provision, the State shall take steps to 85. Which one among the following is the
separate the Judiciary from the Executive in largest State in terms of seats in the Rajya
the public services of the State’ is Sabha?
incorporated in which part of the
A. Andhra Pradesh B. Bihar
Constitution of India?
C. Rajasthan D. Karnataka
A. Part-IV B. Part-V
86. Under which Article of the Constitution of
C. Part-VI D. Part-VII
India has provision been made for
83. Which of the following statements are reservation of seats for women in
correct? Panchayats?

1. Speaker may permit any Member to A. Article 243 B B. Article 243 C


address the House in his/her mother
C. Article 243 D D. Article 243 E
tongue if he/she cannot adequately
express in either Hindi or English. 87. The painted illustration of the moving of the
Ashoka Pillar at-Topra is found in
2. Business of the Parliamentary
Committees is transacted either in Hindi A. Tarikh-i-Firuz Shahi
or in English
B. Tarikh-i-Shahi
3. The minutes of the Parliamentary
C. Sirat-i-Firuz Shahi
Committees are prepared invariably in
Hindi or English. D. Akbar Nama
Select the correct answer using the code 88. Which one among the following is a temple
given below: of the Vaishnavite tradition?
A. 1, 2 and 3 B. 1 and 2 only A. Srirangam

C. 2 and 3 only D. 1 and 3 only B. Chidambaram

84. Which one of the following is the correct C. Gangaikonda Cholapuram


meaning of the term ‘State’, as defined in
D. Thanjavur
Article 12 of the Constitution of India?
89. The biography of Shaikh Muinuddin Chishti,
A. It refers only to the Government of authored by Jahanara, is known as
India.
A. Munis al Arwah B. Fawaid ul Fuwad
B. It refers only to the Government of India
and the Parliament of India. C. Sirat ul Auliya D. Muraqqa-e Dehli

C. It refers only to the Government of 90. Which one of the following is not a political
India, Parliament of India, and method of the moderates in the National
Governments and Legislatures of each Movement?
State.
A. Overthrow of alien rule
D. It refers to the Government of India, B. Constitutional agitation
Parliament of India, Governments and
Legislatures of each State, and all local C. Slow, orderly political progress
or other authorities within the territory
D. Mobilisation of public opinion
of India.

PAGE 25
www.byjusexamprep.com

91. Fertilizers are used to obtain higher yields B. A group of organisms of one species
of crops. However, all nutrients are usually living in different areas during different
not available in fertilizers. Which one of the seasons.
following nutrients is usually not available
C. A unit consisting of biotic and abiotic
in fertilizers?
components.
A. Iron B. Potassium
D. A group of organisms of more than one
C. Nitrogen D. Phosphorus species, living in the same area at the
same time.
92. A bus starting from a bus-stand and moving
with uniform acceleration attains a speed 97. A ray of light travelling from a rarer medium
of 20 km/h in 10 minutes. What is its to a denser medium
acceleration?
A. slows down and bends away from the
A. 200 km/h2 B. 120 km/h2 normal.

C. 100 km/h2 D. 240 km/h2 B. slows down and bends towards the
normal.
93. In which of the following groups of
organisms, is food material broken down C. speeds up and bends away from the
outside the body and absorbed? normal.

A. Yeast, mushroom, bread mould D. speeds up and bends towards the


normal.
B. Mushroom, Amoeba, green plants
98. Which one of the following is the correct
C. Caserta, tapeworm, lice reactivity order of metals reacting with
D. Paramecium, Amoeba, Caserta dilute HCl?

94. A solution turns red litmus blue, its pH is A. Mg > Al > Zn > Fe
likely to be ………… B. Mg < Al < Zn < Fe
A. 1 B. 4 C. Mg > Zn > Fe > Al
C. 5 D. 10 D. Fe > Mg > Al > Zn
95. The magnetic field lines inside a current 99. Which one of the following materials is
carrying long solenoid are in the form of present in a guard tube (drying tube) that is
A. Ellipse. used for preparation of HCl gas?

B. Parabola. A. Calcium chloride B. Calcium bromide


C. Calcium iodide D. Calcium fluoride
C. Hyperbola.
100. What are the constituents of alloy solder ?
D. Parallel straight lines.
A. Pb and Zn B. Pb and Sn
96. While studying vegetation of an area, terms
like ‘population’ and ‘community’ are often C. Pb and Si D. Pb and Co
used. Which one of the following
statements best describes a population? 101. An athlete completes one round of a
circular track of diameter 100 m in 20 s.
A. A group of organisms of one species, What will be the displacements after 1
living in the same area at the same time. minute and 10 s, respectively?

PAGE 26
www.byjusexamprep.com

A. 0 m, 50 m B. 300 m, 100 m A. 1 : 1 B. 1 : 3
C. 300 m, 50 m D. 0 m, 100 m C. 3 : 1 D. 2 : 1
102. Where should an object be placed in front 107. Refraction of light, as it enters from one
of a convex lens to get a real and enlarged transparent medium to another, is due to
image of the object? A. change in temperature of the media.
A. At twice the focal length B. change in the amplitude of light.
B. At infinity C. change in speed of light.
C. Between the principal focus and twice D. internal property of light.
the focal length
108. When copper reacts with moist carbon
D. Beyond twice the focal length dioxide (CO2) in air, it forms a green coating
103. What is the location of intercalary meristem of which one of the following compounds?
in plants? A. Cupric carbonate B. Cuprous oxide
A. Base of the leaves or internodes C. Cupric oxide D. Copper sulphate
B. Stems or roots 109. Rupa and Sachin observed an animal in
C. Tips of stems and leaves their school garden. Rupa called it an insect
while Sachin identified it as an earthworm.
D. Base of flower pedicel Which one of the following characteristics
104. Which of the following limits the number of confirms that it is an insect?
trophic levels in a food chain ? A. The animal had jointed legs.
A. Deficient food supply B. Body of the animal had very little
segmentation.
B. Polluted air
C. Body of the animal was cylindrical.
C. Decrease in the available energy at
higher trophic levels D. Body of the animal was bilaterally
symmetrical.
D. Parasitic organisms
110. What is the name of the process that
105. Which of the following carbon allotropes
converts sulphide ores into oxides by
is/are good conductor(s) of electricity ?
heating strongly in the presence of excess
1) Diamond air?
2) Graphite A. Calcination B. Roasting
3) Fullerene C. Smelting D. Incineration
Select the correct answer using the code 111. What is the effect of pressure of a human
given below: body on sand?
A. 1 only B. 1 and 2 only A. Larger while standing than while lying

C. 2 only D. 1 and 3 only B. Smaller while standing than while lying

106. In pea, a pure tall plant (TT) is crossed with C. Same while standing or lying
a short plant (tt). What will be the ratio of D. Larger while standing during the
pure tall plants to short plants in the F2 daytime and smaller during the night-
generation? time while lying

PAGE 27
www.byjusexamprep.com

112. An electric circuit is consisting of a cell, an A. 0.22 W B. 2.2 W


ammeter, and a nichrome wire of length l.
C. 22 W D. 220 W
If the length of the wire is reduced to half ,
then the ammeter reading 117. A sound wave has a frequency of 4 kHz and
A. decreases to one-half. wavelength 30 cm. How long will it take to
travel 2.4 km?
B. gets doubled.
A. 2.0 s B. 0.6 s
C. decreases to one-third.
C. 1.0 s D. 8.0 s
D. remains unchanged.
118. In the human body, blood flows through a
113. Which one of the following acids is secreted process of double circulation. Which one of
by leaves of Nettle that causes painful the following statements is true in this
stings? regard?
A. Methanoic acid B. Citric acid
A. Oxygenated blood reaches the left side
C. Tartaric acid D. Acetic acid of the heart from the lungs.
114. Which of the following statements about B. Blood in the left side of the heart is poor
universal indicator is/are true ? in oxygen and is brought to the right side
1. It is a mixture of several indicators. of the heart.

2. It shows different colours at different C. Deoxygenated blood from the left side
concentrations of hydrogen ions in of the heart is brought to the lungs for
solution. oxygenation.

3. It helps to determine the strength of D. Oxygenated blood from the right side of
given acid and base in titration. the heart is sent around the body.

Select the correct answer using the code 119. Which one of the following statements
given below: about variations is not true?
A. 1 only B. 1 and 2 only A. Variation is minimum in asexual
reproduction.
C. 2 and 3 only D. 1, 2 and 3
B. All variations in a species have equal
115. Which of the following statements is/are
chances of survival.
correct ?
1. All the bases are alkali. C. Changes in genetic constitution result in
variation.
2. All alkalis dissolve in water.
D. Variants can be selected by
3. Alkalis are soapy to touch, bitter in taste environmental factors.
and corrosive in nature.
120. Animals which are marine, bilaterally
Select the correct answer using the code symmetrical, have a coelom and a
given below: notochord, but never form a vertebral
A. 1 only B. 1 and 3 only column are placed under which one of the
following groups?
C. 2 and 3 only D. 3 only
A. Chordata B. Protochordata
116. An electric bulb is connected to a 110 V
generator. The current is 0.2 A. What is the C. Vertebrata D. Mammalia
power of the bulb?

PAGE 28
www.byjusexamprep.com

Elementary Mathematics
Direction: Consider the following for the next Direction: Consider the following for the next
four (04) items that follow: four (04) items that follow:
The following Pie–Charts show the percentage of The following Bar–Chart gives the production of
different categories (A, B, C, D, E and F) of cars by three different companies X, Y and Z in
employees in a company in the year 2019 and different years:
2020 . The total number of employees was 4000
in 2019 and 5000 in 2020 .

1. There are two categories of employees in


2019 whose total strength remained the
same in 2020 . What are these two
categories?
A. C and B B. E and A 5. The percentage increase in the total
production of cars from 2014 to 2018 was
C. E and F D. C and D
A. 50% B. 100%
2. In which one of the following categories of
employees, the percentage change in C. 150% D. 200%
number of employees in 2020 was 6. Consider the production of each company
maximum as compared to 2019? separately. In how many instances was the
A. A B. B percentage increase in production of cars
over the previous year’s production greater
C. C D. D than 20%?
3. What was the percentage increase of A. 5 B. 6
category–E employees in 2020 as compared
to that in 2019? C. 7 D. 8

A. 20% B. 25% 7. In which year was the percentage increase


in production of cars minimum as
C. 30% D. 40% compared to its previous year?
4. There are two categories of employees A. 2015 B. 2016
whose strength increases by same
percentage in 2020. What are the two C. 2017 D. 2018
categories? 8. In one of the years the percentage increase
in production of cars was minimum as
A. E and F B. C and E
compared to its previous year. What was
C. A and E D. A and C the minimum percentage?

PAGE 29
www.byjusexamprep.com

A. B. A. 5 km/hr B. 8 km/hr

C. D. 20% C. 10 km/hr D. 12 km/hr


14. The present age of a father is equal to sum
Direction: Consider the following for the
of the ages of his 4 children. After ten years
next two (02) items that follow:
the sum of the ages of the children will be
Collect all the sequences of five consecutive 1.6 times the age of their father. What is the
integers such that their product is equal to present age of father?
one of these integers. Let X be the
A. 36 years B. 40 years
collection of all possible such sequences.
Let P be the smallest integers and Q be the C. 42 years D. 45 years
largest integer occurring in these
15. The sum of numerator and denominator of
sequences.
a fraction is 10 . If the numerator is
9. How many such sequences of five increased by 3 and denominator is
consecutive integers are possible? decreased by 1, the fraction becomes 1 .
What is the difference between numerator
A. One B. Two
and denominator of the fraction?
C. Three D. Five
A. 2 B. 3
10. What is the arithmetic mean of P and Q?
C. 4 D. 5
A. 0
16. If the system of equations 7x + ky = 27 and
B. 1 kx + 7y = 19 have unique solution, then
C. 2 which one of the following is correct?
D. Cannot be determined due to A. k ≠ 7 B. k ≠ 13
insufficient data C. k = 7 D. k = 13
11. If one of the roots of the equation
17. A sum of money compounded annually
is , then what is the doubles itself in 5 years. In how many years
sum of the squares of the roots? will it become four times of itself?
A. 10 years B. 12 years
A. B.
C. 15 years D. 20 years
C. D.
18. Between 3 and 4 O’clock, both hour hand
12. A two–digit number is such that the product and minute hand will coincide past 3
of the digits is 8 . If 63 is added to this O’clock between
number, the digits interchange their places. A. 15–16 minutes B. 16–17 minutes
What is the sum of the digits in the
number? C. 17–18 minutes D. 18–19 minutes

A. 6 B. 7 19. Sheela can stitch a suit in 2 days, while


C. 8 D. 9 Meena can stitch a suit in days. How
many days will both take in stitching 30
13. A motor boat has speed 30 km/hr in still
suits?
water. It goes 60 km downstream and
comes back in hours. What is the speed of A. 32 days B. 33 days
C. 35 days D. 40 days
the stream?

PAGE 30
www.byjusexamprep.com

20. If 2x – 3y – 7 = 0, then what is the value Direction: Consider the following for the next
of ? there (03) items that follow:
A. –1 B. 0 A solid consisting of a right circular cone of radius
x and height 2x standing on a hemisphere of
C. 1 D. 3
radius x (take )
21. If p varies directly as q and inversely as
square of r, what is the percentage increase 26. The volume of the solid is equal to that of a
in p due to an increase in q by 20% and a
A. sphere of radius x
decrease in r by 20%?
B. sphere of diameter x
A. 87.5% B. 85%
C. cylinder of radius x
C. 82.5% D. 80%
22. A person agrees to work for 30 days on a D. cylinder of radius
condition that for every day’s work he Direction: Consider the following for the next
should receive Rs 500, and that for every there (03) items that follow:
day’s absence from work he should forfeit
Rs 100. At the end of the time he received A solid consisting of a right circular cone of radius
Rs 11,400. How many days did he work? x and height 2x standing on a hemisphere of
radius x (take )
A. 20 B. 21
C. 24 D. 25 27. What is the approximate total surface area
of solid?
23. A person bought a chair and a table for
`700. He sold the chair at a gain of 5% and A. 11.2x2 B. 12.5x2
the table at a gain of 20%. He gained 16% C. 13.3x2 D. 15.1x2
on the whole. What is original cost of table?
28. The cubical solid is placed upright in a right
A. `400 B. `450 circular cylinder full of water such that it
C. `550 D. `600 touches the bottom. If the internal radius of
cylinder is x and height is 3x, what is the
24. A person rode one–third of a journey at 60 approximate volume of water left in the
km/hr, one–third at 50 km/hr and the rest cylinder?
at 40 km/hr. Had the person ridden half of
the journey at 60 km/hr and the rest at 40 A. 5.04x3 B. 5.09x3
km/hr, he would have taken 4 minutes C. 5.14x3 D. 5.24x3
longer to complete the journey. What
distance did the person ride? Direction: Consider the following for the next
two (02) items that follow:
A. 180 km B. 210 km
A chord of a circle of radius 2.1 cm subtends an
C. 240 km D. 300 km
angle of 120° at the centre. (take
25. A person saves `1000 more than he did the and )
previous years. If he saves `2000 in the first
years, in how many years will he save 29. What is the approximate area of minor
`170000? segment of circle?
A. 16 years B. 17 years A. 2.71cm2 B. 2.42cm2
C. 18 years D. 19 years C. 1.91cm2 D. 1.71cm2

PAGE 31
www.byjusexamprep.com

30. What is the approximate area of major 34. Which on one of the following is correct?
segment of the circle?
A.
2 2
A. 10.05cm B. 10.15cm
B.
C. 11.05cm2 D. 11.15cm2
C.
Direction: Consider the following for the next
D.
three (03) items that follow:
35. If AD = 5 cm, BD = 4 cm and CD = 6 cm, then
ABC is a triangle with sides AB = 6 cm, BC = 10 cm
what is AB equal to?
and CA = 8. With vertices A, B and C as centres,
three circles are drawn each touching the other A. 7 cm B. 6.5 cm
two externally.
C. 6 cm D. 5.5 cm
31. What is the sum of the radii of the circles?
Direction: Consider the following for the next
A. 10.4 cm B. 11.2 cm three (03) items that follow:

C. 12 cm D. 13 cm ABC is a triangle with AB = 1.6 cm, BC = 6.3 cm


and CA = 6.5 . Let P and Q be the mid-points of
32. What is the length of the altitude of the AB and BC respectively.
triangle drawn from vertex A on BC?
36. What is equal to?
A. 2.4 cm B. 3 cm
A. 41.25 cm2 B. 42.25 cm2
C. 4 cm D. 4.8 cm
C. 43.75 cm2 D. 44.25 cm2
Direction: Consider the following for the next
three (03) items that follow: 37. What is equal to ?

ABC is a triangle with sides AB = 6 cm, BC = 10 cm A. AC2 B. 1.2 AC2


and CA = 8 . With vertices A, B and C as centres, C. 1.25 AC2 D. 1.5 AC2
three circles are drawn each touching the other
two externally. 38. What is equal to?

33. If P, Q and R are the areas of sectors at A, B A. 101.39 cm2 B. 111.39 cm2
and C within the triangle respectively, then C. 121.39 cm2 D. 131.39 cm2
which of the following is/are correct?
Direction: Consider the following for the next
1. P =  cm2 two (02) items that follows:
2. 9Q+4R = 36 cm2 AB is a diameter of a circle with centre O. Radius
OP is perpendicular to AB. Let Q be any point on
Select the correct answer using the code
arc PB.
given below
39. What is equal to?
A. 1 only B. 2 only
A. 30° B. 40°
C. Both 1 and 2 D. Neither 1 nor 2
C. 45° D. 60°
Direction: Consider the following for the next
two (02) items that follow: 40. What is equal to?

ABC is triangle in which AB = AC and D is any A. 30° B. 40°


point on BC. C. 45° D. 60°

PAGE 32
www.byjusexamprep.com

41. The radius of circum-circle of a right-angled 47. A hemispherical bowl of internal radius 1
triangle is 10 cm and the altitude drawn to cm contains a liquid. The liquid is filled in
the hypotenuse is 8 cm. What is the area of small cylindrical bottles of internal radius 3
the triangle? cm and internal height 4 cm. What is the
number of bottles used to empty the bowl?
A. 60 cm2 B. 80 cm2
A. 54 B. 81
C. 100 cm2 D. 120 cm2
C. 108 D. 135
42. The circles touch externally. The sum of
their areas is 41 square cm. If the distance 48. A hollow spherical shell is made of a metal
between their centers is 9 cm, then what is of density 7 g/cm3. If its internal and
difference between their diameters? external radii are 3 cm and 6 cm
respectively, then what is the mass of the
A. 1 cm B. 1.5 cm
shell? (take )
C. 2 cm D. 4 cm
A. 2772 g B. 3322 g
43. Let p be the area of a square X and q be the
area of the square formed on the diagonal C. 4433 g D. 5544 g
of the square X. What is the value of ? 49. A cone of height 16 cm and diameter 14 cm
is mounted on a hemisphere of same
diameter. What is the volume of the solid
A. B.
thus formed? (take )
C. D.
A. 1540 cm3 B. 1078 cm3

44. The area of a rhombus is 336 square cm. If C. 1048 cm3 D. 770 cm3
the length of one of its diagonals is 48 cm, 50. 3 cubes each of volume 343 cm3 are joined
then what is the perimeter of the rhombus? end to end. What is the total surface area of
the resulting cuboid?
A. 200 cm B. 120 cm
A. 343 cm2 B. 350 cm2
C. 100 cm D. 90 cm
C. 686 cm2 D. 700 cm2
45. The minute hand of a clock is 21 cm long.
What is the area on the face of the clock 51. A cubical block of side 14 cm is surmounted
described by the minute hand between by a hemisphere of radius 7 cm. What is the
total surface area of the solid thus formed?
10.10 A .M and 10.30 A .M? (take )
(take )
2 2
A. 231 cm B. 331 cm
A. 1330 cm2 B. 1306 cm2
2 2
C. 462 cm D. 492 cm
C. 1296 cm2 D. 1256 cm2
46. The length and breadth of a room are 21 m
52. How many silver coins,3.5 cm in diameter
and 16 m respectively. If the length of the
and of thickness 4 mm, must be melted to
longest rod that can be placed in the room
form a cuboid of dimensions 21 cm × 11 cm
is 29 m, then what is the height of the
room? × 7 cm? (take )

A. 10 m B. 11 m A. 420 B. 210
C. 12 m D. 13 m C. 200 D. 168

PAGE 33
www.byjusexamprep.com

53. A tub is in the shape of a frustum of a cone. the flagstaff is 60°. What is the height of
The radii of two circular ends of the tub are flagstaff?
105 cm and 42 cm. If the vertical height of
A. 12 m B. 15 m
the tub is 16 cm, what is its slant height?
C. 16 m D. 20 m
A. 63.5 cm B. 65 cm
60. What is the maximum value of
C. 73.5 D. 75 cm where
54. ABC is a triangle right–angled at B with ?
AB = 8 and BC = 6 cm. It is made to revolve A. 1 B. 2
about its side BC . What is the approximate
total surface area of the cone so formed? C. 3 D. 4

(take ) 61. From an aeroplane flying above a river at an


altitude of 1200 m, it is observed that the
A. 452 cm2 B. 440 cm2 angles of depression of opposite points on
the two banks of a river are 30° and θ. If the
C. 432 cm2 D. 420 cm2 width of the river is 3000 m, then which one
55. A solid rod consists of a cylinder of height of the following is correct?
20 cm and radius 7 cm. It is surmounted by A. θ < 30° B. 30°<  < 45°
another solid cylinder of height 10 cm and
radius 3.5 cm. If 1 cubic metre of rod weighs C. 45°<  < 60° D. 60°<  < 90°
10000 kg, what is the mass of the rod?
62. , where then what is
(take )
then what is
A. 34.65 kg B. 31.56 kg equal to ?

C. 3.465 kg D. 3.156 kg A. B.
56. What is the minimum value
of where ? C. D. 2

A. 0 B. 1 63. Consider the following :


C. 2 D. None of these 1.
57. If where 2.
, then what is the value
3.
of ?
Which of the above are identities?
A. B.
A. 1 and 2 only
C. D. B. 2 and 3 only
58. What is equal to? C. 1 and 3 only
D. 1, 2 and 3
A. 0 B. 1
64. What is the value of sin24° sin66° – cos24°
C. 2 D. 4 cos66° + tan24° tan66° – cot24° cot66°?
59. A ladder 10 m long reaches a point 10 m A. 0 B. 1
below the top of a vertical flagstaff. From
foot of the ladder, the elevation of top of C. 2 D. 3

PAGE 34
www.byjusexamprep.com

65. If x = p sinA cosB, y = p sinA sinB and z = p 71. 5 pencils, 6 notebooks and 7 erasers cost
cosA, then what is the value of `250; whereas 6 pencils, 4 notebooks and 2
? erasers cost `180 . What is the cost of 2
notebooks and 4 erasers?
A. –p2 B. 0
A. `90 B. `75
C. p2 D. 2 p2
C. `60 D. `40
66. If x = m secA + n tanA and y = m tanA + n
72. How many zeros are there in the product
secA, then what is equal to?
?
A.
A. 262 B. 261
B. C. 246 D. 235
C. 73. If p and q (p > q) are the roots of the
D. equation , then which
one of the following is correct?
67. If for some  lying between 0° and 90°,
A.
tan  = 1, then what is the value of
? B.

A. –1 B. 0 C.
D.
C. D.
74. If the rootsof the equation
68. What is equal are real, then what is
the minimum value of N?
to?
A. 0.1 B. 0.01
A. sin B. cos
C. 0.001 D. 0.0001
C. tan D. cot
75. If , then what is the value
69. If A, B and C are interior angles of a triangle of x?
ABC, then what is A. 1 B. log102
C. log105 D. 2log105
equal to?
76. If , then what
A. 0 B. is equal to?
A. 0 B. 1
C. D.
C. 2 D. 3
70. In a triangle ABC, right–angled at B, AB +
BC = 10(1+ 3 ) cm and length of the 77. If and , then
hypotenuse is 20 cm. What is the value of what is B equal to?
tanA + tanC?
A. B.
A. B.
C. D.
C. 3 D. 2 3

PAGE 35
www.byjusexamprep.com

x4 y4 84. If , then which of


78. What is + +
2 2 2 2 2 2 2 2
(x − y )(y − z ) (y − x )(y − z ) the following is/are correct?
4
z
equal to? 1.
(z2 − x2 )(z2 − y2 )

A. –1 B. 0 2.
C. 1 D.
Select the correct answer using the code
79. If , then what given below:
is equal to? A. 1 only B. 2 only
A. 3 only B. 5 only C. Both 1 and 2 D. Neither 1 nor 2
C. –3 or 3 D. –5 or 5 85. A person wishes to fence 375m2 rectangular
80. If , then what garden. He has 65 m of barbed wire and is
able to fence only three sides of the garden.
is equal to? What is the perimeter of the garden?
A. A. 80 m B. 84 m
B. C. 90 m D. 100 m
C. 86. What are distinct prime factors of the
D. number 26381?
A. 29, 17, 37 B. 31, 17, 47
81. If three positive numbers are in the ratio
2:3:5 and the sum of their squares is 1368, C. 19, 37, 13 D. 23, 31, 37
then what is sum of the numbers?
87. Which one of the following is a factor of the
A. 30 B. 45 polynomial (x–1)(x–2) (x–4)–90?
C. 60 D. 75 A. x + 14 B. x – 14
82. Consider the following inequalities : C. x – 6 D. x – 7
88. What is the square root of
1. where a > b > 0
A. B.

2. only when a > b > 0 C. D.


89. What is the remainder after dividing the
Which of the above is/are correct? number 371000 by 9?
A. 1 only B. 2 only A. 1 B. 3
C. Both 1 and 2 D. Neither 1 nor 2 C. 7 D. 9
83. Let work done by (3n–1) men in (2n+1) days 90. The sum of LCM and HCF of two numbers is
be x and work done by (3n+1) men in 536 and the difference between LCM and
(4n–3) days be y. If x:y = 6:11, then what is HCF is 296. If one of the numbers is 104,
the value of n? then what is the other number?
A. 6 B. 7 A. 420 B. 480
C. 8 D. 9 C. 484 D. 506

PAGE 36
www.byjusexamprep.com

91. 20 men are supposed to complete a work in A. x2–6x+12=0


10 days. After working for 5 days, they
B. x2–8x+12=0
realise that only one–fourth of the work is
done. How many more men they need to C. x2+8x+12=0
employ to finish the work on time? D. x2–10x+12=0
A. 40 B. 30 96. For which values of k, does the equation
C. 20 D. 15 x2 – kx + 2 = 0 have real and distinct
solutions?
92. If x is a negative real number, then which of
the following are not correct? A.

1) There is some natural number k such that B. only


kx > 0
C. only
2) x2 + x > 0 always
D. or
3) 2x<x<–x
97. If , then what
4) x2 is always a rational number is equal to ?
Select the correct answer using the code A.
given below:
B.
A. 1, 2 and 3
C.
B. 1, 2 and 4
D.
C. 1, 3 and 4
98. If log10x + log10x2 = 2log10x+1, then what is
D. 2, 3 and 4 the value of x?
93. What is the sum of the linear factors (in x A. 1 B. 2
and y) of the expression C. 5 D. 10
A. 2x–3y B. 3x–2y 99. The LCM of two prime numbers p and q
C. 3x+2y D. 2x+3y is 2231, where p>q. What is the value of p–
q?
94. Which one of the following equations
does not have real roots? A. 67 B. 70

A. 2x2 + 16x + 3 = 0 C. 74 D. 81

B. 2x2 + 10x – 1 = 0 100. Three runners are running in a circular


track, and they complete one round in 20,
C. x2 – 8x + 1 = 0 30 and 35 minutes respectively. When will
they next meet at the starting point?
D. 4x2 + 9x + 6 = 0
A. After 3 hour 30 minutes
95. The sum and the product of the roots of a
quadratic equation are 7 and 12 B. After 4 hour 30 minutes
respectively. If the bigger root is halved and
C. After 3 hour
the smaller root is doubled, then what is the
resulting quadratic equation? D. After 7 hour

PAGE 37
www.byjusexamprep.com

ANSWERS KEY II 2021


ENGLISH

1 A 21 A 41 B 61 A 81 B 101 D
2 A 22 D 42 B 62 A 82 D 102 A
3 A 23 C 43 B 63 B 83 C 103 A
4 D 24 A 44 B 64 A 84 B 104 B
5 C 25 B 45 C 65 D 85 D 105 A
6 B 26 D 46 B 66 A 86 A 106 B
7 C 27 C 47 D 67 D 87 C 107 D
8 D 28 B 48 A 68 B 88 A 108 C
9 C 29 C 49 C 69 A 89 C 109 D
10 B 30 A 50 C 70 C 90 B 110 B
11 A 31 A 51 C 71 B 91 C 111 A
12 A 32 B 52 D 72 C 92 A 112 C
13 C 33 D 53 C 73 D 93 C 113 C
14 B 34 B 54 B 74 D 94 A 114 B
15 B 35 C 55 A 75 A 95 B 115 A
16 D 36 A 56 D 76 A 96 B 116 D
17 B 37 D 57 A 77 D 97 C 117 D
18 C 38 C 58 C 78 C 98 A 118 C
19 A 39 A 59 B 79 B 99 B 119 C
20 A 40 B 60 B 80 D 100 C 120 B

GENERAL KNOWLEDGE
1 A 21 B 41 B 61 D 81 B 101 D
2 B 22 A 42 A 62 D 82 A 102 C
3 C 23 C 43 D 63 B 83 A 103 A
4 D 24 D 44 C 64 D 84 D 104 C
5 B 25 A 45 B 65 D 85 B 105 C
6 B 26 A 46 B 66 C 86 C 106 C
7 A 27 C 47 B 67 A 87 A 107 C
8 D 28 B 48 B 68 B 88 A 108 A
9 A 29 A 49 B 69 D 89 A 109 A
10 B 30 B 50 B 70 B 90 A 110 B
11 B 31 B 51 B 71 C 91 A 111 A
12 C 32 C 52 A 72 C 92 B 112 B

PAGE 38
www.byjusexamprep.com

13 D 33 B 53 A 73 A 93 A 113 A
14 C 34 B 54 D 74 B 94 D 114 D
15 B 35 D 55 D 75 C 95 D 115 C
16 D 36 B 56 B 76 B 96 A 116 C
17 C 37 A 57 C 77 A 97 B 117 A
18 A 38 C 58 B 78 D 98 A 118 A
19 A 39 B 59 C 79 C 99 A 119 B
20 B 40 B 60 B 80 D 100 B 120 B

Elementary Mathematics
1 D 21 A 41 B 61 D 81 C
2 B 22 C 42 C 62 A 82 A
3 B 23 C 43 D 63 C 83 B
4 D 24 C 44 A 64 A 84 C
5 B 25 B 45 C 65 C 85 A
6 A 26 A 46 C 66 A 86 D
7 D 27 C 47 C 67 D 87 D
8 A 28 D 48 A 68 C 88 C
9 D 29 A 49 A 69 A 89 A
10 A 30 D 50 C 70 A 90 B
11 B 31 C 51 A 71 B 91 A
12 D 32 D 52 A 72 A 92 B
13 C 33 C 53 B 73 B 93 D
14 B 34 B 54 A 74 D 94 D
15 C 35 A 55 A 75 D 95 B
16 A 36 B 56 C 76 C 96 D
17 A 37 C 57 A 77 C 97 A
18 B 38 B 58 C 78 C 98 D
19 C 39 C 59 B 79 B 99 C
20 D 40 C 60 A 80 C 100 D

PAGE 39
CDS II 2021
www.byjusexamprep.com

(Solutions)
ENGLISH
1. Ans. A.
* ‘Bags’ is a plural noun, also ‘they’ is used after the blank. Hence, we understand that plural verb
‘are’ will be used.
2. Ans. A.
By, is the correct answer to fill in the blank.
3. Ans. A.
They are, is the correct usage because ‘polythene bags’ are being talked about.
4. Ans. D.
Option D - Along, is the correct answer for the given blank.
5. Ans. C.
because, is the correct answer to fill in the blank.
6. Ans. B.
but, is the correct answer to fill in the blank.
7. Ans. C.
the, is the correct answer to fill in the blank.
8. Ans. D.
to see to it, is the correct answer to fill in the blank.
9. Ans. C.
don’t, is the correct answer to fill in the blank.
10. Ans. B.
to, is the correct answer to fill in the blank.
11. Ans. A.
by, is the correct preposition to fill in the blank.
12. Ans. A.
On (Preposition), is the correct answer to fill in the blank.

PAGE 40
www.byjusexamprep.com

13. Ans. C.
in, is the correct answer to fill in the blank.
14. Ans. B.
To, is the correct preposition to fill in the blank.
15. Ans. B.
for is the correct preposition.
Additional information- If you are searching someone, correct preposition will be looking for.
16. Ans. D.
on, is the correct answer to fill in the blank.
17. Ans. B.
beside, is the correct answer to fill in the blank.
18. Ans. C.
away from, is the correct answer to fill in the blank.
19. Ans. A.
for, is the correct preposition to fill in the blank.
20. Ans. A.
by, is the correct answer to fill in the blank.
Additional Information - If we are using any kind of vehicle, we use the preposition ‘by’.
21. Ans. A.
but neither of them could help me, is the correct alternative to complete the sentence.
22. Ans. D.
since the AC was in a very low temperature setting, is the correct alternative to complete the
sentence.
23. Ans. C.
depending on what you prefer, is the correct alternative to complete the sentence.
24. Ans. A.
neither of us had the keys, is the correct alternative to complete the sentence.
25. Ans. B.
any one of them, is the correct alternative to complete the sentence.
26. Ans. D.
every ten minutes, is the correct alternative to complete the sentence.

PAGE 41
www.byjusexamprep.com

27. Ans. C.
over our house, is the correct alternative to complete the sentence.
28. Ans. B.
the entire evening, is the correct alternative to complete the sentence.
29. Ans. C.
that have unhappy endings, is the correct alternative to complete the sentence.
30. Ans. A.
that happened was my fault, is the correct alternative to complete the sentence.
31. Ans. A.
The birds die of hunger in winter because they do not move to warmer places.
32. Ans. B.
The birds do not foresee the catastrophe because they overlook a difficult situation.
33. Ans. D.
Human beings cannot be happy because they worry too much about future.
34. Ans. B.
Ill-educated – is the correct antonym of the ‘erudite’.
Erudite means having or showing great knowledge or learning.
35. Ans. C.
The central theme is the passage is - Progress of mankind.
36. Ans. A.
The general tone of the passage is that of – optimism.
37. Ans. D.
The willingness of consumers to go back to normal lifestyle indicates their sense of economic
security.
38. Ans. C.
The statement ‘People are moving on with their lives’, indicates that life must go on.
39. Ans. A.
Pent up demand means - desire (suppressed) to spend money once pandemic is controlled.
40. Ans. B.
Professional insecurity is the reason behind suppression of consumer demands.
41. Ans. B.
The most appropriate antonym is vague.
* Quiet mean with very little noise.

PAGE 42
www.byjusexamprep.com

* Precise means clear and accurate.


* Minute means a very short time.
* Vague means not clear or definite.
42. Ans. B.
The most appropriate antonym is slim.
* Stout means fat.
* Plump means pleasantly fat.
* Slim in an attractive way.
* Paunchy means having a large belly.
43. Ans. B.
The most appropriate antonym is - culminates.
* Inaugurate means to introduce a new official or leader.
* Initiate means to start something.
* Emanate means to produce or show something.
* Culminate means to reach a final result.
44. Ans. B.
The most appropriate antonym is - serious.
* Tragic means which makes someone very sad.
* Uproarious means provoking loud laughter.
* Jovial means happy and friendly.
* Serious means bad or dangerous.
45. Ans. C.
The most appropriate antonym is peace.
* Hullabaloo means a lot of loud noise.
* Uproar means a lot of noise or confusion.
* Accolade means an award or privilege granted as a special honour.
* Peace means a state of being calm.
46. Ans. B.
The most appropriate antonym is released.
* Impeded means to make it difficult for somebody.
* Confined means very small.
* Held means to keep something in a certain position.
* Released means to allow somebody to be free.

PAGE 43
www.byjusexamprep.com

47. Ans. D.
The most appropriate antonym is elation.
* Despondency means losing all hope.
* Determination means the process of deciding something officially.
* Dependant means a person who depends on somebody else for money, food and shelter.
* Elation means full of joy.
48. Ans. A.
The most appropriate antonym is - vindicated.
* Incriminated means to provide evidence that somebody is guilty of a crime.
* Indicted means to officially charge somebody with a crime.
* Arraigned means to bring a person to a court of law in order to formally accuse him/her of crime.
* Vindicated means vengeful.
49. Ans. C.
The most appropriate antonym is deprecate.
* Bluster means talk in a loud, aggressive way with little effect.
* Brag means to talk too proudly about something.
* Flaunt means to show something that you are proud of so that other people will admire it.
* Deprecate means express disapproval of.
50. Ans. C.
The most appropriate antonym is normal.
* Odd means strange.
* Peculiar means unusual.
* Idiosyncratic means unique to an individual.
* Normal means ordinary.
51. Ans. C.
Around – is acting as an intensifier in the given sentence.
* An intensifier is a word that strengthens or weakens another word.
52. Ans. D.
After – is a Conjunction.
Conjunction is a word that is used for joining other words.
53. Ans. C.
Extremely new – is an adjective.
* Adjective means a word that tells you more about a noun.

PAGE 44
www.byjusexamprep.com

54. Ans. B.
Less – is working as an adjective in the given sentence.
55. Ans. A.
Why – is an interjection.
* An interjection is a word or expression that occurs as an utterance of its own.
56. Ans. D.
While – is a Noun in the sentence.
57. Ans. A.
* Too – is an adverb, An adverb is a word or an expression that modifies a verb.
58. Ans. C.
* Opportunity – is a noun. A noun is a word that refers to a thing, a person, place or an animal.
59. Ans. B.
Solo – is working as an adjective in the sentence.
60. Ans. B.
* Definite article is the word ‘the’.
61. Ans. A.
* ‘Everything went well’ is the correct form.
62. Ans. A.
* Simple past tense will be used.
* Lavanya didn’t go
63. Ans. B.
* ‘has worn the same sweater’ is the correct form.
64. Ans. A.
* ‘It has begun to turn dark’ is the correct form.
65. Ans. D.
The given sentence is error-free.
66. Ans. A.
* ‘I have been playing’ is the correct form.
67. Ans. D.
The given sentence is error-free.
68. Ans. B.
* ‘She wasn’t very pleased’

PAGE 45
www.byjusexamprep.com

69. Ans. A.
* ‘Where do you come from?’ is the correct form.
70. Ans. C.
* ‘on a holiday’ is the correct form.
71. Ans. B.
The correct sequence for the jumbled parts of a sentence is – SRPQ.
* Columbus made his first voyage from Europe to America in 1992.
72. Ans. C.
The correct sequence for the jumbled parts of a sentence is – PRQS.
* Scientists have warned us about the effects of climate change.
73. Ans. D.
The correct sequence for the jumbled parts of a sentence is – QPSR.
* Industrial growth at a terrific speed is a potent cause of pollution in a big city.
74. Ans. D.
The correct sequence for the jumbled parts of a sentence is - QSPR.
* Today our environment is in a constant grip of filth and squalor and garbage.
75. Ans. A.
The correct sequence for the jumbled parts of a sentence is – SQRP.
* It is the duty of the government to ease out the monster of corruption everywhere.
76. Ans. A.
The correct sequence for the jumbled parts of a sentence is - QRPS.
* The poor visibility of the vehicles made the movement quite difficult.
77. Ans. D.
The correct sequence for the jumbled parts of a sentence is – SRQP.
* Though he was only a few yard away I couldn’t see Mohan clearly.
78. Ans. C.
The correct sequence for the jumbled parts of a sentence is – SPRQ.
* Here in Indian both winter and summer are truly extreme.
79. Ans. B.
The correct sequence for the jumbled parts of a sentence is - SRQP.
* Public culture is a mental and physical space where the basic ideas of the self and world view get
crystallised.

PAGE 46
www.byjusexamprep.com

80. Ans. D.
The correct sequence for the jumbled parts of a sentence is - RQPS
* The first job he performed on reaching the post was that he ordered my release from jail.
81. Ans. B.
written, is the correct answer to fill in the blank.
82. Ans. D.
off, is the correct answer to fill in the blank.
83. Ans. C.
under, is the correct answer to fill in the blank.
84. Ans. B.
went off, is the correct answer to fill in the blank.
85. Ans. D.
to, is the correct answer to fill in the blank.
86. Ans. A.
at, is the correct answer to fill in the blank.
87. Ans. C.
in the long run, is the correct answer to fill in the blank.
88. Ans. A.
climbing, is the correct answer to fill in the blank.
89. Ans. C.
of, is the correct answer to fill in the blank.
90. Ans. B.
having to, is the correct answer to fill in the blank.
91. Ans. C.
The most appropriate synonym is submerged.
* Swarmed means to fly or move in large numbers.
* Swam means to move your body through water.
* Floated means to move slowly through air or water.
* Submerged means to put under water.
92. Ans. A.
The most appropriate synonym is - arid.
* Humid means containing a lot of water.

PAGE 47
www.byjusexamprep.com

* Rainy means having or bringing a lot of rain.


* Moist means slightly wet.
* Arid means very dry.
93. Ans. C.
The most appropriate synonym is - disagreements.
* Confabulation means to talk informally.
* Interferences means the act of getting involved in a situation you are not a part of.
* Discrepancies means difference between two things that should be the same.
* Disagreements means a situation in which people have a different opinion.
94. Ans. A.
The most appropriate synonym is - impeded.
* Shaded means to protect something from direct light.
* Facilitated means to make something possible or easier.
* Poured means to make a liquid flow steadily.
* Impeded means to make it difficult for somebody to go forward.
95. Ans. B.
The most appropriate synonym is - intricacy.
* Conclusion means an opinion that you reach after thinking about something carefully.
* Grievance means something that you think is unfair and that you want to complain about.
* Complacency means a feeling of calm satisfaction with your own abilities.
* Intricacy means the complicated parts or details of something.
96. Ans. B.
The most appropriate synonym is inordinate.
* Inculpable means free from guilt.
* Unreasonable means unfair.
* Innocuous means not meant to cause harm or upset someone.
* Inordinate means much greater than usual or expected.
97. Ans. C.
The most appropriate synonym is - began.
* Ordered means to use your position to tell somebody to do something.
* Interfered means to get involved in a situation which does not involve you.
* Planned means to decide and organise.
* Began means to start.

PAGE 48
www.byjusexamprep.com

98. Ans. A.
The most appropriate synonym is sallow.
* Ruddy means having a healthy red colour.
* Glowing means something is very good.
* Radiant means greater happiness.
* Sallow means having a slightly yellow colour that does not look healthy.
99. Ans. B.
The most appropriate synonym is veto.
* Wan means looking pale.
* Weaken means to become less strong.
* Supersede means to take the place of somebody which has become old fashioned.
* Veto means to refuse to give official permission for an action or plan when other people have
agreed to it.
100. Ans. C.
The most appropriate synonym is appalled.
* Entreated means to ask somebody to do something in an emotional way.
* Pacified means to make somebody who is angry or upset be calm or quiet.
* Scintillated means sparkle.
* Appalled means feeling disgust at something unpleasant.
101. Ans. D.
A short sleep during the day, is the correct meaning of the given idiom.
102. Ans. A.
Life in its natural, embellished state, is the correct meaning of the given idiom.
103. Ans. A.
A person or a thing that appears threatening but is ineffectual, is the correct meaning of the given
idiom.
104. Ans. B.
careful about small amounts but careless about large amounts, is the correct meaning of the given
idiom.
105. Ans. A.
to listen carefully to something, is the correct meaning of the given idiom.
106. Ans. B.
someone who deserts one group to join another, is the correct meaning of the given idiom.

PAGE 49
www.byjusexamprep.com

107. Ans. D.
express your opinion, is the correct meaning of the given idiom.
108. Ans. C.
of highest possible quality, is the correct meaning of the given idiom.
109. Ans. D.
making or receiving payments surreptitiously, is the correct meaning of the given idiom.
110. Ans. B.
Suddenly and without much thought, is the correct meaning of the given idiom.
111. Ans. A.
The correct sequence for the given jumbled sentences is option A – SRQP.
112. Ans. C.
The correct sequence for the given jumbled sentences is option C – RSQP.
113. Ans. C.
The correct sequence for the given jumbled sentences is option C - SQPR
114. Ans. B.
The correct sequence for the given jumbled sentences is option B - PQSR
115. Ans. A.
The correct sequence for the given jumbled sentences is option A – SQRP.
116. Ans. D.
The correct sequence for the given jumbled sentences is option D - RSQP
117. Ans. D.
The correct sequence for the given jumbled sentences is option D - PRQS
118. Ans. C.
The correct sequence for the given jumbled sentences is option C – SRQP.
119. Ans. C.
The correct sequence for the given jumbled sentences is option C – SRQP
120. Ans. B.
The correct sequence for the given jumbled sentences is option B – PSQR,
Life in a village is ideal. Our villages are no longer dirty, rather, they are highly clean and tidy.
Moreover, they are disease-free. Because the village people are well-educated now. They can no
longer be considered backward.

PAGE 50
www.byjusexamprep.com

General Knowledge
1. Ans. A.
A three-member panel have chosen Odiya poet Rajendra Kishore Panda for the 2020 Kuvempu
Rashtriya Puraskar., which comes with a cash prize of Rs 5 lakh, a silver medal, as well as a citation.
Hence, option A is correct.
2. Ans. B.
Avani Lekhara, a trailblazing shooter, became the first-ever Indian woman to win two Paralympic
medals, adding the 50m Rifle 3 Position SH1 bronze to the gold she had won earlier in the Games
here.
Hence, option B is correct.
3. Ans. C.
Bagram Airfield-BAF, also known as Bagram Air Base, is an Afghan military base that was previously
the largest US military base in Afghanistan.
It is situated next to the ancient city of Bagram, 11 kilometres (6.8 miles) southeast of Charikar in
Afghanistan's Parwan Province.
Hence, option C is correct.
4. Ans. D.
Panchmuli Lake is located in Kevadia, Gujarat, near the Sardar Vallabhbhai Patel 'Statue of Unity.'
It made headlines because 194 crocodiles were relocated from the lake in the last two years for the
safety of tourists.
Hence, option D is correct.
5. Ans. B.
Pinaka is a multiple rocket launcher developed in India for the Indian Army by the DRDO (Defence
Research & Development Organisation).
The system can fire a salvo of 12 HE rockets in 44 seconds and also has a maximum range of 40 km
for the Mark-I and 60 km for the Mark-I enhanced version.
Hence, option B is correct.
6. Ans. B.
The 'ITAT e-dwar' portal aims to improve accountability, accessibility, and transparency in the ITAT's
day-to-day operations.
It would not only result in a reduction in paper use & cost savings, but also in a rationalisation of case
fixation, resulting in faster case disposition.
Hence, option B is correct.

PAGE 51
www.byjusexamprep.com

7. Ans. A.
Agni-P, also known as Agni-Prime, is a medium-range ballistic missile being developed by the DRDO
(Defence Research & Development Organisation) as a replacement for the Agni-I & Agni-II missiles in
the operational service of Strategic Forces Command, with major improvements in the form of the
composite motor casing, manoeuvrable reentry vehicle (MaRV), enhanced propellants, navigation,
& guidance systems.
Hence, option A is correct.
8. Ans. D.
According to Goa Chief Minister Pramod Sawant, the state has not observed a single rabies case in
the last three years.
This makes Goa the nation's first rabies-free state.
Hence, option D is correct.
9. Ans. A.
DPE was elevated to the status of a full-fledged Department in May 1990, when it was renamed the
Department of Public Enterprises (DPE). It is currently part of the Ministry of Finance.
The Department of Public Enterprises (DPE) is the nodal department for all Central Public Sector
Enterprises (CPSEs) and develops CPSE policy.
Hence, option A is correct.
10. Ans. B.
Rakesh Sharma, a former Indian military pilot and cosmonaut, became the first Indian and the
138th person to travel in space on this day in the year 1984.
After Rakesh Sharma, Kalpana Chawla, and Indian-American Sunita Williams, Sirisha Bandla became
the fourth person of Indian origin to accomplish the feat.
Hence, option B is correct.
11. Ans. B.
Jovenel Moise, the president of Haiti, was assassinated recently on 7th July 2021, the killing is being
blamed on a group of 28 foreign mercenaries. First Lady Martine Mose was also shot multiple times
during the attack and also was airlifted to the US for emergency treatment.
Hence, option B is correct.
12. Ans. C.
The Ministry of Cooperation is a govt ministry in India that was established in July 2021. The ministry
provides a distinct administrative, legal, as well as the policy framework for strengthening the
nation's cooperative movement.
Hence, option C is correct.

PAGE 52
www.byjusexamprep.com

13. Ans. D.
Indian Naval Ship INS Tabar had recently participated in a two-day naval exercise with the Italian
Navy. The exercise was conducted the Mediterranean Sea, the exercise benefited both parties by
improving interoperability as well as consolidating combined operations against maritime threats.
Hence, option D is correct.
14. Ans. C.
China has been declared "malaria-free" by the WHO (World Health Organization).
It is the result of a seven-decade-long, multi-pronged health strategy that has successfully eliminated
indigenous cases for four years in a row.
Hence, option C is correct.
15. Ans. B.
Bangabandhu Sheikh Mujibur Rahman has been awarded the Gandhi Peace Prize for 2020. The
Gandhi Peace Prize is an annual award established by the Govt. of India since 1995, the year of
Mahatma Gandhi's 125th birth anniversary. The award is open to all individuals, regardless of
nationality, language, race, caste, creed, or gender.
Hence, option B is correct.
16. Ans. D.
During the 44th meeting of UNESCO's World Heritage Committee, Dholavira, an archaeological site
of a Harappan-era city, was designated a UNESCO World Heritage Site.
Dholavira is also known as Kotada timba in the local dialect.
Hence, option D is correct.
17. Ans. C.
The Durand Cup is well-known for being Asia's oldest football tournament. The first edition of the
competition was held in the year 1888.
Since then, there have been 130 editions of the Durand Cup, with only wars and the COVID-19
pandemic interfering with the tournament's staging.
Hence, option C is correct.
18. Ans. A.
Karnam Malleswari is the first Indian woman to win a medal in the Olympics, she received the Arjuna
Award in the year 1994 and the Rajiv Gandhi Khel Ratna award, India's highest sporting honour, as
well as the civilian Padma Shri award in the year 1999.
Hence, option A is correct.
19. Ans. A.
The Russian Federation participated in the Tokyo Olympics, 2020 under the name ROC (Russian
Olympic Committee), Russia, or the Russian Federation, is the official designation of the world's
largest country, which finished fourth in Rio 2016 after winning 56 medals.
Hence, option A is correct.

PAGE 53
www.byjusexamprep.com

20. Ans. B.
The Union Cabinet, chaired by PM Shri Narendra Modi, has approved the launch of a new oil palm
mission called the national Mission on Edible Oils – Oil Palm (NMEO-OP) as a new Centrally
Sponsored Scheme with a special emphasis on the North East region as well as the Andaman and
Nicobar Islands.
Hence, option B is correct.
21. Ans. B.
The town of Chisht, from which the Sufi Chishti Silsila derives its name, is located in Central
Afghanistan. The Chishti silsila was introduced to India by Sufi saint Khwaja Muin-ud-Din Chishti. He
arrived in India in the year 1161 with Mahmud of Ghazni and stayed in Ajmer until 1236.
Hence, option B is correct.
22. Ans. A.
The rules for congregational worship (Sangat) involving collective recitation were organized by Guru
Nanak. He saw the god as formless and genderless and proposed a simple way to connect with him
by simply remembering and repeating his name.
Hence, option A is correct.
23. Ans. C.
Malti at Pithoragarh will be at more risk with respect to the earthquake hazard zone specified by the
Geological Survey of India. This zone includes Kashmir, the Central and Western Himalayas, North &
Middle Bihar, the Rann of Kutch, the North-East Indian region, and the Andaman & Nicobar Islands.
Hence, option C is correct.
24. Ans. D.
The Government wants to prepare a plan for drought-prone areas of the country. Kutch region will
be predominantly focused. Drought killed tens of millions of people in India during the 18th, 19th, and
20th centuries.
Hence, option D is correct.
25. Ans. A.
Four persons are travelling to the different states of India. I will advise the person travelling to
Arunachal Pradesh to protect oneself from blizzards. The blizzard hit Sela, Ahirgarh, and Nuranang in
the West Kameng district, disrupting traffic from Tezpur to Tawang.
Hence, option A is correct.
26. Ans. A.
Rajasthan state is not prominent for plantation agriculture. Plantation agriculture is described as the
development of one or more crops, usually cash crops, on a large scale. It is most prevalent in tropical
climates, where cash crops grow more naturally.
Hence, option A is correct.

PAGE 54
www.byjusexamprep.com

27. Ans. C.
The distance between Manali to Leh has been reduced by the Atal Tunnel. The tunnel shortens the
distance and time it takes to travel between Manali & Keylong on the way to Leh. The previous route,
which went via Gramphu, was 116 kilometres (72.1 miles) long and took 5 to 6 hours in good
conditions.
Hence, option C is correct.
28. Ans. B.
Gulf of Mannar biosphere reserve is the biosphere reserve that comprises islands with estuaries,
beaches, coral reefs, salt marshes and mangroves. The Gulf of Mannar Biosphere Reserve spans
1,050,000 hectares on India's south-east coast, across from Sri Lanka. In terms of marine biodiversity,
it is among the world's richest regions.
Hence, option B is correct.
29. Ans. A.
To make the right to free & compulsory education a fundamental right, the Constitution (Eighty-third
Amendment) Bill, 1997, not Constitution (Eighty-Fifth) Amendment Act, was introduced in
Parliament to add a new article, namely, article 21 A, conferring the right to free as well as
compulsory education on all children aged 6 to 14 years.
Hence, option A is correct.
30. Ans. B.
Elections for the Constituent Assembly were held for the first time under the Cabinet Mission Plan
of 1946. The Constituent Assembly drafted the Indian Constitution, which was implemented on
16th May 1946, as part of the Cabinet Mission Plan.
Hence, option B is correct.
31. Ans. B.
The procedure for amending the Indian Constitution is outlined in Article 368. A Constitution
Amendment Bill introduced under Article 368 may be introduced in either House of Parliament and
must be passed by a special majority in each House.
Part-xx Article 368 (1) of the Indian Constitution grants constituent power to make formal
amendments & empowers Parliament to amend the Constitution by adding, modifying, or repealing
any provision in accordance with the procedure set out in the Constitution, which differs from the
procedure for the ordinary legislation.
Hence, option B is correct.
32. Ans. C.
The internal disturbance is not a circumstance for the proclamation of Emergency by the President
of India under Article 352 of the Constitution of India. According to Article 352, if the President
believes that a grave emergency exists in which the security of India or any part of its territory is
threatened, whether, by war, external aggression, or armed rebellion, he may issue a proclamation
to that influence in respect of the whole of India or such part of its territory as may be specified in
the Proclamation Explanation.
Hence, option C is correct.

PAGE 55
www.byjusexamprep.com

33. Ans. B.
Akbar issued a Farman in 1598 permitting in the city of Cambay (Khambat), Gujarat, the construction
of a Church. An excerpt from a Farman (an imperial order) issued by Akbar in the year 1598 depicts
a church in Khambat.
Hence, option B is correct.
34. Ans. B.
Domingo Paes, the medieval traveller, has described the city of Vijayanagara. He went there as part
of a group of traders from Goa's former colony. His visit occurred during King Krishna Deva Raya's
reign, and Paes documented his impressions of the Vijayanagara state in his Chronica dos reis de
Bisnaga ('Chronicle of the Vijayanagar kings').
Hence, option B is correct.
35. Ans. D.
The extent of damage caused by an earthquake is not influenced by the climate of the area.
Factors Influencing the Impact of an Earthquake:
• Location
• Magnitude
• Depth
• Distance from epicentre
• Local geological conditions
• Secondary effects
• Architecture
Hence, option D is correct.
36. Ans. B.
An avalanche is a ‘Terrestrial’ type of disaster. Terrestrial disasters include earthquakes, landslides,
volcanic eruptions, avalanches, subsidence, etc.
Hence, option B is correct.
37. Ans. A.
An infection constantly maintained at a baseline level in a geographic area without external input is
known as Endemic. In epidemiology, an infection is seen to be endemic in a population when it is
constantly retained at a baseline level in a geographic area without the presence of external inputs.
Hence, option A is correct.
38. Ans. C.
Bhabar is a narrow belt that runs parallel to the Shiwalik foothills at the slope's break. As an outcome,
streams and rivers flowing from the mountains deposit heavy materials such as rocks & boulders
and, in some cases, disappear in this zone.
Hence, option C is correct.

PAGE 56
www.byjusexamprep.com

39. Ans. B.
There shall be one general electoral roll for each territorial constituency for election to either House
of Parliament or to the House or either House of a State's Legislature, and no person shall be
ineligible for inclusion in any such roll or claim to include in any special electoral roll.
Article 325 states that no one shall be denied inclusion on the electoral roll solely on the basis of
religion, race, caste, sex, or any combination of these factors.
Hence, option B is correct.
40. Ans. B.
Totalitarianism is a recent form of autocracy that is distinguished by the concentration of power in a
single centre, whether it is an individual dictator or a group of power holders such as with a
committee or a party leadership.
Freedom of speech, Freedom of religion, Artistic Freedom, Freedom to love or work wherever are
individual freedoms denied in a totalitarian state.
Hence, option B is correct.
41. Ans. B.
The federation of the United States of America (U.S.A) has been described as “an indestructible union
of indestructible states”. States in India, unlike most other federations, have no right to territorial
integrity. Any state's area, boundaries, or name can be changed unilaterally by the Parliament.
Furthermore, it only requires a simple majority, not a special majority.
Hence, Option B is correct.
42. Ans. A.
The End of History and the Last Man is a 1992 book of political philosophy written by American
political scientist Francis Fukuyama.
Hence, Option A is correct.
43. Ans. D.
The Gauhati High Court's main seat is in Guwahati, Assam. There are three outlying benches on the
court. They are as follows:
• The Kohima bench for Nagaland state (est. on 1stDecember 1972)

• The Aizawl bench for Mizoram state (est. on 5thJuly 1990)

• The Itanagar bench for Arunachal Pradesh state (est. on 12thAugust 2000)

Hence, Option D is correct.


44. Ans. C.
The Member abstains from voting in the House without prior permission is not a condition for the
disqualification of a Member of Parliament.
Hence, Option C is correct.

PAGE 57
www.byjusexamprep.com

45. Ans. B.
Granville Austin considered the Directive Principles of State Policy as aiming at ‘furthering the goals
of social exclusion. According to the famous author Granville Austin, the Indian Constitution is first
& foremost a social document.
Hence, Option B is correct.
46. Ans. B.
D performs her train journey by the shortest route from Bengaluru to New Delhi. She will not cross
river Yamuna while performing the journey.
Hence, Option B is correct.
47. Ans. B.
The tropic of Cancer passes through eight states in India - Gujarat, West Bengal, Madhya Pradesh,
Rajasthan, Chattisgarh, Tripura, Jharkhand, and Mizoram.
Mizoram is the northeast's southernmost landlocked state, sharing boundaries with three of the
seven sister states Tripura, Assam, & Manipur. In addition, the state shares a 722-kilometer border
with the neighbouring nations of Myanmar and Bangladesh.
Hence, Option B is correct.
48. Ans. B.
Lakshadweep is located in the emerald Arabian Sea, between 8º – 12º 13″ North latitude and 71º –
74º East longitude, 220 to 440 kilometres from that of the coastal city of Kochi in Kerala. Given its
lagoon area of approximately 4,200 sq. kms, (20,000 sq.).
Hence, Option B is correct.
49. Ans. B.
According to India's Census 2011, the adolescent population (age group of 10 to 19 years) of India,
account for one-fifth (19.1%) of the total population. By 2020, India's youth population is expected
to account for 34.33 per cent of the total population. Hence, statement 1 is incorrect.
In the adolescent population (age group of 10 to 19 years) of India, according to the 2011 Census,
there is the greater number of males than females. Hence, statement 3 is incorrect.
Hence, Option B is correct.
50. Ans. B.
J from Assam as per Census 2011, has the maximum density of population. Assam's population
density in 2011 was 398 people per square kilometre. Assam's population density increased from
138 people per square kilometre in 1961 to 398 people per square kilometre in the year 2011,
growing at a 19.79 per cent annual rate.
Hence, Option B is correct.

PAGE 58
www.byjusexamprep.com

51. Ans. B.
Swami Dayanand Saraswati took inspiration from Vedas. Swami Dayanand Saraswati was raised in a
devout, prosperous family, where he learned Sanskrit and absorbed the wisdom of Hindu scriptures,
particularly the Vedas.
Hence, Option B is correct.
52. Ans. A.
The call for renunciation of (all) voluntary associations with the (British) Government’ was given
during Non-Cooperation Movement. Gandhiji, emboldened by the movement's success, called for a
campaign of "non-cooperation" with British rule. Indians who wanted colonialism to end were told
not to attend schools, colleges, or law courts, and also not to pay taxes. In short, they were asked to
"renounce all voluntary association with the (British) Government."
Hence, option A is correct.
53. Ans. A.
Because Lakshadweep and Andaman & Nicobar Islands are in the same time zone, you can call
anyone during your normal business hours and the time would be the same in Andaman & Nicobar
Islands as it is in Lakshadweep.
The time difference between IST (India Standard Time) & Greenwich Mean Time (GMT+5.5) is 5:30
hours (5 hours 30 minutes). Indian Standard Time is based on 82.5° E longitude, that's just west of
Mirzapur, near Allahabad in the state of Uttar Pradesh. Hence, statement 3 is incorrect.
Hence, option A is correct.
54. Ans. D.
D had recently visited Khardung La, Nubra Valley and several Buddhist monasteries. She had visited
UT of Ladakh. The northernmost region of Jammu & Kashmir is Nubra Valley. The Valley, located
about 150 kilometres from Leh, is recognised as the Orchard of Ladakh and was originally named as
Ldumra, which means "Valley of Flowers."
Hence, option D is correct.
55. Ans. D.
Tamil Nadu is situated on India's eastern coast. South India typically receives rain from the south-
west monsoons. However, the south-west monsoon from the Arabian Sea is obscured by the
Western Ghats.
Hence, option D is correct.
56. Ans. B.
Soil Order is the most general level of classification in the USDA system of Soil Taxonomy. Entisols,
Mollisols, Alfisols, Inceptisols, Andisols, Spodosols, Ultisols, Oxisols, Histosols, Aridisols, Gelisols, and
Vertisols are the 12 orders of soils on the planet
Hence, option B is correct.

PAGE 59
www.byjusexamprep.com

57. Ans. C.
Shad witnessed contour bunding and contour ploughing while visiting Himachal Pradesh state in
India.
Hence, option C is correct.
58. Ans. B.
All returning migrants are the beneficiary of the “Garib Kalyan Rojgar Abhiyan scheme.
The Pradhan Mantri Garib Kalyan Yojana or Package is a comprehensive relief package worth Rs 1.70
Lakh Cr. Yojana for the poor to assist them in fighting the Corona Virus.
Hence, option B is correct.
59. Ans. C.
Lorenz curve – Inequality in the distribution of income or wealth
Phillips curve – Inflation and employment
Engel curve – Income and proportion of expenditure on food
Laffer curve – Tax rates and tax revenue
Hence, option C is correct.
60. Ans. B.
The market for soaps is a typical example of monopolistic competition. Monopolistic competition is
a market structure in which many firms produce similar, but not interchangeable, products. This type
of competitive market is classified as being somewhere between monopoly & perfect competition in
economics
Hence, option B is correct.
61. Ans. D.
Following the Constitution (One Hundred and First Amendment) Act, 2016, the Parliament of India
enacted quite a few GST Acts in the year 2017.(i) Central GST Bill, 2017, (ii) Union Territory GST Bill,
(iii) Integrated GST Bill, 2017, and (iv) Goods and Services Tax (Compensation to States) Bill, 2017.
The State Goods and Services Tax Act does not fall in this category.
Hence, option D is correct.
62. Ans. D.
The Directorate of Enforcement does not enforce the Prohibition of Benami Property Transaction
Act, 1988.
The Directorate of Enforcement investigates a violation of FEMA, 1999, which went into effect on
1st June 2000. Penalties of up to three times the monetary amount involved may be imposed.
Hence, option D is correct.

PAGE 60
www.byjusexamprep.com

63. Ans. B.
Price control is not a subject under the Concurrent List. The concurrent list allows the government
to make laws & regulations at both the state & federal levels. These laws & regulations are only
applicable to the subjects on the concurrent list.
Hence, option B is correct.
64. Ans. D.
The Indian Constitution grants the union extraordinary powers in response to various types of
emergencies. The Constitution's emergency provisions enable the federal govt. to acquire the
strength of a unitary system when any exigencies of the situation demand it.
Hence, option D is correct.
65. Ans. D.
The Constitution of India guarantees freedom of speech and expression. But the freedom is subject
to certain reasonable restrictions imposed by the State. These restrictions may relate to Defamation,
Decency or morality, and Incitement to an offence.
Hence, option D is correct.
66. Ans. C.
38th Amendment in the Constitution of India made a Proclamation of Emergency immune from
judicial review.
Hence, option C is correct.
67. Ans. A.
Warren Hastings was a British official who was elevated to the position of Governor-General after
the Regulating Act of 1773. The Regulating Act of 1773 was the British government's first step in
regulating & controlling the company's operations in India. It forbade company servants from
engaging in any private trade or accepting gifts or bribes from "natives."
Hence, option A is correct.
68. Ans. B.
Monotheism and internal social reform were preached by the Kherwar or Sapha Har movement of
the 1870s.
• The Kherwar Movement appears to have begun in the year 1868.
• It is also referred to as the Sapha Har movement.
• This movement popularized the concept of One God while also aiming for social reform.
• He demanded the establishment of a Santal raj.
Hence, option B is correct.
69. Ans. D.
In the year 1970, Poona Sarvajanik Sabha was established. Mahadev Govind Ranade founded the
organisation. Ranade is also credited with founding the Hindu reformist organisation Prarthna Samaj.
Hence, option D is correct.

PAGE 61
www.byjusexamprep.com

70. Ans. B.
Pearl fishing was a thriving industry during the Sangam period. The Pandyan port city of Korkai was
the epicentre of the pearl trade.
Hence, option B is correct.
71. Ans. C.
Samudragupta was praised in glowing terms in the Prayaga Prashasti.
In Samudragupta's prashasti, the poet lavished praise on him. According to this prashasti, he was a
wise king and a great poet. He was on par with the gods. He was a valiant ruler of the Gupta Dynasty
whose sphere of influence was vast.
Hence, option C is correct.
72. Ans. C.
The Self-Respect Movement was initiated by E.V. Ramaswamy Naicker. The Self-Respect Movement
is a South Asian movement that aims to create a society in which oppressed castes have equal human
rights, as well as to encourage backward castes to have self-respect in the context of a caste-based
society that considers them to be at the bottom of the hierarchy.
Hence, option C is correct.
73. Ans. A.
The location of the ancient city of Taxila (Takshshila), mentioned in ancient Indian texts, was
identified by Alexander Cunnigham. In the mid-nineteenth century, the ruins of Taxila were
rediscovered by the world-famous archaeologist Sir Alexander Cunningham. Taxila was designated a
UNESCO World Heritage Site in the year 1980.
Hence, option A is correct.
74. Ans. B.
Khema was not a Jain Acharya. Khema practised Buddhism and was a Buddhist nun. She is regarded
as one of the top Buddha's female disciple.
Hence, option B is correct.
75. Ans. C.
In India, the first major public appearance of Mahatma Gandhi was in the Inauguration of Banaras
Hindu University in 1916. Among those invited to this event were the princes and philanthropists
whose contributions had aided in the establishment of the BHU.
Hence, option C is correct.
76. Ans. B.
Patanjali's Mahabhaya is a commentary on selected rules of Sanskrit grammar from the Paini's
treatise, the Aadhyayi, as well as Katyayana's Varttika-stra, an elaboration of Paini's grammar. Hence,
statement 1 is incorrect.
Patanjali’s Mahabhashya is a book on grammar and refers to historical personalities only incidentally.
Hence, statement 2 is correct.
Hence, option B is correct.

PAGE 62
www.byjusexamprep.com

77. Ans. A.
When the quantity demanded changes due to a change in price, there is movement along the
demand curve. It is critical to differentiate between movement across a demand curve and a shift in
a demand curve.
Hence, option A is correct.
78. Ans. D.
The value of the advantages of the foregone alternative, the next best alternative that could have
been preferred but was not, is referred to as the opportunity cost.
Another way of looking at it is that "choosing is refusing;" one option can only be accepted by
rejecting another.
Hence, option D is correct.
79. Ans. C.
There was a stagnation in the spending on education as a per cent of GDP during 2014 – 19 (both
States and the Union Government together). According to the Economic Survey 2020-21, education
spending as a percentage of GDP remained stable at 2.8 per cent from 2014 to 2019 and increased
to 3-3.5 per cent in 2019-21. Experts believe that 6% of GDP should be allocated to education.
Hence, option C is correct.
80. Ans. D.
According to UNDP’s Human Development Report – 2020, India improved in the following
components in recent years.
• Life expectancy at birth
• Expected years of schooling
• GNI per capita
• Mean years of schooling
Hence, option D is correct.
81. Ans. B.
Both gain; it is a win-win situation is true of a pure voluntary exchange between two parties A and B.
A zero-sum game is one in which if one party loses, the other party wins, & the net wealth change is
zero.
Hence, option B is correct.
82. Ans. A.
The provision, the State shall take steps to separate the Judiciary from the Executive in the public
services of the State’ is incorporated Part-IV (Directive Principles of State Policy)of the Constitution
of India.
Hence, option A is correct.

PAGE 63
www.byjusexamprep.com

83. Ans. A.
Speaker may permit any Member to address the House in his/her mother tongue if he/she cannot
adequately express in either Hindi or English. Hence, statement 1 is correct.
Business of the Parliamentary Committees is transacted either in Hindi or in English. Hence,
statement 2 is correct.
The minutes of the Parliamentary Committees are prepared invariably in Hindi or English. Hence,
statement 3 is correct.
Hence, option A is correct.
84. Ans. D.
The term ‘State’, as defined in Article 12 of the Constitution of India refers to the Government of
India, Parliament of India, Governments and Legislatures of each State, and all local or other
authorities within the territory of India.
Hence, option D is correct.
85. Ans. B.
Bihar is the largest State in terms of seats in the Rajya Sabha among the given options.
• Andhra Pradesh has 11 Rajya Sabha seats.

• Bihar has 16 Rajya Sabha seats.

• Rajasthan has 10 Rajya Sabha seats.

• Karnataka has 12 Rajya Sabha seats.

Hence, option B is correct.


86. Ans. C.
Article 243 D of the Constitution of India has provision been made for reservation of seats for women
in Panchayats. Clause (3) of Article 243D of the Indian Constitution ensures women's participation in
Panchayati Raj Institutions by mandating a one-third reservation for the women out of the total
number of seats to be filled by direct election & the number of Panchayat chairpersons.
Hence, option C is correct.
87. Ans. A.
The painted illustration of the moving of the Ashoka Pillar at-Topra is found in Tarikh-i-Firuz Shahi.
The original home of the Delhi-Topra Ashokan pillar, one among many Ashoka pillars moved from
Topra to Feroz Shah Kotla in Delhi in the year 1356 CE by Firuz Shah Tughlaq.
Hence, option A is correct.
88. Ans. A.
Srirangam is a temple of the Vaishnavite tradition. The Sri Ranganathaswamy Temple is a Hindu
temple in Srirangam, Tiruchirapalli, Tamil Nadu, the State of India, dedicated to Ranganatha, a form
of the Supreme God, Maha Vishnu.
Hence, option A is correct.

PAGE 64
www.byjusexamprep.com

89. Ans. A.
The biography of Shaikh Muinuddin Chishti, authored by Jahanara, is known as Munis al Arwah.
Jahanara wrote Mu'nis al-Arw, a biography of Moinuddin Chishti, the pioneer of the Chishti Order in
India, and also Rislah-i ibyah, a biography of Mullah Shah, in that she also described her initiation by
him.
Hence, option A is correct.
90. Ans. A.
Overthrow of alien rule is not a political method of the moderates in the National Movement.
The moderates used the following methods:
• Petitions are being sent to government officials.
• Memorandums are being sent to officers.
• Resolutions are being passed.
• Delegations are being sent to England.
Hence, option A is correct.
91. Ans. A.
Fertilizers provide three main macronutrients: Potassium, Nitrogen, and Phosphorous are normally
which are consumed in larger quantities and derived from the soil in the form of inorganic salts. Iron
is a micronutrient for plant growth which is not available in fertilizers. Hence, the correct answer is
A
92. Ans. B.
Initial velocity, u= 0 km/h
Final velocity, v= 20 km/h
Time= 10 minutes
By using formula,

a= = = 2 km/m

To convert minutes into hour


= 2 × 60 = 120 km/h2
Hence, the correct answer is B
93. Ans. A.
The bread mould, yeast and mushroom shows saprophytic mode of nutrition. Saprophytes undergo
extracellular digestion to digest the dead and decaying matter. They secrete digestive substances
into the surrounding environment and break down the organic matter into simpler substances. The
nutrients thus produced are absorbed directly through the cell membranes of the organisms. Hence,
the correct answer is A

PAGE 65
www.byjusexamprep.com

94. Ans. D.
As the solution turns red litmus blue which suggests that the solution is basic in nature.
As we know that, for an acidic solution pH <7
For a basic solution, pH >7.
Hence, option (D) is the correct answer.
95. Ans. D.
One end of the solenoid acts as a magnetic north pole, while the other end acts as the magnetic
south pole. The magnetic field lines inside a current carrying a long solenoid are in the form of parallel
straight lines. This indicates that the magnetic field is the same at all points inside the solenoid. It
proves that the field is uniform inside the solenoid. Hence, the correct answer is D
96. Ans. A.
A population is the number of organisms of the same species that live in a particular geographic
area at the same time, with the capability of interbreeding.
Hence, the correct answer is A
97. Ans. B.
Refraction is the process of bending of light rays after entering a medium where its speed is different.
During the process, if a ray of light passes from a rarer medium to a denser medium, then it bends
towards the normal to the boundary between the two mediums. The number of bending rays
depends upon the indices of refraction of the two media.
Hence, when a ray of light travels from a rarer medium to a denser medium, it bends towards the
normal.

Hence, the correct answer is B


98. Ans. A.
Dilute acids react with relatively reactive metals such as magnesium, aluminium, zinc and iron. The
products of the reaction are a salt plus hydrogen gas.
The reactivity of these metals with dilute hydrochloric acid depends upon the reduction potential of
the corresponding metal which can also be inferred from the positions of these metals in reactivity
series.
Mg lies at the top , so will be most reactive than comes Al then Zn and Fe least reactive. Hence, the
correct answer is A

PAGE 66
www.byjusexamprep.com

99. Ans. A.
Small amounts of HCl gas for laboratory use can be generated in an HCl generator by dehydrating
hydrochloric acid with either sulfuric acid or anhydrous calcium chloride. Alternatively, HCl can be
generated by the reaction of sulfuric acid with sodium chloride: NaCl + H2SO4 → NaHSO4 + HCl.
Here, other calcium salts with other halogens cannot be used as this may create disturbance in the
reaction due to the presence of different types of halide ions. Hence, the correct answer is A
100. Ans. B.
Alloys commonly used for electrical soldering are 60/40 Sn-Pb, which melts at 188 °C (370 °F), and
63/37 Sn-Pb used principally in electrical/electronic work. This mixture is a eutectic alloy of these
metals, which: has the lowest melting point (183 °C or 361 °F) of all the tin-lead alloys.
Hence, the correct answer is B
101. Ans. D.
Given,
Diameter of a circular track= 100m

Radius = = = 50 m

Time taken by the athlete to complete one round of circular track = 20 seconds
Now, time = 1min 10 sec = = 70 seconds

Number of rounds covered by the athlete = = = 3.5

Circumference of a circle = 2πr


Distance covered in 1 round = Circumference of the circular track

Thus, the circumference of the circular track =

Distance travelled in 70 seconds =

= = = 1100m

Thus, the distance covered by the athlete is 1100m.


For each complete round the displacement is zero.
The displacement will be zero because of 3 complete rounds,.
At the end of his motion, the athlete will be in the diametrically opposite position. That is,
displacement = diameter = 100 m.
Hence, the distance covered is 1100 m and the displacement is 100 m.
Hence, the correct answer is D

PAGE 67
www.byjusexamprep.com

102. Ans. C.
To form a real and enlarged image by a convex lens, then the object must be placed between the
principal focus and twice the focal length in front of the lens.

This ray diagram explains that for the object AB, the image will be A’B’. At twice the focal length, the
image formed by the convex lens is real and from the principal focus it will be an enlarged image.
Hence, the correct answer is C
103. Ans. A.
It is located in the leaves and internodes at the intercalary position.
These help to increase the length of the internode.
Hence, the correct answer is A
104. Ans. C.
All ecosystem is characterized by two basic features- the unidirectional flow of energy and cycling of
materials.
At each tropic level, most of the energy available is utilized for life processes and only ten percent of
the available energy is passed on to the next level so because of this the higher trophic levels have
substantially less energy content and the number of trophic levels in a food chain is limited. Hence,
the correct answer is C
105. Ans. C.
Only allotrope of carbon which is good conductor of electricity is graphite. This electrical
conductivity in graphite arises due to the presence of free electron, generally the 4th electron of the
valence shell.
On the other hand , diamond and fullerene aur insualting as they don’t have any free electron which
can make them conducting. Hence, the correct answer is C
106. Ans. C.
Tall and dwarf in the ratio of 1 : 1. Crossing of homozygous tall TT with a homozygous dwarf (tt), is
called Monohybrid cross. In this, all F1 progeny are heterozygous tall (Tt). In the F2 generation, 1
homozygous tall (TT), 2 heterozygous tall (Tt), homozygous dwarf (tt), genotypically. Phenotypically,
3 tall and 1 dwarf plants are present. Hence, the correct answer is C
107. Ans. C.
The speed of light is different in different media due to the difference in resistance provided by the
medium to the light rays. This changes the speed of light and is termed as refraction. Even though its
phase velocity is changed, the frequency remains constant in refraction. Hence, the correct answer
is C

PAGE 68
www.byjusexamprep.com

108. Ans. A.
The correct answer is Cupric carbonate. Copper reacts with moist carbon dioxide in the air to form
green-colored cupric carbonate. The green color of copper appears due to the corrosion of copper
due to reaction with oxygen and carbon dioxide for copper carbonate. Cupric carbonate is a green
color compound. Hence, the correct answer is A
109. Ans. A.
Insects are characterized by having three pairs of jointed legs from other arthropods; an abdomen
that is divided into 11 segments and lacks any legs or wings; and a body split into three sections with
one pair of antennae on the head (head, thorax, and abdomen). Sometimes, insects also have one
or two wing pairs. Hence, the correct answer is A
110. Ans. B.
Roasting is a process of heating a sulfide ore to a high temperature in the presence of air. It is a step
in the processing of certain ores. More specifically, roasting is often a metallurgical process involving
gas–solid reactions at elevated temperatures with the goal of purifying the metal components.
Calcination, the heating of solids to a high temperature for the purpose of removing volatile
substances, oxidizing a portion of mass, or rendering them friable. Calcination, therefore, is
sometimes considered a process of purification.
Smelting is a form of extractive metallurgy to produce a metal from its ore. Smelting uses heat and
a chemical reducing agent to decompose the ore, driving off other elements as gasses or slag and
leaving just the metal behind.
Incineration is a waste treatment process that involves the combustion of substances contained in
waste materials
Hence, the correct answer is B
111. Ans. A.

Pressure =

Pressure is inversely proportional to area. If the area is smaller, the pressure is greater and when the
area is larger the pressure is smaller.
While standing on loose sand, the area of our feet is smaller, therefore, the pressure of our body is
greater hence, we go deep into the soil but when we lie down on the sand then the area of our body
is larger, therefore it exerts a smaller pressure and we do not go deep in the soil. Hence, we apply
more pressure while standing than the lying down due to less area of contact
(Force=weight=constant).
Hence, the correct answer is A
112. Ans. B.
The ammeter reading will be doubled because, with the decrease in length, the resistance of the
circuit also decreases. Hence, the current in the circuit will be increased.

PAGE 69
www.byjusexamprep.com

Ammeter reading will be increased because of R = .

Where L is the length of the wire and A is an area of the cross-section of the wire. So, When the
length is changed to half by keeping the area of cross-section is constant. Then the resistance will
also be decreased as it is inversely proportional to the area of the cross-section, so with the decrease
in length, the resistance of the wire will decrease, and the ammeter reading will be increased. Hence,
the correct answer is B
113. Ans. A.
Leaves of the nettle plant secrete methanoic acid which causes a painful sting on touching.
Citric acid is an organic compound with the chemical formula HOC(CH₂CO₂H)₂. Usually encountered
as a white solid, it is a weak organic acid. It occurs naturally in citrus fruits.
Tartaric acid is a white, crystalline organic acid that occurs naturally in many fruits, most notably in
grapes, but also in bananas, tamarinds, and citrus.
Acetic acid, systematically named ethanoic acid, is an acidic, colourless liquid and organic compound
with the chemical formula CH₃COOH. Vinegar is no less than 4% acetic acid by volume. Hence, the
correct answer is A
114. Ans. D.
A universal indicator is a pH indicator made of a solution of several compounds that exhibits several
smooth colour changes over a wide range pH values to indicate the acidity or alkalinity of solutions.
Universal indicator is a mixture of many different indicators which gives different colours at different
pH values of the entire pH scale. It is used to obtain an idea of how acidic or basic a substance is.
So, all the three statements are correct for universal indicator hence, option D is correct.
115. Ans. C.
Alkali is a base that dissolves in water. So, all alkali are bases but all bases cannot be an alkali as
some of them may not dissolve in water. So the first statement is incorrect but second statement is
correct.
Also, alkalis are soapy to touch, butter in taste and are corrosive in nature. Hence, the correct answer
is C
116. Ans. C.
Given,
Voltage, V= 110V
Current, I= 0.2A
Power, P = ?
We know that,
P=V×I
= 110 × 0.2

PAGE 70
www.byjusexamprep.com

P = 22W
Hence, the correct answer is C
117. Ans. A.
Speed of sound wave, v = = = 1200 m/s
Distance, d= 2.4km= 2400m

Time, t = = = 2seconds

Hence, the correct answer is A


118. Ans. A.
The oxygenated blood is carried from lungs to the left side of the heart. From lungs, pulmonary vein
carries oxygenated blood to the left atrium. From there oxygenated blood is circulated to the
systemic circulation again.
Hence, the correct answer is A
119. Ans. B.
All variations in a species do not have equal chances of survival. Some variations may be so drastic
that the new DNA copy cannot work with the cellular apparatus it inherits. Such, a newborn cell dies
soon. Variations can be beneficial or harmful according to the ecological niche of the organism.
Hence, the correct answer is B
120. Ans. B.
Characteristics of protochordate are as follows-
1. They are generally found in marine water.
2. Their body is bilaterally symmetrical, triploblastic, and coelomated.
3. At a certain stage of their lives, their body develops a long, rod-like structure for support called
the notochord.
4. They exhibit organ system level of organization.
Hence, the correct answer is B

PAGE 71
www.byjusexamprep.com

Elementary Mathematics
1. Ans. D.
No. of employees in each category are:
Cross check the options we get that total strength of C & D remains same.
C= 400+600 =1000
D= 450+550 = 1000
2. Ans. B.

Percentage increase =

Comparing the % increase of all categories we find that maximum increase is of B i.e. 108.33%
3. Ans. B.

Percentage increase = =

= 25%
4. Ans. D.

On observation from the table, we find that category A&D both have 37.5% increase.
So, option D is correct.
5. Ans. B.
Total car production in 2014 = 300 + 200 + 500 = 1000
Total car production in 2018 = 700 + 500 + 800 = 2000
Percentage increase =

= 100%
So correct option is B
6. Ans. A.

Percentage increase =

PAGE 72
www.byjusexamprep.com

the percentage increase in production of cars over the previous year’s production greater than 20%
= 5 times
7. Ans. D.

Percentage increase =

Minimum is for year 2018 i.e 11.11%


8. Ans. A.

Percentage increase =

Minimum is for year 2018 i.e 11.11%


9. Ans. D.
Let the consecutive numbers be x,x+1,x+2,x+3,x+4
product is equal to one of these integers is only possible when one number is zero.
Zero can be at any of the five places so possible sequence that can be made is 5.
10. Ans. A.
For minimum sequence is -4,-3,-2,-1,0
P = –4
For maximum sequence is 0,1,2,3,4
Q=4
Arithmetic mean = (-4+4)/2 =0
Option A is correct.
11. Ans. B.
Given root will satisfy the equation,

PAGE 73
www.byjusexamprep.com

Putting the value of a in equation we get


4x²-16x+15=0
Sum of root = -b/a
Product of root = c/a

Option B is correct.
12. Ans. D.

13. Ans. C.
Speed of boat = 30kmph
Speed of stream = s kmph

PAGE 74
www.byjusexamprep.com

Downstream speed = (30+s)


upstream speed = (30-s)
according to question
total time = 9/2
time taken upstream+ downstream = 9/2

14. Ans. B.
Present age of father = x
Present age of all children = x
After 10 years
Age of father = x + 10
Age of all children = x + (4 × 10) = x + 40
According to question,

x = 40year
Present age of father = 40 year
15. Ans. C.
Let, Numerator =x
Denominator =y
x + y = 10………equation 1

Correct option is C

PAGE 75
www.byjusexamprep.com

16. Ans. A.
Unique solution condition

Option A is correct.
17. Ans. A.

Second condition

18. Ans. B.

Lies between 16-17 minute.

PAGE 76
www.byjusexamprep.com

19. Ans. C.
Efficiency of Sheela =1/2
Efficiency of Meena = 2/3
Their combined efficiency = 1/2+2/3
= 7/6
Together, they take stitch 7/6 suit in a day.
Time taken to stitch 30 suits = 30×7/6
=35 days
20. Ans. D.

option D is correct.
21. Ans. A.
According to the questions

Now q increase by 20% so new value of q

Now r is decreasing by 20%


So new value of r

Now new value of p

PAGE 77
www.byjusexamprep.com

Now percentage change in p

22. Ans. C.

Correct option is C.
23. Ans. C.

PAGE 78
www.byjusexamprep.com

24. Ans. C.
According to question

25. Ans. B.
Saving are as follows:
2000,3000,4000,5000,6000……
The sequence is following arithmetic progression
a=2000, d= 1000, n=?

Neglect negative value. We get 17 as the answer.


In 17 year the given saving will be done.
26. Ans. A.

PAGE 79
www.byjusexamprep.com

The volume of the solid is equal to that of a sphere of radius x


27. Ans. C.

28. Ans. D.

Volume of the solid =

The volume left in the cylinder = internal volume of cylinder – volume of solid.

29. Ans. A.

PAGE 80
www.byjusexamprep.com

30. Ans. D.
Area of major segment= whole circle- area of minor segment.

31. Ans. C.
Suppose radius of circle with centre is a, b, c
a + b = 6, b +c=8, c +a=10
2(a +b + c) = 6+8+10
2(a+ b+ c) = 24
a + b + c=12
32. Ans. D.

33. Ans. C.

PAGE 81
www.byjusexamprep.com

Both the statement are correct.


34. Ans. B.

Draw AE ⊥ BC
In a triangle AEB and Triangle AEC, we have
AB = AC
AE = AE and because AB = AC

Since △AED and △ABE are right-angled triangles at E.

We can write AB = AC
Then
AC2 – AD2 = CD × BD
35. Ans. A.
By putting the given values in equation

PAGE 82
www.byjusexamprep.com

AB² - 5² = 4×6
AB² = 24+25 =49
AB = 7cm
36. Ans. B.
As given in the questions
AB = 1.6 cm
BQ = BC/2 = 6.3/2 = 3.15
Now

37. Ans. C.

In triangle ABQ
AB²+BQ² = AQ²
In triangle PBC
PB²+BC² = PC²
Adding both the above equation
AB²+BQ²+PB²+BC²=AQ²+PC²
AC²+PQ²= AQ²+PC²
By using mid point theorem
PQ=0.5 AC
AC²+(0.5AC)² = AQ²+PC²
AQ²+PC²=1.25 AC²
38. Ans. B.
4(CP²-AQ²)
= 4(PB²+BC²-AB²-BQ²)

PAGE 83
www.byjusexamprep.com

39. Ans. C.

Triangle AOP is isosceles triangle, so angle opposite to equal side will be equal.
Sum of all three angle of triangle =180

So, Angle BAP = 45


40. Ans. C.

Angle subtended by chord is double to the angle subtended on circumference.


Angle subtended by chord AP on O =90 degree
Angle subtended by chord AP on Q =45 degree
41. Ans. B.
As we know the radius of circum–circle of a right-angled triangle is half of the hypotenuse
So hypotenuse of the right angle triangle = 2 × radius of circum – circle of a right-angled triangle is =
2 × 10 = 20 cm

PAGE 84
www.byjusexamprep.com

42. Ans. C.

Difference of diameter= 10-8 = 2cm


43. Ans. D.
Area of square with side ‘a’ : P=a²

Area of square with side ‘a 2 ’ : Q=2a²

Option D is correct.
44. Ans. A.
Area of rhombus=1/2(product of diagonals of rhombus)

PAGE 85
www.byjusexamprep.com

45. Ans. C.
60 minute is equivalent to 360 degree.
So 20 minute is equivalent to 120 degree.

46. Ans. C.

From the floor part we will find out diagonal of floor i.e. d

As shown in figure, a right-angle triangle is formed with hypotenus as 29 base as d and height as h

47. Ans. C.

number of bottles required=108


48. Ans. A.
Volume of spherical shell=

PAGE 86
www.byjusexamprep.com

49. Ans. A.

50. Ans. C.

51. Ans. A.

PAGE 87
www.byjusexamprep.com

which is equals to 1330 cm²


Option A is correct.
52. Ans. A.

correct option is option A.


53. Ans. B.

54. Ans. A.

55. Ans. A.

PAGE 88
www.byjusexamprep.com

56. Ans. C.

57. Ans. A.

58. Ans. C.

PAGE 89
www.byjusexamprep.com

59. Ans. B.
In the triangle ACD, AD and DC are equal, so

Now,

Height of AB = 10 + 5 = 15 m
60. Ans. A.

PAGE 90
www.byjusexamprep.com

61. Ans. D.

In triangle AOC,

So option D is correct.
62. Ans. A.

PAGE 91
www.byjusexamprep.com

63. Ans. C.

64. Ans. A.

PAGE 92
www.byjusexamprep.com

65. Ans. C.

66. Ans. A.

67. Ans. D.

68. Ans. C.

PAGE 93
www.byjusexamprep.com

69. Ans. A.

=0
70. Ans. A.

71. Ans. B.

PAGE 94
www.byjusexamprep.com

72. Ans. A.
for a zero at the end there should be a product of 2 and 5. The power of 5 is less than the power of
2. So we have to find the number of 5.
We have
Breaking it into power of 5 and 2.
The terms that contain 5 are
Total number of 5’s = 46 + 41 + 36 + 31 + 2(26) + 21 + 16 + 11 + 6 + 2(1) = 262
So, the number of 0’s at the end of the expression = 262
73. Ans. B.

74. Ans. D.

PAGE 95
www.byjusexamprep.com

75. Ans. D.

76. Ans. C.

77. Ans. C.

PAGE 96
www.byjusexamprep.com

78. Ans. C.

79. Ans. C. B.

PAGE 97
www.byjusexamprep.com

80. Ans. C.

81. Ans. C.
Let the number is 2x, 3x and 5x.
According to the question,

By putting the value, the numbers are 12, 18 and 30.


Sum of 12, 18 and 30 = 60
82. Ans. A.

PAGE 98
www.byjusexamprep.com

83. Ans. B.

Value will be multiple of 7/2 so next integer value will be 7.


84. Ans. C.

We have

PAGE 99
www.byjusexamprep.com

So,

Similarly, we can get,

Now,

SO, both ethe options are correct.


85. Ans. C. A.
Area of rectangle = 375
L × B = 375
Factorizing 375 we get 15 × 25
Let us cross check as the perimeter of 3 side is given
Let L = 15 and B = 25
2L + B = 65 (given)
30 + 25 = 65
55 = 65 WRONG
Let L = 15 and B = 25
L + 2B = 65
15 + 50 = 65
65 = 65 correct
Perimeter = 2(l + b) = 2(15 + 25)=80m

PAGE 100
www.byjusexamprep.com

86. Ans. D.

Prime factors are 23,31,37


87. Ans. D.

88. Ans. C.

PAGE 101
www.byjusexamprep.com

89. Ans. A.

Any power of 1 will be 1 so remainder will be 1 as 36 will be completely divided by 9.


90. Ans. B.
LCM + HCF = 536
LCM – HCF = 296
Adding both the equation
2(LCM) = 832
LCM = 416
HCF = 120
LCM X HCF = product of two number
416 x 120 = 104 x second number
Second number = 480
91. Ans. A.

92. Ans. B.
Statement 1: product of negative and positive is always negative. Thus contradict in statement 1
and NOT CORRECT.
Statement 2: is incorrect. Equation doesn’t hold true always. Only when x<-1
Statement 3: is true.
Statement 4: is incorrect as its not necessary that it will be rational always.
Statement 1,2and 4 are incorrect.
93. Ans. C. D.

PAGE 102
www.byjusexamprep.com

94. Ans. D.

95. Ans. C. B.

96. Ans. D.

PAGE 103
www.byjusexamprep.com

97. Ans. A.

98. Ans. D.

99. Ans. C.
Prime number so HCF=1
LCM=2231
LCM×HCF=product of two number
2231×1= p.q
(23)(97)=p.q
P=97 and q= 23
p-q=97-23=74
100. Ans. D.
To get next meet we will find out LCM of (20,30,35) = 420min
420 min= 7 hours
They we meet again after 7 hours.

PAGE 104
www.byjusexamprep.com

CDS I 2021
ENGLISH
Direction: Each item in this section has a 6. Language is (a)/ an essential modes of
sentence with three underlined parts labelled as communication (b) / and every language
(a), (b) and (c). Read each sentence to find out matters in communication. (c)/ No error
whether there is any error in any underlined part (d).
and indicate your response on the Answer Sheet
A. (a) B. (b)
against the corresponding letter i.e., (a) or (b) or
(c). If you find no error, your response should be C. (c) D. (d)
indicated as (d).
7. The matter was been resolved (a)/ after
1. He had been doing (a) / the job of a long deliberations (b)/ among the members
physician (b)/during the 1990s (c)/No error of the team. (c)/ No error (d)
(d)
A. (a) B. (b)
A. (a) B. (b)
C. (c) D. (d)
C. (c) D. (d)
8. Care has been taken (a) / to ensure safety
2. He said (a) / that he preferred (b)/coffee and security (b)/ of the participants in the
than tea (c)/ No error (d). event. (c)/ No error. (d)
A. (a) B. (b) A. (a) B. (b)
C. (c) D. (d) C. (c) D. (d)
3. All that (a) / glitters (b) / are not gold (c) / 9. Indian film industry (a) / is considered to be
No error (d). (b) / one of the symbol of cultural and social
life.(c) / No error. (d)
A. (a) B. (b)
A. (a) B. (b)
C. (c) D. (d)
C. (c) D. (d)
4. A species is considered (a)/ endangered
when it is (b) in a very high risk of extinction 10. It is still true (a) / that the Indian economy
in the world. (c)/ No error (d). (b) / is dependent on monsoon (c) / No
error. (d)
A. (a) B. (b)
A. (a) B. (b)
C. (c) D. (d)
C. (c) D. (d)
5. Our greatest glory (a)/ is not in never falling
(b)/ but in rising every time we fell. (c)/ No Direction: Given below are some idioms/phrases
error(d). followed by four alternative meanings to each.
Choose the response (a), (b), (c) or (d) which is
A. (a) B. (b)
the most appropriate expression and mark you
C. (c) D. (d) response in the Answer Sheet accordingly.

PAGE 105
www.byjusexamprep.com

11. Be in eclipse 18. Run wild


A. Less successful B. Feeling happy A. To run like a wild animal
C. Very successful D. Being defeated B. To treat anyone like a wild
12. Ways and means C. To fell like a wild animal
A. A technique D. To grow without any control
B. Methods of achieving something 19. Wind down
C. Norms and regulations of doing A. To relax after a period of activity
something
B. To act furiously after a period of silence
D. Improving one’s way of doing
C. To speak out the truth to people
13. Up in arms
D. To act on the ground
A. Very happy B. Very satisfied
20. Mellow out
C. Very angry D. Feeling fine
A. To feel bad about other’s enjoyment
14. Big ticket
B. To like and dislike people concurrently
A. Very less B. Very costly
C. To enjoy oneself without doing much
C. Very easy D. Not much
D. To work hard and doing much work
15. Bolt form the blue
Direction: In this section you have two short
A. An event or piece of news which passages. After each passage, you will find some
unexpected items based on the passage. Read the passages
and answer the items based on them. You are
B. Desirable event or news
required to select your answers based on the
C. An event which takes place as planned content of the passage and opinion of the author
only.
D. News which has been long expected,
but arrives late Passage-I
16. Be a law unto yourself Conventional education makes independent
thinking extremely difficult. Conformity leads to
A. Behave unconventional and
mediocrity. To be different from the group or to
unpredictable
resist environment is not easy and is often risky
B. Abide by laws and order as long as we worship success. The urge to be
successful, which is the pursuit of reward
C. Ask others to follow the law whether in the materials or in the so-called
D. Create law and order for others spiritual sphere, the search for inward or
outward security, the desire for comfort — this
17. Spiff up whole process smothers discontent, put an end
A. To make oneself look neat to spontaneity and breed fear; and fear blocks
the intelligent understanding of life. With
B. To make oneself look untidy increasing age, dullness of mind and heart sets
C. To make oneself look arrogant in. In seeking comfort, we generally find a quiet
corner in life where there is a minimum of
D. To appear on the stage as a baboon conflict, and then we are afraid to step out of

PAGE 106
www.byjusexamprep.com

that seclusion. This fear of life, this fear of 25. The word 'uncomprehendingly' in the
struggle and of a new experience, kills in us the passage means
spirit of adventure; our whole upbringing and
A. Achieve success
education have made us afraid to be different
from our neighbour, afraid to think contrary to B. Earning money
the established pattern of society, falsely
C. Without knowing
respectful of authority and tradition.
Fortunately, there are a few who are in earnest, D. With complete knowledge
who are willing to examine our human problems
without the prejudice of the right or of the left; Direction: Noise is a common problem in both
but in the vast majority of us, there is no real urban and rural areas, traffic noise in particular.
spirit of discontent, or revolt. When we yield Trees and shrubs are not only visually attractive
uncomprehendingly to environment, any spirit but also can make effective noise barriers.
of revolt that may have had died down, and our Therefore when space permits we have to plant
responsibilities soon put an end to it. more trees and shrubs in areas where there is
noise problem. By doing this noise can be
21. Why does conventional education make reduced by six decibels over a distance of 30
independent thinking extremely difficult? metres. The only thing we have to keep in mind
is that the planting should particularly dense; the
A. It does not encourage disagreement
trees which have dense foliage and relatively
B. It does not accept conformity vertical orientation of arrangements should be
planted. It will completely screen the source of
C. It is not fact based, but problem based
noise. If there is any gap in barrier it will still
D. It is only religious education allow noise to penetrate. Single layer Of trees is
worthless as a noise barrier. Due to inter-
22. Worshipping success leads to
reflection, multi-rows Of trees are more
A. questioning the established practices effective. The planted trees must be taller. We
can reduce noise level by 1.5 decibels with each
B. accepting the established practices
additional 3 feet of tree height. When the
C. modifying the established practices opportunity to use trees is restricted a
combination of shrubs and tall grass can be used.
D. neither accepting nor rejecting the
Instead of tarmac or gravel surfaces soft grass
established practices
ground cover is preferable. It will absorb noise
23. Why do most people want to find a quiet rather than reflect it. Rows of trees, shrubs,
corner in life? crotons, ground covering plants, climbers and
their flowers not only reduce noise and provide
A. Because they do not like to be in peace
a beautiful backyard they also help block wind,
B. Because they seek pleasure provide shade, and absorb traffic.
C. Because they seek comfort 28. What distance a good number of can cover
in reducing noise?
D. Because they do not worship success
A. 30 metres B. 60 metres
24. Which word in the passage means 'bias'?
C. 3 metres D. 300 metres
A. contrary
27. Trees can reduce noise only when
B. prejudice
A. planted in isolation
C. uncomprehendingly
D. discontent B. planted with flowering shrubs

PAGE 107
www.byjusexamprep.com

C. they are planted dense S. is a direct hindrance


D. planted with shorter ones A. R P S Q B. R Q P S
28. 'Single layer of trees is worthless as a noise C. S P Q R D. Q R S P
barrier.' Why?
33. P. enough potassium,
A. Noise can't penetrate easily
Q. banana provides
B. Help in the growth of shrubs
R. from various sugars
C. Noise is sent back to the source S. and sustained energy
D. Noise can penetrate easily A. P Q R S B. R S Q P
29. Higher the trees C. S Q R P D. Q P S R
A. higher the noise 34. P. of great trouble
B. lower the noise Q. this is a time
C. greater the decibels R. the world to the maximum
D. thinner the air S. as one virus threatens
30. How do shrubs and tall grass reduce noise? A. PQSR B. RSPQ
A. They absorb noise C. SRQP D. QPSR
B. They reflect noise 35. P. are of the opinion
C. They divert noise Q. some archaeologists
D. They are neutral to noise R. and that everybody enjoyed equal status
Direction: Each of the following items in this S. that Harappan society had no rulers,
section consists of a sentence, parts of which
have been jumbled. These parts have been A. S P Q R B. Q S R P
labelled as P, Q, R and S. Given below each C. Q P S R D. R P S Q
sentence are four sequences namely (a), (b), (c)
and (d). You are required to re-arrange the 36. P. in the same locality
jumbled parts of the sentence and mark your Q. marriage within a unit
response accordingly.
R. endogamy refers to
31. P. consideration for others
S. or a group living
Q. for self-gratification
A. Q R S P B. R S P Q
R. freedom does not mean the opportunity
C. Q P S R D. R Q S P
S. or the setting side of
37. P. when steam engines
A. S P Q R B. Q S P R
Q. dominated industrialization
C. R P Q S D. R Q S P
R. coal was the main source
32. P. of any kind
S. Of energy in the initial stages,
Q. to freedom and intelligence
A. S P R Q B. R S P Q
R. domination or compulsion
C. P Q R S D. Q R S P

PAGE 108
www.byjusexamprep.com

38. P. from the scientists? The correct sequence should be


Q. is the information A. SRQP B. RPQS
R. that comes C. RQSP D. QRSP
S. how important 42. S1 : We drink more water during summers.
A. Q S P R B. Q R S P S6 : There are variations in weather
C. S Q R P D. S P Q R conditions during different seasons.

39. P. brought on by destructive pests P : On the other hand in north-eastern


states, winters are mild except in the hills.
Q. humans have suffered
Q : Because in North India, both the
R. frustration and food losses sessions have extreme weather conditions.
S. Since earliest times, R : School clothes during the summer is
A. S P Q R B. Q R S P different from the winter.

C. S Q R P D. R S Q P S : Why do you wear lighter clothes during


summers and heavy woollen clothes during
40. P. most spectacular gold coins the winter in north India?
Q. some of the The correct sequence should be
R. the Gupta rulers in India A. SRPQ B. RSQP
S. Were issued by C. PQRS D. QPRS
A. R S P Q B. Q P S R
43. S1 : Policy, like many other terms, is a
C. P Q R S D. S P Q R complex concept.
Direction: In this section each item consists of six S6 : Public policy is the outcome of the
sentences of a passage. The first and sixth struggle between government and
sentences are given in the beginning as S1 and powerful groups.
S6. The middle four sentences in each have been
P : It is form of government control usually
jumbled up and labeled as P, Q, R and S. You are
expressed in a law, a legislation, a
required to find the proper sequence of the four
regulation or an
sentences and mark your response accordingly
on the Answer Sheet. order.
41. S1 : Man has progressed with science and Q : Public policy is what a government
technology. chooses as guidance for action.
S6 : What was difficult and impossible is R : A policy may be general or specific,
now easy and possible. broad or narrow, simple or complex, public
or private,
P : He is on the road to conquer other
planets. written or unwritten.
Q : Now with scientific knowledge and S : It takes different forms.
technical know-how he has done what was
The correct sequence should be
impossible.
A. SRQP B. SQPR
R : Hitherto he had been backward.
C. RSPQ D. PQRS
S : He has reached the moon.

PAGE 109
www.byjusexamprep.com

44. S1: The dhoti kurta is a traditional lungi- 46. S1 : Urbanisation in India has become . an
type garment worn by men in India. important and irreversible process.
S6 : Gandhi ji usually wore a simple shawl S6 : As per the Census of India 2011, the
with his dhoti. urban population is increasing rapidly.
P : It is commonly worn with a loose long P : The 2030 development agenda of the
tunic. United Nations has emphasised the role of
sustainable cities for making cities
Q : This he started wearing after he
inclusive, safe, resilient and sustainable.
abandoned his western attire of pants and
shirt. Q : This shows that India is in the midst of
transition from a predominantly rural to a
R : A dhoti, made of homespun khadi cotton
quasiurban society.
cloth, was the garment favoured by Gandhi
ji. R : The process of urbanisation has been
characterized by a dramatic increase in the
S : The dhoti is rectangular piece of number of large cities.
unstitched cloth, around five yards long,
wrapped around S : It is an important determinant of
national economic growth and poverty
the midriff and tucked between the legs. reduction.
The correct sequence should be The correct sequence should be
A. PSRQ B. RSPQ A. QRSP B. PSRQ
C. SPRQ D. QRSP C. SRQP D. RQSP
45. S1 : Languages in contact with each other 47. S1 : Ethics is a branch of philosophy
cause change. concerned with the distinction between
S6 : Changes often occur first in informal right and wrong (morals) and the ultimate
and casual language. worth of actions or things (values).

P : The increasing use of English in many S6 : But values can be either inherent or
parts of the world affect both local conferred.
languages and English. P : For many philosophers, only humans are
Q : Language contact has long been moral agents, being capable of acting
recognised as a major engine of change. morally or immorally.

R : This gives rise to new, hybrid language Q : Environmental ethics asks about the
varieties. moral relationships between humans and
the world around us.
S : A historical example is that of Danish and
English which led to a major shift in the R : Ethics evaluated the relationship, rules,
vocabulary and grammar of English. principles or codes that require or forbid
certain conduct.
The correct sequence should be
S : Value is a measure of the worth of
A. RSQP something.
B. QSPR The correct sequence should be
C. QRSP A. RQPS B. PQSR
D. SQRP C. QSRP D. RSPQ

PAGE 110
www.byjusexamprep.com

48. S1 : The Constitution of India, introduced The correct sequence should be


on 26 January 1950, laid down certain basic
A. QSRP B. QRSP
principles and values.
C. RQPS D. SPRQ
S6 : The Constitution guaranteed all citizens
equality before the law and equality of 50. S1 : Ecology, history and geography
opportunity in government employment. determine the distribution of organisms.
P : It was also to be a federation with S6 : Others prefer to interpret bio
demarcation of spheres of action between geographical history through vicariance
the Union Government and the State (range-splitting)events.
Governments.
P : Some historical biogeographers are
Q : India was to be a secular and democratic interested in finding centre of-origins.
republic with a parliamentary system based
Q : Most species distributions result from a
on adult franchise.
combination of all these factors.
R : These include : freedom of speech and
R : But biogeographers tend to specialise in
expression, freedom to assemble
ecological aspects (ecological biography or
peacefully etc.
historical aspects).
S : It guaranteed all Indian citizens certain S : Ecological biogeographers are interested
fundamental rights. in the effects of environmental factors.
The correct sequence should be The correct sequence should be
A. QPSR B. RSPQ A. QRSP B. RQPS
C. PSQR D. SORP C. PQRS D. SPRQ
49. S1 : The National Health Policy 2017 Direction: Each of the following sentences in this
envisages creation of a digital health section has a blank space and four words or
technology eco- system. group of words are given after the sentence.
S6 : Secondly, offering financial protection Select the most appropriate word or group of
up to 35 lakhs per year per family for the words for the blank space and indicate your
deprived 10 crores plus households. response on the Answer Sheet accordingly.

P : Ayushman Bharat comprising twin 51. He said that he _______ when he lost all his
missions is set to transform the investment in the stock market
nation&#39;s health system. A. was broken
Q : This will also foster a linkage across B. was broke
public and private health care.
C. brake
R : The policy aims at developing an
integrated health information system that D. had been collapse
serves the need of all stakeholder and 52. The man in black thought that he was
improves efficiency, transparency. _________ selected for the team to play at
the national level
S : This will be carried out by
operationalising firstly 1.5 lakh health and A. one of the player
wellness centres offering preventive and
primary care. B. one of the players

PAGE 111
www.byjusexamprep.com

C. is one player 60. Indian perspective of life informs that


leading a life of greed ________ in
D. the player of the team
unhappiness
53. When she _______ her friend after a period
A. will lead B. shall be
of two decades her throat choked
C. will result D. would resulted
A. meets B. had been meeting
Direction: Each item consists of sentences
C. met D. has met
with an underlined word followed by four
54. The portrait of the lady conveys _______ it words or group of words. Select the option
appears that is opposite in meaning to the
underlined word and mark your response
A. the best B. more than
on the Answer Sheet accordingly.
C. the most D. nothing
61. His arguments are not valid. People
55. A biography is _____ life history of a consider it bombastic.
person.
A. outdated B. straight forward
A. about a B. above
C. verbose D. not true
C. for a D. with a
62. The decision was absurd for many of the
56. She told me that she _________ her members of the team
degree by 2023
A. bizarre B. meaningless
A. will have completed
C. reasonable D. thoughtful
B. shall be completed
63. Relatives of deceased have been informed
C. will completed about the accident
D. would have completed A. injured B. alive
57. What is the name of the person _______ C. dead D. survived
daughter married your cousin last year
64. At last she was able to get some solace as
A. whose B. who the matter has been resolved amicably
C. whom D. with A. comfort B. relief
58. The world is changing ______ that the rich C. punishment D. aggravation
become richer and the poor become poorer
65. Twenty first century has turned out to be a
A. through B. in many way century of problems contrary to the
thinking that it would be a better time
C. in a way D. in such a way
A. similar B. different
59. Had I been informed beforehand, I ____ it
to the celebrations C. divergent D. good faith
A. could have made 66. The poet said that poetry is a spontaneous
overflow of powerful feelings
B. will have made
A. prepared B. alerted
C. would have
C. develop D. well executed
D. ought to have made

PAGE 112
www.byjusexamprep.com

67. Language is an instrument for asserting 71. It sounds like _______71 great idea.
one's identity, attitude and perspective
A. a B. an
A. declaring B. supporting
C. the D. some
C. denying D. propagating
72. People getting instantly excited when they
68. He has been exonerated as he tendered an hear_______ 72 phrase 'education for
apology peace' or the title of a course that calls itself
'Peace Education’.
A. honoured B. pardoned
A. a B. the
C. convicted D. felicitated
C. that D. some
69. Persuasion is essential for people to work
as a team 73. About a decade ago, I proposed such
______ 73 course in my institute.
A. Dislike B. Discouraging
A. any B. the
C. Convincing D. Deter
C. some D. a
70. Every habitat has some distinctive
vegetation which defines the eco-system 74. The idea of a course _______ 74 'peace
education' had occurred to me on the way
A. unique B. common
back from one of the neighbouring
C. special D. unfamiliar countries where I met students of class IX.
Direction: Each of the following sentences in this A. to B. in
section has a blank space with four words or
C. for D. on
group of words given. Select whichever word or
group of words you consider the most 75. I could not find ______75 difference
appropriate for the blank space and indicate
A. any B. the
your response on the Answer Sheet accordingly.
C. that D. this
It sounds like _______71 great idea. People
getting instantly excited when they hear_______ 76. _______ 76 Indian students and their
72 phrase 'education for peace' or the title of a counterparts in the other
course that calls itself 'Peace Education’. About a
A. with B. between
decade ago, I proposed such ______ 73 course in
my institute. The idea of a course _______ 74 C. among D. to
'peace education' had occurred to me on the
way back from one of the neighbouring 77. country on peace _______ 77. Living
countries where I met students of class IX. I could together.
not find ______75 difference _______ 76 Indian A. with B. and
students and their counterparts in the other
country on peace _______ 77. Living together. It C. because D. but
made me think that if one learns to live with a 78. Living together. It made me think that if one
conflict, life grows across it like grass ________ learns to live with a conflict, life grows
78 trees. The arguments and stances children across it like grass ________ 78 trees.
took demonstrated ________ 79 peace comes
from understanding a conflict well, not A. or B. with
necessarily ______ solving it. C. for D. some

PAGE 113
www.byjusexamprep.com

79. The arguments and stances children took A. core B. part


demonstrated ________ 79 peace comes
C. idea D. tip
from
A. for B. but 87. Each child develops his/her competency
based on the contexts and the inputs for
C. that D. when learning
80. understanding a conflict well, not
A. capability B. thinking
necessarily ______ solving it.
A. by B. with C. knowledge D. ideal

C. to D. for 88. He appears to be very haughty, but he is a


humble person
Direction: Each item in this section consists
of a sentence with an underlined word(s) A. tough B. modest
followed by four words/group of words.
C. arrogant D. knowledgeable
Select the option that is nearest in meaning
to the underlined word and mark your 89. The newly appointed secretary is
response on the Answer Sheet accordingly. industrious
81. Having got excited she opened up the A. diligent B. knowledgeable
Pandora's Box which led to lot of
commotion. C. indolent D. insincere

A. uproar B. peace 90. The indignant attitude of the speaker


made the groups unhappy
C. sound D. furious
82. The inherent danger in the problem is that A. resentful B. congenial
it would lead to many more problems C. unruly D. supportive
A. outward B. difficult Direction: Each of the following sentences has a
C. hallow D. inbuilt word or phrase underlined. Read the sentences
carefully and find which part of speech the
83. The officer was reprimanded by the court
underlined word is. Indicate your response on
for delaying the case
the Answer Sheet accordingly.
A. admonished B. appreciated
91. The preacher said, "No one is above the all
C. praised D. disliked powerful".
84. Some people think that their strength is A. Adverb B. Place value
perpetual
C. Preposition D. Verb
A. temporary B. powerful
C. ever lasting D. all persuasive 92. The most beautiful seven monkeys of the
landlord were taken for a stroll
85. One's actions exemplify one's attitude and
values A. Object B. Adjective

A. devise B. sympathize C. Noun phrase D. Noun


C. asks for D. demonstrate 93. Alas! He has been defeated
86. The crux of the issue was that there was no A. Adjective B. Adverb
evidence to prove the accused guilty of the
C. Interjection D. Conjunction
act

PAGE 114
www.byjusexamprep.com

94. The man who is in the green shift is the 103. Sheela has been at the receiving end
chief of the day
A. but she is very honest
A. Pronoun
B. and she is very honest
B. Relative noun
C. she had been proud of it
C. Demonstrative pronouns
D. she was fine with it
D. Assertive noun
102. Elam has no other option
95. He has been living in the jungle for long and
A. unless he wins
he knows all the animals here
B. but to accept the challenge
A. Subordinating conjunction
C. though he has won challenge
B. Coordinating conjunction
D. he has to accept the challenge
C. Preposition
103. Although the group has been acquitted
D. Personal pronoun
A. they are under the watch of police
96. They have been driving since early morning
B. they are free to do anything
A. Adjective B. Adverb
C. there is no suspicion in them
C. Noun D. Comparison
D. they have not been arrested
97. This time we woke up to the virus related
diseases. 104. Unless you fulfil the essential qualifications
A. Pronoun B. Article A. you will be called for the interview
C. Adverb D. Noun phrase B. you shall be selected for the post
98. Love your neighbour as thyself C. you will not be called for the interview
A. Noun B. Pronoun D. you can't be rejected
C. Preposition D. Adjective 105. The company's bid for the contract is
rejected
99. Ravi won the match with a huge margin
A. since it filled all the conditions
A. Intransitive verb
B. for the company fulfilled the conditions
B. Transitive verb
C. for the company had not been in the
C. Demonstrative verb
goodwill list
D. Adjective
D. since the deposit in the account is
100. There was an exultation in the group found less than required
A. Adjective B. Adverb 106. I could not sleep
C. Adjectival Noun D. Noun A. because being very tired
Direction: Following items have one part of the B. despite being very tired
sentence followed by four alternatives.
C. in spite of being sleepless
Complete the sentences by choosing the correct
alternative. D. since being very tired

PAGE 115
www.byjusexamprep.com

107. She married him A. an B. a


A. since his good nature C. the D. one
B. because his nature 113. Don’t read ________ the lines. Read the
lines.
C. because of his nature
A. in B. between
D. in spite his nature
C. on D. by
108. I will come to meet you
114. The village possesses ________ scenic
A. in case you want beauty.
B. in case of time A. some B. a
C. because you need C. an D. the
D. although you want 115. He is all out to blame everyone ________
109. If you had planned the work well his mistakes.

A. you shall have completed it on time A. for B. in

B. you could complete it on time C. on D. with

C. you will complete it on time 116. This is _______ unique system of


governance where everyone is the
D. you could have completed it on time government.
110. You can use my computer A. an B. the
A. as long as you can't use any external C. a D. some
drive
117. I will take care ________ myself. You don’t
B. as long as you used any external drive bother about me.
C. as long as you do not use any external A. of B. about
drive
C. for D. off
D. as long as you shall use any external
118. She is considered __________ nightingale
drive
of the college for her melodious voice.
Direction: Each of the following sentences in this
A. a B. an
section has a blank space with four options.
Select whichever preposition or determiner you C. some D. the
consider the most appropriate for the blank
119. The room is a bit dark. Could you please
space and indicate your response on the Answer
turn ________ the light?
Sheet accordingly.
A. on B. it
113. Don't go________ the wind, you will face
the music later. C. off D. away
A. with B. for 120. Don’t walk so fast! I can’t keep _________
with you.
C. about D. between
A. in B. up to
112. He has been considered ________ honest
contestant in the competition. C. up D. on

PAGE 116
www.byjusexamprep.com

General Knowledge
1. Which one of the following is a cold ocean 5. Mission Indradhanush aims at
current?
A. reducing child deaths due to
A. Brazilian Current pneumonia

B. Gulf Stream B. reducing the impact of rotavirus

C. North Equatorial Current C. elimination of maternal tetanus.


D. full immunisation of children
D. California Current
6. The Saubhagya Scheme aims at universal
2. India’s territorial limit extends towards the
sea up to A. LPG connection

A. 10 nautical miles B. household electrification

B. 12 nautical miles C. primary school education


D. public health insurance
C. 14 nautical miles
7. NIDHI is an umbrella scheme for the
D. 15 nautical miles
promotion of
3. Which of the following statements about A. young and aspiring innovators.
the Pradhan Mantri Gram Sadak Yojana are
correct? B. scientific research.

1. It is part of Government of India’s C. primary health care.


poverty reduction strategy. D. primary education in rural areas.
2. It is a centrally sponsored scheme for 8. KIRAN (Knowledge Involvement in
rural development Research Advancement through Nurturing)
is an initiative to provide opportunities to
3. It provides connectivity in rural areas.
women scientists who
Select the correct answer using the code A. had a break in their career due to family
given below: reasons.
A. 1, 2 and 3 B. 1 and 3 only B. are pursuing innovative research in life
C. 2 and 3 only D. 1 and 2 only sciences.
C. have collaborative projects with labs in
4. Which one of the following statements
foreign countries
about the DDU-GKY is not correct?
D. have leadingedge projects in Nano-
A. It is a skill training programme.
Science and Technology.
B. It is for rural youth from poor families. 9. The Amara-Nayaka system was a major
C. Its objective is placement in wage political innovation of which Indian imperial
employment. rulers?

D. It relies entirely on skill training by A. Cholas B. Chalukyas


Government agencies. C. Guptas D. Vijayanagara

PAGE 117
www.byjusexamprep.com

10. Which Maratha statesman signed the B. The Government would not release
Treaty of Bassein on December 31, 1802? those prisoners who had indulged in
the peaceful picketing of liquor and
A. Bajirao II
foreign cloth shops.
B. Vithuji Holkar
C. The Congress did not agree to the
C. Daulat Rao Sindhia suspension of the Civil Disobedience
Movement.
D. Madhava Rao Narayan
D. The Congress agreed to take part in the
11. Which on of the following statements
Second Round Table Conference.
about the Act V of 1843 relating to Slavery
in India is correct? 15. The percentage by which the money the
borrower pays back exceeds the money
A. It gave the masters the right to willfully
that was borrowed is called as
keep their slaves ties to their estates.
A. Bank rate
B. It denied the masters the use of Courts
to assert their claims on slaves. B. Nominal interest rate
C. The law Courts and masters worked C. Real interest rate
jointly in resolving the cases of
D. Terms of credit
desertion.
16. The asset or assets that a borrower pledges
D. The slaves became the owners of the
in order to guarantee repayment of a loan
land.
is called as
12. In 1911, who among the following
A. Cheque B. Collateral
introduced a bill in the Imperial Legislative
Council for introduction of compulsory and C. Guarantee card D. Bond
free primary education in India?
17. The increase in private investment
A. DadabhaiNaoroji spending induced by the increase in
Government spending is known as
B. Bal Gangadhar Tilak
A. Crowding in B. Deficit financing
C. Sir Harcourt Butler
C. Crowding out D. Pumping out
D. Gopal Krishna Gokhale
18. The situation in an economy which is
13. Who among the following founded the Bhil
growing slowly along with rapid inflation
Seva Mandal in 1922?
(rising price level) is called
A. Dayaram Gidumal
A. Stagnation B. Deflation
B. GurusadayDutt
C. Stagflation D. Recession
C. Dhondo Keshav Karve
19. The mismatch in the regional or
D. AmritlalVithaldas Thakkar occupational pattern of job vacancies and
the pattern of worker availability results in
14. Which one of the following statements
about the Gandhi-Irwin Pack of 1931 is A. Structural unemployment
correct?
B. Disguised unemployment
A. The Government would not release
C. Altered unemployment
those prisoners who had been non-
violent. D. Cyclical unemployment

PAGE 118
www.byjusexamprep.com

20. Which one of the following statements is 25. Equality before the law or equal protection
not correct? of the laws within the territory of India is
guaranteed under which one of the
A. Real GDP is calculated by valuing
following Articles of the Constitution of
outputs of different years at common
India?
prices.
A. Article 14 B. Article 15
B. Potential GDP is the real GDP that the
economy would produce if its C. Article 16 D. Article 22
resources were fully employed.
26. Which of the following statements
C. Nominal GDP is calculated by valuing regarding the 'casting vote' in the
outputs of different years at constant Parliament is/are correct?
prices.
1. It is cast by the speaker or a person
D. Real GDP per capita is the ratio of real acting as such
GDP divided by population.
2. It is cast in addition to voting in the first
21. Which one of the following approaches of instance.
human development was initially proposed
by the International Labor Organisation 3. It is cast in the case of equality of votes.
(ILO) and emphasized on health, education, 4. It is always cast to maintain the status
food, water supply, sanitation and housing? quo.
A. Welfare approach Select the correct answer using the code
B. Basic needs approach given below:

C. Income approach A. 1, 2 and 3 only B. 1 and 3 only

D. Capability approach C. 2 and 4 only D. 3 only

22. Milpa and Ladang are different names for 27. National Disaster Management Authority is
headed by
A. shifting cultivation.
A. the Prime Minister.
B. mixed farming.
B. the Home Minister.
C. truck farming.
C. the President.
D. plantation agriculture
D. the Health Minister.
23. Who among the following said that
Democracy means a system of 28. When was the monopoly of China trade lost
'Government by Consent'? by East India Company?

A. John Locke A. 1813 B. 1833

B. J.S. Mill C. 1838 D. 1860

C. Jeremy Bentham 29. Which one of the following statements


about the British Indian Medical Service
D. J.J. Rousseau (IMS) is not correct?
24. 'Legal Positivism' theory was developed by A. IMS began in 1764
A. T.H. Green B. Thomas Hobbes B. It recruited health professionals by
C. John Austin D. Patrick Devlin means of a competitive examination.

PAGE 119
www.byjusexamprep.com

C. Indians were never admitted to IMS. Select the correct answer using the code
given below:
D. The IMS was at first meant to look after
the troops. A. 1, 2 and 3 only
30. Which one of the following statements with B. 1, 3 and 4 only
regard to ryotwari settlement is not
C. 2 and 3 only
correct?
D. 2 only
A. In southern and western India, the
ryotwari settlement was adopted. 34. Which one of the following statements
about the composition of the Parliament is
B. Ryotwari was in principle a direct
not correct?
contract between the ryot and the
state. A. Representatives of the states in Rajya
Sabha are elected directly by the
C. It means a tax contract valid for usually
people.
30 years.
B. Representatives from Union Territories
D. In principle, it strengthened the former
in Lok Sabha are chosen by direct
elite, the zamindars and weakened the
elections.
peasantry.
C. Rajya Sabha has 12 nominated
31. A motion of no-confidence is moved against members.
A. an individual Minister. D. Lok Sabha has seats reserved for SCs
B. the Council of Ministers. and STs.

C. the Prime Minister. 35. Which one of the following is not a correct
description of the Union Cabinet?
D. a political party.
A. It is part of the Parliament.
32. Which one of the following statements
about the Vice-President of India is not B. It is responsible to the Parliament.
correct? C. It remains in power till it enjoys the
A. He is not eligible for re-election. confidence of the Parliament.

B. He must not be less than 35 years of D. A person from outside the Parliament
age. can never be appointed a member of
the Cabinet.
C. He is elected by members of an
electoral college consisting of members 36. Which of the following statements is/are
of both Houses of Parliament. correct?

D. His term of office is five years. 1. The election to the Panchayat are
conducted by the State Election
33. Which among the following fundamental Commission.
rights is/are available to non-citizens?
2. The State Election Commissioner is
1. Freedom of speech appointed by the Chief Election
2. Protection against self-incrimination Commissioner of India.

3. Freedom of conscience 3. The State Legislatures have the power


to make laws on all matters relating to
4. Non-discrimination in employment Panchayat elections.

PAGE 120
www.byjusexamprep.com

Select the correct answer using the code Select the correct answer using the code
given below: given below:
A. 2 only A. 1 only B. 2 and 3 only
B. 1 and 2 only C. 1, 2 and 3 D. 1 and 3 only
C. 1, 2 and 3 40. Match List I with List II and select the
correct answer using the code given below
D. 1 and 3 only the lists:
37. Which of the following statements about List I List II
the Panchayati Raj System are correct?
(a) Cyclones 1. Western
1. All seats in a Panchayat are filled by Australia
persons chosen by direct election.
(b) Hurricanes 2. South China Sea
2. The 'Gram Sabha' consists of persons
registered in the village electoral rolls. (c) Typhoons 3. Indian Ocean
3. The Chairperson of a Panchayat is (d) Wily-wilies 4. Atlantic Ocean
elected in accordance with a law
passed by each state.
Code:
4. All states in India have a three-tier
system of Panchayats. A. a-3 b-2 c-4 d-1
Select the correct answer using the code B. a-3 b-4 c-2 d-1
given below:
C. a-1 b-4 c-2 d-3
A. 1 and 4 only
D. a-1 b-2 c-4 d-3
B. 1, 2 and 3 only
41. Which one of the following longitudes is the
C. 2 and 3 only standard meridian of India?
D. 2, 3 and 4 only A. 83° 30' E B. 82° 30' E
38. Which one of the following units is not a C. 82° 30' W D. 83° 30' W
part of All India Radio's Transcription and
42. Which one of the following water bodies
Programme Exchange Service?
separates the Andaman and the Nicobar
A. Central Archives Islands?
B. Sound Archives A. Gulf of Mannar

C. Advanced Research Unit B. Ten Degree Channel

D. Transcription Unit C. Eleven Degree Channel

39. Which of the following is/are social security D. Palk Strait


scheme(s)? 43. Which one of the following is the largest
1. Atal Pension Yojana linguistic group of India?

2. Pradhan MantriJeevanJyotiBimaYojana A. Sino-Tibetan B. Austric

3. Pradhan Mantri Suraksha BimaYojana C. Indo-Aryan D. Dravidian

PAGE 121
www.byjusexamprep.com

44. Which of the following statements with A. Justice M.N. Venkatachaliah


regard to the creation of a new state or
B. Justice J.S. Verma
alteration of boundaries of states is/are
correct? C. Justice Ranganath Mishra
1. An amendment has to be moved in the D. Justice Y.K. Sabharwal
Parliament under Article 368 of the
49. Which one of the following scholars of
Constitution of India.
Akbar's court translated Bhaskaracharya's
2. Ratification by one-half of the states is Lilavati into Persian?
necessary after the amendment is
A. AbulFazl B. Faizi
made by the Parliament.
C. FathullahShirazi D. AtaullahRashidi
3. The legislation can be passed by a
simple majority of both Houses of 50. Match List I with List II and select the
Parliament. correct answer using the code given below
the lists:
Select the correct answer using the code
given below: List I List II
(Author) (Poetical
A. 1 and 3 only B. 2 only
work)
C. 3 only D. 2 and 3 only
(a) Brindavan 1. Shiva
45. Which mega public sector undertaking is Das sankirttan
designated with 'Navratna' status?
(b) Krishnadas 2. Chandimangal
A. Indian Oil Corporation Limited Kaviraj
B. Gas Authority of India Limited (c) Mukundaram 3. Chaitanya
C. Bharat Petroleum Corporation Limited Chakravarti charitamrita

D. Bharat Electronics Limited (d) Rameshwar 4. Chaitanya


Bhattacharya mangal
46. Who among the following founded the
Marathi newspaper 'Kesari'?
A. Lokmanya Tilak Code:

B. Vallabhbhai Patel A. a-4 b-2 c-3 d-1 B. a-4 b-3 c-2 d-1

C. Lala Lajpat Rai C. a-1 b-3 c-2 d-4 D. a-1 b-2 c-3 d-4

D. Mahatma Gandhi 51. Growing of two or more crops


simultaneously on the same filed in a
47. M.M Punchhi Commission on Centre-State definite pattern is known as
Relations had recommended the disposal
of a bill reserved for the consideration of A. mixed cropping B. inter cropping
the Union Executive within C. mixed farming D. hybrid farming
A. four months. B. six months. 52. The Kaleshwaram Lift Irrigation System,
C. eight months. D. five months. which was in the news recently, is
constructed in
48. Who among the following was the Chairman
of the National Commission for Review of A. Kerala B. Gujarat
the Working of the Constitution (2000)? C. Telangana D. Maharashtra

PAGE 122
www.byjusexamprep.com

53. In the year 2020 which one of the following 59. The first phase of Exercise Malabar, 2020
tiger reserves of India was declared by was conducted at
UNESCO as a Biosphere Reserve?
A. Port Blair
A. Panna Tiger Reserve
B. Mumbai
B. Namdapha Tiger Reserve
C. Visakhapatnam
C. Dudhwa Tiger Reserve
D. Kochi
D. Pench Tiger Reserve
60. Which one of the following is not a member
54. Which one of the following ships was of the Quad group of nations?
involved in ‘Mission Sagar-II’?
A. France B. USA
A. INSA Shakti
C. Australia D. Japan
B. INS Vikramaditya
61. Which one of the following is correct about
C. INS Kesari ‘Pinaka’?

D. INSA Airavat A. It is a multi-barrel system.

55. Peninsula Shield Force is a military arm of B. It is a type of battle tank.

A. Organization of Islamic Cooperation C. It is an advanced submarine.

B. Organization of the Petroleum D. It is an indigenously developed drone


Exporting Countries system.

C. Gulf Cooperation Council 62. Osmosis is the process of movement of


water molecules from its
D. Arab League
A. higher concentration to its lower
56. ‘CARAT Bangladesh 2020’ was a joint naval concentration through a cell wall.
exercise between Bangladesh and
B. lower concentration to its higher
A. India B. USA concentration through a fully
C. Japan D. Thailand permeable membrane.

57. The Luhri Hydro-Electric Power Project is C. higher concentration to its lower
being constructed on the river concentration through a fully
permeable membrane.
A. Satluj B. Beas
D. higher concentration to its lower
C. Ravi D. Chenab concentration through a semi-
58. Hollywood actor Sir Sean Connery, who permeable membrane.
died recently, was popularly known for the 63. Which cell organelles have their own DNA
portrayal of the role. and Ribosomes?
A. Rocky A. Golgi body and Endoplasmic Reticulum
B. Captain Jack Sparrow B. Mitochondria and Plastids
C. Terminator C. Lysosome and Golgi body
D. James Bond D. Vacuole and Plastids

PAGE 123
www.byjusexamprep.com

64. A student was doing an experiment on 70. An antibiotic is not useful against of virus
increasing the cell division among plants. whereas a vaccine is. Which one of the
She asked her supervisor to suggest the following is the most appropriate reason
specific plant hormone for the same. Had for this?
you been her supervisor, which plant
A. An antibiotic can break RNA only,
hormone would you suggest?
whereas a virus has DNA
A. Abscisic acid B. Gibberellins B. An antibiotic is a carbohydrate in its
C. Cytokinin D. Auxin chemical nature, whereas a vaccine is a
protein that works well to kill a virus.
65. A child receives a tall beautiful plant as a
birthday gift from his father with a quiz. The C. Only a vaccine can break the genetic
father asked her how she would verify material of a virus.
whether this tall plant was the progeny of D. A virus does not use biochemical
both the tall parents or one tall and one pathways that can be blocked by an
short parent plant. She could verify this antibiotic. But a vaccine can boost an
through. immune system to fight the virus.
A. cross-pollination 71. Aluminum is manufactured from
B. self-pollination A. Copper ore B. Bauxite ore
C. tissue culture C. Mica ore D. Manganese ore
D. negative propagation 72. A solution contains 20 g of solute in 180 g
of solvent. If the solvent is water, what is
66. Cell was is not present in cells of
the concentration of the solution in terms
A. Bacteria B. Plants of mass by mass percentage?
C. Fungi D. Humans A. 11.1% B. 22.2%
67. Which one of the following is the C. 10% D. 20%
structure of a cardiac muscle cell?
73. Which one of the following metals is kept
A. Cylindrical, Unbranched and immersed in Kerosene oil to protect it and
Multinucleated to prevent accidental fire?
B. Spinal shaped, Unbranched and A. Calcium B. Sodium
Uninucleate C. Vanadium D. Magnesium
C. Spinal shaped, Branched, and 74. Which one of the followings do silver
Uninucleate articles reach with, that makes the silver
D. Cylindrical, Branched, and Uninucleate articles black?

68. Which one among the following is a free- A. Sulfur B. Oxygen


living animal? C. Carbon dioxide D. Phosphorus
A. Liver fluke B. Wuchereria 75. When we heat lead nitrate [Pb(NO3)2] in a
C. Plasmodium D. Planaria boiling tube, we observe the emission of
brown fumes. Which one of the followings
69. ‘Sleeping sickness’ is caused by? is the brown gas?
A. Trypanosoma B. Leishmania A. NO B. N2O2
C. Plasmodium D. Paramecium C. N2O2 D. NO2

PAGE 124
www.byjusexamprep.com

76. Calcium oxide reacts vigorously with water Select the correct answer using the code
to produce slaked lime whose solution is given below:
used for white-washing walls. This slaked
A. 1 only B. 2 only
lime reacts with component (A) in the air to
form a thin layer of component (B) to give a C. 1 and 2 only D. 1 and 3 only
shiny finish. What are the components (A)
81. Which one of the following metals has both
and (B)?
malleability and ductility properties?
A. A. Na B. Au
B. C. Ce D. He

C. 82. Which one of the following elements are a


non-metal but is lustrous?
D.
A. Carbon B. Silicon
77. Which one of the followings serum C. Germanium D. Iodine
compounds is used for softening hard
water? 83. The Size of particles being studied in
nanotechnology is about
A. Na2CO3 B. NaHCO3
A. 1 Å-10 nm B. 1-100 nm
C. NaOH D. Na2SO4
C. 1-50 μ D. 1 mm – 10 mm
78. Which one of the followings is used in soda-
acid fire extinguishers? 84. The weight of an object is due to
A. the net force acting on it
A. Sodium chloride
B. the total of all forces acting on it
B. Sodium hydrogen carbonate
irrespective of their directions.
C. Calcium hydroxide
C. the force that it exerts on the ground
D. Acetic acid
D. its inert property
79. Which one of the followings naturally
85. The direction of acceleration in uniform
occurring acids is found in abundance in
circular motion is along the
tomatoes?
A. direction of motion
A. Acetic acid B. Citric acid
B. tangent to the circle at the point of
C. Oxalic acid D. Tartaric acid observation
80. Which of the following statements about C. direction of velocity
‘fission’ is correct?
D. direction perpendicular to velocity
1) It is related to the creation of new
individuals by means of cell division in a 86. How many internal reflections of light take
unicellular organism place in the formation of primary rainbow?

2) It is related to the transformation of A. 0 B. 1


heavier nuclei into smaller nuclei C. 2 D. More than 2
3) It is related to the creation of heavier 87. Which one of the following laws of
nuclei by means of combining two higher electromagnetism does not give the
nuclei direction of magnetic field?

PAGE 125
www.byjusexamprep.com

A. Right hand thumb rule 93. Francois Bernier was physician to


B. Fleming’s left-hand rule A. Prince Murad
C. Fleming’s right-hand rule B. Princess Jahanara
D. Faraday’s law of electromagnetic C. Emperor Shah Jahan
induction
D. Prince Dara Shikoh
88. Who amongst the following is pioneer in
discovering the heating effect of electric 94. Exchange rates state the value of one
current? currency in terms of other currencies.
Which one of the following statements with
A. Isaac Newton B. Galileo Galilei respect to the exchange rate of currency is
C. James P. Joule D. J.J. Thomson correct?

89. A non-spherical shining spoon can generally A. Floating exchange rates are rates in
be considered as a which the Governments interfere by
buying or selling their currencies.
A. Spherical mirror B. Parabolic mirror
B. Fixed exchange rates are rates set by
C. Plane mirror D. Lens
Government decisions and maintained
90. Which one of the following is an electric by Government actions.
conductor?
C. Under the Bretton Woods System, the
A. A plastic sheet exchange rates are floated in terms of
rise or fall in price of gold.
B. Distilled water
D. Under the classical gold standard, the
C. Human body
exchange rates are fixed in terms of
D. A wooden thin sheet price of dollar.
91. We are given three copper wires of 95. The excess of total expenditure of
different lengths and different areas of Government over its total receipts,
cross-section. Which one of the following excluding borrowings, is known as
would have highest resistivity?
A. Primary deficit
A. Copper wire of 50 cm length and 1 mm
diameter B. Fiscal deficit

B. Copper wire of 23 cm length and 0.5 C. Current deficit


mm diameter
D. Capital deficit
C. Copper wire of 10 cm length and 2.0
96. The situation where the equilibrium level of
mm diameter
real GDP falls short of potential GDP is
D. All the wires would have same known as
resistivity
A. Recessionary gap
92. Which one of the following British firms was
taken over by Soorajmull-Nagarmull group? B. Inflationary gap

A. McLeod B. Octavius Steel C. Demand-side inflation

C. Davenport D. Andrew Yule D. Supply-side inflation

PAGE 126
www.byjusexamprep.com

97. Which one of the following is not a function 102. Who among the following socialist activists
of money? persuaded Gandhiji not to restrict the salt
march protest to men alone?
A. Acts as an intermediate in the
exchange process A. Kamaladevi Chattopadhyay
B. Acts as a store of value B. Sarojini Naidu
C. Used as the unit of account C. MatanginiHazra
D. Used for regulating consumption D. Mithuben Petit
98. Which one of the following sea ports gained 103. Who among the following was popularly
significance for handling iron-ore exports to known as 'Lokahitwadi'?
Japan?
A. Gopal Hari Deshmukh
A. Kandla B. Ennore
B. MahadevGovindRanade
C. Kochi D. Mormugao
C. Gopal Krishna Gokhale
99. Arabica, Robusta and Liberica are varieties
D. JyotibaPhule
of
104. Alluvial soils vary in nature from sandy loam
A. Coffee B. Теа
to clay. They are generally
C. Sugarcane D. Cotton
A. poor in potash and rich in phosphorus.
100. Identify the type of soil on the basis of the
B. poor in both potash and phosphorus.
given characteristics:
C. rich in both potash and phosphorus.
1. They are rich in lime, iron, magnesia
and alumina. D. rich in potash and poor in phosphorus,
2. They are generally clayey, deep and 105. Which one of the following statements
impermeable. about primary waves of earthquakes is not
correct?
3. They are mainly found in Maharashtra,
Madhya Pradesh and Gujarat. A. They are similar to sound waves.
Select the correct answer from the given B. They can travel only through solid
alternatives: materials.
A. Laterite soil C. They travel through gaseous, liquid and
solid materials.
B. Red and yellow soil
D. They move faster and are the first to
C. Saline soil
arrive at the surface.
D. Black soil
106. Ozone layer, which absorbs the ultra-violet
101. Which one of the following painters was radiation, is found in which one of the
not associated with Humayun? following layers of the atmosphere?
A. Mir Sayyid Ali A. Ionosphere
B. MaulanaDostMusawir B. Troposphere
C. Maulana Yusuf C. Mesosphere
D. Bihzad D. Stratosphere

PAGE 127
www.byjusexamprep.com

107. Which one of the following is a type of 111. Article 231 of the Constitution of India
igneous rock? grants power to establish a common High
Court for two or more states to
A. Marble B. Halite
A. the Parliament.
C. Granite D. Shale
B. the Supreme Court.
108. Identify the crop on the basis of the
following characteristics: C. the President of India.
1. It is a kharif crop. D. the Union Cabinet.
2. Aus, Aman and Boro are its three 112. Which one of the following is not an
different growing periods in an objective of NITI Aayog?
agricultural year.
A. It provides a critical direction and
3. About one-fourth of the total cropped strategic input for development
area of India is under its cultivation. process.
Select the correct answer using the code B. It functions as a "think tank in providing
given below: key elements of policy.
A. Wheat C. It monitors and evaluates
implementation of the programmes.
B. Rice
D. It offers a platform for resolution of
C. Pulses
inter-state conflicts as provider of first
D. Cotton and last resort'.
109. Which one among the following was 113. Which one among the following is not a
eliminated by the 44th Amendment Act of feature of centralization?
the Parliament?
A. National unity B. Uniformity
A. Right against Exploitation
C. Prosperity D. Liberty
B. Right to Constitutional Remedies
114. Who founded the Central Hindu College at
C. Right to Property Benaras which was later developed into
Benaras Hindu University?
D. Right to Education
A. Madan Mohan Malaviya
110. Which one among the following motions
cannot be made while introducing an B. Annie Besant
ordinary Bill in the Parliament?
C. Ishwar Chandra Vidyasagar
A. That the Bill be taken into
D. Madame H.P. Blavatsky
consideration
115. In 1943, young leaders in which one of the
B. That the Bill be circulated for the
following districts of Maharashtra set up a
purpose of eliciting public opinion
parallel Government (pratisarhar) with
C. That the Bill be referred to a Select volunteer corps (seva dals) and village units
Committee (tufan dals)?
D. That the Bill be referred to a Joint A. Pune B. Nasik
Committee of the House without the
C. Nagpur D. Satara
concurrence of the other House

PAGE 128
www.byjusexamprep.com

116. Which of the following statements about Select the correct answer using the code
the Sanyasi and Faqir given below:
disturbances/rebellions are correct?
A. 4-2-3-1 B. 4-3-2-1
1. Governor-General Warren Hastings
C. 1-3-2-4 D. 1-2-3-4
faced the persistent Sanyasi and Faqir
disturbances in Bengal and Bihar. 118. Which one of the following statements
about the Coriolis force is not correct?
2. There were a number of Shaivite Naga
Sanyasis who formed into armed A. It is maximum at the Poles.
bands.
B. It is absent at the Equator.
3. Majnu Shah, who led bands into Bengal
C. It deflects the wind to the right
from 1771, was their prominent leader.
direction in the southern hemisphere.
Select the correct answer using the code
D. It deflects the wind to the right
given below:
direction in the northern hemisphere.
A. 1 and 2 only B. 1, 2 and 3
119. When the rivers discharge their waters
C. 2 and 3 only D. 1 and 3 only from all directions into a lake or depression,
the pattern is known as
117. Chronologically arrange the following
treaties/conventions signed between the A. Trellis B. Dendritic
Marathas and the British (starting with the
C. Radial D. Centripetal
earliest):
120. Which one of the following tributaries of
1. Treaty of Salbai
river Indus feeds the canal system of the
2. Treaty of Purandar Bhakra Nangal Project?
3. Convention of Wadgaon A. Chenab B. Satluj
4. Treaty of Surat C. Ravi D. Jhelum

PAGE 129
www.byjusexamprep.com

Elementary Mathematics
1. Consider the following data with regard to A. 8.5 B. 9.5
production of cars (in lakhs):
C. 10.2 D. 11.2
5. The marks obtained by 5 students are 21,
27, 19, 26, 32. Later on 5 grace marks are
added to each student. What are the
average marks of the revised marks of the
students?
A. 26 B. 30
In which of the countries, the production of
cars has increased by more than or equal to C. 31 D. 32
5% in 2016 over 2015?
6. Let p be the mean of m observations and q
A. B and E be the mean of n observations, where
. If the combined mean of
B. A, C and D only
observations is c, then which of
C. A, C, D and E
the following is correct?
D. A, D and E only
A. B.
Direction: The following table shows the marks
of 90 students in a test of 80 marks: C. D.

Direction: For the next four items, consider the


following data with regard to different types (I,
II, III, IV, V) of multivitamin tablets produced in a
company (in lakhs):

2. The percentage of students who have


obtained less than or equal to 50% marks is:
A. 30% B. 40%
7. Which product is produced least over the
C. 45% D. 60% years 2000-2005?
3. What is the median of the following data? A. Type II B. Type III
2, 3, -1, 2, 6, 8, 9 C. Type IV D. Type V
A. 2 B. 3 Direction: For the next four items, consider the
C. 4 D. 5 following data with regard to different types (I,
II, III, IV, V) of multivitamin tablets produced in a
4. What is the arithmetic mean of the first ten company (in lakhs):
composite numbers?

PAGE 130
www.byjusexamprep.com

10. In which year, the production of type I is


8. In which one of the following pairs of years,
more than the sum of the production of
the difference in total number of tablets
type III and Type IV?
produced between them is minimum?
A. 2001 B. 2002
A. (2003, 2005)
C. 2003 D. 2004
B. (2001, 2005)
11. The sides of a right-angled triangle are in
C. (2003, 2004)
the ratio . What is the
D. (2000, 2002) perimeter of the triangle?
Direction: For the next four items, consider the A. 28 units B. 42 units
following data with regard to different types (I,
II, III, IV, V) of multivitamin tablets produced in a C. 56 units D. 84 units
company (in lakhs): 12. ABC is a triangle right angled at B. let M and
N be two points on AB such that
. Let P and Q be two points
on AC such that PM is parallel to QN and QN
is parallel to CB. If BC = 12 cm, then what is
(PM + QN) equal to?
A. 10 cm B. 11 cm
C. 12 cm D. 13 cm
13. AB and CD are the diameters of a circle
9. The ratio of percentage drop in total which intersects at P. Join AC, CB, BD and
production in 2004 compared to 2001 to DA. If , then what is
that in 2000 compared to 2001, is equal to?

A. B. A. 30° B. 60°
C. 90° D. 120°

C. D. 14. An equilateral triangle ABC and a scalene


triangle DBC are inscribed in a circle on
same side of the arc. What is equal
Direction: For the next four items, consider the
following data with regard to different types (I, to?
II, III, IV, V) of multivitamin tablets produced in a A. 30o B. 45o
company (in lakhs):
C. 60o D. 90o

PAGE 131
www.byjusexamprep.com

15. The sides of a triangle ABC are 4 cm, 6 cm A. 1:1 B. 2:1


and 8 cm. with the vertices of the triangle
C. 3:2 D. 4:3
as centres three circles are drawn each
touching the other two externally. What is 22. A solid sphere of diameter 60 mm is melted
the sum of the radii of the three circles? to stretch into a wire of length 144 cm.
A. 6 cm B. 7 cm What is the diameter of the wire?

C. 9 cm D. 10 cm A. 0.5 cm B. 1 cm

16. Let PAB be a secant to a circle intersecting C. 1.5 cm D. 2 cm


the circle at A and B. let PT be the tangent 23. The ratio of the radius of base to the height
segment. If PA = 9 cm and PT = 12 cm, them of a cylinder is 2 : 3. If the volume of the
what is AB equal to? cylinder is 1617 cm3 , then what is the
A. 5 cm B. 6 cm curved surface area of the cylinder?
C. 7 cm D. 8 cm
17. If the perimeter of a right-angled triangle is
30 cm and the hypotenuse is 13 cm, them A. 242 cm2 B. 385 cm2
what is the area of the triangle?
C. 462 cm2 D. 770 cm2
2 2
A. 24 cm B. 27 cm
24. The difference between the outside and
C. 30 cm2 D. 36 cm2 the inside surface area of a cylindrical pipe
18. ABC is a triangle right angled at C. let p be 14 cm long is 44 cm2. The pipe is made of 99
the length of the perpendicular drawn from cm3 of metal. If R is the outer radius and r is
C on AB. If BC = 6 cm and CA = 8 cm, then the inner radius of the pipe, then what is
what is the value of p? (R+r) equal to?
A. 5.4 cm B. 5 cm
A. 9 cm B. 7.5 cm
C. 4.8 cm D. 4.2 cm
19. ABCD is a trapezium in which AB is parallel C. 6 cm D. 4.5 cm
to DC and 2AB = 3DC. The diagonals AC and 25. A metal solid cube of edge 24 cm is melted
BD intersect at O. what is the ratio of the and made into three small cubes. If the
area of to that of ? edge of two small cubes are 12 cm and 16
cm, then what is the surface area of the
A. 2:1 B. 3:2
third small cube?
C. 4:1 D. 9:4
A. 1200 cm2 B. 1800 cm2
20. A circle touches all the four sides of a
quadrilateral ABCD. If AB = 9 cm, BC = 8 C. 2400 cm2 D. 3600 cm2
cm and CD = 12 cm, then what is DA equal 26. A conical vessel whose internal radius is 5
to? cm and height 24 cm is full of water. The
A. 14 cm B. 13 cm water is emptied into a cylindrical vessel
with internal radius 10 cm. what is the
C. 12 cm D. 11 cm height to which the water rises?
21. A cone and a hemisphere have equal bases
A. 1 cm B. 2 cm
and equal volumes. What is the ratio of the
height of the cone to the radius of the C. 3 cm D. 4 cm
hemisphere?

PAGE 132
www.byjusexamprep.com

27. A metal solid cube of side 22 cm is melted Which of the above is/are correct?
to make a cone of height 21 cm. what is the
radius of the base of the cone? A. 1 only B. 2 only
C. Both 1 and 2 D. Neither 1 nor 2
32. If and , then
A. 11 cm B. 16.5 cm what is equal to?
C. 22 cm D. 27.5 cm
A. –1 B. 0
28. A cone of height 24 cm has a curved
surface area 550 cm2. What is the ratio of C. 1 D. 2

its radius to slant height? 33. If , then what is equal


to?

A. B. A. B.

C. D.
C. D.

34. Let ABC be a triangle right angled at C, then


29. A rectangular paper is 44 cm long and 22 cm
what is equal to?
wide. Let be the volume of the largest
cylinder formed by rolling the paper along
A. B.
its length and be the volume of the
largest cylinder formed by rolling the paper
along its width. What is the ratio of to C. D.

? 35. Let and ,


where . What is the
A. 1 : 1 B. 2 : 1 value of ?
C. 1 : 2 D. 3 : 2 A. 0
30. A hollow spherical shell is made up of a B. 1
metal of density 3 g/cm3. If the internal and
external radii are 5 cm and 6 cm C. 2
respectively, then what is the mass of the
D. Cannot be determined due to
shell? insufficient data

A. 1144 g B. 1024 g 36. If , where ,

C. 840 g D. 570 g then what is the value of ?

31. Consider the following inequalities:


A. B.
I.
C. D. 1
II.

PAGE 133
www.byjusexamprep.com

37. What is A. 0.5 cm B. 1 cm


equal to? C. 1.5 cm D. 2 cm
A. 4 B. 3 43. A sector is cut from a circle of radius 21 cm.
if the length of the arc of the sector 55 cm,
C. 2 D. 1
what is the area of the sector?
38. If 6 + 8 tan q = sec q and 8 – 6 tan q = sec q,
A. 577.5 cm2 B. 612.5 cm2
then what is the value of k2?
C. 705.5 cm2 D. 725.5 cm2
A. 11 B. 22
44. A wire is in the form of a circle of radius 70
C. 77 D. 99 cm. if it is bent in the form of a rhombus,
39. A vertical tower standing at the corner of a then what is its side length?
rectangular fields subtends angle of 60o and
45o at the two nearer corners. If θ is the
angle that the tower subtends at the
farthest corner, then what is cotθ equal to? A. 55 cm B. 75 cm
C. 95 cm D. 110 cm
A. B. 2
45. If the perimeter of a semicircular park is 360
m, then what is its area?
C. D.
A. 3850 m2 B. 7700 m2
o
40. A pole on the ground leans at 60 with the
C. 11550 m2 D. 15400 m2
vertical. At a point x metre away from the
base of the pole on the ground, two halves 46. In a trapezium ABCD, AB is parallel to DC.
of the pole subtend the same angle. If the The diagonal AC and BD intersect at P. if
pole and the point are in the same vertical AP : PC = 4 : (4x - 4) and BP : PD = (2x - 1) :
plane, then what is the length of the pole? (2x + 4), then what is the value of ?

A. 2x metre B. 3x metre A. 4 B. 3

C. 2x metre D. 2 2x metre C. D. 2
41. A cloth of 3 m width is used to make a
conical tent 12 m in diameter with a slant 47. D ABC is similar to D DEF. The perimeters of
height of 7 m. what is the length of the D ABC and D DEF are 40 cm and 30 cm
respectively. what is the ratio of (BC +
cloth? CA) to (EF + FD) equal to?
A. 5 : 4 B. 4 : 3
A. 21 m B. 28 m
C. 3 : 2 D. 2 : 1
C. 44 m D. 66 m
48. Two isosceles triangles have equal vertical
42. A sphere of diameter 6 cm is dropped into angle and their areas are in the ratio 4.84 :
a cylindrical vessel partly filled with water. 5.29. What is the ratio of their
The radius of the vessel is 6 cm. if the corresponding heights?
sphere is completely submerged in water,
A. 11:23 B. 23:25
then by how much will the surface level of
water be raised? C. 22:23 D. 484:529

PAGE 134
www.byjusexamprep.com

49. ABC is a triangle right angled at A and AD is


56. If , where , then
perpendicular to BC. If BD = 8 cm and DC =
12. 5 cm, then what is AD equal to? which of the following is correct?
A. 7.5 cm B. 8.5 cm A.
C. 9 cm D. 10 cm B. only
50. What is log1031.25 equal to? C. only
A. D.

B. 57. If for then


C. consider the following statements:

D. I. P can be less than .

51. What is the square root of ?


II. P can be more than 1.
A. B. Which of the above statements is/are
correct?
C. D.
A. 1 only B. 2 only
52. The sum of the reciprocals of two alternate
C. Both 1 and 2 D. Neither 1 nor 2
natural numbers is . What is the sum of
58. What is the ratio of the greatest to the
the numbers?
smallest value of ,
A. 12 B. 13
?
C. 14 D. 16
A. –3 B. –2
53. If n is any natural number, then is
always divisible by how many natural C. –1 D. 1
numbers? 59. If the equation
A. 1 B. 4 contains real solution for x and y, then

C. 6 D. 8 A. B.

54. If , then what is x equal to? C. D.


60. A trader gives successive discounts of 20%,
A. B.
10% and 5% respectively. what is the
overall discounts?
C. D. A. 30% B. 31.6%
C. 32.8% D. 35%
55. What is the least value of ?
61. A sum of money was invested ate simple
A. 5 B. 4
interest at a certain rate for 5 years. Had it
C. 3 D. 2 been invested at 5% higher rate, it would

PAGE 135
www.byjusexamprep.com

have fetched Rs.500 more. What was the


67. What should be added to to
principal amount?
A. Rs. 2000 B. Rs. 1800 get ?
C. Rs. 1600 D. Rs. 1200
62. The difference between the compound A.
interest (compounded automatically) and
the simple interest on a certain sum of B.
money at 12% per annum for 2 years is Rs.
72 . What is the principal amount?
C.
A. Rs. 6,500 B. Rs. 6,000
C. Rs. 5,500 D. Rs, 5000 D.

63. Walking at of his usual speed, a man is


68. If and , then
12 minutes late for his office. What is the
usual time taken by him to cover that what is equal to?
distance?
A. 48 minutes A. -2 B. -1
B. 50 minutes C. 0 D. 1
C. 54 minutes 69. If is the HCF of and
D. 60 minutes , then what is the value of k?
64. The cost price of 100 mangoes is equal to A. B.
the selling price of 80 mangoes. What is the
profit percentage?
C. D.
A. 16% B. 20%
C. 24% D. 25% 70. Consider the following statements:
65. In a mixture of 80 litres of a liquid and I. If x is directly proportional to z and y is
water, 25% of the mixture is the liquid. How directly proportional to z, then (x2 –
much water should be added to the mixture y2) is directly proportional to z2.
so that the liquid becomes 20% of the II. If x is inversely proportional to z and y
mixture? is inversely proportional to z, then xy is
A. 15 litres B. 20 litres inversely proportional to z2.

C. 24 litres D. 25 litres Which of the above statements is/are


correct?
66. If 20 persons can clean 20 floors in 20 days,
then in how many days can 16 persons A. 1 only
clean 16 floors? B. 2 only
A. 25 days B. 24 days C. Both 1 and 2
C. 20 days D. 16 days D. Neither 1 nor 2

PAGE 136
www.byjusexamprep.com

78. How many pairs of (x, y) can be chosen


71. What is equal
from the set {2, 3, 6, 8, 9} such that
to? , where x ≠ y?
A. 0 B. 1
A. 0 B. 1
C. 2 D. 4
72. For what integral value of x is C. 2 D. 3
79. Consider the pairs of prime numbers (m, n)
between 50 and 100 such that .
How many such pairs are there?

A. 4 B. 3 A. 2 B. 3

C. 2 D. 1 C. 4 D. 5

73. What is the HCF of and 80. How many terms are there in the following
product?
?
A. B.

C. D.

74. If , then which A. 15 B. 30


one of the following is a possible expression C. 45 D. 60
for k?
81. What is the remainder when is
A. B. divided by 6?
C. D. A. 0 B. 1

75. What is C. 3 D. 4

82. If , then which of the


equal to? following are correct?

I.
A. B. 3
II.
C. D. 0
76. If the number is divisible by III.
13, then what is the value of P?
Select the correct answer using the code
A. 3 B. 6 given below.
C. 7 D. 8 A. 1 and 2 only
77. What is the remainder when is B. 2 and 3 only
divided by 7?
C. 1 and 3 only
A. 1 B. 2
D. 1, 2 and 3
C. 4 D. 6

PAGE 137
www.byjusexamprep.com

90. 4x3 + 12x2 – x – 3 is divisible by


83. If , then what is the
A. only
value of ?
A. 0 B. 1 B. only

C. 2 D. 3 C. Both and
84. What is the unit digit I the expression of
D. Neither nor
6732?
A. 1 B. 3 91. Which one of the following fractions will
have minimum change in its value if 3 is
C. 7 D. 9 added to both the numerator and
denominator of all the fractions?
85. What is the value of x, if ?
A. B.
A. B.
C. D.
C. D.
92. Let the average score of a class of boys and
86. For how many real values of k is girls in an examination be p. the ratio of
a perfect square for boys and girls in the class is 3 : 1. If the
every integer x? average score of the boys is , then
A. 0 B. 1 what is the average score of the girls?

C. 2 D. 4 A. p-1 B. p-2
C. p-3 D. p
87. If , then what is equal
93. The incomes of A, B and C are in the ratio 7
to? : 9 : 12and their expenditures are in the
A. B. ratio 8 : 9 : 15. If A’s saving is one-fourth of
his income, then the ratio of savings of A, B
and C is
C. D. 0
A. 56 : 99 : 69
2
88. If the equation 4x – 2kx + 3k = 0 has equal
B. 99 : 56 : 69
roots, then what are the values of k?
A. 4, 12 B. 4, 8 C. 69 : 56 : 99

C. 0, 12 D. 0, 8 D. 99 : 69 : 56

89. If the sum as well as the product of the


94. If ,
roots of the equation is
6, then what is equal to? then what is the value of n?

A. 8 B. 7 A. 98 B. 99

C. 6 D. 5 C. 100 D. 101

PAGE 138
www.byjusexamprep.com

95. X sells his goods 25% cheaper than Y and 98. A train travels 600 km in 5 hours and the
25% dearer than Z. How much percentage next 900 km in 10 hours. What is the
is Z’s goods cheaper than Y? average speed of the train?
A. 80 kmph B. 90 kmph
A. B. 40%
C. 100 kmph D. 120 kmph
C. 50% D. 99. Let the work done by (x – 1) men in (x + 1)
days be y. Let the work done by (x + 2) men
in (x – 1) days be z. If y : z = 9 : 10, then what
96. The expression
is the value of x?
simplifies to: A. 8 B. 9
A. B. C. 10 D. 12
C. D. 100. The surface area of a cube is equal to that
of a sphere. If x is the volume of the cube
97. A train 200 m long passes a platform 100m
and y is the volume of the sphere, then
long in 10 seconds. What is the speed of the
what is x2 : y2 equal to?
train?
A. 40 m/s B. 30 m/s A. B.

C. 25 m/s D. 20 m/s C. D.

PAGE 139
www.byjusexamprep.com

ANSWERS KEY I 2021


ENGLISH

1 C 21 A 41 A 61 B 81 A 101 A
2 C 22 B 42 B 62 C 82 D 102 B
3 C 23 C 43 A 63 B 83 A 103 A
4 C 24 B 44 C 64 D 84 C 104 C
5 C 25 C 45 B 65 A 85 D 105 D
6 B 26 A 46 A 66 A 86 A 106 B
7 A 27 C 47 A 67 C 87 A 107 C
8 D 28 D 48 A 68 C 88 C 108 A
9 C 29 B 49 C 69 B 89 A 109 D
10 D 30 A 50 A 70 B 90 A 110 C
11 A 31 D 51 B 71 A 91 C 111 A
12 B 32 A 52 B 72 B 92 B 112 A
13 C 33 D 53 C 73 D 93 C 113 B
14 B 34 D 54 B 74 B 94 A 114 A
15 A 35 C 55 A 75 A 95 B 115 A
16 A 36 D 56 D 76 B 96 B 116 C
17 A 37 B 57 A 77 B 97 C 117 A
18 D 38 C 58 D 78 B 98 B 118 D
19 A 39 C 59 A 79 C 99 B 119 A
20 C 40 B 60 C 80 A 100 D 120 C

GENERAL KNOWLEDGE
1 D 21 B 41 B 61 A 81 B 101 D
2 B 22 A 42 B 62 D 82 D 102 A
3 A 23 A 43 C 63 B 83 B 103 A
4 D 24 C 44 C 64 C 84 C 104 D
5 D 25 A 45 D 65 B 85 D 105 B
6 B 26 B 46 A 66 D 86 B 106 D
7 A 27 A 47 B 67 D 87 D 107 C
8 A 28 B 48 A 68 D 88 C 108 B
9 D 29 C 49 B 69 A 89 B 109 C
10 A 30 D 50 B 70 D 90 C 110 D
11 B 31 B 51 B 71 B 91 D 111 A
12 D 32 A 52 C 72 C 92 A 112 D

PAGE 140
www.byjusexamprep.com

13 D 33 C 53 A 73 B 93 D 113 D
14 D 34 A 54 D 74 A 94 B 114 B
15 B 35 D 55 C 75 D 95 B 115 D
16 B 36 D 56 B 76 D 96 A 116 B
17 A 37 B 57 A 77 A 97 D 117 A
18 C 38 C 58 D 78 B 98 D 118 C
19 A 39 C 59 C 79 C 99 A 119 D
20 C 40 B 60 A 80 C 100 D 120 B

Elementary Mathematics
1 C 21 B 41 C 61 A 81 A
2 B 22 B 42 B 62 D 82 D
3 B 23 C 43 A 63 A 83 B
4 D 24 D 44 D 64 D 84 A
5 B 25 C 45 B 65 B 85 C
6 C 26 B 46 B 66 C 86 C
7 B 27 C 47 B 67 A 87 B
8 A 28 C 48 C 68 C 88 C
9 C 29 B 49 D 69 B 89 B
10 C 30 A 50 A 70 C 90 C
11 C 31 A 51 A 71 A 91 D
12 C 32 A 52 C 72 C 92 C
13 D 33 D 53 D 73 D 93 A
14 C 34 D 54 A 74 A 94 B
15 C 35 A 55 C 75 D 95 B
16 C 36 C 56 B 76 D 96 B
17 C 37 C 57 D 77 B 97 B
18 C 38 D 58 B 78 A 98 C
19 D 39 C 59 A 79 D 99 A
20 B 40 B 60 B 80 D 100 A

PAGE 141
CDS I 2021
www.byjusexamprep.com

(Solutions)
ENGLISH
1. Ans. C.
The error is in part C of the given sentence.
'During' must be replaced with 'since'.
2. Ans. C.
The error is in part C of the given sentence.
With ‘preferred’ we use the preposition ‘to’ instead of ‘than’.
3. Ans. C.
The error is in part C of the given sentence.
It should be ‘is’ instead of ‘are.
4. Ans. C.
The error is in part C of the given sentence.
With ‘risk’, we use the preposition ‘at’.
5. Ans. C.
The error is in part C of the given sentence.
The entire sentence is in present tense. And the subject ‘we’ is first person plural form, so the verb
will be in plural i.e., ‘fall’.
6. Ans. B.
The error is in part B of the given sentence.
The entire sentence is in singular form, so singular form of ‘modes’ will be use which is ‘mode’.
7. Ans. A.
The error is in part A of the given sentence.
‘been’ cannot be used with ‘was’. Remove 'been'.
8. Ans. D.
The given sentence is error-free.

PAGE 142
www.byjusexamprep.com

9. Ans. C.
The error is in part C of the given sentence.
Whenever ‘one of’ is used, sentence will be in plural.
10. Ans. D.
The given sentence is error-free.
11. Ans. A.
Idiom 'To be in eclipse’ means - To be less successful.
12. Ans. B.
Idiom ‘Ways and means’ mean - methods of achieving something.
13. Ans. C.
Idiom ‘Up in arms’ means - very angry.
14. Ans. B.
Idiom ‘Big ticket’ means - very costly.
15. Ans. A.
Idiom ‘Bolt from the blue’ means - an event or piece of news which is unexpected.
16. Ans. A.
Idiom ‘Be a law unto yourself’ means - to behave unconventional and unpredictable.
17. Ans. A.
Idiom ‘Spiff up’ means to make oneself look neat.
18. Ans. D.
Idiom ‘Run wild’ means - to grow without control.
19. Ans. A.
Idiom ‘Wind down’ means - to relax after a period of activity.
20. Ans. C.
‘Mellow out’ means - to enjoy oneself without doing much.
21. Ans. A.
the most appropriate answer according to the passage is option A - It does not encourage
disagreement.
22. Ans. B.
Option B - accepting the established practices, is the most appropriate answer.
23. Ans. C.
As clearly mentioned in the passage, 'Because they seek comfort' is the correct answer choice.

PAGE 143
www.byjusexamprep.com

24. Ans. B.
Prejudice means give rise to prejudice in (someone); make biased.
Hence, option is the correct answer.
25. Ans. C.
Uncomprehendingly means showing or having an inability to comprehend something.
Hence, option C - Without knowing, is the most appropriate answer choice.
26. Ans. A.
As clearly mentioned in the passage, The correct answer is option A - 30 metres.
27. Ans. C.
It is clearly mentioned in the passage that Trees can reduce noise when they are planted dense.
Hence, option C is the correct answer.
28. Ans. D.
The single layer of trees is worthless as a noise barrier because noise can easily penetrate.
Hence, option D is the most apropriate answer.
29. Ans. B.
The higher the trees, lower the noise.
Option B is the correct answer.
30. Ans. A.
Shrubs and tall grass reduces noise by absorbing it.
Hence, option A is the correct answer.
31. Ans. D.
The correct sequence is - RQSP.
Correct Sentence- Freedom does not mean the opportunity for self- gratification or the setting aside
of consideration for others.
32. Ans. A.
The correct sequence is - RPSQ.
Correct Sentence- Domination or compulsion of any kind is a direct hindrance to freedom and
intelligence.
33. Ans. D.
The correct sequence - QPSR.
Correct sentence- Banana provides enough potassium, and sustained energy from various sugars.
34. Ans. D.
The correct sequence - QPSR.
This is a time of great trouble as one virus threatens the world to the maximum.

PAGE 144
www.byjusexamprep.com

35. Ans. C.
The correct sequence - QPSR
Some archeologists are of the opinion that Harappan society had no rulers, and that everybody
enjoyed equal status.
36. Ans. D.
The correct sequence - RQSP.
Endogamy refers to marriage within a unit or a group living in the same locality.
37. Ans. B.
The correct sequence - RSPQ
Coal was the main source of energy in the initial stages, when steam engines dominated
industrialization.
38. Ans. C.
The correct sequence - SQRP
How important is the information that comes from the scientists?
39. Ans. C.
The correct sequence - SQRP.
Since earliest times, humans have suffered frustration and food losses brought on by destructive
pests.
40. Ans. B.
The correct sequence - QPSR.
Some of the most spectacular gold coins were issued by the Gupta rulers in India.
41. Ans. A.
Man has progressed with science and technology. He has reached the moon. Hitherto he had been
backward. Now with scientific knowledge and technical know-how he has done what was impossible.
He is one the road to conquer other planets. What was difficult and impossible is now easy and
possible.
42. Ans. B.
We drink more water during summers. School clothes during the summer is different from the
winter. Why do you wear lighter clothes during summers and heavy woolen clothes during the winter
in north India? Because in North India, both the sessions have extreme weather conditions. On the
other hand, in north-eastern states, winters are mild except in the hills. There are variations in
weather conditions during different seasons.
43. Ans. A.
Policy, like many other terms is a complex concept. It takes different forms. A policy may be general
or specific, broad or narrow, simple or complex, public or private, written or unwritten. Public policy
is what a government chooses as guidance for action. It is form of government control usually
expressed in a law, a legislation, a regulation or an order. Public policy is the outcome of the struggle
between government and powerful groups.

PAGE 145
www.byjusexamprep.com

44. Ans. C.
The dhoti kurta is a traditional lungi type garment worn by men in India. The dhoti is rectangular
piece of unstitched cloth, around five yards long, wrapped around the midriff and tucked between
the legs. It is commonly worn with a loose long tunic. A dhoti, made of homespun khadi cotton cloth,
was the garment favored by Gandhi ji. This he started wearing after he abandoned his western attire
of pants and shirt. Gandhiji usually wore a simple shawl with his dhoti.
45. Ans. B.
Languages in contact with each other cause change. Language contact has long been recognized as
a major engine of change. A historical example is that of Danish and English which led to a major shift
in the vocabulary and grammar of English. The increasing use of English in many parts of the world
affect both local languages and English. This gives rise to new, hybrid language varieties. Changes
often occur first in informal and casual language.
46. Ans. A.
QRSP- is the most appropriate sequence for the given jumbled sentences.
Hence, option A is the correct answer.
47. Ans. A.
Ethics is a branch of philosophy concerned with the distinction between right and wrong and the
ultimate worth of actions or things. Ethics evaluated the relationships, rules, principles or codes that
require or forbid certain conduct. Environmental ethics asks about the moral relationships between
humans and the world around us. For many philosophers, only humans are moral agents, being
capable of acting morally or immorally. Value is a measure of the worth of something.
48. Ans. A.
India was to be a secular and a democratic republic with a parliamentary system based on adult
franchise. It was also to be a federation with demarcation of spheres of action between the Union
Governments and the State Governments. It guaranteed all Indian citizens certain fundamental
rights. These include: freedom of speech and expression, freedom to assemble peacefully etc.
49. Ans. C.
The policy aims at developing an integrated health information system that serves the need of all
stakeholder and improves efficiency, transparency. This will also foster a linkage across public and
private healthcare. Ayushman Bharat compromising twin missions is set to transform the nation’s
health system. This will be carried out by operationalizing firstly 1.5 lakh health and wellness centers
offering preventive and primary care.
50. Ans. A.
Most species distributions result from a combination of all these factors. But biogeographers tend to
specialize in ecological aspects. Ecological biogeographers are interested in the effects of
environmental factors. Some historical biogeographers are interested in finding center-of-origins.
51. Ans. B.
‘was broke’ is the correct usage as it means being bankrupt.
52. Ans. B.
‘One of the players’ is the correct option to fill in the blank.

PAGE 146
www.byjusexamprep.com

53. Ans. C.
‘met’ is the correct usage because the sentence is in past tense so verb will also be in past.
Hence, option C is correct.
54. Ans. B.
‘more than’ is the correct option to fill in the blank.
55. Ans. A.
‘about a’ is the correct option to fill in the blank.
56. Ans. D.
‘would have completed’ is the correct usage because the sentence is in past tense so the verb will be
in past tense.
57. Ans. A.
‘whose’ is the correct pronoun to fill the given blank.
58. Ans. D.
‘in such a way’ is the correct usage.
59. Ans. A.
‘could have made’ is the correct usage.
60. Ans. C.
‘will result’ is the correct usage.
61. Ans. B.
The most appropriate antonym for bombastic is straightforward which means direct in your
approach.
Outdated means not useful or common any more.
Verbose means containing more words than necessary.
Not true means inaccurate.
62. Ans. C.
The most appropriate antonym for absurd is reasonable which means fair.
Bizarre means strikingly out of the ordinary.
Meaningless means without meaning.
Thoughtful means thinking deeply.
63. Ans. B.
The most appropriate antonym for Deceased is alive which means not dead.
Injured means physically or mentally hurt.

PAGE 147
www.byjusexamprep.com

Dead means no longer alive.


Survived means to continue to live or exist in or after a difficult or dangerous situation.
64. Ans. D.
The most appropriate antonym for solace is aggravation which means to make a bad situation worse.
Comfort means the state of having everything your body needs, or of having a pleasant life.
Relief means the feeling of happiness that occurs when something unpleasant or distressing stops or
does not happen.
Punishment means the action or way of punishing somebody.
65. Ans. A.
The most appropriate antonym for contrary is similar which means like somebody/something but
not exactly the same.
Different means not the same.
Divergent means moving or extending in different directions from a common point.
Good faith means honesty or sincerity of intention.
66. Ans. A.
Option A is the correct answer.

The most appropriate antonym for spontaneous is prepared which means to get ready or to make
somebody/something ready.
Alerted means to warn somebody of danger or a problem.
Develop means to grow or cause to grow and become more mature, advanced, or elaborate.
Well executed means skillfully carried out.
67. Ans. C.
The most appropriate antonym for asserting is denying which means to state that something is not
true; to refuse to admit or accept something.
Declaring means to state something publicly and officially or to make something known in a firm,
clear way
Supporting means to help somebody by saying that you agree with him/her/it.
Propagating means to multiply.
68. Ans. C.
The most appropriate antonym for exonerated is convicted which means having officially been found
guilty of a crime in a law court.
Honored means to show great (public) respect for somebody/something or to give somebody pride
or pleasure.

PAGE 148
www.byjusexamprep.com

Pardoned is granted to an individual, often by the action of a government official such as a governor,
president.
Felicitated means to congratulate.
69. Ans. B.
The most appropriate antonym for persuasion is discouraging which means to deprive of courage or
confidence.
Dislike means not liked.
Convincing means able to make somebody believe something.
Deter means to turn aside, discourage.
70. Ans. B.
The most appropriate antonym for distinctive is common which means
happening or found often or in many places.
Unique means not like anything else; being the only one of its type.
Special means not usual or ordinary.
Unfamiliar means that you do not know well.
71. Ans. A.
‘a’ is the correct usage.
72. Ans. B.
‘the’ is the correct usage.
73. Ans. D.
Artcile ‘a’ is the correct usage.
74. Ans. B.
Preposition ‘in’ is the correct usage.
75. Ans. A.
‘any’ is the correct usage.
76. Ans. B.
‘between’ is the correct usage.
77. Ans. B.
‘and’ is the correct usage.
78. Ans. B.
‘with’ is the correct usage.
79. Ans. C.
‘that’ is the correct usage.

PAGE 149
www.byjusexamprep.com

80. Ans. A.
‘by’ is the correct usage.
81. Ans. A.
The most appropriate synonym for commotion is uproar which means a state of commotion,
excitement, or violent disturbance.
Peace means societal friendship and harmony in the absence of hostility and violence.
Sound means something that you hear or that can be heard.
Furious means full of fury, violent passion, or rage.
82. Ans. D.
The most appropriate synonym for inherent is inbuilt which means existing as an original or essential
part of something or someone.
Outward means on the outside.
Difficult means not easy to do or understand.
Hallow means honor as holy.
83. Ans. A.
The most appropriate synonym for reprimanded is admonished which means to tell somebody firmly
that you do not approve of something that he/she has done.
Appreciated means to enjoy something or to understand the value of somebody/something.
Praised means to say that somebody/something is good and should be admired.
Disliked means to think that somebody/something is unpleasant.
84. Ans. C.
The most appropriate synonym for perpetual is ever lasting which means continuing forever; never
changing.
Temporary means lasting for a short time; not permanent.
Powerful means having a lot of control or influence over other people.
All persuasive means able to cause people to do or believe something.
85. Ans. D.
The most appropriate synonym for exemplify is demonstrate which means to show something clearly
by giving proof.
Devise means to invent a new way of doing something.
Sympathize means to feel sorry for somebody.
Asks for means to put a question to somebody in order to find out some information.
86. Ans. A.
The most appropriate synonym for crux is core which means the central or most important part of
something.

PAGE 150
www.byjusexamprep.com

Part means one of the pieces, areas, periods, things, etc. that together with others forms the whole
of something.
Idea means a plan, thought or suggestion, especially about what to do in a particular situation.
Tip means the thin or pointed end of something.
87. Ans. A.
The most appropriate synonym for competency is capability which means the quality of being able
to do something.
Thinking means using your mind to think about something.
Knowledge means a familiarity, awareness, or understanding of someone or something.
Ideal means a standard of perfection, beauty, or excellence.
88. Ans. C.
The most appropriate synonym for haughty is arrogant which means exaggerating or disposed to
exaggerate one's own worth or importance often by an overbearing manner.
Tough means strong.
Modest means not talking too much about your own abilities, good qualities, etc.
Knowledgeable means having a lot of knowledge.
89. Ans. A.
The most appropriate synonym for industrious is diligent which means showing care and effort in
your work or duties.
Knowledgeable means having a lot of knowledge.
Indolent means averse to activity, effort, or movement.
Insincere means saying or doing something that you do not really believe.
90. Ans. A.
The most appropriate synonym for indignant is resentful which means having or showing a feeling of
anger or displeasure about someone or something unfair.
Congenial means pleasant.
Unruly means behave badly and are difficult to control.
Supportive means to give help or assistance.
91. Ans. C.
‘above’ is a preposition.
92. Ans. B.
‘most beautiful’ is an adjective.
93. Ans. C.
‘Alas!’ is an interjection.

PAGE 151
www.byjusexamprep.com

94. Ans. A.
Option A is the correct answer.
‘Who’ is a pronoun (Subject pronoun).
95. Ans. B.
‘And’ is a coordinating conjunction.
A coordinating conjunction is a word that joins two elements of equal grammatical rank and syntactic
importance. The seven coordinating conjunctions are: for, and, nor, but, or, yet, and so.
96. Ans. B.
‘since’ is an adverb.
97. Ans. C.
‘This’ is an Adverb.
98. Ans. B.
‘thyself’ is a pronoun.
99. Ans. B.
‘won’ is a transitive verb.
A Transitive verb is a verb that takes a direct object, i.e. the action of a transitive verb is done to
something or someone whereas in Intransitive verb the action is not done to someone or something
but only involves subject in it.
100. Ans. D.
‘exultation’ is a noun.
101. Ans. A.
If someone is at the receiving end then that person is suffering so we will use a contrary.
Hence, the correct answer is - Sheela has been at the receiving end but she is very honest.
102. Ans. B.
‘no other’ is always followed by ‘but to’.
Hence, option B - Elam has no other option but to accept the challenge.
103. Ans. A.
The correct complete sentence is - Although the group has been acquitted they are under the watch
of police meaning that the group has already been free from a criminal charge but they are under
the watch of police.
104. Ans. C.
Complete sentence is - Unless you fulfil the essential qualifications you will not be called for the
interview meaning Unless you will fulfill the essential qualification then only you will be called for
the interview.

PAGE 152
www.byjusexamprep.com

105. Ans. D.
The company’s bid for the contract is rejected since the deposit in the account is found less than
required.
106. Ans. B.
The correct complete sentence - I could not sleep despite being very tired.
107. Ans. C.
The correct complete sentence - She married him because of his nature.
108. Ans. A.
The correct complete sentence - I will come to meet you in case you want.
109. Ans. D.
The correct complete sentence - If you had planned the work well you could have completed it on
time.
110. Ans. C.
The correct complete sentence - You can use my computer as long as you do not use any external
drive.
111. Ans. A.
‘with’ is the correct usage.
112. Ans. A.
‘an’ is the correct usage.
113. Ans. B.
‘between’ is the correct usage.
114. Ans. A.
‘some’ is the correct usage.
115. Ans. A.
‘for’ is the correct usage.
116. Ans. C.
‘a’ is the correct usage.
117. Ans. A.
‘of’ is the correct usage.
118. Ans. D.
‘the’ is the correct usage.
119. Ans. A.
‘on’ is the correct usage.
120. Ans. C.
‘up’ is the correct usage.

PAGE 153
www.byjusexamprep.com

General Knowledge
1. Ans. D.
California Current - The California Current is a cold water Pacific Ocean current that moves southward
along the western coast of North America, beginning off southern British Columbia and ending off
southern Baja California Sur.
Hence, option D is correct.
2. Ans. B.
Territorial jurisdiction extends up to 12 nautical miles from territorial waters closest to the baseline;
Beyond the territorial waters is a contiguous zone extending 24 nautical miles; And beyond that is
the Special Economic Zone of India at a distance of 200 nautical miles.
Hence, option B is correct.
3. Ans. A.
Pradhan Mantri Gram Sadak Yojana (PMGSY) is a nationwide scheme in India to provide good all-
weather road connectivity to unconnected village settlements with a population of over 500 in the
plains and over 250 hilly areas to all There are plans to connect weather roads. This centrally
sponsored scheme was launched in 2000 by the late Prime Minister of India, Late Shri Atal Bihari
Vajpayee.
Hence, option A is correct.
4. Ans. D.
DDU-GKY was launched by Union Ministers Nitin Gadkari and Venkaiah Naidu on 25 September 2014
on the 98th birth anniversary of Pandit Deendayal Upadhyay. The vision of DDU-GKY is to "transform
rural poor youth into an economically independent and globally relevant workforce". It aims to target
youth aged 15–35 years. DDU-GKY is a part of the National Rural Livelihood Mission (NRLM), which
works with the twin objectives of adding diversity to the income of rural poor families and fulfills the
career aspirations of rural youth.
Hence, option D is correct.
5. Ans. D.
Mission Indradhanush (MI) was one of the world's largest public health programs in 2014 by the
Ministry of Health and Family Welfare, which was strategically and strategically prepared. Since then,
vaccination has taken a percentage as an important pillar of public health. Mission Indradhanush is
targeted to immunize children who are either unvaccinated or partially vaccinated, ie those who
have not been covered under regular immunization due to various reasons.
Hence, option D is correct.
6. Ans. B.
The Pradhan Mantri Sahaj Bijli Har Ghar Yojana (Saubhagya) scheme aims to achieve universal
household electrification in India by providing last-mile connectivity and electricity connections to all
households in rural and urban areas by 31 March 2019. Its goal is 100% electrification nationwide.
The scheme was launched in September 2017.
Hence, option B is correct.

PAGE 154
www.byjusexamprep.com

7. Ans. A.
NATIONAL INITIATIVE FOR DEVELOPING AND HARNESSING INNOVATIONS (NIDHI) - PRomoting and
Accelerating Young and ASpiring innovators & startups (NIDHI-PRAYAS), is an umbrellaprogramme
conceived and developed by the Innovation & Entrepreneurship division, Department of Science &
Technology, Government of India, for nurturing ideas and innovations (knowledge-based and
technology-driven) into successful startups.
Hence option A is correct.
8. Ans. A.
Knowledge Involvement Research Advancement through Nurturing (KIRAN) program by the
Department of Science and Technology to provide various career opportunities to women scientists.
It is primarily aimed at bringing gender equality in the field of science and technology by involving
more women talent in the field of research and development through various programs. It provides
fellowship assistance from 25000 to 55000 rupees to women in the age group of 27 to 57 to continue
higher education in Science and Technology after a break in career.
Hence, option A is correct.
9. Ans. D.
The Amara-nayaka system was a major political innovation of the Vijayanagara Empire. Most
probably many features of this system were derived from the iqta system of Delhi Sultanate. Nayakas
were warriors holding an office bestowed on them by the central government on the condition of
rendering military service.
Hence, option D is correct.
10. Ans. A.
Treaty of Bassein, a pact between Baji Rao II, the Maratha Peshwa of Poona (now Pune) in India, and
the British was signed on Dec. 31, 1802. This was a decisive step in the breakup of the Maratha
confederacy. The treaty led directly to the East India Company’s annexation of the Peshwa's
territories in western India in 1818.
Hence, option A is correct.
11. Ans. B.
ACT No. V. OF 1843 relating to Slavery in India: Passed on the 7th April 1843, An Act for declaring and
amending the Law regarding the condition of Slavery within the Territories of the East India
Company. According to this ACT, No rights arising out of an alleged property in the person and
services of another as a slave shall be enforced by any Civil or Criminal Court or Magistrate within
the territories of the East India Company.
Hence, option B is correct.
12. Ans. D.
Gopal Krishna Gokhale was a member of the Imperial Legislative Council. He was popular in India for
the first time as a great champion of compulsory primary education. He publicly calls for the
introduction of "compulsory primary education" in India.
Hence, option D is correct.

PAGE 155
www.byjusexamprep.com

13. Ans. D.
AmritlalVitthaldas Thakkar, also known as Thakkar Bapa, was an Indian social worker who worked
for the upliftment of tribal people in the state of Gujarat in India. He became a member of the
Servants of India Society founded in 1905 by Gopal Krishna Gokhale. In 1922, he founded the Bhil
Seva Mandal.
Hence, option D is correct.
14. Ans. D.
Gandhi-Irwin Pact: Indian National Congress (INC) agreed to participate in the Second Round Table
Conference. INC promised to stop the civil disobedience movement. The Govt on the other hand
agreed for:
• Withdrawal of all ordinances that curb Congress activities.
• Return of all prosecutions except for violent crimes.
• The release of those arrested for participating in the Civil Disobedience Movement.
• Removal of the salt tax.
Hence, option D is correct.
15. Ans. B.
The nominal interest rate is either of two distinct things:
the rate of interest before adjustment for inflation, OR for interest rates "as stated" without
adjustment for the full effect of compounding (also referred to as the nominal annual rate). An
interest rate is called nominal if the frequency of compounding (e.g. a month) is not identical to the
basic time unit in which the nominal rate is quoted (normally a year).
Hence, option B is correct.
16. Ans. B.
The term collateral refers to an asset that a lender accepts as security for a loan. Depending on the
purpose of the loan, collateral can take the form of real estate or other types of property. Collateral
serves as a security for the lender.
Hence, option B is correct.
17. Ans. A.
The crowding out effect suggests rising public sector spending drives down private sector spending.
There are three main reasons for the crowding out effect to take place: economics, social welfare,
and infrastructure. Crowding in, on the other hand, suggests government borrowing can actually
increase demand and hence private spending.
Hence, option A is correct.
18. Ans. C.
Stagflation is characterized by slow economic growth and relatively high unemployment or economic
stagnation - coupled with rising prices (i.e. inflation) at the same time. The period of inflation can
also be defined alternatively with the fall in the gross domestic product (GDP).
Hence, option C is correct.

PAGE 156
www.byjusexamprep.com

19. Ans. A.
Structural unemployment is a form of unemployment caused by a mismatch between the skills that
workers in the economy can offer, and the skills demanded of workers by employers (also known as
the skills gap), mismatch in the regional or occupational pattern of job vacancies and the pattern of
worker availability. However it requires either migration or re-training, structural unemployment can
be chronic or long-term and slow to fix.
Hence, option A is correct.
20. Ans. C.
Nominal GDP measures a country's gross domestic product using current prices, without adjusting
for inflation. Contrast this with real GDP, which measures a country's economic output adjusted for
the impact of inflation.
Hence, option C is correct.
21. Ans. B.
The 'basic needs' approach introduced by the International Labour Organization's World
Employment Conference in 1976. According to ILO report, it indicates two crucial elements in the
Basic Needs approach: “First, they include certain minimum requirements of a family for private
consumption: adequate food, shelter and clothing, as well as certain household equipment and
furniture. Second, they include essential services provided by and for the community at large, such
as safe drinking water, sanitation, public transport and health, education and cultural facilities.”
Hence, option B is correct.
22. Ans. A.
Shifting Cultivation is known as Milpa in Central America & Mexico, Ray in Vietnam, Taungya In
Myanmar, Tamrai in Thailand, Chena in Sri Lanka, Conuco in Venezuela, Roca in Brazil, Masole in
central Africa and Ladang in Indonesia, Caingin in the Philippines. In India, Shifting Cultivation is
known by various local names like 'Jhum' cultivation.
Hence, option A is correct.
23. Ans. A.
The English philosopher John Locke created the philosophical school empiricism. John Locke wrote,
"Democracy is a government based on law, that is created after long consideration by appropriate
elected representatives of the people and proclaimed in a way, that all men can become acquainted
with them."
Hence, option A is correct.
24. Ans. C.
Legal positivism is a school of thought of analytical jurisprudence developed largely by legal
philosophers during the 18th and 19th centuries, such as Jeremy Bentham and John Austin. While
Bentham and John Austin developed legal positivist theory, empiricism provided the theoretical basis
for such developments to occur.
Hence, option C is correct.

PAGE 157
www.byjusexamprep.com

25. Ans. A.
Article 14: equality before the law, the state shall not deny to any person equality before the law or
the equal protection of the laws within the territory of India. The equality before the law is
guaranteed to all without regard to race, colour, or nationality.
Hence, option A is correct.
26. Ans. B.
A casting vote is a vote that someone may exercise to resolve a deadlock or in the case of equality of
votes. A casting vote is typically by the presiding officer of a council, legislative body, committee,
etc., In some legislatures, a casting vote may be exercised however the presiding officer wishes,
according to her party affiliation or according to her personal beliefs.
Hence, option B is correct.
27. Ans. A.
The Government of India (GOI) set up a High-Powered Committee (HPC) in August 1999 and a
National Committee after the Gujarat earthquake, for making recommendations on the preparation
of Disaster Management plans and suggesting effective mitigation mechanisms. The Tenth Five-Year
Plan document also had, for the first time, a detailed chapter on Disaster Management. The Twelfth
Finance Commission was also mandated to review the financial arrangements for Disaster
Management.
Hence, option A is correct.
28. Ans. B.
The Regulating Act (1773) and William Pitt the Younger India Act (1784) established government
control of political policy through a regulatory board for Parliament. The company then gradually lost
commercial and political control. Its commercial monopoly was broken in 1813. In 1833, the
jealously-protected monopoly of the East India Company was finally abolished and the China trade
was opened to the competition of dozens of British companies, who had been petitioning the
government, and from 1834 it was the only managing agency for the British Government of India.
Hence, option B is correct.
29. Ans. C.
British Indian Medical Service (IMS) organization began with the establishment of the Bengal Medical
Service on 20 October 1763 after that by 1764 in both Madras and Bombay also. In Bengal, increasing
military actions required the separation of Military Surgeons from Civil Surgeons.
Hence, option C is correct.
30. Ans. D.
Three major systems of land revenue collection existed in India. They were – Zamindari, Ryotwari
and Mahalwari. Where Ryotwari System was introduced by Thomas Munro in 1820. Mainly
introduced in Madras, Bombay, parts of Assam and Coorg provinces of British India. according to this,
the ownership rights were handed over to the peasants. British Government collected taxes directly
from the peasants. The revenue rates of Ryotwari System were 50% where the lands were dry and
60% in irrigated land.
Hence, option D is correct.

PAGE 158
www.byjusexamprep.com

31. Ans. B.
A no-confidence motion can be passed against the entire council of ministers only.
As a parliamentary motion, it exhibits to the head of state that the elected parliament no longer has
confidence in (i.e. one or more members of) the appointed government. In some nations, if a no-
confidence motion is proceeded against an individual minister they have to resign along with the
entire council of ministers.
Hence, option B is correct.
32. Ans. A.
The Vice President’s election is slightly different from that of the election of the President as the
members of state legislatures are not involved in the electoral college but the nominated members
of both the houses are part of the electoral college for the vice-presidential election.
• The Vice President holds office for a period of five years.
• He/She may be re-elected any number of times.
Hence, option A is correct.
33. Ans. C.
Fundamental Rights (FRs)are the social and political claims without which, a man cannot give his best
to the society or can not realize his/her best self.
The Fundamental Rights guaranteed by the Articles 14, 20, 21, 21A, 22, 23, 24, 25, 26, 27 and 28 are
available to all persons whether he/she is citizens or foreigners.
The Fundamental Rights guaranteed by Articles 15, 16, 19, 29, and 30 are provided only to citizens
of India.
Hence, option C is correct.
34. Ans. A.
Elections to the Rajya Sabha are indirect; members those represents states are elected by elected
members of legislative assemblies of the States comparatively with the system of proportional
representation by means of the single transferable vote, STV, and those representing Union
Territories are chosen in such manner as Parliament may by law prescribe.
Hence, option A is correct.
35. Ans. D.
A person from outside the Parliament can be appointed a member of the Cabinet.
Hence, option D is correct.
36. Ans. D.
According to Article 243K, elections to the Panchayats, the superintendence, direction & control of
the preparation of electoral rolls for, and the conduct of, whole elections to the Panchayats shall be
vested in a State Election Commission comprising of a State Election Commissioner to be appointed
by the Governor.
Hence, option D is correct.

PAGE 159
www.byjusexamprep.com

37. Ans. B.
• All the Panchayat’s seats shall be filled by persons chosen by a direct election from territorial
constituencies in the Panchayat area. The electorate has been named Grama Sabha comprising
of persons registered in the electoral rolls which relate to a particular village comprised within
the area of a Panchayat.
• The "Gram Sabha" means a body comprising of persons registered in the electoral rolls which
relate to a village comprised within the area of Panchayat at the village level;
Hence, option B is correct.
38. Ans. C.
The Transcription Service had started on 3rd April 1954 and entrusted with the major function of
preparing transcription of speeches of all dignitaries with main focus on the Prime Ministers &
Presidents of the country.
This office has the following functional units –
• Central Archives
• Programme Exchange Unit (PEU)
• Transcription Unit
• Refurbishing Unit
• Digital Sound Archives
• Commercial Release & Marketing
Hence, option C is correct.
39. Ans. C.
* All three, i.e., Atal Pension Yojana, Pradhan Mantri Jeevan Jyoti Bima Yojana and Pradhan Mantri
Suraksha BimaYojana are social security scheme (s).
* Atal Pension Yojana is a pension scheme under the Govt. of India. This scheme replaces the
SwavalambanYojana and was set up to provide old age income security to the workers belonging
to the unorganised sector. The scheme was launched by PM Modi on 9th May 2015 in Kolkata. The
scheme mainly focuses on encouraging workers of unorganised sectors to save for their future.
Hence, option C is correct.
40. Ans. B.
Different names of cyclones in Different countries
• West Indies & the coast of Florida. In these areas, tropical cyclones are known as Hurricanes.
• Philippine Islands, The coast of China & Japan. These are known as Typhoons.
• Bay of Bengal (in India) & the Arabian Sea, where these are known as Cyclones.
• North-east & North-western coast of Australia, these are known as Willy-Willies.
• USA & Mexico, these are known as Tornadoes.

PAGE 160
www.byjusexamprep.com

Hence, option B is correct.


41. Ans. B.
• The standard meridian of India is 82.5 degrees east of the Greenwich Meridian, i.e., 82°30' E. This
meridian passes through Mirzapur in Uttar Pradesh.
• It runs through the centre of the nation. In the year 1905, this longitude was opted as the
meridian to create the Indian Standard Time.
Hence, option B is correct.
42. Ans. B.
• The Ten Degree Channel, a segment of the Parallel corresponding to 10 degree North Latitude,
separates the Andaman &Nicobar Islands in the Bay of Bengal.
• The two groups of islands together form the Indian Union Territory of Andaman & Nicobar
Islands.
• This channel is 150 km wide.
• It is also said that when Western Trade wind was used by Arabians they crossed via this channel
in the fifteenth Century.

Hence, option B is correct.

PAGE 161
www.byjusexamprep.com

43. Ans. C.
The Indo-Aryan is the largest linguistic group of India. The Indo-Aryan is considered as a language
spoken by close to one billion people of India & it is derived from Indo-European language family.
This particular language is a group of 219 different languages spoken in countries such as India,
Pakistan, Bangladesh, Nepal, and Sri Lanka.
Hence, option C is correct.
44. Ans. C.
• Article 3 deals with the formation of new states & alteration of areas, boundaries or names of
existing States: -
• The Parliament may by law form a new State by separation of any territory from any State or by
uniting two or more States or even parts of States or by uniting any territory to a part of any
State;
➢ increase the area of any State;
➢ diminish the area of any State;
➢ alter the boundaries of any State;
➢ alter the name of any State;
Hence, option C is correct.
45. Ans. D.
There are currently 16 Navratna CPSEs like:
• Bharat Electronics Limited (BEL)
• Container Corporation of India Limited
• Engineers India Limited
• Hindustan Aeronautics Limited, etc.
Rest are Maharatnas.
Hence, option D is correct.
46. Ans. A.
Towards his nationalistic aims, Bal GangadharTilak published two newspapers, i.e., 'Mahratta' in
English and 'Kesari' in Marathi. Both these newspapers stressed on making the Indians aware of the
glorious past and also encouraged the masses to be self-reliant.
Hence, option A is correct.
47. Ans. B.
M.M Punchhi Commission on Centre-State Relations had made a recommendation for the disposal
of a bill reserved for the consideration of the Union Executive within six months.
In the month of April 2007, a three-member commission headed by the former chief justice of India
M.M. Punchhi was formed by the UPA Government to take a fresh look at relative roles &
responsibilities of several levels of government and their inter-relations.
Hence, option B is correct.

PAGE 162
www.byjusexamprep.com

48. Ans. A.
The NCRWC (National Commission to review the working of the Constitution) also known as Justice
Manepalli Narayana Rao Venkatachaliah Commission was established by a resolution of the NDA
Government of India headed by Atal Bihari Vajpayee on 22nd February 2000 for suggesting possible
amendments to the Indian Constitution.
Hence, option A is correct.
49. Ans. B.
• It was Faizi, who translated ‘Lilavati’, the celebrated Sanskrit work in Maths by Bhaskaracharya,
into Persian. The celebrated work Akbarnama was accomplished by his brother Abul Fazal.
• Faizi was a Persian poet who joined Akbar’s suite during the seize of Chittor in the year 1568. In
1588 was given the status of Malik-ush-Shu’ara (i.e. Court Poet) of Akbar.
Hence, option B is correct.
50. Ans. B.
The Shiva Sankirttan was written by Rameshwar Bhattacharya, a disciple of Vyasa belonging to Suta
class. The Chandimangal of Kavikankan Mukundaram Chakravarti is the story of the Goddess Chandi’s
continuous battle to establish her cult among humans.
The Chaitanya Charitamrita composed by Krishnadasa Kaviraja is written in Bengali literature. The
Chaitanya Mangala of Brindavan Dasa is a critical hagiographical work on the Supreme Personality of
Godhead, Sri Krishna Chaitanya, i.e., Chaitanya Mahaprabhu in Bengali.
Hence, option B is correct.
51. Ans. B.
About Intercropping: It is a crop cultivation method where two or more different kinds of crops are
cultivated together within the same field but following a specific pattern, such as planting in rows.
Hence, option B is correct.
52. Ans. C.
The KLIP (Kaleshwaram Lift Irrigation Project) is a multi-purpose irrigation project on the river
Godavari in Kaleshwaram, Bhupalpally, Telangana, India.
* Presently the world's largest multi-stage lift irrigation project, it’s farthest upstream influence is at
the convergence of the Pranhita & Godavari rivers.
Hence, option C is correct.
53. Ans. A.
In the year 2020, Panna National Park of Madhya Pradesh was declared a UNESCO Biosphere
Reserve.
About Panna Tiger Reserve: The Panna tiger reserve is located in the Vindhya mountain range in
Madhya Pradesh’s northern part. The Ken river (which is a tributary of the Yamuna River) flows
through the reserve.
Hence, option A is correct.

PAGE 163
www.byjusexamprep.com

54. Ans. D.
• INS Airavat arrived in Port Djibouti on 10thNovember 2020, as part of the ongoing humanitarian
mission 'Sagar-II.'
• It is in line with 'Mission Sagar,' which will take place in May-June 2020.
Hence, option D is correct.
55. Ans. C.
The Peninsula Shield Force is the military arm of the GCC (Gulf Cooperation Council).
• The shield force purpose is to deter and respond to, military aggression against any of the GCC
member countries: Bahrain, Saudi Arabia, Kuwait, Oman, Qatar, and the United Arab Emirates.
Hence, option C is correct.
56. Ans. B.
Bangladesh & United States navies launched the ‘CARAT (Cooperation Afloat Readiness and Training)
Bangladesh 2020’ to expand relationships and to broaden maritime awareness between the two
countries.
Hence, option B is correct.
57. Ans. A.
The CCEA (Cabinet Committee on Economic Affairs) chaired by Prime Minister Modi has approved
the investment of Rs.1810.56 crore for 210 MW Luhri Stage-I Hydro Electric Project situated on river
Satluj which is located in Shimla & Kullu districts of Himachal Pradesh. This project will generate
758.20 million units of electricity yearly.
Hence, option A is correct.
58. Ans. D.
Sir Sean Connery has died at the age of 90. The Scottish actor was best remembered for his portrayal
of James Bond, being the first-ever to bring the role to the big screen & appearing in seven of the spy
thrillers.
Hence, option D is correct.
59. Ans. C.
The 24th edition of the MALABAR naval exercise was scheduled in two phases in November 2020.
The Phase 1 of the Exercise MALABAR 2020 involving participation by IN (Indian Navy), USN (United
States Navy), JMSDF (Japan Maritime Self Defence Force), and RAN (Royal Australian Navy) had
commenced off Visakhapatnam in the Bay of Bengal from 03rd to 06th November 2020..
Hence, option C is correct.
60. Ans. A.
Quad group consisting of Japan, India, Australia and the United States. Quad is not an alliance but a
grouping of nations which are interested in strengthening a rule-based order in the region of Indo-
Pacific.
Hence, option A is correct.

PAGE 164
www.byjusexamprep.com

61. Ans. A.
Pinaka is an indigenous MBRL (Multi-Barrel Rocket Launch) system developed by DRDO for the Indian
Army. It is one of the most reliable and effective weapon systems of the Indian Army.
Whole Pinaka MBRL has six launcher vehicles, each comprising 12 rockets with six loader-
replenishment vehicles & two command post vehicles with DIGICORA MET radar and a fire control
computer.
Hence, option A is correct.
62. Ans. D.
Osmosis can be defined as the spontaneous movement of solvent molecules from a region of high
solvent potential to a region of lower solvent potential through a partially semi-permeable
membrane. This is a process that can be used to purify water.
Hence, option D is correct.
63. Ans. B.
Mitochondria and plastids contain their own DNA and ribosomes because of that they are able to
synthesize some of their own proteins and replicate independently of the nucleus. This genetic
material is also called mt DNA or mitochondrial DNA.
Hence, option B is correct.
64. Ans. C.
Cytokinins are essential to plant hormones that influence growth, and the stimulation of cell division.
Cytokinins are known to regulate axillary bud growth and apical dominance. They help in the
production of new leaves, chloroplasts, and adventitious shoots.
Hence, option C is correct.
65. Ans. B.
In the self-pollination process, the pollen grains transfer from the stigma of the same or genetically
similar flower. A flower is self-pollinated if pollen is transferred to it from any flower of the same
plant. Genes are not diverse in nature, therefore, the purity of the race is maintained. The plants do
not depend on external factors for pollination.
Hence, option B is correct.
66. Ans. D.
Cell wall is made up of cellulose, and they are present in most prokaryotes (except mollicute
bacteria), in fungi, algae, and eukaryotes including plants cell. Cell walls are absent in animals.
Humans have skeleton support and cell walls function is to provide skeletal support.
Hence, option D is correct.
67. Ans. D.
Cardiac muscles (the heart muscle) are branched and cylindrical in shape. They are uninucleated
cells, and they are under involuntary control. The cardiac muscle performs coordinated contractions
which allowed the heart to pump blood through the circulatory system.
Hence, option D is correct.

PAGE 165
www.byjusexamprep.com

68. Ans. D.
Planarians are free-living flatworms, they are not directly dependent on another organism for
survival. They inhabit freshwater, and they are carnivorous (even they don't have teeth) or
scavengers.
Hence, option D is correct.
69. Ans. A.
African trypanosomiasis is also known as sleeping sickness. It is due to microscopic parasites of the
Trypanosoma brucei species, transmitted through the tsetse fly. That is found only in sub-Saharan
Africa.
Hence, option A is correct.
70. Ans. D.
Viruses are different from bacteria, they have a different structure and a different way of surviving.
They also don’t have cell walls that can be attacked by antibiotics, instead, viruses are surrounded
by a protective protein coat. Viruses actually digress into, live in, and make new copies of themselves
within a host cell. They can't reproduce on their own, as bacteria do, Instead, they attach themselves
to healthy cells and reprogram those cells to make new viruses. It is because of these reasons
antibiotics are not effective on viruses.
Hence, option D is correct.
71. Ans. B.
Aluminum is manufactured through two phases- the Bayer process and the Hall-Heroult process.
Aluminum originates from bauxite ore to obtain aluminum oxide. By refining and smelting, the
aluminum oxide then releases pure aluminum. A white powder from which aluminum can be
extracted. The extraction is done by electrolysis. During electrolysis- positively charged aluminum
ions gain electrons and form molten aluminum from the cathode, and oxide ions lose electrons and
form oxygen molecules at the anode.
Hence, option B is correct.
72. Ans. C.
20g / (20g + 180g) * 100 = 10%
Hence, option C is correct.
73. Ans. B.
Sodium is kept immersed in Kerosene oil to protect it and to prevent accidental fire. Sodium is a
highly reactive metal. It vigorously reacts with the oxygen, carbon dioxide, and moisture present in
the air that causes a fire. For preventing this explosive reaction, Sodium is kept immersed in kerosene
because it does not react in kerosene oil. Like sodium, potassium is also a highly reactive metal.
Hence, option B is correct.
74. Ans. A.
The silver article becomes black after reacts with sulfur which is present in the atmosphere. Then
sulfur forms silver sulfide on the surface of the silver article, so they appear as dull or black. This layer

PAGE 166
www.byjusexamprep.com

of silver sulfide can be removed by rubbing the article with toothpaste because toothpaste consists
of several abrasive substances like Aluminum hydroxide that makes silver metal all shiny.
Hence, option A is correct.
75. Ans. D.
When we heat lead nitrate Pb(NO3) in a boiling tube, we observe the emission of brown fumes of. It
is a thermal decomposition reaction. When Pb(NO3) is broken down into lead oxide, nitrogen dioxide,
and oxygen. In this decomposition reaction, Lead Nitrate gives a yellow-colored Lead oxide, and
Nitrogen dioxide is released as brown fumes.
Hence, option D is correct.
76. Ans. D.
This slaked lime reacts with CO2 in the air to form a thin layer of CaCO3 to give a shiny finish. The
calcium oxide (CaO) is also called quicklime. It is used for whitewashing by producing calcium
hydroxide (CaOH). Calcium hydroxide reacts with water and absorbs carbon dioxide (CO2) from the
environment. Then it produces calcium carbonate (CaCO3) which creates a shiny finish on the walls.
Hence, option D is correct.
77. Ans. A.
Hard water can be softened by washing soda or sodium carbonate Na2CO3. It removes the calcium
ions in a precipitation reaction from the hard water. And then the hard water passed through an ion-
exchange resin in a column. In the water, sodium ions replace the calcium ions when it passes
through the column.
Hence, option A is correct.
78. Ans. B.
The formula of soda acid is NaHCO3. The soda acid fire extinguisher contains sodium bicarbonate and
sulfuric acid. It is the most effective household fire extinguisher. Sulfuric acid is kept inside the iron
vessel. Any fire extinguisher is a cooling of the combustible material, cut off the air supply. Soda acid
fire extinguisher acts on all fire except due to electrical and inflammable liquid.
Hence, option B is correct.
79. Ans. C.
Oxalic acid naturally occurring acids is found in abundance in tomato.
• The main acid found in tomatoes is oxalic acid.
• Tomatoes contain over ten different types of acids, including citric acid, malic acid, ascorbic
acid, and oxalic acid.
• Tomato contains a variety of acids, but two in particular account for a significant portion of the
total: citric acid and malic acid.
Hence, option C is correct.
80. Ans. C.
Fission is related to the creation of new individuals through cell division in the unicellular organism
of the living cell. In nuclear fission, it is related to the transformation of heavier nuclei into smaller

PAGE 167
www.byjusexamprep.com

nuclei. Nuclear fission in an atom splits into two or smaller atoms, as a result of neutron
bombardment. It is an exothermic reaction. Most fission are binary fission. And they produce two
charged fragments.
Hence, option C is correct.
81. Ans. B.
Gold (Au) has both malleability and ductility properties. The elements which tend to form positive
ions are called metals. They are luster, ductile, malleable, a good conductor of heat and electricity,
and high melting and boiling points. Metals are also solid at room temperature except for mercury.
They have high tensile strength. They are usually hard, except sodium, potassium. They act as
reducing agents. They displace hydrogen from acids. They react with water to evolve hydrogen gas.
Hence, option B is correct.
82. Ans. D.
Iodine is a non-metal, but it is lustrous. It's because iodine going down the size of the atom is increase
and shielding effect of the atom and outermost electron decrease. So that iodine becomes excited
by absorbing heat energy, and they emit light. So iodine appears to have a lustrous surface but this
element is not a metal.
Note-Carbon is also showing lustrous but only in certain forms of diamond, and graphite.
Hence, option D is correct.
83. Ans. B.
Nanotechnology is the field of research and study of very minute things which can be applied across
all the science fields, such as chemistry, biology, physics, materials science, etc. Nanotechnology can
be defined as the study and use of those structures whose size is in between 1 nanometre and 100
nanometres.
Hence, option B is correct.
84. Ans. C.
Weight is a synonym of force that acts at all the times on all objects on the ground. It is the universal
force of attraction. The Earth pulls on all objects with a force of gravity downward towards the centre
of the Earth. This shows that the weight of an object is due to the force that it exerts on the ground.
Hence, option C is correct.
85. Ans. D.
Objects in uniform circular motion move along with a circular pathway at a constant speed, so the
acceleration can be point perpendicular to the velocity for a change in the direction only. The
acceleration vector must point inward toward the centre to turn the object back onto the circular
path.
Hence, option D is correct.
86. Ans. B.
A Primary rainbow is a coloured bow or arc that appears on a “screen” of water drops when light
from a bright light source fall upon them. It happens when a sunbeam is being refracted twice and

PAGE 168
www.byjusexamprep.com

reflected once by the droplet, a primary rainbow will form. So, there is only one total internal
reflection of light in water droplets that takes place in the formation of the primary rainbow.
Hence, option B is correct.
87. Ans. D.
Faraday’s law is the basic law of electromagnetism which explains that a magnetic field interacts with
an electric circuit to produce an electromotive force (EMF) and this phenomenon is known as
electromagnetic induction. It doesn't tell about the direction.
Hence, option D is correct.
88. Ans. C.
The heating effect of electric current was discovered by James Prescott Joules. When an electric
current pass through a conductor, the conductor becomes hot and gives rise to heat after some time.
It is due to the conversion of some electric energy while passing through the conductor into heat
energy. This effect of electric current is known as the heating effect of current.
Hence, option C is correct.
89. Ans. B.
A parabolic mirror is a specially designed mirror that is used to capture and focus energy onto a single
point. These are also used for distributing the energy from one point to multiple points. Parabolic
mirrors can be defined as parabolic reflectors or as parabolic dishes. A parabolic mirror is the most
common non-spherical mirror. That’s why a non-spherical shining spoon can generally be considered
as a parabolic mirror.
Hence, option B is correct.
90. Ans. C.
Electric charges can flow easily through conductors. The human body is an electric conductor as it
contains various salts dissolved in their blood. The cells of a human body contain various ions such
as sodium ion, potassium ion, chloride ion, etc. which helps to conduct electricity. A plastic sheet,
distilled water and wood are good electrical insulator but not conductors.
Hence, option C is correct.
91. Ans. D.
Resistivity is defined as an electric resistance of a conductor of the unit cross- sectional area and unit
length. Its unit is ohm meter. A characteristic property of each material, resistivity is used to
differentiate various materials based on their ability to conduct electric currents. Thus, all three
copper wires of different lengths and different areas of the cross-section will have the same
resistivity.
Hence, option D is correct.
92. Ans. A.
McLeod British firms were taken over by Soorajmull-Nagarmull group.
McLeod is an Indian based tea company. It is currently the largest tea-growing company in the world.

PAGE 169
www.byjusexamprep.com

It is part of the Williamson Magor Group. It has 48 tea estates in the Brahmaputra Valley of Assam &
five in the Dooars area of West Bengal, 3 factories in Vietnam, six estates in Uganda and the
management control of the world Gisovu estate in Rwanda.
Hence, option A is correct.
93. Ans. D.
François Bernier was a French physician and traveller. He was born in Joue-Etiau in Anjou. He was
briefly personal physician to Mughal prince Dara Shikoh, the eldest son of the Mughal emperor Shah
Jahan, and after the execution of Dara Shikoh, was attached to the court of the Mughal emperor
Aurangzeb, for around twelve years during his stay in India.
Hence, option D is correct.
94. Ans. B.
Typically, a govt. maintains a fixed exchange rate by either buying/selling its currency on the open
market. This is one of the reasons why governments maintain reserves of foreign currencies. If the
rate of exchange drifts too far below the desired rate, the govt. buys its currency in the market using
its reserves. This results in greater demand on the market and pushes up the price of the currency.
If the rate of exchange drifts too far above the desired rate, the govt. sells its currency, thus
increasing its foreign reserves.
Hence, option B is correct.
95. Ans. B.
Fiscal Deficit is the excess of total expenditure over total receipts excluding borrowings. In other
words, it gives the amount needed by the government to meet its expenses. Therefore, a large Fiscal
Deficit signifies a large borrowings.
Primary Deficit is Fiscal Deficit of the present year minus interest payments on previous borrowings.
Hence, option B is correct.
96. Ans. A.
A recessionary gap occurs when real GDP is less than potential GDP and which brings a falling price
level. A recessionary gap occurs when the SRAS (Short Run Aggregated Supply) curve and the AD
(Aggregate Demand) curve intersect to the left of the potential GDP line.
An inflationary gap is a macroeconomic phenomenon that measures the difference between the
current level of real GDP and the GDP that would exist if an economy was operating at full
employment.
Hence, option A is correct.
97. Ans. D.
Money has taken various forms through the ages, but money consistently has three functions, they
are:
• Store of value
• Unit of account
• Medium of exchange

PAGE 170
www.byjusexamprep.com

The present-day economies use fiat money-money that is neither a commodity nor represented or
‘backed’ by a particular commodity. Even forms of money which share these function may be more
or less useful based on the characteristics of money.
Hence, option D is correct.
98. Ans. D.
Marmagao Port, located at the entrance of the Zuari estuary, is a natural harbour in Goa. The port
gained significance after its remodelling in the year 1961 to handle iron-ore exports to Japan. The
construction of Konkan railway has considerably extended the hinterland of this particular port. Goa,
Karnataka, Southern Maharashtra constitute its hinterland.
Hence, option D is correct.
99. Ans. A.
There are more than 100 coffee species, but the main commercial varieties are Robusta, Arabica,
and – to a lesser extent – Liberica. Robusta, Arabica, and Liberica are three common coffee beans
used all over the world.
Hence, option A is correct.
100. Ans. D.
The Black soil stretch over the parts of Maharashtra, Gujarat, Western parts of Madhya Pradesh,
North- Western Andhra Pradesh, Karnataka, Rajasthan, Tamil Nadu, Chhattisgarh, Jharkhand up to
Raj Mahal hills. The Black soil has clayey texture and is highly fertile. It is rich in calcium carbonate,
magnesium, potash, and lime but poor in nitrogen &phosphorous.
Hence, option D is correct.
101. Ans. D.
Bihzad was not associated with Humayun. Bihzadwas a Persian painter and head of the royal ateliers
in Tabriz and Herat during the late Timurid and in early Safavid Persian periods.
Hence, option D is correct.
102. Ans. A.
The socialist activist KamaladeviChattopadhyay had persuaded Mahatma Gandhi not to restrict the
protests to men alone. She was herself one of the numerous women who courted arrest by breaking
the salt/liquor laws.
KamaladeviChattopadhyay was a key figure in the international socialist feminist movement. From
the late part of the 1920s to the 1940s and beyond, she became an emissary for Indian women and
political independence.
Hence, option A is correct.
103. Ans. A.
Gopal Hari Deshmukh, popularly known as Lokhitwadi. He was the pioneer of all-sided reformism in
the present state of Maharashtra. Deshmukh was a fine product of the new learning of the West.
Deshmukh laid the foundation of a wide-based intellectual renaissance in western India.
Hence, option A is correct.

PAGE 171
www.byjusexamprep.com

104. Ans. D.
The alluvial soils vary in nature from that of sandy loam to clay. They are usually rich in potash but
poor in phosphorous.
Even otherwise phosphates are available in water or soil in a very small fraction. The reasons are
phosphates take the longest time among all other minerals for its complete recycling. Even during
the recycling process if not immediately used, a portion of available phosphates gets converted into
other compounds which again takes much time for it to come into the cycle.
Hence, option D is correct.
105. Ans. B.
S waves or secondary waves can travel only through solid materials, not P waves or primary waves.
P waves (or Primary waves) are the first waves to arrive at a seismograph. The P waves are the fastest
seismic waves and can move via solid, liquid, or gas. They are similar to sound waves. They leave
behind a trail of rarefactions and compressions on the medium they move through. The P waves are
also known as pressure waves for this reason.
Hence, option B is correct.
106. Ans. D.
The ozone layer is a part of high ozone concentration in the stratosphere, 15-35 kilometres above
Earth's surface. The ozone layer performs as an invisible shield & protects us from harmful UV
radiation from the sun. In particular, the ozone layer protects us from the ultraviolet radiation,
known as UV-B, which causes sunburn. The long-term exposure to high levels of UV-B threatens
human health and it damages most animals, plants & microbes, so the ozone layer protects all life
on Earth.
Hence, option D is correct.
107. Ans. C.
Igneous rocks are defined as the kind of rocks that are formed when molten rock (i.e. rock liquefied
by intense heat & pressure) cools to a solid-state.
Some examples of Igneous rocks are:
• Granite
• Basalt
Hence, option C is correct.
108. Ans. B.
Characteristics of rice crop:
• Rice is a Kharif crop.
• Rice requires the temperature, above 25 Degree Celcius and annual rainfall above 100 cm.
• The major rice-producing states are Punjab, Haryana, Bihar, Uttar Pradesh, Rajasthan and
Madhya Pradesh.
• Rice is the staple food crop.
Hence, option B is correct.

PAGE 172
www.byjusexamprep.com

109. Ans. C.
The 44th amendment act was passed in the year 1978 by the Morarji Desai government. It eliminated
the right to property, which was a fundamental right according to Article 19. This particular
amendment made right to property only a legal right. It is to be considered that the 25th amendment,
1971 curtailed the fundamental right to property that allowed the government to obtain private
property for public use after paying compensation to the owner.
Hence, option C is correct.
110. Ans. D.
The motion can be made while introducing an ordinary Bill in the Parliament except in statement D.
The member-in-charge of the ordinary Bill moves a motion for consideration of the Bill, it is
permissible for any other member of parliament to move, as an amendment to the motion for
consideration, that the particular Bill be referred to a Select or Joint Committee or be circulated for
eliciting public opinion thereon. If the motion is considered for reference of the Bill to a Joint Commit-
tee comprising of members of both the Houses, the concurrence of the other House is compulsory.
Hence, option D is correct.
111. Ans. A.
Clause (1) of the Article 231 empowers the Parliament of India to set up a common High Court for
two or more States or for two or more states & a Union territory. Currently, there are 25 High Courts
in India having jurisdiction over 28 states & 8 Union territories. Whereas most of the states have a
High Court of their own, few states share a common High Court within the purview of Article 231,
such High Courts having jurisdiction over two or more states or two or more states & a Union
territory.
Hence, option A is correct.
112. Ans. D.
The major objectives of NITI Aayog are mentioned below:
• To develop a shared vision of national development sectors, priorities, and strategies with the
active involvement of States in the light of national goals.
• To foster cooperative federalism via structured support initiatives & mechanisms with the
States on a uniform basis, recognizing that strong States make a strong nation.
• To evolve mechanisms to formulate credible plans at the village level &aggregate these
progressively at higher levels of govt.
Hence, option D is correct.
113. Ans. D.
Centralization refers to the phenomenon in which activities involving planning & decision-making
within an organization are concentrated to a particular leader or location.
Some features of Centralization:
• Uniformity in process
• Better quality of work & high productivity
• Experienced people to take important decisions and to guide

PAGE 173
www.byjusexamprep.com

• Consistent focus on the vision of the institution


• Quick decision making & implementation
• Less costly
• National unity
• Prosperity
Hence, option D is correct.
114. Ans. B.
Madan Mohan Malviya had set up BHU in the year 1915 with the support of few leaders, mainly
Annie Besant. Annie Besant had founded the Central Hindu College in the year 1898 in Varanasi and
this college was then later gifted to Pandit Madan Mohan Malviya; it transferred into Banaras Hindu
University. Also, a notable point is that foundation for the BHU’s main campus was laid by Lord
Hardinge on 4th February. 1916, on the occasion of Vasant Panchami.
Hence, option B is correct.
115. Ans. D.
The leader of the Satara Parallel govt. was "Kratisimha" Nana Patil, who later joined the Communist
party & was elected to the Lok Sabha on the CPI ticket from Satara in the year 1957 and again from
Beed in the year 1967. The British rule was effectively overthrown in large parts of Satara district of
the western side of Maharashtra during those three years.
The Parallel govt. the movement was a guerrilla kind of struggle, and it operated in over 150 villages
with solid peasant support.
Hence, option D is correct.
116. Ans. B.
The Sannyasi& Fakir Rebellion took place in the early colonial rule in Bengal during the rule of Warren
Hasting. The rebellion started in the year 1750 onwards but took a violent turn since 1773 when
Warren Hastings attained the Governor-Generalship of Bengal. The movement covered an ample
range of Bengal and Bihar and continued for a long time.
Hence, option B is correct.
117. Ans. A.
On 6th March 1775, the Treaty of Surat was signed between Raghunathrao who was a claimant to
the throne of the Peshwa& the British East India Company.
On 1st March 1776, the Treaty of Purandar was signed between Nana Phadnavis who was a Maratha
minister and the Calcutta Council of the East India Company.
Convention of Wadgaon, (on 13th January 1779), compact concluded after the 1st Maratha War in
India (i.e. 1775–82)..
The Treaty of Salbai by which First Anglo-Maratha War was ended, was signed in 17th May 1782
between the Marathas and the British East India Company.
Hence, option A is correct.

PAGE 174
www.byjusexamprep.com

118. Ans. C.
• The earth’s rotation about its axis affects the direction of the wind and this force is known as
the Coriolis force.
• Coriolis force is directly proportional to the angle of latitude.
• It deflects the wind to the right direction in the northern hemisphere and the left direction in
the southern hemisphere.
Hence, option C is correct.
119. Ans. D.
When rivers discharge their waters from all directions in a lake/depression, the pattern is known as
‘centripetal’. This drainage pattern is even called endorheic drainage. For instance, Loktak lake in
Manipur.
When the primary tributaries of rivers travel parallel to each other and secondary tributaries join
them at 90 Degree, the pattern is known as ‘trellis’. For instance, rivers in the upper part of the
Himalayan part and the old folded mountains of the Singhbhum (i.e. Chotanagpur Plateau) have
drainage of trellis pattern.
Hence, option D is correct.
120. Ans. B.
Bhakra Nangal Dam is a concrete gravity dam on the Sutlej River in Bilaspur, Himachal Pradesh in
northern India. This dam forms the Gobind Sagar reservoir. The dam is situated at a gorge near the
(now submerged) upstream Bhakra village in Himachal Pradesh’s Bilaspur district at a height of 226
m.
Hence, option B is correct.

PAGE 175
www.byjusexamprep.com

Elementary Mathematics
1. Ans. C.

Only A, C, D and E
2. Ans. B.
total number of students are 90.
Number of students who scored less than or equals to 50% marks (40 mark) = 5+8+10+13 = 36
% age of such students = 36/90 * 100%= 40%
3. Ans. B.
arranging the number in the ascending order
-1, 2,2,3,6, 8, 9

There are 7 terms, so the median

4. Ans. D.
first ten composite numbers are

Mean

5. Ans. B.

average marks of the students

After adding 5 grace marks to each student.


The revised average marks = 25 +5 = 30
6. Ans. C.

given that the average , , where and

Sum of m observation and sum of the n observation and

So, sum of observation

PAGE 176
www.byjusexamprep.com

Where
7. Ans. B.
total production of each tablet over years 2000-2005.
Type I: 160 + 200 + 135+ 240 + 180 + 210 = 1125
Type II: 80+150+35+95+110+150 = 620
Type III: 70+85+44+120+85+100 = 504
Type IV: 90+160+95+80+95+92 = 612
Type V: 75+100+85+120+115+110 = 605
So the least is type III
8. Ans. A.
total number of tablets produced each year
2000: 160+80+70+90+75 = 475
2001: 200+150+85+160+100 = 695
2002: 135+35+44+95+85 = 394
2003: 240+95+120+80+120 = 655
2004: 180+110+85+95+115 = 585
2005: 210+150+100+92+110 = 662
(2003, 2005): |655 – 662| = 7
(2001, 2005): |695 – 662| = 33
(2003, 2004): |655 – 585| = 70
(2000, 2002): |475 – 394| = 81
(2003, 2005) is minimum
9. Ans. C.

percentage drop in total production in 2004 compared to 2001

percentage drop in total production in 2000 compared to 2001

PAGE 177
www.byjusexamprep.com

required ratio

10. Ans. C.

In year 2003 and 2005.


11. Ans. C.

The sides of a right-angled triangle are in the ratio

So, the ratio of sides


So, perimeter units.
12. Ans. C.
given that and

PAGE 178
www.byjusexamprep.com

13. Ans. D.

we know that the intersecting points of diameter of circle intersects at center of circle.
We know that angle made by a chord on centre is two times to the angle made on it circumference.

So,
14. Ans. C.

ABC is a equilateral triangle then each angle will be . We know that angle made by same chord
on the circumference of circle is equal. So = .
15. Ans. C.

let the sides AB, AC and BC of a triangle ABC are 4 cm, 6 cm and 8 cm. let the touching pointing of
the circles are L, M and N such that AN = AL, BL = BM and CN = CM.
Let AN = a cm then CN = CM = 6 – a cm
Then BM = BL = 8 – (6 – a) = 2 + a
And AL = AN = 4 – ( 2 + a) = 2 - a

PAGE 179
www.byjusexamprep.com

But AN = a then
a=2–a
a = 1 cm
sum of the radii of the circle = AN + BM + CL = 1 + 5 + 3 = 9 cm
16. Ans. C.

we know that

17. Ans. C.
let the base and height of the right-angled triangle is b and h.

…(i)

Perimeter …(ii)

Or

Now, the area of the triangle

18. Ans. C.

If BC = 6 cm and CA = 8 cm, then

Area of the triangle ABC

PAGE 180
www.byjusexamprep.com

19. Ans. D.

given that ABCD is a trapezium in which AB is parallel to DC.

(vert. opposite angles)

(Alternate angles)

So, (AAA)

Now,

20. Ans. B.
let the circle touches the side of quadrilateral AB, BC, CD and DA at P, Q, R and S.

we know that the tangents from a point to the circle are equal. So,

Now,

PAGE 181
www.byjusexamprep.com

21. Ans. B.
let the height and radius of cone are h and r. Since bases of cone and hemisphere are equal then
radius of hemisphere is r.
And given that the volumes are equal, then

22. Ans. B.
let the radius of the wire is r. since volume remains equal.
sphere of radius = 30 mm = 3 cm

23. Ans. C.

let the radius of base and height of the cylinder is and respectively.
And the volume of the cylinder

Curved surface area of the con

PAGE 182
www.byjusexamprep.com

24. Ans. D.
If R is the outer radius and r is the inner radius of the pipe and length is 14 cm. then

Difference between outside and inside surface area =

Volume of the pipe

25. Ans. C.
let the edge of third cube is h, and the volume of the cube remains constant

Now, the surface area of the third cube

26. Ans. B.
volume of both the vessel should be equal

27. Ans. C.
let the radius of the cone is r. since the volume remains equal.

PAGE 183
www.byjusexamprep.com

28. Ans. C.
curved surface area = 550 cm2

height of cone = 24 cm

Divide equation (ii) by equation (i)

29. Ans. B.
the volume of the largest cylinder when it is rolled along its width, so

And the volume of the largest cylinder when it is rolled along its length, so

Required ratio,

PAGE 184
www.byjusexamprep.com

30. Ans. A.
If the internal and external radii are 5 cm and 6 cm respectively, then volume of the sphere

Mass of the sphere,

31. Ans. A.

So, statement I is true.

So, statement is not correct.


32. Ans. A.

and

33. Ans. D.

given that, …(i)

From equation (i) and (ii)

PAGE 185
www.byjusexamprep.com

34. Ans. D.

given ABC be a triangle right angled at C, then

35. Ans. A.

let that satisfy both the equation and .

Then

36. Ans. C.

given , where

37. Ans. C.

PAGE 186
www.byjusexamprep.com

38. Ans. D.
given that 6 + 8 tanθ = secθ and 8 – 6 tanθ = k secθ
Add both after squaring,

39. Ans. C.
let the angle B and angle D are 60° and 45° as shown in the figure. Then,

And

Now,

40. Ans. B.
let the pole is BC which makes an angle of 60o with the vertical, then
And CD = BD

AB = x meter and AD is common in both triangles

PAGE 187
www.byjusexamprep.com

Then both the triangles are congruence then


It means,

Now as we now AD cut the CB then

Now Triangle ADB is right angle triangle


Then

Now length of pole =

41. Ans. C.
surface area of the cloth and the tent remains same/

42. Ans. B.
the height of the water in the vessel raised due the volume of the sphere.

43. Ans. A.
angle obtained by sector on the centre of the circle

Area of the sector,

44. Ans. D.
perimeter of the rhombus = circumference of circle

[ = side of rhombus]

PAGE 188
www.byjusexamprep.com

45. Ans. B.
perimeter of a semi-circular park with radius r,

Area of semicircle

46. Ans. B.
If The diagonal AC and BD intersect at P. then

47. Ans. B.
ΔABC is similar to ΔDEF. Then

then

48. Ans. C.
Two isosceles triangles have equal vertical angles and let their heights and bases are h1, h2 and

b1, b2, then . Now,

49. Ans. D.

ABC is a triangle right angled at A, then

PAGE 189
www.byjusexamprep.com

area of the triangle ABC,

50. Ans. A.

we can write

51. Ans. A.

Or let is the square root of

By comparing we get

By observation we get or

So, square root of is or .

PAGE 190
www.byjusexamprep.com

52. Ans. C.
let the let two alternate natural numbers are a and a+2, then according to the question,

So, the numbers are 6 and 8. And their sum is 14.


53. Ans. D.
let n = 1 then

n = 2 then

n = 3 then

we can observe that we get always factor of 24 and factors of 24 are 1,2,3,4,6,8,12,24.
So, number of natural numbers that divides are 8.
54. Ans. A.
we have
Taking log both side at base 10

55. Ans. C.

we have

PAGE 191
www.byjusexamprep.com

56. Ans. B.

given that

We can write it as

Taking sine

57. Ans. D.
Given, p = sin2x + cos4x
= sin2x + (1 – sin2x)2
= sin4x + sin2x – 2sin2x + 1
= sin4x – sin2x + 1
= (sin2x – 1/2)2 + 3/4
p ≥ 3/4
= sin2x + cos4x
= sin2x + cos2x(1 – sin2x)
= (sin2x + cos2x) – sin2x cos2x
= 1 – sin2x cos2x
p≤1
Therefore, 3/4 ≤ p ≤ 1
So both statements are incorrect.
58. Ans. B.

let ,

So, the required ratio will be -2.

PAGE 192
www.byjusexamprep.com

59. Ans. A.

we know that for real solution

But,

So,

60. Ans. B.
if successive discounts are 20%, 10% and 5% then,

overall discounts

61. Ans. A.
let the sum of amount is Rs. P and rate of simple interest is r% and invested it for 5 years, then

According to the question if the rate of interest was 5% more then, new S.I.

62. Ans. D.
let the principal amount is Rs. P and the rate of interest is 12% for 2 years. Then,

PAGE 193
www.byjusexamprep.com

63. Ans. A.
let the speed of the man is v and time take by him is t.
Then, distance

64. Ans. D.
let the cost price is CP and the selling price is SP.
According to question
100CP = 80SP
SP/CP=100/80=5/4
SP/CP - 1 = 5/4 - 1
(SP-CP)/CP = (5-4)/4 = 1/4
(SP-CP)/CP * 100 = 25
percentage profit = 25%
65. Ans. B.
liquid in 80 litres of mixture =

And water in the mixture = 80 – 20 = 60


Let litres of water are added in the mixture, so

66. Ans. C.
let a person can clean 1 unit of floor in one day. then,
20 persons clean 20 floors 20 days
20 persons clean 1 floor 1 day
1 person cleans 1 floor 20 days
1 person cleans 16 floors days = 320 days

PAGE 194
www.byjusexamprep.com

16 persons clean 16 floors days = 20 days

Or

67. Ans. A.
we should add

68. Ans. C.

let then it satisfies both the equations, now

Or
You can also put .
69. Ans. B.
If (x – k) is the HCF of and , then will satisfy both and
equations.

Then, …(i)

And …(ii)
By equation (i) and (ii)

PAGE 195
www.byjusexamprep.com

70. Ans. C.

Statement-I: and

Statement-I: and

71. Ans. A.

72. Ans. C.

Integer value of x = 2.

PAGE 196
www.byjusexamprep.com

73. Ans. D.
factorizing both polynomials

HCF
74. Ans. A.
given that

75. Ans. D.

76. Ans. D.
we know that the rule of divisibility by 13 is “if is a number N, then if the number formed
by the alternative sum and difference of blocks of 3-3 digits from right to left is divisible by 13 then
N is divisible by 13”.
So, for is divisible by 13
must be divisible by 13.
must be divisible by 13. So, P must be 8.

77. Ans. B.

we have to find remainder of

We know

PAGE 197
www.byjusexamprep.com

78. Ans. A.
here

Since then, there are no such pairs.

79. Ans. D.
prime numbers between 50 to 100 are 53, 59, 61, 67, 71, 73, 79, 83, 89 and 97.
89-83=79-73=73-67 = 67-61=59-53 = 6
Such number of pairs are 5.
80. Ans. D.
since all the numbers are different then total number of terms
81. Ans. A.

Here, we can see that it is always divisible by 6.


82. Ans. D.
we have

Squaring both sides

Cubing both sides

PAGE 198
www.byjusexamprep.com

83. Ans. B.

[by componendo dividendo]

[by squaring both sides]

[by componendo-dividendo]

84. Ans. A.

we know that the unit digits of

85. Ans. C.

86. Ans. C.
for a quadratic equation to be perfect square,

the discriminant,

PAGE 199
www.byjusexamprep.com

87. Ans. B.

given that

88. Ans. C.

if a quadratic equation has equal roots,

the discriminant, D = 0 ⇒ b2 = 4ac

89. Ans. B.
given that the sum of the roots and the product of roots are 6.

So,

90. Ans. C.

let

Now

PAGE 200
www.byjusexamprep.com

Hence is a factor of f(x)

And

Hence is a factor of f(x)

91. Ans. D.
change after adding 3 to both the numerator and denominator of all the fractions

We can see the minimum change is with fraction .

92. Ans. C.
the ratio of boys and girls in the class is 3:1, let the number of boys and girls in the class are 3k and
k. Given that the average score of the class is p and average score of the boys is (p + 1), let the average
score of the girls is , then total score of the class

93. Ans. A.
let their incomes are 7m, 9m and 12m and their expenditures are 8n, 9n and 15n. then their savings
are and (12m - 15n). Now, given that

…(i)

Now ratio their savings are (7m - 8n) : (9m - 9n) : (12m - 15n)

PAGE 201
www.byjusexamprep.com

94. Ans. B.

95. Ans. B.
let the price of X’s goods is then,

Y’s goods price

Z’s goods price

Percentage cheaper of Z’s goods than Y’s good

96. Ans. B.

97. Ans. B.

speed of the train

98. Ans. C.

Average speed,

99. Ans. A.
let a man works a unit of work in days then,

and

And

PAGE 202
www.byjusexamprep.com

100. Ans. A.
let the side of the cube is s and radius of the cube is r. Then

If is the volume of the cube and v is the volume of the sphere.

PAGE 203
www.byjusexamprep.com

CDS II 2020
ENGLISH
Direction: In this section a word is spelt in four 6. Which one of the following alternatives has
different ways. Identify the one which is correct. the correct spelling?
Choose the correct response (a), (b), (c) or (d)
A. Twelfth B. Twelfth
and indicate on the Answer Sheet accordingly.
C. Tweluth D. Twelthe
1. Which one of the following alternatives has
the correct spelling? 7. Which one of the following alternatives has
the correct spelling?
A. Mountaineous
A. Snobbery B. Snoberry
B. Mountenous
C. Snabbery D. Snobbory
C. Mountaineus
8. Which one of the following alternatives has
D. Mountainous
the correct spelling?
2. Which one of the following alternatives has
A. Neurasis B. Nuroesis
the correct spelling?
C. Neurosis D. Neuresis
A. Etiquette B. Etiquete
9. Which one of the following alternatives has
C. Etiequtte D. Etequtte
the correct spelling?
3. Which one of the following alternatives has
A. Dipthteria B. Diptheria
the correct spelling?
C. Diphtheria D. Diphthria
A. Curriculum B. Curiculum
10. Which one of the following alternatives has
C. Curiculeum D. Curriculum
the correct spelling?
4. Which one of the following alternatives has
A. Meagre B. Megare
the correct spelling?
C. Meagr D. Megear
A. Magnificent B. Magnificant
Direction: Each of the following sentences in this
C. Magneficent D. Magenficient
section has a blank space with four words or
5. Which one of the following alternatives has group of words given. Select whichever word or
the correct spelling? group of words you consider the most
appropriate for the blank space and indicate
A. Felecitation
your response on the Answer Sheet accordingly.
B. Felicitation
11. The difficult thing about ________ the
C. Falicitation science of habits is that
D. Felicitasion A. studying B. study
C. studies D. are studying

PAGE 204
www.byjusexamprep.com

12. most people, when they hear about this 19. different, and so the specifics of from
field of research, ___________12. diagnosing and changing the patterns in our
lives differ person to ____________19. and
A. wanting B. wanted
behaviour to behaviour. Giving up
C. wants D. want
A. people B. persons
13. to know the secret formula for quickly
C. personnel D. person
changing any habit. If scientists have
discovered how 13._________ 20. cigarettes is different _________20.
curbing overeating, which is different
A. those patterns work,
A. from B. since
B. this
C. to D. into
C. these
21. From changing how you communicate with
D. that
your spouse, 21. _______
14. then it stands to reason that they
A. it B. this
____________14. have also found a recipe
for rapid change, right? C. what D. which
A. must B. will 22. is different from how you prioritize tasks at
work. What’s more, each person’s habits
C. could D. might
are ____________ 22. by different cravings.
15. If only it ____________15.
A. broken B. given
A. are B. were
C. driven D. prescribed
C. was D. will be
23. As a result, this book does not ________23.
16. that easy. It's not __________16. one prescription. Rather, I hoped to deliver
something
A. these formulas don't
A. contain B. contains
B. this
C. contained D. containing
C. that
24. else: a framework for understanding
D. which
_________24. habits work and a
17. exist. The problem is that there isn't one
A. how B. what
formula for 17. ________
C. where D. whose
A. changing
25. guide to experimenting with how they
B. changed
_________25. change. Some
C. having changed
A. might B. would
D. changes for
C. will D. must
18. habits. There are thousands. Individuals
26. habits yield easily to analysis and influence.
and habits are 18. ________
Others are _________ 26.
A. full B. all
A. quiet B. most
C. complete D. most
C. better D. more

PAGE 205
www.byjusexamprep.com

27. Complex and obstinate, and require eye. For centuries it was assumed that women
prolonged study. And for others, change is were less intelligent than men and that they did
a __________27. that never fully not merit the same degree of education. They
concludes. were not allowed a vote in the political system.
By the same token, any kind of knowledge
A. process B. processing
developed by women was regarded as non-
C. processed D. processes serious, trivial, gossip or alternatively as
knowledge that had been discredited by science,
28. But that does not __________28. it can't
such as superstition or traditional practices of
occur. Each chapter in this book explains
childbirth or healing. All these attitudes were
A. means B. meant part of a larger system in which women were
dominated, exploited, and physically abused by
C. meaning D. mean
men. Slowly, but increasingly, from the end of
29. a different aspect of why habits exist and 18th century, feminists began to contest this
how they function. The framework situation. The more they contested it, the more
_________29. in this section is an attempt it became increasingly obvious that these
to distil, attitudes extended into the whole of the culture;
A. describing B. described social relations, politics, law, medicine, the arts,
popular and academic knowledge.
C. will describe D. description
31. Post colonialism is
30. in _______30. very basic way, the tactics
that researchers have found A. a contestation of the then existing
dominant western practices
A. a B. any
B. a contestation of western practices in
C. the D. rather colonial states
Direction: In this section you have two short
C. a contestation of the superstitious
passage. After each passage, you will find some
practices
items based on the passage. Read the passage
and answer the items based on them. You are D. an approval of indigenous practices
required to select your answer based on the
content of the passage and opinion of the author 32. What does ……. She was always an object,
only. never a subject; mean?

Passage -I A. Women were given respect and


worshipped
Post-colonial cultural analysis has been
concerned with the elaboration of theoretical B. Women were not given any right
structures that contest the previous dominant equation to men
western ways of seeing things. A simple analogy
C. Women were treated at par with men
would be with feminism, which has involved a
comparable kind of project: there was a time D. Women liked to be treated inferior to
when any book you might read, any speech you men
might hear, any film that you saw, was always
told from the point of view of male. The woman 33. Why was 'she never the observing eye'?
was there, but she was always an object, never a A. She was beautiful, so she was observed
subject. From what you would read, or the films by men
you would see, the woman was always the one
who was looked at. She was never the observing B. She liked to be observed by men

PAGE 206
www.byjusexamprep.com

C. Women were assumed to be less What is living? When we try to define "living, we
intelligent that men conventionally look for distinctive characteristics
exhibited by living organisms. Growth,
D. Women were assumed to be more
reproduction, ability to sense environment and
intelligent that men
mount a suitable response come to our mind
34. The contestation to dominance of the male immediately as unique features of living
resulted in organisms. One can add a few more features like
A. participation of women in social metabolism, ability to self-replicate, self-
relations, politics, law medicine, the organize, interact and emergence to this list.
arts, popular and academic knowledge 36. Why are the living types amazing?
B. participation of men in social relation A. The extraordinary diversity of habitats
politics, law, medicine, the arts, makes it amazing
popular and academic knowledge
B. The living organisms are acting as per
C. participation of women in social
their interests
movements
D. contestations with males in life leading C. The human thinking makes the living
to divorce types amazing

35. Which word in the passage is opposite of D. the evolution of life makes it amazing
'contrast'? 37. Why does the author say, 'ecological
A. Contestations conflict and cooperation'?
B. Trivial A. Because living organisms are structured
C. Discredited this way

D. Analogy B. Because ecological mechanism worms


with conflict and cooperation
Direction: Read the passage given below and
answer the questions that follow. C. Because humans want to fight and live
together
How wonderful is the living world! The wide
range of the living types is amazing. The D. Because living organisms sometimes
extraordinary habitats in which we find living fight and sometimes live tougher
organisms, be it cold mountains, deciduous
forests, oceans, fresh water lakes, deserts or hot 38. Which of the following statements is true of
springs, leave us speechless. The beauty of a the passage?
galloping horse, or a migrating bird, the valley of A. Meaning of life could be reflected as to
flowers or the attacking shark evokes awe and a what living is as opposed to the non-
deep sense of wonder. The ecological conflict living and what the purpose of life is
and cooperation among members of a
population and among populations of a B. Meaning of life could be reflected as to
community or even a molecular traffic inside a how living organisms live and non-living
cell make us deeply reflect on- what indeed is life organisms exist
? This question has two implicit questions within
C. Meaning of life could be reflected as to
it. The first is a technical one and seeks answer
where the life begins and where it ends
td living is as opposed to the non-living, and the
second is the philosophical one, and seeks D. Meaning of life could be reflected on
answer to what the purpose of life is. how various living organisms differ

PAGE 207
www.byjusexamprep.com

39. Distinctive characteristics exhibited by 44. Indian feminism grew out of the women’s
organisms indicate that movements (a)/ of the late nineteenth
century (b)/ reached full maturity in the
A. they are living organisms early twentieth century (c)/ No error (d).
B. they are non-living organisms A. (a) B. (b)
C. they can be either living organisms or C. (c) D. (d)
non-living organisms
45. The greatest merit of democracy is that
D. the know the purpose of life everyone feels free

40. Which word in the passage means 'unique'? (a) (b)


and can pursues his/her interest. No error
A. common
(c) (d)
B. characteristics
A. (a) B. (b)
C. distinctive
C. (c) D. (d)
D. general 46. All stake holders of education have the right
Direction: Each time in this section has a to ask for accountability
sentence with three underlined parts labelled as (a) (b)
(a), (b) and (c). Read each sentence to find out
in every aspects of its implementation. No
whether three in any error in any underlined part
error
and indicate your response on the Answer Sheet
against the corresponding letter, i.e., (a) or (b) or (c) (d)
(c). If you no error, your response should be
A. (a) B. (b)
indicated as(d).
C. (c) D. (d)
41. He has been (a) one the most revered
member (b) of the committee of enquiry. 47. Learning many languages promotes
(c) No error (d) linguistic, cultural and harmonies
(a) (b)
A. (a) B. (b)
Among people speaking different
C. (c) D. (d) languages. No error
42. Rahul asked me (a) whether I was (c) (d)
interested (b) to joining the group for the
trip. (c) No error (d) A. (a) B. (b)
C. (c) D. (d)
A. (a) B. (b)
48. One should not act according to one’s
C. (c) D. (d)
(a) (b)
43. ‘Where there is a will then there is a way’ is
whims and fancies on public places. No
an old epithet. No error
error
(a) (b) (c) (d) (c) (d)
A. (a) B. (b) A. (a) B. (b)
C. (c) D. (d) C. (c) D. (d)

PAGE 208
www.byjusexamprep.com

49. Economists believe that India had taken a 54. When people found that the jewel was in
new turn in 1990 records of Rahim, they gave it to him.
(a) (b) A. Pronoun B. Nominative

with the liberalization to her economy. No C. Noun D. Adverb


error 55. It is eleven O'clock now and all of us should
(c) (d) retire to bed.
A. Personal pronoun
A. (a) B. (b)
B. Relative pronoun
C. (c) D. (d)
C. Impersonal pronoun
50. Irrigation works have a special importance
in an agricultural countries like India, D. Verb

(a) (b) 56. The flower is very beautiful.

Where rainfall is unequally distributed A. Adjective B. Adverb


throughout the seasons. No error C. Preposition D. Conjunction
(c) (d) 57. This boy is strong than Ramesh.
A. (a) B. (b) A. Noun B. Adjective

C. (c) D. (d) C. Article D. Adverb

Direction: Each of the following sentences has a 58. I hurt myself.


word or phrase underlined. Read the sentences A. Noun
carefully and which part of speech the
underlined word is. Indicate your response on B. Pronoun
the Answer Sheet accordingly. C. Demonstrative preposition
51. All the pilgrims rested for a while under the D. Adjective
banyan tree.
59. The ants fought the wasps.
A. Adverb B. Place value A. Intransitive verb
C. Preposition D. Verb B. Transitive verb
52. The wonderful statue of the leader C. Demonstrative verb
welcomes all people to city.
D. Adjective
A. Object B. Adjective
60. I can hardly believe it.
C. Noun phrase D. Noun A. Adjective B. Preposition
53. This is his pen. C. Adverb D. Verb
A. Possessive pronoun Direction: Each item in this section consists of a
sentence with an underlined words(s) followed
B. Possessive adjective
by four words/group of words. Select the option
C. Adverb that is nearest in meaning to the underlined
word and mark your response on the Answer
D. Verb Sheet Accordingly.

PAGE 209
www.byjusexamprep.com

61. Emboldened by its success, the leader now 68. People avoided him for his high
plans to go ahead with the plan and mindedness.
implementation.
A. toughness B. strong principles
A. Encouraged B. Disgruntled
C. anger D. whims
C. Succeeded D. Failed.
69. There is a tendency to treat social changes
62. It is encouraging to see India's indigenous as mere development in terms of
cinema is going places. accumulation of wealth.
A. homogenous
A. position B. predisposition
B. classical
C. thinking D. idea
C. home-grown
70. During the ancient period poets were
D. Non-native language patronized through various institutions.
63. The ability to imagine and connective a
A. supported B. respected
common good is inconsistent with what is
known as 'pleonexia' is a major struggle for C. opposed D. scolded
a good democracy to realize.
Direction: Each item in this section consists of
A. Greed to grab everything for oneself sentences with an underlined word following by
B. Greed to accumulate more and more four words or group of words. Select the option
wealth that is opposite in meaning to the underlined
word and mark your response on the Answer
C. Dislike for others Sheet accordingly.
D. Over ambitious
71. The archaic thinking leads to unfounded
64. He tried to avoid saying something that beliefs.
would implicate him further.
A. antiquated
A. reward B. incriminate
B. outmoded
C. encourage D. incite
C. beyond the times
65. The statutory corporate tax which forms
the major income of the government has D. modern
not changed this year.
72. Police had to resort to tear gas to diffuse
A. legislature B. unlawful tension among the crowd.
C. government D. legal A. concentrate B. scatter
66. He has been part of the all dissident C. disperse D. strew
activities.
73. Unrest in some pockets made the city
A. rebellious B. supportive
dwellers confine themselves at home.
C. conformist D. legal
A. Turbulence
67. Advocacy is one major component of any
new programme. B. Unease

A. promotion B. opposition C. Apprehension


C. critique D. liking D. Calm

PAGE 210
www.byjusexamprep.com

74. Peace and tranquility are instruments required to find the proper sequence of the four
which would boost the development of sentences and mark your response accordingly
society. on the Answer Sheet.
A. uproar B. calm 81. SI : The country's economy is growing and
would continue to grow at a rapid pace in
C. serenity D. sound
the coming years.
75. Barring a decision of such disputes, other
S6 : The market share of electrical vehicles
matters relating to the election of President
increases with increasing availability of
or Vice-President may be regulated by law
infrastructure.
made by Parliament.
P: It also provides us an opportunity to
A. excepting B. without
grow as manufacturer of electric vehicles.
C. including D. excluding
Q: According to NITI Aayog (2019), if India
76. His speech was full of emotions, and it was reaches an electric vehicles sales
an extempore. penetration, emission and oil savings can
A. prepared B. ready-made be achieved.

C. unrehearsed D. ad lib R: Given the commitments that India has


made on the climate front as a nation and
77. The teacher asked her students to on environmental aspect, it is likely that
understand the ensuing problems and larger and larger share of automobiles
address them suitably. sector would be in the form of electric
A. subsequent B. consequent vehicles.

C. retrospective D. en suite S: This presents a great opportunity for the


automobile industry, as the demand for
78. All the allegations against the actor were automobiles would only increase.
expunged by the committee of inquiry.
The correct sequence should be
A. got rid of B. part of
A. S R Q P B. R Q S P
C. accepted D. rejected
C. Q P S R D. Q S R P
79. His relatives dissuaded him from giving up
the job. 82. SI: Central government receipts can broadly
be divided into non-debt and debt receipts.
A. persuaded B. discouraged
S6: This is also evident from the
C. advised against D. deter composition of non-debt receipts.
80. He is one of the confidants of the leader P: Debt receipts mostly consist of market
and can influence the decision of the borrowing and other liabilities which the
government. government I obliged to repay in the future.
A. opponents B. intimate Q: The non-debt receipts comprise of tax
C. close friend D. colleague revenue, non-tax revenue, recovery of
loans and disinvestment receipts.
Direction: In this section each item consists of six
sentences of a passage. The first and sixth R: The outcomes as reflected in the
sentences are given in the beginning as SI and S6. Provisional Actual figures is lower than the
The middle four sentences in each have been budget estimate owing to reduction in the
jumbled up and labelled as P, Q, R and S. You are net tax revenue.

PAGE 211
www.byjusexamprep.com

S: The Budget 2018-19 targeted S: The role of insulin in keeping the blood
significantly high growth in non-debt glucose level within the narrow limit is an
receipts of the Central Government, which example of this function.
was driven by robust growth.
The correct sequence should be
The correct sequence should be
A. P S R Q B. R S P Q
A. S R P Q B. R S Q P
C. S R Q P D. Q R S P
C. P Q R S D. Q P R S 85. S1: All living things affect living and non-
83. S1 : Paleontology is the study of the living things around them.
remains of dead organisms over enormous S6: This inter dependability needs to be
spans of time. understood when we, humans consume
S6: Faunal analysis gives information about much more than required and abuse
the animal people hunted and nature.
domesticated, the age of animal at death, P: This can also affect the population of fox,
and the diseases the afflicted them. if foxes depend on rabbits for food.
P: Bones provide a great information Q: For example, earthworms make burrows
Q: The distribution of faunal remains and worm casts,
(animal bones) at a site can indicate which R: This act of earthworms affect the soil,
area were used for butchering, cooking, and therefore the plants growing in it.
eating bone tool making and refuse
dumping. S: Rabbit's fleas carry the virus which causes
myxomatosis, so they can affect the size of
R: Within this discipline, molecular biology the rabbit population.
and DNA studies have been used to
understand hominid evolution. The correct sequence should be

S. Hominid evolution answers the questions A. R S Q P B. P S R Q


about what ancient people looked like, and C. Q R S P D. S Q R P
to plot patterns of migration.
86. S1: The ecosystem of water is complex and
The correct sequence should be many environmental factors are intricately
linked.
A. Q P R S B. S P Q R
S6: The trees slowly transfer rainwater into
C. R S P Q D. P Q R S
the sub-soil and this is critical for sustaining
84. S1: Hormone have several functions in the water for months after the rains.
body.
P: Thick forests make for excellent
S6: The two hormones together regulate catchments.
the glucose level in the blood.
Q: The problems we see are because we
P: They help to maintain the balance of have undermined these links over decades.
biological activities in the body.
R: First rain and snowfall are the only
Q: Insulin is released in response to the sources of water – about 99%.
rapid rise in blood glucose level. S: In the four months of monsoon, there are
R: On the other hand hormone glucagon about 30-35 downpours and the challenge
tends to increase the glucose level in the is to hold this water in systems that can last
blood. us over 365 days.

PAGE 212
www.byjusexamprep.com

The correct sequence should be The correct sequence should be


A. Q R S P B. P S R Q A. Q R S P B. R S P Q

C. S R Q P D. R Q S P C. P S Q R D. S Q R P

87. S1 : Politics is exciting because people 89. S1: On increasing the temperature of solids,
disagree. the kinetic energy of the particles increases.

S6: It is not solitary people who make S6: The temperature at which a solid melts
politics and a good society, it is the people to become a liquid at the atmospheric
together which make good politics and pressure is called its melting point.
society. P: A stage is reached when the solid melts
P: For Aristotle politics is an attempt to and is converted to a liquid.
create a good society because politics is, Q: Due to the increase in kinetic energy, the
above all, a social activity. particles start vibrating with greater speed.
Q: They also disagree about how such R: The particles leave their fixed positions
matters should be resolved, how collective and start moving more freely.
decision should be made and who should
have a say. S: The energy supplied by heat overcomes
the forces of attraction between the
R: They disagree about how they should particles.
live.
The correct sequence should be
S: Who should get what? How should
A. Q S R P B. Q R S P
power and other resource be distributed?
Should society be based on cooperation or C. P R S Q D. S P R Q
conflict? And so on.
90. S1: Things are often not what they seem.
The correct sequence should be
S6: This happened without you even
A. R S Q P B. P Q S R knowing it. So imagine the changes that
occurs to this earth and humanity.
C. Q S R P D. R S P Q
P: But you are really not, because the Milky
88. S1: Regular exercise makes many of the Way galaxy, of which you are a part, is
organ systems become more efficient. moving through space at 2.1 million
S6: Different activities require different kilometres an hour.
levels of fitness. Q: So, in roughly twenty-second that it
P: It can improve your strength: make your would have taken you to read this
body-more flexible and less likely to suffer paragraph, you have already moved
from sprain. thousands of kilometre.

Q: It can also improve your endurance. R: And that is without taking into account
the effects of earth's ration on its won axis.
R: It also uses up energy and helps to Its orbiting around the sun and sun's
prevent large amounts of fat building up in journey around the Milky Way.
the body.
S: As you read this sentence, perhaps sitting
S: Exercise can increase your fitness in three in a comfortable chair in your study, you
ways. would probably consider yourself at rest.

PAGE 213
www.byjusexamprep.com

The correct sequence should be 97. Very few of the texts from very early Vedic
period are _______ now.
A. Q R P S B. R Q P S
A. extant B. exit
C. P Q R S D. S P R Q
C. exempt D. redundant
Direction: Each of the following sentences in this
section has a blank space, and four words or 98. A speech is a _______ address, delivered to
group of words are given after the sentence. an audience that seeks to convince,
Select the most appropriate word or group of persuade, inspire or inform.
words for the blank space and indicate your
A. formal B. informal
response on the Answer Sheet accordingly.
C. humorous D. political
91. If I __________ a good match I would have
got married. 99. All that _______ is no gold

A. had found B. have found A. glitter B. glitters

C. found D. have C. glittering D. gliding

92. The lady has been declared as on of the top 100. Having been in politics for about 40 years,
ten __________ of the community. the party now treats him like________.

A. more powerful members A. a have-been

B. most powerful members B. a had-been

C. most powerful member C. a has-been

D. more powerful member D. would have been

93. When I visited the villages nearby the city, I Direction: Each of the following items in this
__________ many water bodies intact. section consists of a sentence, parts of which
have been jumbled. These parts have been
A. came across B. come across labelled as P, Q, R and S. Given below each
sentence are four sequences namely (a), (b), (c)
C. came D. came in
and (d). You are required to re-arrange the
94. He has lost all his investments and he is jumbled parts of the sentence and mark you
________. response accordingly.

A. broke B. broken 101. history of life (P) evolutionary Biology is (Q)


forms on earth (R) the study of (S)
C. discredited D. defunct
The correct sequence should be
95. He ______ whether he could get any
certificate for the course. A. S P Q R B. Q S P R

A. said B. told C. R P Q S D. P S Q R

C. thought of D. asked 102. life is considered (P) the origin of (Q) the
history of universe (R) a unique event in (S)
96. I _______ farewell to all my course mates
last year. The correct sequence should be

A. bid B. bade A. Q P S R B. P S Q R

C. said D. bad C. S Q P R D. R S P Q

PAGE 214
www.byjusexamprep.com

103. productive resources is (P) how we manage 109. during the last century (P) Indian social,
(Q) and competitiveness (R) critical to political and cultural life (Q) as a testimony
strategic growth (S) of (R) Indian cinema stands (S)
The correct sequence should be The correct sequence should be
A. P Q R S B. R S P Q A. S P Q R
C. S R P Q D. Q P S R B. Q R S P
104. in service firms (P) operation strategy (Q) C. P Q R S
from the corporate strategy (R) is generally
inseparable (S) D. S R Q P

The correct sequence should be 110. of all searches for knowledge (P) should be
the beginning (Q) an exploration into truth
A. S R Q P B. Q P S R (R) and experiments of life (S)
C. R S P Q D. P S Q R The correct sequence should be
105. are travelling, (P) a recent survey has A. R Q P S B. S P Q R
revealed (Q) that they are worried about
their safety (R) even as more Indians (S) C. R S P Q D. Q R S P

The correct sequence should be Direction: Given below are some idioms/phrases
followed by four alternative meanings to each.
A. S P Q R B. Q S R P Choose the response A, B, C or D, which is the
C. P R S Q D. R P S Q most appropriate expression and mark you
response in the Answer Sheet accordingly.
106. the imagination of children (P) stories can
exercise (Q) more than the stories (R) 111. Get the jitters
because they tell (S) A. Feeling anxious
The correct sequence should be B. Feeling happy
A. Q R S P B. S P Q R
C. Stammering
C. Q P S R D. R S Q P
D. Feeling exposed
107. as a record of (P) and suffering of humans
112. French leave
(Q) the achievements, experiments (R)
history is considered (S) A. Absent from work without asking for
permission in French
The correct sequence should be
B. Asking for permission before leaving
A. S P R Q B. R Q S P
work
C. P Q R S D. Q R S P
C. Work for permission to get leave
108. can be invented (P) it appears (Q) has been
invented (R) that all that (S) D. Absent from work without asking for
permission
The correct sequence should be
113. Take a stand
A. Q S P R B. Q R S P
A. To publicly express an opinion about
C. R S Q P D. S P Q R something

PAGE 215
www.byjusexamprep.com

B. To make a stand for one to sit 117. Around the corner


C. To be firm on your work A. A thing which is at the end of the corner

D. To be part of the work B. An event or thing which is going to


happen soon
114. Cut and run
C. An event that corners someone for his
A. To avoid a difficult situation by leaving wrong
suddenly
D. An event that happens in the corner of
B. To avoid an event suddenly powerful place
C. To meet some danger suddenly 118. With Heavy Heart
D. To ask for sudden meeting with A. With heavyweight
someone
B. With joy and humour
115. Cut the cord
C. With a sense of shame
A. To stop needing your parents for
D. With pain and regret
money
119. Cost a bomb
B. To stop needing someone else to look
after you start acting independently A. To be very arrogant

C. To be safe on your own B. To be with rich people

D. To be a married person C. To be very expensive

116. Cupboard love D. To be stingy

A. Loving someone to get something from 120. Roll your sleeves up


the person A. To prepare for wrestling
B. Loving the cupboards B. To prepare for hard work
C. innocent love C. To make someone work for you
D. Loving to be free of all conditions D. To work with others.

PAGE 216
www.byjusexamprep.com

General Knowledge
1. Lead nitrate on heating gives B. one X chromosome from mother and
one Y chromosome from father
A. PbO2 and NO2
C. two X chromosomes from mother and
B. PbO and NO2 one X chromosome from father
C. PbO and NO D. one X chromosome and one Y
D. PbO2 and NO chromosome from father and one X
chromosome from mother
2. The valency of phosphorus is
8. Mature sclerenchyma cells have
A. 2, 3 B. 3, 4
A. cellulose wall and are living
C. 4, 5 D. 3, 5
B. lignified wall and are living
3. Which one of the following does not form C. suberized wall and are dead
an oxide on reaction with oxygen?
D. lignified wall and are dead
A. Magnesium B. Lead
9. Which one of the following statements
C. Tin D. Silver about phloem is correct?
4. Which one of the following is used for A. Phloem transports water and minerals.
storing biological tissues?
B. Phloem transports photosynthetic
A. Liquid nitrogen products.
B. Liquid helium C. Phloem is a simple tissue.
C. Liquid argon D. phloem gives support to the plant.

D. Liquid bromine 10. Which one among the following is a non-


conventional source of energy?
5. The radioactive isotype of hydrogen is
A. Petroleum
A. protium B. deuterium
B. Coal
C. tritium D. hydronium
C. Radioactive elements
6. Antibiotic such as penicillin blocks D. Solar energy
A. cell wall formation in bacteria 11. Which one of the following combinations of
B. RNA synthesis in bacteria source and screen would produce the
sharpest shadow of an opaque object?
C. DNA synthesis in bacteria
A. A point source and an opaque screen
D. division in bacteria
B. An extended source and an opaque
7. In human beings, the chromosomes that screen
determine the birth of a normal female
C. A point source and a transparent
child are
screen
A. one X chromosome from mother and
D. An extended source and a transparent
one X chromosome from father
screen

PAGE 217
www.byjusexamprep.com

12. Which one of the following phenomena A. Zinc oxide


verifies the fact that light travels much
B. Copper oxide
faster than sound?
C. Magnesium oxide
A. Twinkling of stars in the night sky
D. Calcium
B. Lighting of a matchstick
18. Identify the correct pair of elements among
C. Thunderstorm the following which are liquid at room
temperature and standard pressure.
D. Mirage
A. Bromine and fluorine
13. When a light ray enters into the glass
medium from water at an angle of B. Mercury and rubidium
incidence 0°, what would be the angle of C. Bromine and thallium
refraction?
D. Bromine and mercury
A. 90°
19. Symbol of an element was Introduced by
B. 45°
A. John Dalton
C. 0°
B. Antoine Lavoisier
D. The ray will not enter at all C. Jones Jakob Berzelius
14. A luminous object is placed at a distance of D. Robert Doyle
40 cm from a converging lens of focal length
25 cm. The image obtained on the screen is 20. A mixture of sodium chloride (salt) and
ammonium chloride can be separated by
A. erect and magnified
A. sublimation
B. erect and smaller
B. filtration
C. inverted and magnified C. chromatography
D. inverted and smaller D. distillation
15. Which of the following lenses will bend the 21. Apart from hyper acid secretion, peptic
light rays through the largest angle? ulcers are also developed due to bacterial
infection. The causative agent is
A. Lens with power +2.0 D
A. Helicobacter pylori
B. Lens with power +2.5 D
B. E. coli
C. Lens with power –1.5 D
C. Streptococcus pneumonia
D. Lens with power –2.0 D
D. Salmonella typhimurium
16. Silver articles turn black when kept in the
22. Wings of birds and bats are considered
open for a longer time due to the formation
analogous structures because they have
of
A. common origin and common function
A. H2S B. AgS
B. different origin and common function
C. AgSO4 D. Ag2S
C. common origin and different function
17. Which one of the following oxides shows
both acidic and basic behavior? D. different origin and different function

PAGE 218
www.byjusexamprep.com

23. Pearls are harvested from 29. If the distance between two objects is
increased by two times, the gravitational
A. Prawn B. Pila
force between them will
C. Tuna D. Oyster
A. remains same
24. All the individuals of a particular organism,
such as rose plants, belong to a taxonomic B. increase by two times
category called C. decrease by two times
A. species B. genus D. decrease by four times
C. family D. order 30. If the linear momentum of a moving object
25. Under the Kingdom Plantae, which of the gets doubled due to the application of a
following individuals are predominantly force, then its kinetic energy will
aquatic? A. remains same
A. Bryophytes B. Algae
B. increase by four times
C. Pteridophyte D. Gymnosperms
C. increase by two times
26. If the speed of light in air is represented by
D. increase by eight times
c and the speed in a medium is v, then the
refractive index of the medium can be 31. The Pragyan rover installed in the
calculated using the formula Chandrayaan-2 mission had how many
wheels?
A. B.
A. 2 B. 3
C. 4 D. 6
C. D.
32. Which one of the following lakes in India '
has a large quantity of a substance found in
27. Which one of the following statements with
the Moon?
regard to the ultraviolet light is not correct?
A. Lonar Lake, Maharashtra
A. It Is an electromagnetic wave.
B. It can travel through a vacuum. B. Pangong Lake, Ladakh

C. It is a longitudinal wave. C. Chilika Lake, Odisha

D. Its wavelength is shorter/smaller than D. Loktak Lake, Manipur


that of visible light. 33. The Government of India programme
28. Which one of the following statements regarding 'Stay in India and Study in India' is
about the properties of neutrons is not initiated by
correct? A. the Ministry of Youth Affairs and Sports
A. Neutron mass is almost equal to proton B. the Ministry of Culture
mass.
C. the Ministry of Education
B. Neutrons possess zero charges.
D. the Ministry of Tourism
C. Neutrons are located inside the atomic
nuclei. 34. ASEEM is
D. Neutrons revolve around the atomic A. Aatmanirbhar Skilled Employee
nuclei. Employer Measurement

PAGE 219
www.byjusexamprep.com

B. Aatmanirbhar Skilled Employee 39. Which one among the following was India’s
Employer Mapping first trade union in the proper sense of the
term?
C. Aatmanirbhar Skilled Employee
Enterprises Medium A. Bombay Labour Union
D. Automatic Skilled Employee Employer B. Ahmedabad Labour Union
Mission
C. Madras Labour Union
35. Recently, a rare kind of yellow turtle was
D. Allahabad Labour Union
discovered in India. The State in which it
was seen is 40. Who among the following created the first
all India Trade Union Congress in 1920?
A. Uttarakhand
A. B .P. Wadia
B. Odisha
B. S.A. Dange
C. Tamil Nadu
C. N.M. Joshi
D. Arunachal Pradesh
D. B .T. Ranadive
36. Who among the following moved the
motion of Secret sitting session of the 41. With whom did Subhas Chandra Bose form
Assembly (1942)? an alliance to destroy the Holwell
Monument in Calcutta during 1939-40?
A. M.S. Aney
A. The Communist Party of India
B. G.V. Mavalankar
B. The Muslim League
C. C. M. Stephen
C. The Hindu Mahasabha
D. A. Ayyangar
D. The Unionist Party
37. Which one among the following is not
correct about the Secretary-General of the 42. In which one of the following places was the
Lok Sabha? Ahmadiyya Movement started by Mirza
Ghulam Ahmad?
A. The Secretary-General is the advisor to
the Speaker. A. Patna B. Aligarh
B. The Secretary-General acts under the C. Bhopal D. Gurdaspur
authority in the name of the Speaker.
43. Who among the following gave evidence
C. The Secretary-General works under the before the Joint Select Committee on the
Speaker with delegated authority. Government of India Bill, 1919 in favour of
female franchise?
D. The secretary General passes orders in
the name of the Speaker. 1. Mrs. Annie Besant
38. Who among the following formed the 2. Mrs. Sarojini Naidu
Seva Samiti Boy Scouts Association in 1914?
3. Mrs. Hirabai Tata
A. Hriday Nath Kunzru
Select the correct answer using the code
B. S.G. Vaze given below –
C. Annie Besant A. 1 only B. 1 and 2 only
D. Shri Ram Bajpai C. 2 and 3 only D. 1, 2 and 3

PAGE 220
www.byjusexamprep.com

44. Which one of the following cities is closest A. urea subsidy


to the Equator?
B. petroleum subsidy
A. Mogadishu B. Singapore
C. food subsidy
C. Colombo D. Manila
D. fertilizer subsidy
45. The largest geographical area of India is
covered by which one of the following 52. Which one of the following statements
types of soils? about the Indian economy during 2019-20
is not correct?
A. Inceptisols B. Entisols
A. There has been a deceleration in
C. Alfisols D. Vertisols growth rate.
46. Which one of the following indicates the B. There has been sluggish growth in tax
Tropical Savannah climate? revenue relative to the Budget
A. Aw B. Dfc Estimates.

C. Cwg D. Am C. Fiscal deficit as a percentage of GDP


has been as per the Budget Estimates.
47. The Isotherm Line, which divides India
North-South into almost two equal parts in D. The non-tax revenue registered a
the month of January, is considerably higher growth.

A. 10°C B. 25°C 53. Which one of the following statements with


regard to the National Food Security Act is
C. 15°C D. 20°C
not correct?
48. Decadal growth rate of population in
A. The Act was enacted in the year 2013
percentage was highest in India in the year
B. The Act was rolled out in the year 2014
A. 1991 B. 1981
C. 1971 D. 1961 C. The act legally entitles 67 percent of
the population to receive highly
49. Timber vegetation is generally not found in subsidised food grains.
which of the following regions?
D. The Act is not being implemented in all
A. Subtropical region the States/Union Territories.
B. Temperate region E. None of the above
C. Alpine region 54. As per the World Bank’s Ease of Doing
D. Tundra region Business Ranking. India’s rank had
improved from 142 in 2014 to 63 in 2019.
50. Which of the following Indian places During this period, in which of the following
receives minimum rainfall in a year? parameters has India’s rank deteriorated?
A. Jodhpur B. Leh A. Ease of starting a business
C. New Delhi D. Bengaluru B. Getting electricity
51. As per the Budget Estimates of expenditure C. Registering property
on major subsidies during 2019-20, the
maximum expenditure was likely to be on D. Paying taxes

PAGE 221
www.byjusexamprep.com

55. As per the Budget Estimates of 2019-20, the 59. The recent explosion near OIL will in
following are some of the important Baghjan is due to
sources of tax receipts for the Union
A. removing the spool during the blowout
Government:
control operations
1. Corporation: Tax
B. transfer of oil from its depot to a
2. Taxes on Income other than pipeline
Corporation Tax
C. the leakage of methyl isocyanide
3. Goods and Services Tax
D. the leakage of radiations from the
4. Union Excise Duties radioactive substance
Which one of the following is the correct 60. Which one of the following Indian institutes
descending order of the aforesaid tax was approved by the Drugs Controller
receipts as a percentage of GDP? General of India for conducting human
A. 1, 2, 3, 4 B. 1, 3, 2, 4 trials of the Oxford-Astra-Zeneca Covid-19
vaccine candidate?
C. 3, 2, 1, 4 D. 2, 4, 3, 1
A. Bharat Biotech
56. Match List-I with List-II and select the
correct answer using the code given below B. AIIMS
the lists: C. Serum Institute of India
List-I List-II D. National Institute of Epidemiology
(Active Volcano) (Location)
61. What is 'Little Boy'?
(a) Mount Merapi 1. Hawaii
A. The fission bomb dropped at Hiroshima
(b) Sakurajima 2. Italy
B. The fusion bomb dropped at Nagasaki
(c) Mount Vesuvius 3. Japan
C. The first nuclear bomb tested by
(d) Mauna Loa 4. Indonesia America
D. The first nuclear bomb tested by North
A. 1 2 3 4 B. 1 3 2 4 Korea
C. 4 2 3 1 D. 4 3 2 1 62. In August 2020, a blast has taken place at
Beirut, killing about one hundred people
57. Which one of the following countries is not
and thousands wounded. The blast was
located on the Tropic of Capricorn?
caused by
A. Chile B. Brazil
A. dynamite
C. Paraguay D. Uruguay
B. ammonium nitrate
58. Who among the following is the architect of
C. RDX
the Ram Temple being constructed at
Ayodhya? D. mercury nitride
A. P. O. Sompura 63. Which one of the following statements with
B. Chandrakant Sompura regard to 'protective democracy' is not
correct?
C. Brinda Somaya
A. It propounds that citizen participation
D. B.V. Doshi is essential in democracies.

PAGE 222
www.byjusexamprep.com

B. Citizens must be able to protect 67. The socialist idea of Sapta Kranti (Seven
themselves from governmental Revolutions) was proposed by
encroachments.
A. Ram Manohar Lohia
C. It is compatible with laissez-faire
capitalism B. Jawaharlal Nehru

D. Political equality is understood in C. M. G. Ranade


formal terms as equal voting rights. D. Jayappakash Narayan
64. Which one of the following statements in
68. The National Disaster Management
relation to Panchayats is not correct?
Authority functions under the Ministry of
A. Legislature of a State may, by law, make
A. Environment, forest and Climate
provisions with respect to the
Change
composition of Panchayats
B. Home Affairs
B. Panchayat area means the territorial
area of a Panchayat. C. Commerce and Industry
C. Gram Sabha includes all persons in the D. Finance
electoral rolls of the village within a
Panchayat 69. Who among the following formed the
National Liberation Federation (Liberal
D. Reservation of seats of SCs and STs has Party)?
nothing to do with the proportion of
their population. A. Motilal Nehru and C. R. Das
65. A special address by the Governor refers to B. Muhammad Ali and C. R. Das
the address delivered by the Governor
C. T. B. Sapru and M. R. Jayakar
A. when President's Rule is called for
D. M. R. Jayakar and C. R. Das
B. when a national emergency
necessitates dissolution of Legislative 70. Who among the following was the first to
Assembly accept a ministerial position in the central
provinces in October 1925?
C. at the commencement of the first
session after the general election and A. B.S. Moonje
at the first session of each year B. M. R. Jayakar
D. whenever he/she has concluded that
C. S. B.Tambe
such is necessary
D. B.N. Sasmal
66. Which one among the following is not a
character of a secular state? 71. After the first world war, the Triveni Sangh
A. It refuses theocracy was formed by

B. It separates religion from the State A. the Jats and Gujjars

C. A state in order to be secular must be B. the Rajputs and Yadavs


democratic C. the jats and Yadavs
D. It must prevent religious conflict and
D. the Ahirs and Kurmis
promote religious harmony

PAGE 223
www.byjusexamprep.com

72. Who among the following founded the D. the Supreme Court, High Courts and
Mohammedan Anglo-Oriental Defence Lower Courts
Association (1893)?
78. Who among the following was the advisor
A. Auckland Colvin to the Constituent Assembly?
B. Badruddin Tyabji A. B .N. Rau
C. Theodore Beck B. B .R. Ambedkar
D. Sir Syed Ahmad Khan C. Pattabhi Sitaramayya
73. Which one of the following was not a D. Alladi Krishanswamy
demand made by the Congress Moderates?
79. Which of the following terms were added
A. Universal adult franchise
to the Preamble of the Constitution of India
B. Repeal of the Arms Act by the Constitutional Amendment, 1976?
C. Extension of Permanent Settlement 1. Socialist 2. Secular
D. Higher jobs for Indians in the army 3. Integrity 4. Fraternity
74. During the 19th century, who among the Select the correct answer using the code
following wrote Satapatra Series? given below.
A. M. G. Ranade A. 1 and 2 only B. 1, 2 and 3
B. G. Tilak C. 2 and 4 D. 1, 3 and 4
C. Bankim Chandra Chatterjee
80. Which one of the following is not a
D. G. H. Deshmukh classified category of political parties as
outlined by the Election Commission of
75. The largest barrier reef system in the world
India?
is found as
A. East Australian Coast A. National Parties

B. West Australian Coast B. State Parties

C. North Australian Coast C. Regional Parties

D. South Australian Coast D. Registered Unrecognised Parties


76. The Citizenship (Amendment) Act falls 81. Which one of the following statements with
under which one of the following Parts of regard to the appointment of the Members
the Constitution of India? of the Parliamentary Committees is
correct?
A. Part I B. Part II
A. The Members are only appointed.
C. Part IV D. Part VI
77. In the Indian judicial system, writs are B. The Members are only elected
issued by C. The members are only nominated.
A. the Supreme Court only D. The members are appointed or elected
B. the High Courts only on a motion made and adopted or
nominated by the Speaker of the Lok
C. the Supreme Court and High Courts Sabha or the Chairman of the Rajya
only Sabha.

PAGE 224
www.byjusexamprep.com

82. Which of the following statements with 86. Which one of the following towns was not
regard to the privileges of the Members of a centre of the Revolt of 1857?
the Parliament are correct?
A. Ayodhya B. Agra
1. Privileges would not be fettered by the
Article 19(1)(a) of the Constitution of C. Delhi D. Kanpur
India. 87. At which of the following sessions of the
2. Privileges must be read subject to the Indian National Congress was the
Articles 20-22 and Article 32 of the resolution on Fundamental Rights and
Constitution of India. Economic Policy passed?

3. Immunity is available in relation to both A. Tripuri session


civil and criminal prosecution. B. Lahore session
4. Immunity is available in relation to C. Lucknow session
freedom of speech even in his/her
private or personal capacity. D. Karachi session
Select the correct answer using the code 88. Who among the following wrote The
given below. Philosophy of the Bomb?
A. 1, 2 and 4 B. 1 and 2 only A. Sukhdev
C. 2 and 3 D. 1 and 4 only B. Chandrashekar Azad
83. Al - Biruni’s Kitab-ul-Hind was written in C. Bhagwati Charan Vohra
which language?
D. Bhagat Singh
A. Arabic B. Persian
89. Which one of the following statements
C. Urdu D. Turkish about Gupta coins is not correct?
84. Ibn Batuta went to China as the envoy of A. Gupta kings issued large number of
which one of the following Delhi Sultans? gold coins known as Dinar.
A. Alauddin Khilji
B. Chandragupta II, Kumaragupta I,
B. Muhammad bin Tughluq Skandagupta and Budhagupta issued
silver coins.
C. Iltutmish
D. Firoz Shah Tughluq C. The observes of coins are carved with
the images of the kings and on the
85. Consider the following statements: revers are carved deities.
The Azamgarh Proclamation refers to D. The largest number of coins issued by
1. the declaration by the rebels of 1857 the Guptas were of copper.
2. the statement by the leader of the 90. In the Gandhara School of Art, initially blue
underground movement in the Revolt schist and green phyllite were used. When
of 1942 did stucco completely replace stone as
main material used by Gandhara School
Which of the statements given above is/are
sculptors?
not correct?
A. 1st century CE B. 2nd century CE
A. 1 only B. 2 only
C. Both 1 and 2 D. Neither 1 nor 2 C. 3rd century CE D. 5th century CE

PAGE 225
www.byjusexamprep.com

91. Which one of the following is not a fluvial 97. In August 2020, who among the following
landform? was administered the oath as the Prime
Minster of Sri Lanka for the fourth time?
A. Cirque B. Gorge
A. Gotabaya Rajapaksa
C. Braids D. Canyon
B. Basil Rajapaksa
92. Which one of the following countries does
not have Tundra vegetation? C. Mahinda Rajapaksa
A. Belarus B. USA D. Namal Rajapaksa
C. Russia D. Canada 98. Recently islands of Andaman and Nicobar
were connected with mainland by
93. The four planets closest to the Sun are
Submarine Optical Fibre Cable. Which one
called
of the following islands was not connected
A. terrestrial planets initially?
B. giant planets A. Shaheed Island
C. dwarf planets B. Swaraj Island
D. gas planets C. Little Andaman
94. Which one of the following is considered as D. Port Blair
the deepest point of the oceans?
99. Who among the following has won the
A. Tonga Trench Singles Title in Wimbledon Tennis
Championship (Women) in the year 2019?
B. Mariana Trench
A. Karolina Pliskova
C. Philippine Trench
B. Simona Halep
D. Kermadec Trench
C. Serna Williams
95. Which one of the following is not a major
tectonic plate? D. Naomi Osaka
A. Saudi Arabian plate 100. Which one of the following is
decommissioned aircraft carrier?
B. Antarctica and the surrounding oceanic
plate A. INS Rajput B. INS Chakra
C. India-Australia-New Zealand plate C. INS Khanderi D. INS Viraat
D. Pacific plate 101. Buenos Aires and Montevideo are situated
across the banks of
96. Which one of the following Indian Ocean
island nations has recently declared a state A. River Plate B. Orinoco River
of environmental emergency due to oil spill
C. Purus River D. Madeira River
from grounded ship?
102. Which one among the following Union
A. Maldives
Territories of India is the smallest in
B. Mauritius geographical area?
C. Madagascar A. Chandigarh
D. Sri Lanka B. Puducherry

PAGE 226
www.byjusexamprep.com

C. Dadra and Nagar Haveli and Daman and B. consumers want to buy more at any
Diu given price
D. Lakshadweep C. average income rises

103. Climax mine, the largest producer of D. population grows


molybdenum, is located in 108. As per the use-based classification of the
A. Canada B. USA Index of Industrial Production (IIP), the
maximum weight has been assigned to
C. Australia D. South Africa
A. primary goods
104. Sea of Azov is connected to
B. intermediate goods
A. Black Sea
C. consumer durables
B. Baltic Sea
D. consumer non-durables
C. Mediterranean Sea
109. Arrange the following countries in
D. North Sea descending order as per the Global Human
Development Index, 2019:
105. Which one of the following statements with
regard to ozone is not correct? 1. Germany

A. Ozone is found mostly at 15-55 km in 2. USA


the atmosphere. 3. South Africa
B. Ozone is produced by gaseous chemical 4. India
reactions.
Select the correct answer using the code
C. 16th November is celebrated as the given below.
International Day for the Preservation
of the Ozone Layer. A. 1, 2, 3, 4 B. 1, 3, 2, 4

D. Ozone is a form of oxygen in which C. 3, 2, 1, 4 D. 4, 3, 2, 1


three oxygen atoms are bounded 110. Since 2014-15, India has consistently run
together. trade surplus with which one among the
106. A market, in which there are a large number following countries?
of firms, homogeneous product infinite A. China
elasticity of demand for ari individual firm
and no control over price by firms, is B. Saudi Arabia
termed as C. USA
A. oligopoly D. Germany
B. imperfect competition 111. Which one among the following is not a
coral reef island?
C. monopolistic competition
A. Great Barrier Reef, Australia
D. perfect competition
B. Rainbow Reef, Fiji
107. Normally, there will not be a shift in the
demand curve when C. Swaraj Island, India

A. price of a commodity falls D. Kyushu Island, Japan

PAGE 227
www.byjusexamprep.com

112. Which one of the following States is B. The National Commission for
planned to host the Khelo India Youth Scheduled Tribes
Games (4th Edition)?
C. The National Commission for Backward
A. Kerala B. Haryana Classes
C. Gujarat D. Manipur D. The National Commission for Women
113. G.C Murmu, who was appointed as the 118. Which one of the following is not the
Comptroller and Auditor General of India in necessary condition for the issue of a writ
August 2020, was the Lieutenant of Quo Warranto?
Governor/Administrator of which one of
A. The office must be a public office.
the following Union Territories prior to this
appointment? B. The office must be created by the
Statue or by the Constitution itself.
A. Ladakh
C. The office must not be a substantive
B. Jammu and Kashmir one.
C. Chandigarh D. There has been a contravention of the
D. Puducherry Constitution or a Statue in
appropriating such person to that
114. Who among the following played the role office.
of Shakuntala Devi in the biopic movie
based on the life of the famous 119. Which of the following Articles in the
mathematician? Constitution of India are exceptions to the
Fundamental Rights enumerated in Article
A. Madhuri Dixit B. Rani Mukherjee 14 and Article 19?
C. Tabu D. Vidya Balan A. Article 31A and Article 31C
115. Which one of the following countries had B. Article 31B and Article 31D
chosen the name ‘Nisarga’ for the cyclone
which devastated the coastline of C. Article 12 and Article 13
Maharashtra and Gujarat in June 2020? D. Article 16 and Article 17
A. Maldives B. Bangladesh 120. Which one among the following statements
pertaining to the President’s term of office
C. Thailand D. Japan
is not correct?
116. August 12 is celebrated as
A. The President holds office for a term of
A. the World Environment Day five years.

B. the World No-Tobacco Day B. The President may be removed from


the office by way of impeachment.
C. the International Day against Drug
Abuse and Illicit Trafficking C. The President may resign before the
expiration of his/her term by writing to
D. the International Youth Day the Speaker of the Lok Sabha.
117. Which one of the following Commissions is D. The President shall, not with-standing
related to Article 338A? the expiration of his/her term, continue
A. The National Commission for to hold office until his/her successor
Scheduled Castes enters upon his/her office.

PAGE 228
www.byjusexamprep.com

Elementary Mathematics
1. A car did a journey in t hours. Had the 6. If cosec θ – sin θ = m and sec θ – cos θ = n,
average speed been x kmph greater, the then what is + equal to?
journey would have taken y hours less. How
long was the journey? A. 0 B. 1
–1
A. x (t – y)ty B. x (t – y)ty C. mn D. m2n2
C. x (t – y)th–2 D. x (t + y)ty 7. If cos θ + sec θ = k, then what is the value of
2. When a ball is allowed to fall, the time it sin2θ – tan2θ ?
takes to fall any distance varies as the A. 4 – k B. 4 – k2
square root of the distance and it takes 4
seconds to fall 78.40 m. How long would it C. k2 – 4 D. k2 + 2
take to fall 122.50 m?
8. ABC is a triangle inscribed in a semicircle of
A. 5 seconds B. 5.5 seconds diameter AB. What is cos (A + B) +
sin(A + B) equal to?
C. 6 seconds D. 6.5 seconds
3. If 63 – 4x 4x + 5 = 8 (Given log102 = 0.301 and A. 0 B.
log103 = 0.477), then which one of the
following is correct? C. D. 1
A. 0 < x < 1 B. 1 < x < 2
9. Consider the following statements :
C. 2 < x < 3 D. 3 < x < 4
4. The Euclidean algorithm is used to calculate 1. sin θ = x + is possible for some real
the value of x.
A. Square root of an integer
2. cos θ = x + is possible for some real
B. Cube root of an integer
value of x.
C. Square of an integer
Which of the above statements is/are
D. HCF of two integers correct?
5. If radius of a sphere is rational, then which A. 1 only
of the following is/are correct?
B. 2 only
1. Its surface area is rational.
C. Both 1 and 2
2. Its volume is rational.
D. Neither 1 nor 2
Select the correct answer using the code
given below: 10. What is the magnitude (in radian) of the
interior angle of a regular pentagon?
A. 1 only
B. 2 only A. B.

C. Both 1 and 2
C. D.
D. Neither 1 nor 2

PAGE 229
www.byjusexamprep.com

11. What is the HCF of the polynomials 18. X, Y and Z travel from the same place with
uniform speeds 4 km/hr, 5 km/hr and 6
x6 – 3x4 + 3x2 – 1 and x3 + 3x2 + 3x + 1 ?
km/hr respectively. Y starts 2 hours after X.
A. (x + 1) m B. (x + 1)2 How long after Y must Z start in order that
they overtake X at the same instant?
C. x2 + 1 D. (x + 1)3
12. The HCF and LCM of two polynomials are 3x A. hours B. hours
+ 1 and 30x3 + 7x2 – 10x – 3 respectively. If
one polynomial is 6x2 + 5x + 1, then what is C. hours D. hours
the other polynomial?
A. 15x2 + 4x + 3 B. 15x2 + 4x – 3 19. 1 – x – xn + xn + 1, where n is a natural
number, is divisible by
C. 15x2 – 4x + 3 D. 15x2 – 4x – 3
A. (1 + x)2
13. If (p + 2 ) (2q – 1) = 2pq – 10 and (p – 2) (2q
– 1) = 2 pq – 10, then what is pq equal to? B. (1 – x)2

A. –10 B. –5 C. 1– 2x – x2

C. 5 D. 10 D. 1 + 2x – x2

14. What is the value of 20. A person sold an article for Rs. 75 which
cost him Rs. x. He finds that he realised x%
profit on his outlay. What is x equal to?
A. 20% B. 25%
A. 0 B. 1
C. 50% D. 100%
C. D. 21. If 172020 is divided by 18, then what is the
remainder?
15. What is the square root of 4x4 + 8x3 – 4x +
1? A. 1 B. 2

A. 2x2 – 2x – 1 B. 2x2 – x – 1 C. 16 D. 17

C. 2x2 + 2x + 1 D. 2x2 + 2x – 1 22. What is the value of

16. The sum of the digits of a two digit number + + + ... +


is 13 and the difference between the
number and that formed by reversing the
digits is 27. What is the product of the digits
of the number? A. 1 B. 5

A. 35 B. 40 C. 9 D. 10

C. 45 D. 54 23. If xm =
14
x x x then what is the value of
17. If then m?

which one of the following is correct? A. B.


A. x + y + z = 0 B. x – y – z = 0
C. D.
C. x + y – z = 0 D. x + 2y + 3z = 0

PAGE 230
www.byjusexamprep.com

24. The sum of all possible products taken two Select the correct answer using the code
at a time out of the numbers +1, +2, +3, + 4, given below:
+ 5 is
A. 1 and 3 only B. 1 and 2 only
A. 0 B. –30
C. 2 and 3 only D. 1, 2 and 3
C. –55 D. 55
30. If An = Pn + 1, where Pn is the product of the
25. A train of length 110 m is moving at a first n prime numbers, then consider the
uniform speed of 132 km/hr. The time following statements:
required to cross a bridge of length 165 m
1. An is always a composite number.
is
2. An + 2 is always an odd number.
A. 6.5 seconds B. 7 seconds
3. An + 1 is always an even number.
C. 7.5 seconds D. 8.5 seconds
Which of the above statements is/are
26. The simple interest on a certain sum is one-
correct?
fourth of the sum. If the number of years
and the rate of annual interest are A. 1 only B. 2 only
numerically equal, then the number of
C. 3 only D. 2 and 3 only
years is
31. x3 + x2 + 16 is exactly divisible by x, where x
A. 2.5 B. 3
is a positive integer. The number of all such
C. 3.5 D. 5 possible values of x is
27. A 60-page book has n lines per page. If the A. 3 B. 4
number of lines were reduced by 3 in each
C. 5 D. 6
page, the number of pages would have to
be increased by 10 to give the same writing 32. The number of set (a, b, c), where a, b, c are
space. What is the value of n? positive integers such that abc = 30, is
A. 18 B. 21 A. 30 B. 27
C. 24 D. 30 C. 9 D. 8
28. If x men working x hours per day can do x 33. If the roots of the quadratic equation x2 –
units of works in x days, then y men working 4x – log10 N = 0 are real, then what is the
y hours per day in y days would be able to minimum value of N?
do k units of work. What is the value of k?
A. 1 B.
A. x2y–3 B. x3y–2
C. y2x–3 D. y3x–2
C. D.
29. Let d(n) denote the number of positive
divisors of a positive integer n. Which of the 34. The number of different solutions of the
following are correct? equation x + y + z = 12, where each of x, y
and z is a positive integer, is
1. d(5) = d(11)
A. 53 B. 54
2. d(5).d(11) = d(55)
C. 55 D. 56
3. d(5) + d(11) = d(55)

PAGE 231
www.byjusexamprep.com

35. If I = a2 + b2 + c2, where a and b are A. 20% B. 25%


consecutive integers and c = ab, then I is
C. 30% D. 40%
A. An even number and it is not a square
42. A river 3 m deep and 40 m wide is flowing
of an integer
at the rate of 2 km/hr and falls into the sea.
B. An odd number and it is not a square of What is the amount of water in litres that
an integer will fall into the sea from this river in a
minute?
C. Square of an even integer
A. 40, 00, 000 litres
D. Square of an odd integer
B. 4, 00, 000 litres
36. If the number 23P62971335 is divisible by
the smallest, odd composite number, then C. 40, 000 litres
what is the value of P? D. 4, 000 litres
A. 4 B. 5 43. If a television set is sold at Rs. x, a loss of
C. 6 D. 7 28% would be incurred. If television is sold
at y Rs, a profit of 12% would be incurred.
37. What is the remainder when the sum 15 + What is the ratio of y to x?
25 + 35 + 45 + 55 is divided by 4?
A. 41 : 9 B. 31 : 9
A. 0 B. 1
C. 23 : 9 D. 14 : 9
C. 2 D. 3
44. By increasing the speed of his car by 15
38. What is the digit in the unit place of 399 ? km/hr, a person covers a distance of 300 km
by taking an hour less than before. What
A. 1 B. 3
was the original speed of the car?
C. 7 D. 9
A. 45 km/hr B. 50 km/hr
39. LCM of two numbers is 28 times their HCF.
C. 60 km/hr D. 75 km/hr
The sum of the HCF and the LCM is 1740. If
one of these numbers is 240, then what is 45. Three persons start a business with capitals
the other number? in the ratio . The first person
A. 420 B. 640 withdraws half his capital after 4 months.
C. 820 D. 1040 What is his share of profit if the business
fetches an annual profit of Rs. 96,800?
40. (xn – an) is divisible by (x – a), where x a,
for every A. Rs. 32, 000 B. Rs. 34, 500

A. Natural number n C. Rs. 36, 000 D. Rs. 36, 800

B. Even natural number n only 46. If x varies as y, then which of the following
is/are correct?
C. odd natural number n only
1. x2 + y2 varies as x2 – y2
D. Prime number n only
41. A shopkeeper sells his articles at their cost 2. varies inversely as y
price but uses a faulty balance which reads
1000 gm for 800 gm. What is the actual
profit percentage? 3. varies as

PAGE 232
www.byjusexamprep.com

Select the correct answer using the code A. 6 B. 6.5


given below:
C. 7 D. 7.5
A. 1 and 2 only B. 2 and 3 only
52. If the yield (in gm) of barley from 7 plots of
C. 3 only D. 1, 2 and 3 size one square yard each, were found to be
47. Ena was born 4 years after her parents 180, 191, 175, 111, 154, 141 and 176 then
marriage. Her mother is 3 years younger what is the median yield?
than her father and 24 years older than Ena, A. 111 gm B. 154 gm
who is 13 years old. At what age did Ena’s
father get married? C. 175 gm D. 176 gm
A. 25 years B. 24 years 53. Which one of the following measures of
central tendency will be used to determine
C. 23 years D. 22 years
the average size of the shoe sold in the
48. Mahesh is 60 years old. Ram is 5 years shop?
younger to Mahesh and 4 years eldest to
Raju. Babu is a younger brother of Raju and A. Arithmetic mean
he is 6 years younger. What is the age B. Geometric mean
difference between Mahesh and Babu?
C. Median
A. 18 years B. 15 years
D. Mode
C. 14 years D. 11 years
54. When the class intervals have equal width,
49. The number of items in a booklet is N. In the
the height of a rectangle in a histogram
first year there is an increase of x% in this
represents
number and in the subsequent year there is
a decrease of x%. At the end of the two A. Width of the class
years, what will be the number of items in
the booklet? B. Lower class limit

A. Less than N C. Upper class limit

B. Equal to N D. Frequency of the class


C. More than N 55. The ages of 7 family members are 2, 5, 12,
18, 38, 40 and 60 years respectively. After 5
D. It depends on the value of N
years a new member aged x years is added.
50. If ab + xy – xb = 0 and bc + yz – cy = 0, then If the mean age of the family now goes up
by 1.5 years, then what is the value of x?
what is equal to?
A. 1 B. 2
C. 3 D. 4
A. B.
56. The mean weight of 100 students in a class
C. 1 D. 0 is 46 kg. The mean weight of boys is 50 kg
and that of girls is 40 kg. The number of
51. Fifteen candidates appeared in an
boys exceeds the number of girls by
examination. The marks of the candidates
who passed in the examination are 9, 6, 7, A. 10 B. 15
8, 8, 9, 6, 5, 4 and 7. What is the median of
marks of all the fifteen candidates? C. 20 D. 25

PAGE 233
www.byjusexamprep.com

57. What is the algebraic sum of the deviations 64. Areas of two squares are in the ratio m2 : n4.
from the mean of a set of values 25, 65, 73, What is the ratio of their perimeters?
75, 83, 76, 17, 26, 7, 14?
A. m : n B. n : m
A. –1 B. 0 C. m : n2 D. m2 : n
C. 1 D. 2 65. AD is the median of the triangle ABC. If P is
58. The mean of five observations x, x + 2, x + any point on AD, then which one of the
4, x + 6, x + 8 is m. What is the mean of the following is correct?
first three observations? A. Area of triangle PAB is greater than the
A. m B. m – 1 area of triangle PAC

C. m – 2 D. m – 3 B. Area of triangle PAB is equal to area of


triangle PAC
59. What is the median of 2, 4, 6, …, 100?
C. Area of triangle PAB is one-fourth of the
A. 48 B. 49 area of triangle PAC
C. 50 D. 51 D. Area of triangle PAB is half of the area
60. The harmonic mean and the geometric of triangle PAC
mean of two numbers are 10 and 12 66. What is the area of a segment of a circle of
respectively. What is their arithmetic
radius r subtending an angle at the
means?
centre?
A. B.
A.
C. 11 D. 14.4
61. A circle is inscribed in a triangle ABC. It B.
touches the sides AB and AC at M and N
respectively. If O is the centre of the circle
C.
and ∠A = 70°, then what is ∠MON equal to?
A. 90° B. 100° D.
C. 110° D. 120°
67. ABC is a triangle right-angled at C. Let P be
62. The sum of the squares of sides of a right- any point on AC and Q be any point on BC.
angled triangle is 8,450 square units. What Which of the following statements is/are
is the length of its hypotenuse? correct?
A. 50 units B. 55 units 1. AQ2 + BP2 = AB2 + PQ2
C. 60 units D. 65 units 2. AB = 2PQ
63. A triangle and a parallelogram have equal Select the correct answer using the code
areas and equal bases. If the altitude of the given below :
triangle is k times the altitude of the
parallelogram, then what is the value of k? A. 1 only

A. 4 B. 2 B. 2 only
C. Both 1 and 2
C. 1 D. D. Neither 1 nor 2

PAGE 234
www.byjusexamprep.com

68. Four circular coins of equal radius are A. 90° B. 135°


placed with their centres coinciding with
C. 180° D. 270°
four vertices of a square. Each coin touches
two other coins. If the uncovered area of 74. ABC is an equilateral triangle. The side
the square is 42 cm2, then what is the radius BC is trisected at D such that BC = 3 BD.
What is the ratio of AD2 to AB2?
of each coin? (Assume )
A. 7 : 9 B. 1 : 3
A. 5 cm B. 7 cm C. 5 : 7 D. 1 : 2
C. 10 cm D. 14 cm
75. Consider the following statements :
69. The radii of the flat circular faces of a
1. The diagonals of a trapezium divide
bucket are x and 2x. If the height of the
each other proportionally.
bucket is 3x, what is the capacity of the
2. Any line drawn parallel to the parallel
bucket? (Assume )
sides of a trapezium divides the non-
parallel sides proportionally.
A. 11x3 B. 22x3
Which of the above statements is/are
C. 44x3 D. 55x3 correct?
70. If p, q, r, s and t represent length, breadth,
A. 1 only B. 2 only
height, surface area and volume of a cuboid
C. Both 1 and 2 D. Neither 1 nor 2
respectively, then what is equal
76. If H, C and V are respectively the height,
to? curved surface area and volume of a cone,
then 3πVH3 + 9V2 is equal to
A. B.
A. C2H2 B. 2 C2H2

C. D. C. 5 C2H2 D. 7 C2H2
77. How many solid lead balls each of diameter
71. In a quadrilateral ABCD, ∠B = 90° and AB + 2 2 mm can be made from a solid lead ball of
BC2 + CD2 – AD2 = 0, then what is ∠ACD radius 8 cm.
equal to? A. 512 B. 1024
A. 30° B. 60°
C. 256000 D. 512000
C. 90° D. 120°
78. The two sides of a tringle are 40 cm and
72. In a ΔABC, AC = 12 cm, AB = 16 cm and AD 41cm. If the perimeter of the triangle is 90
is the bisector of ΔA. If BD = 4 cm, then what cm, what is its area?
is DC equal to?
A. 90 cm2 B. 135 cm2
A. 2 cm B. 3 cm
C. 150 cm2 D. 180 cm2
C. 4 cm D. 5 cm
79. The diagonals of a rhombus differ by 2 units
73. ABCD is a cyclic quadrilateral. The bisectors and its perimeter exceeds the sum of the
of the angels A, B, C and D cut the circle at diagonals by 6 units. What is the area of the
P, Q, R and S respectively. What is ∠PQR + rhombus?
∠RSP equal to ?

PAGE 235
www.byjusexamprep.com

A. 48 square units 85. If sin θ + cos θ = 2 , then what is sin6θ +


B. 36 square units cos6θ + 6 sin2θ cos2θ equal to?

C. 24 square units A. B.
D. 12 square units
C. 1 D.
80. What is the area of a right-angled triangle,
if the radius of the circumcircle is 5 cm and 86. What is the least value of 9 sin2θ + 16 cos2θ?
altitude drawn to the hypotenuse is 4 cm?
A. 0 B. 9
A. 20 cm2 B. 18 cm2
C. 16 D. 25
C. 16 cm2 D. 10 cm2
87. If cos47° + sin 47° = k, then what is the value
81. The difference between two angles is 15° of cos2 47° – sin2 47°?
and sum of the angles in radian is . The
A. B.
bigger angle is k times the smaller angle.
What is k equal to? C. D.

A. B. 88. If cosec θ – sin θ = p3 and sec θ – cos θ = q3,


then what is the value of tan θ?
C. D. A. B.

82. Consider the following statements : C. pq D. p2q2


1. The equation 2 sin2θ – cos θ + 4 = 0 is 89. If 0 < α, β < 90° such that cos (α – β) = 1,
possible for all θ. then what is sin α – sin β + cos α – cos β
2. tan θ + cot θ cannot be less than 2, equal to?
where 0 < θ < . A. –1 B. 0

Which of the above statements is/are C. 1 D. 2


correct? 90. Consider the following statements :
A. 1 only B. 2 only 1. The value of cos 61° + sin 29° cannot
C. Both 1 and 2 D. Neither 1 nor 2 exceed 1

83. A road curve is to be laid out on a circle. 2. The value of tan 23° – cot 67° is less
What radius should be used if the track is to than 0.
change direction by 42° in distance of 44 m? Which of the above statements is\are
(Assume π = 22/7) correct?

A. 60 m B. 66 m A. 1 only B. 2 only

C. 75 m D. 80 m C. Bothe 1 and 2 D. Neither 1 nor 2

84. What is the maximum value of 3 sin θ – 4? 91. In a triangle, values of all the angles are
integers (in degree measure). Which one of
A. –4 B. –1 the following cannot be the proportion of
C. 0 D. 1 their measures?

PAGE 236
www.byjusexamprep.com

A. 1 : 2 : 3 B. 3 : 4 : 5 A. Hexagon B. Octagon
C. 5 : 6 : 7 D. 6 : 7 : 8 C. Nonagon D. Decagon
92. The length of a rectangle is increased by 96. A bicycle wheel makes 5000 revolutions in
10% and breadth is decreased by 10%. Then moving 11 km. What is the radius of the
the area of the new rectangle is wheel? (Assume π = 22/7)
A. Neither increased nor decreased A. 17.5 cm B. 35 cm
B. Increased by 1% C. 70 cm D. 140 cm
C. Decreased by 1% 97. The volumes of two cones are in the ratio
1:4 and their diameters are in the ratio 4 :
D. Decreased by 10%
5. What is the ratio of their heights?
93. The surface areas of two spheres are in the
A. 25 : 64 B. 16 : 25
ratio 1 : 4. What is the ratio of their
volumes? C. 9 : 16 D. 5 : 9
A. 1 : 16 B. 1 : 12 98. In a triangle ABC, if 2 ∠A = 3 ∠B = 6 ∠C, then
what is ∠A + ∠C equal to?
C. 1 : 10 D. 1 : 8
A. 90° B. 120°
94. The length, breadth and height of a brick
are 20 cm, 15 cm and 10 cm respectively. C. 135° D. 150°
The number of bricks required to construct
99. If the perimeter of a circle and a square
a wall with dimensions 45 m length, 0.15 m
are equal, then what is the ratio of the area
breadth and 3 m height is
of the circle to that of the square?
A. 12450
A. 1 : π B. 2 : π
B. 11250
C. 3 : π D. 4 : π
C. 6750
100. The lengths of the sides of a right-angled
D. None of the above triangle are consecutive even integers (in
cm). What is the product of these integers?
95. If the sum of all interior angles of a regular
polygon is twice the sum of all its exterior A. 60 B. 120
angles, then the polygon is
C. 360 D. 480

PAGE 237
www.byjusexamprep.com

ANSWERS KEY II 2020


ENGLISH

1 D 21 D 41 B 61 A 81 A 101 B
2 A 22 C 42 C 62 C 82 D 102 A
3 D 23 A 43 B 63 B 83 C 103 D
4 A 24 A 44 A 64 B 84 A 104 B
5 B 25 A 45 C 65 D 85 C 105 A
6 B 26 D 46 C 66 A 86 A 106 C
7 A 27 A 47 B 67 A 87 A 107 A
8 C 28 D 48 C 68 B 88 B 108 A
9 C 29 B 49 C 69 B 89 A 109 D
10 A 30 A 50 B 70 A 90 D 110 A
11 A 31 A 51 C 71 D 91 A 111 A
12 D 32 B 52 D 72 A 92 B 112 D
13 C 33 C 53 B 73 D 93 A 113 A
14 A 34 A 54 A 74 A 94 A 114 A
15 B 35 D 55 C 75 C 95 D 115 B
16 C 36 A 56 B 76 A 96 B 116 A
17 A 37 D 57 B 77 C 97 A 117 B
18 B 38 A 58 B 78 C 98 A 118 D
19 D 39 A 59 B 79 A 99 B 119 C
20 A 40 C 60 C 80 A 100 C 120 B

GENERAL KNOWLEDGE
1 B 21 A 41 B 61 A 81 D 101 A
2 D 22 B 42 D 62 B 82 B 102 D
3 D 23 D 43 D 63 C 83 A 103 B
4 A 24 B 44 B 64 D 84 B 104 A
5 C 25 B 45 A 65 C 85 B 105 C
6 A 26 B 46 A 66 C 86 B 106 D
7 A 27 C 47 D 67 A 87 D 107 A
8 D 28 D 48 C 68 B 88 C 108 A
9 B 29 D 49 D 69 C 89 D 109 A
10 D 30 B 50 B 70 C 90 C 110 C
11 A 31 D 51 C 71 D 91 A 111 D
12 C 32 A 52 C 72 D 92 A 112 B

PAGE 238
www.byjusexamprep.com

13 C 33 C 53 D 73 C 93 A 113 B
14 C 34 B 54 C 74 D 94 B 114 D
15 B 35 B 55 B 75 A 95 A 115 B
16 D 36 A 56 D 76 B 96 B 116 D
17 A 37 D 57 D 77 C 97 C 117 B
18 D 38 D 58 B 78 A 98 A 118 C
19 A 39 C 59 A 79 B 99 B 119 A
20 A 40 C 60 C 80 D 100 D 120 C

Elementary Mathematics
1 B 21 A 41 B 61 C 81 B
2 A 22 C 42 A 62 D 82 B
3 B 23 A 43 D 63 B 83 A
4 D 24 C 44 C 64 C 84 B
5 D 25 C 45 A 65 B 85 D
6 B 26 D 46 D 66 B 86 B
7 B 27 B 47 C 67 A 87 A
8 D 28 D 48 B 68 B 88 B
9 D 29 B 49 A 69 B 89 B
10 C 30 D 50 C 70 C 90 A
11 D 31 C 51 A 71 C 91 D
12 D 32 B 52 C 72 B 92 C
13 C 33 C 53 D 73 C 93 D
14 D 34 C 54 D 74 A 94 C
15 D 35 D 55 B 75 C 95 A
16 B 36 A 56 C 76 A 96 B
17 B 37 B 57 B 77 D 97 A
18 B 38 C 58 C 78 D 98 B
19 B 39 A 59 D 79 C 99 D
20 C 40 A 60 D 80 A 100 D

PAGE 239
www.byjusexamprep.com

PAGE 240
CDS II 2020
www.byjusexamprep.com

(Solutions)
ENGLISH
1. Ans. D.
Mountainous is the correct spelling.
Mountainous means having many mountains or something very large in size or amount.
2. Ans. A.
Etiquette is the correct spelling. Etiquettes are the rules of polite and correct behaviour.
3. Ans. D.
Option D - Curriculum is the correct spelling.
Curriculum - all the subjects that are taught in a school, college or university; the contents of a
particular course of study
4. Ans. A.
Option A - Magnificent is the correct spelling.
5. Ans. B.
Option B - Felicitation is the correct spelling. Felicitation means words expressing praise for an
achievement or good wishes on a special occasion.
6. Ans. B.
Option B - Twelfth is the correct spelling.
7. Ans. A.
Option A - Snobbery is the correct spelling. Snobbery refers to the behaviour or attitudes typical of
people who think they are better than other people in society, for example because they have more
money, better education, etc.
8. Ans. C.
Option C - Neurosis is the correct spelling.
9. Ans. C.
Option C - Diphtheria is the correct spelling.
Diphtheria means Diphtheria is a serious infection caused by strains of bacteria called
Corynebacterium diphtheriae that make toxin.

PAGE 241
www.byjusexamprep.com

10. Ans. A.
Meagre is the correct spelling.
11. Ans. A.
'Studying' is the correct usage as it matches the tense of the sentence.
12. Ans. D.
'Want' is the correct usage.
13. Ans. C.
'These' is the correct usage.
14. Ans. A.
'Must' is the correct usage.
15. Ans. B.
'Were' is the correct usage.
16. Ans. C.
'That' is the correct usage.
17. Ans. A.
'Changing' is the correct usage.
18. Ans. B.
'All' is the correct usage.
19. Ans. D.
'person' is the correct usage.
20. Ans. A.
'From' is the correct usage.
21. Ans. D.
'which' is the correct usage.
22. Ans. C.
'driven' is the correct usage.
23. Ans. A.
'contain' is the correct usage.
24. Ans. A.
'how' is the correct usage.
25. Ans. A.
'might' is the correct usage.

PAGE 242
www.byjusexamprep.com

26. Ans. D.
'more' is the correct usage.
27. Ans. A.
'process' is the correct usage.
28. Ans. D.
'mean' is the correct usage.
29. Ans. B.
'described' is the correct usage.
30. Ans. A.
'a' is the correct usage.
31. Ans. A.
Option A - a contestation of the then existing dominant western practices is the correct answer.
32. Ans. B.
The meaning of the sentence is explained in option B - Women were not given any right equation to
men.
33. Ans. C.
Option C - Women were assumed to be less intelligent that men
Is the correct answer according to the given passage.
34. Ans. A.
Option A - participation of women in social relations, politics, law medicine, the arts, popular and
academic knowledge is the correct answer as per the given passage.
35. Ans. D.
Contestation means the action or process of disputing or arguing.
Trivial means of very little importance or value.
Discredited means brought into disrepute; having lost respect or credibility.
Analogy means a comparison between one thing and another, typically for the purpose of
explanation or clarification.
36. Ans. A.
The extraordinary diversity of habitats makes it amazing. Hence, option A is the correct answer.
37. Ans. D.
The author says 'ecological conflict and cooperation' because living organisms sometimes fight and
sometimes live tougher.

PAGE 243
www.byjusexamprep.com

38. Ans. A.
Statement A- Meaning of life could be reflected as to what living is as opposed to the non-living and
what the purpose of life is.
39. Ans. A.
Distinctive characteristics exhibited by organisms indicate that they are living organisms.
40. Ans. C.
Common means the same in a lot of places or for a lot of people.
Characteristics means a feature or quality belonging typically to a person, place, or thing and serving
to identify them.
Distinctive means characteristic of one person or thing, and so serving to distinguish it from others.
General means affecting or concerning all or most people or things; widespread.
41. Ans. B.
The error is in part B of the given sentence, the correct part should be 'one of the most revered
members'
There should be a plural form.
42. Ans. C.
The error is in part C of the given sentence, It should be 'in' joining the group instead of 'to'.
43. Ans. B.
The error is in part B of the given sentence.
Where there is a will, there is a way, is the correct sentence.
44. Ans. A.
The error is in part A of the given sentence. 'movements' should be in singular form.
45. Ans. C.
The error is in part C of the given sentence. 'Pursue' should be used instead of pursues.
46. Ans. C.
The error is in part C of the given sentence. 'Aspect' should be used instead of aspects.
47. Ans. B.
The error is in part B of the given sentence. The correct sentence should be 'Learning many languages
promotes linguistic, cultural and harmony among people speaking different languages'
48. Ans. C.
The error is in part C of the given sentence. Preposition 'At' should be used instead of 'on'.
49. Ans. C.
The error is in part C of the given sentence. 'to' must be replaced with 'of her economy'.
Hence, option C is the correct answer.

PAGE 244
www.byjusexamprep.com

50. Ans. B.
The error is in part B of the given sentence. 'Countries' should be I singular form.
51. Ans. C.
'under' is a preposition.
52. Ans. D.
'statue' is a noun.
53. Ans. B.
'his' is a possessive adjective because it indicates the possession of the noun to a person.
54. Ans. A.
'It' is a pronoun.
55. Ans. C.
'It' is a personal pronoun. When we have a certain structure in a sentence, we can use "it" as an
impersonal pronoun.
56. Ans. B.
'very' is an adverb as it qualifying the adjective 'beautiful'.
57. Ans. B.
'This' is an adjective as it is attached with a noun 'boy'.
58. Ans. B.
'myself' is a pronoun. (reflexive pronoun)
59. Ans. B.
'fought' is a transitive verb as it is used with an object 'wasps'.
60. Ans. C.
'hardly' is an adverb.
61. Ans. A.
The most appropriate synonym for emboldened is encouraged which means to give support,
confidence, or hope to (someone).
Disgruntled means angry or dissatisfied.
Succeeded means to achieve the desired aim or result.
Failed means unsuccessful.
62. Ans. C.
The most appropriate synonym for indigenous is home-grown which means grown or produced in
one's own garden or country.
Homogenous means consisting of parts or elements that are all the same.

PAGE 245
www.byjusexamprep.com

Classical means representing an exemplary standard within a traditional and long-established form
or style.
Non-native language means not born or raised in the place where a particular language is spoken
63. Ans. B.
The most appropriate synonym for pleonexia is greed to accumulate more and more wealth.
64. Ans. B.
The most appropriate synonym for implicate is incriminate which means to make someone appear
guilty of a crime or wrongdoing.
Reward means a thing given in recognition of service, effort, or achievement.
Encourage means to give support, confidence, or hope to someone.
Incite means to encourage or stir up a violent or unlawful behavior.
65. Ans. D.
The most appropriate synonym for statutory is legal, which means relating to the law.
Legislature means the legislative body of a country or state.
Unlawful means not conforming to, permitted by, or recognized by law or rules.
Government means the group of people with the authority to govern a country or state.
66. Ans. A.
The most appropriate synonym for dissident is rebellious which means showing a desire to resist
authority, control, or convention.
Supportive means providing encouragement or emotional help.
Conformist means a person who conforms to accepted behavior or established practices.
Legal means relating to the law.
67. Ans. A.
The most appropriate synonym for Advocacy is promotion which means a raising or a state of being
raised to a higher rank or position.
Opposition means resistance or dissent, expressed in action or argument.
Critique a detailed analysis and assessment of something, especially a literary, philosophical, or
political theory.
Liking means a feeling of regard or fondness.
68. Ans. B.
The most appropriate synonym for high mindedness is strong principles which means that someone
who believes in a principle very strongly.
Toughness means the state of being strong enough to withstand adverse conditions or rough
handling.

PAGE 246
www.byjusexamprep.com

Anger means a strong feeling of annoyance, displeasure, or hostility.


Whims means a sudden desire or change of mind, especially one that is unusual or unexplained.
69. Ans. B.
The most appropriate synonym for tendency is predisposition which means a liability or tendency to
suffer from a particular condition, hold a particular attitude, or act in a particular way.
Position means a place where someone or something is located or has been put.
Thinking means the process of considering or reasoning about something.
Idea means a thought or suggestion as to a possible course of action.
70. Ans. A.
The most appropriate synonym for patronized is supported which means bear all or part of the
weight of; hold up.
Respected means to admire someone or something deeply, as a result of their abilities, qualities, or
achievements.
Opposed means to be anxious to prevent or put an end to.
Scolded means to remonstrate with or rebuke someone angrily.
71. Ans. D.
The most appropriate antonym for archaic is modern which means relating to the present or recent
times as opposed to the remote past.
Antiquated means old-fashioned or outdated.
Outmoded means old-fashioned.
Beyond the times means for a period of time that continues after (a particular date, age, etc.)
72. Ans. A.
The most appropriate antonym for diffuse is concentrate which means focus all one's attention on a
particular object or activity.
Scatter means to throw in various random directions.
Disperse means to distribute or spread over a wide area.
Strew means to scatter or spread things untidily over a surface or area.
73. Ans. D.
The most appropriate antonym for Unrest is calm, which means not showing or feeling nervousness,
anger, or other strong emotions.
Turbulence means violent or unsteady movement of air or water, or of some other fluid.
Unease means mental or spiritual discomfort
Apprehension means anxiety or fear that something bad or unpleasant will happen.

PAGE 247
www.byjusexamprep.com

74. Ans. A.
The most appropriate antonym for tranquility is uproar which means a loud and impassioned noise
or disturbance.
Calm means not showing or feeling nervousness, anger, or other strong emotions.
Serenity means a state of being calm, peaceful and untroubled.
Sound means a noise or a voice.
75. Ans. C.
The most appropriate antonym for Barring is including which means containing as part of the whole
being considered.
Excepting means except for; apart from.
Without means in the absence of.
Excluding means not taking someone or something into account; except.
76. Ans. A.
The most appropriate antonym for extempore is prepared, which means made ready for use.
Ready-made means made to a standard size or specification rather than to order.
Unrehearsed means not practiced before a performance.
Ad lib means without previous preparation.
77. Ans. C.
The most appropriate antonym for ensuing is retrospective which means a look back at events that
took place.
Subsequent means coming after something in time.
Consequent means following as a result or effect.
En-suite means immediately adjoining a bedroom and forming part of the same set of rooms.
78. Ans. C.
The most appropriate antonym for expunged is accepted which means generally believed or
recognized to be valid or correct.
got rid of means take action so as to be free of a troublesome or unwanted person or thing.
Part of means some but not all of a thing
Rejected means to dismiss as inadequate, unacceptable, or faulty.
79. Ans. A.
The most appropriate antonym for dissuaded is persuaded, which means to induce (someone) to do
something through reasoning or argument.
Discouraged means having lost confidence or enthusiasm; disheartened.

PAGE 248
www.byjusexamprep.com

Advised against means to make a recommendation to one to not pursue a particular plan or action.
Deter means to discourage someone from doing something by instilling doubt or fear of the
consequences.
80. Ans. A.
The most appropriate antonym for confidants is opponents which means someone who competes
with or opposes another in a contest, game, or argument.
Intimate means closely acquainted; familiar.
Close friend means someone you rely on and can trust.
Colleague means someone you work with or someone who's in the same profession as you.
81. Ans. A.
SRQP - is the most appropriate sequence for the given jumbled sentences.
Hence, option A is the correct answer.
82. Ans. D.
QPRS- The non-debt receipts comprise of tax revenue, non-tax revenue, recovery of loans and
disinvestment receipts. Debt receipts mostly consist of market borrowing and other liabilities which
the government is obliged to repay in the future. The outcomes as reflected in the Provisional Actual
figures is lower than the budget estimate owing reduction in the net tax revenue. The Budget 2018-
19 targeted significantly high growth in non-debt receipts of the Central Government, which was
driven by robust growth.
83. Ans. C.
RSPQ- Within this discipline, molecular biology and DNA studies have been used to understand
hominid evolution. Hominid evolution answers the questions about what ancient people looked like,
and to plot patterns of migration. Bones provide a great information. The distribution of faunal
remains (animal bones) at a site can indicate which areas were used for butchering, cooking, eating,
bone tool making and refuse dumping.
84. Ans. A.
PSRQ- They help to maintain the balance of biological activities in the body. The role of insulin in
keeping the blood glucose level within the narrow limit is an example of this function. On the other
hand, hormone glucagon tends to increase the glucose level in the blood. Insulin is released in
response to the rapid rise in blood glucose level.
85. Ans. C.
QRSP- For example, burrows and earthworms make burrows and worm casts. This act of earthworms
affects the soil, and therefore the plants growing in it. Rabbit's fleas carry the virus which causes
myxomatosis, so they can affect the size of the rabbit population. This can also affect the population
of fox, if foxes depend on rabbits for food.
86. Ans. A.
QRSP- The problems we see are because we have undermined these links over decades. First, rain
and snowfall are the only sources of water – about 99%. In the four months of monsoon, there are
about 30-35 downpours and the challenge is to hold this water in systems that can last us over 365
days. Thick forests make for excellent catchments.

PAGE 249
www.byjusexamprep.com

87. Ans. A.
RSQP- They disagree about how they should live. Who should get what? How should power and other
resource be distributed? Should society be based on cooperation or conflict? And so on. They also
disagree about how such matters should be resolved, how collective decision should be made and
who should have a say. For Aristotle politics is an attempt to create a good society because politics
is, above all, a social activity.
88. Ans. B.
RSPQ- It also uses up energy and helps to prevent large amounts of fat building up in the body.
Exercise can increase your fitness in three ways. It can improve your strength; make your body more
flexible and less likely to suffer from sprain. It can also improve your endurance.
89. Ans. A.
QSRP- Due to the increase in kinetic energy, the particles start vibrating with greater speed. The
energy supplied by heat overcomes the forces of attraction between the particles. The particles leave
their fixed positions and start moving more freely. A stage is reached when the solid melts and is
converted to a liquid.
90. Ans. D.
SPRQ- As you read this sentence, perhaps sitting in a comfortable chair in your study, you would
probably consider yourself at rest. But you are really not, because the Milky Way galaxy, of which
you are a part, is moving through space at 2.1 million kilometers an hour. And that is without taking
into account the effects of earth's rotation on its own axis, its orbiting around the sun and sun's
journey around the Milky Way. So, in roughly twenty second that it would have taken you to read
this paragraph, you have already moved thousands of kilometers.
91. Ans. A.
'had found' is the correct usage as it matches with the tense of the sentence.
92. Ans. B.
'most powerful members' is the correct usage as ten is plural and most is used when referring to
greatest in amount or degree
93. Ans. A.
'came across' is the correct usage because the sentence is in the past tense.
94. Ans. A.
'broke' is the correct usage because broke means having completely run out of money.
95. Ans. D.
Option D - 'asked' is the correct usage.
96. Ans. B.
'bade' is the correct usage as the sentence is talking about the past and the past tense of bade is bid.
97. Ans. A.
'Extant' is the correct usage. Extant means still in existence; surviving.

PAGE 250
www.byjusexamprep.com

98. Ans. A.
Option A- 'formal' is the correct usage.
99. Ans. B.
Option B 'glitters' is the correct usage.
100. Ans. C.
Option C 'a has been' is the correct usage.
101. Ans. B.
The correct and complete sentence is -Evolutionary Biology is the study of history of life forms on
earth.
102. Ans. A.
The correct and complete sentence is - The origin of life is considered a unique event in the history
of universe.
103. Ans. D.
The correct and complete sentence is - How we manage productive resources is critical to strategic
growth and competitiveness.
104. Ans. B.
The correct and complete sentence is - Operations strategy in service firms is generally inseparable
from the corporate strategy.
105. Ans. A.
The correct and complete sentence is - Even as more and more Indians are travelling, a recent survey
has revealed that they are worried about their safety.
106. Ans. C.
The correct and complete sentence is - Stories can exercise the immigration of children because they
tell more than the stories.
107. Ans. A.
The correct and complete sentence is - History is considered as a record of the achievements,
experiments and suffering of humans.
108. Ans. A.
The correct and complete sentence is - It appears that all that can be invented has been invented.
109. Ans. D.
The correct and complete sentence is - Indian cinema stands as a testimony of Indian social, political
and cultural life during the last century.
110. Ans. A.
The correct and complete sentence is - An exploration into truth should be the beginning of all
searches for knowledge and experiments of life.

PAGE 251
www.byjusexamprep.com

111. Ans. A.
Idiom Get the jitters means - feeling anxious.
112. Ans. D.
Idiom' French leave' means absent from work without asking for permission.
113. Ans. A.
Idiom 'Take a stand' means to publicly express an opinion about something.
114. Ans. A.
Idiom' Cut and run' means to avoid a difficult situation by leaving suddenly.
115. Ans. B.
Idiom 'Cut the cord' means to stop needing someone else to look after you and start acting
independently.
116. Ans. A.
Idiom' Cupboard love' means loving someone to get something from the person.
117. Ans. B.
Idiom 'Around the corner' means an event or thing which is going to happen soon.
118. Ans. D.
Idiom' With Heavy Heart' means with pain and regret.
119. Ans. C.
'Cost a bomb' means to be very expensive.
120. Ans. B.
Idiom' Roll your sleeves up' means to prepare for hard work.

PAGE 252
www.byjusexamprep.com

General Knowledge
1. Ans. B.
On heating, lead nitrate is converted into yellow color lead oxide, brown color nitrogen dioxide, and
colorless gas of oxygen. The reaction will be

This reaction is an example of a decomposition reaction.


Hence, Option B is correct.
2. Ans. D.
Valency is the combining capacity of the atom. The atomic number of phosphorus is 15. So, the
electronic configuration will be 2,8,5. To complete the outer orbit, the atom has to gain 3 electrons
or lose 5 electrons. Therefore, phosphorus has both valency 3 and 5.
Hence, Option D is correct.
3. Ans. D.
Silver does not react with oxygen under normal circumstances, because it is less reactive and comes
downwards in the reactivity series. Silver reacts with hydrogen sulfide in the air and forms silver
sulfide which is responsible for black coating over the silver.
Hence, Option D is correct.
4. Ans. A.
Liquid nitrogen is used to keep samples at –196°C which is considered the gold standard for
biosamples preservation. It is used to store the biological tissues at a temperature below the glass
transition point of water which is about –135° C, stops virtually all biological activity, and minimizes
the sample degradation over time.
Hence, Option A is correct.
5. Ans. C.
The most stable radioactive isotype of hydrogen is Tritium with a half-life of 12.32 years. It is a
hydrogen isotope that consists of one proton, two neutrons, and one electron.
Hence, Option C is correct.
6. Ans. A.
Most of the bacteria generates a cell wall which is composed partly of a macromolecule called
peptidoglycan, it is made up of amino sugars and short peptides. Penicillin is one of those antibiotics
which prevents the final cross-linkage step with the coordination of this macromolecule which results
in the burst wall and kills the bacteria. It is safe for the host.
Hence, Option A is correct.

PAGE 253
www.byjusexamprep.com

7. Ans. A.
A human being has 23 pairs of chromosomes in which one pair of X and Y chromosomes determine
the sex of the human being. If there is one X chromosome from mother and one X chromosome from
father, then two X chromosomes (XX) makes a girl whereas X chromosome from mother and Y
chromosome from father then these chromosomes (XY) makes a boy.
Hence, Option A is correct.
8. Ans. D.
Mature sclerenchyma cells are generally having dead cells which are made up of heavily thickened
secondary walls containing lignin. The cells are rigid and are generally found in nongrowing regions
of the plant, such as the bark or mature stems.
Hence, Option D is correct.
9. Ans. B.
Phloem is the living tissue in vascular plants that transports the photosynthetic products which are
the soluble organic compounds obtained during photosynthesis, mainly sugar sucrose to the parts of
the plants where it is needed. This process is called translocation.
Hence, Option B is correct.
10. Ans. D.
Solar energy is a non-conventional source of energy because it does not produce any pollution which
helps in reducing the greenhouse effect. Solar energy is the most readily available source of energy
that does not belong to anybody as it is free from nature.
Hence, Option D is correct.
11. Ans. A.
A point light source is single and a small one that gives off the light in all directions, such as a lit
candle. It is a source of light that has no screen or lampshade to block the direction in which the light
can travel. As light rays travel from the point source and strike an opaque object, the light rays do
not travel through an opaque object. There is an area behind the object where the light rays do not
reach, and this is the place where the dark shadow forms. If the distance between the object and the
light source changes, the scattering of the light rays’ changes.
Hence, Option A is correct.
12. Ans. C.
Light and sound are very different. Sound is a mechanical disturbance that travels through air or
another medium. Sound needs a medium to travel through a medium that determines its speed. A
thunderstorm is a loud noise that follows a flash of lightning. Lightning can be seen before the sound
of thunder is heard as light travels faster than sound. The speed of sound in air is 300m/s while Light
travels at 186,282 meters per second. This indicates that the reason why we see lightning first in a
thunderstorm is that the visible light waves formed by the plasma in the lightning travel to our eyes
much faster than the crack of thunder sound reach to our ears.
Hence, Option C is correct.

PAGE 254
www.byjusexamprep.com

13. Ans. C.

In this case, the angle of incidence is and

=0

=0

In this case, the angle of incidence is 0°. This diagram shows that if the light ray is incident at 0°, then
the angle of refraction is also 0°. There is no change in the direction of the light ray, thus, the speed
of the light will change as it moves into the new medium.
Hence, Option C is correct.
14. Ans. C.
Given,
Object distance, u = 40cm
Focal length, f = 25cm
From mirror formula

= 200cm

Hence, the image will be real, inverted, and magnified.


Hence, Option C is correct.

PAGE 255
www.byjusexamprep.com

15. Ans. B.
Lens with power +2.5 D will bend the light rays through the largest angle because a positive sign
shows a convex lens, a convex lens concentrates the light rays that are incident on it, so it is called a
converging lens. The bending of light rays is maximal at the top and bottom of the convex lens
because the top and bottom prisms have the greatest angle between the refracting surfaces.
Hence, Option B is correct.
16. Ans. D.
Silver articles turn black when kept in the open for a longer time due to the formation of (Ag2S). Silver
reacts with hydrogen sulfide in the air and forms silver sulfide (Ag2S) which is responsible for black
coating over the silver.
Hence, Option D is correct.
17. Ans. A.
Zinc oxide is an amphoteric oxide that shows both properties of acidic and basic behavior. It dissolves
in most of the acids but insoluble in water. An amphoteric oxide reacts with both acid and base to
form salt and water.
Hence, Option A is correct.
18. Ans. D.
The only liquid elements at standard temperature (298 K) and pressure (1 atm.) are bromine (Br) and
mercury (Hg). Bromine (Br), a reddish-brown liquid which melts at 265.9 K while Mercury (Hg), a toxic
metal, melts at 234.32 K.
Hence, Option D is correct.
19. Ans. A.
John Dalton proposed the tenets of his atomic theory that chemical elements are made up of atoms.
The atoms of an element are identical in mass while different elements have different masses. He
proposed that atoms can not be created nor be destroyed. Dalton proposed the symbols of those
elements, but these symbols are not used nowadays. He used them to represent elements and in
combination compounds.
Hence, Option A is correct.
20. Ans. A.
Ammonium chloride is isolated from a mixture of sodium chloride and ammonium chloride by the
process of sublimation. Ammonium chloride can be sublimed whereas sodium chloride cannot be
sublimed. When a mixture of sodium chloride and ammonium chloride is heated, ammonium
chloride changes its state from solid to vapor. The vapors of ammonium chloride are condensed on
the cold surface. Sodium chloride is left behind.
Hence, Option A is correct.

PAGE 256
www.byjusexamprep.com

21. Ans. A.
Helicobacter pylori is a gram-negative bacillus that is found in the gastric epithelial cells. These
bacteria are responsible for 90% of duodenal ulcers and 70%-90% of gastric ulcers.
Hence, Option A is correct.
22. Ans. B.
Wings of birds and bats are considered analogous structures because they have similar functions and
similar structures but different origins as they developed independently as an adaptation for flying.
Hence, Option B is correct.
23. Ans. D.
Pearls are harvested from Oyster through a harvesting process. Only Oysters have this quality to
produce pearls with good shapes, lusters and qualities will be grafted a subsequent time. It takes
much time to harvest a cultured pearl from the birth of oyster larvae and this is mainly done in
winters.
Hence, Option D is correct.
24. Ans. B.
Taxonomy is that branch of biology which manages the identification and naming of organisms. A
genus is a taxonomic category that deals with the biological classification of living and fossil
organisms, as well as viruses in biology. In the ladder of biological classification, the genus is above
species and below family category. In binomial nomenclature, the genus name forms the first part of
the binomial species name for each species within the genus taxonomic category. All the individuals
of a particular organism, such as rose plants, belong to a taxonomic category called genus.
Hence, Option B is correct.
25. Ans. B.
Algae is a member of a group of predominantly aquatic photosynthetic organisms that belongs to
the kingdom Protista. Algae is having many types of life cycles, and they are various in size from
microscopic Micro monas species to giant kelps that reach around 60 meters (200 feet) in length.
Hence, Option B is correct.
26. Ans. B.
In physics optics, the refractive index of a material is a unitless number that describes how faster the
light travels through the material. It is defined as

n=

where,
c = speed of light in vacuum
v = phase velocity of light in the medium
So, If the speed of light in air is represented by c and the speed in a medium is v, then the refractive
index of the medium can be calculated using the formula.
Hence, Option B is correct.

PAGE 257
www.byjusexamprep.com

27. Ans. C.
Ultraviolet light is a form of electromagnetic radiation whose wavelength ranges from 10 nm to 400
nm, which is shorter than that of visible light, but longer than X-rays. Two types of waves are there
i.e., transverse waves and longitudinal waves. Ultraviolet light is an example of transverse waves. If
the particles are displaced perpendicular to the direction the wave travels, then this is defined as
transverse waves. If the particles are displaced parallel to the direction the wave travel is defined as
longitude waves.
Hence, Option C is correct.
28. Ans. D.
The neutron is a subatomic particle, the symbol is n or n°, which has a zero charge and a mass slightly
greater than that of a proton. Protons and neutrons constitute the nuclei of atoms. Protons and
neutrons make up the nucleus in the center of the atom, but neutrons do not revolve around the
atomic nuclei.
Hence, Option D is correct.
29. Ans. D.
The gravitational force between the two objects is inversely proportional to the square of the
distance between them.

The gravitational force will be if the distance between these two objects is doubled.

That is, .

If the distance between two objects is increased by two times, the gravitational force between them

will be .

Hence, Option D is correct.


30. Ans. B.
As we know, Momentum is directly proportional to velocity. So, If the momentum of that object is
doubled, but its mass does not increase then its velocity will be doubled. If the velocity will be
doubled, then the kinetic energy increases by four times. If the linear momentum of a moving object
gets doubled due to the application of a force, then its kinetic energy will be increased by four times.
Hence, Option B is correct.
31. Ans. D.
Pragyan, which signifies 'wisdom' in Sanskrit was a rover and the third component of the
Chandrayaan-2 mission. Pragyan was a robotic vehicle that would traverse the lunar surface on the
six wheels that were painted in the colours of the tiranga.
Hence, Option D is correct.

PAGE 258
www.byjusexamprep.com

32. Ans. A.
The Mineral contents of samples from the Lonar crater-cum-lake in the Buldhana district,
Maharashtra, which was established after a meteorite crash around 50,000 years ago, are similar to
the moon rocks obtained from the first and the third manned moon missions, as per the study led
by the IIT (Indian Institute of Technology), Bombay.
Hence, Option A is correct.
33. Ans. C.
The Ministry of Human Resource Development has approved the 'Study in India' programme with,
inter-alia, the following objectives to improve India's soft power with a focus on the neighbouring
countries and utilize it as a tool in diplomacy.
Hence, Option C is correct.
34. Ans. B.
The MSDE (Ministry of Skill Development & Entrepreneurship) has launched 'Aatamanirbhar Skilled
Employee Employer Mapping (ASEEM)' portal to help skilled people find the sustainable livelihood
opportunities. ASEEM portal will provide employers a platform to assess the availability of the skilled
workforce and formulate their hiring plans.
Hence, Option B is correct.
35. Ans. B.
In the Odisha's Balasore district, a farmer, Basudev Mahapatra, spotted a turtle which is yellow in
colour and according to the experts, it is the product of albinism. The turtle is known as the 'Indian
flap shell turtle'. The turtle is commonly found in India, Pakistan, Sri Lanka, Bangladesh, Nepal and
Myanmar. It is omnivorous & its diet consists of frogs, snails and even some aquatic vegetation.
Hence, Option B is correct.
36. Ans. A.
It was M.S. Aney who moved the motion of Secret sitting session of the Assembly in the year 1942.
Dr. Madhav Shrihari Aney popularly referred to as Loknayak Bapuji Aney/Bapuji Aney, was an ardent
educationist, statesman, freedom fighter, a modern Sanskrit poet and a politician. He was even
designated with the title of ‘Loknayak Bapuji’, which means ‘The People's Leader and Respected
Father’. He was one among the founder of the Congress Nationalist Party.
Hence, Option A is correct.
37. Ans. D.
• In the discharge of his constitutional & statutory responsibilities, the Speaker of the Lok Sabha is
assisted by the Secretary-General, Lok Sabha.
• The Secretary-General remains in the office till his/her retirement at the age of 60. He or she is
answerable only to the Speaker; his/her actions can’t be discussed or criticised in or outside the
Lok Sabha.
• On the behalf of the President of India, he or she summons members to attend the session of
Parliament and authenticates bills in the absence of the Speaker.
Hence, Option D is correct.

PAGE 259
www.byjusexamprep.com

38. Ans. D.
Shri Ram Bajpai founded the Seva Samiti Boy Scouts Association in the year 1914 at Bombay on the
lines of the world-wide Baden-Powell Organisations, which at that time banned Indians from joining
it. Though later Baden-Powell, after a private visit to India, lifted the colour bar, Bajpai’s organisation
continued its separate existence, for it had the aim of bringing about the complete Indianisation of
the Boy Scout movement in India.
Hence, Option D is correct.
39. Ans. C.
1918 – First Registered Organized Trade Union – Madras Labour Union led by B P Wadia and V.
Kalyanasundaram Mudaliar, in the year 1918. The strike was started by the workers of the
Buckingham and Carnatic Mills in Madras, against the managing company, Binny & Co.
Hence, Option C is correct.
40. Ans. C.
Narayan Malhar Joshi was an Indian trade union leader and the follower of Gopal Krishna Gokhale.
Joshi got involved in labour issues and initiated the All India Trade Union Congress in the year 1920
along with the Lala Lajpat Rai.
Hence, Option C is correct.
41. Ans. B.
Subhash Chandra Bose formed an alliance with the Muslim League to destroy the Holwell Monument
in Calcutta during 1939-40. After the failure of Bose-Mahasabha alliance, Bose entitled his alliance
with the Muslim League of being a larger vision and strategic purpose.
Hence, Option B is correct.
42. Ans. D.
A remote and unknown town, Qadian in Gurdaspur emerged as a centre of religious learning in the
year 1889, when Mirza Ghulam Ahmad established the Ahmadiyya Muslim Community, and in the
year 1891, it became the venue for the annual gatherings of the communities.
Hence, Option D is correct.
43. Ans. D.
Sarojini Naidu, Annie Besant, Hirabai Tata, Mithan Tata went to London in the year 1919 to give
evidence before the Joint Parliamentary Committee on the Government of India Bill and received a
warm support from the British Women's organisations like the Women's freedom League and the
Women's International League36.
Hence, Option D is correct.
44. Ans. B.
Among the given cities, Singapore is closest to the equator.
Some other cities which lie closest to the equator:
• Quito, Ecuador
• Kampala, Uganda

PAGE 260
www.byjusexamprep.com

• Pekanbaru, Indonesia
• Padang, Indonesia
• Libreville, Gabon
• Pontianak, Indonesia
Hence, Option B is correct.
45. Ans. A.
Inceptisols:
General characteristics: These are usually the weakly developed young soil though they are more
developed than entisols.
Area (in Thousand Hectares): 130372.9
Percentage of the total area of India: 39.74
Hence, Option A is correct.
46. Ans. A.
The tropical Savannah climate is even called the tropical wet and dry climate. It is the climate
experienced in the savannah or tropical grassland regions of the world. These places are situated
near the equator, and they lie between the Southern and the Northern Tropics.
It shares some similar characteristics with that of the tropical monsoon climate, but it receives less
annual rainfall as compared to the tropical monsoon climate. In the Koppen classification, the climate
is designated as Aw or As.
Hence, Option A is correct.
47. Ans. D.
The Isotherm Line, which divides the India North-South into almost two equal parts in the month of
January is 20°C.
It can even be done for the other parameters like barometric pressure (isobars), dew point
temperature (isodrosotherms), geopotential height (isohypses), wind speed (isotachs), and salinity
(isohalines). Isotherms are always smooth, labelled with the values, and mostly parallel to each
other.
Hence, Option D is correct.
48. Ans. C.
The Decadal growth rate of population in terms of percentage was highest in India in 1971.
The population growth rate peaked during the decade 1971-81, perhaps in the year’s 1972-73
(based on the Sample Registration Scheme data). The avg. annual exponential growth rate declined
marginally to 2.11 per cent (4.5%) after having remained at a plateau for the previous two decades
of 1961-71 & 1971-81. At this point in time, the fertility and mortality trends indicate that the India
will reach the replacement level fertility (Net Reproductive Rate of Unity) by the years 2010-2015.
Hence, Option C is correct.

PAGE 261
www.byjusexamprep.com

49. Ans. D.
Timber is a type of wood that has been processed into beams and planks. It is also known as ‘lumber’
in the US and Canada. Any wood capable of yielding a minimum dimensional size can be termed as
timber or lumber. It is a stage in the process of wood production. The Timber vegetation is generally
found in Subtropical, Temperate and Alpine region.
Hence, Option D is correct.
50. Ans. B.
The place in India receiving the lowest rainfall is Leh. The avg. annual rainfall in the region is less
than 50 cm. The cities like Jaisalmer in Rajasthan and Leh in Ladakh receives the least rainfall.
Hence, Option B is correct.
51. Ans. C.
As per the Budget Estimates of the expenditure on major subsidies during 2019-20, the maximum
expenditure was likely to be on food subsidy.
• Fertiliser subsidy: Expenditure on fertiliser subsidy is estimated at Rs 79,996 crore in 2019-20.
This is estimated to increase by Rs 9,910 crore (1%) over the revised estimate of 2018-19.
Allocation to the subsidy in 2019-20 budget is Rs 5,010 crore higher than the allocation made in
2019-20 interim budget.
Hence, Option C is correct.
52. Ans. C.
The substantial deceleration in the nominal GDP growth reflects poorly on the government’s fiscal
marksmanship. In other words, it signifies that the government was not able to assess the magnitude
of economic growth deceleration that was underway. The poor fiscal marksmanship, in turn, leads
to inaccurate policymaking because a government could end up making policies for an economy that
does not actually exist on the ground.
Hence, Option C is correct.
53. Ans. D.
The enactment of the NFSA (National Food Security Act) 2013 on 5th July 2013 marks a paradigm shift
in the approach to food security from welfare to a rights-based approach.
The NFSA Act was rolled out in the year 2014.
The Act legally entitles up to 75 percent of the rural population and 50 percent of the urban
population to receive subsidized foodgrains under the Targeted Public Distribution System.
Hence, Option D is correct.
54. Ans. C.
Ease of Doing Business report 2020 :
• India has recorded an increase of 14 positions at 63rd rank against its rank of 77 last year.
• For the 3rd consecutive year, India was also amongst the top 10 improvers.

PAGE 262
www.byjusexamprep.com

• India’s improved ranking was on the back of reforms like (a)dealing with construction permits,
(b)trading across borders and (c)resolving insolvency.
Hence, Option C is correct.
55. Ans. B.
For every rupee with the government –offer, 68 paise came from the direct & indirect taxes while
the states' share of taxes & duties is the single-largest expen’e head accounting for 23% of the total
spending as per the 1019-20 budget.
As per the Budget for 2019-20 presented in the Parliament by Finance Minister Nirmala Sitharaman,
GST collections contributed 19 paise in every rupee revenue collection.
The Corporation tax is the single largest source of the income, contributing 21 paise to each rupee
earned.
The collection from borrowings & other liabilities will be 20 paise whereas income tax will yield 16
paise to every rupee collection.
Hence, Option B is correct.
56. Ans. D.
Active Volcanos - Location
Mauna Loa - Hawaii
Mount Vesuvius - Italy
Mount Merapi - Indonesia
Sakurajima - Japan
Hence, Option D is correct.
57. Ans. D.
The Tropic of Capricorn, also called the Southern Tropic, is an imaginary line (a latitude circle) which
indicates the southernmost point where the Sun is directly overhead at noon. This occurs during the
solstice on December 21.
The Tropic of Capricorn is one among the five crucial latitudinal circles marked on the globe. It is
23°26’13.3″ south of the Equator.
Hence, Option D is correct.
58. Ans. B.
Chandrakant Sompura, the Ahmedabad-based architect whose design has been adopted by Shri Ram
Janmabhoomi Teerth Kshetra for the construction of the temple. The temple will retain its original
plan of 'Nagara’ style of temple architecture with the Ram Mandir being three-storeys and of 161
feet height. The temple will have five-domed mandaps, and one Shikhar, all adhering to the principles
of Vastu Shastra.
Hence, Option B is correct.

PAGE 263
www.byjusexamprep.com

59. Ans. A.
The recent explosion near OIL will in Baghjan is due to removing the spool during the blowout control
operations. In a written statement, OIL said, 'The three experts were provided with medical attention
immediately at the well site, and later on they were taken to the burn unit section at Astha Hospital
in Dibrugarh district. They were released after dressing of their injuries.
Hence, Option A is correct.
60. Ans. C.
The Serum Institute of India was halting the trial of the Oxford University-Astrazeneca COVID-19
vaccine in India until the British-Swedish drug maker restarts the trial. The move came a day after
the Pune-based vaccine maker received a show-cause notice from the DGCI (Drug Controller's
General of India).
Hence, Option C is correct.
61. Ans. A.
August 6, 2020, marks the 75th anniversary of the dropping of the first atomic bomb. Two American
atomic bombs ended World War II in August 1945, and the devastation will be forever remembered.
In an instant when the first bomb was dropped, tens of thousands of residents of Hiroshima, Japan
were killed by "Little Boy," the code name for the first atomic bomb used in warfare in world history.
Hence, Option A is correct.
62. Ans. B.
President Michel Aoun said 2,750 tonnes of ammonium nitrate, used in fertilisers and bombs, had
been stored for six years at the port without safety measures after it was seized.
The explosion was the strongest ever to rip through Beirut, a city still scarred by the civil war that
ended three decades ago and reeling from an economic meltdown and a surge in coronavirus
infections. The blast rattled buildings on the Mediterranean island of Cyprus, about 160 km away.
Hence, Option B is correct.
63. Ans. C.
The following below are the basic features of protective democracy:
• Protective democracy believes in popular sovereignty. But since people can't directly take part
in the processes of the state, they do it through their representatives.
• Both the popular sovereignty and the representative form of the government are very much
legitimate.
• It is the basic duty of the state to secure the rights & liberties of citizens and whether this is
properly performed or not people keep the strong vigilance over the functions of the state.
Hence, Option C is correct.
64. Ans. D.
Article – 243 :
'Panchayat' means an institution (by whatever name called) of self-government constituted under
article 243B, for the rural areas;

PAGE 264
www.byjusexamprep.com

Article – 243C : the Legislature of a State may, by the law, make provisions with respect to the
composition of Panchayats.
Hence, Option D is correct.
65. Ans. C.
Article 176 provides for the special address by the Governor.
• At the initiation of the 1st session after every general election to the Legislative Assembly and at
the commencement of the 1st session of each year, the Governor shall address the Legislative
Assembly or, in the case of a State having a Legislative Council, both House assemble together.
• Provision are made by the rules regulating the procedure of the House or either House for the
allotment of time for discussion of the matters referred to in such address.
Hence, Option C is correct.
66. Ans. C.
Secularism as a Western term is defined as the separation of powers of church & State. In the context
of the other societies, it is near-complete freedom of religion and the lack of authority of religious
leaders over the political matters. Democracy is not a pre-requisite for establishing a secular state
though secularism works best when adapted with a democratic system.
Hence, Option C is correct.
67. Ans. A.
Ram Manohar Lohiya was an activist for the Indian freedom struggle and the socialist movement in
India.
Ram Manohar Lohiya' Seven Revolutions (i.e. Sapta Kranti):
• for equality between man and woman;
• against the political, economic and race-based inequalities;
• for the destruction of castes;
• against foreign domination
• for economic equality, planned production and against the private property;
• against interference in private life;
• against the arms & weapons and for Satyagraha.
Hence, Option A is correct.
68. Ans. B.
The MHA (Ministry of Home Affairs) is the 'Nodal Ministry' for the management of the natural
disasters in India. It is headquartered in New Delhi. The PM of India is the chairman of the NDMA. It
has five members.
Hence, Option B is correct.
69. Ans. C.
When the Montagu report of the year 1918 was made public, there was a split in the Congress over
it. The moderates greeted it while the extremists opposed it. This results in a schism in the Congress
with moderate leaders forming the 'Indian National Liberal Federation' in the year 1919. The party

PAGE 265
www.byjusexamprep.com

was founded by Surendra Nath Banarjee, and some among its prominent leaders were Tej Bahadur
Sapru, V. S. Srinivasa Sastri and M. R. Jayakar.
Hence, Option C is correct.
70. Ans. C.
S.B. Tambe was the first one to accept a ministerial position in the central provinces in Oct. 1925.
Shripad Balwant Tambe was the pledger from Amravati in the Berar division of Central Provinces. He
was a member of the Swaraj Party and the President of the Central Provinces Legislative Council.
Hence, Option C is correct.
71. Ans. D.
After the first world war, the Triveni Sangh was formed by the Ahirs and the Kurmis. The 'Triveni
Sangh' was a caste coalition and political party established in Shahabad District of Bihar in pre-
independence India to voice the political solidarity of 'middle peasant castes' as well as to carve a
space in democratic politics for the lower castes.
Hence, Option D is correct.
72. Ans. D.
Mohammedan Anglo-Oriental Defence Association was founded in the year 1893 by Sir Syed Ahmad
Khan as a part of the Aligarh movement. He established it as he considered competence in English
and 'Western sciences' necessary skills for maintaining Muslims' political influence, especially in
Northern India.
Hence, Option D is correct.
73. Ans. C.
The Moderates constitutional demands were:
• Expansion of the Legislative Council and the Legislative Assemblies, both Central and Provincial.
• Increase in the membership of Indians by involving some members elected by local bodies such
as chambers of commerce, universities, etc.
• By the starting of the 20th century, they demanded for Swaraj (self-rule) within the British Empire
similar to that of the self-governing colonies in Australia and Canada.
• Adequate representation of the Indians in the Executive Council of the Viceroy and those of the
Governors.
• Reformation and expansion of the legislative councils formed by the act of 1861. They demanded
the rise in the membership of these councils and all legislative and financial matters involving
the Budget should be submitted to these councils.
• The legislative council's members to be directly elected by the people of India, i.e. they
demanded Universal Adult Franchise.
• A complete separation of the executive and the judicial branches of administration.
• Complete self-government modelled on self-governing British colonies like Australia and Canada.
Hence, Option C is correct.

PAGE 266
www.byjusexamprep.com

74. Ans. D.
During the 19th century, 'Satapatra Series' was written by G.H. Deshmukh. Gopal Hari Deshmukh was
an Indian activist, thinker, social reformer and writer from Maharashtra. His original surname was
Shidhaye. Because of 'Vatan' (i.e. the right of Tax collection) that the family had received, the family
was later called Deshmukh. Deshmukh is regarded as a prominent figure of the Social Reform
Movement in Maharashtra.
Hence, Option D is correct.
75. Ans. A.
• The Great Barrier Reef is the largest among the living structure on the Earth.
• It is a site of the remarkable variety and beauty on the north-east coast of Australia. The reef is
situated off the coast of Queensland, Australia, in the Coral Sea.
Hence, Option A is correct.
76. Ans. B.
• Part II of the Indian Constitution contains provisions related to citizenship. So, any amendment
to the citizenship act will fall under part II of the Constitution.
• Articles 5 - 11 contain provisions related to citizenship in India.
• Part I - States and union territories.
• Part IV – Directive Principles of State Policy.
• Part VI – The states.
Hence, Option B is correct.
77. Ans. C.
• Article 32 of Indian Constitution empowers the Supreme Court of India to issue writs: habeas
corpus, mandamus, prohibition, quo warranto and certiorari, for the enforcement of
fundamental rights.
• By Article 226, the High Courts have also been empowered to issue writs, for the implementation
of Fundamental Rights.
• Hence both Supreme Court and High Courts can issue writs.
Hence, Option C is correct.
78. Ans. A.
Benegal Narsing Rau was appointed as the advisor to the Constituent Assembly. He played a crucial
role in framing the general structure of the Constitution and preparation of the first draft. He
travelled to different countries and had extensive meetings with eminent jurists and political experts
throughout the world.
Hence, Option A is correct.
79. Ans. B.
• The 42nd Amendment to the Constitution of India was enacted in 1976. The Amendment:
attempted to reduce the power of the Supreme Court and High Courts, added Article 51 A, which
contained the Fundamental Duties for citizens and added Socialist, Secular, and Integrity added
to the Preamble of the Constitution.

PAGE 267
www.byjusexamprep.com

• The Amendment transferred five subjects (Education, Forests, Weight & Measures, Protection
of Wild Animals and Birds, and Administration of Justice) from the state list to the concurrent
list.
• The Amendment also added the four new Directive principles of the state policy.
• Due to the scale of changes introduced, this amendment is also known as ‘mini-Constitution’.
Hence, Option B is correct.
80. Ans. D.
• India is a multi-party democracy. The Election Commission is gives recognition to political parties
as national or state political parties on the basis of their performance in the elections.
• Election Commission of India recognises three type of political parties: National Party, State Party
and registered unrecognized parties.
• As of November 2020, India has eight national political party: All India Trinamool Congress,
Bahujan Samaj Party, Bharatiya Janata Party, Communist Party of India, Communist Party of India
(Marxist), Indian National Congress, National People’s Party and Nationalist Congress Party.
Hence, Option D is correct.
81. Ans. D.
• Parliamentary Committee means a Committee appointed or elected by the House or nominated
by the Speaker and which works under the direction of the Speaker and presents its report to
the House or to the Speaker.
• Parliamentary Committees are of two kinds: Standing Committees and Ad hoc Committees.
Standing Committees are permanent and regular committees which are constituted from time
to time. Ad hoc Committees are appointed for a specific purpose and they cease to exist when
they finish the task assigned to them and submit a report.
• The members of these committees are appointed or elected on a motion made and adopted or
nominated by the Speaker of the Lok Sabha or the Chairman of the Rajya Sabha.
Hence, Option D is correct.
82. Ans. B.
• Parliamentary privilege are rights and immunities enjoyed by Members of Parliament in their
individual capacity.
• Article 105 of the Constitution of India which provides for powers, privileges and immunities of
the Houses of Parliament and of the members and the Committee.
• The privileges of the MPs are not fettered by the Article 19(1)(a) of the Constitution of India and
they must be read subject to the Articles 20-22 and Article 32 of the Constitution of India.
Hence, Option B is correct.
83. Ans. A.
• Al-Biruni was a scholar from Uzbekistan, who visited India in the 11th century CE. He was captured
as slave by Mahmud of Ghazni and spent most of his years in city of Ghazni. He visited India in
1017 CE.
• Al-Biruni’s Kitab-ul-Hind is written in Arabic and is divided into 80 chapters on subjects such as
religion and philosophy, festivals, astronomy, alchemy, manners and customs, social life, weights
and measures, iconography, laws and metrology.
Hence, Option A is correct.

PAGE 268
www.byjusexamprep.com

84. Ans. B.
• Ibn Batuta was a Moroccan traveller who visited India in 14th century CE.
• Before he set off for India in 1332-33, he had made pilgrimage trips to Mecca, and had already
travelled extensively in Syria, Iraq, Persia, Yemen, Oman and a few trading ports on the coast of
East Africa.
• Muhammad Bin Tughlaq, the sultan of Delhi, was impressed by his scholarship and appointed
him the qazi or judge of Delhi.
• He also went to China as the envoy of Muhammad Bin Tughlaq.
Hence, Option B is correct.
85. Ans. B.
• The Azamgarh proclamations, issued on 25th August 1857 stressed on the issue of unity among
Hindus and the Muslims of the country.
• While the real author of the proclamation is not known, many sources have indicated that the
proclamation was written by Firoz Shah, grandson of the Mughal emperor Bahadur Shah Zafar.
Hence, Option B is correct.
86. Ans. B.
• The revolt of 1857 was initiated by the Indian soldiers employed by the East India Company. The
soldiers were aggrieved by many reasons, the most immediate being introduction of the greased
cartridges of the Enfield Rifle.
• Lord Canning was the Governor General of India at the time of Revolt of 1857.
• Important leaders and places of the revolt were:

Hence, Option B is correct.


87. Ans. D.
• The resolution on Fundamental Rights and Economic Policy was passed at the Karachi session of
Indian National Congress in 1931. The session was presided by Sardar Patel.
• The Congress also endorsed the Gandhi-Irwin pact at the Karachi Session.
• The Congress passed the historic ‘Purna Swaraj’ resolution at its Lahore session in 1929.
Hence, Option D is correct.
88. Ans. C.
• The Philosophy of the Bomb was written by Bhagwati Charan Vohra in a response to a critical
essay written by Mahatma Gandhi against the methods used by revolutionaries.
• Gandhiji had written that if we continue to use violent means to achieve independence, we will
not achieve true independence.
Hence, Option C is correct.

PAGE 269
www.byjusexamprep.com

89. Ans. D.
• The Gupta Empire, established by Sri Gupta rose on the ruins of the Kushan Empire and ruled for
more than 200 years.
• The rulers of the Gupta dynasty issued largest number of gold coins (called Dinars) in India. The
Gupta rulers also issued silver coins but the coins made of copper are very scarce.
• The gold coins of the Gupta rulers are the extraordinary examples of artistic excellence. The coins
depicted the ruling monarch on the obverse and carried legends with the figure of a goddess on
the reverse.
Hence, Option D is correct.
90. Ans. C.
• In the first century CE, Gandhara (now in Pakistan), Mathura in northern India and Vengi in
Andhra Pradesh emerged as important centres of art production.
• The Gandhara School of Art was highly influenced by Greco-roman style.
• Initially, grey-blue schist and green phyllite were predominantly used for making the sculptors
but by the 3rd century CE stucco completely replaced stone as material used for sculptors. The
sculptures were originally painted and gilded.
Hence, Option C is correct.
91. Ans. A.
• The landforms formed as a result of erosional and depositional action of running water (mainly
rivers) are known as fluvial landforms.
• George, braids, Canyon, Alluvial Fans and Cones, Natural Levee, Waterfalls, potholes, Meanders,
oxbow lakes and deltas are examples of fluvial landforms.
• When the discharge of a river is low, the river takes the route of less resistance, thus forming
braided channels.
• Cirque is a landform formed by glacial erosion.
Hence, Option A is correct.
92. Ans. A.
• Tundra vegetation is characterised by extremely low temperatures and low biodiversity.
• It is found in the polar areas of Europe, Asia and North America.
• The vegetation is composed of dwarf shrubs, sedges and grasses, mosses, and lichens.
• Belarus does not have a Tundra Vegetation.
Hence, Option A is correct.
93. Ans. A.
• The four innermost planets (planets closer to the Sun i.e. Mercury, Venus, Earth and Mars) are
referred as terrestrial planets.
• Jupiter, Saturn, Uranus and Neptune are referred as gas giants as well as Jovian planets.
Hence, Option A is correct.
94. Ans. B.
• Mariana Trench located in the Pacific Ocean is considered as the deepest point (11,034 meters
deep) of the oceans.

PAGE 270
www.byjusexamprep.com

• Tonga Trench, located in the south-west Pacific Ocean is the second deepest trench of the world.
• The Kermadec Trench is an ocean trench in the south Pacific Ocean.
Hence, Option B is correct.
95. Ans. A.
• A tectonic plate (also called lithospheric plate) is a massive, irregularly-shaped slab of solid rock,
generally composed of both continental and oceanic lithosphere.
• The plate tectonics theory proposes that the earth’s lithosphere is divided into seven major and
some minor plates.
Hence, Option A is correct.
96. Ans. B.
• Mauritius Prime Minister Pravind Jugnauth declared a state of emergency after MV Wakashio, a
Japanese-owned ship that ran aground offshore days ago began spilling tons of fuel.
• Coral reefs, seagrasses and mangroves make Mauritian waters extraordinarily rich in
biodiversity. The oil spill threatens to ruin the blue waters and can cause irreversible damage to
marine biodiversity.
Hence, Option B is correct.
97. Ans. C.
• Percy Mahinda Rajapaksa was sworn in as the Prime Minister of Sri Lanka for the fourth time in
August 2020.
• Prior to this, he had served as the President of Sri Lanka from 2005 to 2015.
• Gotabaya Rajapaksa is the eight President of Sri Lanka.
Hence, Option C is correct.
98. Ans. A.
• The 2300 km-long submarine Optical Fibre Cable (OFC) which connects Andaman and Nicobar
Islands with mainland India was inaugurated by Indian Prime Minister in August 2020.
• The OFC was laid to provide better connectivity from Chennai to Port Blair and seven other
Islands - Swaraj Deep (Havelock), Long Island, Rangat, Hutbay (Little Andaman), Kamorta, Car
Nicobar and Campbell Bay (Great Nicobar).
• Initially, the Shaheed island was not connected to mainland through this programme.
Hence, Option A is correct.
99. Ans. B.
• The Wimbledon is one of the four grand slam tournaments of Lawn Tennis, with other three
being Australian Open, French open and US Open.
• Simona Halep of Romania defeated Serena Williams in the final to win the Ladies' Singles tennis
title at the 2019 Wimbledon Championships.
• Halep also became the first Romanian female player to win the Wimbledon.
Hence, Option B is correct.

PAGE 271
www.byjusexamprep.com

100. Ans. D.
• INS Viraat was a Centaur-class aircraft carrier of the Indian Navy. It was acquired by India from
Royal Navy of the United Kingdom in 1987.
• INS Viraat was formally decommissioned from Indian Navy in 2017.
• Presently INS Vikramaditya is the flagship aircraft carrier of the Indian Navy.
• INS Rajput is a guided-missile destroyer. INS Chakra and INS Khanderi are attack submarines of
the Indian Navy.
Hence, Option D is correct.
101. Ans. A.
• The Rio de la Plata or River Plate forms part of the border between Argentina and Uruguay. The
cities of Buenos Aires and Montevideo are located on its western and northern shores,
respectively.
• The Orinoco River flows through Venezuela and Colombia and is the fourth largest river in the
world by discharge volume of water.
• The Purus river is a tributary of Amazon and flows through Peru and Brazil.
• The Madeira river flows through Bolivia and Brazil and is the biggest tributary of the Amazon.
Hence, Option A is correct.
102. Ans. D.
As of November 2020, India has eight Union Territories. The following table mentions Union
Territories and their geographical area:

Hence, Option D is correct.


103. Ans. B.
• The Climax mine, is a major molybdenum mine located in Colorado (United States of America).
• Molybdenum is an essential trace mineral. China, Chile and the US are the three largest
producers of molybdenum.
Hence, Option B is correct.
104. Ans. A.
• The Sea of Azov is a connected to the Black Sea by the Strait of Kerch. The Don River and Kuban
River flow into the Sea of Azov.
• The Sea of Azov, a northern extension of the Black Sea, is an important geopolitical region located
on the southern coastlines of Russia and Ukraine.
Hence, Option A is correct.

PAGE 272
www.byjusexamprep.com

105. Ans. C.
• Ozone layer in Earth’s stratosphere absorbs most of the ultraviolet radiation of the Sun and thus
stops them from reaching Earth’s surface.
• The thickness of the ozone in a column of air from the ground to the top of the atmosphere is
measured in terms of Dobson units (DU).
• Ozone depletion (thinning of the ozone layer) is a result of an increase in Chlorofluorocarbons
(CFCs) in the Earth’s atmosphere.
• International Day for the Preservation of the Ozone Layer is observed on 16th September every
year.
Hence, Option C is correct.
106. Ans. D.
• A perfectly competitive market has the following defining features:
• The market consists of a large number of buyers and sellers
• Each firm produces and sells a homogenous product. i.e., the product of one firm cannot be
differentiated from the product of any other firm.
• Entry into the market as well as exit from the market are free for firms.
• Information is perfect
Hence, Option D is correct.
107. Ans. A.
• A demand curve graph depicts the relationship between price of a commodity (the y-axis) and
the quantity of that commodity that is demanded at that price (the x-axis).
• Demand curve will shift to left or right as a result of change in any non-price determinant of
demand. Thus, normally there is no shift in demand curve when price of a commodity falls.
• Several factors like rising income, change in population, changes in expectations and changes in
tastes and preferences affect individual and market demand.
Hence, Option A is correct.
108. Ans. A.
• The Index of Industrial production measures variations in the production of various industries
and is released by the Ministry of Statistics & Programme Implementation.
• IIP divides industries into six use-based sectors: Primary Goods, Capital Goods, Intermediate
Goods, Infrastructure/ Construction goods, Consumer durables and Consumer nondurables.
• Weights of the different sectors under the used based classification – 2011-12 series are:

Hence, Option A is correct.

PAGE 273
www.byjusexamprep.com

109. Ans. A.
• The concept of Human Development Index was developed in 1990 by the Pakistani economist
Mahbub ul Haq.
• The HDI measures average achievement of a country in three basic dimensions of human
development: a long and healthy life, access to knowledge, and a decent standard of living.
Hence, Option A is correct.
110. Ans. C.
• Trade surplus refers to a situation when the value of exports is greater than import.
• Trade deficit refers to a situation when the amount of import exceeds exports.
• India's overall trade deficit, including both goods and services, has increased to USD 103.63
billion in 2018-19 from USD 84.45 billion in 2017-18.
Hence, Option C is correct.
111. Ans. D.
Coral reefs are one of the most biologically diverse marine ecosystems on the Earth. Corals grow over
geological time scales and have been in existence for about 200 million years.
• Coral reefs play an important role in marine ecosystem and support the habitats of flora and
fauna in the sea.
• Australian Great Barrier Reef, Belize Barrier Reef and the New Caledonian Barrier Reef, The
Mesoamerican Barrier Reef System, The Florida Reef Tract and Rainbow Reef are some examples
of coral reef.
• Kyushu Island (Japan) is not a coral reef island.
• In India, Coral reefs are present in the areas of Gulf of Kutch, Gulf of Mannar, Andaman &
Nicobar, Lakshadweep Islands and Malvan.
Hence, Option D is correct.
112. Ans. B.
• Haryana will host the 4th edition of the Khelo India Youth Games after the Tokyo Olympics 2021.

Hence, Option B is correct.


113. Ans. B.
• Girish Chandra Murmu, a 1985-batch IAS officer of Gujarat cadre became the first Lieutenant
Governor of newly created Union Territory of Jammu and Kashmir on 31st October 2019.

PAGE 274
www.byjusexamprep.com

• He resigned from the post on 5 August 2020 and was appointed as the 14th Comptroller and
Auditor General of India.
Hence, Option B is correct.
114. Ans. D.
• Actress Vidya Balan portrayed the role of Shakuntala Devi in the biopic movie based on the life
the noted mathematician.
• Shakuntala Devi (1929-2013), also known as ‘the Human Computer’ due to her quick calculation
was a famous mathematician and writer.
• She also wrote a number of books including Figuring: The Joy of Numbers, The World of
Homosexuals, Astrology for You, Mathability: Awaken the Math Genius in Your Child, Perfect
Murder.
Hence, Option D is correct.
115. Ans. B.
• The idea to designate a name to cyclone to ensure smooth communication was conceived in
2000 by a group of tropical countries including Bangladesh, India, the Maldives, Myanmar,
Oman, Pakistan, Sri Lanka and Thailand.
• The regional specialised meteorological centres (RSMCs) and Tropical Cyclone Warning Centres
(TCWCs) are responsible for naming tropical Cyclones that form in every ocean basin across the
world.
Hence, Option B is correct.
116. Ans. D.
• The United Nations has designated 12 August as the International Youth Day. The theme of
International Youth Day 2020 was “Youth Engagement for Global Action”.

Hence, Option D is correct.


117. Ans. B.

Hence, Option B is correct.


118. Ans. C.
• Article 32 of Indian Constitution empowers the Supreme Court of India to issue writs: habeas
corpus, mandamus, prohibition, quo warranto and certiorari, for the enforcement of
fundamental rights.

PAGE 275
www.byjusexamprep.com

• By Article 226, the High Courts have also been empowered to issue writs, for the
implementation of Fundamental Rights.
Hence, Option C is correct.
119. Ans. A.
Articles 31A, 31 B and 31 C deals with right to property and they are exception to the Fundamental
Rights.
• Article 31A contains laws regarding the acquisition of any estate or right by the government or
management of any property.
• Article 31B states that the acts which are present in the ninth schedule and are inconsistent
with the provisions laid down by the constitution or resist any decree or order will be left on
the competent legislature to amend, revoke or to let it be in force.
• Article 31C states that no law which gives effect to the provisions contained in Article 39B and
Article 39C can be challenged under Article 14, Article 19, or Article 31.
Hence, Option A is correct.
120. Ans. C.
The President is the first citizen of India (head of the Indian state). The Indian has following terms of
office:
• The President holds the office for a term of five years from the date of entering his/her office.
• The President may resign before the expiration of his/her term by writing to the Vice President.
• The President can also be removed from the office before the completion of his/her term by
the way of Impeachment.
• The President can hold the office beyond his term of five years till his/ her successor assumes
charge.
• The Indian President is also eligible for re-election to the office of President.
Hence, Option C is correct.

PAGE 276
www.byjusexamprep.com

Elementary Mathematics
1. Ans. B.
let the distance is s km and time taken to complete it is t hr.

Then the speed kmph

New speed kmph now it takes hr.

Distance,

2. Ans. A.

Given that the fall of distance,

It takes 4 sec to fall 78.40 m. let it will take t sec to fall 122.5 m

So,

sec

3. Ans. B.
given that

We can see that,

PAGE 277
www.byjusexamprep.com

4. Ans. D.
Euclidean algorithm, procedure for finding the Highest Common Factor (HCF) of two numbers,
described by the Greek mathematician Euclid in his Elements (c. 300 bc).
5. Ans. D.
let the radius of a sphere is p (a rational number)

Surface area of sphere

Volume of sphere

Here both are incorrect.


6. Ans. B.

we have and

7. Ans. B.

given that

8. Ans. D.

since triangle is in semicircle, AB is diameter then (inside a semicircle)

PAGE 278
www.byjusexamprep.com

Now,

And

9. Ans. D.

We know that

No such value of exist.

Same for the statement -2.


10. Ans. C.

interior angle of a polygon,

For pentagon n = 5,

11. Ans. D.

Let

And

So,

12. Ans. D.

let

Given that,

And

We know that

PAGE 279
www.byjusexamprep.com

So,

13. Ans. C.

we have and

and

After solving both and

14. Ans. D.

we have

15. Ans. D.

let

PAGE 280
www.byjusexamprep.com

By comparison

, and

So, the function is

16. Ans. B.
let the number is then the reverse number is .

Given that …(i)

From equation (i) and (ii)


The product of the digit
17. Ans. B.

given that

Now,
18. Ans. B.
Let X takes t hr. to travel a distance with speed of 4 kmph.
Then distance travelled by X, km.

Now Y takes hr. to complete the same distance with speed of 5 kmph.

Then distance

And

Let Z starts time after Y starts then

PAGE 281
www.byjusexamprep.com

19. Ans. B.

let

20. Ans. C.

given that and and profit

21. Ans. A.

we have to find

All the term is divisible by 18 except last which will be remainder.

So, remainder

22. Ans. C.

we have

Rationalization of each term

PAGE 282
www.byjusexamprep.com

23. Ans. A.

we have

After comparing the power, we get,

24. Ans. C.
we have 10 numbers

Let S is the sum of all possible products taken two at a time.

We know that, for two numbers

For three numbers

So,

25. Ans. C.
total distance

Speed of train

Time taken to cross the bridge

26. Ans. D.

let the sum of amount is . Then and let .

27. Ans. B.
given that the number of pages = 60 and number of lines per page = n
So, total number of lines in book = 60n

PAGE 283
www.byjusexamprep.com

After reducing 3 lines per page and increasing 10 pages gives the same writing space, then

28. Ans. D.
total units of work when x men working x hours per day in x day = units x
total units of work when y men working y hours per day in y day = units k

29. Ans. B.
given that d(n) = positive divisor of n
5 = 1,5
11 = 1,11
55 = 1,5,11,55
d(5) = 2
d(11) = 2
d(55) = 4
So statement 1 and 2 are correct.
30. Ans. D.

we have ; where is the product of first n prime numbers.

we know that when we add one in product of first n prime numbers it becomes also a prime number.

So is a prime numbers not composite number.

All the prime number (except 2) are odd number. So +1 is always even numbers

And +2 is always a odd numbers.

31. Ans. C.

let is exactly divisible by x. When we divide is divided by x then


remainder will be zero.

PAGE 284
www.byjusexamprep.com

For is exactly divisible by x, x should be divisor of 16

And

32. Ans. B.

given that ; a, b and c are positive integer

Total number of solutions = 3 + 6 + 6 + 6 + 6 = 27


33. Ans. C.
given that the roots of the equation is real then

34. Ans. C.
given that ; where

We have to choose 3 numbers to be sum 12, the number of ways


Or

Total number of possible solution = 3 + 6 + 6 + 6 + 6 + 3 + 6 + 6 + 3 + 3 + 6 + 1 = 55

PAGE 285
www.byjusexamprep.com

35. Ans. D.

given that ; where a and b are consecutive integer and c = ab.

So, and

The product of any two consecutive number is always an even number, so c is a even number and
I is a square of an odd integer.
36. Ans. A.
the smallest odd composite number is 9. And 23P62971335 is exactly divisible by 9. So sum of all the
digits of the number is also divisible by 9.
Hence ;
So P should be 4 to 23P62971335 is exactly divisible by 9.
37. Ans. B.

let

All the term is divisible by 4 except 1, So remainder is 1.


38. Ans. C.
we know that cycle of 3 is 4.

So, unit digit of is 7.


39. Ans. A.
given that and

PAGE 286
www.byjusexamprep.com

If one of the numbers is 240 then another number is

40. Ans. A.

we know that is always divisible by ; where n is a natural number.

41. Ans. B.
Given that a shopkeeper sells his articles at their cost price but by default balance which reads 1000
gm for 800 gm.
If he purchases and sell 1 kg article at 100 Rs. but due to faulty balance he gain 20 Rs at every 1 kg
article because it only sell 800 gm. and its cost price for 800 gm is 80 Rs. and its sell price is 100 Rs
for 800 gm because of faulty balance.
Then gain = 20

So, gain % =

⇒ gain % =

= 25 %
42. Ans. A.
We have, the depth of the river means height = 3 m
the width of the river = 40 m
Velocity of the river = 2 km/hour
So, it means in one hour water in river can flow = 2 km
And in 60 minutes = 2000 m

So, in 1 minute water can flow = m

So, for 1 minute river length =

Volume of water falling in a sea = volume of the river which is like cuboid

= 4000 m3 or 40,00,000 litres

PAGE 287
www.byjusexamprep.com

43. Ans. D.
Let television is purchase at 100 Rs.
If television is sold at Rs, a loss of 28% would be incurred.

We know that


If television is sold at y Rs, a profit of 12% would be incurred.

We know that

So,

44. Ans. C.
Let original speed of car = x km/hr
And the distance that covers = 300 km

So, time = hr

If speed of his car = (x + 15) km/hr

Then time =

So, distance =

PAGE 288
www.byjusexamprep.com

so speed cannot negative.


Then original speed = 60 k/hr.
45. Ans. A.

We have, three persons start a business with capitals in the ratio = .

= 20:15:12
So, first investment = 20x
second investment = 15x
third investment = 12x
Given that first withdraws half its capitals after 4 months.
So, first investment after 12 months =
second investment after 12 months =
third investment after 12 months =
Total investment =
Total annual profit = 96,800 rs

Then first share = rs.

46. Ans. D.
We have, x varies with y means x = ky.

(i)

and

so,

It means varies as

(ii)

So is inversely proportional to .

PAGE 289
www.byjusexamprep.com

(iii)

So, varies with .

Hence (i), (ii) and (iii) all are correct.


47. Ans. C.
We have, Ena is the daughter.
Her mother is 24 years older than Ena and Ena was born 4 years after her parents marriage.
So, her mother age is 20 years old at marriage.
And her mother is 3 years younger than her father.
So, Ena’s father age at marriage = 23 years
48. Ans. B.
We have, age of Mahesh = 60 years.
Ram age = Mahesh age – 5 = Raju age + 4
Babu age = Raju age – 6
So, Mahesh age = Raju age + 9
Then the difference of age between Mahesh and babu = 9 + 6 =15 years
49. Ans. A.
We have, the number of items in a booklet =
If first year x % increase in this number.
So, now the number of items in a booklet =

In subsequent year there is a decrease of x%.

So, now the number of items in a booklet =

PAGE 290
www.byjusexamprep.com

So, No. of items is less than N.


50. Ans. C.
We have,

...……(i)

And,

…….(ii)

Adding eq (i) and eq (ii) , we get

51. Ans. A.
We have, the marks of 10 passed students are 9,6,7,8,8,9,6,5,4 and 7.
In ascending order , 4,5,6,6,7,7,8,8,9,9.
And appeared students = 15

So, number 15 is odd then median is the value of means 8th observations.

but we don’t have the marks of 5 failed students which are obviously marks less than 4.
So, including the unknown 5 observations, 8th observations is 6.

PAGE 291
www.byjusexamprep.com

52. Ans. C.
We have, the yield of barely from the 7 plots of size one square yard each are
180, 191,175,111,154,141 and 176
In ascending order, 111, 141, 154, 175, 176, 180, 191

Here, 7 is the odd number of items. Therefore, median is the value of observations.

i.e. 4th observations. then 175gm is the median.


53. Ans. D.
We want to average size of shoe sold in the shop it means what is the popular or maximum demand
in shoe size.
So, in a measure in central tendency, mode is the most appropriate measure because it is repeated
the highest number of times in the series.
54. Ans. D.
A histogram is the most used graph to show frequency distributions and its display of statistical
information. A histogram uses rectangle to show frequency of data items of successive class interval
having equal width.
Hence, Height of a rectangle in a histogram represents frequency of the class.
55. Ans. B.
We have the 7 family members are 2, 5, 12, 18, 38, 40 and 60 years respectively.

So, mean =

= 25
After 5 years a new member x age is added. and mean is increase 1.5 years.
So, now mean = 26.5 years

56. Ans. C.

Let no of boys and no of girls .

So, …….(i)
and the mean weight of the boys = 50 kg

PAGE 292
www.byjusexamprep.com

total weight of boys = 50x

And the mean weight of the girls = 40 kg

total weight of girls = 40y


We have, the mean weight of 100 students = 46 kg

…….(ii)
Solving equation (i) and (ii), we get
x = 60 and y = 40
So, no of boys is exceed by 20 from no the girls.
57. Ans. B.
We have the values 25, 65,73,75,83,76,17,15,7,14.
In ascending order, 7, 14,15,17,25,65, 73,75, 76, 83

Mean =

= 45
deviations from the mean of the set of values = -38, -31, -30, -28, -20, 20,28,30,31,38
So algebraic sum of deviation of the mean of the set of values
= -38-31-30-28-20+20+28+30+31+38
=0
58. Ans. C.

We have, five observations


And mean of five observations =

And mean of first three observations =

PAGE 293
www.byjusexamprep.com

=x+2
=m–4+2
=m–2
59. Ans. D.
We have,
2, 4, 6, …… , 100
So, here n numbers in this A.P. series.

We know that

Since there are 50, an even number of items. Therefore median is the arithmetic mean of

and observations.

So, 25th observation =

& 26th observation =

Median =

60. Ans. D.
We have, H.M. = 10 and G.M. = 12

Let and two numbers.

We know that G.M. = and H.M. =

And

PAGE 294
www.byjusexamprep.com

We know that A.M. =

A.M. = 14.4
61. Ans. C.
We know that the radius from the centre of the circle to the point of tangency is perpendicular to
the tangent line.

Hence,

Also,

It is also given that


Now in quadrilateral AMON,


62. Ans. D.
Let the sides of right-angled triangle are a unit and b unit.
Length of its hypotenuse = h units
Using Pythagoras theorem

⇒ ………………..(1)
According to question

From (1)

PAGE 295
www.byjusexamprep.com

Hence, Length of hypotenuse = 65 units


63. Ans. B.
Let base of triangle = base of parallelogram = b
Altitude of parallelogram = a
Altitude of triangle= ka
Area of parallelogram = Area of triangle

⇒k=2
64. Ans. C.

Ratio of area of two squares =

Ratio of sides of two squares = Ratio of perimeter of two squares =


65. Ans. B.

As AD is the median of triangle ABC.


We know that each median divides the triangle into two similar triangles of equal area.
⇒ ar(ABD) = ar(ACD) ……………….(1)
Similarly, PD is the median of triangle PBC.
We know that each median divides the triangle into two similar triangles of equal area.
⇒ ar(PBD) = ar(PCD) ……………….(2)
From (1) and (2)
⇒ ar(PAB) = ar(PAC)
Hence, option B is correct.

PAGE 296
www.byjusexamprep.com

66. Ans. B.
Area of segment circle formula is given by

67. Ans. A.

In right angled triangle PCQ

……………..(1)

In Right angled triangle ABC

………………(2)
Adding (1) and (2)

Hence, statement (1) is correct.


Consider statement (2)
Now,
AB = 2PQ is true only if P and Q are midpoints of AC and BC respectively.
But it is not mentioned in question that P and Q are midpoints of AC and BC respectively.
Hence, statement (2) is not correct.
68. Ans. B.
Let radius of each circular coin = r cm

PAGE 297
www.byjusexamprep.com

the uncovered area of the square = 42 cm2

Uncovered area of the square = Area of square – Area of 4 circles that is covering square.


69. Ans. B.
We can say that bucket is in the form of frustum of a cone.
It is given that radii of the flat circular faces of a bucket are x and 2x.
The height of the bucket (h) = 3x

Required capacity of the bucket =

70. Ans. C.
p, q, r, s and t represent length, breadth, height, surface area and volume of a cuboid respectively
Volume of a cuboid (t)= pqr

Surface area of a cuboid (s) =

PAGE 298
www.byjusexamprep.com

71. Ans. C.
In a quadrilateral ABCD, ∠B = 90°

Also,
AB2 + BC2 + CD2 – AD2 = 0 …………….(1)
In right angled triangle ABC

⇒ ……………(2)
Put the value of (2) in (1)
⇒ AC2 + CD2 – AD2 = 0
⇒ AC2 + CD2 = AD2 ……………..(3)
From (3), we can conclude that triangle ACD is a right angled triangle.
⇒ ∠ACD = 90°
72. Ans. B.
If A ray bisects an angle of a triangle , then it divides the opposite sides of the triangle into segments
that are proportional to the other two sides.

PAGE 299
www.byjusexamprep.com

73. Ans. C.

Now, PQRS is also a cyclic quadrilateral.


⇒ Sum of opposite angle of cyclic quadrilateral is 180°.
Hence, ∠PQR + ∠RSP = 180°
74. Ans. A.

Let side = x
Draw
As triangle ABC is equilateral triangle.

BE =

BD =

DE = BE – BD =

In triangle ADE by Pythagoras theorem


⇒ =

PAGE 300
www.byjusexamprep.com

75. Ans. C.

Given :
A trapezium ABCD in which AB∥DC and its diagonals AC and BD intersect at O.
Construction :
Through O, draw EO∥AB, meeting AD at E.
Proof :
In △ADC, EO∥DC
Therefore,

⇒ ……………..(1)

In △DAB,EO∥AB
Therefore,

⇒ ..........(2)

From 1 and 2, we get,

Hence, We can say that The diagonals of a trapezium divide each other proportionally.
Consider second statement :

Here, ABCD is a trapezium such that EF||AB||DC


Join EF to cut AC at G.
ΔADC , EG||DC , so by basic proportionality theorem , we have

PAGE 301
www.byjusexamprep.com

⇒ ………………(1)

ΔACB, GF||AB , so by basic proportionality theorem , we have

⇒ ………………(2)

From (1) and (2)

Hence, we can say that Any line drawn parallel to the parallel sides of a trapezium divides the non-
parallel sides proportionally.
Hence, both the statements are true.
76. Ans. A.
H, C and V are respectively the height, curved surface area and volume of a cone.

77. Ans. D.

Required number of solid lead balls =

78. Ans. D.
The two sides of a tringle are 40 cm and 41cm and the perimeter of the triangle is 90 cm.
Hence, the third side of the triangle = 90 cm - (40 + 41 ) cm = 90 cm – 81 cm = 9 cm
Clearly,

Hence, the triangle is right angled triangle.

So, Area of triangle =

PAGE 302
www.byjusexamprep.com

79. Ans. C.
Let length of one diagonal is a unit and length of another diagonal is b unit.
It is given that diagonals of a rhombus differ by 2 units

Perimeter of the rhombus =


According to question





Length of diagonal cannot be negative.
Hence, Length of diagonal is 6 cm
Also, a = b +2 = 6 + 2 = 8 units

Hence, Area of rhombus =

80. Ans. A.
Radius of the circumcircle = 5 cm

Altitude drawn to the hypotenuse = 4 cm

PAGE 303
www.byjusexamprep.com

As, ABC is a right-angled triangle with


AND we know that the angle inscribed in a semicircle is always a right angle.
The hypotenuse of the triangle is the diameter of the circle.

Hence, Hypotenuse =

Area of a right-angled triangle =

81. Ans. B.

Clearly,

Hence, k =

82. Ans. B.
Consider statement 1 :

=>

and

We know that can never take value of -4 and

Hence, there is no value of for which the above equation holds.


Consider statement 2 :

PAGE 304
www.byjusexamprep.com

tan θ + cot θ is always greater than or equal to 2, where 0 < θ < .

Hence, only statement (2) is correct.


83. Ans. A.

We know that
Perimeter of the arc =

84. Ans. B.
Maximum value of

We know that maximum value of is 1

Hence, Maximum value of =


85. Ans. D.

If
Squaring both sides

PAGE 305
www.byjusexamprep.com

86. Ans. B.
Least value of

⇒ =

We know that maximum value of is 1.

⇒ Least value of = 16 – 7 (1) = 9


87. Ans. A.

= k ………………(1)

Hence, =k ……(2)

Squaring equation (1)



Now,

⇒ ……………(3)

From (2) and (3)

⇒ =
88. Ans. B.

PAGE 306
www.byjusexamprep.com

⇒ ………….(1)

⇒ ……………(2)

Divide (2) by (1)

89. Ans. B.

Hence, =

90. Ans. A.
Consider statement (1)

For,

PAGE 307
www.byjusexamprep.com


We know that

⇒ <1
Hence, statement (1) is correct.
Consider statement (2)

Hence, statement (2) is not correct.


91. Ans. D.
For option (A) ratio of angles 1 : 2 : 3, So
x + 2x + 3x = 180°
6x = 180°
x = 30
Hence all are integers.
For option (B) ratio of angles 3 : 4 : 5
3x + 4x + 5x = 180°
12x = 180°
x = 15°
Hence all angles, being multiple of 15 , so will be integers.
For option (c) ratio of angles 5 : 6 : 7
5x + 6x + 7x = 180°
18x = 180°
x = 10°
Hence all angles will be integers.
For option (D) ratio is 6 : 7 : 8
6x + 7x + 8x = 180°
21x = 180°
x = 8.57°
Hence for this ratio of angles , the angles would not be integers.

PAGE 308
www.byjusexamprep.com

92. Ans. C.
Assuming length is denoted by L
And breadth by b.
Then initial area of rectangle = L.b
But as given condition final length (L’) = 1.1L …………..(10% increase).
Final breadth (b’) = 0.9b …………..(10% decrease) .
Hence final area of the rectangle = L’.b’
= 1.1L × 0.9b
= 0.99 Lb
Thus it is 1% less than initial area.
93. Ans. D.

as surface area of sphere =

So =

Hence now =

Volume of sphere 1 : Volume of sphere 1 = 1 : 8


94. Ans. C.
Given for single brick L = 20 cm , b = 15 cm and h=10 cm
Volume of a brick = L.b.h
= 20 × 15 × 10 cm3
= 3000 cm3

PAGE 309
www.byjusexamprep.com

Volume of the wall = length of wall × breadth of wall × height of wall


=4500 × 15 × 300 cm3

Hence total number of bricks required =

95. Ans. A.
As we know sum of all interior angles = (n – 2) × 180°
And sum of all exterior angles = 360°
Now the given condition ⇒ (n – 2) × 180° = 2× 360°
n-2 = 4
n=6
hence the polygon is Hexagon.
96. Ans. B.
if the perimeter of the wheel be then it will cover distance in one revolution.

So the number of total revolutions =

……………(11km=1100000 cm)

97. Ans. A.

as volume of a cone =

So = =

PAGE 310
www.byjusexamprep.com

………………..(1)

But it is given that

Putting the ratio in eqn. (1)

98. Ans. B.
Given that 2 ∠A = 3 ∠B = 6 ∠C ………..(1)
As we know ∠A + ∠B + ∠C = 180°
From equ.(1)

Similarly for

As ∠A + ∠B + ∠C = 180°
From equ.(1)

PAGE 311
www.byjusexamprep.com

°
°
°
Hence ∠A + ∠C = 120°
99. Ans. D.
it is given that perimeter of circle = perimeter of square
……… ( where r is radius of circle and a is side of square)

………….(1)

Then =

100. Ans. D.

Assuming sides of right-angled triangle as

As in triangle the biggest side is the hypotenuse for the right-angled triangle
So by Pythagoras theorem - hypotenuse2 = base2 + hight2

But as a cannot be negative.

Hence the sides are 6 , 8 and 10


So their product = 6×8×10
= 480

PAGE 312
www.byjusexamprep.com

CDS I 2020
ENGLISH
Direction: In this section direct speech sentences 3. He said to his manager, “Could you please
are given and you are required to find the correct pass the bill this week?”
indirect speech sentence of the same. Choose
A. He told his manager that bill to be
the correct response (a), (b), (c) or (d) and
passed.
indicate on the Answer Sheet accordingly.
1. The grandfather said to the baby, “May you B. He requested his manager to pass the
live long with all good things of life.” bill that week.

A. The grandfather blessed to the baby C. He ordered his manager to pass the bill
with long life and all good things of life. that week.

B. The grandfather asked the baby that D. He requested his manager to pass the
she would live long with all good things bill this week.
of life. 4. “Those who sowed the seeds last season
C. The grandfather wanted the baby to will reap the harvest this season,” said the
live long with all good things of life. leader to her followers.
D. The grandfather blessed the baby that A. The leader said to her followers that
she would live long with all good things those who sowed the seeds the
of life. previous season would reap the
harvest that season.
2. The village chief said to the villagers, “All of
us need to adopt new regulations. We will B. The leader addressed her followers
protect our Earth forever.” that those who have sown the seeds
A. The village chief ordered the villagers the previous season would reap the
that all of them needed to adopt new harvest this season.
regulations and they would protect C. The leader addressed her followers
their Earth forever. that those who had sown the seeds the
B. The village chief told the villagers that previous season would reap the
all of them need to adopt new harvest that season.
regulations and they will protect their D. The leader advised her followers that
Earth forever. those who sow the seeds the previous
C. The village chief wanted the villagers season would reap the harvest this
needed to adopt new regulations and season.
they would protect their Earth forever. 5. “Where were you last evening?” said the
D. The village chief told the villagers that lady to her maid.
all of them needed to adopt new
A. The lady asked her maid where she had
regulations and they would protect
been the previous evening.
their Earth forever.

PAGE 313
www.byjusexamprep.com

B. The lady asked her maid where she had 9. He said to his friend, “Could you please
been in the last evening. close the door?”
C. The lady asked her maid where had she A. He requested his friend to close the
been the evening before. door.
D. The lady told her maid where she had B. He requested his friend to please close
been to the last evening. the door.
6. The Captain said to the soldiers, “March C. He ordered his friend to close the door.
forward and aim at the peak of the hill
today.” D. He wanted his friend to close the door
for him.
A. The Captain requested the soldiers to
march forward and aim at the peak of 10. Rahul said to his teacher, “Madam, what is
the hill that day. the way to solve the question?”
B. The Camptain ordered the soldiers to A. Rahul asked his teacher what the way
march forward and aim at the peak of to solve the question was.
the hill today. B. Rahul told his teacher what was the
C. The Captain ordered the soldiers to way to solve the question.
march forward and aim at the peak of
C. Rahul asked to his teacher what the
the hill that day.
way was to solve the question.
D. The Captain told the soldiers that they
D. Rahul told his teacher what the way
should march forward and aim at the
was to solve the question.
peak of the hill that day.
Directions: In this section two sentences are
7. Navanitha said to her friends, “What a
given and you are required to find the correct
scintillating beauty it is !”
sentence which combines both the sentences.
A. Navanitha told to her friends that it was Choose the correct response (a), (b), (c) or (d)
a scintillating beauty. and indicate on the Answer Sheet accordingly.
B. Navanitha exclaimed to her friends 11. Which one of the following is the correct
what a scintillating beauty it was. statement of the combination of the two
C. Navanitha asked her friends whether it sentences given below using ‘whereas’?
was a scintillating beauty. Kavya is interested in reading books. Her
D. Navanitha exclaimed to her friends that sister shows interest in outdoor games.
it was a scintillating beauty. A. Kavya is interested in reading books
8. Raj said to Sheela, “The Sun rises in the whereas her sister’s interest is outdoor
east”. games.
A. Raj told Sheela that the Sun rose in the B. Kavya is interested in reading books
east. whereas her sister is not interested in
B. Raj told Sheela that the Sun rises in the it.
east. C. Kavya is interested is reading whereas
C. Raj asked Sheela that the Sun rises in her sister’s interest is outdoor games.
the east. D. Kavya is interested in reading books
D. Raj said to Sheela that the Sun has whereas her sister’s interest is to play
arisen in the east. outside.

PAGE 314
www.byjusexamprep.com

12. Which is the correct combination of the A. Gandhiji who preached peace is an
given two sentences? apostle of peace.
The teacher entered the classroom. All B. Gandhiji preached peace because he is
students stopped talking. an apostle of peace.
A. No sooner did the teacher enter the C. Gandhiji who preached peace is called
classroom than the students stopped an apostle of peace.
talking.
D. Gandhiji is an apostle of peace because
B. As soon as the teacher entered the he preached peace.
classroom all students were asked to
16. Which one of the following is the correct
stop talking.
statement combining the two statements
C. All student stopped talking as the using ‘though’?
teacher enters the classroom.
He has been trying his level best to wins. He
D. No sooner did the students stop talking could not succeed.
than the teacher entered the
classroom. A. Though he is trying his level best to win,
he could not succeed.
13. Which is the correct combination of the
given two sentences? B. He is trying his level best to win, though
he could not succeed.
He is too tired. He could not stand.
C. Though he has been trying his level best
A. He is so tired that he could scarcely to win, he could not succeed.
stand
D. Though he had been trying his level
B. He is too tired and cannot stand. best to win, he could not succeed.
C. He will not stand and he is very tired. 17. Which is the correct combination of the
D. He is so tired that he could not be given two simple sentences using ‘If’
standing. clause?
14. Which is the correct combination of the Minchi should have worked hard. She
given two sentences? would have cleared the test.
Priya reached the station. The bus left A. If Minchi had worked hard, she would
before her. have cleared the test.
A. When Priya reached the station, the B. Had not Minchi worked hard, she could
bus had already left. not have cleared the test.
B. When Priya had reached the station, C. If Minchi has worked hard, she would
the bus already left. have cleared the test.
C. Priya reached the station, when the bus D. If Minchi had worked hard, she will
already left. have cleared the test.
D. When Priya had reached the station, 18. Which is the correct combination of the
the bus had already left. given two sentences?
15. Which is the correct combination of the Parents have been waiting since morning.
given two sentences using ‘relative clause’? They want to meet the counsellor.
Gandhiji preached peace. He is an apostle A. The counsellor had been waiting to
of peace. meet the parents since morning.

PAGE 315
www.byjusexamprep.com

B. Parents had been waiting to meet the 22. Which one of the following alternatives has
counsellor since the morning. the correct spelling?
C. Parents are waiting to meet the A. Resilient B. Resilint
counsellor in the morning. C. Risilient D. Realisent
D. Parents have been waiting since 23. Which one of the following alternatives has
morning to meet the counsellor. the correct spelling?
19. Which is the correct combination of the A. Pnumonia B. Neumonia
given two sentences?
C. Pneumonia D. Numania
He is hard-working. He is honest too. 24. Which one of the following alternatives has
A. He is not only hard-working, but also the correct spelling?
honest. A. Psudonym B. Pseudonym
B. He is only hard-working and honest. C. Pseudanym D. Seeudonym
C. He is hard-working but honest too. 25. Which one of the following alternatives has
the correct spelling?
D. He is not hard-working but also honest.
A. Neuphrology B. Nephrology
20. Which is the correct combination of the
C. Neprology D. Neaprology
given two sentences?
26. Which one of the following alternatives has
The officer will return from China on the correct spelling?
Monday. You can meet him.
A. Psephology B. Psefoloagy
A. You can meet the officer when he
returned from China on Monday. C. Sephology D. Psyphology
27. Which one of the following alternatives has
B. You can meet the officer when he will
the correct spelling?
return from China on Monday.
A. Vacum B. Vacuum
C. You can meet the officer when he
returns from China on Monday. C. Vacuem D. Vacam

D. The officer will meet you when you 28. Which one of the following alternatives has
return from China on Monday. the correct spelling?
A. Commisionor B. Commisioner
Directions: In this section a word is spelt in four
different ways. Identify the one which is correct. C. Commissioner D. Comissioner
Choose the correct response (a), (b), (c) or (d)
29. Which one of the following alternatives has
and indicate on the Answer Sheet accordingly.
the correct spelling?
21. Which one of the following alternatives has A. Stretegy B. Stretagy
the correct spelling?
C. Strategy D. Strategy
A. suplementary
30. Which one of the following alternatives has
B. supplementary the correct spelling?
C. supplementery A. Continuum B. Continuem

D. supplemantory C. Contuneim D. Continueiam

PAGE 316
www.byjusexamprep.com

Directions: Each of the following sentences has 37. “What is the latest news?” asked the
a word or phrase underlined. Read the sentences Captain.
carefully and find which part of speech the
A. Relative pronoun
underlined word belongs to. Indicate your
response on the Answer Sheet accordingly. B. Adjective
31. Ravi was declared as the winner in the tie C. Adverb
because he had hit the most number of
fours and sixes. D. Adjectival clause
A. Conjunction B. Interjection 38. The most beautiful actor of the industry
was awarded today.
C. Adverb D. Cause
32. Hurrah! What scintillating beauty the A. Adjective B. Numeral
landscape is! C. Adverb D. Noun
A. Conjunction B. Adjective 39. The man in dark blue is the one who made
C. Adverb D. Interjection us win the match.
33. The building is very ancient. A. Relative clause
A. Transitive verb B. Interrogative pronoun
B. Intransitive verb C. Relative pronoun
C. Phrasal verb D. Affirmative
D. Auxiliary verb
40. He has been working in the Department of
34. He has offered her another chance. Foreign Affairs since 2002.
A. Intransitive verb A. Preposition B. Adjective
B. Past tense C. Intensifier D. Noun
C. Perfect tense
Directions: Each item in this section consists of
D. Transitive verb sentences with an underlined word followed by
35. When he reached the department, the four words or group of words. select the option
officials had left for the meeting. that is opposite in meaning to the underlined
word and mark your response on the Answer
A. Past perfect verb Sheet accordingly. |||End|||
B. Past tense 41. All business activities need not result in
C. Dependent clause profit-making. There is a need to be
charitable.
D. Independent clause
A. Lenient B. Malevolent
36. Noticing the change in the behaviour of the
officer, the cadets returned to their C. Unforeseen D. Gracious
position.
42. One feels elated when someone praises
A. Participle one’s work.
B. Present continuous A. Feels good B. Excited
C. Noun phrase
C. Depressed D. Sober
D. Noun

PAGE 317
www.byjusexamprep.com

43. The highest award was bestowed upon her Directions: Each item in this section consists of a
for her yeoman service. sentence with an underlined word followed by
four words/group of words. Select the option
A. Conferred B. Withdrawn
that is nearest in meaning to the underlined
C. Imparted D. Imbibed word and mark your response on the Answer
44. Wooing everyone over an issue for support Sheet accordingly.
will not serve much purpose. 51. The cause of the accident is yet to be
A. Discouraging ascertained, but police officials suspect the
driver of the vehicle allegedly fell asleep.
B. Encouraging
A. Determined B. Curtained
C. Pursuing
C. Thought of D. Being known
D. Persuading
52. The leader said, “I am aghast with the
45. Tempestuous behaviour would not yield developments so far. I will take time to
much in any place. understand this.”
A. Relaxed B. Passionate A. Satisfied B. Sad
C. Intense D. Windy C. Amused D. Horrified
53. Uninterrupted rain had fatigued the
46. What we lack in the current times is commuters from the outskirts to the city
compassion. and work suffered.

A. Empathy B. Carefulness A. Excited B. Refreshed

C. Indifference D. Hardship C. Slowed D. Exhausted

47. There is a need to promote philanthropy in 54. Not everyone finds a vocation which suits
education. one’s aptitude.

A. Charity B. Benevolence A. Attitude B. Approach

C. Nastiness D. Likeliness C. Liking D. Occupation

48. Everyone has to fight the inertia in the 55. A new show is trying to change the clichéd
system. depictions of women in animation.

A. Sluggishness B. Indolence A. Original B. Hackneyed

C. Activity D. Torpor C. Crony D. Artificial

49. He speaks eloquently and can pull crowds. 56. The captain produced yet another stellar
show to make her team enter the semi-
A. Confusingly B. Expressively finals.
C. Powerfully D. Fluently A. Extraordinary B. Eclipse
50. Early medieval period was not a C. Poor D. Not a great
combination of urban and rural civilization.
It was not a period of urban decay as 57. His thesis makes all generic statements
claimed by some. which have already been proved.

A. Survival B. Waste away A. Specific B. Crude

C. Decomposition D. Spoil C. Broad D. Non-standard

PAGE 318
www.byjusexamprep.com

58. The budget incorporated a number of tax 66. It shows that she has ________ many years
reforms which included higher taxes for the of service.
very rich.
A. Put in B. Put out
A. Excluded B. Integrated C. Put D. Put on
C. Laid down D. Removed 67. It is best to _________ politics when in the
59. He is such a leader that his actions are classroom.
contagious. A. Keep out B. Keep on
A. Complicated B. Transmittable C. Keep off D. Keeping
C. Effective D. Unthinkable 68. Ravi has proved that he can __________ on
his promise by winning the match.
60. All the developments that took place in the
20th century have had implications for the A. Carry through B. Carry out
next century.
C. Carry D. Carry off
A. Consequences Directions: Each of the following sentences in
B. Interferences this section has a blank space and four words or
group of words are given after the sentence.
C. Feedback Select the most appropriate word or group of
D. Planning words for the blank space and indicate your
response on the Answer Sheet accordingly.
61. I _________ my old friend after twenty
years. 69. The fire brigade fought for four hours to
_______ the fire in the building.
A. Ran into B. Ran in
A. Put in B. Put out
C. Run in D. Run on
C. Put on D. Put off
62. The police could not establish how the
70. On his way to the capital, the minister
accident ________.
______ the eminent social worker at his
A. Came off B. Came about residence.
C. Came on D. Came out A. Called on B. Called
63. Where there is a _______, there is a way. C. Calling for D. Call off

A. Way B. Road Directions: In this section each item consists of


six sentences of a passage. The first and the sixth
C. Wing D. Will sentences are given in the beginning as S1 and S6
64. If I ________ an angel, I would solve the The middle four sentences in each have been
problems of people. jumbled up and labelled as P, Q, R and S You are
required to find out the proper sequence of the
A. Am B. Were four sentences and mark you response
C. Was D. Have accordingly on Answer Sheet.

65. The chairperson said that the group was 71. S1: Buddhism continued to spread into
________ of time. many lands of Asia during the period of 5th
and 6th century.
A. Running out B. Running
S6: he translated several scriptural
C. Running with D. Run out commentaries into Pali and wrote a work

PAGE 319
www.byjusexamprep.com

called the Visuddhimagga, which soon 73. S1: For several million years, humans lived
attained the status of a classic work of by hunting wild animals and gathering wild
Theravada doctrine and meditation. plants.
P: While this can be understood as a part of S6: As a result, conditions were favourable
larger processes of cultural interaction, for the growth of grasses such as wild
especially trade, a key role was played by barley and wheat.
monks.
P: This led to the development of farming
Q: We know a little bit about some of them, and pastoralism as a way of life.
but there must have been countless men
whose commitment to the Buddhist path Q: This change took place because the last
gave them the courage and determination ice age came to an end about 13,000 years
to preserve in the face of the long, hard ago and with that warmer, wetter
journey to India and back. conditions prevailed.

R: Buddhism had made its way to Sri Lanka R: Then, between 10,000 and 4,500 years
many centuries earlier, during the time of ago, people in different parts of the world
Ashoka, and a thriving Buddhist community learnt to domesticate certain plants and
soon took root. animals.

S: In the 5th century, the monk S: The shift from foraging to farming was a
Buddhaghosha travelled to Sri Lanka. major turning point in the human history.

The correct sequence should be The correct sequence should be

A. RQPS B. QRPS A. QSPR B. SPQR

C. PQRS D. PSRQ C. PSQR D. RPSQ


72. S1: All governments claim eternal 74. S1: Britain was the first country to
consistency and success. experience modern industrialization.
S2: Diplomacy offers choices, and those S6: This gave people a wider choice for
choices must be negotiated with other ways to spend their earnings and expanded
sovereign actors. the market for the sale of goods.
P: Choices involved uncertainty, risk and P: this meant that the kingdom had
immediacy; those who must take the common laws, a single currency and a
choices operate in the contemporary market that was not fragmented by local
political milieu. authorities and uneven taxation.
Q: And yet the essence of governance is Q: It had been politically stable since the
choice. seventeenth century, with England, Wales
and Scotland unified under a monarchy.
R: Nowhere is this more true than in foreign
policy decision-making. R: By then a large section of the people
received their income in the form of wages
S: Some even claim omniscience.
and salaries than in goods.
The correct sequence should be
S: By the end of the seventeenth century,
A. SQPR B. QSRP money was widely used as the medium of
exchange.
C. SRPQ D. RSPQ

PAGE 320
www.byjusexamprep.com

The correct sequence should be The correct sequence should be


A. QPSR B. PSQR A. PQRS B. RSPQ
C. RSQP D. SRQP C. QPSR D. SRQP
75. S1: Farming is the main production activity 77. S1: Biology is the youngest of the
in the village. formalized disciplines of natural science.
S6: The new ways of farming need less land,
S6: Life expectancy of human beings has
but much more capital.
dramatically changed over the years.
P: These have allowed the farmers to
P: However, the twentieth century and
produce more crops from the same amount
certainly the twenty-first century has
of land.
demonstrated the utility of biological
Q: Over the years there have been many knowledge in furthering human welfare, be
important changes in the way farming is it in health sector or agriculture.
practiced.
Q: The discovery of antibiotics, and
R: But in raising production, a great deal of synthetic plant-derived drugs, Anaesthetics
pressure has been put of land and other have changed medical practice on one hand
natural resources. and human health on the other hands.
S: This is an important achievement, since R: Applications of physics and chemistry in
land is fixed and scarce. our daily life also have a higher visibility
The correct sequence should be than those of biology.
A. QPSR B. RSPQ S: Progress in physics and chemistry
C. SRPQ D. PRSQ proceeded much faster than in biology.

76. S1: People in society need many goods and The correct sequence should be
services in their everyday life including A. QPRS B. PRQS
food, clothing, shelter, transport, etc.
C. RPQS D. SRPQ
S6: The teacher in the local school has the
skills required to impart education to the 78. S1: Biology is the study of life in its entirety.
students.
S6: Classical descriptive and clueless
P: A weaver may have some yarn, some biology found a theoretical framework in
cotton and other instruments required for the evolutionary theory of Darwin.
weaving cloth.
P: In later years, the focus was physiology
Q: A family farm may own a plot of land, and internal morphology or anatomy.
some grains, farming implements, maybe a
pair of bullock s and also the labour services Q: Darwinian ideas of evolution by natural
of the family members. changed the perception completely.

R: Every individual has some amount of the R: The growth of biology as a natural
goods and services that one would like to science during the last 1000 years is
use. interesting from many points of view.
S: In fact, the list of goods and services that S: One feature of this growth is changing
any individual need is so large that no emphasis from mere description of life
individual in society, to begin with, has all forms to identification and classification of
the things one needs. all recorded living forms.

PAGE 321
www.byjusexamprep.com

The correct sequence should be The correct sequence should be


A. RSPQ B. SPRQ A. SRPQ B. RPQS
C. QRPS D. PQRS C. PQRS D. QRSP
79. S1: “Every person carries in his head a Directions: Given below are some
mental model of the world – a subjective idioms/phrases followed by four alternative
representation of external reality,” writes meanings to each. Choose the response (a), (b),
Alvin Toffler in Future Shock. (c) or (d) which is the most appropriate
expression and mark your response in the
S6: When we begin to think we can do so
Answer Sheet accordingly.
only because our mind is already filled with
all sorts of ideas with which to think. 81. Call it a day
P: It organizes our knowledge and gives us A. End of the day
a place from which to argue.
B. Completion of work
Q: This mental model is, he says, like a giant
C. Stop doing something
filing cabinet.
D. A beautiful day
R: It contains a slot for every item of
information coming to us. 82. Rise to the occasion
S: As EF Schumacher says, “When we think, A. To celebrate a success in a difficult
we do not just think; we think ideas.” situation
The correct sequence should be B. To regret a situation which ended in
failure
A. PSRQ B. SPRQ
C. To succeed in dealing with a difficult
C. QRPS D. RQPS
situation
80. S1: Chinua Achebe was born in 1930 and
D. To motivate people to succeed in a
educated at the Government College in
difficult situation
Umuahia, Nigeria.
83. The green-eyed monster
S2: Chinua Achebe has written over twenty
books, including novels, stories, essays and A. Feeling of being joyous
collections of poetry, and won the Nobel
B. Feeling of being jealous
Prize for literature.
C. Feeling bad about happenings
P: During the Civil War in Nigeria, he
worked for the Biafran government service. D. Feeling lucky about something
Q: After the War, he was appointed Senior 84. Prime the pump
Research Fellow at the University of
Nigeria, Nsukka A. To do something in order to make
something succeed
R: He jointed the Nigerian Broadcasting
Company in Lagos in 1954, later becoming B. To do good things to succeed in life
its Director of External Broadcasting. C. To do something in order to get bad
S: He received a BA from London University things done
in 1953 and in 1956 he studied D. Asking people to do things to make
broadcasting in London at the BBC something succeed

PAGE 322
www.byjusexamprep.com

85. The pros and cons C. People who campaign for the
protection of tigers
A. The good and bad parts of a situation
D. A daredevil
B. Like and dislike of a situation
Directions: Each of the following items in this
C. A bad experience in an event section consists of a sentence, parts of which
D. A good moment of an event have been jumbled. These parts have been
labelled as P, Q, R and S. Given below each
86. A banana republic sentence are four sequences, namely (a), (b), C
and (d). You are required to re-arrange the
A. A small or poor country with a weak
jumbled parts of the sentence and mark your
government
response accordingly.
B. A small or poor country which produces
91. P. historical identity and a common
banana
descent
C. A country which has been occupied by Q. a group of people
a big country
R. is called an ethnic group
D. A country without any government
S. who share a common culture
87. The dog days
A. QSPR B. QRPS
A. Days celebrating dogs
C. PSQR D. RQPS
B. The bitter days 92. P. poetry is
C. The hottest days Q. and ideas
D. The coldest days R. powerful feelings
88. Eat like a bird S. the spontaneous overflow of
A. East fast A. SRQP B. PQRS

B. Eat very little C. RSQP D. PSRQ

C. Eat a lot 93. P. against the officer


Q. reason for the accusation
D. Pretending to be eating
R. there should have been
89. Lily-livered
S. who was in-charge at that time
A. Brave and courageous
A. RPSQ B. RQPS
B. Not brave
C. PQRS D. SPRQ
C. Comical
94. P. several of our food
D. Outrageous
Q. are being extensively cultivated
90. A paper Tiger R. and vegetable crops
A. Person or organization that appears S. hybrid varieties of
powerful, but actually is not
A. QRSP B. SPQR
B. Person or organization that acts like a
C. QPRS D. SPRQ
tiger

PAGE 323
www.byjusexamprep.com

95. P. in modern societies A. SPQR B. SQPR


Q. or merely suppressed C. QRSP D. QSRP
R. has class conflict 101. Each item in this section has a sentence
S. Been resolved with three underlined parts labelled as (a),
(b) and (c). Read each sentence to find out
A. RPSQ B. RSPQ whether there is any error in any
C. PRSQ D. QRSP underlined part and indicate your response
on the Answer Sheet against the
96. P. can express a view on
corresponding letter i.e., (a) or (b) or (c). If
Q. in which the electorate you find no error, your response should be
R. a particular issue of public policy indicated as (d).

S. a referendum is a vote A. At the beginning of the nineteenth


century
A. SQPR B. RPQS
B. female literacy was extremely lowed
C. QRSP D. PQRS
97. P. a majority of the vote C. in comparison to male literacy

Q. the party that received D. No error


R. of the government 102. Each item in this section has a sentence
with three underlined parts labelled as (a),
S. must take control
(b) and (c). Read each sentence to find out
A. QPSR B. PSRQ whether there is any error in any
C. RSPQ D. SQPR underlined part and indicate your response
on the Answer Sheet against the
98. P. before it starts corresponding letter i.e., (a) or (b) or (c). If
Q. of the government is you find no error, your response should be
indicated as (d).
R. the essential power
A. if I were you B. I would not go for
S. the power to manage conflict
A. RSPQ B. SQRP C. change of job D. No error

C. RQSP D. QRSP 103. Each item in this section has a sentence


with three underlined parts labelled as (a),
99. P. The cry of general public (b) and (c). Read each sentence to find out
Q. agenda in any country whether there is any error in any
underlined part and indicate your response
R. public policy making
on the Answer Sheet against the
S. is generally driven by corresponding letter i.e., (a) or (b) or (c). If
A. RQSP B. RPSQ you find no error, your response should be
indicated as (d).
C. PSRQ D. QRSP
A. When everything starts working for you
100. P. for long and
B. you will find
Q. the backbone of India
R. will continue to be the same C. things are achieve and delivered

S. agriculture has been D. No error

PAGE 324
www.byjusexamprep.com

104. Each item in this section has a sentence C. of a minor road traffic accident
with three underlined parts labelled as (a), D. No error
(b) and (c). Read each sentence to find out
whether there is any error in any 107. Each item in this section has a sentence
underlined part and indicate your response with three underlined parts labelled as (a),
on the Answer Sheet against the (b) and (c). Read each sentence to find out
corresponding letter i.e., (a) or (b) or (c). If whether there is any error in any
you find no error, your response should be underlined part and indicate your response
indicated as (d). on the Answer Sheet against the
corresponding letter i.e., (a) or (b) or (c). If
A. Evolutionary biology leave us you find no error, your response should be
B. distinctly pessimistic about the indicated as (d).
possibility A. Government Stock offers
C. that altruism can arise naturally among B. safety, liquidity and attractive returned
humans
C. for long duration
D. No error D. No error
105. Each item in this section has a sentence 108. Each item in this section has a sentence
with three underlined parts labelled as (a), with three underlined parts labelled as (a),
(b) and (c). Read each sentence to find out (b) and (c). Read each sentence to find out
whether there is any error in any whether there is any error in any
underlined part and indicate your response underlined part and indicate your response
on the Answer Sheet against the on the Answer Sheet against the
corresponding letter i.e., (a) or (b) or (c). If corresponding letter i.e., (a) or (b) or (c). If
you find no error, your response should be you find no error, your response should be
indicated as (d). indicated as (d).
A. The fascination with gold at least A. If the scheme would have been
B. seems to be a case were traditional implemented effectively
belief and B. All affected
C. modern finance would point the same C. would have benefitted
way D. No error
D. no error 109. Each item in this section has a sentence
106. Each item in this section has a sentence with three underlined parts labelled as (a),
with three underlined parts labelled as (a), (b) and (c). Read each sentence to find out
(b) and (c). Read each sentence to find out whether there is any error in any
whether there is any error in any underlined part and indicate your response
underlined part and indicate your response on the Answer Sheet against the
on the Answer Sheet against the corresponding letter i.e., (a) or (b) or (c). If
corresponding letter i.e., (a) or (b) or (c). If you find no error, your response should be
you find no error, your response should be indicated as (d).
indicated as (d). A. The auction, conducted by the bank
A. Scrolling thorough my social media B. will be price based
timeline C. using multiple period method
B. I hovered over a video D. No error

PAGE 325
www.byjusexamprep.com

110. Each item in this section has a sentence C. Pattern


with three underlined parts labelled as (a),
D. Perspectives
(b) and (c). Read each sentence to find out
whether there is any error in any 112. Which quality does the author here
underlined part and indicate your response advocate, to be a good reader?
on the Answer Sheet against the A. Being objective to the ideas of the
corresponding letter i.e., (a) or (b) or (c). If author
you find no error, your response should be
indicated as (d). B. Having preconceptions and
predispositions
A. After mysteriously expanding for
decades C. Having continuous routines
B. Antarctica’s sea ice cover D. Disagreeing with the author
C. starting melting 113. One of the ways to improve our reading
D. No error habits is to

Passage-II A. break the routine by changing the time


of reading.
When we pick up a newspaper, a book, or an
article, we come to our task with certain B. change the types of topics we read.
preconceptions and predispositions. We expect C. break the routine of reading the same
to find a specific piece of information or be newspaper.
presented with an argument or an analysis of
something, say, the likelihood of recession in the D. stop reading for some time and then
next six months or the reasons why children restart reading.
can’t read. We probably know a little about the 114. Our expectations and predispositions may,
book or article we are reading even before we however, blind us because
start. There was, after all, some reason why we
A. we may not get the actual ideas of the
chose to read one piece of writing rather than
author.
another.
Our expectations and predispositions may, B. we will get the actual ideas of the
however, blind us to what the article and its author.
author is actually saying. If, for example, we are C. we may disagree with the author.
used to disagreeing with the author, we may see
D. we will agree with all the ideas of the
only what we expect to see and not what is
author.
actually there. Day after day in our routine
pattern of life we expose ourselves to the same 115. According to the author, which one of the
newspaper, the same magazine, even books by following statements is not true?
authors with the same perspectives. In order to
A. Reader’s preconceptions influence
reflect on our reading habits and improve our
their reading.
skills we need to break out of this routine, step
back and look at what we are doing when we B. Readers have expectations when they
read. read an article or a book.
111. Which word in the passage means C. Readers look for specific information in
‘viewpoints’? any of their readings.
A. Preconceptions D. Readers assume that everything they
read will have new information.
B. Predispositions

PAGE 326
www.byjusexamprep.com

Directions: In this section you have a few short B. ideologies, hierarchies and power
passage. After each passage, you will find some structures.
items based on the passage. First, read a passage
C. excessive agricultural products.
and answer the items based on it. You are
required to select your answers based on the D. very high rewards.
contents of the passage and opinion of the
118. What kind of agriculture based societies
author only.
would emerge as civilizations?
Passage-I
A. Societies which achieved high
Not all agricultural societies become civilizations, productivity in agriculture had the
but no civilization can become one without opportunity to find time for other work
passing through the stage of agriculture. This is
B. Societies which depended on
because at some stage in the development of
agriculture completely moved to other
agriculture, as productivity improves, not all
fruitful work so as to move to many
people would need to be engaged in producing
places.
or procuring food. A significant number of
people could be freed up to pursue other C. Societies which transitioned from one
activities such as building walls or monuments stage of agriculture to another.
for new cities; making new tools, weapons and
jewellery; organizing long-distance trade; D. Societies which could not do
creating new artistic masterpieces; coming up agriculture for lack of resources moved
with new inventions; keeping accounts; and to other work.
perhaps constructing new public infrastructure 119. A significant number of people were sent to
such as irrigation canals that further improve the carry out other work from agriculture
productivity of agriculture, thus realizing even because
more people to do new things.
A. There were insufficient agricultural
This can happen, of course, only if a society that products.
has transitioned to high-productivity agriculture
has also, at some stage in its evolution, found a B. People were needed to build
way to channel the bonanza of free time into monuments, weapons, jewellery, etc.
other work fruitfully. In the ancient world, this C. There were sufficient agricultural
often involved creating new ideologies and new products.
hierarchies or power structures to coerce or
otherwise convince large groups of people to D. This enabled the development of
devote their time to the new tasks for very little civilizations.
reward. 120. Which one of the following statements is
116. Which word in the passage means true according to the author?
‘changeover’? A. Agriculture has always been part of all
A. Transitioned B. Channel civilizations.

C. Coerce D. Hierarchies B. Not all civilizations have undergone the


processes of agriculture.
117. People as groups were convinced to do new
work through C. Agriculture gave birth to new
civilizations.
A. reward, force and community
persuasions. D. Communities discontinued agriculture
to become civilizations

PAGE 327
www.byjusexamprep.com

General Knowledge
1. The Government of India has recently (d) Oligopoly 4. Many
constituted a civilian award in the name of producers
Sardar Vallabhbhai Patel in the field of selling similar
contribution to products
A. Unity and integrity of India
B. Art and culture Code:

C. Social work A. A-4 B-3 C-1 D-2


D. Entrepreneurship B. A-4 B-1 C-3 D-2
2. BRICS Summit, 2020 will be hosted by C. A-2 B-1 C-3 D-4
A. India B. China D. A-2 B-3 C-1 D-4
C. Russia D. Brazil 5. Suppose an agricultural laborer earns ₹ 400
3. Which one of the following was the host per day in her village. She gets a job to work
country for World Tourism Day, 2019? as a babysitter in a nearby town @ ₹ 700
per day. She chose to work as an
A. USA B. India agricultural laborer. Which one of the
C. Russia D. Canada following is the opportunity cost of the
agricultural laborer?
4. Match List-I with List-II and select the
correct answer using the code given below A. ₹1,100 B. ₹700
the Lists: C. ₹400 D. ₹300
List-I List-II 6. According to the Census 2011, in India,
(Market (Characteristic) what is the percentage of people
structure) (approximately) considered to be migrants
(a) Perfect 1. Only one (internal), i.e., now settled in a place
competition different from their previous residence?
producer
A. 25% B. 35%
selling one
commodity C. 45% D. 55%
(b) Monopoly 2. Few producers 7. Which one of the following is the nodal
selling similar agency in India for the United Nations
or almost Environment Programme?
similar
products A. The Ministry of Environment, Forest
(c) Monopolistic 3. Many and Climate Change
producers B. The Ministry of Science and Technology
Competition
selling C. The Ministry of Earth Sciences
Differentiated D. The Ministry of Home Affairs
products

PAGE 328
www.byjusexamprep.com

8. Which one of the following is the latest 13. The provisions of the Constitution of India
addition to the AYUSH group of health care pertaining to the institution of Panchayat
systems? do not apply to which one of the following
States?
A. Unani B. Siddha
A. Meghalaya
C. Sowa-Rigpa D. Reiki
B. Tripura
9. The formulation of policy with respect to
Intellectual Property Rights (IPRs) is the C. Assam
responsibility of
D. Goa
A. The Ministry of Law and Justice
14. The 11th Schedule of the Constitution of
B. The Department of Science and India distributes powers between
Technology
A. The Union and the State Legislatures
C. The Department for Promotion of
Industry and Internal Trade B. The State Legislatures and the
Panchayat
D. The Ministry of Human Resource
Development C. The Municipal Corporation and the
Panchayat
10. Which one of the following is the correct
sequence of formation of the Commissions D. The Gram Sabha and the Panchayat
starting from the earliest?
15. The power to legislate on all matters
A. Finance Commission, Planning relating to elections of Panchayats lies with
Commission, Investment Commission,
A. The Parliament of India
Election Commission
B. The State Legislatures
B. Election Commission, Planning
Commission, Finance Commission, C. The State Election Commission
Investment Commission
D. The Election Commission of India
C. Planning Commission, Election
Commission, Finance Commission, 16. The Hunter Commission (1882) appointed
Investment Commission to survey the state of education in India

D. Investment Commission, Finance A. Deprecated University education


Commission, Planning Commission, B. Overruled the Despatch of 1854
Election Commission
C. Enclosed the Despatch of 1854 with
11. The National Water Academy (NWA) is greater emphasis on primary education
located at
D. Criticized the grants-in-aid system of
A. Dehradun B. Hyderabad schooling
C. Bhopal D. Khadakwasla 17. The State of Hyderabad in the Deccan
12. Which one of the following rivers does not officially acceded to the Indian Union in the
drain into the Black Sea? year

A. Volga B. Dnieper A. 1948 B. 1950

C. Don D. Danube C. 1949 D. 1947

PAGE 329
www.byjusexamprep.com

18. Who founded the ‘Seva Samiti’ at Allahabad 24. Who among the following is the Chairman
in 1914? of the Economic Advisory Council to the
Prime Minister (EAC-PM)?
A. Hridayanath Kunzru
A. Ratan P. Watal B. Bibek Debroy
B. G. K. Gokhale
C. Ashima Goyal D. Sajjad Chinoy
C. Shri Ram Bajpai
25. The power of the Supreme Court to decide
D. T. B. Sapru in the case of a dispute between two or
19. The creation of a Federal Court in India was more States is called
advocated by which of the following A. Original jurisdiction
Acts/Commissions?
B. Inherent Jurisdiction
A. The Government of India Act, 1919
C. Plenary jurisdiction
B. The Lee Commission, 1923
D. Advisory jurisdiction
C. The Government of India act, 1935
26. Who among the following stated in the
D. The Indian Councils Act, 1909 Constituent Assembly that on 26th January
1950, India was going to enter a life of
20. The maiden trilateral naval exercise
contradictions?
involving India, Singapore and Thailand was
held at A. Dr. B. R. Ambedkar
A. Port Blair B. Chennai B. Jawaharlal Nehru
C. Panaji D. Kochi C. Mahatma Gandhi
21. Rustom-2, which crashed in Karnataka D. S. P. Mukherjee
recently, was a/an
27. The First Delimitation Commission in India
A. Fighter aircraft was constituted in
B. Helicopter A. 1949 B. 1950

C. Transport aircraft C. 1951 D. 1952

D. Unmanned aerial vehicle 28. The Ministry of Heavy Industries and Public
Enterprises consists of
22. The Vijaynagar Advanced Landing Ground
of the Indian Air Force, which was reopened A. The Department of Heavy Industry and
recently, is located in the Department for Promotion of
Industry and Internal Trade
A. Jammu and Kashmir
B. The Department of Public Enterprises
B. Arunachal Pradesh and the Department for Promotion of
C. Karnataka Industry and Internal Trade

D. Himachal Pradesh C. The Department of Scientific and


Industrial Research and the
23. Hilsa is the national fish of Department of Heavy Industry
A. Pakistan B. India D. The Department of Heavy Industry and
the Department of Public Enterprises
C. Bangladesh D. Nepal

PAGE 330
www.byjusexamprep.com

29. What is the ground on which the Supreme II. CIL operates through 82 mining areas
Court can refuse relief under Article 32? spread over twenty provincial States of
India.
A. The aggrieved person can get remedy
from another court III. CIL is the single largest coal-producing
company in the world.
B. That disputed facts have to be
investigated Select the correct answer using the code
given below.
C. That no fundamental right has been
infringed A. 1 only B. 1 and 3 only
D. That the petitioner has not asked for C. 2 and 3 only D. 1, 2 and 3
the proper writ applicable to his/her
35. The College of Military Engineering
case
affiliated to Jawaharlal Nehru University is
30. Which one of the following Articles were situated at
defended by Dr. B. R. Ambedkar on the
A. New Delhi B. Dehradun
plea that it would be used as ‘a matter of
last resort’? C. Nainital D. Pune
A. Article 352 B. Article 359 36. Who among the following was the first to
arrive in Africa as traders that eventually
C. Article 356 D. Article 368
led to European colonization of Africa?
31. The major part of Central Asia is dominated
A. French B. Spanish
by which one of the following language
families? C. Portuguese D. Dutch
A. Indo-European B. Sino-Tibetan 37. ‘Gandhi Solar Park’ is located at
C. Austric D. Altaic A. New York B. Vladivostok
32. Which one of the following is not among C. Thimphu D. Houston
the principal languages of Jammu and
38. The Global Goalkeeper Award is given by
Kashmir?
A. The Bill and Melinda Gates Foundation
A. Urdu B. Gujari
B. The United Nations Environment
C. Koshur D. Monpa
Programme
33. Which one of the following climatic types is
C. The Kellogg School of Management
found in Central Spain?
D. The World Meteorological Organization
A. Subarctic
39. Which one of the following is India’s official
B. Mediterranean dry hot summer
entry for the Best International Feature
C. Subtropical Steppe Film category in the 92nd Academy Awards?
D. Humid continental warm summer A. Bulbul Can Sing
34. Which of the following statements with B. Super Deluxe
regard to Coal India Limited (CIL) is/are
C. Gully Boy
true?
D. And The Oscar Goes To
I. CIL has its headquarter at Kolkata.

PAGE 331
www.byjusexamprep.com

40. ‘Naropa’ is an annual festival of A. Sukkur and Rohri Hills


A. Sikkim B. Khetri in Rajasthan
B. Ladakh C. Chagai Hills
C. Arunachal Pradesh D. Hills of Baluchistan
D. Nagaland 46. Consider the following statements about
41. What was the code name given to the first Stone Age in India:
ever tri-service military exercise between I. Different periods are identified on the
India and USA? basis of the type and technology of
A. Lion Triumph stone tools.

B. Elephant Triumph II. There are no regional variations in the


type and technology of tools in
C. Tiger Triumph different periods.
D. Bison Triumph III. Stone Age cultures of different periods
42. The ‘Tattvabodhini Sabha’ was established evolved uniformly in a neat unilinear
by fashion all over the subcontinent.

A. Devendranath Tagore in 1839 Which of the statements given above is/are


correct?
B. Keshab Chandra Sen in 1857
A. 1 only B. 1 and 2 only
C. Akshay Kumar Datta in 1850
C. 3 only D. 1, 2 and 3
D. Dwarakanath Tagorer in 1840
47. Which one of the following Articles of the
43. Who believed that the Russian designs
Constitution of India protects a person
were ‘an imminent peril to the security and
against double jeopardy?
tranquility’ of the Indian Empire in 1836?
A. Article 20 B. Article 21
A. Lord Auckland
C. Article 22 D. Article 23
B. Lord Palmerston
48. Which one of the following was added as a
C. Lord Canning
fundamental duty through the Constitution
D. Alexander Burnes (86th Amendment) Act, 2002?
44. The work Siyar-ul-Mutakherin, which A. To strive towards excellence in
describes the Battle of Plassey, 1757, was individual and collective activity
written by
B. To provide opportunities for education
A. Salabat Jung to one’s child between the age of 6 and
B. Qasim Khan 14 years

C. Ghulam Husain C. To work for the welfare of women and


children
D. Ram Mohan Roy
D. To promote peace and harmony
45. From which one of the following factory
sites were limestone and chert blades mass 49. Which of the following are considered to be
produced and sent to various Harappan the four pillars of human development?
settlements in Sindh?

PAGE 332
www.byjusexamprep.com

A. Equity, inclusion, productivity and C. Subtropical high-pressure cells


empowerment
D. Equatorial troughs
B. Equity, productivity, empowerment
and sustainability 54. In India, how many States/Union Territories
have more than two international
C. Productivity, gender, inclusion and boundaries?
equity
A. 1 B. 2
D. Labour, productivity, inclusion and
equity C. 3 D. 4
50. Which one of the following is not a feature 55. Which of the following are warm ocean
of the Ayushman Bharat Scheme? currents?
A. There is no cap on family size and age. A. Kuroshio and California Current
B. The scheme includes pre- and post- B. North Atlantic Drift and Brazil Current
hospitalization expenses.
C. Canaries and Benguela Current
C. A defined transport allowance per
hospitalization will also be paid to the D. West Wind Drift and Falkland Current
beneficiary.
56. Which one of the following groups of cities
D. The scheme provides a benefit cover of does not have Sclerophyll as its natural
₹ 10 lakh per family. vegetation cover?
51. Overseas Indians can exercise franchise in A. Valparaiso and Cape Town
an election to the Lok Sabha under which of
the following conditions? B. Lisbon and Perth

I. They must be citizens of India. C. Los Angeles and Adelaide

II. Their names must figure in the D. Las Vegas and Queensland
electoral roll. 57. Geomorphic factors influencing plant and
III. They must be present in India to vote. animal distributions are
Select the correct answer using the code A. Slope angle and relief only
given below.
B. Slope aspect and relative relief
A. 1, 2 and 3 B. 2 and 3 only
C. Slope angle, slope aspect and relief
C. 1 and 2 only D. 1 only
D. Slope angle, slope aspect and relative
52. Which one of the following soils is relief
characterised by very high content of
organic matter? 58. Which one of the following statements
about the famous text of Panchatantra is
A. Vertisol B. Histosol correct?
C. Gelisol D. Spodosol
A. It is a philosophical text reflecting the
53. In the Hadley cell thermal circulation, air debates of the time and refuting rival
rises up and finally descends at positions.
A. Intertropical convergence zone B. It is a text ushering in linguistics as a
B. Doldrums formal science.

PAGE 333
www.byjusexamprep.com

C. It is a text discussing developments in A. Abd al-Samad B. Abul Hassan


various spheres of natural sciences.
C. Dasavant D. Bishandas
D. It is a text showing through illustration
63. Amitabh Bachchan was recently conferred
what should and should not be done.
with the prestigious Dada Saheb Phalke
59. Which of the following rulers were Award. Who among the following was the
identified through matronymics (names first recipient of the award?
derived from that of the mother)? A. Prithviraj Kapoor
A. Mallas of Pava B. Devika Rani
B. Videhas of Mithila C. Sohrab Modi
C. Yaudheyas D. Naushad
D. Satavahanas 64. Recently the Reserve Bank of India has
imposed limitations, initially for a period of
60. Which one of the following pairs is not
six months, on the withdrawal of amount
correctly matched?
by account holders of which one of the
A. Kuddapah-kar : Rocky Wastelands following banks?

B. Nancai : Wet fields A. IndusInd Bank

C. Puncai : Dry fields B. Dhanlaxmi Bank

D. Tottakal : Garden lands C. Punjab and Maharashtra Cooperative


Bank
61. Which of the following statements about
‘Mughal Mansab’ system are correct? D. South Indian Bank
65. Greta Thunberg, a teenaged environment
1. ‘Zat’ rank was an indicator of a
activist who was in the news recently, hails
Mansabdar’s position in the imperial
from
hierarchy and the salary of the
Mansabdar. A. Sweden B. Germany
2. ‘Sawar’ rank indicated the number of C. USA D. Canada
horsemen the Mansabdar was
66. In September 2019, which one of the
required to maintain.
following travel giants declared itself
3. In the seventeenth century, Mansabdars bankrupt?
holding 1000 or above ‘Sawar’ rank A. Expedia B. Cox & Kings
were designated as nobles (Umara).
C. SOTC D. Thomas Cook
Select the correct answer using the code
given below. 67. Who among the following won the Best
Men’s Player Award of FIFA Football
A. 1 and 2 only B. 1 and 3 only Awards, 2019?
C. 2 and 3 only D. 1, 2 and 3 A. Cristiano Ronaldo
62. The famous Mughal painting, depicting B. Virgil van Dijk
Jahangir embracing the Safavid king Shah
C. Lionel Messi
Abbas, was painted by which one of the
following Mughal painters? D. Xavi

PAGE 334
www.byjusexamprep.com

68. In economics, if a diagram has a line passing 72. The “Basel Convention’ is aimed at
through the origin and has 45° angle with protecting human health and environment
either axis and it is asserted that along the against the adverse effect of which of the
line X = Y, what is tacitly assumed? following?
A. Both variables are pure numbers. A. Hazardous wastes
B. Both variables are in the same unit. B. Persistent Organic Pollutants (POPs)

C. Both variables are in different units. C. Mercury

D. At least one variable is a pure number. D. Chemicals and pesticides

69. The Cash Reserve Ratio refers to 73. Saubhagya, a Government of India Scheme,
relates to which of the following areas?
A. The share of Net Demand and Time
Liabilities that banks have to hold as A. Achieving universal household
liquid assets electrification

B. The share of Net Demand and Time B. Providing clean cooking fuel to poor
Liabilities that banks have to hold as households
balances with the RBI C. Rationalizing subsidies on LPG
C. The share of Net Demand and Time D. Stopping female foeticide
Liabilities that banks have to hold as
74. Which of the following statements is not
part of their cash reserves
correct regarding the Members of
D. The ratio of cash holding to reserves of Parliament Local Area Development
banks Scheme (MPLADS)?
70. Which one of the following is not correct A. Members of the Parliament (MPs)
about Repo rate? sanction, execute and complete works
under the scheme.
A. It is the interest rate charged by the
Central Bank of overnight loan. B. Nominated Members of the Parliament
can recommend works for
B. It is the interest rate paid by the implementation anywhere in the
commercial banks on overnight country.
borrowing.
C. The scheme is fully funded by the
C. It is the interest rate agreed upon in the Government of India.
loan contract between a commercial
bank and the Central Bank. D. The annual entitlement per MP is ₹ 5
crore.
D. It is the cost of collateral security.
75. Which one of the following is the earliest
71. Which one of the following is the biggest launched scheme of the Government of
cause of the incidence of migration of India?
female persons in India?
A. Deendayal Antyodaya Yojana
A. Employment
B. Pradhan Mantri Gram Sadak Yojana
B. Education
C. Saansad Adarsh Gram Yojana
C. Marriage
D. Deendayal Upadhyaya Grameen
D. Business Kaushalya Yojana

PAGE 335
www.byjusexamprep.com

76. Which of the following statements with 81. The number of people per unit area of
regard to the functioning of the Panchayats arable land is termed as
is not correct?
A. Agricultural density
A. Panchayats may levy, collect and
appropriate taxes, duties, tolls, etc. B. Arithmetic density

B. A person who has attained the age of C. Physiological density


25 years will be eligible to be a member
D. Economic density
of a Panchayat.
C. Every Panchayat shall ordinarily 82. In the field of tourism, which one of the
continue for five years from the date of following Indian States is described as ‘One
its first meeting. State Many Worlds’?

D. A Panchayat reconstituted after A. Assam B. West Bengal


premature dissolution shall continue C. Karnataka D. Rajasthan
only for the remainder of the full
period. 83. Nyishi tribe is found mainly in
77. Which one of the following is not an A. Andaman and Nicobar
objective of the MGNREGA?
B. Arunachal Pradesh
A. Providing up to 100 days of skilled
labour in a financial year C. Nilgiri-Kerala
B. Creation of productive assets D. Kashmir Valley
C. Enhancing livelihood security 84. In the region of the eastern shore of
D. Ensuring empowerment to women Adriatic Sea, a cold and dry wind blowing
down from the mountain is known as
78. Which one of the following is not a land-use
category? A. Mistral B. Bora
A. Forestland C. Bise D. Blizzard
B. Pasture land 85. Who among the following Mongol
C. Marginal land leaders/commanders did not cross Indus to
attack India?
D. Barren and wasteland
A. Chenghiz Khan B. Tair Bahadur
79. Which one of the following is not a type
of commercial agriculture? C. Abdullah D. Qutlugh Khwaja
A. Dairy farming 86. Who among the following Sultans
B. Grain farming succeeded in finally breaking and
destroying the power of Turkan-i-
C. Livestock ranching Chihalgani?
D. Intensive subsistence agriculture
A. Iltutmish
80. Which one of the following rivers joins
B. Balban
Ganga directly?
A. Chambal B. Son C. Alauddin Khalji

C. Betwa D. Ken D. Muhammad bin Tughluq

PAGE 336
www.byjusexamprep.com

87. Who among the following was one of the 92. Which one of the following cell organelles
founders of the Indian Society of Oriental contains DNA?
Art?
A. Golgi apparatus
A. Rabindranath Tagore
B. Mitochondrion
B. Abanindranath Tagore
C. Lysosome
C. Dwarakanath Tagore
D. Endoplasmic reticulum
D. Bankim Chandra Chattopadhyaya 93. Which of the following endocrine glands is
88. Which one among the following was not found in pair in humans?
demanded by the All India Depressed A. Adrenal B. Pituitary
classes Leaders’ Conference at Bombay in
1931? C. Testis D. Ovary

A. Universal adult suffrage 94. Which one of the following is the correct
sequence of organs that occur in the path
B. Separate electorates for untouchables of urine flow in the human body?
C. Reserved seats for the minorities A. Kidney, ureter, urinary bladder, urethra
D. A unitary State in India B. Kidney, urinary bladder, ureter, urethra
89. Which one of the following Acts reserved C. Kidney, ureter, urethra, urinary bladder
seats for women in Legislatures in
accordance with the allocation of seats for D. Urinary bladder, kidney, urethra, ureter
different communities? 95. The electromagnetic waves, which are used
A. The Government of India Act, 1858 for satellite communication, are

B. The Indian Councils Act, 1909 A. infrared radiations


B. ultraviolet radiations
C. The Government of India Act, 1919
C. radio waves
D. The Government of India Act, 1935
D. visible lights
90. On 31st December, 1929, in which one of
the following Congress Sessions was the 96. The resistivity p of a material may be
proclamation of Purna Swaraj made? expressed in units of
A. Ahmedabad B. Calcutta A. ohm B. ohm/cm
C. Lahore D. Lucknow C. ohm-cm D. ohm-cm2
91. Who among the following scientists 97. If x is the temperature of a system in Kelvin
introduced the concept of immunization to and y is the temperature of the system in
the medical world? °C, then the correct relation between them
is
A. Edward Jenner
A. x = 273 – y
B. Robert Koch
B. x = 273 + y
C. Robert Hooke
C. x = 173 + y
D. Carl Linnaeus
D. x = 173 – y

PAGE 337
www.byjusexamprep.com

98. Silver articles become black after some 102. Which one of the following is the first
time when exposed to air because enzyme to mix with food in the digestive
tract?
A. Silver gets oxidized to silver oxide
A. Trypsin
B. Silver reacts with moist carbon dioxide
in the air to form silver carbonate B. Cellulose
C. Silver reacts with sulphur in the air to C. Pepsin
form a coating of silver sulphide
D. Amylase
D. Silver reacts with nitrogen oxides in the
air to form silver nitrate 103. Which one of the following is the ‘energy
currency’ for cellular processes?
99. The number of water molecules associated
with copper sulphate molecule to form A. Glucose
crystals is
B. ATP
A. 2 B. 4
C. ADP
C. 5 D. 6
D. Pyruvic acid
100. Match List-I with List-II and select the
correct answer using the code given below 104. When air is blown from the mouth into a
the Lists: test tube containing limewater, the
limewater turns milky. This is due to the
List-I List-II presence of
(Name) (Formula)
A. Water vapour
A. Bleaching 1. NaHCO3
powder B. Oxygen

B. Baking soda 2. Na2CO3.10H2O C. Carbon dioxide

C. Washing 3. Ca(OH)2 D. Carbon monoxide


soda
105. Which one of the following is the correct
D. Slaked lime 4. CaOCl2 sequence of events during sexual
reproduction in plants?
A. Seeding, formation of embryo,
Codes:
pollination, fertilization, division of the
A. A-4 B-1 C-2 D-3 zygote
B. A-4 B-2 C-1 D-3 B. Formation of embryo, seedling,
pollination, fertilization, division of the
C. A-3 B-2 C-1 D-4
zygote
D. A-3 B-1 C-2 D-4
C. Pollination, fertilization, division of the
101. Mahatma Gandhi’s Dandi March, a great zygote, the formation of embryo,
event in India freedom struggle, was seedling
associated with
D. Seedling, the formation of the embryo,
A. Iron B. Sodium chloride division of zygote, pollination,
C. Sulphur D. Aluminium fertilization

PAGE 338
www.byjusexamprep.com

106. Which one of the following animals has a C. At an angle of 60° with each other
three-chambered heart?
D. At an angle of 45° with each other
A. Scoliodon
112. Which one of the following is not true
B. Salamander about the image formed by a plane mirror?
C. Pigeon A. It is of the same size as the subject.
D. Human being B. It is laterally inverted.
107. Which one of the following is not an C. It is real image.
epidemic disease?
D. It is formed as far behind the mirror as
A. Cholera
the object is in front.
B. Malaria
113. Which one of the following is not a property
C. Smallpox of the X-rays?
D. Elephantiasis A. They are deflected by electric fields.
108. The unit of the force constant k of a spring B. They are not deflected by magnetic
is fields.
A. N-m B. N/m C. They have high penetration length in
C. N-m 2
D. N/m2 matter.
109. Which one of the following types of D. Their wavelength is much smaller than
radiation has the shortest wavelength? that of visible light.
A. Radio waves 114. The property of the sound waves that
determines the pitch of the sound is its
B. Visible light
C. Infrared (IR) A. Frequency B. Amplitude

D. Ultraviolet (UV) C. Wavelength D. Intensity

110. If the speed of light in air is 3 x 108 m/s, then 115. Match List-I with List-II and select the
the speed of light in a medium of refractive correct answer using the code given below
3 the Lists:
index is
2 List-I List-II
A. 2 x 10 m/s 8 (Compound) (Use)

9 A. Boric acid 1. Antiseptic


B. x 108 m/s
4 B. Citric acid 2. Food
3 preservative
C. x 108 m/s
2 C. Magnesium 3. Antacid
D. 3 x 10 m/s 8 Hydroxide
111. In a periscope, the two plane mirrors are D. Acetic acid 4. Pickle
kept
A. Parallel to each other
A. A-1 B-2 C-3 D-4 B. A-1 B-3 C-2 D-4
B. Perpendicular to each other
C. A-4 B-3 C-1 D-2 D. A-4 B-2 C-3 D-1

PAGE 339
www.byjusexamprep.com

116. Which one of the following acids is C. Cu + PbCl2→ CuCl2 + Pb


produced in human stomach?
D. Mg + CuSO4→ MgSO4 + Cu
A. Formic acid
119. The elements of which of the following
B. Sulphuric acid pairs are isobars?
C. Nitric acid A. 1
and 13 H
1H
D. Hydrochloric acid 1
B. 1H and 12 H
117. A solution having pH equal to zero is known
12 14
as C. 6C and 6C

A. Highly alkaline solution D. 40


and 40
18 Ar 20 Ca
B. Highly acidic solution
120. Scattering of α-particles by a thin gold foil
C. Weakly acidic solution suggests the presence of
D. Neutral solution A. Electron in an atom
118. Which one of the following chemical B. Proton in an atom
reactions is not feasible?
C. Positively charged nucleus at the
A. Fe + CuSO4→ FeSO4 + Cu centre of an atom
B. Zn + CuSO4→ ZnSO4 + Cu D. Isotopes of gold

PAGE 340
www.byjusexamprep.com

Elementary Mathematics
1. A circle of diameter 8 cm is placed in such a II. Area of ΔQAB is equal to that of ΔPCQ
manner that it touches two perpendicular
Select the correct answer using the code
lines. Then another smaller circle is place in
given below :
the gap such that it touches the lines and
the circle. What is the diameter of the A. 1 only B. 2 only
smaller circle?
C. Both 1 and 2 D. Neither 1 nor 2
A. 4(3- 2 ) cm B. 4(3-2 2 ) cm 6. The lengths of sides of a triangle are 3x,
C. 8(3- 2 ) cm D. 8(3-2 2 ) cm 4 y , 5 3 z , where 3x <4 y < 5 3 z . If
one of the angles is 90°, then what are the
2. A square and a rectangle have equal areas.
minimum integral values of x, y, z
If one side of the rectangle is of length
respectively?
numerically equal to the square of the
length of the side of the square then the A. 1, 2, 3 B. 2, 3, 4
other side of the rectangle is
C. 1, 1, 1 D. 3, 4, 5
A. square root of the side of the square
7. What is the maximum number of circum-
B. half the side of the square circles that a triangle can have?
C. of unit length A. 1 B. 2
D. double the side of the square C. 3 D. Infinite
3. The length and breadth of a rectangle are in 8. If an arc of a circle of radius 6 cm subtends
the ratio 4: 3. Then what is the ratio of the a central angle measuring 30°, then which
area of the triangle formed by the parts of one of the following is an approximate
the diagonals with a long side to the area of length of the are?
the triangle formed by the parts of
diagonals with a short side? A. 3.14 cm B. 2.15 cm

A. 3 : 4 B. 4 : 3 C. 2.14 cm D. 2 cm

C. 16 : 9 D. 1 : 1 9. A ladder 5 m long is placed in a room so as


to reach a point 4.8 m high on a wall and on
4. ABCD is a parallelogram where AC and turning the ladder over to the opposite side
BD are the diagonals. If ∠BAD = 60°, ∠ ADB of the wall without moving the base, it
= 90°, then what is BD2 equal to ? reaches a point 1.4 m high. What is the
3 3 breadth of the room?
A. AB2 B. AB2
5 4 A. 5.8 m B. 6 m
1 2 C. 6.2 m D. 7.5 m
C. AB2 D. AB2
2 3
10. What is the area of the largest square
5. A line through the vertex A of a plate cut from a circular disk of radius one
parallelogram, ABCD meets DC in P and BC unit ?
produced in Q. If P is the mid-point of DC,
then which of the following is/are correct? A. 4 square units B. 2 2 square units
I. Area of ΔPDA is equal to that of ΔPCQ C. π square units D. 2 square units

PAGE 341
www.byjusexamprep.com

11. ABCD is a quadrilateral such that AD = DC = 14.


CA = 20 units, BC = 12 units and ∠ABC = 90°.
What is the approximate area of the
quadrilateral ABCD?
A. 269 sq units B. 300 sq units
C. 325 sq units D. 349 sq units
12. y z
In the given figure, if = 6 and = 5, then
x x
what is the value of x?
A. 45° B. 30°
C. 15° D. 10°
15. ABCD is a trapezium, where AB is
parallel to DC. If AB = 4 cm, BC = 3 cm,
CD = 7 cm and DA = 2 cm, then what is the
What is the area of the shaded region in the area of the trapezium?
given figure, if the radium of each of the
2
circles is 2 cm? A. 22 cm2
3
A. 4 3 – 2π cm2 B. 3 – π cm2
3
B. 22 cm2
 2
C. 3− cm2 D. 2π – 2 3 cm2
2
C. 22 3 cm2
13.
22 2
D. cm2
3
16.

In the given figure, what is the area of the


shaded region?

A. 9 (  − 3 ) sq units
Angles are shown in the given figure. What
B. 3 ( 4  − 3 3 ) sq units is value of ∠1 +
?
C. 3 ( 3 − 4 3 ) sq units A. 240° B. 360°
D. 9 ( 3 −  ) sq units C. 540° D. 720°

PAGE 342
www.byjusexamprep.com

17. What is the approximate area of the shaded


region in the figure given?
A. 15.3 cm2 B. 25.5cm2
C. 28.4cm2 D. 30.5cm2
20. The number 2 × 3 × 5 × 7 × 11 + 1 is
A. a prime number
B. not a prime, but power of a prime
C. not a power of a prime, but a
composite even number
In the given figure PQ is parallel to RS,
95°, = 110°, and ∠ABC = x°. D. not a power of a prime, but a
Then what is the value of x? composite odd number

A. 15 B. 25 21. How many digits are there in (54)10 ? (Given


that log102=0.301 and log103=0.477)
C. 30 D. 35
A. 16 B. 18
18.
C. 19 D. 27
22. In a competitive examination, 250
students have registered. Out of these, 50
students have registered for Physics, 75
students for Mathematics and 35 students
for both Mathematics and Physics. What is
the number of students who have
registered neither for Physics nor for
In the given figure, there are three semi Mathematics ?
circles ABC, AEF and CDF. The distance
between A and C is 25 units and F is the A. 90 B. 100
mid-point of AC. What is the total area C. 150 D. 160
of the three semi circles?
23. Which one of the following is the largest
A. 924 square units number among 22222, 22222, 22222, 22222 ?
B. 824 square units A. 22222 B. 22222
C. 624 square units C. 22222 D. 22222
D. 369 square units 24. A two-digit number is 9 more than four
19. times of the number obtained by
interchanging its digits. If the product of
digits in the two-digit number is 8, then
what is the number?
A. 81 B. 42
C. 24 D. 18

PAGE 343
www.byjusexamprep.com

25. If α and ß are the roots of the quadratic A. B.


equation x2 + kx – 15 = 0 such that α – ß = 8,
then what is the positive value of k? C. D.
A. 2 B. 3
34. Students of a class are made to sit in rows
C. 4 D. 5 of equal number of chairs. If number of
26. If (x + k) is the HCF of x2 + 5x + 6 and x2 + 8x students is increased by 2 in each row, then
+ 15, then what is the value of k ? the number of rows decreases by 3. If
number of students is increased by 4 in
A. 5 B. 3
each row, then the number of rows
C. 2 D. 1 decreases by 5. What is the number of
students in the class?
27. If x2+ 9y2=6xy, then what is y:x equal to?
A. 100 B. 105
A. 1:3 B. 1:2
C. 110 D. 120
C. 2:1 D. 3:1
35. What is the median of the data 3, 5, 9,
28. What is equal to? 4, 6, 11, 18?
A. 6 B. 6.5
A. 1 B. –1
C. 7 D. 7.5
C. 0 D. 2am-n
36. In a pi-diagram there are three sectors. If
33 66 the ratio of the angles of the sectors is 1 : 2
29. If x = 2 , y = and z = , then which
one of the following is correct? : 3, then what is the angle of the largest
sector?
A. Y < x < z B. z< x < y
A. 200° B. 180°
C. z< y < x D. x< y <z
C. 150° D. 120°
30. If log x = 1.2500 and y = xlogx, then what is
log y equal to ? 37. The maximum marks in a Test are
converted from 250 to 50 for the purpose
A. 4.2500 B. 2.5625 of an Internal Assessment. The highest
C. 1.5625 D. 1.2500 marks scored were 170 and lowest marks
were 70. What is the difference between
31. If the area of a square is 2401x4 + 196x2 + 4, the maximum and minimum marks scored
then what is its side length? in the Internal Assessment?
A. 49x2 + 3x + 2 B. 49x2 – 3x + 2 A. 15 B. 17
C. 49x2 + 2 D. 59x2 + 2 C. 20 D. 24
32. If x varies as yz, then y varies inversely as Directions for the following three items:
x Read the following information and answer the
A. xz B.
z three items that follow:

z 1 The following data presents count of released


C. D. convicts who have served prison terms (X), those
x (xz)
who have received some educational or
33. What is the point on the xy-plane satisfying technical training during their term (Y) and those
5x + 2y = 7xy and 10x + 3y = 8xy? who were offered Company placement (Z)

PAGE 344
www.byjusexamprep.com

respectively, from six different jails A, B, C, D, E two towers so that the tip of the towers and
and F, in the year 2010. tip of the pole are also on the same line. If
the distance of the pole from the shorter
tower is 75 feet, then what is the height of
the taller tower (approximately)?
A. 85 feet B. 110 feet
C. 125 feet D. 140 feet
44. What is the value of
sin26°+sin212°+sin218°+….+sin284°+sin290°?
38. Jails with highest and smallest percentage
A. 1 B. 2
of trained convicts are respectively
C. 4 D. 8
A. F and D B. D and F
45. Two circles of radii 20 cm and 16 cm
C. C and A D. D and A intersect and the length of common chord
39. Jail with highest placement rate of trained is 24 cm. If d is the distance between their
convicts is centres, then which one of the following is
correct ?
A. F B. D
A. d < 26 cm
C. B D. A
B. 26 cm < d < 27 cm
40. Jails from which more than half of the
trained convicts are offered jobs, are C. 27 cm < d < 28 cm

A. A, B and C B. A, B and D D. d > 28 cm

C. A, D and E D. A, E and F 46. A rectangle is 48 cm long and 14 cm wide. If


the diagonal makes an angle θ with the
41. In a water tank there are two outlets. It longer side, then what is (secθ + cosecθ)
takes 20 minutes to empty the tank if both equal to?
the outlets are opened. If the first outlet is
opened, the tank is emptied in 30 minutes. 775 725
A. B.
What is the time taken to empty the tank 168 168
by second outlet? 375 325
C. D.
A. 30 minutes B. 40 minutes 84 84
C. 50 minutes D. 60 minutes 47. Two circles touch internally. The sum of
their areas is 136π cm2 and distance
 1 
42. If  x 8 + 2  = 47, what is the value of between their centres is 4 cm. What are the
 x  radii of the circles ?
 6 1  A. 11 cm, 7 cm B. 10 cm, 6 cm
x + 6  = ?
 x 
C. 9 cm, 5 cm D. 8 cm, 4 cm
A. 36 B. 27 48. If area of a circle and a square are same,
C. 18 D. 9 then what is the ratio of their perimeters?
43. On a plane area, there are two vertical A. 2  B. 
towers separated by 100 feet apart. The
shorter tower is 40 feet tall. A pole of length  
C. D.
6 feet stands on the line joining the base of 2 4

PAGE 345
www.byjusexamprep.com

49. The thickness of cylinder is 1 foot, the inner 54. A right circular cylinder just encloses a
radius of the cylinder is 3 feet and height is sphere. If p is the surface area of the sphere
7 feet. To paint the inner surface it requires and q is the curved surface area of the
one litre of a particular colour. How much cylinder, then which one of the following is
quantity of the same colour is required to correct?
paint all the surfaces of the cylinder?
A. p = q B. p = 2q
7 3
A. litre B. litre C. 2p = q D. 2p = 3q
3 2
8 10 55.
C. litre D. litre
3 3
50. Suppose a region is formed by removing a
sector of 20° from a circular region of radius
30 feet. What is the area of this new
region?
A. 150π square feet
B. 5150π square feet
C. 650π square feet
D. 850π square feet Let PQRS be the diameter of a circle of
radium 9 cm. The length PQ, QR and RS are
51. How many cubic metres of the earth is to
equal. Semi-circle is drawn with QS as
be dug out to dig a well of radius 1.4 m and
diameter (as shown in the given figure).
depth 5 m ?
What is the ratio of the shaded region to
A. 30.2 cubic metre that of the unshaded region?
B. 30.4 cubic metre A. 25 : 121 B. 5 : 13
C. 30.6 cubic metre
C. 5 : 18 D. 1 : 2
D. 30.8 cubic metre
56.
52. If the diagonals of a rhombus are x and y,
then what is its area?
A. B.

C. xy D. x2-y2
53. Out of 4 identical balls of radius r, 3 balls are
placed on a plane such that each ball
touches the other two balls. The 4th ball is
placed on them such that this ball touches
all the three balls. What is the distance of
centre of 4th ball from the plane?
In the given figure AB is parallel to CD and
2 3 +2 2 AC is parallel to BD. If ∠EAC = 40°, ∠FDG =
A. 2 r unit B. r unit
3 2 55°, ∠HAB = x°, then what is the value of x?
r 3 +2 2 A. 85 B. 80
C. unit D. r unit
3−2 2 3
C. 75 D. 65

PAGE 346
www.byjusexamprep.com

57. A. 3 B. 13
C. 39 D. 117
61. What are the values of p and q respectively,
if (x – 1) and (x + 2) divide the polynomial
x3 + 4x2 + px + q?
A. 1, –6 B. 2, –6
C. 1, 6 D. 2, 6
Consider the following statements with 62. If 5x+1 – 5x-1 = 600, then what is the value of
reference to the given figure: 102x ?
I. The sum of the areas of ΔAOD and ΔBOC A. 1 B. 1000
is equal to the sum of the areas of ΔAOB
and ΔDOC. C. 100000 D. 1000000

II. ∠AOD = ∠BOC 63. If f(x) is divided by (x – α) (x – ß) where


α ≠ ß, then what is the remainder ?
III. AB + BC + CD + DA > AC + BD
Which of the above statements are A.
correct?
A. 1 and 2 only B. 2 and 3 only B.

C. 1 and 3 only D. 1, 2 and 3


C.
58. Two unequal pairs of numbers satisfy the
following conditions :
D.
I. The product of the two numbers in
each pair is 2160 64. If the points P and Q represent real
II. The HCF of the two numbers in each numbers 0.73 and 0.56 on the number
pair is 12. line, then what is the distance between P
and Q ?
If x is the mean of the numbers in the first
pair and y is the mean of the numbers in the 1 1
second pair, then what is the mean of x and A. B.
6 5
y?
16 11
A. 60 B. 72 C. D.
45 90
C. 75 D. 78
65. The ratio of speeds of X and Y is 5 : 6. If Y
59. If m is the number of prime numbers allows X a start of 70 m in a 1.2 km race,
between 0 and 50; and n is the number of then who will win the race and by what
prime numbers between 50 and 100, then distance?
what is (m-n) equal to ?
A. X wins the race by 30 m
A. 4 B. 5
B. Y wins the race by 90 m
C. 6 D. 7
60. Which one of the following is the C. Y wins the race by 130 m
largest divisor of 3x + 3x+1 + 3x+2, if x is any D. The race finishes in a dead heat
natural number?

PAGE 347
www.byjusexamprep.com

66. A train takes two hours less for a journey of


300 km if its speed is increased by 5 km/hr 72. What is equal to?
from is usual speed. What is its usual
speed?
A. B.
A. 50 km/hr B. 40 km/hr
C. 35 km/hr D. 25 km/hr C. D.
67. If 6 men and 8 women can do a piece of
work in 10 days; and 13 men and 24 women 63
73. If θ lies in the first quadrant and cot  = ,
can do the same work in 4 days, then what 16
is the ratio of daily work done by a man to then what is the value of (sinθ + cosθ)?
that of a woman?
69
A. 2 : 1 B. 1 : 2 A. 1 B.
65
C. 4 : 3 D. 3 : 4
79
68. In a class room the ratio of number of girls C. D. 2
65
to that of boys is 3 : 4. The average height
of students in the class is 4.6 feet. If the 74. What is the value of +4
average height of the boys in the class is 4.8
feet, then what is the average height of the equal to?
girls in the class?
A. 0 B. 1
A. Less than 4.2 feet
C. 2 D. 5
B. More than 4.2 feet but less than 4.3
feet 75. What is the area of the triangle having side
lengths ?
C. More than 4.3 feet but less than 4.4
feet
D. More than 4.4 feet but less than 4.5 A. B.
feet
69. The number of three-digit numbers (all
digits are different) which are divisible by 7 C. D.
and also divisible by 7 on reversing the
order of the digits, is 76. If the angles of a triangle are 30° and 45°
A. Six B. Five and the included side is ( 3 + 1) cm, then
what is the area of the triangle?
C. Four D. Three
A. ( 3 + 1) cm2 B. ( 3 + 3) cm2
70. What is equal
1
to? C. ( 3 + 1) cm2 D. 2( 3 + 1) cm2
2
A. 0 B. 1
77. ABCD is a plate in the shape of a
C. 2sinθ D. 2 cosθ
parallelogram. EF is the line parallel to DA
71. What is (tan x + tan y)(1 – cot x cot y) + (cot and passing through the point of
x + cot y)(1 – tan x tan y) equal to? intersection O of the diagonals AC and BD.
A. 0 B. 1 Further, E lies on DC, and F lies on AB. The
triangular portion DOE is cut out from the
C. 2 D. 4

PAGE 348
www.byjusexamprep.com

plate ABCD. What is the ratio of area of the A. 2012 B. 2013


remaining portion of the plate to the
C. 2014 D. 2015
whole?
84. A sum was put at simple interest at certain
5 5
A. B. rate for 2 years. Had it been put at 1%
8 7 higher rate of interest, it would have
3 7 fetched ₹24 more. What is the sum?
C. D.
4 8 A. ₹500 B. ₹600
78. In a circle of radius 5 cm, AB and AC are two
C. ₹800 D. ₹1200
chords such that AB = AC = 8 cm. What is
the length of chord BC? 85. The population of two villages is 1525 and
2600 respectively. If the ratio of male to
A. 9 cm B. 9.2 cm
female population in the first village is 27 :
C. 9.6 cm D. 9.8 cm 34 and the ratio of male to female
79. Which one of the following is a set of population in the second village is 6 : 7,
then what is the ratio of male to female
n
solutions of the equation x x = x x , if n is population of these two villages taken
a positive integer ? together?
A. {1, n2} B. {1, n} 33 85
A. B.
C. {1, } D. {n, n2} 41 82

80. If the sum of the digits of a number 71 5


C. D.
(10)n – 1, where n is a natural number, is 90 6
equal to 3798, then what is the value of n? 86. How many integral values of x and y satisfy
A. 421 B. 422 the equation 5x + 9y = 7, where -500 <x
<500 and -500 <y<500?
C. 423 D. 424
A. 110 B. 111
81. A number divides 12288, 28200 and 44333
so as to leave the same remainder in each C. 112 D. None of these
case. What is that number?
87. Let XYZ be a 3-digit number. Let S = XYZ +
A. 272 B. 232 YZX + ZXY. Which of the following
C. 221 D. 120 statements is/are correct?

82. If m and n are positive integers such that I. S is always divisible by 3 and (X + Y + Z)
mn = 1331, then what is the value of II. S is always divisible by 9
(m – 1)n-1 ?
III. S is always divisible by 37
A. 1 B. 100
Select the correct answer using the code
C. 121 D. 125
given below:
83. The price of an article X increases by 20%
A. 1 only B. 3 only
every year and price of article Y increases
by 10% every year. In the year 2010, the C. 1 and 2 D. 1 and 3
price of article X was ₹ 5000 and price of
88. For two observations, the sum is S and
article Y was ₹ 2000. In which year the
product is P. what is the harmonic mean of
difference in their prices exceeded ₹ 5000
these two observations?
for the first time?

PAGE 349
www.byjusexamprep.com

A. B. A. 10m B. 7m
C. 5m D. 3 m
C. D. 96. The angles of elevation of the top of a tower
from two points at distances p and q from
89. What is the least perfect square which is the base and on the same straight line are
divisible by 3, 4, 5, 6 and 7? 27° and 63° respectively. What is the height
A. 1764 B. 17640 of the tower?
C. 44100 D. 176400 A. pq B. pq
90. If (x2 – 1) is a factor of ax4+bx3+cx2+dx+e,
pq pq
Then which one of the following is correct? C. D.
2 2
A. a + b + c = d + e B. a + b + e = c + d
cos  1
97. What is + equal to?
C. b + c + d = a + e D. a + c + e = b + d 1 + sin  cot 
91. A wheel makes 360 revolutions in one A. cosec θ B. sec θ
minute. What is the number of radians it
turns in one second? C. sec θ + cosec θ D. cosec θ – cot θ

A. 4π B. 6π a 1 b c 1 d e 1
98. If = , = 2, = , = 3 and = ,
C. 12π D. 16π b 3 c d 2 e f 4
abc
92. What is the least value of (25 cosec2x + then what is the value of ?
def
sec2x) ?
1 3
A. 40 B. 36 A. B.
4 4
C. 26 D. 24
3 27
C. D.
93. Let 0 < θ < 90° and 100 θ = 90°. If α = n99−1 8 4
cotnθ, then which one of the following is
99. In covering a certain distance, the average
correct?
speeds of X and Y are in the ratio 4 : 5. If X
A. α = 1 B. α = 0 takes 45 minutes more than Y to reach the
C. α > 1 D. 0 < α < 1 destination, then what is the time taken by
Y to reach the destination?

94. If tan 6θ = cot2θ, where 0 < 6θ < , then A. 135 minutes B. 150 minutes
2
what is the value of sec 4θ? C. 180 minutes D. 225 minutes
A. 2 B. 2 100. If the annual income of X is 20% more than
2 4
that of Y, then the income of Y is less than
C. D. that of X by p%. what is the value of p?
3 3
2
95. A tree of height 15 m is broken by wind in A. 10 B. 16
such a way that its top touches the ground 3
and makes an angle 30° with the ground. 1
What is the height from the ground to the C. 17 D. 20
3
point where tree is broken?

PAGE 350
www.byjusexamprep.com

ANSWERS KEY I 2020


ENGLISH

1 D 21 B 41 B 61 A 81 C 101 B
2 D 22 A 42 C 62 B 82 C 102 D
3 B 23 C 43 B 63 D 83 B 103 C
4 C 24 B 44 A 64 B 84 A 104 A
5 A 25 B 45 A 65 A 85 A 105 B
6 C 26 A 46 C 66 A 86 A 106 A
7 D 27 B 47 C 67 A 87 C 107 B
8 B 28 C 48 C 68 A 88 B 108 A
9 A 29 D 49 A 69 B 89 B 109 C
10 A 30 A 50 A 70 A 90 A 110 C
11 A 31 A 51 A 71 C 91 A 111 D
12 A 32 D 52 D 72 A 92 D 112 A
13 A 33 D 53 D 73 D 93 B 113 C
14 A 34 C 54 D 74 A 94 D 114 A
15 C 35 A 55 B 75 A 95 C 115 D
16 D 36 A 56 A 76 D 96 A 116 A
17 A 37 B 57 C 77 D 97 A 117 B
18 D 38 C 58 B 78 A 98 C 118 A
19 A 39 C 59 B 79 C 99 A 119 C
20 C 40 A 60 A 80 A 100 B 120 A

GENERAL KNOWLEDGE
1 A 21 D 41 C 61 A 81 C 101 B
2 C 22 B 42 A 62 B 82 C 102 D
3 B 23 C 43 B 63 B 83 B 103 B
4 B 24 B 44 C 64 C 84 B 104 C
5 D 25 A 45 A 65 A 85 A 105 C
6 B 26 A 46 A 66 D 86 B 106 B
7 A 27 D 47 A 67 C 87 B 107 D
8 C 28 D 48 B 68 B 88 B 108 B
9 C 29 C 49 B 69 B 89 D 109 D
10 B 30 C 50 D 70 C 90 C 110 A
11 D 31 D 51 A 71 D 91 A 111 A
12 A 32 D 52 B 72 A 92 B 112 C

PAGE 351
www.byjusexamprep.com

13 A 33 B 53 C 73 A 93 B 113 A
14 B 34 B 54 D 74 A 94 A 114 A
15 B 35 D 55 B 75 B 95 C 115 A
16 C 36 C 56 D 76 B 96 C 116 D
17 A 37 A 57 D 77 A 97 B 117 B
18 A 38 A 58 D 78 D 98 C 118 C
19 C 39 C 59 D 79 D 99 C 119 D
20 A 40 B 60 A 80 B 100 A 120 C

Elementary Mathematics
1 D 21 B 41 D 61 A 81 C
2 C 22 D 42 C 62 D 82 B
3 D 23 A 43 A 63 C 83 B
4 B 24 A 44 D 64 A 84 D
5 A 25 A 45 B 65 C 85 D
6 C 26 B 46 A 66 D 86 B
7 A 27 A 47 B 67 A 87 D
8 A 28 B 48 C 68 C 88 C
9 C 29 B 49 C 69 C 89 C
10 D 30 C 50 D 70 A 90 D
11 A 31 C 51 D 71 A 91 C
12 A 32 C 52 A 72 C 92 B
13 B 33 B 53 D 73 C 93 A
14 C 34 D 54 A 74 D 94 A
15 D 35 A 55 B 75 C 95 C
16 B 36 B 56 A 76 C 96 B
17 B 37 C 57 B 77 D 97 B
18 D 38 B 58 B 78 C 98 C
19 D 39 D 59 B 79 A 99 C
20 A 40 A 60 C 80 B 100 B

PAGE 352
CDS I 2020
www.byjusexamprep.com

(Solutions)
ENGLISH
1. Ans. D.
The sentence is in direct speech the correct indirect speech of the given sentence will be ‘The
grandfather blessed the baby that she would live long with all good things of life.’ Hence, option D is
the correct answer.
2. Ans. D.
The sentence is in direct speech, the correct indirect speech of the given sentence should be ‘The
village chief told the villagers that all of them needed to adopt new regulations and they would
protect their Earth forever.’ Thus, option D is the correct answer.
3. Ans. B.
The given sentence is in direct speech and interrogative form, The reporting verb is in the past tense
so the reported speech will change accordingly, Reporting verb ‘said’ will change into ‘requested’ ,
The sentence in indirect speech will be ‘He requested his manager to pass the bill that week.’ Thus,
option B is the correct answer.
4. Ans. C.
The sentence is given in the direct speech and is in past tense, Correct indirect speech of the given
sentence will be option C ‘The leader addressed her followers that those who had sown the seeds
the previous season would reap the harvest that season.’
5. Ans. A.
The given sentence is in Direct Speech. The given sentence is in Direct Speech because it quotes
actual words. Therefore, it must be changed into Indirect Speech. We notice that the reporting verb
is in the Past tense (‘said’). Hence, a change will be made in the tense of the reported speech. The
correct indirect speech will be option A ‘The lady asked her maid where she had been the previous
evening.’
6. Ans. C.
The given sentence is in direct speech and in past tense, the reported speech is an order so the
sentence in indirect speech will change accordingly. The correct option is ‘The captain ordered the
soldiers to march forward and aim at the peak of the hill that day.’ Hence, option C is the correct
answer.

PAGE 353
www.byjusexamprep.com

7. Ans. D.
The above sentence is in the present continuous tense and exclamatory in form. Therefore, as per
the rules, it will get converted into past continuous and inverted commas will be replaced with the
word ‘exclaimed & that’. Only option D fits correct to this rule and therefore, it is the correct answer.
8. Ans. B.
The given sentence is in assertive form. The reporting speech is in past tense, but the reported speech
is a universal truth. When reported speech is a universal truth/idiom/phrase/historical fact, then the
tense of the reported speech remains the same.
The conjunction ‘that’ will be used to connect the reported and reporting part of the sentence.
Hence, the correct answer is option B.
9. Ans. A.
The given sentence is in direct speech and interrogative form, The reporting verb is in the past tense
so the reported speech will change accordingly, Reporting verb ‘said’ will change into ‘requested’ ,
The sentence in indirect speech will be ‘He requested his friends to close the door’. Thus, option A is
the correct answer.
10. Ans. A.
The sentence is given in direct speech is in interrogative form, The reporting verb is in past tense and
therefore the reported speech will change according to the rules. Reporting verb “said” will change
to “asked”. The reported speech is in simple present tense which will change to past perfect tense in
the indirect speech. the sentence in indirect speech will be ‘Rahul asked his teacher what the way to
solve the question was?’ Thus, option A is the correct answer.
11. Ans. A.
The correct combination of the two sentences among the given options is ‘Kavya is interested in
reading books whereas her sister’s interest is outdoor games.’ Hence, we can say option A is the
correct option.
12. Ans. A.
The correct combination of the two sentences among the given options is ‘No sooner did the teacher
enter the classroom than the students stopped talking’. Hence, option A is the correct answer.
13. Ans. A.
The correct combination of the two sentences among the given options is ‘He is so tired that he could
scarcely stand’ Hence, option A is the correct choice.
14. Ans. A.
The most appropriate way of combining the two sentences is ‘When Priya reached the station, the
bus had already left.’, Option A is the correct answer.
15. Ans. C.
The most appropriate way of combining the two sentences using a relative clause is ‘Gandhiji who
preached peace is called an apostle of peace'. Hence, we can say option C is the right answer.

PAGE 354
www.byjusexamprep.com

16. Ans. D.
The best way to combine the two given sentences using ‘though’ is ‘Though he had been trying his
level best to win, he could not succeed.’ Hence, option D is the correct choice.
17. Ans. A.
The most appropriate way of combining both the given sentences is ‘If Minchi had worked hard, she
would have cleared the test.’ Thus, option A is the correct answer.
18. Ans. D.
The most appropriate way of combining both the given sentences is ‘Parents have been waiting since
morning to meet the counsellor. Option is D is the correct answer.
19. Ans. A.
The correct combination of the two given sentences is ‘He is not only hard-working, but also honest.’
Hence, option A is the right answer.
20. Ans. C.
The correct combination of the two sentences is option C i.e. ‘You can meet the officer when he
returns from china on Monday.’ Thus, C is the right answer.
21. Ans. B.
Option B has the correct spelling of the given word Supplementary meaning completing or enhancing
something.
22. Ans. A.
Option A is the correct spelling of the given word Resilient meaning (of a person or animal) able to
withstand or recover quickly from difficult conditions.
23. Ans. C.
Option C has the correct spelling of the given word, Pneumonia is a lung inflammation caused by
bacterial or viral infection, in which the air sacs fill with pus and may become solid.
24. Ans. B.
Option B is the correct spelling of the given word, A Pseudonym is a fictitious name, especially one
used by an author.
25. Ans. B.
Option B has the correct spelling of the given word, Nephrology is the branch of medicine that deals
with the physiology and diseases of the kidneys.
26. Ans. A.
Option A has the correct spelling of the given word, Psephology means the statistical study of
elections and trends in voting.
27. Ans. B.
Option B ‘Vacuum’ is the correct spelling of the given word, Vacuum means a space entirely devoid
of matter.

PAGE 355
www.byjusexamprep.com

28. Ans. C.
The Correct spelling of the given word is ‘Commissioner’ meaning a person appointed to a role on or
by a commission, a representative of the supreme authority in an area.
29. Ans. D.
Option D is the correct spelling of the given word ‘Strategy’ meaning a plan of action designed to
achieve a long-term or overall aim.
30. Ans. A.
The correct spelling of the given word is ‘Continuum’ meaning a continuous sequence in which
adjacent elements are not perceptibly different from each other, but the extremes are quite distinct.
31. Ans. A.
The underlined word in the given sentence is a conjunction. Conjunctions is a part of speech that is
used to connect words, phrases, sentences or clauses, conjunctions may or may not stand between
the items they conjoin. Thus, option A is the correct choice.
32. Ans. D.
The underlined word ‘Hurrah!’ is an Interjection, Interjections are words or phrases that convey
emotion in an abrupt and exclamatory way like ‘wow’ ‘oh’ ‘ouch’ etc.
33. Ans. D.
The underlined word ‘is’ in the given sentence is an auxiliary verb. An auxiliary verb is used with the
main verb to help express the main verb’s tense, mood or voice, They are also known as the main
verb.
34. Ans. C.
The underlined word ‘has offered’ in the given sentence is in Perfect tense, The perfect form is the
verb tense used to indicate a completed, or "perfected," action or condition. Hence, option C is the
correct answer.
35. Ans. A.
The underlined word in the given sentence is a Past perfect verb, The past perfect verbs is a verb
tense used to talk about actions that were completed before some point in the past. Thus, option A
is the most appropriate answer.
36. Ans. A.
The underlined word in the given sentence is a Participle, A participle is formed from a verb and
modifies a noun, noun phrase, verb or verb phrase. A participle most commonly ends in -ed or -ing.
Hence, option A is the correct answer.
37. Ans. B.
The underlined word in the given sentence is an Adjective, Adjectives are the words that describes
or modifies other words. Here the word ‘latest’ is describing ‘news’. Thus, option B is the most
appropriate answer.

PAGE 356
www.byjusexamprep.com

38. Ans. C.
The underlined word in the given sentence is an Adverb. Thus, option C is the correct choice.
39. Ans. C.
The underlined word in the sentence is a relative pronoun. Relative pronouns are used to refer to
noun that are mentioned, whether they are people, places, ideas, animal or things. most common
relative pronouns are whom, who, which, that, whose. Thus, option c is the correct answer.
40. Ans. A.
The underlined word ‘Since’ is a Preposition.
41. Ans. B.
The most appropriate antonym for the given word is ‘Malevolent’
Malevolent means having or showing a wish to do evil to others.
Meaning of the given words are:
Lenient means more merciful or tolerant than expected.
Unforeseen means not anticipated or predicted.
Gracious means courteous, kind, and pleasant, especially towards someone of lower social status.
42. Ans. C.
The most appropriate antonym of the given word ‘Elated’ is ‘Depressed’
Depressed means in a state of unhappiness or despondency.
Meaning of the other words are:
Feels good means something that evokes positive emotions or feelings.
Excited means very enthusiastic and eager.
Sober means not affected by alcohol; not drunk.
43. Ans. B.
The most appropriate antonym for the given word ‘bestowed’ is ‘withdrawn.’
Meaning of the other given words are:
Conferred means grant (a title, degree, benefit, or right). Hence, the correct answer.
Withdraw means remove or take away (something) from a particular place or position.
Imparted means make (information) known.
Imbibe means Imbibed means absorb or assimilate (ideas or knowledge), bestow.
44. Ans. A.
The most appropriate antonym of the given word ‘Wooing’ is ‘Discouraging’
Discouraging means causing someone to lose confidence or enthusiasm; depressing.

PAGE 357
www.byjusexamprep.com

Encouraging means giving someone support or confidence; supportive.


Pursuing means follow or chase (someone or something)
Persuading means induce (someone) to do something through reasoning or argument.
45. Ans. A.
The most appropriate antonym of the given word ‘tempestuous’ is ‘Relaxed’.
Relaxed means free from tension and anxiety.
Meaning of the other words are:
Passionate means having, showing, or caused by strong feelings or beliefs.
Intense means of extreme force, degree, or strength.
Windy means characterized by or exposed to strong winds.
46. Ans. C.
The most appropriate antonym for the given word ‘compassion’ is ‘Indifference’.
Indifference means lack of interest, concern, or sympathy.
Meaning of the other words are:
Empathy means the ability to understand and share the feelings of another.
Carefully means in a way that deliberately avoids harm or errors; cautiously.
Hardship means severe suffering or privation.
47. Ans. C.
The most appropriate antonym for the given word is ‘Nastiness’ as it refers to the state of the state
or quality of being nasty.
Meaning of the other words are:
Charity means an organization set up to provide help and raise money for those in need.
Benevolence means the quality of being well meaning, kindness.
Likeliness means a feeling of regard or fondness.
48. Ans. C.
The most appropriate antonym of the given word is ‘Activity’. Activity means the condition in which
things are happening or being done.
Meaning of the other words are:
Sluggishness means slow-moving or inactiveness.
Indolence means avoidance of activity or exertion; laziness.
Torpor means a state of physical or mental inactivity; lethargy.

PAGE 358
www.byjusexamprep.com

49. Ans. A.
Option A ‘Confusingly’ is the most appropriate antonym of the given word ‘eloquently’.
Meaning of the other words are:
Confusing means bewildering or perplexing. Hence, the answer.
Expressively means effectively conveying thought or feeling.
Powerfully means with great power or strength.
Fluently means with an ability to express oneself easily and articulately.
50. Ans. A.
Option A ‘Survival’ is the most appropriate antonym of the given word ‘decay’.
Meaning of the other words are:
Waste away means they become extremely thin or weak because they are ill or worried
Decomposition means the state or process of rotting; decay.
Spoil means diminish or destroy the value or quality of.
51. Ans. A.
Option A ‘Determined’ is the most appropriate synonym of the underlined word.
Meaning of the other words are:
Determined means having made a firm decision and being resolved not to change it.
Curtained means provide with a curtain or curtains.
Thought of means something to be thought of or something occurring to someone.
Being known means something to having knowledge or information concerning something.
52. Ans. D.
Option D is the most appropriate synonym for the given word.
Meaning of the other words are:
Satisfied means contented; pleased.
Sad means feeling or showing sorrow; unhappy.
Amused means finding something funny or entertaining.
Horrified means filled with horror; extremely shocked. Hence, The correct answer.
53. Ans. D.
Option D ‘Exhausted’ is the most appropriate synonym of the given word ‘fatigued’.
Meaning of the other words are:
Excited: very enthusiastic and eager.
Refreshed: having regained strength or energy.

PAGE 359
www.byjusexamprep.com

Slowed: reduce one's speed or the speed of a vehicle or process.


Exhausted: worn out, very tired. Hence, the correct answer.
54. Ans. D.
Option D ‘Occupation’ is the most appropriate synonym for the underlined word.
Meanings of the other words are:
Attitude means a settled way of thinking or feeling about something.
Approach means come near or nearer to (someone or something) in distance or time.
Liking means a feeling of regard or fondness.
Occupation means a job or profession, work. Hence, the correct answer.
55. Ans. B.
Option B ‘Hackneyed’ is the most appropriate synonym for the underlined word ‘cliched’.
Original means present or existing from the beginning; first or earliest.
Hackneyed means having been overused; unoriginal and trite, stereotyped. Hence, the correct
answer.
Crony means a close friend or companion.
Artificial means made or produced by human beings rather than occurring naturally, especially as a
copy of something natural.
56. Ans. A.
Option A ‘Extraordinary is the most appropriate synonym of the given word ‘Stellar’.
Meaning of the other words are:
Extraordinary means remarkable, very unusual or remarkable.
Eclipse means an obscuring of the light from one celestial body by the passage of another between
it and the observer or between it and its source of illumination.
Poor means lacking sufficient money to live at a standard considered comfortable or normal in a
society.
Not a great means something which is not great, ordinary.
57. Ans. C.
Option C ‘broad’ is the most appropriate synonym of the underlined word.
Meaning of the other words are:
Crude means unrefined, in a natural or raw state; not yet processed or refined.
Broad means a distance larger than usual from side to side; wide.
Non-standard means not average or usual.

PAGE 360
www.byjusexamprep.com

58. Ans. B.
Option B ‘Integrated’ is the most appropriate synonym for the underlined word.
Meaning of other words are:
Excluded means remove from consideration.
Laid down means to put or place in a horizontal position or position of rest; set down: to lay a book
on a desk. 2. to knock or beat down, as from an erect position; strike or throw to the ground: One
punch laid him low.
Removed means take (something) away or off from the position occupied, abolish or get rid of.
59. Ans. B.
Option B ‘Transmittable’ is the most appropriate synonym of the word given in the sentence.
Meaning of the other words are:
Complicated means consisting of many interconnecting parts or elements; intricate.
Transmittable means (of an emotion, feeling, or attitude) likely to spread to and affect others,
catching.
Effective means successful in producing a desired or intended result.
Unthinkable means (of a situation or event) too unlikely or undesirable to be considered a possibility.
60. Ans. A.
The most appropriate synonym of the underlined word is ‘Consequences’ meaning a result or effect.
Let us understand the meaning of the other words:
Interferences means the action of interfering or the process of being interfered with.
Feedback means information about reactions to a product, a person's performance of a task, etc.
which is used as a basis for improvement.
Planning means the process of making plans for something.
61. Ans. A.
The correct option to fill the blank is ‘Ran into’ as ‘run out’ means (of a supply of something) be used
up. Thus, option A is the right answer.
62. Ans. B.
The correct option to fill the blank is ‘Came about’ as ‘came about’ means to reach a condition or
conclusion. Thus, option B is the answer.
63. Ans. D.
The most appropriate option to fill the blank is ‘Will’ as the complete sentence will be Where there
is a will, there is a way’ it is a proverb that means if someone is determined to do something, he will
find a way to accomplish it regardless of obstacles.

PAGE 361
www.byjusexamprep.com

64. Ans. B.
The correct option to fill the blank is ‘Were’, the complete sentence will be ‘If I were an angel, I would
solve the problem of people’.
As ‘were’ is used when you are talking about something that isn’t true or when you wish something
was true.
65. Ans. A.
The correct option to fill the blank is ‘Running out’ as ‘running out’ means (of a supply of something)
be used up. Thus, option A is the right answer.
66. Ans. A.
The most appropriate answer to fill the blank is ‘Put in’ as ‘put in’ means to do, make effort or make
time. Thus, option A is the most correct choice.
67. Ans. A.
The most appropriate answer to fill the blank with is ‘Keep out’ as ‘keep out’ means remain (or cause
someone or something to remain) outside. Thus, option A is the correct choice.
68. Ans. A.
The correct answer to fill the blank is ‘Carry through’ as it means to put a plan or idea into effect.:
complete, accomplish. Thus, option A is the correct answer.
69. Ans. B.
The most appropriate option to fill the blank is ‘Put out’ as ‘put out’ means to extinguish. Thus, B is
the correct choice.
70. Ans. A.
The correct option to fill the blank is ‘Called on’ as ‘called on’ means to pay a visit to, it is the most
appropriate answer.
71. Ans. C.
Sentence P should come first after S1 as it is telling about the key role the monks played, following
that sentence Q should come as it is telling about the journey of some people who committed to
Buddhist paths which gave them courage and determination, Next should come sentence R Telling
about the roots and the journey of Buddhism, Sentence S should come next in the sequence which
is stating that the Buddhaghosha travelled to Sri Lanka. Thus, option C is the correct sequence.
72. Ans. A.
Sentence S comes first after S1 telling about what else some governments claim is omniscience.
Following it sentence Q comes next mentioning the main essence of governance is choice, Sentence
P is next telling about the choices in the different types of things, at last comes sentence R stating
that this is more true in foreign policy making. Thus, the correct sequence is option A (SQPR).
73. Ans. D.
Sentence R should follow S1 as it is telling about the time period when humans learnt to domesticate
certain plants and animals, following that Sentence P mentioning that domestication of plants and
animals led to development of farming and pastoralism, Sentence S should come next telling about

PAGE 362
www.byjusexamprep.com

a major turning point in human history, at last should come sentence Q telling about the favourable
conditions for farming.
Thus, option D (RPSQ) is the correct sequence for the given jumbled sentences.
74. Ans. A.
Sentence Q should come after S1 as it is telling about the political stable position in England, wales
and Scotland, Next should come sentence P telling about what it meant to be unified under a
monarchy, following that sentence S mentioning the usage of money at a wide scale, and at last
sentence R about how the people received their incomes in wages and salaries and not in goods.
Thus, option A (QPSR) is the correct sequence.
75. Ans. A.
Sentence Q should follow S1 as it tells that ‘there have been many important changes in the way the
farming is practiced’, Sentence P should come next telling about the farmers trying to produce more
crops on the same land with the help of new changes, Sentence S telling about the positive aspect of
the new changes in the farming following that should come sentence R at last telling about one of
the negativity aspect of the raised production. Thus, option A (QPSR) is the correct sequence.
76. Ans. D.
Sentence S will come first after S1, telling about the needs of individuals, Sentences R and Q come
next respectively mentioning the types of goods and their usages for different families or individuals.
At last sentence P will come in the sequence as it is further explaining the usages and types of goods
an individual may have. Thus, option D (SRQP) is the correct sequence.
77. Ans. D.
Sentence S will follow S1 as it is telling about the progress of physics and chemistry was much faster
than biology. Sentence R comes next as it tells about the applications of physics and chemistry,
following that sentence P is next telling about the importance and utility of biology, at last sentence
Q which is telling about the contribution of biology in health sector majorly. Thus, option D (SRPQ) is
the correct sequence.
78. Ans. A.
Sentence R should come first after S1 mentioning that the growth of biology is interesting, following
that, sentence S which tells about the features of the growth of biology, Sentence P telling about the
focus points and at last sentence Q telling about the Darwinian ideas that changed the perceptions
completely. Thus, option A is the correct sequence.
79. Ans. C.
Sentence Q should be the first after S1 as it is telling about what the mental model is like. Following
that sentence R should follow which is explaining further about the mental model. Sentence P comes
next telling the qualities of the model and at last Sentence S quoting EF Schumacher, Thus, option C
is the correct sequence for the jumbles sentences.
80. Ans. A.
Sentence S should follow S1 as it tells about the degrees Chinua Achebe received in their respective
years. The sentence R should be the next sentence as it tells about his career and role in the
broadcasting company. Sentence P and Q comes next in the sequence. Hence, Option A (SRPQ) is the
correct answer.

PAGE 363
www.byjusexamprep.com

81. Ans. C.
Idiom Call it a day refers to stop what you are doing because you do not want to do any more or
think you have done enough. Thus, Option C ‘stop doing something’ is the correct answer.
82. Ans. C.
Idiom Rise to the occasion means to successfully overcome a difficult situation. Option C is clearly
closest to that meaning. Hence the correct answer.
83. Ans. B.
The green-eyed monster a feeling of anger or unhappiness because somebody you like or love is
showing interest in somebody else; jealousy.
Hence, option B is the correct answer.
84. Ans. A.
Prime the pump idiom means prime the pump. stimulate or support the growth or success of
something, especially by supplying it with money. Option A is closest to that meaning, Hence, it is
the correct answer.
85. Ans. A.
The pros and cons idioms refers to advantages and disadvantages of something, when the pros and
cons of something are weighed, the good and bad parts of it are considered. Thus, option A is clearly
the correct choice.
86. Ans. A.
The idiom A banana republic refers to a small, poor country with a weak, corrupt or dishonest
government. A small country, run by corrupt politicians and they are usually dependent on one
commodity. Option A is closest to this meaning thus, it is the correct choice.
87. Ans. C.
The dog days idiom refers to a period stagnation or inactivity, alluding to how people do not want to
do much during those hot sultry months, except to lie around and try to stay cool. Option C is clearly
the correct choice.
88. Ans. B.
Eat like a bird is an idiom which means to ‘eat very little’. Therefore, option B is the correct choice.
89. Ans. B.
The idiom ‘Lily-livered’ means lacking courage, cowardly. Thus, option B is the correct answer.
90. Ans. A.
The meaning of the idiom ‘A paper tiger’ refers to something or someone that claims or appears to
be powerful or threatening, but is actually ineffectual and unable to withstand challenge. Thus,
option A is the correct choice.
91. Ans. A.
The correct sequence for the jumbled sentences is ‘A group people who share a common culture,
historical identity and a common descent is called an ethnic group.’ Hence, option A is the right
answer.

PAGE 364
www.byjusexamprep.com

92. Ans. D.
First sentence should be sentence P mentioning the subject of the para ‘poetry’, next should be
sentence S explaining the poetry as the ‘the spontaneous overflow of’, following that sentences R &
Q at last. Hence the correct option is D.
93. Ans. B.
The correct sequence for the jumbled sentences should be ‘There should have been reason for the
accusation against the officer who was in-charge at that time’. Hence, option B is the right answer.
94. Ans. D.
The correct sequence for the jumbled sentences should be ‘Hybrid varieties of several of our food
and vegetable crops are being extensively cultivated’. Hence, option D is right.
95. Ans. C.
The correct sequence for the jumbled sentences is ‘In modern societies has class conflict been
resolved or merely suppressed.’ Thus, option C is the correct choice.
96. Ans. A.
The correct sequence is ‘A referendum is a vote in which the electorate can express a view on a
particular issue of public policy’.
97. Ans. A.
The correct and complete sentence should be ‘The party that received a majority of the vote must
take control of the government’. Hence, the right sequence is QPSR.
98. Ans. C.
First Sentence should be sentence R ‘the essential power’ following it should come sentence Q
mentioning the body which has the essential power ‘the government’. Then comes sentence S stating
the kind of power and at last sentence P. Thus, option C is correct.
99. Ans. A.
First sentence is sentence R as the subject of the para ‘public policy making agenda’ is mentioned in
it, following that is Sentence Q which is further elaborating the topic of the passage. Next comes
sentences S and at last sentence P which is mentioning the reason behind the public policy making
agenda.
100. Ans. B.
The correct sequence is SQPR. The first sentence should be S as it introduces the subject of the
passage i.e. ‘agriculture’, second sentence should be Q as it tells agriculture is the backbone of India.
P should be the third sentence as it mentions that agriculture has been the backbone of India for a
long period and at last comes sentence R. Thus, option B is the answer.
101. Ans. B.
The error is in part B of the given sentence as the past tense ‘lowed’ is given in the sentence.
It will be replaced with “low”, so that the correct sentence will be ‘At the beginning of the nineteenth
century female literacy was extremely low in comparison to male literacy.’

PAGE 365
www.byjusexamprep.com

102. Ans. D.
There is no error in the given sentence. Hence, option D is the correct choice.
103. Ans. C.
The Error is in the part C of the given sentence, “Achieve” will be replaced by “achieved” as achieved
is the past tense of ‘achieve’. The correct sentence will be ‘When everything starts working for you,
you will find things are achieved and delivered.’
104. Ans. A.
The error is in part A of the given sentence. “Leave” will be replaced with “Leaves” as “leaves” is the
third person singular simple present form of ‘leave’. The correct sentence will be ‘Evolutionary
biology leaves us distinctly pessimistic about the possibility that altruism can arise naturally among
humans.’
105. Ans. B.
The error is in the part B of the given sentence, “were” will be replaced with “where”. The complete
sentence will be ‘The fascination with gold at least seems to be a case where traditional belief and
modern finance would point the same way.’ Thus, option B is the correct choice.
106. Ans. A.
The error is in part A of the given sentence, “thorough” is incorrectly used here it will be replaced by
“through” as meaning of both the words are completely different.
Thorough means complete with regard to every detail; not superficial or partial.
Through means moving in one side and out of the other side of (an opening, channel, or location).
107. Ans. B.
The error is in the part B of the given sentence, “Returned” will be replaced by “returns” as “returns”
is 3rd person singular present tense. Thus, the complete sentence will be ‘Government stock offers
safety, liquidity and attractive returns for long duration.’
108. Ans. A.
The error is in the part A of the given sentence, “would have been” will be replaced by “had been”
as “had been” is the past perfect tense. Thus, option A is correct choice.
109. Ans. C.
The error is in the part C of the given sentence, “Period” will be replaced by “price” as “period”
method is incorrectly written.
The correct complete sentence will be ‘The auction, conducted by the bank will be price based using
multiple price methods.’
110. Ans. C.
The error is in part C of the sentence. “starting melting” will be replaced with “has started” as "Has
started" is an active voice, present tense, perfect aspect construction.

PAGE 366
www.byjusexamprep.com

111. Ans. D.
The word Perspectives is the correct word that means same as ‘viewpoints’ in the passage. Thus,
option D is right.
112. Ans. A.
In the given passage, The author advocates the quality of being objective to the ideas of the author.
Hence, option A is the correct answer.
113. Ans. C.
As per the passage ‘Day after day in our routine pattern of life we expose ourselves to the same
newspaper, the same magazine, even books by authors with the same perspectives.’ The Correct
option is C ‘Break the routine of reading the same newspaper’.
114. Ans. A.
According to the passage ‘Our expectations and predispositions may, however, blind us to what the
article and its author is actually saying. If, for example, we are used to disagreeing with the author,
we may see only what we expect to see and not what is actually there’ Thus, We may not get the
actual ideas of the author. Hence, option A
115. Ans. D.
According to the author ‘When we pick up a newspaper, book, or an article. We expect to find a
specific piece of information or be presented with an argument or an analysis of something.’ Hence,
option D is the correct answer.
116. Ans. A.
Option A is the correct answer, as transitioned means same as ‘changeover’ in the passage.
Changeover means a change from one system or situation to another.
117. Ans. B.
It is clearly stated in the passage that ‘In the ancient world, this often involved creating new
ideologies and new hierarchies or power structures to coerce or otherwise convince large groups of
people to devote their time to the new tasks for very little reward.’ Hence, option B is the correct
choice.
118. Ans. A.
‘A society that has transitioned to high-productivity agriculture has also, at some stage in its
evolution, found a way to channel the bonanza of free time into other work fruitfully.’ Thus, option
A is the correct choice.
119. Ans. C.
It is mentioned in the passage that as productivity improves, not all people would need to be engaged
in producing or procuring food. A significant number of people could be freed up to pursue other
activities, Thus, option C is the correct answer.
120. Ans. A.
As stated in the passage “Not all agricultural societies become civilizations, but no civilization can
become one without passing through the stage of agriculture” we can clearly make out that
Agriculture has always been part of all civilization, Hence, option A is the correct answer.

PAGE 367
www.byjusexamprep.com

General Knowledge
1. Ans. A.
The Government of India has recently constituted a civilian award in the name of Sardar Vallabhbhai
Patel in the field of contribution to the Unity and integrity of India. It was to be given in a very rare
and highly deserving case. It consists of a medal and a citation. It does not have any monetary
rewards associated with it. It can be given only 3 three times in a year it was announced on the birth
anniversary of Sardar Patel on October 31st which is declared as the National Unity Day.
Hence, option A is correct.
2. Ans. C.
BRICS is an acronym for Brazil, Russia, India, China, and South Africa. It was formed in 2009. The
Summit was going to be held in Saint Petersburg in mid-July 2020 in Russia.
Hence, option C is correct.
3. Ans. B.
World Tourism Day is celebrated on 27 September every year. It was hosted by India in 2019. The
theme of the event was tourism and job: A better future for all. The main aim of this day is to create
awareness on the importance of tourism within the international community and to show how it
affects the social, cultural political, and economic values. The color of the World Tourism day is blue.
Hence, option B is correct.
4. Ans. B.
1. Perfect competition: Many producers selling a similar product.
2. Monopoly: Only 1 producer selling 1 commodity.
3. Monopolistic Competition: Many producers selling differentiated products
4. Oligopoly: Few producers selling similar or almost similar products.
So, price rigidity exists in the oligopoly form of the market.
Hence, option B is correct.
5. Ans. D.
Opportunity cost is an important concept in microeconomics. It refers to the benefit or the value
which is missed by a business owner, investor, organisation, or individual because they chose
something else. If agricultural labour earns Rs400 per day and also gets another job as a babysitter
in a nearby town that offers 700 rupees per day. If she chose to work as an agricultural labour
opportunity cost would be 700 – 00 = 300.
Hence, option D is correct.
6. Ans. B.
Census refers to the procedure of acquiring and recording information about the members of a
country in a systematic procedure. It is taken every 10 years in India. The term migrant means a
person who moves from one place to other especially in search of livelihood. Internal migration or

PAGE 368
www.byjusexamprep.com

domestic migration refers to the migration of people from one place to another but within the
geographical territory of the country. According to the census 2011, 35% of the internal migrants are
now set in a place different from the previous residence.
Hence, option B is correct.
7. Ans. A.
It is a Ministry of the Indian government. It is currently held by Prakash Javadekar. It has the
responsibility of planning, promoting, coordinating the implementation of Environment and Forest
related programs in the country. It also monitors the flora and fauna of India along with its
conservation. Other important activities include control of pollution, deforestation, and land
degradation mitigation. It also has the responsibility of maintaining the national parks of India. It is
controlled by the Indian forest services.
Hence, option A is correct.
8. Ans. C.
Ayush is an acronym of the Medical Systems practiced in India search as Ayurveda, yoga,
naturopathy, Unani, and homeopathy. These systems are based on different medical philosophies
and represent a way of living which can help in the prevention of diseases and the promotion of
health. It is headed by Shripad Yesso Naik in India. It has many Health Care programs in the rural
population such as the Ayushman Bharat Yojana.
Hence, option C is correct.
9. Ans. C.
Department of industrial policy and promotion was renamed as the Department for the promotion
of industry and industrial internal trade in 2019. It is administered by the Ministry of Commerce and
industry. Its main function is to formulate and implement growth strategies for the industrial sector
along with achieving National goals and social-economic objectives. Intellectual Property Rights
means providing property right through patents, copyrights, and trademarks.
Hence, option C is correct.
10. Ans. B.
Election Commission: This is a permanent and independent body established by the constitution of
India to ensure free and fair elections in the country.
Planning Commission: It helps the government of India to formulate India's five-year plans. It was
established in 1950 with Jawaharlal Nehru as a chairman.
Finance Commission: It is a constitutional body which has the responsibility of allocating finance
between the central and the state government. It was formed in 1951.
Investment Commission: It was formed by the government of India in December 2004.
Hence, option B is correct.
11. Ans. D.
The National Water Academy has the responsibility of streaming the water resources personnel. It
addresses the training needs of the water resource engineering of Central and state agencies in terms

PAGE 369
www.byjusexamprep.com

of water resource projects. It includes planning, design, construction, evaluation, operation, and
monitoring. Its located at Khadakwasla.
Hence, option D is correct.
12. Ans. A.
The Black Sea is a water body and marginal sea of the Atlantic Ocean. It lies between Eastern Europe,
the Caucasus, and Western Asia. There are many rivers that flow into the Black Sea. These are the
Danube, Dnieper, Southern Bug, Dniester, Don, and Rioni. It is 1,68,500 sqm with a depth of 7257
feet and a volume of 5,47,000 km cube.
Hence, option A is correct.
13. Ans. A.
The provisions of the Constitution of India pertaining to the institution of Panchayat do not apply to
Meghalaya. Part IX of the Constitution of India deals with the Panchayati Raj system in the rural
areas of the country. It includes terms, composition, selection procedure, duration, etc. Apart from
Meghalaya, it also is not applied to the states of Nagaland and Mizoram.
Hence, option A is correct.
14. Ans. B.
The 11th Schedule of the Constitution of India distributes powers between the State Legislatures and
the Panchayat. According to this amendment, the state has to organize the village Panchayat as well
as give the required powers to help them to function as self-government. It is compulsory for the
state to accept this system. However, the election process in the Panchayati Raj Institutions will be
independently held of the state government will.
Hence, option B is correct.
15. Ans. B.
The Panchayat system is a process that helps to encourage people to participate in their own
government. It is a unit of local administration. The seats are filled by the people chosen by direct
election from territorial constituencies in the panchayat area. The electorate is called Gram Sabha
consisting of people registered in the electoral rolls relating to the territory within the panchayat
area.
Hence, option B is correct.
16. Ans. C.
Hunter was a Commission formed by the then Governor-General of India on 3rd February 1882. It
was under the leadership of Sir William Hunter, who was an Indian Civil Service officer and a member
of the Executive Council. It was mainly formed to consider the different aspects of education,
especially primary education.
Hence, option C is correct.
17. Ans. A.
• In 1948, the Deccan state of Hyderabad formally acceded to the Indian Union.
• Nizam al-Mulk established it.

PAGE 370
www.byjusexamprep.com

• Mir Osman Ali Khan, the seventh Nizam, was advocating for Hyderabad to become an
independent sovereign state.
Hence, option A is correct.
18. Ans. A.
Hridayanath Kunzru formed the organisation called Seva Samiti in Allahabad in 1949. He was a
member of the servants of Indian society. The main aim of this organisation was to organize social
services during natural disasters and epidemics for the promotion of education, sanitation,
upliftment of depressed classes, as3 formation of criminal, and promoting the spirit of cooperation
among all.
Hence, option A is correct.
19. Ans. C.
The Government of India Act 1935 was formed due to the increasing demand for constitutional
reforms by the Indian leaders. This act was based on the Simon Commission, recommendations on
the round table conferences, report of the joint table committee, and the white paper published by
the British government. This act had many provisions related to the division of powers, the Indian
Council, and the reorganization. It also advocated the setting up of a federal court.
Hence, option C is correct.
20. Ans. A.
The maiden trilateral naval exercise involving India, Singapore, and Thailand was held at Port Blair.
The five-day-long exercise was aimed at improving the maritime interrelationship among the three
countries and to contribute to improving the overall maritime security in the regions. It would
improve mutual confidence among them and develop a common understanding of procedures. It
also involved sports fixtures, professional exchanges, and cross deck familiarisation visits.
Hence, option A is correct.
21. Ans. D.
Rustom 2 was an unmanned aerial vehicle that is used by the Indian armed forces to carry out
surveillance. It is a medium altitude and long endurance vehicle. It was designed by the defense
research and development organisation for the three services of the Indian Navy, Indian Army, and
Indian Air Force. It crashed in the Chitradurga District of Karnataka in an agricultural field at 6 am.
The first trial was being conducted.
Hence, option D is correct.
22. Ans. B.
The Vijaynagar Advanced Landing Ground of the Indian Air Force, which was reopened recently, is
located in Arunachal Pradesh. It was first opened in 1962 by the Indian Airforce. It started its
operation with Dakota and Otter aircraft. It was the year that the India China war took place. It was
operational until 2016. It was then declared unfit for the fixed-wing aircraft such as transport planes.
Its runway was damaged.
Hence, option B is correct.

PAGE 371
www.byjusexamprep.com

23. Ans. C.
Hilsa a species of fish related to Herring. It is a very popular food in the Indian subcontinent. It is the
national fish of Bangladesh and the state symbol of West Bengal and Tripura. It contributes to 12%
of total fish production and 1.15% of GDP in Bangladesh. On 6th August 2017, the Department of
patents, designs, and trademarks under the Ministry of Industry of Bangladesh, the recognition of
Hilsa was declared as the product of Bangladesh.
Hence, option C is correct.
24. Ans. B.
The Chairman of the Economic Advisory Council to the Prime Minister (EAC-PM) is Bibek Debroy. It
is nonconstitutional. nonpermanent and independent body. It advises the prime minister on
economic issues such as inflation, market prices, foreign exchange, etc. It highlights the concerning
issues from a neutral viewpoint. Currently, it was revived on 25 September 2017. It is headed by a
chairman and other prominent economists as members.
Hence, option B is correct.
25. Ans. A.
The Chairman of the Economic Advisory Council to the Prime Minister (EAC-PM) is Bibek Debroy. It
is nonconstitutional. nonpermanent and independent body. It advises the prime minister on
economic issues such as inflation, market prices, foreign exchange, etc. It highlights the concerning
issues from a neutral viewpoint. Currently, it was revived on 25 September 2017. It is headed by a
chairman and other prominent economists as members.
Hence, option B is correct.
26. Ans. A.
Dr. B.R.Ambedkar was the Chairman of the Drafting Committee set up to frame the Indian
constitution. While concluding a debate in the Constituent Assembly on 26th January 1950 he said
that India was going to enter a life of contradictions.
Hence, option A is correct.
27. Ans. D.
The First Delimitation Commission in India was constituted in 1952. It is also called a Boundary
Commission. It is set up to fix limitations or boundaries in a country or a state with a legislative body.
It has been constituted 4 times – in 1952 under the Delimitation Commission Act, 1952, in 1963 under
Delimitation Commission Act, 1962, in 1973 under Delimitation Act, 1972 and in 2002 under
Delimitation Act, 2002. The action imposed by it cannot be questioned in any court.
Hence, option D is correct.
28. Ans. D.
Ministry of Heavy Industries and Public Enterprises is an agency of the government of India that
administers 48 public sector enterprises. It consists of the Department of Heavy Industry and the
Department of Public Enterprises. It helps to improve capacity utilization, increase profits, generate
resources, and form strategies to become more efficient. The current Minister is Prakash Javadekar
and the Minister of State is Arjun Ram Meghwal. It is responsible for many PSEs such as BBUNL, BHEL
CCI, HEC, etc.
Hence, option D is correct.

PAGE 372
www.byjusexamprep.com

29. Ans. C.
Article 32 of the Indian Consitution is related to the remedies for the enforcement of the
Fundamental Rights. The aggrieved person can get remedy from another court such as the Supreme
Court or High Court if his issues aren’t solved. It is called the soul of the Constitution. The Supreme
court becomes the defender and guarantor of fundamental rights. The person can also approach the
supreme court directly without any way of appeal. It can be invoked only to get a remedy related to
fundamental rights and not any other legal right.
Hence, option C is correct.
30. Ans. C.
Article 356 states that the president’s rule can be imposed on any state of India due to the failure of
the constitutional machinery. In other words, the state government will cease to function and the
central government will directly administer the state. This is done if the President receives a report
from the Governor of the state or is convinced that the situation of the state is such that the state
government cannot function without the interference of the central government.
Hence, option C is correct.
31. Ans. A.
• The Indo-European language families dominate the majority of Central Asia.
• The Indo-European languages are a large family of languages native to western Eurasia. It
includes the majority of European languages, as well as those of the northern Indian
Subcontinent and the Iranian Plateau.
Hence, option A is correct.
32. Ans. D.
Urdu was the official language of J and K (when it was a state). It’s widely spoken there. Koshur or
Kashmiri is a widely spoken local language of people in Jammu and Kashmir.More than 50% can speak
or understand the Koshur language. Gujari is spoken by the gujari tribes of JnK. Gujars and
Bakkarwals are the popular tribes of Jammu and Kashmir. Monpa is a language which is spoken by
Monpa people of Arunachal Pradesh. The Monpa are believed to be the only Nomadic tribe in North
East India.
Hence, option D is correct.
33. Ans. B.
A Mediterranean climate is that in which the summers are dry and warm. Winters observe rainfall
(uniqueness of this climate). This climate is named after Mediterranean Sea which plays a very
important role in controlling the weather of the adjacent areas. The Spanish mainland is bordered to
the south and east almost entirely by the Mediterranean Sea. Hence, the central Spain also observes
Mediterranean climate.
Hence, option B is correct.
34. Ans. B.
Coal India Limited (CIL) the state owned coal mining corporate came into being in November 1975.
With a modest production of 79 Million Tonnes (MTs) at the year of its inception CIL today is the

PAGE 373
www.byjusexamprep.com

single largest coal producer in the world and one of the largest corporate employer. Operating
through 83 mining areas and spread over eight (8) provincial states of India. CIL has its headquarter
at Kolkata.
Hence, option B is correct.
35. Ans. D.
There are many specialized army training institutions, such as:
1) College of military engineering in Pune. 2) High altitude warfare school (HAWS) at Gulmarg,Jnk. 3)
Counter Insurgency and Jungle Warfare School at vairengte,Mizoram.4) Army war college at Mhow,
Madhya Pradesh.
Hence, option D is correct.
36. Ans. C.
Portuguese were the first one to arrive in Africa as traders which eventually lead to European
colonization of Africa. Their voyage to find India led them to reach Africa in the early 15th century.
Rest of the European countries such as Spain Dutch etc. followed the path of Portuguese.
Hence, option C is correct.
37. Ans. A.
"Gandhi solar park is located at New York. It was inaugurated by Narendra Modi at UN headquarters,
New York City. Along with it a special commemorative event was held to celebrate 150th anniversary
of Mahatma Gandhi. A special UN postage stamp on Gandhi 150 years was also released.
Hence, option A is correct.
38. Ans. A.
The global goalkeeper award is given by buy the bill and Melinda gates foundation. This year the
prime minister of India Mr Narendra Modi was conferred with this award he received this award for
swachh Bharat abhiyan. Mr Modi dedicated this award to all the people of this country.
Hence, option A is correct.
39. Ans. C.
India's entry to 92nd academy awards was Gully Boy. Gully boy I was not able to make it to top 10
films at the academy awards. Parasite a South Korean was awarded the best film. It became the first
ever non English movie to win the best film award.
Hence, option C is correct.
40. Ans. B.
Naropa festival also known as Kumbh of Himalayas began in Ladakh ( September,2019). Very famous
Shondol dance is performed by many artists in this festival. Naropa festival usually takes place after
12 years in the Tibetan calendar. This festival celebrates the life of great Buddhist philosopher and
scholar Naropa.
Hence, option B is correct.

PAGE 374
www.byjusexamprep.com

41. Ans. C.
Tiger triumph was the first ever tri service military exercise between India and USA. It was a nine day
humanitarian assistance and disaster relief (HADR) exercise. The main aim was to develop synergy
between both the military in HADR operations. It was a two phase exercise, one at Vishakhapatnam
and second at Kakinada.
Hence, option C is correct.
42. Ans. A.
Tattvabodhini Sabha: the founding member was Debendranath Tagore. The main aim was to reform
Hinduism and Indian society. It worked as a splinter group of Brahmo Samaj. It propagated the ideas
of Ram Mohan Roy. They were also known as searching society or truth propagating.
Hence, option A is correct.
43. Ans. B.
Lord Palmerston was the former prime minister of the Britain. He had dominated the foreign policy
of Britain in mid-19th century. Lord palmerston saw in Russian designs imminent peril to the security
and tranquility of the Indian empire and goaded on the British government of India to take effective
steps to check mate them.
Hence, option B is correct.
44. Ans. C.
The famous book “Siyar ul mutakherin” was written by Ghulam Husain. He had written specifically
about Battle of Plassey in this book. Salabat jung was the 4th Nizam of Hyderabad. Qasim khan was
a Mughal General and a nobleman at Jahangir and Shah Jahan's court. Raja Ram Mohan Roy also
known as Father of Bengal Renaissance. He founded Brahmo Samaj and was a man of high intellect.
He had written books like The Percepts of Jesus, Gift to Monotheists etc.
Hence, option C is correct.
45. Ans. A.
Rohri hills (Sindh, Pakistan) are home to many archaeological sites and the abundant presence of
limestone there proves the point that it had a factory and from here limestone was sent to various
Harappan civilizations in Sindh. No nearby places to Sindh had the presence of limestone.
Hence, option A is correct.
46. Ans. A.
The Stone Age basically refers to a pre historic period where stones were used as tools to derive food
from the nature. From early Stone Age to later Stone Age, humans kept on innovating in stone
technology by using raw stones to using ivory in the later Stone Age.
Hence, option A is correct.
47. Ans. A.
A partial protection against double jeopardy is a Fundamental Right guaranteed under Article 20 (2)
of the Constitution of India, which states, "No person shall be prosecuted and punished for the same
offence more than once". This provision enshrines the concept of autrefois convict, that no one

PAGE 375
www.byjusexamprep.com

convicted of an offence can be tried or punished a second time. However it does not extend to
autrefois acquit, and so if a person is acquitted of a crime, he can be retried. In India, protection
against autrefois acquit is a statutory right, not a fundamental one.
Hence, option A is correct.
48. Ans. B.
86th Amendment had a direct effect on FR, DPSP AND FD.86th amendment act inserted a new article
21A, i.e. Right to Education. According to it “The State shall provide free and compulsory education
to all children of the age of six to fourteen years in such manner as the State may, by law, determine”.
Hence, option B is correct.
49. Ans. B.
Human development is basically based on six pillars equity, cooperation, sustainability,
empowerment, security and productivity. Equity is how fair we are to others especially w.r.t. health
care and education. We must be fair to both men and women with regard to this. Sustainability is
nothing but our right to earn and run our livelihood. Productivity means the mass participation of
people in income generation. Empowerment gives you choice to take your own decisions and
influence development.
Hence, option B is correct.
50. Ans. D.
Ayushman Bharat is a flagship scheme of Government of India to achieve the vision of Universal
Health Coverage (UHC). The main aim is to meet Sustainable Development Goals and its underlining
commitment, which is to "leave no one behind. Following are the main points of the scheme: 1) It
will provide a cover of 5 lakh per family per year for medical treatment in every empaneled hospitals,
both public and private. It is providing cashless and paperless service to its beneficiaries at the point
of service, i.e the hospital. 3) E-cards are being provided to the eligible beneficiaries based on the
deprivation and occupational criteria of Socio-Economic Caste Census 2011.
Hence, option D is correct.
51. Ans. A.
An overseas Indian can vote only if he has not given up the citizenship of India. He has to first register
himself as overseas voter in his hometown. He has to be physically present to vote in India. There is
no provision to vote online in Indian election norms. To vote one has to personally visit a polling
booth. Postal ballots and proxy voting are available only for government or armed forces personnel.
Hence, option A is correct.
52. Ans. B.
Gelisols, Histosols, Spodosols and Vertisols are some of the different types of soils.
Histosols, also known as peats, are mainly composed of organic materials. Restricted drainage
doesn't allow the remains of various plants and animals to decompose and hence, acts as carbon
sink. Hence they have ecological importance to us.
Hence, option B is correct.

PAGE 376
www.byjusexamprep.com

53. Ans. C.
Hadley cell is an atmospheric circulation in which the air rises near the equator due to intense heating
and then it flows poleward and due to coriolis force it descends at the subtropical high pressure belt.
Then again the descended air returns equatorward and forms the Hadley cell. This also leads to the
creation of trade winds.
Hence, option C is correct.
54. Ans. D.
In total, there are 3 states and 1 Union territory that share more than two international boundaries.1)
Ladakh shares international boundaries with Pakistan, Afghanistan and China. 2) West Bengal shares
international boundaries with Nepal, Bhutan and Bangladesh. 3) Sikkim shares international
boundaries with Nepal, Bhutan and China. 4) Arunachal Pradesh shares international boundaries
with China, Bhutan and Myanmar.
Hence, option D is correct.
55. Ans. B.
Kuroshio current of North Pacific Ocean is a warm current whereas California current is a cold current
which flows in North Pacific Ocean. North Atlantic current is a warm current which flows in Northern
Atlantic Ocean. Brazil current is a warm current which flows southward along the east coast of South
America.
Hence, option B is correct.
56. Ans. D.
Sclerophyllous plants are predominantly found in Chaparral Biomes. They are very prominent in
Australia (Perth region, Sydney region and Adelaide region). These are also found in California
Chaparral, Chile and Cape Town. Hence option D doesn't have any sclerophyllous vegetation.
Hence, option D is correct.
57. Ans. D.
Slope angle, slope aspect and relative relief are the most important geomorphic factors that
influence plants and animals distribution. Relative relief causes a sudden change in distribution of
flora and fauna. Slope plays a vital role in deciding what type of vegetation would occur and what
kind of animals would sustain there.
Hence, option D is correct.
58. Ans. D.
Panchatantra is a compilation of interrelated animal fables in Sanskrit Language. The collection is
attributed to Vishnusharma but the original author is unknown. Every Panchatantra story ends with
a moral message. The moral messages always distinguish between right and wrong.
Hence, option D is correct.
59. Ans. D.
Satavahana rulers had been using their mothers' names before their own names. Though very
patriarchal in nature but they kept their mothers' names as sign of deep respect and honour for

PAGE 377
www.byjusexamprep.com

women. Gautamiputa Satkarni, Vasishthiputra Puluvavi, Haritiputra Sakasena, Madhariputra


Satakarni are some of the names of the rulers who had always used their mothers' name first.
Hence, option D is correct.
60. Ans. A.
Kudappah kar was the name for one of the main seasons for Rice cultivation in Southern India.
Nancai, Puncai and Tottakal are land classifications. Nancai were the wet lands which were always
irrigated by the natural streams. Puncai were the dry lands which were always dependent on
seasonal rainfalls. Tottakal were the garden lands which were irrigated artificially by the local means
such as wells, ponds, etc.
Hence, option A is correct.
61. Ans. A.
The mansab or rank was designated by the dual representation - one by personal rank (known as zat)
and the other by cavalry rank (known as sawar). ‘Zat’ rank was an indicator of the Mansabdar’s
position in the imperial hierarchy and also the salary of the Mansabdar whereas ‘Sawar’ rank
indicated the number of horsemen the Mansabdar was required to maintain. Every mansabdar was
given the rank of both zat and sawar and a mansabdar was paid rupees two per horse.
Hence, option A is correct.
62. Ans. B.
Abu’l Hassan’s ancient painting, Jahangir Embracing Shah Abbas (act. 1600-30) is a small composition
filled with exquisite detail of opaque watercolor and ink on gold paper.
The scene portrays two men of royalty in very detailed and lavish clothing and jewellery embracing
standing on top of a globe with a lamb and a lion at their feet. Encompassing them is a large golden
halo that surrounds both of them being held up by two angel heads. In the centre of the halo, there
is microscopic scripture that is illegible unless looked at with a magnifying glass.
Hence, option B is correct.
63. Ans. B.
The very first recipient of the Dada Saheb Phalke award was Devika Rani who received it in 1969 on
the occasion of the 17th National Films Awards.
Recently, Amitabh Bachchan is selected for the 66th Dadasaheb Phalke Award in 2018 while in 2017
this award was given to Vinod Khanna.
Hence, option B is correct.
64. Ans. C.
Recently, the RBI (Reserve Bank of India) has imposed limitations on the Mumbai-based Punjab &
Maharashtra Cooperative Bank (PMC Bank) from carrying out a majority of its routine business
transactions for a time period of six months.
Hence, option C is correct.

PAGE 378
www.byjusexamprep.com

65. Ans. A.
Greta Thunberg is a Swedish environmental activist who has gained international recognition for
promoting the view that humanity is facing an existential crisis arising from climate change. She is
recognized for her work against climate change, a popular example of youth activism. She started
protesting on 20th August 2018, outside of the Swedish parliament in Stockholm.
Hence, option A is correct.
66. Ans. D.
Thomas Cook, the 178-year-old British travel giant, has declared bankruptcy, leaving 150000 British
citizens stranded abroad, triggering the largest-ever peacetime emergency rescue operation in
British history. Brexit and Bankruptcy are the reasons for its collapse.
Hence, option D is correct.
67. Ans. C.
The 2019 FIFA FIFPro Men’s and Women’s Players of the Year were announced at a gala celebration
held at the Teatro all Scala in Milan.
Lionel Messi won the Best Men’s Player Award of FIFA Football Awards, 2019. He beat off
competition from Juventus’ Cristiano Ronaldo and Liverpool’s Virgil van Dijk to take the
Hence, option C is correct.
68. Ans. B.
If in a diagram has a line passing through the origin and has 45° angle with either axis and also it is
asserted that along the line X equals to Y, then it is assumed that both variables X and Y are in the
same unit.
Hence, option B is correct.
69. Ans. B.
• The Cash Reserve Ratio (CRR) is the percentage of Net Demand and Time Liabilities that banks
must hold as reserves with the RBI.
• CRR's goal is to keep inflation under control.
• During periods of high inflation, the central bank raises the CRR to reduce the bank's loanable
funds.
Hence, option B is correct.
70. Ans. C.
• The repo rate is not a loan agreement between a commercial bank and the central bank.
• The central bank has the authority to alter it in order to regulate in accordance with the situation.
Hence, option C is correct.
71. Ans. C.
Marriage is the biggest reason because of which women migrate from one place to another. The
findings from the 2011 Census, which were published recently show that marriage made up for 46
percent of the total migrations in India, of which 97 percent are women.
Hence, option C is correct.

PAGE 379
www.byjusexamprep.com

72. Ans. A.
The Basel Convention on the Control of the Trans-boundary Movements of the Hazardous Wastes
and their Disposal was adopted in the year 1989, and it came into force in 1992.
Hence, option A is correct.
73. Ans. A.
Pradhan Mantri Saubhagya Yojana has been launched by the Prime Minister of India, Shri Narendra
Modi to provide electricity to the poor families of the country. Under this scheme, the financially
poor people of the country who are unable to get electricity connection and are living without
electricity, will be given free electricity connection by the Central Government. This scheme is also
known as Pradhan Mantri Sahaj Bijli Har Ghar Yojana.
Hence, option A is correct.
74. Ans. A.
The Local Area Development Scheme called as MPLADS is a government scheme launched on 23rd
December 1993. This was a central sector scheme and was developed as an initiative to enable the
parliament members to recommend the developmental work in their constituencies based on locally
felt needs. These developmental works majorly focused on the areas of national priorities like
drinking water, education, sanitation, public health, roads, etc.
Hence, option A is correct.
75. Ans. B.
Pradhan Mantri Gram Sadak Yojana: The PMGSY is under the authority of the Ministry of Rural
Development and was begun on 25th December 2000. It is completely funded by the central
government. During the period of November 2015, following the recommendations of the 14th
Finance Commission, the Sub-Group of the Chief Ministers on Rationalization of Centrally Sponsored
Schemes, it was announced that the project would be funded by both the central government (60%)
and states (40%). In the 2000 government was announced.
Hence, option B is correct.
76. Ans. B.
All Panchayati Raj Institutions perform such functions which are specified in state laws relating to
Panchayati raj. Some States distinguish between compulsory and optional functions of Gram
Panchayats while other States don’t make this distinction.
The civic functions of the panchayats relating to sanitation, cleaning of public roads, public toilets
and lavatories, primary health care, minor irrigation, vaccination, the supply of drinking water,
constructing public wells, rural electrification, social health and primary and adult education, etc. are
obligatory functions of village panchayats.
Hence, option B is correct.
77. Ans. A.
Mahatma Gandhi Employment Guarantee Act of 2005 is Indian labour law and social security
measure that aims to guarantee the 'right to work'. This act was passed in September 2005.

PAGE 380
www.byjusexamprep.com

Its main aim is to provide employment to the unemployed. It provides social protection to the most
vulnerable people living in rural India by giving them employment opportunities. (i.e. the main
objective of MNREGA is to enhance livelihood security.)
Hence, option A is correct.
78. Ans. D.
Barren and wasteland is not a land-use category land.
Barren and wasteland: The land which may be classified as a wasteland such as barren hilly terrains,
desert lands, ravines, etc. normally cannot be brought under cultivation with the available
technology.
Hence, option D is correct.
79. Ans. D.
The type of farming in which crops are grown especially for commercial use only is known as
commercial farming/agriculture. It is produced for the selling purposes only for which a large amount
of labour is required. Commercial agriculture is a large-scale production of crops for sale, intended
for widespread distribution to the wholesalers or retail outlets. In this farming, crops such as wheat,
maize, tea, sugarcane, cashew, coffee, rubber, banana, cotton are harvested and sold in the world
markets.
Hence, option D is correct
80. Ans. B.
Son river joins Ganga directly while Chambal, Betwa and Ken are tributaries of Yamuna. Chambal,
Ken and Betwa rivers do not join the Ganga river. They are tributaries of Yamuna which in turn is a
tributary of Ganga and joins Ganga at Prayagraj, U.P.
The Son originates near Amarkantak in Madhya Pradesh, and it meets the Ganges near Patna in Bihar.
Hence, option B is correct.
81. Ans. C.
The total number of people per unit area of arable land is known as Physiological density.
A higher the physiological density suggests that the available agricultural land is being used by more
and may reach its output limit sooner than a country that has a lower physiological density.
Hence, option C is correct.
82. Ans. C.
The Karnataka government commenced on an ambitious plan ‘One State Many Worlds’ to make the
State one of the most preferred tourist destinations in India and the world at large.
Karnataka offers a range of tourism spots that cover everything from dense forests to lush green
grasslands, from beautiful beaches to historic monuments, grand palaces to centuries-old religious
temples.
Hence, option C is correct.

PAGE 381
www.byjusexamprep.com

83. Ans. B.
The Nyishi community are the largest ethnic group in Arunachal Pradesh in north-eastern India. In
Nyishi, their traditional language, ‘Nyi’ signifies to "a human" and the word ‘shi’ denotes "a being",
which united together refers to a human being. They are spread across eight districts of Arunachal
Pradesh: Kra Daadi, East Kameng, West Kameng, Kurung Kumey, Papum Pare, parts of Lower
Subansiri, Kamle and Pakke Kesang district. The most famous being the Akang and Leil community of
Papum Pare districts. They also live in the Sonitpur and North Lakhimpur districts of Assam.
Hence, option B is correct.
84. Ans. B.
Bora winds are cold and dry north-easterly winds that blow from the mountains towards the eastern
shore of the Adriatic Sea.
Bora is more effective in North Italy as here the wind get descends the southern slopes of the Alps,
although due to descend it gets adiabatically heated still its temperature is very low in comparison
to the coastal area, and these are the typical instance of fall winds.
Hence, option B is correct.
85. Ans. A.
Chenghiz Khan/Genghiz Khan did not cross the Indus to attack India while rest other Mongol leaders
like Tair Bahadur, Abdullah and Qutlugh Khwaja crossed the Indus to attack India.
Genghiz Khan was at the bank of Indus in the year 1221 chasing Jalal ud-Din of Samarkhand, but he
had a formidable ruler of Iltutmish to face in India if he attempted to go into Delhi. Few Indian towns
on the periphery such as Multan and Lahore were attacked.
Hence, option A is correct.
86. Ans. B.
Ghias-ud-Oin Balban (AD 1266-1287): He was an Ilbari-turk. Bahauddin was his original name. He was
the slave of the Iltutmish and gradually with time he occupied the throne of sultan. He destroyed the
power of Turkan-i-Chihalgani.
In order to crash the power of the Turk-e- Chihalgani and to raise the prestige and power of the
monarchy, he propounded a new theory of kingship.
Hence, option B is correct.
87. Ans. B.
Gaganendranath Tagore, along with his brother Abanindranath, is known for founding the Indian
Society of Oriental Art in the year 1907. This ISOA, sponsored by Europeans, much popularized
Tagore’s Bengal School, as well as art and crafts of other Asian nations. It held regular exhibitions in
India and abroad, and came out with exquisite colour reproductions of original paintings by
Abanindranath Tagore, Surendranath Ganguly, Nandalal Bose and other old masters of Mughal and
Rajput art.
Hence, option B is correct.

PAGE 382
www.byjusexamprep.com

88. Ans. B.
The All India Depressed Classes Leader’s Conference held at Bombay in 1931 backed by Dr.
Ambedkar's demand for separate electorates, rejecting compromises proposed by others. Before the
Round Table Conference of the year 1930-31 Ambedkar emerged as the main leader of the depressed
classes. He took a separatist stand and demanded constitutional safeguards for the depressed
classes. The untouchables demanded separate electorates in the 1930s, which led to a conflict
between Ambedkar and Gandhiji, with the former feeling cheated by the Poona Pact.
Hence, option B is correct.
89. Ans. D.
The following are the key features of the Government of India Act, 1935:
• The act calls for the formation of an All India Federation, but it never materialised because the
princely states refused to join.
• The act divided power between the central government and the provinces.
• There were three lists, one for each government: the Federal List (Centre), the Provincial List
(Provinces), and the Concurrent List (Both).
Hence, option D is correct.
90. Ans. C.
The Indian National Congress, on 19th December 1929, passed the historic ‘Purna Swaraj’ – (i.e. total
independence) resolution – at its Lahore session. A public declaration was made on 26th January 1930
– a day which the Congress Party urged Indians to celebrate as ‘Independence Day’. The declaration
was passed due to the breakdown of negotiations between the leaders of the freedom movement
and the British over the question of the dominion status for India.
Hence, option C is correct.
91. Ans. A.
Edward Jenner is a well known around the world for his innovative contribution to the immunization
concept and the ultimate eradication of the smallpox. Jenner's work is broadly regarded as the
foundation of the immunology—despite the fact that he was neither the first one to suggest that
infection with cowpox conferred specific immunity to smallpox nor the first one to attempt cowpox
inoculation for this purpose.
Hence, option A is correct.
92. Ans. B.
Beside nucleus, Mitochondria is another cell organelle which has its own self-replicating DNA. It is
known as mitochondrial DNA and thus called as a semi-autonomous organelle. Mitochondrial is an
oval-shaped semi-autonomous organelle as it contains circular DNA.
Lysosomes are known as garbage disposal as it contains a hydrolytic enzyme that digests wastes
material, cell debris, foreign particles and dead or weak organelles and excretes out from the cells.
Hence, option B is correct.

PAGE 383
www.byjusexamprep.com

93. Ans. B.
The gland which is the master gland is the Pituitary gland that controls the activity of other endocrine
glands. The pituitary gland is linked to the base of the brain. It secretes growth hormone.
Hence, the pituitary gland does not exist in pairs.
Hence, option B is correct.
94. Ans. A.
The correct sequence is: Kidney, ureter, urinary bladder, urethra
The organs, muscles, tubes and nerves that work together to form, store, and carry urine are
considered as the urinary system, which is likewise named for the renal system. The renal system
filters the plasma of the blood and regulates blood volume by excreting the excess water in the form
of urine. Urine transport observes a path through the kidneys, ureters, bladder, and urethra, which
are combinedly known as the urinary tract.
Hence, option A is correct.
95. Ans. C.
Radio frequencies allow information to be transmitted over large distances by radio waves. The
essential element to high-quality satellite communications is the assignment of radio-frequency
spectrum to various types of services. Only a limited amount of such spectra is assigned to Earth-
space radio links, and thus the available bandwidth must be used with a high degree of efficiency.
There are many technical elements associated with the efficient use of RF spectra for satellite
communications and navigation.
Hence, option C is correct.
96. Ans. C.
Resistance R (in Ohm), of a wire of length L (in cm) is directly proportional to its length, and inversely
proportional to its area of cross-section a (in cm²). Then
R = resistivity × length/ area.
In terms of units in CGS system, we have,
Ohm = resistivity × cm/ cm²= resistivity / cm.
Unit of resistivity = Ohm- cm (in CGS system of units)
= Ohm-meter (in SI units).
Definition of resistivity: It is the resistance between the ends of a conductor 1 cm apart, when the
ends have an area of cross section 1 cm².
Hence, option C is correct.
97. Ans. B.
Both the Celsius and Kelvin are the units of temperature. If you add 273 with the value of
temperature in Celcius, you will get the value of temperature in kelvin.
Hence, option B is correct.

PAGE 384
www.byjusexamprep.com

98. Ans. C.
Silver articles, when exposed to air become black after sometime. This is because the silver metal
reacts with sulphur present in the atmosphere and forms silver sulphide. Thus, a layer of silver
sulphide is formed on the surface of silver articles, because of which they appear dull and black.
Hence, option C is correct.
99. Ans. C.
The copper sulphate crystals contains 5 molecules of water of crystallization. CuSO₄.5H₂O which are
blue in colour.
Hence, option C is correct.
100. Ans. A.
• Washing soda is sodium carbonate (Na2CO3 . 10H2O), a strong alkali.
• while baking soda is sodium bicarbonate : sodium hydrogen carbonate (NaHCO3), a mild alkali.
• Calcium hypochlorite is an inorganic compound with formula Caocl2. . It is the main active
ingredient of commercial products called bleaching powder, chlorine powder, or chlorinated
lime, used for water treatment and as a bleaching agents.
• Calcium hydroxide (traditionally called slaked lime) is an inorganic compound with the chemical
formula Ca(OH)2. It is a colorless crystal or white powder and is produced when quicklime
(calcium oxide) is mixed, or slaked with water.
Hence, option A is correct.
101. Ans. B.
Mahatma Gandhi’s Dandi March, a great event in India freedom struggle, was associated with
Sodium chloride. The Salt March also called as the Salt Satyagraha, Dandi March and the Dandi
Satyagraha, was an act of non-violent civil disobedience in colonial India led by Mahatma Gandhi.
One of the major significant events in India’s struggle for independence took place when Mahatma
Gandhi launched the Civil Disobedience Movement with his famous Dandi March, which began on
12th March 1930 and ended on 6th April 1930.
Hence, option B is correct.
102. Ans. D.
An enzyme is a biological catalyst which speeds up chemical reactions. During digestion, food is
broken down to release nutrients which are absorbed. Salivary Amylase is the first enzyme that helps
in the digestive process. Amylase gets mixed with food in the oral cavity during the process of
mastication and bolus formation. Salivary Amylase is secreted in the saliva inside our mouth, and its
first acts on the carbohydrates found in the food that we chew. It breaks the complex compounds
like bread and rice into simpler forms such as glucose.
Hence, option D is correct.
103. Ans. B.
ATP: Adenosine triphosphate is known as the energy currency of the cell.
ATP is the organic compound composed of the phosphate groups, adenine, and the sugar ribose.
These molecules give energy for various biochemical processes in the body. Hence, it is known as the

PAGE 385
www.byjusexamprep.com

“Energy Currency of the Cell”. These ATP molecules are then synthesized by the Mitochondria;
therefore, it is known as the powerhouse of the cell.
Hence, option B is correct.
104. Ans. C.
Carbon dioxide gas turns lime water milky. We exhale carbon dioxide, so when air is blown from the
mouth, i.e., carbon dioxide, into a test tube that contains lime water, the lime water turns milky due
to presence of carbon dioxide. This is due to the formation of calcium carbonate when carbon dioxide
combines with calcium hydroxide (limewater). Lime water is a calcium hydroxide solution. On
reacting with carbon dioxide, it forms insoluble calcium carbonate which remains in water as a
suspension rendering a milky white colour to it.
Hence, option C is correct.
105. Ans. C.
The sequence of the events of sexual reproduction in flowers are:
Pollination: transfer of pollen to the stigma.
Fertilization: fusion of male gamete and egg cell produce zygote and fusion of polar nuclei with male
gamete produce triploid endosperm.
Division of zygote: The zygote divides through a process known as mitosis, in which each cell doubles
by dividing into two cells.
Embryo formation: zygote divides much time by mitosis and produces embryo.
Seedling: embryo differentiates and than becomes plumule, radicle and one or two cotyledons.
Hence, option C is correct.
106. Ans. B.
Salamander being an amphibian, has a three-chambered heart. Also, Lizard has a three-chambered
heart. Except for crocodilians, all the reptiles have a three-chambered heart. Crocodiles have a four-
chambered heart. Three-chambered consists of two atria and one ventricle.
Hence, option B is correct.
107. Ans. D.
Examples of major epidemics include cholera and diarrhoeal diseases, measles, malaria, smallpox
and dengue fever.
An Elephantiasis describes as a parasitic infection that causes extreme swelling in the arms and legs.
The disease is caused because of the filarial worm, which is transmitted from human to human via
the female mosquito when it takes a blood meal. The parasite results into an adult worm that lives
in the lymphatic system of humans.
Hence, option D is correct.
108. Ans. B.
We know that,
F = –kx
Therefore, k = F/x

PAGE 386
www.byjusexamprep.com

Dimension of F = [MLT–2]
Dimension of x = [L]
Therefore, dimension of k = [MLT–2]/[L]=[ MT–2]

The Spring Constant Formula is given as, k = Fx where,


F = Force applied,
x = displacement by the spring
The negative sign shows that the restoring force is opposite to the displacement
It is expressed in Newton per meter (N/m).
Hence, option B is correct.
109. Ans. D.

Hence, option D is correct.


110. Ans. A.
We know that speed of light in any medium is inversely proportional to the refractive index of that
medium.
Then, v1 = 3 x 108 m/s and n1 = 1 for air & n2 = 3/2 = 1.5 for the medium

Then,

PAGE 387
www.byjusexamprep.com

V2 = 2 x 108 m/s
Hence, option A is correct.
111. Ans. A.
The plane mirrors in a periscope are parallel with each other, which helps to reflect the light rays
from the upper end of the periscope to the lower end of the periscope. The plane mirrors in a
Periscope incline at 45° angle with the surface of the periscope

Hence, option A is correct.


112. Ans. C.
Plane mirrors are the only type of mirror for which a real object always produces an image that is
virtual, erect and of the same size as the object.
Hence, option C is correct.
113. Ans. A.
X−rays aren't deflected by electric and magnetic fields because x−rays do not carry any charge. They
are electromagnetic radiations and, therefore, cannot be deflected by electric or any magnetic fields.
Hence, option A is correct.
114. Ans. A.
Pitch describes how high or low a tone is and depends upon the rapidity with which a sounding body
vibrates, i.e., upon the frequency of vibration. The higher the frequency of vibration, the higher the
tone or pitch.
The frequency of a sound wave is what your ear understands as pitch. A higher frequency sound has
a higher pitch, and a lower frequency sound has a lower pitch.
Hence, option A is correct.

PAGE 388
www.byjusexamprep.com

115. Ans. A.
Boric acid: Boric acid is often used as an antiseptic, insecticide, flame retardant, neutron absorber,
or precursor to other chemical compounds.
Citric acid: Citric acid is used in food as a flavouring agent and preservative.
Magnesium hydroxide: It is used as a laxative to relieve occasional constipation.
Acetic acid: Acetic acid is used in many industrial processes for the production of substrates, and it
is often used as a chemical reagent.
Hence, option A is correct.
116. Ans. D.
The major constituent of gastric acid is hydrochloric acid produced by parietal cells in the gastric
glands in the stomach. Its secretion is a quite complex and relatively energetically expensive process.
Parietal cells comprise an extensive secretory network (known as canaliculi) from which the
hydrochloric acid is secreted into the lumen of the stomach. The pH of the gastric acid is 1.5-3.5 in
the human stomach. The parietal cell releases bicarbonate within the bloodstream in the process,
which results in a temporary rise of pH in the blood, known as an alkaline tide.
Hence, option D is correct.
117. Ans. B.

• At pH 7, the substance or solution is at neutral and means that the concentration of H+ and OH-
ion is the same.
• If pH < 7, the solution is acidic. There are more H+ than OH- in an acidic solution.
• If pH >7, the solution is basic.
So, option B is correct.
118. Ans. C.
These are the displacement reactions.

PAGE 389
www.byjusexamprep.com

A single-displacement reaction, also known as a single-replacement reaction, is a reaction by which


one element replaces an/other element in a compound. It can be represented generically as: A + B-
C → A-C + B This will most often occur if A is more reactive than B, thus giving a more stable product

Cu + PbCl2→ CuCl2 + Pb
The following reaction is not possible because in the metal reactivity series, copper being below
hydrogen and it is unable to displace lead from its solution.
So, option C is correct.
119. Ans. D.

Ar and Ca are isobars as they have same mass number 40 but different atomic number.

Hence, option D is correct.


120. Ans. C.
In 1909, Ernest Rutherford’s student reported some unexpected results from an experiment
Rutherford had assigned him. Rutherford called this news the most incredible event of his life.
In the now well-known experiment, alpha particles were observed to scatter backwards from a gold
foil. Rutherford’s explanation, which he published in May 1911, was that the scattering was caused
by a hard, dense core at the center of the atom–the nucleus.

Hence, option C is correct.

PAGE 390
www.byjusexamprep.com

Elementary Mathematics
1. Ans. D.

Let a circle of radius R = 8 cm and centre O touches the axis and a small circle of radius r and centre
at P are in the gap.
Let the distance between the small circle and the origin is x.
Then in small circle

And in big circle

2. Ans. C.
Let the side of the square is a, then one side of the rectangle is a2, and another side is b.
According to the question, area of square = area of the rectangle

Another side of the rectangle is of unit length.

PAGE 391
www.byjusexamprep.com

3. Ans. D.

If the length and breadth of a rectangle are in the ratio 4: 3.


Let the sides of the rectangle are 4x and 3x.

Now the diagonal of the rectangle

The sides of the big triangle are 2.5x,2.5x and 4x

And area =

The sides of the small triangle are 2.5x,2.5x and 3x

And area

So the required ratio is 1:1


4. Ans. B.
D A

C B
In the right-angled triangle ABD

5. Ans. A.
Let a parallelogram ABCD where line AQ meets at the middle point of CD at P.

PAGE 392
www.byjusexamprep.com

i. Since BC and AD is Parallel then

And CP = PD, both the triangle APD and CPQ are congruent. So their areas are equal.
ii. In the triangle AQB and CPQ
Angle BQA = Angle CQP = same angle
Since the triangle PCQ is a part of the triangle BQA. So their areas can’t equal.

6. Ans. C.
The lengths of sides of a triangle are 3x, ,5 , where 3x < <5 . Since it makes a right-
angle triangle. Then,

If the value of x,y and z are 1 then it satisfies the condition of the right-angle triangle.
7. Ans. A.
A triangle has a maximum of 1 circum-circle that touches all the vertex of the triangle.
8. Ans. A.

The length of the arc

9. Ans. C.

5m
5m 4.8m
1.4m

x y
As we can see in the above figure, the ladder makes a right-angle triangle, and the sum of the base
of the triangle is the breath of the room.

PAGE 393
www.byjusexamprep.com

So the breath of the room = x + y = 4.8 + 1.4 = 6.2m


10. Ans. D.
If a square of maximum area is cut from the circular disk, then the vertex of the square touch the
circle.
Let the side of the square is a, and the radius of circle is 1 unit, then
The diagonal of the square = diameter of the circle

So the area of the largest square plate

11. Ans. A.
Let ABCD is a quadrilateral such that AD = DC = CA = 20 units, BC = 12 units and = 90°
A B

12

D 20 C
Since we can see the triangle ABC are a triangle and ABC is a right-angle triangle. In right angle
triangle ABC

Now the total area

12. Ans. A.
The radius of the circle is 2 cm. when we join the centre of each circle it makes equilateral triangle

PAGE 394
www.byjusexamprep.com

The area of the circle which is inside the circle

The areas of the shaded region = area of triangle – area of the part of the circle

13. Ans. B.
The sides of the rectangle are and 6 unit

Radius of the circle

A P 3 3 B

60 3
6 Q
O

D C
6 3

The angle at the centre by long arc

So
The shaded area = Area of part of circle OAB – Area of triangle AOB

14. Ans. C.

if = 6 and = 5, then y = 6x and z = 5x,

As we can see that, x + y + z = 180°


or x + 6x + 5x =180°
or 12x = 180°

PAGE 395
www.byjusexamprep.com

or x = 15°
15. Ans. D.
In the trapezium AB = PQ = 4 cm
Let AP = BQ = h and DP = x, then QC = CD – DP - PQ = 7 – 4 – x = 3 – x
Since APD and BCQ is right angle triangle, so

Now, the area of the trapezium

16. Ans. B.
In each triangle, 3rd angle = 180° - sum of other two angle
Since opposite angle are equal then,

17. Ans. B.
Since PQ and RS are parallel then Angle AEF = Angle EGH = 95°
Angle BGH = 180° – 95° = 85°

PAGE 396
www.byjusexamprep.com

Angle BHG = 180° – 110° = 70°


Now in triangle BGH, Angle BGH +Angle GHB + Angle GBH = 180°
85° + 70° + x = 180°
x = 180° – 155°
x = 25°
18. Ans. D.
Given that AC = 25 units
Since F is the midpoint of AC , so AF = FC = 12.5 units
Radius of semicircle ABC = 25/2 = 12.5 unit
Radius of semicircle AEF and CDF = 12.5/2 = 6.25 cm
Total area of the figure

19. Ans. D.
The sides of the rectangle are 8cm and 6cm, the diagonal of the rectangle

And the radius of the circle =

Area of the shaded region = Area of the circle – Area of the rectangle

20. Ans. A.
We know that if we add one to the multiplication of all prime numbers from starting then result
obtained is also a prime number.
Consider 2x3x5x7x11+1
Note : 2,3,5,7 and 11 all are prime numbers and 1 is also added to their product.
So, 2x3x5x7x11+1 is a prime number.
21. Ans. B.
Let x = (54)10
Taking logarithm both sides

PAGE 397
www.byjusexamprep.com

Since the characteristic of log x is 17


So, number of digits in (54)10 = 17 + 1 = 18
22. Ans. D.
Total students registered = 250 students
Students registered for Maths = 75
Students registered for Physics = 50
Students registered for both subjects = 35
Venn Diagram:

Number of Students registered for at least one subject = 15 + 35 + 40 = 90


Hence, Number of students who have registered neither for Physics nor for Mathematics = 250-90
= 160
23. Ans. A.
Consider 22222, 22222, 22222, 22222
Take logarithm

Now, log 2 = 0.301

PAGE 398
www.byjusexamprep.com

log (1111) = 3(approx.)


log(111) = 2 (approx.)
log(11) = 1 (approx.)
If we put these values, we can clearly see that is largest.

⇒ 22222 is largest.
24. Ans. A.
Let unit digit of number = y
Ten’s digit of a number = x
Number = 10x + y………….(1)
If digits are interchanged, then
unit digit of number = x
Ten’s digit of a number = y
Interchanged Number = 10y + x………….(2)
According to question
10x + y = 4(10y + x) + 9
⇒ 10x +y = 40 y + 4x +9
⇒ 6x-39y = 9
⇒ 2x – 13y = 3………………(3)
Also, product of digits in the two-digit number is 8
⇒ xy = 8

⇒ ……………(4)

Put value of y from (4) to equation (3)


Solve quadratic equation using quadratic formula

⇒ x = 8,

PAGE 399
www.byjusexamprep.com

As x is a digit so only possible value of x = 8.


Put value of x in equation (4)
y=1
Required Number =

25. Ans. A.
Given α and ß are the roots of the quadratic equation x2 + kx – 15 = 0

Sum of roots =

⇒ α +ß = -k ………………..(1)

Product of roots =

⇒ (α.ß) = - 15…………….(2)
Also , α-ß=8 (Given)

We know that

According to question, k =2
26. Ans. B.
If (x+k) is the HCF of x2+5x+6 and x2+8x+15 then
x = -k will satisfy both equations x2+5x+6 and x2+8x+15
Put x = -k in equation x2+5x+6
⇒ k2-5k+6=0……………(1)
Put x = -k in equation x2+8x+15
⇒ k2-8k+15=0………..(2)
Subtract (1) from (2)
⇒ –3k +9 = 0
⇒ –3k = -9
⇒k=3
27. Ans. A.
Consider x2+ 9y2=6xy

PAGE 400
www.byjusexamprep.com

Hence, y : x = 1 : 3
28. Ans. B.

Consider

29. Ans. B.

Consider x = ,y= and z =

Now we will take LCM of 2,3,6


⇒ LCM(2,3,6) = 6

Clearly, …………(1)

Also x , y and z all are positive numbers.


So from equation (1) we can conclude that z < x < y
30. Ans. C.

PAGE 401
www.byjusexamprep.com

Given y = xlogx

But log x=1.2500

Hence, log y = = 1.5625

31. Ans. C.

Area of a square = 2401x4 + 196x2 + 4 =

We know that , Area of square =

32. Ans. C.
Given, x varies as yz

y varies inversely as

33. Ans. B.
5x + 2y = 7xy ……………(1)
10x + 3y = 8xy ………………(2)
Multiply equation (1) by 2
10x + 4y = 14xy ……………(3)
Subtract equation (3) from equation (2)
⇒ –y = –6 xy
⇒ y = 6xy

⇒x=

Put value of x in equation (1)

PAGE 402
www.byjusexamprep.com


34. Ans. D.
Let number of rows= x
Number of students in each row = y
Hence, Number of students = xy
If number of students is increased by 2 in each row, then the number of rows decreases by 3
⇒ (x-3)(y+2)= xy…………(1)

⇒ ……………..(2)
If number of students is increased by 4 in each row, then the number of rows decreases by 5
⇒ (x-5)(y+4) = xy

⇒ ………….(3)
Multiply equation (2) by 2 and then subtract from equation (3)
⇒y–8=0
⇒y=8
Put the value of y in (2)
⇒ x = 15
Hence number of students in the class = xy =

35. Ans. A.
Consider 3, 5, 9, 4, 6, 11, 18
Arrange the data in ascending order
3,4,5,6,9,11,18
Here, Total number of observation (n) = 7

As n is odd, median =

PAGE 403
www.byjusexamprep.com

4th term is 6.
Hence, median will be 6.
36. Ans. B.
Ratio of the angles of the sectors is 1 : 2 : 3
Let angle of the first sector = x
Angle of the second sector = 2x
Angle of the third sector = 3x
We know that complete angle of a pi-diagram is 360°
⇒ x + 2x + 3x = 360°
⇒ 6x = 360°
⇒ x = 60°
Angle of the largest sector = 3x = 3 × 60° = 180°
37. Ans. C.
Given the maximum marks in a Test are converted from 250 to 50 for the purpose of an Internal
Assessment.

This means marks are reduced to th for the purpose of an Internal Assessment.

The highest marks scored were 170 and lowest marks were 70.
For internal assessment:

Equivalent of 170 =

Equivalent of 70 =

Required Difference = 34 – 14 = 20
38. Ans. B.
Total released convicts from Jail A = 86 + 45 +25 = 156

% of trained convicts of jail A =

Total released convicts from Jail B = 1305 + 903 +461 = 2669

PAGE 404
www.byjusexamprep.com

% of trained convicts of jail B =

Total released convicts from Jail C = 2019 +940+474=3433

% of trained convicts of jail C =

Total released convicts from Jail D =1166 +869+416 =2451

% of trained convicts of jail D =

Total released convicts from Jail E =954+544+254=1752

% of trained convicts of jail E =

Total released convicts from Jail F = 1198+465+174=1837

% of trained convicts of jail F =

Jails with highest and smallest percentage of trained convicts are respectively are D and F.
39. Ans. D.

Placement rate of trained convicts of Jail A = 55.55

Placement rate of trained convicts of Jail B = = 51.05

Placement rate of trained convicts of Jail C = = 50.43

Placement rate of trained convicts of Jail D = = 47.87

Placement rate of trained convicts of Jail E = = 46.69

Placement rate of trained convicts of Jail F = = 37.41

Clearly, Jail A have highest placement rate of trained convicts


40. Ans. A.
Trained convicts of Jail A = 45
Half of the trained convicts = 22.5
Convicts who got job from Jail A = 25
Trained convicts of Jail B = 903

PAGE 405
www.byjusexamprep.com

Half of the trained convicts = 451.5


Convicts who got job from Jail B = 461
Trained convicts of Jail C = 940
Half of the trained convicts = 470
Convicts who got job from Jail C = 474
Trained convicts of Jail D = 869
Half of the trained convicts = 434.5
Convicts who got job from Jail D = 416
Trained convicts of Jail E = 544
Half of the trained convicts = 272
Convicts who got job from Jail E = 254
Trained convicts of Jail E = 465
Half of the trained convicts = 232.5
Convicts who got job from Jail E = 174
Jails from which more than half of the trained convicts are offered jobs, are A, B, C
41. Ans. D.
1st outlet: A & 2nd outlet: B
A & B both opened together takes 20 min. to empty the tank
While when A opens, it takes 30 min. to empty the tank.
A+B A
20 min 30 min

3 2

60 min
B takes : 1 unit à60 min
Hence, option D is correct.
2nd method:
By using the formula
y is the time taken when both outlets are open
And x is the time taken by 1st outlet.

Then B takes = min.

42. Ans. C.

PAGE 406
www.byjusexamprep.com

1
Given: (x8 + ) = 47
x8
1
(x8 + ) + 2 = 47 + 2
x8
Make perfect square
1
Then (x4 + )=7
x4
Again add 2 both side and try to make a perfect square
1
(x4 + )+2=7+2
x4
1
Then you will get, (x2 + )=3=k
x2
1
Then you know that (x6 + )3 = k3 – 3k = 33 – 3x3
6
x
= 27 – 9 = 18
Hence, option C is correct.
43. Ans. A.

Here, angle CBI = angle BAD

Then , BD = 40 – 6 = 34 cm

Then, IF = 40 feet’s
Now, CF = IF + CI

PAGE 407
www.byjusexamprep.com

CF = 40 + 45.33
CF = 85 feet’s (approximately)
Hence option A is correct.
44. Ans. D.
Given: sin26°+sin212°+sin218°+….+sin284°+sin290°
sin290° = 1,
sin26°+sin212°+sin218°+….+sin284°+1
observe angles are in A.P : 6, 12, 18, 24, 30, 36, 42 ………., 84
convert last half of sin term in to cos term, i.e,
+1

1 + 1 + 1 …. + 1 = 8
Hence option D is correct.
45. Ans. B.

We have AD = 12 cm, BD AC (property)


Now just apply the Pythagoras theorem in triangles ABD and BDC to get AD and DC respectively.

In triangle ADB: AD =

Similarly, DC = 16 cm ( use Pythagoras in ADB)

AC (D) = AD + DC = + 16 cm

26 cm < d < 27 cm
Hence, option B is correct.
46. Ans. A.
Length = 48 cm

PAGE 408
www.byjusexamprep.com

And breadth = 14 cm. it is given that the diagonal makes an angle with the longer side

Then,

Perpendicular = 14 cm, base = 48 cm then we can find hypotenuse by Pythagoras theorem

Hypotenuse (H) =

H = 50 cm

So, option A is correct.


47. Ans. B.

4 cm r

Let a circle of radii R and a small circle of radii r touches internally.


Hence R= r+4 (i)

Acc. to question, sum of their areas is 136 πcm2

(r-6) = 0 or r = 6
Hence the radii of the small circle is 6 cm, and the big circle is 10 cm.

PAGE 409
www.byjusexamprep.com

48. Ans. C.
Let the radius of the circle is r and the sides of the square is a.
Then according to the question
Area of circle = Area of the square

Hence the ratio of their perimeter is

49. Ans. C.
If the thickness of cylinder is 1 foot and the inner radius of the cylinder is r1 = 3 feet, then the outer
radius is r2 = 4 feet. And height is h = 7 feet.
The inner surface of the circle is

The total surface of the cylinder is

So total required point is

50. Ans. D.
The radius of the circle is 30 feet.

Then the area of the circle

PAGE 410
www.byjusexamprep.com

If a sector of 20° is removed, then the only sector of 340° area remains.

51. Ans. D.
When we dug in such a way, then it makes a cylinder
The cubic metre volume of earth to be dig is

52. Ans. A.
Let a rhombus which diagonals are x and y. Since the diagonal of rhombus cut each other at right
angle and at the midpoint. Then area of the rhombus

y/2
x/2

53. Ans. D.

let A, B and C are 3 balls of radius r are placed in the plane, and 4th ball P is placed on the top of them.
Let h is the distance of centre of ball P from the ground.
In the triangle ABC, let a point Q at the mid point of BC. Since triangle ABC is a equilateral then

PAGE 411
www.byjusexamprep.com

Angle ACB = 60 and OCB = 30 so in the OCQ

Again in the triangle POC which also a right angle triangle

Observe that the taken distance from the plane is situated at above radius r frim the given point.
So desired distance will be addition of radius r into obtained value of h.

54. Ans. A.
Let h is the hight of the cylinder and r is the radius.
If a right circular cylinder just encloses a sphere. Then
height of cylinder = diameter of the sphere = diameter of the cylinder
h=2r
now,

p = the surface area of the sphere =

q = the curved surface area of the cylinder =

here we can see the, p = q

PAGE 412
www.byjusexamprep.com

55. Ans. B.
The diameter PS is divided in three equal parts
PS = 2(9) = 18 cm
PQ = QR = RS = PS/3 = 6 cm
The area of the shaded region

The area of the unshaded region

So the required ratio

56. Ans. A.
Since ABCD is a rhombus, then

angle BAC = angle FDG = 55


now, at point A

angle HAB + Angle BAC + angle CAE = 180

x + 55 + 40 = 180

x = 180 - 95

x = 85

PAGE 413
www.byjusexamprep.com

57. Ans. B.
ABCD is a trapezoid where AD is parallel to BC and perpendicular to the line CD. Hence ∠ADC and
∠BCD is right angle.
II. At point O ∠AOD and ∠BOC are opposite angle.
Hence, ∠AOD = ∠BOC
III. In ΔADC, AD + CD > AC ...(i)
and In ΔBDC, BC + CD > BD …(ii)
By adding both equation
AD + CD + BC + CD > AC + BD
58. Ans. B.
Given that the product of the two numbers in each pair is 2160 and the HCF of the two numbers in
each pair is 12.
Let those numbers are ‘a’ and ‘b’.
HCF(a,b) = 12
We can write, a = 12m and b = 12n where m and n are co-prime.
Also , ab = 2160
⇒ (12m)(12n)=2160
⇒ 144mn = 2160
⇒ mn = 15
Now there are two cases
Either m = 5 and n = 3
Or m = 15 and n =1
So, Either a = 60 and b = 36
Or a = 180 and b = 12
Hence possible pairs of unequal number satisfying both the conditions are (60,36) and (180,12)

Now, mean of 60 and 36 (x) =

Mean of 180 and 12 (y) =

Mean of x and y =

59. Ans. B.
m is the number of prime numbers between 0 and 50

PAGE 414
www.byjusexamprep.com

Prime numbers between 0 and 50 =2,3,5,7,11,13,17,19,23,29,31,37,41,43,47


Number of prime numbers between 0 and 50 = 15
⇒ m = 15
Prime numbers between 50 and 100 = 53 , 59 , 61 , 67 , 71 , 73 , 79 , 83 , 89 , 97
Number of prime numbers between 50 and 100 = 10
⇒ n = 10
Hence , m – n = 15 – 10 = 5
60. Ans. C.

3x+3 x+1 +3 x+2 =

Here, x is a natural number.


Put x = 1

Put x = 2

= , which is also divisible by 39

Hence, largest divisor of 3x+3 x+1 +3 x+2, if x is any natural number will be 39.
61. Ans. A.
If (x-1) and (x+2) divide the polynomial x3+4x2+px+q
Then x = 1 and x = -2 both will satisfy equation x3+4x2+px+q
Put x= 1 in equation x3+4x2+px+q

⇒ ………….(1)
Put x= -2 in equation x3+4x2+px+q

⇒ …………..(2)

Subtract (2) from (1)


⇒ 3p=3
⇒p=1
Put the value of p in equation (1)
q=-6
62. Ans. D.

PAGE 415
www.byjusexamprep.com

Given 5x+1 -5x-1=600………..(1)


Also, ………….(2)

From (1) and (2)


⇒x=3

Hence , 102x =

63. Ans. C.
f(x) is divided by (x-α) (x-ß) where α≠ß
Then f(x) = (x-α) (x-ß) g(x) + (ax+b) ………(1)
where g(x) is quotient and (ax+b) is linear remainder
Put in (1)

…..(2)

Put in (1)

………(3)

Subtract (3) from (2)

……….(4)

Put the value of ‘a’ in equation (2)

………..(5)

Remainder = (ax+b) = =

64. Ans. A.

Points P and Q represent real numbers and on the number line.

Let =x
Multiply by 10 both side
7.333333….. = 10x…………(1)

PAGE 416
www.byjusexamprep.com

Multiply by 10 both side again


⇒ 73.3333333….. = 100x…………..(2)
Subtract (1) from (2)
⇒ 90x = 66

Let =y
Multiply by 10 both side
5.666666….. = 10y…………(1)
Multiply by 10 both side again
⇒ 56.666666….. = 100y…………..(2)
Subtract (1) from (2)
⇒ 90y = 51

Distance between P and Q =

65. Ans. C.
Ratio of speeds of X and Y is 5 : 6.
Let speed of X = 5x
Speed of y =6x
Distance travelled by X = (1200-70) = 1130 m
Distance travelled by Y = 1200 m

Time taken by X to travel 1130 m =

Time taken by Y to travel 1200 m=

As

Hence , Y wins the race.


Let Y completes the race ‘t’ time
And X covers a distance of l metre in ‘t’ time.

PAGE 417
www.byjusexamprep.com

If time is constant then Ratio of speed = Ratio of distance

Hence , Distance by which Y wins the race = 1200 – 1000-70 = 130 m


66. Ans. D.
Let usual speed of train = x km/hr
Distance travelled = 300 km

Time taken by train = hours

It is given that train takes two hours less for a journey of 300 km if its speed is increased by 5 km/hr
from is usual speed.
New Speed = (x+5) km/hr

So, time taken by train = hours

According to question


⇒ x = - 30 or x = 25
But speed cannot be negative.
Hence, x = 25 km/hr
Usual speed = 25 km/hr

PAGE 418
www.byjusexamprep.com

67. Ans. A.
Let a man’s one day work = x unit
Woman’s one day work = y unit
6 men and 8 women can do a piece of work in 10 days

Work done by 6 men and 8 women in one day =

⇒ 6x + 8y = ……………..(1)

Now, 13 men and 24 women can do the same work in 4 days.

⇒ 13x + 24 y = …………….(2)

Multiply equation (1) by 3 and subtract from (2)

⇒ -5x =

⇒x=

Put the value of x in (1)

Hence , Ratio of x and y = 2:1.


68. Ans. C.
In a class room the ratio of number of girls to that of boys is 3 : 4.
Number of boys = 4x
Number of girls = 3x
Total number of students =7x
Average height of students in the class is 4.6 feet.

PAGE 419
www.byjusexamprep.com

Sum of heights of all the students = =


Average height of the boys in the class is 4.8 feet

Sum of heights of all the boys =

Sum of heights of all the girls =

Average height of the girls in the class =

Hence, Average height of the girls in the class More than 4.3 feet but less than 4.4 feet
69. Ans. C.
Let the three digits number = abc
Then, the reverse of it will be cba
Both the numbers are divisible by 7 then both the number should be multiple of 7
We can write both the number as
7X = 100a + 10b + c ……………1st eq.
7Y = 100c + 10b + a ……………..2nd eq.
Subtract 2nd from 1st, we get
7(x-y) = 99(a – c)
Here, 99 is not divisible by 7 then, it is compulsory that (a-c) should be a factor of 7.
Then the possible value of (a – c) are a = 2 & c = 9, a = 1 & c = 8
Then possible numbers are 168, 861, 259 &952
Note: 343 will not be the required number as the three digits should be different
Hence C will be correct option.
70. Ans. A.

Divide numerator and denominator by cos θ, then we get

PAGE 420
www.byjusexamprep.com

So, option A is correct.


71. Ans. A.

Convert every term in tan form


Then we get

Hence, option A is correct.


72. Ans. C.

Multiply numerator and denominator by


Then,

Hence, option C is correct.

PAGE 421
www.byjusexamprep.com

73. Ans. C.

Now, here we have base = 63, perpendicular = 16, then with the help of Pythagoras theorem, we can
find the hypotenuse as 65.

Hypo = 65

Then

Then

Hence, option C is correct.


74. Ans. D.
Either you can directly check it by just putting some values of θ for ex: as θ = 0 or 90°, you will get 5
as your answer.
Or go through the conventional method

Put 1 as and expand it.

Then +4=1+4=5

Hence, option D is correct.


75. Ans. C.

triangle having side lengths

As we can see from here all sides are of different length, then the triangle will be a scalene triangle,

PAGE 422
www.byjusexamprep.com

Then, the area of triangle = where a, b, c are


the sides of the triangle.

Here, s =

S=

Put the value of each term, then you will get,

Area = =

Hence, option C is correct.


76. Ans. C.

The angles of a triangle are 30° and 45° and the included side is ( 3 +1) cm
1st angle = 30°
2nd angle = 45°
3rd angle = 180 –(30 + 45) = 105°

Then angle 1050 will be opposite to ( 3 +1) cm side

Similarly x cm is the side opposite to 45o angle.


We can use here sin rule.

After solving you will get X = 2 cm

PAGE 423
www.byjusexamprep.com

Area =

Area =

1
Area = ( 3 +1) cm2
2
Hence, option C is correct.
77. Ans. D.

Now, as we know that the diagonal of a parallelogram cut the parallelogram in to four equal parts
i.e, the area of the triangles formed by the diagonals should always be equal in a parallelogram
(property of a parallelogram)
i.e, Ar(ABO) = Ar(BCO) = Ar( DOC) = Ar(DOA)
we can also easily prove that the triangles DOE and EOC are congruent to each other

i.e Ar( DOE) = Ar( EOC) = Ar(DOC) = Ar(ABCD)

then Ar(remaining portion after removal of the triangle DOE) = Ar(ABCD)

, now , we have to find the ratio of area of remaining portion of the plate to the whole =

Hence, option D is correct.

78. Ans. C.

PAGE 424
www.byjusexamprep.com

AD ⊥ BC & AO = 5 cm (Radius)
Assume OD = x cm & DC = BD = y cm, AD = 5 + x
Now, in triangle ADC & triangle ODC : DC is common.
82 -(5 + x)2 = 52 – x2

From here after simplification, we get x =

Then y =

Then, BC = 2y = 2 x 4.8 = 9.6 cm


Hence, option C is correct.
79. Ans. A.

Given ………………..(1)

⇒ …………………(1)

Note : If then a = b
From (1)


Squaring both sides

PAGE 425
www.byjusexamprep.com

⇒ x = n2
If x = n2 then correct option will be either A or D
Now we will check whether x = 1 or x = n satisfy equation (1)
Put x = 1 in equation (1)

⇒1=1
This means x = 1 satisfies equation (1)
Put x = n in equation (1)

This means x = n does not satisfy equation (1)


Hence correct option will be A.
80. Ans. B.

Number under consideration:

Put n = 1 ,

Put n = 2 ,

Put n = 3 ,

Clearly, Sum of digits of = 9n , where n is a natural number.

⇒ 9n = 3798
⇒ n = 422
81. Ans. C.
Given that a number divides 12288, 28200 and 44333 so as to leave the same remainder in each
case.
Let when 12288, 28200 and 44333 are divided by x and r is obtained as a remainder.

Subtract (1) from (2)

PAGE 426
www.byjusexamprep.com

⇒ ……………(4)

Subtract (1) from (3)

Subtract (2) from (3)

From (4), (5) & (6) we can say that 15912 , 32045 and 16133 all are divisible by x
HCF (15912 , 16133, 32045) = HCF {(16133-15912) , ( 32045 – 16133) , (32045 – 15912)}
= HCF(221 , 15912 , 16133) = 221
Hence , x = 221
221 divides 12288, 28200 and 44333 so as to leave the same remainder in each case.
82. Ans. B.
Given m and n are positive integers such that mn = 1331.

We know that

Hence, m = 11 and n = 3

⇒ (m-1)n-1 =

83. Ans. B.
Price of article X was ₹ 5000 and price of article Y was ₹ 2000 in the year 2010.
Difference = 5000 – 2000 = 3000 Rs.

In 2012, Price of article X =

Price of article Y =

Difference = 7200 – 2420 = Rs. 4780 which is less than 5000

In 2013 , Price of article X =

Price of article Y =

PAGE 427
www.byjusexamprep.com

Difference = 8640 – 2662 = Rs. 5978 which is more than 5000


Hence , Required year = 2013
84. Ans. D.
Let a sum of Rs. x was put at simple interest at rate of r % per annum for 2 years.

⇒ …………..(1)

Had it been put at 1% higher rate of interest, it would have fetched ₹24 more.

⇒ ……….(2)

Subtract (1) from (2)

⇒ 24 =


Hence, Required sum = Rs. 1200
85. Ans. D.
Population of first village = 1525
Ratio of male to female population in the first village is 27 : 34
If male population in the first village is 27x then number of female in the first village will be 34x
⇒ 61x = 1525
⇒ x = 25
Hence, population of male in the first village =
population of female in the first village =
Population of second village = 2600
Ratio of male to female population in the second village is 6:
If male population in the second village is 6y then number of females in the second village will be 7y
⇒ 13y = 2600
⇒ y = 200
Hence, population of male in the second village =

PAGE 428
www.byjusexamprep.com

population of female in the second village =


Total number of male in both villages = 675 +1200 = 1875
Total number of Female in both villages = 850 +1400 = 2250
Required Ratio = 1875 : 2250 = 5:6
86. Ans. B.
Here, the given equation is 5x + 9y = 7
Under the condition: x & y should be integer b/w -500 to 500.
Now, we have to check the value by hit and trial method and try to observe the relation

Here, 1st convert the given equation as y =

Try to put some value of x (integer) to get y as integer

Here, we can see that when we put x = -4, -13, -22, -31…………..(common difference b/w each term =
9). Then y = integer value.
As x [-500, 500]
X may be -495, -486, ……………..-4, -13 , -22, ………….., 495 ( common difference = 9)
It is nothing but an A.P having 1st term = -495 & last term = 495 and d ( common difference) = 9
Then, no. of possible value (n) = no. of terms of the A.P

n=

Hence, option B is correct.

87. Ans. D.
It is given that, S = XYZ + YZX + ZXY

PAGE 429
www.byjusexamprep.com

Then it can be written as:S = 100x + 10y +z + 100y + 10z + x + 100z + 10x + y
S = 111(x + y + z)
= 3 × 37(x + y + z)
Now, check the given condition,
I. S is always divisible by 3 and (X + Y + Z) : true
II. S is always divisible by 9 : not necessary (if x+y+z = multiple of 3. Only then s is divisible by 9.)
III. S is always divisible by 37 : true
Hence, option D is correct.
88. Ans. C.
It is given that: For two observations, the sum is S and product is P
Assume two observations are x & y then x + y = S and x.y = P
And we know that H.M =2xy/(x+y) = 2P/S
Hence, option C is correct.
89. Ans. C.
We have to find the least perfect square, which is divisible by 3, 4, 5, 6 and 7
A number which is divisible by 3,4,5,6 and 7 should be a multiple of 3,4,5,6 and 7
N=3×4×5×6×7×A
Now, the number should also be a least perfect square, then to make ‘N’ a perfect square, we have
to take A = 3 × 4 × 5 × 6 × 7
Then N become 32 × 42 × 52 × 62 × 72 = 44100
Hence, option C is correct.
90. Ans. D.
Given: (x2 – 1) is a factor of ax4+bx3+cx2+dx+e
i.e, (x-1)(x+1) is a factor of ax4+bx3+cx2+dx+e
then x = 1 & -1 will be the rots of the equation and will satisfy the given equation i.e remainder = 0
f(-1) = 0
f(-1) = a(-1)4+b(-1)3+c(-1)2+d(-1)+e = 0
a – b + c – d +e = 0
a+c+e=b+d
hence, option D is correct.
91. Ans. C.
A wheel makes 360 revolutions in one minute

PAGE 430
www.byjusexamprep.com

Means 6 revolution per second


And we know that 1 revolution = 2π radians
Then in 1 sec the number of radians it turns = 6 x 2π radians = 12π
Hence, option C is correct.
92. Ans. B.
Given: (25 cosec2x + sec2x)

We have,

Then, minimum value of the given function = (remember it)

i.e,

Hence, option B is correct.


Note: Other method is to go through the conventional method i.e, the concept of maxima minima,
first, find the angle at which dy/dx = 0 then check the condition for maxima or minima. But for the
exam point of view, this method is not suitable. ( too much lengthy)
Hence, it is convenient to remember the result for the given particular type of function.
93. Ans. A.
Given: α cotnθ

It means …………….1st eq.

An it is given that 100 θ = 90° it implies that 900 = 100θ


Then,

Now, rearrange the terms

Then α = 1
Hence, option A is correct.

PAGE 431
www.byjusexamprep.com

94. Ans. A.
If tan 6θ = cot2θ
It is only possible when ‘6θ + 2θ = 90o
i.e, 8θ = 900
4θ = 45o

Then sec4θ = sec45o = 2


Hence, option A is correct.
95. Ans. C.

x + y = 15 meter ………….1st eq.

From 1st , we get : 2y + y = 15


3y = 15
y = 5 meter
hence, height = 5 meter.
So, option C is correct.
96. Ans. B.

Assume H is the height of the tower.

PAGE 432
www.byjusexamprep.com

Then, AB = H, ……1st eq. & ……2nd eq.

Multiply e. 1st and 2nd ,

Tan63o.tan27o =

Here, tan(90o – 63) = cot27o


Now, AB2 = BC.BD

Hence, option B is correct.


97. Ans. B.

Hence, option B is correct.


98. Ans. C.

Given = ………………(1)

=2 …………..(2)

= ……………….(3)

=3 ………………….(4)

= ……………….(5)

Multiply (1) , (2) and (3)

⇒ ………………(6)

PAGE 433
www.byjusexamprep.com

Multiply (2) , (3) and (4)

⇒ ………………(7)

Multiply (3) , (4) and (5)

⇒ ………………(8)

Multiply (6) , (7) and (8)

= =

99. Ans. C.
We know that when the distance is same, then speed is inversely proportional to time i.e,

ty: tx = 4:5
=> Then 1 unit difference 45 minutes
Time taken by y = 4 units = 4 x 45 = 180 minutes
Hence, option C is correct.
100. Ans. B.
Let Y’s income = 100 units
Then X’s income = 100 + 20 = 120 units (X’s income is 20% more than that of Y)

Now, Y’s income less than that of X’s income =

Hence, option B is correct.

PAGE 434
www.byjusexamprep.com

CDS II 2019
ENGLISH
Direction: In this section, a word is spelled in four 7. Choose the alternative bearing the correct
different ways. You are to identify the one which spelling from (a), (b), (c) and (d).
is correct.
A. Embarassment
1. Choose the alternative bearing the correct
spelling from (a), (b), (c) and (d). B. Embbarasment

A. Accommodate B. Acomodate C. Embrasement

C. Accommodate D. Acomodait D. Embarrassment


2. Choose the alternative bearing the correct Directions: Given below are some
spelling from (a), (b), (c) and (d). idioms/phrases followed by four alternative
meanings to each.
A. Recommand B. Reccommend
C. Recommend D. Reccomand Choose the response (a), (b), (c) or (d) which is
the most appropriate meaning.
3. Choose the alternative bearing the correct
spelling from (a), (b), (c) and (d). 8. Dirt cheap

A. Argyument B. Argument A. Extremely cheap


C. Argument D. Argyooment B. Extremely costly
4. Choose the alternative bearing the correct C. Very cheap person
spelling from (a), (b), (c) and (d).
D. Very cheap item
A. Decisive B. Desisive
9. A shrinking violet
C. Descisive D. Desicive
A. A lean person
5. Choose the alternative bearing the correct
spelling from (a), (b), (c) and (d). B. A shy person
A. Aggressive B. Agresive C. A happy person
C. Aggressive D. Aggresive D. A sad person
6. Choose the alternative bearing the correct 10. Gordian knot
spelling from (a), (b), (c) and (d).
A. Undoable job
A. Assassination
B. Asassination B. A difficult problem

C. Asasination C. A different problem

D. Assasination D. Doable job

PAGE 435
www.byjusexamprep.com

11. Fall in a heap 17. A pearl of wisdom


A. To be at the mercy of someone else A. An important piece of news
B. To be thinking about someone B. An important person
C. To lose control of one’s own feelings C. An important thing for life

D. To be in control of one’s own feelings D. An important piece of advice

12. Have a conniption fit Directions: Each of the following passages in this
section has some blank spaces with four words
A. To be very angry or groups of words given. Select whichever word
B. To be very happy or group of words you consider most
appropriate for the blank space and indicate
C. To be very sad your response on the Answer Sheet accordingly
D. To be a jubilant person 18. The founders of the Indian Republic
________ the farsightedness and the
13. Be in seventh heaven
courage to
A. To be extremely happy
A. Had B. Has
B. To be extremely upset
C. Has had D. Were
C. To be extremely adventurous
19. commit ___________ to two major
D. To be extremely silent innovations of historical significance in

14. Hand in glove A. them B. themselves

A. Working separately C. the people D. the course

B. Working together 20. nation-building and social engineering :


first, to ______ a democratic and civil
C. Working for someone
A. build B. building
D. Not willing to work
C. constructing D. built
15. Nip in the bud
21. ________ society among illiterate people
A. Prevent a small problem before it and, second, to undertake economic
becomes severe
A. libertarian B. liberation
B. Prevent the big problems
C. liberating D. liberty
C. Make it severe
22. development _________ democratic
D. Beating the problem political structure. Hitherto, in all
16. Like a shag on a rock A. with a B. within a
A. Completely alone C. for the D. without a

B. Completely idle 23. societies in which an economic take off or


an early industrial and agricultural ______
C. Complete silence had occurred, effective democracy,
D. Complete happy especially from the working

PAGE 436
www.byjusexamprep.com

A. breakthrough B. breakout A. Through B. In spite of


C. breaking D. investment C. Though D. Because
24. people, had been extremely limited. On the 31. German biologist Ernst Haeckel in 1869, a
other hand, _______ the beginning, few conceptual terms ________

A. with B. from A. are B. were

C. within D. for C. have been D. have

25. India was committed to ________ 32. already proposed to reveal relationships
democratic and civil libertarian political _______ organisms and their environment.
order A. among B. those
A. few B. some C. of D. between
C. a D. an 33. For example, French zoologist I.G. Hilaire
used the term ‘ethology’ _______ the
26. and a representative system of government
_________ on free and fair elections to be A. for B. to
conducted on the basis of universal adult C. with D. in
franchise.
34. study of relations of ________ organisms
A. basing B. basis of within the family and society in the
C. based D. function A. the B. a
Direction: Each of the following passages in this C. live D. dead
section has some blank spaces with four words
35. aggregate and in the community. British
or groups of words given. Select whichever word
naturalist St. George Jackson Mivart
or group of words you consider most
proposed the term ‘hexicolog’ with regard
appropriate for the blank space and indicate
to the study of the relations _______ living
your response on the Answer Sheet accordingly
creatures
27. Ecology, in a very simple term, is a science
A. for B. of
that _______ the interdependent,
C. within D. in
A. studies B. study
36. to other organisms and their environment
C. studying D. exploring as regards the nature of the locality they
28. mutually reactive and interconnected frequent, the temperatures and the ______
relationships _______ the organisms and of light which suit them and their relations
to other organisms as enemies, rivals, or
A. among B. between accidental and involuntary benefactors.
C. to D. for A. amount B. focus
29. ________ physical environment on the one C. share D. quality
hand and among the organisms on the
Directions: Each item in this section consists of a
A. their B. its sentence with an underlined word followed by
four words/groups of words. Select the option
C. theirs D. all
that is nearest in meaning to the underlined
30. other hand. ________ the term ‘ecology’ word and mark your response on your Answer
was first coined and used by the Sheet accordingly.

PAGE 437
www.byjusexamprep.com

37. The properties of the family have been 45. The exemptions grated to State institutions
impounded by the order of the court. for acquiring informed consent from
processing personal data in many cases
A. Confiscated B. Permitted
appear to be too blanket.
C. Sold D. Put on hold
A. Obtain B. Lose
38. The officer in charge of the operations has
C. Giving D. Thinking
been impugned for the excesses.
46. The manner in which this exercise has been
A. Expelled
undertaken leaves much to be desired.
B. Rewarded
A. Disliked B. Unlikely
C. Challenged
C. Wish for D. Asked for
D. Given allowance
Directions: In this section, each item consists of
39. Cognitivist and linguists believe that every six sentences of a passage. The first and sixth
child is born with innate qualities. sentences are given in the beginning as S1 and
S6. The middle four sentences in each item have
A. Biological B. Intrinsic
been jumbled up and labelled as P, Q, R and S.
C. Extrinsic D. Unnatural You are required to find the proper sequence of
the four sentences and mark your response
40. It was obligatory for the board to
accordingly on the Answer Sheet.
implement the rule.
47. S1: The master always says, “Refuse to be
A. Compulsory B. Unnecessary
miserable”.
C. By chance D. Problematic
S6: This is the art of right contact in life.
41. They describe the act as a blatant betrayal
P: Before you fall into self-pity and blame
of faith.
games, remember that responsibility
A. Loyal B. Faithfulness comes to only those who feel responsible.
C. Treachery D. Honesty Q: Challenges are faced by the strong and
42. However, if it must decide, then it should courageous, and if life brings you such
do so on the narrowest ground possible. opportunities, then turn failures into
success.
A. Widest B. Slightly
R: Life can be painful, but it need not be
C. Smallest D. Thick sorrowful.
43. This is akin to a contractual relationship S: If you want to be happy, find occasions to
that places obligations on the entities be cheerful.
entrusted with data.
The correct sequence should be
A. Removed B. Narrow
A. RSPQ B. SQPR
C. Similar D. Unparallel
C. QRSP D. RQSP
44. Many communication problems can be
attributed directly to misunderstandings 48. The first and sixth sentences are given in
and inaccuracies. the beginning as S1 and S6. The middle four
sentences in each item have been jumbled
A. Disapproved B. Unofficial up and labelled as P, Q, R and S. You are
C. Ascribed D. Tribute required to find the proper sequence of the

PAGE 438
www.byjusexamprep.com

four sentences and mark your response Q: The gulf between the ‘haves’ and the
accordingly on the Answer Sheet. ‘have nots’ continued to increase and out of
this gap between the rich and poor sprang
S1: Gandhiji reached Newcastle and took
disputes.
charge of the agitation.
R: It generated new wealth but as this new
S6: The treatment that was meted out to
wealth only went to a minority, it could not
these brave men and women in jail
solve the question of distribution
included starvation and shipping, and being
forced to work in the mines by mounted S: The Industrial Revolution solved the
military police. question of production.
P: During the course of the march, Gandhiji The correct sequence should be
was arrested twice, released, arrested a
A. PQRS B. SRQP
third time and sent to jail.
C. SRPQ D. RQSP
Q: The employers retaliated by cutting off
water and electricity to the workers’ 50. The first and sixth sentences are given in
quarters, thus forcing them to leave their the beginning as S1 and S6. The middle four
homes. sentences in each item have been jumbled
up and labelled as P, Q, R and S. You are
R: Gandhiji decided to march this army of
required to find the proper sequence of the
over two thousand men, women and
four sentences and mark your response
children over the border and thus see them
accordingly on the Answer Sheet.
lodged in Transvaal jails.
S1: Institutions define and play a regulatory
S: The morale of the workers, however, was
role with regard to human behaviour.
very high and they continued to march till
they were prosecuted and sent to jail. S6: It shows how important it is for a nation
to build institutions for nurturing
The correct sequence should be
democracy.
A. QRPS B. SRQP
P: Once established, institutions set a
C. QPSR D. RQSP dynamic relationship with the members
constituting them and they mutually affect
49. The first and sixth sentences are given in
each other.
the beginning as S1 and S6. The middle four
sentences in each item have been jumbled Q: They shape preferences, power and
up and labelled as P, Q, R and S. You are privilege.
required to find the proper sequence of the
R: At the same time, institutions
four sentences and mark your response
themselves can be transformed by the
accordingly on the Answer Sheet.
politics they produce and such
S1: One of the most important forces in the transformation can affect social norms and
modern world, socialism was a direct result behaviours.
of the Industrial Revolution.
S: They also provide a sense of order and
S6: This is how socialism as a theory and predictability.
practice came into being.
The correct sequence should be
P: Socialism was a direct challenge to
A. RPQS B. QRSP
capitalism and sought to put an end to such
an exploitative economic structure. C. PSRQ D. QSRP

PAGE 439
www.byjusexamprep.com

51. The first and sixth sentences are given in P: There is a range of strategies by which
the beginning as S1 and S6. The middle four the food is taken in and used by the
sentences in each item have been jumbled organism.
up and labelled as P, Q, R and S. You are
Q: For example, whether the food source is
required to find the proper sequence of the
stationary (such as grass) or mobile (such as
four sentences and mark your response
deer), would allow for differences in how
accordingly on the Answer Sheet.
the food is accessed and what is nutritive
S1: Idioms are a colourful and fascinating apparatus used by a cow or a lion.
aspect of language.
R: The form of nutrition differs depending
S6: Idioms may also suggest a particular on the type and availability of food material
attitude of the person using them, for as well as how it is obtained by an organism.
example, disapproval, humour, S: Some organisms break down the food
exasperation or admiration, so you must material outside the body and then absorb
use them carefully. it and others take in the whole material and
P: Your language skills will increase rapidly break it down inside their bodies.
if you can understand idioms and use them The correct sequence should be
confidently and correctly.
A. RQPS B. QPSR
Q: They are commonly used in all types of
language, informal and formal, spoken and C. SQPR D. QPRS
written. 53. The first and sixth sentences are given in
R: In addition, idioms often have a stronger the beginning as S1 and S6. The middle four
meaning than non-idiomatic phrases. sentences in each item have been jumbled
up and labelled as P, Q, R and S. You are
S: One of the main problems students have required to find the proper sequence of the
with idioms is that it is often impossible to four sentences and mark your response
guess the meaning of an idioms from the accordingly on the Answer Sheet.
words in contains.
S1: “When I was alive and had a human
The correct sequence should be heart,” answered the statue, “I did not
know what tears were, for I lived in the
A. RQPS B. RSPQ
Palace of Sans-souci where sorrow is not
C. SRQP D. QPSR allowed to enter.

52. The first and sixth sentences are given in S6: And now that I am dead they have set
the beginning as S1 and S6. The middle four me up here so high that I can see all the
sentences in each item have been jumbled ugliness and all the misery of my city, and
up and labelled as P, Q, R and S. You are though my heart is made of lead yet I
required to find the proper sequence of the cannot choose but weep.”
four sentences and mark your response P: So I lived, and so I died.
accordingly on the Answer Sheet.
Q: Round the garden ran a very lofty wall,
S1: Each organism is adapted to its but I never cared to ask what lay beyond it,
environment. everything about me was so beautiful.
S6: What can be taken in and broken down R: My courtiers called me the Happy Prince,
depends on the body design and the happy indeed I was, if pleasure be
functioning. happiness.

PAGE 440
www.byjusexamprep.com

S: In the daytime I played with my four sentences and mark your response
companions in the garden, and in the accordingly on the Answer Sheet.
evening I led the dance in the Great Hall.
S1: I had spent many nights in the jungle
The correct sequence should be looking for game, but this was the first time
I had every spent a night looking for a man-
A. QSRP B. PQRS
eater.
C. PRQS D. RPQS S6: It was in this position my men an hour
54. The first and sixth sentences are given in later found me fast asleep; of the tiger I had
the beginning as S1 and S6. The middle four neither heard nor seen anything.
sentences in each item have been jumbled P: I bitterly regretted the impulse that had
up and labelled as P, Q, R and S. You are induced me to place myself at the man-
required to find the proper sequence of the eater’s mercy.
four sentences and mark your response
accordingly on the Answer Sheet. Q: The length of road immediately in front
of me was brilliantly lit by the moon, but to
S1: One day her mother, having made some right and left the overhanging trees cast
cakes, said to her, “Go, my dear, and see dark shadows, and when the night wind
how your grandmother is doing, for I hear agitated the branches and the shadows
she has been very ill. Take her a cake, and moved, I saw a dozen tigers advancing on
this little pot of butter.” me.
S6: “Does she live far off?” said the wolf. R: As the grey dawn was lighting up the
snowy range which I was facing, I rested my
P: He asked her where she was going.
head on my drawn-up knees.
Q: The poor child, who did not know that it
S: I lacked the courage to return to the
was dangerous to stay and talk to a wolf,
village and admit I was too frightened to
said to him, “I am going to see my
carry out my self-imposed task, and with
grandmother and carry her a cake and a
teeth chattering, as much from fear as from
little pot of butter from my mother.”
cold, I sat out the long night.
R: As she was going through the wood, she
The correct sequence should be
met with a wolf, who had a very great mind
to eat her up, but he dared not, because of A. QPSR B. SRPQ
some woodcutters working nearby in the C. PRSQ D. RPQS
forest.
Directions: Given below are a few sentences.
S: She set out immediately to go to her Identify the part of speech of the underlined
grandmother, who lived in another village. words. Choose the response (a), (b), (c) or (d)
The correct sequence should be which is the most appropriate expression.

A. PRQS B. SRPQ 56. Rita eats her dinner quickly.

C. PRSQ D. RPQS A. Verb B. Preposition


C. Adjective D. Adverb
55. The first and sixth sentences are given in
the beginning as S1 and S6. The middle four 57. He thought the movie ended abruptly.
sentences in each item have been jumbled
A. Noun B. Adverb
up and labelled as P, Q, R and S. You are
required to find the proper sequence of the C. Verb D. Adjective

PAGE 441
www.byjusexamprep.com

58. I will meet you in the third week of August. 66. Beauty lies in the eyes of the beholder.
A. Pronoun B. Verb A. Allure B. Charm
C. Preposition D. Noun C. Inelegance D. Ideal
59. Jasmines and roses are my favourite 67. Reading details about suicide cases can
flowers. push vulnerable people taking the extreme
A. Verb B. Preposition step.

C. Conjunction D. Interjection A. Imperious B. Impervious

60. She truthfully answered the detective’s C. Helpless D. Defenseless


questions.
68. Standing before a judge in a courtroom can
A. Verb B. Adjective be daunting for anyone.
C. Noun D. Adverb A. Uncomfortable B. Encouraging
61. Hurrah ! We won the game! C. Demoralizing D. Off-putting
A. Interjection B. Conjunction 69. He has been facing a kind of intimidation by
C. Noun D. Pronoun his friends for last two years.

62. The son writes meaningless letters to his A. Wiles B. Conviction


father. C. Persuasion D. Support
A. Adverb B. Verb
70. There are many factors that constrain the
C. Pronoun D. Adjective philosophy of job enrichment in practice.
63. The secretary himself visited the affected A. Oblige B. Pressure
families.
C. Restrict D. Support
A. Verb B. Noun
71. People look for plausible remedies to the
C. Adverb D. Pronoun problems which they do not know.
64. The children were walking through the A. Acceptable B. Unthinkable
forest.
C. Solvable D. Believable
A. Verb B. Adverb
72. The departing speech of the
C. Adjective D. Preposition Chairperson ended with a plaintive note.
65. The Presiding Officer walked slowly to the
A. Melancholic B. Gleeful
dais.
C. Doleful D. Adventurous
A. Adverb B. Adjective
C. Verb D. Noun 73. The members have taken a unanimous
decision to discord some of the rulings of
Directions: Each item in this section consists of a the Managing Committee on problems
sentence with an underlined word followed by relating to maintenance.
four words/groups of words. Select the option
that is opposite in meaning to the underlined A. Accord B. Dissension
word and mark your response on your Answer C. Dispute D. Friction
Sheet accordingly.

PAGE 442
www.byjusexamprep.com

74. The insolent nature of the speaker had A. RQPS B. SPRQ


provoked the members of the house and
C. QPSR D. PSQR
this led to pandemonium.
79. P: for guest teachers
A. Respectful
B. Autocratic Q: in the department of Biotechnology

C. Impudent R: was also held

D. Thought provoking S: a Selection Committee meeting

75. Incessant rains have resulted in failure of A. SPRQ B. QRSP


crops during this season. C. PRQS D. RSPQ
A. Sporadic B. Persistent 80. P: for contractual assignment at Cultural
C. Continual D. Ceaseless Centres abroad

Directions: Each of the following items in this Q: as Teacher of Indian Culture for two
section consists of a sentence, the parts of which years
have been jumbled. These parts have been R: applications are invited in a prescribed
labelled as P, Q, R and S. Given below each format
sentence are four sequences namely (a), (b), (c)
and (d). You are required to rearrange the S: from Indian Nationals for deployment
jumbled parts of the sentence and mark your A. QPRS B. SRPQ
response on the Answer Sheet accordingly.
C. PQRS D. RSQP
76. P: the company are often asked
81. P: while they are small
Q: the formal or informal interviews
Q: and do the great things
R: employees who are leaving
R: while they are easy
S: for their opinions during
S: do the difficult things
A. RPSQ B. RQPS
C. PSQR D. PQSR A. SRQP B. PSQR

77. P: a hailstorm activity in the evenings C. SRPQ D. QPSR

Q: there is a possibility of 82. P: then you sure

R: while there could be Q: if you can’t

S: heavy rain towards the weekend R: don’t deserve me at my best

A. SQPR B. QSRP S: handle me at my worst

C. QRPS D. SPRQ A. PRQS B. QSPR

78. P: has been below normal since last week C. RQSP D. PSRQ
Q: the minimum temperature 83. P: you will be more disappointed
R: in some parts of the city Q: than by the ones you did do
S: when rain and hailstorm activity was R: by the things you didn’t do
recorded
S: twenty years from now

PAGE 443
www.byjusexamprep.com

A. PRSQ B. PRQS Directions: Each item in this section has a


sentence which has multiple parts. Find out the
C. PQSR D. SPRQ
error/no error and indicate your response from
84. P: man is one who can lay the options (a), (b), (c) and (d) on the Answer
Sheet.
Q: a firm foundation with the bricks
89. Experience has shown that the change-over
R: a successful
from a closed economy to a mercantile
S: others have thrown at him economy has presented in human society
innumerable problems.
A. PQSR B. RQSP
A. Experience has shown that
C. RPQS D. QSPR
B. The change-over from a closed
85. P: what we may be economy
Q: but we know not C. To a mercantile economy has presented
R: we know D. In human society innumerable
S: what we are problems

A. RSQP B. QPRS 90. A closed economy is identified as a human


community which produces all it consumes
C. QRPS D. RQPS and consumed all it produces.
86. P: for the ordinary A. A closed economy is identified
Q: not willing to risk the unusual B. As a human community
R: if you are C. Which produces all it consumes
S: you will have to settle D. And consumed all it produces
A. PRQS B. SPQR 91. Iron is the most useful against all metals.
C. RQSP D. QSRP A. Iron is
87. P: as mere stepping stones B. The most useful
Q: his major achievements C. Against all metals
R: for the next advance D. No error
S: he regarded 92. Mumbai is largest cotton centre in the
A. SPQR B. SQPR country.

C. SPRQ D. RPQS A. Mumbai is

88. P: have a great influence B. Largest cotton centre

Q: and they often shape our personality C. In the country

R: on our adult lives D. No error

S: events in our childhood 93. While every care have been taken in
preparing the results, the company
A. SPRQ B. SQRP reserves the right to correct any
C. SRQP D. PQRS inadvertent errors at a later stage.

PAGE 444
www.byjusexamprep.com

A. While every care have been taken 99. You look as if you have ran all the way
home.
B. In preparing the results,
C. The company reserves the right to A. You look as if
correct B. you have ran
D. Any inadvertent errors at a later stage C. all the way home
94. My sister and me are planning a trip from
D. No error
Jaipur to Delhi.
A. My sister and me are
B. Planning a trip 100. The real voyage of discovery consist not in
seeking new landscapes, but in having new
C. From Jaipur to Delhi eyes.
D. No error
A. The real voyage of discovery
95. Despite the thrill of winning the lottery last
B. consist not in seeking new landscapes,
week, my neighbour still seems happy.
A. Despite the thrill of winning C. but in having new eyes

B. The lottery last week, D. No error


C. My neighbor 101. No struggle can ever succeeded without
women participating side by side with men.
D. Still seems happy
96. Children are not allowed to use the A. No struggle can ever succeeded
swimming pool unless they are with an B. without women participating
adult.
C. side by side with men
A. Children are not allowed
D. No error
B. To use the swimming pool
C. Unless they are with an adult 102. Education is the passport to the future, for
tomorrow belong to those who prepare for
D. No error it today.
97. Her knowledge of Indian languages are far A. Education is the passport to the future,
beyond the common.
B. for tomorrow belong to those
A. Her knowledge
C. who prepare for it today
B. of Indian languages
C. are far beyond the common D. No error

D. No error 103. There come a time when you have to


choose between turning the page and
98. The care, as well as the love of a father, closing the book.
were missing in her life.
A. There come a time
A. The care, as well as the love
B. of a father, B. when you have to choose

C. were missing in her life C. between turning the page

D. No error D. and closing the book

PAGE 445
www.byjusexamprep.com

Directions: In this section, you have few short of these developments, ‘the handicrafts stage’, a
passages. After each passage, you will find some number of social and economic changes took
items based on the passage. First, read a passage place which marked the beginning of the labour
and answer the items based on it. You are problem in the world. The self-sufficient
required to select your answers based on the economy of the village underwent a drastic
contents of the passage and opinion of the change. The community of traders and
author only. merchants emerged.
Passage-I 104. Humanity’s evolution from primitive stage
to the present has been
Mankind’s experience of various evolutionary
changes from primitive times to the present day A. static and smooth
has been extensive and varied. However, man’s
problems were never before as complicated as B. huge and diversified
they seem to be today. Man’s economic activity C. always violent
centres primarily around production. Labour is
said to be the primary factor of production; its D. always peaceful
role, therefore, has been given a lot of 105. ---“man’s problems were never before as
importance. It should be useful to have an complicated as they seem to be today”
overall view of the economic history of man- means
from the nomadic times to the modern factory
system-and study its relevance to the various A. The present times are the best times of
labour problems of today. humanity

Initially, man passed through ‘the hunting and B. The present times are the crucial period
fishing stage’. During this period, his basic needs for humanity
were adequately met by Nature. Wild animals, C. The present times pose much more
birds and fruits satisfied his hunger, and his thirst challenges to humans than the previous
was quenched by the waters of springs and times
rivers. Caves gave him shelter and barks of trees
were used as clothing. During this stage of man’s D. The present times provide much more
progress, labour problems did not exist because facilities than the previous times
of the absence of any economic, political and 106. Why does the author say that labour
social systems. problems did not exist during ‘the hunting
Then came ‘the pastoral stage’, which was and fishing stage’?
marked by a certain amount of economic A. There was no nation existing at that
activity. The nomadic and migratory nature of time
man persisted, and, together with his goats and
cattle, he moved on to fresh pastures and B. There were no economic, political and
meadows. Some conflicts would sometimes take social systems
place among herd-owners, for, during this
C. There was no capitalism and market
period, the institution of nominal private
property ownership was not known. D. There was no labour law
This stage paves the way for ‘the agricultural 107. “The pastoral stage was marked by a
stage’, during which the class system began to certain amount of economic activity.”
develop. There was a small artisan class mostly How?
self-employed; and there were also landed
A. Humans started migrating and held
proprietors or Zamindars as well as slaves. Thus,
goat-herds
arose the feudal system. During the fourth stage

PAGE 446
www.byjusexamprep.com

B. Humans started owning land their possession. In some cases, Pattas were
issued to the beneficiaries, but possession of
C. Conflicts started as humans owned
land shown in the Pattas was not given, or
goats
corresponding changes were not made in the
D. Humans started doing agriculture records of right.
108. Which word in the passage means The balance of power in rural India is so heavily
‘surfaced’? weighed against the landless and the poor that
A. Quenched B. Emerged implementing land ceiling laws is difficult. It is
clear that without massive mobilization of the
C. Nomadic D. Adequately rural poor and depending on democratic
Directions: In this section, you have few short governance in rural India, very little can be
passages. After each passage, you will find some achieved in this direction.
items based on the passage. First, read a passage Although half of India’s population continues to
and answer the items based on it. You are depend on agriculture as its primary source of
required to select your answers based on the livelihood, 83 percent of farmers operate
contents of the passage and opinion of the holdings of less than 2 hectares in size, and the
author only. average holding size is only 1.23 hectares. This is
Passage – II often in fragments and unirrigated. There are
also those who are entirely landless, although
Ever since independence, land reforms have agriculture is their main source of livelihood.
been a major instrument of State policy to They have inadequate financial resources to
promote both equity and agricultural purchase and often depend on leasing in small
investment. Unfortunately, progress on land plots, on insecure terms, for short periods,
reforms has been slow, reflecting the resilience sometimes only for one season. Hence, may face
of structures of power that gave rise to the insecurity of tenure and the growing threat of
problem in the first place. land alienation and pressure from urbanization,
The main instrument for realizing more industrialization and powerful interest.
equitable distribution of land is the land ceiling 109. Why does the land reform prove to be
laws. These laws were enacted by several States slow?
during the late 1950s and 1960s, and the early
1970s saw more stringent amendments in the A. Because of the disparity in power
laws to plug loopholes in the earlier laws. But the structure
record of implementation has not been B. Because of the power of the
satisfactory. Around 3 million hectares of land government
has been declared surplus so far, which is hardly
2 percent of net sown area in India. About 30 C. Because States have different laws
percent of this land has not yet been distributed D. Because of the scarcity of land in the
as it is caught up in the litigations. Besides, a country
number of Benami and clandestine transactions
have resulted in illegal possession of significant 110. Which of the following statements is/are
amounts of land above ceiling limits. There are correct?
widespread reports of allotment of inferior, 1. Land ceiling laws have proved to be
unproductive, barren and wasteland to landless unsatisfactory.
household, many of whom have been forced to
sell it off, in the absence of resources to make it 2. The democratic structure of the
productive. In many instances, lands allotted to government cannot provide solution to
the rural poor under the ceiling laws are not in the problem of land reforms.

PAGE 447
www.byjusexamprep.com

3. The owners of land have abundant investment. Unfortunately, progress on land


natural resources. reforms has been slow, reflecting the resilience
of structures of power that gave rise to the
4. Identified land for distribution has not
problem in the first place.
been distributed due to court cases
against it. The main instrument for realizing more
equitable distribution of land is the land ceiling
Select the correct answer using the code
laws. These laws were enacted by several States
given below.
during the late 1950s and 1960s, and the early
A. 1 and 4 B. 1 only 1970s saw more stringent amendments in the
laws to plug loopholes in the earlier laws. But the
C. 3 and 4 D. 2 and 4
record of implementation has not been
111. One of the reasons of selling off the lands satisfactory. Around 3 million hectares of land
by the allottees is that the lands were has been declared surplus so far, which is hardly
2 percent of net sown area in India. About 30
A. unproductive and barren percent of this land has not yet been distributed
B. salty, not getting water as it is caught up in the litigations. Besides, a
number of Benami and clandestine transactions
C. fertile, but uncultivable have resulted in illegal possession of significant
D. with the powerful people amounts of land above ceiling limits. There are
widespread reports of allotment of inferior,
112. Which word/group of words in the passage unproductive, barren and wasteland to landless
means ‘lawsuit’? household, many of whom have been forced to
A. Amendments sell it off, in the absence of resources to make it
productive. In many instances, lands allotted to
B. Litigations the rural poor under the ceiling laws are not in
C. Illegal possession their possession. In some cases, Pattas were
issued to the beneficiaries, but possession of
D. Fragments land shown in the Pattas was not given, or
113. According to the author, what is the corresponding changes were not made in the
primary source of livelihood of majority of records of right.
India’s population? The balance of power in rural India is so heavily
A. Industry B. Forest weighed against the landless and the poor that
implementing land ceiling laws is difficult. It is
C. Agriculture D. None clear that without massive mobilization of the
Directions: In this section, you have few short rural poor and depending on democratic
passages. After each passage, you will find some governance in rural India, very little can be
items based on the passage. First, read a passage achieved in this direction.
and answer the items based on it. You are Although half of India’s population continues to
required to select your answers based on the depend on agriculture as its primary source of
contents of the passage and opinion of the livelihood, 83 percent of farmers operate
author only. holdings of less than 2 hectares in size, and the
Passage – II average holding size is only 1.23 hectares. This is
often in fragments and unirrigated. There are
Ever since independence, land reforms have also those who are entirely landless, although
been a major instrument of State policy to agriculture is their main source of livelihood.
promote both equity and agricultural They have inadequate financial resources to

PAGE 448
www.byjusexamprep.com

purchase and often depend on leasing in small rather than written or typed on an identifiable
plots, on insecure terms, for short periods, machine.
sometimes only for one season. Hence, may face
Survey software packages are available that
insecurity of tenure and the growing threat of
generate questions for a number of standard
land alienation and pressure from urbanization,
topics and can be customized by modifying
industrialization and powerful interest.
existing questions or by adding questions. If the
114. “There are also those who are entirely survey is computerized, reports can be
landless, although agriculture is their main generated with ease to provide snapshots of a
source of livelihood” means given period of time, trend analysis, and
breakdowns according to various demographics.
A. they do not have money to buy lands
You may be interested in responses by age, sex,
B. they have sold off their lands to others job categories, departments, division, functions
or geography.
C. most of them are agriculture labourers
The survey can be conducted by placing
D. they are migrant labourers from other
microcomputers in several locations convenient
places
for employees’ use. Employees are advised
Directions: In this section, you have few short where the computers will be, for how long, and
passages. After each passage, you will find some when he data will be collected (for instance, daily
items based on the passage. First, read a passage at 5:00 p.m. for three weeks). The screens should
and answer the items based on it. You are not be viewable to supervisors or passers-by.
required to select your answers based on the While there may be some risk that employees
contents of the passage and opinion of the will take the survey more than once, there are
author only. comparable risks with other methods too.
Passage – III Managers may be interested in knowing how
Despite downsizings, workers’ overall job they are perceived by their peers and
satisfaction actually improved between 1988 subordinates. Packages are available that can be
and 1994. Some reasons given were improved customized, which allow the manager to
work flow, better cooperation between complete a self-assessment tool used to
departments, and increases fairness in compare self-perceptions to the anonymous
supervision. Many firms today rely on attitude opinions of others. This comparison may assist in
surveys to monitor how employees feel about the development of a more effective manager.
working in their firms. 115. Which one of the following is not the
reason for improved job satisfaction of
The use of employee attitude surveys had grown
employees?
since 944 when the National Industrial
Conference Board “had difficulty finding fifty A. Improved work flow
companies that had conducted opinion surveys”. B. Better cooperation between
Today, most companies are aware of the need departments
for employees’ anonymity, the impact of both
the design of the questions and their sequence, C. Supervisors’ fairness
the importance of effective communication, D. Increased remuneration
including knowing the purpose of the survey
before it is taken and getting feedback to the 116. Companies feel that it is necessary to
employees after it is completed. A. maintain anonymity of the employees
Computerization of surveys can provide and to have effective design and
anonymity, if there is no audit trail to the user, sequence of questions and effective
especially for short answers that are entered communication

PAGE 449
www.byjusexamprep.com

B. maintain the fairness of the managers A. Trend B. Breakdowns


to be part of the survey
C. Convenient D. Perceptions
C. conduct surveys from their employees
119. “The screens should not be viewable to
D. maintain anonymity of the employees supervisors or passers-by.” Why?
and not to have effective design and
A. To maintain the secrecy of a person
sequence of questions and effective
communication B. The main problem is to enable
everyone to participate
117. One major benefit of using survey software
packages is C. The manager has to be fair enough
A. reports can be generated easily D. To maintain the problems faced by
women in job market
B. privacy of a person is exposed to the
supervisors 120. What does the word ‘customized’ mean
here?
C. employees would like to take up the
test on computer A. Adapted
D. employer can get to know the B. Take as it is
information immediately
C. Fixed
118. Which word in the passage means
D. Mass produced
‘tendency’?

PAGE 450
www.byjusexamprep.com

General Knowledge
1. Which one of the following is the motto of A. Colombo B. Islamabad
NCC?
C. Kathmandu D. Dhaka
A. Unity and Discipline
6. Which one of the following countries is not
B. Unity and Integrity a founding member of the New
Development Bank?
C. Unity and Command
A. Brazil B. Canada
D. Unity and Service
C. Russia D. India
2. Which one of the following departments is
not under the Ministry of Home Affairs? 7. The Public Financial Management System
(PFMS) is a web-based online software
A. Department of Official Languages
application designed, developed, owned
B. Department of Border Management and implemented by the
C. Department of Jammu and Kashmir A. Department of Financial Services
Affairs
B. Institute of Government Accounts and
D. Department of Legal Affairs Finance
3. Which of the following statements is/are C. Controller General of Accounts
correct?
D. National Institute of Financial
1. India is a signatory to the United Management
Nations Convention to Combat
8. Match List I with List II and select the
Desertification (UNCCD).
correct answer using the code given below
2. Ministry of Home Affairs is the nodal the lists:
Ministry in the Government of India for
the UNCCD List I List II
(Institute) (Location)
Select the correct answer using the code
given below: A. National Institute 1. Chennai
of Ayurveda
A. 1 only B. 2 only
B. National Institute 2. Bengaluru
C. Both 1 and 2 D. Neither 1 nor 2 of Homoeopathy
4. Under which one of the following Articles of C. National Institute 3. Kolkata
the Constitution of India, a statement of of Unani Medicine
estimated receipts and expenditure of the
Government of India has to be laid before D. National Institute 4. Jaipur
the Parliament in respect of every financial
year? Code :
A. Article 110 B. Article 111 A. a-1 b-2 c-3 d-4
C. Article 112 D. Article 113 B. a- 1 b-3 c-2 d-4
5. The South Asian Association for Regional C. a- 4 b-3 c-2 d-1
Cooperation was founded in
D. a-4 b-2 c-3 d-1

PAGE 451
www.byjusexamprep.com

9. Which of the following statements about C. Naomi Osaka


‘Invest India’ is/are correct?
D. Serena Williams
1. It is a joint venture (not for profit) 14. Which among the following IN ship(s)
company. participated in the SIMBEX-19?
2. It is the National Investment Promotion 1. INS Kolkata 2. INS Shakti
and Facilitation Agency of India .
3. INS Vikrant
Select the correct answer using the code
given below: Select the correct answer using the code
given below:
A. 1 only B. 2 only
A. 1, 2 and 3 B. 1 and 2 only
C. Both 1 and 2 D. Neither 1 nor 2
C. 2 and 3 only D. 1 only
10. The National Dope Testing Laboratory
functions under 15. “Triples” is a new format of

A. Ministry of Health and Family Welfare A. Boxing B. Judo

B. Ministry of Science and Technology C. Chess D. Badminton


16. Who among the following was the
C. Ministry of Youth Affairs and Sports
Chairman of the Committee on Deepening
D. Ministry of Home Affairs Digital Payments appointed by the RBI?
11. In how many phases was the general A. H.R. Khan
election, 2019 conducted in India?
B. Nandan Nilekani
A. 6 phases B. 7 phases
C. N.R. Narayana Murthy
C. 8 phases D. 9 phases
D. Sanjay Jain
12. Which one of the following statements 17. ‘The Sasakawa Award’ of United Nations is
about the Organization of Islamic given in recognition of the work done in the
Cooperation is not correct? field of
A. Its permanent Secretarist is located at A. Disaster Reduction
Jeddah.
B. Peace Keeping
B. It endeavours to safeguard and protect
interests of the Muslim world in the C. Health Services
spirit of promoting international peace D. Poverty Alleviation
and harmony among various people of
the world . 18. Why was India’s G.S. Lakshmi in news
recently?
C. It is the largest inter-governmental
organization of the world A. She was the first Indian to play cricket
for an English County Club.
D. It has consulative and cooperative
B. She became the first female ICC match
relations with the UN.
referee.
13. Who among the following won the Italian
C. She was awarded the Ramon
Open Women’s Tennis Singles Title, 2019?
Maggsayssy Award for the year 2019
A. Karolins Pliskova
D. She was the recipient of the Booker
B. Johanna Konta Prize in the year 2019

PAGE 452
www.byjusexamprep.com

19. Who among the following was elected as A. Article 53 B. Article 74


the President of Indonesia for the second
C. Article 361 D. Article 363
term?
25. Which law prescribes that all proceedings in
A. Joko Widodo
the Supreme Court shall be in English
B. Prabowo Subianto language?
C. Sandiaga Uno A. Article 145 of the Constitution of India
D. Jusuf Kalla B. Article 348 of the Constitution of India
20. In India, May 21 is observed as C. The Supreme Court Rules, 1966
A. NRI Day D. An Act passed by the Parliament
B. National Youth Day 26. The total number of members in the Union
Council of Ministers in India shall not
C. National Technology Day exceed
D. National Anti Terrorism Day A. 10% of the total number of members of
21. Arrange the following in the chronological the Parliament.
order of their implementation: B. 15% of the total number of members of
1. The Indian Factory Act (First) the Parliament.

2. The Vernacular Press Act C. 10% of the total number of members of


the Lok Sabha.
3. The Morley Minto Reforms
D. 15% of the total number of members of
4. The Cornwallis Code the Lok Sabha.
Select the correct answer using the code 27. Which one of the following is the most
given below: noticeable characteristic of the
A. 4, 2, 1,3 B. 2, 4, 1, 3 Mediterranean climate?

C. 3, 4, 1, 2 D. 2, 1, 3, 4 A. Limited geographical extent

22. Article 371 A of the constitution of India B. Dry summer


provides special privileges to C. Dry winter
A. Nagaland B. Misoram D. Moderate temperature
C. Sikkim D. Manipur 28. Which one of the following rivers takes a
23. How many Zonal Councils were set up vide “U” turn at Namcha Barwa and enters
Part-III of the states Re-organisation Act, India?
1956? A. Ganga B. Tisrta
A. Eight B. Seven C. Barak D. Brahamaputra
C. Six D. Five 29. What was the Dutt-Bradley thesis?
24. Which provision of the Constitution of India A. The Working Committee of the Indian
Provides that the President shall not be National Congress decided that
answerable to any Court in India for the Congress should play a crucial role in
exercise of powers of his office? realising the independent of India.

PAGE 453
www.byjusexamprep.com

B. The Socialist party decided to play 34. In December 1962, which Soviet leader
foremost part in anti-imperialist declared that China was responsible for the
struggle. Sino-Indian War of 1962?
C. Revloutionary socialist A. Khrushchev
BatukeshwarDutt put forth a ten-point
B. Bulganin
plan to work for the success of anti-
imperialist front. C. Suslov
D. It was a Communist party document, D. Malenkov
according to which the National
35. Which of the following statements with
Congress could play a great part and a
regard to the ‘Make in India’ initiative is/are
foremost part in realising the anti-
correct?
imperialist people’s front.
1. It was launched in the year 2018
30. The khuntkatti tenure was prevalent in
which one of the following regions of India 2. Its objects to foster innovation.
during the British Colonial Rule? Select the correct answer using the code
A. Bundelkhand given below:

B. Karnataka A. 1 only

C. Chota Nagpur B. 2 only


C. Both 1 and 2
D. Madras Presidency
D. Neither 1 nor 2
31. Who was the author of the book ‘Plogues
and Peoples’? 36. Which one of the following States does not
have a legislative Council?
A. William H. McNeill
A. Karnataka
B. W.I. Thomas
B. Telangana
C. Rachel Carson
C. Jammu and Kashmir
D. David Cannadine
D. Arunachal Pradesh
32. Who among the following started the
Indian Agriculture Service? 37. What is SWAYAM?
A. Study Webs of Active-Learning for
A. Lord Curson
Young Aspiring Minds
B. William Bentinck
B. Study Webs of Active-Learning for
C. Lord Minto Youth Aspiring Minds.

D. Lord Rippon C. Study Webs of Active-Learning for


Young Aspiration Minds
33. ‘Chandimangala’ was composed in which
one of the following languages during the D. Study Webs of Active-Learning for
16th century CE? Youth Aspiration Minds

A. Sanskrit B. Tamil 38. Which one of the following is not


enumerated in the Constitution of India as
C. Bengali D. Oriya a fundamental duty of citizens of India?

PAGE 454
www.byjusexamprep.com

A. To safeguard public property 42. The working of the price mechanism in a


free-market economy refers to which one
B. To protect and improve the natural
of the following?
environment
A. The interplay of the forces of demand
C. To develop the scientific temper and
and supply
spirit of inquiry
B. Determination of the inflation rate in
D. To promote international peace and
the economy.
security
C. Determination of the economy’s
39. Who among the following in his book. ‘The
propensity to consume.
Manageral Revolution’ argued that a
managerial class dominated all industrial D. Determination of the economy’s full
societies, both capitalist and communist, by employment output.
virtue of its technical and scientific 43. Indexation is a method whose use can be
knowledge and its administrative skills? associated with which one of the following?
A. James Burnham A. Controlling inflation
B. Robert Michels B. Nominal GDP estimation
C. Gaetano Mosca C. Measurement of savings rate
D. Vilfredo Pareto D. Fixing of wage compensation
40. Which one of the following conditions laid 44. A car undergoes a uniform circular motion.
down in the Constitution of India for the The acceleration of the car is
issue of a writ of Quo-Warranto is not
correct? A. zero

A. The office must be public and it must be B. a non-zero constant


created by a Statute C. a non-zero but not a constant
B. The office most be a substantive one D. None of the above
C. There has been a contravention of the 45. An echo is heard after 5 seconds of the
Constitution or a Statute in sppointing production of sound which moves with a
such person to that office speed of 340 m/s. What is the distance of
D. The appointment is in tone with a the mountain from the sources of sound
statutory provision. which produced the echo?

41. Which one of the following equals Personal A. 0.085 km B. 0.85 km


Disposable Income? C. 0.17 km D. 1.7 km
A. Personal Income – Direct taxes paid by 46. A 100 W electric bulb is used for 10 hours a
households and miscellaneous fees, day. How many units of energy are
fines, etc. consumed in 30 days?
B. Private Income – Saving of Private A. 1 B. 10
Corporate Sectors – Corporation Tax
C. 30 D. 300
C. Private Income – Taxes
47. Which of the following statements relating
D. Total expenditure of Households – to the Fifth Schedule of the Constitution of
Income Tax – Gifts received India is not correct?

PAGE 455
www.byjusexamprep.com

A. It relates to the special provision for Which of the statements given above is/are
administration of certain areas in the correct?
States other than Assam, Meghalaya,
A. 1 only B. 2 only
Tripura and Mizoram.
C. Both 1 and 2 D. Neither 1 nor 2
B. Tribal advisory councils are to be
constituted to give advice under the 50. Which one of the following forms of
Fifth Schedule. Constitution contains the features of both
the Unitary and Federal Constitution?
C. The Governor is not authorized to make
regulations to prohibit or restrict the A. Unitary B. Federal
transfer of land by, or among members
of the Scheduled Tribes. C. Quasi-Federal D. Quasi Unitary

D. The Governors of the States in which 51. Which one of the following Indian States
there are scheduled areas have to gas not international boundary
submit reports to the President A. Bihar B. Chhattisgarh
regarding the administration of such
areas. C. Uttarakhand D. Meghalaya

48. Consider the following statements with 52. Which one of the following Indian cities is
regard to the formation of new States and not located on a river bank?
alteration of boundaries of existing States : A. Agra B. Bhagalpur
1. Parliament may increase the area of C. Bhopal D. Kanpur
any State.
53. Where are Jhumri Telaiya and Mandar Hills
2. Parliament may diminish the area of situated?
any State.
A. Jharkhand B. Bihar
3. Parliament cannot alter the boundary
of any State. C. Assam D. West Bengal

4. Parliament cannot alter the name of 54. Which one of the following is not correct
any State. regarding South India?

Which of the statements given above is/are A. Diurnal range of temperature is less
not correct?
B. Annual range of temperature is less
A. 1 and 2 B. 2 and 3
C. Temperature is high throughout the
C. 3 and 4 D. 4 only year

49. Consider the following statements : D. Extreme climatic conditions are found

1. The Advocate General of a State in 55. Which one of the following statements
India is appointed by the President of India regarding sex composition is not correct?
upon the recommendations of the
A. In some countries, sex ratio is
Governor of the concerned State.
expressed as number of males per
2. As provided in the Code of Civil thousand females.
Procedure, High Courts have original
B. In India, sex ratio is expressed as
appellate advisory jurisdiction at the
number of females per thousand males.
State level.

PAGE 456
www.byjusexamprep.com

C. At world level, sex ratio is about 102 Codes:


males per 100 females.
A. A-3 B-1 C-4 D-2
D. In Asia, there is high sex ratio.
B. A-2 B-1 C-4 D-3
56. Who among the following has given the
C. A-3 B-4 C-1 D-2
concept of Human Development?
D. A-2 B-4 C-1 D-3
A. Amartya Sen
60. Who among the following Mughal
B. Mahbub-ul-Haq
emperors was a following of the
C. Sukhamoy Chakravarty Naqshbandiyya leader, Lhwaja Ubaydullah
Ahrar?
D. G.S. Chaddha
A. Babur B. Humayun
57. Which one of the following regions is an
important supplier of citrus fruits? C. Akbar D. Jahangir
A. Equatorial region 61. Which one of the following statements
about the Government of India Act, 1919 is
B. Mediterranean region
not correct?
C. Desert region
A. It extended the practice of communal
D. Sub-humid region representation.
58. Who were the Nayanars? B. It made the Central Executive
responsible to the Legislature.
A. Those who were immersed in devotion
to Vishnu C. It is also known as the Montague-
Chelmsford Reforms.
B. Those who were devotes of Buddha
D. It paved the way for federalism by
C. Leaders who were devotees of Shiva
clearly separating the responsibilities of
D. Leaders who were devotees of the Centre
Basveshwara
62. The concept of “Four Pillar State”, free from
59. Match List I with List Ii and select the district magistracy for India was suggested
correct answer using the code given below by
the lists :
A. Lala Lajpat Rai
List I List II B. Ram Manohar Lohia
(Ethnic (Related
Territorial Occupational) C. Raja Ram Mohan Roy
Segment)
D. Subash Chandra Bose
A. Maruta 1. Pastoralists 63. Which one among the following is not a
Makkal part of the Fundamental Rights (Part III) of
B. Kuravan 2. Fishing the Constitution of India?
Makkal people A. Prohibition of traffic in human beings
C. Mullai Makkal 3. Ploughmen and forced labour

D. Neytal Makkal 4. Hill people B. Prohibition of employment of children


in factories

PAGE 457
www.byjusexamprep.com

C. Participation of workers in 67. Coral reefs are not found in which one of
management of industries the following regions?
D. Practice any profession, or to carry on A. Lakshadweep Islands
any occupation, trade or business B. Gulf of Kachchh
64. Which one of the following is not a C. Gulf of Mannar
geographical requirement for cultivation of
cotton? D. Gulf of Cambay
68. In which one of the following States in jute
A. Temperature reaching 25-degree
not significantly cultivated?
centigrade or more in summer
A. Assam B. West Bengal
B. Moderate to light rainfall
C. Odisha D. Andhra Pradesh
C. Medium loam soil with good drainage
69. Consider the following statements:
D. A growing period of at least 100 frost
1. According to Mahavamsa, Ashoka turned
free days
to the Buddha’s dhamma when his nephew
65. Which one of the following statements
Nigrodha preached the doctrine to him.
regarding temperature coniferous forest
biome is not correct? 2. Divyavadana ascribes Ashoka being
drawn to the Buddha’s teaching to the
A. They are characterized by very little influence of
undergrowth.
Samudra, a merchant-turned monk.
B. They have a growing period of 50 to
100 days in a year. 3. Dipavamsa speaks of Samudra, the 12-
year-old son of a merchant, as the key
C. There is low variation in annual figure in
temperature.
Ashoka’s coming under the influence of the
D. There is high range in spatial Buddhist dhamma.
distribution of annual precipitation. Which of the statements given above is/are
66. Match List-I with List-II and select the correct?
correct answer using the code given below A. 1 only B. 2 only
the lists:
C. 1 and 2 D. 1 and 3
List I List II
70. Name the site that gives us valuable
(Peak) (Name of Hill)
information about India’s maritime links on
A. Anaimudi 1. Nilgiri the Coromandel Coast.
B. Doddabetta 2. Satpura A. Bharukachchha B. Karur

C. Dhupgarh 3. Aravali C. Arikamedu D. Anuradhapura


71. Where was the largest quantity of cichlids
D. Guru Shikhar 4. Annamali
found in India?
A. Backwaters of Kerala
Codes: B. Sundarbans
A. 3 2 1 4 B. 3 1 2 4 C. Narmada
C. 4 1 2 3 D. 4 2 1 3 D. Godavari

PAGE 458
www.byjusexamprep.com

72. Which Greek philosopher coined the term 78. Which one of the following is active
“Geography” in the 3-rd. century B.C.E.? transport?
A. Euclid B. Plato A. It is the movement of a substance
against a diffusion gradient with the
C. Eratosthenes D. Clio
use of energy
73. Who is the author of the 16th century
B. It is the movement of a substance
Sanskrit text, the Vraja Bhakti Vilasa which
against a diffusion gradient without the
focuses on: the Braj region in North India?
use of energy.
A. Todar Mal B. Narayana Bhatta
C. It is the movement of a substance
C. Chaitanya D. Rupa Goswami against a diffusion gradient with the
74. Bose-Einstein Condensate is: use of energy from photosynthesis.

A. solid state of matter D. It is the movement of a substance along


a diffusion gradient with the use of
B. fifth state of matter energy from respiration.
C. plasma 79. Chlorophy-II in photosynthetic prokaryotic
D. state of condensed matter bacteria is associated with:

75. The rate of evaporation of liquid does not A. plastids


depend upon: B. membranous vesicles
A. temperature C. nucleoids
B. its surface area exposed to the D. chromosomes
atmosphere
80. What do you mean by ‘Demographic
C. its mass
Dividend’?
D. humidity
A. A rise in the rate of economic growth
76. Rutherford’s alpha particle scattering due to a higher share of working age
experiment on thin gold foil was people in a
responsible for the
B. A rise in the rate of literacy due to
discovery of: development of educational
institutions in different
A. electron B. proton
C. A rise in the standard of living of the
C. atomic nucleus D. neutron
people due to the growth of alternative
77. Food chain is: livelihood
A. relationship between autotrophic D. A rise in the gross employment ratio of
organisms a country due to government policies
B. exchange of genetic material between E. None of the above
two organisms
81. Which one of the following organisms
C. passage of food (and thus energy) from responsible for sleeping sickness?
one organism to another
A. Leishmania B. Trypanosoma
D. modern entrepreneur establishment
providing food outlets C. Ascaris D. Helicobacter

PAGE 459
www.byjusexamprep.com

82. Which one of the following body 87. The value of the slope of a normal demand
parts/organs of the human body does not curve is:
have smooth muscles?
A. positive B. negative
A. Ureters
C. zero D. infinity
B. Iris of eye
88. Which one of the following is an example of
C. Bronchi of lungs a price floor?
D. Biceps A. Minimum Support Price (MSP) for
83. What is Inter-cropping? Jowar in India

A. It is the time period between two B. Subsidy given to farmers to buy


cropping seasons. fertilizers
B. It is growing of two or more crops in C. Price paid by people to buy goods from
random mixture. ration shops
C. It is growing of two or more crops in D. Maximum Retail Price (MRP) printed on
definite row patterns. the covers/packets of goods sold in
India
D. It is growing of different crops on a
piece of land in a pre-planned 89. Which one of the following factors is not
succession. considered in determining the Minimum
Support Price (MSP) in India?
84. Magnification is
A. Cost of production
A. actual size of specimen/observed size
B. Price trends in international and
B. observed size of specimen/actual size
domestic markets
C. actual size of specimen – observed size
C. Cost of living index
D. actual size of specimen × observed size
D. Inter-crop price parity
85. Which one of the following cell organelles
is known as ‘suicide bags’ of a cell? 90. Which one of the following is not a
dimension of the Human Development
A. Lysosomes Index?
B. Plastids A. A long and healthy life
C. Endoplasmic reticulum B. Knowledge
D. Mitochondria C. Access to banking and other financial
86. Which one of the following statements with provisions
regard to economic models is not correct? D. A decent standard of living
A. They involve simplification of complex
91. Gini Coefficient or Gini Ratio can be
processes.
associated with which one of the following
B. They represent the whole or a part of a measurements in an economy?
theory.
A. Rate of inflation
C. They can be expressed only through
B. Poverty index
equations.
C. Income inequality
D. They help in gaining an insight into
cause and effect. D. Personal income

PAGE 460
www.byjusexamprep.com

92. Consider the following statements: B. Isobars are atoms having the same
atomic number but a different mass
1. Particles of Matter intermix on their
number.
own.
C. The mass number of an atom is equal to
2. Particles of Matter have a force acting
the number of nucleons in its nucleus.
between them.
D. Valency is the combining capacity of an
Which of the statements given above is/are atom.
correct?
97. If the speed of a moving magnet inside a
A. 1 only B. 2 only coil increases, the electric current in the coil
C. Both 1 and 2 D. Neither 1 nor 2 A. increase B. decreases
93. Rate of evaporation increases with C. reverses D. remains the same
A. an increase of surface area 98. The frequency (in Hz) of a note that is one
B. an increase in humidity octave higher than 500 Hz is

C. a decrease in wind speed A. 375 B. 750

D. a decrease in temperature C. 1000 D. 2000

94. If an object is at rest, then the time (X-axis) 99. Which of the following statements as per
versus distance (Y-axis) graph the Constitution of India are not correct?
1. The President tenders his resignation to
A. is vertical
the Chief Justice of India.
B. is horizontal
2. The Vice-President tender his
C. has 45o positive slope resignation to the President of India.
D. has 45o negative slope 3. The Comptroller and Auditor General of
India are removed from his office in the
95. Consider the following statements about
like manner as the President of India.
mixture:
4. A Judge of the Supreme Court can
1. A substance can be separated into
resign his office by writing under his
other kinds of Matter by any physical
hand addressed to the Chief Justice of
process.
India.
2. Dissolved sodium chloride can be Select the correct answer using the code
separated from water by the physical given below :
process of evaporation.
A. 1 and 2 only B. 3 and 4 only
Which of the statements given above is/are
correct? C. 1, 2 and 3 D. 1, 3 and 4

A. 1 only B. 2 only 100. Rajya Sabha has exclusive jurisdiction in

C. Both 1 and 2 D. Neither 1 nor 2 A. creation of new States

96. Which of the following statements is not B. declaring a war


correct? C. financial emergency
A. Elements are defined by the number of D. authorizing Parliament to legislate on a
protons they possess. subject in the State List

PAGE 461
www.byjusexamprep.com

101. The term soil impoverishment relates to 104. Which one of the following mountains
which one of the following? separates the Black Sea and the Caspian
Sea?
A. Soil erosion
A. Urals
B. Soil deposition
B. Caucasus
C. Soil getting very deficient in plant
nutrients C. Carpathians

D. Soil getting enriched with plant D. Balkan mountains


nutrients 105. Rains caused by thunderstorms during the
102. Which one of the following is the correct hot weather season (mid-March to mid-
sequential phase in the successional June) in Karnataka are called
development of vegetation community in a A. Kalbaisakhi
habitat?
B. Mango showers
A. Migration, Reaction, Stabilization and
C. Loo
Nudation
D. Cherry blossoms
B. Migration, Stabilization, Reaction and
Nudation 106. Which one of the following is the largest
freshwater lake in India?
C. Nudation, Migration, Reaction and
Stabilization A. Chilika B. Loktak

D. Reaction, Migration, Stabilization and C. Dal D. Wular


Nudation Directions: The following 4 (four) items
103. Match List I with List II and select the consist of two statements, Statement I and
correct answer using the code given below Statement II. Examine these two
the lists : statements carefully and select the correct
answer using the code given below :
List I List II
107. Statement I: The Greek travellers were
(Soil (Major
most impressed by the fertility of India's
type) characteristic)
soil and the energy and ability of her
A. Oxisols 1. Very rich in Organic cultivators.
matter Statement II: Ancient India knew the use of
B. Vertisols 2. Soil lacking horizons manure.

C. Histosols 3. Very old and Highly A. Both the statements are individually
weathered true, and Statement II is the correct
explanation of Statement I.
D. Entisols 4. Rich in clay content
B. Both the statements are individually
and highly basic
true, and Statement II is not the correct
explanation of Statement I.
Code: C. Statement I is true, but Statement II is
false.
A. A-3 B-1 c-4 D-2 B. A-3 B-4 c-1 D-2
D. Statement I is false, but Statement II is
C. A-2 B-1 c-4 D-3 D. A-2 B-4 c-1 D-2 true.

PAGE 462
www.byjusexamprep.com

108. Statement I: Non-cooperation began in A. Both the statements are individually


Punjab with the student movement true, and Statement II is the correct
inspired by Lala Lajpat Rai in January 1921 explanation of Statement I.
Statement II: The Sikh dominated central B. Both the statements are individually
Punjab countryside was stirred by the true, and Statement II is not the correct
powerful Akali upsurge. explanation of Statement I.

A. Both the statements are individually C. Statement I is true, but Statement II is


true, and Statement II is the correct false.
explanation of Statement I. D. Statement I is false, but Statement II is
B. Both the statements are individually true.
true, and Statement II is not the correct 111. Who were Alvars?
explanation of Statement I.
A. Those who immersed in devotion to
C. Statement I is true, but Statement II is Vishnu
false.
B. Devotees of Shiva
D. Statement I is false, but Statement II is
true. C. Those who worshipped the abstract
form of God
109. Statement I: The Oudh Kisan Sabha
established in 1920 failed to bring under is D. Devotees of Shakti
wing any Kisan Sabhas. 112. Which one of the following is monatomic?
Statement II: The Oudh Kisan Sabha asked A. Hydrogen B. Sulphur
the Kisans to refuse to till bedakhli land, not
C. Phosphorus D. Helium
to offer hari and begar.
113. In Graphite, each carbon atom is bonded to
A. Both the statements are individually
three other carbon atoms
true, and Statement II is the correct
explanation of Statement I. A. forming a three-dimensional structure
B. Both the statements are individually B. in the same plane giving a hexagonal
true, and Statement II is not the correct array
explanation of Statement I.
C. in the same plane giving a square array
C. Statement I is true, but Statement II is D. in the same plane giving a pentagonal
false. array
D. Statement I is false, but Statement II is 114. Soap solution used for cleaning purpose
true. appears cloudy. This is due to the fact that
110. Statement I: The United Provinces during soap micelles can
Non-Cooperation became one of the A. refract light
strongest bases of the Congress.
B. scatter light
Statement II: The literary outcrop of Non-
Cooperation in Bengal was quite meager C. diffract light
compared to the days of the Swadeshi D. polarize light
agitation.

PAGE 463
www.byjusexamprep.com

115. People prefer to wear cotton clothes in the 118. Which one of the following acids is also
summer season. This is due to the fact that known as Vitamin C?
cotton clothes are
A. Methanoic acid
A. good absorbers of water
B. Ascorbic acid
B. good conveyors of heat
C. Lactic acid
C. good radiators of heat
D. Tartaric acid
D. good absorbers of heat
119. Which one of the following is not found in
116. Employing Chromatography, one cannot
animal cells?
separate
A. radio-isotopes A. Free ribosomes

B. colours from a dye B. Mitochondria


C. pigments from a natural colour C. Nucleolus
D. drugs from blood D. Cell wall
117. Consider the following statement : 120. Marsilea, Fern and Horse-tail are examples
"The atomic number of an element is a of which one of the following plant groups?
more fundamental property than its atomic
A. Pteridophyta
mass." Who among the following scientists
has made the above statement? B. Bryophyta
A. Dmitri Mendeleev B.Henry Moseley C. Gymnosperms
C. J. J. Thomson D. Ernest Rutherford D. Angiosperms

PAGE 464
www.byjusexamprep.com

Elementary Mathematics
1. Given y is inversely proportional to √x, and 6. Six cubes, each with 12 cm edge are joined
x = 36 when y = 36. What is the value of x end to end. What is the surface area of
when y = 54? resulting cuboid?
A. 54 B. 27 A. 3000 cm2 B. 3600 cm2
C. 16 D. 8 C. 3744 cm2 D. 3777 cm2
2. A person carries Rs. 500 and wants to buy 7. A right circular cone has a height 8 cm. If the
apples and oranges out of it. If the cost of radius of its base is 6 cm, then what is its
one apple is Rs. 5 and the cost of one total surface area?
orange is Rs. 7, then what is the number of
ways in which a person can buy both apples A. 96π cm2 B. 69π cm2
and oranges using total amount? C. 54π cm2 D. 48π cm2
A. 10 B. 14 8. A bucket is in the form of a truncated cone.
C. 15 D. 17 The diameters of the base and top of the
bucket are 6 cm and 12 cm respectively. If
3. Radha and Rani are sisters. Five years back the height of the bucket is 7 cm, what is the
the age of Radha was three times that of capacity of the bucket?
Rani, but one year back the age of Radha
was two times that of Rani. What is the age A. 535 cm3 B. 462 cm3
difference between them?
C. 234 cm3 D. 166 cm3
A. 8 B. 9
9. The volume of a hemisphere is 155232 cm3.
C. 10 D. 11 What is the radius of the hemisphere?
4. Consider the following statements : A. 40 cm B. 42 cm
1. If p is relatively prime to each of q and C. 38 cm D. 36 cm
r, then p is relatively prime to the
product qr. 10. Three copper spheres of radii 3 cm, 4 cm
and 5 cm are melted to form a large sphere.
2. If p divides the product qr and if p What is its radius?
divides q, then p must divide r.
A. 12 cm B. 10 cm
Which of the above statements is/are
correct? C. 8 cm D. 6 cm

A. 1 only B. 2 only 11. Consider a trapezium ABCD, in which AB is


parallel to CD and AD is perpendicular to
C. Both 1 and 2 D. Neither 1 nor 2 AB. If the trapezium has an incircle which
5. If the sum of a real number and its touches AB at E and CD at F, where EB = 25
reciprocal is 26/5, then how many such cm and FC = 16 cm, then what is the
numbers are possible? diameter of the circle?

A. None B. One A. 16 cm B. 25 cm

C. Two D. Four C. 36 cm D. 40 cm

PAGE 465
www.byjusexamprep.com

12. A thin rod of length 24 feet is cut into rods 18. If log101995 = 3.3000, then what is the value
of equal size and joined so as to form a of (0.001995)1/8?
skeleton cube. What is the area of one of
the faces of the largest cube thus A. B.
constructed?
A. 25 square feet B. 24 square feet C. D.
C. 9 square feet D. 4 square feet
19. The sides of a triangle are 30 cm, 28 cm and
13. If one side of a right-angled triangle (with 16 cm respectively. In order to determine
all sides integers) is 15 cm, then what is the its area, the logarithm of which of the
maximum perimeter of the triangle? quantities are required?
A. 240 cm B. 225 cm A. 37, 11, 28, 16
C. 113 cm D. 112 cm B. 21, 30, 28, 7
14. A solid metallic cylinder of height 10 cm and C. 37, 21, 11, 9
radius 6 cm is melted to make two cones in
the ratio of volume 1 : 2 and of same height D. 37, 21, 9, 7
as 10 cm. What is the percentage increase
20. The quotient when x4 – x2 + 7x + 5 is divided
in the flat surface area?
by (x + 2) is ax3 + bx2 + cx + d. What are the
A. 25% B. 50% values of a, b, c and d respectively?
C. 75% D. 100% A. 1, -2, 3, 1 B. -1, 2, 3, 1
15. A hollow sphere of external and internal C. 1, -2, -3, -1 D. -1, 2, -3, -1
diameters 6 cm and 4 cm respectively is
melted into a cone of base diameter 8 cm. 21. What is the ratio of the area of a square
what is the height of the cone? inscribed in a semicircle of radius r to the
area of square inscribed in a circle of radius
A. 4.75 cm B. 5.50 cm r?
C. 6.25 cm D. 6.75 cm A. 1 : 2 B. 2 : 5
16. Let XYZ be an equilateral triangle in which C. 2 : 3 D. 3 : 5
XY = 7 cm. If A denotes the area of the
triangle, then what is the value of log10A4? 22. A piece of wire of length 33 cm is bent into
(Given that log101050 = 3.0212 and log1035 an arc of a circle of radius 14 cm. what is the
= 1.5441) angle subtended by the arc at the centre of
the circle?
A. 5.3070 B. 5.3700
A. 75o B. 90o
C. 5.5635 D. 5.6535
C. 135o D. 150o
17. What is (x – a)(x – b) (x – c) equal to?
23. A right circular cylinder has a diameter of 20
A. x3 – (a+b+c)x2 + (bc+ca+ab)x – abc cm and its curved surface area is 1000 cm2.
B. x3 + (a+b+c)x2 + (bc+ca+ab)x + abc What is the volume of the cylinder?

C. x3 – (a+b+c)x2 + (a+b+c)x – abc A. 4000 cm3 B. 4500 cm3

D. x3 + (a+b+c)x2 + (a + b + c)x + abc C. 5000 cm3 D. 5200 cm3

PAGE 466
www.byjusexamprep.com

24. A square is drawn such that its vertices are 30. What is the sum of all integer value of n for
lying on a circle of radius 201 mm. What is which n2 + 19n + 92 is a perfect square?
the ratio of area of circle to that of square?
A. 21 B. 19
A. 11 : 7 B. 7 : 11
C. 0 D. -19
C. 20 : 19 D. 19 : 20
31. Which one of the following is not correct?
25. If the length of the hypotenuse of a right
A. 1 is neither prime nor composite
angled triangle is 10 cm, then what is the
maximum area of such a right angled B. 0 is neither positive nor negative.
triangle?
C. If p × q is even, then p and q are always
A. 100 cm2 B. 50 cm2 even
C. 25 cm2 D. 10 cm2 D. √2 is an irrational number
26. Two numbers p and q are such that the 32. If the ratio of the work done by (x + 2)
quadratic equation px2 + 3x + 2q = 0 has -6 workers in (x – 3) days to the work done by
as the sum and the product of the roots. (x + 4) workers in (x – 2) days is 3 : 4, then
What is the value of (p – q)? what is the value of x?
A. -1 B. 1 A. 8 B. 10
C. 2 D. 3 C. 12 D. 15
27. If X = {a, {b}, c}, 33. What is the maximum value of the
Y = {{a}, b, c} and expression ?
Z = {a, b, {c)},
A. B.
then (X ∩ Y) ∩ Z equals to
A. {a, b, c} B. {{a}, {b}, {c}}
C. 1 D.
C. {Φ} D. Φ
28. What is the value of 34. If , then what is the value

? of ?

A. 2 B. 3
A. 0 B.
C. 4 D. 7

C. D. 1 35. If (b - 6) is one root of the quadratic


equation x2 - 6x + b = 0, where b is an
29. What is the LCM of the polynomials x3 + 3x2 integer, then what is the maximum value of
+ 3x + 1, x3 + 5x2 + 5x + 4 and x2 + 5x + 4? b2?

A. (x + 1)3(x + 4) (x2 + x + 1) A. 36 B. 49

B. (x + 4) (x2 + x + 1) C. 64 D. 81

C. (x + 1)(x2 + x + 1) 36. How many Pairs of natural numbers are


there such that the difference of their
D. (x + 1)2(x + 4)(x2 + x + 1) squares is 35?

PAGE 467
www.byjusexamprep.com

A. 1 B. 2 A. 1% loss
C. 3 D. 4 B. 1% profit

37. The equation x2 + px + q = 0 has roots equal C. No profit no loss


to p and q where q ≠ 0. What are the values D. 0.5% profit
of p and q respectively?
44. If , where x, y and z are
A. 1, -2 B. 1, 2
C. -1, 2 D. -1, -2
natural numbers, then what is (x + y + z)
38. The sum of the squares of four consecutive
equal to?
natural numbers is 294. What is the sum of
the numbers? A. 6 B. 7
A. 38 B. 34 C. 8 D. 9

C. 30 D. 26 45. A library has an avenge number, of 510


visitors on Sunday and 240 on other days.
39. What is the digit in the unit’s place of the What is the average number of visitors per
number represented by 398 – 389? day in a month of 30 day beginning with
Saturday?
A. 3 B. 6
A. 276 B. 282
C. 7 D. 9
C. 285 D. 375
40. If 10n divides 623 × 759 × 1052, then what is
the largest value of n? 46. Consider the following statements:

A. 20 B. 22 1. √75 is a rational number.


2. There exists at least a positive integer x
C. 23 D. 28
such that
41. A real number x is such that (x – x2) is
maximum. What is x equal to?
3. for all real values of x.
A. -1.5 B. -0.5
C. 0.5 D. 1.5 4. 4.232323……. can be expressed in the
form where p and q are integers.
42. It takes 11 hours for a 600 km journey if 120
km is done by train and the rest by car. It
Which of the above statements are
takes 40 minutes more if 200 km are
correct?
covered by train and the rest by car. What
is the ratio of speed of the car to that of the A. 1 and 2 B. 2 and 3
train?
C. 3 and 4 D. 2 and 4
A. 3 : 2 B. 2 : 3 47. Two taps X and Y are fixed to a water tank.
C. 3 : 4 D. 4 : 3 If only X is opened, it drains out the full tank
of water in 20 minutes. If both X and Y are
43. A person sells two items each at Rs. 990, opened, then they drain out the full tank of
one at a profit of 10% and another at a loss water in 15 minutes. If only Y is opened,
of 10% What is the combined percentage of how long does it take to drain out the full
profit or loss for the two items? tank of water

PAGE 468
www.byjusexamprep.com

A. 30 minutes B. 45 minutes 54. All the four sides of a parallelogram area of


equal length. The diagonals are in the ratio
C. 60 minutes D. 90 minutes
1 : 2. If the sum of the lengths of the
48. Let x be the smallest positive integer such diagonals is 12 cm, then what is the area of
that when 14 divides x the remainder is 7; the parallelogram?
and when 15 divides x, the remainder is 5.
A. 9 cm2 B. 12 cm2
Which one of the following is correct?
C. 16 cm2 D. 25 cm2
A.
55. An equilateral triangle and a square are
B. constructed using metallic wires of equal
C. length. What is the ratio of area of triangle
to that of square?
D.
A. 3 : 4 B. 2 : 3
49. What is the number of digits in 725, 823 and
920 respectively? [Given log102 = 0.301, C. 4√3 : 9 D. 2√3 : 9
log103 = 0.477, log107 = 0.845] 56. Let two lines p and q be parallel. Consider
A. 21, 20, 19 B. 20, 19, 18 two points B and C on the line p and two
points D and E on the line q. The line
C. 22, 21, 20 D. 22, 20, 21 through B and E intersects the line through
50. What is the square root of 16 + 6 √7? C and D at A in between the two lines p and
q. If AC : AD = 4 : 9, then what is the ratio of
A. 4 + √7 B. 4 - √7 area of triangle ABC to that of triangle ADE?
C. 3 + √7 D. 3 - √7 A. 2 : 3 B. 4 : 9
51. The length and breadth of a rectangle are C. 16 : 81 D. 1 : 2
increased by 20% and 10% respectively.
What is the percentage increase in the area 57. Suppose P, Q and R are the mid-points of
of the rectangle? sides of a triangle of area 128 cm2. If a
triangle ABC is drawn by joining the mid-
A. 32% B. 30% points of sides of triangle PQR, then what is
C. 25% D. 15% the area of triangle ABC?

52. Two cylinders of equal volume have their A. 4 cm2 B. 8 cm2


heights in the ratio 2 : 3. What is the ratio C. 16 cm2 D. 32 cm2
of their radii?
58. A piece of wire is in the form of a sector of
A. √3 : 1 B. √3 : √2 a circle of radius 20 cm, subtending an
C. 2 : √3 D. √3 : 2 angle 150o at the centre. If it is bent in the
form of a circle, then what will be its radius?
53. ABC is a triangle right angled at B. If AB = 5
cm and BC = 10 cm, then what is the length A. cm
of the perpendicular drawn from the vertex
B to the hypotenuse? B. 7 cm
A. 4 cm B. 2√5 cm C. 8 cm

C. cm D. 8 cm D. None of these

PAGE 469
www.byjusexamprep.com

59. If l is the length of the median of an A. 8 cm2 B. 5 cm2


equilateral triangle, then what is its area?
C. 3.6 cm2 D. 1.8 cm2
A. B. 65. The perimeter of a triangle is 22 cm.
through each vertex of the triangle, a
straight line parallel to the opposite side is
C. D. 2l2
drawn. What is the perimeter of triangle
60. The areas of three adjacent faces of a formed by these lines?
cuboid are x, y and z. If V is the volume of
the cuboid, then which one of the following A. 33 cm B. 44 cm
is correct? C. 66 cm D. 88 cm
2
A. V = xyz B. V = xyz
66. What is the value of ?
C. V3 = xyz D. V = (xyz)2
61. Let PQRS be a parallelogram whose A. 0 B. 1
diagonals PR and QS intersect at O. If
triangle QRS is an equilateral triangle C. 2 D. 4
having a side of length 10 cm, then what is 67. The angles of elevation of the tops of two
the length of the diagonal PR? pillars of heights h and 2h from a point P on
A. 5√3 cm B. 10√3 cm the line joining the feet of the two pillars
are complementary. If the distances of the
C. 15√3 cm D. 20√3 cm
foot of the pillars from the point P are x and
62. The perimeters of two similar triangles ABC y respectively, then which one of the
and PQR are 75 cm and 50 cm respectively. following is correct?
If the length of one side of the triangle PQR
is 20 cm, then what is the length of A. 2h2 = x2y B. 2h2 = xy2
corresponding side of the triangle ABD? C. 2h2 = xy D. 2h2 = x2y2
A. 25 cm B. 30 cm 68. If 0 < θ < 90o, sinθ = 3/5 and x = cotθ, then
C. 40 cm D. 45 cm what is the value of 1 + 3x + 9x2 + 27x3 + 81x4
+ 243x5?
63. A line segment AB is the diameter of a circle
with centre at O having radius 6.5 cm. Point A. 941 B. 1000
P is in the plane of the circle such that AP =
x and BP = y. In which one of the following C. 1220 D. 1365
cases the point P does not lie on the circle? 69. If cos2x + cos x = 1, then what is the value of
A. x = 6.5 cm and y = 6.5 cm sin12x + 3sin10x + 3sin8x + sin6x?

B. x = 12 cm and y = 5 cm A. 1 B. 2
C. x = 5 cm and y = 12 cm C. 4 D. 8
D. x = 0 c and y = 13 cm 70. What is the value of log10(cosθ) + log10(sinθ)
64. The sides AD, BC of a trapezium ABCD are + log10(tanθ) + log10(cotθ) + log10(secθ) +
parallel and the diagonals AC and BD meet log10(cosecθ)?
at O. If the area of triangle AOB is 3 cm2 and A. -1 B. 0
the area of triangle BDC is 8 cm2, then what
is the area of triangle AOD? C. 0.5 D. 1

PAGE 470
www.byjusexamprep.com

71. Let the bisector of the angle BAC of a A. 72 days B. 64 days


triangle ABC meet BC in X. Which one of the
C. 60 days D. 54 days
following is correct?
77. When N is divided by 17, the quotient is
A. AB < BX B. AB > BX
equal to 182. The difference between the
C. AX = CX D. None of these quotient and the remainder is 175. What is
the value of N?
72. Which one of the following is correct in
reaped of angled triangle? A. 2975 B. 3094
A. Its orthocentre lies inside the triangle C. 3101 D. 3269

B. Its orthocentre lies outside the triangle 78. The train fare and bus fare between two
stations is in the ratio 3 : 4. If the train fare
C. Its oithocentre lies on the triangle increases by 20% and bus fare increases by
D. It has no orthocentre 30%, then what is the ratio between revised
train fare and revised bus fare?
73. The area of a sector of a circle of radius 4
cm is 25.6 cm2. What is the radian measure A. B.
of the arc of the sector?
A. 2.3 B. 3.2 C. D.
C. 3.3 D. 3.4
79. The monthly incomes of A and B are in the
74. Three parallel lines x, y and z are cut by two ratio 4 : 3. Each saves Rs. 600. If their
transversals m and n. Transversal m cuts expenditures are in the ratio 3 : 2, then
the lines x, y, z at P, Q, R respectively; and what is the monthly incomes of A?
Transversal n cuts the lines x, y, z at L, M, N
respectively. If PQ = 3 cm, QR = 9 cm and A. Rs. 1800 B. Rs. 2000
MN = 10.5 cm, then what is the length of C. Rs. 2400 D. Rs. 3600
LM?
80. A trader sells two computers at the same
A. 3 cm B. 3.5 cm price, making a profit of 30% on one and a
loss of 30% on the other. What is the net
C. 4 cm D. 4.5 cm
loss or profit percentage on the
75. A hollow right circular cylindrical vessel of transaction?
volume V whose diameter is equal to its
A. 6% loss B. 6% gain
height, is completely filled with water. A
heavy sphere of maximum possible volume C. 9% loss D. 9% gain
is then completely immersed in the vessel.
81. A lent B to some amount of Rs. 25000 and
What volume of water remains in the
lent some amount to C at same 7% simple
vessel?
interest. After 4 years A received Rs. 11200
as interest from B and C. How much did A
A. B.
lend to C?
A. Rs. 20000 B. Rs. 25000
C. D.
C. Rs. 15000 D. Rs. 10000
76. A stock of food grains is enough for 240 82. If the equations x2 + 5x + 6 = 0 and x2 + kx +
men for 48 days. How long will the same 1 = 0 have a common root, then what is the
stock last for 160 men? value of k?

PAGE 471
www.byjusexamprep.com

3. Adding 1 to the product of two


A. or B. or
consecutive odd numbers makes it a
perfect square.
C. or D. or
Which of the above statements are
correct?
83. What is the LCM of ?
A. 1, 2 and 3 B. 1 and 2 only

A. B. C. 2 and 3 only D. 1 and 3 only


88. HCF of two numbers is 12. Which one of the
C. D. following can never be their LCM?

84. X, Y and Z start at same point and same time A. 80 B. 60


in the same direction to run around a C. 36 D. 24
circular stadium. X completes a round in
252 seconds, Y in 308 seconds and Z in 198 89. If and y = x3, then y satisfies
seconds. After what time will they meet
again at the starting point? which one of the following equations?
A. 26 minutes 18 seconds A. 8y2 – 20y – 1 = 0
B. 42 minutes 36 seconds B. 8y2 + 20y – 1 = 0
C. 45 minutes C. 8y2 + 20y + 1 = 0
D. 46 minutes 12 seconds
D. 8y2 – 20y + 1 = 0
3
85. For what value of k can be expression x +
kx2 – 7x + 6 be resolved into three linear 90. Let a and b be two positive real numbers
factors? such that a a + b b = 32 and
A. 0 B. 1 a b + b a = 31 . What is the value of
C. 2 D. 3 ?
86. The rate of interest on two different
A. 5
schemes is the same and it is 20%. But in
one of the schemes, the interest is B. 7
compounded half yearly and in the other
the interest is compounded annually. Equal C. 9
amounts are invested in the schemes. If the D. Cannot be determined
difference of the returns after 2 years is Rs.
482, then what is the principal amount in Direction: Read the following information and
each scheme? answer the four items that follow:
A. Rs. 10,000 B. Rs. 16,000 The data shows that Indian roads are timing
C. Rs. 20,000 D. Rs. 24,000 deadlier over the years.

87. Consider the following statements:


1. Unit digit in 17174 is 7
2. Difference of the squares of any two
odd numbers is always divisible by 8

PAGE 472
www.byjusexamprep.com

91. What is the average number of cyclists


A. B.
killed daily in road accidents in 2017?
A. 10 B. 12
C. D.
C. 19 D. 21
92. What is the average number of bikers killed 97. Number of scooters of both the companies
daily in road accidents in the year 2017? sold by showroom B is what per cent more
than the number of scooters of company X
A. 163 B. 152 sold by showroom A?
C. 147 D. 134
A. B.
93. What is the approximate percentage
change in the pedestrians’ fatalities during
the period 2014-17? C. D.
A. 66% B. 68%
98. Number of scooters of company Y sold by
C. 71% D. 76% showroom E is what per cent of the number
94. What was the average number of of scooters of both companies sold by
pedestrians killed per day in the year 2017? showroom C?

A. 51 B. 53 A. 52 B. 54

C. 54 D. 56 C. 55 D. 56
Direction: Read the following information and Direction: Read the following frequency
answer the four items that follow: distribution for two series of observations and
answer the two items that follow :
Let the distribution of number of scooters of
companies X and Y sold by % showrooms (A,B,CD
and E) in a certain year be denoted by S1 and the
distribution of number of scooters of only
company X sold by the five showrooms in the
same year be denoted by S2

95. What is the difference between the 99. What is the mode of the frequency
number of scooters of both companies sold distribution of Series-II?
by showroom A and total number of
scooters of company X sold by showrooms A. 26 B. 36
B and E together? C. 46 D. 56
A. 416 B. 426 100. What is the mean of frequency distribution
C. 432 D. 436 of Series-I?
96. What is the average number of scooters of A. 33.6 B. 35.6
company Y sold by the showrooms A, C and
E? C. 37.6 D. 39.6

PAGE 473
www.byjusexamprep.com

ANSWERS KEY II 2019


ENGLISH

1 A 21 A 41 C 61 A 81 A 101 A
2 C 22 B 42 C 62 D 82 B 102 B
3 B 23 A 43 C 63 D 83 D 103 A
4 A 24 B 44 C 64 D 84 C 104 B
5 A 25 C 45 A 65 A 85 A 105 C
6 A 26 C 46 C 66 C 86 C 106 B
7 D 27 A 47 A 67 B 87 B 107 A
8 A 28 B 48 A 68 B 88 A 108 B
9 B 29 A 49 B 69 C 89 D 109 A
10 B 30 C 50 C 70 D 90 D 110 A
11 C 31 B 51 D 71 B 91 C 111 A
12 A 32 D 52 A 72 B 92 B 112 B
13 A 33 A 53 A 73 A 93 A 113 C
14 B 34 A 54 B 74 A 94 A 114 C
15 A 35 B 55 A 75 A 95 D 115 D
16 A 36 A 56 A 76 A 96 D 116 A
17 D 37 A 57 B 77 B 97 C 117 A
18 A 38 C 58 C 78 C 98 C 118 A
19 B 39 B 59 C 79 A 99 B 119 A
20 A 40 A 60 D 80 D 100 B 120 A

GENERAL KNOWLEDGE
1 A 21 A 41 A 61 B 81 B 101 C
2 D 22 A 42 A 62 B 82 D 102 C
3 A 23 D 43 A 63 C 83 C 103 B
4 C 24 C 44 C 64 D 84 B 104 B
5 D 25 B 45 B 65 C 85 A 105 B
6 B 26 D 46 C 66 C 86 B 106 D
7 C 27 B 47 A 67 D 87 B 107 A
8 C 28 D 48 C 68 D 88 D 108 B
9 C 29 D 49 D 69 C 89 C 109 D
10 C 30 C 50 C 70 C 90 C 110 B
11 B 31 A 51 B 71 A 91 C 111 A
12 C 32 C 52 C 72 C 92 C 112 D

PAGE 474
www.byjusexamprep.com

13 A 33 C 53 A 73 B 93 A 113 B
14 B 34 A 54 D 74 B 94 B 114 B
15 D 35 D 55 D 75 C 95 B 115 A
16 B 36 D 56 B 76 C 96 B 116 A
17 A 37 A 57 B 77 C 97 A 117 B
18 B 38 D 58 C 78 A 98 C 118 B
19 A 39 A 59 C 79 B 99 D 119 D
20 D 40 D 60 A 80 A 100 D 120 A

Elementary Mathematics
1 C 21 B 41 C 61 B 81 C
2 B 22 C 42 B 62 B 82 B
3 A 23 C 43 A 63 A 83 D
4 A 24 A 44 A 64 D 84 D
5 C 25 C 45 C 65 B 85 A
6 C 26 C 46 D 66 C 86 C
7 A 27 D 47 C 67 C 87 C
8 B 28 B 48 B 68 D 88 A
9 B 29 A 49 C 69 A 89 A
10 D 30 D 50 C 70 B 90 C
11 D 31 C 51 A 71 B 91 A
12 D 32 B 52 B 72 C 92 D
13 A 33 D 53 B 73 B 93 A
14 B 34 C 54 C 74 B 94 D
15 A 35 D 55 C 75 B 95 D
16 A 36 B 56 C 76 A 96 A
17 A 37 A 57 B 77 C 97 C
18 B 38 B 58 D 78 A 98 C
19 D 39 B 59 A 79 C 99 C
20 A 40 A 60 B 80 C 100 C

PAGE 475
CDS II 2019
www.byjusexamprep.com

(Solutions)
ENGLISH
1. Ans. A.
The correct spelling for the given word is option A – Accommodate.
Accommodate means to provide lodging or sufficient space for.
2. Ans. C.
The correct spelling for the given word is option C- Recommend.
Recommend means put forward (someone or something) with approval as being suitable for a
particular purpose or role.
3. Ans. B.
The correct spelling is option B – Argument.
Argument means an exchange of diverging or opposite views, typically a heated or angry one.
4. Ans. A.
The correct spelling for the given word is option A – Decisive.
Decisive means settling an issue; producing a definite result.
5. Ans. A.
The correct spelling for the given word is option A – Aggressive.
Aggressive means ready or likely to attack or confront; characterized by or resulting from
aggression.
6. Ans. A.
The correct spelling for the given word is option A – Assassination.
Assassination means the action of assassinating someone.
7. Ans. D.
The correct spelling for the given word is option D – Embarrassment.
Embarrassment means a feeling of self-consciousness, shame, or awkwardness.

PAGE 476
www.byjusexamprep.com

8. Ans. A.
The most appropriate meaning of the given idiom is option A – Extremely cheap.
9. Ans. B.
The most appropriate meaning of the given idiom is option B – A shy person.
10. Ans. B.
The most appropriate meaning of the given idiom is option B – A difficult job.
11. Ans. C.
The most appropriate meaning of the given idiom is option C – To lose control of one’s own feelings.
12. Ans. A.
The most appropriate meaning of the given idiom is option A – To be very angry.
13. Ans. A.
The most appropriate meaning of the given idiom is option A- To be extremely happy.
14. Ans. B.
The most appropriate meaning of the given idiom is option B – Working together.
15. Ans. A.
The most appropriate meaning of the given idiom is option A- Prevent a small problem before it
becomes severe.
16. Ans. A.
The most appropriate meaning of the given idiom is option A- Completely alone.
17. Ans. D.
The most appropriate meaning of the given idiom is option D- An important piece of advice.
18. Ans. A.
Option A – Had is the most appropriate answer to fill the blank.
‘Had’ past simple and past participle of have.
19. Ans. B.
Option B – Themselves is the most appropriate answer to fill the blanks.
Themselves is a reflexive pronoun, reflexive pronouns are used when the subject and the object of
the verb refer to the same person or thing.
20. Ans. A.
Option A – build is the most appropriate answer to fill the blank.
21. Ans. A.
Option A – Libertarian is the most appropriate answer to fill the blank.
Libertarian - is a political philosophy and movement that upholds liberty as a core principle.
Liberation- the act of setting someone free from imprisonment, slavery, or oppression; release.

PAGE 477
www.byjusexamprep.com

Liberating- freeing a place or people from enemy occupation.


Liberty- the state of being free within society from oppressive restrictions imposed by authority on
one's way of life, behavior, or political views.
22. Ans. B.
Option B – Within a is the most appropriate answer to fill the blank.
23. Ans. A.
Option A- Breakthrough is the most appropriate answer to fill the blank.
A breakthrough is a sudden, dramatic, and important discovery or development.
An investment is - the action or process of investing money for profit.
A breakout is a forcible escape, especially from prison.
Breaking - separate or cause to separate into pieces as a result of a blow, shock, or strain.
24. Ans. B.
Option B – From is the most appropriate answer to fill the blank.
25. Ans. C.
Option C – A is the most appropriate answer to fill the blank.
26. Ans. C.
Option C – Based is the most appropriate answer to fill the blank.
27. Ans. A.
Option A – Studies is the most appropriate answer to fill the blank.
28. Ans. B.
Option B – Between is the most appropriate answer to fill the blank.
29. Ans. A.
Option A – Their, is the most appropriate answer to fill the blank.
30. Ans. C.
Option C – Though is the most appropriate answer to fill the blank.
31. Ans. B.
Option B – Were is the most appropriate answer to fill the blank.
32. Ans. D.
Option D – Between is the most appropriate answer to fill the blank.
33. Ans. A.
Option A – For is the most appropriate answer to fill the blank.

PAGE 478
www.byjusexamprep.com

34. Ans. A.
Option A – The is the most appropriate answer to fill the blank.
35. Ans. B.
Option B – of is the most appropriate answer to fill the blank.
36. Ans. A.
Option A – amount is the most appropriate answer to fill the blank.
37. Ans. A.
Confiscated is the most appropriate synonym for impounded.
Meaning of the other words:
Confiscated- (of property) taken or seized with authority.
Permitted- officially allow (someone) to do something.
Put on hold- a deception; a hoax.
38. Ans. C.
Challenged is the most appropriate synonym for impugned.
Meaning of the other words:
Expelled- officially make (someone) leave a school or other organization.
Challenged – used euphemistically to indicate that someone suffers disability in a specified respect.
Given allowance- the amount of something that is permitted, especially within a set of regulations
or for a specified purpose.
Rewarded - give something to (someone) in recognition of their services, efforts, or achievements.
39. Ans. B.
Intrinsic is the correct synonym for the given word.
Meaning of other words:
Biological – relating to biology or living organisms.
Extrinsic- not part of the essential nature of someone or something; coming or operating from
outside.
Unnatural- contrary to the ordinary course of nature; abnormal.
40. Ans. A.
Compulsory is the most appropriate synonym for the given word.
Meaning of other words:
Unnecessary – not needed.
By chance- Something that happens by chance was not planned by anyone.
Problematic- constituting or presenting a problem.

PAGE 479
www.byjusexamprep.com

41. Ans. C.
Treachery is the correct synonym for betrayal.
Meaning of the other words:
Loyal- giving or showing firm and constant support or allegiance to a person or institution.
Faithfulness- the quality of being faithful; fidelity.
Honesty- the quality of being honest.
42. Ans. C.
Smallest is the correct synonym for the given word.
Widest - of great or more than average width.
Slightly – to a small degree; not considerably.
Thick - with opposite sides or surfaces that are far or relatively far apart.
43. Ans. C.
Similar is the correct synonym for akin.
Meaning of the other words:
Removed - take (something) away or off from the position occupied.
Narrow – of small width in relation to length.
Unparallel - not parallel.
44. Ans. C.
Ascribe Is the correct synonym for the given word.
Meaning of given words:
Disapproved- have or express an unfavourable opinion.
Unofficial- not officially authorized or confirmed.
Tribute- an act, statement, or gift that is intended to show gratitude, respect, or admiration.
45. Ans. A.
Obtain is the most appropriate synonym for the given word acquiring.
Meaning of the other words:
Lose- be deprived of or cease to have or retain (something).
Giving- providing love or other emotional support; caring.
Thinking- the process of considering or reasoning about something.
46. Ans. C.
Wish for is the correct synonym for the given word desired.
Meaning of the other words:

PAGE 480
www.byjusexamprep.com

Disliked- feel distaste for or hostility towards.


Unlikely- be regarded with distaste or hostility; be unpopular.
Asked for - make a request or demand for something to somebody.
47. Ans. A.
The most appropriate sequence for the jumbled sentences is option A – RSPQ.
48. Ans. A.
The most appropriate sequence for the jumbled sentences is option A- QRPS.
49. Ans. B.
The most appropriate sequence for the jumbled sentences is option B- SRQP.
50. Ans. C.
The most appropriate sequence for the jumbled sentences is option C- PSRQ.
51. Ans. D.
The most appropriate sequence for the jumbled sentences is option D – QPSR.
52. Ans. A.
The most appropriate sequence for the jumbled sentences is option A – RQPS.
53. Ans. A.
The most appropriate sequence for the jumbled sentences is option A- QSRP.
54. Ans. B.
The most appropriate sequence for the jumbled sentences is option B – SRPQ.
55. Ans. A.
The most appropriate sequence for the jumbled sentences is option A- QPSR.
56. Ans. A.
‘Eats’ is a verb. A verb is a a word used to describe an action, state, or occurrence, and forming the
main part of the predicate of a sentence.
57. Ans. B.
‘Abruptly’ is an adverb.
A word or phrase that modifies or qualifies an adjective, verb, or other adverb or a word group,
expressing a relation of place, time, circumstance, manner, cause, degree, etc.
58. Ans. C.
‘In’ is a preposition.
A preposition a word governing, and usually preceding, a noun or pronoun and expressing a relation
to another word or element in the clause.

PAGE 481
www.byjusexamprep.com

59. Ans. C.
‘And’ is a conjunction.
A conjunction is a word used to connect clauses or sentences or to coordinate words in the same
clause.
60. Ans. D.
Truthfully is an adverb.
A word or phrase that modifies or qualifies an adjective, verb, or other adverb or a word group,
expressing a relation of place, time, circumstance, manner, cause, degree, etc
61. Ans. A.
‘Hurrah’ is an interjection.
Hence, option D is the correct answer.
62. Ans. D.
‘Meaningless’ is an adjective.
Hence, option D is the most appropriate answer.
63. Ans. D.
‘himself’ is a (reflexive) pronoun.
64. Ans. D.
‘Through’ is a preposition.
Hence, option D is the correct answer.
65. Ans. A.
‘Slowly’ is an adverb.
Hence, option a is the correct answer.
66. Ans. C.
Inelegance is the most appropriate antonym for the word.
Allure- the quality of being powerfully and mysteriously attractive or fascinating.
Charm – the power or quality of delighting, attracting, or fascinating others.
Ideal- satisfying one's conception of what is perfect; most suitable.
67. Ans. B.
Impervious is the correct antonym for the given word vulnerable, meaning - unable to be affected
by.
Imperious- arrogant and domineering.
Helpless- unable to defend oneself or to act without help.
Defenceless- without defence or protection; totally vulnerable.

PAGE 482
www.byjusexamprep.com

68. Ans. B.
Encouraging is the correct antonym.
Uncomfortable- causing or feeling slight pain or physical discomfort.
Demoralising- cause (someone) to lose confidence or hope.
Off putting- unpleasant, disconcerting.
69. Ans. C.
Persuasion is the correct antonym for the given word.
Wiles – devious or cunning stratagems employed in manipulating or persuading someone to do what
one wants.
Conviction- a formal declaration by the verdict of a jury or the decision of a judge in a court of law
that someone is guilty of a criminal offence.
Support- bear all or part of the weight of; hold up.
70. Ans. D.
Support is the correct antonym for the given word constrain.
Oblige- make (someone) legally or morally bound to do something.
Pressure- the use of persuasion or intimidation to make someone do something.
Restrict- put a limit on; keep under control.
71. Ans. B.
Unthinkable is the best antonym for the given word.
Acceptable- able to be agreed on; suitable.
Solvable – susceptible of solution or of being solved.
Believable- able to be believed; credible.
72. Ans. B.
Gleeful is the correct antonym for plaintive.
Gleeful means exuberantly or triumphantly joyful.
Melancholic- feeling or expressing pensive sadness.
Adventurous- willing to take risks or to try out new methods, ideas, or experiences.
Doleful- expressing sorrow; mournful.
73. Ans. A.
Accord is the most appropriate antonym for discord.
Dissension- disagreement that leads to discord.
Dispute- a disagreement or argument.
Friction- the resistance that one surface or object encounters when moving over another.

PAGE 483
www.byjusexamprep.com

74. Ans. A.
Respectful is the most appropriate antonym for insolent.
Autocratic- relating to a ruler who has absolute power.
Impudent- not showing due respect for another person; impertinent.
Thought provoking- stimulating careful consideration or attention.
75. Ans. A.
Sporadic is the most appropriate antonym.
Persistent- continuing to exist or occur over a prolonged period.
Continual- forming a sequence in which the same action or event is repeated frequently.
Ceaseless- constant and unending.
76. Ans. A.
The most appropriate sequence is option A -RPSQ.
77. Ans. B.
The most appropriate sequence is option B – QSRP.
78. Ans. C.
The most appropriate sequence is option C- QPSR.
79. Ans. A.
The most appropriate sequence is option A – SPRQ.
80. Ans. D.
The most appropriate sequence is option D – RSQP.
81. Ans. A.
The most appropriate sequence is option B- SRQP.
"Do the difficult things while they are easy and do the great things while they are small"
82. Ans. B.
The most appropriate sequence is option B – QSPR.
83. Ans. D.
The most appropriate sequence is option D – SPRQ.
84. Ans. C.
The most appropriate sequence is option C – RPQS.
85. Ans. A.
The most appropriate sequence is option A – RSQP.

PAGE 484
www.byjusexamprep.com

86. Ans. C.
The most appropriate sequence is option C – RQSP.
87. Ans. B.
The most appropriate sequence is option B -SQPR.
88. Ans. A.
The most appropriate sequence is option A – SPRQ.
89. Ans. D.
The error is in part D of the given sentence.
Here, “in human society innumerable problems” should be written as “innumerable problems in
human society”.
90. Ans. D.
The error is in part D of the given sentence.
‘Consumed’ should be replaced with ‘Consumes.
91. Ans. C.
The error is in part C of the given sentence.
“against’ should be replaced with ‘of’.
92. Ans. B.
The error is in part B of the given sentence.
‘The’ should be added before ‘Largest’.
93. Ans. A.
“have been” should be replaced with “has been”.
94. Ans. A.
The correct sentence should be – “My sister and I are planning a trip from Jaipur to Delhi.”
95. Ans. D.
“Happy” should be replaced with ‘Unhappy.”
96. Ans. D.
The given sentence is error free.
97. Ans. C.
The error is in part C.
“are” should be replaced with ‘is’ as knowledge is considered as singular.
98. Ans. C.
The error is in part C.
‘Were’ should be replaced with ‘was’ as these are abstract noun and singular.

PAGE 485
www.byjusexamprep.com

99. Ans. B.
The error is in Part B.
‘Have’ in the sentence will be removed.
100. Ans. B.
‘Consist’ Should be replaced with ‘Consists’.
101. Ans. A.
The error is in part B.
‘succeeded’ should be replaced with ‘Succeed’
102. Ans. B.
The error is in part B.
“Belong” should be replaced with “belongs”.
103. Ans. A.
The error is in part A.
“Come’ should be replaced with “comes”.
104. Ans. B.
Option B is the correct answer as it is mentioned in the passage that human’s evolution from
primitive stage to the present has been extensive and varied i.e. huge and diversified.
105. Ans. C.
The statement from the passage means that the present times pose much more challenges to
humans than the previous times.”
Hence, option C is correct.
106. Ans. B.
The author says that the labour problems did not exist during the hunting and fishing stage because
there were no economic, political and social systems. Hence, option B is correct.
107. Ans. A.
We understand from the third para that the pastoral age was marked by a certain amount of
economic activity as humans started migrating and held goat herds.” Hence, option A is correct.
108. Ans. B.
‘Emerged’ in the passage means ‘surfaced’.
109. Ans. A.
From the first para we can figure out that land reform proved to be slow because of “the disparity in
power structure.” Hence, option A is the most appropriate answer.

PAGE 486
www.byjusexamprep.com

110. Ans. A.
Option A (1 and 4) “Land ceiling laws have proved to be unsatisfactory.” And “identified land for
distribution has not been distributed due to court cases against it” is correct.
111. Ans. A.
“The lands were unproductive and barren” is the most appropriate answer.
112. Ans. B.
“Litigations” means “lawsuit”
113. Ans. C.
According to the author, agriculture is the primary source of livelihood of majority of India’s
population.
114. Ans. C.
The statement from the passage means that most of them are agriculture labourers.
Hence, option C is the correct answer.
115. Ans. D.
Option D ‘Increased remuneration’ is the correct answer.
116. Ans. A.
Companies feel that it is necessary to maintain anonymity of the employees and to have effective
design and sequence of questions and effective communication.
Hence, option A is correct.
117. Ans. A.
One major benefit of using survey software package is the reports can be generated easily.
Hence, option a is correct.
118. Ans. A.
“Trend” from the passage means “Tendency”.
119. Ans. A.
To maintain the secrecy of the person the screen should not be viewable to supervisors or passers-
by.
120. Ans. A.
‘Adapted’ means customized.
Hence, option A is the most appropriate answer.

PAGE 487
www.byjusexamprep.com

General Knowledge
1. Ans. A.
• The NCC is the largest uniformed youth organization. Its Motto is 'Unity and Discipline'. The NCC
in India was formed the National Cadet Corps Act in 1948.
• Governer of United kingdom was the founder of the NCC.
Hence, Option A is correct.
2. Ans. D.
DEPARTMENTS OF MHA (Ministry of home affairs)
• Department Of Border Management
• Department Of Internal Security
• Department Of Jammu, Kashmir And Ladakh Affairs
• Department Of Home
• Department Of Official Language
• Department Of States
Hence, Option D is correct.
3. Ans. A.
• India is a signatory to the United Nations Convention for Combating Desertification (UNCCD).
• The Ministry of Environment, Forest and Climate Change (MoEFCC) is the nodal Ministry of
Government of India (GoI) that oversees implementation of the Convention in the country.
Hence, Option A is correct.
4. Ans. C.
Under Article 112 of the Constitution, a statement of estimated receipts and expenditure of the
Government of India has to be laid before Parliament in respect of every financial year which runs
from 1st April to 31st March. Annual Financial Statement is the main Budget document and is
commonly referred to as the Budget Statement.
Hence, Option C is correct.
5. Ans. D.
• The South Asian Association for Regional Cooperation (SAARC) was established with the signing
of the SAARC Charter in Dhaka on 8 December 1985.
• SAARC comprises of eight Member States: Afghanistan, Bangladesh, Bhutan, India, Maldives,
Nepal, Pakistan and Sri Lanka.
Hence, Option D is correct.
6. Ans. B.
• The five founding members of the Bank include Brazil, Russia, India, China and South Africa.
• Parent organization: BRICS
• Headquarters: Shanghai, China
Hence, Option B is correct.

PAGE 488
www.byjusexamprep.com

7. Ans. C.
The Public Financial Management System (PFMS) is a web-based online software application
designed, developed, owned and implemented by the Controller General of Accounts.
Hence, Option C is correct.
8. Ans. C.
• The National Institute of Ayurveda (NIA) at Jaipur
• National Institute of Homeopathy was established at Amherst Street, Kolkata and later shifted
to the present location.
• National Institute of Unani Medicine (NIUM) was established in 1984 at Bangalore .
• National Institute of Siddha was established in 2005 at Tambaram, Chennai, India.
Hence, Option C is correct.
9. Ans. C.
• ‘Invest India’ is India’s official agency dedicated to investment promotion and facilitation.
• In early 2010, Invest India is set up as a joint venture company between the Department of
Industrial Policy & Promotion (DIPP), Ministry of Commerce & Industry (35% equity), Federation
of Indian Chambers of Commerce and Industry (FICCI) (51% equity), and State Governments of
India (0.5% each).
Hence, Option C is correct.
10. Ans. C.
• The National Dope Testing Laboratory (NDTL) is a premier analytical testing & research
organization established as an autonomous body under the Ministry of Youth Affairs and Sports,
Government of India.
Hence, Option C is correct.
11. Ans. B.
The 2019 Indian general election was held in seven phases from 11 April to 19 May 2019 to constitute
the 17th Lok Sabha.
End date: 19 May 2019
Start date: 11 April
Hence, Option B is correct.
12. Ans. C.
• Its Headquarters is in Jeddah, Kingdom of Saudi Arabia.
• The Organisation of Islamic Cooperation (OIC) is the second largest inter-governmental
organisation after the United Nations, with the membership of 57 states, covering four
continents
Hence, Option C is correct.

PAGE 489
www.byjusexamprep.com

13. Ans. A.
Karolins pliskova won the title, defeating Johanna Konta in the final by 6–3, 6–4.
Hence, Option A is correct.
14. Ans. B.
• India, Singapore have completed SIMBEX 2019 Naval Exercises in South China Sea
• INS Kolkata, the lead ship of the Kolkata-class guided missile destroyers, and INS Shakti, a
Deepak-class fleet tanker, participated in the exercise.
Hence, Option B is correct.
15. Ans. D.
The Badminton World Federation (BWF) has launched two new formats of badminton game namely,
AirBadminton and Triples. In contrast to traditional competitive badminton which is an indoor game
the Airbadminton is an outdoor game.
Hence, Option D is correct.
16. Ans. B.
The Reserve Bank of India (RBI) announced that it was constituting of a ‘high-level committee for
deepening of digital payments’. Former UIDAI chairman and Infosys Co-founder Nandan Nilekani as
the Chairman of the five-member committee.
Hence, Option B is correct.
17. Ans. A.
• The United Nations Sasakawa Award is the most prestigious international award in the area of
Disaster Risk Management.
• 2019 Sasakawa award was “Building Inclusive and Resilient Societies”. UNDRR received more
than 61 nominations from 31 countries for the 2019 award.
Option A is correct.
18. Ans. B.
• India’s G.S. Lakshmi will become the first woman match referee to officiate at a global ICC event
when the Women’s T20 World Cup starts in Australia from February 21.
Option B is correct
19. Ans. A.
• The President of the Republic of Indonesia (Indonesian: Presiden Republik Indonesia) is the head
of state and also head of government of the Republic of Indonesia. The president leads the
executive branch of the Indonesian government and is the commander-in-chief of the
Indonesian National Armed Forces. Since 2004, the president and vice president are directly
elected to a five-year term.
• Joko Widodo is the 7th and current president of Indonesia. He assumed office on 20 October
2014.
Hence, Option A is correct.

PAGE 490
www.byjusexamprep.com

20. Ans. D.
"Every year May 21 is observed as Anti-Terrorism day in India to wean away the youth from terrorism
and showing as to how it is prejudicial to the national interest,"
Hence, Option D is correct.
21. Ans. A.
• During Lord Ripon's time, the first Factories Act was adopted in 1881.
• In British India, the Vernacular Press Act was proposed by Lord Lytton, then Viceroy of India,
and was unanimously passed by the Viceroy's Council on 14 March 1878.
• The Indian Councils Act 1909, commonly known as the Morley-Minto or Minto-Morley Reforms.
• The Cornwallis Code is a body of legislation was enacted in 1793.
Hence, Option A is correct.
22. Ans. A.
• Article 371A – Nagaland
• Article 371A of the Constitution mainly states that no act of Parliament would apply to the state
of Nagaland in matter relating to religious or social practices of Nagas, Naga customary law and
procedure, administration of civil or criminal justice involving decisions according to Naga
customary law and ownership and transfer of land and its resources.
Hence, Option A is correct.
23. Ans. D.
• The idea of creation of Zonal Councils was mooted by the first Prime Minister of India, Pandit
Jawahar Lal Nehru in 1956.
• In the light of the vision of Pandit Nehru, five Zonal Councils were set up vide Part-III of the
States Re-organisation Act, 1956.
Hence, Option D is correct.
24. Ans. C.
• Article 361 is an exception to Article 14 (Right to Equality) of the Indian Constitution.
• The features are as follows:
• The President or the Governor is not answerable to any court for the exercise of the powers and
duties of his office.
• No criminal proceedings shall be conducted against the President or the Governor during his
term of office.
• No arrest or imprisonment shall be made against the President or Governor during his term of
office.
Hence, Option C is correct.
25. Ans. B.
Article 348 deals with the language to be used in the Supreme Court and in the High Courts and for
Acts, Bills, etc.
the authoritative texts:
I. of all Bills to be introduced or amendments thereto to be moved in either House of Parliament
or in the House or either House of the Legislature of a State,

PAGE 491
www.byjusexamprep.com

II. of all Acts passed by Parliament or the Legislature of a State and of all Ordinances promulgated
by the President or the Governor of a State, and
III. of all orders, rules, regulations and bye laws issued under this Constitution or under any law made
by Parliament or the Legislature of a State, shall be in the English language.
Hence, Option B is correct.
26. Ans. D.
According to the Constitution of India, the total number of ministers in the council of ministers must
not exceed 15% of the total number of members of the Lok Sabha. Ministers must be members of
parliament. Any minister who is not a member of either of the houses of the parliament for six
consecutive months is automatically stripped off his or her ministerial post.
Hence, Option D is correct.
27. Ans. B.
• The Mediterranean climate is characterized by dry and hot summer and cold and rainy winter.
• The regions are located in the western parts of the continent between 30 and 45 degrees north
and south of the equator.
Hence, Option B is correct.
28. Ans. D.
Brahmaputra river takes a ‘U' turn at Namcha Barwa and then continues to flow in Arunachal Pradesh
and after that, it is called as "Dihang river or Siang river".
Hence, Option D is correct.
29. Ans. D.
• The Dutt-Bradley Thesis was published in International Press Correspondence (INPRECOR), the
journal of Communist International, on February 29, 1936.
• The Dutt-Bradley Theses greatly helped the communist movement. It is through the interventions
of the communists during this period that a lot of cadre and new generation of leaders who
contributed immensely in building and developing the communist movement in the country,
emerged.
Hence, Option D is correct.
30. Ans. C.
The Chota Nagpur Tenancy Act, 1908
• the Chhota Nagpur Tenancy Act, 1908 is an important act for the tribal population of Jharkhand.
It restricts transfer of tribal land to non tribals.
• Mundari Khuntkatti is a customary institution found among Mundas of Chotanagpur which
provides ownership of land among all the families of the same killi (clan), who cleared the forest
and made land cultivable.
Hence, Option C is correct.

PAGE 492
www.byjusexamprep.com

31. Ans. A.
Plagues and People is a book on epidemiological history by William H. McNeill published in New york
in 1976.
Hence, Option A is correct.
32. Ans. C.
Lord Minto started the Indian Agriculture Service in India.
Hence, Option C is correct.
33. Ans. C.
The Chandimangala is an important subgenre of Mangalkavya, the most significant genre of medieval
literature in Bengali.
The texts shows primarily some folk goddesses like Chandi and Abhaya.
Hence, Option C is correct.
34. Ans. A.
Sino Indian War also known as Indo china war between china and India that occurred in 1962. Soviet
leader Khrushchev sided with Nehru and held china for the responsible of Sino Indian war of 1962.
Hence, Option A is correct.
35. Ans. D.
Make in India is a kind of swadeshi movement covering 25 sectors of Indian economy launched on
25 September 2014 to encourage companies to manufacture their products in India and investment
in to manufacturing.
Hence, Option D is correct.
36. Ans. D.
The two houses in states called legislative assembly and legislative council. There are only seven
states which have two tier of legislature- Uttar Pradesh, Bihar, Maharashtra, Karnataka, Andhra
Pradesh, Telangana and Jammu Kashmir.
Hence, Option D is correct.
37. Ans. A.
SWAYAM stands for study webs of active learning for young aspiring minds. It comes under ministry
of human resource development. This program initiated by government of India and designed to
achieve the three cardinal principles of education policy namely access, equity and quality.
Hence, Option A is correct.
38. Ans. D.
The fundamental duties in the Indian constitution under article 51 A inspired by the constitution of
erstwhile USSR. There is a list of duties defined in Indian constitution for every citizens of India. 'To
promote international peace and security' is not a fundamental duty
Hence, Option D is correct.

PAGE 493
www.byjusexamprep.com

39. Ans. A.
The managerial revolution was written by James Burnham and published in 1941, speculated on the
fate of capitalism.
Hence, Option A is correct.
40. Ans. D.
Quo Warranto means “by what authority or warrant. It is issued by the court to enquire into the
legality of claim of a person to celcius office. The writ can be issued only in the case of substantive
public office created by a statute or by the constitution. It cannot be issued in the case of ministerial
office or private office.
Hence, Option D is correct.
41. Ans. A.
The total amount of money available for an individual or population to spend or save after taxes have
been paid called personal disposable income. To calculate personal disposable income we have to
subtract taxes from personal income.
Hence, Option A is correct.
42. Ans. A.
The price mechanism is the system of interaction of buyers and sellers in free markets enables goods,
resources and services to be in the allocated price. This system where the forces of demand and
supply determine the prices of commodity and the changes in it. It is the buyers and sellers who
actually determine the prices of commodities.
Hence, Option A is correct.
43. Ans. A.
Indexation is a technique to adjust income payments by means of price index in order to maintain
the purchasing power of the public after inflation, while the de-indexation is the unwinding of
indexation.
Hence, Option A is correct.
44. Ans. C.
Uniform circular motion can be described as a motion of an object in a circle at a constant speed. As
an object moves in a circle it continuously changes its direction. The acceleration of the car is because
of changing directions. The direction of acceleration inwards. So the acceleration is non-zero quantity
and because of change in velocity[changing direction] it is not a constant quantity.
Hence, Option C is correct.
45. Ans. B.
distance
Speed =
time
distance
340 =
5
Distance =340x5=1700meters
1700
So, the distance of the reflecting surface from source is = 850 meters=0.85kms
2
Hence, Option B is correct.

PAGE 494
www.byjusexamprep.com

46. Ans. C.
1 unit= 1 KWh (one kilo watt hour)
Where kilo=1000
It simply means if you have 100 watt bulb and you make on it for 10 hour it consume 1 unit of power.
100 watt bulb x 10hours=100 watt hour= 1 KWh=1 unit
According to the question used for 30 days then it will be 30 unit.
Hence, Option C is correct.
47. Ans. A.
The fifth schedule of the constitution deals with the administration and control of scheduled areas
as well as scheduled tribes residing in any state other then Assam, Meghalaya, Tripura, Mizoram. The
president is empower to declare an area to be scheduled area and he can increase or decrease an
area.
Hence, Option A is correct.
48. Ans. C.
Article 3 authorize the parliament to –
• Form a new state by separation of territory from any state or by uniting two or more states or
parts of states.
• Increase the area of the any state.
• Diminish the area of any state.
• Alter the boundary of any state.
• Alter the name of every state.
Hence, Option C is correct.
49. Ans. D.
• Article 165 has provided the office of advocate general for the state. He is the highest law officer
in the state.
• He is appointed by the governor. He must be a person who is qualified to be appointed a judge
of a high court.
• High court enjoys jurisdiction in original, appellate and supervisory at the state level as provided
in the constitution.
Hence, Option D is correct.
50. Ans. C.
• Unitary constitution governs constitutionally as one single unit with one legislature in center.
• Federal constitution is where powers are divided between national government and regional
government.
• Quasi-Federal refers to government organize similar to union of states under central
government rather then the individual government of the states.
Hence, Option C is correct.

PAGE 495
www.byjusexamprep.com

51. Ans. B.
• Bihar bordered by Nepal, Uttar-Pradesh, Jharkhand and west Bengal.
• Uttarakhand bordered by Tibet, Uttar-Pradesh, Nepal, Haryana and Himachal-Pradesh.
• Meghalaya bordered by Bangladesh, Assam.
• Chhattisgarh bordered by Madhya-Pradesh, Uttar-Pradesh, Jharkhand, Telangana, Maharashtra
and Odisha.
Hence, Option B is correct.
52. Ans. C.
• Agra is the city in the state of Uttar-Pradesh which situates in the bank of river Yamuna.
• Bhagalpur is the city in the state of Bihar situates on the bank river Ganges.
• Kanpur is the city in the state of Uttar-Pradesh and is on the banks of river Ganges.
• Bhopal consist Bhoj lake and is comes under the states of Madhya-Pradesh.
Hence, Option C is correct.
53. Ans. A.
Jhumri Telaiya is a city in Jharkhand situated in the Damodar valley.
Mandar hills is situated in Banka district under Bhagalpur division of state of Bihar.
Hence, Option A is correct.
54. Ans. D.
The South-Indian region has a tropical climate and depends on monsoon and rainfall. It is
characterized by moderate to high year-around temperature and seasonal heavy rainfall.
The diurnal range of temperature is also less and climatic conditions are more likely stable.
Hence, Option D is correct.
55. Ans. D.
The sex ratio is the ratio of males to females in a population usually expressed as the number of
males per thousand females. In India sex ratio is defined as number of females per thousand males.
The sex ratio for the entire world population is 101 males 100 females.
Hence, Option D is correct.
56. Ans. B.
Human development index is the static composite index of life expectancy, education and per capita
income. It was developed by Pakistani economist Mahbub-Ul-Haq.
Hence, Option B is correct.
57. Ans. B.
Citrus fruits are the highest value fruit crops in-terms of International trade. It grows in warm climate
such as Florida. Mediterranean regions are the important supplier of Citrus fruits. This regions enjoys
the mild and moist winters hence the crops not get spoiled.
Hence, Option B is correct.

PAGE 496
www.byjusexamprep.com

58. Ans. C.
The Nayanars were the group of 63 saints in the 6-8th century who are devoted to the Hindu god
Shiva in Tamil Nadu.
Hence, Option C is correct.
59. Ans. C.
Maruta Makkal were the tribes of Ploughmen. Inhabiting fertile, Well watered tracts and living in
villages called ur.
Kuravan Makkal were hill people who are foresters, and tell fortunes to other people.
Mullai Makkal also called Ayar were pastoralists.
Neytal Makkal or fishing people living in large coastal village called Pattinam or small ones called
Pakkam.
Hence, Option C is correct.
60. Ans. A.
Babur was the follower of Naqshbandiyya leader, Lhwaja Ubaydullah Ahrar.
Hence, Option A is correct.
61. Ans. B.
The Government of India Act 1919 was passed to expand participation of Indians in the Government
of India. The Act embodied the reforms recommended in the report of the “Secretary of State for
India, Edwin Montagu, and the Viceroy, Lord Chelmsford”. The Act covered ten years, from 1919 to
1929. This Act represented the end of benevolent despotism and began the genesis of responsible
government in India.
Hence, Option B is correct.
62. Ans. B.
The concept of “Four Pillar State”, free from district magistracy for India was suggested by Ram
Manohar Lohia. He was an activist in the Indian Independence Movement. Lohia believes in
decentralization of economic and political powers. For giving a solution to the malady of Indian
administration, he gives the concept of Four- Pillar.
Hence, Option B is correct.
63. Ans. C.
Participation of workers in management of industries is not a part of the Fundamental Rights (Part
III) of the Constitution of India.
There are six fundamental rights have been recognized by the Indian constitution:
Right to equality (Articles 14-18)
Right to Freedom (Articles 19-22)
Right Against exploitation (Articles 23-24)

PAGE 497
www.byjusexamprep.com

Right to Freedom of Religion (Articles 25- 28)


Cultural and Educational Rights (Articles 29-30)
Right to Constitutional Remedies (Articles. 32-35)
Hence, Option C is correct.
64. Ans. D.
A growing period of at least 100 frost free days is not a geographical requirement for cultivation of
cotton.
Climate: Cotton grows well in warm and moist climate, where summer is long and where there is
salinity in the Soil.
Temperature: Cotton grown well in a temperature of 24°C. High temperature is injurious.
Rainfall: 60-100 cm rainfall is essential for the cultivation of cotton.
Hence, Option D is correct.
65. Ans. C.
The average temperature of Coniferous Forest Biome is 50 degrees Fahrenheit. The average winter
temperature is -40°C to 20°C, while the average summer temperature is 10°C. This biome
experiences long, snowy winter months and hot wet summer months.
Hence, Option C is correct.
66. Ans. C.
Anaimudi Peak is related to Annamali hill (in Tamilnadu and Kerala).
Doddabetta Peak is related to Nilgiri Hill (Western ghat in Western Tamilnadu).
Dhupgrh Peak is related to Satpura Hill (in Madhya Pradesh).
Guru Shikhar is related to Satpura Hill (in the Arbuda mountains of Rajasthan).
Hence, Option C is correct.
67. Ans. C.
Coral reefs are not found in Gulf of Mannar regions.
Coral reefs flourish in ocean waters that provide few nutrients. they are most commonly found at
shallow depths in tropical waters, but not in deep water and cold water.
Hence, Option C is correct.
68. Ans. D.
Andhra Pradesh States in jute not significantly cultivated.
The jute crop is grown in nearly 83 districts of seven states - West Bengal, Assam, Orissa, Bihar, Uttar
Pradesh, Tripura and Meghalaya. West Bengal alone accounts for over 50 percent raw jute
production and the largest producer of the Jute in India. 60% to the total world production of Jute is
cultivated in India.
Hence, Option D is correct.

PAGE 498
www.byjusexamprep.com

69. Ans. C.
A story in Divyavadana attributes Ashoka's conversion to the Buddhist monk Samudra, who was an
ex-merchant from Shravasti. According to this account, Samudra was imprisoned in Ashoka's "Hell",
but saved himself using his miraculous powers. When Ashoka heard about this, he visited the monk,
and was further impressed by a series of miracles performed by the monk. He then became a
Buddhist.
In the Ashokavadana states that Samudra was a merchant's son, and was a 12-year-old boy when he
met Ashoka; Mahavamsa and Dipavamsa seems to be influenced by the Nigrodha story.
Hence, Option C is correct.
70. Ans. C.
Arikamedu is the site that gives us valuable information about India’s maritime links on the
Coromandel coast.
The coastline forms a part of Tamil Nadu and Andhra Pradesh. The important ports include Chennai,
Thoothukkudi, Nellore, Ennore and Nagapattinam, which take advantage of their close proximity
with regions rich in Natural and Mineral resources and good Transport Infrastructure.
Hence, Option C is correct.
71. Ans. A.
The largest quantity of cichlids found in India at Backwaters of Kerala.
Lake Victoria is famous for its enormous diversity of endemic cichlid fish, which although never fully
counted probably once numbered around 450 species.
Every different species of cichlid is only found in Africa and in very specific places. The cichlid is native
to the Victoria, Malawi, and Edward Lakes of eastern Africa.
Hence, Option A is correct.
72. Ans. C.
The first Greek scholar Eratosthenes is credited with the first use of the word Geography in the third
century B.C.E He is also known as the "father of Geography" for his geographical writing and
accomplishments, including the measurement of the circumference of the Earth.
Geography~ the study of the World’s surface, physical qualities, Climate, Population, Products, etc.
Hence, Option C is correct.
73. Ans. B.
the author of the 16 th century Sanskrit text, the Vraja Bhakti Vilasa which focuses on the Braj region
in North India is Narayana Bhatta.
Braj region is also known as Brajbhoomi. It is a region in India on both sides of Yamuna river with its
centre at Mathura-Vrindavan in Uttar-Pradesh. It also includes Hodal in Haryana and Bharatpur
district in Rajasthan.
Hence, Option B is correct.

PAGE 499
www.byjusexamprep.com

74. Ans. B.
Bose - Einstein Condensate is fifth state of Matter.
Bose-Einstein condensate (BEC), a state of matter in which separate atoms or subatomic particles,
Cooled to near Absolute Zero (0 K, − 273.15 °C, or − 459.67 °F; K = kelvin), Coalesce into a single
Quantum mechanical entity—that is, one that can be described by a Wave function — on a near-
macroscopic scale.
Hence, Option B is correct.
75. Ans. C.
The rate of evaporation of liquid does not depend upon its mass.
The rate of evaporation does not depend on the type of Particles because it completely depends
upon the surface area of the liquid, if surface area is large Evaporation will be maximum. Evaporation
also depends upon wind speed, if the Wind is flowing at a higher speed, then the rate of Evaporation
increases.
Hence, Option C is correct.
76. Ans. C.
Rutherford’s alpha particle scattering experiment on thin gold foil was responsible for the discovery
of Proton.
Rutherford's Gold Foil Experiment proved the Existence of a small massive center to atoms, which
would later be known as the nucleus of an atom. Ernest Rutherford, Hans Geiger and Ernest Marsden
carried out their Gold Foil Experiment to observe the effect of Alpha particles on matter.
Hence, Option C is correct.
77. Ans. C.
Food chain is passage of food (and thus energy) from one organism to another.
Food chain, in ecology, the sequence of transfers of Matter and Energy in the form of food from
Organism to Organism. Food chains intertwine locally into a food web because most Organisms
consume more than one type of Animal or Plant. Plants, which convert Solar Energy to food by
Photosynthesis, are the primary food source.
Hence, Option C is correct.
78. Ans. A.
Active transport: It is the movement of a substance against a diffusion gradient with the use of energy
from respiration.
In cellular biology, active transport is the movement of Molecules across a membrane from a region
of lower Concentration to a region of higher Concentration—against the Concentration Gradient.
Active transport requires Cellular energy to achieve this movement.
Hence, Option A is correct.
79. Ans. B.
Chlorophy - II in photosynthetic prokaryotic bacteria is associated with membranous vesicles.

PAGE 500
www.byjusexamprep.com

They contain a pigment called bacteriochlorophyll. This bacteriochlorophyll is not a cell organelle but
is a pigment found in the folds of the cell membranes also called vesicles.
Hence, Option B is correct.
80. Ans. A.
Demographic Dividend’- A rise in the rate of economic growth due to a higher share of working age
people in population.
According to United Nations Population Fund (UNFPA), Demographic Dividend means, "the
Economic growth potential that can result from shifts in a Population’s age structure, mainly when
the share of the Working-Age population (15 to 64) is larger than the Non-Working-age share of the
population (14 and younger, and 65 and older.
India has one of the youngest Populations in an aging world. By 2020, the median age in India will be
just 28, compared to 37 in China and the US, 45 in Western Europe, and 49 in Japan.
Hence, Option A is correct.
81. Ans. B.
Trypanosoma organisms is responsible for sleeping sickness.
African trypanosomiasis, also known as African sleeping sickness or simply sleeping sickness, is an
insect-borne Parasitic infection of Humans and other Animals. It is caused by the species
Trypanosoma Brucei. Humans are infected by two types, Trypanosoma brucei Gambiense (TbG) and
Trypanosoma brucei Rhodesiense (TbR).
Hence, Option B is correct.
82. Ans. D.
Biceps is one of the body parts/organs of the human body does not have smooth Muscles.
The term smooth muscle refers to a muscle of the Human body that is part of an Involuntary muscle
group.
Hence, Option D is correct.
83. Ans. C.
Inter-Cropping - It is growing of two or more crops in definite row patterns.
In other words, intercropping is the Cultivation of two or more crops simultaneously on the same
field. The most common goal of Intercropping is to produce a greater yield on a given piece of land
by making use of resources or Ecological processes that would otherwise not be utilized by a single
Crop.
Hence, Option C is correct.
84. Ans. B.
Magnification is observed size of specimen/actual size. Magnification is the process of enlarging the
apparent size, not physical size, of something. This enlargement is quantified by a calculated number
also called "Magnification". When this number is less than one, it refers to a reduction in size,
sometimes called Minification or De-Magnification.
Hence, Option B is correct.

PAGE 501
www.byjusexamprep.com

85. Ans. A.
Lysosomes are the cells which are known as suicidal-bags and they produce Golgi-body.
It is knowns as suicidal bags because it is containing a large number of enzymes which can digest
foreign particles that enter the cell, and this process is known as autolysis.
Hence, Option A is correct.
86. Ans. C.
An Economic-Model is a simplified version of reality that allows us to observe, understand, and make
predictions about Economic behavior.
The Economic model is a simplified, often Mathematical, framework designed to illustrate complex
Processes.
Hence, Option C is correct.
87. Ans. B.
The value of the slope of a normal demand curve is Negative. As the Price decreases, while the
Quantity increases, the slope of Demand curve is usually negative. It is to be noted that in the case
of a straight-line demand curve the slope is the same on all its points.
There is no specific value of it, the demand curve is downward sloping demonstrating the negative
relationship with Price.
Hence, Option B is correct.
88. Ans. D.
Maximum Retail Price (MRP) printed on the covers/packets of goods sold in India is one of the
following is an example of a price floor.
A Price floor in economics is a minimum price imposed by a Government or Agency, for a particular
product or service.
An example of a price floor is minimum wage laws, where the government sets out the minimum
hourly rate that can be paid for labour, When the minimum wage is set above the Equilibrium Market
price for unskilled or low-skilled Labour, Employers hire fewer Workers.
Hence, Option D is correct.
89. Ans. C.
Cost of living index is Which one of the following factors is not considered in determining the
Minimum Support Price (MSP) in India.
Minimum Support Price is Calculated by Cost of production. Changes in input prices. Input-Output
price difference. Trends in market prices over a period of time (3-6 months) Demand and Supply of
Commodities. Inter-crop price rates. Effect on the Industrial cost structure. Effect on cost of living of
the Population.
Hence, Option C is correct.
90. Ans. C.
Access to banking and other financial provisions one of the following is not a dimension of the Human
Development Index.

PAGE 502
www.byjusexamprep.com

The Human Development Index (HDI) is a Statistical tool used to measure a Country's overall
achievement in its Social and Economic.
Hence, Option C is correct.
91. Ans. C.
In economics, the Gini coefficient is sometimes known as the Gini index, or Gini ratio is a measure of
statistical dispersion intended to represent the income or wealth distribution of a nation's residents
and is most commonly used to measure the income inequality. It was developed by the Italian
statistician and sociologist Corrado Gini and published in his 1912 paper Variability and Mutability.
Hence, Option C is correct.
92. Ans. C.
The essential characteristics of particles of the Matter are the following:
• The particles of the Matter are microscopic.
• The particles of the Matter have space between them.
• Particles of the Matter intermix on their own.
• The particles of the Matter are continually moving.
• Particles of the Matter have a force acting between them
Hence, Option C is correct.
93. Ans. A.
The rate of evaporation is affected by the following factors:
• Temperature: The rate of evaporation increases with an increase in temperature.
• Surface area: The rate of evaporation increases with an increase in surface area.
• Humidity: The amount of water vapour present in the air is called humidity. The rate of
evaporation decreases with an increase in humidity.
• Wind speed: Evaporation increases with an increase in wind speed.
Hence, Option A is correct.
94. Ans. B.
If an object is at rest, then the time (X-axis) versus distance (Y-axis), then the graph is horizontal.
All motion represented on a distance-time graph is unidirectional since direction cannot be
represented by a distance-time graph. If the line is a continuous straight line, then the object has a
uniform speed. If the distance remains constant over time, represented by a straight line parallel to
the x-axis, then the object is at rest at the given distance, i.e. If an object is not moving, the distance-
time graph results in a horizontal line which shows that the object is at rest.
Hence, Option B is correct.
95. Ans. B.
Mixtures are constituted by more than one kind of pure form of Matter, known as a substance. A
substance cannot be separated into other kinds of Matter by any physical process. We know that
dissolved sodium chloride can be separated from water by the physical process of evaporation.
A substance cannot be separated into other kinds of Matter by any physical process. Whatever the
source of a substance may be, it will always have the same characteristic properties. Therefore, we
can say that a mixture contains more than one substance.

PAGE 503
www.byjusexamprep.com

However, sodium chloride is itself a substance and cannot be separated by the physical process into
its chemical constituents.
Hence, Option B is correct.
96. Ans. B.
Isobars: They are atoms of chemical elements having same atomic mass, but a different atomic
number are called Isobars, i.e. the sum of the number of protons and neutrons together from the
atomic mass. Therefore, we can also say the number of nucleons present in the nucleus is equal to
the atomic mass of an atom. It will have the same number of nucleons.
Hence, Option B is correct.
97. Ans. A.
If the speed of a moving magnet inside a coil increases, the electric current in the coil increases. As
the changing magnetic field caused by the material's motion induces a current in the coil of wire
proportional to the change in the field. If a 0 is represented, the magnetic field does not change
between the two domains of a bit, so no current is induced as the magnetic material passes the coil.
Hence, Option A is correct.
98. Ans. C.
An octave can be defined as an interval between two points where the frequency at the second point
is twice the frequency of the first.
The frequency that is one octave higher than 500 Hz is

Thus, the frequency of a note that is one octave higher than 500 Hz is 1000 Hz.
Hence, Option C is correct.
99. Ans. D.
By Article 148, Comptroller and Auditor-General of India. There shall be a Comptroller and Auditor-
General of India who shall be appointed by the President by warrant under his hand and seal and
shall only be removed from office in like manner and on like grounds as a Judge of the Supreme
Court.
The tenure of a High Court judges is not fixed by the Constitution, but they hold office until they
attain the age of 62 years. A judge of the High Court can resign his office by writing to the President
any time after the appointment.
Hence, Option D is correct.

PAGE 504
www.byjusexamprep.com

100. Ans. D.
The Constitution of India established federal State due to which the Indian Parliament is bicameral
in nature which ensures representation of states equally. The Rajya Sabha has been given special
power by the Constitution.
Hence, Option D is correct.
101. Ans. A.
Soil impoverishment actually means a number of conditions that can lead to degradation in the
quality of soil, or degradation in its richness. It relates to soil erosion. This might be caused due to
reckless cutting of trees, leading to deforestation of trees. We know that trees help to bind the soil
to the earth, and hence once these are uprooted, the richness gets diminished. Also, land should not
be used carelessly for other activities that can harm its fertility.
Hence, Option A is correct.
102. Ans. C.
In 1916, Frederic Clements published a descriptive theory of succession and advanced it as a general
ecological concept. His theory of succession had a powerful influence on ecological thought.
Clements' concept is usually termed classical ecological theory. According to Clements, succession is
a process involving several phases.
• Nudation: Succession begins with the development of a bare site, called Nudation (disturbance).
• Migration: refers to the arrival of propagules.
• Ecesis involves the establishment and initial growth of vegetation.
• Competition: as the vegetation becomes well established, grows, and spreads, various species
begin to compete for space, light and nutrients.
• Reaction: during this phase, autogenic changes such as the buildup of humus affect the habitat,
and one plant community replaces another.
• Stabilization: a supposedly stable climax community forms.
Hence, Option C is correct.
103. Ans. B.
Entisols are defined as soils that do not show any profile development other than an A horizon.
Histosols have very low bulk density and are poorly drained because they are rich in organic Matter
and holds the water very well.
Oxisols are very old & highly weathered, uniform texture, high amounts of Fe and Al oxides.
Hence, option B is correct.
104. Ans. B.
The great historic barrier of the Caucasus Mountains rises up across the wide isthmus separating the
Black and Caspian Seas in the region where Europe and Asia converge. The great historic barrier of
the Caucasus Mountains rises up across the wide isthmus separating the Black and Caspian seas in
the region where Europe and Asia converge.
Hence, Option B is correct.

PAGE 505
www.byjusexamprep.com

105. Ans. B.
Due to the apparent northward movement of the Sun temperature start rising from March, toward
the end of the May a low-pressure trough is developed which extends from the Thar Desert in the
northwest to the Chotanagpur plateau in the east. In the heart of this low pressure through in the
North West, dry and hot winds blow in the afternoon. These hot and dry winds are locally known as
Loo. Dust storms in the evening very common during May in the northwestern part of the country.
Sometimes they are accompanied by light rains and pleas and cool breeze which give temporary
relief from the oppressive heat.
Hence, option B is correct.
106. Ans. D.
Wular Lake is the largest freshwater lake in India.
This lake is situated in the Kashmir and 40 km northwest from the Srinagar city of India. The depth
of the lake is 14 meters, and the size of the lake is 189 sq.km, and the length of the river is 16 km,
and its breadth is 10 km. Wular lake is very important to play a role in the hydrographic process of
the Kashmir to absorb the annual freshwater as the basin.
Hence, option D is correct.
107. Ans. A.
The Greek travellers were most impressed by the fertility of India's soil and the energy and ability of
the cultivators. The Greeks found it a great source of wonder that India produced two crops a year.
Ancient India knew the use of manure. The ordinary villagers of Ancient days cultivated the land
much as they do then, ploughing with shallow wooden ploughs drawn by oxen, verified by Harappan
discoveries.
Hence, Option A is correct.
108. Ans. B.
Non-Cooperation began in Punjab with a fairly successful Lahore student walk-out inspired by Lajpat
Rai in January 1921, but the movement in the cities at that time seems to have remained relatively
weak unlike in April 1919; perhaps precisely because memories of ruthless British reprisals at that
time were still very fresh.
The Sikh-dominated central Punjab countryside was stirred to its depths, however, by the powerful
Akali upsurge, initially quite an independent religious reform movement which for a time got closely
identified with Non-Cooperation.
Hence, Option B is correct.
109. Ans. D.
The Oudh Kisan Sabha established in 1920 did not fail to bring under is wing any Kisan Sabhas.
Through the efforts of Misra, Jawaharlal Nehru, Mata Badal Pande, Baba Ramchandra, Deo Narayan
Pande and Kedar Nath, the new organization brought under its wing, by the end of October, over
330 Kisan sabhas.
The Oudh Kisan Sabha asked the Kisans to refuse to till the bedakhli land, not to offer hari and beggar
(forms of unpaid labour), to boycott those who did not accept these conditions and to solve their
disputes through panchayats.
Hence, Option D is correct.

PAGE 506
www.byjusexamprep.com

110. Ans. B.
The United Provinces during the Non-Cooperation became one of the strongest bases of the
Congress, with 3,28,966 members in July 1921 (a figure exceeded only by Bihar which claimed
3,50,000) and U.P. won from this time a leading position in national politics which it has retained till
today.
The literary outcrop of Non-Cooperation in Bengal was quite meagre if compared to that of the 1905
days, and nationalists in the 1920s and 1930s had to make do very often with the old Swadeshi songs.
Hence, Option B is correct.
111. Ans. A.
The Alvars were Tamil poet-saints of South India who espoused bhakti (devotion) to the Hindu
Supreme god Vishnu or his avatar Krishna in their songs of longing, ecstasy and service. They are
venerated especially in Vaishnavism, which regards Vishnu or Krishna as the Supreme Being.
Hence, option A is correct.
112. Ans. D.
Monatomic compounds are composed of single atoms, and there are no chemical bonds between
these atoms. Diatomic compounds are composed of molecules containing two atoms. Therefore,
there are chemical bonds between these atoms.
• Helium – monatomic
• hydrogen – diatomic
• Sulphur – diatomic
• phosphorus – tetratomic
Hence, Option D is correct.
113. Ans. B.
Graphite is an allotrope of carbon which is a semimetal and is also a form of coal. Graphite is the
most the stable form of carbon under standard conditions.
In Graphite, each carbon atom is bonded to three other carbon atoms in the same plane, giving a
hexagonal array. One of these bonds is a double bond, and hence the valency of the carbon is
satisfied.
Hence, option B is correct.
114. Ans. B.
When soap is mixed in the water, a colloidal solution is formed. The soap solution has soap micelles
which are an aggregate of soap molecules. These micelles are large, and they scatter light. That is
why the soap solution appears cloudy. Soap is mixed with water to form micelle, which gives the
solution a cloudy appearance when soap is added to the water the fatty acid ionizes. The polar water-
soluble gets attracted to water while the non-polar tails attract each other. Then the ions are
attracted to each other and form the special clusters and sometimes in the form of emulsions which
gives a cloudy look.
Hence, Option B is correct.

PAGE 507
www.byjusexamprep.com

115. Ans. A.
People prefer to wear cotton clothes in the summer season. This is due to the fact that cotton clothes
are good absorbers of water/sweat. It also protects the body from harmful sun rays.
Hence, Option A is correct.
116. Ans. A.
Chromatography is a technique used to separate components in a mixture on the basis of them being
mixed as mobile phase, i.e. liquid or gas and stationary phase, i.e. solid or liquid.
Hence, Option A is correct.
117. Ans. B.
The scientist who first of all, showed that the atomic number is a more fundamental property of an
element than its atomic mass was Henry Moseley. He declared this concept in the Year, 1913. He is
best known for the Moseley's Law and contributions to Physics. Moseley is also associated with the
publication of the very first Long Form or Modern periodic table that is used to date.
Moseley performed experiments and studied the frequencies of the rays emitted from the elements.
With these experiments, he concluded that the atomic number is a more fundamental property of
an element than its atomic mass.
Hence, Option B is correct.
118. Ans. B.
Vitamin C, also known as ascorbic acid, is a vitamin. It is found fresh fruits, berries and vegetables. It
is one of the water-soluble vitamins. Vitamin C was first discovered in 1928. In 1932, it was proved
to stop the sickness called scurvy.
Hence, Option B is correct.
119. Ans. D.
Animal cells do not have cell walls. Cell walls are found in plant cells. Other given organelles like
Mitochondria, Nucleolus and Free ribosomes are present in animal cells.
The cell parts found only in plants but not in animals are:
• Cell wall
• Large central vacuole
• Chloroplasts (plastids)
Hence, Option D is correct.
120. Ans. A.
Pteridophytes are the plant body which is differentiated into roots, stem and leaves and has
specialized tissue for the conduction of water and other substances from one part of the plant body
to another. In fact, they can be considered as the first terrestrial vascular plants, showing the
presence of the vascular tissue, xylem, and phloem. Mostly, we find these plants in damp and shady
places. Also, most ferns are grown as ornamental plants. Some examples are Marsilea, ferns,
horsetails, Azolla, Ptreium and Salvinia.
Hence, Option A is correct.

PAGE 508
www.byjusexamprep.com

Elementary Mathematics
1. Ans. C.
We have,

y is inversely proportional to x .

Means

Given that then .

So if then,

2. Ans. B.
We have, total amount = 500 rs
We want to buy some apples and some orange using total amount.
One apple rate is 5rs and one orange rate is 7 rs.
Let apples and orange buy by a person.

Then
Now if we vary values of . Then for proper value of means not fraction we have to put y in term
of multiples of 5.
So least value of is 5.

Then

And maximum value of is .

PAGE 509
www.byjusexamprep.com

Then,

So here between 5 and 70 , no of terms which is multiple of 5.we use formula of AP series.

Then there is 14 ways to buy apple and oranges.


3. Ans. A.
We have, Radha and Rani are sisters.
Let present age of Radha is and present age of Rani is .

Before 5 years,

……..(i)

Before 1 year,

……..(ii)
Solving equation (i) and (ii), we get

Then age difference between Radha and Rani is 8.


4. Ans. A.
(i) We have is relatively prime to and .

Means there is nothing common between and .

Example
Then if we product the and means .
So, means 2 has no common factor with 35.
Means is relative prime to the product .
Then statement-1 is correct.
(ii) if divide the means divides by or divides by or both divides with .
But there is not must condition that if divides then must divides .
So, statement-2 is wrong.

PAGE 510
www.byjusexamprep.com

5. Ans. C.
We have a real number is .

And, reciprocal of this number is .

So,

So, there are two types of such real numbers.


6. Ans. C.
We have,
Six cubes with same edge 12 cm are joint end to end.

Then total length of big cuboid = cm

and breadth and height is same 12 cm.

so, total surface area of cuboid =

=
7. Ans. A.
We have,

Radius of base = 6 cm and height of circular cone = 8 cm

We know that slant height =

PAGE 511
www.byjusexamprep.com

cm

total surface area of the cone =

=
8. Ans. B.
We have,
Diameter of base = 6 cm and diameter of top of the bucket = 12 cm

Then radius of base cm and radius of top of the bucket cm

And, height of the bucket = 7 cm

Then capacity or volume of the bucket =

= 462
9. Ans. B.
We have,

the volume of hemisphere = 155232


Let, radius of hemisphere is cm.

We know that the volume of hemisphere =

So,

cm
10. Ans. D.
We have three copper spheres of radius 3 cm, 4 cm and 5 cm.
We melted these spheres into a large sphere.

PAGE 512
www.byjusexamprep.com

We know that volume of sphere is .

Volume of three small spheres = Volume of large sphere

cm

11. Ans. D.
We have, One trapezium ABCD.

and
And, the trapezium has an incircle which touches AB at E and CD at F.
Here cm and cm.
If we draw one line from E to F which is pass through centre of circle then EF is tangent to circle and
it makes angle with AB and CD respectively.

Then
We draw a line from C to EB at position N means
16
D F C

A E N B
25
By the rule of tangent if two tangents which touches at different position in a circle has meet at one
point then length is same from the particular position to tangent.
So, here MB and EB is tangent to circle and meet at B.
cm
And, FC and MC is tangent to circle and meet at C.
cm

Then cm

PAGE 513
www.byjusexamprep.com

In a ,

cm

We can see cm
So. diameter is 40 cm.
12. Ans. D.
We have,
A thin rod of length 24 feet cut into equal size skeleton cube .
In a skeleton cube has 12 pieces. Means size of one piece is 2 feet.
t
ee
2f

Then, area of one of the faces of largest cube =

= square feet
13. Ans. A.

We know that the common triplet sides of a right triangle

So here we have one side is 15 cm.

Then other side is cm

And third side is cm

Then maximum perimeter = cm


14. Ans. B.
We have,
A cylinder height = 10 cm and radius is 6 cm.

Volume of a solid cylinder =

PAGE 514
www.byjusexamprep.com

Flat surface area of cylinder =


A solid cylinder is melted into two cones with same height 10 cm .
The ratio of volume of two cone is 1:2.
Means if we divide into 1:2 then .

So one cone volume is and second has .

Cone volume =

Then first bigger cone has radius. then

Then flat surface area of first cone =

Then second smaller cone has radius. then

Then flat surface area of second cone =


Total surface area of cone = .

So flat surface area from cylinder to cone is increased by which is 50 % increase.


15. Ans. A.
We have,

External radius of the sphere, cm

Internal radius of the sphere, cm

Volume of metal in the hollow sphere =

Given that radius of the cone formed, cm.


Let the height of the cone be .

Volume of the cone =

PAGE 515
www.byjusexamprep.com

Volume of the cone = volume of metal in the sphere

cm

16. Ans. A.

We have an equilateral triangle .


Side cm

We know the area of equilateral triangle =

Now,

PAGE 516
www.byjusexamprep.com

17. Ans. A.
We have,

18. Ans. B.
We have,

19. Ans. D.

We know that Heron’s formula, if we want to calculate area of triangle having sides units
is

Here,

We have,

PAGE 517
www.byjusexamprep.com

cm

cm

cm

Then,

So, we have to know the logarithmic of 37, 7, 9 and 21 for finding the area of triangle.
20. Ans. A.
We have

So, here quotient is compare with .

Then
21. Ans. B.
We have,
Firstly square is inscribed in a semicircle radius cm.
So, here sides of square is cm.

PAGE 518
www.byjusexamprep.com

D C

r a

A O B
a/2

In the figure

In a ,

And, we have a square which is inscribed in a circle of radius cm.


Here, diagonal of square is the same as diameter of circle.
So, here diagonal of square =

D C

2r

A B
Then ratio of area of square inscribed in a semicircle to area of square inscribed in a circle

PAGE 519
www.byjusexamprep.com

22. Ans. C.
We have ,
Length of arc = 33 cm and radius of circle is 14 cm.

Let is the angle subtended by the arc at centre of the circle.

We know that length of arc =

33 cm
B
A
14

23. Ans. C.
We have,
The diameter of right circular cylinder = 20 cm
cm

cm

We know that curved surface area = sq. units

Given that curved surface area =

cm

We know that volume of cylinder = sq. units

PAGE 520
www.byjusexamprep.com

=
24. Ans. A.
We have,
A square which is inscribed in a circle thus square vertices touch the circle .
Radius of circle = 201 mm
Thus, diagonal of the square = diameter of circle = mm

We know that area of circle =

We know that area of square =

D C
m
1m
20

A B

So, the ratio of area of circle to square =

25. Ans. C.
We have,
The length of hypotenuse = 10 cm
We know the Pythagoras theorem,

Let, perpendicular is a cm and base is b cm.

Then,

PAGE 521
www.byjusexamprep.com

We know that

We want maximum area so we do differentiation of area w.r.t. a which is equal to 0.

cm

Then maximum area =

= 25
26. Ans. C.
Consider px2 + 3x + 2q = 0

⇒ Sum of roots =

According to the question

⇒ Product of roots =

According to the question

PAGE 522
www.byjusexamprep.com

⇒q= =

p-q=(1/2)-(-3/2) = 2
27. Ans. D.
X = {a, {b}, c}, Y = {{a}, b, c} and Z = {a, b, {c)}
Now,
Z = {a, b, {c)}
Hence, (X ∩ Y) ∩ Z = ∩ {a, b, {c)} = Φ
28. Ans. B.

Consider

We know that when then

Clearly, (x-y) + (y-z) + (z-x) = 0

Hence,

⇒ =

29. Ans. A.
We are interested in finding LCM of the polynomials
x3 + 3x2 + 3x + 1, x3 + 5x2 + 5x + 4 and x2 + 5x + 4

⇒ x3 + 3x2 + 3x + 1 =

⇒ x3 + 5x2 + 5x + 4 =

⇒ x2 + 5x + 4 =
Hence, LCM of the polynomials x3 + 3x2 + 3x + 1, x3 + 5x2 + 5x + 4
and x2 + 5x + 4 = (x + 1)3(x + 4) (x2 + x + 1)
30. Ans. D.
If n2 + 19n + 92 is a perfect square.

PAGE 523
www.byjusexamprep.com

Then let n2 + 19n + 92 =

Now, n will attain integer value only if is a perfect square of odd integer.


Hence,

………. (1)

………..(2)
Adding (1) and (2)
⇒ 4k = 8
⇒k=2
Put the value of k in (1)



Now, n2 + 19n + 92 = 4
⇒ n2 + 19n + 88 = 0


⇒ n = - 11 or n = -8
Hence, Required Sum = -11 – 8 = - 19
31. Ans. C.
We know that 1 is neither prime nor composite.
Hence, statement (1) is correct.

PAGE 524
www.byjusexamprep.com

We know that 0 is neither positive nor negative.


Hence, statement (2) is correct.
Let p = 2 and q = 3

Here, is even but q is odd.


Hence, if p × q is even, then p and q are not always even.
Hence, statement (3) is not correct.
32. Ans. B.
The ratio of the work done by (x + 2) workers in (x – 3) days to the work done by (x + 4) workers in (x
– 2) days is 3 : 4.



But
Hence , x = 10
33. Ans. D.

Consider

Clearly, will be maximum when is minimum.

Now, will be minimum at x =

Hence, Maximum value of =

34. Ans. C.

Consider

PAGE 525
www.byjusexamprep.com

Now ,

35. Ans. D.
If (b - 6) is one root of the quadratic equation x2 - 6x + b = 0, where b is an integer,


⇒ b = 8 or b = 9

Hence, maximum value of b2 =

36. Ans. B.

Factors of 35 = 35 , 7 , 5 , 1
Required pairs = (6,1) , (18 , 17)
So, two such pairs are possible.
37. Ans. A.
The equation x2 + px + q = 0 has roots equal to p and q where q ≠ 0.

Consider

⇒ Sum of roots =

⇒ 2p = - q ……………(1)

Product of roots =

⇒ p =1
Put the value of p in (1)
⇒q=-2
38. Ans. B.
The sum of the squares of four consecutive natural numbers is 294.

PAGE 526
www.byjusexamprep.com

Let four numbers are x , x+1 , x+2 and x+3

But x is a natural number


So, x = 7
⇒ Sum of the numbers =

39. Ans. B.
Cyclicity of 2,3,7 and 8 is 4
So in case of any number with unit digit 2,3,7 and 8 raised to some power, power is divided by 4 and
remainder is obtained. We can find unit digit in this case using following table:

Consider
When 98 is divided by 4 then 2 is obtained as a remainder.

Hence, Unit digit of will be 9.

Consider
When 89 is divided by 4 then 1 is obtained as a remainder.

Hence, Unit digit of will be 3.


Hence, Unit digit of 398 – 389 = 9 – 3 = 6
40. Ans. A.
Consider

PAGE 527
www.byjusexamprep.com

We know that 10 is formed by the multiplication of 2 and 5.


If 10n divides 623 × 759 × 1052 then n will be the possible pairs of 2 and 5 present in 623 × 759 × 1052 .
Number of 2 present in 623 × 759 × 1052 = 23
Number of 5 present in 623 × 759 × 1052 = 20
Hence, Possible pairs will be 20.
⇒ n = 20
41. Ans. C.

Clearly, f(x) a quadratic polynomial and maximum of this quadratic polynomial occurs at

Now, (x – x2) is maximum at x =

42. Ans. B.
Let speed of car = x kmph
Speed of train = y kmph
According to question

⇒ ……………..(1)

PAGE 528
www.byjusexamprep.com

xy= (120y+240x)/7……………..(2)
From (1) and (2),

= (120y + 240x)/7

(y + 4x)/11 = (y + 2x)/7
28x + 7y = 22x + 11y
6x = 4y
3x = 2y
x/y = 2/3
43. Ans. A.
Selling price of first item = Rs. 990
Profit = 10%

Cost price of first item =

Selling price of second item = Rs. 990


Loss = 10%

Cost price of second item =

Total selling price of both items = 990+990 = Rs. 1980


Total cost price of both items = 900+1100 =Rs. 2000

Loss% =

44. Ans. A.

PAGE 529
www.byjusexamprep.com

Hence, =

After comparison, We can say that


x = 3, y = 1 and z = 2
Hence, x+y+z =3+1+2 = 6
45. Ans. C.
A library bas an avenge number, of 510 visitors on Sunday and 240 on other days.
Total number of days in month = 30
Number of Saturday in this month = 5
Remaining days = 25

Required average =

46. Ans. D.
Consider statement 1 :

Clearly, is an irrational number.


So, statement 1 is not correct.

Consider statement 2: There exists at least a positive integer x such that

⇒ ……..(1)

Clearly, there exist a lot of positive integers that satisfy equation (1) i.e.
x = 2 , 3………………..
So, statement (2) is correct.

Consider statement 3 : for all real values of x.

For x = 4

PAGE 530
www.byjusexamprep.com

So, statement (3) is incorrect.

Consider statement 4 : 4.232323……. can be expressed in the form where p and q are integers.

Let x = 4.2323… ………………(1)


Multiply by 100
⇒ 100x = 423.232323 ………………. ……..(2)
Subtract (2) from (1)

Hence, statement (4) is correct.


47. Ans. C.
Tap X can drain out the full tank in = 20 minutes
Tap (X+Y) can drain out the full tank in = 15 minutes
Total capacity of full tank = LCM (20,15) = 60 units

Efficiency of tap X =

Efficiency of tap (X+Y) =

Efficiency of tap Y = 4 unit/minute – 3 unit/minute = 1 unit/minute

Tap Y alone can drain out the full tank in =

48. Ans. B.
Consider

Put q = 1

But when we divide 21 by 15, remainder will be 6.


So, 21 is not required answer.
Put q = 2

PAGE 531
www.byjusexamprep.com

But when we divide 35 by 15, remainder will be 5.


So, 35 is required answer.
49. Ans. C.
Consider

Since the characteristic of is 19.


Hence, Number of digits = 19 + 1 = 20

Now,

Since the characteristic of is 20.


Hence, Number of digits = 20+ 1 = 21
Now,

Since the characteristic of is 21.


Hence, Number of digits = 21 + 1 = 22
50. Ans. C.

Consider

⇒ =

Hence,

51. Ans. A.

Let the length and the breadth of the rectangle is . Then new length and breadth are
. Now the percentage change in the area

PAGE 532
www.byjusexamprep.com

52. Ans. B.
If the height and radius of two cylinder are respectively.

Then, their volumes are respectively. But it is given that

53. Ans. B.
Given that is a right-angled triangle and

Then,

If BP is perpendicular on AC, the then are of the

54. Ans. C.

PAGE 533
www.byjusexamprep.com

If all the sides of a parallelogram are equal, then it is a rhombus. We know that the diagonals of the
rhombus bisect each other at right-angle. Let the diagonals are , then given that

and

From above two,

Now, area of parallelogram,

55. Ans. C.
Let the side of the equilateral triangle and side of the circle are respectively.

Given that the circumference is equal, then

Now,

56. Ans. C.
Line BE and CD intersects each other at point A. These two lines bisect each other ( situated on same
parallel line). And the is similar.

So,

And

57. Ans. B.

PAGE 534
www.byjusexamprep.com

The area of the triangle formed by the joining of the mid-point of sides of a triangle

is the area of the triangle .

Hence, area of triangle PQR = (XYZ (whose mid points of sides are P, Q and R))/4 = (area of the given
triangle)/4 = 128/4 = 32 cm2
area of triangle ABC = (PQR (whose mid points of sides are A, B and C))/4 = (area of the triangle
PQR)/4 = 32/4 = 8 cm2
58. Ans. D.
If a sector of a circles subtending an angle at the centre and radius is , then length of the sector

If it is bent to form a circle of radius then the circumference of the new circle

59. Ans. A.
Given that the length of the median of an equilateral triangle is , that is the height of the triangle.

In general, the height of an equilateral triangle is equal to times a side of the equilateral triangle.

Let the base of the triangle is then,

Now, the area of the triangle

60. Ans. B.
Let the sides of the cuboid is , then

Now multiplying each together

PAGE 535
www.byjusexamprep.com

61. Ans. B.
As we know that the diagonal of the parallelogram bisect each other. Then

Since it is given that is a equilateral triangle so diagonal PR and QS intersects at right-angle.

So, In ,

Hence,
62. Ans. B.
Given that the area of the are 75 cm and 50 cm respectively.

Then,

Let , then

63. Ans. A.
If AB is the diameter and O is the centre and the radius is 6.5 cm then .

Given that . We know that if a point P lies on the circle then ABP forms a right-
angled triangle. So, .

From the above option does not satisfies the condition.

64. Ans. D.
The sides AD, BC of a trapezium ABCD are parallel, and the diagonals AC and BD meet at O. If the
area of triangle AOB is 3 cm square and the area of triangle BDC id=s 8 cm square. Then what is the
area of the triangle AOD

Given that and

PAGE 536
www.byjusexamprep.com

So for (they are on the same base and between the sam eparallel lines)

(they are on the same base and between the same parallel lines)

From the property of the trapezium

65. Ans. B.
When, we draw such figures as mentioned in the question the vertex of the old triangle are the mid
points of the sides of new triangle and the sides of the old triangle are half of the opposite side.

So the perimeter of such triangle will be doubled to the old one.


Hence, the perimeter of triangle

66. Ans. C.

We have

67. Ans. C.
Given that the angle of elevation from point P to the top of pole are complementary to each other
and .

PAGE 537
www.byjusexamprep.com

Let the

In the ,

And in the ,

From equation (i) and (ii),

68. Ans. D.

If , then all the ratios will be positive.

Given that

So,

PAGE 538
www.byjusexamprep.com

69. Ans. A.
We have

70. Ans. B.
We have

71. Ans. B.
The angle bisector AX divide the side BC in the ratio of side AB and AC

PAGE 539
www.byjusexamprep.com

AX divides BC in the same ratio as the ratio of AB and AC, this will give:

Since we know that sum of two sides of a triangle is always greater than third side, this gives:
c+b>a
Therefore
BX<AB
AB>BX
72. Ans. C.

Orthocenter is the point of intersection of the altitudes. Each leg in a right triangle forms an altitude.
So, in a right-angled triangle, the orthocenter lies at the vertex containing the right angle.
73. Ans. B.

If the radian measure of the arc is , then the area of the arc

Here,

74. Ans. B.

PAGE 540
www.byjusexamprep.com

From the figure, we can see the are similar.

Hence,

75. Ans. B.

Let the height of the cylinder is and the radius is ; , then

The maximum possible volume of sphere (radius R) is immersed into cylinder if

For maximum possible

When, we immersed the sphere into the cylinder the volume of water remains
= volume of cylinder – volume of sphere

PAGE 541
www.byjusexamprep.com

76. Ans. A.

77. Ans. C.
Given that

or

78. Ans. A.

Let the train and bus fair are . Train fare increased by 20% and bus fair increased by 30%.
So new fair of train and new fair of bus .

So, the ratio

79. Ans. C.

Let the income of A and B are . And their expenditure are . The their savings
are . And

PAGE 542
www.byjusexamprep.com

Hence, monthly income of A

80. Ans. C.
If percentage of profit and loss is same on a certain amount then there will always net loss.

81. Ans. C.
lent A lent to C a some of amount of Rs. P and to B Rs.25,000. Rate of interest for both 7% and time

is 4 years. Then simple interest,

82. Ans. B.

We have

It is given that one of the roots of the equation is common with the roots of the
equation

Let is common roots then it satisfy the equation .

So,

Let is common roots then it satisfy the equation .

So,

83. Ans. D.

84. Ans. D.

Time taken by to complete a round are 252 sec, 308 sec and 198 sec.
Now,

So, they meet again after 2772 sec or 46 min 12 sec.


85. Ans. A.

PAGE 543
www.byjusexamprep.com

Let the three linear factors are

So,

After comparing,

Let

Then,

So,
86. Ans. C.
Let the sum of the amount on each investment is and rate of interest is 20%.

ATQ

87. Ans. C.
Statement-1: Unit digit of
Cycle of 7 = 4(7,9,3,1), 174 = 4 x 43 + 2
So, last digit of is 9.

Statement-2: Let two odd numbers are .

So,

Hence, it is always divisible by 8.


Statement-3: Let two consecutive odd numbers are

Then,

Hence, it is always a perfect square.


Here statement 2 and 3 are correct.
88. Ans. A.

PAGE 544
www.byjusexamprep.com

If the HCF of the two numbers is 12, then the numbers are (where are
coprime to each other).
So, the LCM of the numbers = multiple of 12

From the above option we can see that 80 is not multiple of 12.
89. Ans. A.

We have .

So,

From equation (i) and (ii),

90. Ans. C.
a√a + b√b = 32
(√a)3 + (√b)3 = 32
a√b + b√a = 31
√a√b(√a + √b) = 31
Suppose √a = x and √b = y
x3 + y3 = 32 ---- (1)
xy(x + y) = 31 ---- (2)
(x + y)3 = x3 + y3 + 3xy(x + y)
(x + y)3 = 32 + 3 × 31 = 125 = 53
(x + y) = 5
x3 + y3 = (x + y)(x2 + y2 – xy) = 32
(x + y)(x2 + y2 )– xy(x + y) = 32
5(x2 + y2) – 31 = 32

PAGE 545
www.byjusexamprep.com

(x2 + y2) = 63/5


(a + b) = 63/5
5(a + b)/7 = (5/7) × (63/5) = 9
91. Ans. A.

Average number of bikers killed per day in 2017

92. Ans. D.

Average number of bikers killed per day in 2017

93. Ans. A.
Percentage change in the fatalities of the pedestrian during the period 2014-17.

94. Ans. D.
We have

Average number of pedestrians killed per day in 2017

95. Ans. D.

Total number of Scooter of companies X and Y sold by showroom A

Total number of Scooter of company X sold by showroom B

Total number of Scooter of company X sold by showroom E

So, required difference

96. Ans. A.

PAGE 546
www.byjusexamprep.com

Total number of Scooter of companies X and Y sold by showroom A

Total number of Scooter of company X sold by showroom A

Total number of Scooter of company Y sold by showroom A=

Total number of Scooter of companies X and Y sold by showroom C

Total number of Scooter of company X sold by showroom C

Total number of Scooter of company Y sold by showroom C =

Total number of Scooter of companies X and Y sold by showroom E

Total number of Scooter of company X sold by showroom E

Total number of Scooter of company Y sold by showroom E=

So, Average number of scooters sold by showrooms A,C and E

97. Ans. C.

Total number of Scooter of companies X and Y sold by showroom B

Total number of Scooter of company X sold by showroom A

So, required %age

98. Ans. C.

Total number of Scooter of companies X and Y sold by showroom E

Total number of Scooter of company X sold by showroom E

Total number of Scooter of company Y sold by showroom E=

Total number of Scooter of companies X and Y sold by showroom C

PAGE 547
www.byjusexamprep.com

So, required %age

99. Ans. C.
From the given data series-I and series-II have 100 students.
For series-I

For series-II

From equation (i) and (ii),

We can see in series-II the highest frequency range is 40-50.

Estimated Mode =

where:
• L is the lower class boundary of the modal group
• fm-1 is the frequency of the group before the modal group
• fm is the frequency of the modal group
• fm+1 is the frequency of the group after the modal group
• w is the group width

100. Ans. C.

PAGE 548
www.byjusexamprep.com

From the given data series-I and series-II have 100 students.
For series-I

For series-II

From equation (i) and (ii),

Now, mean of the series-I

PAGE 549
www.byjusexamprep.com

CDS I 2019
ENGLISH
Direction: Each item in this section consists of a Direction: Each item in this section consists of a
part with a bold word/words followed by four sentence with an underlined word followed by
words. Select the option that is nearest in four words/group of words. Select the option
meaning to the underlined word/words and that is the nearest in meaning to the underlined
mark your response on your Answer Sheet word and mark your response on your Answer
accordingly. Sheet accordingly.
1. The authorities have reprimanded the 4. Mahesh is mostly prejudiced in his political
subordinate officer for violating the opinion.
protocol.
A. objectionable
A. extolled B. predatory
B. purported C. jaundiced
C. admonished D. intimate
D. required an apology Direction: Each item in this section consists of a
sentence with an underlined word followed by
Direction: Each item in this section consists of a
four words/group of words. Select the option
sentence with an underlined word followed by
that is the nearest in meaning to the underlined
four words/group of words. Select the option
word and mark your response on your Answer
that is the nearest in meaning to the underlined
Sheet accordingly.
word and mark your response on your Answer
Sheet accordingly. 5. Do not indulge in tautology
2. For Gandhiji, India’s religious and linguistic A. truth telling
diversity was an asset, not a liability.
B. prolixity
A. obligation B. advantage
C. foretelling
C. attribute D. reinforcement D. telepathic conversation
Direction: Each item in this section consists of a Direction: Each item in this section consists of a
sentence with an underlined word followed by part with an underlined word followed by four
four options. Select the option that is nearest in words. Select the option that is opposite in
meaning to the underlined word and mark your meaning to the underlined word and mark your
response accordingly. response on your Answer Sheet accordingly.
3. How hysterical he is! 6. His religious views are rather fanatical.
A. berserk B. consistent A. bigoted B. rabid
C. duplicitous D. insincere C. moderate D. militant

PAGE 550
www.byjusexamprep.com

Direction: Each item in this section consists of a Direction: Each item in this section consists of a
sentence with an underlined word followed by sentence with an underlined word followed by
four options. Select the option that is opposite four words. Select the option that is opposite in
in meaning to the underlined word and mark meaning to the underlined word and mark your
your response accordingly. response on your answer sheet accordingly.
7. Religious fundamentalists often consider 11. His handwriting is readable.
the followers of other religions to be
A. well-written B. decipherable
heretics.
C. illegible D. comprehensible
A. dissenter B. believer
Direction: Each item in this section consists of a
C. renegade D. apostate sentence with an underlined word followed by
Direction: Each item in this section consists of a four words. Select the option that is opposite in
part with an underlined word followed by four meaning to the underlined word and mark your
words. Select the option that is opposite in response on your answer sheet accordingly.
meaning to the underlined word and mark your 12. Mohan is his steadfast friend
response on your Answer Sheet accordingly.
A. committed B. unwavering
8. According to G B shaw, men have become
inert. Therefore, life force has chosen C. unfaltering D. unreliable
women to perform its functions. Direction: Each item in this section consists of a
A. lively B. quiescent sentence with an underlined word followed by
four words. Select the option that is opposite in
C. dormant D. apathetic meaning to the underlined word and mark your
Direction: Each item in this section consists of a response on your answer sheet accordingly.
part with an underlined word followed by four 13. Radha often goes tempestuous while
words. Select the option that is opposite in debating.
meaning to the underlined word and mark your
response on your Answer Sheet accordingly. A. calm B. violent

9. Some of the men are highly misanthropic. C. fierce D. vehement

A. anti-social B. philosophic Direction: Each item in this section consists of a


part with an underlined word/words followed by
C. atrophic D. philanthropic four words. Select the option that is opposite in
meaning to the underlined word/words and
Direction: Each item in this section consists of a
mark your response on your Answer Sheet
part with a bold word/words followed by four
accordingly.
words. Select the option that is opposite in
meaning to the underlined word/words and 14. The thief had very vital information to pass
mark your response on your Answer Sheet on to the police.
accordingly.
A. crucial B. inessential
10. The teacher was a very profound man.
C. indispensable D. fundamental
A. sincere
Direction: Each item in this section consists of a
B. erudite sentence with an underlined word followed by
four words. Select the option that is opposite in
C. scholarly meaning to the underlined word and mark your
D. superficial response on your answer sheet accordingly.

PAGE 551
www.byjusexamprep.com

15. His lectures are often wordy and pointless. A. QRSP B. QPSR
A. diffuse B. concise C. SQRP D. RSQP
C. garrulous D. voluble Direction: Each of the following items in this
section consists of a sentence, the parts of which
Direction: Each of the following items in this have been jumbled. These parts have been
section consists of a sentence, the parts of which labelled as P, Q, R and S. Given below each
have been jumbled. These parts have been sentence are four sequences namely (a), (b), (c)
labelled as P, Q, R and S. Given below each and (d). You are required to rearrange the
sentence are four sequences namely (a), (b), (c) jumbled parts of the sentence and mark your
and (d). You are required to rearrange the response accordingly.
jumbled parts of the sentence and mark your
response accordingly. 19. brushed past the latter’s pet dog (P)
stabbed to death by a man (Q) after his
16. the prize money (P) for refusing her (Q) vehicle accidentally (R) a cargo van driver
PepsiCo was ordered (R) to compensate the was allegedly (S)
woman(S)
A. QRSP B. QPSR
A. RSQP B. SPQR
C. SQRP D. SQPR
C. RPSQ D. QRSP
Direction: Each of the following items in this
Direction: Each of the following items in this section consists of a sentence, the parts of which
section consists of a sentence, the parts of which have been jumbled. These parts have been
have been jumbled. These parts have been labelled as P, Q, R and S. Given below each
labelled as P, Q, R and S. Given below each sentence are four sequences namely (a), (b), (c)
sentence are four sequences namely (a), (b), (c) and (d). You are required to rearrange the
and (d). You are required to rearrange the jumbled parts of the sentence and mark your
jumbled parts of the sentence and mark your response accordingly.
response accordingly.
20. an earthquake and tsunami (P) the disaster
17. trade operating from a colony (P) held a mitigation agency (Q) said that the death
meeting (Q) demanding a probe into the toll from (R) In Indonesia has crossed 1500
illegal drug (R) The residents of the city (S) (S)
A. QRSP B. SPQR A. PQSR B. RPSQ

C. SQRP D. RSQP C. SQRP D. QRPS

Direction: Each of the following items in this Direction: Each of the following items in this
section consists of a sentence, the parts of which section consists of a sentence, the parts of which
have been jumbled. These parts have been have been jumbled. These parts have been
labelled as P, Q, R and S. Given below each labelled as P, Q, R and S. Given below each
sentence are four sequences namely (a), (b), (c) sentence are four sequences namely (a), (b), (c)
and (d). You are required to rearrange the and (d). You are required to rearrange the
jumbled parts of the sentence and mark your jumbled parts of the sentence and mark your
response accordingly. response accordingly.

18. the university authorities cancelled the 21. Scientists say they have developed a new
ongoing students’ union election and (P) (P) illnesses such as heart disease and
Following students’ unrest on campus (Q) cancer (Q) DNA tool that uses machine
closed till further orders (R) declared the learning to accurately (R) Predict people’s
institution (S) height and assess their risk for serious (S)

PAGE 552
www.byjusexamprep.com

A. PRSQ B. RPSQ A. PSRQ B. QRPS


C. PSRQ D. QRPS C. QSPR D. PQRS
Direction: Each of the following items in this Direction: Each of the following items in this
section consists of a sentence, the parts of which section consists of a sentence, the parts of which
have been jumbled. These parts have been have been jumbled. These parts have been
labelled as P, Q, R and S. Given below each labelled as P, Q, R and S. Given below each
sentence are four sequences namely (a), (b), (c) sentence are four sequences namely (a), (b), (c)
and (d). You are required to rearrange the and (d). You are required to rearrange the
jumbled parts of the sentence and mark your jumbled parts of the sentence and mark your
response accordingly. response accordingly.
22. a rare evergreen tree in the Southern 25. like a muscle and repeating the process (P)
Western Ghats (P) researchers have found and stable reading circuit (Q) helps the child
that (Q) Common white-footed ants are the build a strong (R) the brain works (S)
best pollinators of (R) Bees might be the
best known pollinators but (S) A. QSRP B. SPRQ

A. PRSQ B. SQRP C. QSPR D. RQPS


C. QSRP D. PQRS Direction: Given below are some idioms/phrases
followed by four alternative meanings to each.
Direction: Each of the following items in this
Choose the response (a), (b), (c) or (d) which is
section consists of a sentence, the parts of which
the most appropriate expression.
have been jumbled. These parts have been
labelled as P, Q, R and S. Given below each 26. A match made in heaven
sentence are four sequences namely (a), (b), (c)
and (d). You are required to rearrange the A. a marriage that is solemnized formally
jumbled parts of the sentence and mark your B. a marriage that is unsuccessful
response accordingly.
C. a marriage that is likely to be happy and
23. say from their forties onwards (P) it is thus successful
a good idea (Q) and continue to exercise
early enough (R) for senior citizens to start D. a marriage of convenience
(S) Direction: Given below are some idioms/phrases
A. PRSQ B. QRSP followed by four alternative meanings to each.
Choose the response (a), (b), (c) or (d) which is
C. QSRP D. PQRS
the most appropriate expression.
Direction: Each of the following items in this
27. A culture vulture
section consists of a sentence, the parts of which
have been jumbled. These parts have been A. someone who is very keen to
labelled as P, Q, R and S. Given below each experience art and literature
sentence are four sequences namely (a), (b), (c)
B. someone who wants to defend ancient
and (d). You are required to rearrange the
culture
jumbled parts of the sentence and mark your
response accordingly. C. someone who is ashamed of one’s own
24. scientists have determined (P) injury in culture
animals and humans (Q) that is linked to the D. someone who looks at her/his culture
severity of spinal cord (R) a gene signature critically
(S)

PAGE 553
www.byjusexamprep.com

Direction: Given below are some idioms/phrases C. a loving and kind person
followed by four alternative meanings to each.
Choose the response (a), (b), (c) or (d) which is D. a disorganized person with bad habits
the most appropriate expression. Direction: Given below are some idioms/phrases
28. A death blow followed by four alternative meanings to each.
Choose the response (a), (b), (c) or (d) which is
A. to be nearly dead the most appropriate expression.
B. to be deeply afraid of death 32. To vote with your feet
C. to beat someone to death
A. to show that you do not support
D. an action or event which causes something
something to end or fail
B. to replace something important
Direction: Given below are some idioms/phrases
followed by four alternative meanings to each. C. to change something you must do
Choose the response (a), (b), (c) or (d) which is D. to express a particular opinion
the most appropriate expression.
Direction: Given below are some idioms/phrases
29. The jewel in the crown followed by four alternative meanings to each.
A. someone who has many skills Choose the response (a), (b), (c) or (d) which is
the most appropriate expression.
B. something that one wants
33. Verbal diarrhoea
C. the most valuable thing in a group of
things A. to be sick
D. the jewel in the crown of the king B. to talk too much
Direction: Given below are some idioms/phrases C. to be in a difficult situation
followed by four alternative meanings to each.
Choose the response (a), (b), (c) or (d) which is D. to be a good orator
the most appropriate expression.
Direction: Given below are some idioms/phrases
30. To live in a fool’s paradise followed by four alternative meanings to each.
Choose the response (a), (b), (c) or (d) which is
A. to live a life that is dishonest
the most appropriate expression.
B. to be happy being a fool
34. To sail close to the wind
C. to be not aware of the current situation
A. to pretend to be something that you
D. to enjoy yourself by spending a lot of are not
money
B. to be in some unpleasant situation
Direction: Given below are some idioms/phrases
followed by four alternative meanings to each. C. to be destroyed by a belief
Choose the response (a), (b), (c) or (d) which is
the most appropriate expression. D. to do something that is dangerous

31. A rotten apple Direction: Given below are some idioms/phrases


followed by four alternative meanings to each.
A. to remove something which is rotten Choose the response (a), (b), (c) or (d) which is
B. one bad person in a group of good the most appropriate expression.
people

PAGE 554
www.byjusexamprep.com

35. A double entendre 39. Blue blood


A. to look at someone or something twice A. to swallow poison
B. a situation in which you cannot succeed B. to be overly interested in someone
C. a word which has two meanings C. to suddenly become jealous
D. something that causes both advantages D. to belong a family of the highest social
and problems. class
Direction: Given below are some idioms/phrases Direction: Given below are some idioms/phrases
followed by four alternative meanings to each. followed by four alternative meanings to each.
Choose the response (a), (b), (c) or (d) which is Choose the response (a), (b), (c) or (d) which is
the most appropriate expression. the most appropriate expression.
36. To cut your own throat 40. Cut the crap
A. to stop doing something A. an impolite way of telling someone to
B. to do something because you are angry stop saying things that are not true

C. to behave in a relaxed manner B. to stop needing someone else to look


after you
D. to allow someone to do something
C. to talk about something important
Direction: Select the most appropriate meaning
of the given idiom. D. to upset someone by criticizing them
37. Cook the books Direction: Each item in this section has a
sentence with three underlined parts labelled as
A. to alter facts dishonestly
(a), (b) and (c). Read each sentence to find out
B. to do something that spoils someone’s whether there is any error in any underlined part
plan and indicate your response on the Answer Sheet
against the corresponding letter i.e., (a) or (b) or
C. to tell an interesting story
(c). If you find no error, your response should be
D. to be very angry indicated as (d).
Direction: Given below are some idioms/phrases 41. Except for few days (a) in a year during the
followed by four alternative meanings to each. monsoon (b) the river cannot flow on its
Choose the response (a), (b), (c) or (d) which is own (c) No error (d)
the most appropriate expression.
A. (a) B. (b)
38. Change your tune
C. (c) D. (d)
A. to listen to good music
42. Being apprised with our approach, (a) the
B. to do things that you are not willing to whole neighbourhood (b)came out to meet
C. to change your opinion completely the minister (c) No error (d)
because it will bring you and advantage A. (a) B. (b)
D. to pretend to be very friendly C. (c) D. (d)
Direction: Given below are some idioms/phrases 43. The celebrated grammarian Patanjali (a)
followed by four alternative meanings to each. was (b) a contemporary to Pushyamitra
Choose the response (a), (b), (c) or (d) which is Sunga (c) No error (d)
the most appropriate expression.

PAGE 555
www.byjusexamprep.com

A. (a) B. (b) 51. The effect of female employment (a) on


gender equality (b) Now appear to be
C. (c) D. (d)
trickling at the next generation (c) No error
44. His appeal for funds (a) met (b) a poor (d)
response (c) No error (d) A. (a) B. (b)
A. (a) B. (b) C. (c) D. (d)
C. (c) D. (d) 52. Since the 15 minutes that she drives (a) she
45. Buddhism teaches that (a) freedom from confesses that she feels like (b) A woman
desires (b) will lead to escape suffering (c) with wings (c) No error (d)
No error (d) A. (a) B. (b)
A. (a) B. (b) C. (c) D. (d)
C. (c) D. (d) 53. India won (a) by an innings (b) and three
runs (c) No error (d)
46. This hardly won liberty (a) was not to be (b)
lightly abandoned (c) No error (d) A. (a) B. (b)
A. (a) B. (b) C. (c) D. (d)
C. (c) D. (d) 54. Each one (a) of these chairs (b) are broken
(c) No error (d)
47. My friend said (a) he never remembered (b)
having read a more enjoyable book (c) No A. (a) B. (b)
error (d) C. (c) D. (d)
A. (a) B. (b) 55. Few creature (a) outwit (b) the fox in
C. (c) D. (d) Aesop’s Fables (c) No error (d)

48. With a population of over one billion, (a) A. (a) B. (b)


India is second most populous country (b) C. (c) D. (d)
In the world after China (c) No error (d)
56. Anywhere in the world (a) when there is
A. (a) B. (b) conflict (b) women and children suffer the
most (c) No error (d)
C. (c) D. (d)
A. (a) B. (b)
49. There are hundred of superstitions (a)
which survive (b) In the various parts of the C. (c) D. (d)
country(c) No error (d)
57. The man is (a) the foundational director (b)
A. (a) B. (b) of this company (c) No error (d)
C. (c) D. (d) A. (a) B. (b)

50. It is (a) in the temperate countries of C. (c) D. (d)


northern Europe (b) that the beneficial 58. Parents of LGBT community members (a)
effects of cold is most manifest (c) No error are coming in (b) with a little help from
(d) NGOs (c) No error (d)
A. (a) B. (b) A. (a) B. (b)
C. (c) D. (d) C. (c) D. (d)

PAGE 556
www.byjusexamprep.com

59. To love one art form is great (a) but to be otherwise reached. The fundamental facts in
able to appreciate another (b) And find this as in all ethical (____7___) are feelings; all
lateral connections are priceless (c) No that thought can do is to clarify and systematize
error (d) the expression of those feelings, and it ¡s such
clarifying and systematizing of my own feelings
A. (a) B. (b)
that I wish to (____8___) in the present article.
C. (c) D. (d) In fact, the question of right and wrongs of a
60. Female literacy rate has gone up by 11% (a) particular war is generally (___9___) from a
in the past decade as opposed to (b) A 3% judicial or quasi-juridical (___10___). |||End|||
increase in male literacy (c) No error (d) 62. Find out the appropriate word for blank (1).
A. (a) B. (b) A. Upon B. On
C. (c) D. (d) C. At D. Over
Direction: Each item in this section consists of a 63. Find out the appropriate word for blank (2).
sentence with an underlined word followed by
A. Delightful B. Painful
four words/group of words. Select the option
that is the nearest in meaning to the underlined C. Pleasant D. Lovely
word and mark your response on your Answer
64. Find out the appropriate word for blank (3).
Sheet accordingly.
A. Duty B. Obligation
61. Doctors are reluctant to take rural postings
despite big salary offers. C. Responsibility D. Crime
A. disinclined B. eager 65. Find out the appropriate word for blank (4).
C. fervent D. unrepentant A. Feeling B. Sentiment
Directions: Each of the following sentences in C. Reason D. Patriotism
this section has a blank space with four words or 66. Find out the appropriate word for blank (5).
group of words given. Select whichever word or
group of words you consider most appropriate A. However B. As well as
for the blank space and indicate your response C. Both D. Despite
on the Answer Sheet accordingly.
67. Find out the appropriate word for blank (6).
The question whether war is ever justified, and
if so under what circumstances, is one which has A. Thought B. Intimated
been forcing itself (___1___) the attention of all C. Suggested D. Held
thoughtful men. On this question I find myself in
Directions: Each of the following sentences in
the somewhat (____2___) position of holding
this section has a blank space with four words or
that no single one of the combatants is justified
group of words given. Select whichever word or
in the present war, while not taking the extreme
group of words you consider most appropriate
Tolstoyan view that war is under all
for the blank space and indicate your response
circumstances a (___3___). Opinions on such a
on the Answer Sheet accordingly.
subject as war are the outcome of (____4___)
rather than of thought: given a man’s emotional The question whether war is ever justified, and if
temperament, his convictions, (___5___) on the so under what circumstances, is one which has
war in general, and on any particular war which been forcing itself (___1___) the attention of all
may occur during his lifetime, can be (____6___) thoughtful men. On this question I find myself in
with tolerable certainty. The arguments used the somewhat (____2___) position of holding
will be mere reinforcements to convictions that no single one of the combatants is justified

PAGE 557
www.byjusexamprep.com

in the present war, while not taking the extreme (###Q2###) with amazing results. Frances H.
Tolstoyan view that war is under all Arnold, an American who was given one-half of
circumstances a (___3___). Opinions on such a the prize, used ‘directed evolution’ to
subject as war are the outcome of (____4___) (###Q3###) variants of naturally occurring
rather than of thought: given a man’s emotional enzymes that could be used to (###Q4###)
temperament, his convictions, (___5___) on the biofuels and pharmaceuticals. The other half
war in general, and on any particular war which went to George P. Smith, also of the U.S., and Sir
may occur during his lifetime, can be (____6___) Gregory P. Winter, from the U.K., who evolved
with tolerable certainty. The arguments used will antibodies to (###Q5###) autoimmune diseases
be mere reinforcements to convictions and even metastatic cancer through a process
otherwise reached. The fundamental facts in this called phase display.
as in all ethical (____7___) are feelings; all that
72. Find out the appropriate word in each case.
thought can do is to clarify and systematize the
expression of those feelings, and it ¡s such A. evolution B. devolution
clarifying and systematizing of my own feelings
C. revolution D. involution
that I wish to (____8___) in the present article.
In fact, the question of right and wrongs of a 73. Find out the appropriate word in each case.
particular war is generally (___9___) from a A. microscope B. field
judicial or quasi-juridical (___10___).
C. market D. laboratory
68. Find out the appropriate word for blank (7).
74. Find out the appropriate word in each case.
A. Questions B. Answers
A. inhibit B. synthesize
C. Statements D. Experiences
C. hamper D. hold back
69. Find out the appropriate word for blank (8).
75. Find out the appropriate word in each case.
A. Engage B. Praise
A. constitute B. sink
C. Attempt D. Commend C. manufacture D. resolve
70. Find out the appropriate word for blank (9). 76. Find out the appropriate word in each case.
A. Considered B. Observed A. combat B. support
C. Transferred D. Opined C. observe D. invite
71. Find out the appropriate word for blank Direction: In this Section each item consists of six
(10). sentences of a passage. The first and sixth
sentences are given in the beginning as S1 and
A. Possibility B. Formula
S6. The middle four sentences in each have been
C. Force D. Standpoint jumbled up and labelled as P, Q, R and S. You are
required to find the proper sequence of the four
Direction: Each of the following sentences in this
sentences and mark your response accordingly
section has a blank space with four words or
on the Answer Sheet.
group of words given. Select whichever word or
group of words you consider most appropriate 77. S1 : He no longer dreamed of storms, nor of
for the blank space and indicate your response women, nor of great occurrences, nor of
on the Answer Sheet accordingly. great fish, nor fights, nor contests of
strength, nor of his wife.
The Nobel Prize for Chemistry this year is a
tribute to the power of (###Q1###). The S6 : He urinated outside the shack and then
laureates harnessed evolution and used it in the went up the road to wake the boy.

PAGE 558
www.byjusexamprep.com

P : He never dreamed about the boy. P : They dare not approach us, they have no
power to get a hold on us, until the mind is
Q : He only dreamed of places and of the
weakened.
lions on the beach now.
R : He simply woke, looked out through the Q : Weakness leads to all kinds of misery,
open door at the moon and unrolled his physical and mental. Weakness is death.
trousers and put them on. R : But they cannot harm us unless we
S : They played like young cats in the dusk become weak, until the body is ready and
and he loved them as he loved the boy. predisposed to receive them.

The correct sequence should be S : There are hundreds of thousands of


microbes surrounding us.
A. R Q P S B. S R Q P
The correct sequence should be
C. R Q S P D. P R S Q
A. P Q R S B. P R Q S
78. S1 : We do not know, after 60 years of
equation, how to protect ourselves against C. Q R S P D. Q S R P
epidemics like cholera and plague.
80. S1 : The Nobel Prize for Economics in 2018
S6 : This is the disastrous result of the was awarded to Paul Romer and William
system under which we are educated. Nordhaus for their work in two separate
P : If our doctors could have started areas : economic growth and
learning medicine at an earlier age, they environmental economics respectively.
would not make such a poor show as they
S6 : Among recent winners of Nobel Prize in
do.
Economics, it’s hard to think of one issue
Q : I have seen hundreds of homes. I cannot which is more topical and relevant to India.
say that I have found any evidence in them
P : But there is a common thread in their
of knowledge of hygiene.
work.
R : I consider it a very serious blot on the
state of our education that our doctors Q : In economic jargon it’s termed as
have not found it possible to eradicate externality.
these diseases. R : Productive activity often has spillovers,
S : I have the greatest doubt whether our meaning that it can impact an unrelated
graduates know what one should do in case party.
one is bitten by a snake. S : Romer and Nordhaus both studied the
The correct sequence should be impact of externalities and came up with
profound insights and economic models.
A. R Q S P B. P R Q S
The correct sequence should be
C. Q R P S D. P Q S R
79. S1 : The weak have no place here, in this life A. P Q R S B. P R Q S
or in any other life. Weakness leads to C. Q S P R D. Q S R P
slavery.
81. S1 : India’s museums tend to be dreary
S6 : This is the great fact : strength is life, experiences.
weakness is death. Strength is felicity, life
eternal, immortal; weakness is constant S6 : Because it’s better to attract crowds
strain and misery : weakness is death. than dust.

PAGE 559
www.byjusexamprep.com

P : Even the Louvre that attracted an eye- 83. S1 : Few scientists manage to break down
popping 8.1 million visitors last year the walls of the so-called ivory tower of
compared to India’s 10.18 million foreign academia and touch and inspire people
tourists, has hooked up with Beyonce and who may not otherwise be interested in
Jay-Z for promotion, where they take a science.
selfie with Mona Lisa. S6: Not many would have survived this, let
Q : Our museums need to get cool too. alone excelled in the manner he did.
R : A change of approach is clearly called P: Stephen Hawking was one of these few.
for. Q: Around this time he was diagnosed with
S : Troops of restless schoolchildren are Amyotrophic Lateral Sclerosis, an incurable
often the most frequent visitors, endlessly motor neuron disease, and given two years
being told to lower their voices and not to live.
touch the art. R: Judging by the odds he faced as a young
The correct sequence should be gradate student of physics at Cambridge
University, nothing could have been a more
A. P Q R S B. P R S Q remote possibility.
C. S R P Q D. Q S R P S: When he was about 20 years old, he got
82. S1 : A decade ago UN recognized that rape the shattering newd that he could not work
can constitute a war crime and a with the great Fred Hoyle for his PhD, as he
constitutive act of genocide. had aspird to.

S6 : The fact that these two peace laureates The correct sequence should be
come from two different nations underlines A. P Q S R B. P R Q S
that this problem has been widespread,
C. S R P Q D. P R S Q
from Rwanda to Myanmar.
84. S1 : The climate question presents a
P : This year’s Nobel peace prize has been
leapfrog era for India’s development
awarded to two exceptional individuals for
paradigm.
their fight to end the use of sexual violence
as a weapon of war. S6 : This presents a good template for India,
building on its existing plans to introduce
Q : Denis Mukwege is a doctor who has electric mobility through buses first, and
spent decades treating rape survivors in the cars by 2030.
Democratic Republic of Congo, where a
long civil war has repeatedly witnessed the P : It is aimed at achieving a shift to
horror of mass rapes. sustainable fuels, getting cities to commit
to eco-friendly mobility and delivering
R : Nadia Murad is herself a survivor of more walkable communities, all of which
sexual war crimes, perpetuated by IS will improve the quality of urban life.
against the Yazidis.
Q : At the Bonn conference, a new
S : Today she campaigns tirelessly to put Transport Decarbonisation Alliance has
those IS leaders in the dock in international been declared.
courts.
R : This has to be resolutely pursued,
The correct sequence should be breaking down the barriers to wider
A. P Q R S B. P R Q S adoption of rooftop solar energy at every
level and implementing net metering
C. S R Q P D. Q R S P systems for all categories of consumers.

PAGE 560
www.byjusexamprep.com

S : Already, the country has chalked out an P : So both the U.S. and China, which have
ambitious policy on renewable energy, blamed each other for the ongoing trade
hoping to generate 175 gigawatts of power war, are doing no good to their own
form green sources by 2022. economic fortunes by engaging in this tit-
The correct sequence should be for-tat tariff battle.

A. S R Q P B. S P R Q Q : Apart from disadvantaging its


consumers, who will have to pay higher
C. P R S Q D. Q R S P prices for certain goods, tariffs will also
85. S1 : The dawn of the information age disrupt the supply chain of producers who
opened up great opportunities for the rely on foreign imports.
beneficial use of data.
R : China, which is fighting an economic
S6 : To some, in this era of Big Data analytics slowdown, will be equally affected.
and automated, algorithm-based
processing of zettabytes of information, the S : The minutes of the U.S. Federal Reserve
fear that their personal data may be June policy meeting show that economic
unprotected may conjure up visions of a uncertainty due to the trade war is already
dystopian world in which individual affecting private investment in the U.S.,
liberties are compromised. with many investors deciding to scale back
or delay their investment plans.
P : But it is the conflict between the massive
scope for progress provided by digital era The correct sequence should be
and the fear of loss of individual autonomy A. S Q P R B. Q P S R
that is foregrounded in any debates about
data protection laws. C. Q R P S D. P S R Q
Q : It also enhanced the perils of Direction: In this section you have few short
unregulated and arbitrary use of personal passages. After each passage, you will find some
data. items based on the passage. First, read a passage
R : It is against this backdrop that the White and answer the items based on it. You a required
Paper made public to elicit views from the to select your answers based on the contents of
public on the shape and substance of a the passage and opinion of the author only.
comprehensive data protection law Passage — I
assumes significances.
From 1600 to 1757 the East India Company’s role
S : Unauthorised leaks, hacking and other in India was that of a trading corporation which
cyber crimes have rendered databases brought goods or precious metals into India and
vulnerable. exchanged them for Indian goods like textiles
The correct sequence should be and spices, which it sold abroad. Its profits came
A. S Q R P B. Q P R S primarily from the sale of Indian goods abroad.
Naturally, it tried constantly to open new
C. S R P Q D. Q S P R markets for Indian goods in Britain and other
86. S1 : In a globalized world, no country can countries. Thereby, it increased the export of
hope to impose tariffs without affecting its Indian manufacturers, and thus encouraged
own economic interests. their production. This is the reason why Indian
rulers tolerated and even encouraged the
S6 : The ongoing trade war also threatens
establishment of the Company’s factories in
the rules-based global trade order which
India. But, from the very beginning, the British
has managed to amicably handle trade
manufacturers were jealous of the popularity
disputes between countries for decades.

PAGE 561
www.byjusexamprep.com

that India textiles enjoyed in Britain. All of a 89. What did the British manufacturer do to
sudden, dress fashions changed and light cotton compete with the Indian manufacturers?
textiles began to replace the coarse woolens of
A. They pressurized the government to
the English, Before, the author of the famous
levy heavy duties on export of Indian
novel, Robinson Crusoe, complained that Indian
clothes
cloth had “crept into our houses, our closets and
bed chambers; curtains, cushions, chairs, and at B. They pressurized the government to
last beds themselves were nothing but calicos or levy heavy duties on import of Indian
India stuffs”. The British manufacturers put clothes
pressure on their government to restrict and
C. They requested people to change their
prohibit the sale of Indian goods in England. By
fashion preferences.
1720, laws had been passed forbidding the wear
or use of printed or dyed cotton cloth, in 1760 a D. They lowered the prices of the Britain
lady had to pay a fine of 200 for possessing an made textile
imported handkerchief! Moreover, heavy duties
were imposed on the import of plain cloth. Other 90. Which source is cited by the author to
European countries, except Holland. also either argue that Indian textile was in huge
prohibited the import of Indian cloth or imposed demand in 18th century England?
heavy import duties. In spite of these laws, A. The archival source
however, Indian silk and cotton textiles still held
their own in foreign markets, until the middle of B. The scientific source
the eighteenth century when the English textile C. The journalistic source
industry began to develop on the basis of new
and advanced technology. D. The literary source

88. The East India Company was encouraging 91. “New and advanced technology” in the
the export of Indian manufacturers paragraph refers to
because A. the French Revolution
A. it was a philanthropic trading B. the Glorious Revolution of England
corporation
C. the Industrial Revolution
B. it wanted Indian manufacturers to
prosper in trade and commerce D. the beginning of colonialism

C. it profited from the sale of Indian goods Direction: In this section you have few short
in foreign markets passages. After each passage, you will find some
items based on the passage. First, read a passage
D. it feared Indian Kings who would not and answer the items based on it. You a required
permit them trade in India to select your answers based on the contents of
88. The people of England used Indian clothes the passage and opinion of the author only.
because Passage — II
A. they loved foreign and imported Zimbabwe’s prolonged political crisis reached
clothes the boiling point earlier this month when
B. the Indian textile was light cotton President Robert Mugabe dismissed the Vice-
President, Emmerson Mnangagwa. A battle to
C. the Indian clothes were cheaper succeed the 93-year-old liberation hero-turned
D. the Indian cloths could be easily President had already been brewing within the
transported ruling Zimbabwe African National Union-

PAGE 562
www.byjusexamprep.com

Patriotic Front (Zanu-PF), with the old guard C. Generation 40


backing Mr. Mnangagwa, himself a freedom
D. Zanu –PF
fighter, and ‘Generation 40’, a grouping of
younger leaders supporting Mr, Mugabe’s 52- 94. Mr. Mugabe’s political weakness became
year-old wife, Grace. Ms. Mugabe, known, for apparent when
her extravagant lifestyle and interfering ways,
A. he endorsed his wife
has been vocal in recent months about her
political ambitions. Mr. Mugabe was seen to B. he turned against the army
have endorsed iii when on November 6 he
dismissed Mr. Mnangagwa. But Mr. Mugabe C. he suffered from health issues
who has ruled Zimbabwe since its independence D. he dismissed Mr. Mnangagwa
in 1980, erred on Two counts: he underimat&1
the deep connections Mr. Mnangagwa has 95. The security forces of Zimbabwe staged a
within the establishment and overestimated his coup against the President because
own power in a system he has helped shape. In A. they wanted Mrs. Mugabe as the
the good old days, Mr. Mugabe was able to rule President
with an iron grip. But those days are gone. Age
and health problems have weakened his hold on B. they were aware of Mugabe’s failing
power, while there is a groundswell of anger wealth
among the public over economic C. they disliked Mugabe’s extravagant
mismanagement. So when he turned against a lifestyle
man long seen by the establishment as his
successor, Mr. Mugabe left little doubt that he D. they did not want a Mugabe dynasty
was acting from a position of political weakness. 96. Why does the military not want to call it a
This gave the security forces the confidence to coup d’etat?
turn against him and make it clear they didn’t
want a Mugabe dynasty. The military doesn’t A. Because coup is immoral
want to call its action a coup d’etat, for obvious B. Because coup is illegal
reasons. A coup would attract international
condemnation, even sanctions. But it is certain C. Because coup would lead to inter-
that the army chief, General. Constantine national censure and sanctions
Chiwenga, is in charge. His plan as it emerges, is D. Because it would make the public revolt
to force Mr. Mugabe to resign and install a
transitional government, perhaps under Mr. Direction: In this section you have few short
Mnangagwa, until elections are held. passages. After each passage, you will find some
items based on the passage. First, read a passage
92. In the paragraph, who has been called and answer the items based on it. You a required
liberation hero? to select your answers based on the contents of
A. Constantino Chiwenga the passage and opinion of the author only.

B. Emmerson Mnangagwa Passage – III

C. Robert Mugabe Over-eating is one of the most wonderful


practices among those who think that they can
D. Army Chief afford it. In fact, authorities say that nearly all
93. Mrs. Mugabe is supported by who can get as much as they desire, over-eat to
their disadvantage. This class of people could
A. Mr. Mnangagwa save a great more food than they can save by
B. Mr. Mugabe missing one meal per week and at the same time

PAGE 563
www.byjusexamprep.com

they could improve their health. A heavy meal at 101. According to the passage, how many times
night, the so-called “dinner”, is the fashion with a day should we have food?
many and often is taken shortly before retiring.
A. Three times
It is unnecessary and could be forgone, not only
once a week but daily without loss of strength. B. Two times
From three to five hours are needed to digest
C. Once
food. While sleeping, this food not being
required to give energy for work, is in many cases D. Has not been specified
converted into excess fat, giving rise to
102. According to the passage, people overeat
overweight. The evening meal should be light,
taken three or four hours before retiring. This A. because they can afford to
prevents overeating, conserves energy and B. because they are hungry
reduces the cost of food.
C. because they have to work more
97. Why should those who over-eat refrain
from doing so? D. because they have to conserve energy

A. Because overeating leads to loss of Direction: In this section you have few short
wealth passages. After each passage, you will find some
items based on the passage. First, read a passage
B. Because overeating is bad for health and answer the items based on it. You a required
C. Because overeating conserves food to select your answers based on the contents of
the passage and opinion of the author only.
D. Because overeating is immoral and
unhealthy Passage – IV
98. Over-eating is more prevalent among Much has been said of the common ground of
religious unity. I am not going just now to
A. the rich venture own theory. But if anyone here hopes
B. the poor that this unity will come by the triumph of any
one of the religions and the destruction of the
C. everybody
others, to him I say, “Bother, yours is an
D. the bourgeoisie impossible hope.” Do I wish that the Christian
would become Hindu? God forbid. Do I wish that
99. The writer is asking the readers
the Hindu or Buddhist would become Christian?
A. to skip the heavy dinner and take light God forbid.
evening meal instead
The seed is put in the ground, and earth and air
B. to stop eating anything at night and water are placed around it. Does the seed
become the earth, or the air, or the water? No.
C. to take food only during the day
It becomes a plant. ¡t develops after the law of
D. to eat food before the sunset its own growth, assimilates the air, the earth,
and the water, converts them into plant
100. What is the most appropriate time for
substance, and grows into a plant.
having evening meal?
Similar is the case with religion. The Christian is
A. An hour after the sunset
not to become a Hindu or a Buddhist, nor a
B. Three or four hours before sleeping Hindu or a Buddhist to become a Christian. But
each must assimilate the spirit of the others and
C. Before the sunset
yet preserve his individuality and grow according
D. Just before sleeping to his own law of growth.

PAGE 564
www.byjusexamprep.com

If the Parliament of Religions has shown B. One day, there will be no religion
anything to the world, it is this: it has proved to
C. Purity and charity are the exclusive
the world that holiness, purity and charity are
possessions
not the exclusive possessions of any church in
the world, and that every system has produced D. Banner of every religion will soon be
men and women of the most exalted character. written
In the face of this evidence, if anybody dreams of
Direction: Each of the following sentences in this
the exclusive survival of his own religion and the
section has a blank space and four words or
destruction of the others, I pity him from the
group of words are given after the sentence.
bottom of my heart, and point out to him that
Select the word or group of words you consider
upon the banner of every religion will soon be
the most appropriate for the blank space and
written in spite of resistance: “Help and not
indicate your response on the Answer Sheet
fight,” “Assimilation and not Destruction,”
accordingly.
“Harmony and Peace and not Dissension.”
103. According to the author of the passage, 107. How we _________ to ageing is a choice we
people should must make wisely.

A. change their religions A. respond B. absolve

B. follow their religions and persuade C. discharge D. overlook


others to follow it Direction: Each of the following sentences in this
C. follow their own religions and respect section has a blank space and four words or
other religions group of words are given after the sentence.
Select the word or group of words you consider
D. disrespect other religions
the most appropriate for the blank space and
104. The Parliament of Religions is indicate your response on the Answer Sheet
accordingly.
A. a Christian organization
B. a Buddhist organization 108. Complementary medicine _______ fewer
risks, since it is used along with standard
C. a Hindu organization remedies, often to lessen side-effects and
D. a platform for discussion about every enhance feelings of well-being.
religion of the world A. reacts B. releases
105. What does the author think about those C. ejects D. carries
who dream about the exclusive survival of
their own religions and the destruction of Direction: Each of the following sentences in this
the others? section has a blank space and four words or
group of words are given after the sentence.
A. He hates them Select the word or group of words you consider
B. He desires to imprison them the most appropriate for the blank space and
indicate your response on the Answer Sheet
C. He pities them
accordingly.
D. He praises them
109. Stress may ___________ fertility in men
106. According to the passage, what is and women.
“impossible hope”?
A. engage B. reduce
A. One day, all the people of the world will
follow only one religion C. inject D. deduce

PAGE 565
www.byjusexamprep.com

110. Which one of the following phrasal verbs 114. You were going to apply for the job, and
may be used in the blank space in the then you decided not to. So what
sentence, "The football match had to be _________?
__________ because of the weather"?
A. put you off B. put you out
A. called on B. called off
C. turned you off D. turned you away
C. called out D. called over
Direction: Each of the following sentences in this
Direction: Each of the following sentences in this section has a blank space and four words or
section has a blank space and four words or group of words are given after the sentence.
group of words are given after the sentence. Select the word or group of words you consider
Select the word or group of words you consider the most appropriate for the blank space and
the most appropriate for the blank space and indicate your response on the Answer Sheet
indicate your response on the Answer Sheet accordingly.
accordingly.
115. _________ it was raining, he went out
111. Nobody believed Ram at first but he without a raincoat.
_____________ to be right.
A. Even B. Since
A. came out
C. Unless D. Although
B. carried out
Direction: Each of the following sentences in this
C. worked out section has a blank space and four words or
group of words given after the sentence. Select
D. turned out
the word or group of words you consider most
Direction: In the following question, a sentence appropriate for the blank space and indicate
is given with a blank to be filled in with an your response on the Answer Sheet accordingly.
appropriate word. Select the correct alternative
116. I parked my car in a no-parking zone, but
out of the four and indicate it by selecting the
I__________ it.
appropriate option.
A. came up with
112. How are you ___________ in your new job?
Are you enjoying it? B. got away with
A. keeping on B. going on C. made off with
C. getting on D. carrying on D. got on with
Direction: Choose the most appropriate Direction: Each item in this section consists of a
alternative to complete the sentence: sentence with an underlined word followed by
four words/group of words. Select the option
113. We live _____ a tower block. Our
that is the nearest in meaning to the underlined
apartment is on the fifteenth floor.
word and mark your response on your Answer
A. at B. in Sheet accordingly.
C. over D. above 117. A provocative message had been doing
rounds on social media to instigate the mob
Direction: In the following question, a sentence
against migrants.
is given with a blank to be filled in with an
appropriate word. Select the correct alternative A. dexterous B. inflammatory
out of the four and indicate it by selecting the
C. valiant D. prudent
appropriate option.

PAGE 566
www.byjusexamprep.com

Direction: Each item in this section consists of a 119. The portal will help victims and
part with an underlined word followed by four complainants to anonymously report
words. Select the option that is nearest in cybercrime.
meaning to the underlined word and mark your
response on your Answer Sheet accordingly. A. incognito

118. The differences include increase in mean B. directly


temperature and heavy precipitation in C. unfailingly
several regions.
D. in situ
A. drought
Direction: Each item in this section consists of a
B. oasis
sentence with an underlined word followed by
C. rainfall four words/group of words. Select the option
that is the nearest in meaning to the underlined
D. snowing word and mark your response on your Answer
Direction: Each item in this section consists of a Sheet accordingly.
sentence with an underlined word followed by
120. He is suffering from a terminal disease.
four words/group of words. Select the option
that is the nearest in meaning to the underlined A. sublunary B. terrific
word and mark your response on your Answer
Sheet accordingly. C. chronic D. incurable

PAGE 567
www.byjusexamprep.com

General Knowledge
1. Henry T. Colebrooke was a Professor of 5. Who among the following was known
Sanskrit in which one of the following during the days of the Revolt of 1857 as
institutions? 'Danka Shah'?
A. Fort William College A. Shah Mal
B. Serampore Mission B. Maulavi Ahmadullah Shah
C. Kashi Vidyapith C. Nana Sahib
D. Asiatic Society D. Tantia Tope
2. The Deccan Agriculturalists 'Relief Act of 6. The Summary Settlement of 1856 was
1879 was enacted with which one of the based on which one of the following
following objectives? assumptions?
A. Restore lands to the dispossessed
A. The Talukdars were the rightful owners
peasants
of the land
B. Ensure financial assistance to peasants
B. The Talukdars were interlopers with no
during social and religious occasions
permanent stakes in the land.
C. Restrict the sale of land for
indebtedness to outsiders C. The Talukdars could evict the peasants
from the lands
D. Give legal aid to insolvent peasants
D. The Talukdars would take a portion of
3. The Damin-i-Koh was created by the British the revenue which flowed to the State
Government to settle which one of the
following communities? 7. The Inter State Council was set up in 1990
on the recommendation of
A. Santals
A. Punchhi Commission
B. Mundas
B. Sarkaria Commission
C. Oraons
C. Rajamannar Commission
D. Saoras
4. The Limitation Law, which passed by the D. Mungerilal Commission
British in 1859, addressed which one of the 8. Which among the following writs is issued
following issues? to quash the order of a Court or Tribunal?
A. Loan bonds would not have any legal A. Mandamus B. Prohibition
validity
C. Quo Warranto D. Certiorari
B. Loan bonds signed between money
lender and Ryots would have validity 9. Which among the following statements
only for three years about the power to change the basic
structure of the Constitution of India is/are
C. Loan bonds could not be executed by correct?
moneylenders
1. It falls outside the scope of the
D. Loan bonds would have validity for ten
amending powers of the Parliament.
years

PAGE 568
www.byjusexamprep.com

2. It can be exercised by the people 14. The mobile app 'CVIGIL' is helpful in
though representatives in a
A. conducting free and fair e-tendering
Constituent Assembly.
process in government offices
3. It falls within the constituent powers of
the Parliament. B. fighting against corruption in public
services
Select the correct answer using the code
given below. C. removing garbage from the muncipal
areas
A. 1 and 3 B. 1 and 2
D. reporting violation of model code of
C. 1 only D. 2 and 3 conduct in election bound States
10. When a Proclamation of Emergency is in 15. Prahaar' is
operation, the right to move a Court for the
enforcement of all Fundamental Rights A. a battle tank
remains suspended, except B. a surface-to-surface missile
A. Article 20 and Article 21 C. an aircraft carrier
B. Article 21 and Article 22 D. a submarine
C. Article 19 and Article 20
16. Who among the following is/are the
D. Article 15 and Article 16 recipient/recipients of Rajiv Gandhi Khel
Ratna Award, 2018?
11. Which one of the following Articles of the
Constituent of India down that no citizen A. Virat Kohli
can be denied the use of wells, tanks and
B. S.Mirabai Chanu and Virat Kohli
bathing Ghats maintained out of State
funds? C. Neeraj Chopra
A. Article 14 D. Hima Das and Neeraj Chopra
B. Article 15 17. Pakyong Airport is located in ____.
C. Article 16 A. Sikkim
D. Article 17 B. Jammu and Kashmir
12. Who amongst the following organized the C. Arunachal Pradesh
All India Scheduled Castes Federation?
D. Mizoram
A. Jyotiba Phule
E. None of these
B. Periyar
18. The United Nations has been observing
C. B.R. Ambedkar International Day of Rural Women on
D. M.K. Karunanidhi A. 15th July
13. Paul Allen, who died in October 2018, was
B. 15th August
the co-founder of _____.
C. 15th September
A. Oracle B. IBM
D. 15th October
C. Microsoft D. SAP

PAGE 569
www.byjusexamprep.com

19. Who among the following is the first Indian Which of the statements given above is/are
to win Pulitzer Prize? correct?
A. Arundhati Roy A. 1 only B. 1 and 2
B. Gobind Behari Lal C. 2 and 3 D. 3 only
C. Vijay Seshadri 24. Consider the following statements about a
joint-stock company:
D. Jhumpa Lahiri
20. Saurabh Chaudhary excels in which one of 1. It has a legal existence.
the following sports? 2. There is limited liability of shareholders.
A. Archery B. Shooting 3. It has a democratic management.
C. Boxing D. Judo 4. It has a collective ownership.
21. Which one of the following is not an Which of the statements given above are
assumption in the law of demand? correct?
A. There are no changes in the taste and A. 1 and 2 only B. 1, 2 and 3 only
preferences of consumers
C. 3 and 4 only D. 1, 2, 3 and 4
B. Income of consumers remains constant
25. When some goods or productive factors are
C. Consumers are affected by
completely fixed in amount, regardless of
demonstration effect
price, the supply curve is
D. There are no changes in the price of
A. horizontal
substitute goods.
B. downward sloping to the right
22. Which one of the following statements is
not correct? C. vertical
A. When total utility is maximum, D. upward sloping to the right
marginal utility is zero
26. Who designed the Bombay Secretariat in
B. When total utility is decreasing, the 1870s?
marginal utility is negative
A. H. St. Clair Wilkins
C. When total utility is increasing,
marginal utility is positive B. Sir Cowasjee Jehangir Readymoney

D. When total utility is maximum, C. Purushottamdas Thakurdas


marginal and average utility are equal D. Nusserwanji Tata
to each other.
27. Who was the founder of Mahakali
23. Consider the following statements about Pathshala in Calcutta?
indifference curves:
A. Her Holiness Mataji Maharani
1. Indifference curves are convex to the Tapaswini
origin.
B. Sister Nivedita
2. Higher indifference curve represents
higher level of satisfaction. C. Madamee Blavatsky
3. Two indifference curves cut each other. D. Sarojini Naidu

PAGE 570
www.byjusexamprep.com

28. Which European ruler had observed 'Bear 33. Which one of the following is not a change
in mind that the commerce of India is the brought about by the Indian Independence
commerce of the world …. he who can Act of 1947?
exclusively command it is the dictator of A. The Government of India Act, 1935 was
Europe? amended to provide an interim
A. Queen Victoria Constitution
B. Peter the Great of Russia B. India ceased to be a dependency
C. Napoleon Bonaparte C. The Crown was the source of authority
till new Constitution was framed
D. Gustav II Adolf
D. The Governor General was the
29. Which European traveller had observed, "A
constitutional head of Indian Dominion
Hindu woman can go anywhere alone, even
in the most crowded places, and she need 34. Which one of the following is not a correct
never fear the impertinent looks and jokes statement regarding the provision of
of idle loungers"? Legislative Council in the State Legislature?
A. Francois Bernier A. The States of Bihar and Telangana have
Legislative Councils
B. Jean-Baptiste Tavernier
B. The total number of members in the
C. Thomas Roe Legislative Council of a State shall not
D. Abbe J. A. Dubois exceed one third of the total number of
members in the Legislative Assembly
30. Who was the author of the book, Plagues
and Peoples? C. One twelfth of all members shall be
elected by electorates consisting of
A. W.L. Thomas
local bodies and authorities
B. Rachel Carson
D. one twelfth of all members shall be
C. David Cannadine elected by graduates residing in the
D. William H.McNeill State
35. Which one of the following is not correct
31. Which Indian sociologist argued that the
about the Panchayats as laid down in Part
idea of a distinct Hinduism was formed by
IX of the Constitution of India?
the 'cultural arrogance of post-
Enlightenment Europe'? A. The Chairperson of a Panchayat needs
to be directly by people in order to
A. Ashis Nandy
exercise the right to vote in the
B. Partha Chatterjee Panchayat meetings.
C. T.K.Oommen B. The State Legislature has the right to
D. Rajni Kothari decide whether or not offices of the
Chairpersons in the Panchayats are
32. Sub-prime crisis' is a term associated with reserved for SCs, STs or women.
which one of the following events?
C. Unless dissolved earlier, every
A. Economic recession Panchayat continues for a period of five
B. Political instability years.
C. Structural adjustment programmes D. The State Legislature may by law make
provisions for audit of accounts of the
D. Growing social inequality Panchayats.

PAGE 571
www.byjusexamprep.com

36. Which one of the following is not correct C. Life Cycle Hypothesis
about Administrative Tribunals?
D. Permanent Income Hypothesis
A. The Parliament may by law constitute
40. According to John Maynard Keynes,
Administrative Tribunals both at the
employment depends upon
Union and State levels
B. Tribunals may look into disputes and A. aggregate demand
complaints with respect to recruitment B. aggregate supply
and conditions of service of persons
appointed to public services. C. effective demand

C. Tribunals established by a law of the D. rate of interest


Parliament can exclude the jurisdiction 41. Which one of the following canons of
of all Courts to allow for special leave to taxation was not advocated by Adam
appeal. Smith?
D. The law establishing the Tribunals may A. Canon of equity
provide for procedures including rules
of evidence to be followed. B. Canon of certainty

37. A market situation when firms sell similar C. Canon of convenience


but not identical products is termed as D. Canon of fiscal adequacy
A. perfect competition 42. Which Arab scientist could be given the
B. imperfect competition credit of christening the mathematical
discipline of algorithm?
C. monopolistic competition
A. Al-Khwarizmi B. Ibn al - Haytham
D. oligopoly
C. Ibn Rushd D. Ibn Sina
38. Consider the following statements:
43. Which one of following developments took
1. Inflation in India continued to be
place because of the Kansas Nebraska Act
moderate during 2017-18
of 1854?
2. There was significant reduction in food
A. The Missouri Compromise was
inflation, particularly pulses and
repealed and people of Kansas and
vegetables during the period.
determine whether they should own
Which of the statements given above is/are slaves or not.
correct?
B. The Act did not permit the territories
A. 1 only B. 2 only the right to voter over the question of
C. Both 1 and 2 D. Neither 1 nor 2 slavery

39. Which one of the following hypotheses C. The voice of the majority in regard to
postulates that individual's consumption in the issue of slavery was muzzled
any time period depends upon resources D. The Federal Government had the sole
available to the individual, rate of return on authority to decide on slavery.
his capital and age of the individual?
44. Which one of the following issues was
A. Absolute Income Hypothesis included in the Indo-US Nuclear Agreement
B. Relative Income Hypothesis of 2007?

PAGE 572
www.byjusexamprep.com

A. India has 'advance right to reprocess' 47. The South China Sea Dispute involves which
US-origin safeguarded spent fuel. of the following countries?
B. India did not have the right to build a 1. China
strategic fuel reserve with the help of
the other supplier countries 2. Vietnam

C. India should not test a nuclear device 3. Malaysia

D. The US will impede with the growth of 4. Indonesia


India's nuclear weapons programme. Select the correct answer using the code
45. Which of the following statements about given below.
Alladi Krishnaswami Ayyar, as a drafting
A. 1 and 4
member of the Constitution of India, are
correct? B. 1 and 2 only
1. He favoured the role of the Supreme C. 1, 2 , 3 and 4
Court in taking important decisions
related to the interpretation of the D. 2, 3 and 4
Constitution of India. 48. The 'Kyoto Protocol' is an international
2. He felt that the Supreme Court had to treaty that commits State parties to
draw the line between liberty and social reduction in
control. A. poverty
3. He believed in the dominance of the B. greenhouse gases emission
executive over the judiciary.
C. nuclear armaments
4. He favoured a dictatorial form of
governance. D. agricultural subsidy
Select the correct answer using the code 49. The 'Beijing Declaration' is concerned with
given below. which one of the following issues?
A. 1 and 2 only B. 1, 2 and 3 A. Rights of children
C. 3 and 4 D. 1, 2 and 4 B. Rights of women
46. Which of the following are the core C. Right to development
functions of the United Nations
multidimensional peace keeping D. Reduction of tariffs
operations? 50. The 'Gujral Doctrine' relates to which one of
1. Stabilization the following issues?
2. Peace consolidation A. Build trust between India and its
neighbours
3. To extend support to a losing State in a
war. B. Initiate dialogue with all insurgent
groups in India
Select the correct answer using the code
given below. C. Undertake development activities in
A. 1, 2 and 3 B. 2 and 3 only Naxal dominated areas

C. 1 and 3 only D. 1 and 2 only D. Ensure food security

PAGE 573
www.byjusexamprep.com

51. Match List - I with List - II and select the B. Newman projection can be
correct answer using the code given below represented in eclipsed, staggered and
the Lists : skew conformations
List-I List-II C. Fischer projection of the molecule is its
(Compound/ (Shape of most stable conformation
Molecule) Molecule)
D. In Sawhorse projections, the lines are
A. CH3F 1. Trigonal planar inclined at an angle of 120o to each
other.
B. HCHO 2. Tetrahedral
56. The monomer/monomers used for the
C. HCN 3. Trigonal
synthesis of Nylon 6 is /are
pyramidal
A. hexamethylenediamine and adipic acid
D. NH3 4. Linear
B. caprolactam
C. urea and formaldehyde
Codes:
D. phenol and formaldehyde
A. A-2, B-4, C-1, D-3
57. Which one among the following stars is
B. A-2, B-1, C-4, D-3
nearest to the earth?
C. A-3, B-4, C-1, D-2
A. Sirius B. Arcturus
D. A-3, B-1, C-4, D-2
C. Spica D. Proxima Centauri
52. Very small insoluble particles in a liquid may
be separated from it by using _______ . 58. Which of the following planets of our solar
system has least mass?
A. crystallization
A. Neptune B. Jupiter
B. fractional distillation
C. Mars D. Mercury
C. centrifugation
59. Two identical solid pieces, one of gold and
D. decantation other of silver, when immersed completely
53. Which one of the following elements in water exhibit equal weights. When
cannot be detected by "Lassaigne's test"? weighed in air (given that density of gold is
greater than that of silver)
A. I B. Cl
A. the gold piece will weigh more
C. S D. F
B. the silver piece will weigh more
54. In which of the following, functional group
isomerism is not possible? C. both silver and gold pieces weigh equal

A. Alcohols B. Aldehydes D. weighing will depend on their masses

C. Alkyl halides D. Cyanides 60. If the wavelengths corresponding to


ultraviolet, visible and infrared radiations
55. Which one of the following statements is are given as lUV, lVIS and lIR respectively,
not correct? then which of the following gives the
A. Fischer projection represents the correct relationship among these
molecule in an eclipsed conformation wavelengths?

PAGE 574
www.byjusexamprep.com

A. lUV < lIR < lVIS A. Tracheid


B. lUV > lVIS > lIR B. Pericycle

C. lUV > lIR > lVIS C. Intercalary meristem

D. lUV < lVIS < lIR D. Lateral meristem

61. An electron and a proton starting from rest 66. Which one of the following organisms is
get accelerated through potential dependent on saprophytic mode of
difference of 100kV. The final speeds of the nutrition?
electron and the proton are Ve and Vp A. Agaricus B. Ulothrix
respectively. Which of the following
relations is correct? C. Riccia D. Cladophora

A. Ve > VP 67. Which one of the following has a bilateral


symmetry in its body organization?
B. Ve < VP
A. Asterias B. Sea anemone
C. Ve = VP
C. Nereis D. Euchinus
D. Cannot be determined 68. Which one of the following pairs of animals
62. If two vectors A and B are at an angle ϴ ≠ 0 is warm blooded?
degree then A. Crocodile and Ostrich
A. B. Hagfish and Dogfish

B. C. Tortoise and Ostrich


D. Peacock and Camel
C.
69. Which one of the following States of India
D. is not covered by Food Forecasting Stations
set up by the Central Water Commission?
63. Which one of the following functions is not A. Rajasthan
carried out by smooth endoplasmic
reticulum? B. Jammu and Kashmir

A. Transport of materials C. Tripura

B. Synthesis of lipid D. Himachal Pradesh


70. The City of Cartagena, which is famous for
C. Synthesis of protein
Protocol on Biosafety, is located in
D. Synthesis of steroid hormone
A. Colombia B. Venezuela
64. Which cell organelles mainly functions as a C. Brazil D. Guyana
storehouse of digestive enzymes?
71. Which one of the following is the most
A. Desmosome B. Ribosome populated State in India as per Census
C. Lysosome D. Vacuoles 2011?

65. Which one of the following tissues is A. Goa B. Mizoram


responsible for increase of girth in the stem C. Meghalaya D. Sikkim
of a plant?

PAGE 575
www.byjusexamprep.com

72. Which among the following countries of A. Rooftop recharge pit


South America does the Tropic of Capricorn
B. Recharge wells
not pass through?
C. Gully plug
A. Chile B. Bolivia
D. Recharge trench
C. Paraguay D. Brazil
77. If one plots the tank irrigation in India and
73. Which one of the following is not correct superimposes it with map of well irrigation,
about Sargasso Sea? one may find that the two are negatively
A. It is characterized with anticyclonic related. Which of the following statements
circulation of ocean currents explain the phenomenon?
B. It records the highest salinity in Atlantic 1. Tank irrigation predates well irrigation.
Ocean 2. Tank irrigation is in the areas with
C. It is located west of Gulf Stream and impervious surface layers.
east of Canary Current 3. Well irrigation requires sufficient
D. It confined in gyre of calm and ground water reserves.
motionless water 4. Other forms of irrigation are not
74. Match List - I with List - II and select the available.
correct answer using the code given below Select the correct answer using the code
the Lists : given below.
List-I List-II A. 1, 2 and 3 B. 2 and 3 only
(City) (Product)
C. 3 and 4 D. 1 and 4
A. Detroit 1. Motorcar
78. When hot water is placed into an empty
B. Antwerp 2. Diamond cutting water bottle, the bottle keeps its shape and
does not soften. What type of plastic is the
C. Tokyo 3. Steel water bottle made from?
D. Harbin 4. Shipbuilding A. Thermoplastic B. PVC
C. Polyurethane D. Thermosetting
Codes: 79. Which of the following is/are state function
A. A-3, B-4, C-2, D-1 / functions?

B. A-3, B-2, C-4, D-1 1. q + w 2. q

C. A-1, B-4, C-2, D-3 3. w 4. H - TS

D. A-1, B-2, C-4, D-3 Select the correct answer using the code
given below.
75. Which one of the following is not situated
on Varanasi - Kanyakumari National A. 1 and 4 only B. 1, 2 and 4
Highway? C. 2, 3 and 4 D. 1 only
A. Satna B. Rewa 80. For a certain reaction, DGq = -45 kJ/mol and
DHq = -90 kJ/mol at 0 °C. What is the
C. Katni D. Jabalpur
minimum temperature at which the
76. Which one of the following methods is not reaction will become spontaneous, if DHq
suitable for urban rainwater harvesting? and DSq are independent of temperature?

PAGE 576
www.byjusexamprep.com

A. 273 K B. 298 K C. entire elastic region of the stress strain


curve
C. 546 K D. 596 K
D. elastic as well as plastic region of the
81. The PCl5 molecule has trigonal bipyramidal stress strain curve
structure. Therefore, the hybridization of p
orbitals should be 86. Which one of the following statements
regarding histone proteins is correct?
A. sp2 B. sp3
A. Histones are proteins that are present
C. dsp2 D. dsp3 in mitochondrial membrane
82. In spherical polar coordinates [γ, β, α], θ B. Histones are proteins that are present
denotes the polar angle around z-axis and a in nucleus in association with DNA
denotes the azimuthal angle raised from x-
C. Histones are proteins associated with
axis. Then the y-component of P is given by lipids in the cytosol
A. P sin θ sin α B. P sin θ cos α D. Histones are proteins associated with
carbohydrates in the cytosol
C. P cos θ sin α D. P cos θ cos α
87. Which one of the following statements
83. For an ideal gas, which one of the following
regarding haemoglobin is correct?
statements does not hold true?
A. Haemoglobin present in RBC can carry
A. The speed of all gas molecules is same only oxygen but not carbon dioxide
B. The kinetic energies of all gas molecules B. Haemoglobin of RBC can carry both
are not same oxygen and carbon dioxide
C. The potential energy of the gas C. Haemoglobin of RBC can carry only
molecules is zero carbon dioxide
D. There is no interactive force between D. Haemoglobin is only used for blood
the molecules. clotting and not for carrying gases.
84. What is constellation? 88. Which one of the following is the correct
sequence of passage of light in a compound
A. A particular pattern of equidistant stars microscope?
from the earth in the sky
A. Condenser - Objective lens - Eye - piece
B. A particular pattern of stars that may - Body tube
not be equidistant from the earth in the
sky B. Objective lens - Condenser - Body tube
- Eye piece
C. A particular pattern of planets of our
C. Condenser - Objective lens - Body tube
solar system in the sky
- Eyepiece
D. A particular pattern and satellites in the
D. Eyepiece - Objective lens - Body tube -
sky due to their position in the space
Mirror
85. The Hooke's law is valid for 89. Which one of the following statements is
A. only proportional region of the stress correct?
strain curve A. Urea is produced in liver
B. entire stress strain curve B. Urea is produced in blood

PAGE 577
www.byjusexamprep.com

C. Urea is produced from digestion of C. West Bengal


starch
D. Andaman and Nicobar Islands
D. Urea is produced in lung and kidney
95. Which one of the following cities was not
90. Which one of the following river valleys of included in the list of smart cities in India?
India is under the influence of intensive
gully erosion? A. Silvassa B. Jorhat

A. Kosi B. Chambal C. Itanagar D. Kavaratti


C. Damodar D. Brahmaputra 96. Find the correct arrangement of the
following urban agglomerations in
91. Which one of the following may be the true
descending order as per their population
characteristic of cyclones?
size according to Census 2011.
A. Temperate cyclones move from west to
east with westerlies whereas tropical A. Delhi - Mumbai - Kolkata - Chennai
cyclones follow trade winds B. Mumbai - Delhi - Kolkata - Chennai
B. The front side of cyclone is known as C. Mumbai - Kolkata - Delhi - Chennai
the 'eye of cyclone'.
D. Kolkata - Chennai - Mumbai – Delhi
C. Cyclones possess a centre of high
pressure surrounded by closed isobars 97. Match List - I with List - II and select the
correct answer using the code given below
D. Hurricanes are well known tropical
the Lists:
cyclones which develop over mid
latitudes. List-I List-II
92. The Headquarters of the International (Type of (Example)
Tropical Timber Organization is located at Lake)

A. New Delhi B. Yokohama A. Tectonic 1. Lonar Lake


C. Madrid D. Jakarta B. Crater 2. Gangabal Lake
93. Atmospheric conditions are well governed C. Glacial 3. Purbasthali Lake
by humidity. Which one among the
following may best define humidity? D. Fluvial 4. Bhimtal Lake

A. Form of suspended water droplets


caused by condensation Codes:
B. Deposition if atmospheric moisture
A. A-4, B-1, C-2, D-3
C. Almost microscopically small drops of
B. A-4, B-2, C-1, D-3
water condensed from and suspended
in air C. A-3, B-1, C-2, D-4
D. The moisture content of the D. A-3, B-2, C-1, D-4
atmosphere at a particular time and
place 98. The Andaman group of islands and the
Nicobar group of islands are separated by
94. The Shompens are the vulnerable tribal which one of the following latitudes?
group of
A. 8° N latitude B. 10° N latitude
A. Jharkhand
C. 12° N latitude D. 13° N latitude
B. Odisha

PAGE 578
www.byjusexamprep.com

99. Damanganga Reservoir Project with about C. It was not given any name
115 km of minor canals and distributaries is
located in D. It was sanctioned by the United Nations

A. NCT 104. Which one of the following statements


about the National Green Tribunal is not
B. Dadra and Nagar Haveli correct?
C. Puducherry
A. It was set up in the year 2010
D. Goa
B. It is involved in effective and
100. Consider the following statements relating expeditious disposal of cases relating to
to Coal India Limited: environmental protection and
1. It is designated as a 'Maha Ratna' conservation of forests.
company under the Ministry of Coal. C. It may consider giving relief and
2. It is the single largest coal producing compensation for damages to persons
company in the world. and property
3. The Headquarters of Coal India Limited D. It is bound by the procedures laid down
is located at Ranchi Jharkhand. under the Code of Civil Procedure, 1908
Which of the statements given above is/are 105. Which of the following statements about
correct? the provisions of the Constitution of India
A. 1 only B. 1 and 2 only with regard to the State of Jammu and
Kashmir is not correct?
C. 2 and 3 only D. 1, 2 and 3
A. The Directive Principles of State Policy
101. Afro-Asian solidarity as a central element of do not apply
India's foreign policy was initiated by which
of the following Prime Ministers? B. Article 35A gives some special rights to
the permanent residents of the State
A. Narendra Modi B. I.K. Gujral
with regard to employment, settlement
C. J.L. Nehru D. Manmohan Singh and property.
102. The Prime Minister's National Relief Fund is C. Article 19(1) (f) has been omitted
operated by which one of the following
bodies? D. Article 368 is not applicable for the
amendment of Constitution of the
A. The Prime Minister's Office (PMO) State.
B. The National Disaster Management
106. In 1921, during which one of the following
Authority
tours, Gandhiji shaved his head and began
C. The Ministry of Finance wearing loincloth in order to identity with
the poor?
D. The National Development Council
(NDC) A. Ahmadabad B. Champaran
103. Which one of the following statements with C. Chauri Chaura D. South India
regard to India's Surgical Strike Mission is
correct? 107. Simla was founded as a hill station to use as
strategic place for billeting troops, guarding
A. It is conducted in the year 2018 frontier and launching campaign during the
B. It was led by the Indian Air Force course of

PAGE 579
www.byjusexamprep.com

A. Anglo-Maratha War 110. Eight States have achieved more than 99%
household electrification prior to the
B. Anglo-Burmese War
launch of 'Saubhagya Scheme'. Which one
C. Anglo-Gurkha War of the following is not among them?
D. Anglo-Afghan War A. Kerala
108. Which politician in British India had B. Punjab
opposed to a Pakistan that would mean
"Muslim Raj here and Hindu Raj C. Himachal Pradesh
elsewhere"?
D. Madhya Pradesh
A. Khan Abdul Ghaffar Khan
111. In October 2018, India was elected as a
B. Sikandar Hayat Khan member to the United Nations Human
C. Maulana Abul Kalam Azad Rights Council for a period of

D. Rafi Ahmed Kidwai A. five years B. four years


109. Match List-I with List-II and select the C. three years D. two years
correct answer using the code given below
the Lists : 112. Consider the following statements about
the Bureau of Pharma PSUs of India (BPPI):
List-I List-II
(Author) (Book) 1. It is the implementing agency of
Pradhan Mantri Bhartiya Janaushadhi
A. Sekhar 1. Jawaharlal Pariyojana (PMBJP).
Bandyopadh Nehru: A
yay Biography, Vol- 2. It has registered as an independent
I, 1889-1947 society under the Societies Registration
Act, 1860.
B. Sarvepalli 2. From Plassey to
Gopal Partition: A Which of the statements given above is/are
History of correct?
Modern India.
A. 1 only B. 2 only
C. David 3. The
Hardiman Ascendancy of C. Both 1 and 2 D. Neither 1 nor 2
the Congress in
113. Consider the following statements about a
Uttar Pradesh,
scheme launched by the Government of
1926 – 1934
India:
D. Gyanendra 4. Gandhi in His
It was launched to provide social security
Pandey Time and Ours
during old age and to protect elderly
persons aged 60 years and above against a
future fall in their interest income due to
Codes:
uncertain market conditions. The scheme
A. A-2, B-4, C-1, D-3 enables old age income security for senior
citizens through provision of assured
B. A-2, B-1, C-4, D-3
pension / return linked to the subscription
C. A-3, B-1, C-4, D-2 amount based on government guarantee to
Life Insurance Corporation of India (LICI).
D. A-3, B-4, C-1, D-2

PAGE 580
www.byjusexamprep.com

Identify the scheme. 118. The 2018, which of the following countries
have legalized the possession and use of
A. Pradhan Mantri Swasthya Suraksha
recreational cannabis?
Yojana
B. Pradhan Mantri Vaya Vandana Yojana 1. America 2. Canada

C. Liveability Index Programme 3. Nigeria 4. Uruguay

D. Rashtriya Vayoshri Yojana Select the correct answer using the code
given below:
114. Who among the following won India's first
ever gold medal in the International Youth A. 1, 2 and 3
Olympic Games (2018) held in Argentina?
B. 2 and 4 only
A. Neeraj Chopra
C. 1 and 4 only
B. Praveen Chitravel
D. 1, 2 and 4
C. Jeremy Lalrinnunga
119. Which of the following are the benefits of
D. Suraj Panwar
the Pradhan Mantri Jan Arogya Yojana
115. E. K. Janaki Ammal National Award on (PMJAY)?
Taxonomy is administered by the
1. Free treatment available at all public
A. Ministry of Agriculture and Farmers and empanelled private hospitals in
Welfare times of need
B. Ministry of New and Renewable Energy 2. Cashless and paperless access to quality
C. Ministry of Health and Family Welfare health care services
D. Ministry of Environment, Forest and 3. Government provides health insurance
Climate Change cover of up to Rs.5,00,000 per family
per year
116. Which one of the following pairs of military
training institute of India and location is not 4. Pre-existing diseases are not covered
correctly matched?
Select the correct answer using the code
A. Army War College: Mhow given below.
B. High Altitude Warfare School: Gulmarg A. 1 and 3 only
C. Army Air Defence College: Pune
B. 1 , 2 and 3
D. Rashtriya Indian Military College:
Dehradun C. 2 and 4 only

117. Which one of the following viruses is D. 2, 3 and 4


responsible for the recent death of lions in 120. The 11th BRICS in 2019 will be hosted by ?
Gir National Park?
A. China
A. Canine Distemper Virus
B. Nipah Virus B. Russia

C. Hendra Virus C. Brazil

D. Foot and Mouth Disease Virus D. India

PAGE 581
www.byjusexamprep.com

Elementary Mathematics
Direction: Consider the following for the next 5. What is the area of quadrilateral ABCD?
three (03) items :
A. 300 CM2 B. 306 CM2
A cube is inscribed in a sphere. A right circular
cylinder is within the cube touching all the C. 312 CM2 D. 316 CM2
vertical faces. A right circular once is inside the 6. What is the difference between perimeter
cylinder. Their heights are same and the of triangle ABC and perimeter of triangle
diameter of the cone is equal to that of the ADC?
cylinder. |||End|||
A. 4 cm B. 5 cm
1. What is the ratio of the volume of the
sphere to that of cone? C. 6 cm D. 7 cm

A. B. 7 : 2 Direction: Consider the following for the next


two (02) items:
C. D. An equilateral triangle ABC is inscribed in a circle
2. What is the ratio of the volume of the cube of radius 20 3
to that of the cylinder?
7. What is the length of the side of the
A. 4 : 3 B. 21 : 16 triangle?
C. 14 : 11 D. 45 : 32 A. 30 cm B. 40 cm
3. Consider the following statements: C. 50 cm D. 60 cm
1. The surface area of the sphere is 5 8. The centroid of the triangle ABC is at a
distance d from the vertex A. What is d
Times the curved surface area of the
equal to ?
cone.
A. 15 cm2 B. 20 cm2
2. The surface area of the cube is equal to
the curved surface area of the cylinder. C. D.
Which of the above statements is/are
correct? Direction: Consider the following for the next
two (02) items:
A. 1 only B. 2 only
The sum of length, breadth and height of a
C. Both 1 and 2 D. Neither 1 nor 2 cuboid is 22 cm and the length of its diagonal is
Direction: Consider the following for the next 14 cm.
three (03) items: 9. What is the surface area of the cuboid?
ABCD is a quadrilateral with AB = 9 cm, BC = 40 A. 288 cm2
cm, CD = 28 cm, DA = 15 cm and angle ABC is a
right – angel B. 216 cm2
3. What is the area of triangle ADC? C. 144 cm 2
A. 126 CM2 B. 124 CM2 D. Cannot be demined due to insufficient
data
C. 122 CM2 D. 120 CM2

PAGE 582
www.byjusexamprep.com

10. If S is sum of the cubes of the dimensions of 16. Three consecutive integers form the
the cuboid and V is its volume, then what is lengths of a right-angled triangle. How
(S-3V) equal to? many sets of such three consecutive
integers is/are possible?
A. 572 cm3 B. 728 cm3
A. Only one
C. 1144 cm3 D. None of these
B. Only two
11. If then which one of C. Only three
the following statements is correct? D. Infinitely many
A. Each fraction is equal to 1 or -1 17. Two circles are drawn with the same
centre. The circumference of the smaller
B. Each fraction is equal to 1/2 or 1
circle is 44 cm and that of the bigger circle
C. Each fraction is equal to 1/2 or -1 is double the smaller one. What is the area
between these two circles?
D. Each fraction is equal to 1/2 only
A. 154 square cm
12. If (4a +7b) (4c-7d) = (4a -7b) (4c +7d, then
which one of the following is correct ? B. 308 square cm
C. 462 square cm
A. B.
D. 616 square cm
C. D. 18. A rectangular red carpet of size 6 ft × 12 ft
has a dark red border 6 inches wide. What
13. Tushar takes 6 hours to complete a piece of is the area of the dark red border?
work, while Amar completes the same work A. 9 square feet
in 10 hours. If both of them work together,
then what is the time required to complete B. 15 square feet
the work? C. 17 square feet
A. 3 hours D. 18 square feet
B. 3 hours 15 minutes 19. The perimeter of a right-angled triangle
C. 3 hours 30 minutes is k times the shortest side. If the ratio of
the other side to hypotenuse is 4 : 5, then
D. 3 hours 45 minutes what is the value of k?
A. 2 B. 3
14. What is the value of ?
C. 4 D. 5
A. 1 B. 2
20. A 12 m long wire is cut into two pieces, one
C. 3 D. 4 of which is bent into a circle and the other
15. In an examination, 52% candidates failed in into a square enclosing the circle. What is
English and 42% failed in Mathematics. If the radius of the circle?
17% failed in both the subjects, then what
percent passed in both the subjects? A. B.
A. 77 B. 58
C. 48 D. 23 C. D.

PAGE 583
www.byjusexamprep.com

21. The angles of a triangle are in the ratio Which of the above statements is/are
1 : 1 : 4. If the perimeter of the triangle is k correct?
times its largest side, then what is the value
A. 1 only B. 2 only
of k?
C. Both 1 and 2 D. Neither 1 nor 2
A. B. 26. Two equal circles intersect such that each
passes through the centre of the other. If
the length of the common chord of the
C. D. 2 circles is 10√3 cm, then what is the
diameter of the circle?
22. The hypotenuse of a right-angled triangle
A. 10 cm B. 15 cm
10 cm and its area is 24 cm2. If the shorts
side is halved and the longer side is double, C. 20 cm D. 30 cm
the new hypotenuse becomes 27. Consider the following grouped frequency
A. B. distribution:

C. D.

23. Consider the following statements:


1. An isosceles trapezium is always cyclic.
2. Any cyclic parallelogram is a rectangle.
Which of the above statements is/are
correct? If the mean of the above data is 25.2, then
A. 1 only what is the value of p?

B. 2 only A. 9 B. 10

C. Both 1 and 2 C. 11 D. 12

D. Neither 1 nor 2 28. Consider the following frequency


distribution:
24. A ladder is resting against a vertical wall and
its bottom is 2.5 m away from the wall. If it
slips 0.8 m down the wall, then its bottom
will move away from the wall by 1.4 m.
What is the length of the ladder?
A. 6.2 m B. 6.5 m
C. 6.8 m D. 7.5 m
25. Consider the following statements:
1. The number of circles that can be
drawn through three non-collinear
points is infinity. What is the median for the distribution?
2. Angle formed in minor segment of a A. 6 B. 7
circle is acute.
C. 8 D. 9

PAGE 584
www.byjusexamprep.com

29. Diagrammatic representation of data C. Abscissa of the vertices of the polygon


includes which of the following? — Class marks of the frequency
distribution
1. Bar diagram
D. Area of the polygon — Total frequency
2. Pie-diagram
of the distribution
3. Pictogram
34. If the lengths of two parallel chords in a
Select the correct answer using the code circle of radius 10 cm are 12 cm and 16 cm,
given below: then what is the distance between these
two chords?
A. 1 and 2 only B. 2 and 3 only
A. 1 cm or 7 cm
C. 1 and 3 only D. 1, 2 and 3
B. 2 cm or 14 cm
30. The data collected from which one of the
following methods is not a primary data? C. 3 cm or 21 cm
A. By direct personal interviews D. 4 cm or 28 cm
B. By indirect personal interviews 35. Considering two opposite vertices of a
square of side ‘a’ as centres, two circular
C. By schedules sent through enumerators
arcs are drawn within the square joining the
D. From published thesis other two vertices, thus forming two
sectors. What is the common area in these
31. If the mean of m observations out of n two sectors?
observations is n and the mean of
remaining observations is m, then what is
the mean of all n observations? A. B.

A. B. C. D.

36. If p and q are the roots of the equation x2 –


C. D. 15x + r = 0 and p – q = 1, then what is the
value of r ?
32. Which one of the following pairs is correctly
A. 55 B. 56
matched?
C. 60 D. 64
A. Median — Graphical location
37. For the inequation x2 – 7x + 12 > 0, which
B. Mean— Graphical location
one of the following is correct ?
C. Geometric mean — Ogive
A. 3 < x < 4
D. Mode — Ogive
B. – ∞ < x < 3 only
33. The following pairs relate to frequency
C. 4 < x < ∞ only
distribution of a discrete variable and its
frequency polygon. Which one of the D. – ∞ < x < 3 or 4 < x < ∞
following pairs is not correctly matched?
38. The expression 52n – 23n has a factor?
A. Base line of the — X-axis polygon
A. 3 B. 7
B. Ordinates of the — Class frequencies
vertices of the polygon C. 17 D. None of these

PAGE 585
www.byjusexamprep.com

39. What is the value of sin 46o cos 44o + cos 46o
C. D.
+ sin 44o?
A. sin 2o B. 0 45. What is the value of sin2 25o + sin2 65o?
C. 1 D. 2 A. 0 B. 1
o
40. Suppose 0 < θ < 90 , then for every θ 4 sin2 C. 2 D. 4
θ + 1 is greater than or equal to ?
46. What is the value of sin6 θ + cos6 θ + 3 sin2
A. 2 B. 4 sin θ θ cos2 θ – 1?
C. 4 cos θ D. 4 tan θ A. 0 B. 1
41. Consider a regular hexagon ABCDEF. Two C. 2 D. 4
towers are situated at B and C. The angle of
elevation from A to the top of the tower at 47. Consider the following for real numbers α,
B is 30o, and the angle of elevation to the β, γ and δ ?
top of the tower at C is 45o. What is the 1. sec α = 1/4
ratio of the height of towers at B and C?
2. tan β = 20
A. 1 : 3 B. 1 : 3 3. cosec γ = 1/2
C. 1 : 2 D. 1 : 2 3 4. cos δ = 2
42. What is the value of tan 1o tan 2o tan 3o ……. How many of the above statements are not
Tan 89o? possible?
A. 0 B. 1 A. one B. two
C. 2 D. ∞ C. three D. four
43. There are two parallel streets each directed Direction: Consider the following for the next
north to south. A person in the first street three (03) items:
travelling from south to north wishes to
In a certain town of population size 1,00,000
take the second street which is on his right
three types of newspapers (I, II and III) are
side. At some place, he makes a 150o turn
available. The percentages of the people in the
to the right and he travels for 15 minutes at
town who read these papers are as follows:
the speed of 20 km/hr. After that he takes
a left turn of 60o and travels for 20 minutes
at the speed of 30 km/hr in order to meet
the second street. What is the distance
between the two streets?
A. 7.5 km B. 10.5 km
C. 12.5 km D. 15 km
48. What is the number of people who read
44. If 3 tan θ = cot θ where 0 < θ < , then only one newspaper?
what is the value of θ ? A. 20,000 B. 25,000
C. 30, 000 D. 35,000
A. B.
49. What is the number of people who read at
least two newspaper?

PAGE 586
www.byjusexamprep.com

A. 12,000 B. 13,000 C. positive only if a ≠ b


C. 14, 000 D. 15,000 D. positive if an only if a > b
50. What is the number of people who do not
55. What is equal to ?
read any of these three newspapers?
A. 62,000 B. 64,000 A. B.

C. 66,000 D. 68,000 C. D. 1.068


56. The inequality 3N > N3 holds when
51. Suppose n is a positive integer such that (n2
+ 48) is a perfect square. What is the A. N is any natural number
number of such n?
B. N is natural number greater than 2
A. One B. Two C. N is a natural number greater than 3
C. Three D. Four D. N is a natural number except 3
57. Which one of the following is an irrational
52. For x = , what is the value of
number?

? A.

B.
A. 1 B. 2
C. 0.45454545
C. 3 D. 2
D. 0.1211221112221111122222
53. Consider the following statements in
respect of two integers p and q (both > 1) 58. What is the largest value of n such that 10n
which are relatively prime: divides the product

1. Both p and q may be prime numbers. 25 × 33 × 48 × 53 × 67 × 76 × 812 × 99 × 106 ×


1512 × 2014 × 2211 × 2515 ?
2. Both p and q may be composite
A. 65 B. 55
numbers
C. 50 D. 45
3. one of p and q may be prime and the
other composite. 59. How many pairs (A, B) are possible in the
number 479865AB if the number is divisible
Which of the above statements are by 9 and it is given that the last digit of the
correct? number is odd?
A. 1 and 2 only B. 2 and 3 only A. 5 B. 6
C. 1 and 3 only D. 1, 2 and 3 C. 9 D. 11
54. For any two real numbers a and b. 60. Priya’s age was cube of an integral number
(different from 1) four years ago and square
is of an integral number after four years. How
long should she wait so that her age
A. always zero becomes square of a number in the
previous year and cube of a number in the
B. never zero next year?

PAGE 587
www.byjusexamprep.com

A. 7 years B. 12 years 66. What is the least number of complete years


in which a sum of money put out at 40%
C. 14 years D. 21 years
annual compound interest will be more
61. Which of the following statements is not than trebled?
true?
A. 3 B. 4
A. The difference of two prime numbers,
both greater than 2, is divisible by 2. C. 5 D. 6

B. For two different integers m, n and a 67. In a circle of radius 8 cm, AB and AC are two
prime number p, if p divides the chords such that AB = AC = 12 cm. What is
product m × n, then p divides either m the length of chord BC?
or n.
A. B.
C. If a number is of the form 6n – 1 (n
being a natural number), then its is a C. D.
prime number.
68. Consider the following inequalities in
D. There is only one set of three prime respect of any triangle ABC:
numbers such that there is a gap of 2
between two adjacent prime numbers. 1. AC – AB < BC

62. Which of the following pair of numbers is 2. BC – AC < AB


the of the equation 3x+2 + 3–x = 10 ?
3. AB – BC < AC
A. 0, 2 B. 0, –2
Which of the above are correct?
C. 1, –1 D. 1, 2
A. 1 and 2 only
63. It is given that log10 2= 0.301 and log10 3 =
0.477. How many digits are there in (108)10? B. 2 and 2 only

A. 19 B. 20 C. 1 and 3 only
C. 21 D. 22 D. 1, 2 and 3
64. The sum of three prime numbers is 100. If 69. Consider the following statements:
one of them exceeds another by 36, then
one of the numbers is 1. The perimeter of a triangle is greater
than the sum of its three medinas.
A. 17 B. 29
2. In any triangle ABC, if D is any point on
C. 43 D. None of these BC, then AB + BC + CA > 2AD.
65. Given that the polynomial (x2 + ax + b)
Which of the above statements is/are
leaves that same remainder when by (x – 1)
correct?
or (x + 1) What are the values of a and b
respectively? A. 1 only B. 2 only
A. 4 and 0 C. Both 1 and 2 D. Neither 1 nor 2
B. 0 and 3 70. In a rectangle, length is three times its
C. 3 and 0 breadth. If the length and the breadth of
the rectangle are increased by 30% and
D. 0 and any interger 10% respectively, then its perimeter
increases by

PAGE 588
www.byjusexamprep.com

B. 8, 6, 8, 6, 7, 8
A. B. 20%
C. 6, 8, 8, 7, 8, 6
C. 25% D. 27% D. 8, 6, 8, 8, 6, 7
71. What is the percentage decrease in the 77. Three cars A, B and C started from a point
area of a triangle if its each side is halved? at 5 pm, 6 pm and 7 pm respectively and
A. 75% B. 50% travelled at uniform speeds of 60 km/hr, 80
km/hr and x km/hr respectively in the same
C. 25% D. No change direction. If all the three met at another
72. The volume of a spherical balloon is point at the same instant during their
increased by 700%. What is the percentage journey, then what is the value of x?
increase in its surface area? A. 120 B. 110
A. 300% B. 400% C. 105 D. 100
C. 450% D. 500% 78. For x > 0, what is the minimum value of

73. What is equal

to ? A. 1
A. 1 B. 0 B. 2
1
C. D. 3 C. 2
2
74. If ax = by = cz and b2 = ac, then what is D. Cannot be determined
equal to ?
79. If , then what are the non-

zero s of x?
A. B.
A.
C. D.
B.
75. If tan x = 1, 0 < x < 90o, then what is the
value of 2 sin x cos x?
C.
A. B. 1
D.
C. D. 3
80. Radha and Hema are neighbours and study
76. Consider the multiplication 999 × abc = in the same school. Both of them use
def132 in decimal notation, where a, b, c, d, bicycles to go to the school. Radha’s speed
e and f are digits. What are the values of a, is 8 km/hr whereas Hema’s speed is 10
b, c, d, e and f respectively? km/hr. Hema takes 9 minutes less than
Radha to reach the school. How far is the
A. 6, 6, 8, 6, 8, 7

PAGE 589
www.byjusexamprep.com

school from the locality of Radha and C. x + 4 D. x + (x/54)


Hema?
87. Consider two-digit numbers which remain
A. 5 km B. 5.5 km the same when the digits interchange their
positions. What is the average of such two-
C. 6 km D. 6.5 km digit numbers?
81. The number 3521 is divided by 8. What is the A. 33 B. 44
remainder?
C. 55 D. 66
A. 1 B. 3
88. The monthly expenditure of a person is Rs.
C. 7 D. 9 6000. The distribution of expenditure on
82. If a, b and c are positive integers such that various items is as follows:

then what is the mean

of a, b and c?
A. 1 B. 2 If the above data is represented by a
percentage bar diagram of height 15 cm,
C. 1.33 D. 2.33 then what are the lengths of the two
83. If an article is sold at a gain of 6% instead of segments of the bar diagram corresponding
a loss of 6% the seller gets Rs.6 more. What to education and miscellaneous
is the cost price of the article? respectively?

A. Rs. 18 B. Rs. 36 A. 1.25 cm and 5 cm

C. Rs. 42 D. Rs. 50 B. 1.2 cm and 4.15 cm


C. 1.2 cm and 3.5 cm
84. If 3x = 4y = 12z, then z is equal to
D. 4.15 cm and 6 cm
A. xy B. x + y
89. x, y and z are three numbers such that x is
C. D. 4x +3y 30% of z and y is 40% of z. If x is p% of y,
then what is the value of p?
85. The corners of a square of side ‘a’ are cut A. 45 B. 55
away so as to form a regular octagon. What
is the side of the octagon? C. 65 D. 75

A. B. 90. In a hostel the rent per room is increased by


20%. If number of rooms in the hostel is
also increased by 20% and the hostel is
C. D. a/3 always full, then what is the percentage
change in the total collection at the cash
86. The average of 50 consecutive natural counter?
numbers is x. What will be the are average
when the next four natural numbers are A. 30% B. 40%
also included? C. 44% D. 48%
A. x + 1 B. x + 2

PAGE 590
www.byjusexamprep.com

91. A race has three parts. The speed and time A. 75 seconds B. 90 seconds
required to complete the individual parts
for a runner is displayed on the following C. 105 seconds D. 135 seconds
chart: 96. In a class of 100 students, the average
weight is 30 kg. If the average weight of the
girls is 24 kg and that of the boys is 32 kg,
then what is the number of girls in the
class?
What is the average speed of this runner?
A. 25 B. 26
A. 8.17 kmph B. 8.00 kmph
C. 7.80 kmph D. 7.77 kmph C. 27 D. 28

92. A field can be reaped by 12 men or 18 97. If a : b = c : d = 1 : 6, then what is the value
women in 14 days. In how many days can 8
men and 16 women reap it? of ?

A. 26 days B. 24 days
C. 9 days D. 8 days A. B.

93. A man who recently died left a sum of Rs.


3,90,000 to be divided among his wife, five C. D.
sons and four daughters. He directed that
each son should receive 3 times as much as 98. What is the remainder when (1729 + 1929) is
each daughter receives and that each divided by 18 ?
daughter receives and that each daughter
should receive twice as much as their A. 6 B. 2
mother receives. What was the wife’s C. 1 D. 0
share?
99. A person divided a sum of Rs. 17,200 into
A. Rs. 14,000 B. Rs. 12,000
three parts and invested at 5%. 6% and 9%
C. Rs. 10,000 D. Rs. 9,000 per annum simple interest. At the end of
two years, he got the same interest on each
94. What is the unit place digit in the expansion
part of money. What is the money invested
of 773?
at 9%?
A. 1 B. 3
A. Rs. 3,200 B. Rs. 4,000
C. 7 D. 9
C. Rs. 4,800 D. Rs. 5000
95. A plane is going in circles around an airport.
The plane takes 3 minutes to complete one 100. A prime number contains the digit X at
round, The angle of elevation of the plane unit’s place. How many such digits of X are
from a point P on the ground at time t possible?
seconds is equal to that at time (t + 30)
seconds. At time (t + x) seconds, the plane A. 3 B. 4
flies vertically above the point P. What is x C. 5 D. 6
equal to?

PAGE 591
www.byjusexamprep.com

ANSWERS KEY I 2019


ENGLISH

1 C 21 A 41 A 61 A 81 D 101 D
2 B 22 A 42 A 62 A 82 A 102 A
3 A 23 C 43 C 63 B 83 D 103 C
4 C 24 A 44 B 64 D 84 A 104 D
5 B 25 B 45 C 65 A 85 D 105 C
6 C 26 C 46 A 66 C 86 B 106 A
7 B 27 A 47 B 67 D 87 C 107 A
8 A 28 D 48 B 68 A 88 B 108 D
9 D 29 C 49 A 69 C 89 B 109 B
10 D 30 B 50 C 70 A 90 D 110 B
11 C 31 B 51 C 71 D 91 C 111 D
12 D 32 A 52 A 72 A 92 C 112 C
13 A 33 B 53 D 73 D 93 C 113 B
14 B 34 D 54 C 74 B 94 D 114 A
15 B 35 C 55 A 75 C 95 D 115 D
16 A 36 B 56 D 76 A 96 C 116 B
17 C 37 A 57 B 77 C 97 B 117 B
18 B 38 C 58 B 78 A 98 A 118 C
19 C 39 D 59 C 79 D 99 A 119 A
20 D 40 A 60 B 80 B 100 B 120 D

GENERAL KNOWLEDGE
1 A 21 C 41 D 61 A 81 D 101 C
2 D 22 D 42 A 62 A 82 A 102 A
3 A 23 B 43 A 63 C 83 A 103 C
4 B 24 D 44 C 64 C 84 B 104 D
5 B 25 D 45 A 65 D 85 A 105 C
6 B 26 A 46 D 66 A 86 B 106 D
7 B 27 A 47 C 67 A 87 B 107 C
8 D 28 B 48 B 68 D 88 C 108 B
9 A 29 D 49 B 69 D 89 A 109 A
10 A 30 D 50 A 70 A 90 B 110 D
11 B 31 A 51 B 71 C 91 A 111 C
12 C 32 A 52 C 72 B 92 B 112 C

PAGE 592
www.byjusexamprep.com

13 C 33 A 53 D 73 C 93 D 113 B
14 D 34 C 54 C 74 D 94 D 114 C
15 B 35 A 55 C 75 A 95 B 115 D
16 B 36 C 56 B 76 C 96 B 116 C
17 A 37 C 57 D 77 B 97 A 117 A
18 D 38 C 58 D 78 D 98 B 118 B
19 B 39 C 59 B 79 A 99 B 119 B
20 B 40 C 60 D 80 C 100 B 120 C

Elementary Mathematics
1 A 21 A 41 B 61 C 81 B
2 C 22 C 42 B 62 B 82 B
3 D 23 C 43 C 63 C 83 D
4 A 24 B 44 A 64 D 84 C
5 B 25 D 45 B 65 D 85 A
6 C 26 C 46 A 66 B 86 B
7 D 27 C 47 C 67 D 87 C
8 C 28 C 48 A 68 D 88 B
9 A 29 D 49 A 69 C 89 D
10 C 30 D 50 D 70 C 90 C
11 D 31 A 51 C 71 A 91 B
12 A 32 A 52 D 72 A 92 C
13 D 33 D 53 D 73 A 93 C
14 D 34 B 54 C 74 C 94 C
15 D 35 C 55 A 75 B 95 C
16 A 36 B 56 D 76 D 96 A
17 C 37 D 57 D 77 A 97 C
18 C 38 C 58 A 78 C 98 D
19 C 39 C 59 A 79 A 99 B
20 B 40 B 60 C 80 C 100 D

PAGE 593
CDS I 2019
www.byjusexamprep.com

(Solutions)
ENGLISH
1. Ans. C.
Let us understand the meaning of the given words :-
Reprimand = to express to someone your strong official disapproval on something.
Extol = praise enthusiastically.
Purported = appearing or stated to be true, though not necessarily so; alleged.
Admonish = warn or reprimand someone firmly.
It is clear from the meanings of the options that “admonished” is a synonym of “reprimanded”.
Hence, option C is the correct answer.
2. Ans. B.
Asset means a useful or valuable thing or person.
Obligation means an act or course of action to which a person is morally or legally bound; a duty or
commitment.
Advantage means a condition or circumstance that puts one in a favourable or superior position.
Attribute means a quality or feature regarded as a characteristic or inherent part of someone or
something.
Reinforcement means the action or process of reinforcing or strengthening.
It is clear from the meanings of the options that “advantage” is a synonym of “asset”. Hence option
B is correct.
3. Ans. A.
Let's us understand the meaning of the given words :-
Hysterical = affected by or deriving from wildly uncontrolled emotion.
Berserk = out of control with anger or excitement; wild or frenzied.
Consistent = staying the same throughout.
Duplicitous = deceitful.
Insincere = not expressing genuine feelings.
Hence, option A is the correct answer.

PAGE 594
www.byjusexamprep.com

4. Ans. C.
Prejudiced means having or showing a dislike or distrust that is derived from prejudice; bigoted.
Objectionable means arousing distaste or opposition; unpleasant or offensive.
Predatory means seeking to exploit others.
Jaundiced means affected by bitterness, resentment, or cynicism.
Intimate means closely acquainted; familiar.
It is clear from the meanings of the options that “jaundiced” is a synonym of “prejudiced”. Hence
option C is correct.
5. Ans. B.
Tautology means the saying of the same thing twice over in different words, generally considered
to be a fault of style.
Prolixity means extended to great, unnecessary, or tedious length; long and wordy.
Foretelling means predict (the future or a future event).
Telepathic means supposedly capable of transmitting thoughts to other people and of knowing
their thoughts; psychic.
It is clear from the meanings of the options that “prolixity” is a synonym of “tautology”. Hence
option C is correct.
6. Ans. C.
Let us understand the meaning of the given words :-
Fanatical = filled with excessive and single-minded zeal.
Bigoted = obstinately or unreasonably attached to a belief, opinion, or faction, and intolerant
towards other people's beliefs and practices.
Rabid = having or proceeding from extreme or fanatical support of or belief in something.
Moderate = average in amount, intensity, quality, or degree.
Militant = favoring confrontational or violent methods in support of a political or social cause.
Hence, option C is the correct answer.
7. Ans. B.
Let us understand the meaning of the given words :-
Heretics = a person holding an opinion at odds with what is generally accepted.
Dissenter = a person who dissents.
Believer = a person who believes in the truth or existence of something.
Renegade = a person who deserts and betrays an organization, country, or set of principles.
Apostate = a person who renounces a religious or political belief or principle.
Hence, option B is correct answer.

PAGE 595
www.byjusexamprep.com

8. Ans. A.
Let us understand the meaning of the given words :-
Inert = lacking the ability or strength to move.
Lively = full of life and energy; active and outgoing.
Quiescent = in a state or period of inactivity or dormancy.
Dormant = (of an animal) having normal physical functions suspended or slowed down for a period
of time; in or as if in a deep sleep.
Apathetic = showing or feeling no interest, enthusiasm, or concern.
Hence, option A is the correct answer.
9. Ans. D.
Let us understand the meaning of the given words :-
Misanthropic = having or showing a dislike of other people; unsociable.
Antisocial = not sociable or wanting the company of others.
Philosophic = relating or devoted to the study of the fundamental nature of knowledge, reality, and
existence.
Atrophic = (of body tissue or an organ) having atrophied.
Philanthropic = (of a person or organization) seeking to promote the welfare of others; generous
and benevolent.
Hence, option D is the correct answer.
10. Ans. D.
Let us understand the meaning of the given words :-
Profound = (of a state, quality, or emotion) very great or intense..
Sincere = free from pretence or deceit; proceeding from genuine feelings.
Erudite = having or showing great knowledge or learning.
Scholarly = involving or relating to serious academic study.
Superficial = not thorough, deep, or complete; cursory.
It is clear from the meanings of the options that “superficial” is an antonym of “profound”.
Hence, option D is the correct answer.
11. Ans. C.
Well-written means written properly in a clear and concise manner.
Decipherable means something that can be read, decoded, and interpreted.
Illegible refers to something that is not clear enough to be read.
Comprehensible means something that can be understood.

PAGE 596
www.byjusexamprep.com

It is therefore clear from the meaning of these words that the meaning of Illegible is the opposite
of ‘readable’. Therefore, option C is the correct answer.
12. Ans. D.
Steadfast means dutifully firm towards something.
Committed means dedicated to or loyal to something wholeheartedly.
Unwavering means steady and firm.
Unfaltering also means firm, steady and resolute.
Unreliable means something/someone who can be trusted or relied upon.
From the meanings of these words, it is clear that ‘Unreliable’ has the meaning opposite to
‘steadfast’. Therefore, D is the correct option.
13. Ans. A.
Tempestuous means characterized by strong or conflicting emotion.
Calm means not showing strong, aggressive or angry emotions.
Violent means involving physical force intended to hurt or damage someone or something.
Fierce means having or displaying an intense or ferocious aggressiveness.
Vehement means showing strong feeling.
It is therefore clear from the meaning of these words that meaning of Tempestuous is the opposite
of Calm. So, the correct answer is A.
14. Ans. B.
Vital means something that is important.
Crucial also refers to something/ someone that is critical and of great importance.
Inessential means not-essential, not absolutely necessary.
Indispensable means absolutely necessary.
Fundamental means something that is central, primary and important.
It is clear from the meanings of these words that Inessential is the antonym of Vital. Therefore, B is
the correct answer.
15. Ans. B.
Wordy means expressed in too many words.
Diffuse means spread out or not concentrated.
Concise means expressing in a clear, comprehensive and brief manner.
Garrulous means very talkative or pointless talking.
Voluble is something characterized by ready or rapid speech.
The meaning of the word ‘concise’ is the opposite of ‘wordy’. So, B is the correct option.

PAGE 597
www.byjusexamprep.com

16. Ans. A.
The verb “ordered” is generally followed by an infinitive and “to compensate” is an infinitive. So, RS
is a pair here. Out of Q and P, only Q makes sense after S. Therefore, we need an option with a
sequence of RSQ and it is only present in option A. Hence option A is correct.
17. Ans. C.
The word “drug” from part R can only be connected to “trade” from part P out of the given options
as “drug trade” is a commonly used phrase and nothing else from the options goes with “drug”. So,
it can be concluded that RP is a pair and it is only present in option C. Hence option C is correct.
18. Ans. B.
Among the given parts, only “the institution” can be the subject of the verb “closed”. So, it can be
concluded that SR is a pair and it is only present in option B. Hence option B is correct.
19. Ans. C.
Among the given parts, only “brushed past” can be the verb of the subject “his vehicle
accidentally”. So, it can be concluded that RP is a pair and it is only present in option C. Hence
option C is correct.
20. Ans. D.
“Death toll” can be only be caused “earthquake or tsunami” out of the given options. So, RP is a
pair here and out of S and Q, only “said” can be the verb for “agency” which is in Q. Therefore, we
need an option with a sequence of QRP and it is only present in option D. Hence option D is correct.
21. Ans. A.
The adjective “serious” goes with only “illness” so SQ is pair and it leaves “new” goes with “DNA”
so PR is a pair. These are only present in option A. Hence option A is correct.
22. Ans. B.
Option B is correct - SQRP.
Correct sentence - Bees might be the best known pollinators but researchers have found that
Common white-footed ants are the best pollinators of a rare evergreen tree in the Southern
Western Ghats.
23. Ans. C.
“And” is a coordinating conjunction. We use 'and' to connect two words, phrases, clauses or
prefixes together (similar grammatical items). R begins with “and” followed by a verb and S ends
with a verb; so “and” can connect the end verb of S and the initial verb of R. Therefore, it can be
concluded that SR is a pair and it is only given in option C. Hence, option C is the correct sequence.
24. Ans. A.
Fraction P has the subject and the verb of the sentence. Next fraction could be either Q or S as both
have objects. However, the sequence PQRS does not form a coherent sentence whereas sequence
PSRQ does. Hence, option A is the correct answer.

PAGE 598
www.byjusexamprep.com

25. Ans. B.
The given sentence when rearranged cannot begin with P, Q and R as none of them have the
subject. The subject of the sentence is given in part S which means the sentence can only begin
with S. Out of the given options, only option B has the sequence beginning with S. Therefore, SPRQ
is the correct sequence.
26. Ans. C.
The phrase “a match made in heaven” means “a marriage that is happy and successful because the
partners are very compatible.” It matches option C. Hence, it is the correct answer.
27. Ans. A.
The phrase “a culture vulture” means “a person who is very interested in the arts.” It matches
option A. Hence option A is correct.
28. Ans. D.
The phrase “a death blow” means “an action or event that causes the sudden and complete end of
something important.” It matches option D. Hence option D is correct.
29. Ans. C.
The phrase “the jewel in the crown” means “the most valuable or successful part of something.” It
matches option C. Hence, it is the correct answer.
30. Ans. B.
If you say that someone is living in a fool's paradise, you are criticizing them because they are not
aware that their present happy situation is likely to change and get worse. Option C is the correct
choice.
31. Ans. B.
The phrase “a rotten apple” means “a bad or corrupt person in a group, especially one whose
behavior is likely to have a detrimental influence on the others.” It matches option B. Hence option
B is correct.
32. Ans. A.
The phrase “to vote with your feet” means “to show your opinion by leaving an organization or by
no longer supporting, using, or buying something.” It matches option A. Hence, it is the correct
answer.
33. Ans. B.
The phrase “verbal diarrhoea” means “the quality or habit of talking too much.” It matches option
B. Hence, it is the correct answer.
34. Ans. D.
The phrase "sail close to the wind” means “to behave in a manner that is on the verge of being
dangerous, improper or illegal.” It matches option D. Hence, it is the correct answer.
35. Ans. C.
The phrase “a double entendre” means “a word or phrase open to two interpretations, one of
which is usually risqué or indecent.” It matches option C. Hence option C is correct.

PAGE 599
www.byjusexamprep.com

36. Ans. B.
The phrase “to cut your own throat” means “to behave in a way that will cause harm to yourself,
especially because you are very offended or angry about something.” It matches option B. Hence it
is the correct answer.
37. Ans. A.
The phrase “cook the books” means alter facts or figures dishonestly or illegally.
e.g. They cooked the books to drive up the company's stock prices.
Hence, option A is the correct answer.
38. Ans. C.
The phrase “change your tune” means “express a very different opinion or behave in a very
different way.” It matches option C. Hence option C is correct.
39. Ans. D.
The phrase “blue blood” means “noble birth.” It matches option D. Hence option D is correct.
40. Ans. A.
The phrase “cut the crap” means “get to the point; state the real situation.” It matches option A.
Hence option A is correct.
41. Ans. A.
Few, when used without a preceding 'a', means "very few" or "none at all". On the other hand, a
few is used to indicate "not a large number". The question talks about the days of monsoon which
are not many in number. So there should be an “a” before few in the question. Hence option A is
correct.
42. Ans. A.
The verb “apprised” does not take “with” as a preposition. It should be replaced with “apprised of”.
So, the error is in part A. Hence option A is correct.
43. Ans. C.
The correct phrase is “contemporary of someone” but the part C “to” is used which is wrong. So,
the error is in part C. Hence option C is correct.
44. Ans. B.
In order to express “to unexpectedly experience trouble, danger, difficulty etc” we use the verb
“meet with” but in part B, only “met” is used which is incorrect. It should be replaced with “met
with”. So, the error is in part B. Hence option B is correct.
45. Ans. C.
The error is in part C of the given sentence.
46. Ans. A.
The correct phrase is “hard won something” but the part A “hardly” is used which is wrong. So, the
error is in part A. Hence option A is correct.

PAGE 600
www.byjusexamprep.com

47. Ans. B.
We can’t replace “no” with “never” as it means “at no time in past”. In part C the sentence
demands “did not” instead of “never”. So, the error is in part B. Hence option B is correct.
48. Ans. B.
We use “the” before superlatives and ordinal numbers. In part B, second is an ordinal number but
there is no “the” before it which is incorrect. So, the error is in part B. Hence option B is correct.
49. Ans. A.
“Hundreds" is used not for a specific number but just to show a large quantity and it is used with
“of”. In part A, the singular form is used instead of correct “hundreds”. So, the error is in part A.
Hence option A is correct.
50. Ans. C.
In part C the subject of clause “beneficial effects” is a plural and the verb used with it is a singular
“is” which is incorrect. It should be replaced with “are”. So the error is in part C. Hence option C is
correct.
51. Ans. C.
In the context of the sentence, the verb” trickling” does not take “out” as a preposition. It should
be replaced with “tricking with”. So, the error is in part C. Hence option C is correct.
52. Ans. A.
“Since” is used to refer to a specific point in time but in part A it is used for period of time which is
incorrect. It should be replaced with “for”. So, the error is in part A. Hence option A is correct.
53. Ans. D.
The given sentence is totally grammatical and has no error. Hence option D is correct.
54. Ans. C.
“Each” takes a singular verb but in part C a plural verb “are” is used. It should be replaced with “is”.
So, the error is in part C. Hence option C is correct.
55. Ans. A.
We always use a plural noun after the word “few” but in part A, singular noun “creature” is used
which is incorrect. So, the error is in part A. Hence option A is correct.
56. Ans. D.
The given sentence is error free. Hence, option D is the correct answer.
57. Ans. B.
The correct phrase is “foundation director” but the part B “foundational director” is used which is
wrong. So, the error is in part B. Hence option B is correct.
58. Ans. B.
The phrasal work “come in” means “to enter” or “to go to work” which is incorrect according to the
context. The context suggests a phrase like “come out” that could mean “to say something in an

PAGE 601
www.byjusexamprep.com

open, honest, or public way that often makes someone feel surprised, embarrassed, or offended”.
So, the error is in part B. Hence option B is correct.
59. Ans. C.
The error is in part C of the given sentence. 'are priceless' should be replaced with 'is priceless'.
60. Ans. B.
“As opposed to” is used to express two contradictory statements but part A and part C are similar
statement. So, the use of “as opposed to” in part B is incorrect. Hence option B is correct.
61. Ans. A.
Reluctant = unwilling and hesitant
Disinclined: unwilling; reluctant
Eager = strongly wanting to do or have something
Fervent = used to describe beliefs that are strongly and sincerely felt or people who have strong
and sincere beliefs
Unrepentant = showing no regret for one's wrongdoings
Thus, option A is the correct answer.
62. Ans. A.
Out of the given options only “Upon and On” are grammatically correct options but the difference
is that 'upon' is far more formal. So, “upon” will be used in the sentence.
63. Ans. B.
Option (b) “painful” is the correct word in context of the passage as war is being discussed and be
its outcome or reason, the word that fits the blank should have a negative context. Thus, the other
options including delightful, pleasant and lovely are incorrect.
64. Ans. D.
Option (d) “crime” is the correct option. As war can never be a duty, responsibility or moral
obligation.
65. Ans. A.
A feeling is an emotion and it is related to thought which is already mentioned in the sentence.
Sentiment is the outcome of feelings. So ‘feeling’ is more contextually correct.
66. Ans. C.
‘and’ and ‘both’ are used in pair as a conjunction. This gives us a hint that “both” is the correct
answer. Thus, other options are eliminated.
67. Ans. D.
“'Held" is the correct option to fill in the blank.
68. Ans. A.
“questions” is the correct option contextually. It is because when one judging a situation ethically,
it is moreover seen as a question of right or wrong.

PAGE 602
www.byjusexamprep.com

69. Ans. C.
“attempt” is the correct option contextually. The author here is admitting his feeling and calls this
process as “attempt”.
70. Ans. A.
“considered” is the correct option contextually. Here, the context is of bringing the question of
right or wrongs into consideration. Rest of the options do not suit the context.
71. Ans. D.
“standpoint” is the correct option contextually. It means a set of beliefs from which opinions are
formed.
72. Ans. A.
“evolution” is the correct option contextually. As in latter part of the passage evolution has been
discussed.
73. Ans. D.
“Laboratory” is the correct option contextually. As in the passage chemical enzymes have been
discussed. Thus, laboratory is the most appropriate option.
74. Ans. B.
“synthesize” is the correct option contextually. It means to produce a substance by a chemical
reaction in plants or animals or other micro-organism. The context here is regarding the evolution
of enzymes, ‘synthesize’ is the correct answer.
75. Ans. C.
“manufacture” is the correct option contextually. It is because the context here is creating biofuels
and pharmaceuticals from enzymes. So, “manufacture” is the correct answer.
76. Ans. A.
The work of antibodies it to fight with the diseases like cancer. The word “combat” is a synonym of
fight and thus, it is the correct option contextually.
77. Ans. C.
RQSP- is the correct order in which sentences should be arranged. When arranged in the correct
sequence, they read as “He no longer dreamed of storms, nor of women, nor of great occurrences,
nor of great fish, nor fights, nor contests of strength, nor of his wife. He simply woke, looked out
through the open door at the moon and unrolled his trousers and put them on. He only dreamed of
places now and of the lions on the beach. They played like young cats in the dusk and he loved
them as he loved the boy. He never dreamed about the boy. He urinated outside the shack and
then went up the road to wake the boy.”
78. Ans. A.
RQSP is the correct order in which sentences should be arranged.
79. Ans. D.
QSRP is the correct order in which sentences should be arranged.

PAGE 603
www.byjusexamprep.com

80. Ans. B.
PRQS is the correct order in which sentences should be arranged.
81. Ans. D.
QSRP is the correct order in which sentences should be arranged.
82. Ans. A.
PQRS is the correct order in which sentences should be arranged.
83. Ans. D.
PRSQ is the correct order in which sentences should be arranged.
84. Ans. A.
SPRQ is the correct order in which sentences should be arranged.
85. Ans. D.
QSPR is the correct order in which sentences should be arranged.
86. Ans. B.
QPSR is the correct order in which sentences should be arranged.
87. Ans. C.
“it profited from the sale of Indian goods in foreign market” is the correct reason as per the
passage.
88. Ans. B.
“the Indian textile was light cotton” is the correct reason as per the passage.
89. Ans. B.
“they pressurized the government….” is the correct option as per the passage.
90. Ans. D.
“literary source” has been cited by the author.
91. Ans. C.
“industrial revolution” is referred in the paragraph.
92. Ans. C.
“Robert Mugabe” is the correct option as per the passage.
93. Ans. C.
“generation forty” is the correct option as per the passage.
94. Ans. D.
“he dismissed Mr. Mnangagwa” is the correct option as per the passage.

PAGE 604
www.byjusexamprep.com

95. Ans. D.
“they did not want…” is the correct option as per the passage.
96. Ans. C.
“ because coup……….. sanction” is the correct option as per the passage.
97. Ans. B.
“because over eating is bad for health” is the correct option as per the passage.
98. Ans. A.
“the bourgeoise” is the correct answer. Bourgeoise refers to the middle class people.
99. Ans. A.
“to escape the heavy…. instead” is the correct option.
100. Ans. B.
“Three or four ….. sleeping” is the correct option.
101. Ans. D.
It has not been specified in the passage. So, option D is the correct answer.
102. Ans. A.
“because they can afford to” is the correct option.
103. Ans. C.
“follow their………. religion” is the correct option.
104. Ans. D.
“A platform….. world” is the correct option.
105. Ans. C.
“he pities them” is the correct option.
106. Ans. A.
“one day…… religion” is the correct option.
107. Ans. A.
The word “respond” is the correct filler for the blank. It is because only “respond” takes the
preposition “to” after it in the given options. Also, the sentence makes complete sense with the use
of the word "respond".
108. Ans. D.
The word “carries” is the correct filler for the blank. It is because the context here is of having
fewer risk and the verb “carry” is also used in context of having something with you all the time.
109. Ans. B.
The word “reduce” is the correct filler for the blank. Reduce means make smaller or less in amount,
degree, or size and the context here is reducing fertility in men and women.

PAGE 605
www.byjusexamprep.com

110. Ans. B.
The sentence gives us a sense that the football match was cancelled due to bad weather. So, the
phrasal verb which means to cancel, will be used in the given context. “Call off” means to cancel
something such a sports event, or to end an activity because it is no longer useful or possible.
111. Ans. D.
The phrase “turned out” is the correct filler for the blank. It means to be disclosed. The context
here is believing something final when no one was doing it earlier. In this context, “turn out” is the
correct answer as it means to be known or discovered finally and surprisingly.
112. Ans. C.
“getting on” is the correct option meaning - perform or make progress in a specified way. The
sentence also talks about enjoying something in a progressive state.
113. Ans. B.
The correct preposition “in”. Preposition “in” is used to denote a location within a larger area.
114. Ans. A.
The sentence shows two state of mind. The person was going to apply for job initially but later he
did not. So, "Put off" is the correct answer, meaning - to delay or move an activity to a later time, or
to stop or prevent someone from doing something
115. Ans. D.
“Although” is the correct option. Although is a conjunction which is used to join two clauses
denoting “despite the fact that”. In the sentence, the person went out without wearing a raincoat
despite the fact that it was raining.
116. Ans. B.
“got away with” is the correct option as it means to escape a situation. In the sentence, the person
parked his car in an un-authorized area but escape the situation without getting into any trouble.
117. Ans. B.
Let's first learn the meanings of the words:
Provocative = causing anger or another strong reaction, especially deliberately.
Inflammatory = intended or likely to cause anger or hate
Dexterous = showing or having skill, especially with the hands
Valiant = possessing or showing courage or determination.
Prudent = acting with or showing care and thought for the future.
Thus, option B is the correct answer.
118. Ans. C.
Let us understand the meaning of the given words :-
Precipitation = water that falls from the clouds towards the ground, especially as rain or snow.

PAGE 606
www.byjusexamprep.com

Drought = a long period when there is little or no rain.


Oasis = a region in desert area where there is greenery due to the presence of water.
Rainfall = water falling in drops from vapour condensed in the atmosphere.
Snowing = fall as snow.
Hence, option C is the correct answer.
119. Ans. A.
Anonymously: Not defined by name; made or done by someone whose name is not known or not
made public
Incognito = having one's true identity concealed
In situ = in the original place
Unfailingly = in a reliable or unchanging way; always.
So, option A is the correct answer.
120. Ans. D.
Terminal = (of a disease) predicted to lead to death, especially slowly; incurable.
Sublunary = situated beneath the moon or between the earth and the moon
Chronic = persisting for a long time or constantly recurring.
Thus, the word “incurable” is the correct answer as terminal diseases are the end-stage incurable
diseases.

PAGE 607
www.byjusexamprep.com

General Knowledge
1. Ans. A.
In 1782 Colebrooke was appointed through his father's influence to a writer-ship with the East a
India Company in Calcutta. Lord Wellesley appointed him honorary professor of Hindu law and
Sanskrit at the college of Fort William.
Hence, Option A is correct.
2. Ans. D.
The Deccan Riots Commission was set up which presented a report to the British Parliament in
1878. In 1879, the Agriculturists Relief Act was passed which ensured that the farmers could not be
arrested and imprisoned if they were unable to pay their debts. It was an Act for the relief of
Indebted Agriculturists in certain parts of the Deccan.
Hence, Option D is correct.
3. Ans. A.
Damini-i-Koh was the land allocated to the Santhals. The Santhals could live on the land and
practise plough agriculture, thus helping them to carry out settled agriculture.
Hence, Option A is correct.
4. Ans. B.
The ryots came to see the moneylender as devious and deceitful. They complained of
moneylenders manipulating laws and forging accounts. In 1859 the British passed a Limitation Law
that stated that the loan bonds signed between moneylenders and ryots would have validity for
only three years. This law was meant to check the accumulation of interest over time. The
moneylender, however, turned the law around, forcing the ryot to sign a new bond every three
years.
Hence, Option B is correct.
5. Ans. B.
Ahmadullah Shah Madrasi alias Danka Shah (1789-1858), a descendant of the Qutub Shahi family of
the Deccan, was in Faizabad when the sepoys mutinied in Meerut. He threw himself into the
movement then and there. This was why he also came to be known as Ahmadullah Shah Faizabadi.
Hence, Option B is correct.
6. Ans. B.
The British land revenue policy further undermined the position and authority of the taluqdars.
After annexation, the first British revenue settlement, known as the Summary Settlement of 1856,
was based on the assumption that the taluqdars were interlopers with no permanent stakes in
land: they had established their hold over land through force and fraud.
Hence, Option B is correct.

PAGE 608
www.byjusexamprep.com

7. Ans. B.
The Inter-State Council was set up in 1990 following the recommendations of the Sarkaria
Commission.
The constitutional roots of the council are to be found in Article 263, which recommends that the
President of India set up such a council to deal with federal issues.
Hence, Option B is correct.
8. Ans. D.
• A writ of certiorari is issued to quash the order passed by an inferior court or tribunal in excess
of jurisdiction.
• A writ of prohibition is issued to prevent an inferior court or tribunal to go ahead with the trial
of a case in which it has assumed excess of jurisdiction.
• Quo Warranto is a legal proceeding during which an individual's right to hold any office or
governmental privilege is challenged.
Hence, Option D is correct.
9. Ans. A.
The Parliament cannot amend those provisions which form the ‘basic structure’ of the Constitution.
This was ruled by the Supreme Court in the Kesavananda Bharati Case. (1973)
Basic Structure of the Indian Constitution is not defined. But according to SC, the basic structure
can be evolved but cannot be destroyed. Hence it falls within the constituent powers of the
Parliament.
Hence, Option A is correct.
10. Ans. A.
The right to protection in respect of conviction for offences (Article 20) and the right to life and
personal liberty (Article 21) remain enforceable even during emergency.
Hence, Option A is correct.
11. Ans. B.
Article 15:
Prohibition of discrimination on grounds of religion, race, caste, sex or place of birth
(1) The State shall not discriminate against any citizen on grounds only of religion, race, caste, sex,
place of birth or any of them
(2) No citizen shall, on grounds only of religion, race, caste, sex, place of birth or any of them, be
subject to any disability, liability, restriction or condition with regard to
(a) access to shops, public restaurants, hotels and palaces of public entertainment; or
(b) the use of wells, tanks, bathing ghats, roads and places of public resort maintained wholly or
partly out of State funds or dedicated to the use of the general public
Hence, Option B is correct.

PAGE 609
www.byjusexamprep.com

12. Ans. C.
Scheduled Caste Federation was founded by Dr. Ambedkar in 1942 to fight for the rights of the
Dalit community. SCF was the successor organization of the Independent Labour Party led by
Ambedkar. SCF later evolved into the Republican Party of India.
Hence, Option C is correct.
13. Ans. C.
Paul Gardner Allen (January 21, 1953 – October 15, 2018) was an American business magnate,
investor, software engineer, humanitarian, and philanthropist. Alongside Bill Gates, Allen co-
founded Microsoft in 1975, which helped spark the microcomputer revolution and later became
the world's largest PC software company.
Hence, Option C is correct.
14. Ans. D.
The Chief Election Commissioner Shri. O.P. Rawat, along with the Election Commissioners Shri Sunil
Arora and Shri Ashok Lavasa launched a mobile app, called ‘Cvigil,’ on July 3, 2018, for citizens to
report any violation of the model code of conduct during elections.
“cVIGIL" is a user-friendly and easy to operate Android application.
Hence, Option D is correct.
15. Ans. B.
The Prahaar is a short-range, solid propellant, road-mobile ballistic missile designed for tactical
strikes against close range targets. It is a surface to surface missile.
Prahaar is expected to replace the Prithvi-I short-range ballistic missile in Indian service.
Hence, Option B is correct.
16. Ans. B.
Indian skipper Virat Kohli was bestowed with Rajiv Khel Ratna award by Indian President Ram Nath
Kovind. Virat Kohli became the third cricketer to receive this honour after Sachin Tendulkar and
Mahendra Singh Dhoni.
Hence, Option B is correct.
17. Ans. A.
Pakyong Airport is a greenfield airport near Gangtok, the state capital of Sikkim, India. The airport,
spread over 400 ha (990 acres), is located at Pakyong town
The airport was inaugurated by India's Prime Minister Narendra Modi on 24th September 2018
Hence, Option A is correct.
18. Ans. D.
On October 15, the United Nations commemorates the International Day of Rural Women.
This year the theme is 'Sustainable infrastructure, services and social protection for gender equality
and the empowerment of rural women and girls.
Hence, Option D is correct.

PAGE 610
www.byjusexamprep.com

19. Ans. B.
• Gobind Behari Lal became the science editor of the San Francisco Examiner.
• He won the Pulitzer Prize for journalism in 1937.
Hence, Option B is correct.
20. Ans. B.
Saurabh Chaudhary is an Indian sport shooter. He comes from a family of farmers living in Yamuna
& Gangetic plains of western Uttar Pradesh. He won the Gold medal at the 2018 Asian Games in 10
m Air Pistol.
Hence, Option B is correct.
21. Ans. C.
There are certain assumptions underlying the law of demand, which are as follows:
i. Assumes that the consumer’s income remains same
ii. Assumes that the preferences of consumer remain same.
iii. Considers that the fashion does not show any changes, because if fashion changes, then people
would not purchase the products that are out of fashion.
iv. Assumes that there would be no change in the age structure, size, and sex ratio of population.
This is because if population size increases, then the number of buyers increases, which, in turn,
affect the demand for a product directly.
Hence, Option C is correct.
22. Ans. D.
Total utility - It is total psychological satisfaction which a consumer derives from the consumption
of a commodity is known as total utility
Marginal utility - It is an addition made in total utility by consuming and additional unit of a
commodity is known as marginal utility.
• When marginal utility is positive, total utility increases
• When marginal utility is zero, total utility is at maximum
• When marginal utility is negative, total utility decreases
Hence, Option D is correct.
23. Ans. B.
The indifference curves cannot intersect each other.
Higher indifference curve represents a higher level of satisfaction because higher IC means a
bundle consisting more of both the goods or same quantity of one good n more quantity of the
other good .
Indifference curves is convex to the point of origin because of diminishing Marginal Rate of
Substitution.
Hence, Option B is correct.

PAGE 611
www.byjusexamprep.com

24. Ans. D.
Joint stock company is a voluntary association of individuals for profit, having a capital divided into
transferable shares, the ownership of which is the condition of membership.
The definition of a joint stock company highlights the following features of a company.
Separate legal entity: From the day of its incorporation, a company acquires an identity, distinct
from its members. Its assets and liabilities are separate from those of its owners. The law does not
recognise the business and owners to be one and the same.
The management and control of the affairs of the company is undertaken by the Board of
Directors.
Hence, Option D is correct.
25. Ans. D.
Although demand curves are typically downward sloping to reflect that consumers’ utility for a
good diminishes with increased consumption, firm supply curves are generally upward sloping.
The upward sloping character reflects that firms will be willing to increase production in response
to a higher market price because the higher price may make additional production profitable.
Hence, Option D is correct.
26. Ans. A.
The Bombay Secretariat was completed in 1874 and designed by Captain Henry St. Clair Wilkins in
the Venetian Gothic style. With its arcaded verandahs and huge gable over the west facade, it was
a monument to the civic pride of Bombay's British rulers.
Hence, Option A is correct.
27. Ans. A.
Mataji Maharani Tapaswini was one of the strongest proponents of female education in India. Her
greatest contribution came in the form of the Mahakali Pathshala which she set up in Kolkata in
1893.
The school was a completely indigenous affair which did not rely on either foreign aid or assistance.
The education of girls was carried out on a strictly national basis in the hopes that they would be
able to revive and regenerate Hindu society.
Hence, Option A is correct.
28. Ans. B.
India was one of the main centres of world trade and industry. Peter the Great of Russia was led to
exclaim:
“Bear in mind that the commerce of India is the commerce of the world and … he who can
exclusively command it is the dictator of Europe.”
Hence, Option B is correct.

PAGE 612
www.byjusexamprep.com

29. Ans. D.
A European traveler, Abbe J.A. Dubois, commented, at the beginning of the 19th century:
"A Hindu woman can go anywhere alone, even in the most crowded places, and she need never
fear the impertinent looks and jokes of idle loungers....A house inhabited solely by women is a
sanctuary which the most shameless libertine would not dream of violating.'
The women of the time possessed title individuality of their own. This does not mean that there
were no exceptions to this rule.
Hence, Option D is correct.
30. Ans. D.
Plagues and Peoples is a book on epidemiological history by William Hardy McNeill published in
New York City in 1976. It was a critical and popular success, offering a radically new interpretation
of the extraordinary impact of infectious disease on cultures as a means of enemy attack. The book
ranges from examining the effects of smallpox in Mexico, the bubonic plague in China, to the
typhoid epidemic in Europe.
Hence, Option D is correct.
31. Ans. A.
Ashish Nandy traces the roots of colonialism to differentiate between culturalism and the cultural
arrogance of Europe. In this way, Indian sociologist Ashish Nandy argued that the idea of a similar
Hindutva was formed by the cultural arrogance of post-Enlightenment Europe.
Hence, Option A is correct.
32. Ans. A.
The United States subprime mortgage crisis was a nationwide financial crisis, occurring between
2007 and 2010, that contributed to the U.S. recession of December 2007 – June 2009.
It was triggered by a large decline in home prices after the collapse of a housing bubble, leading to
mortgage delinquencies and foreclosures and the devaluation of housing-related securities.
Hence, Option A is correct.
33. Ans. B.
Provisions of the Indian Independence Act, 1947
There were six primary provisions in the ‘Indian Independence Act’ 1947.
The third provision made sure the Crown continues its authority in some form or the other; it
stated that the office of the Governor-General will be established in the newly formed countries
and will be the constitutional head of both dominions.
The Governor General was also empowered to adopt and amend the ‘Government of India Act’
1935.
Hence, Option B is correct.

PAGE 613
www.byjusexamprep.com

34. Ans. C.
The total number of members in the Legislative Council of a State having such a Council shall not
exceed one-third of the total number of members in the Legislative Assembly of that State:
Provided that the total number of members in the Legislative Council of a State shall in no case be
less than forty.
Of the total number of members of the Legislative council of a State- as nearly as may be, one-third
shall be elected by electorates consisting of members of municipalities, district boards and such
other local authorities in the State as Parliament may by law specify;
Hence, Option C is correct.
35. Ans. A.
The act provides for a five-year term of office to the panchayat at every level. However, it can be
dissolved before the completion of its term.
the state legislature shall provide for the reservation of offices of chairperson in the panchayat at
the village or any other level for the SCs and STs.
The state legislature may make provisions with respect to the maintenance of accounts by the
panchayats and the auditing of such accounts.
Hence, Option A is correct.
36. Ans. C.
Article 323A provides that Parliament may by law establish tribunals for adjudication of disputes
concerning recruitment and conditions of service of persons appointed to public service under
Central, State or any Local or other authority or a corporation owned or controlled by the
Government of India.
The law made by Parliament for the purpose may specify the jurisdiction and procedure of the
tribunals.
Hence, Option C is correct.
37. Ans. C.
Monopolistic competition is a type of imperfect competition such that many producers sell
products that are differentiated from one another (e.g. by branding or quality) and hence are not
perfect substitutes.
In other words, large sellers selling the products that are similar, but not identical and compete
with each other on other factors besides price.
Hence, Option C is correct.
38. Ans. C.
According to the Survey, inflation in the country continued to moderate during 2017-18 with the
CPI based headline inflation averaging 3.3 per cent during the period -- the lowest in the last six
financial years

PAGE 614
www.byjusexamprep.com

Retail inflation fell to a record low of 2.18% in May as prices of kitchen staples like vegetables and
pulses declined sharply although there was a marginal spike in fruit rates.
Hence, Option C is correct.
39. Ans. C.
The life-cycle theory of consumption, popularly known as life-cycle hypothesis,' was developed by
Ando and Modigliani" in the early 1960s.
The life-cycle hypothesis postulates that individual consumption in any time period depends on (i)
resources available to the individual, (ii) the rate of return on his capital, and (iii) the age of the
individual.
Hence, Option C is correct.
40. Ans. C.
J.M Keynes’ theory of employment is a demand-deficient theory. This means that Keynes visualised
employment from the demand side of the model. His theory is a demand-oriented approach, as
opposed to the classical supply side model.
According to him, the volume of employment in a country depends on the level of effective
demand of people for goods and services. Thus, unemployment is attributed to the deficiency of
effective demand.
Hence, Option C is correct.
41. Ans. D.
Adam Smith propounded the following first four canons of taxation -
(1) Canon of Equity. (2) Canon of Certainty: (3) Canon of Convenience or Ease (4) Canon of
Economy.
Hence, Option D is correct.
42. Ans. A.
Muḥammad ibn Mūsā al-Khwārizmī, formerly Latinized as Algorithmi, was a Persian scholar who
produced works in mathematics, astronomy, and geography under the patronage of the Caliph Al-
Ma'mun of the Abbasid Caliphate.
He also developed the concept of the algorithm in mathematics, which is why some have called
him the "grandfather of computer science".
Hence, Option A is correct.
43. Ans. A.
The Kansas-Nebraska Act was passed by the U.S. Congress on May 30, 1854. It allowed people in
the territories of Kansas and Nebraska to decide for themselves whether or not to allow slavery
within their borders. The Act served to repeal the Missouri Compromise of 1820 which prohibited
slavery north of latitude 36 °30 ´.
Hence, Option A is correct.

PAGE 615
www.byjusexamprep.com

44. Ans. C.
The deal would indirectly bring India under purview of NSG and US laws that would not allow India
to conduct nuclear test in the future. The issue of nuclear test was later clarified as the moratorium
on nuclear test was unilateral and voluntary and there was no pressure on India from outside.
Hence, Option C is correct.
45. Ans. A.
The doctrine of independence is not to be raised to the level of a dogma so as to enable the
judiciary to function as a kind of super-legislature or super-executive. The judiciary is there to
interpret the Constitution or adjudicate upon the rights...”
Hence, Option A is correct.
46. Ans. D.
Peacekeeping is flexible and over the past two decades has been deployed in many configurations.
There are currently 14 UN peacekeeping operations deployed on four continents.
Today's multidimensional peacekeeping operations are called upon not only to maintain peace and
security, but also to facilitate the political process, protect civilians, assist in the disarmament,
demobilization and reintegration of former combatants; support the organization of elections,
protect and promote human rights and assist in restoring the rule of law.
Hence, Option D is correct.
47. Ans. C.
The South China Sea disputes involve both island and maritime claims among several sovereign
states within the region, namely Brunei, the People's Republic of China (PRC), Republic of China
(Taiwan), Malaysia, the Philippines, and Vietnam.
Although Indonesia is not part of claims in the South China Sea dispute, after Joko Widodo became
President of the country in 2014, he instituted a policy in 2015 that, if any foreign fishermen were
caught illegally fishing in Indonesian waters, their vessels would be destroyed.
Hence, Option C is correct.
48. Ans. B.
The Kyoto Protocol is an international agreement linked to the United Nations Framework
Convention on Climate Change, which commits its Parties by setting internationally binding
emission reduction targets.
Recognizing that developed countries are principally responsible for the current high levels of GHG
emissions in the atmosphere as a result of more than 150 years of industrial activity, the Protocol
places a heavier burden on developed nations under the principle of "common but differentiated
responsibilities."
Hence, Option B is correct.

PAGE 616
www.byjusexamprep.com

49. Ans. B.
The Beijing Declaration was a resolution adopted by the UN at the end of the Fourth World
Conference on Women on 15 September 1995. The resolution adopted to promulgate a set of
principles concerning the equality of men and women.
Hence, Option B is correct.
50. Ans. A.
Gujral Doctrine is considered as a milestone in India's foreign policy. It was rendered by Mr. Inder
Kumar Gujral, the Minister of External Affairs in H. D. Deve Gowda Government in 1996. This theory
says that "India" as a bigger country of South Asia should give one-sided concession to her small
neighbours and have cordial relations with them.
Hence, Option A is correct.
51. Ans. B.

HCHO: The carbon and and the oxygen are bonded through a double bond which counts as "one
electron pair". Hence the molecule has three electron pairs and is trigonal planar.
Hence, Option B is correct.
52. Ans. C.
Sometimes the solid particles in a liquid are very small and can pass through a filter paper. For such
particles, the filtration technique cannot be used for separation. Such mixtures are separated by
centrifugation.
Hence, Option C is correct.
53. Ans. D.
The sodium fusion test, or Lassaigne's test, is used in elemental analysis for the qualitative
determination of the presence of foreign elements, namely halogens, nitrogen, and sulphur, in an
organic compound. It was developed by J. L. Lassaigne.
Silver fluoride is soluble in water and does not precipitate and thus this method cannot be used for
detection of fluorine.
Hence, Option D is correct.
54. Ans. C.
The haloalkanes (also known as halogenoalkanes or alkyl halides) are a group of chemical
compounds derived from alkanes containing one or more halogens.
Halogens can’t form more than one bond in the context of common organic compounds, and can’t
form multiple bonds to carbon.
Hence, Option C is correct.

PAGE 617
www.byjusexamprep.com

55. Ans. C.
Fischer projections were originally proposed for the depiction of carbohydrates and used by
chemists, particularly in organic chemistry and biochemistry.
A Fischer projection always uses an eclipsed conformation. A Newman diagram can represent
either an eclipsed conformation or a staggered conformation or a skew conformation.
Hence, Option C is correct.
56. Ans. B.
First of all, nylon 6 is only made from one kind of monomer, a monomer called caprolactam. Nylon
6,6 is made from two monomers, adipoyl chloride and hexamethylene diamine.
Hence, Option B is correct.
57. Ans. D.
• Proxima Centauri is the nearest star to earth after sun.
• It is a red dwarf star with a mass about an eighth of the Sun's mass.
• It is about 4.2 light-years from Earth.
• It was discovered in 1915 by Robert Innes and is the nearest known star to the Sun.
Hence, Option D is correct.
58. Ans. D.

Hence, Option D is correct.

PAGE 618
www.byjusexamprep.com

59. Ans. B.
As it is said that density of gold is greater than that of density of silver, so silver occupies a greater
volume than gold. So it has more upthrust from the water, although they both show the same
weight under water, there is more silver present. Hence silver piece will weigh more.
Hence, Option B is correct.
60. Ans. D.

Infrared radiation, what we experience as heat when we hold our hand near a warm object, is
somewhat longer wavelengths than visible light. Visible light is simply electromagnetic radiation in
a range of wavelengths that our eyes are sensitive to. Visible wavelengths range from 0.0007
milimeters for red light, through orange, yellow, green, and blue, to 0.0004 milimeters for violet
light. Ultraviolet is shorter wavelengths than violet.
Hence, Option D is correct.
61. Ans. A.
Both when accelerated through a potential difference of 100KV, they will have the same energy.
The mass of the electron is less than the mass of a proton, hence it will move faster (momentum).
Hence, Option A is correct.
62. Ans. A.

|A| + |B| = + =A+B

|A + B| = =A+B

Hence option A is correct


63. Ans. C.
Smooth Endoplasmic Reticulum does not contain ribosomes on its surface for which it is regarded
as smooth. So, proteins cannot be synthesised by SER provided protein synthesis is the primary task
of ribosomes.
Hence, Option C is correct.

PAGE 619
www.byjusexamprep.com

64. Ans. C.
The lysosome is found in nearly all types of eukaryotic cells (cells with a clearly defined nucleus)
and is responsible for the digestion of macromolecules, old cell parts, and microorganisms.
Each lysosome is surrounded by a membrane that maintains an acidic environment within the
interior via a proton pump.
Hence, Option C is correct.
65. Ans. D.
• In vascular plants, the lateral meristem is one of two meristems in which growth occurs in order
to increase stem girth.
• A meristem is the region of plant tissue found on the tips of roots and shoots. It is where cell
division occurs to produce new growth.
Hence, Option D is correct.
66. Ans. A.
Saprophytic fungi feed on dead plant and animal remains. Many are extremely beneficial, breaking
down this organic material into humus, minerals and nutrients that can be utilised by plants.
Agaricus is one among them.
Hence, Option A is correct.
67. Ans. A.
Asterias has bilateral symmetry only at larval stage but fivefold symmetry (pentamerism, a special
type of radial symmetry) as adults.
Nereis do not possess any symmetry elements in their body organisation.
Hence, Option A is correct.
68. Ans. D.
Warm-blooded creatures, like mammals and birds, try to keep the inside of their bodies at a
constant temperature. They do this by generating their own heat when they are in a cooler
environment, and by cooling themselves when they are in a hotter environment. To generate heat,
warm-blooded animals convert the food that they eat into energy. Peacock and camel are warm
blooded animals.
Hence, Option D is correct.
69. Ans. D.
Presently, there are 226 flood forecasting stations consisting of 166 level forecasting stations for
villages and towns and 60 inflow forecasting stations for reservoirs/dams/barrages in the country.
Existing flood forecasting network covers 22 States / Union Territories and 19 major river basins /
sub-basins in the country.
Hence, Option D is correct.

PAGE 620
www.byjusexamprep.com

70. Ans. A.
Cartagena is a port city on Colombia’s Caribbean coast. By the sea is the walled Old Town, founded
in the 16th century, with squares, cobblestone streets and colorful colonial buildings.
Hence, Option A is correct.
71. Ans. C.

Hence, Option C is correct.


72. Ans. B.
The South American countries that lie within the Tropic of Capricorn are Argentina, Brazil, Chile and
Paraguay. Brazil is the only country in the world that crosses both the Equator and either tropic.
Hence, Option B is correct.
73. Ans. A.
The calm centre of the anticyclonic gyre in the North Atlantic, comprising a large eddy of surface
water, the boundaries of which are demarcated by major current systems such as the Gulf Stream,
Canaries Current, and North Atlantic Drift. The Sargasso Sea is a large, warm (18 °C), saline, which is
characterized by an abundance of floating brown seaweed (Sargassum).
Hence, Option A is correct.
74. Ans. D.

Hence, Option D is correct.


75. Ans. A.
Satna is not situated on Varanasi-Kanyakumari National Highway.
Hence, Option A is correct.
76. Ans. C.
In urban areas, rain water available from roof tops of buildings, paved and unpaved areas goes
waste. This water can be recharged to aquifer and can be utilized gainfully at the time of need.
In areas where the surface soil is impervious and large quantities of roof water or surface runoff is
available within a very short period of heavy rainfall, the use of trench/ pits is made to store the
water in a filter media and subsequently recharge to ground water through specially constructed
recharge wells.
Hence, Option C is correct.

PAGE 621
www.byjusexamprep.com

77. Ans. B.
The Tank irrigation is more in the rocky plateau area of the county, where the rainfall is uneven and
highly seasonal. The Eastern Madhya Pradesh, Chhattisgarh, Orissa, Interiors of Tamil Nadu and
some parts of Andhra Pradesh have more land under tank irrigation.
Well Irrigation is common in alluvial plains of the country except the deserts of Rajasthan.
Hence, Option B is correct.
78. Ans. D.
Thermoset, or thermosetting, plastics are synthetic materials that strengthen during being heated,
but cannot be successfully remoulded or reheated after their initial heat-forming. This contrasts
with thermoplastics, which soften when heated and harden and strengthen after cooling.
Thermoplastics can be heated, shaped and cooled as often as necessary without causing a chemical
change, while thermosetting plastics will burn when heated after the initial moulding.
Hence, Option D is correct.
79. Ans. A.
Q and w are path functions while q+w i.e.H, enthalpy is state function. Similarly, H-TS i.e. G, Gibbs
free energy is state function.
Hence, Option A is correct.
80. Ans. C.
DGq = -45 KJ/mol
DHq = -90 KJ/mol
T = 273 °K
Using the formula,
DGq = DHq – TDS
-45 = -90 – 273 DS
DS = 0.164
Now since DHq and DSq are independent,
DH 45
T2 = = = 273 °K
DS 0.164
Required Temp = 273 + 273 = 546 °K
Hence, Option C is correct.
81. Ans. D.
The P atom needs five orbitals to form the five P-Cl bonds. It has a 3s and three 3p orbitals, so it
must use one of its 3d orbitals to form the fifth bond. These orbitals are hybridized to form
fivesp3d orbitals that just happen to point in the right directions to form a trigonal bipyramid.
Hence, Option D is correct.

PAGE 622
www.byjusexamprep.com

82. Ans. A.
The Cartesian Coordinates can be retrieved from spherical coordinates (l,q,a) by:
X = r.sinq.cosa
Y = r.sinq.sina
Z = r.sinq
Using the same we get for P,
Y = P.sinq.sina
Hence, Option A is correct.
83. Ans. A.
In an ideal gas, at any particular time, different particles in the gas have different speeds and hence
different kinetic energies. This assumption is reasonable because as the particles collide, we expect
their speed to change. Even if initial speed of all the particles was same, the molecular collisions
will disrupt this uniformity.
Hence, Option A is correct.
84. Ans. B.
A constellation is a group of stars that forms an imaginary outline or pattern on the celestial
sphere, typically representing an animal, mythological person or creature, a god, or an inanimate
object. Stars may or may not be equidistant from the earth.
Hence, Option B is correct.
85. Ans. A.
Hooke's law states that within the elastic limit, stress developed is directly proportional to the
strain produced in a body.
Hooke’s law is valid only in the linear part of stress-strain curve.
Hence, Option A is correct.
86. Ans. B.
Histones are highly alkaline proteins found in eukaryotic cell nucleus that package and order the
DNA into structural units called nucleosomes. They are the chief protein components of chromatin,
acting as spools around which DNA winds, and play role in gene regulation.
Hence, Option B is correct.
87. Ans. B.
During respiration, about 97% of oxygen is transported by Red Blood Cells in blood and the
remaining 3% gets dissolved in the plasma.
Around 20-25% of carbon dioxide is carried by haemoglobin as carbamino-haemoglobin.

PAGE 623
www.byjusexamprep.com

Oxygen enters the blood from the lungs and carbon dioxide is expelled out of the blood into the
lungs. The blood serves to transport both gases. Oxygen is carried to the cells. Carbon dioxide is
carried away from the cells.
Hence, Option B is correct.
88. Ans. C.

Hence, Option C is correct.


89. Ans. A.
Urea is produced in the liver and is a metabolite (breakdown product) of amino acids. Ammonium
ions are formed in the breakdown of amino acids. Some are used in the biosynthesis of nitrogen
compounds. Excess ammonium ions are converted to urea.
Hence, Option A is correct.
90. Ans. B.
The Chambal's lower course is lined by belt of badland gullies resulting from accelerated soil
erosion and is the site of a major project in soil conservation.
Chambal Project is a joint project from both Madhya Pradesh and Rajasthan government, which
aims at harvesting the Chambal River for irrigation, power prevention and control of soil erosion in
the valley.
Hence, Option B is correct.
91. Ans. A.
Temperate Cyclones are cyclones of mid-latitudes and hence are primarily under influence of
permanent winds of mid-latitudes i.e. westerlies. Their movement is therefore eastwards of their
origin with average velocity of 32km per hour in summers and 48km per hour in winters.
Movement of Tropical Cyclones Normally, they move from east to west under the influence of
trade winds because trade winds are permanent winds of tropical latitudes. The general direction is
therefore westwards from their origin.
Hence, Option A is correct.
92. Ans. B.
The International Tropical Timber Organization (ITTO) is an intergovernmental organization that
promotes conservation of tropical forest resources and their sustainable management, use and
trade.
Headquarters: Yokohama, Japan
Hence, Option B is correct.

PAGE 624
www.byjusexamprep.com

93. Ans. D.
"Humidity" refers to the presence of water vapor in the atmosphere. It is measured in either
relative terms (relative humidity) or absolute terms (dewpoint temperature). Humidity indicates
the likelihood for precipitation, dew, or fog to be present.
Hence, Option D is correct.
94. Ans. D.
Shompen or Shom Pen are indigenous people of the interior of the Great Nicobar Island, part of the
Indian union territory of Andaman and Nicobar Islands.
Hence, Option D is correct.
95. Ans. B.
Silvassa is in Dadra and Nagar Haveli.
Kavarati is capital of Lakshadweep.
Itanagar is capital of Arunachal Pradesh
These three were originally included in smart city list.
Assam government has sent a proposal for Jorhat to be included in smart city list. But it has not yet
been included.
Hence, Option B is correct.
96. Ans. B.
Among the Million Plus UAs/Cities, there are three very large UAs with more than 10 million
persons in the country, known as Mega Cities. These are Greater Mumbai UA (18.4 million), Delhi
UA (16.3 million) and Kolkata UA (14.1 million). The largest UA in the country is Greater Mumbai
UA followed by Delhi UA.
Hence, Option B is correct.
97. Ans. A.
Lonar Lake, also known as Lonar crater, is a notified National Geo-heritage Monument saline soda
lake located at Lonar in Buldhana district, Maharashtra.
The oxbow lake of Purbasthali sprawls over an area 3.50 km2. Hence fluvial nature.
The Gangbal Lake also called Gangbal Lake, is a lake situated at the foothills of Mount Haramukh in
Ganderbal district, north of Srinagar city in the state of Jammu and Kashmir.
Hence, Option A is correct.
98. Ans. B.
The Ten Degree Channel is a channel that separates the Andaman Islands and Nicobar Islands from
each other in the Bay of Bengal.
Hence, Option B is correct.

PAGE 625
www.byjusexamprep.com

99. Ans. B.
The Daman Ganga also called Dawan River is a river in western India. The river's headwaters are on
the western slope of the Western Ghats range, and it flows west into the Arabian Sea
There is an existing major irrigation project on the river called the Damanganga Reservoir Project,
which is located near Madhuban village in Dharampur taluka of Valsad district of Gujarat.
Hence, Option B is correct.
100. Ans. B.
Headquarters Kolkata, West Bengal, India
Coal India Limited (CIL) is an Indian state-controlled coal mining company headquartered in
Kolkata, West Bengal, India and the largest coal-producing company in the world and a Maharatna
company.
Hence, Option B is correct.
101. Ans. C.
The tone for that 1955 conference on Afro-Asian solidarity was actually set by then Indian Prime
Minister Jawaharlal Nehru almost a decade earlier.
The first large-scale Asian–African or Afro–Asian Conference—also known as the Bandung
Conference was a meeting of Asian and African states, most of which were newly independent,
which took place on 18-24 April 1955 in Bandung, Indonesia.
Hence, Option C is correct.
102. Ans. A.
PMNRF operates from the Prime Minister's Office, South Block, New Delhi-110011 and does not
pays any license fee. PMNRF is exempt under Income Tax Act, 1961 under Section 10 and 139 for
return purposes. Contributions towards PMNRF are notified for 100% deduction from taxable
income under section 80(G) of the Income Tax Act, 1961.
Hence, Option A is correct.
103. Ans. C.
On 29 September 2016, India announced that it conducted "surgical strikes" against militant launch
pads across the Line of Control and inflicted "significant casualties". The Indian Army conducted
surgical strikes against suspected militants.
However, the operation was not given any name.
Hence, Option C is correct.
104. Ans. D.
The legislate Act of Parliament defines the National Green Tribunal Act, 2010 as follows,
"An Act to provide for the establishment of a National Green Tribunal for the effective and
expeditious disposal of cases relating to environmental protection and conservation of forests and
other natural resources including enforcement of any legal right relating to environment and giving

PAGE 626
www.byjusexamprep.com

relief and compensation for damages to persons and property and for matters connected
therewith or incidental thereto". It follow principle of natural justice.
Hence, Option D is correct.
105. Ans. C.
Part IV (dealing with Directive Principles of State Policy) and Part IVA (dealing with Fundamental
Duties) are not applicable to the state.
Articles 19(1)(f) and 31(2) of the Constitution are still applicable to Jammu and Kashmir.
An amendment made to the Constitution of India does not apply to the state unless it is extended
by a presidential order.
Article 35A of the Indian Constitution is an article that empowers the Jammu and Kashmir state's
legislature to define “permanent residents” of the state and provide special rights and privileges to
those permanent residents
Hence, Option C is correct.
106. Ans. D.
On 22nd September 1921, Gandhi made a momentous decision to change his attire. From the
elaborate Gujarati attire, he decided on a simple dhoti and shawl. This epoch-making decision was
taken by Gandhiji in Madurai after he decided that he has to work for and with the with the poor
people of India and how can he identify with them if he wears different clothes from them.
Hence, Option D is correct.
107. Ans. C.
History of Shimla goes back to the period of Anglo – Gurkha war in the year 1808-09. The British
connection with the area developed only after this battle between the Sikh rulers of these hills and
the Gurkhas from Nepal.
Hence, Option C is correct.
108. Ans. B.
“Pakistan would mean a massacre,” the Premier of Punjab Sir Sikandar Hyat Khan predicted to the
distinguished civilian Penderel Moon as early as in October 1938 ( Divide and Quit, page 20). He
told the Punjab Legislative Assembly, on March 11, 1941, “We do not ask for freedom that there
may be Muslim Raj here and Hindu Raj elsewhere. If that is what Pakistan means I will have nothing
to do with it.”
Hence, Option B is correct.
109. Ans. A.

Hence, Option A is correct.

PAGE 627
www.byjusexamprep.com

110. Ans. D.
Saubhagya – ‘Pradhan Mantri Sahaj Bijli Har Ghar Yojana’ was launched in September 2017 with
the aim to provide access to electricity to all remaining households in the country.
Eight States which have already achieved more than 99 percent household electrification prior to
launch of Saubhagya scheme are ineligible for participation under the award scheme. These eight
states are Andhra Pradesh, Gujarat, Goa, Haryana, Himachal Pradesh, Kerala, Punjab and Tamil
Nadu.
Hence, Option D is correct.
111. Ans. C.
In January India will join China and Nepal, besides Pakistan, which were elected to the 47-member
Council in previous years to serve three-year terms.
India received 188 votes, the highest polled by any of the 18 countries elected in the voting.
This is the fifth time India is elected to the Geneva-based Council, the main body of the UN charged
with promoting and monitoring human rights.
Hence, Option C is correct.
112. Ans. C.
Bureau of Pharma PSUs of India (BPPI) is the implementing agency of Pradhan Mantri Janaushadhi
Pariyojana (PMBJP). BPPI was established in December 2008 under the Department of
Pharmaceuticals, Government of India. The Bureau has been registered as an independent society
under the Societies Registration Act, 1860 as a separate independent legal entity in April, 2010.
BPPI follows the provisions of GFR, 2017 as amended from time to time, the CVC guidelines, and
instructions from the Department of Pharmaceuticals.
Hence, Option C is correct.
113. Ans. B.
Pradhan Mantri Vaya Vandana Yojana (PMVVY) is a Pension Scheme announced by the
Government of India exclusively for the senior citizens aged 60 years and above which is available
from 4th May, 2017 to 31st March, 2020.
This is a simplified version of the VPBY and will be implemented by the Life Insurance Corporation
(LIC) of India.
Hence, Option B is correct.
114. Ans. C.
Having won three silver medals, India won its first-ever gold in the history of Youth Olympics when
weightlifter Jeremy Lalrinnunga claimed the top honours in the men's 62 kg category in Buenos
Aires.
Hence, Option C is correct.
115. Ans. D.
In order to encourage work of excellence in taxonomy and also to encourage young students and
scholars to work in this field of science, this award, named after the late Prof. E.K. Janaki Ammal

PAGE 628
www.byjusexamprep.com

was instituted in the year 1999. Late Prof. Janaki Ammal did taxonomic work of outstanding merit
and excellence, particularly in the area of cytotaxonomy and has been a source of inspiration to
many young scientists.
The award is presented by Ministry of Environment, Forest and Climate Change.
Hence, Option D is correct.
116. Ans. C.
The Army Air Defence College, (abbreviated as AADC), is the training academy for the Army Air
Defence Corps of Indian Army. The college is located in the Gopalpur Military Station in Gopalpur,
Odisha.
Hence, Option C is correct.
117. Ans. A.
Canine distemper may have been responsible for the death of four lions even as two more deaths
were reported in Gir, taking the death toll to 23 since September 12.
Canine distemper is a viral disease that impacts a large number of wild animals including cats and
dogs.
Hence, Option A is correct.
118. Ans. B.
Five countries and three states in United States legalized cannabis in some form. Canada set the
tone when it legalized recreational marijuana, becoming the first G-7 country to do it. United State
-not the country- but some states have legalised the use of cannabis.
Hence, Option B is correct.
119. Ans. B.
BENEFITS UNDER THE SCHEME:
• Ayushman Bharat- Pradhan Mantri Jan ArogyaYojana (PMJAY) will provide a cover of up to Rs. 5
lakhs per family per year, for secondary and tertiary care hospitalization.
• Over 10.74 crore vulnerable entitled families (approximately 50 crore beneficiaries) will be
eligible for these benefits.
• PMJAY will provide cashless and paperless access to services for the beneficiary at the point of
service.
Hence, Option B is correct.
120. Ans. C.
• The 2019 BRICS summit is the eleventh annual BRICS summit, an international relations
conference attended by the heads of state or heads of government of the five-member states
Brazil, Russia, India, China and South Africa.
• The summit will be held in Rio de Janeiro, Brazil, the third time Brazil has hosted the summit
after the 2014 summit.
Hence, Option C is correct.

PAGE 629
www.byjusexamprep.com

Elementary Mathematics
1. Ans. A.

The top view of the given assembly will look like the figure above
Outermost is the sphere. Inside that there is a cube and within that there is a cone and cylinder
with same radius.
Here side of cube = a
Diameter of Sphere = body diagonal = √3 a
Radius of sphere = √3 a/2 = r1
Height of Cylinder = Height of cone = side of cube = a = h
Radius of cylinder = Radius of cone = side of cube/2 = a/2 = r2(as shown in the figure)

Volume of sphere/volume of cone = = 6√3:1

2. Ans. C.

The top view of the given assembly will look like the figure above
Outermost is the sphere. Inside that there is a cube and within that there is a cone and cylinder
with same radius.
Here side of cube = a
Diameter of Sphere = body diagnol = √3 a
Radius of sphere = √3 a/2 = r1
Height of Cylinder = Height of cone = side of cube = a = h

PAGE 630
www.byjusexamprep.com

Radius of cylinder = Radius of cone = side of cube/2 = a/2 = r2(as shown in the figure)

Put π = 22/7
= 14/11
3. Ans. D.

The top view of the given assembly will look like the figure above
Outermost is the sphere. Inside that there is a cube and within that there is a cone and cylinder
with same radius.
Here side of cube = a
Diameter of Sphere = body diagnol = √3 a
Radius of sphere = √3 a/2 = r1
Height of Cylinder = Height of cone = side of cube = a = h
Radius of cylinder = Radius of cone = side of cube/2 = a/2 = r2(as shown in the figure)
Surface area of Sphere = 4πr12 = 3πa2
Curved Surface area of cone = πr2L = πr2(h2+r22)1/2 = √5 πa2/4
Surface area of cube = 6a2
Curved Surface area of cylinder = 2πr2h = πa2
Thus neither 1 nor 2 are true
4. Ans. A.

Area of triangle ADC =

PAGE 631
www.byjusexamprep.com

Where s is the semi perimeter of triangle = (AD + DC + CA) / 2 = 15+28+41 / 2 = 42 cm

Area =

= 126 cm2
5. Ans. B.

Area of quadrilateral ABCD = area of triangle ADC + area of triangle ABC


= 126 + ½ * 9 * 40 = 306 cm2
6. Ans. C.

Perimeter of triangle ABC – Perimeter of triangle ADC = (9+40+41)-(15+28+41) = 6cm


7. Ans. D.

Radius of circumcircle of an equilateral triangle = side / √3


R = a/√3
a = R√3 = 20√3 * √3 = 60cm

PAGE 632
www.byjusexamprep.com

8. Ans. C.

For equilateral triangle circumcentre and centroid are the same points
So distance from vertex = radius of circumcircle = 20√3
9. Ans. A.
Let lengths, breadth and height of cuboid be l, b and h respectively
According to question
l+b+h = 22cm……(i)
and √(l2+b2+h2) = 14cm …..(ii)
Surface area of cuboid = 2(lb+bh+lh)
Squaring eq (i) gives
l2+b2+h2 + 2(lb+bh+lh) = 484
Substituting l2+b2+h2 from eq (i)
2(lb+bh+lh) = 484-196 = 288 cm2
10. Ans. C.
Let lengths, breadth and height of cuboid be l, b and h respectively
According to question
l+b+h = 22cm……(i)
and √(l2+b2+h2) = 14cm …..(ii)
S = l3+b3+h3 and V = lbh
S-3V = l3+b3+h3 - 3 lbh = (l+b+h)( l2+b2+h2-[lb+bh+lh])…(iii)
As we know
Squaring eq (i) gives
l2+b2+h2 + 2(lb+bh+lh) = 484
Substituting l2+b2+h2 from eq (i)
2(lb+bh+lh) = 484-196 = 288 cm2
lb+bh+lh = 144 cm2

PAGE 633
www.byjusexamprep.com

Putting this in eq (iii) we get


22(196-144) = 22*52 = 1144cm2
11. Ans. D.
a/(b+c) = b/(c+a) = c/(a+b)=t(say)
This implies
b+c=a/t (1)
c+a=b/t (2)
a+b=c/t (3)
Add both sides of the equation 1, 2 and 3
2a+2b+2c=1/t(a+b+c)
t=1/2
So 1/2 is a possible fraction.
Now suppose if value 1 is possible
Then
a/(b+c) = b/(c+a) = c/(a+b)=1
a=b+c, b=c+a, c=a+b
This gives
a+b+c=2a+2b+2c
1=2
Which is not possible so value 1 is not possible.
Similarly -1 is not possible also.
12. Ans. A.
(4a+7b)(4c-7d) = (4a-7b)(4c+7d)
(4a+7b)/(4a-7b) = (4c+7d)/(4c-7d)
Using componendo and dividendo
(4a+7b)+(4a-7b) / (4a+7b)-(4a-7b) = (4c+7d)+(4c-7d) / (4c+7d)-(4c-7d)
Or 8a/14b = 8c/14d
Or a/b = c/d
13. Ans. D.
Let them take x hours working together
1/x = 1/10 + 1/6 = 8/30
X= 30/8 hours = 15/4 hours = 3hours 45 minutes

PAGE 634
www.byjusexamprep.com

14. Ans. D.

2+ = t (let)
2 + √t = t
Or t-2 = √t
Squaring both sides
t = t2 – 4t + 4
or t2 – 5t + 4 = 0
Or t = 4,1 Now t cannot be equal to 1 as it is clear that it is always greater than 2
So t = 4
15. Ans. D.

No of students failed in English only = 52 – 17 = 35


No of students failed in maths only = 42 – 17 = 25
Total no of failed students in either of the subjects = 35+17+25 = 77
No of passed student in both subjects = 100 – 77 = 23
16. Ans. A.
let n-1, n, n+1 be 3 consecutive integers
So
(n+1)2 = n2 + (n-1)2
(n+1)2- (n-1)2 = n2
4n = n2
So n = 0 or n = 4
n can’t be 0 as n-1 will be negative then
So 3,4 and 5 is the only triplet formed

PAGE 635
www.byjusexamprep.com

17. Ans. C.

Given C1 = 2πr1 = 44 C2 = 2πr2 = 88


r1 = 7 r2 = 14
Area between circles = πr22 – πr12 = 22/7(142-72)
= 462 cm2
18. Ans. C.
Initially carpet is 6×12 = 72 sq feet
Since red border is 6 inches wide from all 4 side
So area without border = 5 × 11 = 55 sq feet
Area of border = total – area without border = 72 – 55 = 17 sq feet
19. Ans. C.
Let other side and hypotenuse be 4x and 5x respectively
Shortest side2 + (4x)2 = (5x)2
Shortest side = 3x
According to question
K*3x = 12x
So k = 4
20. Ans. B.

As it is clear that 2r = a where a is the side of the square and


R is the radius of circle
It is given that 2πr + 4a = 12
a = 12/(π+4)

PAGE 636
www.byjusexamprep.com

21. Ans. A.
4k + k + k = 6x =180 degrees
k= 30 degrees
So triangle is 30,30 and 120 degrees
Let sides of triangle be x,x and y units with y being the largest side opposite to 120 degree angle
Using cosine law
Cos 120 = - sin 30 = -1/2 = (2x2 – y2)/2x2
So 3x2 = y2 …. (i)
Given Perimeter = k (Largest side)
Or 2x+y = ky
Putting value of x from eq (i)
2y/√3 + y = ky
K = 2/√3 + 1
22. Ans. C.
Hypotenuse = 10cm
Let the other 2 perpendicular sides be a and b
Area ½ a*b = 24
So a*b = 48 cm2
Also using Pythagoras
a2 + b2 = 100
(a+b)2 = a2 + b2 + 2ab = 100 + 96 = 196
a+b = 14
Similarly
a-b = 2
So
a=8 and b=6
Now smaller side is halved and larger side is doubled
So a1 = 16 and b1 = 3
New hypotenuse = √(162+32) = √265

PAGE 637
www.byjusexamprep.com

23. Ans. C.

Since it is an isosceles trapezium


So angle C = angle D = x let
A = 180 – D = 180 – x (since AB is parallel to CD)
B = 180-x
A+C = 180 –x + x = 180 degrees (Property of cyclic quadrilateral)

ABCD is cyclic parallelogram with AB // CD and AD // BC


Considering angles
A = C = y (Property of parallelogram)
And
B=D = x
Also since it is cyclic
A+C = B+D = 180degrees
So x=y=90degrees
And also opposite sides are equal being a parallelogram
Thus ABCD is a rectangle
24. Ans. B.

PAGE 638
www.byjusexamprep.com

AB = CD = x = Length of ladder
Let OC = y m
y2 + 3.92 = x2
(y+0.8)2 + 2.52 = x2
So y2 + 3.92 = (y+0.8)2 + 2.52
y = 5.2m
x= √(5.22+3.92)
x= 6.5m
25. Ans. D.
(1) Only one circle can be drawn through 3 non collinear points
(2) Angle in the minor segment is always obtuse
26. Ans. C.

Let there be 2 circles with centre O1 and O


AB is the common chord
Since both passes through the center of each other as shown in figure
So O1O is the radius of both
Let O1O = r = AO1 = AO
AX = AB / 2 = 5√3 cm (since OX perpendicular to chord bisects it)
AOO1 forms an equilateral triangle with on side = radius = r
Sin 60 = √3/2 = AX / AO = 5√3/r
So r = 10cm
So diameter = 20 cm
27. Ans. C.
Mean = (sum of fixi )/ (sum of f) = (8*5 + 12*15 + 10*25 + P*35 + 9*45) / (8+12+10+P+9) = 25.2
(875 + 35P)/(39+P) = 25.2
P = 11

PAGE 639
www.byjusexamprep.com

28. Ans. C.
Summation of frequencies = 6+4+5+8+9+6+4 = 42
Median = mid value = average of 21st and 22nd value
Arranging data in increasing order we get

So mid value i.e 21st and 22nd value = 8


29. Ans. D.
All three are types of data representation
Pictogram uses pictures so show different identities with different numbers
30. Ans. D.
Primary data is information that you collect specifically for the purpose of your research project. An
advantage of primary data is that it is specifically tailored to your research needs. A disadvantage is
that it is expensive to obtain.
31. Ans. A.
Mean of m observations is n
Mean of n-m observations is m
So total = nm + (n-m)m
Total observations = n
Mean = Total / Total observations = (2mn-m2)/n = 2m – m2/n
32. Ans. A.
An ogive (oh-jive), sometimes called a cumulative frequency polygon, is a type of frequency
polygon that shows cumulative frequencies. In other words, the cumulative percents are added on
the graph from left to right. An ogive graph plots cumulative frequency on the y-axis and class
boundaries along the x-axis. Only median can be traced using frequency polygon curve. Thus it has
a graphical location on the curve. Hence the only option correctly matched is option A.
33. Ans. D.
Area of the polygon gives sum of fixi not summation of fi

PAGE 640
www.byjusexamprep.com

34. Ans. B.

Case – 1
When both the chords are in two different halves of the circle

Distance between chords = OM + ON =

Case – 2
When both the chords are in two different halves of the circle

Distance between chords = OM + ON =

35. Ans. C.

Area of leaf BEDFB = Area of two quarter circle – area of square

PAGE 641
www.byjusexamprep.com

= 2πr2/4 – a2
= πa2/2 – a2 = a2(π/2 - 1)
36. Ans. B.
In the below equation,
x2 – 15x + r = 0
sum of roots = p + q = -(-15)/1 = 15 (sum of roots for equation ax2 + bx + c is -b/a)
product of roots = pq = r/1 = r (product of roots for equation ax2 + bx + c is c/a)
given p – q = 1
also we know that p+q = 15
subtracting the squares of both
(p+q)2 + (p-q)2 = 152 – 1
p2 + q2 + 2pq – p2 – q2 +2pq = 225 -1
4pq = 224
4r = 224
r = 56
37. Ans. D.

As we can see from the graph of the quadratic equation, that the value of the equation is greater
than zero for the values of x < 3 and x > 4
38. Ans. C.
52n – 23n = ( 52 )n – ( 23 )n = (25)n – (8)n
We know that an – bn always have a common factor (a - b)
Therefore one of the factor is 25 – 8 = 17
39. Ans. C.
sin 46o. cos 44o + cos 46o. sin 44o
sin 46o. sin (90 - 44)o + cos 46o. cos (90 - 44)o
= sin2 46o + cos2 46o = 1

PAGE 642
www.byjusexamprep.com

40. Ans. B.
We know that,
Arithmetic mean ≥ Geometric mean

(4sin2 θ + 1)/2 ≥
4sin2 θ + 1 ≥ 2. 2 sin θ
4sin2 θ + 1 ≥ 4sin θ
41. Ans. B.

Let the side of regular hexagon be ‘a’


Let height of the tower1 be h1 and tower 2 be h2

Height of tower 1 = h1 = (distance between A and B)* (tan 30o) =

Distance between A and C = 2* =

Height of tower 2 = h2 = (distance between A and C)* (tan 45o) = .1 =

Ratio of height of towers at B and C respectively = =

42. Ans. B.
tan 1o. tan 89o = tan 1o. cot 1o = 1
similarly,
tan 2o. tan 88o = tan 2o. cot 2o = 1
tan 3o. tan 87o = tan 3o. cot 3o = 1
hence the equation will reduce to
tan 45o = 1

PAGE 643
www.byjusexamprep.com

43. Ans. C.

Initially the person is travelling from south to north i.e. D to A


He takes 150o right turn and moves AB distance and then he takes 60o left turn travels BC
AB = 20km/hr * 15/60 hr = 5km
BC = 30 * 20/60 = 10 km
We know that distance between both the streets is DC = DB + BC
DB = AB cos 60o = 5. ½ = 2.5 km
So the distance between streets = 12.5 km
44. Ans. A.
3tan θ = cot θ
3tan θ = 1/tan θ
tan2 θ = 1/3
tan θ = 1/
θ = π/6
45. Ans. B.
sin2 25o + sin2 65o = sin2 25o + sin2 (90 – 25)o = sin2 25o + cos2 25o = 1
46. Ans. A.
sin6 θ + cos6 θ + 3sin2 θ .cos2 θ - 1
sin6 θ + cos6 θ + 3sin2 θ .cos2 θ. 1 - 1
sin6 θ + cos6 θ + 3sin2 θ .cos2 θ. (sin2 θ + cos2 θ ) - 1
(sin2 θ + cos2 θ )3 – 1 = 1 – 1 =0
47. Ans. C.
Sec of any number can never be less than 1
tan can take any value from -∞ to + ∞
cosec of any number can never be less than 1

PAGE 644
www.byjusexamprep.com

cos of any number can never be greater than 1


so option 1,3,4 are not possible
48. Ans. A.

The number of people who read only I , only II and only II are
1 % + 19% + 0% = 20% of total population = 20/100 * 100000 = 20000
49. Ans. A.

As we can see from the above venn diagram the number of people who read two or more
newspapers are
1% + 1% + 3% + 7% = 12% = 12/100 * 100000 = 12000
50. Ans. D.

PAGE 645
www.byjusexamprep.com

Number of people who do not read any of these newspaper = total population – number of people
who read atleast one of these newspapers.
number of people who read atleast one of these newspapers = 1% + 1% + 3% + 1% + 7% + 19% =
32% of total population = 32000
required number of people = 100000 – 32000 = 68000
51. Ans. C.
N2 + 48 =k2
48 =k2 – N2
(k - N)(k + N) = 48
So the possible number of pairs of (k - N) and (k + N) are
(1,48),(2,24), (3,16), (4,12), (6,8)
On solving the above pairs for (k - N) and (k + N), we get the integer values of N and k as
N=1 ,k= 7
N=4 , k=8
N=11,k=13
So the total possible values of N are three
52. Ans. D.

x=

on rationalizing,

x=

x=
putting the value of x in the equation

2+

2+ =2+0=2

53. Ans. D.
All the given statements are true. The following are the examples for all the statements

PAGE 646
www.byjusexamprep.com

Statement 1: Both p and q may be prime numbers. E.g. 3 and 5


Statement 2 : Both p and q may be composite numbers. E.g. 4 and 9
Statement 3 : One of p and q may be prime and the other composite. E.g. 7 and 12
54. Ans. C.
For the equation,

Where a and b are real numbers,


The roots of number is always positive and hence it can be zero only at a=b
So the above equation is positive only when a=b
55. Ans. A.
+

= 0.5353535353….+0.5333333333…..
= 1.068686868 = 1

56. Ans. D.
3N > N3 holds for all the natural numbers except N = 3 at which 3N = N3
57. Ans. D.
A number that cannot be represented in the form p/q where p and q are two integers, is known as
irrational number
. Hence it is rational

is already in the form of rational number

0.4545454545……. can be represented in the form of p/q as 5/9


0.12112211122211112222……… cannot be represented in the form of p/q as there is no recurring
digits in the given number
58. Ans. A.
For the number to be divisible by 10n, it must contain the same powers for 2 and 5
Power of 2 = 25+2.8+7+3.12+6+2.14 + 11 = 25+16+7+36+6+28+11 = 2109
Power of 5 = 53+6+12+14+2.15 = 565
Hence maximum possible power of 10 can be 65 only.
59. Ans. A.
If the number is divisible by 9 the sum of all its digit is divisible by 9
4+7+9+8+6+5+A+B = 39 + A + B is divisible by 9

PAGE 647
www.byjusexamprep.com

Possible values of B are 1,3,5,7,9 as it is given that last digit is odd


For B= 1, A=5
For B = 3, A= 3
For B= 5, A = 1
For B = 7, A = 8
For B = 9, A= 6
60. Ans. C.
Let present age of Priya be p
p-4 = n3
p+4 = √k
since n is a no >1 on putting n= 2 we get p = 12
So p+4 = 16 which is square of an integral number thus consistent with given information
Now after how many years her age becomes such that age – 1 is a square and age + 1 is a cube
Using option if we add 14 years to current age , we get age = 26 years
Here 25 is a square and 27 is a cube thus making 14 the correct answer
61. Ans. C.
Option C is incorrect as 6n – 1 form can be a prime number but it is not necessarily true.
Example 35 is of form 6n-1 but is not a prime number
62. Ans. B.
3x+2 + 3-x = 10
Only powers of 3 that add upto 10 is
32 + 30 = 10
X+2 = 0
X= -2 is consistent
Or x+2 = 2
X= 0 is consistent
Thus x = 0, -2 are the s
Alternatively, we can put values from the options and check.
63. Ans. C.
No of digits in (108)10
We have to find the log of the given number with base 10 and add one to its integral part to find
the no of digits

PAGE 648
www.byjusexamprep.com

log (108)10 = 10 log 108 = 10 log(22 * 33) = 10[2log2 + 3log3]


= 10[2*0.301 + 3*0.477] = 20.33
Integral part = 20
No of digits = 20+1 = 21
64. Ans. D.
Let the three prime numbers be x, y, y+36
x+y+y+36 = 100
x+2y = 64
2y is an even number always
We know that
Even + even = even or odd + odd = even
So x has to be even to satisfy x+2y = 64
The only even prime no is 2
Put x=2
2y = 62
Or y = 31
So the numbers are 2, 31 , 67
Thus option D is the answer
65. Ans. D.
Since x2 + ax + b when divided by x-1 or x+1 leaves the same remainder
So on putting x=1 and x=-1 we get the same value
1+a+b = 1-a+b
2a=0
a=0
here b can take any value as it will always get cancelled out
66. Ans. B.
A = P(

3P < P(

3 < (1.4)^t
When t = 3 ; 1.4^3 = 2.744
And when t = 4; 1.4^4 = 3.8416
T=4 is the answer

PAGE 649
www.byjusexamprep.com

67. Ans. D.

O is the center of circle


Here ABC forms an isosceles triangle as AB=AC=12cm
So AE (a perpendicular bisector) passes through O as OE also bisects chord BC at right angle
AD = DB = 6
In triangle ADO
AO2 = AD2 + DO2
OD = √64 – 36 = √28
Now using similarity
AEB~ADO
AB/AO = EB/DO
12/8 = (BC/2)/ √28
BC=6√7
68. Ans. D.
AC-AB<BC Or AB+BC>AC
BC-AC<AB Or AB+AC>BC
AB-BC<AC Or AC+BC>AB
Sum of 2 sides of triangle is always greater than the third side
So all three statements are true
69. Ans. C.
1) Perimeter of triangle is greater than the sum of 3 medians

Let ABC be the triangle and D. E and F are midpoints of BC, CA and AB respectively.

PAGE 650
www.byjusexamprep.com

Recall that the sum of two sides of a triangle is greater than twice the median bisecting the third
side,(Theorem to be remembered)
Hence in ΔABD, AD is a median
⇒ AB + AC > 2(AD)
Similarly, we get
BC + AC > 2CF
BC + AB > 2BE
On adding the above inequations, we get
(AB + AC) + (BC + AC) + (BC + AB )> 2AD + 2CD + 2BE
2(AB + BC + AC) > 2(AD + BE + CF)
∴ AB + BC + AC > AD + BE + CF
2) To prove: AB + BC + CA > 2AD
Construction: AD is joined
Proof: In triangle ABD,
AB + BD > AD [because, the sum of any two sides of a triangle is always greater than the
third side] ---- 1
In triangle ADC,
AC + DC > AD [because, the sum of any two sides of a triangle is always greater than the
third side] ---- 2
Adding 1 and 2 we get,
AB + BD + AC + DC > AD + AD
=> AB + (BD + DC) + AC > 2AD
=> AB + BC + AC > 2AD
Hence proved
70. Ans. C.
Let the breadth of the rectangle = x
Length of the rectangle will be = 3 times of breadth = 3x
So the initial perimeter = 2(length + breadth) = 2(x + 3x) = 8x
New breadth after increase = x + 10x/100 = 1.1x
New length after increase = 3x + 30*3x/100 = 3.9x
New perimeter = 2(1.1x + 3.9x) = 10x
Percentage change in perimeter = ( 10x-8x )*100/8x = 25%

PAGE 651
www.byjusexamprep.com

71. Ans. A.
Area of triangle of = ½*a*b* sinθ = A
Where a and b are sides of the triangle and θ be the angle between them
After decreasing each side
New area = ½*(a/2)*(b/2)*sinθ = ¼ A
%decrease = [(A – ¼ A)/A ]*100 = 75%
72. Ans. A.
Let the volume of spherical balloon initially = V
New volume after increase = V + 700*V/100 = 8V
Since we know that volume of sphere is directly proportional to the radius of sphere

Final radius = 2* initial radius


Since surface area of sphere is directly proportional to the square of the radius of sphere,()

Final surface area = 4*initial surface area

% change = = 300%

73. Ans. A.
We know that when a+b+c = 0, then
a3 + b3 + c3 = 3abc
in the above question,
(x-y) + (y-z) + (z-x) = 0
Therefore,
(x-y)3 + (y-z)3 + (z-x)3 = 3(x-y)(y-z)(z-x)

PAGE 652
www.byjusexamprep.com

74. Ans. C.
ax = by = cz = k
a = k1/x
b = k1/y
c = k1/z
given b2 = ac, putting the above values of a,b,c in the equation we get
k2/y = k1/x .k1/z
2/y = 1/x + 1/z
75. Ans. B.
tan x = 1
then
x = 45o
2sin x. cos x = =1

76. Ans. D.
999 x abc = def132
We can write the above equation as
(1000 – 1) x abc = def132
abc000 – abc = def000 + 132 = (def +1)x 1000 - 868
on comparing the LHS and RHS, we get
a = 8, b = 6, and c = 8 and d = a = 8 , e = b = 6 and f = c – 1 = 8 -1 = 7
77. Ans. A.
Distance covered by A till 6pm = 60 km
Distance covered by A till 7 pm = 120 km
Time taken by B to catch A = 60/(80-60 ) = 3 hrs
So A and B will meet at 6pm + 3 hrs = 9pm
Since we know that all three met at the same time
The time taken by C to cover 120 km difference will be = 9pm – 7pm = 2hrs
Therefore,
(x – 60 )*2 = 120
x = 120 km/hr

PAGE 653
www.byjusexamprep.com

78. Ans. C.
For x>0 Min of x + (x+2)/2x = ?
x + (x+2)/2x = x + ½ + 1/x
So we have to find the minimum of x+1/x and add ½ to it
As AM>GM
So (x+1/x)/2 > √(x*1/x)
Or x + 1/x > 2
So min of x + (x+2)/2x = 2+1/2 = 5/2
79. Ans. A.

On squaring and cross multiplying, we get

On applying componendo and dividend

On solving the above equation, we get

80. Ans. C.
Let the distance between be D km
Time taken by radha – Time taken by Hema = 9 mins
So D/8 – D/10 = 9/60 hrs
D = 6km
81. Ans. B.
3521/8
As we know 32=9 will leave remainder = 1 when divided by 8
So 3521/8 = [(32)260 * 3]/8 = 1*3/8 = 3/8 Thus remainder is 3

PAGE 654
www.byjusexamprep.com

82. Ans. B.

On comparing equations we get a= 1 , b = 2 and c = 3


Mean = a+b+c/3 = 6/3 = 2
83. Ans. D.
Let CP be Rs x
Then
1.06x – 0.94x = 6
So x = Rs 50
84. Ans. C.
3x = 4y = 12z
Taking log of all 3 we get
xln3 = yln4 = zln12 = k
z = k/ln12 = k / ln(3*4) = k/ln3 + ln4 = k / (k/x +k/y) = xy / (x+y)
85. Ans. A.

Let side of hexagon be x


AE2 + AL2 = LE2
Since we are forming a regular octagon so AE = AL = FB = BG and so on
So AE = SB = x/√2
AE + EF + FB = side of square = a (Given)
So x/√2 + x + x/√2 = a
X = a/(√2+1) = a(√2 - 1)

PAGE 655
www.byjusexamprep.com

86. Ans. B.
Sum of n consecutive natural numbers = n(n+1)/2
Average of n consecutive natural numbers = (n+1)/2
For first 50 consecutive natural numbers average = 51/2 = x
54 consecutive natural numbers average = 55/2 = 51/2+2 = x+2
87. Ans. C.
All such 2 digit numbers are 11,22,33,44……. upto 99
Forms an AP
So sum = n/2(a+l)
= 9/2(11+99)
Average = sum/9 = ½(11+99) = 55
88. Ans. B.
15 cm corresponds to 6000 rs
Education = 480/6000 * 15 cm = 1.2cm
Miscellaneous = 1660/6000 * 15cm = 4.15 cm
89. Ans. D.
x = 30% of z = 30z/100 = 3z/10
y = 40% of z = 40z/100 = 4z/10
According to the question,
(x/y)*100 = p%

p% = = 75%

90. Ans. C.
Let initial rent be rs 10
And initial rooms be 10
So initial collection = 10*10 = Rs 100
Now new rent = 10 + 20% of 10 = 12
New no of rooms = 10 + 20% of 10 = 12
So new collection = 12*12 = 144
% change in collection = (144-100)/100*100 = 44%

PAGE 656
www.byjusexamprep.com

91. Ans. B.

Average speed = Total Distance / Total time =

= (45+64+75)/23
= 184/23
= 8 kmph
92. Ans. C.
12 men or 18 women can complete in 14 days
8 men and 16 women can complete in how many days
12men = 18 women (Comparing efficiencies)
1men = 18/12 = 1.5 women
8 men and 16 women = 12women + 16 women = 28 women
18 women completes in 14 days
1 woman completes in 14*18 days
28 women completes in (14*18)/28 days = 9 days
93. Ans. C.
Let his wife get a share of Rs x
Each of the 4 daughters get = Rs 2x
Each of the 5 sons get = Rs 6x
So x + 4*2x + 5*6x = 390000
So 39x = 390000
X= 10000 = wife’s share
94. Ans. C.

From the above table we can see that the power 73 is of the form 4x + 1
Therefore the unit digit according to the table = 7

PAGE 657
www.byjusexamprep.com

95. Ans. C.

Let the plane be at point A at t seconds and at point B after t + 30 seconds


Since the motion is uniform, we can say that at time t+15 seconds, the plane is above the point is
diametrically opposite to the point P from where the angle is same.
Now since the time taken to cover the full circle is 3 minutes (180 seconds), the time taken by the
plane to reach the diametrically opposite point will be 90 seconds.
So the time after which the plane reaches the point P will be = t+ 15 + 90 seconds = (t + 105)
seconds
96. Ans. A.
By alligation,

So the number of girls will be =( 1/(1+3))*100 = 25


97. Ans. C.
Let a = x then b = 6x
Also let c = y then d = 6y

98. Ans. D.
The number 1729 = (18 - 1)29 when divided by 18 leaves the remainder (-1)29 = 18-1 = 17
The number 1929 = (18 + 1)29 when divided by 18 leaves the remainder (1)29 = 1
Then after adding these two the remainder will be 17+1 = 18 which is divisible by 18
Hence the remainder will be 0

PAGE 658
www.byjusexamprep.com

99. Ans. B.
Let sum invested @ 5% be P1, @ 6% be P2 then @ 9% = 17200-(P1+P2)
So according to question
P1*5*2/100 = P2*6*2/100 or P1 = (6/5) P2
Also P2*6*2/100 = [17200-(P1+P2)]*9*2/100
Or 2 P2 = [17200 – (11/5)P2] * 3
Or (2 + 33/5)P2 = 17200 * 3
P2 = 17200 * 3 * 5 / 43 = 6000
So P1 = 6/5 P2 = 7200
So Sum invested @ 9% = 17200 – (6000+7200) = Rs 4000
100. Ans. D.
For prime no units place cannot be occupied by even number except for 2
Thus no of digits occupying unit digit of prime numbers = 6 (1,2,3,5,7,9)
Example 2,3,5,7,11,19 in itself are prime numbers

PAGE 659
www.byjusexamprep.com

CDS II 2018
ENGLISH
Direction: Each item in this section has a C. A dozen novels and a number of poetry
sentence in active voice followed by four collections were authored by the writer
sentences one of which is the correct passive who passed away recently
voice statement of the same. Select the correct
D. A dozen novels and a number of poetry
one and mark it in the Answer Sheet accordingly.
collections had been authored by the
1. India won freedom with the blood and writer who passed away recently
sweat of hundreds and thousands of
Indians. 4. All the examinees have answered one
particular question in the long answer
A. India had won freedom with the blood writing section.
and sweat of hundreds and thousands of
Indians A. One particular question is answered by
all the examinees in the long answer
B. Freedom had been won by India with
writing section.
the blood and sweat of hundreds and
thousands of Indians B. One particular question was answered
C. Freedom was won by India with the by all the examinees in the long answer
blood and sweat of hundreds and writing section.
thousands of Indians C. All the examinees answered one
D. Freedom was won by hundreds and particular question in the long answer
thousands of Indians with their blood writing section.
and sweat D. One particular question has been
2. Shut the door. answered by all the examinees in the
long answer writing section.
A. Shut the door
B. Let the door be shut 5. The members of the parliament elect their
group leader either by consensus or by
C. The door be shut voice vote.
D. The door is shut A. The group leader is elected by the
3. The writer who passed away recently has members of the parliament either by
authored a dozen novels and a number of consensus or by voice vote
poetry collections.
B. The group leader was elected by the
A. A dozen novels and a number of poetry members of the parliament either by
collections have been authored by the consensus or by voice vote
writer who passed away recently
C. The group leader has been elected by
B. A dozen novels and a number of poetry the members of the parliament either
collections has been authored by the by consensus or by voice vote
writer who passed away recently

PAGE 660
www.byjusexamprep.com

D. The members of the parliament are A. Ramila asked Rahim where his ideas had
elected by their group leader either by been when they had faced the trouble
consensus or by voice vote the week before.
Direction: Each item in this section has a direct B. Ramila asked Rahim where his ideas had
statement followed by its reported form in been when they faced the trouble the
indirect speech. Select the correct statement in last week.
indirect speech and mark it in the Answer Sheet
C. Ramila asked Rahim where his ideas had
accordingly.
been when they faced the trouble the
6. The preacher said to the crowd, “The Sun week before.
rises everyday for all of us without any D. Ramila told Rahim where his ideas were
expectations in return.” when they faced the trouble the week
A. The preacher told the crowd that the before.
Sun rose everyday for all of them 9. The manager said to his colleagues, “We
without any expectations in return have received a serious threat to our
B. The preacher told the crowd that the business now and we need to act to face it.”
Sun rose everyday for all of us without A. The manager told his colleagues that
any expectations in return they had received a serious threat to
C. The preacher told the crowd that the our business then and they needed to
Sun has risen everyday for all of them act to face it
without any expectations in return B. The manager told his colleagues that
they received a serious threat to their
D. The preacher told the crowd that the
business then and they needed to act to
Sun rises everyday for all of them
face it
without any expectations in return
C. The manager said his colleagues that
7. The actor said to his co-star, Sarita, “Will
they had received a serious threat to
you go with me for a cup of tea in the
our business then and they needed to
evening today?”
act to face it
A. The actor said to his co-star if she would D. The manager told his colleagues that
go for a cup of tea with him in evening they had received a serious threat to
today their business at that time and they
B. The actor told his co-star, Sarita if she needed to act to face it
would go with him for a cup of tea in 10. Vivek said to his friend, “Could you please
evening that day turn off the switch?”
C. The actor requests his co-star, Sarita if A. Vivek told his friend to turn off the
she would go with him for a cup of tea switch
in that evening that day
B. Vivek asked his friend to please turn off
D. The actor asked his co-star, Sarita if she the switch
would go with him for a cup of tea in the
evening that day C. Vivek requested his friend to turn off
the switch
8. Romila said to Rahim, “Where were your
ideas when we faced the troubles last D. Vivek told his friend that he should turn
week?” off the switch

PAGE 661
www.byjusexamprep.com

11. The captain said to his soldiers, “Move 14. Efficacy of the project needs an
forward and face the target now.” examination.
A. The captain ordered his soldiers to A. Inefficiency
move forward and face the target
B. Efficiency
B. The captain informed his soldiers that
C. Value
they should move forward and face the
target now D. Effectiveness
C. The captain asked his soldiers to move Directions: Each item in this section consists of
forward and face the target then a sentence with an underlined word followed by
four words/group of words. Select the option
D. The captain told his soldiers that they
that is opposite in meaning to the underlined
move forward and face the target
word and mark your response on your Answer
immediately.
Sheet accordingly.
Directions : Each item in this section consists of
15. The whole audience showed a disdainful
a sentence with an underlined word followed by
attitude during the match.
four words/group of words. Select the option
that is opposite in meaning to the underlined A. Sneering B. Respectful
word and mark your response on your Answer
C. Mocking D. Cheerful
Sheet accordingly.
Directions: Each item in this section consists of
12. The baby could not move as the place was
a sentence with an underlined word followed by
soggy.
four words/group of words. Select the option
A. Sodden B. Dry that is opposite in meaning to the underlined
word and mark your response on your Answer
C. Hot D. Wet
Sheet accordingly.
Directions : Each item in this section consists of
16. The entry was carried out inadvertently.
a sentence with an underlined word followed by
four words/group of words. Select the option A. Purposely B. Purposively
that is opposite in meaning to the underlined
C. Accidently D. Not noticing
word and mark your response on your Answer
Sheet accordingly. Directions: Each item in this section consists of
a sentence with an underlined word followed by
13. Her rebuttal that she was not involved in
four words. Select the option that is opposite in
the case was considered by the court.
meaning to the underlined word and mark your
A. Refusal response on your answer sheet accordingly.
B. Denial 17. This painting has a distinctive element
which can be noticed well.
C. Acceptance
A. Salient B. Common
D. Kindness
C. Great D. Unique
Directions: Each item in this section consists of
a sentence with an underlined word followed by Directions: Each item in this section consists of
four words/group of words. Select the option a sentence with an underlined word followed by
that is opposite in meaning to the underlined four options. Select the option that is opposite
word and mark your response on your Answer in meaning to the underlined word and mark
Sheet accordingly. your response accordingly.

PAGE 662
www.byjusexamprep.com

18. They consider themselves as foes from 22. His ideas are obscure.
birth.
A. new B. clear
A. protagonists B. opponents
C. infamous D. obscene
C. friends D. soul mates Directions: In this section, each item consists of
Directions: Each item in this section consists of six sentences of a passage. The first and the sixth
a part with a bold word/words followed by four sentences are given as S1 and S6. The middle
words. Select the option that is opposite in four sentences in each have been jumbled up
meaning to the underlined word/words and and labeled P, Q, R and S. You are required to
mark your response on your Answer Sheet find the proper sequences of the four sentences
accordingly. and mark your response accordingly on the
Answer Sheet.
19. It appears that the whole group is
mutinous. 23. S1 : The Constitution of India thus emerged
through a process of intense debate and
A. arrogant B. lucky discussion.
C. obedient D. sincere S6 : In countries such as the United States
Directions: Each item in this section consists of and the United Kingdom, only men with
a sentence with an underlined word followed by education were allowed into the charmed
four options. Select the option that is opposite circle.
in meaning to the underlined word and mark P : This was an unprecedented act of faith,
your response accordingly. for in other democracies the vote had been
20. There prevailed a woebegone feeling in the granted slowly, and in stages.
room. Q : However, on one central feature of the
A. sad B. cheerful Constitution there was substantial
agreement.
C. sleepy D. thoughtful
R : Many of the provisions were arrived at
Directions: Each item in this section consists of through a process of give-and-take, by
a sentence with an underlined word/words forging a middle ground between two
followed by four words. Select the option that is opposed positions.
opposite in meaning to the underlined
word/words and mark your response on your S : This was on the granting of the vote to
Answer Sheet accordingly. every adult Indian.

21. Ravi is jovial and he makes the environment A. P R S Q


sanguine. B. R Q S P
A. Pessimistic C. Q S R P
B. Optimistic D. Q S P R
C. Humorous Directions: In this section, each item consists of
six sentences of a passage. The first and the sixth
D. Rebellious
sentences are given as S1 and S6. The middle
Directions: Each item in this section consists of four sentences in each have been jumbled up
a part with an underlined word followed by four and labeled P, Q, R and S. You are required to
words. Select the option that is opposite in find the proper sequences of the four sentences
meaning to the underlined word and mark your and mark your response accordingly on the
response on your Answer Sheet accordingly. Answer Sheet.

PAGE 663
www.byjusexamprep.com

24. S1 : We cannot understand the power of P : To be dealt with equitably is to be


rumours and prophecies in history by treated fairly in comparison with another
checking whether they are factually correct group of people or a relevant other person.
or not.
Q : Equity involves feelings and perceptions
P : The rumours in 1857 began to make and is always a comparative process.
sense when seen in the context of the R : Equity theory states, in effect, that
policies the British pursued from the late people will be better motivated if they are
1820s. treated equitably and demotivated if they
Q : Rumours circulate only when they are treated inequitably.
resonate with the deeper fears and S : It is not synonymous with equality, which
suspicions of people. means treating everyone the same, since
this would be inequitable if they deserve to
R : Under the leadership of Governor
be treated differently.
General Lord William Bentinck, the British
adopted policies aimed at “reforming” A. P Q R S B. P Q S R
Indian society by introducing Western C. R S Q P D. Q P R S
education, Western ideas and Western
Institutions. Directions: In this section, each item consists of
six sentences of a passage. The first and the sixth
S : We need to see that they reflect about sentences are given as S1 and S6. The middle
the minds of people who believed them – four sentences in each have been jumbled up
their fears and apprehensions, their faiths and labeled P, Q, R and S. You are required to
and convictions. find the proper sequences of the four sentences
and mark your response accordingly on the
S6 : With the cooperation of sections of
Answer Sheet.
Indian society they set up English-medium
schools, colleges and universities which 26. S1 : Measurement is an important concept
taught Western sciences and liberal arts. in performance management.

A. S Q P R B. Q S P R S6 : It is more difficult in the case of


knowledge workers for example scientists
C. P R S Q D. R S P Q and teachers.
Directions: In this section, each item consists of P : It also indicates where things are not
six sentences of a passage. The first and the sixth going so well, so that corrective action can
sentences are given as S1 and S6. The middle be taken.
four sentences in each have been jumbled up Q : It identifies where things are going well
and labeled P, Q, R and S. You are required to to provide the foundations for building
find the proper sequences of the four sentences further success.
and mark your response accordingly on the
Answer Sheet. R : It is the basis for providing and
generating feedback.
25. S1 : Equity theory is concerned with the
S : Measuring performance is relatively easy
perception people have about how they are
for those who are responsible for achieving
being treated compared with others.
quantified targets for example sales.
S6 : This explains only one aspect of the
process of motivation and job satisfaction,
although it may be significant in terms of A. R Q P S B. Q P S R
morale. C. P S Q R D. S P Q R

PAGE 664
www.byjusexamprep.com

Directions: In this section, each item consists of Q : It can be said that it is more connected
six sentences of a passage. The first and the sixth with the capitalistic economies.
sentences are given as S1 and S6. The middle R : It empowers the individuals and small
four sentences in each have been jumbled up groups to carry out their plans for their
and labeled P, Q, R and S. You are required to achievement of a common goal.
find the proper sequences of the four sentences
and mark your response accordingly on the S : The decentralized planning is
Answer Sheet. implemented through market mechanism.

27. S1 : It is doubtful if mankind, throughout his A. Q R S P B. S R Q P


long history, has ever lived at all C. S Q R P D. S R P Q
‘sustainably’.
Directions: In this section, each item consists of
S6 : The concept of ‘sustainable’ is so far six sentences of a passage. The first and the sixth
from reality that it is almost laughable. sentences are given as S1 and S6. The middle
P : But in general mankind has regarded the four sentences in each have been jumbled up
environment as an endless ‘resource’ to be and labeled P, Q, R and S. You are required to
exploited and plundered. find the proper sequences of the four sentences
and mark your response accordingly on the
Q : Maybe a few isolated tribal groups Answer Sheet.
found the necessary balance with nature
lived without the desire for endless ‘more’. 29. S1 : Tolstoy Farm was founded in 1910 by
which time Gandhi had already
R : Now we have reached a point where we conceptualized ideas that he would
are on the verge of destroying ourselves develop in India.
and most of the life on earth.
S6 : Both he and Gandhi often referred to
S : This process has accelerated greatly the time that they spent in Tolstoy Farm as
since the industrial revolution. among the happiest in their lives.
A. P Q R S B. Q P S R P : He was rich and used his money to buy
C. P Q S R D. S R Q P the land and help set up the farm.

Directions: In this section, each item consists of Q : A Jewish architect, Kallenbach was by his
six sentences of a passage. The first and the sixth side through this period.
sentences are given as S1 and S6. The middle R : Tolstoy Farm became the subject of
four sentences in each have been jumbled up research for different kinds of cooperative
and labeled P, Q, R and S. You are required to communities across the world.
find the proper sequences of the four sentences
S : He first put in the social, moral, religious
and mark your response accordingly on the
components of his doctrine.
Answer Sheet.
A. Q R S P B. S Q P R
28. S1 : Decentralized planning is a process of
planning that begins from the grassroots C. S Q R P D. R Q P S
level taking into confidence all the Directions: In this section, each item consists of
beneficiaries. six sentences of a passage. The first and the sixth
S6 : But it cannot be described as sentences are given as S1 and S6. The middle
undemocratic for most national states four sentences in each have been jumbled up
adopt such planning now. and labeled P, Q, R and S. You are required to
find the proper sequences of the four sentences
P : Under decentralized planning, the and mark your response accordingly on the
operation is from bottom to top. Answer Sheet.

PAGE 665
www.byjusexamprep.com

30. S1 : Though most of us talk of discipline, R : In other words, you have to be the
what do we mean by that word? perfect teacher to create a new society; and
to bring the perfect teacher into being, you
S6 : And most of us are interested in mass
have to understand yourself.
movements, large schools with a great
many boys and girls; we are not interested S : Therefore you, who are the seeker of
in creative intelligence, therefore we put up truth, have to be both the pupil and the
huge schools with enormous attendances. teacher.
P : The teacher would understand each A. Q R S P B. S R Q P
child and help him in the way required. C. Q S R P D. R Q S P
Q : But if you have five or six in a class, and Directions: In this section, each item consists of
an intelligent understanding teacher with a six sentences of a passage. The first and the sixth
warm heart, I am sure there would be no sentences are given as S1 and S6. The middle
need for discipline. four sentences in each have been jumbled up
R : When you have a hundred boys in a and labeled P, Q, R and S. You are required to
class, you will have to have discipline; find the proper sequences of the four sentences
otherwise there will be complete chaos. and mark your response accordingly on the
Answer Sheet.
S : Discipline in schools becomes necessary
when there is one teacher to a hundred 32. S1 : The giant wall of the Dhauladhar range
boys and girls. in Himachal Pradesh is one of the most
stunning sights in the Himalayas.
A. Q R S P B. S R Q P
S6 : Looming over the hill stations of
C. Q S R P D. R Q P S Dharmsala and McLeodganj, the
Directions: In this section, each item consists of Dhauladhar is a popular trekking
six sentences of a passage. The first and the sixth destination.
sentences are given as S1 and S6. The middle P : As the lifeline of the region it acts as a
four sentences in each have been jumbled up watershed ridge between Chamba’s Ravi
and labeled P, Q, R and S. You are required to river system and Kangra’s Beas river
find the proper sequences of the four sentences system.
and mark your response accordingly on the
Answer Sheet. |||End||| Q : Although of modest altitude compared
to other Himalayan ranges – the highest
31. S1 : Truth is far more important than the Dhauladhar peak is less than 5,000 m.
teacher.
R : Thus, the Dhauladhar could be stated as
S6 : So a teacher must obviously be one the lifeline of the region.
who is not within the clutches of society,
who does not play power politics or seeks S : Despite that, the range sweeps up an
position or authority. astounding 12,000 ft. from the valley floor,
creating a barrier wall in that is striking to
P : Without self-knowledge, the air-plane look at.
becomes the most destructive instrument
A. Q R P S
in life; but with self-knowledge, it is a
means of human help. B. S P Q R
Q : Wisdom begins with self-knowledge; C. Q S R P
and without self-knowledge, more
D. R Q S P
information leads to destruction.

PAGE 666
www.byjusexamprep.com

Directions: Each of the following items in this 35. Elementary education


section consists of a sentence the parts of which
P
have been jumbled. These parts have been
labeled P, Q, R and S and are given below each ensuring the growth of a nation
sentence in four sequences namely (a), (b), (c) Q
and (d). You are required to rearrange the
jumbled parts of the sentence and mark your is inevitable in developing the children
response accordingly. R
33. The Himalayan range to further education, thereby
P. sacred to the Gaddi people S
Q. is home A. Q R P S B. R P Q S
R. to a chain of high altitude lakes C. S P R Q D. P R S Q
S. that towers over the Kangra valley Directions: Each of the following items in this
section consists of a sentence the parts of which
A. Q R P S B. S P Q R have been jumbled. These parts have been
C. S Q R P D. R Q S P labeled P, Q, R and S and are given below each
sentence in four sequences namely (a), (b), (c)
Directions: Each of the following items in this and (d). You are required to rearrange the
section consists of a sentence the parts of which jumbled parts of the sentence and mark your
have been jumbled. These parts have been response accordingly.
labeled P, Q, R and S and are given below each
sentence in four sequences namely (a), (b), (c) 36. We with real life experiences (P) tend to
and (d). You are required to rearrange the learn with interest (Q) when we see beauty
jumbled parts of the sentence and mark your in our work (R) and connect learning (S)
response accordingly. A. Q R S P B. R P Q S
34. P. National Building Organisation besides C. S P R Q D. P R S Q
conducting surveys on housing
Directions: Each of the following items in this
Q. and disseminates the statistical section consists of a sentence the parts of which
information have been jumbled. These parts have been
labeled P, Q, R and S and are given below each
R. collects, tabulates
sentence in four sequences namely (a), (b), (c)
S. on housing and building construction and (d). You are required to rearrange the
activities jumbled parts of the sentence and mark your
response accordingly.
A. Q R P S B. R P Q S
Children that grow into beautiful trees (P)
C. S P R Q D. R Q S P of a warm home and supportive
Directions: Each of the following items in this surroundings (Q) are like the tender
section consists of a sentence the parts of which saplings (R) with the sunshine and rain (S)
have been jumbled. These parts have been A. Q R P S
labeled P, Q, R and S and are given below each
sentence in four sequences namely (a), (b), (c) B. R P Q S
and (d). You are required to rearrange the C. R P S Q
jumbled parts of the sentence and mark your
response accordingly. D. P R S Q

PAGE 667
www.byjusexamprep.com

Directions: Each of the following items in this A. P S Q R B. Q S R P


section consists of a sentence the parts of which
C. S P R Q D. Q S P R
have been jumbled. These parts have been
labeled P, Q, R and S and are given below each Directions: Each of the following items in this
sentence in four sequences namely (a), (b), (c) section consists of a sentence the parts of which
and (d). You are required to rearrange the have been jumbled. These parts have been
jumbled parts of the sentence and mark your labeled P, Q, R and S and are given below each
response accordingly. sentence in four sequences namely P, Q, R and
38. The best part of it brings back the S. You are required to rearrange the jumbled
importance of writing skills (P) and locates parts of the sentence and mark your response
the story with the larger framework of our accordingly.
world (Q) literary flourishes (R) long- 41. One of the difficulties the whole of mankind
formed journalism is that (S) (P) or affect the masses (Q) the day after
A. Q R P S B. R S P Q tomorrow (R)

C. S P R Q D. P R S Q is that we want to transform (S)

Directions: Each of the following items in this A. S P Q R B. P R S Q


section consists of a sentence the parts of which C. S P R Q D. Q R S P
have been jumbled. These parts have been
labeled P, Q, R and S and are given below each Directions: Each of the following items in this
sentence in four sequences namely (a), (b), (c) section consists of a sentence the parts of which
and (d). You are required to rearrange the have been jumbled. These parts have been
jumbled parts of the sentence and mark your labeled P, Q, R and S and are given below each
response accordingly. sentence in four sequences namely (a), (b), (c)
and (d). You are required to rearrange the
39. The government must offer convincing jumbled parts of the sentence and mark your
solutions response accordingly.
PQ 42. in red who stood first (P) in the competition
and job markets that are causing social (Q) has been elected as the chairperson of
ferment to the crises in the rural economy the sports committee (R) the man (S)

RS A. SPQR B. SRPQ

A. P Q S R B. Q S R P C. PSRQ D. QRSP
C. S P R Q D. P R S Q Directions: Each of the following items in this
section consists of a sentence the parts of which
Directions: Each of the following items in this have been jumbled. These parts have been
section consists of a sentence the parts of which labeled P, Q, R and S and are given below each
have been jumbled. These parts have been
sentence in four sequences namely (a), (b), (c)
labeled P, Q, R and S and are given below each
and (d). You are required to rearrange the
sentence in four sequences namely (a), (b), (c)
jumbled parts of the sentence and mark your
and (d). You are required to rearrange the
response accordingly.
jumbled parts of the sentence and mark your
response accordingly. |||End||| 43. (P) I must say what I feel (Q)I am a votary of
truth and (R)to what I may have said before
40. The speaker of their inaction (P) has
(S) and think at a given moment without
identified (Q) charging the opponents (R)
regards
many issues besides (S)

PAGE 668
www.byjusexamprep.com

A. R Q S P B. Q R P S A. RQSP B. RQPS
C. P S R Q D. Q P S R C. SPQR D. PRQS
Directions: Each of the following items in this Directions: Each of the following items this
section consists of a sentence the parts of which section consists of a sentence the parts of which
have been jumbled. These parts have been have been jumbled These parts have been
labeled P, Q, R and S and are given below each labelled as P Q, R and S. Given below each
sentence in four sequences namely (a), (b), (c) sentence are four sequences namely (a), (b), (c)
and (d). You are required to rearrange the and (d). You are required to rearrange the
jumbled parts of the sentence and mark your jumbled parts of the sentence and mark your
response accordingly. response on the Answer Sheet accordingly.
44. the opinion that a human life (P) and that 47. (P) and they largely relied on agriculture,
he would quite like to live that long (Q) fishing and hunting (Q) the people had a
could span 125 years (R) there was a time subsistence economy (R) from excavation
when Gandhi expressed (S) sites indicate that (S) rich materials found

A. S P R Q B. R Q P S A. RQPS B. QSPR

C. S P Q R D. Q R S P C. SPQR D. SRQP

Directions: Each of the following items in this Directions: The following question consists of a
section consists of a sentence the parts of which sentence, the parts of which have been jumbled.
have been jumbled. These parts have been These parts have been labelled P, Q, R and S. You
labeled P, Q, R and S and are given below each are required to re-arrange the jumbled parts of
sentence in four sequences namely (a), (b), (c) the sentence and mark your response
and (d). You are required to rearrange the accordingly.
jumbled parts of the sentence and mark your 48. (P) It is in this context that (Q) and
response accordingly. prosperity must be viewed (R) the role of
45. or an independent judiciary (P) a free press agriculture (S) as a provider of jobs
is (Q) as essential a limb of democracy (R) A. P Q R S B. R S P Q
as a parliament freely elected by the people
(S) C. P R S Q D. R S Q P

A. RQSP B. QRPS Directions: Each of the following items in this


section consists of a sentence the parts of which
C. SPQR D. QRSP have been jumbled. These parts have been
Directions: Each of the following items in this labeled P, Q, R and S and are given below each
section consists of a sentence the parts of which sentence in four sequences namely (a), (b), (c)
have been jumbled. These parts have been and (d). You are required to rearrange the
labeled P, Q, R and S and are given below each jumbled parts of the sentence and mark your
sentence in four sequences namely (a), (b), (c) response accordingly.
and (d). You are required to rearrange the 49. and reduced into stereotypes (P) when
jumbled parts of the sentence and mark your great historical figures are appropriated (Q)
response accordingly. we are living at a time (R) that suit partisan
46. and that is ‘To learn to say I am sorry’ (P) political objectives (S)
something important enough that (Q) but A. RQPS B. RQSP
surely there must be (R) everyone should
learn it (S) C. SQRP D. PRQS

PAGE 669
www.byjusexamprep.com

Directions: Each of the following items in this Directions: Given below are some
section consists of a sentence the parts of which idioms/phrases followed by four alternative
have been jumbled. These parts have been meanings to each. Choose the response (a), (b),
labeled P, Q, R and S and are given below each (c) or (d) which is the most appropriate
sentence in four sequences namely (a), (b), (c) expression.
and (d). You are required to rearrange the
53. A pearl of wisdom
jumbled parts of the sentence and mark your
response accordingly. A. A wise man

50. is that it is not professional enough (P) have B. An important piece of order
not done their homework (Q) a valid C. An important piece of pearl
criticism of the profession of politics in India
(R) as the majority of its practitioners (S) D. An important piece of advice

A. RSPQ B. RPSQ 54. At the crossroads


A. At important road of a decision
C. SPQR D. PQRS
B. At an important point of journey
Directions: Each of the following items in this
section consists of a sentence the parts of which C. At the important road of a journey
have been jumbled. These parts have been D. At an important stage or decision
labeled P, Q, R and S and are given below each
sentence in four sequences namely (a), (b), (c) 55. Close shave
and (d). You are required to rearrange the A. Shaving every closely
jumbled parts of the sentence and mark your
response accordingly. B. Miraculous escape

51. an integrated individual (P) it is difficult in C. Saving someone from danger


modern (Q) by means of education (R) D. Easy escape
civilization to bring about (S)
56. Follow suit
A. RQSP B. RSPQ
A. Following someone’s suit
C. SPQR D. PRQS B. Suiting to someone
Directions: Each of the following items this C. Doing the same as someone else has
section consists of a sentence the parts of which just done
have been jumbled These parts have been
labelled as P Q, R and S. Given below each D. Doing the same kind of mistake
sentence are four sequences namely (a), (b), (c) 57. He makes decision on the fly.
and (d). You are required to rearrange the
A. He decides quickly without any
jumbled parts of the sentence and mark your
seriousness.
response on the Answer Sheet accordingly.
B. He decides with all seriousness.
52. (P) have become integral to most people's
lives (Q) debate for years as the devices (R) C. He decides nonchalantly
have drawn intense interest and (S) safety D. He is unwilling to decide
questions about cell phones
Directions: Each of the following sentences in
A. P Q R S B. R S P Q this section has a blank space with four words or
C. S P Q R D. S R Q P group of words given. Select whichever word or

PAGE 670
www.byjusexamprep.com

group of words you consider most appropriate 61. parliamentary debater, what have you
for the blank space and indicate your response done? You are probably very clever, full of
on the Answer Sheet accordingly. facts. Anybody can pick up facts; but we are
human beings, not factual machines, not
58. substitution – there will be much more
_________ routine automatons. But again,
control because the totalitarian
sirs, you are not interested.
government knows how to shape the minds
and hearts of the people, they A. beast B. bear
___________ the trick.
C. beastly D. bare
A. had learnt B. learnt
Directions: Each of the following sentences in
C. have learnt D. had been learnt this section has a blank space with four words or
Directions: Each of the following sentences in group of words given. Select whichever word or
this section has a blank space with four words or group of words you consider most appropriate
group of words given. Select whichever word or for the blank space and indicate your response
group of words you consider most appropriate on the Answer Sheet accordingly.
for the blank space and indicate your response 62. Educated to be what? To be bank clerks or
on the Answer Sheet accordingly. super salesmen, capitalists or commissars.
59. going to do a thing about radically changing When
the education system; so it will drag on you are a superman __________ as/or a
__________ a monstrous revolution, which super governor or a subtle
will merely be another
A. of some kind
A. until there are
B. of same kind
B. still there is
C. of some
C. till there was
D. of same
D. till there is
Directions: Each of the following sentences in
Directions: Each of the following sentences in this section has a blank space with four words or
this section has a blank space with four words or group of words given. Select whichever word or
group of words given. Select whichever word or group of words you consider most appropriate
group of words you consider most appropriate for the blank space and indicate your response
for the blank space and indicate your response on the Answer Sheet accordingly.
on the Answer Sheet accordingly.
63. together and ________
60. You are listening to me and __________
each other, you are not A. as quick as possible.
B. as quickly as possible.
A. smiling for B. smiling to C. as possible as.
C. smiling with D. smiling at D. quickly
Directions: Each of the following sentences in Directions: Each of the following sentences in
this section has a blank space with four words or this section has a blank space with four words or
group of words given. Select whichever word or group of words given. Select whichever word or
group of words you consider most appropriate group of words you consider most appropriate
for the blank space and indicate your response for the blank space and indicate your response
on the Answer Sheet accordingly. on the Answer Sheet accordingly.

PAGE 671
www.byjusexamprep.com

64. has become an important factor because Directions: Each of the following sentences in
we want large numbers of children _______ this section has a blank space with four words or
group of words given. Select whichever word or
A. educated
group of words you consider most appropriate
B. to be educated for the blank space and indicate your response
on the Answer Sheet accordingly.
C. to be educating
68. art form follow the same pattern
D. to educate
____________ when English translations
Directions: Each of the following sentences in
A. For B. Beginning
this section has a blank space with four words or
group of words given. Select whichever word or C. During D. Initially
group of words you consider most appropriate
Directions: Each of the following sentences in
for the blank space and indicate your response
this section has a blank space with four words or
on the Answer Sheet accordingly.
group of words given. Select whichever word or
65. So the _________ is that in the present group of words you consider most appropriate
social structure, discipline for the blank space and indicate your response
on the Answer Sheet accordingly.
A. difficulty B. difficult
69. Since the language itself is read from right
C. difference D. different
to left, the books with ________
Directions: Each of the following sentences in
A. that B. this
this section has a blank space with four words or
group of words given. Select whichever word or C. these D. which
group of words you consider most appropriate
Directions: Each of the following sentences in
for the blank space and indicate your response
this section has a blank space with four words or
on the Answer Sheet accordingly.
group of words given. Select whichever word or
66. the purists as it showed left-handed group of words you consider most appropriate
samurai, who did not exist in the original for the blank space and indicate your response
book, Hence, nowadays even English on the Answer Sheet accordingly.
translation follow _________ right to left
70. the 12th century when the first
format
______________ for this art form was seen.
A. the B. a
A. instance B. incident
C. some D. same
C. accident D. events
Directions: Each of the following sentences in
Directions: Each of the following sentences in
this section has a blank space and four words or
this section has a blank space with four words or
group of words given after the sentence. Select
group of words given. Select whichever word or
the word or group of words you consider most
group of words you consider most appropriate
appropriate for the blank space and indicate
for the blank space and indicate your response
your response on the Answer Sheet accordingly.
on the Answer Sheet accordingly.
67. were made, they flipped the pictures and
71. ‘whimsical or impromptu pictures’. It
published it. This ______________
_______ in existence since
A. enrage B. enlarged
A. had been B. has been
C. engraved D. enraged
C. was D. is

PAGE 672
www.byjusexamprep.com

Directions: Each of the following sentences in 77. Our gardener, which is very lazy,(a) says
this section has a blank space with four words or that (b) there will be no apples this year.(c)
group of words given. Select whichever word or No error(d)
group of words you consider most appropriate
A. (a) B. (b)
for the blank space and indicate your response
on the Answer Sheet accordingly. C. (c) D. (d)
72. This cultural form _______ from Japan has 78. The legendary hero(a) laid down his
a name which means precious life (b) for our country(c) No
A. originating B. originates error(d)

C. originated D. organizing A. (a) B. (b)

Directions: Each item in this section has a C. (c) D. (d)


sentence with three underlined parts labeled 79. The visitor’s to the zoo are requested,(a) in
(a), (b) and (c). Read each sentence to find out the interest of all concerned, (b) not to
whether there is any error in any underlined carry sticks, stones or food inside and not to
part and indicate your response on the Answer tease animals.(c) No error(d)
Sheet against the corresponding letter i.e., (a),
(b), (c). If you find no error, your response A. (a) B. (b)
should be indicated as (d). C. (c) D. (d)
73. Please note(a) that the interview for the 80. Sherly wants to know (a) whether you are
post(b) shall be held on June 15, 2019 going(b) to Delhi today night.(c) No
between 10.00 a.m. to 2.00 p.m.(c) No error.(d)
error.(d)
A. (a) B. (b)
A. (a) B. (b)
C. (c) D. (d)
C. (c) D. (d)
81. “I’m tired of my boys,” Said the mother,(a)
74. I did not want to listen to him,(a) but he was “Both of them keep quarrelling all the
adamant and(b) discussed about the time.(b) Right now also they are quarrelling
matter.(c) No error.(d) with one another.”(c) No error.(d)
A. (a) B. (b)
A. (a) B. (b)
C. (c) D. (d)
C. (c) D. (d)
75. No sooner did I reached there(a) the
82. The letter has been written;(a) I insist on(b)
children left the place(b) with their
it being sent at once.(c)No error(d)
parents.(c) No error.(d)
A. (a) B. (b)
A. (a) B. (b)
C. (c) D. (d)
C. (c) D. (d)
76. When I asked the guest(a) What she would Directions: In this section, you have few short
like to drink(b) she replied that she passages. After each passage, you will find some
preferred coffee much more than tea. (c) items based on the passage. First, read a
No error.(d) passage and answer the items based on it. You
are required to select your answer based on the
A. (a) B. (b) contents of the passage and opinion of the
C. (c) D. (d) author only.

PAGE 673
www.byjusexamprep.com

We live in a curious age. We are offered C. they are overlooked by the policy
glimpses of a world civilization slowly emerging, planners themselves
for example, the U.N. special agencies dedicated
D. they are indicative of the complete
to health and education. But along with these
failure of the government in controlling
are sights and sounds that suggest that the
the rebellious students
whole civilization is rapidly being destroyed.
Two official policies clash, and instantly 86. The author calls our age curious because
embassies are attacked by howling mobs of
A. it is an age of science and scientists are
students, at once defying law, custom and
curious by nature
usage. And that this may not be merely so many
hot-headed lads escaping all control, and may B. it is witnessing the emergence of a
itself be part of the policy of the political parties, world civilization
that is, mob antics as additional propaganda to
deceive world opinion, makes the situation even C. it is witnessing incidents that threaten
worse. Parties have always been dishonest, but to shake the very foundations of
now it seems as if power-mania is ready to civilization
destroy those civilities that make international D. it is an age of contradictions consisting
relations possible. There is something even of constructive and destructive
worse. What inspires these students to burn activities
cars and books is not their political enthusiasm
but a frenzied delight in destruction, an urge Directions: Read the following passage carefully
towards violent demolition. and answer the question given below:

83. In the passage the word ‘demolition’ has In good many cases, unnecessary timidity makes
the meaning as the word the trouble worse than it needs to be. Public
opinion is always more tyrannical towards those
A. defying B. antics who obviously fear it than towards those who
C. destruction D. urge feel indifferent to it. A dog will bark more loudly
and bite more easily when people are afraid of
84. One aspect of the mob indulging in violence it than when they treat him with contempt, and
and arson is that they the human herd has something of this same
characteristic. If you show that you are afraid of
A. destroy very costly things like vehicles
them, you give promise of good hunting,
B. destroy very valuable artifacts and whereas if you show indifference, they begin to
books doubt their own power and, therefore, tend to
let you alone.
C. get a mad delight in destruction for the
sake of destruction only 87. ‘.… you give promise of good hunting’
means
D. are motivated by certain political
ideology to resort to destruction A. you are vulnerable
85. It is deplorable to witness mob attacks on B. you are challenging
embassies following a clash of policies of
C. you are indomitable
two official policy makers because
D. you are confused
A. students should not take part in politics,
but should concentrate on their studies 88. The author compares men with dogs in
respect of
B. they may result in the loss of lives of
young and promising students A. attacking others without any reason

PAGE 674
www.byjusexamprep.com

B. attacking others when they are weak A. the letter with no postage meant good
news
C. barking and biting
B. the blank sheet meant being well
D. faithfulness to the master
89. The statement, ‘A dog will bark more loudly C. the blank sheet meant bad news
and bite more easily when people are afraid D. the letter with no postage meant
of him, than when they treat him with unimportant news
contempt….’ Implies that
92. The envelope contained
A. barking dogs seldom bite
A. a currency note
B. we should not be afraid of dogs
B. two written sheets
C. if we are afraid of others, they will leave
us alone C. no sheet at all
D. if we are afraid of people, they will try D. a blank sheet
to scare us more
93. Mr. Hill paid the postage because
90. If we are afraid of public opinion, the
attitude of the people towards us is A. the letter was from her brother

A. sympathetic B. indifferent B. the woman was his relative

C. admiration D. ruthless C. the letter was addressed to him


Directions: In this section you have few short D. he wanted to be kind to her
passages. After each passage, you will find some
items based on the passage. First, read a 94. The woman returned the letter to the
passage and answer the items based on it. You postman because
are required to select your answer based on the A. she could not pay the postage
contents of the passage and opinion of the
author only. B. the letter was not addressed to her

Mr. Rowland Hill, when a young man, was C. she already knew the contents of the
walking through the Lake district, when he one letter
day saw the postman deliver a letter to a woman
D. she hated the person who wrote the
at a cottage door. The woman turned it over and
letter
examined it, and then returned it, saying she
could not pay the postage, which was a shilling. 95. The story uses irony as a technique because
Hearing that the letter was from her brother,
Mr. Hill paid the postage, in spite of the manifest A. the woman returned her own brother’s
unwillingness of the woman. As soon as the letter without opening it
postman was out of sight, she showed Mr. Hill B. the woman broke the agreement of
how his money had been wasted, as far as she receiving blank letters to convey well
was concerned. The sheet was blank. There was being of her brother
an all went well with him, he should send a blank
sheet in this way once a quarter, and she thus C. Mr. Hill accepted the letter addressed
had tidings of him without expense of postage. to the woman

91. The woman add her brother had agreed D. in the modern times a brother has no
that time to write a letter to his own sister

PAGE 675
www.byjusexamprep.com

Directions: In this section you have few short 99. Curcumin has positive effect on people
passages. After each passage, you will find some
A. without dementia
items based on the passage. First, read a
passage and answer the items based on it. You B. with Alzheimer’s disease
are required to select your answer based on the
C. without dementia and with Alzheimer’s
contents of the passage and opinion of the
disease
author only.
D. with dementia and with Alzheimer’s
PASSAGE – 1
disease
Daily consumption of a certain form of curcumin
100. Which of the following statements are
improved memory and mood in people with
true?
mild, age-related memory loss. The research
examined the effects of an easily absorbed 1. Senior citizens in India have high level of
curcumin supplement on memory performance Alzheimer’s disease because of
in people without dementia, as well as consumption of turmeric.
curcumin’s potential impact on the microscopic
2. Senior citizens in India do not have high
plaques and tangled in the brains of people with
prevalence of alzheimer’s because of
Alzheimer’s disease. Found in turmeric,
consumption of turmeric.
curcumin has previously been shown to have
anti-inflammatory and antioxidant properties in 3. Consumption of turmeric enhances
laboratory studies. It has also been suggested as cognitive performance.
a possible reason that senior citizens in India, 4. Curcumin is an antioxidant.
where curcumin is a dietary staple, have a lower
prevalence of Alzheimer’s disease and better Select the correct answer using the code
cognitive performance. given below :
96. ____________ of a disease in a region A. 2, 3 and 4
depends on the food habits too. B. 1, 3 and 4
A. Dominance B. Prevalence C. 1 and 4 only
C. Affection D. Death D. 1 and 3 only
97. Eating turmeric Directions: Each item in this section consists of
A. will reduce the chance of getting a sentence with an underlined word followed by
Alzheimer’s disease four options. Select the option that is nearest in
meaning to the underlined word and mark your
B. will increase curcumin response accordingly.
C. will enhance dementia 101. Any classroom should provide an engaging
D. will reduce chance of getting cancer environment for learners.
98. Which word in the passage means A. carefree B. appealing
‘earlier’? C. thinking D. dreaming
A. Performance Directions: Each item in this section consists of
a sentence with an underlined word followed by
B. Absorbed
four words/groups of word. Select the option
C. Properties that is nearest in meaning to the underlined
word and mark your response on your Answer
D. Previously
Sheet accordingly.

PAGE 676
www.byjusexamprep.com

102. His derisive behaviour has led to the 106. A good work place shall not encourage
situation we face now. ineptitude even in a hidden manner.
A. Mockery B. Conducive A. incompetence
C. Encouraging D. Contemptuous B. courage
Directions: Each item in this section consists of C. gossip
a sentence with an underlined word followed by
D. radical thinking
four words/groups of word. Select the option
that is nearest in meaning to the underlined Directions: Each item in this section consists of
word and mark your response on your Answer a sentence with an underlined word/words
Sheet accordingly. followed by four words. Select the option that is
nearer to meaning to the underlined
103. It was felt that the decision to remove the
word/words and mark your response on your
group from the exercise would be
Answer Sheet accordingly.
detrimental to the organization.
107. Don’t condone such acts which lead to
A. Beneficial B. Harsh
unrest in the country.
C. Harmful D. Demanding
A. regard B. punish
Directions: Each item in this section consists of
C. aware of D. overlook
a sentence with an underlined word followed by
four options. Select the option that is nearest in Directions: Each item in this section consists of
meaning to the underlined word and mark your a sentence with an underlined word/words
response accordingly. followed by four words. Select the option that is
nearer to meaning to the underlined
104. Extradition of the leader of the group was
word/words and mark your response on your
debated for hours in the meeting.
Answer Sheet accordingly.
A. acceptance B. sentence
108. He felt desolated after he lost his business.
C. extension D. deportation
A. deserted B. joyful
Directions: Each item in this section consists of
C. strong D. annoyed
a sentence with an underlined word followed by
four words/groups of word. Select the option Directions: Each item in this section consists of
that is nearest in meaning to the underlined a sentence with an underlined word/words
word and mark your response on your Answer followed by four words. Select the option that is
Sheet accordingly. nearer to meaning to the underlined
word/words and mark your response on your
105. Learning of foreign language should not
Answer Sheet accordingly.
impede one’s mother tongue learning.
109. His salubrious words calmed the students.
A. provoking B. pleasant
A. Facilitate B. Acts for
C. ridiculous D. thanking
C. Hinder D. Accept
Directions: Each item in this section consists of
Directions: Each item in this section consists of
a sentence with an underlined word/words
a sentence with an underlined word followed by
followed by four words. Select the option that is
four words/group of words. Select the option
nearer to meaning to the underlined
that is the nearest in meaning to the underlined
word/words and mark your response on your
word and mark your response on your Answer
Answer Sheet accordingly.
Sheet accordingly.

PAGE 677
www.byjusexamprep.com

110. Rahul is always thrifty. Directions: Each of the following sentences in


this section has a blank space and four words or
A. reckless B. economical
groups of words given after the sentences.
C. naïve D. extravagant Select the words or group of words you consider
most appropriate for the blank space and
Directions: Choose the most appropriate indicate your response on the Answer Sheet
alternative to complete the sentence: accordingly.
111. My sister asked me ____________ wiling to 114. _________ sincere he would have got the
go abroad for my studies. prize.
A. if I were A. Had he been
B. if I could be B. Has he been
C. whether I should be C. Would he have been
D. whether I will D. He is
Directions: Each of the following sentences in Directions: Each of the following sentences in
this section has a blank space and four words or this section has a blank space and four words or
groups of words given after the sentences. group of words given after the sentence. Select
Select the words or group of words you consider the word or group of words you consider most
most appropriate for the blank space and appropriate for the blank space and indicate
indicate your response on the Answer Sheet your response on the Answer Sheet accordingly.
accordingly.
115. The visiting Diplomat __________ the
112. If I __________ you I __________ love to Prime Minister.
accept the offer.
A. called in
A. was ……………. Will
B. called at
B. was ……………… would
C. called on
C. were …………….. would
D. called up on
D. were …………………. will
Directions: Each of the following sentences in
Directions: Each of the following sentences in this section has a blank space and four words or
this section has a blank space and four words or groups of words given after the sentences.
groups of words given after the sentences. Select the words or group of words you consider
Select the words or group of words you consider most appropriate for the blank space and
most appropriate for the blank space and indicate your response on the Answer Sheet
indicate your response on the Answer Sheet accordingly.
accordingly.
116. The new student found it difficult to
113. Ten years __________ for me to live in a _________ with his classmates.
foreign country.
A. get along
A. are a long time
B. get among
B. is a long time
C. get well
C. has a long time
D. get up
D. of time

PAGE 678
www.byjusexamprep.com

Directions: Fill in the blanks with appropriate 119. Sandhya _________ me from the top of the
blanks: house.
117. I always want to go alone for a ride, but my A. shouted to
mother _________ going with my brother.
B. shouted at
A. insists B. insists on
C. shouted on
C. insists in D. insisted
D. shouted
Directions: Choose the most appropriate
alternative to complete the sentence: Directions: Each of the following sentences in
this section has a blank space and four words or
118. Ravi has the habit of __________ a groups of words given after the sentences.
headache. Select the words or group of words you consider
A. complaining most appropriate for the blank space and
indicate your response on the Answer Sheet
B. complain accordingly.
C. complaining to 120. My teacher was ___________ us for being
D. complaining of late.

Directions: In the following question, a sentence A. annoyed at


is given with a blank to be filled in with an B. annoyed with
appropriate word. Select the correct alternative
out of the four and indicate it by selecting the C. annoyed about
appropriate option. D. annoys

PAGE 679
www.byjusexamprep.com

General Knowledge
1. Which organization was started at the 6. Under the PRASAD Tourism Scheme, which
Haridwar Kumbh Mela in 1915? one of the following has not been identified
as a religious site for development?
A. Sanatan Dharma Sabha
A. Ajmer (Rajasthan)
B. Dev Samaj
B. Haridwar (Uttarkhand)
C. Brahmin Sabha
C. Somnath (Gujarat)
D. Hindu Mahasabha
D. Velankanni (Tamil Nadu)
2. Who took over the ‘Eka Movement’ started
by the Congress in Awadh during 1921- 7. SWAYAM is
1922?
A. a network that aims to tap the talent
A. Bhagwan Ahir pool of scientists and entrepreneurs
towards global excellence
B. Madari Pasi
B. a Massive Open Online Courses
C. Baba Ramchandra
(MOOCs) initiative on a national
D. Shah Naeem Ata platform
3. The tagline ‘Invaluable Treasures of C. an empowerment scheme for
Incredible India’ is associated with the logo advancing the participation of girls in
for education
A. Archaeological Survey of India D. a scheme that supports differently
abled children to pursue technical
B. India Tourism Development Corporation
education
C. Geological Survey of India
8. Which one of the following is correct about
D. Geographical Indications (GI) of India ‘Aaykar Setu’?
4. The Central Water Commission has recently A. It is a mechanism for achieving
entered into a collaborative agreement excellence in public sector delivery
with which one of the following entities for related to GST.
flood forecasting?
B. With the use of a mobile app, it
A. Skymet B. Google facilitates online payment of taxes.
C. MetService D. AccuWeather C. It is a communication strategy designed
to collect information and build a
5. Name the Indian cricketer who is not database of tax defaulters.
inducted to the ICC Cricket Hall of Fame (till
July 2018). D. It enables electronic filing and
processing of import and export
A. Rahul Dravid declarations.
B. Sunil Gavaskar 9. According to the updated World Bank data
C. Sachin Tendulkar for 2017, India is the sixth biggest economy
of the world (in terms of GDP). Which one
D. Anil Kumble of the following is not ahead of India?

PAGE 680
www.byjusexamprep.com

A. Japan B. UK 13. Which one of the following regarding the


ordinance-making power of the Governor is
C. France D. Germany not correct?
10. Ace athlete Neeraj Chopra is an A. It is not a discretionary power.
accomplished player in
B. The Governor may withdraw the
A. Hammer throw ordinance anytime.
B. Javelin throw C. The ordinance power can be exercised
C. Shot put throw when the Legislature is not in session.

D. Discus throw D. The aid and advice of ministers is not


required for declaring the ordinance.
11. Which one of the following statements
regarding the Human Rights Council is not 14. Which one of the following is not
correct? considered a part of the Legislature of
States?
A. It is an inter-governmental body within
the United Nations system made up of A. The Governor
all members of the UN. B. The Legislative Assembly
B. It is responsible for the promotion and C. The Legislative Council
protection of all human rights around
the globe. D. The Chief Minister
C. It replaced the former United Nations 15. Which one of the following is not among
Commission on Human Rights. the duties of the Chief Minister?
D. It is made up of 47 UN Member States A. To communicate to the Governor of the
which are elected by the UN General State all decisionos of the Council of
Assembly. Ministers relating to the administration
of the affairs of the State and proposals
12. Which one of the following statements for legislation
regarding the Universal Declaration of
Human Rights is not correct? B. To furnish information relating to the
administration of the State and
A. The UN General Assembly adopted the proposals for legislation as the
Human Rights Charter on 10th Governor may call for
December, 1948.
C. To communicate to the President all
B. Some of the provisions of the decisions of the Council of Ministers
Fundamental Rights enshrined in the relating to the administration of the
Constitution of India are similar to the State in the monthly report
provisions of the Universal Declaration
of Human Rights. D. To submit for the consideration of the
Council of Ministers any matter on
C. The Right of Property is not a part of the which a decision has been taken by a
Universal Declaration of Human Rights. Minister but has not been considered
D. India is a signatory to the Universal by the Council, if the Governor so
Declaration of Human Rights. requires

PAGE 681
www.byjusexamprep.com

16. Which one of the following criteria is not C. Summoning, proroguing and dissolving
required to be qualified for appointment as State Legislature
Judge of the Supreme Court?
D. Power to grant pardons, reprieves,
A. At least five years as a Judge of a High respites or remission of punishments
Court
20. The audit reports of the Comptroller and
B. At least ten year as an Advocate of a Auditor General of India relating to the
High Court accounts of the Union shall be submitted to
C. In the opinion of the President, a A. the President
distinguished Jurist
B. the Speaker of the Lok Sabha
D. At least twenty years as a Sub-Judicial
C. the Prime Minister
Magistrate
D. the Vice President
17. Which one of the following is not under the
powers and functions of the Election 21. Who has the power the annulment or
Commission of India? suspension of Acts and Resolutions of the
Autonomous District and Regional
A. Superintendence, direction and control
Councils?
of the preparation of electoral rolls
A. The Governor
B. Conduct of elections to the Parliament
and to the Legislature of each State B. The President
C. Conduct of election to the office of the C. The Chief Minister of the State
President and the Vice President D. The Prime Minister
D. Appointment of the Regional 22. Who among the following shall cause the
Commissioners to assist the Election accounts of the Autonomous District and
Commission in the performance of the Regional Council Funds to be audited?
functions conferred on the Commission
A. The Comptroller and Auditor General of
18. Which one of the following regarding the India
procedure and conduct of business in the
Parliament is not correct? B. The Chartered Accountant empanelled
by the Government of India
A. To discuss State matters
C. The State Government Auditors
B. To discuss issues of the use of police
force in suppressing the Scheduled D. Any Chartered Accountant
Caste and Scheduled Tribe Communities 23. Which one of the following regarding the
C. To discuss issues in dealing with violent tenure of the elected members of the
disturbances in an undertaking under Autonomous District Council is correct?
the control of the Union Government A. Five years from the date of election
D. To discuss issues for putting down the B. Five years from the date appointed for
demands of the industrial labour the first meeting of the Council after the
19. Which one of the following is not related to election
the powers of the Governor? C. Six years from the date of
A. Diplomatic and military powers administration of Oath

B. Power to appoint Advocate General D. Six years from the date of election

PAGE 682
www.byjusexamprep.com

24. The elected President of the All India Kisan 29. The College of Fort William was established
Sabha, which met in Vijayawada (1944), by which one of the following Governor-
was Generals?
A. Sahajananda Saraswati A. Warren Hastings

B. Vinoba Bhave B. Lord Cornwallis

C. Achyut Rao Patwardhan C. Richard Wellesley

D. Narendra Dev D. William Bentinck

25. Which political party formally accepted the 30. ‘Mission Satyanishtha’, a programme on
Cabinet Mission Plan on 6th June, 1946, ethics in public governance, was launched
which had rejected the demand for a recently by the
sovereign Pakistan? A. Indian Railways
A. The Hindu Mahasabha B. Central Bureau of Investigation
B. The Congress C. Supreme Court
C. The Muslim League D. Enforcement Directorate
D. The Unionist Party 31. Which one of the following is the official
mascot of Tokyo 2020 Olympic Games?
26. Subhas Chandra Bose started the ‘Azad
Hiind Radio’ in which one of the following A. Soohorang
countries?
B. Vinicius de Moraes
A. Japan B. Austria
C. The Hare, the Polar Bear and the
C. Germany D. Malaysia Leopard

27. Tea growing of the Chinese tea plants in D. Miraitowa


India in the 19th century was made possible 32. Which one of the following manufacturers
by is engaged in upgradation of the Swedish
A. Joseph Banks 155-mm Bofors Howitzer under the project
‘Dhanush’?
B. James Cook
A. Bharat Electronic Limited
C. Robert Fortune
B. Ordnance Factory Board
D. Robert Owen
C. Bharat Dynamics Limited
28. The Economic historian, who has used the
D. Mishra Dhatu Nigam
data collected by Buchanan-Hamilton to
support the thesis of deindustrialization in 33. What is India’s first Indigenous Aircraft
the 19th century India, is Carrier (IAC) called?
A. Tirthankar Roy A. Vikrant

B. Amiya Kumar Bagchi B. Virat

C. Sabyasachi Bhattacharya C. Vaibhav

D. Irfan Habib D. Varaha

PAGE 683
www.byjusexamprep.com

34. The two defence industrial corridors 40. According to the French traveller Tavernier,
announced by the Finance Minister in his the majority of houses in Varanasi during
2018 Budget speech are coming up in which the 17th century were made of
of the following States?
A. brick and mud
A. Odisha and West Bengal
B. stone and thatch
B. Punjab and Haryana
C. wood and stone
C. Gujarat and Maharashtra D. brick and stone
D. Uttar Pradesh and Tamil Nadu 41. Who was the first Nawab Wazir of Awadh in
35. In which one of the following States was the 18th century?
‘DEFEXPO 2018’ held in April 2018? A. Nawab Safdarjung
A. Goa B. Karnataka B. Nawab Saadat Ali Khan
C. Tamil Nadu D. Andhra Pradesh C. Nawab Shuja-ud-Daula
36. Which one of the following Schedules to D. Nawab Saadat khan
the Constitution of India provides for
setting up of Autonomous District Councils? 42. The aristocrat Muqarrab khan was a great
favourite of which Mughal Emperor?
A. Third Schedule B. Fourth Schedule
A. Akbar B. Jahangir
C. Fifth Schedule D. Sixth Schedule
C. Farrukhsiyar D. Shah Alam
37. Provisions of which one of the following
Articles of the Constitution of India apply to 43. The important source for Akbar’s reign,
the State of Jammu and kashmir? Tarikh-i-Akbari was written by which one of
the following Persian language scholars?
A. Article 238 B. Article 370
A. Arif Qandahari
C. Article 371 D. Article 371G
B. Bayazid Bayat
38. Which one of the following Articles of the
C. Abdul Qadir Badauni
Constitution of India deals with the special
provision with respect to the State of D. Nizamuddin Ahmad
Assam?
44. The class of Amar Nayakas in Vijayanagara
A. Article 371A B. Article 371B is a reference to which of the following?
C. Article 371C D. Article 371D A. Village Chieftains
39. Where the Governor takes a decision B. Senior Civil Servants
independently of his Council of Ministers or
C. Tributary Chiefs
where he acts as the Chief Executive of the
State under Presidents rule, his actions are D. Military Commanders
subject to scrutiny by the Parliament. So
45. Who among the following European
the statements 2 is correct.
travellers never returned to Europe and
Hence, option B is correct. settled down in India?
A. 1 only B. 2 only A. Duarte Barbosa B. Manucci

C. Both 1 and 2 D. Neither 1 nor 2 C. Tavernier D. Bernier

PAGE 684
www.byjusexamprep.com

Directions: The following four (4) items consist A. Both the statements are individually
of two statements, Statement I and Statement true and Statement II is the correct
II. Examine these two statements carefully and explanation of Statement I
select the correct answer using the code given
B. Both the statements are individually
below.
true but Statement II is not the correct
46. Statement I : Agriculture in India still explanation of Statement I
accounts for a substantial share in total
C. Statement I is true but Statement II is
employment.
false
Statement II : There has been no decline in
volatility of agricultural growth in India. D. Statement I is false but Statement II is
true
A. Both the statements are individually
true and Statement II is the correct 49. Statement I : The overall fiscal deficit of the
explanation of Statement I States in India during 2017-2018 stayed
above the FRBM threshold level of 3
B. Both the statements are individually percent for the third successive year.
true but Statement II is not the correct
explanation of Statement I Statement II : Special Category States had
run up a higher level of fiscal deficit in 2017-
C. Statement I is true but Statement II is 2018 compared to 2016-2017.
false
A. Both the statements are individually
D. Statement I is false but Statement II is true and Statement II is the correct
true explanation of Statement I
47. Statement I : Private investments in B. Both the statements are individually
research have severely lagged public true but Statement II is not the correct
investments in India. explanation of Statement I
Statement II : Universities play a relatively C. Statement I is true but Statement II is
small role in the research activities of the false
country.
D. Statement I is false but Statement II is
A. Both the statements are individually true
true and Statement II is the correct
explanation of Statement I 50. With regard to the cabinet decision in July
2018, the percentage increase in Minimum
B. Both the statements are individually Support Price (MSP) is maximum in which
true but Statement II is not the correct one of the following crops?
explanation of Statement I
A. Jowar (Hybrid) B. Bajra
C. Statement I is true but Statement II is
false C. Maize D. Soya bean
D. Statement I is false but Statement II is 51. Who among the following is the Convener
true of the “Task Force” set up in November
2017 by the Government of India to review
48. Statement I : There has been a sharp
the Income-tax Act and draft a new direct
decline in savings rate in Indian economy
tax law?
between 2007-2008 to 2015-2016.
A. Girish Ahuja B. Mukesh Patel
Statement II : There has been a fall in
household and public savings. C. Arbind Modi D. Mansi Kedia

PAGE 685
www.byjusexamprep.com

52. The acronym ‘CAATSA’ refers to a piece of 57. The 2+2 Bilateral Dialogue was held in
legislation enacted by which one of the September 2018 between
following countries?
A. External Affairs and Defence Ministers
A. United Kingdom of India with their US counterparts
B. United States of America B. Finance and Defence Ministers of India
with their Russian counterparts
C. Russia
C. Home and Defence Ministers of India
D. India
and their counterparts in Pakistan
53. The policy on strategic partnerships in
D. External Affairs and Defence Ministers
defence was approved by the Ministry of
of India with their counterparts in
Defence in May 2017. Which of the
Pakistan
following is not among the four segments
identified by the Ministry for acquisition 58. Which of the following statements relating
through the strategic partnership route? to the historic Objectives Resolution, which
was adopted by the Constituent Assembly,
A. Artillery guns
is/are correct?
B. Fighter aircraft and helicopters
1. The Objectives Resolution inspired the
C. Submarines shaping of the Constitution through all
its subsequent stages.
D. Armoured fighting vehicles and main
battle tanks 2. It was not just a resolution, but a
declaration, a firm resolve and a pledge.
54. As per the extant policy, Foreign Direct
Investment is permitted in the defence 3. It provided the underlying philosophy of
sector under the automatic route up to our Constitution.
which one of the following limits?
Select the correct answer using the code
A. 26 percent B. 74 percent given below.
C. 51 percent D. 49 percent A. 1 and 2 only B. 1 only
55. ‘Tejas’ is the name of which one of the C. 1, 2 and 3 D. 2 and 3 only
following?
59. Which of the following statements relating
A. Main battle tank to the Indian Councils Act, 1861 is/are
correct?
B. Nuclear submarine
1. The Act introduced a grain of popular
C. Light combat aircraft
element by including non-official
D. Aircraft carrier members in the Governor-General’s
Executive Council.
56. Who is the Chairman of the Defence
Planning Committee set up in April 2018? 2. The members were nominated and
their functions were confined
A. The Prime Minister exclusively to consideration of
B. The National Security Advisor legislative proposals placed before it by
the Governor-General.
C. The Defence Minister
3. The Governor-General did not have
D. The Chief of the Army Staff effective legislative power.

PAGE 686
www.byjusexamprep.com

Select the correct answer using the code 63. According to the Manusmriti, women can
given below. acquire wealth through which of the
following means?
A. 1 and 2 only
A. Purchase
B. 2 and 3 only
B. Investment
C. 1, 2 and 3
C. Token of affection
D. 1 only
60. Which of the following statements relating D. Inheritance
to the Government of India Act, 1858 is/are 64. According to the Tamil Sangam texts who
correct? among the following were the large
1. The British Crown assumed sovereignty landowners?
over India from the East India Company. A. Gahapatis B. Uzhavars
2. The British Parliament enacted the first C. Adimais D. Vellalars
statute for the governance of India
under the direct rule of the British. 65. The story Gandatindu Jataka was written in
which language?
3. This Act was dominated by the principle
of absolute imperial control without A. Sanskrit B. Telugu
any popular participation in the
C. Tamil D. pali
administration of the country.
66. The Harappan site at Kot Diji is close to
Select the correct answer using the code
Which one of the following major sites of
given below.
that civilization?
A. 1 and 2 only
A. Harappa B. Mohenjo-daro
B. 2 only
C. Lothal D. Kalibangan
C. 1, 2 and 3
67. The natural rate of unemployment
D. 1 and 3 only hypothesis was advocated by
61. In the first century AD, Which among the A. Milton Friedman
following was not a major item of Indian
exports to Rome? B. A W Phillips

A. Pepper B. Spikenard C. J M Keynes

C. Tortoiseshell D. Nutmeg D. R G Lipsey

62. The dialogue on Varna between king 68. The Fourteenth Finance Commission
Avantiputta and Kachchana a disciple of assigned different weights to the following
Buddha appears in Which one of the parameters for distribution of tax proceeds
following Buddhist texts? to the States :

A. Majjhima Nikaya 1. Income distance

B. Samyutta Nikaya 2. Population

C. Anguttara Nikaya 3. Demographic changes

D. Ambattha Sutta 4. Area

PAGE 687
www.byjusexamprep.com

Arrange the aforesaid parameters in 73. Match List – I with List – II and select the
descending order in terms of their weights. correct answer using the code given below
the Lists :
A. 1-2-3-4 B. 1-2-4-3
List-I List-II
C. 1-3-2-4 D. 4-3-2-1
(Major Dam) (State)
69. According to the World Bank’s Doing
A. Cheruthoni Dam 1. Madhya
Business Report, 2018 India’s ranking has
Pradesh
improved in 2018 as compared to 2017 in
which of the following areas? B. Indira Sagar Dam 2. Tamil Nadu
1. Paying taxes C. Krishnarajasagar 3. Karnataka
Dam
2. Resolving insolvency
D. Mettur Dam 4. Kerala
3. Starting a business
4. Getting electricity
Select the correct answer using the code Codes:
given below. A. A-2 B-1 C-3 D-4
A. 1 only B. 1 and 2 only B. A-2 B-3 C-1 D-4
C. 1, 2 and 3 D. 2, 3 and 4 C. A-4 B-3 C-1 D-2
70. According to the latest Reserve Bank of D. A-4 B-1 C-3 D-2
India study on State finances capital
spending is maximum on 74. Which one among the following is not a
tributary of river Luni?
A. rural development
A. Khari B. Sukri
B. water supply and sanitation
C. Jawai D. Banas
C. urban development
75. The headquarters of Metro Railway
D. education Zone is located in
71. Which one among the following passes A. New Delhi B. Mumbai
links Lhasa with Ladakh?
C. Kolkata D. Chennai
A. Lanak La B. Burzil
76. Khasi language is included in
C. Babusar D. Khyber
A. Munda Branch of Austro-Asiatic sub-
72. Which one among the following Union family
Territories of India shares the shortest
length of National Highways? B. Mon- Khmer branch of Austro- Asiatic
sub-family
A. Chandigarh
C. North Assam branch of Sino-Tibetan
B. Delhi family
C. Daman and Diu D. Assam - Myanmari branch of Sino-
D. Dadra and Nagar Haveli Tibetan family

PAGE 688
www.byjusexamprep.com

77. Which one of the following is a west- 80. Match List – I with List – II and select the
flowing river? correct answer using the code given below
the Lists :
A. Mahanadi
List-I List-II
B. Godavari (Classification of (Example)
C. Krishna Town)

D. Narmada A. Industrial Town 1. Vishakha-


patnam
78. Which one of the following was not a part
B. Transport Town 2. Bhilai
of the strategies followed by the
Government of India to increase food gain C. Mining Tow 3. Singrauli
production in India immediately after
Independence? D. Garrison 4. Ambala
Cantonment Town
A. Intensification of cropping over already
cultivated land
Codes:
B. Increasing cultivable area by bringing
cultivable and fallow land under plough A. A-2 B-1 C-3 D-4

C. Using high-yielding varieties (HYV) B. A-2 B-3 C-1 D-4


seeds C. A-4 B-3 C-1 D-2
D. Switching over from cash crops to food D. A-4 B-1 C-3 D-2
crops
81. The Eight Degree Channel separates Which
79. Which of the following statements with of the following?
regard to the land-use situation in India
A. India from Sri Lanka
is/are correct?
B. Lakshadweep from Maldives
1. There has been a tremendous decline in
area under forest in recent years. C. Andaman from Nicobar Islands

2. The rate of increase in land use in recent D. Indira Point from Indonesia
years is the highest in case of area 82. South Arcot and Ramanathapuram receive
under non-agricultural use. over 50 percent of their annual rainfall from
3. Land use such as barren and wasteland, Which one of the following?
area under pastures and tree crops A. South-west monsoon
have experienced decline in recent
years. B. North-east monsoon
C. Bay of Bengal branch of summer
Select the correct answer using the code
monsoon
given below.
D. Western disturbances
A. 1 only
83. Which one of the following places does not
B. 1 and 2 only fall on leeward slope?
C. 2 and 3 only A. Pune B. Bengaluru
D. 1, 2 and 3 C. Leh D. Mangaluru

PAGE 689
www.byjusexamprep.com

84. Which one of the following is the correct 88. Which one of the following features is an
ascending sequence of States with regard indication for modification of stem of a
to percentage of urban population (2011)? plant?
A. Tamil Nadu- Mizoram- Goa- A. Presence of ‘eye’ on potato
Maharashtra B. ‘Scale’ found in onion
B. Goa- Mizoram- Maharashtra- Kerala C. ‘Tendril’ found in pea
C. Maharashtra- Kerala- Mizoram- Goa D. Hair present in carrot
D. Mizoram- Goa- Maharashtra-Kerala 89. In which of the following, heat loss is
85. Which one of the following statements primarily not due to convection?
explains higher mutation rate and faster A. Boiling water
evolution found in RNA virus?
B. Land and sea breeze
A. RNA is relatively unstable compared to
C. Circulation of air around blast furnace
DNA
D. Heating of glass surface of a bulb due to
B. Virus can multiply only within the living current in filament
cell of a host.
90. Which one of the following can extinguish
C. Metabolic processes are absent in virus. fire more quickly?
D. Virus can remain latent for a long A. Cold water B. Boiling water
period.
C. Hot water D. Ice
86. Which one of the following is the special
type of milk produced by a lactating 91. A circuit has a fuse having a rating of 5 A,
mother, essential for the development of What is the maximum number of 100 W-
immune response of newborn baby in 220 V bulbs that can be safely connected in
human? parallel in the circuit?
A. 20 B. 15
A. Breast milk produced after a month of
childbirth C. 11 D. 10
B. Transitional milk 92. The direction of magnetic field at any
location on the earth’s surface is commonly
C. Colostrum specified in terms of
D. Mineralized milk A. field declination
87. Which of the following roles is /are played B. field inclination
by epididymis vas deferens, seminal
vesicles and prostate in male reproductive C. both field declination and field
system of human? inclination

A. Spermatogenesis and maturation of D. horizontal component of the field


sperms 93. When a convex lens produces a real image
of an object, the minimum distance
B. Maturation and motility of sperms
between the object and image is equal to
C. Spermatogenesis and motility of
A. the focal length of the convex lens
sperms
B. twice the focal length of the convex lens
D. Motility of sperms only

PAGE 690
www.byjusexamprep.com

C. four times the focal length of the covex A. Uttar Pradesh


lens
B. Himachal Pradesh
D. one half of the focal length of the
convex lens C. Punjab

94. In Which one of the following reactions, the D. West Bengal


maximum quantity of H2 gas is produced by 100. Which one of the following statements with
the decomposition of 1 g of compound by regard to growth of coral reefs is not
A. correct?

B. A. Coral can grow abundantly in fresh-


water.
C. B. It requires warm water between 23 °C-
25 °C.
D.
C. It requires shallow saltwater, not
95. Which one of the following is not true for deeper than 50 metres.
the form of carbon known as a diamond?
D. It requires plenty of sunlight to aid
A. It is harder than graphite. photosynthesis.
B. It contains the same percentage of 101. The rate of population growth during 2001-
carbon as graphite. 2011 decade declined over the previous
C. It is a better electric conductor than decade (1991-2001) in all of the following
graphite. States, except
D. It has different carbon to carbon A. Tamil Nadu B. Kerala
distance in all directions.
C. Goa D. Andhra Pradesh
96. Which one of the following nitrogen oxides
has the highest oxidation state of nitrogen? 102. Which one of the following is the correct
ascending sequence of States in terms of
A. NO B. NO2 their population density as per Census
C. N2O D. N2O5 2011?

97. The equivalent weight of Ba(OH)2 is given, A. Arunachal Pradesh-Sikkim-Mizoram-


atomic weight of Ba is 137.3 Himachal Pradesh
A. 85.7 B. 137.3 B. Arunachal Pradesh- Mizoram-Sikkim-
Himachal Pradesh
C. 154.3 D. 171.3
98. Which one of the following States has more C. Mizoram-Arunachal Pradesh-Himachal
than two major ports? Pradesh- Sikkim

A. Maharashtra B. West Bengal D. Arunachal Pradesh- Himachal Pradesh-


Sikkim- Mizoram
C. Odisha D. Tamil Nadu
103. What is the visible portion of the
99. As per Census 2011, the concentration of
electromagnetic spectrum called?
Scheduled caste population (going by
percentage of Scheduled Caste population A. Infrared B. Radiowave
to total population of the state) is the
C. Microwave D. Light
highest in the state of

PAGE 691
www.byjusexamprep.com

104. Which one of the following agents does not 109. Which of the following is /are the main
contribute to propagation of plants through absorbing organ/organs of plants?
seed dispersal?
A. Root only
A. Wind B. Fungus
B. Leaf only
C. Animal D. Water C. Root and leaf
105. If the xylem of a plant is mechanically D. Root, leaf and bark
blocked, which of the following functions of
the plant will be affected? 110. Which one of the following does not
convert electrical energy into light energy?
A. Transport of water only
A. A candle
B. Transport of water and solutes
B. A light-emitting diode
C. Transport of solutes only
C. A laser
D. Transport of gases
D. A television set
106. In Which one of the following physiological
111. Two persons are holding a rope of
processes, excess water escapes in the
negligible mass horizontally. A 20 kg mass is
form of droplets from a plant?
attached to the rope at the midpoint; as a
A. Transpiration result the rope deviates from the horizontal
direction. The tension required to
B. Guttation completely straighten the rope is
C. Secretion
D. Excretion A. 200 N B. 20 N
107. Which one of the following denote a ‘true C. 10 N D. infinitely large
fruit?
112. The position vector of a particle is
A. When only the thalamus of the flower r = 2t2xˆ + 3tyˆ + 4zˆ
grows and develops into a fruit
Then the instantaneous velocity  and
B. When only the receptacle of the flower acceleration a respectively lie
develops into a fruit
A. on xy-plane and along z-direction
C. When fruit originates only from the
calyx of a flower B. on yz-plane and along x-direction

D. When only the ovary of the flower C. on yz-plane and along y-direction
grows into a fruit D. on xy-plane and along x-direction
108. Which of the following is not a primary 113. If two miscible liquids of same volume but
function of a green leaf? different densities P and P are mixed, then
the density of the mixture is given by
A. Manufacture of food
B. Interchange of gases A. B.
C. Evaporation of water
C. D.
D. Conduction of food and water

PAGE 692
www.byjusexamprep.com

114. A particle moves with uniform acceleration A. They are all diatomic
along a straight line from rest. The
B. They are all gases at room temperature.
percentage increase in displacement during
sixth second compared to that in fifth C. They are all coloured.
second is about D. They all have same reactivity.
A. 11% B. 22% 118. Reaction between which of the following
C. 33% D. 44% two reactants will produce hydrogen gas?

115. Which of the following is the general A. Magnesium and hydrochloric acid
formula for saturated hydrocarbons? B. Copper and dilute nitric acid
A. C. Calcium carbonate and hydrochloric
acid
B.
D. Zinc and nitric acid
C.
119. How many moles of CO can be obtained by
D. reacting 2.0 mole of CH4 with 2.0 mole of O2
according to the equation given below?
E. None of the above/More than one of
the above
116. The compound has A. 2.0 B. 0.5
C. 2.5 D. 4.0
A. 17 atoms in a molecule of the
compound 120. Two reactants in a flask at room
temperature are producing bubbles of a gas
B. equal molecules of C and H by mass
that turn limewater milky. The reactants
C. twice the mass of oxygen atoms could be
compared to nitrogen atoms
A. zinc and hydrochloric acid
D. twice the mass of nitrogen atoms B. magnesium carbonate and hydrochloric
compared to hydrogen atoms acid
117. Which of the following characteristics is C. methane and oxygen
common to hydrogen, nitrogen, oxygen
and carbon dioxide? D. copper and dilute hydrochloric acid

PAGE 693
www.byjusexamprep.com

Elementary Mathematics
1. Twelve (12) persons can paint 10 identical 5. A milk vendor bought 28 litres of milk at the
rooms in 16 days. In how many days can 8 rate of Rs 8.50 per litre. After adding some
persons paint 20 such rooms? water he sold the mixture at the same
price. If his gain is 12.5%, how much water
A. 12 B. 24 did he add?
C. 36 D. 48 A. 4.5 litres B. 4 litres
2. X and Y together can finish a job in 6 days. C. 3.5 litres D. 3 litres
X can alone do the same job in 12 days. How
long will Y alone take to do the same job? 6. Twelve (12) men work 8 hours per day and
require 10 days to build a wall. If 8 men are
A. 16 days B. 12 days available, how many hours per day must
they work to finish the work in 8 days?
C. 10 days D. 8 days
A. 10 hours B. 12 hours
3. A thief steals a car parked in a house and
goes away with a speed of 40 kmph. The C. 15 hours D. 18 hours
theft was discovered after half an hour and
7. Two equal amounts were borrowed at 5%
immediately the owner sets off in another
and 4% simple interest. The total interest
car with a speed of 60 kmph. When will the
after 4 years amounted to Rs 405. What
owner meet the thief?
was the total amount borrowed?
A. 55 km from the owner’s house and one
A. Rs 1075 B. Rs 1100
hour after the theft.
C. Rs 1125 D. Rs 2250
B. 60 km from the owner’s house and 1.5
hours after the theft 8. Out of 85 children playing badminton or
table tennis or both, the total number of
C. 60 km from the owner’s house and 1.5 girls in the group is 70% of the total number
hours after the discovery of the theft of boys in the group. The number of boys
D. 55 km from the owner’s house and 1.5 playing only badminton is 50% of the
hours after the theft number of boys and the total number of
boys playing badminton is 60% of the total
4. The minute hand of a clock overtakes the number of boys. The number of children
hour hand after every 72 minutes of correct playing only table tennis is 40% of the total
time. How much time does the clock lose or number of children and a total of 12
gain in a day of normal time? children play badminton and table tennis
both. The number of girls playing only
A. Lose minutes
badminton is
A. 14 B. 16
B. Lose minutes
C. 17 D. 35
C. Gain minutes 9. ‘A’ is thrice as good a workman as ‘B’ and
takes 10 days less to do a piece of work than
‘B’ takes. The number of days taken by ‘B’
D. Gain minutes alone to finish the work is

PAGE 694
www.byjusexamprep.com

A. 12 B. 15 A. 5 cm B. 12 cm
C. 20 D. 30 C. 15 cm D. 10 cm
10. The sum of ages of a father, a mother, a son 13. In the figure given below, the diameter of
Sonu and daughters Savita and Sonia is 96 bigger semicircle is 108 cm. What is the
years. Sonu is the youngest member of the area of the shaded region?
family. The year Sonu was born, the sum of
the ages of all the members of the family
was 66 years. If the father’s age now is 6
times that of Sonu’s present age, then 12
years hence, the father’s age will be
A. 44 years B. 45 years
C. 46 years D. 48 years
A. B.
11. In the figure given below, ABCD is a square
of side 4 cm. Quadrants of a circle of C. D.
diameter 2 cm are removed from the four
corners and a circle of diameter 2 cm is also 14. In the figure given below, ABC is a triangle
removed. What is the area of the shaded with AB perpendicular to BC. Further BD is
region? perpendicular to AC. If AD = 9 cm and
DC = 4 cm, then what is the length of BD?

A. 13.36 cm B. 36/13 cm
C. 13/2 cm D. 6 cm
A. B.
15. In the figure given below, If
C. D. and
then QR and AR are
12. In the figure given below, ABC is an respectively
equilateral triangle with each side of length
30 cm. XY is parallel to BC, XP is parallel to
AC and YQ is parallel to AB. If XY + XP + YQ is
40 cm, then the value of PQ is

A. 8.2 cm, 10.4 cm


B. 4.1 cm, 6 cm
C. 2.6 cm, 5.2 cm
D. 4.1 cm, 10.4 cm

PAGE 695
www.byjusexamprep.com

16. In the figure given below, AB is the A. 6 cm B. 8 cm


diameter of the circle whose centre is at O.
C. 9 cm D. 10 cm
Given that then
and respectively are 20. In the figure given below, ABC is a right-
angled triangle where
and On the three sides as
diameters semicircles are drawn as shown
in the figure. The area of the shaded
portion, in square cm, is

A. 32o, 64o B. 64o, 64o


C. 32o, 32o D. 64o, 32o
17. In the figure given below, what is
equal to?
A. pq B.

C. D. pq/2

21. In a race of 1000 m, A beats B by 150 m,


while in another race of 3000 m, C beats D
by 400 m. Speed of B is equal to that of D.
(Assume that A, B, C and D run with uniform
A. 70o B. 75o speed in all the events). If A and C
C. 80o D. 90o participate in a race of 6000 m, then which
one of the following is correct?
18. In the figure given below, ABCD is the
diameter of a circle of radius 9 cm. The A. A beats C by 250 m
lengths AB, BC and CD are equal. B. C beats A by 250 m
Semicircles are drawn on AB and BD as
diameters as shown in the figure. What is C. A beats C by 115.38 m
the area of the shaded region? D. C beats A by 115.38 m
22. A cyclist covers his first 20 km at an average
speed of 40 kmph, another 10 km at an
average speed of 10 kmph and the last 30
km at an average speed of 40 kmph. Then
the average speed of the entire journey is

A. 9π B. 27π A. 20 kmph

C. 36π D. 81π B. 26.67 kmph


19. In the figure given below, the radius of the C. 28.24 kmph
circle is 6 cm and AT = 4 cm. The length of D. 30 kmph
tangent PT is
23. A train 100 m long passes a platform 100 m
long in 10 seconds. The speed of the train is
A. 36 kmph B. 45 kmph
C. 54 kmph D. 72 kmph

PAGE 696
www.byjusexamprep.com

24. In an examination, 50% of the candidates 31. The number of sides of two regular
failed in English, 40% failed in Hindi and polygons are in the ratio 5 : 4. The
15% failed in both the subjects. The difference between their interior angles is
percentage of candidates who passed in 9o. Consider the following statements :
both English and Hindi is
1. One of them is a pentagon and the
A. 20% B. 25% other is a rectangle.
C. 60% D. 75% 2. One of them is a decagon and the other
is an octagon.
25. What is the principal amount which earns
Rs 210 as compound interest for the second 3. The sum of their exterior angles is 720o.
year at 5% per annum?
Which of the above statements is/are
A. Rs 2000 B. Rs 3200 correct?
C. Rs 4000 D. Rs 4800 A. 1 only B. 2 only
26. If then the value of C. 1 and 3 D. 2 and 3
32. If
then is equal to
is
A. abc B. xyz

A. -1 B. 0 C. 0 D. None of these

C. 1 D. 2 33. The number of all pairs (m, n), where m and


n are positive integers, such that
27. The remainder when 3x3 – 2x2y – 13xy2
+ 10y3 is divided by (x – 2y) is equal to is

A. Zero B. y
A. 6 B. 5
C. y – 5 D. y + 3
C. 4 D. 2
28. The sum of all possible products taken two
34. Consider the following statements in
at a time out of the numbers ±1, ±2, ±3,
respect of three 3-digit numbers XYZ, YZX
±4, is
and ZXY:
A. 0 B. -30
1. The sum of the numbers is not divisible
C. 30 D. 55 by
29. If H is the harmonic mean of P and Q then 2. The sum of the numbers is divisible by
the value of is 111.
Which of the above statements is/are
A. 1 B. 2 correct?

C. D. A. 1 only B. 2 only
C. Both 1 and 2 D. Neither 1 nor 2
30. The minimum value of the expression
35. If α and β are the roots of the equation
is
then the value of
A. 5 B. 15/8
is
C. -15/8 D. 0

PAGE 697
www.byjusexamprep.com

A. 1 : 1 B. 2 : 1
A. B.
C. 3 : 1 D. 9 : 1

C. D. 41. In a triangle ABC, the sides AB, AC are


produced and the bisectors of exterior
36. The area of the region bounded externally angles of and intersect at D.
by a square of side 2a cm and internally by If then is equal to
the circle touching the four sides of the
square is A. 115o B. 65o
C. 55o D. 40o
A. B.
42. The chord of a circle is times its radius.
C. D. The angle subtended by this chord at the
minor arc is k times the angle subtended at
37. If ABC is a right-angled triangle with AC as
the major arc. What is the value of k?
its hypotenuse, then which one of the
following is correct? A. 5 B. 2

A. C. 1/2 D. 1/5
43. The areas of two similar triangles are
B.
and respectively.
C. The ratio of their corresponding sides is
D. A. B.
38. If a point P moves such that the sum of the
squares of its distances from two fixed C. D.
points A and B is a constant, then the locus 44. An equilateral triangle, a square and a circle
of the point P is have equal perimeter. If T, S and C denote
A. A straight line the area of the triangle, area of the square
and area of the circle respectively, then
B. A circle which one of the following is correct?
C. Perpendicular bisector of AB A. T < S < C B. S < T < C
D. An arbitrary curve C. C < S < T D. T < C < S
39. If two lines AB and OD intersect at O such 45. If the diagonal of a cube is of length l, then
that then the four angles the total surface area of the cube is
at O are
A. B.
A. 40o, 40o, 140o, 140o
B. 30o, 30o, 150o, 150o C. D.
o o o o
C. 30 , 45 , 75 , 210 46. A rectangular block of length 20 cm,
o o o
D. 60 , 60 , 120 , 120 o breadth 15 cm and height 10 cm is cut up
into exact number of equal cubes. The least
40. Two cones have their heights in the ratio possible number of cubes will be
1 : 3. If the radii of their bases are in the
ratio 3 : 1, then the ratio of their volumes A. 12 B. 16
will be C. 20 D. 24

PAGE 698
www.byjusexamprep.com

47. A solid right cylinder is of height π cm. If its 52. Let and (where ) be the
lateral surface area is half its total surface means of two sets comprising n1 and n2
area, then the radius of its base if (where n2 < n1) observations respectively. If
A. π/2 cm B. π cm is the mean when they are pooled, then
C. 1/π cm D. 2/π cm which one of the following is correct?

48. A semi-circular plate is rolled up to form a A. B.


conical surface. The angle between the
C. D.
generator and the axis of the cone is
A. 60o B. 45o 53. As the number of observations and classes
increases, the shape of a frequency polygon
C. 30o D. 15o
A. Tends to become jagged
49. The lateral surface area of a cone is
B. Tends to become increasingly smooth
Its slant height is 35 cm. The
radius of the base of the cone is C. Stays the same
A. 8.4 cm B. 6.5 cm D. Varies only if data become more
reliable
C. 4.2 cm D. 3.2 cm
54. The median of 19 observations is 30. Two
50. There is path of width 5 m around a circular
more observations are made and the values
plot of land whose area is The
of these are 8 and 32. What is the median
total area of the circular plot including the of the 21 observations?
path surrounding it is
A. 32
A. B.
B. 30
C. D.
C. 20
51. Consider the following statements :
D. Cannot be determined due to
Statement I : Median can be computed insufficient data
even when the end intervals of a frequency
distribution are open. 55. The average marks of section A are 65 and
that of section B are 70. If the average
Statement II : Median is a positional marks of both the sections combined are
average. 67, then the ratio of number of students of
Which one of the following is correct in section A to that of section B is
respect of the above statements? A. 3 : 2 B. 1 : 3
A. Both Statement I and Statement II are C. 3 : 1 D. 2 : 3
true and Statement II is the correct
explanation of Statement I 56. Data on ratings of hotels in a city is
measured on
B. Both Statement I and Statement II are
true and Statement II is not the correct A. Nominal scale B. Ordinal scale
explanation of Statement I C. Interval scale D. Ratio scale
C. Statement I is true but Statement II is Direction: In an examination of Class XII, 55%
false. students passed in Biology, 62% passed in
D. Statement I is false but Statement II is Physics, 60% passed in Chemistry, 25% passed in
true. Physics and Biology, 30% passed in Physics and

PAGE 699
www.byjusexamprep.com

Chemistry, 28% passed in biology and Chemistry. C. In the second and third quadrants
Only 2% failed in all the subjects.
D. In the third and fourth quadrants
57. What is the ratio of number of students
64. The ratio of the sum and difference of the
who passed in both Physics and Chemistry
ages of the father and the son is 11 : 3.
to number of students who passed in both
Consider the following statements :
Biology and Physics but not Chemistry?
A. 7 : 10 B. 10 : 7 1. The ratio of their ages is 8 : 5.

C. 9 : 7 D. 7 : 9 2. The ratio of their ages after the son


attains twice the present age will be
58. If the number of students is 360, then how 11 : 8.
many passed in at least two subjects?
Which of the statements given above is/are
A. 270 B. 263 correct?
C. 265 D. 260
A. 1 only B. 2 only
59. What percentage of students passed in
C. Both 1 and 2 D. Neither 1 nor 2
exactly one subject?
65. There are n zeros appearing immediately
A. 21 B. 23
after the decimal point in the value of
C. 25 D. 27 (0.2)25. It is given that the value of
60. What percentage of students passed in all The value of n is
the three subjects?
A. 25 B. 19
A. 6 B. 5
C. 18 D. 17
C. 4 D. 3
66. If and then the
61. If α, β and γ are the zeros of the polynomial
then value of will be equal to
is equal to
A. -1 B. 0
A. B.
C. 1 D. 3

C. D. 67. If and
then the value of is equal to
62. It is given that the equations
A. 0.9286 B.
and have single positive
C. 1.8572 D.
solution. For this, the value of ‘a’ is
68. The product of two integers p and q, where
A. B. 2 and is 7168 and their HCF
C. D. 1 is 16. The sum of these two integers is

63. The solution of linear inequalities A. 256 B. 184


and lies C. 176 D. 164
A. Only in the first quadrant 69. If the sum of the squares of three
B. In the first and second quadrants consecutive natural numbers is 110, then
the sum of their cubes is

PAGE 700
www.byjusexamprep.com

A. 625 B. 654 75. If then which one of the


C. 684 D. 725 following is correct?
A. B. or
70. If then k is equal to
C. or D. only

A. B. 76. A five-digit number XY235 is divisible by 3


where X and Y are digits satisfying
What is the number of possible
C. D. pairs of values of (X, Y)?

71. Two workers ‘A’ and ‘B’ working together A. 5 B. 6


completed a job in 5 days. Had ‘A’ worked C. 7 D. 9
twice as efficiently as he actually did and ‘B’
worked one-third as efficiently as he 77. How many five-digit numbers of the form
actually did, the work would have XXYXX is/are divisible by 33?
completed in 3 days. In how many days A. 1 B. 3
could ‘A’ alone complete the job?
C. 5 D. Infinite
A. days B. days 78. If then the value of the
expression will be
C. days D. days A. 2 B. 1

72. If α and β are two real numbers such that C. 0 D. -2

and where 1 < p < q 79. If and then the


value of abc is
< r, then which one of the following is the
greatest? A. 1 B. 2
C. -1 D. 0
A. B.
80. The highest four-digit number which is
divisible by each of the numbers 16, 36, 45,
C. D. 48 is

73. HCF and LCM of two polynomials are (x+3) A. 9180 B. 9360
and respectively. If one C. 9630 D. 9840

of the two polynomials is 81. The sides of a triangle are 5 cm, 6 cm and 7
then the other is cm. The area of the triangle is
approximately
A. B.
A. B.
C. D.
C. D.
74. The number of divisors of the number
38808, exclusive of the divisors 1 and itself, 82. Let S be the parallelogram obtained by
is joining the mid-points of the parallelogram
A. 74 B. 72 T. Consider the following statements:

C. 70 D. 68 1. The ratio of area of T so that of S is 2 : 1.

PAGE 701
www.byjusexamprep.com

2. The perimeter of S is half of the sum of 88. If then what is the value
diagonals of T. of
Which of the above statements is/are
A. -1 B. 0
correct?
C. 1 D. 8
A. 1 only B. 2 only
C. Both 1 and 2 D. Neither 1 nor 2 89. Consider the following statements:

83. Walls (excluding roofs and floors) of 5 1. can never be equal


identical rooms having length, breadth and to 1.5
height 6 m, 4 m and 2.5 m respectively are
to be painted. Out of five rooms, two rooms 2. can never be less
have one square window each having a side than 4
of 2.5 m. Paints are available only in cans of Which of the statements given above is/are
1 litre; and 1 litre of paint can be used for correct?
painting 20 square metres. The number of
cans required for painting is A. 1 only B. 2 only

A. 10 B. 12 C. Both 1 and 2 D. Neither 1 nor 2


C. 13 D. 14 90. Consider the following statements:
84. A circle is inscribed in an equilateral triangle
1. where p, q are non-
of side of length l. The area of any square
inscribed in the circle is zero real numbers, is possible only
when p = q.
A. B.
2. where p, q are
C. D.
non-zero real numbers, is possible only
85. If base and hypotenuse of a right triangle when p = q.
are and respectively and Which of the statements given above is/are
the area of the triangle is 2016 square units, correct?
then the perimeter of the triangle may be A. 1 only B. 2 only
A. 224 units B. 288 units
C. Both 1 and 2 D. Neither 1 nor 2
C. 448 units D. 576 units
91. Consider the following statements:
86. If and
then which one of the following is 1. If where
correct?
then
2 2 2
A. (m + n ) = mn B. 2. If where
C. D. then

87. If then what is the Which of the statements given above is/are
correct?
value of ?
A. 1 only B. 2 only
A. 9 B. 12
C. Both 1 and 2 D. Neither 1 nor 2
C. 16 D. 18

PAGE 702
www.byjusexamprep.com

97. Ten (10) years before, the ages of a mother


92. Let and where
and her daughter were in the ratio 3:1. Ion
A, B are acute angles. What is another 10 years from now, the ratio of
equal to? their ages will be 13:7. What are their
present ages?
A. 1/2 B. A. 39 yrs, 21 yrs B. 55 yrs, 25 yrs

C. D. 1 C. 75 yrs, 25 yrs D. 49 yrs, 31 yrs


98. X bought 4 bottles of lemon juice and Y
93. On the top of a hemispherical dome of bought one bottle of orange juice. Orange
radius r, there stands a flag of height h. juice per bottle costs twice the cost of
From a point on the ground, the elevation lemon juice per bottle. Z bought nothing
of the top of the flag is 30o. After moving a but contributed Rs. 50 for his share of the
distance d towards the dome, when the flag drink which they mixed together and
is just visible, the elevation is 45o. The ratio shared the cost equally. If Z's Rs. 50 is
of h to r is equal to covered from his share, then what is the
cost of one bottle of orange juice?
A. B.
A. Rs. 75 B. Rs. 50
C. Rs. 46 D. Rs. 30
C. D.
99. According to Mr. Sharma's will, half of his
94. If 0 < θ < 90° , 0 <  < 90° and cosθ < cos , property goes to his wife and the rest is
then which one of the following is correct? equally divided between his two sons, Ravi
and Raj. Some years later, Ravi dies and
A. θ <  leaves half of his property to his widow and
B. θ >  rest to his brother Raj. When Raj dies he
leaves half of his property to his widow and
C. θ +  = 90° remaining to his mother, who is still alive.
The mother now owns Rs. 88,000 worth of
D. No conclusion can be drawn
the property. The total worth of the
property of Mr. Sharma was?
95. If where then
A. Rs. 1,00,000 B. Rs. 1,24,000
is equal to C. Rs. 1,28,000 D. Rs. 1,32,000
100. A person bought two articles X and Y from
A. 4/3 B. -4/3
a departmental store. The sum of prices
C. 1/3 D. -2/3 before sales tax was Rs. 130. There was no
sales tax on the article X and 9% sales tax
96. In a class of 60 boys, there are 45 boys who
on the article Y. The total amount the
play chess and 30 boys who play carrom. If
person paid, including the sales tax was Rs.
every boy of the class plays at least one of
136.75. What was the price of the article Y
the two games, then how many boys play
before sales tax?
carrom only?
A. 30 B. 20 A. Rs. 75 B. Rs. 85

C. 15 D. 10 C. Rs. 122 D. Rs. 125

PAGE 703
www.byjusexamprep.com

ANSWERS KEY II 2018


ENGLISH

1 C 21 A 41 A 61 C 81 C 101 B
2 B 22 B 42 A 62 A 82 C 102 D
3 A 23 B 43 D 63 B 83 C 103 C
4 D 24 A 44 A 64 B 84 D 104 D
5 A 25 B 45 D 65 A 85 D 105 C
6 D 26 A 46 A 66 A 86 D 106 A
7 D 27 B 47 D 67 D 87 A 107 D
8 A 28 D 48 C 68 D 88 B 108 A
9 D 29 B 49 B 69 B 89 D 109 B
10 C 30 D 50 B 70 A 90 D 110 B
11 A 31 B 51 A 71 B 91 B 111 A
12 B 32 C 52 D 72 A 92 D 112 C
13 C 33 C 53 D 73 C 93 A 113 B
14 A 34 D 54 D 74 C 94 C 114 A
15 B 35 D 55 B 75 A 95 A 115 C
16 A 36 A 56 C 76 C 96 B 116 A
17 B 37 A 57 A 77 A 97 A 117 B
18 C 38 B 58 C 78 D 98 D 118 D
19 C 39 A 59 D 79 A 99 C 119 A
20 B 40 B 60 D 80 C 100 A 120 B

GENERAL KNOWLEDGE
1 D 21 A 41 B 61 C 81 B 101 A
2 B 22 A 42 B 62 A 82 B 102 B
3 D 23 B 43 A 63 C 83 D 103 D
4 B 24 A 44 D 64 D 84 C 104 B
5 C 25 C 45 B 65 D 85 A 105 B
6 B 26 C 46 C 66 B 86 C 106 B
7 B 27 C 47 B 67 A 87 B 107 D
8 B 28 B 48 A 68 B 88 A 108 D
9 C 29 C 49 B 69 D 89 D 109 A
10 B 30 A 50 A 70 D 90 B 110 A
11 A 31 D 51 C 71 A 91 C 111 D
12 C 32 B 52 B 72 A 92 C 112 D

PAGE 704
www.byjusexamprep.com

13 D 33 A 53 A 73 D 93 C 113 A
14 D 34 D 54 D 74 D 94 A 114 B
15 C 35 C 55 C 75 C 95 C 115 A
16 D 36 D 56 B 76 B 96 D 116 D
17 D 37 B 57 A 77 D 97 A 117 B
18 B 38 B 58 C 78 C 98 D 118 A
19 A 39 B 59 A 79 C 99 C 119 A
20 A 40 A 60 C 80 A 100 A 120 B

Elementary Mathematics
1 D 21 C 41 B 61 D 81 B
2 B 22 B 42 B 62 A 82 A
3 B 23 D 43 A 63 B 83 B
4 A 24 B 44 A 64 B 84 D
5 C 25 C 45 D 65 D 85 B
6 C 26 B 46 D 66 B 86 B
7 D 27 A 47 B 67 B 87 D
8 A 28 C 48 C 68 C 88 C
9 B 29 B 49 C 69 C 89 C
10 D 30 B 50 B 70 B 90 C
11 C 31 D 51 B 71 D 91 C
12 D 32 D 52 A 72 C 92 C
13 C 33 C 53 B 73 C 93 A
14 D 34 B 54 B 74 C 94 B
15 D 35 B 55 A 75 C 95 B
16 B 36 A 56 B 76 C 96 C
17 C 37 B 57 B 77 B 97 B
18 B 38 B 58 A 78 A 98 B
19 B 39 B 59 B 79 A 99 C
20 D 40 C 60 C 80 B 100 A

PAGE 705
www.byjusexamprep.com

PAGE 706
CDS II 2018
www.byjusexamprep.com

(Solutions)
ENGLISH
1. Ans. C.
The given sentence is in active form of simple past tense. The structures for active/passive voices
are:
Active: Subject + verb (IInd form) + object...
Passive: Object + was/were + verb (IIIrd form) + by + subject...
The question statement is in simple past tense, so the converted statement too should be in the
same. This eliminates options A and B.
Out of options C and D, the latter has ambiguities in the meaning. So, the correct response is option
C.
2. Ans. B.
The passive voice of imperative sentences which suggest order, suggestion or request can be made
in two ways:
Active: Verb + object
Passive: 1. Let + object + be + past participle
2. You are requested/ordered/suggested + to + verb (Ist form) + object
Going by first way of passive voice, the passive voice of the given sentence would be:
Let the door be shut.
3. Ans. A.
The given sentence is of present perfect tense and it is in active form. The structures for
active/passive voices are:
Active: Subject + has/have + verb (IIIrd form) + object...
Passive: Object + has/have + been + verb (IIIrd form) + by + subject...
Options B and D are incorrect as it uses the incorrect helping verb ‘has’, while the subject is plural.
Option C is incorrect as the original statement is in present perfect tense and the converted
statement too needs to be in the same.
Option A is the correct answer.

PAGE 707
www.byjusexamprep.com

4. Ans. D.
The given sentence is of present perfect tense and it is in active form. The structures for
active/passive voices are:
Active: Subject + has/have + verb (IIIrd form) + object...
Passive: Object + has/have + been + verb (IIIrd form) + by + subject...
The active voice statement is in present perfect tense, thus the passive voice statement has to be in
present perfect tense itself. Out of the available options, only option D satisfies this criterion and
thus, is the correct response.
5. Ans. A.
Basic rules to be followed for Active/Passive conversions are:
1. The object of the active verb becomes the subject of the passive verb.
2. The finite form of the verb is changed (to be+ past participle).
3. The subject of the active sentence becomes the object of the passive sentence (or is dropped).
4. Preposition "by" is used before object.
The given sentence is in active voice. It is simple form of present tense. The structures for
active/passive voices are:
Active: Subject + verb (“s“ or “es” with singular noun) + object…
Passive: Object + Is/are/am + verb (IIIrd form) + by + subject…
6. Ans. D.
As the information in the direct speech statement is a universal fact thus it remains in simple
present tense even after conversion to indirect speech. This only leaves options B and D possible.
Option B is eliminated as it uses the phrase ‘all of us.’
So, option D is the correct response.
7. Ans. D.
The direct speech statement is in the form of a question, so the verb ‘ask’ in past participle form is
the most viable option. Thus, option D is the correct response.
8. Ans. A.
The statement in direct speech is a question which Romila is asking to Rahim. So, only options A
and B are suitable. The past indefinite tense in direct speech gets converted to past perfect tense in
indirect speech.
Out of the available options, only option A satisfies both these criteria and thus is the correct
response.
9. Ans. D.
Option A is incorrect because of the use of ‘our business’ phrase.
Option B is incorrect on multiple counts like missing helping verb and inappropriate conversion of
time.

PAGE 708
www.byjusexamprep.com

Option C uses the verb ‘said’, which is inappropriate in the context of this conversion.
Option D is the correct conversion of the given sentence into indirect speech.
10. Ans. C.
The statement in the direct speech is a request made in the interrogative form, so the use of the
verb ‘requested’ will be suitable in indirect speech. So, the correct response is option C.
11. Ans. A.
The conversion to reported speech requires the understanding of the intention and the tone of the
action. The Captain is entitled to order his soldiers. So, option A is the correct response.
12. Ans. B.
Soggy means very moist; unpleasantly wet and soft.
Sodden means saturated with liquid, especially water; soaked through.
Thus, the word “dry” is the correct antonym of the given word.
13. Ans. C.
Rebuttal means an instance of refusing evidence or an accusation.
Acceptance means the action of consenting to receive or undertake something offered.
Thus, the word “acceptance” is the correct antonym of the given word.
14. Ans. A.
Efficacy means efficiency; the ability to produce a desired or intended result.
Inefficiency is the state of not achieving maximum productivity; failure to make the best use of
time or resources.
Thus, the word “inefficiency” is the correct antonym of the given word.
15. Ans. B.
Disdainful means disgusting or hateworthy.
Sneering means smile or speak in a contemptuous or mocking manner.
Mocking means making fun of someone or something in a cruel way; derisive.
Cheerful means noticeably happy and optimistic.
Thus, the word “respectful” is the correct antonym of the given word.
16. Ans. A.
Inadvertently means without intention; accidentally.
Purposely means on purpose; intentionally.
Purposively is not a valid word.
Thus, the word “purposely” is the correct antonym of the given word.

PAGE 709
www.byjusexamprep.com

17. Ans. B.
Distinctive means characteristic of one person or thing, and so serving to distinguish it from others.
Salient means unique or characteristic feature.
Thus, the word “common” is the correct antonym of the given word.
18. Ans. C.
Let us understand the meaning of the above words:-
Foe = enemy.
Protagonist = the leading character or one of the major characters in a play, film, novel, etc.
Opponent = someone who competes with or opposes another in a contest, game, or argument.
Soul mates = a person with whom one has a feeling of deep or natural affinity.
Hence, option C is the correct answer.
19. Ans. C.
Let us understand the meaning of the given words :-
Mutinous = rebellious.
Arrogant = rude.
Obedient = complying or willing to comply with an order or request; submissive to another's
authority.
Sincere =free from pretence or deceit; proceeding from genuine feelings.
Thus, the word “obedient” is the correct antonym of the given word.
Hence, option C is the correct answer.
20. Ans. B.
Let us understand the meaning of the given words :-
Woebegone = sad or miserable in appearance.
Cheerful = joyous.
Sleepy = ready to fall asleep.
Thoughtful = having intellectual depth.
Hence, option B is the correct answer.
21. Ans. A.
Sanguine means optimistic or positive, especially in an apparently bad or difficult situation.
Pessimistic means having a negative approach.
Optimistic means having a positive approach to something.
Humorous means happy, joyful.

PAGE 710
www.byjusexamprep.com

Rebellious means showing a desire to resist authority, control, or convention.


Thus, the word “pessimistic” is the correct antonym of the given word.
22. Ans. B.
Let us understand the meaning of the given words :-
Obscure = unclear, irrelevant.
Infamous = notorious and ill-famed.
Obscene = vulgar and unpleasant.
Clear = readily apparent to the mind.
New = not of long duration; having just (or relatively recently) come into being or been made or
acquired or discovered.
Hence, option B is the correct answer.
23. Ans. B.
RQ is a mandatory pair as R introduces the idea of mutual discussions before arriving at a decision,
while Q mentions the contrast. S addresses the feature mentioned in Q and so is the next
statement. P points out to the overall process and thus is the last statement. So, the correct
response is option B.
24. Ans. A.
The most appropriate sequence for the given jumbled sentences is SQPR.
Hence, option A is the correct answer.
25. Ans. B.
The most appropriate sequence for the given jumbled sentences is PQSR.
Hence, option B is the correct answer.
26. Ans. A.
R is the first statement as it points out to the subject in S1 and provides some extra anecdotes
about its functions. Out of the available options, only A starts with R and thus is the correct
response.
27. Ans. B.
Q is the first statement as it derives a contrast from the discussion in S1. P is the next statement as
it collates the data from S1 and Q to give a conclusion. S points out to a process that has been
mentioned in P, making R the last statement of the sequence. So, the correct response is option B.
28. Ans. D.
This sequence can be solved in reverse order. S6 clearly tries to draw a contrast about the nature of
decentralized planning; this has been discussed only in statement Q making it the last statement
before S6. Out of the available options, only option D satisfies this criterion and thus is the correct
response.

PAGE 711
www.byjusexamprep.com

29. Ans. B.
The most appropriate sequence for the given jumbled sentences is SQPR.
Hence, option B is the correct answer.
30. Ans. D.
RQ is a mandatory pair as both these statements together form an example to explain the question
posed in S. P is the next statement as it is the consequence of RQ. S and S6 are effectively the
extensions of each other. So, the correct response is Option D.
31. Ans. B.
S is the first statement as it uses the information mentioned in S1 and draws a conclusion out of it.
R is the next statement as it puts the information and conclusion attained in the previous
statements in a more comprehensive form. QP is a mandatory pair as P is the extension of the data
provided in Q. So, the correct response is option B.
32. Ans. C.
Q provides the description of the Dhauladhar range that has been mentioned in S1. QS is a
mandatory pair because they have the ‘although-despite that’ pairing. S will come after P as S is the
consequence of the information provided in P. Thus, the correct response is Option C.
33. Ans. C.
S is the first part as it mentions the characteristic of the subject. It is followed by Q. The only option
that satisfies both these criteria is option C. Thus, option C is the correct response. The complete
sentence is:
“The Himalayan range that towers over the Kangra valley sacred to the Gaddi people is home to a
chain of high-altitude lakes.”
34. Ans. D.
RQ is a mandatory pair as it talks about the functions of the subject in continuation. S is the next
part as it provides an object to the functions stated in RQ. P is the last statement as it mentions the
secondary subject of the subject. The complete sentence is:
“National Building Organisation collects, tabulates and disseminates the statistical information on
housing and building construction activities besides conducting surveys on housing.”
35. Ans. D.
The correct response is option D. The complete sentence is:
“Elementary education is inevitable in developing the children to further education, thereby
ensuring the growth of a nation.”
36. Ans. A.
This statement should be ordered using the options. Out of the available options, only option A
gives a grammatically and contextually viable output. Thus, option A is the correct response.
We tend to learn with interest when we see beauty in our work and connect learning with real life
experiences.

PAGE 712
www.byjusexamprep.com

37. Ans. A.
The correct sequence is option A.
"Children of a warm home and supportive surroundings are like the tender saplings that grow into
beautiful trees with the sunshine and rain."
38. Ans. B.
R is the first part as the fixed portion ends with ‘of’ and none of the statements begin in way that
can be preceded by ‘of’; P too seems probable till you analyse the whole sequence, which will
render the complete statement illogical.
So, the correct response is option B. The complete sentence is:
“The best part of literary flourishes long formed journalism is that it brings back the importance of
writing skills and locates the story with the larger framework of our world.”
39. Ans. A.
So, option A is the correct response. The complete sentence is: PQSR
“The government must offer convincing solutions to the crises in the rural economy and job
markets that are causing social ferment.”
40. Ans. B.
‘The speaker’ is the fixed subject. Out of the available options, only part Q can follow the subject in
this case.
RP is a mandatory pair as P is the explanation of R.
So, the correct response is option B. The complete sentence is:
“The speaker has identified many issues besides charging the opponents of their inaction.”
41. Ans. A.
The subject is fixed, so the first part should be S as it elaborates on the subject; this leaves only
options A and C in the contention.
If we put the parts in order given in option A and C, we end up realising that the sequence in option
C fails to make sense. Thus, option A is the correct response. The complete sentence is:
“One of the difficulties is that we want to transform the whole of mankind or affect the masses the
day after tomorrow.”
42. Ans. A.
The subject of the sentence is given in part (S) as "the man". Part S would be followed by P as more
information about the man has been given in P. Q and R together completes the given sentence.
Thus, the correct sequence is SPQR and the sentence would read as:
“The man in red who stood first in the competition has been elected as the chairperson of the
sports committee.”
43. Ans. D.
This statement is best ordered if we analyse and try to fit the given parts in the options provided.
Only option D justifies the statement and thus is the correct response. The complete sentence is:

PAGE 713
www.byjusexamprep.com

“I am a votary of truth and I must say what I feel and think at a given moment without regards to
what I may have said before.”
44. Ans. A.
This ordering is based on the sequencing of the events in the statement. Firstly, the subject ‘time’ is
introduced through part S. It is followed immediately by P as it is in continuation with the
expression from Gandhi in part S. PR is a mandatory pair. So, option A is the correct response. The
complete sentence is:
“There was a time when Gandhi expressed the opinion that a human life could span 125 years and
that he would quite like to live that long.”
45. Ans. D.
Q is the first part as it has the subject ‘free press in it’; this leaves only options B and D.
RS is a mandatory pair as parliament is elected by people, while the judiciary is not. So, option D is
the correct response. The complete sentence is:
“A free press is as essential a limb of democracy as a parliament freely elected by the people or an
independent judiciary.”
Thus, the correct sequence is Q R S P.
46. Ans. A.
The statement can’t start with P as it clearly is the follow-up part of the statement. This eliminates
option D. The statement can’t start with S as the whole sequence fail to generate any viable
meaning. Now, out of the two available options, only option that makes sense as P is the follow-up
statement of S. The complete sentence is:
“But surely there must be something important enough that everyone should learn it and that is
‘To learn to say I am sorry.”
Thus, the correct order of the sentence is R Q S P.
47. Ans. D.
Out of the available parts, only parts S or Q have viable subjects, so option A is eliminated.
Only option B starts with Q, but the whole sequence fails to make sense and thus is eliminated.
S and R form a mandatory pair; thus, option D is the correct response. The complete sentence is:
“Rich materials found from excavation sites indicate that the people had a subsistence economy
and they largely relied on agriculture, fishing and hunting.”
48. Ans. C.
Using the options to check the contextual accuracy is the best way to go along in small statements
like this. Out of the available options, only option C gives a proper meaning and thus is the correct
response. The complete sentence is:
“It is in this context that the role of agriculture as a provider of jobs and prosperity must be
viewed.”

PAGE 714
www.byjusexamprep.com

49. Ans. B.
The statement needs to start with part R in order to have a viable subject. RQ is a mandatory pair.
The arrangement of S and P is done based on their suitability in making the sentence complete.
The complete sentence is:
“We are living at a time when great historical figures are appropriated, that suit partisan political
objectives and reduced into stereotypes.”
50. Ans. B.
Analysing the given parts of statements, only part R starts with a valid subject and thus is the
opening statement. Now analyse the two options starting with part R. Only option B makes sense
and thus is the correct response. The complete sentence is:
“A valid criticism of the profession of politics in India is that it is not professional enough as the
majority of its practitioners have not done their homework.”
51. Ans. A.
Using the options to check the contextual accuracy is the best way to go along in small statements
like this. Out of the available options, only option A gives a proper meaning and thus is the correct
response. The complete sentence is:
“By means of education, it is difficult in modern civilization to bring about an integrated individual.”
52. Ans. D.
S is the first statement as it mentions the subject of the statement; safety questions about cell
phones.’ RQ forms a mandatory pair as R ends with ‘and’ and Q is the only statement that can
suitably follow this connector. Only option D satisfies both these criteria, so the correct response is
option D. The correct sentence would be:
Safety questions about cell phones have drawn intense interest and debate for years as the devices
have become integral to most people's lives
53. Ans. D.
‘A pearl of wisdom’ means a succinct, insightful saying, piece of advice, or moral precept. Thus,
option D is the correct answer.
54. Ans. D.
‘At the crossroads” means a situation that requires some important choice to be made. Thus,
option D is the correct answer.
55. Ans. B.
‘Close shave’ is a situation in which you come very close to a dangerous situation. Thus, option B is
the correct answer.
56. Ans. C.
‘To follow suit’ means to emulate somebody else’s actions. So, option C is the correct answer.
57. Ans. A.
On the fly means doing something without due preparedness or thought. Thus, option A is the
correct answer.

PAGE 715
www.byjusexamprep.com

58. Ans. C.
The blank can be solved by comparing the numerical suitability of the blank and the subject in the
statement. Only option C has plural helping verb and thus is the correct response.
59. Ans. D.
Option A is incorrect as it takes a plural helping verb while the object is clearly singular.
Option C is incorrect as it takes the incorrect tense.
Still is used to denote the continuity of the action at that present time and till is used to describe
the action up to a definite period of time.
So, option D is the correct response.
60. Ans. D.
Here, ‘smiling’ will take the preposition ‘at’ as the idea is to show the subjects are smiling towards
each other.
61. Ans. C.
Beastly - This fits contextually in the passage as the idea is to signify that humans are better than
machines which simply follow orders and automatons. Hence, option C is the correct answer.
62. Ans. A.
This blank is a phrasal filler. ‘Superman of some kind’ phrase is used to as a jibe to signify the level
of intellect or power greater than the human standards.
63. Ans. B.
Options C and D are incorrect as they fail to make any sense in the context of the passage. Option B
is the correct response as it uses the adverb form of noun ‘quick’, which is the requirement in the
context of the statement.
64. Ans. B.
‘To be’ is required as the context is of an aspiration. As a rule, ‘to be’ is always followed by a past
participle form of verb’. So, the correct response is option B.
65. Ans. A.
‘Different’ or ‘difference’ can’t be used in the blank as there are no comparisons made. Also, the
blank needs an adverb form to make it contextually and grammatically correct. So, the correct
response is option A.
66. Ans. A.
The blank points to a singular definite style of writing, so the article ‘the’ is the most appropriate
response.
67. Ans. D.
Enrage means to make (someone) very angry.
Enlarge means to make or become larger or more extensive.
Engrave means to cut or carve (a text or design) on the surface of a hard object.

PAGE 716
www.byjusexamprep.com

The past participle form of ‘enrage’ will satisfy the blank grammatically and contextually. So, option
D is the correct response.
68. Ans. D.
The context of the statement and its successor makes it clear that the idea is to create a
chronological comparison. This particular blank is meant to convey a meaning of start; options B
and D convey this meaning.
Looking closely, option B is eliminated as it doesn’t complete the sentence grammatically. So, the
correct response is option D.
69. Ans. B.
‘That’ is used to point to an object or item that is away from the point of speech, which is not the
case here.
‘These’ is used for multiple subjects, which is not the case here.
‘Which’ is used to introduce a non-restrictive relative clause, which is not the case here.
So, the correct response is option B.
70. Ans. A.
The context of the blank is to show the first example or occurrence of the art form. Out of the
available options only ‘instance’ serves this purpose. Thus, option A is the correct response.
71. Ans. B.
The previous sentence has set the tense of the conversation as simple present. So, option B is the
correct response.
72. Ans. A.
Option B would have been correct if ‘Japan’ was followed by ‘and’. In this case, the first sentence
would be a different sentence and would belong to simple present tense.
Option C is incorrect because it violates parallelism.
Option D is contextually incorrect.
So, the correct response is option A.
73. Ans. C.
When ‘between’ is used to point out two terminals then the two terminals are connected using
‘and’ instead of ‘to’.
74. Ans. C.
The error is in the last part has the verb ‘discuss’ does not require a preposition to be used after it.
75. Ans. A.
The error is in the verb form usage of ‘reached’; instead of the past participle form ‘reached’, the
base form ‘reach’ is used with ‘did’.

PAGE 717
www.byjusexamprep.com

76. Ans. C.
Replace “preferred coffee much more than tea” with “preferred coffee much more to tea”.
“Prefer” takes preposition “to” with it.
77. Ans. A.
The error is in the usage of ‘which’. ‘Which’ is used for non-living things. It needs to be replaced by
‘who’ as the subject is a living noun.
78. Ans. D.
There is no error in the statement. So, the correct response is option D.
79. Ans. A.
The error is the incorrect usage of possessive noun ‘visitor’s’; it needs to be replaced by visitors.
80. Ans. C.
‘Today night’ is the incorrect usage. It needs to be replaced by ‘tonight’.
81. Ans. C.
The clause ‘both of them…….’ in part B deduces that number of boys is two and for two persons,
we use “each other” instead of ‘one another’.
82. Ans. C.
The error is in part (C) of the sentence. With gerund, we use possessive pronoun. Therefore, the
use of “it” is incorrect and it should be replaced by “its”.
83. Ans. C.
Demolition means destruction, damage etc.
84. Ans. D.
The author has very critically analysed and mentioned that the mob is constantly being used as a
political tool for the destructive actions.
85. Ans. D.
The author mentions that there have been instances when embassies have been attacked by
students as a part of fulfilling a political agenda. The governments have been using this kind of
measures to force the hands of other nations to abide by their ideologies and decisions.
86. Ans. D.
The opening lines of the passage states that while there are so many constructive and
humanitarian activities going on, a lot of destructive activities too are being done alongside. So, the
correct response is option D.
87. Ans. A.
Vulnerable means exposed to the possibility of being attacked or harmed, either physically or
emotionally.
In the context of the passage, the phrase has been used to point out that if someone is showing
weakness then they are vulnerable.

PAGE 718
www.byjusexamprep.com

88. Ans. B.
The passage is woven around the idea that one is more susceptible to being attacked when they
show hesitation or weakness. The comparison with dogs is a means to emphasize on the fact that
the both men and dogs attack when the other person is weak.
89. Ans. D.
The author has used simile figure of speech as a means to show that if one shows fear and
hesitation just like dogs, others tend to try to scare him/her further.
90. Ans. D.
Sympathetic means feeling, showing, or expressing sympathy.
Indifferent means having no particular interest or sympathy; unconcerned.
Admiration means respect and warm approval.
Ruthless means having or showing no pity or compassion for others.
In the passage, it has been mentioned that if one shows fear then the others tend to approach all
the more mercilessly. Therefore, “ruthless” is the correct answer.
91. Ans. B.
The agreement between the woman and her brother was that if everything was going well, then
he’ll just send a blank paper in the letter.
92. Ans. D.
It has been mentioned in the passage that the letter contained a blank paper.
93. Ans. A.
Mr Hill got very anxious that the letter was from the woman’s brother and yet she is returning it
instead of paying for the postage. So, with an intention to help the woman he paid for the postage.
94. Ans. C.
The woman had an agreement with her brother that if the letter was blank then there is no need to
pay for the postage. The woman thus already knew the content of the letter, so she returned the
letter.
95. Ans. A.
Irony is when a statement means the opposite of its literal meaning.
In the story, the woman returns her brother’s letters, which seemed very weird as it is very obvious
as a sibling to want to know about the wellbeing of the other. To the outsider, it seemed like as if
she didn’t care. But in reality, she was just using her intellect to avoid paying for the postage while
getting the news of brother as well.
96. Ans. B.
Dominance means power and influence over others.
Prevalence means commonness.
Affection means the act of affecting or the state of being affected.

PAGE 719
www.byjusexamprep.com

In the context of the passage, we can say that the idea that the frequency with which a disease is
seen amongst the people is directly dependent on the food habits of the people. For example, as
the senior citizens of India consume turmeric in their staple diet thus, they have lower tendency of
Alzheimer’s disease.
97. Ans. A.
It has been mentioned in the passage that turmeric is a source of curcumin, which helps in
countering the Alzheimer’s disease, thus the correct response is option A.
98. Ans. D.
Earlier is used to point to a time before the moment at which the sentence is being spoken. Out of
the available options, only ‘previously’ has this effect and thus the correct response is option D.
99. Ans. C.
In the second line of the passage, it has been mentioned that the research has shown that
curcumin has positive effects on the people without dementia as well as those who are suffering
from the Alzheimer’s disease.
100. Ans. A.
Turmeric has been mentioned as a medical option for the persons suffering from Alzheimer’s
disease. In the last line it has been mentioned that as the Indian senior citizens have turmeric in
their staple diet, thus the numbers of Alzheimer’s patients are less. Statement 1 is in contradiction
to this statement.
All the other statements have been mentioned in the passage, thus the correct response is option
A.
101. Ans. B.
Let's first learn the meanings of the words:
Engaging = charming and attractive.
Carefree = free from anxiety or responsibility.
Appealing = attractive or interesting.
Thinking = the process of considering or reasoning about something.
Dreaming = to experience dreams during sleep.
Hence, option B is the correct answer.
102. Ans. D.
Derisive means expressing contempt or ridicule.
Mockery means teasing and contemptuous language or behaviour directed at a particular person
or thing.
Conducive means making a certain situation or outcome likely or possible.
Encouraging means giving someone support or confidence; supportive.
Contemptuous means showing contempt; scornful.
So, option D is the correct answer.

PAGE 720
www.byjusexamprep.com

103. Ans. C.
Detrimental means tending to cause harm.
Beneficial means resulting in good; favourable or advantageous.
Harsh means unpleasantly rough or jarring to the senses.
Harmful means causing or capable of causing harm.
Demanding means (of a task) requiring much skill or effort.
So, option C is the correct answer.
104. Ans. D.
Let's first learn the meanings of the words:
Extradition = the action of extraditing a person accused or convicted of a crime.
Acceptance = the action of consenting to receive or undertake something offered.
Sentence = to declare the punishment decided for (an offender).
Extension = a part that is added to something to enlarge or prolong it.
Deportation = the action of deporting a foreigner from a country.
Hence, option D is the correct answer.
105. Ans. C.
Impede means to delay or prevent (someone or something) by obstructing them; hinder.
Facilitate means to make (an action or process) easy or easier.
Hinder means top to make it difficult for (someone) to do something or for (something) to happen.
Accept means to consent to receive or undertake (something offered).
Thus, option C is the correct answer.
106. Ans. A.
Ineptitude means the lack of skill or ability.
Incompetence means inability to do something successfully; ineptitude.
Courage means the ability to do something that frightens one; bravery.
Gossip means casual or unconstrained conversation or reports about other people, typically
involving details which are not confirmed as true.
Thus, option A is the correct answer.
107. Ans. D.
Condone means approve or sanction (something), especially with reluctance.
Regard means to respect and acknowledge.
Punish means to inflict a penalty or sanction on someone for (such an offence).

PAGE 721
www.byjusexamprep.com

Overlook means to ignore or disregard (something, especially a fault or offence).


Thus, option D is the correct answer.
108. Ans. A.
Desolated means utterly wretched and unhappy.
Deserted means feeling very unhappy and lonely.
Joyful means feeling, expressing, or causing great pleasure and happiness.
109. Ans. B.
Salubrious means pleasant; not run-down.
Provoking means causing annoyance; irritating.
Pleasant means causing happiness.
Ridiculous means deserving or inviting derision or mockery; absurd.
Thanking means showing gratitude to someone or something.
110. Ans. B.
‘Thrifty’ means using money and other resources carefully and not wastefully.
‘Reckless’ means careless.
‘Economical’ means spending resources carefully.
‘Naïve’ means natural and unaffected; innocent.
‘Extravagant’ means lacking restraint in spending money or using resources.
111. Ans. A.
There are some phrases in which we commonly use subjunctive mood. Such phrases include ask
that, it is important that, we insist that. In past tense, these changes to asked that, it was important
that and we insisted that. If “I” is used in such sentence, we use the verb “were” with them. The
given sentence starts with “My sister asked me” which gives us a hint that subjunctive mood will be
used in the sentence. Out of the given options, only option A uses subjunctive mood. So, it is the
correct response.
112. Ans. C.
"I were" is called the subjunctive mood, and is used when you're are talking about something that
isn't true or when you wish something was true. This is the case here, so options A and B are
eliminated. In such sentences, the first clause uses “were” and the second clause uses “would” in
the sentence. For example:
If I were the president, I would ensure the safety of the women first.
So, option C is the correct response.
113. Ans. B.
The statement needs to be in indefinite form (past or present) to convey any meaning; this
eliminates options C and D.

PAGE 722
www.byjusexamprep.com

When used to refer to a period of time, "years" takes a singular verb.


Five years is a long time to spend in jail.
Ten years was the time he took to build the robot.
When referring to more than one entity, "years" takes a plural verb just like other plural count
nouns:
Two years are added to the training programme.
The next three years are going to pass quickly.
Besides years, other plural unit words involving money and distance also obey notional concord:
Money
Forty dollars is a good price. (one entity)
Two dollars are on her study table. (more than one entity, like two pens)
Distance
Ten kilometres is not that long. (one entity)
Five miles are to be added to this road. (more than one entity)
114. Ans. A.
The sentence is a past conditional sentence. Such sentence describes a situation which did not
occur in real but the speaker only wishes about them to have happened. We use the past perfect
tense in the sub-clause and “would have + past participle” in the main clause. See below example:
If I had known about the rainy weather, I would not have gone hiking.
So, option A is the correct answer.
115. Ans. C.
‘To call on someone’ means to visit someone.
‘To call someone in’ means to ask someone to come and help you or do something for you.
‘To call at’ means to stop at a place for a short time.
There is no such phrasal verb such as “call up on”. So, option D is incorrect.
The context of the statement is of the diplomat making a visit to the PM, so the phrasal verb
“called on” suits the context perfectly and is the correct response.
116. Ans. A.
‘To get along’ phrase means to be friendly with someone.
‘To get well’ means to improve on someone’s health.
‘To get up’ means to stand.
‘To get among’ doesn’t has a phrasal meaning.
Option A is the most suitable in the context of the statement, and so is the correct response.

PAGE 723
www.byjusexamprep.com

117. Ans. B.
The first part of the sentence uses ‘want’ verb form, thus the later verb too needs to be in simple
present form for parallelism. This eliminates option D.
Option A is incorrect because the verb ‘insists’ requires a preposition to give meaning in the
context of this statement.
Between ‘in’ and ‘on’, the latter is the obvious choice as ‘insist in’ is grammatically inaccurate.
So, the correct answer is option B.
118. Ans. D.
Option D is the correct answer. A verb followed by ‘of’ is always used in gerund form. This
eliminates option B.
Option A is incorrect as it fails to convey proper meaning of the statement.
Option C is incorrect because the use of ‘to’ preposition would mean that ‘headache’ is the
recipient of the complaint, which would render the statement senseless.
119. Ans. A.
The context of the statement makes it clear that the subject ‘Sandhya’ is doing the action (shout in
this case), which is directed promptly and clearly at one person.
When we shout to someone, we want them to hear us.
Hence, option A is the correct answer.
120. Ans. B.
The sentence talks about the object ‘us’ being the reason for the subject ‘My teacher’ being
annoyed. So, the most appropriate option to fill the blank would be ‘annoyed with’.
Example :- I was so annoyed with him for turning up late.
Hence, option B is the correct answer.

PAGE 724
www.byjusexamprep.com

General Knowledge
1. Ans. D.
Hindu Mahasabha was founded in 1914 by Madan Mohan Malviya. It worked with Arya Samaj and
other Hindu organizations. It was directly link with Rashtriya Swam SevakSangh founded in 1925 at
Nagpur by K.B.Hegewar. The first All India Hindu Mahasabha Conference was organized at Hardwar
in 1915. The Sabha ecame more aggressive after 1929 and started propagating Hindu Rashtra
which was totally differ from Gandhiji’s Ram Rajya.
Hence, option D is correct.
2. Ans. B.
Eka Movement or Unity Movement is a peasant movement which surfaced in Hardoi, Bahraich and
Sitapur during the end of 1921 by MadariPasi, an offshoot of Non Cooperation Movement. The
initial thrust was given by the leaders of Congress and Khilafat movement. The main reason for the
movement was high rent, which was generally higher than 50% of recorded rent in some areas.
Hence, option B is correct.
3. Ans. D.
Department of Industrial Policy and Promotion (DIPP) under Ministry of Commerce and Industry
has unveiled tricolour logo for geographical indication (GI) certified products. The logo has tagline
“Invaluable Treasures of Incredible India” printed below it.
Hence, option D is correct.
4. Ans. B.
Central Water Commission (CWC), India’s apex technical organization in the field of Water
Resources, has entered into a Collaboration Agreement with Google. CWC would use state-of-the-
art advances made by Google in the in the field of Artificial Intelligence, Machine Learning and geo
spatial mapping for effective management of water resources particularly in the field of flood
forecasting and dissemination of flood related information to the masses widely using the
dissemination platforms developed by Google.
Hence, option B is correct.
5. Ans. C.

Hence, option C is correct.


6. Ans. B.
PRASAD Scheme was launched in 2014-15 with 12 identified sites namely Ajmer (Rajasthan),
Amritsar (Punjab), Amaravati (Andhra Pradesh), Dwarka (Gujarat), Gaya (Bihar), Kedarnath
(Uttarakhand), Kamakhya (Assam), Kanchipuram (Tamil Nadu), Mathura (Uttar Pradesh), Puri
(Odisha), Varanasi (Uttar Pradesh) and Vellankani (Tamil Nadu).

PAGE 725
www.byjusexamprep.com

During 2015-16, Patna (Bihar) and in 2016-17, 12 more sites were added in the scheme for
development namely Ayodhya (Uttar Pradesh), Badrinath (Uttarakhand), Belur (West Bengal),
Deoghar (Jharkhand), Guruvayur (Kerala), Hazratbal and Katra (Jammu & Kashmir), Omkareshwar
(Madhya Pradesh), Somnath (Gujarat), Srisailam and Tirupati (Andhra Pradesh), Trimbakeshwar
(Maharashtra).
Hence, option B is correct.
7. Ans. B.
SWAYAM is a programme initiated by Government of India and designed to achieve the three
cardinal principles of Education Policy viz., access, equity and quality. The objective of this effort is
to take the best teaching learning resources to all, including the most disadvantaged. SWAYAM
seeks to bridge the digital divide for students who have hitherto remained untouched by the digital
revolution and have not been able to join the mainstream of the knowledge economy.
Hence, option B is correct.
8. Ans. B.
The app takes users to snapshots of info available on the income tax website. The app helps to
zero-in to one's tax concern when away from desktop.
The app helps you to file income tax return (ITR) online, locate the nearest Tax Return Prepares
(TRP), provides calculators and other tools, helps you manage your PAN and TDS, calculate and lets
you pay tax, if any.
Hence, option B is correct.
9. Ans. C.
India is now the world’s sixth largest economy, displacing France. The five economies ahead are the
United States, China, Japan, Germany and United Kingdom.
Hence, option C is correct.
10. Ans. B.
Neeraj Chopra is an Indian track and field athlete, who competes in the javelin throw. He
represented India at the 2018 Asian Games where he won a gold medal.
Hence, option B is correct.
11. Ans. A.
The United Nations Human Rights Council (UNHRC) is a United Nations body whose mission is to
promote and protect human rights around the world.
The UNHRC has 47 members elected for staggered three-year terms on a regional group basis.
Hence, option A is correct.
12. Ans. C.
The Universal Declaration of Human Rights (UDHR) is a historic document that was adopted by the
United Nations General Assembly at its third session on 10 December 1948 as Resolution 217 at the
Palais de Chaillot in Paris, France.

PAGE 726
www.byjusexamprep.com

Article 17 of UDHR
(1) Everyone has the right to own property alone as well as in association with others.
(2) No one shall be arbitrarily deprived of his property.
Hence, option C is correct.
13. Ans. D.
His ordinance-making power is not a discretionary power. This means that he can promulgate or
withdraw an ordinance only on the advice of the council of ministers headed by the Chief Minister.
He can withdraw an ordinance at any time.
Hence statement A is correct and statement D is incorrect.
14. Ans. D.
For every state, there is a legislature, which consists of Governor and one House or, two Houses as
the case may be.
Hence, option D is correct.
15. Ans. C.
The Chief Minister enjoys the following powers in relation to the governor:
(a) He is the principal channel of communication between the governor and the council of
ministers. It is the duty of the Chief Minister:
(i) to communicate to the Governor of the state all decisions of the council of ministers relating to
the administration of the affairs of the state and proposals for legislation;
(ii) to furnish such information relating to the administration of the affairs of the state and
proposals for legislation as the governor may call for; and
(iii) if the governor so requires, to submit for the consideration of the council of ministers any
matter on which a decision has been taken by a minister but which has not been considered by the
council.
Hence, option C is correct.
16. Ans. D.
A person to be appointed as a judge of the Supreme Court should have the following qualifications:
• He should be a citizen of India.
• He should have been a judge of a High Court (or high courts in succession) for five years; or
• He should have been an advocate of a High Court (or High Courts in succession) for ten
years; or
• He should be a distinguished jurist in the opinion of the president.
Hence, option D is correct.
17. Ans. D.
To organize and periodically amend electoral rolls and to register all qualified voters.

PAGE 727
www.byjusexamprep.com

The major aim of election commission of India is to define and control the process for elections
conducted at various levels, Parliament, State Legislatures, and the offices of the President and
Vice President of India. It can be said that the Election Commission of India ensures smooth and
successful operation of the democracy. The President appoints regional commissioners after
consultation with the election commission to assist the election commission.
Hence, option D is correct.
18. Ans. B.
Police force is a subject comes under state list of Indian Constitution so this matter may or will be
discussed in the state assembly. Parliament can discuss any matter which it's members bring it to
its notice. Parliament can discuss and enact legal framework only. Implementation of the rule is
subject matter of the government. The court has to ensure the law in enforced and the state is
ruled by the law but the most relevant answer is B so we can say that police force lies under stste
list so it is the right answer.
Hence, option B is correct.
19. Ans. A.
Diplomatic Powers
The international treaties and agreements are negotiated and concluded on behalf of the
President. However, they are subject to the approval of the Parliament. He represents India in
international forums and affairs and sends and receives diplomats like ambassadors, high
commissioners, and so on.
Military Powers
He is the supreme commander of the defence forces of India. In that capacity, he appoints the
chiefs of the Army, the Navy and the Air Force. He can declare war or conclude peace, subject to
the approval of the Parliament.
These powers are exclusive to the President and not given to the Governor.
Hence, option A is correct.
20. Ans. A.
The reports of the Comptroller and Auditor-General of India relating to the accounts of the Union
shall be submitted to the president, who shall cause them to be laid before each House of
Parliament.
The reports of the Comptroller and Auditor-General of India relating to the accounts of a State shall
be submitted to the Governor of the State, who shall cause them to be laid before the Legislature
of the State.
Hence, option A is correct.
21. Ans. A.
If at any time the Governor is satisfied that an act or resolution of a District or a Regional Council is
likely to endanger the safety of India or is likely to be prejudicial to public order, he may annul or
suspend such act or resolution and take such steps as he may consider necessary (including the
suspension of the Council and the assumption to himself of all or any of the powers vested in or

PAGE 728
www.byjusexamprep.com

exercisable by the Council) to prevent the commission or continuance of such act, or the giving of
effect to such resolution.
Hence, option A is correct.
22. Ans. A.
The accounts of the District Council or, as the case may be, the Regional Council shall be kept in
such form as the Comptroller and Auditor-General of India may, with the approval of the President,
prescribe.
The Comptroller and Auditor-General shall cause the accounts of the District and Regional Councils
to be audited in such manner as he may think fit, and the reports of the Comptroller and Auditor
General relating to such accounts shall be submitted to the Governor who shall cause them to be
laid before the Council.
Hence, option A is correct.
23. Ans. B.
The elected members of the District Council shall hold office for a term of five years from the date
appointed for the first meeting of the Council after the general elections to the Council, unless the
District Council is sooner dissolved under paragraph 16 and a nominated member shall hold office
at the pleasure of the Governor.
Hence, option B is correct.
24. Ans. A.
All India Kisan Sabha (All India Peasants Union, also known as the AkhilBharatiyaKisan Sabha), was
the name of the peasants front of the undivided Communist Party of India, an important peasant
movement formed by Sahajanand Saraswati in 1936.
Hence, option A is correct.
25. Ans. C.
On 6th June, the All India Muslim League passed a resolution which, while critical of the contents of
the statement of 16th May, particularly on the Pakistan issue, and while reserving opinions on that
point, yet definitely accepted the scheme put forward by the Mission
Hence, option C is correct.
26. Ans. C.
Azad Hind Radio was a propaganda radio service that was started under the leadership of Netaji
Subhas Chandra Bose in Germany in 1942 to encourage Indians to fight for freedom.
Hence, option C is correct.
27. Ans. C.
The introduction of Chinese tea plants, different from Indian Tea, to India is commonly credited to
Robert Fortune. In 1778, the East India Company requested Sir Joseph Banks to prepare a feasibility
report and as per his suggestion, experimental planting took place between the 26th and 30th
parallels of latitude. But only as late of 1793 did the East India Company take steps to plant Tea in a
Private Botanical Garden of Colonel Robert Kyd at Sibpur, Calcutta.
Hence, option C is correct.

PAGE 729
www.byjusexamprep.com

28. Ans. B.
Amiya Bagchi examined evidence on handloom spinning and other traditional industry in Gangetic
Bihar, an area of eastern India, collected between 1809 and 1813 by the East India Company
surveyor Dr. Francis Buchanan Hamilton.
Bagchi compared Hamilton’s data with the 1901 Census estimates of the population dependent on
industry for the same area.
Hence, option B is correct.
29. Ans. C.
Fort William College (also called the College of Fort William) was an academy and learning centre of
Oriental studies established by Lord Wellesley, then Governor-General of British India.
Hence, option C is correct.
30. Ans. A.
In first of its kind event held by any government organization, the Indian Railways organized a
programme on Ethics in Public Governance and Launched “Mission Satyanishtha”.
The objectives of the Mission are:
1) To train every employee to understand the need and value of ethics in Personal and Public life.
2) To deal with ethical dilemmas in life and Public Governance.
3) To help understand the policies of Indian Railways on ethics and integrity and the employee’s
role in upholding the same.
4) To develop inner governance through tapping inner resources.
Hence, option A is correct.
31. Ans. D.
The mascot for the Olympics is named Miraitowa, and the Paralympic mascot is Someity.
Miraitowa is a combination of the Japanese words for future and eternity; Someity comes from a
popular cherry blossom variety "Someiyoshino" and echoes the English phrase "so mighty."
Hence, option D is correct.
32. Ans. B.
The Dhanush is a 155 mm towed howitzer used by the Indian Army. Under the first phase, the army
will place an order worth over Rs 1,200 crore for 114 such guns with the Ordnance Factories Board.
Hence, option B is correct.
33. Ans. A.
India undocked its first indigenously-built aircraft carrier INS Vikrant at Cochin Shipyard Limited. INS
Vikrant, also known as Indigenous Aircraft Carrier 1 (IAC-1), is an aircraft carrier under construction
by Cochin Shipyard in Kochi, Kerala for the Indian Navy. It is the first aircraft carrier to be built in
India.

PAGE 730
www.byjusexamprep.com

The ship, built at CSL, will undergo a series of fitment and trial processes before it is ready for
propulsion and inducted into the Navy.
Hence, option A is correct.
34. Ans. D.
The Uttar Pradesh government’s showcase defence industrial corridor project will get off the
ground with a manufacturing facility to be built by state-run Bharat Electronics Limited.
BEL is the first company to have approached the state government for allotment of land under the
scheme.
The corridor, one of the two announced by the central government, with the other being in Tamil
Nadu, is expected to benefit the politically significant Bundelkhand region.
Hence, option D is correct.
35. Ans. C.
India’s mega defence exhibition, the DEFEXPO 2018, was conducted in Chennai. The event will
underscore India’s first serious attempt to project itself as a major military manufacturer.
Hence, option C is correct.
36. Ans. D.
While 5th schedule envisages creation of Tribal Advisory Council, 6th schedule provides for District
Councils and Regional Councils with certain legislative and judicial powers.
The Constitution of India makes special provisions for the administration of the tribal dominated
areas in four states viz. Assam, Meghalaya, Tripura and Mizoram.
Hence, option D is correct.
37. Ans. B.
Article 370 of the Indian constitution is an article that gives autonomous status to the state of
Jammu and Kashmir. The article is drafted in Part XXI of the Constitution: Temporary, Transitional
and Special Provisions.
Hence, option B is correct.
38. Ans. B.
Art 371A: Nagaland
Art 371B: Assam
Art 371C: Manipur
Art 371D: Andhra Pradesh
Hence, option B is correct.
39. Ans. B.
Where the Governor takes a decision independently of his Council of Ministers or where he acts as
the Chief Executive of the State under Presidents rule, his actions are subject to scrutiny by the
Parliament. So the statements 2 is correct.
Hence, option B is correct.

PAGE 731
www.byjusexamprep.com

40. Ans. A.
• In the 17th century, Varanasi's houses were made of mud and brick.
• Jean Baptiste Tavernier arrived in Varanasi in 1655.
• He wrote extensively about the art and culture of Varanasi.
• He also wrote about the Ganga, expressing his concern about the river's pollution.
• He was the French jeweller who bought the Hope diamond.
• He gives a detailed account in his book 'Travels of India.'
Hence, option A is correct.
41. Ans. B.
The Nawab of Awadh or the Nawab of Oudh was the title of the rulers who governed the state of
Awadh (anglicised as Oudh) in north India during the 18th and 19th centuries. The Nawabs of
Awadh belonged to a dynasty of Persian origin from Nishapur, Iran. In 1724, NawabSaadat Ali Khan
established the Oudh State with their capital in Faizabad and Lucknow.
Hence, option B is correct.
42. Ans. B.
Physicians did not lack recognition in the Mughal Empire, and their names are often listed in
Mughal histories along with those of the learned and men of letters of the time.
In spite of this they were not regarded as part of the Mughal ruling aristocracy. It was therefore
remarkable that Muqarrab Khan, a surgeon and physician, who was known for his skill in treating
elephants as much as for treating men, should have attained the high mansab of 5,000 zat and
5,000 sawdr, and been appointed governor of three provinces during Jahanglr's reign.
Hence, option B is correct.
43. Ans. A.
Tarikh-i Akbari or Tarikh-iQandhari of ArifQandhari is the Persian text of the well history of Akbar's
period.
Hence, option A is correct.
44. Ans. D.
Nayakas were military chiefs usually mentioned law and order in their areas of control. They
maintained forests and kept armed supporters.
The amara-nayaka system was a major political innovation of the Vijayanagara Empire. It is likely
that many features of this system were derived from the lqta system of the Delhi Sultanate. The
amara-nayakas were military commanders who were given territories to govern by the raya.
Hence, option D is correct.
45. Ans. B.
Niccolao Manucci (19 April 1638–1717) was an Italian writer and traveller.
Manucci spent almost his entire life in India. He would then send home the manuscript for "Storia
do Mogor" which was lent to the French historian François Catrou in 1707.
Hence, option B is correct.

PAGE 732
www.byjusexamprep.com

46. Ans. C.
Indian agriculture sector accounts for 18 percent of India's gross domestic product (GDP) and
provides employment to 50%+ of the countries workforce. So, statement a is true.
Indian agricultural output growth has grown from about 2% average in the first decades to over 3%
now and the annual volatility (measured by standard deviation) has been reduced from 6% to
about 3%. So, statement II is not true.
Hence, option C is correct.
47. Ans. B.
• According to the Economic Survey 2018-19, private investments in research in India were
significantly lower than public investments in research.
• In comparison, private investments were a major source of research activities in countries such
as the United States and China.
• According to the Survey, public investment in research has remained stagnant at 0.6-07
percent of GDP over the last two decades.
Hence, option B is correct.
48. Ans. A.
According to IMF, India's gross savings rate has fallen to 31 per cent of GDP from 37 per cent in
2007-08. The slowdown is more a function of low incomes, tough economic conditions, falling real
returns and high inflation, rather than structural impediments of a rising dependency ratio or a fall
in working age population. so, fall in household and public savings is responsible for sharp decline
in saving rates.
Hence, option A is correct.
49. Ans. B.
With the UDAY burden off their shoulders, state governments in aggregate were to revert to well
below 3% fiscal deficit threshold in FY18 — 2.7% to be precise — but the Reserve Bank of India
(RBI), analysing the revised estimates (REs) of 29 states, put their combined deficit in the year at
3.1%. but in the fiscal year 2015, In 2015-16, the gross state fiscal deficit (GSFD) came at 3.6% after
revised estimates against a budgeted estimate of 2.6%.
As per economic survey, the statement ‘special Category States had run up a higher level of fiscal
deficit in 2017-2018 compared to 2016-2017’ is true.
Hence, option B is correct.
50. Ans. A.

Hence, option A is correct.


51. Ans. C.
To review the Act and to draft a new Direct Tax Law in consonance with economic needs of the
country, the government has constituted the task force.

PAGE 733
www.byjusexamprep.com

The convenor of the eight-member committee will be Arbind Modi, Member, Central Board of
Direct taxes (CBDT).
Hence, option C is correct.
52. Ans. B.
The Countering America's Adversaries Through Sanctions Act, CAATSA, is a United States federal
law that imposed sanctions on Iran, North Korea, and Russia.
Hence, option B is correct.
53. Ans. A.
The policy, whose finer details are still to be formalized, will initially allow the entry of private
companies into the manufacture of submarines, fighter aircrafts, helocopters and armored vehicles
through foreign partnerships.
Hence, option A is correct.
54. Ans. D.
As per the extant FDI policy, foreign investment up to 49% is permitted under the automatic route,
foreign investment beyond 49% and upto 100% is permitted through Government approval
Hence, option D is correct.
55. Ans. C.
The HAL Tejas is an Indian single-seat, single-jet engine, multirole light fighter designed by the
Aeronautical Development Agency (ADA) and Hindustan Aeronautics Limited (HAL) for the Indian
Air Force and Indian Navy. It came from the Light Combat Aircraft (LCA) programme.
Hence, option C is correct.
56. Ans. B.
The Centre has set up a Defence Planning Committee (DPC) chaired by National Security Advisor
(NSA) AjitDoval to facilitate "comprehensive" planning for the defence forces besides focusing on
military doctrines to deal with emerging security challenges.
Hence, option B is correct.
57. Ans. A.
India is going to host the inaugural 2+2 Dialogue with the United States on September 6-7 in New
Delhi. The dialogue, which has been postponed twice earlier, will see External Affairs Minister
SushmaSwaraj and Defence Minister Nirmala Sitharaman holding extensive talks with US Secretary
of State Michael R. Pompeo and Secretary of Defence James Mattis, respectively.
Hence, option A is correct.
58. Ans. C.
Before the framing of the constitution started, an Objectives Resolution (the resolution that
defined the aims of the Assembly) was moved by Nehru in 1946. This resolution enshrined the
aspirations and values behind the Constitution making. On the basis of the Objectives Resolution,
India’s Constitution gave institutional expression to the fundamental commitments:

PAGE 734
www.byjusexamprep.com

equality, liberty, democracy, sovereignty and a cosmopolitan identity. This made the moral
commitment to establish a government that will fulfil the many promises that the nationalist
movement held before the people of India.
Hence, option C is correct.
59. Ans. A.
Indian Council Act, 1861:
The Indian Councils Act of 1861 introduced a grain of popular element insofar as it provided that
the Governor-General's Executive Council, which was so long composed exclusively of officials,
should include certain additional non-official members, while transacting legislative business as a
Legislative Council. The Indian Councils Act 1861 was an act of the parliament of UK that
transformed the India’s executive council into a cabinet run on the portfolio system.
But this Legislative Council was neither representative nor deliberative in any sense. The members
were nominated and their functions were confined exclusively to a consideration of the legislative
proposals placed before it by the Governor-General.
Hence, option A is correct.
60. Ans. C.
• The Act for Better Government of India, 1858, is another name for the Government of India
Act, 1858.
• The title of Governor-General of India was changed to Viceroy of India.
• The administration of India was transferred from the East India Company to the British Crown.
• As a result, the country remained under the direct rule of the British and the Cabinet.
Hence, option C is correct.
61. Ans. C.
Rome imported Indian tigers, rhinoceros, elephants, and serpents to use for circus shows – a
method employed as entertainment to prevent riots in Rome. It has been noted in the Periplus that
Roman women also wore Indian Ocean pearls and used a supply of herbs, spices, pepper, lyceum,
costus, sesame oil and sugar for food. Indigo was used as a color while cotton cloth was used as
articles of clothing. Furthermore, the subcontinent exported ebony for fashioned furniture in
Rome. The Roman Empire also imported Indian lime, peach, and various other fruits for medicine.
Western India, as a result, was the recipient of large amounts of Roman gold during this time.
Tortoiseshell was not a major item of Indian exports to Rome.
Hence, option C is correct.
62. Ans. A.
The story, based on a Buddhist text in Pali known as the MajjhimaNikaya, is part of a dialogue
between a king named Avantiputta and a disciple of the Buddha named Kachchana. While it may
not be literally true, it reveals Buddhist attitudes towards varna.
Hence, option A is correct.
63. Ans. C.
For men, the Manusmriti declares, there are seven means of acquiring wealth. These are:
Inheritance, finding, purchase, conquest, investment, work, and acceptance of gifts from good
people.

PAGE 735
www.byjusexamprep.com

And the six means for women:


What was given in front of the fire (marriage) or the bridal procession, or as a token of affection,
and what she got from her brother, mother or father. In addition, any subsequent gift and
whatever her affectionate husband might give her.
Hence, option C is correct.
64. Ans. D.
Vellalars (also, Velalars, Vellalas) were, originally an elite caste of Tamil agricultural landlords in
Tamil Nadu, Kerala states in India and in neighbouring Sri Lanka; they were the aristocracy of the
ancient Tamil order (Chera/Chola/Pandya/Sangam era) and had close relations with the different
royal dynasties.
Uzhavar: Ordinary ploughmen were called uzhavars.
Kadaisiyar and adimai: Landless labourers and slaves were known as kadaisiyar and adimai
Hence, option D is correct.
65. Ans. D.
The scholars opine that the Jatakas were written in Pali around the middle of the first millennium
A.D. (CE.). The Jatakas tell us about the stories of the previous births of Gautama Buddha. The
number of the collected and published Jatakas is more than five hundred. The Jatakas throw
significant light on the social and economic conditions of the day. They also provide information
about the political events in the age of Buddha. Panchatantra was authored by Vishnu Sharma. It
also furnishes us with important information regarding the life of the ordinary people during the
period under consideration.
For example, a story known as the GandatinduJataka tells us about the deplorable condition of the
subjects—including elderly women and men, cultivators, herders, village boys and even animals—
of a wicked king.
Hence, option D is correct.
66. Ans. B.
Across the Indus from MohenjoDaro is KotDiji, another 3rd millennium and early Harappan farming
community. Distance between MohenjoDaro and KotDiji is 86 kms
Lothal is in Gujarat.
Kalibangan is in Rajasthan.
Approximate driving distance between Harappa and MohenjoDaro is 688 kms
Hence, option B is correct.
67. Ans. A.
Any unemployment not considered to be natural is often referred to as cyclical, institutional or
policy-based unemployment. Variables exogenous to the labor market cause an increase in the
natural rate of unemployment; for example, a steep recession might increase the natural
unemployment rate if workers begin to lose skills or the motivation to find full-time work again.
Economists sometimes call this “hysteresis.”

PAGE 736
www.byjusexamprep.com

Important contributors to the theory of natural unemployment include Milton Friedman, Edmund
Phelps and F.A. Hayek, all Nobel winners.
Hence, option A is correct.
68. Ans. B.

Hence, option B is correct.


69. Ans. D.
India ranked 77th among 190 countries by leapfrogging 23 ranks in World Bank’s Ease of Doing
Business (EODB) 2018 ranking. In the 2017 rankings, India ranked 100th and had jumped 30 places.
Rank as per world bank doing business report 2018 increase in Resolving insolvency, Starting a
business and getting electricity from 183,156 and 29 in 2017 to 108,137 and 24 in 2018

Hence, option D is correct.


70. Ans. D.
According to the latest Reserve Bank of India study on State finances, capital spending is maximum
on education.
Hence, option D is correct.

PAGE 737
www.byjusexamprep.com

71. Ans. A.
Lanak La is a well-established frontier point between Ladakh and Tibet confirmed by travellers
Hence, option A is correct.
72. Ans. A.

Hence, option A is correct.


73. Ans. D.

The Mettur Dam is one of the largest dams in India and the largest in Tamilnadu located across the
river Cauvery where it enters the plains.
The Indira Sagar Dam is a multipurpose project of Madhya Pradesh on the Narmada River.
Krishna Raja Sagara, also popularly known as KRS, is a lake and the dam that creates it. They are
close to the settlement of Krishnarajasagara in the Indian State of Karnataka.
Hence, option D is correct.
74. Ans. D.

The Banas is a river of Rajasthan state in western India. It is a tributary of the Chambal River.
Hence, option D is correct.
75. Ans. C.
The answer is Kolkata. Apart from the headquarters of the Eastern and the South Eastern Railways,
it also has the headquarters of the Kolkata Metro Railways, which is now a zone of the Indian
Railways. There is no city that has 3 zonal headquarters. However, Kolkata and Mumbai are
headquarters of 2 zones each
Hence, option C is correct.

PAGE 738
www.byjusexamprep.com

76. Ans. B.
Austric Languages - The Austric languages category is further sub-divided into Munda and Mon-
Khmer.
Munda or Kol Languages: Munda languages are the largest of the Austric group of languages. They
consist of fourteen tribal languages. The Kherwari is the major group, which is current in Eastern
India (Chota Nagpur, Orissa, Chhattisgarh and West Bengal) and includes Santhali, Mundari, Ho,
Birhor, Bhumiej, Korwa and Korku (or Kurku).
Mon-Khmer Languages: Mon-Khmer group of Austric languages has two sub-groups— Khasi and
Nicobari. Khasi languages are spoken by Khasi tribal people of Meghalaya, while Nicobari languages
are the languages of the tribal people of the Nicobar Islands.
Hence, option B is correct.
Mon-Khmer group of Austric languages has two sub-groups— Khasi and Nicobari. Khasi languages
are spoken by Khasi tribal people of Meghalaya, while Nicobari languages are the languages of the
tribal people of the Nicobar Islands.
77. Ans. D.
Although the general direction of flow of the Peninsular Rivers is from west to east, because slope
is towards west to east, a careful study reveals at least three main directions of flow:
(i) The Mahanadi, the Godavari, the Krishna, the Cauvery and several smaller rivers draining south-
east into the Bay of Bengal. '
(ii) The Narmada and the Tapi flowing west as well as several small streams originating from the
Western Ghats flow westwards into the Arabian Sea.
(iii) Tributaries of the Ganga and the Yamuna such as the Chambal, the Betwa, the Ken, the Son and
the Damodar flow in the north-easterly direction.
Hence, option D is correct.
78. Ans. C.
Post-independence, the policy vis a vis agriculture was to increase cultivable area as much as
possible. This was done by bringing any kind of fallow land available under cultivation.
Over already cultivated lands, intensification practices were followed. After independence
immediate goal of government to increase food grain production by Switching over from cash crops
to food crops.
It was only after the Green revolution of 1960s that the use of HYV seeds was promoted.
Hence, option C is correct.
79. Ans. C.
India’s tree and forest cover has registered an increase of 1% or 8, 021 sq. km in two years since
2015, according to the latest assessment by the government.so, the statement 1 is not correct.
Other two statements are correct.
Hence, option C is correct.

PAGE 739
www.byjusexamprep.com

80. Ans. A.

Hence, option A is correct.


81. Ans. B.

Hence, option B is correct.


82. Ans. B.
“Following withdrawal of southwest monsoon, the wind direction has changed, favouring setting in
of northeast monsoon. The winds are stronger and because of this, systems will be formed one
after another in the Bay of Bengal.”
Tamil Nadu gets 48% of its annual rainfall during the northeast monsoon which normally lasts from
around October 15 to December-end.
Hence, option B is correct.
83. Ans. D.
Mangalore is situated on the west coast of India, and is bounded by the Arabian Sea to its west and
the Western Ghats to its east. So, Mangalore does not fall on leeward slope.
Hence, option D is correct.
84. Ans. C.
Percentage of urban population in Maharashtra (45.23), Kerala (47.72%), Mizoram (51.51%) and in
Goa (62.17%).
Hence, option C is correct.
85. Ans. A.
As a consequence of the lack of proofreading activity of RNA virus polymerases, new viral genetic
variants are constantly created. RNA viruses readily adapt to changing environmental conditions.
Hence, option A is correct.
86. Ans. C.
Colostrum is the first form of milk produced by mammary glands of mammals(including many
humans) immediately following the delivery of the new born. It is high in carbohydrates, high in
protein, high in antibodies but low in fat. Due to high concentration of antibodies, it is essential for
the development of immune response of baby.
Hence, option C is correct.

PAGE 740
www.byjusexamprep.com

87. Ans. B.
The epididymis transports and stores sperm cells that are produced in the testes. The vas deferens
transports mature sperm to the urethra, the tube that carries urine or sperm to outside of the body.
The seminal vesicles produce a sugar-rich fluid (fructose) that provides sperm with a source of energy
to help them move. The prostate gland contributes additional fluid to the ejaculate. Prostate fluids
also help to nourish the sperm.
Hence, option B is correct.
88. Ans. A.
‘‘hair’ in carrot are modified roots and ‘tendril’ in pea are modified leaves. ‘Scale’ found in onion are
modified leaves. So, ‘eye’ on potato is an indication for modification of stem of a plant.
Hence, option A is correct.
89. Ans. D.
Convection is transfer of heat due to the bulk movement of molecules within fluids such as gases and
liquids. But in bulb transfer is due to radiation.
Hence, option D is correct.
90. Ans. B.
The process of fire extinguishing involves absorption of heat. Absorption of heat in converting hot
water to steam is more than the heat absorbed in heating cold water to the boiling temperature.
Hence boiling water can extinguish fire more quickly then ice or cold water.
Hence, option B is correct.
91. Ans. C.
Total permissible power of fuse= 5A×220V = 1100W; power of 1 bulb= 100W; so number of bulb
1100
= = 11.
100
Hence, option C is correct.
92. Ans. C.
The direction of the magnetic field at any location on Earth's surface is commonly specified in terms
of two angles, field declination and field inclination. Magnetic inclination is the angle between the
horizontal plane and the total field vector, measured positive into Earth and Magnetic declination is
the angle between magnetic north and true north.
Hence, option C is correct.
93. Ans. C.
For real image to be formed by convex lens, the minimum distance between object and image should
be 4f, where f is focal length of the lens.
Hence, option C is correct.

PAGE 741
www.byjusexamprep.com

94. Ans. A.
Option A is correct.
95. Ans. C.
Diamond is an insulator. It does not conduct electricity due to absence of free electrons for the
movement of charge. Diamond is a poor conductor of electricity.
Hence, option C is correct.
96. Ans. D.
Oxidation state of nitrogen in NO, NO2, N2O and N2O5 are +2, +4, +1 and +5 respectively. So, N2O5
has highest oxidation state of nitrogen.
Hence, option D is correct.
97. Ans. A.
Molecular mass of Ba(OH)2= 137.3 + 32 + 2 = 171.3 ; mole factor = 2; so,
171.5
equivalent weight = = 85.7
2
Hence, option A is correct.
98. Ans. D.
Tamil Nadu has three major ports, namely Ennore port, Tuticorin port and the Chennai port;
Maharashtra has two, namely, NhavaSheva port and Bombay port; West Bengal has one, the Haldia
port; and Odisha has also one, namely Paradip port.
Hence, option D is correct.
99. Ans. C.

According to 2011 census it was Punjab with nearly 28.9% of the total population.
Hence, option C is correct.

PAGE 742
www.byjusexamprep.com

100. Ans. A.
Corals need saltwater to survive and require a certain balance in the ratio of salt to water. So, they
cannot grow abundantly in freshwater. Corals rarely develop in water deeper than 50 meters.
corals generally live in water temperatures of 20–32 ° C.
Hence, option A is correct.
101. Ans. A.
||| Growth of Population from 2001-2011 in Tamil Nadu was 15.6%, in Kerala 4.9%, in Goa 8.2%
and in Andhra Pradesh 11.1%.
From 1991-2001, the figures were 11.7% in Tamil Nadu, 9.4% in Kerala, 15.2% in Goa and 14.6% in
Andhra Pradesh.
So, Tamil Nadu is the state where Population growth increased from 1991-2001 to 2001-2011.
Hence, option A is correct.
102. Ans. B.
The population density of Arunachal Pradesh, Mizoram, Sikkim and Himachal Pradesh are 17, 52,
86 and 123 per square kilometre respectively. So, Arunachal Pradesh - Mizoram - Sikkim - Himachal
Pradesh is the correct sequence.
Hence, option B is correct.
103. Ans. D.
• The visible light spectrum is the segment of the electromagnetic spectrum that the human eye
can view.
• This range of wavelengths is called visible light or light.
• Typically, the human eye can detect wavelengths from 390 to 700 nm.
Hence, option D is correct.
104. Ans. B.
Fungus is a microorganism which is too small to disperse seeds for propagation of plants. Fungi can
be single celled or very complex multicellular organisms. They are found in just about any habitat
but most live on the land, mainly in soil or on plant material rather than in sea or fresh water
Hence, option B is correct.
105. Ans. B.
Xylem transports water and dissolved minerals from the roots to the rest of the plant and also
provides physical support. So, if it is blocked, transportation of water and solutes would be
affected.
Hence, option B is correct.
106. Ans. B.
In this process water esacpes through hydrothodes on the margin of the leaves in the form of
water droplets but in transpiration water escapes in the form of water vapour.
Hence, option B is correct.

PAGE 743
www.byjusexamprep.com

107. Ans. D.
After pollination of the flower, the fertilized ovules develop into seeds while the surrounding ovary
wall forms the fruit tissue. So true fruit is developed by ovaries only. A true fruit or eucarp is a
mature or ripened ovary, developed after fertilization, e.g., Mango, Maize, Grape etc
Hence, option D is correct.
108. Ans. D.

• Conduction of food and water is the function of vascular tissues- phloem and xylem
respectively.
• Leaf manufacture food with the process called photosynthesis; undergo transpiration
through stomatal openings; and interchange of gases via diffusion.
Hence, option D is correct.
109. Ans. A.
Roots absorb water and minerals from the soil. Leaf prepares food and bark provides strength to
plants. So, the main absorbing organ of plants is root only.
Hence, option A is correct.
110. Ans. A.
Candle converts electrical energy into heat energy (not into light energy). Light is just produced as a
side product of heat generated in the process.
Hence, option A is correct.
111. Ans. D.
let T be the tension in string. For equilibrium 2TCosx= mg string will be straight if x = 90 degrees.
mg
and Cos 90 = 0; T = Cos90 . So, T = infinity.
2
Hence, option D is correct.
112. Ans. D.
dr
v= = 4tx + 3y; i.e. xy plane
dt
dv
a= = 4x; i.e. x direction
dt
Hence, option D is correct.
113. Ans. A.
(P1 + P2 )
Total mass= P1V+P2V; total volume= 2V; density of mixture= total mass/total volume i.e.
2
Hence, option A is correct.

PAGE 744
www.byjusexamprep.com

114. Ans. B.
Sn= u + 0.5a (2n-1); here u=0
9a
For n=5, S5= and
2
11a
for n=6, S6= ;
2
so, %age displacement change= (S6-S5)/S5*100 = 22.22%
Hence, option B is correct.
115. Ans. A.
• Saturated hydrocarbons are the hydrocarbons with no double or triple bonds.
• These include alkanes.
• The general formula for saturated hydrocarbons is CnH2n+2.
Hence, option A is correct.
116. Ans. D.
A white crystalline organic compound (C₇H₇NO₂) composed of a benzene ring with an amino group
and a carboxyl group attached to the carbon atoms 1 and 4, respectively and having Twice the
mass of nitrogen atoms compared to hydrogen atoms.
Hence, option D is correct.
117. Ans. B.
All exist as gases at room temperature. All except carbon dioxide are diatomic. None of the given
gases is coloured. They vary in reactivity also. So, b is the correct answer.
Hence, option B is correct.
118. Ans. A.
Reaction of metals with acid yield hydrogen gas. Reaction involved is:
Mg + 2HCl → MgCl2 + H2
Hence, option A is correct.
119. Ans. A.
2 moles of methane will yield 2 moles of CO. Here CH4 is limiting reagent. So, quantity of O2 which
is present in excess.
Hence, option A is correct.
120. Ans. B.
The reaction involved in the question is:
MgCO3 + HCl = MgCl2 + H2O+ CO2
Since CO2 is produced in the reaction, it turns lime water milky.
Hence, option B is correct.

PAGE 745
www.byjusexamprep.com

Elementary Mathematics
1. Ans. D.

As we know,

2. Ans. B.
Let Y takes y days to finish the work alone

3. Ans. B.
Let the owner meets thief x hours after discovering theft.

Distance travelled by thief till then

Distance travelled by owner till then = 60(x)


According to question,

Distance travelled
They meet 60km from owner’s house & 1.5 hour after theft.
4. Ans. A.
In a correct clock, the minute hand gains 55 minutes(space) over the hour hand in 60 minutes.
To be together again, the minute hand must gain 60 minutes over the hour hand.
55 minutes are gained in 60 minutes
60 minutes are gained in (60/55) x 60 min =720/11 min.

PAGE 746
www.byjusexamprep.com

But, they are together after 72 min.


Lose in 72 min =72-(720/11) =72/11min.
Lose in 24 hours =(72/11 * (60*24)/72)min = 1440/11
The clock loses 1440/11 = 130(10/11) minutes in 24 hours.
Alternative way to solve above kind of problems in exams is using a direct formula.
Whenever there is an overtaking of a minute hand and loss/gain of time involved:

M= intervals of M minutes of correct time


Plugging, M=72 in the above formula we will get the same answer. Negative means clock loses.
5. Ans. C.
Let the water added be x litre

Cost price

New Volume

Selling price

Profit

Profit %

6. Ans. C.

7. Ans. D.
Let total amount is P

PAGE 747
www.byjusexamprep.com

After 4 years,

total amount borrowed = 2p = 2250


8. Ans. A.
Let total number of boys = x
Let total number of girls = 70% of x = 0.7x
Total = x + 0.7x

Number of boys = 50
Number of girls
Number of boys playing only badminton
No. of children playing TT only

No. of children playing both

No. of girls playing only Badminton = Total students – Boys playing only Badminton –Children
playing both games - Children playing only TT

9. Ans. B.
Let A takes days to finish work
Let B takes days to finish work
As A is thrice more efficient, hence B will take 3 times the time taken by A.

Solving, we get:
Time taken by B
10. Ans. D.
We shall use F for father, M for mother, A for Sonu, B for Savita & C for Sonia (for present ages)

PAGE 748
www.byjusexamprep.com

When Sonu was born

Also,

After 12 years
Father’s age

11. Ans. C.
Area of shaded region = area of square – area of circle – 4 x area of quadrant

12. Ans. D.
In the given figure, as
(Corresponding angles)

& (corresponding angles)


is also an equilateral

Similarly,

PAGE 749
www.byjusexamprep.com

13. Ans. C.

In

Area of shaded region = area of bigger semicircle –area of 2 small semi circle –area of small circle

14. Ans. D.
In
(common)
(90 degree)

In

15. Ans. D.
As

PAGE 750
www.byjusexamprep.com

&

16. Ans. B.
(exterior angle supplementary)

(degree measure theorem)

17. Ans. C.
(angles formed in same segment are equal)
In

18. Ans. B.
Shaded area = area of (semi circle Y +semi circle Z – semi circle X)
As (diameter)

Shaded area

19. Ans. B.

In

20. Ans. D.
Shaded area = area of semi circles with dia AB & AC + area of triangle – area of of semi circle with
dia BC.

PAGE 751
www.byjusexamprep.com

In

Putting in above equations

Shaded area

21. Ans. C.
As A beats B by 150m, hence

Speed of A (SA)/Speed by B

As C beats D by 400m, hence

Speed of C (SC)/Speed by D

Let speed of A & B be 20x & 17x


Let speed of C & D be 15y & 13y
As SB & SD hence

Let A beats C by K metres, thus

22. Ans. B.
Let & be & & be & .

Hence,

PAGE 752
www.byjusexamprep.com

Total distance

Total time

Speed = Distance/Time

23. Ans. D.
Total distance
Time
Speed = Distance / Time

24. Ans. B.

Percentage pass

25. Ans. C.
total amount after first year is the principle for second year
and for one year compound interest and simple interest are equal.

PAGE 753
www.byjusexamprep.com

26. Ans. B.

… (i)

(using (i))

Similarly,
Equation becomes

27. Ans. A.

Using remainder theorem

28. Ans. B.
Square of sum of n number= sum of their square +2(sum of all possible products taken two at a
time)
Sum of square of number =1+1+4+4+9+9+16+16
=60

PAGE 754
www.byjusexamprep.com

Square of sum of n number=1-1+2-2+3-3+4-4=0


60+2x=0
2x=-60
x=-30
29. Ans. B.
If H is harmonic mean of P & Q, then

30. Ans. B.

If a > 0 Min value

31. Ans. D.

Let the sides be 5x & 4x interior angle of regular polygon

Sides are 10 & 8


Hence, point 1 is wrong & point 2 is right.
Sum of exterior angles of any regular polygon is 360.
Hence, the sum of their exterior angles will be
Hence, point 3 is also correct.

PAGE 755
www.byjusexamprep.com

32. Ans. D.
Right hand side

33. Ans. C.
We can, rewrite this equation 1/m+1/n−1/mn=2/5
5m+5n−5=2mn
If m=n, then we have 2m2−10m+5=0. There are no integer roots for this.
Now suppose m<n. Then we can say that 10n>2mn⇒5>m.
If m=1 we get 5n=2n; n=0, no solutions.
If m=2 we get 5n+5=4n; n=−5, no solutions
If m=3 we get 5n+10=6n; n=10.
If m=4 we get 5n+15=8n; 3n=15, so n=5.
Therefore, there are 4 positive solutions, (m,n)∈{(10,3),(3,10),(4,5),(5,4)}.
34. Ans. B.
The three numbers can be written as:

Adding above three, we get

Hence, this number is divisible by both x + y + z & 111.


Hence, option C is correct.

PAGE 756
www.byjusexamprep.com

35. Ans. B.
As α & β are roots of equation
Then, α + β = -b/a
And , α × β = -c/a
From the given:

= )+ )]/[ ×( )]
Solving the above, and using above, we get b/ac
36. Ans. A.

Side of square = 2a
Area = (2a)2
= 4a2
Radius of circle = a
Area = πa2
Remaining area

37. Ans. B.
Let us take a set of number which satisfy Pythagoras triplet. Say 3,4,5 (AB, BC & AC respectively)

Now,

option B is correct.
38. Ans. B.
The locus is a circle.

PAGE 757
www.byjusexamprep.com

Standard equation of circle is


where (a, b) is centre & r is the radius of circle.

39. Ans. B.

(Vertically opposite)

(Vertically Opposites)

4 angles are
40. Ans. C.

Let

&

PAGE 758
www.byjusexamprep.com

41. Ans. B.

In

&

Adding both, we get

In

42. Ans. B.

Let radius be r

In

PAGE 759
www.byjusexamprep.com

Minor angle
Major angle

43. Ans. A.
For similar triangles,
Area of triangle 1/Area of triangle 2

44. Ans. A.
Let a be the side of square
Let l be the side of equilateral triangle.
Let r be the radius of circle.
Let P be the perimeter of each one.
For square P = 4a.
For triangle P = 1 × 3

PAGE 760
www.byjusexamprep.com

Area

For square, a=P/4, area is p^2/16


For circle,

Area

Area

Area of circle>area of square > area of triangle

45. Ans. D.
Let the side of cube = a

Diagonal = 3a

l= 3a

Total surface area (TSA) = 6a2

46. Ans. D.
HCF of 10, 15, 20 is 5
Hence, squares are of side 5 cm
Volume of cuboid = lbh

Volume of cube

No. of cubes

47. Ans. B.
Let radius of cylinder be r
LSA = 2πrh

PAGE 761
www.byjusexamprep.com

TSA

According to question,
TSA

48. Ans. C.

Let radius of semi circle = R


Perimeter of semi circle = circumference of base of cone

In

49. Ans. C.
LSA of cone
(22/7) x r x 35 = 462
r = 4.2 cm
50. Ans. B.
Let r be radius of inner circle.

PAGE 762
www.byjusexamprep.com

Let R be outer radius of path

Area

51. Ans. A.
Median is independent from the magnitude of all the terms since median is a magnitude of the
middle term when terms of a series is arranged in ascending or descending order so median is the
positional average of a data.
Extreme value distribution does not affect the value of median this implies median is the most
suitable measurement for an open ended frequency distribution.
Hence option A is the correct.
52. Ans. A.

Since
On pooling n1 & n2, the larger set of observation (n1 + n2) will have a mean lower than (because
of n, observations) but more than (because of n2 observations terms). Thus, it will be in between
&

53. Ans. B.
With increasing number of observations, the shape of frequency polygon tends to become
increasingly smooth.
54. Ans. B.
Since, of the two added observations one is lesser than the median & another is more.
Hence, they will have no effect on the median. Median will be stay 30.
55. Ans. A.
Let the ratio be x : y
Average marks of section A = 65
Average marks of section A = 65x
Average marks of section B = 70
Average marks of section B = 70y
Total Average = 67
Total marks of both sections 67(x+y)

PAGE 763
www.byjusexamprep.com

According to question,

56. Ans. B.
In ordinal scale, the various categories can be logically arranged in a meaningful order; however the
difference between categories is not meaningful.
Example: 1st, 2nd , 3rd etc.
57. Ans. B.
Students who passed both B & C = 30%
Students who passes both A & B but not C = 21%

Ratio

58. Ans. A.
At least 2 pass
= 75% of total students

59. Ans. B.
Students passed in only Biology = 6%
Students passed in only Physics =11%
Students passed in only Chemistry = 6%
Total 23%

60. Ans. C.

PAGE 764
www.byjusexamprep.com

A=Biology, B=Physics, C=Chemistry

Using this, we can find all values of Venn diagram.


61. Ans. D.

For a cubic eq.

62. Ans. A.

For a single positive solution, D=0

As it has a single positive solution, that is only possible when a =

PAGE 765
www.byjusexamprep.com

63. Ans. B.

the equation is not satisfied at origin, thus it lies away from origin

the equation is satisfied by origin; thus solution lies towards origin.

As per graph, solution lies in quadrant I & II.


64. Ans. B.
Let age of father x
Let age of son = y
Ac. To question

Age of son after son attains twice his present age = 2y


Age of father after son attains, twice his present age = y + x

PAGE 766
www.byjusexamprep.com

Hence, only B is correct.


65. Ans. D.
Lets say, (0.2)25 = x
log (0.2)25 = log x
25log (0.2) = log x
25(log 2-log 10 = log x
25(0.30101-1) = log x
log x = -17.47475
Therefore, Characteristic=17=number of zeroes immediately after decimal point in (0.2)25
66. Ans. B.
Let

67. Ans. B.

PAGE 767
www.byjusexamprep.com

68. Ans. C.
As 16 is their HCF, hence
Let

As HCF is 16, hence the sum of the two numbers must be a multiple of 16, this removes option (b)
& (d)
Using option (a),

&

Thus, which is less than 60.

Using option (b),

&

PAGE 768
www.byjusexamprep.com

69. Ans. C.
Let numbers be

As x is a natural number, hence x = 5.


Number are 5, 6, 7

70. Ans. B.

Using identity

71. Ans. D.
Let total work = 1
For full work let A takes days = a
1
1 day work of A =
a
Similarly, B takes days b
1
1 day work of B =
b

5 days work of … (i)

If A worked twice the original efficiency, then 1 day of work of A = 2/a

PAGE 769
www.byjusexamprep.com

1 1
If B worked rd effectively, then 1 day work of B =
3 3b

3 days work both

Acc. to the question,

Putting above eq. in (i)

72. Ans. C.
As
Hence, is negative

is both positive & its magnitude is greater than one.

Option a) is negative

Option b)

is negative

Option c)

Option d)

As
Hence, option a is bigger than option d
As q > p

PAGE 770
www.byjusexamprep.com

Hence, option c > option b


As r > q

So,

Hence,

Thus, option c is biggest.


73. Ans. C.

Thus, answer is
74. Ans. C.

No. of divisors

Excluding one & itself,

75. Ans. C.

Either &

Or and
Taking &
&

Taking &
&

PAGE 771
www.byjusexamprep.com

Hence, or
76. Ans. C.
As XY235 is divisible by 3
X+Y+2+3+5 is multiple of 3
X+Y+10 is multiple of 3
X+Y+1 is multiple of 3
Also,X+Y ≤ 5 hence
X+Y+1 ≤ 6
So X+Y+1 may be 3,6
So the possible pair of X and Y are 11,23,32,14,41,50,05
77. Ans. B.

Number XXYXX
Divisible by 11
=y
Hence, y being a single digit number must be divisible by 11 or else be zero.
For divisibility by 3, number must be divisible by 3

For , we will get as multiple of 3

Hence, number can be one of below three

Hence, answer is 3.
78. Ans. A.

Cubing both sides

PAGE 772
www.byjusexamprep.com

79. Ans. A.

Putting value of z in y & y in x, we get

On comparing powers, we get

80. Ans. B.
LCM of 16,36,45 & 48

LCM

On dividing 9999 (highest 4 digit no.) by 720, we get


639 as remainder,
Hence, required number

81. Ans. B.

Area

PAGE 773
www.byjusexamprep.com

82. Ans. A.
Let us draw a parallel line joining opposite vertices of parallelogram S & is parallel to other two
sides

Area (If a triangle & a parallelogram lie on the same base & between same
parallel lines then the area of the triangle will be half of the area of parallelogram)

Similarly,

Area

Adding above two, we get


Area

Thus, (1) is true.


(2) ab = dc (opposites sides of parallelogram)

Also, (diagonals of parallelogram bisects each other)

& (corresponding angles)


is congruent to

Similarly, &
Adding the above four, we get

Perimeter of
Perimeter of Sum of diagonal of T
Hence (2) is false.
83. Ans. B.
Length (L)
Breadth (B) = 4m

PAGE 774
www.byjusexamprep.com

Height (H) = 2.5 m

CSA of room

CSA of 5 rooms

Side of window

Area of window

Area of 2 window
Net area to be painted
require = 1l

1
1m2 require = l
20

require

12 cans are required.


84. Ans. D.

As ABC is an equilateral triangle, then

In

PAGE 775
www.byjusexamprep.com

… (i)

Also AO:OL=2:1 (property of equilateral triangle, O is the center of the circle)

Put above value in eq. (i)

(OL, OR radius and SQ is diameter)

So other diagonal PR is also of same length:

Area of square PQRS in terms of diagonal is given by:


Area

85. Ans. B.
The only possible triplet for the above data, with the given area of right-angled triangle is:
(P,B,H): (32,126,130)
So the perimeter is (32+126+130) = 288 units
86. Ans. B.
We can write that:
m=cot ϴ(1+sinϴ)/4
n=cot ϴ(1-sinϴ)/4
Then,
mn=[cot ϴ(1+sinϴ)/4][cot ϴ(1-sinϴ)/4]
mn=cot ² ϴ(1-sin ²ϴ)/16

PAGE 776
www.byjusexamprep.com

(using cot ² ϴ=cos ²ϴ/sin ²ϴ and 1-sin ²ϴ=cos ²ϴ)


mn=[(cos ²ϴ/sin ²ϴ)* cos ²ϴ ]/16
mn= cos4ϴ /16sin ²ϴ---------- (1)
Now, let us try to evaluate:
(m ²-n ²) ²
[cot ² ϴ(1+sin ²ϴ)/16- cot ²ϴ(1-sin ²ϴ)/16] ²
{[cot ² ϴ(1+sin ²ϴ) - cot ²ϴ(1-sin ²ϴ)]/16} ²
[(4sinϴcot ² ϴ)/16] ²
cos4ϴ /16sin ²ϴ----------------(2)
As equation (1) is equal to equation (2), so option B is the answer
87. Ans. D.

If
Then
Taking & we get

Squaring both sides

88. Ans. C.

89. Ans. C.
(1) As we know that

PAGE 777
www.byjusexamprep.com

Hence, (1) is correct.

(2)

----------(1)
Also, we know
using equation (1), we have:

Hence (2) is also correct.


90. Ans. C.

(1) … (i)

As
Hence,

cannot be less than 0, hence which is possible only when

PAGE 778
www.byjusexamprep.com

(2)

LHS cannot be negative as it is square term

91. Ans. C.

1)

Put

Hence, (1) is true.


2)
Put in LHS & RHS separately

PAGE 779
www.byjusexamprep.com

Hence (2) is also true, so correct option is C.


92. Ans. C.

… (i)

Putting in eq. (i)

93. Ans. A.

In

PAGE 780
www.byjusexamprep.com

In

94. Ans. B.

& both & are between 0 & 90 as increases, decreases.

Hence >
95. Ans. B.

… (i)

Squaring both sides

… (ii)

Dividing (ii) by (i)

Putting in given equation

96. Ans. C.
Total number of boys = 60
Number of boys who play chess = 45
Number of boys who play carrom = 30
Since all the 60 boys play at least one game
Number of boys who play carrom only will be = 60 – (number of boys who play chess) = 60 – 45 =
15.

PAGE 781
www.byjusexamprep.com

97. Ans. B.
Let the present ages of mother and daughter be x and y
So, according to the question

Also

Putting the value of x in the above equation, we get

Therefore x = 3*25 – 20 = 55
98. Ans. B.
Let the price of one lemon juice bottle = Rs. x
So, the price of one orange juice bottle = Rs. 2x
So the price of one orange and 4 lemon juice bottle will be = 2x + 4*x = 6x
Z’share in this will be = 6x/3 = 2x = 50
Therefore 2x = price of orange juice bottle = Rs. 50
99. Ans. C.
Initially according to the will
Mr. Sharma’s wife share = 50%
Ravi’s share = 25% = Raj’s share
After Ravi’s death
Ravi’s widow share = 25%/2 = 12.5%
Raj’s share = Raj’s initial share + remaining share of raj’s = 25% + 12.5% = 37.5%
After Raj’s Death,
Raj’s widow share = 37.5/2 % = 18.75%

PAGE 782
www.byjusexamprep.com

Mr. Sharma’s wife’s share = 50% + remaining share of Raj = 50% + 18.75% = 68.75%
68.75% of Mr. Sharma’s Property = Rs. 88000
100% of Mr. Sharma’s Property = 88000* 100/68.75 = Rs. 128000
100. Ans. A.
Let the price of article X and Y be x and y respectively
Price of articles before sales tax= Rs. 130
Price of article after sales tax = Rs. 136.75
Difference in prices = sales tax on article Y
136.75 – 130 = 9% of y
6.75 = y* 9/100
y = 6.75*100/9 = Rs. 75

PAGE 783
www.byjusexamprep.com

CDS I 2018
ENGLISH
Direction: In the following question, out of the meanings to each. Choose the response (A), (B),
four alternatives, select the alternative which (C) or (D) which is the most appropriate
best expresses the meaning of the Idiom/Phrase. expression and mark your response in the
Answer Sheet accordingly.
1. Cost an arm and a leg
5. You scratch my back. I’ll scratch yours
A. Severe punishment to someone
A. Mutual favour
B. Too much consciousness about one’s
body B. Mutual understanding
C. Two difficult alternatives C. Mutual respect
D. Something very expensive D. Mutual disliking
2. Best of both worlds. Direction: Given below are some idioms/phrases
followed by four alternative meanings to each.
A. A happy person who is the best with all
Choose the response (A), (B), (C) or (D) which is
B. All the advantages the most appropriate expression and mark your
response in the Answer Sheet accordingly.
C. To take on a task that is way too big
6. The Holy Grail
D. Someone whom everybody likes
A. The pious place of worship
3. Ball is in your court.
B. An important object or goal
A. Be happy at the dance / ball room
C. A very important place
B. It’s up to you to make the decision
D. Someone’s destination of life
C. A very powerful person
Direction: Given below are some idioms/phrases
D. Not speaking directly about an issue
followed by four alternative meanings to each.
4. At the drop of a hat. Choose the response (A), (B), (C) or (D) which is
the most appropriate expression and mark your
A. Without any hesitation
response in the Answer Sheet accordingly.
B. When attempt fails and it’s time to start
7. Step up the plate
all over
A. Take control
C. To further a loss with mockery
B. Take a job
D. Judging other’s intentions too much
C. Take a responsibility
Direction: Given below are some
idioms/phrases followed by four alternative D. Take an opportunity

PAGE 784
www.byjusexamprep.com

Direction: In the following question, out of the Direction: In the following question, out of the
four alternatives, select the alternative which four alternatives, select the alternative which
best expresses the meaning of the idiom/phrase. best expresses the meaning of the idiom/phrase.
8. Raising eyebrows. 12. My two cents.
A. To show surprise A. My money B. My opinion
B. Criticize C. My decision D. My explanation
C. Support Direction: Given below are some idioms/phrases
followed by four alternative meanings to each.
D. Instruct
Choose the response (A), (B), (C) or (D) which is
Direction: Given below are some idioms/phrases the most appropriate expression and mark your
followed by four alternative meanings to each. response in the Answer Sheet accordingly.
Choose the response (A), (B), (C) or (D) which is
13. I don’t buy it
the most appropriate expression and mark your
response in the Answer Sheet accordingly. A. I don’t believe it
9. Down the road B. I have no money
A. In future C. I summarize it
B. In the past D. I don’t need it
C. At present Direction: Given below are some idioms/phrases
followed by four alternative meanings to each.
D. No particular time
Choose the response (A), (B), (C) or (D) which is
Direction: Given below are some idioms/phrases the most appropriate expression and mark your
followed by four alternative meanings to each. response in the Answer Sheet accordingly.
Choose the response (A), (B), (C) or (D) which is
14. You snooze, you lose
the most appropriate expression and mark your
response in the Answer Sheet accordingly. A. Don’t take it lightly
10. What a small world B. Don’t be over-enthusiastic
A. What a coincidence C. Don’t hesitate to do it
B. What a challenging task D. Don’t be pessimistic
C. What a narrow space Direction: In the following question, out of the
four alternatives, select the alternative which
D. What a beautiful place
best expresses the meaning of the
Direction: In the following question, out of the idiom/phrase.
four alternatives, select the alternative which
15. A hot potato.
best expresses the meaning of the idiom/phrase.
A. A dish to relish when it is hot
11. Out of the blue.
B. A very important person in a gathering
A. Undoubtedly
C. An issue which is disputed, and catching
B. Unexpectedly
the attention of people
C. Unbelievably
D. A way of thinking what someone is
D. Unconcerned thinking

PAGE 785
www.byjusexamprep.com

Direction: Each of the following sentences in this Direction: Each of the following sentences in this
section has a blank space with four words or section has a blank space with four words or
group of words given. Select whichever word or group of words given. Select whichever word or
group of words you consider most appropriate group of words you consider most appropriate
for the blank space and indicate your response for the blank space and indicate your response
accordingly. accordingly.
The Second Anglo-Maratha War had shattered One of India’s greatest musicians is M.S.
the __16__ of the Maratha chiefs, but not their Subbulakshmi, affectionately known to most
spirit. The ___17___ of their freedom rankled in people as M.S. Her singing has brought
their hearts. They made a last (###Q3###) ___21___ to millions of people not only
attempt to regain their independence and old ___22___ all parts of India, but in ___23___.
(###Q4###) in 1817. The lead in organizing a countries around the world as well. ___24___
united front of the Maratha chiefs was taken by October 1966 Subbulakshmi was invited to
the Peshwa who was smarting under the ___25___. in New York, where people of
(###Q5###) control exercised by the British ___26___ foreign countries listened to her music
Resident. ___27___. This was one of the greatest
___28___ ever given to any musician. For
16. The Second Anglo-Maratha War had
shattered the __16__ of the Maratha ___29___ together M.S. kept that international
___30___ spell-bound with the beauty of her
chiefs,
voice and her style of singing.
A. pleasant B. satisfying
21. One of India’s greatest musicians is M.S.
C. rigid D. orthodox Subbulakshmi, affectionately known to
17. but not their spirit. The __17__ of their most people as M.S. Her singing has
freedom rankled in their hearts. brought ___21___ to millions

A. prestige B. army A. sorrow B. joy

C. rebellion D. infantry C. boredom D. pain

18. They made a last ___18___ attempt to 22. of people not only ___22___ all parts of
regain their India,
A. horrible B. desperate A. over B. on
C. poor D. strong C. in D. with
19. independence and old ___19___ in 1817. 23. but in ___23___. countries around the
world as well.
A. disappearance
A. strange countries B.unknown
B. empowerment
C. other D. familiar
C. loss
D. disappointment 24. ___24___ October 1966 Subbulakshmi was

20. The lead in organizing a united front of the A. Within B. On


Maratha chiefs was taken by the Peshwa C. In D. By
who was smarting under the ___20___
control exercised by the British Resident. 25. invited to ___25___ in New York,

A. power B. dignity A. dance B. sing

C. time D. patience C. speak D. enjoy

PAGE 786
www.byjusexamprep.com

26. where people of ___26___ foreign 32. Read the sentence to find out whether
countries listened there is any error in any part and mark your
response accordingly. If you find no error,
A. many B. few
your response should be indicated as D.
C. backward D. all
A. He has great fascination
27. to her music ___27___.
B. for each and every thing
A. attentively.
C. that are connected with drama,
B. quietly.
D. No error.
C. indifferently.
Direction: In this question, a sentence is divided
D. boldly. into three parts, each part being mentioned in
options A, B and C.
28. This was one of the greatest ___28___ever
given to any musician. 33. Read the sentence to find out whether
there is any error in any part and mark your
A. awards B. honours
response accordingly. If you find no error,
C. prizes D. recognitions your response should be indicated as D.
29. For ___29___ together M.S. kept A. He was
A. seconds B. minutes B. held in
C. hours D. days C. the prevention of Terrorism act.
30. that international ___30___ spell-bound D. No error.
with the beauty of her voice and her style
of singing. Direction: In this question, a sentence is divided
into three parts, each part being mentioned in
A. spectator B. audience options A, B and C.
C. viewer D. businessmen 34. Read the sentence to find out whether
Direction: In this question, a sentence is divided there is any error in any part and mark your
into three parts, each part being mentioned in response accordingly. If you find no error,
options A, B and C. your response should be indicated as D.

31. Read the sentence to find out whether A. The party


there is any error in any part and mark your B. was ousted in power
response accordingly. If you find no error,
your response should be indicated as D. C. after twelve years.

A. It’s no secret D. No error.

B. that the President wants to Direction: In this question, a sentence is divided


into three parts, each part being mentioned in
C. have a second term of office. options A, B and C.
D. No error.
35. Read the sentence to find out whether
Direction: In this question, a sentence is divided there is any error in any part and mark your
into three parts, each part being mentioned in response accordingly. If you find no error,
options A, B and C. your response should be indicated as D.

PAGE 787
www.byjusexamprep.com

A. Gandhi ji always regretted for the fact Direction: In this question, a sentence is divided
into three parts, each part being mentioned in
B. that people gave him adulation while options A, B and C.
what he wanted
39. Read the sentence to find out whether
C. was acceptance of his way of life. there is any error in any part and mark your
D. No error. response accordingly. If you find no error,
your response should be indicated as D.
Direction: In the following question, some part
of the sentence may have errors. Find out which A. Although the Red Cross accepts blood
part of the sentence has an error and select the from the donors
appropriate option. If the sentence is free from B. the nurses will not leave you give blood,
error, select 'No error'.
C. if you have just had cold.
36. The new model costs (A)/ twice more than
(B)/ last year’s model. (C)/ No Error D. No error.
Direction: In this question, a sentence is divided
A. A B. B
into three parts, each part being mentioned in
C. C D. No Error options A, B and C.

Direction: In this question, a sentence is divided 40. Read the sentence to find out whether
into three parts, each part being mentioned in there is any error in any part and mark your
options A, B and C. response accordingly. If you find no error,
your response should be indicated as D.
37. Read the sentence to find out whether
there is any error in any part and mark your A. There have been a tomado watch
response accordingly. If you find no error, B. issued for Texas country
your response should be indicated as D.
C. until eleven O’clock tonight.
A. Because of the movement of a glacier,
D. No error.
B. the form
Direction: In this question, a sentence is divided
C. of the Great Lakes was very slow. into three parts, each part being mentioned in
options A, B and C.
D. No error.
41. Read the sentence to find out whether
Direction: In this question, a sentence is divided
there is any error in any part and mark your
into three parts, each part being mentioned in
response accordingly. If you find no error,
options A, B and C.
your response should be indicated as D.
38. Read the sentence to find out whether A. I asked him B. what he
there is any error in any part and mark your
response accordingly. If you find no error, C. has done. D. No error.
your response should be indicated as D. Direction: In this question, a sentence is divided
A. A prism is used to refract white light into three parts, each part being mentioned in
options A, B and C.
B. so it spreads out
42. Read the sentence to find out whether
C. in a continuous spectrum. there is any error in any part and mark your
response accordingly. If you find no error,
D. No error.
your response should be indicated as D.

PAGE 788
www.byjusexamprep.com

A. The fruit B. can be made Direction: In this question, a sentence is divided


into three parts, each part being mentioned in
C. to jam. D. No error. options A, B and C.
Direction: Each item in this section has a 46. Read the sentence to find out whether
sentence with three underlined parts there is any error in any part and mark your
labelled (a), (b) and (c). Read each sentence response accordingly. If you find no error,
to find out whether there is any error in any your response should be indicated as D.
underlined part and indicate your response
in the answer sheet against the A. Two thirds of the book
corresponding letter i.e. (a) or (b) or (c). If
you find no error, your response should be B. were
indicated as (d). C. rubbish.
43. I don’t agree (A)/ with smacking children D. No error.
(B)/ if they do something wrong. (C)/ No
error (D) Direction: In this question, a sentence is divided
into three parts, each part being mentioned in
A. A B. B options A, B and C.
C. C D. D 47. Read the sentence to find out whether
Direction: In this question, a sentence is divided there is any error in any part and mark your
into three parts, each part being mentioned in response accordingly. If you find no error,
options A, B and C. your response should be indicated as D.

44. Read the sentence to find out whether A. The students' B. test results
there is any error in any part and mark your C. were pleasant. D. No error.
response accordingly. If you find no error,
your response should be indicated as D. Direction: In this question, a sentence is divided
into three parts, each part being mentioned in
A. She felt options A, B and C.
B. terribly anxious for have to sing 48. Read the sentence to find out whether
C. in front of such a large audience. there is any error in any part and mark your
response accordingly. If you find no error,
D. No error. your response should be indicated as D.
Direction: In this question, a sentence is divided A. Electricity companies are
into three parts, each part being mentioned in
options A, B and C. B. working throughout

45. Read the sentence to find out whether C. days and nights to repair the damage.
there is any error in any part and mark your D. No error.
response accordingly. If you find no error,
your response should be indicated as D. Direction: In this question, a sentence is divided
into three parts, each part being mentioned in
A. You will be options A, B and C.
B. answerable for the court with 49. Read the sentence to find out whether
C. any lies you have told. there is any error in any part and mark your
response accordingly. If you find no error,
D. No error. your response should be indicated as D.

PAGE 789
www.byjusexamprep.com

A. Somebody 52. Which one of the following correctly


expresses the meaning of “wishing to evade
B. who I enjoy reading the law” ?
C. is Tagore. A. Wish to avoid following the law
D. No error. B. Desire to follow the law blindly

Direction: In this question, a sentence is divided C. Reluctance to break the law


into three parts, each part being mentioned in D. Wish to change the law
options A, B and C.
53. The pick-pocket travelling with the lady
50. Read the sentence to find out whether
A. reported the matter to the conductor
there is any error in any part and mark your
response accordingly. If you find no error, B. put his hand in her pocket
your response should be indicated as D. C. took out the dog
A. The best way in which you can D. asked the lady to get off
B. open the bottle is 54. In order to evade the law, the lady

C. by putting it into hot water first. A. hid the dog under the seat
B. got off the bus
D. No error.
C. gave the dog to a fellow passenger
Direction: Given below is a short passage. After
the passage, you will find some items based on D. put the dog in her pocket
the passage. First, read a passage and answer the 55. Once the law in Delhi did not permit the
items based on it. You are required to select your people to
answers based on the contents of the passage
A. board a bus without ticket
and opinion of the author only.
B. carry dogs into private vehicles
In Delhi, it was forbidden by the law, at one time,
to take a Dog into a public vehicle. One day a C. carry animals in public vehicles
lady, accompanied by a pet dog, entered a bus. D. carry animals with them
Wishing to evade the law, she placed her tiny
dog in her dress pocket. It so happened that the Direction: Given below is a short passage. After
person next to this lady was a pick-pocket; and the passage, you will find some items based on
during the journey he carefully placed his hand the passage. First, read a passage and answer the
into her pocket in search of her purse. Great was items based on it. You are required to select your
the horror to find instead a pair of sharp teeth answers based on the contents of the passage
inserted into his fingers. His exclamation of pain and opinion of the author only.
and surprise drew the attention of other I have always opposed the idea of dividing the
passengers to him. world into the Orient and the Occident. It is,
however, the tremendous industrial growth that
51. Why did the pick-pocket exclaim with pain?
has made the West what it is. I think the
A. He was hit by the lady difference, say, between India and Europe in the
12th or 13th century would not have been very
B. He was caught by the fellow- great. Differences have been intensified by this
process of industrialization which has promoted
C. He was bitten by the dog
material well-being tremendously and which is
D. He fell of the bus destroying the life of the mind, which is in a

PAGE 790
www.byjusexamprep.com

process of deterioration, chiefly because the 60. The words “the Orient and the Occident”
environment that has been created by it does mean
not give time or opportunity to individuals to
A. the West and the East respectively
think. If the life of the mind is not encouraged,
then inevitably civilization collapses. B. the East and the West respectively
56. The title that best expresses the central C. the North and the South respectively
idea of the passage is
D. the South and the North respectively
A. difference between the Occident and
Direction: Given below is a short passage. After
the Orient
the passage, you will find some items based on
B. impact of Industrialization on our the passage. First, read a passage and answer the
civilization items based on it. You are required to select your
C. advantages of Industrialization answers based on the contents of the passage
and opinion of the author only.
D. disadvantages of Industrialization
To eat and not be eaten - that’s the imperative
57. The author says that the mental life of the of a caterpillar’s existence. The leaf roller
world is in a process of deterioration reduces its risks of being picked off by predators
because the modem generation is by silking together a temporary shelter in which
A. endowed with low mental powers to feed and rest. Adopting a different line of
defense, the jelly slug extrudes a sticky
B. too lazy to exert its mental powers translucent coating that may foul the mouth-
C. taught that physical activities are more parts of marauding ants. For its part, the aquatic
important than mental larva, by its watery element, fashions a portable
hideout from fragments of aquatic leaves.
D. brought up in an environment.
Cutting a serpentine trail as it feeds on tender
unfavourable to the growth of the
young leaves, the minute citrus leaf miner
mental life
spends its entire larval life inside its host plant,
58. In the opinion of the author, thus keeping its appetizing body safely under
Industrialization is wraps.
A. an absolute blessing 61. The main idea of the passage is that
B. an absolute curse caterpillars

C. neither a blessing nor a curse A. like to eat a lot

D. more of a curse than a blessing B. have to protect themselves while


feeding
59. The author behaves that the difference
between India and Europe in the 12th or C. are good to eat
13th century was not very great because D. are not good to eat
A. Indians and Europeans mixed freely 62. Which one of the following makes itself
B. Indians imitated the European way of unpalatable?
living A. Leaf roller
C. Europeans imitated the Indian way of B. Jelly slug
living
C. Aquatic larva
D. Industrialization had not yet taken
place D. Leaf miner

PAGE 791
www.byjusexamprep.com

63. Which one of the following pairs of words 66. What is the irony behind the over-
in the passage describes enemies of the population of India?
caterpillar?
A. Over-population gives birth to poverty,
A. Serpentine and host which (poverty) itself is the cause of
over-population
B. Predator and marauding
C. Serpentine and marauding B. Under nourishment and
unemployment are outcomes of flawed
D. Predator and host economic progress
64. Which one of the following caterpillars C. Fragmentation of land is leading to
produces a sticky covering? over-population
A. Leaf roller D. Fruits of the remarkable economic
B. Jelly slug progress are trickling down to the Poor
C. Aquatic larva 67. What is the general tone of the passage?
D. Citrus leaf miner A. funny/ humorous B.sombre
65. Which varieties of caterpillars ‘build’ C. didactic D. tragic
shelters to protect themselves?
68. What, in the author’s view, severely affects
A. Leaf roller and aquatic larva the economic growth of our country?
B. Leaf roller and jelly slug A. poverty B. illiteracy
C. Jelly slug and aquatic larva C. over-population D. None of these
D. Jelly slug and citrus leaf miner 69. What, according to the author, is the
Direction: Given below is a short passage. After biggest reason behind over-population?
the passage, you will find some items based on
A. under-nourishment
the passage. First, read a passage and answer the
items based on it. You are required to select your B. unemployment
answers based on the contents of the passage
C. excessive fragmentation of land
and opinion of the author only.
Over-population is the most pressing of India’s D. poverty
numerous and multi-faceted problems. In fact it 70. “It has largely diluted the fruits of the
has caused equally complex problems such as remarkable economic progress”. Find
poverty, under-nourishment, unemployment antonym of the underlined word
and excessive fragmentation of land.
Indisputably, this country has been facing a A. coalesced B. compounded
population explosion of crisis dimensions. It has C. cheapened D. consolidated
largely diluted the fruits of the remarkable
economic progress that the nation has made Direction: In the given question six sentences of
during the last four decades or so. The entire a passage are a given. The first and sixth
battle against poverty is thwarted by the rapid sentences are given in the beginning as S1 and
increase in the population. The tragedy is that S6. The middle four sentences in each have been
while over-population accentuates poverty, the jumbled up and labelled P, Q, R and S. You are
country’s stark poverty itself is in many areas a required to find the proper sequence of the four
major cause of over-population. sentences and mark your response accordingly.

PAGE 792
www.byjusexamprep.com

71. SI: Mass production has increased the A. P Q R S B. S R Q P


tendency to view things as useful rather
C. S P R Q D. S R P Q
than delightful.
S6: Indeed a lowering of quality usually Direction: In this section, each item consists of
results when mass production is six sentences of a passage. The first and sixth
substituted for more primitive methods. sentence are given in the beginning as S1 and S6.
The middle four sentences in each have been
P: These various things share nothing with jumbled up and labelled P, Q, R and S. You are
the buttons except money value. required to find the proper sequence of the four
Q: All the rest you wish to exchange for sentences and mark your response accordingly.
food, shelter, and many other things. 73. SI: At 4 o'clock this morning, Hitler attacked
R: Suppose you are a manufacturer of and invaded Russia.
buttons: however excellent your buttons S6: Under its cloak of false confidence, the
may be, you do not want more than a few German armies drew up in immense
for your own use. strength along a line which stretches from
S: And it is not even the money value of the the White Sea to the Black Sea.
buttons that is important to you: what is P: No complaint had been made by
important is profit which may be increased Germany of its non-fulfilment.
by lowering their quality.
Q: All his usual formalities of perfidy were
The correct sequence should be
observed with scrupulous technique.
A. P Q R S B. R Q P S
R: No one could have expected that Hitler
C. S P Q R D. Q R P S would do it.
Direction: In the given question six sentences of S: A non-aggression treaty had been
a passage are a given. The first and sixth solemnly signed and was in force between
sentences are given in the beginning as S1 and the two countries.
S6. The middle four sentences in each have been
jumbled up and labelled P, Q, R and S. You are A. R Q S P B. R S Q P
required to find the proper sequence of the four C. P S Q R D. Q P S R
sentences and mark your response accordingly.
Direction: In this section, each item consists of
72. SI: Roderick Usher has always been a quiet six sentences of a passage. The first and sixth
person who talked little of himself sentences are given in the beginning as S1 and
S6: In the part of the country where he S6. The middle four sentences in each have been
lived, the “House of Usher” had come to jumbled up and labelled as P, Q, R and S. You are
mean both the family and its ancestral required to find the proper sequence of the four
mansion. sentences and mark your response accordingly
on the Answer Sheet.
P: Many of his ancestors had been famous
for their artistic and musical abilities. 74. S1: Union finance ministry announced a
series of concessions to trade and industry
Q: Others were known for their exceptional
last month.
generosity and charity.
S6: Manufacturers "feel that prices of
R: Yet I did know that his family was an old
certain components may not be brought
one.
down because of the imposition of a 30
S: So I did not know too much about him. percent duty where there was none earlier.

PAGE 793
www.byjusexamprep.com

P: Together, these will result in a loss of required to find the proper sequence of the four
revenue of Rs. 100 crore to the exchequer. sentences and mark your response accordingly.
Q: Earlier, these were attracting customs 76. SI: To most people, the term technology
duty varying from zero to 100 percent. conjures up images of mills or machines.
R: The chunk of the relief of Rs. 60 crore has S6; It includes ways to make chemical
gone to the electronics industry. reactions occur, ways to breed fish, plant
forests or teach history.
S: Raw materials and piece parts now carry
customs duty of 30 percent and 40 percent P: The classic symbol of technology is still
ad valorem respectively. the assembly line created by Henry Ford
half a century ago.
A. RSQP B. PRSQ
Q: The invention of the horse collar in the
C. SQPR D. QPRS
Middle Ages led to changes in agricultural
Direction: In the given question six sentences of methods and was as such a technological
a passage are a given. The first and sixth advance.
sentences are given in the beginning as S1 and
R: Moreover, technology includes
S6. The middle four sentences in each have been
techniques, as well as the machines that
jumbled up and labelled P, Q, R and S. You are
may or may not be necessary to apply
required to find the proper sequence of the four
them.
sentences and mark your response accordingly.
S: This symbol, however, has always been
75. SI: I was the secretary of the Philosophical
inadequate, for technology has always
Society of the Patna College.
been more than factories and machines.
S6: I have been to Kolkata many times
The correct sequence should be
since, but never has it been more pleasant
than that first visit. A. S P R Q B. P S Q R
P: It was my first visit to the city and its C. R S P Q D. Q S R P
impression on my mind was indelible.
Direction: In the following items, each passage
Q: In that capacity, I once led a trip to consists of six sentences. The first sentence (S1)
Kolkata. and the final sentence (S6) are given in the
R: I felt I had landed in the midst of beginning. The Middle four sentences in each
beautiful dream world of a fairy land. have been removed and jumbled up. These are
labelled P, Q, R and S. You are required to find
S: I saw the roads, the trams, the out the proper sequence of the four sentences
skyscrapers and the magnificent shops at and mark accordingly on the Answer Sheet.
the Chowranghee lane.
77. SI: It would be possible to adduce many
The correct sequence should be examples showing what could be done with
A. Q P S R B. P S Q R the limited means' at our ancestor’s
disposal in the way of making life
C. S R P Q D. S Q R P comfortable.
Direction: In the given question six sentences of S6: I hope, in this essay, to make that
a passage are a given. The first and sixth connection manifest.
sentences are given in the beginning as S1 and
S6. The middle four sentences in each have been P: What have comfort and cleanliness to do
jumbled up and labelled P, Q, R and S. You are with politics, morals, and religion?

PAGE 794
www.byjusexamprep.com

Q: But look more closely and you will S6. The middle four sentences in each have been
discover that there exists the closest jumbled up and labelled P, Q, R and S. You are
connection between the recent growth of required to find the proper sequence of the four
comfort and the recent history of ideas. sentences and mark your response accordingly.
R: They show that if they lived in filth and 79. SI: In democratic countries, men are equal
discomfort, it was because filth and before the law.
discomfort fitted in with their principles,
S6: And they live like this not for fun, but
political, moral and religious.
because they are too poor to afford
S: At a first glance one would say that there another room.
could be no causal connection between
P: While some few people live in luxury,
armchairs and democracies, sofas and the
many have not enough to eat, drink and
family system, hot baths and religious
wear.
orthodoxy.
Q: There are many families of five or six
A. PRQS B. RPSQ
persons who live in a single room.
C. QSRP D. QSPR
R: But the sharing-out of money - which
Direction: In this section, each item consists of means the sharing-out of food and clothing
six sentences of a passage. The first and sixth and houses - is still very unfair.
sentence are given in the beginning as S1 and S6.
The middle four sentences in each have been S: In this room they sleep and dress and
jumbled up and labelled P, Q, R and S. You are wash and eat, and in this same room they
required to find the proper sequence of the four die.
sentences and mark your response accordingly. The correct sequence should be
78. SI: Tomorrow it will be a year since we lost A. R P Q S B. P R S Q
our great leader.
C. Q S P R D. S P R Q
S6: Though he is no more with us, the
qualities he possessed and the ideals he Direction: In the given question six sentences of
cherished remain with us. a passage are a given. The first and sixth
sentences are given in the beginning as S1 and
P: To these, he added a feminine S6. The middle four sentences in each have been
sensitiveness to atmosphere. jumbled up and labelled P, Q, R and S. You are
Q: He was involved in the major events of required to find the proper sequence of the four
his time. sentences and mark your response accordingly.

R: He participated in them all while 80. SI: First and foremost, there are order and
maintaining the highest standards of public safety in our civilization.
conduct. S6: Nobody may come and break into my
S: He was incomparably the greatest figure house and steal my goods.
in our history - a man of dynamic force, P: Thus in disputes between man and man,
intellectual power and profound vision. right has taken the place of might.
A. P S R Q B. R Q P S Q: If today I have a quarrel with another
C. R P Q S D. S P Q R man, I do not get beaten merely because I
am physically weaker.
Direction: In the given question six sentences of
a passage are a given. The first and sixth R: I go to law, and the law will decide fairly
sentences are given in the beginning as S1 and between the two of us.

PAGE 795
www.byjusexamprep.com

S: Moreover, the law protects me from Direction: The following question consists of a
robbery and violence. sentence with an underlined word/words
followed by four words. Select the option that is
The correct sequence should be
opposite in meaning to the underlined
A. R Q P S B. S R Q P word/words and mark your response
C. Q R P S D. P R S Q accordingly.
Direction: The following question consists of a 85. Indian culture has been, from time
sentence with an underlined word/words immemorial, of a peculiar cast and mould.
followed by four words. Select the option that is A. ordinary B. customary
opposite in meaning to the underlined
word/words and mark your response C. natural D. fascinating
accordingly. Direction: The following question consists of a
81. Rakesh is vulnerable to political pressure. sentence with an underlined word/words
followed by four words. Select the option that is
A. weak B. unguarded opposite in meaning to the underlined
C. exposed D. resilient word/words and mark your response
accordingly.
Direction: In the following question, out of the
five alternatives, select the word opposite in 86. It could not have been expected that, with
meaning to the given word. such a bent of mind of the people, there
should have been much activity for the
82. Terrorists profess Fanatical ideology.
cultivation of the physical sciences in this
A. Bigoted B. Dogmatic part of the world.
C. Insouciant D. Fervid A. dull B. dormant
E. Infatuated C. indolence D. idle
Direction: The following question consists of a Direction: The following question consists of a
sentence with an underlined word/words sentence with an underlined word/words
followed by four words. Select the option that is followed by four words. Select the option that is
opposite in meaning to the underlined opposite in meaning to the underlined
word/words and mark your response word/words and mark your response
accordingly. accordingly.
83. Macbeth is a/an abominable figure. 87. I would beg of all friends not to rush to Birla
A. abhorrent B. repugnant house nor try to dissuade me or be anxious
about me.
C. reputable D. attractive
A. certain B. composed
Direction: The following question consists of a
sentence with an underlined word/words C. careless D. heedless
followed by four words. Select the option that is Direction: The following question consists of a
opposite in meaning to the underlined sentence with an underlined word/words
word/words and mark your response followed by four words. Select the option that is
accordingly. opposite in meaning to the underlined
84. The princess charming was the centre of word/words and mark your response
attraction today. accordingly.
A. enchanting B. hypnotic 88. Some of the criticisms which they had to
C. repulsive D. fascinating put up were very unfair.

PAGE 796
www.byjusexamprep.com

A. scold B. scorn Direction: Choose the most appropriate


alternative to complete the sentence:
C. appreciation D. censure
93. Every rash driver becomes a _____ killer.
Direction: The following question consists of a
sentence with an underlined word/words A. sure B. reckless
followed by four words. Select the option that is
C. potential D. powerful
opposite in meaning to the underlined
word/words and mark your response Direction: Select the most appropriate option to
accordingly. fill in the blank.
89. It’s the only treatment suitable for cancer. 94. You must go to the station now, your
brother_______ go just yet as his train
A. insufficient B. impertinent
leaves after three hours.
C. befitting D. congenial
A. shouldn’t B. mustn’t
Direction: The following question consists of a
C. wouldn’t D. needn’t
sentence with an underlined word/words
followed by four words. Select the option that is Direction: Fill in the blanks with appropriate
opposite in meaning to the underlined blanks:
word/words and mark your response 95. It is no use in crying over ____________.
accordingly.
A. spoiled milk B. spirited milk
90. It was a mystery as to where the young girl
had acquired such a naive belief. C. split milks D. spilt milk

A. credulous B. childlike Direction: Each of the following sentences in this


section has a blank space and four words or
C. wise' D. innocent group of words are given after the sentence.
Direction: The following sentence has a blank Select the word or group of words you consider
space and four words are given below it. Select the most appropriate for the blank space and
the word you consider most appropriate for the indicate your response on the Answer Sheet
blank and indicate your choice accordingly. accordingly.
91. The whole lot of young men was very 96. I do not think he will ever_______ the shock
enthusiastic but your friend alone was of his wife’s death,
_____ A. get by B. get off
A. quarrelsome B. sincerity C. get through D. get over
C. a wet blanket D. sleepy Direction: Fill in the blanks with appropriate
Direction: Each of the following sentences in this blanks:
section has a blank space and four words or 97. Scarcely _____ the teacher entered the
group of words are given after the sentence. class when he heard the noise.
Select the most appropriate word or group of
words for the blank space and indicate your A. did B. has
response on the Answer Sheet accordingly. C. had D. will have
92. The country owes a deep debt of________ Direction: Each of the following sentences in this
for the freedom fighters. section has a blank space and. four words or
A. patriotism B. complaining group of words given after the sentence. Select
whichever word or group of words you consider
C. remembrance D. gratitude most appropriate for the blank space and

PAGE 797
www.byjusexamprep.com

indicate your response on the Answer Sheet These parts have been labelled P, Q, R and S. You
accordingly. are required to re-arrange the jumbled parts of
the sentence and mark your response
98. After the marathon, some of the
accordingly.
competitors felt completely __________.
102. and the current Hindu emphasis (P) and
A. cut up B. done in
urged the people to imbibe the spirit of
C. done out D. run out free-thinking (Q) on rituals, ceremonies and
Direction: Each of the following sentences in this superstitions (R) Vivekananda condemned
section has a blank space and four words or the caste system (S).
group of words are given after the sentence. A. P Q R S B. S P R Q
Select the word or group of words you consider
C. S P Q R D. R P S Q
the most appropriate for the blank space and
indicate your response on the Answer Sheet Direction: The following question consists of a
accordingly. sentence, the parts of which have been jumbled.
These parts have been labelled P, Q, R and S. You
99. I hope that the rain will _______ for our
are required to re-arrange the jumbled parts of
picnic tomorrow.
the sentence and mark your response
A. keep off B. put off accordingly.
C. set back D. stay out 103. showed that a backward (P) the rise of (Q)
Direction: Select the most appropriate option to modern Japan after 1868 (R) Asian country
fill in the blank. could develop itself without Western
control (S).
100. In the face of the overwhelming mass of
evidence against hint, we cannot ______ A. P Q R S B. S R Q P
him of the crime. C. P R Q S D. Q R P S
A. punish B. absolve Direction: The following question consists of a
C. release D. ignore sentence, the parts of which have been jumbled.
These parts have been labelled P, Q, R and S.
Direction: The following question consists of a Given below each sentence are four sequences
sentence, the parts of which have been jumbled. namely A, B, C and D. You are required to re-
These parts have been labelled P, Q, R and S. arrange the jumbled parts of the sentence and
Given below each sentence are four sequences mark your response accordingly.
namely A, B, C and D. You are required to re-
arrange the jumbled parts of the sentence and 104. P: to accept any of the important
mark your response accordingly. Q: disillusionment
101. Mr. John R: demands of the nationalists produced
P: who was hardly six months old S: the failure of the British govemment
Q: Charles A. S P R Q B. P Q R S
R: as his son C. S R Q P D. Q R P S
S: adopted Direction: The following question consists of a
A. S Q R P B. P S Q R sentence, the parts of which have been jumbled.
These parts have been labelled P, Q, R and S. You
C. R S P Q D. P R S Q are required to re-arrange the jumbled parts of
Direction: The following question consists of a the sentence and mark your response
sentence, the parts of which have been jumbled. accordingly.

PAGE 798
www.byjusexamprep.com

105. Muhammad Iqbal (P) the philosophical and manufacturers (R)/ to restrict and prohibit
religious outlook of people through his (S)
poetry (Q) profoundly influenced (R) one of
A. P Q R S B. S P R Q
the greatest poets of modem India (S).
C. R P S Q D. Q R S P
A. Q R S P B. S R Q P
Direction: The following question consists of a
C. S R P Q D. S P R Q sentence, the parts of which have been jumbled.
Direction: Each of the following items in this These parts have been labelled P, Q, R and S. You
section consists of a sentence the parts of which are required to re-arrange the jumbled parts of
have been jumbled. These parts have been the sentence and mark your response
labelled as P Q, R and S. Given below each accordingly.
sentence are four sequences namely (a), (b), (c) 109. dress fashions changed and light cotton (P)
and (d). You are required to rearrange the of the English (Q) textiles began to replace
jumbled parts of the sentence and mark your (R) the coarse woolens (S).
response on the Answer Sheet accordingly.
A. P R S Q B. R S P Q
106. and it led to rapid economic development
(P) the Industrial Revolution (Q) C. Q P R S D. S P R Q
transformed the British (R) society in a Direction: The following question consists of a
fundamental manner (S) sentence, the parts of which have been jumbled.
A. SPRQ B. QPRS These parts have been labelled P, Q, R and S. You
are required to re-arrange the jumbled parts of
C. QRSP D. SRPQ the sentence and mark your response
Direction: Each of the following items in this accordingly.
section consists of a sentence the parts of which 110. the British manufacturers (P) popularity of
have been jumbled. These parts have been Indian textiles (Q) were jealous of the (R)
labelled as P Q, R and S. Given below each from the very beginning (S).
sentence are four sequences namely (a), (b), (c)
and (d). You are required to rearrange the A. P Q R S B. S P Q R
jumbled parts of the sentence and mark your C. S P R Q D. Q R S P
response on the Answer Sheet accordingly.
Direction: The following question consists of a
107. however (P) still held their own in foreign sentence with an underlined word/words
markets (Q) in spite of these laws (R) Indian followed by four words. Select the option that is
silk and cotton textiles (S) nearest in meaning to the underlined
A. Q P R S B. S P R Q word/words and mark your response
accordingly.
C. S R P Q D. R P S Q
111. Can medicines save us from death?
Direction: Each of the following items in this
A. hide B. rescue
section consists of a sentence the parts of which
have been jumbled. These parts have been C. protect D. liberal
labelled P, Q, R and S. Given below each part are
Direction: The following question consists of a
four sequences namely A., B., C. and D.. You are
sentence with an underlined word/words
required to rearrange the jumbled part of the
followed by four words. Select the option that is
sentence and mark your response accordingly.
nearest in meaning to the underlined
108. put pressure on their government (P)/ word/words and mark your response
Indian goods in England (Q)/ the British accordingly.

PAGE 799
www.byjusexamprep.com

112. The committee should recommend his Direction: Each item in this section consists of a
name to the government. sentence with an underlined word followed by
four options. Select the option that is nearest in
A. praise B. advice
meaning to the underlined word and mark your
C. counsel D. suggest response accordingly.
Direction: Each item in this section consists of a 117. Vendors must have licence.
sentence with an underlined word followed by
four options. Select the option that is nearest in A. One who drives a car
meaning to the underlined word and mark your B. One who works in a hospital
response accordingly.
C. One who is employed in food serving
113. They served fruits after the dinner.
D. One engaged in selling
A. assisted B. obliged
Direction: The following question consists of a
C. waited D. offered
sentence with an underlined word/words
Direction: The following question consists of a followed by four words. Select the option that is
sentence with an underlined word/words nearest in meaning to the underlined
followed by four words. Select the option that is word/words and mark your response
nearest in meaning to the underlined accordingly.
word/words and mark your response
118. She deftly masked her feelings.
accordingly.
114. The soldier showed an exemplary courage. A. hid B. flaunted

A. flawed B. faulty C. oblique D. obscured

C. ideal D. boisterous Direction: Each item in this section consists of a


sentence with an underlined word followed by
Direction: Each item in this section consists of a four words/group of words. Select the option
sentence with an underlined word followed by that is nearest in meaning to the underlined
four options. Select the option that is nearest in word/words and mark your response in your
meaning to the underlined word and mark your Answer Sheet accordingly.
response accordingly.
119. The soldiers repulsed the enemy.
115. The jewels have been stolen from her
bedroom. A. Defeated B. Destroyed
A. embezzled B. asserted C. Rejected D. Repelled
C. yielded D. abdicated Direction: Each item in this section consists of a
Direction: The following question consists of a sentence with an underlined word followed by
sentence with an underlined word/words four options. Select the option that is nearest in
followed by four words. Select the option that is meaning to the underlined word and mark your
nearest in meaning to the underlined response accordingly.
word/words and mark your response 120. A truly respectable old man is a ripe person.
accordingly.
A. senior
116. They will not admit children under
fourteen. B. mature

A. avow B. receive C. perfect

C. accept D. concede D. seasoned

PAGE 800
www.byjusexamprep.com

General Knowledge
1. According to the law of diminishing 4. Which one of the following statements
marginal utility, as the amount of a good about the Elephanta Island is correct?
consumed increases, the marginal utility or
that good tends to A. It was given its name by the British after
a large elephant structure located
A. improve there.
B. diminish B. It contains one large cave.
C. remain constant C. It is well-known for a spectacular
D. first diminish and then improve carving of Vishnu described in the
Vishnudharmattara Purana
2. Consider the following statements about
Impact of tax D. It is associated with the Pashupata sect.
1. A tax is shifted forward to Consumers if 5. Who is the author of Manimekala?
the demand is inelastic relative to
supply. A. Kovalan B. Sathanar

2. A tax is shifted backward to producers C. Hango Adigal D. Tirutakkatevar


if the supply is relatively more inelastic 6. Which of the following is/are the feature(s)
than demand. of the Brahmadeya Grants during c 600-
Which of the statements given above is/are 1200 AD?
correct?
1. Their creation meant a renunciation of
A. 1 only B. 2 only actual or potential sources of revenue
by the State.
C. Both 1 and 2 D. Neither 1 nor 2
3. Which of the following statements about 2. These grants could vary from a small
Sir Syed Ahmad Khan is/are correct? plot to several villages.

1. He argued that India was federation of 3. Most grants were made in unsettled
ethnic communities based on common areas
descent. Select the correct answer using the code
2. His Philosophy was very similar to that given below.
of the Indian National Congress.
A. 1 only B. 2 and 3 only
3. He imagined India as a Nation State
C. 1 and 2 only D. 1, 2 and 3
based on individual citizen’s rights.
4. The curriculum at the Mohammedan 7. Who deciphered the Brahmi and
Anglo-Oriental College blended Muslim Kharoshthi scripts?
theology and European empiricism. A. Piyadassi
Select the correct answer using the code B. Colin Mackenzie
given below.
C. Alexander Cunningham
A. 1 only B. 2 and 3
C. 3 only D. 1 and 4 D. James Prinsep

PAGE 801
www.byjusexamprep.com

8. The first BRICS Summit, after the inclusion 14. Which one of the following statements
of South Africa, was held at about the Deen Dayal Upadhyaya Grameen
Kaushalya Yojana (DDU-GKY) is not correct?
A. Brasilia
A. It is a placement-linked skill training
B. Sanya
programme exclusively for rural girls.
C. Yekaterinburng
B. It aims to convert India’s demographic
D. Durban surplus into a demographic dividend.
9. The judgement of the Supreme Court of C. The scheme aims to benefit more than
India in the Vishakha Case pertains to 55 million poor rural folks.
A. sexual harassment in the work-place D. It is a generational poverty alleviation
programme.
B. Sati
C. dowry death 15. Which one of the following statements
about ‘Niryat Bandhu Scheme’ is correct?
D. rape
A. It is a scheme for mentoring first
10. ‘SAMPRITI-2017’ is a joint military exercise generation entrepreneurs.
between armed forces of India and
B. It is a scheme for crop protection.
A. Bhutan B. Bangladesh
C. It is a scheme for the vulnerable section
C. Pakistan D. Myanmar of the society.
11. Which one of the following statements D. It is a scheme for monitoring rural poor.
about the National Adaptation Fund for
Climate Change is not correct? 16. Which one of the following is not an
objective of the Pradhan Mantri Krishi
A. The fund is meant to assist national and Sinchayi Yojana (PMKSY)?
State level activities to meet the cost of
adaptation measures. A. To achieve convergence of investment
in irrigation at the field level
B. This scheme has been taken as a
Central Sector Scheme. B. To expand cultivable area under
irrigation
C. The Indian Council of Agricultural
Research is the national implementing C. To improve on-farm water use
efficiency to reduce wastage of water
D. The scheme has been in force since
2015-2016 D. To protect farmers against crop failure
due to natural calamities.
12. The projects under Coastal Berth Scheme of
the flagship Sagarmala Programme are 17. In the context of elections in India, which
distributed over how many States? one of the following is the correct full form
of VVPAT?
A. Eight B. Ten
A. Voter Verifiable Poll Audit Trail
C. Twelve D. Fourteen
B. Voter Verifing Paper Audit Trail
13. Where is Hambantota Port located?
C. Voter Verifiable Paper Audit Trail
A. Iran B. Sri Lanka
C. Japan D. Pakistan D. Voter Verifiable Paper Account Trail

PAGE 802
www.byjusexamprep.com

18. Which one of the following is not an 21. Which of the following statements about
International Humans Rights Treaty? the Ordinance-making power of the
Governor is correct?
A. International Covenant on Civil and
Political Rights 1. It is a discretionary power.

B. Convention on the Elimination of All 2. The Governor himself is not competent


forms of Discrimination against to withdraw the Ordinance at any time.
Women Select the correct answer using the codes
C. Convention on the Rights of Persons given below.
with Disabilities A. 1 only B. 2 only
D. Declaration on the Right to C. Both 1 and 2 D. Neither 1 nor 2
Development
22. Which one among the following States of
19. Consider the following statements about India has the largest number of seats in its
Indo-Tibetan Border Police (ITBP): Legislative Assembly?
1. ITBP was raised in 1962. A. West Bengal
2. ITBP is basically a mountain raised B. Bihar
force.
C. Madhya Pradesh
3. ITBP replaced Assam rifles in Sikkim and
D. Tamil Nadu
Arunachal Pradesh in 2004-05 for
border guarding duty. 23. Which one of the following statements
about Buddhist Stupas in India is not
4. ITBP presently has 52 service correct?
battalions.
A. Ashoka played an important role in
Which of the statements given above are popularizing the Stupa cult.
correct?
B. They were repositories of relics of
A. 1, 2 and 4 only B. 3 and 4 only Buddha and other monks.
C. 1, 2,3 and 4 D. 1, 2 and 3 only C. They were located in rural areas.
20. Which one of the following events is not D. They were located close to trade
correctly matched with the year in which it routes.
happened?
24. Which Buddhist text contains an account of
A. Inauguration of the SWIFT system of the Mauryan Emperor Ashoka?
electronic interbank fund transfer
A. Vinaya Pitaka
worldwide-1985
B. Sutta Pitaka
B. Conclusion of Uruguay Round of GATT-
1994 C. Abhidhama Pitaka
C. Inauguration of the World Trade D. Mahavamsa
Organization-1995
25. The idea of ‘Farr-I Izadi’, on which the
D. Establishment of the first wholly Mughal Kingship was based, was first
electronic stock exchange (Nasdaq) – developed by which one of the following
1971 Sufi saints?

PAGE 803
www.byjusexamprep.com

A. Shihabuddin Suhrawadi Select the correct answer using the code


given below.
B. Nizamuddin Auliya
A. 1, 2 and 3 B. 1 and 2 only
C. Ibn al-Arabi
C. 2 and 3 only D. 1 only
D. Bayazid Bistani
30. eBiz is one of the integrated services
26. Name the platform used for ritual purposes projects and part of the 31 Mission Mode
by the kings of the Vijayanagara Empire. Projects (MMPs) under the National e-
A. Mahanavami Dibba Governance Plan of the Government of
India. eBiz is being implemented under the
B. Lotus Mahal| guidance and aegis of the Department of
C. Hazara Rama Industrial Policy and Promotion, Ministry of
Commerce and Industry by
D. Virupaksha
A. Tata Consultancy Services
27. Who among the following was the founder
B. Infosys Technologies Limited
of Phoenix Settlement?
C. Wipro
A. Mahatma Gandhi
D. HCL Technologies
B. B . R. Ambedkar
31. Growth in production (in percent) of which
C. Rabindranath Tagore one of the following core industries in India
D. Swami Vivekananda during the period 2015-2016 was negative?

28. Consider the following statement: A. Natural gas B. Refinery products

“A sound body means one which bends C. Fertilizer D. Coal


itself to the spirit and is always a ready 32. Consider the following statements about
instrument at its service.” National Wildlife Action Plan (NWAP) of
India for 2017-2031:
The above statement is attributed to
1. This is the Third National Wildlife Action
A. Sardar Patel
Plan.
B. Winston Churchill
2. The NWAP is unique as this is the first
C. Mahatma Gandhi time India has recognized- the concerns
relating to climate change impact on
D. Baden-Powell wildlife.
29. Which of the following statements about 3. The NWAP has ten components.
the olive ridley turtles is/are correct?
Which of the statements given above is/are
1. They are the smallest and most correct?
abundant of all sea turtles found in the
world. A. 1 only B. 1 and 2 only

2. They live in warm waters of Pacific, C. 2 and 3 only D. 1, 2 and 3


Atlantic and Indian Oceans 33. Which one of the following statements in
respect of the States of India is not correct?
3. The Coromandel Coast in India is the
largest mass nesting site for the olive A. States in India cannot have their own
ridley turtles. Constitutions.

PAGE 804
www.byjusexamprep.com

B. The State of Jammu and Kashmir has its B. Flying Officer, Flight Lieutenant,
own Constitution. Squadron Leader, Wing Commander,
Group Captain, Air Commodore, Air
C. States in India do not have the right to
Vice Marshal, Air Marshal, Air Chief
secede from the Union of India.
Marshal
D. The maximum number of members in
C. Flying Officer, Flight Lieutenant,
the Council of Ministers of Delhi can be
Squadron Leader, Group Captain, Wing
15 percent of the total number of
Commander, Air Commodore, Air Vice
members in the Legislative Assembly.
Marshal, Air Marshal, Air Chief Marshal
34. Which one of the following became a part
of China in 1997 following the principle of D. Sub Lieutenant, Lieutenant, Lieutenant
'one country, two systems? Commander, Commander, Captain,
Commodore, Rear Admiral, Vice
A. Tibet B. Hong Kong Admiral, Admiral
C. Xinjiang D. Inner Mongolia 38. Who was/were the 10th century
35. Under which one of the following composer(s) of the Nalayira Divya
Amendment Acts was Sikkim admitted into Prabandham?
the Union of India? A. Alvars B. Nayanars
th th
A. 35 B. 36 C. Appar D. Sarnbandar
th th
C. 37 D. 38 39. What is the name of the literary genre
36. Which of the following statements about developed by the Khojas who are a branch
the Attorney General of India is/are not of the Ismaili sect?
correct? A. Ginan B. Ziyarat
1. He is the first Law Officer of the C. Raag D. Shahada
Government of India.
40. Consider the following statement:
2. He is entitled to the privileges of a
Member of the Parliament. "So much is wrung from the peasants, that
even dry bread is scarcely left to fill their
3. He is a whole-time counsel for the
stomachs."
Government.
Who among the following European
4. He must have the same qualifications
travellers had made the above statement
as are required to be a judge of the
about the condition of peasantry in the
Supreme Court.
Mughal Empire?
Select the correct answer using the code
A. Francisco Pelsaert
given below.
A. 1, 2 and 3 B. 2 and 4 B. Francois Bernier

C. 3 only D. 4 only C. Jean-Baptiste Tavernier

37. Which one of the following is not a correct D. Niccolao Manucci


ascending order of commissioned ranks in 41. Which one of the following statements is
the defence forces of India? not correct?
A. Lieutenant, Captain, Major, Lieutenant A. Al-Biruni identifies the Sufi doctrine of
Colonel, Colonel, Brigadier, Major divine love as self-annihilation with
General, Lieutenant General, General parallel passages from Bhagavad Gita.

PAGE 805
www.byjusexamprep.com

B. According to Al-Biruni, Sufi theories of City in Vietnam. In which one of the


Soul were similar to those in Patanjali's following categories was she declared
Yoga Sutra. winner?
C. The Hatha Yogic Treatise Amrita Kunda A. 48 kg B. 51 kg
had lasting impact on Sufism.
C. 54 kg D. 57 kg
D. Hujwiri's conversation with the Yogis
46. Indian short film ‘The School Bag’, which
shows that he was impressed with their
tells a story based in Pakistan, has won the
theory of the division of the human
Best Short Film Award at the South Asian
body.
Film Festival of Montreal (SAFFM).
42. Which of the following statements about
It is directed by Dheeraj Jindal.
the usage of the term 'barbarian' is/are
correct? Hence, option B is correct.
1. It is derived from the Greek word A. Anurag Kashyap
‘barbaros' which means a non-Greek.
B. Dheeraj Jindal
2. Romans used the term for the
C. Sujoy Ghosh|
Germanic tribes, the Gauls and the
Huns. D. Samvida Nanda
Select the correct answer using the code 47. The Ministry of Power, Government of
given below. India has recently constituted a Committee
to investigate the causes of the accident
A. 1 only B. 2 only
that occurred on 1st November 2017 at
C. Both 1 and 2 D. Neither 1 nor 2 Feroze Gandhi Thermal Power Plant Ltd.,
Unchahar, in Uttar Pradesh. Who among
43. Who among the following is the winner of
the following is the Chairman of the
the National Badminton Championship
Committee?
(Men), 2017?
A. Dr. L. D. Papney
A. Kidambi Srikanth
B. Shri Dhawal Prakash Antapurkar
B. H. S. Prannoy|
C. Shri Subir Chakraborty
C. Ajay Jayararn
D. Shri P. D. Siwal
D. Sai Praneeth
48. Which one of the following is India’s first
44. Which one of the following is the theme of
indigenously designed and developed long-
the World Soil Day, 2017?
range subsonic cruise missile which can be
A. Soils and pulses, a symbol for life deployed from multiple platforms?
B. Caring for the Planet starts from the A. Astra B. Akash
Ground
C. Nirbhay D. Shankhnaad
C. Soils, a solid ground for life
49. Which of the following about the role of
D. Soils, foundation for family farming Indian Coast Guard is/are correct?
45. In November 2017, India's MC Mary Kom 1. Indian Coast Guard has been entrusted
won the gold medal at the Asian Boxing with the offshore security coordination
Championship, 2017 held at Ho Chi Minh authority

PAGE 806
www.byjusexamprep.com

2. Lead intelligence agency for coastal 53. Which One of the following teams was
and sea border defeated by India to win the Women’s
Hockey Asia Cup title, 2017?
3. Coastal security in territorial waters
A. Japan B. China
Select the correct answer using the code
given below. C. South Korea D. Pakistan
A. 1 and 3 only B. 1, 2 and 3 54. Which one of the following temples of India
C. 1 and 2 only D. 3 only has won the ‘UNESCO Asia Pacific Award of
Merit, 2017’ for cultural heritage
50. Which one of the following Amendments to conservation?
the Constitution of India has prescribed
that the Council of Ministers shall not A. Kamakhya Temple, Guwahati
exceed 15 percent of total number for B. Sri Ranganathaswami Temple,
members of the House of the People or Srirangam
Legislative Assembly in the States?
C. Meenakshi Temple, Madurai
A. 91st Amendment
D. Kedarnath Temple, Kedarnath
B. 87th Amendment
C. 97th Amendment 55. The Ministry of Communication has
recently launched a Pan India scholarship
D. 90th Amendment programme for school-children called
51. Which one of the following statements is ‘Deen Dayal SPARSH Yojana’. The objective
not correct? of the scheme is to increase the reach of

A. India joined MTCR in 2016. A. sports B. philately

B. India submitted a formal application for C. music D. web designing


membership of the NSG in 2016. 56. Which one of the following States will host
C. India proposed the Comprehensive the Nobel Prize Series-India 2018 (Science
Convention on International Terrorism Impacts Life) Exhibition?
in 1996.
A. Gujarat B. Rajasthan
D. The Commonwealth Heads of
C. Goa D. Madhya Pradesh
Government Meeting (CHOGM) was
held in 2016 at Malta. 57. Which one of the following was a focus
country of the ‘World Food India’, a mega
52. Which one of the following statements
food event held in November 2017 in New
about India is not correct?
Delhi?
A. India has 12 major ports and about 200
non-major ports. A. Germany B. Japan

B. 95 Percent of India’s trade by volume C. Denmark D. Italy


and 68 Percent by value are moved 58. Which of the following statements about
through maritime transport. the Non-Cooperation Movement is/are
C. India has a coastline or about 7500 km. Correct?

D. In the Maritime Agenda, 2010-2020, a 1. It was marked by significant


target or 300 MT Port capacity has been participation of peasants from
set for the year 2020 Karnataka.

PAGE 807
www.byjusexamprep.com

2. It was marked by non-Brahmin lower Select the correct answer using the code
Caste participation in Madras and given below.
Maharashtra
A. 1 and 2 only B. 1, 2 and 3
3. It was marked by the lack of labour
C. 2 and 3 only D. 3 only
unrest in Places like Assam, Bengal and
Madras. 62. Which one of the following statements with
regard to Antrix Corporation Limited is
4. It was badly shaken by the Chauri
correct?
Chaura incident in 1922 after which
Gandhiji decided to continue with the A. It is a commercial arm of the
movement on a much smaller scale. Department of Industrial Policy and
Promotion.
Select the correct answer using the code
given below. B. It is under the administrative control of
the Department of Space.
A. 1 only B. 1, 2 and 4
C. It is under the administrative control of
C. 2 and 3 D. 2 only
the Ministry of Health and Family
59. In the 10th Mandala of the Rigveda, which Welfare.
one of the following hymns reflects upon
D. It is a commercial arm of the
the marriage ceremonies?
department of Science and Technology.
A. Surya Sukta B. Purusha Sukta
63. According to the Election Commission of
C. Dana Stutis D. Urna Sutra India, in order to be recognized as a
`National Party', a political party must be
60. Around twelfth century Sufi Silsilas began
treated as a recognized political party in
to crystallize in different parts of the Islamic
how many States?
world to signify
A. At least two States
1. continuous link between the master
and disciple B. At least three States
2. unbroken spiritual genealogy to the C. At least four States
Prophet Muhammad
D. At least five States
3. the transmission of spiritual power and
64. A person is disqualified for being chosen as,
blessings to devotees
and for being, a Member of either House of
Select the correct answer using the code the Parliament if the person
given below.
1. holds any office of profit under
A. 1 and 2 only B. 2 only Government of India or the
Government of any State other than an
C. 1 and 3 only D. 1, 2 and 3
office declared by the Parliament by
61. Verses ascribed to poet-saint Kabir have law not to disqualify its holder
been compiled in which of the following
2. is an undischarged insolvent
traditions?
3. is so disqualified under the Tenth
1. Bijak in Varanasi
Schedule of the Constitution of India
2. Kabir Granthavali in Rajasthan
4. is of unsound mind and stands so
3. Adi Granth Sahib declared by a competent Court

PAGE 808
www.byjusexamprep.com

Select the correct answer using the code 68. The Reserve Bank of India has recently
given below. constituted a high-level task force on Public
Credit Registry (PCR) to suggest a road map
A. 1, 2 and 4 only B. 1, 2, 3 and 4
for developing a transparent,
C. 3 and 4 only D. 1, 2 and 3 only comprehensive and near-real-time PCR for
India. The task force is headed by
65. Which of the following are the functions of
the National Human Rights Commission A. Sekar Karnam
(NHRC)?
B. Vishakha Mulye
1. Inquiry at its own initiative on the
violation of human rights C. Sriram Kalyanaraman

2. Inquiry on a petition presented to it by D. Y. M. Deosthalee


a victim 69. Which one of the following is the maximum
3. Visit to jails to study the condition of age of joining National Pension System
the inmates (NPS) under the NPS-Private Sector?
4. Undertaking and promoting research in A. 55 years B. 60 years
the field of human rights
C. 65 years D. 70 years
Select the correct answer using the code
given below. 70. Which one of the following statements
about Exchange-Traded Fund (ETF) is not
A. 1 and 2 only B. 2, 3 and 4 only correct?
C. 1, 3 and 4 only D. 1, 2, 3 and 4 A. It is a marketable security.
66. Which of the following statements about B. It experiences price changes
the India Post Payments Bank (IPPB. is/are throughout the day.
correct?
C. It typically has lower daily liquidity and
1. It has been incorporated as a Public higher fees than mutual fund shares.
Limited Company.
D. An ETF does not have its net asset value
2. It started its operation by establishing
calculated once at the end of every day.
two pilot branches at Hyderabad and
Varanasi. E. None of the above
Select the correct answer using the code 71. Accumulation of which one of the following
given below. in the muscles of sprinters leads to cramp?
A. 1 only B. 2 only A. Lactic acid B. Ethanol
C. Both 1 and 2 D. Neither 1 nor 2 C. Pyruvic acid D. Glucose
67. In October 2017, India sent its first 72. Which of the following classes of animals
shipment of wheat to Afghanistan as a part has/have three-chambered heart?
of commitment made by the Government
of India to supply 1.1 million tonnes of A. Pisces and Amphibia
wheat to that country on grant basis. The B. Amphibia and Reptilia
shipment was sent through
C. Reptilia only
A. Iran B. Pakistan
D. Amphibia only
C. Tajikistan D. China

PAGE 809
www.byjusexamprep.com

73. Which one of the following statements Which one of the following is the correct
about lymph is correct? order of the above rivers from east to west?
A. Lymph is formed due to leakage of A. 3-4-1-2 B. 2-1-3-4
blood through capillaries.
C. 2-3-1-4 D. 1-2-4-3
B. Lymph contains blood cells such as
WBC. 77. Which one of the following crops is not
cultivated in Karewas, the lacustrine
C. Lymph is also circulated by the blood deposits of sand, clay, loam, silt and
circulating heart. boulders?
D. Lymph only transports hormones. A. Saffron B. Almond
74. Match List-I with List-II and select the C. Walnut D. Ling nut
correct answer using the code given below
the Lists: 78. Extrusive volcanoes are not found in which
one of the following mountains?
A. Alaska B. Rocky
C. Andes| D. Himalayas
79. Which one of the following Indian States
has the highest percentage of Scheduled
A. A-4, B-3, C-2, D-1 Tribe population to its total population?
B. A-4, B-2, C-3, D-1 A. Mizoram
C. A-1, B-2, C-3, D-4 B. Nagaland
D. A-1, B-3, C-2, D-4 C. Meghalaya
75. Which of the following National Parks of D. Arunachal Pradesh
India are declared as World Heritage by
UNESCO? 80. Consider the following constituent gases of
the atmosphere:
1. Keoladeo National Park
1. Argon
2. Sundarbans National Park
2. Neon
3. Kaziranga National Park
3. Helium
4. Ranthambore National Park
4. Carbon dioxide
Select the correct answer using the code
given below. Which one of the following is the correct
descending sequence of the above gases, in
A. 1 and 2 only B. 1, 2 and 3 only terms of the volume percentage?
C. 3 and 4 only D. 1, 2, 3 and 4 A. 1-3-2-4 B. 1-4-2-3
76. Consider the following tributaries of river C. 4-2-3-1 D. 2-4-1-3
Ganga:
81. The pressure of a fluid Varies depth h as P =
1. Gandak 2. Kosi| P + rgh, where r is the fluid density. This
3. Ghaghara 4. Gomti expression is associated with

PAGE 810
www.byjusexamprep.com

A. Pascal's law 85. Which one of the following is not an


exclusive right of the concerned coastal
B. Newton's law
nations over Exclusive Economic Zone
C. Bernoulli's principle (EEZ)?
D. Archimedes' principle A. Survey and exploitation of mineral
resources of ocean deposits
82. A wire of copper having length l and area of
cross-section A is taken and a current i is B. Exploitation of marine water energy
flown through it. The power dissipated in and marine organisms including fishing
the wire is P. If we take an aluminium wire
C. Conservation and management of
having same dimensions and pass the same
marine resources
current through it, the power dissipated
will be D. Navigation of ships and laying down
submarine cables
A. P B. < P
86. Match List-I with List-II and select the
C. > P D. 2P
correct answer using the code given below
83. Which of the following statements about the Lists:
the National Water Academy (NWA. is/are
correct?
1. The primary objective of the NWA is to
function as Centre of Excellence in
training for in-service engineers from
Central and State organizations on A. A-3, B-1, C-4, D-2
various aspects of water resource B. A-2, B-1, C-4, D-3
planning.
C. A-2, B-4, C-1, D-3
2. The NWA is located in New Delhi.
D. A-3, B-4, C-1, D-2
Select the correct answer using the code
given below. 87. When the sun is near the horizon during
morning & evening, it appears reddish. This
A. 1 only phenomenon occurs due to which of
B. 2 only following?

C. Both 1 and 2 A. Reflection of light

D. Neither 1 nor 2 B. Refraction of light

84. In which one of the following groups of C. Dispersion of light


States in India is the Integrated Coastal D. Scattering of light
Zone Management (ICZM) Project being
implemented as a pilot investment? 88. Two metallic wires made from copper have
the same length but the radius of wire 1 is
A. Gujarat, Kerala and Goa half of that of wire 2. The resistance of wire
B. Kerala, Karnataka and Andhra Pradesh 1 is R. If both the wires are joined together
in series, the total resistance becomes
C. Gujarat, Odisha and West Bengal
A. 2R B. R/2
D. Maharashtra, Andhra Pradesh and
Tamil Nadu C. 5/4 R D. 3/4 R

PAGE 811
www.byjusexamprep.com

89. Which of the following represents a A. Argon B. Xenon


relation for 'heat lost = heat gained?
C. Neon D. Helium
A. Principle of thermal equilibrium
94. Match List-I with List-II and select the
B. Principle of colours correct answer using the code given below
the Lists:
C. Principle of calorimetry
D. Principle of vaporization List-I List-II
(Hypothesis/ (Propounder)
90. Which one of the following elements will be Theory)
an isobar of calcium if the atomic number
of calcium is 20 and its mass number is 40? A. Planetesimal 1. Kober
hypothesis
A. Element with 20 protons and 18
neutrons B. Thermal 2. Chamberlin
contraction
B. Element with 18 protons and 19 theory
neutrons
C. Geosynclinals 3. Daly
C. Element with 20 protons and 19 Orogen theory
neutrons
D. Hypothesis of 4. Jeffreys
D. Element with 18 protons and 22 sliding
neutrons continent
91. Which of the following substances cause
temporary hardness in water? Codes:
1. Mg(HCO3)2 2. Ca(HCO3)2 A. A-2, B-4, C-1, D-3
3. CaCl2 4. MgSO4 B. A-2, B-1, C-4, D-3
Select the correct answer using the code C. A-3, B-1, C-4, D-2
given below.
D. A-3, B-4, C-1, D-2
A. 3 and 4 B. 2 and 3 95. Which one of the following places of India
C. 1 and 4 D. 1 and 2 experiences highest pressure during
winter?
92. Which one of the following is not a
characteristic of a compound? A. Jaisalmer B. Leh

A. Composition is variable. C. Chennai D. Guwahati

B. All particles of compound are of only 96. Which one of the following is applicable to
one type. collision-coalescence process of
precipitation?
C. Particles of compound have two or
A. Clouds which do not extend beyond the
more elements.
freezing level
D. Both A & B
B. Clouds which extend beyond the
93. Bright light is found to emit from freezing level
photographer's flashgun. This brightness is C. All types of clouds
due to the presence of which one of the
following noble gases? D. Cirrocumulus cloud

PAGE 812
www.byjusexamprep.com

97. Which of the following causes adiabatic Direction: The following items consist of two
temperature changes in atmosphere? statements, Statement I and Statement II.
Examine these two statements carefully and
A. Deflection and advection
select the correct answer using the code given
B. Latent heat of condensation below.
C. Expansion and compression of the air 103. Statement I: Bioaccumulation is a process
of progressive accumulation of heavy
D. Partial absorption of solar radiation by
metals and pesticides in an organism.
the atmosphere
Statement II: Large fishes of the pond are
98. A protein is synthesized in the endoplasmic
found to have higher concentration of
reticulum bound ribosomes and it targets
pesticides than planktons of the same
to the inner thylakoid space of chloroplast.
pond.
How many double-layered membrane
layers it has to pass to reach its destination? A. Both the statements are individually
true, and Statement-II is the correct
A. 2 B. 3
explanation of Statement-I
C. 4 D. 5
B. Both the statements are individually
99. Lysosome is formed from which of the true, but Statement-II is not the correct
following cell organelles? explanation of Statement-I
A. Nucleus C. Statement-I is true but Statement-II is
false
B. Endoplasmic reticulum
D. Statement-I is false but Statement-II is
C. Golgi bodies true
D. Ribosomes 104. Statement I: A person may suffer from
100. Which one of the following pairs about tuberculosis if she/he frequently visits
organ/part that helps in locomotion is not crowded place.
correctly matched? Statement II: Bacteria of tuberculosis
A. Euglena: Flagellum spread through droplets by sneezing or
coughing.
B. Paramecium: Cilia
A. Both the statements are individually
C. Nereis: Pseudopodia true, and Statement-II is the correct
D. Starfish: Tubefeet explanation of Statement-I

101. In which one of the following types of B. Both the statements are individually
connective tissues in animals does fat get true, but Statement-II is not the correct
stored? explanation of Statement-I

A. Adipocyte B. Chondrocyte C. Statement-I is true but Statement-II is


false
C. Osteocyte D. Reticulocyte
D. Statement-I is false but Statement-II is
102. Which one of the following is a true fish as true
per the biological system of classification?
105. Statement I: Tides are the rise and fall of
A. Silverfish B. Jellyfish sea levels caused by the combined effects
C. Cuttlefish D. Flying fish of the gravitational forces exerted by the

PAGE 813
www.byjusexamprep.com

Moon and the Sun and the rotation of the C. Statement-I is true but Statement-II is
Earth. false
Statement II: The Earth rotates from west D. Statement-I is false but Statement-II is
to east. true

A. Both the statements are individually 108. Statement I: Incised meanders are formed
true, and Statement-II is the correct in the mature stage of a river.
explanation of Statement-I Statement II: Incised meanders are
B. Both the statements are individually characterized by rejuvenation and
true, but Statement-II is not the correct upliftment of land.
explanation of Statement-I A. Both the statements are individually
true, and Statement-II is the correct
C. Statement-I is true but Statement-II is
explanation of Statement-I
false
B. Both the statements are individually
D. Statement-I is false but Statement-II is true, but Statement-II is not the correct
true explanation of Statement-I
106. Statement I: In Tundra climate, biodiversity C. Statement-I is true but Statement-II is
is comparatively less. false
Statement II: Tundra climate has less D. Statement-I is false but Statement-II is
reproductive warm period. true
A. Both the statements are individually 109. Statement I: By far the most common
true, and Statement-II is the correct topographic form in a Karst terrain is the
explanation of Statement-I sinkhole.

B. Both the statements are individually Statement II: Topographically, a sinkhole is


true, but Statement-II is not the correct depression that varies in depth from less
explanation of Statement-I than a meter to few hundred meters.
A. Both the statements are individually
C. Statement-I is true but Statement-II is
true, and Statement-II is the correct
false
explanation of Statement-I
D. Statement-I is false but Statement-II is
B. Both the statements are individually
true
true, but Statement-II is not the correct
107. Statement I: Portions of glacial troughs may explanation of Statement-I
exhibit remarkably flat floors. C. Statement-I is true but Statement-II is
Statement II: The flat floor in a glacial false
trough is produced by uniform glacial D. Statement-I is false but Statement-II is
erosion. true
A. Both the statements are individually 110. Working of safety fuses depends upon
true, and Statement-II is the correct
1. Magnetic effect of the current
explanation of Statement-I
2. Chemical effect of the current
B. Both the statements are individually
true, but Statement-II is not the correct 3. The magnitude of the current
explanation of Statement-I 4. Heating effect of the current

PAGE 814
www.byjusexamprep.com

Select the correct answer using the code 115. Why is argon gas used along with tungsten
given below. wire in an electric bulb?
A. 1, 2, 3 and 4 A. To increase the life of the bulb
B. 1, 2 and 3 only B. To reduce the consumption of
C. 3 and 4 only electricity

D. 4 only C. To make the emitted light coloured

111. Consider the following statements. D. To reduce the cost of the bulb

1. There is no net moment on a body 116. Consider the following chemical reaction:
which is in equilibrium aFe2O3(s) + bCO(g) → cFe(s) + dCO2
2. The momentum of a body is always In the balanced chemical equation of the
conserved above, which of the following will be the
3. The kinetic energy of an object is values of the coefficients a, b, c and d
always conserved. respectively?

Which of the statements given above is/are A. 3, 2, 3, 1 B. 1, 3, 2, 3


correct?
C. 2, 3, 3, 1 D. 3, 3, 2, 1
A. 1, 2 and 3 B. 2 and 3 only
117. Which one of the following elements is
C. 1 and 2 only D. 1 only essential for the formation of chlorophyll in
green plants?
112. Which one of the following is the value of
one nanometer? A. Calcium B. Iron
A. 10-7 cm B. 10-6 cm C. Magnesium D. Potassium
C. 10-4 cm D. 10-3 cm 118. Which one of the following gases dissolve in
113. Sound waves cannot travel through a water to give the acidic solution?

A. Copper wire placed in air A. Carbon dioxide B. Oxygen

B. Silver slab placed in air C. Nitrogen D. Hydrogen

C. Glass prism placed in water 119. Which one of the following elements is
involved in the water control of the blood?
D. Wooden hollow pipe loaded in a
vacuum A. Potassium B. Lithium
114. Which one of the following is the correct C. Rubidium D. Caesium
relation between the Kelvin temperature
120. Which one of the following elements is
(T) and the Celsius temperature (tc)?
used as a time keeper in atomic clocks?
A. There are two independent
A. Potassium
temperature scales
B. Caesium
B. T = tc
C. T = tc – 273.15 C. Calcium

D. T = tc + 273.15 D. Magnesium

PAGE 815
www.byjusexamprep.com

Elementary Mathematics
1. If 2b = a + c and , then what is A.
equal to?
B.
A. 3 B. 2
C. 24xyz
C. 1 D. –1
D. -24xyz
2. The length of a line segment AB is 2 cm. It is
divided into two parts at a point C such that 7. If , then what is the value of
AC2 = AB × CB. What is the length of CB?
?
A. B.
A. 125 B. –125
C. D.
C. 140 D. –140
3. The curved surface area of a right circular
cone is 1-76 m2 and its base diameter is 140 8. Which one of the following is a zero of the
cm. What is the height of the cone? polynomial ?

A. 10 cm B. A. 0 B. 1
C. 2 D. -1
C. D.
4. If the length of a side of a square is 9. The remainder when is
increased by 8 cm, its area increases by 120 divided by (x + 1) is –7. What is the value of
square cm. What is the length of a side of k?
the square? A. –14 B. 14
A. 2.5 cm B. 3.5 cm C. –7 D. 7
C. 4.5 cm D. 5.5 cm 10. If f(x) and g(x) are polynomials of degree p
5. Which one of the following is correct? and q respectively, then the degree

A. Decimal expansion of a rational {f(x) ± g(x)} (if it is non-zero) is


number is terminating. A. Greater than min(p, q)
B. Decimal expansion of a rational B. Greater than max(p, q)
number is non-terminating.
C. Less than or equal to max(p, q)
C. Decimal expansion of an irrational
number is terminating. D. Equal to min(p, q)

D. Decimal expansion of an irrational 11. What is the value of ?


number is non-terminating and non-
repeating.
A. B.
6. If then what is
C. D.
equal to?

PAGE 816
www.byjusexamprep.com

12. What is equal to for any 18. ABC is a right angled triangle with base BC
and height AB. The hypotenuse AC is four
real number θ?
times the length of the perpendicular
A. 1 drawn to it from the opposite vertex. What
is tan C equal to?
B.
A. B.
C.
C. D.
D. 19. ABC is a triangle right angled at C with
13. In the following table of inverse variation, BC = a and AC = b. If p is the length of the
what are the values of A, B and C perpendicular from C on AB, then which
respectively? one of the following is correct?

A.

B.
A. 10, -30, -1 B. 10, -1, 30
C.
C. -30, 10, -1 D. -1, -30, 10
D.
14. What is the value of
20. Two equal circular regions of greatest
possible area are cut off from a given
Where circular sheet of area A. What is the
remaining area of the sheet?
x ≠ 0? A. A/2 B. A/3
A. –1 B. 0 C. 3A/5 D. 2A/5
C. 1 D. 3 21. A right circular cone is sliced into a smaller
cone and a frustum of a cone by a plane
15. What is cot 1o cot 23 ° cot 45o cot 67 ° cot perpendicular to its axis. The volume of the
89 ° equal to? smaller cone and the frustum of the cone
A. 0 B. 1 are in the ratio 64 : 61. Then their curved
surface areas are in the ratio
C. D. A. 4 :1 B. 16 : 9
C. 64 : 61 D. 81: 64
16. If then what is
22. In a room whose floor is a square of side 10
equal to?
m, an equilateral triangular table of side 2
A. 2 B. m is placed. Four book-shelves of size 4 m ×
1 m × 9 m are also placed in the room. If half
C. D. of the rest of the area in the room is to be
carpeted at the rate of ₹ 100 per square
17. If then what is
metre, what is the cost of carpeting
equal to? (approximately)?
A. 1 B. 1.5 A. ₹ 7,600 B. ₹ 5,635
C. 2 D. 3 C. ₹ 4,113 D. ₹ 3,200

PAGE 817
www.byjusexamprep.com

23. A region of area A bounded by a circle C is 26. The locus of a point equidistant from two
divided into n regions, each of area A/n, by intersecting lines is
drawing circles of radii such
A. A straight line
that concentric with
B. A circle
the circle C. If where m = 1, 2, 3, C. A pair of straight lines
... (n - 2), then which one of the following is D. None of the above
correct?
27. Consider the following statements :
A. p increases as m increases
Two triangles are said to be congruent, if
B. p decreases as m increases
1. Three angles of one triangle are equal
C. p remains constant as m increases to the corresponding three angles of
D. p increases for some values of m as m the other triangle.
increases and then decreases 2. Three sides of one triangle are equal to
thereafter the corresponding three sides of the
24. The locus of the mid-points of the radii of other triangle,
length 16 cm of a circle is 3. Two sides and the included angle of
A. A concentric circle of radius 8 cm one triangle are equal to the
corresponding two sides and the
B. A concentric circle of radius 16 cm included angle of the other triangle.
C. The diameter of the circle 4. Two angles and the included side of
one triangle are equal to the
D. A straight line passing through the
corresponding two angles and the
centre of the circle
included side of the other triangle.
25. Consider the following statements :
Which of the above statements are
1. The orthocentre of a triangle always correct?
lies inside the triangle.
A. 1, 2 and 3
2. The centroid of a triangle always lies
B. 1, 3 and 4
inside the triangle.
C. 1, 2 and 4
3. The orthocentre of a right angled
triangle lies on the triangle. D. 2, 3 and 4
4. The centroid of a right angled triangle 28. Walls (excluding their roofs and floors) of 5
lies on the triangle. identical rooms having length, breadth and
Which of the above statements are height 6 m, 4 m and 2.5 m respectively are
correct? to be painted. Paints are available only in
cans of 1 L and one litre of paint can be used
A. 1 and 2 for painting 20 square metres. What is the
number of cans required for painting?
B. 1 and 4
A. 10 B. 12
C. 2 and 3
C. 13 D. 14
D. 2 and 4

PAGE 818
www.byjusexamprep.com

29. A rectangular pathway having width 4.5 m 34. The surface area of a closed cylindrical box
and length 10 m will have to be tiled using is 352 square cm. If its height is 10 cm, then
square tiles of side 50 cm. Each packet of what is its diameter?
such tiles contains 20 pieces and costs
₹ 100. What will be the total cost of tiles for
the pathway?
A. 4 cm B. 8 cm
A. ₹ 1,200 B. ₹ 1,100
C. 9.12 cm D. 19.26 cm
C. ₹ 1,000 D. ₹ 900
35. A square and an equilateral triangle have
30. A wire is in the form of a circle of radius 98 the same perimeter. If the diagonal of the
cm. A square is formed out of the wire.
square is cm, then what is the area of
What is the length of a side of the square?
the triangle?
(Use π = 22/7)
A. 146 cm B. 152 cm A. B.

C. 154 cm D. 156 cm C. D.

Direction: In a triangle ABC, a, b and c are the 36. In the figure given below, XA and XB are two
lengths of the sides and p, q and r are the lengths tangents to a circle, If and AC
of its medians. is parallel to XB, then what is equal
to?
31. Which one of the following is correct?

A.

B.

C.

D.
A. 70o B. 65o
32. Which one of the following is correct? C. 60o D. 55o
A. 37. In the equilateral triangle ABC given below,
AD = DB and AE = EC. If I is the length of a
B. side of the triangle, then what is the area of
the shaded region?
C.

D.

33. The product of the lengths of the diagonals


of a square is 50 square units. What is the
length of a side of the square?

A. units
B. 5 units A. B.
C. 10 units
C. D.
D. units

PAGE 819
www.byjusexamprep.com

38. SPT is a tangent to the circle at P and O is 44. If (x + 3) is a factor of ,


the centre of the circle. If and then what is the value of k?
PQ is a chord then what is equal to? A. 12 B. 24
A. α B. 2α C. 36 D. 72
C. 90° – α D. 180° – 2α 45. The smallest integer with 4 digits which is a
perfect square is
39. In the figure given below, two equal chords
cut at point P. If AB = CD = 10 cm, A. 1000 B. 1024
OC = 13 cm (O is the centre of the circle) and
C. 1089 D. None of these
PB = 3 cm, then what is the length of OP?
46. If α and β are the roots of the equation
, then what is the value of
the expression ?

A. B.

C. D.
A. 5 cm B. 6 cm

C. D. 47. What is the difference between and

40. If a + b = 2c, then what is the value of 0.9?


A. 0 B. 0.099..
?
C. 0.1 D. 0.09
A. -1 B. 0
48. Which one of the following decimal
C. 1 D. 2 numbers is a rational number with
denominator 37?
41. If the roots of the equation
are reciprocal to each other, then which A. 0.459459459 . . .
one of the following is correct? B. 4.59459459.......
A. P = 2r B. P = r C. 0.0459459459 . . .
C. 2p = r D. P = 4r D. 0.00459459 . . .
42. If and , 49. Three numbers which are co-prime to each
then what is xy equal to? other, are such that the product of the first
two is 286 and that of the last two is 770.
A. 2 B. 3 What is the sum of the three numbers?
C. –2 D. –3 A. 85 B. 80

43. If and , then what is C. 75 D. 70


50. The age of a woman is a two-digit integer.
equal to? On reversing this integer, the new integer is
the age of her husband who is elder to her.
A. 1 B. 2 The difference between their ages is one-
eleventh of their sum. What is the
C. 0 D. –1 difference between their ages?

PAGE 820
www.byjusexamprep.com

A. 8 years B. 9 years 56. There are as many square centimetres in


the surface area of a sphere as there are
C. 10 years D. 11 years
cubic centimetres in its volume. What is the
51. The arithmetic mean of 11 observations is radius of the sphere?
11. The arithmetic mean of the first 6
A. 4 cm B. 3 cm
observations is 10.5 and the arithmetic
mean of the last 6 observations is 11.5. C. 2 cm D. 1 cm
What is the sixth observation? 57. If the ratio of the circumference of the base
A. 10.0 B. 10.5 of a right circular cone of radius r to its
height is 3 : 1, then what is the area of the
C. 11.0 D. 11.5
curved surface of the cone?
52. Consider the following statements :
A. B.
1.

2. C. D.

Which of the above is/are correct? 58. What is the area of the largest circular disc

A. 1 only cut from a square of side units?

B. 2 only A. π square units


C. Both 1 and 2 B. 1 square units
D. Neither 1 nor 2 C. π2 square units
53. The radius and slant height of a right D. 2 square units
circular cone are 5 cm and 13 cm
59. What is the area of the region bounded
respectively. What is the volume of the
internally by a square of side of length ‘a’
cone?
and externally by a circle passing through
A. B. the four comers of the square?

C. D. A. square units.
54. If the ratio of the radius of the base of a
right circular cone to its slant height is 1 : 3, B. square units.
what is the ratio of the total surface area to
the curved surface area? C. square units.
A. 5 : 3 B. 3 : 1
D. square units.
C. 4 : 1 D. 4 : 3
55. What is the volume of a cone of maximum 60. is divisible by
volume cut out from a cube of edge 2a such
A. 7 B. 9
that their bases are on the same plane?
C. 11 D. 13
A. B.
61. If and where
, then what is the
C. D.
value of abc?

PAGE 821
www.byjusexamprep.com

A. –1 B. 1 68. At present the average of the ages of a


father and a son is 25 years. After seven
C. 0 D. 3
years the son will be 17 years old. What will
62. There are two numbers which are greater be the age of the father after 10 years?
than 21 and their LCM and HCF are 3003
and 21 respectively. What is the sum of A. 44 years B. 45 years
these numbers? C. 50 years D. 52 years
A. 504 B. 508 69. A merchant commences with a certain
C. 514 D. 528 capital and gains annually at the rate of
25%. At the end of 3 years he has ₹ 10,000.
63. of the students who registered did not What is the original amount that the
merchant invested?
appear for the examination, of those A. ₹ 5,120 B. ₹ 5,210
who appeared passed. If the number of C. ₹ 5,350 D. ₹ 5,500
registered students is 2000, the number
who passed is 70. For 0 < m < 1, which one of the following is
correct?
A. 1920 B. 1056
C. 1020 D. 864 A.

64. If A : B = 1 : 2, B : C = 3 : 4, C : D = 2 : 3 and B.
D : E = 3 : 4, then what is B : E equal to?
A. 3 : 2 B. 1 : 8 C.
C. 3 : 8 D. 4 : 1
D.
65. A work when done by 10 women is
completed in 12 days. The same work can 71. All odd prime numbers upto 110 are
be completed in 8 days when done by 5 multiplied together. What is the unit digit in
men. How many days will it take to this product?
complete when 6 women and 3 men are A. 0 B. 3
employed to perform the same job?
C. 5 D. None of these
A. 12 B. 10
72. A quadratic polynomial ax2 + bx + c is such
C. 8 D. 5
that when it is divided by x, (x - 1) and (x +
66. A train moving with a speed of 60 km per 1), the remainders are 3, 6 and 4
hour crosses an electric pole in 30 seconds. respectively. What is the value of (a + b)?
What is the length of the train in metres?
A. 3 B. 2
A. 300 B. 400
C. 1 D. –1
C. 500 D. 600
73. If the average of 9 consecutive positive
67. ₹ 120 is distributed among A, B and C so
integers is 55, then what is the largest
that A’s share is ₹ 20 more than B’s and 20
integer?
less than C’s. What is B’s share?
A. 57 B. 58
A. ₹ 10 B. ₹ 15
C. ₹ 20 D. ₹ 25 C. 59 D. 60

PAGE 822
www.byjusexamprep.com

74. A man can row at a speed of x km/hr in still standing at the centre of the square. If l is
water. If in a stream which is flowing at a the length of each side of the square, then
speed of y km/hr it takes him z hours to row what is h2 equal to?
to a place and back, then what is the
distance between the two places? A. B.

A. B.
C. D.

C. D. 79. From a height of h units, a man observes


the angle of elevation as α and angle of
75. A car has an average speed of 60 km per depression as p of the top and the bottom
hour while going from Delhi to Agra and has respectively of a tower of height H (> 4h).
an average speed of y km per hour while To what further height should he climb so
returning to Delhi from Agra (by travelling that the values of angle of elevation and
the same distance). If the average speed of angle of depression get interchanged for
the car for the whole journey is 48 km per the top and bottom of the tower?
hour, then what is the value of y? A. H – h units B. H – 2h units
A. 30 km per hour C. H – 3h units D. H – 4h units
B. 35 km per hour 80. If sin A + cos A = p and sin3A + cos3A = q,
C. 40 km per hour then which one of the following is correct?
D. 45 km per hour A.
76. A, B, C, D and E start a partnership firm. B.
Capital contributed by A is three times that
contributed by D. E contributes half of C.
A’s contribution, B contributes one-third of
D.
E’s contribution and C contributes two-
third of A’s contribution. If the difference
between the combined shares of A, D and E 81. If then which one of the
and the combined shares of B and C in the following is correct?
total profit of the firm is ₹ 13,500, what is
the combined share of B, C and E? (The A. B.
shares are supposed to be proportional to
the contributions) C. D.
A. ₹ 13,500 B. ₹ 18,000
82. What is the largest power of 10 that divides
C. ₹ 19,750 D. ₹ 20,250
the product 1 x 2 x 3 x 4 . . . x 23 x 24 x 25?
77. The arithmetic mean of two numbers is
A. 2 B. 4
10 and their geometric mean is 8. What are
the two numbers? C. 5 D. None of these
A. 15,5 B. 12,8 83. The sum of a number and its square is 20.
C. 16,4 D. 18,2 Then the number is

78. Each side of a square subtends an angle of A. –5 or 4 B. 2 or 3


60° at the tip of a tower of height h metres C. –5 only D. 5 or –4

PAGE 823
www.byjusexamprep.com

84. A cube of maximum volume (each corner 90. A gentleman left a sum of 39,000 to be
touching the surface from inside) is cut distributed after his death among his
from a sphere. What is the ratio of the widow, five sons and four daughters. If each
volume of the cube to that of the sphere? son receives 3 times as much as a daughter
receives, and each daughter receives twice
A. B. as much as their mother receives, then
what is the widow’s share?
C. D.
A. ₹ 1,000 B. ₹ 1,200
85. If the price of wheat rises by 25%, then by
how much percent must a man reduce his C. ₹ 1,500 D. None of these
consumption in order to keep his budget 91. A passenger train and a goods train are
the same as before? running in the same direction on parallel
A. 15% B. 20% railway tracks. If the passenger train now
takes three times as long to pass the goods
C. 25% D. 30% train, as when they are running in opposite
directions, then what is the ratio of the
86. A man undertakes to do a certain work in
speed of the passenger train to that of the
150 days. He employs 200 men. He finds
goods train?
that only a quarter of the work is done in 50
days. How many additional men should he (Assume that the trains nm at uniform
employ so that the whole work is finished speeds)
in time? A. 2 : 1 B. 3 : 2
A. 75 B. 85 C. 4 : 3 D. 1 : 1
C. 100 D. 120 92. The average of the ages of 15 students in a
class is 19 years. When 5 new students are
87. A person borrowed ₹ 5,000 at 5% rate of
admitted to the class, the average age of
interest per annum and immediately lent it
the class becomes 18.5 years. What is the
at 5.5%. After two years he collected the
average age of the 5 newly admitted
amount and settled his loan. What is the
students?
amount gained by him in this transaction?
A. 17 years B. 17.5 years
A. ₹ 25 B. ₹ 50
C. 18 years D. 18.5 years
C. ₹ 100 D. ₹ 200
93. A water tank has been fitted with two taps
88. If 5 tractors can plough 5 hectares of land in P and Q and a drain pipe R. Taps P and Q fill
5 days, then what is the number of tractors at the rate of 12 litres per minute and 10
required to plough 100 hectares in 50 days? litres per minute respectively.
A. 100 B. 20 Consider the following statements SI, S2
and S3 :
C. 10 D. 5
S1 : Pipe R drains out at the rate of 6 litres
89. The annual income of a person decreases per minute.
by ₹ 64 if the annual rate of interest
decreases from 4% to 3.75%. What is his S2 : If both the taps and the drain pipe are
original annual income? opened simultaneously, then the tank is
filled in 5 hours 45 minutes.
A. ₹ 24,000 B. ₹ 25,000
S3 : Pipe R drains out (fully) the filled tank
C. ₹ 25,600 D. ₹ 24,600 in 15 hours 20 minutes.

PAGE 824
www.byjusexamprep.com

To know what is the capacity of the tank, Statement-2: The sum of the angles of a
which one of the following is correct? polygon having n sides is (3n - 8) right
angles.
A. S2 is only sufficient
Which one of the following is correct in
B. S1, S2 and S3 are necessary
respect of the above statements?
C. Any two out of SI, S2 and S3 are
A. Both Statement-1 and Statement-2 are
sufficient
true and Statement-2 is the correct
D. None of the above explanation of Statement-1
94. An article is sold at a profit of 32%. If the B. Both Statement-1 and Statement-2 are
cost price is increased by 20% and the sale true but Statement-2 is not the correct
price remains the same, then the profit explanation of Statement-1
percentage becomes
C. Statement-1 is true but Statement-2 is
A. 10% B. 12% false
C. 15% D. 20% D. Statement-1 is false but Statement-2 is
true
95. What angle does the hour hand, of a clock
describe in 10 minutes of time? 99. In the figure given below, p, q, r are parallel
lines; I and m are two transversals.
A. 1o B. 5o
C. 6o D. 10o
96. The radii of two circles are 4.5 cm and 3.5
cm respectively. The distance between the
centres of the circles is 10 cm. What is the
length of the transverse common tangent?
A. 4 cm B. 5 cm
C. 6 cm D. 7 cm 1. AB : AC = DE : DF

97. A Pie Chart is drawn for the following data : 2. AB × EF = BC × DE


Which of the above is/are correct?
A. 1 only B. 2 only
C. Both 1 and 2 D. Neither 1 nor 2
100. An alloy A contains two elements, copper
and tin in the ratio of 2 : 3, whereas an alloy
What is the angle (approximately)
B contains the same elements in the ratio
subtended by the Social Services Sector at
of 3 : 4. If 20 kg of alloy A, 28 kg of alloy B
the centre of the circle?
and some more pure copper are mixed to
A. 45o B. 46o form a third alloy C which now contains
copper and tin in the ratio of 6 ; 7, then
C. 58o D. 98o
what is the quantity of pure copper mixed
98. Given that the angles of a polygon are all in the alloy C?
equal and each angle is a right angle.
A. 3 kg B. 4 kg
Statement-1 : The polygon has exactly four
C. 5 kg D. 7 kg
sides.

PAGE 825
www.byjusexamprep.com

ANSWERS KEY I 2018


ENGLISH

1 D 21 B 41 C 61 B 81 D 101 A
2 B 22 C 42 C 62 B 82 C 102 B
3 B 23 B 43 D 63 B 83 D 103 D
4 A 24 A 44 B 64 B 84 C 104 A
5 A 25 A 45 B 65 A 85 A 105 D
6 B 26 B 46 B 66 A 86 C 106 C
7 C 27 C 47 C 67 B 87 B 107 D
8 A 28 C 48 B 68 C 88 C 108 C
9 A 29 C 49 B 69 D 89 B 109 A
10 A 30 B 50 C 70 D 90 C 110 C
11 B 31 D 51 C 71 B 91 C 111 C
12 B 32 C 52 A 72 D 92 D 112 D
13 A 33 B 53 B 73 A 93 C 113 D
14 C 34 B 54 D 74 B 94 D 114 C
15 C 35 A 55 C 75 A 95 D 115 A
16 C 36 B 56 B 76 B 96 D 116 C
17 A 37 B 57 D 77 B 97 C 117 D
18 B 38 B 58 D 78 D 98 B 118 A
19 C 39 B 59 D 79 A 99 A 119 D
20 A 40 A 60 B 80 C 100 B 120 B

GENERAL KNOWLEDGE
1 B 21 D 41 D 61 B 81 A 101 A
2 C 22 A 42 C 62 B 82 C 102 D
3 D 23 C 43 B 63 C 83 A 103 B
4 B 24 D 44 B 64 B 84 C 104 A
5 B 25 A 45 A 65 D 85 D 105 B
6 C 26 A 46 B 66 A 86 C 106 B
7 D 27 A 47 D 67 A 87 D 107 A
8 B 28 C 48 C 68 D 88 C 108 D
9 A 29 B 49 B 69 C 89 C 109 B
10 B 30 B 50 A 70 C 90 D 110 C
11 C 31 A 51 D 71 A 91 D 111 C
12 A 32 B 52 D 72 B 92 B 112 A

PAGE 826
www.byjusexamprep.com

13 B 33 D 53 B 73 C 93 B 113 D
14 A 34 B 54 B 74 A 94 A 114 D
15 A 35 A 55 B 75 B 95 B 115 A
16 D 36 C 56 C 76 B 96 A 116 B
17 C 37 C 57 D 77 D 97 C 117 C
18 A 38 A 58 C 78 D 98 B 118 A
19 D 39 A 59 B 79 A 99 C 119 A
20 A 40 A 60 D 80 B 100 C 120 B

Elementary Mathematics
1 C 21 B 41 B 61 B 81 D
2 B 22 C 42 A 62 A 82 D
3 D 23 B 43 A 63 B 83 A
4 B 24 A 44 A 64 C 84 C
5 D 25 C 45 B 65 C 85 B
6 D 26 C 46 C 66 C 86 C
7 C 27 D 47 C 67 C 87 B
8 B 28 C 48 A 68 C 88 C
9 D 29 D 49 D 69 A 89 C
10 C 30 C 50 B 70 A 90 A
11 A 31 C 51 C 71 C 91 A
12 D 32 B 52 B 72 A 92 A
13 A 33 B 53 A 73 C 93 C
14 C 34 B 54 D 74 B 94 A
15 B 35 D 55 C 75 C 95 B
16 A 36 B 56 B 76 B 96 C
17 A 37 A 57 D 77 C 97 C
18 A 38 B 58 B 78 B 98 C
19 A 39 D 59 D 79 B 99 C
20 A 40 C 60 D 80 C 100 B

PAGE 827
CDS I 2018
www.byjusexamprep.com

(Solutions)
ENGLISH
1. Ans. D.
The idiom “costs an arm and a leg” is used to refer to something that is extremely expensive. Thus,
option D is the correct answer.
2. Ans. B.
The idiom “best of both worlds” means a win-win situation or a situation where one can enjoy the
benefit of two different opportunities. Thus, option B is the correct answer.
3. Ans. B.
The idiom “ball is in your court” is used to say that it is now someone else's move, play, or turn, so it
is up to them to make the decision. Thus, option B is the correct answer.
4. Ans. A.
The idiom “At the drop of a hat” means without hesitation. Thus, option A is the correct answer.
5. Ans. A.
The idiom “You scratch my back. I’ll scratch yours” is used to tell someone that if they help you, you
will help them. So, with the usage of the phrase, a mutual favor is agreed upon.
Hence, option A is the correct answer.
6. Ans. B.
The idiom “Holy Grail” means something that you want very much but that is very hard to get or
achieve. Such things can be referred to as an important object or goal.
Hence, option B is the correct answer.
7. Ans. C.
The idiom “step up the plate” means to take responsibility for doing something, even though it is
difficult.
Hence, option C is the correct answer.
8. Ans. A.
The phrase “raising eyebrows” means to cause other people to react with surprise or mild
disapproval. Thus, option A is the correct answer.

PAGE 828
www.byjusexamprep.com

9. Ans. A.
The phrase “down the road” is used for talking about the future and what may happen.
Hence, option A is the correct answer.
10. Ans. A.
The phrase “what a small world” is used when a person knows someone you know by chance; or
when you meet someone unexpectedly. Both of these show coincidence or chance.
Hence, option A is the correct answer.
11. Ans. B.
The idiom “out of blue” means without warning or unexpectedly. Thus, option B is the correct
answer.
12. Ans. B.
The idiom “my two cents” means to give or share your opinion.
Thus, option B is the correct answer.
13. Ans. A.
“To not buy it” is an idiom used to convey that you do not believe in what has been said, i.e you are
not convinced.
Hence, option A is the correct answer.
14. Ans. C.
“You snooze, you lose” is an idiom used to convey that if you do not pay attention and do something
quickly, someone else will do it instead of you.
Hence, option C - Don't hesitate to do it is the correct answer.
15. Ans. C.
The idiom “a hot potato” means a controversial issue or situation which is awkward to deal with.
Option C conveys the same meaning as a ‘disputed issue’ means a controversial issue.
16. Ans. C.
Since the enemies are being talked about, the control exercised by their chief would be tough. Hence,
“rigid” is the most appropriate word in the given context.
17. Ans. A.
With independence, one will try to regain their old honour or prestige that had been lost owing to
the defeat. Thus, option A is the most appropriate answer.
18. Ans. B.
A ‘desperate attempt’ refers to an act of trying in despair or when everything else has failed. This
term fits most appropriately in the given context. Thus, option B is the correct answer.

PAGE 829
www.byjusexamprep.com

19. Ans. C.
“Rankled” means “annoyed” or “upset”. So, since the Marathas were defeated in the war, their “loss”
of freedom must have upset them. Hence, option C is the most appropriate answer.
20. Ans. A.
A defeat would primarily hamper the power of the concerned party. Options B and C are irrelevant
here. Option D is incorrect as the first line states that the spirits of the Marathas had not shattered,
so they must have not lost patience. The correct answer is option A.
21. Ans. B.
Option D is out of context, hence discarded. Options A and C are singular nouns, hence cannot signify
the numerous people who were listening to M.S. “Audience” is the best response as it is a collective
noun for all the spectators. Thus, option B is the correct answer.
22. Ans. C.
Since a singing performance is being talked about, “hours” will be the best unit to describe the stretch
of the event. Thus, option C is the correct answer.
23. Ans. B.
Since no memento or title were given, “awards”, “prizes” and “recognitions” are inappropriate in the
given context. The fact that people belonging to different countries listened to her attentively, is no
less than an honour. Thus, option B is the correct answer.
24. Ans. A.
With reference to listening to a famous singer, “attentively” is the best adverb. Thus, option A is the
correct answer. Option C conveys a negative meaning.
25. Ans. A.
Since it has already been stated that people hailing from countries around the world appreciated
M.S.’s singing skills, “many” will be the best determiner in the blank space. B and C carry negative
tones and “all” is inappropriate. Thus, option A is the correct answer.
26. Ans. B.
Since the profession of M.S. is to sing, she would be invited for the same. Thus, option B is the correct
answer.
27. Ans. C.
Since a specific time (October 1966) is stated after the blank “in” is the most appropriate preposition
to be used. Thus, option C is the correct answer.
28. Ans. C.
The context of the concerned blank states that M.S.’s singing has not just brought joy to the people
of India but also in various countries of the world. Thus, “other” is the most appropriate word for the
blank. So, option C is the answer.

PAGE 830
www.byjusexamprep.com

29. Ans. C.
The most appropriate preposition here would be “in” as the joy has been brought to people within
the country. Thus, option C is the correct answer.
30. Ans. B.
Since the passage opens by stating that M.S. Subbulakshmi is one of India’s greatest musicians, the
people will definitely not be bored or pained by her singing. Her singing will bring joy to them.
Thus, option B is the correct answer.
31. Ans. D.
The given sentence is correct as it is.
32. Ans. C.
The error is in the third part,
Are should be replaced by is since the subject is ‘each and everything’. Everything is followed by a
singular verb.
33. Ans. B.
The error is in the second part.
He was held, here, indicates that a person was found guilty.
When something happens as a result of it being aligned with an act/rule, it happens ‘under’ the Act.
Hence, in should be replaced by under.
34. Ans. B.
The error lies in the second part.
Ousted means drive out or expel (someone) from a position or place. It is always followed by ‘from’.
35. Ans. A.
The error is in the first part.
The use of for is redundant. Gandhiji didn’t regret for the fact but the fact itself.
36. Ans. B.
The error lies in part (B) of the sentence. In the given sentence, the words "twice" and "more" both
are acting as adverbs. Grammatically we cannot place two adverbs close together without placing
any other entity in between them. So, it is incorrect to write "twice more" in the sentence and
therefore, it should be written either "twice as much as" or "two times more than."
37. Ans. B.
The error lies in the second part.
Form should be replaced by formation since a process is being talked about.
38. Ans. B.
The error is in the second part.

PAGE 831
www.byjusexamprep.com

The given sentence means that since prism is used to refract white light, prism spreads out in a
spectrum which is not the case. That should be used between so and it to indicate that it is the light
that spreads out and a prism is used to refract it in order to spread the light in a continuous spectrum.
39. Ans. B.
The error is in the second part.
Leave doesn’t make sense, it should be replaced by ‘let’. The segment means that the nurses don’t
let someone donate blood if they have the minutest of deviation from the healthy status.
40. Ans. A.
The error is in the first part.
Have should be replaced by has since the subject is singular.
41. Ans. C.
The error is in the third part of the sentence.
When the first segment of the sentence is in simple past tense, the second should be in past perfect
tense.
42. Ans. C.
The error is in the third part.
To should be replaced by into in order to express the conversion of fruit to jam.
43. Ans. D.
The given sentence is correct as it is.
Difference between agree with, agree to and agree on.
Agree with someone/something – accept the point of someone/something.
Agree on some issue.
Agree to demands/ queries in order to do them.
Hence, option D is the correct answer.
44. Ans. B.
The error is in the second part of the sentence.
Anxious is followed by about when the reason for being anxious follows. Also, have to sing will be
changed to having to sing in order to make the sentence grammatically correct.
45. Ans. B.
The error lies in the second part of the sentence.
There are two errors in this segment.
For should be replaced by to since one is answerable for something but to someone.
For the same reason, with should be replaced by for.

PAGE 832
www.byjusexamprep.com

46. Ans. B.
The error is in the second part.
‘Were’ should be replaced by is since it is a general statement with no contextual reference of the
past. Moreover, when such quantities (proportions) are stated, they are followed by a singular verb.
47. Ans. C.
The error is in part C of the given sentence.
Hence, option C is the correct answer.
48. Ans. B.
The error lies in the Part B of the given sentence. Remove 'throughout' from the sentence.
49. Ans. B.
The error is in part B of the given sentence. Replace 'who' with 'whom'.
50. Ans. C.
The error lies in the third segment. Hence, option C is the correct choice.
51. Ans. C.
According to the passage, “Great was the horror to find instead a pair of sharp teeth inserted into
his fingers”. This means that the pick-pocket was bitten by the dog as he put his hand in the lady’s
pocket. Thus, option C is the correct answer.
52. Ans. A.
“Evade” means to escape or avoid (someone or something). Thus, option A is the correct answer.
53. Ans. B.
According to the passage, “It so happened that the person next to this lady was a pickpocket; and
during the journey he carefully placed his hand into her pocket in search of her purse.” Thus, option
B is the correct answer.
54. Ans. D.
According to the passage, “Wishing to evade the law, she placed her tiny dog in her dress pocket.”
Thus, option D is the correct answer.
55. Ans. C.
According to the passage, “In Delhi, it was forbidden by the law, at one time, to take a Dog into a
public vehicle.” Forbidden means not permitted. Thus, option D is the correct answer.
56. Ans. B.
Option B - impact of Industrialization on our civilization is correct.
57. Ans. D.
According to the passage, “…the environment that has been created by it does not give time or
opportunity to individuals to think. If the life of the mind is not encouraged, then inevitably
civilization collapses.” So, the lack of the growth of mental life promotes deterioration. Thus, option
D is the correct answer.

PAGE 833
www.byjusexamprep.com

58. Ans. D.
In the opinion of the author, Industrialization is more of a curse than a blessing.
59. Ans. D.
According to the passage, “Differences have been intensified by this process of industrialization
which has promoted material well-being tremendously…” So, since industrialization had not taken
place, difference between India and Europe in the 12th or 13th century was not very great. Thus,
option D is the correct answer.
60. Ans. B.
“Orient” refers to the countries of the East, especially East Asia. “Occident”, on the other hand refers
to “the countries of the West, especially Europe and America. Thus, option B is the correct answer.
61. Ans. B.
As the first line of the passage states, “To eat and not be eaten - that’s the imperative of a caterpillar’s
existence.” The passage goes on to explain how different kinds of caterpillars protect themselves
from predators. Thus, option B best catches the main idea of the passage.
62. Ans. B.
“Unpalatable” means not pleasant to taste. According to the passage, “… jelly slug extrudes a sticky
translucent coating that may foul the mouth-parts of marauding ants.” Here, the expression ‘to foul
the mouth part means to make the taste bad. Thus, option B is the correct answer.
63. Ans. B.
According to the passage, the leaf roller protects itself from predators and the jelly slug protects
itself from marauding ants. Thus, ‘predator and marauding’ describe the enemies of the caterpillar.
Hence option B is the correct answer.
64. Ans. B.
According to the passage, “the jelly slug extrudes a sticky translucent coating that may foul the
mouth-parts of marauding ants.” Thus, option B is the correct answer.
65. Ans. A.
According to the passage, “The leaf roller reduces its risks of being picked off by predators by silking
together a temporary shelter in which to feed and rest… For its part, the aquatic larva, by its watery
element, fashions a portable hideout from fragments of aquatic leaves. Thus, leaf roller and aquatic
larva ‘build’ shelters to protect themselves. Option A is the correct answer.
66. Ans. A.
An irony is an expression of one's meaning by using language that normally signifies the opposite,
typically for humorous effect. As stated in the last line of the passage, the irony is that over-
population that causes poverty is a result of poverty itself. Thus, option A is the correct answer.
67. Ans. B.
Sombre— dark or dull in colour or tone
Didactic— intended to teach, instructive

PAGE 834
www.byjusexamprep.com

Tragic— causing or characterized by extreme distress or sorrow


The given passage gives information on over- population, its causes and ill- effects. Thus, option B is
the correct answer.
68. Ans. C.
According to the passage, “It has largely diluted the fruits of the remarkable economic progress that
the nation has made during the last four decades or so.” Here, “it” refers to over- population. Thus,
option C is the correct answer.
69. Ans. D.
According to the passage, “stark poverty itself is in many areas a major cause of over-population.”
Thus, option D is the correct answer.
70. Ans. D.
Diluted— make (something) weaker in force
Coalesced— come together to form one mass or whole
Compounded— combined
Cheapened— reduced
Consolidated— strengthened
Clearly, option D is the antonym of “diluted”.
71. Ans. B.
S1 states a fact and R goes on to explain it with the help of an example. Thus, is the first sentence
after S1. Q follows as a consequence as it states what will one do with the buttons that are left and
unrequired. P comes next as it talks about the things mentioned in Q. Both S and S6 talks about
lowering of quality, and hence are S6 follows S. Thus, the correct sequence is RQPS.
72. Ans. D.
S1 states that Roderick Usher did not talk much of himself. S presents the consequence of this fact
that because of this quality of Roderick Usher, the speaker did not know too much about him. R with
the usage of “yet” follows next as it presents a contrast to S. P follows as it talks about the ancestors
of Roderick Usher, people who have been referred to as “family” in R. Q mentions the further
characteristics of his ancestors. Thus, the correct sequence is SRPQ.
73. Ans. A.
R has to be the first statement as it is the introductory statement which implies that despite any
anticipation, Hitler attacked Russia. only options A and B have R as the first statement. Also, both
statements end in P, which means P should precede S6. P uses the pronoun “its” which must refer
to a thing. Thus, between S and Q, S should precede P as it talks about “A non-aggression treaty”. Q,
on the other hand, talks about Hitler. Thus, the correct sequence is RQSP.
74. Ans. B.
The correct sequence is option B - PRSQ.

PAGE 835
www.byjusexamprep.com

75. Ans. A.
S1 mentions the designation of the concerned person, i.e., “secretary of the Philosophical Society of
the Patna College”, and Q mentions the term, “in that capacity”, which means, “as the secretary” of
the concerned institute. Thus, Q becomes the first statement after S1. P talks about the visit and S
states what he saw. R puts forth a comment with reference to the visit and logically preceded S2.
The correct sequence is QPRS.
76. Ans. B.
Option B, The correct sequence is PSQR.
77. Ans. B.
Option B, The correct sequence is - RPSQ.
78. Ans. D.
Either statement Q or S can follow S1 logically. But since none of the options mentions Q as the first
statement after S1, we will go with S which talks about the figure mentioned in S1. P mentions an
additional quality of the figure, few of which have already been mentioned in S. Q and R form a
mandatory pair as Q talks about ‘events’ and R uses the pronoun “them” to refer to them. Thus, the
correct sequence is SPQR.
79. Ans. A.
R follows S1 as it gives a contrary idea to what has been stated in S1. R shows how despite legal
equality, out of food and clothing and houses – is still very unfair. P goes on to elaborate on this
unfairness and compares the living conditions of rich and poor people. Q and S form a mandatory
pair, where the former talks many people living in a single room and S talks about their activities in
the same room. Thus, the correct sequence is R P Q S.
80. Ans. C.
Q is the first sentence as it takes forward the idea stated in S1. Order and safety in our civilization is
reflected by the fact that one cannot get beaten merely because I am physically weaker. R states the
legal procedure that will take place. With the use of “thus”, P states how justice is not arbitrary and
can settle disputes without violence. S is linked with S6, as in both talk about robbery (breaking into
one’s house). Thus, the correct sequence is Q R P S.
81. Ans. D.
Vulnerable— unguarded, unsafe; weak
Exposed— uncovered
Resilient— strong
Clearly, option D is the antonym of “vulnerable”.
82. Ans. C.
Fanatical (adj.): obsessively concerned with something.
Bigoted (adj.): obstinately or unreasonably attached to a belief, opinion, or faction, and intolerant
towards other people's beliefs and practices.

PAGE 836
www.byjusexamprep.com

Dogmatic (adj.): inclined to lay down principles as undeniably true.


Insouciant (adj.): showing a casual lack of concern.
Fervid (adj.): intensely enthusiastic or passionate.
Infatuated (adj.): possessed with an intense but short-lived passion or admiration for someone.
So, the correct answer is option C.
83. Ans. D.
Abominable— hateful
Abhorrent— disgusting
Repugnant— extremely distasteful; unacceptable
Reputable— having a good reputation; well respected
Attractive— good looking
Option D is the antonym of “abominable”.
84. Ans. C.
Charming— very pleasant or attractive
Enchanting— delightfully charming or attractive
Hypnotic— irresistible
Repulsive— disgusting
Fascinating— extremely interesting
Clearly, option C is the antonym is the “repulsive”.
85. Ans. A.
Peculiar— unusual
Customary— usual; normal
Fascinating— extremely interesting
Ordinary - Normal, with no special features.
Thus, option A is the antonym of “peculiar”.
86. Ans. C.
Activity— the condition in which things are happening or being done
Dormant— a state of inactivity
Indolence— laziness
Hence, option C is the correct antonym.
87. Ans. B.
Anxious— worried

PAGE 837
www.byjusexamprep.com

Certain— unquestionable
Composed— calm
Careless— irresponsible
Heedless— showing a lack of care or attention
Clearly, option B is the antonym of “anxious”.
88. Ans. C.
Criticism— the act of passing harsh judgment
Scold— to criticize angrily
Scorn— mock
Appreciation— recognition; respect
Censure— express severe disapproval of
Clearly, option C is the antonym of “criticism”.
89. Ans. B.
Suitable— acceptable, appropriate
Insufficient— not in required amount
Impertinent— rude
Befitting— appropriate to the occasion
Congenial— like- minded, compatible
Clearly, “impertinent” is the antonym of “suitable”.
90. Ans. C.
Naïve— innocent
Credulous— having or showing too great a readiness to believe things.
Clearly, “wise” is the antonym of “naïve”.
91. Ans. C.
Wet Blanket refers to “a person who spoils other people's fun by failing to join in with or by
disapproving of their activities”.
Since first half of the sentence refers to others being enthusiastic, this phrase fits the blank most
aptly.
92. Ans. D.
Let's understand the meaning of the given words:
Patriotism: love of country and willingness to sacrifice for it
Complaining: expressing pain or dissatisfaction of resentment

PAGE 838
www.byjusexamprep.com

Remembrance: the ability to recall past occurrences


Gratitude: a feeling of thankfulness and appreciation
Hence, option D is the correct answer.
93. Ans. C.
Rash driving may potentially lead to the death of any person; however, it might not always be true.
Thus, powerful and reckless can be ruled out since they rule out the possibility and give a definite
statement. ‘Sure’ does not fir the blank. The correct answer would be ‘potential’.
94. Ans. D.
Let's understand the meanings of the given modals:-
Should = expresses obligation.
Needn't- is the correct answer choice.
Hence, option D is the correct answer.
95. Ans. D.
The correct phrase is 'to cry over spilled/ spilt milk'. It means being upset over something that has
already happened and cannot be changed. Hence, option D is the correct answer.
96. Ans. D.
Get by = manage with difficulty to live or accomplish something.
Get off = escape a punishment; be acquitted.
Get through = succeed, use
Get over = recover, overcome
Hence, option D is the correct answer.
97. Ans. C.
Whenever a sentence begins with negative adverbial phrases like "scarcely, hardly, no sooner etc,
inversion is used. It means that the verb is used before the subject. Usually, we use the verb "had"
with adverbs like scarcely, hardly. So, option C is the correct choice.
98. Ans. B.
Let's understand the meanings of the given phrasal verbs:
Cut up = to be upset
Done in = extremely tired
Done out = not a valid phrase.
Run out = to finish, use or sell all of something or the period of time for which something lasts finishes
The sentence talks about the incident after the marathon. It is evident that after a marathon, the
runner will feel extremely tired as they have run a lot. So, the phrasal verb "done in" is the correct
answer. Hence, option B is the correct answer.

PAGE 839
www.byjusexamprep.com

99. Ans. A.
Keep off = (of bad weather) fail to occur.
Put off = an off-putting or unpleasant quality or feature.
Set back = a reversal or check in progress.
Stay out = to keep out of something or some place or to remain uninvolved in some piece of business.
Hence, option A is the correct answer.
100. Ans. B.
The given sentence indicates that there is evidence against the concerned person which means that
it is not possible to let the person free. Punish, release and ignore impart an opposite meaning to the
sentence due to the presence of ‘cannot’. Absolve means declare (someone) free from guilt,
obligation, or punishment. Hence, option B is the correct answer.
101. Ans. A.
Option A: The segments form a coherent structure.
Option B: It is illogical; someone who is six months old can’t adopt some other person.
Option C: R doesn’t fit after the fixed segment.
Option D: PR after the fixed segment render it illogical.
102. Ans. B.
The sentence can clearly not begin with P and Q. Hence, option A is eliminated.
Option B: The segments make a structurally and grammatically sound sentence.
Option C: QR seems a bit off in terms of structure; hence it is incorrect.
Option D: RPS is not a well-structured sequence; hence it is incorrect.
Option B is the correct answer.
103. Ans. D.
P can clearly not be the opening segment; hence options A and C get eliminated.
Option B: SR doesn’t make any sense; hence it is eliminated.
Option D: The segments make a coherent sentence and hence the answer.
104. Ans. A.
Option A: The segments form a coherent sentence.
Option B: The sentence can clearly not begin with P; hence it is incorrect.
Option C: SR is incoherent.
Option D: QRP is incoherent. The demands can be disillusioned or disillusioning but not
disillusionment.

PAGE 840
www.byjusexamprep.com

105. Ans. D.
Option A: though it doesn’t seem incoherent in the first go, the hence formed sentence doesn’t
convey a sound meaning since whose poetry influenced is ambiguous.
Option B and C: QP and PQ are incoherent.
Option D: It forms a coherent sentence.
Option D is the correct answer.
106. Ans. C.
SP as an introductory pair is incoherent; hence option A is eliminated.
QP as a pair doesn’t make any sense; hence option B is eliminated.
Option C: The segments form a coherent sentence. So, it is the correct answer.
Option D: PQ doesn’t make any sense; hence it is eliminated.
107. Ans. D.
Option A: The sentence can clearly not begin with Q since it is incomplete to be a starter; hence it is
incorrect.
Option B: This order does not make a coherent sense.
Option C: This order does not convey a proper sense either.
Option D: This is the correct sequence as it conveys the sense that despite the laws, the Indian silk
and cotton textiles held their positions in the foreign markets.
108. Ans. C.
Option A: The sentence can clearly not begin with P; hence it is incorrect.
Option B: SP as a pair doesn’t make any sense, ‘to restrict and prohibit put pressure…’
Option C: It forms a coherent sentence.
Option D: QR is structurally incorrect and makes no sense, ‘Indian goods in England the British
manufacturers….’
109. Ans. A.
Option A: The segments make a coherent sentence; hence it can be put on hold.
Option B: SP as a pair can be ruled out. No possessive case is present; hence ‘the coarse woollens
dress fashions’ is incorrect.
Option C: The sentence can’t begin with ‘of the English’; hence it is incorrect.
Option D: SP as a pair can be ruled out again; hence it is incorrect.
Option A is the correct answer.
110. Ans. C.
Option A: No possessive link between manufacturers and popularity has been established; hence Q
can’t follow P.

PAGE 841
www.byjusexamprep.com

Option B: PQ as a pair can be ruled out again.


Option C: The segments form a coherent sentence ‘From the very beginning, the British
manufacturers were jealous of the popularity of Indian textiles.’
Option D: The popularity of Indian textiles can’t be jealous, but some subject can be jealous of the
popularity; hence it gets eliminated.
Hence, option C is the correct answer.
111. Ans. C.
Hide and liberate don’t fit the context.
Protect is the correct answer choice.
112. Ans. D.
Recommend = advise or suggest (something) as a course of action.
Counsel = recommend (a course of action); give advice to someone.
Advice would be unfit here as it is a noun whereas we require a verb.
Therefore, the correct synonym would be "suggest".
113. Ans. D.
Let's first learn the meanings of the words:
Assisted = helped
Obliged = make (someone) legally or morally bound to do something.
Though not completely synonymous to serve; offered is the best contextual fit. Option D is the
correct answer.
114. Ans. C.
Sol; Exemplary = serving as a desirable model; ideal
Flawed = having or characterized by a fundamental weakness or imperfection.
Boisterous = noisy, energetic, and cheerful.
Hence, ideal is the correct answer.
115. Ans. A.
Let's first learn the meanings of the words:
Embezzle = steal or misappropriate.
Assert = state a fact or belief confidently and forcefully.
Yield = produce.
Abdicate = resign, disown.
Hence, option A is the correct answer.

PAGE 842
www.byjusexamprep.com

116. Ans. C.
Admit = acknowledge/ accept
Avow = assert or confess openly.
Concede = admit or agree that something is true after first denying or resisting it.
Though admit, accept and concede are synonyms, concede has a contextual meaning which doesn’t
fit the given sentence
117. Ans. D.
Let's first learn the one words for the followings:
One who drives a car = chauffer
One who works in a hospital = doctor/nurse
One who is employed in food serving = caterer/ waiting staff
One who is engaged in selling = vendor
Hence, option D is the correct answer.
118. Ans. A.
Mask means to conceal or hide something.
Hence, hid is the correct answer.
Flaunt means to show off.
Oblique means not expressed or done in a direct way.
Obscure means not discovered or known about; uncertain.
119. Ans. D.
The meanings of the words:
Repulse = drive back (an attack or attacker) by force
Repelled = to force someone or something to stop moving towards you or attacking you
Reject = to refuse to accept, use, or believe something or someone
Defeat = to win against someone in a fight, war, or competition
Destroy = to damage something so badly that it cannot be used
Hence, option D is the correct answer.
120. Ans. B.
Let us understand the meaning of the above words:-
Ripe = fully grown, mature.
Seasoned = Aged or processed.
Moreover, only being senior can’t make a person respectable.
Hence, option B is the correct answer.

PAGE 843
www.byjusexamprep.com

General Knowledge
1. Ans. B.
Economists use the concept of marginal utility to measure happiness and pleasure and how that
affects consumer decision making. They have also identified the law of diminishing marginal utility,
which means that the first unit of consumption of a good or service has more utility than the next
units of consumption.
Hence, the marginal utility of the good diminishes as the amount of good consumed increases.
Hence, option B is correct.
2. Ans. C.
Only if either demand or supply was either completely elastic or inelastic will the tax burden fall
entirely on either the buyer or the seller.
Between these 2 extremes, tax incidence varies continuously from a perfectly inelastic supply or
perfectly elastic demand, where the sellers assumes the entire burden of the tax to the perfectly
elastic supply or perfectly inelastic demand where the buyers bear the entire burden.
To better see how the elasticity of supply and demand affects tax incidence, consider a 20% tax on a
can of soda. Suppose the government decides that the buyer should pay the 20% tax.
Hence, option C is correct.
3. Ans. D.
Statements 1 & 4 are true with respect to Sir Syed Ahmad Khan.
Hence, option D is correct.
4. Ans. B.
The Elephanta Caves are located in Western India on Elephanta Island (otherwise known as the Island
of Gharapuri), which features two hillocks separated by a narrow valley.
The name Ardhanarishvara (famous statue of Elephanta) means the Lord Who is half woman,
Ardhanarishvara is also known by other names. The Gupta-era writer Pushpadanta in his
Mahimnastava refers to form as dehardhaghatana. Utpala, commenting on the Brihat Samhita, calls
this form Ardha-gaurishvara, the VishnudharmottaraPurana simply calls this form Gaurishvara.
Hence, option B is correct.
5. Ans. B.
One of the finest jewels of Tamil poetry, the epic poem Manimekalai was written by Sathanar in 2nd
century A.D.
It is unique for the deep spirituality and mysticism it unfolds against the historical and geographical
background of South India and of adjacent Jaffna.
Hence, option B is correct.

PAGE 844
www.byjusexamprep.com

6. Ans. C.
The first 2 statements are true with respect to the Brahmadeya Grants during 600-1200 AD.
Hence, option C is correct.
7. Ans. D.
The inscriptions of Asoka were first deciphered by James Princep in 1837. They are written in Pali
language and in some places Prakrit was used. The Brahmi script was employed for writing.
In the northwestern India Asokan inscriptions were found in Karoshti script.
Hence, option D is correct.
8. Ans. B.
The Summit at Sanya will mark the admission of South Africa to the group, which will extend the
geographic representation of the mechanism in a moment when the financial system reform is
sought worldwide, as well as a greater democratization of global governance in general.
On 14 April 2011, 3rd BRICS (Brazil, Russia, China and South Africa) Summit is to be held in the
Chinese city of Sanya, alongside the President of China, Hu Jintao, the President of Russia, Dmitri
Medvedev, the Prime Minister of India, Manmohan Singh, and the President of South Africa, Jacob
Zuma.
The previous Summits were held in Yekaterinburg (June 2009) and Brasília (April 2010).
Hence, option B is correct.
9. Ans. A.
Vishakha and others v State of Rajasthan was a 1997 Indian Supreme Court case where Vishakha and
other women groups filed Public Interest Litigation (PIL) against State of Rajasthan and Union of India
to enforce the fundamental rights of working women under Articles 14, 19 and 21 of the Constitution
of India.
The court decided that the consideration of "International Conventions and norms are significant for
the purpose of interpretation of the guarantee of gender equality, right to work with human dignity
in Articles 14, 15, 19(1)(g) and 21 of the Constitution and the safeguards against sexual harassment
implicit therein.
Hence, option A is correct.
10. Ans. B.
Closing ceremony for the seventh Indo-Bangladesh joint exercise SAMPRITI was held at Counter
Insurgency and Jungle Warfare School, Vairengte in Mizoram.
Exercise SAMPRITI is an important bilateral defence cooperation endeavour between India and
Bangladesh.
The exercise is aimed at strengthening and broadening the aspects of interoperability and
cooperation between the Indian and Bangladesh Armies while working together in a counter-
insurgency and counter-terrorism environment under the UN mandate.
Hence, option B is correct.

PAGE 845
www.byjusexamprep.com

11. Ans. C.
Government has established the National Adaptation Fund on Climate Change (NAFCC) with a budget
provision of Rs.350 crores for the year 2015-16 and 2016-17, with an estimated requirement of
Rs.181.5 crores for financial year 2017-18.
The objective of the fund is to assist State and Union Territories that are particularly vulnerable to
the adverse effects of climate change in meeting the cost of adaptation.
The National Bank for Agriculture and Rural Development (NABARD) has been appointed as National
Implementing Entity (NIE) responsible for implementation of adaptation projects under the (NAFCC).
Hence, option C is correct.
12. Ans. A.
The Union Ministry of Shipping has extended period of Coastal Berth scheme of flagship Sagarmala
Programme for three years upto March, 2020. Besides, its scope was also expanded to cover capital
dredging at Major Ports and preparation of detailed project report (DPR) for coastal berth project.
The projects under Coastal Berth Scheme are distributed over eight states with highest number of
projects in Maharashtra (12 projects), Andhra Pradesh and Goa (10 projects), Karnataka (6 projects),
Kerala and Tamil Nadu (3 projects), Gujarat (2 projects) and West Bengal (1 project).
Hence, option A is correct.
The projects under Coastal Berth Scheme are distributed over eight states with highest number of
projects in Maharashtra (12 projects), Andhra Pradesh and Goa (10 projects), Karnataka (6 projects),
Kerala and Tamil Nadu (3 projects), Gujarat (2 projects) and West Bengal (1 project).
13. Ans. B.
The Hambantota Port (also known as the Port of Hambantota) is a maritime port in Hambantota, Sri
Lanka.
Hence, option B is correct.
14. Ans. A.
The Ministry of Rural Development (MoRD) announced the Deen Dayal Upadhyaya Grameen
Kaushalya Yojana (DDU-GKY) Antyodaya Diwas, on 25th September 2014.
DDU-GKY is a part of the National Rural Livelihood Mission (NRLM), tasked with the dual objectives
of adding diversity to the incomes of rural poor families and cater to the career aspirations of rural
youth.
DDU-GKY is uniquely focused on rural youth between the ages of 15 and 35 years from poor families.
Over 180 million or 69% of the country’s youth population between the ages of 18 and 34 years, live
in its rural areas. Of these, the bottom of the pyramid youth from poor families with no or marginal
employment number about 55 million.
Hence, option A is correct.

PAGE 846
www.byjusexamprep.com

15. Ans. A.
The objective of the NiryatBandhu Scheme is to reach out to the new and potential exporters and
mentor them through orientation programmes, counselling sessions, individual facilitation, etc., for
being able to get into international trade and boost exports from India.
Hence, option A is correct.
16. Ans. D.
The major objective of the PMKSY is to achieve
• convergence of investments in irrigation at the field level,
• expand cultivable area under assured irrigation (HarKhetkopani),
• improve on-farm water use efficiency to reduce wastage of water,
• enhance the adoption of precision-irrigation and other water saving technologies (More crop per
drop),
• enhance recharge of aquifers and introduce sustainable water conservation practices by
exploring the feasibility of reusing treated municipal based water for peri-urban agriculture and
attract greater private investment in precision irrigation system.
Hence, option D is correct.
17. Ans. C.
Voter Verifiable Paper Audit Trail (VVPAT) machines are used during election process to verify that
the vote polled by a voter goes to the correct candidate. VVPATs are a second line of verification
particularly and are particularly useful in the time when allegations around Electronic Voting
Machines' tampering crop up.
Hence, option C is correct.
18. Ans. A.
The International Bill of Human Rights has been further supplemented by various other international
treaties, conventions and declarations. They are usually regarded as “human rights instruments”.
The important among them are as follows…
• Convention on the Elimination of All Forms of Discrimination Against Women (1979)
• Declaration on the Right to Development (1986)
• Convention on the Rights of Persons with Disabilities (2006).
Hence, option A is correct.
19. Ans. D.
At present, the ITBP has 62 (including 4 service battalions) operational battalions which has close to
62,000 personnel and officers in them. So 4th statement is incorrect.
The Indo-Tibetan Border Police (ITBP) is one of the five Central Armed Police Forces of India, raised
on 24 October 1962, under the CRPF Act, in the wake of the Sino-Indian War of 1962.
Hence, option D is correct.

PAGE 847
www.byjusexamprep.com

20. Ans. A.
• SWIFT was founded in the 1970s, based on the ambitious and innovative vision of creating a
global financial messaging service, and a common language for international financial
messaging.
• The Uruguay Round was the 8th round of multilateral trade negotiations (MTN) conducted
within the framework of the General Agreement on Tariffs and Trade (GATT), spanning from
1986 to 1994 and embracing 123 countries as "contracting parties".
• The World Trade Organization (WTO) is an intergovernmental organization that regulates
international trade. The WTO officially commenced on 1 January 1995 under the Marrakesh
Agreement.
• Founded by the National Association of Securities Dealers, the NASDAQ began trading on
February 8, 1971, as the world's first electronic stock market.
Hence, option A is correct.
21. Ans. D.
The State Executive has temporary law-making power in emergent situations under the Constitution
of India. And the Governor is the head of the State Executive. So, the ordinance making power
confers on him by the Constitution itself. But this is not a discretionary power.
The ordinance making power of the Governor is co-extensive with the legislative power of the State
Legislature to make laws. He can promulgate ordinances only on the subjects on which the State
Legislature has power to make laws under the Constitution.
Hence, option D is correct.
22. Ans. A.
West Bengal: 295
Bihar: 243
Madhya Pradesh: 231
Tamil Nadu: 235
Hence, option A is correct.
23. Ans. C.
A stupa is a mound-like or hemispherical structure containing relics that is used as a place of
meditation.
Buddhist sources claim that during the 3rd century BCE, the Mauryan Emperor Ashoka the Great
ordered these eight stupas to be opened, further distributed the relics of the Buddha into 84,000
portions, and had stupas built over them all over the expanding Buddhist world.
They were located on trade routes in order to propagate Buddhism.
However, there were no evidence of them being located in rural areas.
Hence, option C is correct.

PAGE 848
www.byjusexamprep.com

24. Ans. D.
The contents of the Mahavamsa can be broadly divided into four categories:
The Buddha's Visits to Ceylon: This material recounts three legendary visits by the Buddha to the
island of Ceylon.
Chronicles of Kings of Ceylon: This material consists of genealogies and lineages of kings of Ceylon,
sometimes with stories about their succession or notable incidents in their reigns
History of the Buddhist Sangha: This section of the Mahavamsa deals with the mission sent by
Emperor Ashoka to Ceylon, the transplantation of the bodhi tree, and the founding of the
Mahavihara.
Chronicles of Ceylon: This material begins with the immigration of King Vijaya from India with his
retinue and continues until the reign of King Mahasena, recounting wars, succession disputes,
building of stupas and reliquaries, and other notable incidents.
Hence, option D is correct.
25. Ans. A.
Abul Fazl placed Mughal kingship as the highest station in the hierarchy of objects receiving light
emanating from God (farr-iizadi).
Abul Fazl was inspired by a famous Iranian Sufi, Shihabuddin Suhrawardi who first developed this
idea.
According to this idea, there was a hierarchy in which the Divine Light was transmitted to the king
who then became the source of spiritual guidance for his subjects.
Hence, option A is correct.
26. Ans. A.
Dasara Dibba or the Mahanavami Dibba is a beautiful stone platform located within the Royal
Enclosure of Hampi.
It was built during the Vijayanagara period by King Krishnadevaraya to commemorate his victory over
Udaygiri.
It was here where the King of Vijayanagar used to celebrate the festival of Dasara (Dussehra).
Hence, option A is correct.
27. Ans. A.
The Phoenix Settlement, established by Gandhiji near Durban in 1904, was formally reopened on
February 27, 2000, at a ceremony attended by the President of South Africa, the Zulu King Goodwill
Zwelithini and many other leaders.
The settlement - the first Ashram of Gandhiji - had been damaged in 1985 riots when some African
squatters occupied much of the settlement and named it Bambayi.
Though the Indian community was deeply distressed, it refrained from seeking the forcible eviction
of the squatters.

PAGE 849
www.byjusexamprep.com

The Phoenix Settlement Trust, with financial assistance from the Government of India, recently
restored Gandhiji's house and established a clinic, an HIV/Aids Centre and other facilities to serve all
the people in the area, African and Indian
Hence, option A is correct.
28. Ans. C.
According to Gandhiji,
A sound body means one which bends itself to the spirit and is always a ready instrument at its
service. Such bodies are not made, in my opinion, on the football field. They are made on cornfields
and farms.
Hence, option C is correct.
29. Ans. B.
The Olive ridley turtles are the smallest and most abundant of all sea turtles found in the world,
inhabiting warm waters of the Pacific, Atlantic and Indian oceans.
These turtles, along with their cousin the Kemps ridley turtle, are best known for their unique mass
nesting called Arribada, where thousands of females come together on the same beach to lay eggs.
Though found in abundance, their numbers have been declining over the past few years, and the
species is recognized as Vulnerable by the IUCN Red list.
The coast of Orissa in India is the largest mass nesting site for the Olive-ridley, followed by the coasts
of Mexico and Costa Rica.
Hence, option B is correct.
30. Ans. B.
eBiz – India’s Government-to-Business (G2B) portal was conceptualized with support from National
Institute of Smart Government (NISG) as the consulting partner and developed by M/s. Infosys Ltd.,
Bangalore in a Public Private Partnership (PPP) Model for a period of 10 years.
Through eBiz portal, a business user can fill the e-Forms online/offline, upload the attachments,
make payment online and submit the forms for processing of the department.
Hence, option B is correct.
31. Ans. A.
Natural Gas
The Natural Gas production (weight: 1.71 %) decreased by 4.5 % in June 2016 over June 2015. Its
cumulative index during April to June 2016-17 declined by 6.1 % over the corresponding period of
previous year.
Hence, option A is correct.
32. Ans. B.
India unveiled the third National Wildlife Action Plan for 2017-2031. The plan was unveiled by
environment minister Dr Harsh Vardhan on the inaugural day of the Global Wildlife Programme
(GWP) conference

PAGE 850
www.byjusexamprep.com

The third National Wildlife Action Plan is unique as this is the first time India has recognised the
concerns relating to climate change impact on wildlife and stressed on integrating actions that need
to be taken for its mitigation and adaptation into wildlife management planning processes.
Hence, option B is correct.
33. Ans. D.
There shall be a Council of Ministers consisting of not more than ten per cent (a special provision
for Delhi as against the 15 percent clause under 91st Amendment Act) of the total number of
members in the Legislative Assembly, with the Chief Minister at the head to aid and advise the
Lieutenant Governor in the exercise of his functions in relation to matters with respect to which the
Legislative Assembly has power to make laws.
Option A is correct because the Constitution of India grants special status to Jammu and Kashmir
among Indian states, and it is the only state in India to have a separate constitution.
Hence, option D is correct.
34. Ans. B.
Originally designed for Taiwan, Beijing applied the “One Country, Two Systems” principle to Hong
Kong on July 1, 1997 when sovereignty of the territory was passed from Britain to China.
But “One Country, Two Systems,” which guarantees a degree of autonomy under Chinese
supervision, isn’t applicable to Taiwan. The island has evolved into a self-ruled, hyper-democracy
that enjoys de facto operational independence
Hence, option B is correct.
35. Ans. A.
• Sikkim was admitted to the Union of India through the 36th Amendment Act of 1974.
• Sikkim became a state of India on April 26, 1975, as a result of the 36 Amendment Act.
• In 1975, the Sikkimese Parliament petitioned the Indian Parliament to recognize Sikkim as a
state of India.
• Sikkim has four districts, each with one Rajya Sabha and one Lok Sabha seat.
Hence, option A is correct.
36. Ans. C.
The Attorney General is responsible for giving advice to the Government of India upon such legal
matters and to perform such other duties of legal character as may be referred or assigned to him
by the President.
The Attorney General has the right of audience in all Courts in India as well as the right to participate
in the proceedings of the Parliament, though not to vote.
The Attorney General appears on behalf of Government of India in all cases (including suits, appeals
and other proceedings) in the Supreme Court in which Government of India is concerned.
Hence, option C is correct.

PAGE 851
www.byjusexamprep.com

37. Ans. C.

Hence, option C is correct.


38. Ans. A.
The Nalayira Divya Prabandham is a collection of 4,000 Tamil verses composed by the 12 Alvars and
was compiled in its present form by Nathamunigal during the 9th - 10th centuries.
These were sung by the Alwars in devotional estacy in varrious shirineas of Vishnu, made famous as
Magalashasana sthalas.
Hence, option A is correct.
39. Ans. A.
Ginans are devotional hymns or poems recited by Shia Ismaili Muslims.
Although ginans can be recited, studied, and listened to by non-NizariIsmailis, ginans hold a special
role in the cultural practice and rituals of NizariIsmailis, specifically the community of Khojas, a caste
of south asians of whom the majority now identify as Nizari Ismaili.
Hence, option A is correct.
40. Ans. A.
But while the average Mughal farmer produced more than in later times, he most probably produced
less than in earlier times.
On the whole, the Mughal period was marked by agricultural stagnation, if not slump.
The per capita yield was declining, and the average man in Mughal India probably had less to eat
than before.
'The surplus income left to the peasant was tending to decrease, where it had not already vanished,'
says Moreland.
Hence, option A is correct.

PAGE 852
www.byjusexamprep.com

41. Ans. D.
According to al-Biruni, the sufi theories of the soul were similar to those in Patanjali’s Yoga Sutra.
Like the Yoga Sutra, sufi works also stated that ‘the bodies are the snares of the souls for the purpose
of acquiring recompense’.
Al-Biruni also identifies the sufi doctrine of divine love as self-annihilation with parallel passages from
the Bhagavad Gita.
There was also much exchange of ideas between the Sufis and Indian yogis. In fact the hatha-yoga
treatise Amrita Kunda was translated into Arabic and Persian, So C is right.
Hujwiri was known as DattaGanjBaksh. Hujwiri comments that before he settled in Lahore some sufis
believed in theories that he calls brahmanical.
Hence, option D is correct.
42. Ans. C.
The ancient Greek word “bárbaros,” from which it derives, meant “babbler,”.
In the Greek ear, speakers of a foreign tongue made unintelligible sounds (“bar bar bar”).
Similar words exist in other Indo-European languages, including the Sanskrit “barbara,” which means
“stammering.”
The word “barbarian” originated in ancient Greece and was initially used to describe all non-Greek-
speaking peoples, including Persians, Egyptians, Medes and Phoenicians.
Hence, option C is correct.
43. Ans. B.
H S Prannoy has won the men’s singles title at the 82nd Senior National Championship Senior
National Badminton Championships in Nagpur, Maharashtra on November 8, 2017. He defeated
higher ranked KidambiSrikanth in the final by 21- 15, 16- 21, 21-7.
Hence, option B is correct.
44. Ans. B.
World Soil Day 2017 is celebrated on 5 December under the theme "Caring for the Planet starts from
the Ground," with activities aiming to communicate messages on the importance of soil quality for
food security, healthy ecosystems and human well-being.
Soils not only produce 95% of the food we consume, they also have a huge potential for taking carbon
from the atmosphere and offsetting the greenhouse gas emissions.
Keeping the soil healthy can bring a multitude of gains, from food security to sustainable livelihoods
and climate benefits.
Hence, option B is correct.
45. Ans. A.
Indian sporting legend M.C. Mary Kom (48kg) clinched an unprecedented fifth gold medal but Sonia
Lather (57kg) settled for silver at the ASBC Asian Confederation women’s boxing championships.
This was Mary Kom’s first international gold medal since the 2014 Asian Games.
Hence, option A is correct.

PAGE 853
www.byjusexamprep.com

46. Ans. B.
Indian short film ‘The School Bag’, which tells a story based in Pakistan, has won the Best Short Film
Award at the South Asian Film Festival of Montreal (SAFFM).
It is directed by Dheeraj Jindal.
Hence, option B is correct.
47. Ans. D.
Power Ministry constitutes a Committee to investigate into the causes of the NTPC, Unchahar
Thermal Power Plant accident
The composition of the committee would be as follows:

Hence, option D is correct.


48. Ans. C.
Nirbhay missile is India's first indigenously designed and developed long range sub-sonic cruise
missile.
It is designed and made by DRDO (Defence Research & Development Organisation).
The Nirbhay missile can be deployed from multiple platforms.
Hence, option C is correct.
49. Ans. B.
After the Mumbai attacks in 2008, there has been a paradigm shift in the maritime security apparatus
that increased emphasis on surveillance, intelligence gathering and information sharing amongst the
various stakeholders to ensure an effective response to any emerging situation.
In Feb 2009, the Indian Coast Guard (ICG) was additionally designated as the authority responsible
for coastal security in territorial waters, including areas to be patrolled by the Coastal Police.
The Coast Guard is also responsible for overall coordination between Central and State agencies in
matters relating to Coastal Security.
In order to achieve near gap-free surveillance of the entire coastline, 38 additional Radar Stations
and 08 Mobile Surveillance Systems are being installed.
Hence, option B is correct.

PAGE 854
www.byjusexamprep.com

50. Ans. A.
On July 7, the 91st Amendment to the Constitution, limiting the size of the Council of Ministers at
the Centre and the States to no more than 15 per cent of the numbers in the Lok Sabha or the State
Legislature, came into effect.
Hence, option A is correct.
51. Ans. D.
The Commonwealth Heads of Government Meeting is held every 2 years. It is the association’s
ultimate policy and decision-making forum.
The November 2015 Meeting, to be held in Malta, had the theme of ‘Commonwealth – Adding Global
Value’.
Hence, option D is correct.
52. Ans. D.
Statement 1 and 3 are factual and are correct.
The Maritime Agenda projects a total traffic of 2494.95 million tonnes for all major and non-major
ports taken together and a capacity of 3280.04 million tonnes. Hence, statement 4 is incorrect.
Hence, option D is correct.
53. Ans. B.
• The 2017 Women's Hockey Asia Cup was the ninth Hockey Asia Cup for women.
• It was held from 28 October to 5 November 2017 in Kakamigahara, Gifu, Japan.
• The winner of this tournament qualified for the 2018 World Cup in England.
• India won their second title, after beating China in the final
Hence, option B is correct.
54. Ans. B.
The massive renovation and restoration effort at the Sri Ranganathaswamy Temple in Srirangam has
won the UNESCO Asia Pacific Award of Merit 2017 for cultural heritage conservation.
Hence, option B is correct.
55. Ans. B.
Minister of Communications Shri Manoj Sinha today launched a Pan India scholarship program for
school children called Deen Dayal SPARSH Yojana to increase the reach of Philately.
Under the scheme of SPARSH (Scholarship for Promotion of Aptitude & Research in Stamps as a
Hobby), it is proposed to award annual scholarships to children of Standard VI to IX having good
academic record and also pursuing Philately as a hobby through a competitive selection process in
all postal circles.
The Minister said that to avail this scholarship, a child must be a student of a recognized school within
India and the concerned school should have a Philately Club and the candidate should be a member
of the Club.
Hence, option B is correct.

PAGE 855
www.byjusexamprep.com

56. Ans. C.
The Department of Biotechnology (DBT) of the Ministry of Science and Technology, Government of
India, has collaborated with Nobel Media AB, Sweden to hold the Nobel Prize Series in India for five
years.
The second edition of the Nobel Prize Series - India 2018 will be held at Panjim, Goa in partnership
with Government of Goa on February 1-2, 2018. It will start on February 1, 2018 with the formal
inauguration of the Nobel Prize Series - India 2018, Science Exhibition by the Hon'ble Chief Minister
Mr. Manohar Parrikar.
Hence, option C is correct.
57. Ans. D.
World food India is shaping up to be one of the premier food exhibitions for the food processing
industry in India.
An initiative of the Indian government, this year’s edition is set to feature 400+ exhibitors both local
and international.
Italy due to its excellence in all aspects of the food processing chain will be a focus country.
Hence, option D is correct.
58. Ans. C.
Non-Brahmin lower caste movement took place in Madras and Maharashtra
Karnataka was not affected by the NCM.
There was labour unrest in Assam, Bengal and Madras
Statement 4 is incorrect as after the Chauri-Chaura incident the movement was completely stopped.
Hence, option C is correct.
59. Ans. B.
• surya sukta-- a hymn in the praise of the sun god
• dana stuti-- these are hymns in the praise of those who donate
• urna sutra-- is the dot/mark on the forehead that is put before performing rituals.
• purusha sukta--- contains hymns about marriage ceremonies
Hence, option B is correct.
60. Ans. D.
Sufi silsilas began to crystallise in different parts of the Islamic world around the twelfth century.
The word silsila literally means a chain, signifying a continuous link between master and disciple,
stretching as an unbroken spiritual genealogy to the Prophet Muhammad.
It was through this channel that spiritual power and blessings were transmitted to devotees.
Special rituals of initiation were developed in which initiates took an oath of allegiance, wore a
patched garment, and shaved their hair. Thus, all statements are correct.
Hence, option D is correct.

PAGE 856
www.byjusexamprep.com

61. Ans. B.
Verses ascribed to Kabir have been compiled in three distinct but overlapping traditions. The Kabir
Bijak is preserved by the Kabirpanth (the path or sect of Kabir) in Varanasi and elsewhere in Uttar
Pradesh, the Kabir Granthavali is associated with the Dadupanth in Rajasthan, and many of his
compositions are found in the Adi Granth Sahib.
Bijak is the best known of the compilations of the Kabir, and as such is the holy scripture for followers
of the Kabirpanthi religion. The Bijak, an anthology of Kabir’s verses compiled in Eastern Uttar
Pradesh and/or Bihar, is one of Kabir’s most influential works.
In the case of the Guru Granth Sahib, Kabir’s poems were collected to be sung and are organized in
ragas.
Hence, option B is correct.
62. Ans. B.
Antrix Corporation Limited (ACL), Bengaluru is a wholly owned Government of India Company under
the administrative control of the Department of Space.
Antrix Corporation Limited was incorporated as a private limited company owned by Government of
India in September 1992 as a Marketing arm of ISRO for promotion and commercial exploitation of
space products, technical consultancy services and transfer of technologies developed by ISRO.
As the commercial and marketing arm of ISRO, Antrix is engaged in providing Space products and
services to international customers worldwide.
Hence, option B is correct.
63. Ans. C.
Trinamool Congress became the 7th political party to be recognized as a National Party in September
2016.

Hence, option C is correct.


64. Ans. B.
Disqualification of MP's and MLA's on the ground of holding office of profit is a controversial topic of
nation-wide debate. It began with the disqualification of Samajwadi Party MP Jaya Bachchan from
Rajya Sabha on March 17, 2006 with retrospective effect from July 14, 2004.

PAGE 857
www.byjusexamprep.com

There are separate provisions of disqualification for the Members of Parliament and MLAs. Our
constitution clearly mentions that the Parliament has the power to make a law in respect to the issue
of qualification and disqualification of the membership in case:
(i) He holds any office of profit under the Government of India or of any state, other than an office
declared by a law of Parliament not to disqualify its holder.
(ii) He is of unsound mind and stand so declared by a competent court.
(iii) He is an undischarged insolvent.
(iv) He is not a citizen of India or has voluntarily acquired the citizenship of a foreign state, or is under
any acknowledgement or allegiance or adherence to a foreign state, and
(v) It is he so disqualified by or under any law of Parliament. It applies to the case of 'defection' as
prescribed in tenth schedule.
Hence, option B is correct.
65. Ans. D.
The Commission performs the following functions, namely:
Inquire, Suomotu or on a petition presented to it by a victim or any person on his behalf, into
complaint of
a) violation of human rights – this is extremely obvious
b) negligence in the prevention of such violation, by a public servant.
c) intervene in any proceeding involving any allegation of violation of human rights pending before a
court with the approval of such court.
Hence, option D is correct.
66. Ans. A.
With simple, diverse and growth-oriented offerings, IPPB aims to give every Indian access to efficient
banking services.
Incorporated as a Public-Sector Bank under the Department of Posts with 100% GOI equity, IPPB has
launched on January 30th, 2017 in Ranchi and Raipur with the objective of being present in all
corners of India by the end of the year.
Hence, option A is correct
67. Ans. A.
October 2017: A press release from the Ministry of External Affairs (MEA) noted that the
consignment would be the first to use the new route via Chabahar (Iran) to access Afghanistan, even
as India plans similar transfers in the coming months.
The shipment of wheat is a landmark moment as it will pave the way for operationalisation of the
Chabahar port as an alternative, reliable and robust connectivity for Afghanistan.
It will open up new opportunities for trade and transit from and to Afghanistan and enhance trade
and commerce between the three countries (India, Iran and Afghanistan) and the wider region.
https://www.thehindu.com/news/national/india-ships-wheat-to-afghanistan-via-
chabahar/article19945498.ece

PAGE 858
www.byjusexamprep.com

68. Ans. D.
The Reserve Bank of India has constituted a 10-member ‘High Level Task Force on Public Credit
Registry (PCR) for India’, which will, among other things, suggest a roadmap for developing a
transparent, comprehensive and near-real-time PCR for India.
Headed by YM Deosthalee, ex-CMD, L&T Finance Holdings, the task force includes Sekar Karnam,
DMD & Chief Credit Officer, SBI; Vishaka Mulye, ED, ICICI Bank; Rashesh Shah, Chairman and CEO,
Edelweiss Group; and Sriram Kalyanaraman, MD and CEO, National Housing Bank.
https://www.thehindubusinessline.com/money-and-banking/rbi-sets-up-task-force-on-public-
credit-registry/article9920197.ece
Hence, option D is correct
69. Ans. C.
https://pib.nic.in/newsite/PrintRelease.aspx?relid=173147
In continuance of the several initiatives under taken by Pension Fund Regulatory and Development
Authority (PFRDA) during the last few years to increase the pension coverage in the country, PFRDA
has now increased the maximum age of joining under NPS-Private Sector (i.e. All Citizen and
Corporate Model) from the existing 60 years to 65 years of age.
Now, any Indian Citizen, resident or non-resident, between the age of 60- 65 years, can also join NPS
and continue up to the age of 70 years in NPS. With this increase of joining age, the subscribers who
are willing to join NPS at the later stage of life will be able to avail the benefits of NPS.
Hence, option C is correct
70. Ans. C.
• An ETF, or exchange-traded fund, is a marketable security that tracks an index, a commodity,
bonds, or a basket of assets like an index fund.
• Unlike mutual funds, an ETF trades like a common stock on a stock exchange.
• ETFs experience price changes throughout the day as they are bought and sold.
• ETFs typically have higher daily liquidity and lower fees than mutual fund shares, making them
an attractive alternative for individual investors.
Hence, option C is correct
71. Ans. A.
Muscle cramping is a common problem encountered by athletes and nonathletes alike. Defined as
painful involuntary skeletal muscle contractions, cramps may be categorized as either nonexercise
related or exercise related.
Traditionally, such cramping was believed to arise from dehydration, electrolyte imbalances
(including magnesium, potassium and sodium), accumulation of lactic acid, or low cellular energy
levels.
Hence, option A is correct
72. Ans. B.
Mammalian and avian hearts have four chambers two atria and two ventricles.

PAGE 859
www.byjusexamprep.com

Amphibians and reptiles, by contrast, have a three-chambered heart. The three-chambered heart
consists of two atria and one ventricle.
Fish possess the simplest type of true heart a two-chambered organ composed of one atrium and
one ventricle.
Hence, option B is correct
73. Ans. C.
The lymphatic system is a complex system of fluid drainage and transport, and immune response and
disease resistance.
Fluid that is forced out of the bloodstream during normal circulation is filtered through lymph nodes
to remove bacteria, abnormal cells and other matter.
This fluid is then transported back into the bloodstream via the lymph vessels.
Lymph only moves in one direction, toward the heart.
Hence, option C is correct
74. Ans. A.

Hence, option A is correct

PAGE 860
www.byjusexamprep.com

75. Ans. B.
List of Natural World Heritage Sites. Name of UNESCO World Heritage Site, location, year of
notification, and total area is as follows:

Hence, option B is correct


76. Ans. B.

Hence, option B is correct

PAGE 861
www.byjusexamprep.com

77. Ans. D.
Karewa sediments are treasures of many human civilizations and habitations. In fact, the agriculture
of the valley dominantly survives and sustains on Karewa soils. The world famous saffron from
Pampore and apples from Shopian are best examples.
The famous almond orchids are best grown in the soils of karewas.
The karewas are mainly devoted to the cultivation of saffron, almond, walnut, apple and orchards.
Ling nuts (singharas) grow throughout the East of India: West Bengal, Jharkhand, and Bihar are
examples of such regions.
Hence, option D is correct
78. Ans. D.
Volcanic mountains are very common along the destructive plate boundaries i.e. Oceanic-continental
plate boundaries which are yet uncovered in the rock folded mountains formed due to continental –
to continental plate collision.
The Himalayas are also a plate-to-plate collision tectonic boundary. In this case, the Indian Plate [of
the Indian Subcontinent] is colliding head-on with the Eurasian Plate. Both plates are comprised of
continental lithospheric crust, so there is no appreciable distinction in density. Hence, option D is
correct
79. Ans. A.

Hence, option A is correct


80. Ans. B.
Composition of Dry Air (Substance, % by volume)
Nitrogen, N2 78.08
Oxygen, O2 20.95
Argon, Ar 0.93
Carbon dioxide, CO2 0.033
Neon, Ne 0.0018
Helium, He 0.00052
Methane, CH4 0.0002
Krypton, Kr 0.00011
Dinitrogen oxide, N2O 0.00005

PAGE 862
www.byjusexamprep.com

Hydrogen, H2 0.00005
Ozone, O3 0.000001
Hence, option B is correct
81. Ans. A.
Pascal's law is a principle in fluid mechanics that states that a pressure change occurring anywhere
in a confined incompressible fluid is transmitted throughout the fluid such that the same change
occurs everywhere.
This principle is stated mathematically as:
P= P + rgh
Hence, option A is correct
82. Ans. C.
Aluminum has a resistivity varying from 2.65 to 2.82 × 10−8 Ω m.
The resistivity of copper is generally given as: 1.72 x 10-8 Ω.m.
So, since resistivity of aluminium is higher than that of copper, so Power dissipated will be greater
than P. this is because rest of the factors (l, A and i) are constant.
Hence, option C is correct
83. Ans. A.
National Water Academy (formerly known as Central Training Unit) was set up in Central Water
Commission Opens in a new window by the Ministry of Water Resources, RD and GR, opens in a new
window Govt. of India in the year 1988, to impart training to the in-service engineers of various
Central/State organizations involved in the Development & Management of Water Resources.
It was established under USAID assistance and strengthened with the subsequent assistance received
from the World Bank.
It is located at Pune.
Hence, option A is correct
84. Ans. C.
India Integrated Coastal Zone Management Project
• To conserve, project and manage the coastal and marine environment, the Ministry is
implementing the India ICZM Project with the World Bank assistance.
• For the implementation of the project the Ministry has established a Society of Integrated
Coastal Management (SICOM) as nodal body under the Society Registration Act, in Delhi.
• The ICZM Project is being implemented as pilot investments in the coastal states of Gujarat,
Odisha and West Bengal.
• National activities are implemented through SICOM along with monitoring of activities carried
out by three selected states.

PAGE 863
www.byjusexamprep.com

• The project has four implementing agencies-MoEF at the national level with lead
responsibilities, and the Departments of Forests and Environment (DoEF) of the three
participating states.
Hence, option C is correct
85. Ans. D.
In the exclusive economic zone, the coastal nation has sovereign rights for the purpose of exploring
and exploiting, conserving and managing the natural resources, whether living or non-living,
including production of energy from the water, currents and winds.
However, In the exclusive economic zone, all nations, whether coastal or land-locked shall enjoy the
freedom of navigation and overflight and of the laying of submarine cables and pipelines, and other
internationally lawful uses of the sea. That means fourth right is not exclusive only for the coastal
nations.
Hence, option D is correct
86. Ans. C.
Cirrus clouds are formed at high altitudes (8,000 – 12,000m). They are thin and detached clouds
having a feathery appearance. They are always white in colour.
Clouds with extensive vertical development are cumulus and cumulonimbus. And they cause heavy
rainfall and thunderstorms.
Stratus clouds are the lowest forming and are often called fog or mists when they are earth-bound
Stratus clouds are formed when a large air mass cools at the same time (e.g. – a warm air parcel
drifts into or above a cooler region). They are therefore, horizontally forming.
Hence, option C is correct
87. Ans. D.
• According to Rayleigh's law, the intensity of scattered light varies inversely as the fourth power
of its wavelength. Sunlight consists of seven colors. Of these, red has the maximum wavelength.
• During sunrise and sunset, the rays have to travel a larger part of the atmosphere because they
are very close to the horizon.
• Therefore, light other than red is mostly scattered away. Most of the red light, which is the least
scattered, enters our eyes. Hence, the sun and the sky appear red.
Hence, option D is correct
88. Ans. C.

So, resistance of Wire 2 is R/4 as radius is half of wire 1, so area would be 4 times of wire 1.
Now for resistance in series. R= R1 + R2.
Therefore, Final R= R + R/4 = 5R/4
Hence, option C is correct

PAGE 864
www.byjusexamprep.com

89. Ans. C.
Heat lost by hot object = Heat gained by cold object
where we ignore heat gained or lost from/to the surroundings. The study of heat gained and lost in
this manner is often called Calorimetry.
Hence, option C is correct
90. Ans. D.
Isobar: one of two or more atoms having equal atomic weights but different atomic numbers.

Hence, option D is correct


91. Ans. D.
There are two types of water hardness: temporary and permanent hardness.
Temporary hardness is due to the presence of calcium hydrogen carbonate Ca(HCO3)2(aq) and
magnesium hydrogen carbonate Mg(HCO3)2(aq).
Hence, option D is correct
92. Ans. B.
• A compound is a substance formed by the chemical bonding of two or more elements. As a
result, none of the particles in the compound can be the same.
• If it contains the same particles, it is referred to as an element.
• Because the elements of the compound are chemically combined, they cannot be separated
using traditional physical methods.
Hence, option B is correct
93. Ans. B.
Electronic Flash differs from the consumable bulb flash in important aspects. Instead of creating light
by burning up in an atmosphere containing oxygen, a consumable material (each bulb being used
therefore only once), it produces a brilliant incandescent by discharging a current for a brief instant
through a rarified gas, usually xenon.
The gas glows every time the discharge is made through it but it is not consumed, so that many
thousands of flashes are obtainable from a single tube, which helps to offset the much higher cost
of the initial apparatus.
Hence, option B is correct
94. Ans. A.
The Chamberlin–Moulton planetesimal hypothesis was proposed in 1905 by geologist Thomas
Chrowder Chamberlin and astronomer Forest Ray Moulton to describe the formation of the solar
system.

PAGE 865
www.byjusexamprep.com

Mechanism of the Thermal Contraction Theory: Jeffrey's theory is based essentially on the history of
the contraction of the earth
Famous German geologist Kober has presented a detailed and systematic description of the surface
features of the earth in his book 'Der Bau der Erde'. Kober'sgeosynclinal theory is based on the forces
of contraction produced by the cooling of the earth
Hypothesis of sliding continent was given by Daly.
Hence, option A is correct
95. Ans. B.
As temperature of Leh will be lowest during winters from the given options, the highest pressure will
be at Leh.
Hence, option B is correct
96. Ans. A.
In colder clouds: droplets formed by Bergeron mechanism. But, in many parts of the world the air is
too warm for ice crystals to form.
This being the case, rain and snow cannot develop following the Bergeron Process.
Instead, tiny droplets form as they collide into one another creating larger and larger droplet, this is
known as collision- coalescence process. So, warmer clouds are associated with this keyword.
Hence, option A is correct
97. Ans. C.
Application of temperature changes in the atmosphere is in adiabatics, or temperature change which
are caused by compressing or allowing air to expand.
Hence, option C is correct
98. Ans. B.
There are 3 double-layered membrane layers, which the protein has to pass to reach its destination.
Hence, option B is correct
99. Ans. C.
• Lysosomes are spherical membranous sacs of enzymes.
• The lysosome membrane helps to keep its internal compartment acidic and separates the
digestive enzymes from the rest of the cell.
• Lysosomes are formed by budding from the Golgi complex.
Hence, option C is correct
100. Ans. C.
Euglena move by a flagellum (plural ‚ flagella), which is a long whip-like structure that acts like a little
motor.
The paramecium uses its cilia to sweep the food along with some water into the cell mouth after it
falls into the oral groove.

PAGE 866
www.byjusexamprep.com

The underside of the starfish is covered with hundreds of tube feet, which it uses for walking around,
for attaching tightly to rocks, and for holding on to prey.
Nereis possess setae and parapodia for locomotion
Hence, option C is correct
101. Ans. A.
• Adipocytes are the major source of leptin production, and plasma leptin concentrations
directly correlate with the amount of total body fat.
• Adipocytes are highly specialized cells that play a crucial role in the energy balance of most
vertebrates.
• Adipocytes convert excess energy to tri-acylglycerol and deposit it during feeding in
preparation for periods of food deprivation.
• Adipocytes may become enlarged by increased fat storage.
Hence, option A is correct
102. Ans. D.
Cuttlefish or cuttles are marine animals of the order Sepiida. They belong to the class Cephalopoda,
which also includes squid, octopuses, and nautiluses. Cuttlefish have a unique internal shell, the
cuttlebone. Despite their name, cuttlefish are not fish but molluscs.
Jellyfish are soft bodied, free-swimming aquatic animals with a gelatinous umbrella-shaped bell and
trailing tentacles.
Lepismasacharina commonly known as silver fish is small wingless insect belongs to phylum-
Arthropoda.
The Exocoetidae are a family of marine fishes in the order Beloniformes class Actinopterygii. Fish of
this family are known as flying fish.
Hence, option D is correct
103. Ans. B.
• Bioaccumulation is the process by which chemicals such as pesticides, heavy metals, and toxic
substances accumulate in an organism, posing a threat to health, life, and the environment.
• Planktons are primary producers, and large fishes are tertiary consumers, consuming
producers such as planktons & other small fishes, resulting in biomagnification or an increase
in pesticide concentrations. As a result of this intake, large fishes in the pond have higher
pesticide concentrations than planktons in the same pond.
Hence, option B is correct
104. Ans. A.
Tuberculosis (TB) is caused by a type of bacterium called Mycobacterium tuberculosis.
It's spread when a person with active TB disease in their lungs coughs or sneezes and someone else
inhales the expelled droplets, which contain TB bacteria.
One would have to spend prolonged periods (several hours) in close contact with an infected person
to catch the infection yourself.

PAGE 867
www.byjusexamprep.com

For example, TB infections usually spread between family members who live in the same house. It
would be highly unlikely for you to become infected by sitting next to an infected person on a bus or
train. Hence, statement 1 is incorrect. Hence, option A is correct
105. Ans. B.
Tides are the rise and fall of sea levels caused by the combined effects of the gravitational forces
exerted by the Moon and the Sun and the rotation of Earth.
Statement 2 is also correct. But it is not the correct explanation for the 1st statement.
Hence, option B is correct
106. Ans. B.
Species richness is generally lower in the Arctic than at lower latitudes, and richness also tends to
decline from the low to high Arctic.
Many hypotheses have been advanced to explain the overall decline of biodiversity with increasing
latitude, although there is still no consensus about a mechanistic explanation.
Abiotic factors include lower available energy and area at high latitudes, and the relatively young age
of Arctic ecosystems.
Among biotic factors, latitudinal differences in rates of diversification have been suggested, but
empirical evidence for this as a general principle is lacking.
Hence, option B is correct
107. Ans. A.
Glaciers cut distinctive U-shaped valleys with a flat floor and steep sides. These are an example of a
glacial erosion landform.
Example: As the main glacier erodes deeper into the valley, the tributary is left higher up the steep
sides of the glacier.
Hence, option A is correct
108. Ans. D.
Rejuvenation occurs when the river’s base level falls (i.e. when sea level falls). This can be a
consequence of either a fall in the amount of sea water or the land rising. Incised meanders are
meanders which are particularly well developed and occur when a river’s base level has fallen giving
the river a large amount of vertical erosion power, allowing it to down cut. Hence, 2nd statement is
correct.
There are two types of incised meanders, entrenched meanders and ingrown meanders.
As they are products of rejuvenation, they are not formed in mature stage of a river.
Hence, option D is correct
109. Ans. B.
A sinkhole (in Karst topography) is a hole in the ground that forms when water dissolves surface rock.
Often, this surface rock is limestone, which is easily eroded, or worn away, by the movement of
water.

PAGE 868
www.byjusexamprep.com

Formation of a Sinkhole: In a landscape where limestone sits underneath the soil, water from rainfall
collects in cracks in the stone. These cracks are called joints. Slowly, as the limestone dissolves and
is carried away, the joints widen until the ground above them becomes unstable and collapses. The
collapse often happens very suddenly and without very much warning. Water collects in these
collapsed sections, forming sinkholes.
Sinkholes also form when the roofs of caves collapse. Sinkholes are often funnel-shaped, with the
wide end open at the surface and the narrow end at the bottom of the pool.
Sinkholes vary from shallow holes about 1 meter (3 feet) deep, to pits more than 50 meters (165
feet) deep.
Hence, option B is correct
110. Ans. C.
An electric fuse is a device which is used to limit the current in an electric circuit. The fuse safeguards
the circuit and the electrical appliances from being damaged.
The fuse wire is generally an alloy of lead and tin. It has a low melting point and breaks the circuit if
the current exceeds a safe value.
The thickness and length of the fuse wire depends on the maximum current allowed through the
circuit.
It is connected in series in the beginning of the electric circuits.
If too many electrical appliances of high power rating (like electric iron, water heater etc) are
switched on at the same time, they draw an extremely large current from the circuit. This condition
is called overloading and it can cause overheating of the wiring and lead to a fire. (heating effect and
magnitude of the current)
Hence, option C is correct
111. Ans. C.
There are two conditions that must be met for an object to be in equilibrium. The first condition is
that the net force on the object must be zero for the object to be in equilibrium. If net force is zero,
then net force along any direction is zero.
Kinetic energy (KE) is the energy of motion, and kinetic energy is not always conserved in a collision
Energy and momentum are always conserved.
Hence, option C is correct
112. Ans. A.
Amount: 1 nanometer (nm) of length
Equals: 0.0000001 centimetres (cm) in length. This is equal to 10-7 cm.
Hence, option A is correct
113. Ans. D.
Sound is a kind of energy created when something vibrates.

PAGE 869
www.byjusexamprep.com

When this vibration reaches an ear, it is translated into what we recognize as a sound.
Sound vibration must travel through matter. Therefore, it can travel through air and water but not
through vacuum.
Hence, option D is correct
114. Ans. D.
Lord Kelvin, working in Scotland, developed the Kelvin scale in 1848. His scale uses molecular energy
to define the extremes of hot and cold. Absolute zero, or 0 K, corresponds to the point at which
molecular energy is at a minimum. The Kelvin scale is preferred in scientific work, although the
Celsius scale is also commonly used. Temperatures measured on the Kelvin scale are reported simply
as K, not °K.
T (in °C) + 273.15 = T (in K)
T (in K) − 273.15 = T (in °C)
Hence, option D is correct
115. Ans. A.
• Argon gas is used in fluorescent and incandescent light bulbs to stop the oxygen in the light
bulbs from corroding the hot tungsten filament.
• The use of argon in light bulbs prevents the evaporation of the tungsten filaments, which results
in increased light bulb life.
• Argon is also used to create an inert atmosphere for growing semiconductor crystals, arc
welding and for processes that need protection from other kinds of atmospheric gases.
Hence, option A is correct
116. Ans. B.
Fe2O3 + 3CO = 2Fe + 3CO2
Hematite + Carbon Monoxide = Iron + Carbon Dioxide
In this reaction, the iron oxide is reduced to iron, and the carbon is oxidised to carbon dioxide. Hence,
option B is correct
117. Ans. C.
Magnesium is essential for the formation of chlorophyll in green plants. Without magnesium,
chlorophyll cannot capture sun energy needed for photosynthesis. In short, magnesium is required
to give leaves their green color.
Hence, option C is correct
118. Ans. A.
Carbon dioxide (CO2) gas dissolved in water can cause water to become acidic.
The acidity of water from dissolved CO2 can be reduced by a base such as baking soda (sodium
bicarbonate).
Hence, option A is correct

PAGE 870
www.byjusexamprep.com

119. Ans. A.
Aldosterone is a key steroid hormone critical for maintaining salt and water balance. It regulates the
concentration of minerals, like sodium and potassium, in the fluid outside your cells.
When your sodium levels drop, or potassium levels rise, your body signals your adrenal glands to
release aldosterone into your blood.
Aldosterone signals your kidneys to reabsorb sodium into the blood, and acts on your sweat glands
to reduce sodium loss through sweat. Hence, option A is correct
120. Ans. B.
• When exposed to certain frequencies of radiation, such as radio waves, the subatomic particles
called electrons that orbit an atom's nucleus will "jump" back and forth between energy states.
• Clocks based on this jumping within atoms can, therefore, provide an extremely precise way to
count seconds.
• It is no surprise then that the international standard for the length of one second is based on
atoms.
• Since 1967, the official definition of a second is 9,192,631,770 cycles of the radiation that gets
an atom of the element called caesium to vibrate between two energy states.
Hence, option B is correct

PAGE 871
www.byjusexamprep.com

Elementary Mathematics
1. Ans. C.
2b = a + c and y2 = xz
Let the value of a. b and c to satisfy the equation be a = b = c = 1
Also, let the values of x, y and z
x=y=z=2
Xb-c yc-a za-b
= 21-1 21-1 21-1
= 20 20 20
= 1*1*1
=1
OR

2. Ans. B.
AB line segment is divided into two parts at point C, let CB = x
AC = 2-x
As per equation given in the statement (AC2 = AB * CB)
(2-x)2 = 2 * (x)
x2 +4-4x= 2x
x2 -6x + 4 = 0

PAGE 872
www.byjusexamprep.com

on solving this equation; x = 3 - √5


3. Ans. D.
πrl =1.76 * 104 cm2
22/7 * 70*l = 1.76 * 104
l = 80 cm
l2 = 6400
Also, l2 = r2 +h2
h2 = 6400 - 4900 = 1500
h = 10 √15
4. Ans. B.
Let side of square = x
Area = x2
After increasing; (X+8)2 = x2 +120
On solving this equation, we get, x = 3.5 cm
5. Ans. D.
Decimal expansion of an irrational number is non-terminating and non-repeating.
6. Ans. D.
x+y+z=0
From this,
y + z = -x
z + x = -y
x + y = -z
Putting these values on eq
(y + z - x)3 + (z + x - y)3 + (x + y - z)3
=(-x-x)3+(-y-y)3 +(-z-z)3
= (-2x)3+(-2y)3+(-2z)3
= -8(x3+y3+z3)
{using identity for (x + y + z)3 ,
x3 + y3 + z3 – 3xyz = (x + y + z) (x2 + y2 + z2 – xy – yz – zx)
If x + y + z = 0, then x3 + y3 + z3 – 3xyz = 0 or x3 + y3 + z3 = 3xyz}
=-8*3xyz
= -24xyz

PAGE 873
www.byjusexamprep.com

7. Ans. C.

8. Ans. B.
3x3 +4x2-7
Putting x =1;
3+4-7 = 0
Since x = 1 satisfies the equation then
(x-1) is a factor of the polynomial
Hence the correct answer is 1.
9. Ans. D.
3x3 + kx2 + 5x - 6 divided by (x+1)

If remainder is -7 so,
-6 - 8 + k = -7
Thus, K = 7
10. Ans. C.
Less than or equal to max(p, q)
if none of the highest degree in both polynomial cancel out in addition then degree of addition
polynomial will be max(p,q).

PAGE 874
www.byjusexamprep.com

If highest degrees in polynomial f(x) and g(x) cancel out then the degree of addition polynomial will
be less than max(p,q).
So option C is corrcet.
11. Ans. A.
{(√5-√3)/ √5+√3} – {(√5+√3)/ √5-√3}
= {(√5-√3)2 – (√5+√3)2}/5-3
=-4√5*√3/2
= -2√15
12. Ans. D.
Sin4θ -cos4θ
= (sin2θ - cos2θ) * (sin2θ + cos2θ)
= -cos2θ *1
= 1- 2cos2θ
13. Ans. A.
M ∝ 1/N
MN = K (CONSTANT)
15*-4 = -6*A
6A = 60
A = 10
Similarly,
-6*10 = 2B
B = -30
Similarly,
2*-30 = C*60
C = -1
14. Ans. C.
1/(1+xb-a+xc-a) + 1/(1+xa-b+xc-b) + 1/(1+xa-c+xb-c)
=xa/(xa+xb+xc) + xb/(xa+xb+xc) + xc/(xa+xb+xc)
(xa+xb+xc)/(xa+xb+xc) = 1
15. Ans. B.
cot 1 ° cot 23 ° cot 45 ° cot 67 ° cot 89 °
cot10 cot890 =1

PAGE 875
www.byjusexamprep.com

cot230 cot670=1
cot450 = 1
Thus, cot 1 ° cot 23 ° cot 45 ° cot 67 ° cot 89 ° = 1
16. Ans. A.
sec x cosec x = 2
sinx cosx = 1/2
sin2x=1=sin90
lthe value of x = 450
Putting this value in the above equation, √2 * √2 = 2
Tann x + Cotn x
Tann 45 + Cotn 45
= 1n+1n
=2
17. Ans. A.
Cos x + Cos2x = 1........(a)
Sin2x + Cos2x = 1
From both equations
Cos x = Sin2x
Putting this value in equation
Cos x + Cos2x = 1
Sin2x + Sin4x = 1
18. Ans. A.

Let AB = a and BC = b. Now, 2 cases may be possible in this question


Case I: a>b
Case II: a<b
In triangle ABC, Area = 1/2 AB*BC = 1/2 * OB*AC
=> 1/2 ab = 1/2 x*4x

PAGE 876
www.byjusexamprep.com

=> ab = 4x2 => 2ab = 8x2


Applying Pythagoras theorem in this triangle,
a2 + b2 = (4x)2 = 16x2
Now, (a + b)2 = a2 + b2 + 2ab => (a + b)2 = 16x2 + 8x2 = 24x2
Thus, (a + b) = 2√6 x
Similarly, (a - b)2 = a2 + b2 - 2ab
Thus, (a – b) = 2√2 x and -2√2 x (Considering Case I & II mentioned above)
Now,
Case I: (a + b) + (a - b) = 2√6 x + 2√2 x & (a + b) - (a - b) = 2√6 x - 2√2 x
=> a = (√6 + √2) x & b = (√6 - √2) x
Case II: (a + b) + (a - b) = 2√6 x - 2√2 x & (a + b) - (a - b) = 2√6 x + 2√2 x
=> a = (√6 - √2) x & b = (√6 + √2) x
Now, Tan C = a/b
So, Tan c = (√6 - √2) x/ (√6 - √2) x (Case I)
On rationalizing, Tan C = 2 + √3
For Case II, Tan C = 2 - √3
19. Ans. A.
Area of ΔABC = 1/2 * a*b
Also Area of ΔABC = 1/2* p*√ (a2+b2) , AB =√( a2+b2)
1/2ab = 1/2 * p * √( a2+b2)
a2b2 = p2(a2+b2)
20. Ans. A.
Area of a circle A = πr2
Area of greatest possible circle A’ = πr2/4
A-2A’ = πr2/2 = A/2
21. Ans. B.

For small cone,

PAGE 877
www.byjusexamprep.com

radius = r, height = h, slant height = l


For big cone,
radius = R, height = H, slant height = L
The triangles formed in the smaller and bigger cones are similar, hence,
r/R = h/H = l/L --- eq (1)
Now, (Volume of small cone/ Volume of Frustum) = 64/61 = k(constant)
Thus, volume of big cone = 64k + 61k = 125k
Volume of small come; V1/ Volume of big cone, V2 = (1/3 πr2h)/ (1/3 πR2H)
Also, V1/ V2 = 64m/125m = 64/125
So, r2h/ R2H = 64/125
From eq (1), r3/ R3 = 64/125; r/R = 4/5
Now, Ratio of curved surface area of small cone/ Ratio of curved surface area of big cone = πrl/ πRL
= (4/5)*(4/5) (from eq 1) = 16/25 = k constant
So, Curved surface area of frustum = 25k – 16k = 9k
Thus, Ratio of curved surface area of small cone/ Ratio of curved surface area of frustum = 16k/9k =
16:9
22. Ans. C.
Total area of room = 100 m2
Area of triangular table = √3
Area of 4 bookshelves = 4*4*1 = 16
Area of rest of room = 100 – (√3 - 16) = 82.268
Half of this area = 41.134
Cost of carpeting = 41.134 *100 = Rs. 4113
23. Ans. B.
pm = (rm + 1)/rm = 1 + 1/rm
For m=1, p1 = 1 + 1/r1
For m=2, p2 = 1 + 1/r2
For m=3, p3 = 1 + 1/r3
Also, r3 > r2 > r1
Thus, 1/r1 > 1/r2 > 1/r2
or, 1 + 1/r1 > 1 + 1/r2 > 1 +1/r2
or, p1 > p2 > p3
Thus, when m increases, value of p decreases. Hence, Option b is correct

PAGE 878
www.byjusexamprep.com

24. Ans. A.

The locus of the mid-points of the radii of length 16 cm of a circle is a concentric circle of radius 8 cm
Hence Option ‘a’ is correct.
25. Ans. C.
Among the given statements, Sentence 2 (The centroid of a triangle always lies inside the triangle)
and statement 3 (The orthocenter of a right-angled triangle lies on the triangle) are correct. Hence,
Option (c) is correct.
26. Ans. C.

The locus of a point equidistant from two intersecting lines is a straight line. Hence, option (a) is
correct
27. Ans. D.
There are three conditions of congruency. These are:
(a) Side-Angle-Side
(b) Angle-Side-Angle
(c) Side-Side-Side
Statement 1 says Angle-Angle-Angle property, which is not true. Statements 2,3,4 are correct. Hence
the answer is option (d).
28. Ans. C.
Area of one room to be painted
= 2(bh + hl) = 2((4*2.5) + (2.5*6)) = 50 m2

PAGE 879
www.byjusexamprep.com

Area of 5 rooms = 50*5 = 250m2


For painting 20m2 = 1 can is used
So, for painting 250m2, number of cans used:
= 250/20 = 12.5, i.e., approximately 13 cans
29. Ans. D.
Side of tiles = 50cm
Area of each tile = 50*50 = 2500 cm2
Area of rectangular pathway = (1000*450) cm2
Total tiles required for the pathway = 450000/2500 = 180 tiles.
Cost of 20 tiles = Rs. 100
Cost of 180 tiles = 100 * 180 / 20 = Rs. 900.
30. Ans. C.
2πr = 4a
πr = 2a
22/7 *98 = 2a
a = 154 cm
31. Ans. C.

As per question, AB = a; BC = b; CA = c; AD = p; BE = q and CF = r


AB+BD > AD
BC + CE > BE
CA +AF > CF
Adding the above 3 equations:
AB + BC + CA + (BC/2 +AC/2 +AB/2) > AD + BE + CF
3/2(AB + BC + CA) > (AD + BE + CF)
3(a + b + c) > 2(p + q + r)

PAGE 880
www.byjusexamprep.com

32. Ans. B.

As per question, AB = a; BC = b; CA = c; AD = p; BE = q and CF = r. Let G be the mid-point/ intersection


point of the 3 medians.
Now, in triangle AGC, using the triangle inequality property:
2/3r + 2/3p > c ---- eq (1)
In triangle BGC; 2/3q + 2/3 r > b ----- eq (2)
In triangle AGB, 2/3p + 2/3q > a ----- eq (3)
Adding (1), (2) & (3):
2/3r + 2/3p + 2/3q + 2/3 r + 2/3p + 2/3q > a + b + c
4 (p + q + r) > 3 (a + b + c)
Hence option (b) is correct.
33. Ans. B.
D1 *D2 = 50
Area of square = a2 = 1/2 D1*D2 (where ‘a’ is the side of the square)
a2 = 1/2 * 50 = 25
a = 5 units
34. Ans. B.
Surface area of cylindrical box = 2πrh + 2πr2
= 2* π/4 *d2 + πdh =352 (where d is the diameter = 2r)
d2 +2d *10 = 352/ π * 2
d2 +20d = 352/22 *7 *2 = 224
d2 + 20d - 224 = 0
d = 8 cm
35. Ans. D.
Let Side of triangle = a & Side of Square = b

PAGE 881
www.byjusexamprep.com

According to question,
3a =4b (Since their perimeters are same)
Diagonal of square = b√2 = 6√2
Hence, b= 6
So, a = 8
Area of triangle = √3/4 a2
= √3/4 * 64
=16 √3
36. Ans. B.

Let O be the center of the circle.


Now, angle OAX = angle OBX = 90o
In polygon AOBX,
Angles (AOB + OBX + BXA + XAO) = 360o
Thus, angle AOB = 360 – (90 + 90 + 50) = 130o
angle ACB = 1/2 angle AOB = 1/2 of 130 = 65o
37. Ans. A.
AD =DB = l/2 (since AB = l, given in the question)
Area of shaded region = Area of triangle (ABC – ADE)
= (√3l2/4) – (√3/4 * (l/2)2)
= 3 √3l2/16
38. Ans. B.
∠ QPT = α, ∠ OPT = 90o
∠OPQ = ∠OQP = 90 - α (isosceles triangle)
∠POQ = 180 – (90- α + 90 - α) (Sum of all the angles of triangle OPQ is 180o)
=2α

PAGE 882
www.byjusexamprep.com

Hence, option (b) is correct

–3 –2 –1 0 1 2 3

–1

–2

–3

39. Ans. D.

AB = CD = 10 cm, PB = 3 cm; AP = (10-3) = 7 cm, OC = 13 cm


Extending line MN, such that it is the diameter of the circle.
Since MN is the diameter; MN = 2OC = 2*13 = 26 cm
As per theorem of chords intersecting each other in a circle:
AP*PB = MP*PN
=> 7*3 = (MN – PN) *PN
=> 21 = (26 – PN) * PN
=> PN2 – 26 PN + 21 = 0
Applying formula,
PN = {-(-26) (+/-) √(-26)2 - (4*21)}/ 2 (Discarding the negative root)
PN = 13 + 2√37 or 13 - 2√37
Now, OP = ON – PN = 13 – (13 + 2√37) or 13 – (13 - 2√37)
Since the first case will yield a negative value, so we will discard it.
Thus, OP = 2√37

PAGE 883
www.byjusexamprep.com

40. Ans. C.
Given equation: a + b = 2c
Let the values of a =0, b= 1, c =1/2 to satisfy the above equation
Now, putting the value of a, b, c in the equation given in question
a/(a-c) + c/(b-c)
= 0/ (0-1/2) + 1/2/ (1-1/2)
= 0 + 1/2/ (1/2)
=1
OR
a+b=2c
a-c=c-b
Given equation
a/(a-c) + c/(b-c)
=a/(a-c)-c/(c-b)
=a/(a-c)-c/(a-c)
=(a-c)/(a-c)
=1
41. Ans. B.
Let the roots be x and 1/x.
Use the property of quadratic equations {x+y = -b/a , xy = c/a};
x*1/x = r/p
1 = r/p; r = p
42. Ans. A.
65x -33y = 97 => equation 1
33x – 65y = 1 => equation 2
adding both equation
98x-98y=98
x-y=1
subtracting both equation
32x+32y=96
x+y=3
After solving these two equations

PAGE 884
www.byjusexamprep.com

we get x=2 and y=1


So xy=2
43. Ans. A.
From eq 1
b/y + z/c = 1
z/c = 1- b/y
z/c = (y-b)/y …..(i)
from eq 2
c/z =1-x/a
z/c =a/ a-x…… (ii)
equating equations (i) & (ii)
(y-b)/y =a/(a-x)
Cross-multiplying
(ay – ab – xy + xb) = ay
Xb = ab + xy
(ab+xy)/xb = 1
44. Ans. A.
If (x+3) is a factor of x3 + 3x2 + 4x + k
So, equation will be completely divided by (x+3)

if k =12 so equation completely divided by x+3


From this we find that k = 12
45. Ans. B.
Let the smallest number of 4 digits is 1000

So the smallest 4 digits perfect square is, 322 = 1024


46. Ans. C.
ax2+ bx +c = 0; roots of this equation are α and β

PAGE 885
www.byjusexamprep.com

Thus, α + β = -b/a and α β = c/a


Now, (α+1)(β+1) = α β+ α +β+1
Putting the values;
= c/a –b/a +1
= (c – b + a)/ a
47. Ans. C.
let

Multiply both side by 10 and simplify.

48. Ans. A.
0.459459459……
= 459/999
= 51/111
= 17/37
49. Ans. D.
Let the numbers be = p, q and r
Now, p*q = 286 = 2 *13 * 11 (After factorization)
q*r = 770 = 11 * 7 *5 *2
Since the numbers are co-prime, so,
q = 2*11 = 22
p = 13
r = 35
Sum of the three numbers = 22 + 13 + 35 = 70
50. Ans. B.
Let the age of women = 10x + y
The age of husband = 10y +x

PAGE 886
www.byjusexamprep.com

According to question
(10y+x) – (10x +y) = 1/11 (10x+y+10y+x)
9y-9x = 1/11 (11x+11y)
9y - 9x = x + y ……(a)
8y = 10x
x = 8y/10
x= 4/5 y
Difference of their ages
9y - 9x = 4y/5 +y
9y - 9x = 9y/5
So, the difference of their ages is multiple of 9. Hence, option (b) is correct.
51. Ans. C.
Sum of 11 observation = 11*11 = 121
Sum of first 6 observation = 10.5*6 = 63
Sum of last 6 observation = 11.5 *6 = 69
Sum of first 6 & last 6 observations = 63 + 69 = 132
Thus, Sixth observation = 132-121 = 11
52. Ans. B.
Taking statement 1:
(Sec2θ - 1) * (1- cosec2θ) = 1
(1+tan2θ - 1) * (-cot2θ) = 1
tan2θ * (-cot2θ) = 1
-1 = 1 is not possible, Hence, statement 1 is wrong.
Taking statement 2,
sin θ (1 + cos θ)-1+ (1+cos θ) (sin θ)-1 = 2 cosec θ
sin θ/ (1 + cos θ) + (1 + cos θ)/sin θ
= (sin2θ +1 + cos2θ + 2cosθ)/ (sin θ + sin θ cos θ)
= 2 (1+cosθ)/ sin θ (1+cosθ)
= 2 cosec θ
So, statement 2 is true

PAGE 887
www.byjusexamprep.com

53. Ans. A.

Height of cone = √ (169-25) =12


Volume of cone = 1/3 π r2 * h
=1/3*22/7*25*12
=100 π
54. Ans. D.
r =1
l =3
Ratio of total surface area to curved surface rea = (πr2+ πrl)/ πrl = πr2/ πrl + 1
= r/l +1
= 1/3 + 1
= 4/3
Required ratio = 4:3
55. Ans. C.
Edge of cube = 2a
So, height of cone = 2a
Radius of cone = a (for maximum volume)
Volume of cone = 1/3 π a2 *2a = 2 πa3/3
56. Ans. B.
According to question
4 πr2 = 4/3 πr3
r = 3 cm
57. Ans. D.
According to question
2 πr/h = 3/1

PAGE 888
www.byjusexamprep.com

h = 2/3 πr
Curved surface area of cone = πrl = πr √(h2+r2)
= πr √ (4/9*π2 r2+ r2)
= {πr2√4 π2+9}/3
58. Ans. B.
Side of square; a = 2/√ π
For largest circular disc; Side of square = Diameter of disc
i.e., a = 2r
Area of circle; πr2 = π(a/2)2
= π*1/ π = 1
59. Ans. D.
In this case, Diagonal of square = Diameter of circle
√2a = 2r
Area of required region = πr2 – a2
= π(√2a/2)2 - a2
= πa2/2 – a2
= (π-2) a2/2
60. Ans. D.
517 + 518 + 519 + 520
= 517(1 + 5 + 52+ 53)
= 517(156)
156 is divisible by 13, and not by 7, 9 or 11. Hence the correct option is 13.
61. Ans. B.
Given that, x = y1/a, y = z1/b, z= x1/c
Taking log in the above-given equations.
log x = 1/a log y, log y = 1/b log z, log z = 1/c log x
Calculating the values of a, b and c:
a = log y/ log x
b = log z/ log y
c = log x/ log z
Now, a*b*c = 1

PAGE 889
www.byjusexamprep.com

62. Ans. A.
Let the numbers be x and y
x*y = LCM *HCF
xy = 21*3003
xy = 21* 273*11
According to the question both the numbers are greater than 21, hence the two numbers are:
xy = 231*(21*11), i.e. xy = 231*273
x + y = (231+273) = 504
63. Ans. B.
Total registered students = 2000
Students who did not appear = 2000/25 = 80
Total students who appeared = 2000 - 80 = 1920
Total students who passed = 1920*11/20 = 1056
64. Ans. C.
A:B = 1:2 → 3:6
B:C = 3:4 → 6:8
C:D = 2:3 → 8:12
D:E = 3:4 → 12:16
B:E = 6:16
B:E = 3:8
65. Ans. C.
10W * 12 = 8 * 5M
M = 3W
Let total days required to complete the complete work by 6 women and 3 men be ‘y’.
(6W+3M) y = 10W*12
(10W*12 is equal to the total work)
(6W+9W) y = 10W *12
15Wy = 10W *12
Y = 8 days
66. Ans. C.
Speed =60 km/hr = 60*5/18 m/sec
Time = Distance / Speed (distance is a length of train)

PAGE 890
www.byjusexamprep.com

30 = L/ 60*5/18
L = 500m
67. Ans. C.
A + B + C = 120
According to the question
B = A – 20
C = A + 20
Also, A+B+C = 120
Solving the above 3 equations.
A + A – 20 + A + 20 = 120
3A = 120
A = 40
Thus, B = 20
And C = 60
68. Ans. C.
Total age of father and son = 25*2 = 50
After 7 years,
Son’s age = s+7 = 17
Present age of son = 10
Present age of father = 40
After 10 years, Age of father = 50 years (40 + 10)
69. Ans. A.
Let the certain capital = x
x * 125/100 *125/100 *125/100 = 10000
x * 5/4 *5/4 *5/4 = 10000
x = 10000*4*4*4/125
x = 5120
70. Ans. A.
Given that, m’s value lies between 0 and 1.
Let the value of m = 1/2
⇒ m2 = 1/4
And m-1 = 2

PAGE 891
www.byjusexamprep.com

So, log(1/2) = -0.3010


Now taking the value of m = 1/4
⇒ m2 = 1/16
And m-1 = 4
So, log(1/4) = -0.6020
So, option (A) is true.
log m < m2 < m < m-1
71. Ans. C.
In all odd prime numbers, the unit digits are 1, 3, 5, 7, 9
So after multiplying these numbers, we get = 945
Hence, the unit’s digit is 5.
72. Ans. A.
ax2 + bx + c
When divided by x, dividend = ax + b & reminder = c
So, the value of c = 3
When divided by (x-1), dividend = ax + b + a & reminder => c + a + b = 6
Thus, a + b = 3 (since c = 3)
73. Ans. C.
Let the integers be x, x+1, x+2, x+3, x+4, x+5, x+6, x+7, x+8
Now, as per question,
(x + x+1+ x+2 + x+3 + x+4 + x+5 + x+6 + x+7 + x+8)/ 9 = 55
9x + 36 = 55*9
x = 51
Largest integer = x+8 = 59
74. Ans. B.
Speed in still water, Vb = x
Speed in flowing water, Vs = y
Total time taken by the man to row to & fro = z
Thus, z = d/ (x + y) + d/ (x - y) (where d is the distance between the two places)
z =d {x – y + x + y} /x2-y2
d = z (x2-y2)/ 2x

PAGE 892
www.byjusexamprep.com

75. Ans. C.
Total distance = 2d
So according to the question
2d/48 = d/60 + d/y
1/24 = 1/60 + 1/y
1/24 = (y+60)/60y
5y = 2y +120
3y = 120
y = 40 km per hour
76. Ans. B.
Let D’s share = x
E = 3x/2
B= x/2
C= 2x
A=3x
Shares of A+D+E = 3x + x + 3x/2 = 11x/2
Shares of B+C = 2x +x/2 = 5x/2
Difference = 3x = 13500
x= 4500
Shares of B+C+E = 4x = 4 * 4500 = 18000
77. Ans. C.
Let the two numbers be a and b
(a + b)/2 = 10
a + b = 20
Also, √ab= 8
ab = 64
a=64/b
Solving the above 2 equations,
a = 16 and b = 4

PAGE 893
www.byjusexamprep.com

78. Ans. B.

let a=l is the side of square


h is the height of tower

In triangle mPB

79. Ans. B.

In the initial figure, Tan α = (H - h)/AB, tan β = h/AB

PAGE 894
www.byjusexamprep.com

From 2nd figure


Tan α = (h + x)/AB
Equating both the values of tan α
(H - h)/AB = (h + x)/AB
x = H-2h
80. Ans. C.
Sin A + Cos A = p
Squaring both sides
Sin2A + Cos2A +2 sin A cos A = p2
1 + 2 sin A cos A = p2
sin A Cos A = (p2 -1)/2
Sin A + Cos A = p
Cubing both sides;
(Sin A + Cos A)3 = Sin3A + Cos3A + 3 Sin A Cos A (Sin A + Cos A)
P3 = q + 3(p2-1)/2 *p
P3 = q + 3p3/2 -3p/2
2p3 - 3p3 + 3p - 2q = 0
P3 - 3p + 2q = 0
81. Ans. D.
If x = (sec2θ - tan θ)/ (sec2θ + tan θ)
Let the values of θ = 45
So, x = (2-1)/ (2+1) =1/3
Since this holds true only for option 4, hence, it’s the correct answer.
(1st in incorrect since it states that the value of x lies between 1/3 & 3, excluding 1/3, hence it is not
correct)
82. Ans. D.
The highest power of 10 which would divide 25! Is greater than 5, hence, option (d) is correct.
25!=1*2*3*4*5...10...15...20...24*25
factor of 5 = 56
so highest power of 10 is 6
83. Ans. A.
Let the numbers are x and x2

PAGE 895
www.byjusexamprep.com

x2 + x = 20
x2 +x - 20 = 0
Solving this quadratic equation.
x= -5 and 4
84. Ans. C.
For cube to be of maximum volume, Diagonal of cube = Diameter of sphere
√3a = 2r
r = √3a/2
According to question,
Volume of Cube / Volume of sphere = a3/(4/3 πr3 )
Putting the value of r;
= a3 / (4/3 π(√3a/2)3)
On solving this ratio, we get 2/√ 3π
85. Ans. B.
Old price x and new price 1.25x
x * y = k (where y is the total consumption and ‘k’ is the budget)
1.25x * y’ = k (y’ is the new consumption)
Equating both the equations:
y’ = 100/125 y
y’ = 4/5 y
i.e. the new consumption of 4/5 of the original consumption. If original consumption was 100, new
consumption = 4/5 of 100 i.e. 80. So, the consumption must be reduced by 20%.
86. Ans. C.
Let total work = X
200M *150 = X
After 50 days
200M *50 = X/4
Remaining work = 3X/4
After 50 days, let ‘y’ workers be added to complete the work on time.
(200+y) M*100= 3X/4
(200+y) M *100 = 3 *200M *50
200+y = 300
Y = 100 men

PAGE 896
www.byjusexamprep.com

87. Ans. B.
Total gain by the person
= {5000*2 * 5.5 / 100} – {5000*2*5/100}
= {5000*2/100} * {5.5-5}
= 5000*2 *5/1000
= Rs. 50
88. Ans. C.
M *D/W = M’*D’/W’
5*5/5 = M’*50/100
M’ = 10 tractors
89. Ans. C.
Let the annual income = x
As per the conditions given in the question,
(x*1*4/100) – (x *1*3.75/100) = 64
x/100 *(4-3.75) = 64
x= 64*100/.25
x = Rs. 25600
90. Ans. A.
Total sum = Rs. 39000
Let the share of wife = x
So, share of each daughter = 2x
Share of each son = 6x
As per question,
5(6x) + 4(2x) + x = 39000
30x + 8x + x = 39000
39x = 39000
X = Rs. 1000
91. Ans. A.
Let the length of train A be l1 and the length of train B be l2. Let their respective speeds be Ua & Ub
Now, according to the question,
3{(l1+l2)/(Ua+Ub)} = (l1 +l2)/Ua-Ub
On solving the above equation,

PAGE 897
www.byjusexamprep.com

2Ua = 4Ub
Ua/Ub =2/1
92. Ans. A.
Total age of 15 students = 19*15 = 285
After 5 new students added, total age = 20*18.5 = 370
Sum of the ages of 5 new students = 370 -285 = 85
Average age of the 5 new students = (85/5) = 17
93. Ans. C.
P= 12
Q =10
R= -6
Taking S1 or S2
P + Q – R = 12 + 10 - 6 = 16lt/min
5 hours 45 min = 345 min
Volume of tank = 16*345 = 5520 liters
Now taking S1 and S3
15 hrs. 20 min = 920 min
Volume of tank = 6*920 = 5520 liters
Now taking S2, S3
Let the volume of tank v
v= [10+12 – v/920] *345
v = 22 * 345 – 345v/920
v + 69v/184 = 22*345
253v/184 = 22*345
V = 22*345*184/253
V = 5520 liters
Thus, any two of S1, S2 and S3 are sufficient.
94. Ans. A.
Let CP of the article = Rs. 100
Then, SP = 100 * 132/100 = 132
According to question,
CP is increased by 20% and SP remains same

PAGE 898
www.byjusexamprep.com

New CP = 100 *120/100 = 120


Profit % = (132 – 120)/ 120 * 100 = 12/120*100 = 10%
95. Ans. B.
The hour hand completes 3600 in (60*12) i.e. 720 minutes
Thus, it completes 1/20 in a minute.
So, in 10 minutes it covers 50
96. Ans. C.
Length of transverse common tangent = √{center distance2 – (r1 + r2)2}
= √ (100 - 64) = √36 = 6 cm
97. Ans. C.
100% corresponds to 3600
16.1% corresponds to 3600/100* 16.1 = 57.960 = 580
98. Ans. C.
The given statement is; The angles of the polygon are all equal and each angle is 90o.
This means that it is either a rectangle or a square. This makes statement 1 correct (i.e. the polygon
has exactly 4 sides).
Sum of interior angles of a polygon having ‘n’ sides is (n – 2) * 180o = (n-2) * 2 * 90o
i.e. sum of interior angles of a polygon having n sides is (2n – 4) right angles.
Hence only statement 1 is correct
99. Ans. C.
Both the given properties of lines are correct. Hence, option (c) is correct.
100. Ans. B.
Ratio of copper and tin in alloy A = 2:3
Ratio of copper and tin in alloy B = 3:4
20 kg taken from A:
Copper = 8 kg and tin = 12 kg
28 kg taken from B:
Copper = 12 kg and tin =16 kg
This is mixed with some pure cooper = x kg
Ratio of copper and tin in alloy C = 6:7
Total copper in alloy C/ total tin in alloy C = 6/7
(8 + 12 + x)/ (12 + 16) = 6/7
(20 + x)/ 28 = 6/7
x = 4 kg

PAGE 899
www.byjusexamprep.com

CDS II 2017
ENGLISH
Direction: In the given question six sentences of Q : This is so because the object to be
a passage are a given. The first and sixth attained is a matter of actual experiment in
sentences are given in the beginning as S1 and which you either succeed or fail.
S6. The middle four sentences in each have been
R : He must either correct his aim, or
jumbled up and labelled P, Q, R and S. You are
persevere in his error with his eyes open.
required to find the proper sequence of the four
sentences and mark your response accordingly. S : If a man aims at a mark with bow and
arrow, he must either hit it or aim it.
1. S1: Egypt lies in the north-east corner of
Africa. A. P S Q R B. R P S Q
S6: The whole country depends on the C. S Q R P D. Q S P R
water of Nile.
Direction: In the given question six sentences of
P : Most of it is desert or semi-desert. a passage are a given. The first and sixth
sentences are given in the beginning as S1 and
Q : It has very little rainfall.
S6. The middle four sentences in each have been
R : It is four times as big as Great Britain in jumbled up and labelled P, Q, R and S. You are
size. required to find the proper sequence of the four
sentences and mark your response accordingly.
S : Only a twenty-fifth of the total area is
cultivable. 3. S1 : Isaac’s mother married again.
A. P Q R S B. S R P Q S6 : He had a set of little tools and saw of
various sizes made by himself.
C. R P S Q D. Q P R S
P : But he was known to be very clever at
Direction: In the given question six sentences of making things.
a passage are a given. The first and sixth
sentences are given in the beginning as S1 and Q : She sent him to school.
S6. The middle four sentences in each have been
R : Isaac was left to the care of his good old
jumbled up and labelled P, Q, R and S. You are
grandmother.
required to find the proper sequence of the four
sentences and mark your response accordingly. S : In his early years Isaac did not appear to
be a very bright student.
2. S1 : In mechanical efforts, you improve by
Perpetual practice. A. R Q S P B. Q R S P
S6 : There is neither excuse nor temptation C. S Q R P D. R P Q S
for the latter.
Direction: In the given question six sentences of
P : He cannot go on shooting wide or falling a passage are a given. The first and sixth
short, and still fancy that he is making sentences are given in the beginning as S1 and
progress. S6. The middle four sentences in each have been

PAGE 900
www.byjusexamprep.com

jumbled up and labelled P, Q, R and S. You are S : Though he used the recorded history of
required to find the proper sequence of the four mankind, but he was interested not merely
sentences and mark your response accordingly. in the chronology of single states or group
but in the rise and fall of whole civilizations.
4. S1: The examination system must be
regarded as the chief wrecker of young A. P S Q R B. Q S P R
nerves.
C. S Q R P D. P Q S R
S6: If I become a Vice-Chancellor, my first
Direction: In the given question six sentences of
act would be to abolish all Examinations in
a passage are a given. The first and sixth
my university.
sentences are given in the beginning as S1 and
P: It makes me jump out of the bed, all in a S6. The middle four sentences in each have been
sweat. jumbled up and labelled P, Q, R and S. You are
required to find the proper sequence of the four
Q: It does this by building up a tension, for
sentences and mark your response accordingly.
a part of the year, all through one’s youth.
6. S1 : Science first began to become
R: And after four decades, the same
important after A.D. 1500.
nervousness sometimes recurs to me in
nightmares. S6 : Men read them, became inquisitive
again, and began to want to find things out.
S: I remember the desperate nervousness
that used to grip me from January to April P : As a result of this, books came to be
every year. circulated.
A. R P Q S B. R S P Q Q : During the Middle Ages the coming of
Science was hindered by the Church.
C. R Q P S D. Q S R P
R : In the middle of the fifteenth century,
Direction: In the given question six sentences of
however, the Turks captured the city of
a passage are a given. The first and sixth
Constantinople and the Greek books were
sentences are given in the beginning as S1 and
scattered far and wide.
S6. The middle four sentences in each have been
jumbled up and labelled P, Q, R and S. You are S : It considered free inquiry into the nature
required to find the proper sequence of the four of things to be wicked.
sentences and mark your response accordingly.
A. P Q S R B. Q S R P
5. S1 : History is a subject that is so little
C. S R P Q D. R P Q S
valued today that it is almost impossible to
win world fame as a historian; yet that is Direction: In the given question six sentences of
exactly what Toynbee was able to. a passage are a given. The first and sixth
sentences are given in the beginning as S1 and
S6 : Among the civilizations that he studied
S6. The middle four sentences in each have been
was that of India.
jumbled up and labelled P, Q, R and S. You are
P : We usually think of history as a required to find the proper sequence of the four
chronological account of the development sentences and mark your response accordingly.
of various states and empires under ruler.
7. S1 : Phatik was a mischievous boy of
Q : Toynbee’s view of history was different. fourteen.
R : He tries to find the pattern behind the S6 : Ultimately he distinguished himself as a
birth, growth and decay of civilization. scholar.

PAGE 901
www.byjusexamprep.com

P : It was then that Phatik’s uncle offered to 9. S1 : Ross sent an account of his work,
take the boy to Kolkata. together with slide and specimens
Q : She was much worried about his to Manson.
education. S6 : Ross was elected a fellow of the Royal
R : His mother found it difficult to bring him Society in 1901.
up. P : They produced a profound sensation.
S : Away from his home Phatik became Q : In July 1898, Manson described Ross’s
sober and industrious. results to the British Medical Association.
A. P Q R S B. S R Q P R : The President of the Royal Society came
to Manson’s house and inspected Ross’s
C. R S P Q D. R Q P S materials and said that ‘it was of
Direction: In the given question six sentences of remarkable interest and value’.
a passage are a given. The first and sixth S : When Manson had finished, the whole
sentences are given in the beginning as S1 and audience rose and cheered.
S6. The middle four sentences in each have been
A. R S P Q B. P S R Q
jumbled up and labelled P, Q, R and S. You are
required to find the proper sequence of the four C. Q P S R D. S P Q R
sentences and mark your response accordingly.
Direction: In the given question six sentences of
8. S1 : Whenever I met Baba Amte I was a passage are a given. The first and sixth
reminded of an anecdote my grand-mother sentences are given in the beginning as S1 and
used to tell me. S6. The middle four sentences in each have been
jumbled up and labelled P, Q, R and S. You are
S6 : He forgot that he had made it. required to find the proper sequence of the four
P : He once made an idol of God. sentences and mark your response accordingly.
10. S1 : Civilization dawned when early man
Q : As the idol was nearing completion, the
learnt how to produce heat and energy by
sculptor was becoming more and more
burning wood.
withdrawn into himself.
S6 : When they have been used, they
R : And the moment it was complete, he cannot be replaced.
threw away his chisel and hammer and
bowed to the idol of God he had just P : Then steam was used to produce
created. electricity.

S : There was a great sculptor. Q : In this century great use has been made
of oil and natural gas and the use of atomic
A. P Q R S B. S R P Q reactors also has provided another source
of energy.
C. S P Q R D. Q P R S
R : Much later, the first industrial revolution
Direction: In the given question six sentences of was based on the production of steam by
a passage are a given. The first and sixth burning coal.
sentences are given in the beginning as S1 and
S6. The middle four sentences in each have been S : But none of these fuels is renewable.
jumbled up and labelled P, Q, R and S. You are A. P R Q S B. R Q S P
required to find the proper sequence of the four
sentences and mark your response accordingly. C. R P Q S D. R P S Q

PAGE 902
www.byjusexamprep.com

Direction: In the given question six sentences of R : The first school maintains that if
a passage are a given. The first and sixth teaching methods are perfect, errors will
sentences are given in the beginning as S1 and never occur.
S6. The middle four sentences in each have been
S : They argue that we should concentrate
jumbled up and labelled P, Q, R and S. You are
on how to deal with errors, instead of on
required to find the proper sequence of the four
method of teaching.
sentences and mark your response accordingly.
11. S1 : Ghost is a subject which baffles A. Q S R P B. P S Q R
everyone everywhere throughout the C. Q P S R D. R Q P S
world.
Direction: In the given question six sentences of
S6 : Yet it is a subject which has held people a passage are a given. The first and sixth
spellbound and the belief in them sentences are given in the beginning as S1 and
continues to flourish. S6. The middle four sentences in each have been
P : But human beings have always been jumbled up and labelled P, Q, R and S. You are
curious to know more about them. required to find the proper sequence of the four
sentences and mark your response accordingly.
Q : Needless to say, such attempts have
proved to be useless. 13. S1 : Down the stairway of the Holiday Inn
hotel, I enter the conference hall.
R : There have been attempts even to
photograph these creatures of darkness. S6 : Some are learning against the sidewall.

S : Even after the advancement of science, P : I take a seat in the back row as more
the reality of ghosts remains a mystery till chairs
this day. Q : The hall is already packed with
A. Q R P S B. S Q P R delegates.

C. S P R Q D. S Q R P R : Still quite a few people are left standing.

Direction: In the given question six sentences of S : Most of the delegates are executives of
a passage are a given. The first and sixth Indian or Indo-US companies.
sentences are given in the beginning as S1 and
A. S Q R P B. P R Q S
S6. The middle four sentences in each have been
jumbled up and labelled P, Q, R and S. You are C. S R Q P D. Q S P R
required to find the proper sequence of the four
Direction: In the given question six sentences of
sentences and mark your response accordingly.
a passage are a given. The first and sixth
12. S1 : There have been two schools of sentences are given in the beginning as S1 and
thought which deal with the errors of S6. The middle four sentences in each have been
learners. jumbled up and labelled P, Q, R and S. You are
required to find the proper sequence of the four
S6 : Both views are popular today but the
sentences and mark your response accordingly.
second is gaining ground fast.
14. S1 : A sportsman is noted for his sense of
P : The philosophy of the second school is
discipline.
that errors are natural and they will occur
in any learning. S6 : Once discipline is accrued in the play
Q : So errors, they say, is a sign of faulty field, it can be applied and practised in
teaching methods. other spheres of life.

PAGE 903
www.byjusexamprep.com

P : The first lesson in discipline is to win 16. S1 : The miseries of the world cannot be
without pride and to lose without cured by physical help only.
bitterness. S6 : Then alone will misery ease in the
Q : One is no longer swayed by the sudden world.
gusts of passion. P : Let men have light, let them be strong
R : Then, one must learn that error or and educated.
selfishness will disgrace and endanger the Q : No amount of physical help will
rest.
remove them completely.
S : A sense of equanimity brings order and
method into the life of the people. R : Until man’s nature changes, his
A. Q P R S B. R Q S P physical needs will always rise, and miseries
will always be felt.
C. P Q S R D. P R S Q
S : The only solution is to make mankind
Direction: In the given question six sentences of
enlightened.
a passage are a given. The first and sixth
sentences are given in the beginning as S1 and A. Q P R S B. R Q S P
S6. The middle four sentences in each have been
C. S P Q R D. P Q R S
jumbled up and labelled P, Q, R and S. You are
required to find the proper sequence of the four Direction: In the given question six sentences of
sentences and mark your response accordingly. a passage are a given. The first and sixth
sentences are given in the beginning as S1 and
15. S1 : Mr. and Mrs. Robert went home late
S6. The middle four sentences in each have been
last night.
jumbled up and labelled P, Q, R and S. You are
S6 : Mr. Robert rushed to the police station required to find the proper sequence of the four
immediately. sentences and mark your response accordingly.
P : Somebody had broken open the lock. 17. S1 : Aristotle worked under limitations.
Q : To their dismay they found all their S6 : The age was not a period of
things missing. experiment.
R : They got into the house with a lot of fear. P : Physical events were mostly attributed
S : When they reached home they found to the intervention of God.
the front door open. Q : There had been little industrial
A. R S P Q invention in Greece, perhaps because slave
labour was cheap and plentiful.
B. S P R Q
R : The only equipment he had for his study
C. Q S R P was a ruler and compass and
D. R Q P S some crude instruments.
Direction: In the given question six sentences of S : The facts on which modern theories of
a passage are a given. The first and sixth science have been based had not
sentences are given in the beginning as S1 and
S6. The middle four sentences in each have been been discovered.
jumbled up and labelled P, Q, R and S. You are A. R P Q S B. R S P Q
required to find the proper sequence of the four
sentences and mark your response accordingly. C. Q S R P D. S Q R P

PAGE 904
www.byjusexamprep.com

Direction: In the given question six sentences of Direction: In the given question six sentences of
a passage are a given. The first and sixth a passage are a given. The first and sixth
sentences are given in the beginning as S1 and sentences are given in the beginning as S1 and
S6. The middle four sentences in each have been S6. The middle four sentences in each have been
jumbled up and labelled P, Q, R and S. You are jumbled up and labelled P, Q, R and S. You are
required to find the proper sequence of the four required to find the proper sequence of the four
sentences and mark your response accordingly. sentences and mark your response accordingly.
18. S1 : The bus sped along the road. 20. S1 : Hiuen-tsang became a Buddhist monk
at the age of twelve and soon discovered
S6 : The dog wailed for a long time.
that the Buddhist texts available in China
P : But the bus could stop only after were insufficient.
covering a few yards.
S6 : But he was on a quest and returned
Q : It injured the dog in the leg. after a while to his motherland with a rich
R : The driver applied the brake. collection of texts, documents and relics.

S : Suddenly a stray dog ran on to the P : Wherever he went, he was asked by the
middle of the road. local rulers and monks to stay in the place.

A. S P R Q B. S R P Q Q : He entered India through Kashmir,


where he spent some time in Srinagar.
C. R P Q S D. P R S Q
R : He therefore decided to go on a
Direction: In the given question six sentences of pilgrimage to India to collect further
a passage are a given. The first and sixth material.
sentences are given in the beginning as S1 and
S6. The middle four sentences in each have been S : From India, he attempted to go to
jumbled up and labelled P, Q, R and S. You are Ceylon, but gave up the attempt.
required to find the proper sequence of the four
A. P Q S R B. R S P Q
sentences and mark your response accordingly.
C. Q S R P D. R Q S P
19. S1 : The status of women in our country is,
on the whole, far from high. 21. Select the most appropriate option to fill in
the blank.
S6 : Education can lift women out of the
depths of misery and ignorance into which With the less rapid expansion of the
they have sunk. economy, we should make ____ progress
P : But the plight of women in villages is still toward stable price levels.
miserable. A. detailed B. substantial
Q : The educated women in cities enjoy C. definite D. infinite
equality with the men folk.
Direction: Complete the sentence below by
R : The movement for the freedom and choosing the correct word that will fit in the
right of women has certainly been steadily blank.
gaining momentum.
22. At times he gets very angry, and then no
S : Their education has been thoroughly one can _____________ him.
neglected.
A. prevent B. humour
A. R P Q S B. R Q P S
C. mollify D. satisfy
C. S Q P R D. S P Q R

PAGE 905
www.byjusexamprep.com

Direction: The following sentence has a blank 27. The clouds of suspicion will clear ____ soon.
space and four words are given below it. Select
A. up B. away
the word you consider most appropriate for the
blank and indicate your choice accordingly. C. off D. by
23. Many people today have fallen into utter Direction: The following sentence has a blank
confusion of values with the result that they space and four words are given below it. Select
cannot ____ the good from the bad. the word you consider most appropriate for the
A. divide B. differentiate blank and indicate your choice accordingly.

C. see D. alter 28. The teachers said that they were no longer
prepared to ____ the ways of the new
Direction: The following sentence has a blank Headmaster.
space and four words are given below it. Select
the word you consider most appropriate for the A. put over with B. put on with
blank and indicate your choice accordingly. C. put up with D. put up to
24. If Mohan ____ at 5 a.m., he would not have Direction: Select the most appropriate option to
missed the train. fill in the blank.
A. started B. had started 29. ___________ the construction of new
C. would start D. has started housing units at the rate of one every month,
there is still a shortage of accommodation.
Direction: In the following question, a sentence
is given with a blank to be filled in with an A. Through B. Despite
appropriate word. Select the correct alternative C. By D. For
out of the four and indicate it by selecting the
appropriate option. Direction: Select the most appropriate option to
fill in the blank.
25. His property was divided _________ his
daughters and sons. 30. Democracy requires the equal right of all to
the development of such capacity for good
A. between B. among
as nature has ___________ them with.
C. from D. with
A. presented B. endowed
Direction: The following sentence has a blank
C. fortified D. replenished
space and four words are given below it. Select
the word you consider most appropriate for the Direction: The following question consists of a
blank and indicate your choice accordingly. sentence, the parts of which have been jumbled.
These parts have been labelled P, Q, R and S.
26. His persistence in his misdemeanours has
Given below each sentence are four sequences
lowered him in the ____ of everyone who
namely A, B, C and D. You are required to
knows him.
re-arrange the jumbled parts of the sentence
A. eyes B. estimation and mark your response accordingly.
C. estimate D. esteem 31. My unmarried aunt, is creating a lot of
Direction: In the following question, a sentence problems for us (P)/ who stays with us, (Q)/
is given with a blank to be filled in with an because of her interfering nature in our
appropriate word. Select the correct alternative personal lives (R)/ and there is a
out of the four and indicate it by selecting the misunderstanding among family members
appropriate option. (S)

PAGE 906
www.byjusexamprep.com

A. Q S R P B. Q P S R The correct sequence should be


C. P Q S R D. S P Q R A. P Q R S B. Q P R S
Direction: The following question consists of a C. R S P Q D. R Q P S
sentence, the parts of which have been jumbled.
Direction: The following question consists of a
These parts have been labelled P, Q, R and S.
sentence, the parts of which have been jumbled.
Given below each sentence are four sequences
These parts have been labelled P, Q, R and S.
namely A, B, C and D. You are required to
Given below each sentence are four sequences
re-arrange the jumbled parts of the sentence
namely A, B, C and D. You are required to
and mark your response accordingly.
re-arrange the jumbled parts of the sentence
32. No criminal proceeding and mark your response accordingly.
in any Court during his term of office 34. The actress,
P has been selected
whatsoever shall be initiated P
Q as the best heroine
or continued against the President Q
R who is the daughter of a famous male
singer
or a Governor
R
S
of the year
The correct sequence should be
S
A. Q R S P B. P Q R S
The correct sequence should be
C. Q P S R D. S Q P R
A. R P Q S B. Q P S R
Direction: The following question consists of a
sentence, the parts of which have been jumbled. C. R S P Q D. Q S R P
These parts have been labelled P, Q, R and S.
Direction: The following question consists of a
Given below each sentence are four sequences
sentence, the parts of which have been jumbled.
namely A, B, C and D. You are required to re-
These parts have been labelled P, Q, R and S.
arrange the jumbled parts of the sentence and
Given below each sentence are four sequences
mark your response accordingly.
namely A, B, C and D. You are required to
33. The dentist with a severe tooth-ache re-arrange the jumbled parts of the sentence
and mark your response accordingly.
P
35. Last summer everyday
when he was brought to hospital
P
Q
kept pestering a pretty girl
extracted Manish’s tooth
Q
R
one persistent admirer
and relieved his pain
R
S

PAGE 907
www.byjusexamprep.com

with phone calls Direction: The following question consists of a


sentence, the parts of which have been jumbled.
S
These parts have been labelled P, Q, R and S.
The correct sequence should be Given below each sentence are four sequences
namely A, B, C and D. You are required to
A. Q R S P B. R S Q P re-arrange the jumbled parts of the sentence
C. R Q S P D. P Q R S and mark your response accordingly.

Direction: Each of the following items in this 38. My father retired at the age of 68,
section consists of a sentence, the parts of which where he had served,
have been jumbled. These parts have been
P
labelled as P, Q, R and S. Given below each
sentence are four sequences namely (a), (b), (c) in South Carolina
and (d). You are required to rearrange the
Q
jumbled parts of the sentence and mark your
response accordingly. as Pastor for 12 years,

36. to make wishes come true (P) that blue has R


the power (Q) in many cultures people from a Baptist Church
believe (R) and be successful in life (S)
S
A. PSRQ B. RSPQ
The correct sequence should be
C. RQPS D. SQPR
A. P R S Q B. S Q P R
Direction: The following question consists of a C. S P Q R D. Q P R S
sentence, the parts of which have been jumbled.
These parts have been labelled P, Q, R and S. Direction: The following question consists of a
Given below each sentence are four sequences sentence, the parts of which have been jumbled.
namely A, B, C and D. You are required to These parts have been labelled P, Q, R and S.
re-arrange the jumbled parts of the sentence Given below each sentence are four sequences
and mark your response accordingly. namely A, B, C and D. You are required to re-
arrange the jumbled parts of the sentence and
37. From a picnic table through the playground mark your response accordingly.
P 39. The completion
while we unpacked a basket It enables employees to feel a sense of
accomplishment
Q
P
we watched them laugh and leap
and makes them take pride in their work;
R
Q
bulging with sandwiches and cookies
of high quality products
S
R
The correct sequence should be also enhances employee satisfaction,
A. R S Q P B. Q P R S because

C. R P Q S D. Q S R P S

PAGE 908
www.byjusexamprep.com

The correct sequence should be Direction: The following question consists of a


sentence, the parts of which have been jumbled.
A. P Q R S B. R Q P S
These parts have been labelled P, Q, R and S.
C. P S R Q D. R S P Q Given below each sentence are four sequences
namely A, B, C and D. You are required to re-
Direction: The following question consists of a arrange the jumbled parts of the sentence and
sentence, the parts of which have been jumbled. mark your response accordingly.
These parts have been labelled P, Q, R and S.
Given below each sentence are four sequences 42. He said that a small college
namely A, B, C and D. You are required to re- P
arrange the jumbled parts of the sentence and
mark your response accordingly. he'd rather go to

40. But, Kuala Lumpur Q

where modern Malay executives not studying at all

P R

but will never miss Friday prayers instead of

Q S

might have a cellular phone in hand, The correct sequence should beA. Q S P R

R B. P R Q S
C. Q P R S
is a city firmly rooted in tradition
D. Q P S R
S
Direction: Each of the following item in this
The correct sequence should be
section consists of a sentence, the parts of which
A. R Q S P B. S P R Q have been jumbled. These parts have been
labelled P, Q, R and S. Given below each
C. R P S Q D. S Q R P sentence are four sequences namely (a), (b), (c)
Direction: Each of the following item in this and (d). You are required to rearrange the
section consists of a sentence, the parts of which jumbled parts of the sentence and mark your
have been jumbled. These parts have been responses accordingly.
labelled P, Q, R and S. Given below each 43. swim is becoming difficult (P)/ are few open
sentence are four sequences namely (a), (b), (c) ponds around (Q)/ and expensive since
and (d). You are required to rearrange the there (R)/ teaching a child how to (S)
jumbled parts of the sentence and mark your
responses accordingly. A. S P R Q B. Q R S P

41. is a failure of planning and (P)/ perhaps the C. S R Q P D. P R Q S


most significant factor in the growth of all Direction: The following question consists of a
metropolitan crimes, (Q)/ governance in sentence, the parts of which have been jumbled.
the urban sprawl (R)/ including crimes These parts have been labelled P, Q, R and S.
against the elderly (S) Given below each sentence are four sequences
A. Q S P R B. P R Q S namely A, B, C and D. You are required to re-
arrange the jumbled parts of the sentence and
C. Q R P S D. P S Q R mark your response accordingly.

PAGE 909
www.byjusexamprep.com

44. The doctor able to find out 46. She gave her old coat
P P
what had caused to a beggar
Q Q
the food poisoning the one with the brown fur on it
R R
had not been shivering with cold

S S

The correct sequence should be The correct sequence should be

A. S P R Q B. P R Q S A. P R Q S B. S Q P R

C. P R S Q D. S P Q R C. P Q R S D. R P Q S

Direction: The following question consists of a Direction: The following question consists of a
sentence, the parts of which have been jumbled. sentence, the parts of which have been jumbled.
These parts have been labelled P, Q, R and S. These parts have been labelled P, Q, R and S.
Given below each sentence are four sequences Given below each sentence are four sequences
namely A, B, C and D. You are required to re- namely A, B, C and D. You are required to re-
arrange the jumbled parts of the sentence and arrange the jumbled parts of the sentence and
mark your response accordingly. mark your response accordingly.

45. The officer was suspended 47. The medical teams

P at the ground said that the injured

being corrupt P

Q by the surging crowds,

from service Q

R included women and children

before his dismissal R


who were trampled
S
S
The correct sequence should be
The correct sequence should be
A. Q P S R B. Q P R S
A. P R Q S B. P Q R S
C. R S Q P D. R S P Q
C. Q P R S D. P S Q R
Direction: The following question consists of a
sentence, the parts of which have been jumbled. Direction: Each of the following item in this
These parts have been labelled P, Q, R and S. section consists of a sentence, the parts of which
Given below each sentence are four sequences have been jumbled. These parts have been
namely A, B, C and D. You are required to labelled P, Q, R and S. Given below each
re-arrange the jumbled parts of the sentence sentence are four sequences namely (a), (b), (c)
and mark your response accordingly. and (d). You are required to rearrange the

PAGE 910
www.byjusexamprep.com

jumbled parts of the sentence and mark your a university is essentially a community of
responses accordingly. students and teachers
48. goal does not look at (P)/ he who has (Q)/ S
the hurdles on his way (R)/ his eyes fixed on
The correct sequence should be
the (S)
A. S R Q P B. S P Q R
A. P Q R S B. S R P Q
C. P Q R S D. S Q P R
C. Q S P R D. R Q P S
Direction: Given below is a short passage. After
Direction: The following question consists of a
the passage, you will find some items based on
sentence, the parts of which have been jumbled.
the passage. First, read a passage and answer the
These parts have been labelled P, Q, R and S.
items based on it. You are required to select your
Given below each sentence are four sequences
answers based on the contents of the passage
namely A, B, C and D. You are required to re-
and opinion of the author only.
arrange the jumbled parts of the sentence and
mark your response accordingly. For many years, ship captains navigating the
waters of Antarctica have been intrigued by
49. We do not know when but we know
sightings of emerald icebergs. Scientists have
P now explained their mystery. There icebergs are
turned upside down. Icebergs are blocks of ice
The exact date of his death
that have broken off huge slabs of frozen snow
Q called ice shelves. Their green appearance
results from sea water that has frozen at the
for certain
bottom over hundreds of years. The frozen sea
R water has dissolved organic matter which gives
it a yellow tone and the fresh water ‘ice shelf’
Shakespeare was born
above has a blue tinge. When the iceberg turns
S upside down, it appears green through the visual
mix of yellow with the blue from below.
A. S P R Q B. P Q R S
51. What is the meaning of ‘intrigued’?
C. P S Q R D. S R Q P
A. Surprised B. Fascinated
Direction: The following question consists of a
sentence, the parts of which have been jumbled. C. Muffled D. Repulsed
These parts have been labelled P, Q, R and S.
52. What are ice shelves?
Given below each sentence are four sequences
namely A, B, C and D. You are required to re- A. They are huge pieces of chunks of ice
arrange the jumbled parts of the sentence and
B. They are frozen sea water
mark your response accordingly.
C. They are pieces of ice which look like
50. The purpose is to advance knowledge
shelves
P
D. They are huge pieces of ice which are
the two have to work together very old
Q 53. What are icebergs?
and disseminate it A. Huge chunks of ice floating on water
R B. Froze sea water

PAGE 911
www.byjusexamprep.com

C. Green ice 57. Forests are being destroyed in order to


D. Green yellow water below and blue A. provide land for agriculture
above
B. provide wood for fuel
54. When the iceberg turns upside down, it
C. kill dangerous animals
appears
D. provide necessities as well as needless
A. green B. yellow
comforts and pleasures
C. blue D. white
58. The evil effect of destroying Nature instead
Direction: Given below is a short passage. After of using it is seen in
the passage, you will find some items based on
A. the fall in production of our factories
the passage. First, read a passage and answer the
items based on it. You are required to select your B. the fall in our standard of living
answers based on the contents of the passage
C. the unfavourable changes in climate
and opinion of the author only.
D. frequent occurrence of epidemics
In its simple form, science has helped man to
protect himself from Nature and to overcome 59. Climatic changes and soil erosion are
natural obstacles to movement. But with the results of
advance of science, a situation has arisen in
which Nature need to be protected from man. A. scientific developments
He has used Nature’s own gifts, not only of metal B. nuclear explosion
but even the human brain, to attack Nature.
Forests are being destroyed not only to satisfy C. natural calamity
need but to provide luxuries. The evil effects of D. deforestation
deforestation are already making themselves
clearly felt in climatic changes and soil erosion. Direction: Given below is a short passage. After
Man has at last begun to learn that he has to the passage, you will find some items based on
protect if he wants Nature to protect him. the passage. First, read a passage and answer the
items based on it. You are required to select your
55. The use of science in its simple form has answers based on the contents of the passage
helped man to and opinion of the author only.
A. do such things as building shelter and According to the civil laws of most countries
make carts, boats, etc. obedience is no longer the duty of a wife; every
B. make bombs and missiles woman has the political right to vote; but these
liberties remain theoretical as long as she does
C. build factories using machinery not have economic freedom. A woman
D. make planes supported by a man is not liberated from the
man.
56. Nature now needs to be protected from
man because It is through gainful employment that woman
has travelled most of the distance that separated
A. nature has become weak her from the male; and nothing else can
guarantee her liberty in practice.
B. man is rapidly destroying Nature
I once heard a maidservant declare, while
C. man is cruel to animals
cleaning the stone floor of a hotel lobby, “I never
D. man has become irrational asked anybody for anything; I succeeded all by

PAGE 912
www.byjusexamprep.com

myself.” She was as proud of her self-sufficiency 63. “These liberties” in the first paragraph refer
as a Rockefeller, Ford or Birla. to
However, the mere combination of the right to A. The right to vote, not to obey and right
vote and a job does not mean complete to a job
liberation : working, today, is not a liberty.
B. The right to vote and not to obey
A recent study of women workers in a car factory C. The rights of servants to disobey their
shows that they would prefer to stay in the home master and the right of the master to
rather than work in the factory. The majority of punish them
women do not escape from the traditional
feminine world. Their jobs at the factory do not D. Women’s right to vote and earn money
relieve them of housekeeping burdens; they get Direction: Given below is a short passage. After
from neither society nor their husbands, the the passage, you will find some items based on
assistance they need to become in concrete fact the passage. First, read a passage and answer the
eh equals of men. items based on it. You are required to select your
60. Which of the following helps women most answers based on the contents of the passage
to achieve equality with men? and opinion of the author only.

A. The right to vote During the past three generations the diseases
affecting western societies have undergone
B. Civil liberties dramatic changes. Polio, diphtheria,
tuberculosis, commonly known as TB, are
C. A job
vanishing; one injection of an antibiotic often
D. Wealthy husbands cures deadly diseases such as pneumonia or
syphilis; and so many mass killers have come
61. Why does the writer talk about the under control that two-thirds of all death are
maidservant in the hotel lobby? now associated with the diseases of old age.
A. The servants of today will one day be Thos who die young are more often than not
freed from their rich masters victims of accidents, violence, or suicide.
The changes in health status are generally
B. A servant can become as rich as
equated with the decrease in suffering and
Rockefeller or Birla
attributed to more or better medical care.
C. Even with a low paid job women can Almost everyone believes that at least one of his
achieve equality friends would not be alive and well except for the
skill of a doctor. But there is in fact no evidence
D. Economic independence is necessary of any direct relationship between this change in
for women’s liberation the pattern or nature of sicknesses on the one
62. In which paragraph does the writer say that hand and the so-called progress of medicine on
it is revealed that some women would not the other hand. These changes are the results of
like to work in the factory? political technological changes. They are not
related to the activities that require the
A. In paragraph four preparation and status of doctors or the costly
equipment in which doctors take pride. In
B. In paragraph three
addition, an increase in the number of new
C. In paragraph two diseases in the last fifteen years are themselves
the result for medical intervention. They are
D. In paragraph one doctor-made or iatrogenic.

PAGE 913
www.byjusexamprep.com

64. In the western societies, the occurrence of basic services like nutritional diet, health care
polio, diphtheria and tuberculosis has and education. It results into a loss of cultural
identity and destroys traditional knowledge.
A. increased
Poor people become marginalised and suffer
B. completely stopped from exploitation and loss of human dignity.
C. decreased 67. Which of the following sentences comes
close to the meaning of the sentence,
D. continued without changes
“Poverty embraces a whole range of
65. More death are now associated with old circumstances.”
age than in the past because
A. There are lot of angles to poverty
A. iatrogenic diseases are spreading faster
B. There are several section in the society
now
which are poor
B. deadly diseases affecting the young
have been well controlled C. There are several types of poverty

C. accidents, violence and suicide that D. Poverty is solely an economic issue


killed many youths in the past are now 68. What way do you think ‘lack of access to
under control information’ affects poor people?
D. political and technological changes A. They don’t get information about how
now take better care of the young than to improve their conditions
the old
B. They didn’t get newspapers to read at
66. The writer probably is arguing for all
A. stopping the practice of western C. They can’t go to school and read books
medicines completely
D. They don’t get information about
B. stopping the use of costly equipment schemes of getting rich
and medicines
69. Why do you think ‘cultural identity’ is
C. rethinking about the successes and important?
failures of the western medicines
A. A sense of cultural identity gives people
D. giving greater attention to new, self-respect and confidence
iatrogenic diseases than to the old
diseases such as polio, diphtheria and B. Cultural identity defines the character
pneumonia of poor people

Direction: Given below is a short passage. After C. It is important to have cultural identity
the passage, you will find some items based on to get jobs
the passage. First, read a passage and answer the D. It is useful to have cultural identity
items based on it. You are required to select your because it brings your success
answers based on the contents of the passage
70. Which of the following sentences comes
and opinion of the author only.
closest to the sentence ‘Poor people
Poverty is a complex problem. It is far more than become marginalised’?
an economical condition. We measure it usually
in terms of income but forget that poverty A. They are not given any benefit of any
embraces a whole range of circumstances, government schemes
including lack of access to information and to B. They are ignored by the rich people

PAGE 914
www.byjusexamprep.com

C. They are the most ignored elements of 73. CV Raman was one of the greatest sons of
the society India
D. They are the most disposed elements A
of the society
who has earned everlasting fame
71. What do you think is the tone of the
B
passage?
for scientific researches.
A. Objective but querulous
C
B. Descriptive and impassioned
No error.
C. Argumentative and critical
D
D. Objective and critical
A. A B. B
Direction: The following question has a sentence
with three parts labelled a, b and c. Read each C. C D. D
sentence to find out whether there is any error
Direction: The following question has a sentence
in any underlined part and indicate your
with three parts labelled a, b and c. Read each
response on the Answer Sheet against the
sentence to find out whether there is any error
corresponding letter i.e., A or B or C. If you find
in any underlined part and indicate your
no error, your response should be indicated as D.
response on the Answer Sheet against the
72. It is identification with the audience corresponding letter i.e., A or B or C. If you find
no error, your response should be indicated as D.
A
74. This box
that makes one come home from the play
so much more A

B is heavy than

satisfied than one ever is after merely B


passive enjoyment of the show. the other one.
C C
No error. No error.
D D
A. A B. B A. A B. B
C. C D. D C. C D. D
Direction: The following question has a sentence Direction: The following question has a sentence
with three parts labelled a, b and c. Read each with three parts labelled a, b and c. Read each
sentence to find out whether there is any error sentence to find out whether there is any error
in any underlined part and indicate your in any underlined part and indicate your
response on the Answer Sheet against the response on the Answer Sheet against the
corresponding letter i.e., A or B or C. If you corresponding letter i.e., A or B or C. If you find
find no error, your response should be indicated no error, your response should be indicated as D.
as D.

PAGE 915
www.byjusexamprep.com

75. The writer does not have the freedom in this profession?
A C
to choose his own themes, No error.
B D
society thrusts them on him. A. A B. B
C C. C D. D
No error.
Direction: The following question has a sentence
D with three parts labelled a, b and c. Read each
A. A B. B sentence to find out whether there is any error
in any underlined part and indicate your
C. C D. D response on the Answer Sheet against the
Direction: The following question has a sentence corresponding letter i.e., A or B or C. If you find
with three parts labelled a, b and c. Read each no error, your response should be indicated as D.
sentence to find out whether there is any error 78. I know that
in any underlined part and indicate your
response on the Answer Sheet against the A
corresponding letter i.e., A or B or C. If you find ignorance is not bliss,
no error, your response should be indicated as D.
B
76. No one knows
yet I am ignorant in many things.
A
C
as to why he did it,
No error.
B
or who was behind his doing it. D

C A. A B. B

No error. C. C D. D
D Direction: The following question has a sentence
with three parts labelled a, b and c. Read each
A. A B. B
sentence to find out whether there is any error
C. C D. D in any underlined part and indicate your
Direction: The following question has a sentence response on the Answer Sheet against the
with three parts labelled a, b and c. Read each corresponding letter i.e., A or B or C. If you find
sentence to find out whether there is any error no error, your response should be indicated as D.
in any underlined part and indicate your 79. You are (A) just sixteen years old (B), isn’t
response on the Answer Sheet against the it? (C) No error (D)
corresponding letter i.e., A or B or C. If you find
no error, your response should be indicated as D. A. A B. B

77. How long C. C D. D

A Direction: Each item in this section has a


sentence with three underlined parts labelled
you are (A), (B) and (C). Read each sentence to find out
B whether there is any error in any underlined part

PAGE 916
www.byjusexamprep.com

and indicate your response in the answer sheet Direction: The following question has a sentence
against the corresponding letter i.e. (A) or (B) or with three parts labelled a, b and c. Read each
(C). If you find no error, your response should be sentence to find out whether there is any error
indicated as (D). in any underlined part and indicate your
response on the Answer Sheet against the
80. The old widower (A) living in remittances
corresponding letter i.e., A or B or C. If you find
from his sons, (B) could not make both ends
no error, your response should be indicated as D.
meet(C) No error(D)
83. Each student
A. A B. B
C. C D. D A

Direction: Each item in this section has a from amongst the hundred students in the
sentence with three underlined parts labelled class
(A), (B) and (C). Read each sentence to find out B
whether there is any error in any underlined part
and indicate your response in the answer sheet want to watch this movie.
against the corresponding letter i.e. (A) or (B) or C
(C). If you find no error, your response should be
indicated as (D). No error.
81. Debate about biotechnology and genetic D
engineering (A) is under way around the
A. A B. B
world, (B) and India is fully engrossed with
the discussion. (C) No error. (D) C. C D. D
A. A B. B Direction: The following question has a sentence
with three parts labelled a, b and c. Read each
C. C D. D
sentence to find out whether there is any error
Direction: The following question has a sentence in any underlined part and indicate your
with three parts labelled a, b and c. Read each response on the Answer Sheet against the
sentence to find out whether there is any error corresponding letter i.e., A or B or C. If you find
in any underlined part and indicate your no error, your response should be indicated as D.
response on the Answer Sheet against the
corresponding letter i.e., A or B or C. If you find 84. Although there is virtually no production in
no error, your response should be indicated as D. India,

82. I like to A

A the Encyclopaedia Britannica estimate

listen the song of the nightingale B


B that India has perhaps the largest
accumulated stocks of silver in the world.
in the evening.
C
C
No error. No error.

D D

A. A B. B A. A B. B

C. C D. D C. C D. D

PAGE 917
www.byjusexamprep.com

Direction: The following question has a sentence Direction: The following question has a sentence
with three parts labelled a, b and c. Read each with three parts labelled a, b and c. Read each
sentence to find out whether there is any error sentence to find out whether there is any error
in any underlined part and indicate your in any underlined part and indicate your
response on the Answer Sheet against the response on the Answer Sheet against the
corresponding letter i.e., A or B or C. If you find corresponding letter i.e., A or B or C. If you find
no error, your response should be indicated as D. no error, your response should be indicated as D.
85. We have to reach 87. What most students need, above all else

A A

there at ten is practice in writing


B
B
and particularly in writing things that
will you please walk little faster.
matter to them.
C C
No error. No error.
D D
A. A B. B A. A B. B
C. C D. D C. C D. D

Direction: The following question has a sentence Direction: The following question has a sentence
with three parts labelled a, b and c. Read each with three parts labelled a, b and c. Read each
sentence to find out whether there is any error sentence to find out whether there is any error
in any underlined part and indicate your in any underlined part and indicate your
response on the Answer Sheet against the response on the Answer Sheet against the
corresponding letter i.e., A or B or C. If you corresponding letter i.e., A or B or C. If you
find no error, your response should be indicated find no error, your response should be indicated
as D. as D.

86. It is almost difficult, 88. She was out of the mind,


A
A
when she made that plan to go abroad
in case impossible,
B
B
without taking into consideration her
to keep awake late after dinner. present family position.
C C
No error. No error.
D D

A. A B. B A. A B. B

C. C D. D C. C D. D

PAGE 918
www.byjusexamprep.com

Direction: The following question has a sentence 91. The Department of Fine Arts has been
with three parts labelled a, b and c. Read each criticised
sentence to find out whether there is any error
in any underlined part and indicate your A
response on the Answer Sheet against the for not having much required courses
corresponding letter i.e., A or B or C. If you find
no error, your response should be indicated as D. B

89. Are you scheduled for this semester.

A C

through with No error.

B D

that newspaper? A. A B. B

C C. C D. D

No error. Direction: The following question has a sentence


with three parts labelled a, b and c. Read each
D sentence to find out whether there is any error
A. A B. B in any underlined part and indicate your
response on the Answer Sheet against the
C. C D. D corresponding letter i.e., A or B or C. If you find
no error, your response should be indicated as D.
Direction: The following question has a sentence
with three parts labelled a, b and c. Read each 92. If you have thought about the alternatives,
sentence to find out whether there is any error
in any underlined part and indicate your A
response on the Answer Sheet against the you would not have chosen
corresponding letter i.e., A or B or C. If you find
no error, your response should be indicated as D. B

90. A. My college such difficult topic for the term paper.

B. is besides C

C. the lake. No error.

D. No error. D

A. A B. B A. A B. B

C. C D. D C. C D. D

Direction: The following question has a sentence Direction: The following question has a sentence
with three parts labelled a, b and c. Read each with three parts labelled a, b and c. Read each
sentence to find out whether there is any error sentence to find out whether there is any error
in any underlined part and indicate your in any underlined part and indicate your
response on the Answer Sheet against the response on the Answer Sheet against the
corresponding letter i.e., A or B or C. If you find corresponding letter i.e., A or B or C. If you find
no error, your response should be indicated as D. no error, your response should be indicated as D.

PAGE 919
www.byjusexamprep.com

93. The duties of the secretary are A. A B. B


A C. C D. D
to take the minutes, mailing the Direction: The following question has a sentence
correspondence, with three parts labelled a, b and c. Read each
sentence to find out whether there is any error
B in any underlined part and indicate your
and calling the members before meeting. response on the Answer Sheet against the
corresponding letter i.e., A or B or C. If you find
C no error, your response should be indicated as D.
No error. 96. A. The old furnitures
D B. was disposed of
A. A B. B C. And the new ones were placed.
C. C D. D D. No error.
Direction: In the sentence, identify the segment A. A B. B
which contains the grammatical error.
C. C D. D
94. If I was you, I would not go to watch a film Direction: Each of the following sentences in this
in my mother’s absence. section has a blank space with four words or
A. If I was you group of words given. Select whichever word or
group of words you consider most appropriate
B. I would not go for the blank space and indicate your response
C. to watch a film in accordingly.

D. my mother’s absence Whenever I go into a bank, I feel scared.


Everybody and everything that I see there
Direction: The following question has a sentence (Q_97) me. As for the manager the sight (Q_98)
with three parts labelled a, b and c. Read each him simply terrifies me and (Q_99) me want to
sentence to find out whether there is any error runaway (Q_100) I can. As soon as I (Q_101) the
in any underlined part and indicate your door of the bank I lose my head (Q_102) when I
response on the Answer Sheet against the try to do any (Q_103) there, I behave like an
corresponding letter i.e., A or B or C. If you find idiot. I cannot explain (Q_104) for this but that is
no error, your response should be indicated as D. how it (Q_105) has been that is how it is (Q_106).
95. Those of us who have a family history of 97. Find the appropriate word in each case.
heart disease
A. pleases B. frightens
A
C. saddens D. terrifies
should make a yearly appointment
98. Find the appropriate word in each case.
B A. of B. at
with their doctors. C. by D. on
C 99. Find the appropriate word in each case.
No error. A. shapes B. makes
D C. shields D. asks

PAGE 920
www.byjusexamprep.com

100. Find the appropriate word in each case. A. died after retirement
A. as slow as B. died before retirement
B. as fast as C. died while riding a horse
C. as steadily as D. died for a great cause
D. as actively as Direction: For the expression which has been
underlined in each of the following sentences,
101. Find the appropriate word in each case.
choose the response (A), (B), (C) or (D) which
A. open B. close most nearly expresses its meaning.

C. shut D. see 108. Your ambition should be in consonance


with your capabilities.
102. Find the appropriate word in each case.
A. parallel to
A. or B. and
B. in accordance with
C. either D. neither
C. in harmony with
103. Find the appropriate word in each case.
D. in tune with
A. service B. business
Direction: For the expression which has been
C. deed D. act underlined in each of the following sentences,
104. Find the appropriate word in each case. choose the response (A), (B), (C) or (D) which
most nearly expresses its meaning.
A. the reasons
109. I could not go up in life for want of proper
B. the responses guidance.
C. the answers A. for lack of
D. the causes B. for need of
105. Find the appropriate word in each case. C. for the desire of
A. always B. no time D. for the necessity of
C. any time D. many times Direction: For the expression which has been
106. Find the appropriate word in each case. underlined in each of the following sentences,
choose the response (A), (B), (C) or (D) which
A. then B. now most nearly expresses its meaning.
C. later D. after 110. Though he was arrogant, I could not
dispense with his services.
Direction: For the expression which has been
underlined in each of the following sentences, A. terminate B. align with
choose the response (A), (B), (C) or (D) which
most nearly expresses its meaning. C. claim D. disregard with

107. The institution decided to give Mrs. Roy’s Direction: For the expression which has been
job to her son not because she was a social underlined in each of the following sentences,
worker, but because she had died in choose the response (A), (B), (C) or (D) which
harness. most nearly expresses its meaning.

PAGE 921
www.byjusexamprep.com

111. In spite of his hard work, he was made a A. hefty B. thin


scapegoat for the failure of the project.
C. disillusioned D. determined
A. freed from any responsibility for
Direction: The following question consists of a
B. suspected of causing sentence with an emboldened word/words
followed by four words. Select the option that is
C. blamed without reason for
opposite in meaning to the emboldened
D. was severely punished for word/words and mark your response
accordingly.
Direction: For the expression which has been
underlined in each of the following sentences, 115. There was a mammoth gathering to listen
choose the response (A), (B), (C) or (D) which to the leader.
most nearly expresses its meaning.
A. negligible
112. If food supply fails to keep pace with
B. tiny
population, civilization will collapse.
C. poor
A. to grow along with
D. large
B. to walk side by side
Direction: The following question consists of a
C. to gain momentum
sentence with an emboldened word/words
D. to move at the same speed as followed by four words. Select the option that is
opposite in meaning to the emboldened
Direction: The following question consists of a
word/words and mark your response
sentence with an emboldened word/words
accordingly.
followed by four words. Select the option that is
opposite in meaning to the emboldened 116. The audience thoroughly enjoyed the
word/words and mark your response hilarious drama.
accordingly.
A. amusing
113. Whether the rewards are in commensurate
B. delightful
with the efforts or not, a society will always
have workaholics and the shirk work C. serious
groups.
D. momentous
A. disproportionate
Direction: The following question consists of a
B. equal to sentence with an emboldened word/words
followed by four words. Select the option that is
C. matched
opposite in meaning to the emboldened
D. unparalleled word/words and mark your response
accordingly.
Direction: The following question consists of a
sentence with an emboldened word/words 117. The writer’s erudition in science is revealed
followed by four words. Select the option that is in every page of the book.
opposite in meaning to the emboldened
A. unenlightened
word/words and mark your response
accordingly. B. ignorance
114. Wars leave behind a large number of C. intelligence
emaciated soldiers in the camps of both the
D. hollowness
victorious and the vanquished.

PAGE 922
www.byjusexamprep.com

Direction: The following question consists of a 119. The question is not whether the court
sentence with an emboldened word/words vindicates him with regard to his
followed by four words. Select the option that is involvement in the case, but how he feels
opposite in meaning to the emboldened about it.
word/words and mark your response
accordingly. A. reprieves B. absolves

118. The seminar which Ravi organised proved C. indicts D. summons


to be momentous event.
Direction: The following question consists of a
A. trivial sentence with an emboldened word/words
followed by four words. Select the option that is
B. futile
opposite in meaning to the emboldened
C. vain word/words and mark your response
D. useless accordingly.

Direction: The following question consists of a 120. In those days many monarchs enjoyed vast
sentence with an emboldened word/words ecclesiastical powers.
followed by four words. Select the option that is
A. permanent B. temporal
opposite in meaning to the emboldened
word/words and mark your response C. contemporary D. constitutional
accordingly.

PAGE 923
www.byjusexamprep.com

General Knowledge
1. Which one of the following is not included A. Union Basic Income
in the National Air Quality Index?
B. Undefined Basic Income
A. Sulphur
C. Unconditional Basic Income
B. Nitrogen dioxide
D. Universal Basic Income
C. Lead
6. Which one of the following statements
D. Methane about the Global Environment Facility
2. Which one of the following is not a feature Grant Agreement signed by India in August
of the Saansad Adarsh Gram Yojana 2017 with the World Bank for ‘Ecosystems
(SAGY)? Service Improvement Project’, is not
correct?
A. It focuses on community participation.
A. The size of the project is bout USD 25
B. It is to be guided by a Member of the million?
Parliament
B. The duration of the project is 15 years.
C. It aims at creating infrastructure for the
village. C. The project will entirely be financed by
the World Bank out of is GEF Trust
D. A Village Development Plan would be
Fund.
prepared for every identified Gram
Panchayat. D. The Ministry of Environment, Forest
and Climate Change will implement the
3. ‘Pragati’ scholarship scheme of the
project.
Government of India is meant for
A. higher education of girls 7. The Setu Bharatam Programme of the
Government of India is a programme for
B. technical education of girls
A. building bridges across major rivers for
C. secondary education of girls transport of goods
D. elementary education of girls B. building bridges for safe and seamless
4. Which one of the following authorities has travel on national highways
recently launched the mobile apps ‘My C. building roads to connect coastal areas
FASTag’ and FASTag Partner’?
D. training of highway engineers at the
A. Telecom Regulatory Authority entry level and service level
B. National Highways Authority
8. Where is the world’s first Partition Museum
C. Airports Economic Regulatory situated?
Authority
A. New Delhi B. Lahore
D. National Disaster Management
Authority C. Amritsar D. Islamabad

5. UBI, as an alternative for subsidies in 9. Why was constitutional amendment


poverty alleviation, stands for needed for introducing GST?

PAGE 924
www.byjusexamprep.com

A. States were not willing to agree with A. The powers of the Union Executive
the Union for introduction of GST extend to giving directions to the States
without amendment in the concerning the exercise of their
Constitution. powers.
B. GST was to be implemented on B. The Union Executive can issue a
concurrent base and Article 246 was provision relating to reduction of
inadequate for such a case salaries of employees of the State
Governments.
C. The Empowered Committee of Finance
Ministers had recommended for C. Governors have no emergency power
constitutional amendment like the President of India
D. The GST Council had recommended for D. If the Governor of a State is satisfied
constitutional amendment so that its that a situation has arisen whereby the
power enhances financial stability or credit of the Sate is
threatened, he may declare financial
10. The all-women expedition of Indian Navy to
emergency in the State
circumnavigate the globe on the sailing
vessel, INSV Tarini, is scheduled to have 15. Which one of the following statements
four stopovers. Which one of the following about the All India Services is correct?
is not one of them?
A. The All India Services may be created by
A. Fremantle B. Lyttelton an Act of the Parliament
C. Port Stanley D. Durban B. The endorsement of the Rajya Sabha is
11. Which one of the following was the theme not essential for the creation of the All
of the World Environment Day, 2017? India Services

A. Green Economy : Does it include you? C. The rules of recruitment to the All India
Services are determined by the UPSC
B. Connecting People to Nature
D. The conditions of service to the All India
C. Think, Eat, Save Services may be altered by the UPSC
D. Many Species, One Planet, One Future 16. Which one of the following statements
12. Venus, the first environmental research about the Gupta period in Indian History is
satellite, was launched in August 2017 by not correct?
which one of the following countries? A. Sanskrit language and literature, after
A. India B. Russia centuries of evolution, reached what
has been described as a level of
C. China D. Israel classical excellence through royal
patronage.
13. The National Handloom Day is observed on
____________. B. The status of women was redefined.
A. 7th June B. 17th July They were entitled to formal education
and hence there were women
C. 7th August D. 17th September teachers, philosophers and doctors.
Early marriage was prohibited by law
14. Which one of the following statements
and they were given the right to
about emergency provisions under the
property.
Constitution of India is not correct?

PAGE 925
www.byjusexamprep.com

C. Decentralization of administrative 19. Which one of the following statements


authority was impacted by increased regarding Uniform Civil Code as provided
grants of land and villages with fiscal under Article 44 of the Constitution of India
and administrative immunities to is not correct?
priests and temples.
A. It is Fundamental Right of every Indian
D. Land grants paved the way for feudal citizen.
developments and emergence of B. The State shall endeavour to secure it
serfdom in India, resulting in the for citizens throughout the territory of
depression of the peasantry. India.
17. Match List-I with List-II and select the C. It is not enforceable by any Court.
correct answer using the code given below
the Lists: D. It is not enforceable by a Court yet the
Constitution requires that as a principle
List-I List-II it should be fundamental in the
(Harappan (Modern governance of our country.
site) Name)
20. Match List-I with List-II and select the
A. Dholavira 1. Saurashtra correct answer using the code given below
the Lists:
B. Rakhigarhi 2. Hisar
List-I List-II
C. Bhirrana 3. Kadir Island
(States of NE (Location in
D. Bhogavo 4. Haryana India) Map)
A. Tripura

Codes: B. Mioram

A B C D C. Nagaland

A. 1 4 2 3 D. Manipur

B. 1 2 4 3
C. 3 2 4 1 Codes:

D. 3 4 2 1 A B C D

18. Which of the following was/were founded A. 2 1 4 3


by Raja Ram Mohan Roy? B. 2 4 1 3
1. Atmiya Sabha C. 3 1 4 2
2. Brahmo Samaj D. 3 4 1 2
3. Prarthana Samaj 21. Which of the following statements is/are
correct?
4. Arya Samaj
1. Himalayan rivers have their origin in
Select the correct answer using the code the snow-covered areas, hence are dry
given below. in winter season.
A. 1, 2 and 3 B. 2 only 2. Rivers of the Peninsular Plateau have
C. 1 and 2 only D. 1, 3 and 4 reached maturity.

PAGE 926
www.byjusexamprep.com

3. Himalayan rivers depict all the three 24. ‘Tuvalu’ has become a point of discussion
stages of normal cycle of erosion. recently. Why?
Select the correct answer using the code A. Potato plant that could grow in high
given below. altitude

A. 2 only B. 1 and 3 only B. Place in equatorial Africa, where snow


is found
C. 2 and 3 only D. 1, 2 and 3
C. New innovative technology to meet
22. Match List-I with List-II and select the global warming
correct answer using the code given below
D. A country under threat of submergence
and Lists:
due to ice melting and sea level rise
List-I List-II 25. Which of the following lakes is/are situated
(Lake) (Type of Lake) in Ladakh?
A. Bhimtal 1. Lagoon 1. Tso Kar
B. Ashtamudi 2. Landslide 2. Pangong Tso
C. Gohna 3. Tectonic 3. Tsomgo

D. Lonar 4. Crater 4. Tso Moriri


Select the correct answer using the code
given below.
Codes:
A. 1 only B. 2 and 3 only
A B C D
C. 1, 2 and 4 D. 2 and 4 only
A. 3 1 2 4 26. Harshacharita has references to various
B. 3 2 1 4 presents sent by a ruler named Bhaskara to
Harshavardhan A. Bhaskara belonged to
C. 4 2 1 3
A. Haryanka Dynasty of Magadha
D. 4 1 2 3
B. Varman Dynasty of Assam
23. Which of the following statements related
C. Nanda Dynasty of North India
to latitude are true?
D. None of the above
1. Rainfall, temperature and vegetation
vary with latitude 27. Which one of the following statements
about the Harappan Culture is not correct?
2. The difference between the longest day
and the shortest day increases with A. It witnessed the first cities in the
latitude subcontinent.
B. It marks the first use of script, written
3. Indira Point is located approximately at
from right to lift.
6 °45’ N latitude
C. It marks the earliest known use of iron
Select the correct answer using the code
as a medium for the art of sculpting.
given below.
D. It marks the earliest known use of
A. 1 and 2 only B. 1 and 3 only stone as a medium for the art of
C. 2 and 3 only D. 1, 2 and 3 sculpting.

PAGE 927
www.byjusexamprep.com

28. Which river is praised in the fifth century 34. A writ issued to secure the release of a
Tamil epic, Silappadikaram? person found to be detained illegally is
A. Cauvery B. Godavari A. Mandamus B. Habeas corpus
C. Saraswati D. Ganges C. Certiorari D. Prohibition
29. Who among the following scholars argued 35. Constitutional safeguards available to Civil
that “capital created underdevelopment Servants are ensured by
not because it exploited the A. Article 310 B. Article 311
underdeveloped world, but because it did
not exploit it enough”? C. Article 312 D. Article 317
A. Bill Warren B. Paul Baran 36. A Money Bill passed by the Lok Sabha can
be held up by the Rajya Sabha for how
C. Geoffrey Kay D. Lenin many weeks?
30. Which twelfth century Sanskrit scholar was A. Two B. Three
first responsible for the compilation of
‘Nibandhas’ or digests of epic and Puranic C. Four D. Five
texts? 37. Which one of the following criteria got the
A. Harsha B. Govindachandra highest weight for determination of shares
of States in the formula given by the 14th
C. Lakshmidhara D. Kalidasa Finance Commission?
31. Who among the following was the Chief A. Population
Justice of India when Public Interest
Litigation (PIL) was introduced in the Indian B. Income distance
Judicial System? C. Area
A. M. Hidayatullah B. A.S. Anand D. forest cover
C. A.M. Ahmadi D. P.N. Bhagwati 38. Which of the following statements in
32. The National Commission for Women was respect to the Indian Peninsular Plateau are
created by correct?
1. The Deccan Plateau gradually rises
A. an amendment in the Constitution of
from North to South.
India
2. The Malwa Plateau dominates the
B. a decision of the Union Cabinet
Vindhyan scarps, forming the Eastern
C. an Act passed by the Parliament flank of the plateau.
D. an order of the President of India 3. The Western Satpuras separate the
Narmada and Tapi rivers.
33. Which one of the following cannot be
introduced first in the Rajya Sabha? 4. The Chota Nagpur Plateau is composed
of Archaean granite and gneiss rocks.
A. Constitutional Amendment
Select the correct answer using the code
B. CAG Report given below.
C. Annual Financial Statement A. 1, 2, 3 and 4 B. 1, 3 and 4 only
D. Bill to alter the boundaries of any State C. 2 and 4 only D. 1 and 3 only

PAGE 928
www.byjusexamprep.com

39. Match List-I with List-II and select the 41. Who was the editor of the journal Indian
correct answer using the code given below Social Reformer that was started in 1890?
the Lists:
A. K.T. Telang
List-I List-II
B. Veeresalingam
(Mountain (Location in Map)
Range) C. N.G. Chandavarkar
A. Satpura D. K.N. Natarajan
B. Vindhya 42. Match List-I with List-II and select the
correct answer using the code given below
C. Aravalli the Lists:
D. Mahadeo List-I List-II
(Type of (Meaning)
Land)
Codes:
A. Urvara 1. Land watered
A B C D by a river
A. 4 2 1 3 B. Maru 2. Fertile land
B. 3 1 2 4 C. Nadimatrika 3. Land watered
C. 3 2 1 4 by rain

D. 4 1 2 3 D. Devamatrika 4. Desert land


40. Match List-I with List-II and select the
correct answer using the code given below
Codes:
the Lists:
A B C D
List-I List-II
(Industrial (Country) A. 2 1 4 3
Region)
B. 3 4 1 2
A. Leipzig 1. USA
C. 3 1 4 2
B. Detroit 2. UK D. 2 4 1 3
C. Lorranine 3. France 43. With regard to nature of Mughal State, who
D. Cumberland 4. Germany among the following scholars argued that
“the peculiar feature of the State in Mughal
India was that it served not merely as the
Codes: protective arm of the exploiting classes, but
was itself the principal instrument of
A B C D exploitation”?
A. 2 1 3 4 A. Irfan Habib
B. 4 3 1 2 B. Satish Chandra
C. 4 1 3 2 C. Athar Ali
D. 2 3 1 4 D. J.F. Richards

PAGE 929
www.byjusexamprep.com

44. Consider the following: 46. In India, the base year of the new GDP
series has been shifted from 2004-05 to
1. Tughlaqabad Fort
A. 2007-08 B. 2008-09
2. Bada Gumbad in Lodhi Garden
C. 2010-11 D. 2011-12
3. Qutub Minar
47. Which of the following indicators have
4. Fatehpur Sikri been used by the World Economic Forum to
calculate Global Competitiveness Index for
Which one of the following is the correct
2016-17?
chronological order of building the above
monuments? 1. Efficiency enhancer subindex
A. 3, 1, 4, 2 B. 3, 1, 2, 4 2. Innovation and sophistication factors
subindex
C. 1, 3, 2, 4 D. 1, 3, 4, 2
3. Life expectancy enhancer subindex
45. Match List-I with List-II and select the
correct answer using the code given below Select the correct answer using the code
the Lists: given below.
A. 1 and 2 only
List-I List-II
(Type of (Explanation) B. 2 and 3 only
Deficit)
C. 1 and 3 only
A. Fiscal 1. Total Expenditure- D. 1, 2 and 3
Deficit Revenue Receipts
and Non-debt 48. Due to which of the following factors, the
Capital Receipts Industrial Revolution took place in England
in the eighteenth century?
B. Revenue 2. Revenue
Deficit Expenditure- 1. The discovery of coal and iron deposits
Revenue Receipts 2. The discovery of steam power
C. Effective 3. Revenue Deficit- 3. The introduction of railways
Revenue Deficit
4. The regular supply of raw materials
Grants for Creations
of Capital Assets Select the correct answer using the code
given below.
D. Primary 4. Fiscal Deficit-
Deficit Interest Payments A. 1 and 2 only B. 2, 3 and 4 only
C. 1 and 3 only D. 1, 2, 3 and 4
49. Who among the following was not a
Codes:
member of the Drafting Committee of the
A B C D Constituent Assembly?

A. 1 2 3 4 A. N.G. Ayyangar

B. 1 3 2 4 B. K.M. Munshi

C. 4 2 3 1 C. B.N. Rau
D. Muhammad Saadulah
D. 4 3 2 1

PAGE 930
www.byjusexamprep.com

50. Article 21 of the Constitution of India 4. The Legislative Assemblies of NCT of


includes Delhi and Puducherry
1. Rights of transgenders Select the correct answer using the code
given below.
2. Rights of craniopagus twins
3. Rights of mentally retarded women to A. 1 and 2 only B. 1, 2 and 3
bear a child C. 1, 2 and 4 D. 3 and 4
Select the correct answer using the code 53. Which of the following statements
given below. is/are correct?
A. 1 only B. 1 and 2 only 1. The Directive Principles of State Policy
C. 2 and 3 only D. 1, 2 and 3 are meant for promoting social and
economic democracy in India.
51. Consider the following statements about
the Scheduled Castes and the Scheduled 2. The Fundamental Rights enshrined in
Tribes under the provisions of the Par III of the Constitution of India are
Constitution of India: ordinarily subject to reasonable
restrictions.
1. State can make any special provision
relating to their admission to the 3. Secularism is one of the basic features
Government educational institutions. of Constitution of any country.

2. State can make any special provision Select the correct answer using the code
relating to their admission to the given below.
private educational institutions aided A. 1 only B. 2 only
by the State.
C. 1 and 2 only D. 1, 2 and 3
3. State can make any special provision
relating to their admission to the 54. A Joint Sitting of the Parliament is resorted
private educational institutions not to, for resolving the deadlock between two
aided by the State. Houses of the Parliament for passing which
of the following Bills?
4. State can make any special provision
relating to their admission to the 1. Money Bill
minority educational institutions as
2. Constitutional Amendment Bill
described in Article 30.
3. Ordinary Bill
Which of the statements given above are
correct? Select the correct answer using the code
given below.
A. 1 and 2 only B. 3 and 4 only
A. 1 only B. 2 and 3 only
C. 1, 2 and 3 only D. 1, 2, 3 and 4
52. The President of India is elected by an C. 3 only D. 1, 2 and 3
Electoral College comprising of elected 55. Which of the following statements about
members of which of the following? the 73rd and 74th Constitution Amendment
1. Both the Houses of the Parliament Act is/are correct?

2. The Legislative Assemblies of States 1. It makes it mandatory for all States to


establish a three-tier system of
3. The Legislative Councils of States Government.

PAGE 931
www.byjusexamprep.com

2. Representatives should be directly A. urea into sulphur dioxide


elected for five years.
B. sugar into carbon dioxide
3. There should be mandatory reservation
of one-third of all seats in all C. lipids into methane
Panchayats at all levels for women. D. urea into ammonia
Select the correct answer using the code
61. Consider the following statements:
given below.
1. The chain reaction process is used in
A. 1 and 2 only B. 2 and 3 only
nuclear bombs to release a vast amount
C. 1, 2 and 3 D. 3 only of energy, but in nuclear reactors, there
is no chain reaction.
56. The Fundamental Rights guaranteed in the
Constitution of India can be suspended only 2. In a nuclear reactor, the reaction is
by _________. controlled, while in nuclear bombs, the
A. A proclamation of National Emergency reaction is uncontrolled.

B. An Act passed by the Parliament 3. In a nuclear reactor, all operating


reactors are ‘critical’, while there is no
C. An amendment to the Constitution of question of ‘criticality’ in case of a
India nuclear bomb.
D. The judicial decisions of the Supreme 4. Nuclear reactors do not use
Court moderators, while nuclear bombs use
57. Tincture of iodine is an antiseptic for fresh them.
wounds. It is a dilute solution of elemental
Which of the above statements about
iodine, which does not contain
operational principles of a nuclear reactor
A. water B. acetone and a nuclear bomb is/are correct?
C. alcohol D. potassium iodine A. 1 and 3 B. 2 and 3
58. Which one of the following polymers does C. 4 only D. 1 and 4
not contain glucose units?
62. A person throws an object on a horizontal
A. Glycogen B. Starch frictionless plane surface. It is noticed that
C. Cellulose D. Rubber there are two forces acting on this object –
(i) gravitational pull and (ii) normal reaction
59. Desalination of seawater is done by using of the surface. According to the third law of
reverse osmosis. The pressure applied to motion, the net resultant force is zero.
the solution is Which one of the following can be said for
A. larger than osmotic pressure the motion of the object?
B. smaller than osmotic pressure A. The object will move with acceleration.
C. equal to osmotic pressure B. The object will move with deceleration.
D. equal to atmospheric pressure C. The object will move with constant
60. The foul smell of urine of a healthy man speed, but varying direction.
having healthy food, when spilled on floor, D. The object will move with constant
is mainly due to the bacterial velocity.
decomposition of ______.

PAGE 932
www.byjusexamprep.com

63. Ultrasonic waves are produced by making 69. Match List-I with List-II and select the
use of correct answer using the code given below
the Lists:
A. ferromagnetic material
List-I List-II
B. magnetic material
(National (Route)
C. piezoelectric material Highway)
D. pyroelectric material A. NH-2 1. Delhi-Jaipur-
Ahmedabad-
64. Joule-Thomson process is extremely useful
Mumbai
and economical for attaining low
temperature. The process can be B. NH-4 2. Thane-Pune-
categorized as Bengaluru-Chennai
A. isobaric process C. NH-7 3. Delhi-Agra-
Allahabad-Kolkata
B. isoenthalpic process
D. NH-8 4. Varanasi-Jabalpur-
C. adiabatic process Nagpur-
D. isochoric process Hyderabad-
Bengaluru-Maduri-
65. ‘Xeriscaping’ is a concept related to Kanyakumari
A. landscaping related to save water
Codes:
B. landscaping related to save soil
A B C D
C. weathering of rock surface
A. 3 4 2 1
D. All of the above B. 1 2 4 3
66. Which of the following radioactive C. 1 4 2 3
substances enters/enter the human body
D. 3 2 4 1
through food chain and causes/cause many
physiological disorders? 70. Match List-I with List-II and select the
correct answer using the code given below
A. Strontium-90 B. Iodine-131 the Lists:
C. Cesium-137 D. All of these List-I List-II
67. An emulsion consists of (Concern) (Product)
A. one liquid and one solid A. TISCO 1. Chemicals

B. one liquid and one gas B. BALCO 2. Iron and Steel


C. BPCL 3. Electronics
C. two liquids
D. BEL 4. Aluminium
D. two solids
68. One carbon credit is accepted as equivalent Codes:
to A B C D
A. 100 kg of carbon A. 2 1 4 3
B. 100 kg of carbon dioxide B. 2 4 1 3
C. 1000 kg of carbon C. 3 4 1 2
D. 1000 kg of carbon dioxide D. 3 1 4 2

PAGE 933
www.byjusexamprep.com

71. Match List-I with List-II and select the 73. Which one of the following planets was
correct answer using the code given below explored by Cassini Mission launched by
the Lists: NASA, which ended in September 2017?

List-I List-II A. Sun


(Pass) (Place)
B. Neptune
A. Zoji La 1. Himachal Pradesh
C. Saturn
B. Shipki La 2. Uttrakhand
D. Jupiter
C. Lipulekh 3. Jammu and
Kashmir 74. Which one of the following political parties
was launched by Irom Sharmila in Manipur?
D. Nathu La 4. Sikkim
A. People’s Resurgence and Justice
Alliance
Codes:
B. Manipur Resistance Alliance
A B C D
C. Tribal Resistance Party
A. 4 2 1 3
B. 4 1 2 3 D. Revolutionary People’s Party

C. 3 1 2 4 75. The Nobel Prize in Physics for the year 2016


was given to
D. 3 2 1 4
A. David J. Thouless
72. Match List-I with List-II and select the
correct answer using the code given below B. F. Duncan M. Haldane
the Lists:
C. J. Michael Kosterlitz
List-I List-II
(Strait) (Countries) D. All of them

A. Bass 1. UK and France 76. Who among the following is the recipient of
Strait the Dadasaheb Phalke Award, 2016?
B. Davis 2. Australia and A. Nana Patekar
Strait Tasmania
B. Manoj Kumar
C. Dover 3. USA and Cuba
Strait C. Javed Akhtar
D. Florida 4. Canada and D. K. Viswanath
Strait Greenland
77. Teejan Bai, a recipient of the M.S.
Subbulakshmi Centenary Award, 2016, is an
Codes: exponent in
A B C D
A. Kannada classical vocal
A. 2 4 1 3
B. Kajari dance
B. 2 1 4 3
C. Bihu dance
C. 3 4 1 2
D. Pandavani, a traditional performing art
D. 3 1 4 2

PAGE 934
www.byjusexamprep.com

78. The Sustainable Development Goals 4. Buddha held that nuns could attain
(SDGs), which were adopted by the UNO in spiritual liberation just like a monk and
place of the Millennium Development granted them an equal status in the
Goals (MDGs), 2015, aim to achieve the 17 mendicant order.
goals by the year___.
Select the correct answer using the code
A. 2020 B. 2030 given below.
C. 2040 D. 2050 A. 1, 2, 3 and 4 B. 1, 2 and 3 only

79. Who among the following is the recipient of C. 1, 2 and 4 only D. 3 and 4 only
the Jnanpith Award, 2016? 83. Which one of the following statements
A. Shankha Ghosh about the Sayyid brothers during the period
of Later Mughals is not true?
B. Raghuveer Chaudhari
A. They brought Jahandar Shah to power.
C. Pratibha Ray
B. They wielded administrative power.
D. Rehman Rahi
C. They followed a tolerant religious
80. Who among the following is the author of policy.
the book, The Social Contract?
D. They reached an agreement with King
A. Voltaire B. Hobbes Shahu.
C. Locke D. Rousseau E. None of the above/More than one of
the above
81. Which one of the following combinations of
year and event concerning the French 84. Which one of the following peasant
Revolution is correctly matched? struggles was an outcome of British opium
policy?
A. 1789 : Napoleonic Code
A. Phulaguri Dhawa (1861)
B. 1791 : Tennis Court Oath
B. Birsaite Ulgulan (1899-1900)
C. 1792 : National Convention
C. Pabna Revolt (1873)
D. 1804 : New Constitution of France
D. Maratha Peasant Uprising (1875)
82. Which of the following statements with
regard to the heterodox sect between sixth Direction: The following five (5) items consist of
and fourth century BC are correct? two statements, Statement I and Statement II.
Examine these two statements and select the
1. The Jain ideas were already being correct answer using the code given below.
circulated in the seventh century BC by
Parshva. 85. Statement I: The early Aryans, who were
essentially pastoral, did not develop any
2 Although Buddhism, and to a lesser political structure which could measure up
extent Jainism, took account of the to a State in either ancient or modern
changes in material life and reacted sense.
against orthodoxy, neither of these
Statement II: Kingship was the same as
sought to abolish the caste system.
tribal chiefship; the term Rajan being used
3. The first female disciple of Mahavira is for tribal chief who was primarily a military
said to have been a captured slave leader and who ruled over his people and
woman. not over any specified area.

PAGE 935
www.byjusexamprep.com

A. Both the statements are individually 88. Statement I: The interior part of
true and Statement II is the correct Maharashtra does not receive adequate
explanation of Statement I rain in the summer season.
B. Both the statements are individually Statement II: The interior part of
true but Statement II is not the correct Maharashtra lies in the rain shadow of the
explanation of Statement Western Ghats.
C. Statement I is true but Statement II is A. Both the statements are individually
false true and Statement II is the correct
D. Statement I is false but Statement II is explanation of Statement I
true
B. Both the statements are individually
86. Statement I: The communists left the All true but Statement II is not the correct
India Trade Union Congress in 1931. explanation of Statement
Statement II: By 1928, the Communists C. Statement I is true but Statement II is
were no longer working with the false
mainstream national movement.
D. Statement I is false but Statement II is
A. Both the statements are individually true
true and Statement II is the correct
explanation of Statement I 89. Statement I: Skin cancer is generally caused
by the ultraviolet radiation.
B. Both the statements are individually
true but Statement II is not the correct Statement II: Stratosphere allows
explanation of Statement ultraviolet radiation to enter the earth from
C. Statement I is true but Statement II is the sun.
false A. Both the statements are individually
D. Statement I is false but Statement II is true and Statement II is the correct
true explanation of Statement I

87. Statement I: Global warming signifies the B. Both the statements are individually
rise in global surface temperature. true but Statement II is not the correct
explanation of Statement
Statement II: The increase of concentration
of greenhouse gases in the atmosphere C. Statement I is true but Statement II is
causes the rise in global surface false
temperature.
D. Statement I is false but Statement II is
A. Both the statements are individually true
true and Statement II is the correct
explanation of Statement I 90. Devaluation of currency will be more
beneficial if prices of
B. Both the statements are individually
true but Statement II is not the correct A. domestic goods remain constant
explanation of Statement
B. exports become cheaper to importers
C. Statement I is true but Statement II is
false C. imports remain constant

D. Statement I is false but Statement II is D. exports rise proportionately


true

PAGE 936
www.byjusexamprep.com

91. The monetary policy in India uses which of C. Rate of increase in onion production is
the following tools? more than potato production in the
current year in comparison to the
1. Bank rate previous year.
2. Open market operations D. The major tomato-growing States are
3. Public debt Madhya Pradesh, Karnataka, Odisha
and Gujarat.
4. Public revenue
95. Which of the following Schedules of the
Select the correct answer using the code Constitution of India has fixed the number
given below. of Members of the Rajya Sabha to be
elected from each State?
A. 1 and 2 only
A. Fifth Schedule B. Third Schedule
B. 2 and 3 only
C. Sixth Schedule D. Fourth Schedule
C. 1 and 4 only
96. Which of the following features were
D. 1, 2, 3 and 4 borrowed by the Constitution of India from
92. Arrange the following events in sequential the British Constitution?
order as they happened in India: 1. Rule of Law
1. Mahalanobis Model 2. Law-making Procedure
2. Plan Holiday 3. Independence of Judiciary
3. Rolling Plan 4. Parliamentary System
Select the correct answer using the code Select the correct answer using the code
given below. given below.
A. 1, 2, 3 B. 3, 2, 1 A. 1 and 2 only B. 2, 3 and 4

C. 2, 3, 1 D. 1, 3, 2 C. 1 and 4 only D. 1, 2 and 4

93. The phenomenon of ‘demographic 97. Which of the following statements about
dividend’ of a country relates to ‘delegation’ is/are correct?

A. a sharp decline in total population 1. It is the abdication of responsibility.

B. an increase in working age population 2. It means conferring of specified


authority by a lower authority to a
C. a decline in infant mortality rate higher one.
D. an increase in sex ration 3. It is subject to supervision and review.
94. Which one of the following statements 4. It is a method of dividing authority in
about various horticulture crops of India for the organization.
the year 2016-17 is not correct?
Select the correct answer using the code
A. The area under horticulture crops has given below.
increased over previous year.
A. 3 only B. 2 and 4 only
B. Fruit production during the current
year is higher than the previous year. C. 3 and 4 D. 1, 2 and 4

PAGE 937
www.byjusexamprep.com

98. In which two Indian States of the four 104. In the reaction between hydrogen sulphate
mentioned below, it is necessary to hold ion and water
certain minimum educational qualifications the water acts as
to be eligible to contest Panchayat
Elections? A. an acid

1. Punjab 2. Haryana B. a base

3. Karnataka 4. Rajasthan C. amphoteric


Select the correct answer using the code D. an inert medium
given below.
105. A person is standing on a frictionless
A. 1 and 2 B. 2 and 4 horizontal ground. How can he move by a
C. 2 and 3 D. 1 and 4 certain distance on this ground?

99. What are the main constituents of biogas? A. By sneezing


A. Methane and sulphur dioxide B. By jumping
B. Methane and carbon dioxide C. By running
C. Methane, hydrogen and nitric oxide D. By rolling
D. Methane and nitric oxide
106. After using for some time, big transformers
100. The most ozone gas (about 90%) is located get heated up. This is due to the fact that
in the atmospheric layer of
1. current produces heat in the
A. ionosphere transformers
B. troposphere 2. hysteresis loss occurs in the
C. stratosphere transformers
D. mesosphere 3. liquid used for cooling gets heated
101. Which one of types of glasses is used for Select the correct answer using the code
making optical instruments? given below.
A. Pyrex glass B. Soft glass A. 1 only
C. Hard glass D. Flint glass
B. 2 and 3 only
102. A white powder which is used for repairing
fractured bones ? C. 1 and 2 only

A. bleaching powder D. 1, 2 and 3

B. plaster of Paris 107. Infrared, visible and ultraviolet radiations/


light have different properties. Which one
C. powder of zinc oxide
of the following statements related to
D. lime powder these radiations/light is not correct?
103. How many hydrogen atoms are contained A. The wavelength of infrared is more
in 1.50 g of glucose (C6H12O6)? than that of ultraviolet radiation.
A. B. B. The wavelength of ultraviolet is smaller
C. D. than that of visible light.

PAGE 938
www.byjusexamprep.com

C. The photon energy of visible light is 111. A biological community in its environment
more than that of infrared light. such as a pond, an ocean, a forest, even an
aquarium is known as
D. The photon energy of ultraviolet is
lesser than that of visible light. A. biome

108. A ray of light is incident on a plane mirror at B. community


an angle of 40 ° with respect to surface C. abiotic environment
normal. When it gets reflected from the
mirror, what will be the angle of deviation D. ecosystem
with respect to normal?
112. Pearl is a hard object produced within the
A. 40° B. 100° soft tissues of a mollusc. Which one of the
following is the main constituent of pearl?
C. 90° D. 80°
A. Calcium carbonate
109. Two wires are made having same length l
and area of cross-section A. Wire 1 is made B. Calcium oxide
of copper and wire 2 is made of aluminium. C. Calcium nitrate
It is given that the electrical conductivity of
copper is more than that of aluminium. In D. Calcium sulphate
this context, which one of the following 113. Human insulin molecule is composed of
statements is correct? one α-chain having 21 amino acids and one
A. The resistance of wire 1 will be higher β-chain having 30 amino acids. How many
than that of wire 2 functional insulin genes occur in adult
humans?
B. The resistance of wire 2 will be higher
A. One
than that of wire 1
B. Two
C. The resistance of both the wires will be
the same. C. Three
D. If same current is flown through both D. Four
the wires, the power dissipated in both
114. Match List-I with List-II and select the
the wires will be the same.
correct answer using the code given below
110. An electron and a proton starting from rest the Lists:
are accelerated through a potential
List-I List-II
difference of 1000 V. Which one of the
(Cell (Function)
following statements in this regard is
Organelle)
correct?
A. The kinetic energy of both the particles A. Mitochond 1. Photosynthesis
will be same. ria

B. The speed of the electron will be higher B. Chloroplast 2. Protein


than that of the proton. synthesis
C. The speed of the proton will be higher C. Ribosomes 3. Intracellular
than that of the electron. Digestion
D. The speed of the electron and the D. Lysosomes 4. ATP formation
proton will be equal.

PAGE 939
www.byjusexamprep.com

Codes: Select the correct answer using the code


given below.
A B C D
A. 4, 3, 2, 1 B. 2, 1, 4, 3
A. 3 1 2 4
C. 1, 2, 3, 4 D. 1, 3, 2, 4
B. 3 2 1 4
118. Which one of the following islands is the
C. 4 1 2 3
largest?
D. 4 2 1 3
A. Borneo
115. Blue Baby Syndrome is cause by the
contamination of B. Madagascar

A. nitrite (NO2– ) C. New Guinea

B. sulphite (SO32– ) D. Sumatra

C. nitrate (NO3– ) 119. Who among the following geographers is


related to ‘primate city’ concept?
D. sulphate (SO42– )
A. August Losch
116. Which one of the following proteins given
lustrous shiny appearance to silk fibre? B. Mark Jefferson

A. Fibrin B. Sericin C. Griffith Taylor


C. Collagen D. Nectin D. W. Christaller
117. Arrange the following Tiger Reserves of 120. Which one of the following tribal groups is
India from North to South: dominantly found in the ‘Blue Mountains’?
1. Dudhwa A. Lambadas

2. Panna B. Gonds

3. Pench C. Jarawas

4. Indravati D. Todas

PAGE 940
www.byjusexamprep.com

Elementary Mathematics
1. The average height of 22 students of a class Statement II: Mode is unique.
is 140 cm and the average height of 28
Which one of the following is correct in
students of another class is 152 cm. What is
respect of the above statements?
the average height of students of both the
classes? A. Both Statement I and Statement II are
true and Statement II is the correct
A. 144.32 cm B. 145.52 cm
explanation of Statement I
C. 146.72 cm D. 147.92 cm
B. Both Statement I and Statement II are
2. The heights (in cm) of 5 students are 150, true but Statement II is not the correct
165, 161, 144 and 155. What are the values explanation of Statement I
of mean and median (in cm) respectively?
C. Statement I is true but Statement II is
A. 165 and 161 B. 155 and 155 false
C. 160 and 155 D. 155 and 161 D. Statement I is false but Statement II is
true
3. The total number of live births in a specific
locality during different months of a 7. The pie diagrams on the monthly
specific year was obtained from the office expenditure of two families A and B are
of the Birth Registrar. This set of data may drawn with radii of two circles taken in the
be called ratio 16 : 9 to compare their expenditures.
A. Primary data B. Secondary data Which one of the following is the
appropriate data used for the above-
C. Recorded data D. Countable data
mentioned pie diagrams?
4. The geometric mean of x and y is 6 and the
A. ₹ 16,000 and ₹ 9,000
geometric mean of x, y and z is also 6. Then
the value of z is B. ₹ 8,000 and ₹ 4,500

A. 12 B. C. ₹ 25,600 and ₹ 8,100


D. ₹ 4,000 and ₹ 3,000
C. 6 D.
8. A man travelled 12 km at a speed of 4 km/hr
5. Which one of the following is not correct?
and further 10 km at a speed of 5 km/hr.
The proportion of various items in a pie What was his average speed?
diagram is proportional to the
A. 4.4 km/hr B. 4.5 km/hr
A. Areas of slices
C. 5.0 km/hr D. 2.5 km/hr
B. Angles of slices
9. A small company pays each of its 5
C. Lengths of the curved arcs of the slices categories ‘C’ workers ₹ 20,000, each of its
D. Perimeters of the slices 3 category ‘B’ workers ₹ 25,000 and a
category ‘A’ worker ₹ 65,000. The number
6. Consider the following statements: of workers earning less than the mean
Statement I: The value of a random salary is

variable having the A. 8 B. 5

highest frequency is mode. C. 4 D. 3

PAGE 941
www.byjusexamprep.com

10. Frequency density of a class is computed by 15. Consider the following statements:
the ratio
1. The point of intersection of the
A. Class frequency to the class width perpendicular bisectors of the sides of
a triangle may lie outside the triangle.
B. Class frequency to total frequency
2. The point of intersection of the
C. Class frequency to total number of perpendiculars drawn from the vertices
classes to the opposite sides of a triangle may
D. Cumulative frequency up to that class lie on two sides.
to total frequency Which of the above statements is/are
correct?
Direction: In a University there are 1200
students studying four difference subjects, A. 1 only B. 2 only
Mathematics, Statistics, Physics and Chemistry.
20% of the total number of students are studying C. Both 1 and 2 D. Neither 1 nor 2
Mathematics, one-fourth of the total number of 16. A closed polygon has six sides and one of its
students are studying Physics, 320 students are angles is 30 ° greater than each of the other
studying Statistics and remaining students are five equal angles. What is the value of one
studying Chemistry. Three-fifth of the total of the equal angles?
number of students studying Chemistry are girls.
150 boys are studying Mathematics. 60% of A. 55° B. 115°
students studying Physics are boys. 250 girls are C. 150° D. 175°
studying Statistics.
17. There are 8 lines in a plane, no two of which
11. What is the ratio of the total number of are parallel. What is the maximum number
boys to the total number of girls? of points at which they can intersect?
A. 67 : 83 B. 17 : 26 A. 15 B. 21
C. 27 : 19 D. 189 : 179 C. 28 D. None of these
12. What is the difference between the 18. One-fifth of the area of a triangle ABC is cut
number of boys studying Mathematics and off by a line DE drawn parallel to BC such
the number of girls studying Physics? that D is on AB and E is on AC. If BC = 10 cm,
then what is DE equal to?
A. 20 B. 30
A. cm B. cm
C. 60 D. 80
C. cm D. cm
13. The number of girls studying Statistics is
what percent (approximate) of the total 19. An arc of a circle subtends an angle π at the
number of students studying Chemistry? centre. If the length of the arc is 22 cm, then
what is the radius of the circle?
A. 58.8 B. 73.5
(Take )
C. 78.7 D. 80.6
A. 5 cm B. 7 cm
14. What is the total number of boys studying
Statistics and Physics? C. 9 cm D. 11 cm

A. 180 B. 240 20. The diagonals of a rhombus are of length 20


cm and 48 cm. What is the length of a side
C. 250 D. 310 of the rhombus?

PAGE 942
www.byjusexamprep.com

A. 13 cm B. 26 cm 25. A tent has been constructed which is in the


C. 36 cm D. 39 cm form of a right circular cylinder surmounted
by a right circular cone whose axis coincides
21. Consider the following statements in with the axis of the cylinder. If the radius of
respect of three straight lines A, B and C on the base of the cylinder is 50 m, the height
a plane: of the cylinder is 10 m and the total height
1. If A and C are parallel and B and C are of the tent is 15 m, then what is the capacity
parallel; then A and B are parallel. of the tent in cubic metres?
2. If A is perpendicular to C and B is 87500
A. 37500π B.
perpendicular to C; then A and B are 3
parallel.
26500
3. If the acute angle between A and C is C. D. 25000π
3
equal to the acute angle between B
and C; then A and B are parallel. 26. From a solid cylinder whose height is 8 cm
and of base radius 6 cm, a conical cavity of
Which of the above statements are
height 8 cm and of base radius 6 cm is
correct?
formed by hollowing out. What is the inner
A. 1, 2 and 3 B. 1 and 2 only surface area of the cavity?
C. 1 and 3 only D. 2 and 3 only A. 6π square cm
22. In a triangle PQR, X is a point on PR Y is a B. 8π square cm
point on QR such that PR = 10 cm, RX = 4
cm, YR = 2 cm, QR = 5 cm. Which one the C. 10π square cm
following is correct? D. 60π square cm
A. XY is parallel to PQ
27. The radii of the frustum of a right circular
B. PQ = 2XY cone are in the ratio 2 : 1. What is the ratio
C. PX = QY of the volume of the frustum of the cone to
that of the whole cone?
D. PQ = 3XY
A. 1 : 8 B. 1 : 4
23. In a triangle ABC, the medians AD and BE
intersect at G. A line DF is drawn parallel to C. 3 : 4 D. 7 : 8
BE such that F is on AC . If AC = 6 cm, then 28. The vertical angle of right circular cone is
what is CF equal to? 
and the slant height is 2 r cm. What is
A. 2.25 cm B. 3 cm 2
C. 4.5 cm D. 6 cm the volume of the cone in cubic cm?

24. Two rectangular sheets of sizes 2π × 4 A. πr3 B. 9πr3


π and π × 5π are available. A hollow right
r3
circular cylinder can be formed by joining a C. D. 3πr3
pair of parallel sides of any sheet. What is 3
the maximum possible volume of the 29. A cylindrical vessel 60 cm in diameter is
circular cylinder that can be formed this partially filled with water. A sphere 30 cm
way? in diameter is gently dropped into the
A. 4π2 B. 8π2 vessel and is completely immersed. To
what further height will the water in the
C. 1.25π2 D. 6.25π2 cylinder rise?

PAGE 943
www.byjusexamprep.com

A. 20 cm B. 15 cm A. AD2 B. 2AD2
C. 10 cm D. 5 cm C. 3AD2 D. 4AD2
30. A cylinder of height 2x is circumscribed by a 36. The diagonals of a cyclic quadrilateral ABCD
sphere of radius 2x such that the circular intersect at P and the area of the triangle
ends of the cylinder are two small circles on APB is 24 square cm. If AB = 8 cm and CD =
the sphere. What is the ratio of the curved 5 cm and CD is parallel to AB, then what is
surface are of the cylinder to the surface the area of the triangle CPD?
area of the sphere?
A. 24 square cm
A. B. B. 25 square cm
C. D. C. 12.5 square cm
31. If a point O in the interior of a rectangle D. 9.375 square cm
ABCD is joined with each of the vertices A,
B, C and D, then OB2 + OD2 will be equal to 37. AB and CD are parallel chords of a circle 3
cm apart. If AB = 4 cm, CD = 10 cm, then
A. B. what is the radius of the circle?
C. D. A. 7 cm B. 19 cm
32. In a triangle ABC, ∠C = 90 ° and CD is the
C. 29 cm D. 14 cm
perpendicular from C to AB.
38. In a triangle ABC, AD is perpendicular on BC.
If , then which
If ∠BAC = 90 °, AB = c, BC = a, CA = b and AD
one of the following is correct?
= p, then which one of the following is
A. BC.CD = AB.CA correct?
B. A A. p = abc B. p2 = bc
B. BC = CD.CA bc ab
C. p= D. p =
C. a c
39. Segment QR of length r is a tangent at Q to
D. AB.CD = BC.CA
a circle of radius r with centre at P. What is
33. If PL, QM and RN are the altitudes of the area of the part of the triangle PQR,
triangle PQR whose orthocentre is O, then which is outside the circular region?
Q is the orthocentre of the triangle
r2 r2 r2
A. OPQ B. OQR A. B. −
16 2 8
C. PLR D. OPR
r2 r2 r2 r2
34. The distance between the centres of two C. − D. −
2 16 4 8
circles having radii 9 cm and 4 cm is 13 cm.
What is the length of the direct common 40. What is the area of the triangle whose sides
tangent of these circles? are 51 cm, 37 cm, and 20 cm?

A. 12 cm B. 11 cm A. 300 square cm

C. 10 cm D. 9.5 cm B. 305 square cm

35. In an equilateral triangle ABC, BD is drawn C. 306 square cm


perpendicular to AC. What is BD2 equal to? D. 307 square cm

PAGE 944
www.byjusexamprep.com

41. An isosceles triangle is drawn outside on C. Greater than 1 but less than 2
one of the sides of a square as base in such
D. Greater than 2
a way that the perimeter of the complete
7 46. A man, standing at a point X on the bank XY
figure is times the perimeter of the of a river that cannot be crossed, observes
6
original square. What is the ratio of area of a tower to be N α0 E on the opposite
the triangle to the area of the original parallel bank. He then walks 200 m along
square? the bank to the point Y towards East, and
finds the tower to be N β0 W. From these
A. 1 : 1 B. 2 : 3 observations, the breadth of the river will
C. 1 : 2 D. 1 : 3 be
42. Two straight lines AB and AC include an (Given that and )
angle. A circle is drawn in this angle which A. 60 m B. 70 m
touches both these lines. One more circle is
drawn which touches both these lines as C. 80 m D. 90 m
well as the previous circle. If the area of the 47. Let AB represent a building of height h
bigger circle is 9 times the area of the metre with A being its top, B being its
smaller circle, then what must be the angle bottom. Let A’B’ represent a tower of
A? height (h+x) metre (x>0) with A’ being its
A. 45° B. 60° top and B’ being its bottom. Let BB’ = d
metre. Let the angle of elevation of A’ as
C. 75° D. 90° seen from A be 45 °.
43. A hollow cube is formed by joining six Consider the following statements:
identical squares. A rectangular cello tape
of length 4 cm and breadth 0.5 cm is used Statement I: h + x > d
for joining each pair of edges. What is the Statement II: The angle of depression of
total area of cello tape used?
B as seen from A’ is less
A. 12 square cm B. 24 square cm
than 45 °.
C. 36 square cm D. 48 square cm
Which one of the following is correct in
44. The diameters of two given circles are in the respect of the above statements?
ratio 12:5 and the sum of their areas is
equal to the area of a circle of diameter 65 A. Both Statement I and Statement II are
cm. What are their radii? true, and Statement II is the correct
explanation of Statement I
A. 12 cm and 5 cm
B. Both Statement I and Statement II are
B. 24 cm and 10 cm true, but Statement II is not the correct
C. 60 cm and 25 cm explanation of Statement I

D. 30 cm and 12.5 cm C. Statement I is true but Statement II is


false
sin 1
45. The value of where 1c represents 1 D. Statement I is false but Statement II is
c
sin 1
true
radian is
48. Consider the following statements:
A. Equal to 1
1. If 45 ° < θ < 60 °, then sec2θ + cosec2θ
B. Less than 1 = α2 for some real number α > 1.

PAGE 945
www.byjusexamprep.com

1 + cos  2 
2. If 0 ° < θ < 45 °, then = x for 53. If 0  x  , then (sinx + cosx) is
1 − cos  2
some real number x > 2.
A. > 2 B. < 2
cos 
3. If 0 ° < θ < 45°, then + C. ≥ 2 D. ≤ 2
1 − tan 
sin  54. If a triangle has sides 5, 13 and 12 units and
2
1 − cot  θ is the acute angle of the triangle, then
what is value of (sinθ + cosθ)?
What is the number of true statements?
A. Zero B. One 5 7
A. B.
13 13
C. Two D. Three
49. If θ measured in radians is the angle 12 17
C. D.
between the hour hand and the minute 13 13
hand of a clock when the time is 4 : 36 pm, 55. A man rows down a river 18 km in 4 hours
then which one of the following is correct? with the stream and returns in 10 hours.
A. Consider the following statements:
1. The speed of the man against the
B.
stream is 1.8 km/hr.
C. 2. The speed of the man in still water is
3.15 km/hr.
D.
3. The speed of the stream is 1.35 km/hr.
sin45 − sin30
50. If A = and Which of the above statements are
cos 45 + cos60
sec 45 − tan45 correct?
B= , then which one of
cosec 45 + cot45 A. 1 and 2 only B. 2 and 3 only
the following is correct? C. 1 and 3 only D. 1, 2 and 3
A. A = B B. A > B > 0
56. When prices rise by 12%, if the expenditure
C. A < B D. B < A < 0 is to be the same, what is the percentage of
51. If angle A of triangle ABC is 30° and circum- consumption to be reduced?
radius of the triangle is 10 cm, then what is 2 2
the length of side BC? A. 16 % B. 10 %
3 7
A. 5 cm B. 10 cm
3 5
C. cm D. cm C. 16 % D. 10 %
5 7
m2 − n2 
52. If sin  = and 0    , then what 57. The difference between the compound
2 2
m +n 2 interest (compounded annually) and simple
is the value of cos  ? interest on a sum of money deposited for 2
2mn 2mn years at 5% per annum is ₹15. What is the
A. B. sum of money deposited?
2 2
m +n m2 − n2
A. ₹6,000 B. ₹4,800
m2 + n2 m2 − n2
C. D. C. ₹3,600 D. ₹2,400
2mn 2mn

PAGE 946
www.byjusexamprep.com

58. There are 350 boys in the first three the job. The number of men who left the
standards. The ratio of the number of boys job is
in first and second standards is 2:3, while
A. 24 B. 18
that of boys in second and third standards
is 4:5. What is the total number of boys in C. 12 D. 6
first and third standards? 64. If log106 = 0.7782 and log108 = 0.9031, then
A. 302 B. 280 what is the value of log108000 + log10600?
C. 242 D. 230 A. 4.6813 B. 5.5813
59. If 78 is divided into 3 parts which are C. 1.5813 D. 6.6813
1 1
proportional to 1, , , then the middle 65. The value of
3 6
part is:
A. Equals to 1
28 B. Lies between 0 and 1
A. B. 13
3
C. Lies between 1 and 2
52 55
C. D. D. Is greater than 2
3 3
60. A boy went to his school at a speed of 12 (x − y)(y − z)(z − x)
66. What is equal to?
km/hr and returned to his house at a speed (x − y)3 + (y − z)3 + (z − x)3
of 8 km/hr. If he has taken 50 minutes for
1 1
the whole journey, what was the total A. − B.
distance walked? 3 3
C. 3 D. –3
A. 4 km B. 8 km
67. In an examination, 35% students failed in
C. 16 km D. 20 km
Hindi, 45% students failed in English and
61. A tap can fill a tub in 10 hours. After 20% students failed in both the subjects.
opening the tap for 5 hours it was found What is the percentage of students passing
that a small outlet at the bottom of the tub in both the subjects?
was open and water was leaking through it.
A. 0 B. 20
It was then immediately closed. It took 7
hours to fill the tub after closing the outlet. C. 30 D. 40
What time will be taken by the outlet to
68. If , then what is the value
empty the full tub of water?
of ?
A. 35 hours B. 25 hours
C. 20 hours D. 17 hours A. 3 B. 0

62. 4 goats or 6 sheep can graze a field in C. 1 D. –1


50 days. 2 goats and 3 sheep will graze it in 69. Which one of the following is a correct
A. 200 days B. 150 days statement?

C. 100 days D. 50 days A. {x : x + 5 = 5} = 𝜙

63. 30 men can complete a job in 40 days. B. {x : x + 5 = 5} = {0}


However, after 24 days some men out of C. {x : x + 5 = 5} = 0
the assigned 30 left the job. The remaining
D. {x : x + 5 = 5} = {𝜙}
people took another 40 days to complete

PAGE 947
www.byjusexamprep.com

70. Ram buys 4 chairs and 9 stools for ₹1,340. A. Both integral roots
If he sells chairs at 10% profit and stools at
B. At least one integral root
20% profit, he earns a total profit of ₹188.
How much money did he have to pay for C. No integral root
the chairs? D. Both positive integral roots
A. ₹200 B. ₹400 77. The non-zero solution of the equation
C. ₹800 D. ₹1,600 a − x2 b − x c − x b − x 2
− = − , where b ≠ 0,
71. Five years ago, Ram was three times as old bx c b cx
as Shyam. Four years from now, Ram will be c ≠ 0, is
only twice as old as Shyam. What is the
A. B.
present age of Ram?
A. 30 years B. 32 years C. D.
C. 36 years D. 40 years
78. If the roots of the equation
72. What is the value of u in the system of
are
equations 3(2u + ) = 7u, 3(u + 3) = 11u ?
equal, then which one of the following is
1 correct?
A. 0 B.
4
A. B.
1
C. D. 1 C. D.
2
73. A student has to secure 40% of marks to 79. a, b, c, d are non-zero integers such that
pass an examination. He gets only 45 marks (ab) divides (cd). If a and c are coprime,
and fails by 5 marks. The maximum marks then which one of the following is correct?
are A. a is a factor of c
A. 120 B. 125 B. a is a factor of b
C. 130 D. 150 C. a is a factor of d
74. To maintain 8 cows for 60 days, a milkman D. d is a factor of a
has to spend ₹6,400. To maintain 5 cows for
n days, he has to spend ₹4,800. What is the 80. If the HCF of polynomials
value of n? and
A. 46 days B. 50 days is
C. 58 days D. 72 days , then what are the values of
a and b respectively?
75. How many numbers between 500 and 1000
are divisible by 13? A. 2, 2 B. 2, -3

A. 36 B. 37 C. -1, -3 D. -2, -1

C. 38 D. 39 81. The factors of


are
76. If k is an integer, then
A. and
 7
x2 + 7x − 14  k2 −  = 0 has B. and
 8

PAGE 948
www.byjusexamprep.com

C. and 88. If a train crosses a km-stone in 12 seconds,


how long will it take to cross 91 km-stones
D. and completely if its speed is 60 km/hr?
82. The product of the polynomials A. 1 hr 30 min
and
is B. 1 hr 30 min 12 sec

A. B. C. 1 hr 51 min
D. 1 hr 1 min 3 sec
C. D.
89. A fruit seller has a certain number of
83. Consider the following numbers:
mangoes of which 5% are rotten. He sells
1. 2222 2. 11664 75% of the remainder and he is left with 95
mangoes. How many mangoes did he have
3. 343343 4. 220347 originally?
Which of the above are not perfect A. 500 B. 450
squares?
C. 400 D. 350
A. 1, 2 and 3 B. 1, 2 and 4
90. Sudhir purchased a chair with three
C. 2, 3 and 4 D. 1, 3 and 4 consecutive discounts of 20%, 12.5% and
5%. The actual deduction will be
84. Let …
, where … are A. 33.5% B. 30%
real numbers. If f(x) is divided by (ax – b), C. 32% D. 35%
then the remainder is
91. If the points P and Q represent the real
A. B. numbers 0.83 and 0.62 on the number
line, then the distance between P and Q is
C. D. 21 19
A. B.
85. If (x + 2) is a common factor of x2 + ax + b 90 90
and x2 + bx + a, then the ratio a : b is equal 21 56
C. D.
to 100 90
A. 1 B. 2 92. The product of two non-zero expressions is
. If their HCF is , then their
C. 3 D. 4
LCM is
86. The quotient of when divided by
A.
is
B.
A. B.
C.
C. D.
D.
87. In a 100 m race, A runs at 6 km/hr. If A gives
B a start of 8 m and still beats him by 9 93. is divided by ,
seconds, what is the speed of B? then the remainder is

A. 4.6 km/hr B. 4.8 km/hr A. 135 B. –135


C. 63 D. –63
C. 5.2 km/hr D. 5.4 km/hr

PAGE 949
www.byjusexamprep.com

94. Let f(x) and g(x) be two polynomials (with 98. How many numbers from 1 to 1000 are
real coefficients) having degrees 3 and 4 divisible by 2, 3, 4 and 5?
respectively. What is the degree of f(x)g(x)?
A. 16 B. 17
A. 12 B. 7
C. 32 D. None of these
C. 4 D. 3 99. What is the value of α(α ≠ 0) for which
95. What is the positive value of m for which x2 – 5x + α and x2 – 7x + 2α have a common
the roots of the equation 12x2 + mx + 5 factor?
= 0 are in the ratio 3:2? A. 6 B. 4

5 10 C. 3 D. 2
A. 5 10 B.
12 Direction: In a University there are 1200
students studying four difference subjects,
5 12
C. D. Mathematics, Statistics, Physics and Chemistry.
12 5
20% of the total number of students are studying
96. If x = 111 …… 1 (20 digits), y = 333….3 (10 Mathematics, one-fourth of the total number of
digits) and z = 222…….2 (10 digits), then students are studying Physics, 320 students are
x − y2 studying Statistics and remaining students are
what is equal to? studying Chemistry. Three-fifth of the total
z
number of students studying Chemistry are girls.
1 150 boys are studying Mathematics. 60% of
A. B. 1 students studying Physics are boys. 250 girls are
2
studying Statistics.
C. 2 D. 3
Question No. 88
97. What are the factors of
100. In Which subject is the difference between
?
the number of boys and girls are equal?
A. and
A. Mathematics and Chemistry
B. and B. Statistics and Chemistry
C. and C. Mathematics and Physics
D. and D. Mathematics and Statistics

PAGE 950
www.byjusexamprep.com

ANSWERS KEY II 2017


ENGLISH

1 C 21 B 41 A 61 D 81 C 101 A
2 D 22 C 42 D 62 A 82 B 102 B
3 A 23 B 43 A 63 B 83 C 103 B
4 D 24 B 44 D 64 C 84 B 104 A
5 D 25 B 45 B 65 B 85 C 105 A
6 B 26 A 46 A 66 C 86 B 106 B
7 D 27 C 47 D 67 A 87 D 107 B
8 C 28 C 48 C 68 A 88 A 108 B
9 C 29 B 49 A 69 A 89 B 109 A
10 C 30 B 50 D 70 C 90 D 110 A
11 C 31 B 51 B 71 D 91 B 111 C
12 D 32 A 52 A 72 C 92 A 112 D
13 D 33 D 53 A 73 B 93 B 113 A
14 D 34 A 54 A 74 B 94 A 114 A
15 B 35 C 55 A 75 A 95 C 115 B
16 B 36 C 56 B 76 D 96 A 116 C
17 B 37 C 57 D 77 B 97 B 117 B
18 B 38 B 58 C 78 C 98 A 118 A
19 B 39 D 59 D 79 C 99 B 119 C
20 D 40 B 60 B 80 B 100 B 120 B

GENERAL KNOWLEDGE
1 D 21 C 41 D 61 B 81 C 101 D
2 B 22 A 42 D 62 D 82 C 102 B
3 B 23 D 43 A 63 C 83 A 103 D
4 B 24 D 44 B 64 C 84 A 104 C
5 D 25 C 45 A 65 A 85 A 105 A
6 B 26 B 46 D 66 D 86 A 106 C
7 B 27 C 47 A 67 C 87 A 107 D
8 C 28 A 48 D 68 D 88 A 108 A
9 B 29 C 49 C 69 D 89 C 109 B
10 D 30 C 50 D 70 B 90 B 110 B
11 B 31 D 51 C 71 C 91 A 111 D
12 D 32 C 52 C 72 A 92 A 112 A

PAGE 951
www.byjusexamprep.com

13 C 33 C 53 C 73 C 93 B 113 B
14 D 34 B 54 C 74 A 94 C 114 C
15 A 35 B 55 B 75 D 95 D 115 C
16 B 36 A 56 A 76 D 96 D 116 A
17 C 37 B 57 B 77 D 97 C 117 C
18 C 38 B 58 D 78 B 98 B 118 C
19 A 39 C 59 A 79 A 99 B 119 B
20 D 40 C 60 D 80 D 100 C 120 D

Elementary Mathematics
1 C 21 B 41 D 61 B 81 C
2 B 22 A 42 B 62 D 82 B
3 B 23 A 43 B 63 B 83 D
4 C 24 B 44 D 64 D 84 A
5 D 25 B 45 B 65 C 85 A
6 C 26 D 46 C 66 B 86 C
7 C 27 D 47 C 67 D 87 B
8 A 28 C 48 C 68 C 88 B
9 A 29 D 49 B 69 B 89 C
10 B 30 A 50 A 70 C 90 A
11 A 31 C 51 B 71 B 91 B
12 B 32 D 52 A 72 D 92 D
13 B 33 D 53 B 73 B 93 D
14 C 34 A 54 D 74 D 94 B
15 C 35 C 55 D 75 C 95 A
16 B 36 D 56 D 76 C 96 B
17 C 37 C 57 A 77 A 97 C
18 B 38 C 58 D 78 C 98 A
19 B 39 B 59 C 79 C 99 A
20 B 40 C 60 B 80 B 100 C

PAGE 952
CDS II 2017
www.byjusexamprep.com

(Solutions)
ENGLISH
1. Ans. C.
The introductory sentence is, ‘Egypt lies in the north-east corner of Africa’ & the concluding sentence
is, ‘The whole country depends on the water of Nile’.
If we look at the last sentence, clearly the second last sentence would be related to the condition of
water in the area, hence the second last sentence would be ‘Q’.
This narrows down to options to ‘B’ & ’C’.
Also, logically, sentence ‘P’ should be followed by sentence ‘Q’, since ‘S’ is basically an effect of ‘P’.
Thus, the correct sequence will be S1, R, P, S, Q, S6.
Hence, option ‘C’ is correct.
2. Ans. D.
Sentences ‘P’ and ‘R’ make use of pronoun ‘he’, who has been referred to in sentence ‘S’. Hence, ‘P’
& ‘R’ will clearly come after ‘S’. This narrows our option to ‘C’ & ‘D’.
Now, sentence ‘Q’ does not refer to the ‘man’ in sentence ‘S’, but is a general statement referring to
the perpetual practice mentioned in S1.
Hence, the correct sequence of the sentences will be: S1, Q, S, P, R, S6
Thus, option ‘D’ is correct.
3. Ans. A.
Sentence ‘R’ must come after S1, since it refers to what happened with Isaac when his mother
remarried. The next sentence would be ‘Q’, since ‘she’ refers to the grandmother, who sent him to
school. Sentence ‘P’ would follow sentence ‘S’, since both are connected, i.e. although Isaac was not
a bright student, he was clever at making things.
Thus, the correct sequence would be S1, R, Q, S, P, S6.
Hence, option ‘A’ is correct.
4. Ans. D.
‘Me’ used in sentences ‘P’ & ‘R’ refer to the first person ‘I’ used in sentence ‘S’. Hence both ‘P’ & ‘R’
would come after ‘S’. The only option with this arrangement is ‘D’.
Sentence ‘Q’ is a general statement referring to the educational system, hence it would come after
S1.

PAGE 953
www.byjusexamprep.com

As far as arrangement of ‘P’ and ‘R’ is concerned, sentence ‘P’ is in present tense (makes me) and
hence cannot be used after ‘S’ which is in past tense (used to grip). Sentence ‘R’ refers to the
nightmares which the author has at present and hence, ‘P’ will come after ‘R’.
So, the correct sequence is, S1, Q, S, R, P, S6.
Hence, option ‘D’ is correct.
5. Ans. D.
Sentences ‘R’ & ‘S’ would come after ‘Q’, since the pronoun ‘he’ used in these sentences refer to
Toynbee, whose mention comes in sentence ‘Q’.
Also, sentence ‘R’ would follow sentence ‘S’, because sentence ‘R’ refers to the birth, growth and
decay of civilization and the idea of civilization has been introduced in sentence ‘S’.
Sentence ‘P’ is general in nature and sentence ‘Q’ would follow sentence ‘P’ because sentence ‘P’
mentions the idea of History in general and sentence ‘Q’ mentions how Toynbee’s view was different
from this general idea.
Hence, the correct sequence will be S1, P, Q, S, R, S6.
6. Ans. B.
The most appropriate sequence for the given jumbled sentences is QSRP.
Hence, option B is the correct answer.
7. Ans. D.
Sentence ‘R’ would follow S1 since it shows the effect of S1, i.e., since Phatik was a mischievous boy,
so his mother found it difficult to bring him up. Sentence ‘Q’ will follow ‘R’, since the pronoun ‘she’
used in ‘R’ refers to his mother who has been mentioned in ‘R’.
Judging by the options, the correct sequence would be S1, R, Q, P, S, S6.
8. Ans. C.
‘S’ is the opening sentence of the anecdote mentioned in ‘S1’. ‘P’ would follow ‘S’. ‘Q’ would come
before ‘R’ since it would justify the correct sequence of making of the idol by the sculpture.
Thus, the correct sequence would be: S1, S, P, Q, R, S6.
9. Ans. C.
Sentence ‘Q’ would follow ‘S1’ since it shows what Manson did after receiving Ross’s work. ‘P’ would
follow ‘Q’ since it shows the effect his work had on the audience to which it was presented, which
will be followed by ‘S’, which shows the action of the audience.
Thus, the correct sequence would be S1, Q, P, S, R, S6.
10. Ans. C.
Sentence ‘S’ would clearly come before ‘S6’, since it describes renewable resources which cannot be
reused, as mentioned in ‘S6’. ‘Q’ would come before ‘S’, since it mentions oil and natural gas, the
renewable resources described in ‘S’. Looking at the remaining sentences, ‘R’ would follow ‘S1’ and
‘P’ would follow ‘R’.
Thus, the correct sequence would be S1, R, P, Q, S, S6

PAGE 954
www.byjusexamprep.com

11. Ans. C.
Sentence S1 talks about ghosts. Sentence S points out the fact that even after the advancements in
science, it still remains a mystery. Sentence P and R shows the curiosity of humans for ghosts and
the attempts made by humans to take their photographs in which they've failed too (Sentence Q).
Thus, the correct sequence would be S1, S, P, R, Q, S6.
12. Ans. D.
Sentence S1 talks about 2 schools of thought. ‘R’ and 'Q' tell us about the first school of thoughts
that focuses on teaching methods. Sentence 'P' and 'S' talk about the second school of thoughts that
focuses on how one should focus more on dealing with errors than on teaching methods.
Thus, the correct sequence would be: S1, R, Q, P, S, S6.
13. Ans. D.
‘Q’ would follow ‘S1’ since it shows the state of the conference hall as the author enters it. ‘S’ would
follow ‘Q’ since it describes the delegates mentioned in ‘Q’. The next statement would be ‘P’, which
describes the action taken by the author, followed by ‘R’.
Thus, the correct sequence would be: S1, Q, S, P, R, S6.
14. Ans. D.
‘P’ would follow ‘S1’ as it describes the first lesson of discipline mentioned in S1. ‘R’ would follow ‘P’
since it points the next thing one must focus on after discipline.
Looking at the options, the correct sequence would be S1, P, R, S, Q, S6.
15. Ans. B.
‘S’ would follow ‘S1’, since it shows it goes with the correct sequence of events, i.e. what happened
when the couple reached home. Next statement would be ‘P’, followed by ‘R’, since ‘P’ shows what
they saw as soon as they reached home and then the action which they took, i.e., they entered the
house. The last statement would be ‘Q’.
Thus, the correct sequence would be: S1, S, P, R, Q, S6.
16. Ans. B.
Sentence ‘R’ would follow ‘S1’, since it connects with S1 showing why physical help cannot cure
miseries. The next statement would be ‘Q’, since it concludes ‘R’. It would be followed by ‘S’, since it
presents the solution to the issue described in ‘R’ & ‘Q’. The last statement would be ‘P’.
Thus the correct sequence would be: S1, R, Q, S, P, S6.
17. Ans. B.
The most appropriate sequence for the given jumbled sentences is RSPQ.
Hence, option B is the correct answer.
18. Ans. B.
‘S’ would follow ‘S1’, since it shows what happened when the bus was moving. The next statement
would be ‘R’ since it shows the action of the driver as soon as he spotted the dog. ‘P’ would follow
‘R’ as it shows what happened after the driver applied the brakes and the last statement would be
‘Q’ as it shows the final impact.
Thus, the correct sequence would be: S1, S, R, P, Q, S6.

PAGE 955
www.byjusexamprep.com

19. Ans. B.
‘P’ would follow ‘Q’ since it shows the contrast between the women of rural and urban areas. ‘Q’
would be followed by ‘S’ as it refers to the status of education of the women in villages, whose
reference comes in ‘P’. ‘R’ would follow S1.
Thus, the correct sequence would be: S1, R, Q, P, S, S6.
20. Ans. D.
‘R’ would follow ‘S1’ as it shows the decision which Hieun-tsang took after realising about the
Buddhist texts. The next statement would be ‘Q’ as it shows how he entered India, which has been
conveyed in ‘R’. Next statement would be ‘S’ as it shows the next phase of his journey after India.
The last statement would be ‘P’.
Thus, the correct sequence would be: S1, R, Q, S, P, S6.
21. Ans. B.
The sentence implies that we need to make sufficient/ considerable progress given the state of
economic expansion. Thus, the correct word would be ‘substantial’, which means considerable.
Other words, i.e. detailed, definite and infinite do not fit the given blank.
Hence, option B is the correct answer.
22. Ans. C.
The statement implies that at times his anger is so high that no one can appease/ lessen it. Thus, the
correct word would be ‘mollify’, which means ‘reduce the severity of’.
23. Ans. B.
The apt word would be ‘Differentiate’, since the sentence refers to the fact that most people cannot
recognise the good from the bad.
24. Ans. B.
We need to use a past perfect form in this sentence, since an action that took place in the past before
another past action has been described. Thus, the correct option would be ‘had started’.
25. Ans. B.
Between is used to say that somebody or something is between two or more clearly separate people
or things.
Among is used when somebody or something is in a group of people or things which we do not see
separately.
Here, 'daughters & sons'
26. Ans. A.
The correct word to fill the blank is ‘eyes’. The idiom ‘in the eyes of’ means in someone’s opinion or
view.
27. Ans. C.
The correct option would be ‘off’. The phrase ‘Clear off’ means to go away, which is in accordance
with the meaning of the given statement.

PAGE 956
www.byjusexamprep.com

28. Ans. C.
The sentence implies that the teachers could not tolerate the ways of the headmaster.
Thus, the correct option would be the phrase ‘put up with’, which means to tolerate or endure.
Put over- To postpone; delay.
Put on - To clothe oneself with, to don.
Put up to - To engage in; carry on.
29. Ans. B.
The two parts of the given sentence are contradictory, i.e., the first part shows the rate of
construction of housing units and the next denotes shortage of accommodation. Thus, the correct
option would be ‘despite’.
30. Ans. B.
Let's understand the meanings of the given words:
Present = formally introduce (someone) to someone else.
Endow = provide with a quality, ability, or asset.
Fortify = to make something stronger, especially in order to protect it
Replenish = fill (something) up again.
The sentence talks about providing some quality to someone. So, the correct filler here would be
"endow". The verb "endow" is usually followed by preposition "with" which is already given in the
sentence. This is also an indication that only the verb "endow" would fit here.
31. Ans. B.
To solve such para jumbles quickly, we should take the help of options. According to option B, the
correct sentence is "My unmarried aunt, who stays with us is creating a lot of problems for us and
there is a misunderstanding among family members because of her interfering nature in our personal
lives."
32. Ans. A.
The correct sequence would be:
No criminal proceeding whatsoever shall be initiated or continued against the President or a
Governor in any court during his term if office.
‘Q’ refers to the criminal proceedings and hence would appear first. ‘R’ would come after ‘Q’ and will
be followed by ‘S’, since they identify the people against whom the proceedings are initiated/
continued.
33. Ans. D.
The correct sequence would be:
The dentist extracted Manish’s tooth when he was brought to hospital with a severe tooth-ache and
relieved his pain.
‘R’ describes the action taken by the dentist and hence would appear first. ‘P’ would appear before
‘S’ because the pain described in ‘S’ is because of the severe tooth-ache which has been described in
‘P”.

PAGE 957
www.byjusexamprep.com

34. Ans. A.
The correct sequence would be:
The actress, who is the daughter of a famous male singer, has been selected as the heroine of the
year.
‘S’ would clearly come after ‘Q’. ‘R’ would appear first since it describes the actress (who refers to
the actress).
35. Ans. C.
The correct sequence would be:
Last summer, one persistent admirer kept pestering a pretty girl with phone calls everyday.
‘Q’ would follow ‘R’ since it answers who kept pestering the girl. ‘S’ would follow ‘Q’, which would
be followed by ‘P’.
36. Ans. C.
First part of the sentence would be R as the subject is given here. Q is connected by R with the
conjunction "that". P and S together complete the sentence. The correct sequence is RQPS.
37. Ans. C.
The correct sequence would be:
From a picnic table we watched them laugh and leap through the playground while we unpacked a
basket bulging with sandwiches and cookies.
‘P’ would follow ‘R’, which would appear first. ‘P’ describes where they laughed and leaped. ‘Q’ would
follow ‘P’ since it completes the second part of the sentence, i.e. what the author did while watching.
‘S’ would come after ‘Q’, since it describes the basket.
38. Ans. B.
The correct sequence would be:
My father retired at the age of 6, from a Baptist Church in South Carolina where he had served as
Pastor for 12 years.
‘S’ would appear first since it links the retirement of author’s father to the place of his service and it
would be followed by ‘Q’, since it describes the location of Church in ‘S’. ‘P’ would follow ‘Q’ and it
would be followed by ‘R’. ‘Q’ connects the second part of the sentence, i.e. what did author’s father
do.
39. Ans. D.
The correct sequence would be:
The completion of high quality products also enhances employee satisfaction, because it enables
employees to feel a sense of accomplishment and makes them take pride in their work.
‘P’ & ‘Q’ would come after ‘S’ since they describe how the employee satisfaction is enhanced, also,
‘Q’ consists of the conjunction, i.e. it joins a sentence before it, thus ‘P’ would come before ‘Q’. ‘R’
would appear first.

PAGE 958
www.byjusexamprep.com

40. Ans. B.
The correct sequence would be:
But, Kuala Lumpur is a city firmly rooted in tradition, where modern Malaya executives might have a
cellular phone in hand but will never miss Friday prayers.
‘S’ would appear first since it has the reference of city, i.e, Kuala Lumpur. ‘P’ would follow ‘S’ and it
will be followed by ‘R’ and then ‘Q’ since it shows the correct sequence of how the city is firmly
rooted in tradition.
41. Ans. A.
‘S’ would follow ‘Q’ which would appear first. ‘S’ refers to the ‘crimes’ against the elderly, whose first
mention comes in ‘Q’. ‘P’ would come after ‘S’ since it shows the cause of crimes. ‘R’ would follow
‘S’ since it joins the second part of these causes.
The correct sequence would be:
Perhaps the most significant factor in the growth of all metropolitan crimes, including crimes against
the elderly, is a failure of planning and governance in the urban sprawl.
42. Ans. D.
The correct sequence would be:
He said that he’d rather go to a small college instead of not studying at all.
‘Q’ would appear first since it begins the statement of the author as mentioned in the opening line
of the sentence. ‘P’ would come after ‘Q’ since it shows where the author would go. ‘S’ would come
after ‘P’ and it will be followed by ‘R’.
43. Ans. A.
‘S’ would appear first since it describes teaching what is becoming difficult & expensive. It will be
followed by ‘P’. ‘R’ would come after ‘P’ since it describes why teaching children how to swim is
becoming difficult. R would be followed by ‘Q’.
The correct sequence would be:
Teaching a child how to swim is becoming difficult and expensive since there are few open ponds
around.
44. Ans. D.
The correct sequence would be:
The doctor had not been able to find out what had caused the food poisoning.
‘P’ would follow ‘S’ since it completes the sentence that the doctor had not been able to find out. ‘S’
would come after ‘R’.
45. Ans. B.
The correct sequence would be:
The officer, being corrupt, was suspended from service before his dismissal.

PAGE 959
www.byjusexamprep.com

‘Q’ would appear first since it describes the nature of the officer and also points out the reason for
his suspension. ‘P’ would come after it and would be followed by ‘R’ describing where was he
suspended from. ‘S’ would come at the end.
46. Ans. A.
The correct sequence would be:
She gave her coat, the one with the brown fur on it, to a beggar shivering with cold.
‘P’ would appear first since it shows what she gave away. ‘R’ describes the coat and hence would
come after ‘P’. ‘Q’ would come after ‘R’ since it shows to whom the coat was given and ‘S’ would
come after ‘Q’ since it describes the state of the beggar.
47. Ans. D.
The correct sequence would be:
The medical team at the ground said that the injured who were trampled by the surging crowds,
included women and children.
‘P’ would appear first since it refers to the medical team mentioned in the opening of the sentence.
‘S’ would appear next, followed by ‘Q’ and ‘R’ would come at the end. ‘Q’ describes who was
responsible for the trampling.
48. Ans. C.
‘Q’ would appear first followed by ‘S’ since it shows where one’s eyes are fixed. ‘P’ would come next
followed by 'R’ showing how the hurdles don’t affect the ones who have their eyes set on the goal.
The correct sequence would be:
He who has his eyes fixed on the goal does not look at the hurdles on his way.
49. Ans. A.
The correct sequence would be:
We do not know when Shakespeare was born but we know for certain the exact date of his death.
‘S’ would appear first since it shows the uncertainty of the statement. ‘P’ would come next followed
by ‘R’ and then ‘Q’ showing the certain part, i.e. the exact date of Shakespeare’s death.
50. Ans. D.
The correct sequence would be:
A University is essentially a community of students and teachers; the two have to work together; the
purpose if to advance knowledge and disseminate it.
There are multiple sentences here which have been joined together. The opening sentence would
be ‘P’, since it is the independent statement of all. The next would be ‘Q’ since the ‘two’ in this
statement refers to students and teachers mentioned in ‘S’. The next statement would be ‘P’
followed by ‘R’
51. Ans. B.
‘Intrigue’ means arouse curiosity/ interest or fascinate. Hence ‘B’ would be correct.

PAGE 960
www.byjusexamprep.com

52. Ans. A.
As per the given passage, ‘Icebergs are blocks of ice that have broken off huge slabs of frozen snow
called ice shelves’. Thus, from the given options, ‘A’ would be correct, i.e. they are huge pieces of
chunks of ice.
53. Ans. A.
Icebergs are chunks of ice floating on water. Hence, ‘A’ is correct.
54. Ans. A.
It has been clearly mentioned in the passage that when the iceberg turn upside down, it appears
green. Thus, the correct answer would be ‘A’.
55. Ans. A.
The first line of the passage clearly mentions that science has helped man to protect himself from
nature and to overcome natural obstacles to movement.
Thus, option ‘A’ would be correct. Because shelters help man protect himself from nature (rain,
storm etc) and making carts, boats etc help him overcome the natural obstacles to movement.
56. Ans. B.
The 3rd line of the passage implies that man has been attacking nature. Thus, the correct option
would be ‘B’.
57. Ans. D.
The 4th line of the passage mentions that the forests have been destroyed not only to satisfy the need
but to provide luxuries as well. So, the correct option here would be ‘D’, i.e. to provide necessary as
well as needless comforts and pleasures.
58. Ans. C.
The 5th line of the passage answers the question, i.e. the evil effects of deforestation are already
making themselves clearly felt by climatic changes and soil erosion, i.e. unfavourable changes in
climate. Hence, ‘C’ is correct.
59. Ans. D.
Again, this has been clearly answered in 5th line itself. Climatic changes and soil erosion are results
of deforestation.
60. Ans. B.
Civil liberties like right to vote, employment etc help women achieve equality with men. This has
been implied in various sentences of the given passage.
61. Ans. D.
The writer talks about the maidservant in the hotel lobby to imply the importance of economic
independence. Thus, the correct answer would be ‘D’.
62. Ans. A.
The author, in paragraph 4 mentions that women workers in a car factory prefer to stay at home
rather than work in the factory as per a recent study.

PAGE 961
www.byjusexamprep.com

63. Ans. B.
The 1st line of the passage clearly mentions that obedience is no longer the duty of a wife and every
woman has the political right to vote. So, the correct option would be ‘B’.
64. Ans. C.
The 2nd line of the passage clearly mentions that diseases like polio, diphtheria, TB etc are vanishing.
So, the correct option would be ‘C’.
65. Ans. B.
The answer has been provided in the 3rd line of the passage, which clearly mentions that 2/3rd of all
deaths are now associated with diseases of old age and many mass killers have been brought under
control.
So, the correct option would be ‘B’.
66. Ans. C.
The writer has discussed the successes and failures of the western medicines in the passage. So, the
correct option would be ‘C’.
67. Ans. A.
By the statement, ‘Poverty embraces a whole range of circumstances’, the author means that there
are various angles to poverty like lack of access to information, basic services etc.
Thus, the correct option would be ‘A’.
68. Ans. A.
By ‘lack of access to information’, the author means that as a result of poverty, people usually don’t
get information on how to improve their condition.
Thus, option ‘A’ is correct.
69. Ans. A.
Out of the given options, ‘A’ defines ‘cultural identity’ aptly, i.e. cultural identity gives people self-
respect and confidence.
70. Ans. C.
By marginalisation of poor people, the author means that those people are most ignored in the
society.
Hence, option ‘C’ is correct.
71. Ans. D.
The tone used in the passage is objective and critical.
The passage is not influenced by sentiments but presents the actual facts.
72. Ans. C.
The error is in the third part of the sentence. The satisfaction is in the context of the past, thus, "was"
should replace "is".

PAGE 962
www.byjusexamprep.com

73. Ans. B.
The error is in part ‘B’. The correct usage would be: ‘who earned everlasting fame’. Present form of
the tense cannot be used here.
74. Ans. B.
Here there is a comparison between the two boxes mentioned in the sentence, hence a comparative
degree should be used. So, use of ‘heavy’ is incorrect. ‘Heavier’ should have been used here.
75. Ans. A.
The error is in part A of the given sentence. Article "the" should be replaced with 'A'.
The correct sentence: A writer does not have the freedom to choose his own themes, society thrusts
them on him.
Hence, option A is the correct sentence.
76. Ans. D.
The sentence is error-free. Hence, option D is correct
77. Ans. B.
Use of ‘you are’ is incorrect. The correct usage would have been:
How long have you been in this profession?
(Present perfect tense would be used here).
78. Ans. C.
‘Ignorant in’ is incorrect. The correct usage should have been ‘ignorant to’.
79. Ans. C.
The incorrect part is C. This is a sentence of question tag. Since the helping verb used in the sentence
is "are", the question tag will be formed accordingly. Hence, "isn't it?" will get replaced by "aren't
you?".
Thus, option C is the correct answer.
80. Ans. B.
The given sentence is grammatically incorrect and an error lies in option B. Use of ‘living in’ is
incorrect. The correct usage should have been ‘living on’.
81. Ans. C.
The given sentence is grammatically incorrect and an error lies in option C. ‘Engrossed with the
discussion’ is incorrect.
The correct usage should have been ‘engrossed in the discussion’.
82. Ans. B.
‘Listen the song’ is incorrect. The correct usage should have been ‘Listen to the song’.
83. Ans. C.
‘want to watch’ is incorrect. The correct usage should have been ‘wants to watch’.

PAGE 963
www.byjusexamprep.com

84. Ans. B.
Use of ‘estimate’ is wrong here. The correct usage should have been ‘the encyclopaedia Britannica
estimates’.
85. Ans. C.
‘Walk little faster’ is incorrect.
‘Walk a little faster’ should have been used instead.
Little signifies not much/ almost none while ‘a little’ signifies a small amount/ some. Thus, use of
walk little faster is not correct here.
86. Ans. B.
Use of ‘in case impossible’ is incorrect, since it is not in line with the first part of the sentence, which
says ‘almost difficult’.
‘in most cases impossible’ could have been used here to correct the error.
87. Ans. D.
There is no grammatical error in the sentence.
88. Ans. A.
Use of ‘out of the mind’ is incorrect.
‘She was out of her mind’ should have been used here.
89. Ans. D.
There is no grammatical error in the sentence.
90. Ans. B.
Use of ‘besides’ is incorrect. Besides means ‘except’ or ‘in addition to’.
The correct usage should have been:
My college is beside the lake.
Beside is a preposition which means ‘next to’.
91. Ans. B.
The use of ‘much required courses’ is incorrect.
The correct usage should have been:
‘for not having the required courses’.
92. Ans. A.
Use of ‘have thought’ is incorrect. Present tense cannot be used here.
The correct usage should have been:
‘If you had thought about the alternatives’.

PAGE 964
www.byjusexamprep.com

93. Ans. B.
Use of ‘take the minutes’ is incorrect. The entire sentence is in the present continuous form.
The correct usage should have been:
‘Taking the minutes’.
94. Ans. A.
Option A has the grammatically incorrect part. The given sentence is of subjunctive mood and in such
sentences, we do not use "was" even with "I" or singular subjects. We use "were" in place of "was".
For example: If she were the queen, she would pardon your crime.
95. Ans. C.
The error is in part C of the sentence.
Hence, option C is the correct answer.
96. Ans. A.
Use of ‘Furniture’s’ is incorrect.
Furniture should have been used instead.
97. Ans. B.
The correct word would be ‘frightens’ because in the opening sentence, the author mentions that
everything about the bank scares him.
98. Ans. A.
The appropriate use would be ‘sight of the manager’.
Other options, i.e. at, by and on cannot be used with sight here.
99. Ans. B.
The appropriate word would be ‘makes’ me want to run away.
The use of other words, i.e. shapes, shields and asks is not logical here.
100. Ans. B.
The correct use would be ‘as fast as’ I can.
As mentioned in the passage, the author is terrified and so would run as fast as possible.
101. Ans. A.
The complete sentence is that the author loses his mind, hence the appropriate use would be: ‘As
soon as I open the door of the bank.
102. Ans. B.
The second part of the sentence is to be connected here with the first part, i.e. the author loses his
mind and so is not able to do anything there. Hence the appropriate word to join them would be
‘and’.

PAGE 965
www.byjusexamprep.com

103. Ans. B.
The correct word would be when I try to do any ‘business’.
The other words, i.e. service, deed, act do not fit here.
(Service is provided by the bank, not done at the bank).
104. Ans. A.
The appropriate words would be ‘the reasons’, i.e. the author fails to explain why he behaves this
way, i.e. he cannot explain the reason behind it.
105. Ans. A.
The appropriate word would be ‘always’.
The use of other options, i.e. no time, any time, many time is not logical here.
106. Ans. B.
The author is trying to convey that it is still the same for him. Present tense will be used here, given
the form of the sentence. So, the correct word would be ‘Now’.
107. Ans. B.
Died in harness means died before retirement. Hence, option ‘B’ is correct.
108. Ans. B.
‘In consonance with’ means in agreement with.
Thus, the appropriate use here would be ‘B’, i.e. in accordance with.
109. Ans. A.
‘For want of’ is used to in the context of not having (something). So, the correct option here would
be ‘A’, i.e. for lack of.
110. Ans. A.
The phrasal verb ‘dispense with’ means ‘to no longer use someone or something because you no
longer want or need them’. Thus, the correct use here would be ‘terminate, i.e. option ‘A’.
111. Ans. C.
To make someone a scapegoat means to blame someone for a fault although it may not be their
fault.
Hence, the correct option would be ‘C’, i.e. blame without reason.
112. Ans. D.
‘To keep pace with’ means to move or progress at the same speed or rate as.
thus, the correct option here would be ‘D’.
113. Ans. A.
‘In commensurate with’ means ‘corresponding in amount/ magnitude’.
Thus, the correct antonym would be ‘Disproportionate’.

PAGE 966
www.byjusexamprep.com

114. Ans. A.
Emaciated means ‘abnormally thin or weak’.
Thus, out of the given options, the most appropriate antonym for ‘emaciated’ would be ‘hefty’, which
means large and heavy.
115. Ans. B.
“Mammoth’ means huge. Thus, the correct antonym would be ‘Tiny.
116. Ans. C.
‘Hilarious’ means extremely amusing. So, the correct antonym for the word would be ‘Serious’.
117. Ans. B.
‘Erudition’ means the quality of having or showing great knowledge or learning’.
So, the correct antonym here would be ‘Ignorance’, which means ‘lack of knowledge or information’.
118. Ans. A.
Momentous means ‘of great importance or significance’. So, the correct antonym would be ‘Trivial’,
which means ‘of little value or importance’.
119. Ans. C.
Vindicate means ‘Clear of blame or suspicion’. Thus, the correct antonym would be ‘Indict’, which
means formally accuse of or charge with a crime.
120. Ans. B.
‘Ecclesiastical’ means relating to the Christian church or its clergy.
Thus, the correct antonym would be ‘Temporal’ which means ‘relating to worldly as opposed to
spiritual affairs; secular.’.

PAGE 967
www.byjusexamprep.com

General Knowledge
1. Ans. D.
Ministry of environment has launched the air quality index with a view to make people aware about
the quality of air they are inhaling. The quality of air is being judged from good to severe. Eight
pollutants have been included in the list of pollutants namely-PM10, PM2.5, NO2, SO2, CO, O3, NH3
And Pb.
Hence, Option D is correct.
2. Ans. B.
Sansad Adarsh gram yojana is launched by PM Narendra Modi on 11 October 2014 to focus upon the
development in a village. The Member of Parliament is to identify the Adarsh gram, facilitate the
planning process, mobilise funds and monitor the scheme.
Hence, Option B is correct.
3. Ans. B.
Pragati scholarship scheme is launched by Ministry of human source and development. It aimed to
provide assistance for advancement for girls pursuing technical education. 4000 total scholarship per
year will be given.
Hence, Option B is correct.
4. Ans. B.
It facilitates the electronic collection of toll. It is a consumer application. One lane in every toll plaza
will be a dedicated FASTag lane. It can be recharged electronically.
Hence, Option B is correct.
5. Ans. D.
UBI- universal basic income, ensures that every citizen has a right to a minimum income to meet
their basic needs as a long-term solution to reduce poverty.
Hence, Option D is correct.
6. Ans. B.
India has signed the Global Environment Facility Grant Agreement in August 2017 with a view to
enhance forest ecosystem services and to improve the livelihood of forest dwellers. The project
duration is 5 years.
Hence, Option B is correct.
7. Ans. B.
Prime minister Narendra Modi has launched Setu Bharatam on 4th March 2016. The idea behind it
is to make all national highways free of railway crossing by 2019.
Hence, Option B is correct.

PAGE 968
www.byjusexamprep.com

8. Ans. C.
The first partition museum came to existence in Amritsar. The main aim is to showcase the stories,
documents related to riots pre and post partition. The art and cultural heritage trust of UK with the
collaboration of Punjab government has founded the museum.
Hence, Option C is correct.
9. Ans. B.
Before amendment Centre did not have power to tax sale of goods, except in case of inter-state sale,
state cannot levy tax on service. So, article 246 was amended to give provision to the GST bill.
Hence, Option B is correct.
10. Ans. D.
INS Tarini is an all-female led navy vogue. It is a 254-day journey and cover about 21600 miles.
Hence, Option D is correct.
11. Ans. B.
World environment day is celebrated on 5th June every year. It is also called Eco day or WED. Over
143 countries all around the world participates in it. The main aim is to promote the awareness about
our nature and to put efforts to protect it.
Hence, Option B is correct.
12. Ans. D.
On 2nd August 2017 the Israel has launched its first environmental research satellite on Venus from
European spaceport in Kourou. It is a joint venture of Israel space agency and French space agency.
Hence, Option D is correct.
13. Ans. C.
• 7 August is being celebrated as the national handloom day.
• The idea behind is to honour the weavers of the country and to highlight the contribution of
handloom to socioeconomic development of our country.
Hence, Option C is correct.
14. Ans. D.
The executive powers of state are vested with Governor. Unlike president of India, Governor has no
such power to declare emergency in the state except under article 356 that empowers a Governor
to report the President about the failure of government in his state, thus to take over the
administration of that particular state.
Hence, Option D is correct.
15. Ans. A.
All India services is created by declaration by Rajya Sabha with two-thirds support of majority. Article
312 Provides for the creation of All India Services.
Hence, Option A is correct.

PAGE 969
www.byjusexamprep.com

16. Ans. B.
Gupta empire (240-590 CE) is also known as golden age of India. In Gupta empire the women were
given a secondary status. She was confined within the restriction of male member of the family
(father or husband). They did not have any right in parental property. Early marriage was a prominent
aspect of the society also restricting them to gain education.
Hence, Option B is correct.
17. Ans. C.
Dholavira is located in Kadir bet Island, in the Kutch district of Gujarat. It is one of the most important
Harappan sites and one of the five largest cities. Rakhigarhi is situated in the Hisar district of Haryana.
It has its origin in pre-Indus valley civilization. Bhirrana is situated in Fatehabad district of Haryana. It
is one of the ancient towns of Indus valley civilization. Bhogavo is one of the southernmost cities of
Harappan civilization. It is located in the Peninsular region of Gujarat known as Saurashtra.
Hence, Option C is correct.
18. Ans. C.
Raja Ram Mohan Roy laid the foundation of Atmiya Sabha in 1815 in Kolkata. The motto behind was
to provide a platform for discussion on philosophical topics and promote free collective thinking to
lead social reforms. Brahmo Samaj was founded by Raja Ram Mohan Roy in 1828 in Kolkata. It
discarded the old orthodoxy prevailing in the Hindu society by abandoning the rituals, Vedas and had
no faith in avatars.
Hence, Option C is correct.
19. Ans. A.
universal civil code is incorporated in the directive principles of state policy in article 44. According
to it, India strives to achieve them in its future policies and laws to make our society more social and
justice. They are not enforceable by courts unlike fundamental rights.
Hence, Option A is correct.
20. Ans. D.
Tripura is a north-eastern state. Agartala is its capital. Biplab Kumar deb is the chief minister of
Tripura.
Mizoram is a north-eastern state. Aizawl is its capital. Pu Laithanhawala is the chief minister of
Mizoram.
Nagaland is a north-eastern state bordering Myanmar. Kohima is its capital. Neiphiu Rio is the chief
minister of Nagaland.
Manipur is a north-eastern state. Imphal is its capital. N. Biren Heptulla is the chief minister of
Manipur.
Hence, Option D is correct.
21. Ans. C.
The peninsular rivers are much older than that of Himalayan Rivers. Peninsular rivers have reached
the mature stage. They make fluvial landforms. The normal cycle of erosion includes three stages
namely youthful stage, mature stage and old stage.
Hence, Option C is correct.

PAGE 970
www.byjusexamprep.com

22. Ans. A.
Bhimtal lake is located in the state if Uttarakhand. It is a freshwater lake. The formation of lake is due
to earth’s tectonic activities.
Ashtamudi lake is located in the state of Kerala. It is a brackish water lake. It supports a wetland
ecosystem and is included in the Ramasar convection of wetlands.
Gohna lake is located in the state of Uttarakhand. It was formed when a landslide dammed the river,
thus forming a giant lake.
Lonar lake is located in the state if Maharashtra. It is a saline lake created by the meteor impact.
Hence, Option A is correct.
23. Ans. D.
Latitudes are imaginary horizontal lines on Earth running parallel to each other. Latitude of a place
determines its climatic conditions. It affects the solar intensity, temperature, seasonal variations and
much more. Equator divides the Earth in two equal halves.
Hence, Option D is correct.
24. Ans. D.
Tuvalu is one the world’s lowest-lying countries. It is located in South Pacific. Due to melting ice sea
level is rising and according to IPCC projection over the next century the water level would rose by
0.8m making Tuvalu inhabitable.
Hence, Option D is correct.
25. Ans. C.
Ladakh is a cold desert. Lakes in Ladhak are known as Tso. Pangong Tso is the world’s highest salt
lake and it is located in Changtang plateau. Tso moriri is also located in Changtang plateau and it has
been declared a wetland of international importance under Ramasar convection in 2002. Tso Kar is
also called twin lake as a part of it is salty and other is fresh water. It is located in southern Ladakh.
Hence, Option C is correct.
26. Ans. B.
The Varman dynasty of Assam is the first historical dynasty of Karampura dynasty. Pragjyotishpura
was its capital city. Bhaskaravarman 13th king of the dynasty, is known to historians for his alliance
with Harshvardhana.
Hence, Option B is correct.
27. Ans. C.
Harappa civilization is one of the oldest civilizations known to mankind. It is also known as bronze
civilization. The metals known to them were copper, gold, silver and bronze.
Hence, Option C is correct.

PAGE 971
www.byjusexamprep.com

28. Ans. A.
In Tamil epic Silappadikaram, the distinctive features of sun, moon river Cauvery and the city of
Poompuhar, begin to praise a deity. Silappadikaran is one of the five great epics of Tamil literature.
Hence, Option A is correct.
29. Ans. C.
Geoffrey Kay in his book Development and Underdevelopment: A Marxist analysis, has tried to give
a insight view of the impact of capitalism on our society.
Hence, Option C is correct.
30. Ans. C.
The completion of nibandhas is also called Kalpatru, which has a total 14 volumes with different
themes like Puranas, Dharma literatureWorship, pilgrimage etc. Lakshmidhara was responsible for
the completion of nibhandas.
Hence, Option C is correct.
31. Ans. D.
Public interest litigation (PIL) was introduced in 1986. Justice P.N Bhagwati introduced it in Indian
judicial system. It is in the larger interest of society as it is filled by the court itself or a private
individual instead of aggrieved person.
Hence, Option D is correct.
32. Ans. C.
The national commission for women is established in 1992 by an act passed by the parliament. Its
main objective is to review the constitutional and legal status of women.
Hence, Option C is correct.
33. Ans. C.
Every year as a part of the budget process under article 112 of the Indian constitution, it is presented
to the parliament. Rajya Sabha may discuss the annual financial statement but cannot vote on the
demand of grants.
Hence, Option C is correct.
34. Ans. B.
“you have the body” is the literate meaning of habeas corpus. It is issued by the court for the
protection of an individual against unlawful detention.
Hence, Option B is correct.
35. Ans. B.
• Article 311 protests the civil servants from arbitrary dismissal, removal and demotions.
• It safeguards the right of civil servants. Such protection is necessary in order to carry out the
duty fearlessly and efficiently.
Hence, Option B is correct.

PAGE 972
www.byjusexamprep.com

36. Ans. A.
A money bill is generated in Lok Sabha only. Lok Sabha holds exclusive power in the matter of money
bill. A money bill is related to imposition, remission and alteration of any tax. Rajya Sabha is bound
to pass the money bill within 14 days else it is assumed to be passed.
Hence, Option A is correct.
37. Ans. B.
According to article 280 the finance commission is constituted for every five years. The fourteenth
finance commission has come to the conclusion that tax devolution should be the primary route of
transfer of resources to states.
Criteria and weights for the horizontal distribution of the tax is as follows;
Criteria Weight (%)
1. Income Distance 50
2. Population (1971) 17.5
3. Area 15

4. Demographic Change 10
(2011)
5. Forest Cover 7.5

Hence, Option B is correct.


38. Ans. B.
The Deccan plateau heights up to 100 meters in the north and reaches at the height of 1,000 meters
in the south.
The Satpura Range is flanked by Narmada gardens in the north and Parallels Tapi gardens in south.
The Chota Nagpur plateau in eastern India spread in a larger area of Jharkhand and also in Odisha,
Wear Bengal, Bihar and Chhattisgarh. It is composed of Archaean granite and gneiss rocks.
Hence, Option B is correct.
39. Ans. C.
The Satpura range is located in central India. The Satpura range parallels the Vindhya range that lies
north to the Satpura. The Satpura rises in eastern Gujarat and runs to the east of Chhatisgarh. The
Mahadeo hills are located in northern section of Satpura in Madhya Pradesh. Aravalli hills starts from
Delhi and runs from Haryana, Gujarat and ends in Gujarat.
Hence, Option C is correct.
40. Ans. C.
Leipzig is located in Germany. Since the time of Holy Rome Empire, it has been a trade city. It is spread
in 114.81 sq.miles.

PAGE 973
www.byjusexamprep.com

Detroit industrial region is located in the United States of America. It is spread in 142.87 sq.miles. It
is also known as motor city.
Lorraine industrial area is located in France. It is spread in 9,092 sq.miles. It generates about 3.4% of
France GDP.
Cumberland industrial area is located in UK. It is spread in 28.3 sq.miles.
Hence, Option C is correct.
41. Ans. D.
K.N Natrajan founded the Indian Social Reformer in 1890 in Madras but later he moved the journal
to Bombay, an epic centre of social and political review.
Hence, Option D is correct.
42. Ans. D.
All are Sanskrit words. The terms were widely used during Vedic period
Urvaru means fertile land. The term was widely used during Vedic period.
Maru refers to a desert land.
Nadimaatraka is a land watered by river.
Devamatrika is a land watered by rain.
Hence, Option D is correct.
43. Ans. A.
Irfan Habib in his book, ‘The partition of memory’, describes the binary relationship between
extractive state and oppressed peasantry which was initiated by Delhi Sultanate but give a
acceptance under Mughal ruler Akbar.
Hence, Option A is correct.
44. Ans. B.
Tughlaqabad fort is situated in Delhi. Ggiyas-ud-din Tughluq was the founder of it. Its work started
in 1321.
Bada Gumbad in Lodhi Garden, Delhi. It was built in 1490 CE by the ruler of Delhi Sultanate.
Qutub Minar is situated in Delhi. It is a 73-meter-tall Minar built by Qutub-ud-din-aibak and Iitutmish
in 1192.
Fatehpur Sikri is situated in Agra. It was founded as the capital city of Mughals in 1571.
Hence, Option B is correct.
45. Ans. A.
Fiscal deficit is when the actual revenue generated fails to meet the expenditure.
Revenue deficit is basically the mismatch between revenue and the expenditure. Here the amount
of revenue and expenditure do not correspond with each other.

PAGE 974
www.byjusexamprep.com

The amount of capital receipts supposed to be used for actual consumption expenditure of the
government is called Effective revenue deficit.
We get Primary deficit by deducting interest payment from fiscal deficit. It is a part of fiscal deficit.
Hence, Option A is correct.
46. Ans. D.
India changes the base year for its GDP calculation Once in a five year. At present 2011-2012 is used
as the base year for calculating the GDP. This change is made by the central statistics office under
the minister of statistic and programme.
Hence, Option D is correct.
47. Ans. A.
Global Competitiveness index is released by the World economic forum to indicate a country’s rank
in the scale of global competitiveness. The rank has been given to a country after assessing 12
indicators that directly or indirectly impacts the country’s competitiveness.
Hence, Option A is correct.
48. Ans. D.
Industrial revolution gained momentum in 18th century because of the favourable factors it had. Not
only labour but supply an active of raw material, invention and discoveries, change in the method of
transportation all together made a powerful impact on the economy of England.
Hence, Option D is correct.
49. Ans. C.
Drafting committee was set up by the constituent assembly on 29, August 1947 and was headed by
B.R Ambedkar. The task given to it was to draft a new constitution for India. Its members were- N.G
Ayyangar, Krishnaswamy Ayyangar, Dr D.K munshi, Syed Mohammad Saadullah, B.L Mitter(replaced
by N Madhava Rau) and D.P Khaitan( replaced by T T Krishanchari).
Hence, Option C is correct.
50. Ans. D.
According to article 21 “everyone has the right to life, liberty and the security of person”. Article 21
secures two most important rights i.e. right to life and right to personal liberty. It protects us from
the state.
Hence, Option D is correct.
51. Ans. C.
Constitution reads as follow “ Nothing in this article of sub-clause(g) of clause (1) of article 19 shall
prevent the state from making any special provision, by law, for the advancement of any socially and
educationally backward classes of citizens or for scheduled caste or scheduled tribe in so far as such
special provision related to their admission to educational institutions including private educational
institutions, whether aided or unaided by state, other than minority educational institutions referred
to in clause (1) of article 30”.
Hence, Option C is correct.

PAGE 975
www.byjusexamprep.com

52. Ans. C.
The method of presidential election is borrowed from Ireland. The president of India is elected by a
electoral college consist of both the houses of parliament, elected members of legislative assemblies
of states and union territory. Article 58 of Indian constitution sets the qualification and eligibility
criteria for candidate contesting for the post of president.
Hence, Option C is correct.
53. Ans. C.
Directive principles of State Policy, contained in Part - IV in the Constitution are not enforceable by
court but it gives certain guidelines to the government make it their duty to incorporate them in near
future to make society more social, economic and morally justifiable.
Fundamental Right (Article 14 -35) are the basic rights given to every citizen of India irrespective of
sex, colour, caste, creed and religion. To be particular a reasonable restriction has been imposed by
the government on the exercise of right to freedom of speech and expression.
Secularism was incorporated in the Preamble of our Constitution by the 42nd Amendment, making
it an integral part of our Constitution. But, Secularism is not a basic feature of all the countries eg.
Iran. That's why Statement - 3 is incorrect.
Hence, Option C is correct.
54. Ans. C.
Parliament of India consists of Lok Sabha, Rajya Sabha and President. Mutual agreement of both the
houses is required to pass a bill. Therefore, the need of joint session arises to break the deadlock and
to keep the balls rolling. It is preceded by the speaker of Lok Sabha after being called by the President.
Hence, Option C is correct.
55. Ans. B.
Few states like Nagaland, Assam have different provisions regarding Local Self Government. It does
not make mandatory for all states to establish three-tier system of government. However, it doesn’t
apply to a state having a population not exceeding 20 lakhs.
Panchayat is elected directly for the term of five years. However, it can be dissolved before the
completion of the term.
This act reserves the one-third seats for women.
Hence, Option B is correct.
56. Ans. A.
• Fundamental rights are guaranteed to citizens of India by the constitution and it cannot be
taken away except in the time of national emergency.
• During the proclamation of a national emergency, all the basic freedom given to us by article
19 get suspended.
• During emergency, President can suspend other fundamental rights also except article 20 and
article 21.
Hence, Option A is correct.

PAGE 976
www.byjusexamprep.com

57. Ans. B.
Tincture of iodine is an antiseptic solution. It contains 2% iodine, 2.4% potassium iodine in mixture
of 47% alcohol (ethanol + Water).
It was in widely used as disinfectant before surgeries but now it has been replaced by organic form
of iodine.
Hence, Option B is correct.
58. Ans. D.
• Rubber consists of polymers of the organic compound called isoprene.
• Turpentine and Naptha are two major solvents of rubber.
• Rubber is harvested in the form of latex(a white milky substance).
Hence, Option D is correct.
59. Ans. A.
Desalination is a process of removing dissolved salt from seawater. It then become fit for human
consumption and irrigation. It has a great future as our world is going through water crisis. For
desalination of seawater pressure ranges from 800-1000 psi.
Hence, Option As correct.
60. Ans. D.
Urine has urea in it which is basically a waste product. Urea can be broken down into ammonia,
which gives urine a foul smell. The more concentrated urine gets, the more foul smell it gives.
Hence, Option B is correct.
61. Ans. B.
Controlled chain reaction do not lead to the explosive effects whereas uncontrolled chain reaction
lead to explosion. In the context of nuclear power, “critically” means that a reactor is operating
safely.
Hence, Option B is correct.
62. Ans. D.
When an object is said to be moving with constant velocity the resultant force acting on the body is
zero. To bring change in momentum an external force is required to change the momentum.
Hence, Option D is correct.
63. Ans. C.
In numbers of applications, ultrasonic waves are generated by a transducer that consists of a
piezoelectric crystal which converts electrical current to sound waves.
Hence, Option C is correct.
64. Ans. C.
In the adiabatic process no heat is gained or lost by the system. The heat is instead transferred in
surrounding only as work. It is a key concept of thermodynamics.
Hence, Option C is correct.

PAGE 977
www.byjusexamprep.com

65. Ans. A.
Xeriscaping has been developed for drought-affected areas. It has become popular in modern
architectural designs. Its major benefits are: water conservation, less maintenance and improve
property value.
Hence, Option A is correct.
66. Ans. D.
Strontium-90 is close to calcium and when it enters our body it gets fixed with bone mass. It emits
only beta rays with a short range, which makes it harmful. It is often found in seafood that has shell
bodies.
Iodine 131 if swallowed is absorbed into the bloodstream in the gastrointestinal tract. It is
concentrated from the blood by the thyroid gland, where it begins destroying the gland’s cells.
Cesium-137 when inhaled allows the radioactive material to disturb the soft tissue. Exposure of these
tissues to beta particles and gamma radiations increases the risk of cancer.
Hence, Option D is correct.
67. Ans. C.
Emulsion is a mixture of two or more immiscible liquids. Milk, mayonnaise, lemonade are few
examples of emulsions.
Hence, Option C is correct.
68. Ans. D.
One tone (1,000 kg) of carbon dioxide is equal to one carbon credit. Carbon credit is a tradable
certificate that permits to emit carbon dioxide or any other greenhouse gases equal to one carbon
credit.
Hence, Option D is correct.
69. Ans. D.
National highway 2 constitutes major part of grand trunk road. It is of 1,465 km in length. The
highway connects Delhi, Faridabad, Mathura, Agra, Kanpur, Allahabad, Varanasi, Dhanbad and
Kolkata.
National highway 4 is 330.7kms in length. After the renumbering of national highway, this highway
is known as NH-48.
National highway-7 is 770 km in length. It passes through the Indian states of Punjab, Chandigarh,
Haryana, Himachal and Uttarakhand.
National highway-8 is 2,807 km in length. It is one of the busiest highways as it connects two
metropolitan cities Delhi and Mumbai.
Hence, Option D is correct.
70. Ans. B.
Tata iron and steel company (Tisco) is an iron and steel company established by Jamshetji
Nusserwanji Tata in 1907. Its headquarter is located in Mumbai.

PAGE 978
www.byjusexamprep.com

Bharat Aluminium Company Ltd. (BALCO) was established in 1965. Its headquarter is located in New
Delhi.
Bharat Petroleum Corporation LTD.(BPCL). Its headquarter is located in Mumbai. It was established
in 1928 with the name Burmah Shell Oil Storage and distribution company of India.
Bharat electronics limited (BEL) is a state-owned enterprise. Its headquarter is located in Bengaluru.
Hence, Option B is correct.
71. Ans. C.
zoji la pass is located between National Highway 1 between Srinagar and Leh. It is a high mountain
pass in the western section of Himalayan mountain range.
Shipki la is a high mountain pass on the India-China border located in Himachal Pradesh, at a height
of 5,669m. The national highway 5 is passed through it.
Lipulekh pass is located in Uttrakhand at an elevation of 5,099m above the sea level. It is located on
the India-China border.
Napthu la Napthula pass is located at an elevation of 4,310m. It connects the east Sikkim with China’s
Tibet autonomous region.
Hence, Option C is correct.
72. Ans. A.
A strait is a narrow water body that connect two larger water bodies. It is also a water channel lying
between two landmasses.
Australia and Tasmania are separated by a sea strait called Bass Strait. It is approximately 240 km
wide and contains over 50 islands.
Davis Strait separates the Baffin Island of Canada from Greenland. It is about 950 km wide and never
less than 300kms.
Dover Strait separates the UK and France. It is the narrowest part of the English Channel. 33.3 km is
the maximum width.
Florida Strait separates USA and Cuba. It is 150km wide at the narrowest point.
Hence, Option A is correct.
73. Ans. C.
Cassini a common name for Cassini-Huygens mission was a joint venture of NASA, the European
space agency and Italian space agency. The motive behind was to study the Saturn and its system. It
was active in space for nearly 20 years. It was the first space probe to enter its orbit.
Hence, Option C is correct.
74. Ans. A.
Irom Chanu Sharmila belongs to Manipur. She is a civil right activist and political activist. She began
her hunger strike against the AFSPA act in 2000 and continued it for 16 years. The main motive behind
the formation of People’s Resurgence and justice alliance is to Gain power and to pressurize the
central government to nullify the AFPSA from the state.
Hence, Option A is correct.

PAGE 979
www.byjusexamprep.com

75. Ans. D.
The Royal Swedish Academy of science annually awards The Noble Prize in Physics. David J. Thouless,
F.Duncan M. Haldane and J. Michael Kosterlitz were awarded for “theoretical discoveries of
topological phase and topological phases of matter”.
Hence, Option D is correct.
76. Ans. D.
In the field of cinema, Dadashaheb Phalke award is one of the highest and most prestigious awards.
It was instituted in 1969. A cash prize of Rs.1,000,000 is given to the winner with a Swarna Kamal.
Kasinathuni Vishwanath was honoured by Dadasaheb Phalke award in 2016. He has directed 53
feature films in various genres. Also, he has bagged 5 national film awards.
Hence, Option D is correct.
77. Ans. D.
Hailing from Chhattisgarh, Teejan Bai is a master of Pandavani, a traditional performing art form.
Pandavani literally means stories of Pandavas. Instruments like ektara, Kartal and tambura is often
used while performing the dance form. He has been honoured by the Padma Bhushan (2003) and
Padma Shri (1987) award.
Hence, Option D is correct.
78. Ans. B.
The Sustainable Development Goals (SDGs) are a collection of 17 global goals set by the United
Nations General Assembly in 2015 for the year 2030. The SDGs are part of Resolution 70/1 of the
United Nations General Assembly, the 2030 Agenda.
Hence, Option B is correct.
79. Ans. A.
Jnanpith award is awarded annually to an author for the “outstanding contribution towards
literature”. It was instituted in 1961. It is awarded to Indian authors for writing in one of the language
included in the Eight Schedule to the constitution of India. Shankha Ghosh was 52nd recipient of this
award and was awarded for writing in Bengali.
Hence, Option A is correct.
80. Ans. D.
When the government gets its rights to exist and to govern from the people to be governed it is
called “Social Contract”. In his book he has explained various types of government. His concern
always remains to make society as democratic as possible.
Hence, Option D is correct.
81. Ans. C.
National convection was a more radical legislative body formed in 1792. It remained in power for
three years. It had membership of 749 members from France and also from other countries. Only
men 25 or above were allowed its membership.
Hence, Option C is correct.

PAGE 980
www.byjusexamprep.com

82. Ans. C.
Parshva also known as Parshvanatha was 23rd Tirthankara. His biography is uncertain and by some
historians he may have lived in 8th or 7th century.
Buddha introduced the idea of placing the morality and equality as the principal value not the caste
system and so did the Jainism but neither of them tried to abolish the caste system.
Monks and nuns both had equal rights. Both could attain Nirvana.
Hence, Option C is correct.
83. Ans. A.
After the death of Aurangzeb, the Sayyid brothers became very influential. Jahandar Shah was the
successor of Bahadur Shah I, who succeeded him after his death in 1712. But Jahandar Shah was
assassinated on the order of Sayyid brothers.
Hence, Option A is correct.
84. Ans. A.
Phulaguri Dhawa was the first organised peasant movement in Assam. The Britishers started
levying taxes on items of regular use like bamboo, opium, water, wood, etc. It annoyed the local
people and for the first time, the British administration was shaken. It took a violent turn when
Britishers killed and hanged the peasants to took over the control.
Hence, Option A is correct.
85. Ans. A.
Aryans is a term used for Indo-Iranian people. They migrated into Indian subcontinent around 1500
BCE. The Aryan society was patriarchal. They were primarily nomadic people. Vedas were recognised
as the most valued religious text by them.
Hence, Option A is correct.
86. Ans. A.
In 1928 communist party decided to fight on two fronts for Country’s freedom - To oppose Congress'
nationalism and To oppose British imperialism. However, in 1934, because of the radical approach
by communist the government put a ban on its functioning. Thereafter, they were no longer working
with the mainstream national movement.
Hence, Option A is correct.
87. Ans. A.
An increase in the average temperature of earth is called global warming. Since the commencement
of industrialization in 18th century the temperature of earth has been significantly increased. Gases
like methane, carbon dioxide, water vapour etc are the greenhouse gases which trap the heat from
sun and don’t let it go from earth, resulting increase in temperature.
Hence, Option A is correct.

PAGE 981
www.byjusexamprep.com

88. Ans. A.
A rain shadow area is an area which lies in the leeward side of mountain. The passage of rain-
producing clouds is blocked by the mountains casting a shadow of dryness. State of Maharashtra
comprises of four regions out of which 30% comes under rain shadow area. Interior of Maharashtra
represent one of the rain shadow area as it is situated on the foot of Sahyadri.
Hence, Option A is correct.
89. Ans. C.
One of the major cause of skin cancer is exposure to ultraviolet (UV) rays. Sun is the main source of
UV rays. UV rays are responsible for damaging the skin cells which hampers the growth regulation of
the skin cells.
Stratosphere protects the earth from the harmful effects of UV rays by absorbing it. But because of
environmental misbalance and anti-ozone compounds, the stratosphere has developed a hole in it
which allows UV rays to reach the earth.
Hence, Option C is correct.
90. Ans. B.
Devaluation is one of the monetary policy tools. It is often used to combat the trade imbalance.
Devaluation is beneficial for the economy of a country as it makes the country more competitive in
global market.
Hence, Option B is correct.
91. Ans. A.
Monetary policy is a process through which RBI controls the input and output of money in the
economy, thus regulating the inflation.
Bank rate is the rate of interest on which the RBI provides loan or funds to the commercial banks. It
is also called discount rate. The present bank rate is 6.25%
Bank needs to keep a certain percentage of bank deposits with the RBI in the form of reserves and
balance. The present bank rate is 4%.
Hence, Option A is correct.
92. Ans. A.
Mahalanobis model was an integral part of India’s second five-year plan (1956-1961). The zest of this
model was to build up a domestic consumption goods sector and to modify the pattern of industrial
development.
After the failure of third five-year plan because of Indo-Sino war, Indo-Pakistan war and a severe
drought in 1965 the Government had to finally announce a plan holiday in 1966-1969.
The fifth five-year plan was rejected by the Janta party government and sixth five-year plan was
introduced. It was further rejected by the Indian National Congress in 1980 and a new plan was again
proposed by them. Now the rolling plan came to the rescue which had essence of all the three
previous plans.
Hence, Option A is correct.

PAGE 982
www.byjusexamprep.com

93. Ans. B.
Demographic dividend refers to the working age group people that a country has that can be the
potential working force of the country. A favourable demographic dividend can take a country’s
economy to the path of success. India has a positive demographic dividend while country like Japan
and UK has dominantly old population.
Hence, Option B is correct.
94. Ans. C.
The country has produced 21.4 million tons in 2017-2018 while the country has harvested 22.4
million tons in 2016-2017.
The production of potatoes is estimated to be 49.3 million tons while 48.6 million tons in 2016-2017.
Hence, Option C is correct.
95. Ans. D.
Fourth schedule [Article 4(1) and 80(2)] of Indian constitution allocates the seats of states and union
territory in the Rajya Sabha (council of state). Members of Rajya Sabha are indirectly elected by the
members of Legislative Assembly of State and Union Territory by mean of the single transferable
vote through proportional representation.
Hence, Option D is correct.
96. Ans. D.
Indian constitution is also called ‘bag of borrowing’ as many of its features are taken from the
constitution of other countries. UK is one such country from where our constitution has borrowed
the features like President to be the nominal head, parliamentary type of government, single
citizenship, rule of law, cabinet of ministers, law-making procedures and speaker in Lok Sabha.
Hence, Option D is correct.
97. Ans. C.
The act of distributing responsibilities or authority among the subordinates or co-workers. The
member to whom the work is being given is called a delegate. The person who tends to delegate the
work remain responsible for the final outcome.
Hence, Option C is correct.
98. Ans. B.
As per Article 40 and 246(3), the Constitution empowers the state to make laws for functioning of
self-governing unit.
Haryana and Rajasthan are two such states that has fixed a minimum educational qualification
mandatory for those contesting the Panchayat Elections.
Hence, Option B is correct.
99. Ans. B.
Biogas is an environmental friendly fuel; produced by anaerobic action of microorganism that feeds
on animal and agricultural waste. It is a renewable source of energy. It is made up of methane (60-
70%), carbon dioxide (25-50%), nitrogen (0-10%) and hydrogen (0-1%).
Hence, Option B is correct.

PAGE 983
www.byjusexamprep.com

100. Ans. C.
About 90% of ozone gas is present in the second atmospheric layer called Stratosphere. It is about
15-30 km above the Earth’s surface. It plays a vital role by protecting us from the harmful effects of
UV rays. Because of environmental pollution and excessive use of CFCs the ozone layer is in danger.
Hence, Option C is correct.
101. Ans. D.
Flint glass is softer than any other glass; it is also called optical glass. Two major components used to
make it is silicates and potassium. It is transparent and clear. It has high refractive index. It is widely
used for making lenses and prism.
Hence, Option D is correct.
102. Ans. B.
• POP is a white powdery slightly hydrated calcium sulfate CaSO4·¹/₂H2O or 2CaSO4·H2O made by
calcining gypsum and used chiefly for casts and molds in the form of a quick-setting paste with
water.
• It gets hardens when moistened and allowed to dry slowly.
• It came in orthopaedics use in 1852.
• It does not produce allergies.
• It is a perfect solution to prevent pain caused due to fractures as it immobilises the movement.
Hence, Option B is correct.
103. Ans. D.
12 atoms of hydrogen is present in per molecules of glucose. It has a molar mass of 180g/mol. Thus,
the number of moles of hydrogen is 12*1.50/180= 0.10 mol. So, when we multiply the numbers of
hydrogen atom by the Avogadro constant, 6.02*10^23, we get 6.02*10^22 atoms.
Hence, Option D is correct.
104. Ans. C.
Protons is often released by an acidic substance and it is accepted by a basic substance. Here Water
acts like an amphoteric.
Hence, Option C is correct.
105. Ans. A.
When we sneeze, air expiates with force through our nose and mouth. According to third law of
motion, there is an equal and opposite reaction for every action. So, a frictionless horizon which
cannot grip us will make us move in opposite direction.
Hence, Option A is correct.
106. Ans. C.
A transformer is a device that regulates the fluctuation of electricity, thus protecting the electrical
equipment. Transformers get heated up because of numbers of reasons. Excess of heating may
reduce the life of a transformer and also weakens the insulation.
Hence, Option C is correct.

PAGE 984
www.byjusexamprep.com

107. Ans. D.
Photons energy is inversely proportional to wavelength. Wavelength of visible light is 10^-5 whereas
wavelength of ultraviolet is 10^-6. Thus, it clearly shows that photons energy of ultraviolet is greater
than that of visible light.
Hence, Option D is correct.
108. Ans. A.
A ray of light which is incident to surface of the object is called Ray of Incidence or Incident ray and
the ray of light which get reflected after striking to the surface of the object is called Ray of reflection
or Reflected ray. The ray of incidence and the ray of reflection make the same angle with respect to
normal i.e Angle of Incidence (i)= Angle of Reflection (r). We measure these angles with respect to
the normal of surface.

Hence, Option A is correct.


109. Ans. B.
The resistance to the flow of electrons is called resistivity. Conductivity is just inverse of resistivity.
Both are the property of conductor. The interesting relationship between them is, as the conductivity
increase resistivity decrease.
Hence, Option B is correct.
110. Ans. B.
When accelerated from rest the energy is fully kinetic so
KE= charge*potential difference
i.e 1/2 Mv^2=Q.V ……………………1
From the above equation it is obvious that velocity of particle is inversely proportional to the mass
of particle so, electron will gain more speed as it has lower mass than that of proton. Also, as the
energy is fully kinetic so electron will have greater energy as a result of greater velocity.
Hence, Option B is correct.
111. Ans. D.
An ecosystem is a community of plants and animals also includes abiotic components, interacting
with each other in a given area. Organisms live together to benefit one another. In an ecosystem
each component has its own role to play. An ecosystem can be taken as the foundation of biosphere.
It has all the essentials that support the life of a particular type of niche.
Hence, Option D is correct.

PAGE 985
www.byjusexamprep.com

112. Ans. A.
A pearl is a hard substance which is formed by the concentric layer of Calcium carbonate. Pearl is
found inside a living shelled mollusc. Calcium carbonate is the building block of shells. Concentration
of Calcium carbonate can be higher in deep sea water than the surface water. It is higher in seawater
than in fresh water. Corals are highly dependent on it for the formation of rigid outer shell.
Hence, Option A is correct.
113. Ans. B.
Insulin is a protein composed of two chains, A chain (with 21 amino acids) and a B chain (with 30
amino acids) and sulphur atoms link them together.
Hence, Option B is correct.
114. Ans. C.
Mitochondria is also known as “powerhouse of cell”. It generates the energy which is required by a
cell to perform its activities. The energy is produced in the form of adenosine triphosphate (ATP).
Chloroplast is found in plants to perform the photosynthesis.
Ribosomes is made up of ribosomal RNA molecules and proteins which together forms a factory for
protein synthesis in cells. It is found in cytoplasm. It is present in both eukaryotic cell and prokaryotes
cell.
Lysosomes are also called “suicide bags”. It is found in animal cells. It is bounded by only a single
membrane.
Hence, Option C is correct.
115. Ans. C.
Excessive nitrates in drinking water can cause the blue baby syndrome. When it enters in an infant’s
body it gets converted into nitrite in its digestive system. It then reacts with the haemoglobin present
in the blood with together forms methaemoglobin, which cannot carry oxygen, thus organs may
deprive of oxygen. As a result, thin-skinned areas like lips, earlobes develop a bluish colour. It may
also cause respiratory and digestive problems in adults.
Hence, Option C is correct.
116. Ans. A.
Silk is a fibre produced by the silkworm . It consists mainly of two proteins, fibroin and sericin.
Silk consists of 70–80% fibroin and 20–30% sericin; fibroin being the structural center of the silk, and
sericin being the gum coating the fibres and allowing them to stick to each other.
Hence, Option A is correct.
117. Ans. C.
Dudhwa national park is located in Terai of Uttar Pradesh and is spread across 490.3 sq.km area.
Panna national park is located in Panna and Chhatarpur districts of Madhya Pradesh. It is spread
across 542.67 sq.km area.

PAGE 986
www.byjusexamprep.com

Pench national park is spread across Madhya Pradesh and Maharashtra.


Indravati is located in Bijapur district of Chhattisgarh.
Hence, Option C is correct.
118. Ans. C.
New Guinea is located off the continent of Australia. It is the second-largest island in the world. It is
spread across 785,753 sq.km area.
Borneo is located in Southeast Asia. It is the largest island in Asia and third largest in the world. It is
spread across 748,168 sq.km area.
Madagascar is located in Africa. It is spread across 587,041 sq.km are. It is widely known for its
biodiversity.
Sumatra is located in southeast Asia. It is an Indonesian island. It is spread across 443,065 sq.km area.
Hence, Option C is correct.
119. Ans. B.
A primate city incommensurably large city in the urban pecking order of a country or region.
Geographer Mark Jefferson developed the ideology of the same in 1939. He defined it as the city
that is twice as large as the next city and twice as significant. It has the amenities of international
importance.
Hence, Option B is correct.
120. Ans. D.
The Blue Mountains also called Nilgiris are spread across Tamil Nadu, Kerala and Karnataka. They are
the meeting point of Eastern Ghats and Western Ghats mountains. The highest peak of Blue
mountain is Doddabetta (2,637meters).
Toda tribe is an ancient and unusual tribe of Blue mountains. They belong to Dravidian ethnic group.
They are basically involved in cattle-rearing and dairy work. The total population of Todas as per 2001
census is 1560.
Hence, Option D is correct.

PAGE 987
www.byjusexamprep.com

Elementary Mathematics
1. Ans. C.
Given average height of 22 students of a class is 140 cm

total height of 22 students =


And average height of 28 students of another class is 152 cm

total height of 28 students


=

Now average height of total students = =

Hence option (c)


2. Ans. B.

The "Mean" is the "average" =

The "Median" is the "middle" value in the list of numbers, given number is 144,150,155,161 and 165,
middle value of given
numbers is 155 i.e. median of given number is 155
Mean = 155 and median = 155
Hence option (b)
3. Ans. B.
As we know that secondary data is the data collected from sources other than user itself.
Thus, The total number of live births in a specific locality during different months of a specific
year was obtained from the office of the Birth Registrar. This set of data may be called
secondary data.
Hence option (b)
4. Ans. C.

As, geometric mean of x and y = = 6 …..(i)

Also, geometric mean of x, y and z = = 6 …..(ii)


From (i) and (ii), we get

PAGE 988
www.byjusexamprep.com

⇒ x3y3 = x2y2z2

⇒ xy = z2
⇒ 62 = z2 (From equation (i))
Hence, z = 6
Hence option (c)
5. Ans. D.
A pie chart is a circular statistical graphic which is divided into slices to illustrate numerical
proportion. In a pie chart, the arc length of each slice (and consequently its central angle and area),
is proportional to the quantity it represents.
Hence the proportion of various items in a pie diagram is not proportional to the Perimeters of
the slices.
Hence option (d)
6. Ans. C.
Statement 1:
As we know that the value of a random variable having the highest frequency is mode.
Hence statement 1 is correct
Statement 2:
A distribution having single mode is known as Unimodal and the distribution having more than
one mode bimodal, trimodal etc or in general multimodal.
Thus, mode is not unique.
Hence statement 2 is incorrect.
Hence option (c)
7. Ans. C.
ratio of radius of pie diagram for expenditure = 16 : 9

the ratio of their expenditure should be in ratio of their area

so, the ratio of their expenditure

Hence option (c)

PAGE 989
www.byjusexamprep.com

8. Ans. A.
As we know that,

Hence option (a)


9. Ans. A.

Mean Salary of workers =

Clearly, workers of B and C categories are gaining salary which is less than the mean salary of
the workers.
Hence the number of workers earning less than the mean salary = 3 + 5 = 8
Hence option (a)
10. Ans. A.
As, we know that Frequency density of a class is the ratio of Class frequency to the class length
Or, Frequency density of a class is the ratio of Class frequency to the class width.
Hence option (a)
11. Ans. A.
Total number of Boys = 150 + 180 + 70 + 136 = 536
Total number of Girls = 90 + 120 + 250 + 204 = 664
The ratio of the total number of boys to the total number of girls = 536 : 664 = 67 : 83 (After dividing
by 8)
Hence option (a)
12. Ans. B.
Number of boys studying Mathematics = 150
Number of girls studying Physics = 120

PAGE 990
www.byjusexamprep.com

Difference between the number of boys studying Mathematics and the number of girls studying
Physics = 150 – 120 = 30
Hence option (b)
13. Ans. B.
Number of girls studying Statistics = 250
Total Number of students studying Chemistry = 340
Thus, required percentage =

Hence option (b)


14. Ans. C.

Total number of boys studying Statistics and Physics = 70 + 180 = 250


Hence option (c)
15. Ans. C.
Statement 1:
Since the point of intersection of the perpendicular bisectors of the sides of a triangle is called
circumcentre and the circumcentre for an obtuse triangle lie outside the triangle.
Hence statement 1 is correct.
Statement 2:
In a right triangle perpendicular drawn from two vertices of hypotenuse are the other two side of
the triangle observe that they meet on the vertex and lie on the side , so both statements are correct.
16. Ans. B.
For finding the sum of the interior angles of a polygon is the same, whether the polygon is regular or
irregular.
So, we would use the formula (n — 2) x 180 °.
Where n is the number of sides in the polygon.
Let one angle of the polygon be x and other 5 equal angles be y.
According to the question,
x = y + 30…….(i)
(n - 2)×1800 = x + 5y……(ii)

PAGE 991
www.byjusexamprep.com

From (i) and (ii), we get


(n - 2)×1800 = y + 30 + 5y
⇒ (6 - 2) ×1800 = 30 + 6y
⇒ 4×1800 = 30 + 6y
⇒ 6y = 690

⇒y=

Hence value of equal angles be 1150


Hence option (b)
17. Ans. C.
As we know that, two non-parallel lines are always intersect at a point.
Thus, the maximum number of points at which they can intersect = 8c2

Hence option (c)


18. Ans. B.

According to question,
By basic proportionality theorem
and angle A is common

So, Triangle ADE ~ triangle ABC [By SAS similarity]


As, we know that the ratio of the areas of two similar triangles is equal to the square of its
proportional sides

Thus, …..(i)

Given that,
…….(ii)

From (i) and (ii), we get

PAGE 992
www.byjusexamprep.com

⇒ =

Hence option (b)


19. Ans. B.

Length of arc =

For finding the value in degree, multiple with with RHS

Thus, 22 =

Hence radius of circle be 7 cm


Hence option (b)
20. Ans. B.

Since the diagonals of rhombus bisect each other.


Thus, AO = OC = 10 cm and BO = OD = 24 cm
In triangle AOB:
Using Pythagoras theorem, we get

PAGE 993
www.byjusexamprep.com



Hence side of rhombus be 26 cm
Hence option (b)
21. Ans. B.
Statement 1:

Clearly, statement 1 is correct.


Statement 2:

Here, A is perpendicular to C and B is also perpendicular to B.


So, A must be parallel to B
Statement 3:

Clearly, A is parallel to B. but if is on another side then it is not necessary to be parallel


Hence all the three statements are correct.
Hence option b
22. Ans. A.

PAGE 994
www.byjusexamprep.com

As,

So, by converse of basic proportionality theorem


XY parallel to PQ
Hence option (a)
23. Ans. A.

Since BE is the median of AC.


Therefore, AE = EC
Also, AC = 9 cm
So, AE = EC = 3 cm
i.e. FC < EC
Thus, according to option FC = 2.25 cm
Hence option (a)
24. Ans. B.
The maximum possible volume of the circular cylinder that can be formed from a rectangular sheet
will have the largest length and breadth.
So, we will consider the rectangular sheet with length 4π and breadth 2π.
The length of the rectangular sheet = circumference of the cylinder
=>4 π = 2 πr
=>r = 2
Volume of cylinder = πr2h = π×4×2 π = 8 π2
Hence option (b)
25. Ans. B.
As, volume of cylinder = [As, r = 50 m and h = 10 m]

PAGE 995
www.byjusexamprep.com

Volume of cone = [As, h =


15 – 10 = 5]
Total volume =

Hence option (b)


26. Ans. D.
As, the conical cavity in the cylinder is hollowed out.
Therefore, inner surface area of the cavity is curved surface area of the cone.

Thus, curved surface area of cone =

= =
Hence inner surface area of cavity = sq. cm
Hence option (d)
27. Ans. D.

Let the radii of frustum of a cone be R and r

Given that, ………(i)

Let angle AC’O’ = angle ACO = θ


Now, in triangle AC’O’:

tan θ = [As, ] …….(ii)

In triangle ACO,

tan θ = ……(iii)

From (ii) and (iii), we get

PAGE 996
www.byjusexamprep.com

⇒ 2h = h + x
⇒h=x
Therefore, H = h + x = h + h = 2h

Now, volume of frustum of cone = and volume of cone =

Hence required ratio =

Hence required ratio be 7 : 8


Hence option (d)
28. Ans. C.
Let the radius of base circle of cone be r cm
Given that slant height(l) = √2r cm

Then, height of cone (h) = = r2 = =r

Hence volume of cone =

Hence option (c)


29. Ans. D.
Since the sphere is dropped in the cylindrical vessel partially filled with water and is completely
immersed.
Therefore, the volumes of both will be equal.
Let r be the radius of cylinder and R be the radius of the sphere.
Thus, volume of cylinder = volume of sphere

⇒ [As, r = 30 and R = 15]

Hence height of water in the cylinder rise = 5 cm


Hence option (d)

PAGE 997
www.byjusexamprep.com

30. Ans. A.

In triangle OAB, by using Pythagoras theorem

Radius of triangle = = = = x
Curved surface area of cylinder = 2πrh = 2π(√3x)(2x) [As, r = √3x and h = 2x]
= 4√3πx2
Surface area of the sphere = 4πr2 = 4π(2x)2 = 16πx2

Hence required ratio =

Hence required ratio be √3 : 4


Hence option (a)
31. Ans. C.

As, ABCD is a rectangle with point O inside the rectangle.


Draw lines OA, OB, OC and OD
Again, draw from point O perpendicular to the sides i.e. OE, OF, OG and OH.
We can use Pythagorean theorem in different right angled triangle in above figure.
[As, OH =AE]…….(i)
[As, CG = EB and OG = HD]……(ii)
[As, EO = AH]………(iii)
[As, OH = AE]……(iv)

PAGE 998
www.byjusexamprep.com

Adding (i) and (ii), we get


…….(v)
Adding (iii) and (iv), we get
…….(vi)
From (v) and (vi), we get

Hence option (c)


32. Ans. D.

⇒ [As, ]



Thus,
Hence option (d)
33. Ans. D.

PAGE 999
www.byjusexamprep.com

From the figure, clearly Q is outside the triangle OPR.


Also, triangle OPR is an obtuse angled triangle.
Since, orthocentre of an obtuse angled triangle is always outside the triangle.
Thus, Q is orthocentre of the triangle OPR.
Hence option (a)
34. Ans. A.
Given that radius of first circle(R) = 9 cm
radius of second circle(r) = 4 cm
Distance between the centres of two circles(d) = 13 cm

The length of the direct common tangent of these circles =

= = = = 12 cm
Hence option (a)
35. Ans. C.

Since ABC is an equilateral triangle and BD is a perpendicular, therefore AD = DC


In triangle BCD, using Pythagoras theorem,


⇒ [As, BC = AC]


⇒ [As, CD = AD]
⇒ [As, DC = AD]

Hence option (c)

PAGE 1000
www.byjusexamprep.com

36. Ans. D.

Since ABCD is a cyclic quadrilateral and a trapezium so, AB parallel CD


In triangle APB and triangle CPD:
Angle CDP = angle ABP (Alternate interior angle)
Angle DCP = angle PAB (Alternate interior angle)
Thus, by AA similarity criteria triangle APB ~ triangle CPD
Now, the ratio of areas of similar triangles is equal to ratio of the squares of one of its proportional
sides
Thus,

⇒ area of triangle CPD =

Hence option (d)


37. Ans. C.

Given that AB = 4 cm and CD = 10 cm


Let the radius of the circle be r cm

PAGE 1001
www.byjusexamprep.com

Since the perpendicular from the centre of a circle to a chord bisects the chord
So, AO = OB = 2 cm and CP = PD = 5 cm
In triangle AOX:
By using Pythagoras theorem,
22 + (3 + x)2 = r2
⇒ 4 + 9 + x2 +6x = r2
⇒ 13 + x2 +6x = r2 …….(i)
In triangle CPX:
52 + x2 = r2
⇒ 25 + x2 = r2 …….(ii)
From equation (i) and (ii), we get
13 + x2 +6x = 25 + x2
⇒ 6x = 12
⇒x=2
From equation (ii), we get
r2 = 25 + 22 = 25 + 4 = 29
⇒r= cm
Hence option (c)
38. Ans. C.

In triangle ABC:
AD is perpendicular on BC
Angle BAC = 900
As, AB = c, BC = a, CA = b and AD = p
Area of triangle ABD = area of triangle ACD

PAGE 1002
www.byjusexamprep.com



Hence option (c)


39. Ans. B.

As, PQR is a triangle and QR = r


Radius of circle = r
Area of shaded region = area of triangle PQR – area of sector PQSP

⇒ Area of shaded region =

Hence option (b)


40. Ans. C.
By using Heron’s formula:

Semi perimeter(S) = [As, a = 51, b = 37 and c = 20]

Area of triangle = =

= = =3
Hence option (c)
41. Ans. D.

PAGE 1003
www.byjusexamprep.com

Let the side of each of the square be a and the other two sides of the triangle be x
7
Given, Perimeter of the complete figure = (perimeter of the original square)
6
7
⇒ 3a + 2x = (4a)
6
7
⇒ 3a + 2x = (2a)
3
⇒ 3(3a + 2x) = 14a
⇒ 9a + 6x = 14a
⇒ 6x = 5a ………(i)
Using Pythagoras theorem
In upper triangle:

h= = = = = = =

Again,

Ratio of the area of triangle to the original square = = =

Hence required ratio be 1 : 3


42. Ans. B.

Let the radius of bigger circle be x and radius of the smaller circle be y.
Then the angle made by direct common tangents when two circles of radius x and y touch externally
is given by

Θ=

Given that area of the bigger circle = 9 ×(area of the smaller circle)


Let us consider Angle BAC = θ

PAGE 1004
www.byjusexamprep.com

Then, θ = = 2×300 = 600

Hence option (b)


43. Ans. B.
Given that a rectangular cello tape of length 4 cm and breadth 0.5 cm is used for joining each pair of
edges.
there are 12 edges in a cube
each edge required 4x.5 sq. cm. of tape
so total area = 12(4x.5) = 24 sq. cm.
44. Ans. D.
Let the radius of two circle are 12r and 5r
And, sum of area of two circles =

Area of circles whose diameter is 65 cm =

According to question,

Hence radius of the circles are


Hence option (d)
45. Ans. B.
As,
1c = 180/ π0 = 180x7/22 = 57.320
As we know sin increases from 00 to 900
Thus sin 10 < sin 57.320
or sin 10/sin 57.320 <1
Hence option (b)
46. Ans. C.

PAGE 1005
www.byjusexamprep.com

Here, AB is the breadth of river.


In triangle ABX:

tanα0 = => 2 = =>

In triangle ABY:

tanβ0 = =>

Given, x + y = 200

Hence the breadth of river be 80 m


Hence option (c)
47. Ans. C.

In triangle AA’C:

tan450 = ⇒ ⇒ AC = BB’ = x ⇒ x = d

Adding h on both sides, we get


h+x=h+d
So, h + x > d
Hence statement 1 is correct.
In triangle A’BB’:
If we take angle be 450

Then, tan450 = ⇒

⇒ d=h+x
But, by statement 1, this is not possible.
Thus, θ ≠ 450

PAGE 1006
www.byjusexamprep.com

Now, either θ < 450 or θ > 450


Let, θ = 600 > 450
In triangle A’BB’

tan 600 = => √3 = => d√3 = h + x ……. (i)

Let, θ = 300 < 450


In triangle A’BB’

tan 450 = => = => d = √3(h + x) ……. (ii)

From (i), we can conclude that either LHS = RHS or, LHS > RHS
But, from (ii), clearly LHS < RHS
Hence, we cannot conclude that the angle of depression of B from A’ is less than 450
Hence statement 2 is incorrect.
Hence option (c)
48. Ans. C.
Statement 1:
As,
If we consider θ = 450
Then, sec2θ + cosec2θ = sec2450 +cosec2450 = 2 + 2 = 4
So, α2 = 4 => α = 2 > 1
If we consider θ = 600

Then, sec2θ + cosec2θ = sec2600 +cosec2600 = 4+ =

So, α2 = => α = 2.31 > 1

Thus, statement 1 is correct.


Statement 2:
As,
If we consider θ = 00
Then,

So, x2 = => x =
If we consider θ = 450

Then,

PAGE 1007
www.byjusexamprep.com

= = 3 + 2√2 = 5.828

So, x2 = => x =2.414 > 2


Hence statement 2 is correct.
Statement 3:
As,
If we consider θ = 00 then it will give 1 which is less than 2
If we consider θ = 450

Then, = =∞≥2

Hence statement 3 is correct.


Hence all the three statements are correct.
Hence option (d)
49. Ans. B.
For calculating the angle between the hour hand and the minute hand of a clock when the time is
4 : 36 pm, we can say that the angle will be approximately equal to the angle made from 4 :20 pm to
4 : 36 pm.
Thus, we need to calculate the angle made by the hands of a clock in 16 minutes.
In 60 minutes, the angle made by the hands of a clock is 360 °.

So, the angle made by the hands of a clock in 16 minutes =

Thus, the angle lies between 72 ° to 108 ° i.e.

Hence,

Hence option (b)


50. Ans. A.

As,

= = (3 - 2×1.41) = 0.18

Now,

= (3-2√2) = (3 - 2×1.41) = 0.18


Hence, A = B
Hence option (a)

PAGE 1008
www.byjusexamprep.com

51. Ans. B.

Since and we know that the angle subtended by an arc at the centre of a circle is double
the angle subtended by it at any point the remaining part of the circle, therefore in the centre,

Also, since triangle OBC is an isosceles triangle so, its base angles will he equal i.e. B and C
are equal.
Let these angles be x.
Therefore, by angle sum property of a triangle,

O+ B+ C = 180 degree => 60

→ 60
Thus, we say that triangle OBC is an equilateral triangle and hence, BC is also equal to 10 cm.
Hence option (b)
52. Ans. A.
Suppose we have right angled triangle with sides a, b and c where c is the longest side.

Now, we see that

Here, we can see that in both the denominators we have the same hypotenuse which means from
all the given options,
Only option (a) has the same hypotenuse as given in the question i.e.
Hence option (a)

PAGE 1009
www.byjusexamprep.com

53. Ans. B.
As, 0 < x <

Then, => 0 < sinx < 1 ……(i)

0<x<

Then, => 1 > cosx > 0 ……(ii)

Adding (i) and (ii), we get,

Hence option (b)


54. Ans. D.

Since we know that 5, 13 and 12 forms a Pythagorean triplet, the side with 13 units is the longest
side and the angle between the other two sides is 900

Therefore

Thus

Hence option (d)


55. Ans. D.
Let the speed of the man in still water be x km/hr and
let the speed of the stream be y km/hr
Speed of the man downstream = x + y km/hr
Speed of the man upstream = x -y km/hr

Therefore ….. (i)

…….. (ii)

PAGE 1010
www.byjusexamprep.com

Solving these equations by elimination method, we get


…….(iii)

………(iv)

Therefore, equations (ii), (iii) and (iv) implies that all the given statements are correct
Hence option (d)
56. Ans. D.

Decrease in consumption =

Hence, consumption of onion should be decreased by % so that there is no change in

the expenditure
Hence option (d)
57. Ans. A.
The difference between the compound interest (compounded annually) and simple interest on a

sum or money deposited for 2 years at R% p.a. be

⇒ P = 6000
Hence option (a)
58. Ans. D.
Given that the ratio of the number of boys in the first and the second standards is 2 : 3 and
the ratio
The number of boys in the second and third standards is 4 :5
Now,
we calculate a common ratio for all the three standards
2 : 3 and 4 : 5will be 2 × 4:3 × 4 = 8 : 12 and 4 × 3 : 5 × 3 = 12 : 15
Therefore, the common ratio for all the three standards 8 : 12 : 15
Sum of the ratio pats = 8 + 12+ 15 = 35

PAGE 1011
www.byjusexamprep.com

Numbers of the boys in the first standard = = 80

Number of boys in third standard =

Total number of boys in the both standards = 80+150= 230


Hence option (d)
59. Ans. C.
Given that 3 parts are proportional to 1,

LCM of the denominator is 6.


Therefore the ratio will be i.e. 6 : 2 : 1

Sum of the ratio part is 9, the middle part of 78 is

Hence option (c)


60. Ans. B.
the speed of boys to go to school is 12km/hr
it's mean he travelled 12 km I in 1 hour or
12 km = 1 hr = 60 min
1 km = 5 min
on returning his speed is 8km/hr
it's mean he travelled 8 km in 1 hour
8 km = 1 hr = 60 min
1 km = 7.5 min
so to go to 2 km he takes (5 + 7.5) min= 12.5 min
since the distances are same and he takes 50 min
12.5 min = 2 km
50 min = 8 km
61. Ans. B.
Let us assume the capacity of the tub is 100L.
It is given that a tap can fill 100L in 10 hrs.
This means, in 1 hr. a tap can fill only 10L.
Therefore, in 7 hrs a tap can fill only 70L.
This means in 5 hrs a tap fills only 30L but actually the tap should fill 50L in 5 hrs.
This means that there is a leakage of 20L which has duration of 5 hrs.

PAGE 1012
www.byjusexamprep.com

If 20L of water is leaked in 5 hrs, then 1L water is leaked in

This means 100L water is leaked in = 25 hrs.

Hence option (b)


62. Ans. D.
Given 4 goat = 6 sheep i.e. efficiency of goat = 6 and efficiency of sheep = 4
Total field required to graze = 4 × 6 × 50 = 1200

Required time =

Hence option (d)


or 4 goat = 6 sheep = 50 days
2 goat = 3 sheep
2 goat + 3 sheep = 4 goat = 50 days
63. Ans. B.
If M1 can do the work in D1 days and M2 man can do the job in M2 days (where all man can
work at the same rate),

Then

According to the question,


M1 = 30, D1 = 40 W1 = 1, M2 = x, D2 = 40 W2 = 16/40

Thus =

Thus, the number of man who left the job = (30-12) = 18


Hence option (b)
64. Ans. D.
Given
=
=
=
= 0.9031 + 3+ 2 +0.7782
= 6.6813
Hence option (d)

PAGE 1013
www.byjusexamprep.com

65. Ans. C.
As,

= y(say)

Squaring both sides,

y2 = x + y => y2 – y – x = 0 => y =

if then its value is

According to question = = = 1.618

which lies between 1 and 2


Hence option (c)
66. Ans. B.
Let, (x - y) = a, (y - z) = b and (z - x) = c
Now, a + b + c = x – y + y – z + z – x = 0
So, a3 + b3 + c3 – 3abc = (a + b + c)(a2 + b2 + c2 + ab + bc + ca)
=> a3 + b3 + c3 – 3abc = 0 [As, a + b + c = 0]
Again,
a3 + b3 + c3 = 3abc

Hence,

Hence option (b)


67. Ans. D.
Let the total number of students be 100%
Numbers of student failed in Hindi = 35%
Numbers of student failed in English = 45%
Number of students failed in both subject = 20%
Total numbers of students failed = (35+45 - 20)% = 60%
Numbers of students passed in both the subject = (100- 60)% = 40 %
Hence option (d)

PAGE 1014
www.byjusexamprep.com

68. Ans. C.
As, ab + bc + ca = 0
⇒ ab + ca = -bc

⇒ and

Now,

=1
Hence option (c)
69. Ans. B.
When we solve the given equation for x,
Then x + 5 = 5 => x = 5 – 5 = 0
Thus, in the given set we have only one element viz. 0
Element φ is a null set i.e. no element in the set.
Hence option (b)
70. Ans. C.
Let CP of chair be x and CP of stools be y,
According to question,
4x+9y = 1340 ….. (i)
10%of 4x + 20% of 9y = 188 => 4x + 18 y = 1880 …… (ii)

PAGE 1015
www.byjusexamprep.com

Solving the equation (i) and (ii) by elimination method, we get


y = 60
Putting the value of y in equation (i), we get
4x = 800,
Thus, the money paid for the chair be Rs. 800
Hence option (c)
71. Ans. B.
Let present age of Ram be x and Shyam be y
From question,
(x - 5) = 3(y – 5 ) => x – 3y = -10 ……. (i)
(x+ 4 ) = 2(y + 4) => x – 2y = 4 …….. (ii)
After solving equation (i) and (ii) we get
x = 32
Hence present age of Ram be 32 years
Hence option (b)
72. Ans. D.

…… (1)
…….(2)

…….. (3)
……… (4)
…….. (5)

&
Hence option (d)

PAGE 1016
www.byjusexamprep.com

73. Ans. B.
Here, the maximum marks are 100% and according to the question,
45+5 = 40%
i.e. 50= 40%
Therefore, by unitary method, 1% =

100% =

Hence option (b)


74. Ans. D.
To maintain M1 cows for D1 days a milkman spends W1 and to maintain M2 cows for D2 days,
a milkman spends W2:

Then

⇒ n= = 72

Hence milkman need 72 days for maintenance.


Hence option (d)
75. Ans. C.
Using the concept of AP,
As, a = 507, b = 988, d = 13

Hence option (C)


76. Ans. C.

we are given that let us check the nature of the roots, we have

D=

= k2

PAGE 1017
www.byjusexamprep.com

Now,

x= = =

Since is an irrational number, therefore any value of k will give an irrational number

only.
Therefore, the given equation has no integral roots.
Hence option (c)
77. Ans. A.

As,



Hence option (a)


78. Ans. C.
Since the roots of the given equation are equal, therefore the discriminant of the given equation is
zero. Thus, b2-4ac = 0
→ =0
→ =0

→ 0

PAGE 1018
www.byjusexamprep.com

Hence option (c)


79. Ans. C.
Since we are given that a and c are co-prime
i.e. HCF of a and c is 1, therefore we can say that a definitely divides d exactly.
So, a is a factor of d.
Hence option (c)
80. Ans. B.
Since HCF of two polynomials is , therefore splitting this polynomial by middle term, we
get
x(x+2)-(x+2) = (x-1)(x+2)
Being the HCF of the given polynomial, we conclude that (x-1) and (x+2) is a factor of f(x)
and g(x)
By HCF we give the value of a and b

Now, =

And =

Since x is a factor of , therefore x = - 2 will satisfied the polynomial. Thus

=
=

Also, since (x - 1) is a factor of , therefore x = 1 will satisfied this polynomial. Thus,


, b = -3
Hence a = 2 and b = - 3
Hence option (b)
81. Ans. C.
As,
=
=
=
=

PAGE 1019
www.byjusexamprep.com

=
Hence factor of =
Hence option (c)
82. Ans. B.
As,

………(i)

=
=
=
……(ii)
From (i) and (ii)

=
=
=
Thus, product of and be
Hence option (b)
83. Ans. D.
As, square root of 2222 = 47.13, so, 2222 is not a perfect square.
Square root of 11664 = 108, so, 11664 is a perfect square.
Square root of 343343 = 585.95, so, 343343 is not a perfect square.
Square root of 220347 = 469.41, so, 220347 is not a perfect square.
Thus, A, C and D are not a perfect square.
Hence option (d)
84. Ans. A.
As, … ,

PAGE 1020
www.byjusexamprep.com

where … are real numbers.


When f(x) will be divided by (ax - b) then from Remainder theorem,

ax – b = 0 => x =

b
Thus, remainder = f  
a
Hence option (a)
85. Ans. A.
Given that (x+2) is a common factor of and

Let f(x) =
And g(x) =
Let p(x) = x+2 this means x+2 = 0
=>x = - 2, so -2 is a zero of f(x) and g(x)
Therefore and

Both polynomials are same


Thus,
4-2a+b – 4 -2b+a

=> b+2b = a+2a => 3a = 3b =>

=> a:b = 1:1


Hence option (a)
86. Ans. C.

Quotient = = 2x – y

Hence option (c)


87. Ans. B.

Given that A gives B a start of 8 m.


This means B starts from the point where A finishes its 8 m. therefore B covers 92 m.
Given that A runs at 6 km/hr.

PAGE 1021
www.byjusexamprep.com

i.e. A runs at

It is also given that even after giving B a start of 8 m, A reaches early than B by 9 seconds.
Therefore, if A takes 60 seconds to complete 100 m race, then B takes (60 + 9) seconds
i.e. 69 seconds to complete 92 m.

= = 4.8km/hr

Hence option(b)
88. Ans. B.
Here, we need to find the time that will take to cross 91 km stones completely.
Given that, in 1 hr. train travels 60 km i.e. 60 km is travelled in 60 min.
This means in 1 minute 1 km is travelled.
Therefore, 90 km is travelled in 90 minutes i.e. 1 hour 30 minutes
And the remaining 1 km in 12 seconds.
Thus, the total time taken is 1 hr. 30 min. 12 sec.
Hence option (b)
89. Ans. C.
Let the number of mangoes the fruit seller has originally be 100x
5% of total mangoes are rotten i.e. 5x mangoes are rotten, remaining mangoes = 95x
Seller sells 75% mangoes of remaining i.e.

Remaining mangoes = 95x – 71.25x = 95 => x = 4


Seller has initially 100x mangoes = 100 × 4 = 400 mangoes
Hence option (c)
90. Ans. A.
As, the discount taking two at a time 20% and 12.5 %,

Single equivalent discount = %=

No consider 30% and 5%

Final reduction =

Hence option (a)


91. Ans. B.
We have
P= and Q =

PAGE 1022
www.byjusexamprep.com

The distance between P and Q = =x


Expressing this distance in the form of rational numbers, we assume =x

Number of digit with bar = 1


Number of digit without bar = 1
Therefore, the denominator would be 90
Number of digit after the decimal = 2
Therefore, the nominator = 21 – 2 = 19

Thud x =

Hence option (b)


92. Ans. D.
We are given that HCF = p2 and product of two non-zero expressions = (x+y+z)p3
We know that HCF × LCM = product of two numbers
Therefore,
P2 × LCM = (x + y + z)p3

=> LCM =

Hence option (d)


93. Ans. D.
Let f(x) = and g(x) = x+3
To find the remainder g(x) should be equal to zero
Therefore g(x) = x+3 => x = -3
Putting this value in f(x) we get
f(x) =
f(x) = - 63
Hence option (d)
94. Ans. B.
Let f(x) =
And g(x) =

Then f(x)g(x) = ) (
=
Thus, it is clear that degree of f(x)g(x) = 7
Hence option (b)

PAGE 1023
www.byjusexamprep.com

95. Ans. A.
Let the two roots be 3x and 2x,
Let sum of root a + b = 3x + 2x =

⇒5x=

⇒ m = – 60x ------ (i)


Products of roots,

⇒ 6x2 =

⇒ x2 =

⇒x= =

Putting this value of x in (i), we get m = -60

Since we need positive value of m therefore

m = 60

Hence option (a)


96. Ans. B.
Given that
x = 111……1 (20 digits)
y = 333……3 (10 digits)

Therefore,

Since

Now, since 1001-999 = 2


Therefore

Hence option (b)

PAGE 1024
www.byjusexamprep.com

97. Ans. C.
The given 'polynomial is of the form
Let A, B and C be three zeroes of the given polynomial

Then, sum of the zeroes i.e. A + B + C = =-4

Product of the zeroes (taken two at a time) i e. = AB+BC+CA = = -11

Product of the zeroes (individual) i.e. ABC = = 30

Now, we will check each option for the correct


In option (a) we have 2, -3 and -5 as three zeroes. Sum of these zeroes is - 6 and product of these
zeroes is 30.
In option (b), we have -2, -3 and 5 as three zeroes. Sum of these zeroes is 0 and product of these
zeroes is 30
In option (c) we have -2, 3 and -5 as three zeroes. Sum of these zeroes is - 4 and product of these
zeroes is 30.
In option (d) we have -2, 3 and 5 as three zeroes. Sum of these zeroes is 6 and product of these zeroes
is 30.
Out of these options, only results of option (c) matches with the results calculated above.
Thus, our correct option is (c)
Hence option (c)
98. Ans. A.
LCM of 2, 3, 4 and 5 is 60.
Number or numbers divisible by 60 from 1 to 600 = 10
Number of numbers divisible by 60 from 601 to 900 i.e. from 300 numbers= 5
Number divisible by 60 from 901 to 1000 = 1
Total numbers = 10+5+1 = 16
Hence option (a)
99. Ans. A.
Let (x - a) is the factor of both quadratic equation.
i.e. x = a is the root of both equation.
Then x = a will satisfy both the equation.
So, ………(i)

………(ii)

PAGE 1025
www.byjusexamprep.com

subtracting eqn. 1 from eqn. 2

100. Ans. C.
Difference between the number of boys and girls in Mathematics = 150 – 90 = 60
Difference between the number of boys and girls in Physics = 180 – 120 = 60
Difference between the number of boys and girls in Statistics = 250 – 70 = 180
Difference between the number of boys and girls in Chemistry = 204 – 136 = 68
Hence Difference between the number of boys and girls in Mathematics and Physics are equal.
Hence option (c)

PAGE 1026
www.byjusexamprep.com

CDS I 2017
ENGLISH
Direction (1-10): The following sentence has a 6. The Governor will _______ the oath of
blank space and four words are given below it. office to the thirty-five new ministers at
Select the word you consider most appropriate 9.30 p.m. tomorrow.
for the blank and indicate your choice
accordingly. A. confer B. present

1. We should give everyone training in C. execute D. administer


citizenship but we have ____________ this 7. He is quite __________; you can never
aspect till now. count on him in matters of secrecy.
A. denied B. neglected
A. unjust B. unbearable
C. refused D. disallowed
C. inefficient D. unreliable
2. He _________ the role of the organization
in creating environmental awareness 8. He may not come, but we’ll get ready in
among the people. case he _________
A. collaborated B. commanded A. may B. will
C. contrasted D. commended C. shall D. does
3. Mrs. Ramsay did not know whether Miss
9. Can you pay ________ all these articles?
Jane knew ___________
A. swimming A. out B. of

B. to swim C. for D. off


C. how to swim 10. In this University, there is no __________
D. how to swimming for awarding scholarships on the basic of
merit in examination alone.
4. The consequences of economic growth
have now __________ to the lowest level. A. precedent B. opportunity
A. drawn B. slipped C. chance D. possibility
C. percolated D. crept Direction (11-17): Given below is a short
5. Most of the students have forgotten to passage. After the passage, you will find some
bring their certificates; they will be _______ items based on the passage. First, read a passage
by the Principal for this reason. and answer the items based on it. You are
required to select your answers based on the
A. rehearsed B. abused contents of the passage and opinion of the
C. accused D. reprimanded author only.

PAGE 1027
www.byjusexamprep.com

Passage 1 13. Why did Tolstoy disparage Eiffel Tower?


I must say a word about the Eiffel Tower. I do not 1. Man was foolish to build it.
know what purpose it serves today. But I then
2. Huge man-made structures did not
heard it greatly disparaged as well as praised. I
appeal to him.
remember that Tolstoy was the chief among
those who disparaged it. he said that the Eiffel 3. Men flocked to see it.
Tower was a monument of man’s folly, not of his
Which of the statements given above is/are
wisdom. Tobacco, he argued, was the worst of all
correct?
intoxicants, inasmuch as a man addicted to it
was tempted to commit crimes which a drunkard A. 1 only B. 1 and 2 only
never dared to do; liquor made a man mad, but
C. 1 and 3 only D. 2 and 3 only
tobacco clouded his intellect and made him build
castles in the air. The Eiffel Tower was one of the Passage 2
creations of a man under such influence. There
is no art about the Eiffel Tower. In no way can it Urbanization and industrialization have often
be said to have contributed to the real beauty of resulted in whole areas of forests being cleared
the Exhibition. Men flocked to see it and to gain new land and to obtain timber for the
ascended it as it was a novelty and of unique various building projects. Large areas of fields
dimensions. It was the toy of the Exhibition. So and forests have disappeared to make way for
long as we are children we are attracted by toys, concrete jungles many of which are fitted with
and the Tower was a good demonstration of the huge plants and chimney stacks. Industrial
fact that we are children attracted by trinkets. growth has necessitated the increased demand
That may be claimed to be the purpose served for fuel oil to run the machines and in doing so
by the Eiffel Tower. produces industrial gases and fumes which belch
through the chimney and pollute the
11. Why did men flock to the Eiffel Tower? atmosphere. The most evident elements in the
A. Men were attracted to the castles built contamination of the atmosphere are dust,
in the air. sulphur dioxide, carbon monoxide and nitrous
oxide.
B. Men lost their wisdom under the
influence of intoxicants. 14. The phrase ‘concrete jungle’ in the
paragraph refers to the factories and
C. Men were attracted to childish things. houses built as a result of urbanization and
industrialization. This phrase suggests that
D. Men were attracted to things of no
the author
value.
A. regrets that fields and forests have
12. Why did Tolstoy believe that tobacco was
been replaced by city buildings.
the worst of all intoxicants?
B. believes that too much cement has
A. Man lost his intellectual abilities under
been used in building factories.
the influence of tobacoo.
C. disapproves of modern industrial
B. Tobacco kept man in a state of
expansion.
inebriation.
D. would like to go back to life in the
C. People who commit crimes are
jungle.
invariably addicted to tobacoo.
15. The result of industrial development,
D. Statements (a) and (b) above are
according to the writer, are
correct in this context.

PAGE 1028
www.byjusexamprep.com

A. urbanization 19. From the passage, Yuri appears to be a man


B. no shortage of fuel oil. who is

C. greater availability of domestic gas. A. calm and courageous

D. greater fuel consumption and B. cunning and crafty


pollution. C. noisy and dangerous
16. The effect on forest areas produced by the D. active and jumpy
activity described in the first sentence is
called 20. When the writer saw Yuri holding a cobra
by the neck, he was ‘struck dumb’. This
A. devastation B. deforestation means that he was
C. disfiguration D. devaluation A. extremely delighted
17. The writer express the belief that B. very much helpless
A. there is plenty of scope for further C. rather surprised
industrialization.
D. absolutely shocked
B. unplanned growth of industry has done
more harm than good. Passage 4
C. the change from rural to urban growth I was lying down in a dark, lonely compartment
is a change for the better. of the speeding train, trying to sleep. But, quite
unusually, sleep eluded me. A vague uneasiness
D. the timber obtained from the forests
gripped me. It was pitch dark outside. A few
has been beneficially used.
points of light flashed by as we sped through a
Passage 3 small station and in the dim light I thought I saw
a hand gripping the bars of my window. Once
I was abruptly awakened by a noisy scuffle. The
again the train was swallowed up by the
sun, a mere fringe over the horizon, immediately
chased away the grey half-darkness. I was too impenetrable darkness. My heart pounded. My
mouth was parched. I could not get up. I do not
sleepy to notice what was happening. Yuri was
know how long I remained thus before the train
rolling over on the ground. I ran up to him but
began to slow down. The reassuring bright lights
was struck dumb. With his right hand he was
holding a cobra by the neck. Two sharp fangs of the station we were entering revealed no
showed from its jaws. The battle was over in a intruder. I breathed again.
few minutes. A hollow hissing and convulsive 21. Which of the following words best
jerks were then only reminders of a just-ended describes the condition of the traveller?
tussle. The catcher half-opened the lid of the box
A. Cautious B. Imaginative
and calmly put the quarry in.
C. Observant D. Nervous
18. With reference to the passage, the
following assumptions have been made : 22. In the dim light he saw
1. The incident took place early in the A. someone trying to climb into the train.
morning
B. someone clinging to the bars of the
2. Yuri threw the snake away window.
Which of these assumptions is/are correct? C. Someone was attempting to steal his
A. 1 only B. 2 only bag.

C. Both 1 and 2 D. Neither 1 nor 2 D. someone standing outside the window.

PAGE 1029
www.byjusexamprep.com

23. The narrator could not sleep because 27. Which of the following statements is
A. he usually found it difficult to fall closest to the truth?
asleep. A. The writer does not like TV as it gives a
B. he could not find a place to lie down. narrow view of things.
C. he was disturbed by some unspecified B. The writer likes TV but he does not like
thoughts. watching it from car windows.
D. the people near him were disturbing C. The writer does not like TV because the
him. picture is in a frame.
Passage 5 D. The writer does not like TV because the
Vacationing on a motorcycle, you see things in a programmes are boring.
way that is completely different from any other.
28. The writer likes travelling on the
In a car you are always in a compartment, and
motorcycle. What is the most likely reason
because you are used to it you do not realize that
for this?
through that car window everything you see is
just more TV. You are a passive observer and it is A. The motorcycle has no windows.
all moving by you boringly in a frame. On a
motorcycle, however, the frame is gone. You are B. The motorcycle does not go as fast as a
completely in contact with it all. You are in the car.
scene, not just watching it anymore, and the C. As the traveller is used to cars,
sense of presence is overwhelming. travelling by motorcycle is a change.
24. The word ‘overwhelming’ means D. Travelling by motorcycle, the writer
A. very strong B. unavoidable feels that he is part of the scenery.
C. interesting D. humorous Passage 6
25. In the last sentence, the writer talks of a
When Jonathan (the seagull) came, it was well
‘sense of presence’. He is referring to the
after dark, and he floated in moonlight on the
presence of
surface of the ocean. His wings were ragged bars
A. his own self as part of the scene. of lead, but the weight of failure was even
B. the time that is now passing. heavier on his back. He wished, feebly, that the
weight would be just enough to drag him gently
C. the scene and the beauty. down to the bottom and end it all. But soon he
D. senses with which one feels. came back to normal. He pushed wearily away
from the dark water and flew towards the land,
26. “In a car you are always in a compartment,
grateful for what he had learned about work-
and because you are used to it you do not
saving low-altitude flying.
realize that ………. .” In this sentence, ‘it’
refers to 29. The lesson that he had learnt that day was
A. travelling in a car. about

B. always being in a compartment, e.g. A. not fighting with stronger birds.


one’s room, office. B. flying carrying bars of lead on his wings.
C. seeing the scenery through the window
C. diving too deep into the sea.
frame.
D. seeing so much TV at home. D. flying at low altitudes.

PAGE 1030
www.byjusexamprep.com

30. ‘His wings were ragged bars of lead’ means 34. You must accustom yourself with new
that ideas.
A. his wings were damaged and supported A. accustomed with
by bars of lead. B. accustom to
B. his wings were damaged and therefore C. accustom yourself to
very heavy.
D. No improvement
C. he had rags and bars of lead on his
35. Poor Tom laid in the shade of a tree before
wings.
he could walk further.
D. his wings were broken like pieces of A. lied
lead.
B. lain
31. The seagull suffered because
C. lay
A. he had tried to do something that other
D. No improvement
seagulls had not done.
36. Mahatma Gandhi is called the father of the
B. probably he had been attacked by a Nation.
stronger bird.
A. called father of the Nation
C. probably he had been attacked by
some strong creature in the sea. B. called as the father of the Nation
C. father of the Nation
D. he had swooned and fallen into the
water. D. No improvement

32. The word ‘wearily’ means 37. The chairman with the other members of
the board are touring Europe these days.
A. tireless B. exhausted
A. have been on touring
C. sadly D. unconscious
B. is touring
Direction: Look at the underlined part of each C. have toured
sentence. Below each sentence are given three
possible substitutions for the underlined part. If D. No improvement
one of them (A), (B) or (C) is better than the 38. The angry neighbours never passed from
underlined part, indicate your response against each other without making rude remarks.
the corresponding letter (A), (B) or (C). If none of
the substitutions improves the sentence, A. passed on
indicate (D) as your response. Thus, a “No B. passed against
improvement” response will be signified by the
C. passed
letter (D).
D. No improvement
33. This telephone number is not existing.
39. I didn’t go to office because I was ill.
A. does not exist
A. felt
B. has not been existing
B. had become
C. has had no existence C. had felt
D. No improvement D. No improvement

PAGE 1031
www.byjusexamprep.com

40. Your services are dispensed for. 48. The man disappeared after he was rescuing
a boy from drowning.
A. from
B. with A. was rescued

C. off B. has been rescued

D. No improvement C. had rescued

41. I didn’t feel like going out yesterday, but on D. No improvement


account of my son’s illness, I had to go to
49. The Prime Minister had wide-ranging
the doctor.
discussions on the international situation.
A. have B. might have
A. widely-ranged B. wide-ranged
C. ought D. No improvement
C. wide-range D. No improvement
42. I was living in Chennai for ten years when I
was a child. 50. The teacher, along with her three children,
were taken to hospital.
A. had lived B. lived
A. was taken
C. had been living D. No improvement
B. had taken
43. He did not abide with my decision.
C. had being taken
A. to B. by
C. for D. No improvement D. No improvement

44. He is now looking about a job. 51. From the last five days it has been raining
torrentially.
A. for B. after
A. Since the last B. For the last
C. into D. No improvement
C. Since last D. No improvement
45. But for one witness the accused ought to
have been sent to jail. 52. He took a loan of hundred rupees from me.

A. would have B. had A. debt B. demand

C. should have D. No improvement C. advance D. No improvement

46. By 8.00 in the morning he wrote four letters 53. It is hard these days to cope with the rising
to his friends. prices.

A. had written A. cope by


B. had been writing B. cope up with
C. was writing C. to be coped with
D. No improvement D. No improvement
47. I shall be obliged if you could grant me an 54. Go north-east across the mountains till you
interview. will reach an island.
A. give B. allow A. reached B. reach
C. permit D. No improvement C. have reached D. No improvement

PAGE 1032
www.byjusexamprep.com

Direction: In the given question six sentences of Q : The cost of minting a 5 paise coin is said
a passage are a given. The first and sixth to be 7 paise while the cost of minting a 10
sentences are given in the beginning as S1 and paise coin is 10.5 paise.
S6. The middle four sentences in each have been
R : Moreover their removal would cause
jumbled up and labelled P, Q, R and S. You are
tremendous hardship to some people.
required to find the proper sequence of the four
sentences and mark your response accordingly. S : The RBI had opposed this saying that
55. S1 : It is common knowledge that people go they figure largely in public transactions.
after different object in the world to get The proper sequence should be
happiness.
A. S R Q P B. Q P S R
S6 : He is conscious of the fact that
happiness is within and not without. C. P S R Q D. P Q S R
P : The wise man with a properly attuned Direction: In the given question six sentences of
mind is happy with them, in spite of them a passage are a given. The first and sixth
and without them too. sentences are given in the beginning as S1 and
Q : Can a condemned prisoner, awaiting S6. The middle four sentences in each have been
execution on the marrow, relish food, jumbled up and labelled P, Q, R and S. You are
however delicious? required to find the proper sequence of the four
sentences and mark your response accordingly.
R : But a little reflection will prove that in
reality these sense-objects, by themselves, 57. S1 : The teaching work for the term is over.
can never make a person happy.
S6 : For this, weekend trips do not suffice,
S : It is folly to quote objects with and a longer stay is necessary.
happiness.
P : That will end on October 13 and the
The proper sequence should be
Diwali vacation will begin from October 14.
A. R Q S P B. Q P S R
Q : After I complete the assessment of
C. P S R Q D. P Q S R examination papers, I plan to go and stay at
Direction: In the given question six sentences of our house in Lonavla for at least a week.
a passage are a given. The first and sixth
R : Next week the terminal examination
sentences are given in the beginning as S1 and
begins.
S6. The middle four sentences in each have been
jumbled up and labelled P, Q, R and S. You are S : There is a lot of repair work that needs
required to find the proper sequence of the four to be carried out in the house.
sentences and mark your response accordingly.
The proper sequence should be
56. S1 : A proposal to remove from circulation
5 paise coins has been given up by the A. S Q R P B. Q S P R
Centre on advice from the Reserve Bank of C. R P S Q D. R P Q S
India.
Direction: In the given question six sentences of
S6 : It is, however, proposed to reduce the
a passage are a given. The first and sixth
costs of minting these coins by changing
sentences are given in the beginning as S1 and
their metallic content.
S6. The middle four sentences in each have been
P : The Government had been thinking of jumbled up and labelled P, Q, R and S. You are
removing from circulation even the 10 required to find the proper sequence of the four
paise coin. sentences and mark your response accordingly.

PAGE 1033
www.byjusexamprep.com

58. S1 : Nobody likes staying at home on a The proper sequence should be


public holiday – especially if the weather is
fine. A. P R S Q B. R S Q P

S6 : It was very peaceful in the cool grass – C. R P S Q D. P Q R S


until we heard bells ringing at the top of the Direction: In the given question six sentences of
hill. a passage are a given. The first and sixth
P : We had brought plenty of food with us sentences are given in the beginning as S1 and
and we got it out of the car. S6. The middle four sentences in each have been
jumbled up and labelled P, Q, R and S. You are
Q : The only difficulty was that millions of required to find the proper sequence of the four
other people had the same idea. sentences and mark your response accordingly.
R : Now everything was ready so we sat 60. S1 : I remember, some year ago, the library
down near a path at the foot of a hill. of a famous divine and literary critic, who
had died, being sold.
S : We moved out of the city slowly behind
a long line of cars, but at last we came to a S6 : Yet, he was a holy man and preached
quiet country road and after some time, admirable sermons.
stopped at a lonely farm.
P : Multitudes of the books had the marks
The proper sequence should be of libraries all over the country.
A. P S Q R B. Q S P R Q : It was a splendid library of rare books,
chiefly concerned with seventeenth
C. P Q R S D. S P Q R century writers.
Direction: In the given question six sentences of R : Evidently, he was very possessive about
a passage are a given. The first and sixth the books he borrowed.
sentences are given in the beginning as S1 and
S6. The middle four sentences in each have been S : He had borrowed them and never found
jumbled up and labelled P, Q, R and S. You are a convenient opportunity of returning
required to find the proper sequence of the four them.
sentences and mark your response accordingly.
The proper sequence should be
59. S1 : Of course, it is silly to try to overcome
fears that keep us from destroying A. R P Q S B. Q P S R
ourselves. C. S P Q R D. P S R Q
S6 : The only fears you need to avoid are Direction: In the given question six sentences of
silly fears which prevent you from doing a passage are a given. The first and sixth
what you should do. sentences are given in the beginning as S1 and
P : This is sensible. S6. The middle four sentences in each have been
jumbled up and labelled P, Q, R and S. You are
Q : You wait until it is out of the way before required to find the proper sequence of the four
crossing. sentences and mark your response accordingly.
R : You need some fears to keep you from 61. S1 : The houses in the Indus Valley were
doing foolish things. built of baked bricks.
S : You are afraid of an automobile coming S6 : They led outside into covered sewers
rapidly down the street you wish to cross. which ran down the side of the streets.

PAGE 1034
www.byjusexamprep.com

P : This staircase sometimes continued Direction: In the given question six sentences of
upwards on to the roof. a passage are a given. The first and sixth
sentences are given in the beginning as S1 and
Q : Access to the upstairs rooms was by a
S6. The middle four sentences in each have been
narrow stone staircase at the back of the
jumbled up and labelled P, Q, R and S. You are
house.
required to find the proper sequence of the four
R : The drains were built in the walls. sentences and mark your response accordingly.
S : The houses had bathrooms and water 63. S1 : There isn’t a cricketer worth his salt
closets, rubbish chutes and excellent who does not aspire for captaincy.
drainage systems.
S6 : It is against this background that any
The proper sequence should be: emotions surrounding the captaincy should
A. S P Q R B. P S Q R be viewed.

C. Q R P S D. Q P S R P : Even parents are proud when their sons


become the captain.
Direction: In the given question six sentences of
a passage are a given. The first and sixth Q : At the lower levels, it is the best player
sentences are given in the beginning as S1 and who gets to lead the team.
S6. The middle four sentences in each have been R : In cricket, the greatest honour any
jumbled up and labelled P, Q, R and S. You are player can get is the captaincy.
required to find the proper sequence of the four
sentences and mark your response accordingly. S : Right from their school days, boys dream
of leading the team.
62. S1 : Systematic and caseless efforts are
being made to tap scientifically the The proper sequence should be
abundant solar energy available in the
country. A. P Q R S B. S R Q P

S6 : Installation of solar thermal systems C. P Q S R D. S Q P R


and devices has helped to save or generate Direction: In the given question six sentences of
energy to the extent of 350 kWh per a passage are a given. The first and sixth
annum. sentences are given in the beginning as S1 and
P : These include cooking, water heating, S6. The middle four sentences in each have been
water desalination, space heating, etc. jumbled up and labelled P, Q, R and S. You are
required to find the proper sequence of the four
Q : A large number of applications in the sentences and mark your response accordingly.
area have already become commercial.
64. S1 : Some students may feel that fast
R : Efforts are also afoot to develop readers do not understand as much as slow
economically viable solar collectors for high readers.
temperature applications.
S6 : This statement, however, needs to be
S : A simple and common mode in solar
made with caution.
energy utilization is solar thermal
conversion. P : Some slow readers will have good
comprehension and others poor.
The proper sequence should be
Q : A quick glance at the scores will show
A. Q S R P B. Q P R S
that fast readers sometimes have very good
C. S Q P R D. R Q S P comprehension and sometimes poor.

PAGE 1035
www.byjusexamprep.com

R : This fallacy can easily be disproved when P: I immediately sent a message requesting
you give the first reading test in a class. a few days of grace as I had to book the
return ticket and attend sundry matters
S : In short, there is little relation between
before winding up my establishment here.
reading speed and comprehension.
The proper sequence should be Q: On the way, I went to the laundry and
made sure I would get my clothes in time.
A. R Q P S B. S Q R P
R: Then I rushed to the bank, collected all
C. P Q R S D. Q S R P my money and made reservations for my
Direction: In the given question six sentences of return journey.
a passage are a given. The first and sixth S: From the shop next to it, I bought a
sentences are given in the beginning as S1 and strong box to dump my books and other
S6. The middle four sentences in each have been odd articles so that I could send them away
jumbled up and labelled P, Q, R and S. You are in advance.
required to find the proper sequence of the four
sentences and mark your response accordingly. A. Q R S P B. R Q P S

65. S1 : Harsten’s theory was that plants C. S P Q R D. P R Q S


definitely react to music. Direction: In the given question six sentences of
S6 : He found that this plant grew faster and a passage are a given. The first and sixth
70 percent taller than the other plant. sentences are given in the beginning as S1 and
S6. The middle four sentences in each have been
P : In his experiment, he used two banana
jumbled up and labelled P, Q, R and S. You are
plants.
required to find the proper sequence of the four
Q : The music was in fact a high-pitched sentences and mark your response accordingly.
humming sound.
67. S1: A common disease of the eye is
R : He gave both plants the same light, heat conjunctivitis or ‘pink eye’, which often
and water. occurs in school-children.
S : But for about an hour a day, one of the S6: Students with this condition should be
plants ‘listened’ to some music. sent to the doctor.
The proper sequence should be P: This disease spreads rapidly among
A. Q S R P B. P Q R S school children.

C. P R S Q D. Q R P S Q: A white discharge also appears in the


corners of the eyes.
Direction: In the given question six sentences of
a passage are a given. The first and sixth R: One or both eyeballs turn quite red and
sentences are given in the beginning as S1 and have a feeling of irritation
S6. The middle four sentences in each have been S: This discharge forms a crust which can
jumbled up and labelled P, Q, R and S. You are often be seen in the morning after the eyes
required to find the proper sequence of the four have been shut all night.
sentences and mark your response accordingly.
A. P S Q R B. R S Q P
66. S1: My office sent a message with a terrific
urgency asking me to return. C. R Q S P D. Q S R P

S6: It was the evening before I could sit and Direction: In the given question six sentences of
write to my parents that I would join them a passage are a given. The first and sixth
soon. sentences are given in the beginning as S1 and

PAGE 1036
www.byjusexamprep.com

S6. The middle four sentences in each have been A. R Q P S B. P R Q S


jumbled up and labelled P, Q, R and S. You are
C. P S Q R D. R Q S P
required to find the proper sequence of the four
sentences and mark your response accordingly. Direction: In the given question six sentences of
a passage are a given. The first and sixth
68. S1 : The first great discovery that man
sentences are given in the beginning as S1 and
probably made was that of fire.
S6. The middle four sentences in each have been
S6 : The animals were not clever enough to jumbled up and labelled P, Q, R and S. You are
learn anything from this. required to find the proper sequence of the four
sentences and mark your response accordingly.
P : Fires sometimes occur by themselves in
the forests perhaps by rubbing together of 70. S1 : The sun-dried stalks of rice had caught
flints or something else. fire and burst into flames.
Q : We light a fire now by a match. S6 : Even those who were too feeble to
keep up with the first rush were on their
R : In olden times fires were made by feet, eager to join the fire-fighters.
striking two flints against each other till a
spark came for setting to a dry thing. P : Then came most of the older people, and
mothers with babies at their backs.
S : But matches are quite recent things.
Q : And all the active women and girls
The proper sequence should be followed them to assist them in fighting the
A. P R S Q B. Q S P R fire.

C. P Q S R D. R P Q S R : All the young men and boys were soon


on the spot.
Direction: In the given question six sentences of
S : Staring wildly at the blazing rice, the
a passage are a given. The first and sixth
people of the village ran to extinguish the
sentences are given in the beginning as S1 and
fire.
S6. The middle four sentences in each have been
jumbled up and labelled P, Q, R and S. You are The proper sequence should be
required to find the proper sequence of the four
A. P S Q R B. Q P R S
sentences and mark your response accordingly.
C. S R P Q D. S R Q P
69. S1: People’s fear of snakes seems to be
based on a series of misconceptions. Direction: In the given question six sentences of
a passage are a given. The first and sixth
S6: Yet people will insist that they cannot
sentences are given in the beginning as S1 and
touch a snake because of its sliminess.
S6. The middle four sentences in each have been
P: The most common one is the conviction jumbled up and labelled P, Q, R and S. You are
that all these creatures are poisonous. required to find the proper sequence of the four
sentences and mark your response accordingly.
Q: Another very popular idea is that these
reptiles are slimy to touch. 71. S1 : One fine morning, I heard a sound of
fire engines with loud alarm bells.
R: Snakes are really dry and cold; not slimy
like a wet cake of soap but rather like a S6 : It was a terrible scene, as huge flames
crocodile-skin handbag to the touch. of fire were coming out of the house.
S: In fact, the non-poisonous ones P : I thought, perhaps a house was on fire.
outnumber the poisonous ones by about Q : They were regulating the crowd.
ten to one.

PAGE 1037
www.byjusexamprep.com

R : The policemen were standing around a Q : The examination was to begin in five
big house. minutes time.
S : I immediately rushed out to see what R : He just rushed across the road.
was happening. S : So he forgot to look to the left or right as
The proper sequence should be he always did.

A. P Q R S B. R S Q P The proper sequence should be

C. S R P Q D. P S R Q A. R Q S P B. S R P Q

Direction: In the given question six sentences of C. R S P Q D. Q S R P


a passage are a given. The first and sixth Direction: In the given question six sentences of
sentences are given in the beginning as S1 and a passage are a given. The first and sixth
S6. The middle four sentences in each have been sentences are given in the beginning as S1 and
jumbled up and labelled P, Q, R and S. You are S6. The middle four sentences in each have been
required to find the proper sequence of the four jumbled up and labelled P, Q, R and S. You are
sentences and mark your response accordingly. required to find the proper sequence of the four
sentences and mark your response accordingly.
72. S1 : Chandran was terribly angry.
74. S1 : According to the legend it was during a
S6 : Saro ran screaming.
famine in China many thousands of years
P : His pen was in her hand and Chandran ago that rice first came into the lives of the
rushed at her. Asians.
Q : Just then his sister, Saro, appeared at S6 : And the little girl gathered a harvest of
the door-step. rice.

R : His pen was missing from the place P : The golden sunbeams through the
where he had left it. meshes and transformed themselves into
golden grains.
S : He thought it was lost.
Q : A little girl went fishing.
The proper sequence should be
R : But instead of catching fish she netted
A. P R S Q B. R S Q P the King of Frogs.
C. P S Q R D. S Q P R S. The King told her to hold out the net to
the sunbeams while he sang a magic song.
Direction: In the given question six sentences of
a passage are a given. The first and sixth The proper sequence should be
sentences are given in the beginning as S1 and
A. P S Q R
S6. The middle four sentences in each have been
jumbled up and labelled P, Q, R and S. You are B. R P S Q
required to find the proper sequence of the four C. Q R S P
sentences and mark your response accordingly.
D. P Q S R
73. S1 : Ramu was in a great hurry.
Direction: In the following question, some part
S6 : Fortunately, it wasn’t a very serious of the sentence may have errors. Find out which
one. part of the sentence has an error and select the
P : As a result to this carelessness, he met appropriate option. If the sentence is free from
with an accident. error, select 'No error'.

PAGE 1038
www.byjusexamprep.com

75. Every student (1)/ should be asked (2)/ to Direction: In this question, a sentence with three
give their ideas on the subject. (3)/ No error parts labelled a, b and c is given. Read the
(4) sentence to find out whether there is any error
in any underlined part and indicate your
A. 1 B. 2
response on the Answer Sheet against the
C. 3 D. 4 corresponding letter i.e., A or B or C. If you find
no error, your response should be indicated as D.
Direction: In this question, a sentence with three
parts labelled a, b and c is given. Read the 79. All of them (a) speak (b) good English (c). No
sentence to find out whether there is any error error. (d)
in any underlined part and indicate your A. (a) B. (b)
response on the Answer Sheet against the
corresponding letter i.e., A or B or C. If you C. (c) D. (d)
find no error, your response should be indicated Direction: In this question, a sentence with three
as D. parts labelled a, b and c is given. Read the
76. Emperor Ashoka (a) have conquered sentence to find out whether there is any error
Kalinga (b) before he embraced Buddhism in any underlined part and indicate your
(c). No error. (d) response on the Answer Sheet against the
corresponding letter i.e., A or B or C. If you
A. (a) B. (b) find no error, your response should be indicated
as D.
C. (c) D. (d)
80. Of the many problems that confront the
Direction: In this question, a sentence with three
leaders of the world (a) none are of grave
parts labelled a, b and c is given. Read the
consequence (b) than the problem of
sentence to find out whether there is any error
saving the normal human race from
in any underlined part and indicate your
extinction (c). No error.(d)
response on the Answer Sheet against the
corresponding letter i.e., A or B or C. If you find A. (a) B. (b)
no error, your response should be indicated as D.
C. (c) D. (d)
77. Her parents (a) has not permitted her (b) to Direction: In this question, a sentence with three
marry Sunil (c). No error. (d) parts labelled a, b and c is given. Read the
A. (a) B. (b) sentence to find out whether there is any error
in any underlined part and indicate your
C. (c) D. (d) response on the Answer Sheet against the
Direction: In this question, a sentence with three corresponding letter i.e., A or B or C. If you find
parts labelled a, b and c is given. Read the no error, your response should be indicated as D.
sentence to find out whether there is any error 81. Whom you think (a) will be dismissed (b)
in any underlined part and indicate your first? (c) No error. (d)
response on the Answer Sheet against the
corresponding letter i.e., A or B or C. If you find A. (a) B. (b)
no error, your response should be indicated as D. C. (c) D. (d)
78. The peon (a) has not sweeped (b) the floor Direction: In this question, a sentence with three
today (c). No error. (d) parts labelled a, b and c is given. Read the
A. (a) B. (b) sentence to find out whether there is any error
in any underlined part and indicate your
C. (c) D. (d) response on the Answer Sheet against the

PAGE 1039
www.byjusexamprep.com

corresponding letter i.e., A or B or C. If you find Direction: In this question, a sentence with three
no error, your response should be indicated as D. parts labelled a, b and c is given. Read the
sentence to find out whether there is any error
82. He couldn’t (a) find (b) an answer (c). No
in any underlined part and indicate your
error. (d)
response on the Answer Sheet against the
A. (a) B. (b) corresponding letter i.e., A or B or C. If you find
no error, your response should be indicated as D.
C. (c) D. (d)
86. Tell him to take (a) another photograph (b)
Direction: In this question, a sentence with three of the group (c). No error (d)
parts labelled a, b and c is given. Read the
sentence to find out whether there is any error A. (a) B. (b)
in any underlined part and indicate your
response on the Answer Sheet against the C. (c) D. (d)
corresponding letter i.e., A or B or C. If you find Direction: In this question, a sentence with three
no error, your response should be indicated as D. parts labelled a, b and c is given. Read the
83. Nandita asked me (a) if I was working (b) sentence to find out whether there is any error
hardly these days (c). No error. (d) in any underlined part and indicate your
response on the Answer Sheet against the
A. (a) B. (b) corresponding letter i.e., A or B or C. If you find
no error, your response should be indicated as D.
C. (c) D. (d)
Direction: In this question, a sentence with three 87. Their belongings (a) were lost (b) in the fire
parts labelled a, b and c is given. Read the (c). No error.(d)
sentence to find out whether there is any error A. (a) B. (b)
in any underlined part and indicate your
response on the Answer Sheet against the C. (c) D. (d)
corresponding letter i.e., A or B or C. If you find
Direction: In this question, a sentence with three
no error, your response should be indicated as D.
parts labelled a, b and c is given. Read the
84. The memoranda (a) are (b) on the table (c). sentence to find out whether there is any error
No error. (d) in any underlined part and indicate your
response on the Answer Sheet against the
A. (a) B. (b)
corresponding letter i.e., A or B or C. If you find
C. (c) D. (d) no error, your response should be indicated as D.
Direction: In this question, a sentence with three 88. I prefer (a) this book (b) than that one (c).
parts labelled a, b and c is given. Read the No error.(d)
sentence to find out whether there is any error
in any underlined part and indicate your A. (a) B. (b)
response on the Answer Sheet against the
C. (c) D. (d)
corresponding letter i.e., A or B or C. If you find
no error, your response should be indicated as D. Direction: In this question, a sentence with three
85. I courteously asked him (a) where was he parts labelled a, b and c is given. Read the
going (b) but he did not reply (c). No error. sentence to find out whether there is any error
(d) in any underlined part and indicate your
response on the Answer Sheet against the
A. (a) B. (b) corresponding letter i.e., A or B or C. If you find
C. (c) D. (d) no error, your response should be indicated as D.

PAGE 1040
www.byjusexamprep.com

89. It (a) was raining (b) cats and dogs (c). No corresponding letter i.e., A or B or C. If you find
error. (d) no error, your response should be indicated as D.
A. (a) B. (b) 93. Tell me the name of (a) a country where
C. (c) D. (d) every citizen is law-abiding (b) and no
trouble is there (c). No error.(d)
Direction: In this question, a sentence with three
parts labelled a, b and c is given. Read the A. (a) B. (b)
sentence to find out whether there is any error C. (c) D. (d)
in any underlined part and indicate your Direction: In this question, a sentence with three
response on the Answer Sheet against the parts labelled a, b and c is given. Read the
corresponding letter i.e., A or B or C. If you find sentence to find out whether there is any error
no error, your response should be indicated as D. in any underlined part and indicate your
90. One of the most interesting feature of response on the Answer Sheet against the
travel in Himachal Pradesh is (a) the large corresponding letter i.e., A or B or C. If you find
number of travellers’ lodges (b) provided by no error, your response should be indicated as D.
the State Government (c). No error. (d) 94. All the boys (a) returned back home (b) well
A. (a) B. (b) in time for lunch (c). No error(d).
C. (c) D. (d) A. (a) B. (b)
Direction: In this question, a sentence with three C. (c) D. (d)
parts labelled a, b and c is given. Read the
Direction: In this question, a sentence with three
sentence to find out whether there is any error
parts labelled a, b and c is given. Read the
in any underlined part and indicate your
sentence to find out whether there is any error
response on the Answer Sheet against the
in any underlined part and indicate your
corresponding letter i.e., A or B or C. If you find
response on the Answer Sheet against the
no error, your response should be indicated as D.
corresponding letter i.e., A or B or C. If you find
91. His friends feel that (a) he will be no error, your response should be indicated as D.
suspended(b) unless he does not report for
95. If a thing (a) is worth doing at all (b) it is
duty Immediately(c). No error.(d)
worth done well (c). No error (d).
A. (a) B. (b)
A. (a) B. (b)
C. (c) D. (d)
C. (c) D. (d)
Direction: In the sentence identify the segment
96. Although we reached his house on time, he
which contains the grammatical error.
was left for the airport.
92. Considering about these facts the principal
A. his house on time
has offered him a seat
B. he was left for the airport.
A. Considering about these facts
C. Although we reached
B. him a seat
D. No error
C. No error
97. It is not difficult to believe that a man (a)
D. principal has offered
who has lived in this for a long time (b) he
Direction: In this question, a sentence with three will never feel at home anywhere else in the
parts labelled a, b and c is given. Read the world (c). No error.(d)
sentence to find out whether there is any error
A. (a) B. (b)
in any underlined part and indicate your
response on the Answer Sheet against the C. (c) D. (d)

PAGE 1041
www.byjusexamprep.com

98. Being a rainy day (a), we did not feel like Direction: The following question consists of a
going out or doing anything (b) except sentence, the parts of which have been jumbled.
playing chess in our room (c). No error.(d) These parts have been labelled P, Q, R and S.
Given below each sentence are four sequences
A. (a) B. (b)
namely A, B, C and D. You are required to re-
C. (c) D. (d) arrange the jumbled parts of the sentence and
mark your response accordingly.
Direction: In this question, a sentence with three
parts labelled a, b and c is given. Read the 102. People do not realize (P) that modern
sentence to find out whether there is any error society (Q) who object to driver education
in any underlined part and indicate your (R) is built around the automobile (S)
response on the Answer Sheet against the A. R P Q S B. Q R S P
corresponding letter i.e., A or B or C. If you find
no error, your response should be indicated as D. C. R S P Q D. S P Q R

99. I do not understand why (a) in spite of my Direction: The following question consists of a
best efforts to please him (b) my boss is so sentence, the parts of which have been jumbled.
angry at me (c). No error(d) These parts have been labelled P, Q, R and S.
Given below each sentence are four sequences
A. (a) B. (b) namely A, B, C and D. You are required to re-
C. (c) D. (d) arrange the jumbled parts of the sentence and
mark your response accordingly.
Direction: In this question, a sentence with three
103. the school masters and the professors tend
parts labelled a, b and c is given. Read the
to believe (P) that innate intelligence is a
sentence to find out whether there is any error
quality (Q) but that you can’t do much
in any underlined part and indicate your
about it except measure it (R) which varies
response on the Answer Sheet against the
very greatly from one individual to another
corresponding letter i.e., A or B or C. If you find
(S)
no error, your response should be indicated as D.
A. P S Q R B. P R S Q
100. I waited (a) for her return (b) with growing
unease (c) No error. (d) C. P Q S R D. Q S R P

A. (a) B. (b) Direction: The following question consists of a


sentence, the parts of which have been jumbled.
C. (c) D. (d) These parts have been labelled P, Q, R and S.
Direction: The following question consists of a Given below each sentence are four sequences
sentence, the parts of which have been jumbled. namely A, B, C and D. You are required to re-
These parts have been labelled P, Q, R and S. arrange the jumbled parts of the sentence and
Given below each sentence are four sequences mark your response accordingly.
namely A, B, C and D. You are required to re- 104. the workaholic often (P) becomes a
arrange the jumbled parts of the sentence and prisoner of success (Q) inadvertently (R)
mark your response accordingly. reaping greater and greater rewards (S)
101. with an idea to reach the deprived child (P) A. S P R Q B. S P Q R
by the United Nations Organization (Q) the
C. R P Q S D. P Q R S
year 1979 has been declared as (R) the
International Year of the Child (S) Direction: The following question consists of a
sentence, the parts of which have been jumbled.
A. P Q R S B. P S Q R These parts have been labelled P, Q, R and S.
C. P R S Q D. R S P Q Given below each sentence are four sequences

PAGE 1042
www.byjusexamprep.com

namely A, B, C and D. You are required to re- A. R P S Q B. Q R P S


arrange the jumbled parts of the sentence and
C. R Q S P D. P S Q R
mark your response accordingly.
105. Friendship has its place in (P) truth and Direction: The following question consists of a
justice (Q) but it cannot override (R) life, (S) sentence, the parts of which have been jumbled.
These parts have been labelled P, Q, R and S.
A. P Q R S B. R S P Q Given below each sentence are four sequences
C. P S R Q D. R Q P S namely A, B, C and D. You are required to re-
arrange the jumbled parts of the sentence and
Direction: The following question consists of a
mark your response accordingly.
sentence, the parts of which have been jumbled.
These parts have been labelled P, Q, R and S. 109. Many people inaudible to others (P) in the
Given below each sentence are four sequences ears or head (Q) suffer persistent noises (R)
namely A, B, C and D. You are required to re- with hearing problems (S)
arrange the jumbled parts of the sentence and
A. R Q S P B. P S Q R
mark your response accordingly.
106. sports cars (P) appeal (Q) to some motorists C. S R Q P D. R S Q P
only (R) with noisy exhausts (S) Direction: The following question consists of a
A. R Q S P B. P S Q R sentence, the parts of which have been jumbled.
These parts have been labelled P, Q, R and S.
C. R S P Q D. P Q S R
Given below each sentence are four sequences
Direction: The following question consists of a namely A, B, C and D. You are required to re-
sentence, the parts of which have been jumbled. arrange the jumbled parts of the sentence and
These parts have been labelled P, Q, R and S. mark your response accordingly.
Given below each sentence are four sequences
namely A, B, C and D. You are required to re- 110. Animals are able to measure (P) the passing
arrange the jumbled parts of the sentence and of the seasons (Q) changes in day-length (R)
mark your response accordingly. by recognizing (S)

107. Attempts at transplanting other organs, but A. P Q S R B. S Q P R


that which most caught the public fancy (P) C. P R S Q D. S R P Q
was the heart transplant (Q) such as the
lungs or the liver, (R) have been made (S) Direction: The following question consists of a
sentence, the parts of which have been jumbled.
A. S R Q P B. R S P Q
These parts have been labelled P, Q, R and S.
C. R P S Q D. Q R P S Given below each sentence are four sequences
Direction: The following question consists of a namely A, B, C and D. You are required to re-
sentence, the parts of which have been jumbled. arrange the jumbled parts of the sentence and
These parts have been labelled P, Q, R and S. mark your response accordingly.
Given below each sentence are four sequences 111. The lady, today (P) is visiting (Q) us (R) who
namely A, B, C and D. You are required to re- composed this poem (S)
arrange the jumbled parts of the sentence and
mark your response accordingly. A. R Q P S B. P R Q S
108. Researchers found that allergic reactions C. Q R P S D. S Q R P
are brought about (P) in the bloodstream
Direction: The following question consists of a
(Q) in the 1940s (R) by the liberation of
sentence, the parts of which have been jumbled.
small quantities of a substance called
These parts have been labelled P, Q, R and S.
histamine (S)

PAGE 1043
www.byjusexamprep.com

Given below each sentence are four sequences A. P Q R S B. P S R Q


namely A, B, C and D. You are required to re-
C. R S P Q D. Q R P S
arrange the jumbled parts of the sentence and
mark your response accordingly. Direction: The following question consists of a
sentence, the parts of which have been jumbled.
112. The man for a moment, (P) looked at me (Q) These parts have been labelled P, Q, R and S.
and thus emptied my pockets (R) turned me Given below each sentence are four sequences
upside down (S) namely A, B, C and D. You are required to re-
A. P Q R S B. Q P R S arrange the jumbled parts of the sentence and
mark your response accordingly.
C. Q P S R D. P Q S R
116. The traveller, sat down (P) to rest (Q) by the
Direction: The following question consists of a
roadside (R) being weary, (S)
sentence, the parts of which have been jumbled.
These parts have been labelled P, Q, R and S. A. P Q R S B. R S P Q
Given below each sentence are four sequences C. S P Q R D. R P Q S
namely A, B, C and D. You are required to re-
arrange the jumbled parts of the sentence and Direction: The following question consists of a
mark your response accordingly. |||End||| sentence, the parts of which have been jumbled.
These parts have been labelled P, Q, R and S.
113. rules and regulations (P) he cheerfully Given below each sentence are four sequences
ignored its demands (Q) a life of (R) namely A, B, C and D. You are required to re-
unaccustomed to (S) arrange the jumbled parts of the sentence and
A. P Q R S B. Q R P S mark your response accordingly.
C. S R P Q D. Q S R P 117. a person bitten by a rabid dog (P) would be
seized by violent symptoms (Q) after an
Direction: Each of the following items in this
incubation period of a month or two (R) and
section consists of a sentence the parts of which
die an agonizing death (S)
have been jumbled. These parts have been
labeled P, Q, R and S and are given below each
sentence in four sequences namely (a), (b), (c) A. P R S Q B. S R Q P
and (d). You are required to rearrange the
jumbled parts of the sentence and mark your C. P R Q S D. P Q R S
response accordingly. Direction: Each of the following items in this
114. he was a tiny man (P) with a sprightly walk section consists of a sentence the parts of which
(Q) tall (R) barely five set (S) have been jumbled. These parts have been
labeled P, Q, R and S and are given below each
A. R S Q P B. S R Q P sentence in four sequences namely (a), (b), (c)
C. Q S P R D. P S R Q and (d). You are required to rearrange the
jumbled parts of the sentence and mark your
Direction: The following question consists of a
response accordingly.
sentence, the parts of which have been jumbled.
These parts have been labelled P, Q, R and S. 118. up to the end of the eighteenth century, not
Given below each sentence are four sequences only because it was often fatal, (P) smallpox
namely A, B, C and D. You are required to re- was a particularly dreaded disease, (Q) but
arrange the jumbled parts of the sentence and also because those who recovered (R) were
mark your response accordingly. permanently disfigured (S)
115. the house was (P) away (Q) with its liveliest A. P Q R S B. Q R S P
member (R) gloomy (S) C. P R S Q D. Q P R S

PAGE 1044
www.byjusexamprep.com

Direction: The following question consists of a Direction: The following question consists of a
sentence, the parts of which have been jumbled. sentence, the parts of which have been jumbled.
These parts have been labelled P, Q, R and S. These parts have been labelled P, Q, R and S.
Given below each sentence are four sequences Given below each sentence are four sequences
namely A, B, C and D. You are required to re- namely A, B, C and D. You are required to re-
arrange the jumbled parts of the sentence and arrange the jumbled parts of the sentence and
mark your response accordingly. mark your response accordingly.
119. it is a fact that rice is one of the most prolific 120. he (P) almost (Q) planned the entire
(P) than almost any other crop (Q) yielding strategy of operation (R) single handed (S)
a greater return per acre (R) of food crops A. R S P Q
(S)
B. P R Q S
A. S R Q P B. R P S Q C. S Q R P
C. Q R S P D. P S R Q D. Q P S R

PAGE 1045
www.byjusexamprep.com

General Knowledge
1. Which one of the following constitutional D. The Directive Principle are directed in
authorities inquires and decides in case of making India an advanced capitalist
doubts and disputes arising out of election country of the world.
of the President and Vice President of
4. Which one of the following statements
India?
relating to cultural and educational rights in
A. The Supreme Court of India India is not correct?
B. The Election Commission of India A. Every section of the citizens has the
C. The Parliamentary Committee right to conserve its language, script or
culture.
D. The High Court of Delhi
B. No citizen shall be denied admission
2. Which one of the following statements is into any educational institution
not correct regarding the Office of the Vice maintained by the State or receiving aid
President of India? out of State funds on grounds of
A. The vice President is elected by an religion, race or language
electoral college consisting of the C. The state shall, in granting aid to
elected members of both the Houses of educational institutions, discriminate
the Parliament. against any educational institution on
B. The vice President is elected in the ground that it is under the
accordance with the system of management of a majority community.
proportional representation by means
D. all minorities whether based on religion
of single transferable vote.
or language, shall have the right to
C. The Vice President shall not be a establish and administer educational
member of either House of the institutions of their choice.
Parliament or of a House of the
5. Which of the following statements is/are
Legislature of any State.
correct regarding the Preamble of the
D. The Vice President of India shall be ex Indian Constitution?
officio Chairman of the Council of
States and shall not hold any office of 1. The Preamble by itself is not
profit. enforceable in a Court of Law.

3. Which one of the following statements 2. The preamble states the objectives
relating to the Directive Principles of State which the Constitution seeks to
Policy is not correct? establish and promote.

A. The provisions contained in part IV of 3. The Preamble indicates the source


the Constitution of India shall not be from which the Constitution derives its
enforceable by any Court. authority.

B. The Directive Principle of State policy Select the correct answer using the code
are fundamental in the governance of given below.
the country. A. 1, 2 and 3 B. 1 and 2 only
C. It shall be the duty of the State to apply
C. 1 and 3 only D. 2 only
the Directive Principle in making laws.

PAGE 1046
www.byjusexamprep.com

6. The Ryotwari experiment in land revenue Examine these two statements carefully and
was started by select the correct answer using the code given
below.
A. Henry Dundas
9. Statement I: The Poona Pact provided for
B. Alexander Reed
151 reserved seats for the scheduled castes
C. David Ricardo to be elected by a joint, not separate,
D. Mountstuart Elphinstone electorate.

7. Which one of the following statements Statement II: Dr. B.R. Ambedkar withdrew
about the justice Party is not correct? from active politics for almost a decade
when the Communal Award was revoked.
A. It clamoured for the same kind of
separate communal representation for A. Both the statements are individually
the Non-Brahmins as had been granted true and Statement II is the correct
to the Muslims by the Morley-Minto explanation of Statement I
Reforms. B. Both the statements are individually
B. It was patronized mainly by richer true but Statement II is not the correct
landowning and urban middle class explanation of Statement I
Non- Brahmins. C. Statement I is true but Statement II is
C. It succeeded in getting the provision for false
28 reserved seats for the Non-Brahmins D. Statement I is false but Statement II is
in the Montagu-Chelmsford Reforms. true
D. It supported the call given by the 10. Statement I: The passing of the Coercive
Congress to boycott elections in 1920 Acts made reconciliation between Britain
8. Which one of the following statements and her American colonies virtually
about the All India Depressed Classes impossible.
Association is not correct? Statement II: The British Parliament, having
A. The All India Depressed Classes issued the Stamp Act in 1765, repealed it
Association was formed in Nagpur with later.
M. C. Rajah as its first A. Both the statements are individually
elected President. true and Statement II is the correct
B. The All India Depressed Classes explanation of Statement I
Association was not attended by Dr. B. B. Both the statements are individually
R. Ambedkar in 1926 true but Statement II is not the correct
C. Ambedkar resigned from the All India explanation of Statement I
Depressed Classes Association and C. Statement I is true but Statement II is
formed his own All India Depressed false
Classes Congress in 1930.
D. Statement I is false but Statement II is
D. The All India Depressed Classes true
association favoured Ambedkar’s
11. Who among the following is the President
demand for separate electorate for the
of Republic of the Union of Myanmar?
depressed classes.
A. Aung San Suu Kyi
Direction: The following two(2) items consist of
two statements, statement I and statement II. B. Htin Kyaw

PAGE 1047
www.byjusexamprep.com

C. Myint Swe 17. The rank of Captain of the Indian Navy is


equivalent to which one of the following?
D. Henry Van Thio
12. In which one of the following cities are the A. Captain of the Indian Army
Summer Olympics, 2020 going to be held? B. Group Captain in the Indian Air Force
A. London B. Paris C. Lieutenant Colonel of the Indian Army
C. Tokyo D. Moscow D. Wing Commander of the Indian Air
13. BREXIT refers to the Great Britain leaving Force
which one of the following? 18. Amartya Sen was awarded the Nobel Prize
A. International Monetary Fund for his contribution to
B. Commonwealth A. Monetary Economics
C. World Trade Organization B. Welfare Economics
D. European Union C. Environmental Economics
14. Which one of the following sectors is not D. Development Economics
affected by the changes made in the
Foreign Direct Investment Policy in June 19. Goods and Services Tax likely to be levied in
2016? India is not a

A. Multi-brand retailing A. gross value tax


B. Defence B. value-added tax
C. Private security agencies C. consumption tax
D. Manufacturing of small arms and D. destination-based tax
ammunitions covered under the Arms
20. The Most Favoured Nation (MFN) Clause
Act, 1959
under WTO regime is based on the principle
15. The Indian Navy in October 2016 of
commissioned the highly maneuverable
fast attack craft INS Tihayu. INS Tihayu A. non-discrimination between nations
would be based in B. discrimination between nations
A. Kochi B. Panaji C. differential treatment between locals
C. Visakhapatnam D. Mumbai and foreigners

16. Which one of the following statements is D. uniform tariff across commodities
correct in relation to the Indian Air Force? 21. Which one of the following indices is now
A. It has bases in many friendly countries. used by the Reserve Bank of India to
measure the rate of inflation in India?
B. It does not carry out joint exercises with
any country. A. NASDAQ Index
C. It has a separate Maintenance B. BSE Index
Command.
C. Consumer Price Index
D. No officer has ever been elevated to
the rank of Marshal of the Indian Air D. Wholesale Price Index
Force.

PAGE 1048
www.byjusexamprep.com

22. Which one of the following intellectual 25. Which of the following statements is/are
property rights is protected without making correct regarding Right to Education in
any registration? India?
A. Copyright 1. Free and compulsory education should
B. Patent be provided to all children of the age of
six to fourteen years.
C. Industrial design
2. The imperative of the provision of the
D. Trademark Right to Education Act, 2009 is that
23. Which one of the following statements schools must have qualified teachers
relating to protection against arrest and and basic infrastructure.
detention of individuals under Article 22 is
3. There should be quality education
not correct?
without any discrimination on the
A. No person who is arrested shall be ground of economic, social and cultural
detained in custody without being background.
informed of the grounds for such
arrest. Select the correct answer using the code
given below.
B. No person shall be denied the right to
consult, and be defended by, a legal A. 1, 2 and 3 B. 1 and 2 only
practitioner of his/her choice.
C. 1 and 3 only D. 3 only
C. Every person who is arrested and
detained in custody shall be produced 26. Which of the following statements
before the nearest magistrate within a regarding Article 21 of the Constitution of
period of one week of such arrest. India is/are correct?

D. The right to protection against is not 1. Article 21 is violated when the under-
available to a person in jail pursuant to trial prisoners are detained under
a judicial order. judicial custody for an indefinite period.

24. Which one of the following statements 2. Right to life is one of the basic human
regarding freedom to manage religious rights and not even the State has the
affairs as per the Constitution of India is not authority to violate that right.
correct?
3. Under Article 21, the right of a woman
A. Every religious denomination shall have to make reproductive choices is not a
the right to manage its own affairs in dimension of personal liberty.
matters of religion except some minor
communities. Select the correct answer using the code
given below.
B. Every religion or any section thereof
shall have the right to own and acquire A. 1, 2 and 3 B. 1 and 2 only
movable and Immovable property. C. 1 and 3 only D. 2 only
C. Every religious community has the right
27. Which of the following statements relating
to establish and maintain institutions
to the powers of the President of India
for religious and charitable purpose.
is/are correct?
D. Every community has the right to
manage its own affairs in matters of 1. The executive power of the Union shall
religion. be vested in the President.

PAGE 1049
www.byjusexamprep.com

2. The executive power shall be exercised 31. The Twelfth Five-year Plan focused on
by the President only through officers inclusive growth. Which of the following
subordinate to him. were considered as challenges for
inclusiveness?
3. The supreme command of the defence
forces of the Union shall be vested in 1. Poverty
the President.
2. Group inequality
Select the correct answer using the code
given below. 3. Regional imbalance

A. 1, 2 and 3 B. 1 and 2 only 4. Unemployment

C. 1 and 3 only D. 3 only Select the correct answer using the code
given below.
28. Which country signed a Memorandum of
Understanding with India for promoting the A. 1, 3 and 4 only B. 1, 2, 3 and 4
production of Pigeon peas/Tur and other
C. 1, 2 and 4 only D. 2 and 3 only
pulses in July 2016?
A. South Africa B. Syria 32. Alexander Kerensky was

C. Egypt D. Mozambique A. the head of the Provisional


Government in Russia before the
29. The 8th BRICS summit 2016 was held in October Revolution
____.
B. a close confidant of Lenin, with whose
A. Brazil B. China help the Czar was dethroned
C. India D. South Africa C. the head of the Czar’s army
E. Russia
D. an advisor of Joseph Stalin
30. Which of the following statements are
33. George Washington was made the
correct about ‘Saakshar Bharat’ scheme?
commander in chief of the American forces
1. It is a centrally sponsored scheme
A. in December 1773, after the Boston tea
which was launched during the
party
Eleventh Five-year Plan.
2. The scheme applies to women in B. at the First Continental Congress in
particular and disadvantaged groups in September 1774
general. C. at the Second Continental Congress in
3. The scheme applies to persons above 1775
the age of 10 years. D. by the Continental Congress at the
4. The scheme is anchored with Declaration of Independence on 4th
Panchayati raj Institutions and Local July, 1776
self-Government. 34. During the mid 19th century Industrial
Select the correct answer using the code Revolution, the average life span of workers
given below. in Manchester was

A. 1, 2 and 3 B. 2, 3 and 4 A. 17 years B. 30 years


C. 1, 2 and 4 D. 1, 3 and 4 C. 55 years D. 62 years

PAGE 1050
www.byjusexamprep.com

35. The fact that the planets move around the Select the correct answer using the code
Sun, not in circles but in ellipses, was first given below.
demonstrated by
A. 1, 3 and 4 B. 1 and 4 only
A. Galileo C. 1 and 3 only D. 2, 3 and 4
B. Martin Luther 39. Consider the following statements about
C. Johannes Kepler the Salt March :
1. The Salt March was deliberately
D. Copernicus
ignored by the European media.
36. Who among the following is the author of
2. The Salt March was widely covered by
the book, A comparison between Woman
the American and European press.
and Men?
3. The Salt March was the first nationalist
A. Pandita Ramabai activity in which women participated in
B. Sarojini Naidu large numbers.

C. Tarabai Shinde 4. The Salt March impressed upon the


British the urgent need to devolve
D. Rameshwari Nehru more power to Indians.
37. Which of the following statements about Which of the statements given above are
the social base of the Arya Samaj in British correct?
India is/are correct?
A. 1, 2 and 4 B. 2, 3 and 4
1. It was located mainly in Punjab and C. 3 and 4 only D. 2 and 3 only
western Uttar Pradesh.
40. Match List-I with List-II and select the
2. It mainly comprised the trading castes. correct answer using the code given below
3. It was much more limited than that of the Lists :
the Brahmo Samaj. List-I List-II
(Act/Event) (Year)
Select the correct answer using the code
given below. A. Rowlatt Act 1. 1922
A. 1 and 2 only B. 2 and 3 only B. Salt March 2. 1931
C. 1 only D. 1, 2 and 3 C. Chauri Chaura 3. 1930
Incident
38. In the elections to the provincial
legislatures in 1937 in British India D. Second Round Table 4. 1919
Conference
1. only about 10 to 12 percent of the
population had the right to vote
2. the untouchables had no right to vote Codes:
A. A-4 B-3 C-1 D-2
3. the Congress won an absolute majority
in five out of eleven provinces B. A-3 B-1 C-4 D-2
4. the Muslim League won more than 80 C. A-4 B-3 C-2 D-1
percent of the seats reserved for D. A-3 B-4 C-2 D-1
Muslims

PAGE 1051
www.byjusexamprep.com

41. The Headquarters of the Southern Naval 2. Gravitational waves were generated
Command of the Indian Navy is located at due to merger of two black holes at
which one of the following places? about 1.4 billion light-years distant.
A. Karwar 3. Gravitational waves were inserted due
to collision of two white dwarf stars at
B. Kochi about 1.4 billion light-years distant.
C. Thiruvananthapuram Select the correct answer using the code
D. Chennai given below.

42. Which one of the following is a Peacetime A. 1 only


Gallantry Award? B. 1 and 2 only
A. Shaurya Chakra C. 2 and 3 only
B. Vir Chakra D. 1, 2 and 3
C. Yudh Seva Medal 46. Consider the following statement about a
famous football player :
D. Param Vir Chakra
“He was diagnosed with a growth hormone
43. Which one of the following is not an Inter- deficiency during his childhood but went on
Services establishment? to win three European Golden Shoes in his
A. Officers Training Academy career’.

B. National Defence Academy Who is that player?


A. Andres Iniesta
C. National Defence College
B. Lionel Messi
D. Armed Forces Medical College
C. Cristiano Ronaldo
44. In the year 2016, the Government of India
announced a ` 6,000 crore special package D. Zinedine Zidane
for the textile and apparel sector to 47. Hindustan Aeronautics Limited handed
1. create one crore jobs within three years over the first two indigenously designed
and manufactured light combat aircrafts to
2. create jobs equally for men and women the Indian Air Force in 2016. What is the
3. provide tax and production incentives name of these new combat aircrafts?
for the entrepreneurs A. Marut B. Garud
Select the correct answer using the code C. Tejas D. Pushpak
given below.
E. Mirage
A. 1 only B. 2 and 3 only
48. Which of the following statements about
C. 1 and 3 only D. 1, 2 and 3 Bitcoin is/are correct?
45. Which of the following statements is/are 1. It is a decentralized virtual currency.
correct? 2. It is generated through complex
1. In June 2016, the LIGO group of computer software systems.
scientists announced the detection of a 3. The Reserve Bank of India recognized it
second set of gravitational waves. as a legal tender in January 2016.

PAGE 1052
www.byjusexamprep.com

Select the correct answer using the code C. Disinvestment receipts


given below.
D. Interest receipts
A. 1 only B. 1 and 2 only
E. None of these
C. 2 and 3 only D. 1, 2 and 3
53. Which of the following are the powers of
49. Which of the following statements about the Supreme Court of India?
the Trans-Pacific Partnership (TPP) is /are
correct? 1. Original jurisdiction in a dispute
between the Government of India and
1. The TPP was signed by 12 Pacific Rim one or more States.
nations in the year 2015.
2. The power to hear appeals from the
2. The TPP is likely to be a game-changer High Courts
in global trade as member countries
account for about 40 percent of global 3. Passing decrees and orders for doing
GDP. justice in any matter before it
3. India is a founder member of TPP. 4 Render advice to the President of India
in matters of law
Select the correct answer using the code
given below. Select the correct answer using the code
A. 1, 2 and 3 B. 1 and 2 only given below.

C. 2 and 3 only D. 1 only A. 1, 2, 3 and 4

50. The 7.6% growth rate registered by Indian B. 1, 2 and 3 only


economy during the year 2015-16 is based C. 1 and 2 only
on
D. 3 and 4 only
A. Gross National Product at market prices
54. Which of the following statements
B. Gross Value Added at constant prices
regarding Indian federal system is/are
C. Gross Domestic Product at market correct?
prices
1. All States have equal representation in
D. Gross Domestic Product at constant the Rajya Sabha.
prices
2. Consent of a State is not required for
51. TRIPS Agreement pertains to altering its boundaries.
A. International tariff regime 3. There is no dual citizenship in India.
B. Intellectual property protection Select the correct answer using the code
C. International practices on trade given below.
facilitation A. 1, 2 and 3 B. 2 and 3 only
D. International taxation of property
C. 1 and 3 only D. 2 only
52. Which one of the following is not a
55. Which of the following statements with
component of Revenue Receipts of the
regard to Panchyats in India are correct?
Union Government?
1. Seats in a Panchayat are filled by direct
A. Corporate tax receipts
election from the territorial
B. Dividends and profits constituencies in the Panchayat area.

PAGE 1053
www.byjusexamprep.com

2. The Gram Sabha is the body of persons 3. not be a cooperative loan defaulter
registered in the electoral rolls relating
4. have studied minimum matriculation
to a village within the Panchayat area.
irrespective of category.
3. The Panchayats work on the principle
of constitutional autonomy. Select the correct answer using the code
given below.
4. The State Legislature may be law
endow the Panchayats with the power A. 1, 2 and 3 only
and authority to enable them to B. 1, 2 and 4 only
function.
C. 1, 2, 3 and 4
Select the correct answer using the code
given below. D. 3 and 4 only

A. 1, 2 and 3 B. 2, 3 and 4 59. Which of the following statements with


regard to the speech of Mahatma Gandhi at
C. 1, 2 and 4 D. 1 and 4 only the opening of the Banaras Hindu
56. Who among the following won a Gold University are correct?
medal for India in Men’s Javelin Throw 1. he charged the Indian elite with a lack
event at the 2016 Rio Paralympic Games? of concern for the laboring poor.
A. Rinku Hooda 2. He asserted that our salvation can
B. Devendra Jhajharia come only through the farmers.

C. Sundar Singh Gurjar 3. He highlighted the plight of the


untouchables.
D. Mariyappan Thangavelu
4. He promised to take up the cause of the
57. Arrange the following Commissions
mill owners of Ahmedabad.
chronologically on the basis of their date of
setting : Select the correct answer using the code
given below.
1. The Second Administrative Reforms
Commission A. 1, 2, 3 and 4
2. The Eleventh Finance Commission B. 1 and 2 only
3. Punchhi Commission C. 1, 2 and 3 only
4. Sarkaria Commission D. 3 and 4 only
Select the correct answer using the code 60. Which of the following statements about
given below. the Shiromani Gurudwara Prabandhak
A. 4, 2, 1, 3 B. 4, 3, 2, 1 Committee (SGPC) are correct?

C. 3, 2, 4, 1 D. 3, 4, 1, 2 1. It began as the political wing of the


Singh Sabhas in the late 19th century.
58. To be eligible to contest election under the
Haryana Panchayati Raj (Amendment) Act, 2. It was formed in 1920 as part of the
2015, a candidate should upcoming Akali movement.

1. have a functional toilet at home 3. It was founded to reclaim control of the


Sikh shrines from the government
2. have payment slips of power bills manipulated loyalist committees.

PAGE 1054
www.byjusexamprep.com

4. It formed the Akali Dal to coordinate 2. It proposed that industries like


groups (Jathas) to reclaim control of the electricity, railways, steel, machineries
shrines. and communication could be
developed in the public sector.
Select the correct answer using the code
given below. 3. The drafters found balancing industry
A. 1, 3 and 4 B. 2, 3 and 4 and agriculture very difficult.

C. 2 and 4 only D. 2 and 3 only 4. The drafters found balancing industry


and agriculture really easy.
61. What was/were the formative influence(s)
on the philosophy of Mahatma Gandhi? Which of the statements given above is/are
1. Gandhiji was influenced by the 18th correct?
century Pranami sect that advocated A. 1 only B. 1 and 2
the unity of faiths.
C. 2 and 3 D. 3 and 4
2. Gandhiji was influenced by the
theosophists. 64. Which of the following statements about
the Ilbert Bill (1883) is/are correct?
3. Gandhiji was an admirer of the writings
of Romantics like Wordworth. 1. It proposed to grant limited criminal
Select the correct answer using the code jurisdiction to native officials.
given below. 2. It proposed to grant complete civil and
A. 1 and 2 only B. 2 and 3 only criminal jurisdiction to native officials.
C. 1, 2 and 3 D. 1 only 3. The proposed Bill generated opposition
from England’s European subjects in India.
62. Consider the following statements about
the different meanings of ‘Swaraj’ as 4. In spite of opposition to the Bill, it was
articulated by Mahatma Gandhi : passed without any modifications.
1. Swaraj is intimately linked with Ahimsa Select the correct answer using the code
(non-violence) and Satyagraha given below.
(adherence to truth).
A. 1 and 2 B. 2 only
2. Swaraj has two senses-one political and
one beyond the realm of politics. C. 1 and 3 D. 3 and 4
3. Swaraj is something that requires time 65. Which one of the following statements is
and patience to acquire. not correct in respect of the South Asian
4. With determination, Swaraj could be Association for Regional Cooperation
obtained easily and quickly. (SAARC)?

Which of the statements given above is/are A. Its Headquarters is located in


correct? Kathmandu.
A. 1 only B. 1 and 2 only B. China is the only country with an
Observer status in SAARC.
C. 3 and 4 D. 1, 2 and 3
63. Consider the following statements about C. The first SAARC Summit was held in
the Second Five-Year Plan : Dhaka.

1. It was drafted under the leadership of D. The Eighteenth SAARC Summit was
K. N. Raj held in Nepal.

PAGE 1055
www.byjusexamprep.com

66. India signed an agreement in 2016 to 71. Which one of the following statements is
develop a strategic port in one of its not correct?
neighbouring countries. What is the name
A. Pavo cristatus is the national bird of
of the port?
India.
A. Chabahar B. Gwadar
B. Nelumbo nucifera gaertn is the national
C. Hambantota D. Mongla flower of India.
67. India became a member of which one of the C. Ficus benghalensis is the national tree
following in 2016? of India.
A. Non-Proliferation Treaty D. Mangifera indica is the national animal
B. Missile Technology Control Regime of India.

C. Nuclear Suppliers Group 72. Which one of the following ports is located
on estuary?
D. Wassenaar Arrangement
A. Kandala B. Marmagao
68. How many medals were won by India in
2016 Summer Olympics? C. Kolkata-Haldia D. Tuticorin

A. One silver and one Bronze 73. ‘Rand/ZAR’ is the currency of

B. Two Gold A. Burundi B. Libya


C. Two Bronze C. Sudan D. South Africa
D. None 74. Which one of the following about sugar
industry of Peninsular India is not correct?
69. The sensitive information leaked in August
2016 concerns which one of the following A. High yield per hectare of sugarcane
defence platforms of India?
B. Higher sucrose content
A. Fifty Generation Fighter Aircraft
C. Long crushing season
B. Combat Helicopters
D. Most of the mills in the Peninsula are
C. Submarines located mainly along the east coast
D. Unmanned Aerial Vehicles 75. The Gulf Stream is a poleword flowing
70. Which one of the following statements is current in the Atlantic Ocean. Which one of
correct in relation to the GST Bill passed by the following statements with regard to this
the Rajya Sabha in August 2016? is not correct?

A. It will replace all central taxes, duties, A. It is similar to the Kuroshio current in
etc, only by a single tax. the North Pacific Ocean.

B. It will subsume central as well as State B. It transports warm, tropical water


taxes, duties, etc. towards polar region.

C. GST will be levied on alcoholic liquor for C. This current is a major factor in weather
human consumption at a uniform rate along the east coast of the USA.
of 25 percent. D. The warm water of the Gulf Stream
D. Petroleum and petroleum products sustains the coral reefs of West Pacific
shall not be subject to the levy of GST. Coast.

PAGE 1056
www.byjusexamprep.com

76. Which one of the following statements with 80. What is the maximum number of states of
regard to Jet stream, an upper level matter?
tropospheric wave, is not correct?
A. Three B. Four
A. It is narrow band of high-velocity wind. C. Five D. Variable
B. It follows the wave path near the 81. Which one of the following with regard to
tropopause at elevations of 12 KM or Aridisol, one of the soil orders, is not
Above 12 KM. correct?
C. Jet streams are typically continuous A. Lack of water for plants during most
over long distances. part of the years
D. In summer, the polar front jet achieves B. High organic matter
its maximum force.
C. Large accumulation of carbonates at
77. The molecular mass of sulphuric acid is 98. depth
If 49 g of the acid is dissolved in water to D. Absence of deep wide cracks
make one litre of solution, what will be the
strength of the acid? 82. Which of the following statements with
regard to cloudburst is/are correct?
A. Two normal B. One normal
1. It is defined as sudden localizedvery
C. 0.5 normal D. Four normal heavy downpour with cloud thunder
and lightning.
78. Which one of the following statements is
not correct? 2. It mostly occurs in the hilly areas.

A. The cathode rays originate from 3. It results into very high intensity of
cathode and proceed towards the rainfall, i.e., 250 mm-300 mm in a
anode in a cathode-ray discharge tube. couple of hours.

B. The television picture tubes are nothing 4. It occurs only during daytime.
but cathode ray tubes. Select the correct answer using the code
given below.
C. The cathode rays themselves are not
visible. A. 1, 2 and 3 B. 1, 3 and 4

D. The characteristics of cathode rays C. 2 and 3 only D. 2 only


depend upon the nature of the gas 83. Tsunami waves are the undersea
present in the cathode-ray tube. occurrence of earthquake exceeding 7.5 on
79. The chemical properties of an element Richter scale. Which one of the following
depend upon statements regarding this is not correct?
A. It often generates strong waves
A. the number of isotopes of the element
B. The pacific coasts are most vulnerable
B. the mass number of the element
to Tsunami waves.
C. the total number of neutrons in the C. Tsunami waves are also called high-
element energy tidal waves or seismic sea
D. the number of electrons in the waves.
outermost shell of the element D. Tsunami is a Latin word.

PAGE 1057
www.byjusexamprep.com

84. Tropical evergreen rain forest biome 87. Which one of the following statements with
provides optimum environmental regard to the Indian Railways is not correct?
conditions for the growth of plants and
A. The Indian rail networks have been
animals. Which one among the following
developed throughout the Konkan
statements regarding this is not correct?
coast.
A. It has heavy rainfall and high B. There is very low density of railway
temperature throughout the year. lines in the Himalayan region.
B. This biome is also called mega-thermal C. The north Indian plain has a dense
biome. network of railway.
C. The evergreen rain forest biome D. At present, India has the largest railway
extended between 10o N and 10o S network in the world.
latitudes.
88. Which one of the following is not a
D. The maximum development of this biosphere reserve of India?
biome has taken place in central and A. Agasthyamalai B. Nokrek
southern California, and north-western
coastal lands of Africa. C. Great Nicobar D. Great Himalayan

85. Which one of the following statements with 89. With regard to ‘Project Tiger’, which one of
regard to the winter solstice is correct? the following statements is not correct?
A. It was launched in India in the year
A. The South Pole experiences 24 hours of
1973.
darkness.
B. The objective of the project is to
B. It occurs on June 21. preserve the habitats and tigers therein
C. The North Pole experiences 24 hours of as natural heritage.
darkness. C. The project emphasized to ensure
viable population of tigers in India.
D. The Sun is at aphelion.
D. There are no tiger reserves in north-
86. Which of the following statements with eastern part of India.
regard to the Mediterraneam agriculture
is/are correct? 90. The optical phenomenon that is primarily
responsible for the observation of rainbow
1. The Mediterranean agriculture is highly on a rainy day is
specialized commercial agriculture.
A. diffraction B. interference
2. It is intensive subsistence agriculture
C. dispersion D. reflection
dominated by wet paddy cultivation.
91. A photon of X-ray has energy of 1 keV. A
3. It is a primitive subsistence agriculture. photon of visible radiation has energy of
4. Viticulture is a speciality of the 3 eV. In this context, which one of the
Mediterranean region. following statements is not correct?

Select the correct answer using the code A. The wavelength of X-ray photon is less
given below. than the wavelength of visible radiation
photon.
A. 1 only B. 2 and 3
B. Both the photons have different
C. 1 and 4 only D. 1, 3 and 4 energies.

PAGE 1058
www.byjusexamprep.com

C. The speeds of both the photons in C. fatty acids, glucose and vitamins
vacuum are different D. vitamin C, vitamin D and glucose
D. The frequency of X-ray photon is higher 96. The mammalian heart is myogenic and it is
than the frequency of visible radiation regulated by nerves. The heartbeat
photon. originates from
92. A parallel-plate capacitor of capacitance C1 A. sinoatrial node
is made using two gold plates. Another
parallel-plate capacitor of capacitance C2 is B. QRS wave
made using two aluminium plates with C. T wave
same plate separation, and all the four
D. hepatic portal system
plates are of same area. If ρg and Pa are
respectively the electrical resistivities of 97. Kwashiorkor disease in children is caused
gold and aluminium, then which one of the by
following relations is correct? A. sufficient carbohydrates but less fats in
A. C1 > C2 B. C2 > C1 diet

C. C1ρa = C2ρg D. C1 = C2 B. Deficient proteins in diet

93. A copper wire of radius r and length l has a C. sufficient vitamins but deficient fats in
resistance of R. A second copper wire with diet
radius 2r and length l is taken and the two D. sufficient fats but deficient vitamins in
wires are joined in a parallel combination. diet
The resultant resistance of the parallel
98. Which one of the following statements with
combination of the two wires will be
regard to volcanoes is not correct?
A. 5 R B. A. Stratovolcanoes produce lava flows
that initially follow valleys but are
C. D. highly resistant to erosion.
B. The surrounding areas can remain
94. Consider the electromagnetic radiations highlands, lava ridges or mesas.
having wavelengths 200 nm, 500 nm and
1000 nm. Which wavelength (s) of the C. Hawaiian shield volcanoes are eroded
following can make visual sensation to a by steams that form deeply carved
human eye? valleys with steeply sloping heads.
D. The system of streams on a dissected
A. 200 nm and 500 nm
volcano cone is not a radial drainage
B. 500 nm and 1000 nm pattern.
C. 500 nm only 99. Which one of the following is the correct
sequence of occurrences of rivers from
D. 200 nm and 1000 nm
Chennai to Kolkata when travelled by road
95. The plant growth regulators are small, (shortest distance)?
simple molecules of diverse chemical
composition. They are: A. Krishna, Kaveri, Godavari, Mahanadi,
Subarnarekha
A. carbohydrates, fats and proteins
B. Krishna, Periyar, Godavari, Mahanadi,
B. indole compounds, adenine
Subarnarekha
derivatives, carotenoids and terpenes

PAGE 1059
www.byjusexamprep.com

C. Penneru, Krishna, Godavari, Mahanadi, 104. ‘Plum Pudding Model’ for an atom was
Subarnarekha proposed by
D. Penneru, Mahanadi, Subarnarekha, A. Antoine Lavoisier
Godavari, Krishna B. Robert Boyle
100. The ratio of gross cropped area to the net C. Ernest Rutherford
sown area is called
D. J. J. Thomson
A. cropping intensity
105. There are two elements-calcium (atomic
B. intensity of crop rotation number 20) and argon (atomic number 18).
The mass number of both the elements is
C. crop productivity
40. They are therefore known as
D. cropping diversity
A. isotones B. isochores
101. In which one of the following States is C. isobars D. isotopes
Loktak Lake situated?
106. Which one of the following is the most
A. Sikkim B. Himachal Pradesh characteristic property of an element?
C. Manipur D. Meghalaya A. Density B. Boiling point
E. Assam C. Mass number D. Atomic number
102. Which one of the following statements is 107. The pH value of a sample of multiple-
correct? distilled water is
For the purpose of Census 2011 A. zero

A. a person aged seven and above who B. 14


can both read and write with C. very near to zero
understanding in any language is
treated as a literate D. very near to seven
108. Which one of the following is a physical
B. a person aged eight and above who can
change?
both read and write with
understanding in any language is A. Burning of coal
treated as a literate B. Burning of wood
C. a person aged nine and above who can C. Heating of a platinum crucible
both read and write with
understanding in any language is D. Heating of potassium chlorate
treated as a literate 109. For which one of the following does the
centre of mass lie outside the body?
D. a person aged ten and above who can
both read and write with A. A fountain pen B. A cricket ball
understanding in any language is
C. A ring D. A book
treated as a literate.
110. A parallel-plate capacitor, with air in
103. What is the number of atoms in 46 g of between the plates, has capacitance C.
sodium-23 (N = Avogadro constant)? Now the space between the two plates of
A. N/2 B. N the capacitor is filled with a dielectric of
dielectric constant 7. Then the value of the
C. 2 N D. 23 N capacitance will become

PAGE 1060
www.byjusexamprep.com

A. C B. B. single-stranded RNA
C. double-stranded DNA and RNA
C. 7C D. 14C
D. double-stranded RNA only
111. Which one of the following is not a
semiconductor? 117. In human body, the cell growth and
differentiation are highly controlled and
A. Silicon B. Germanium
regulated, but in cancer cells
C. Quartz D. Gallium arsenide
A. there is breakdown of these regulatory
112. Suppose voltage V is applied across a mechanisms leading to formation of
resistance R. The power dissipated in the benign and malignant tumors
resistance is P. Now the same voltage V is
applied across a parallel combination of B. controlled cell division and over-
three equal resistors each of resistance R. production of genetic material occur
Then the power dissipated in the second C. RNA is mutated and produced in less
case will be amount
A. P B. 3P D. DNA is mutated and produced in less
C. p/3 D. 2p/ 3 amount
113. When a piece of pure silicon is doped with 118. AIDS is caused by the Human
aluminium, then Immunodeficiency Virus (HIV). The
transmission of HIV infection generally
A. the conductivity of the doped silicon
occurs through
piece will remain the same
B. the doped silicon piece will become n- A. eating contaminated food and water
type B. transfusion of contaminated blood and
C. the doped silicon piece will become p- blood products
type C. inhaling polluted air
D. the resistivity of the doped silicon piece D. shaking hand with infected person
will increase
119. The process of copying genetic information
114. Bleeding of gums, falling of teeth, fragile
from one strand of DNA into RNA is termed
bones and delayed wound healing occur
as
due to the deficiency of which one of the
following vitamins? A. Translation B. Transcription
A. Vitamin C B. Vitamin K C. Replication D. Mutation
C. Vitamin D D. Vitamin B 120. Polynucleotide chain of DNA contains
115. Syngamy results information of A. a nitrogenous base, deoxyribose sugar
A. Haploid Zygote and phosphate group
B. Diploid Zygote B. a nitrogenous base, ribose sugar and
phosphate group
C. Non - Motile Male Gametes
D. Motile Male Gametes C. deoxyribose sugar, ribose sugar and
phosphate group
116. Most viruses that infect plants possess
D. a nitrogenous base and phosphate
A. single-stranded DNA group only

PAGE 1061
www.byjusexamprep.com

Elementary Mathematics

1. If and , then what is 6. If and ,


then what is equal to?
the value of xy?
A. 2ab B.
A. 5 B. 15
C. D.
C. 25 D. 30
7. What is the square root of
2. If and , then what is
the value of (given that and ?
)?
A. 1 B. 2
A. 7 B. 9
C. 3 D. 4
C. 16 D. 49
8. ABCD is a square. X is the mid-point of AB
3. Consider the following statements : and Y is the mid-point of BC
1. Of two consecutive integers, one is Consider the following statements :
even.
1. Triangles ADX and BAY are congruent.
2. Square of an odd integer is of the form
2.

Which of the above statements is/are 3. DX is inclined at an angle 60o with AY


correct? 4. DX is not perpendicular to AY
A. 1 only Which of the above statements are
B. 2 only correct?

C. Both 1 and 2 A. 2, 3 and 4 only

D. Neither 1 nor 2 B. 1, 2 and 4 only

4. The mean of 5 numbers is 15. If one more C. 1, 3 and 4 only


number is included, the mean of the 6 D. 1 and 2 only
numbers becomes 17. What is the included
number? 9. What is the number of digits in ?

A. 24 B. 25 (Given that )
C. 26 D. 27 A. 14 B. 13
5. In an asymmetrical distribution, if the mean C. 12 D. 11
and median of the distribution are 270 and
220 respectively, then the mode of the data 10. If , then what is the
is value of ?

A. 120 B. 220 A. 3 B. 2

C. 280 D. 370 C. 1 D. 0

PAGE 1062
www.byjusexamprep.com

11. Which one among the following is the C. Ratio of AD to AB


largest?
D. Ratio of AC to BD
A. B. 18. The radius of a circle is increased so that its
circumference increases by 15%. The area
C. D. of the circle will increase by
A. 31.25% B. 32.25%
12. If then what is
equal to? C. 33.25% D. 34.25%
19. ABCD is a rectangle. The diagonals AC and
A. 0 B. 1
BD intersect at O. If AB = 32 cm and AD = 24
C. 2 D. 3 cm, then what is OD equal to?
13. What is the value of θ which satisfies the A. 22 cm B. 20 cm
equation ?
C. 18 cm D. 16 cm
A. 0o B. 30o
20. In the figure given below, PQRS is a
C. 45o D. 60o parallelogram. PA bisects angle P and SA
14. A ball of radius 1 cm is put into a cylindrical bisects angle S. What is angle PAS equal to?
pipe so that it fits inside the pipe. If the
length of the pipe is 14 m, what is the
surface area of the pipe?
A. 2200 square cm
B. 4400 square cm
C. 8800 square cm A. 60o B. 75o
D. 17600 square cm C. 90o D. 100o
15. If the perimeter of a rectangle is 10 cm and 21. In the figure given below, and
the area is 4 cm2, then its length is BD and CD bisect angles B and
A. 6 cm B. 5 cm C respectively. What are the values of X and
Y respectively?
C. 4.5 cm D. 4 cm
16. The areas of two circular fields are in the
ratio 16 : 49. If the radius of the bigger field
is 14 m, then what is the radius of the
smaller field?
A. 4 m B. 8 m
C. 9 m D. 10 m A. 10 and 130 B. 10 and 125
17. In a trapezium ABCD, AB is parallel to CD C. 20 and 130 D. 20 and 125
and the diagonals intersect each other at O.
What is the ratio of OA to OC equal to? 22. In the figures given below, PQR is a non-
isosceles right-angled triangle, right angled
A. Ratio of OB to OD at Q. If LM and QT are parallel and
B. Ratio of BC to CD then what is equal to?

PAGE 1063
www.byjusexamprep.com

C. square units

D. 1 square units

26. If then what is

equal to ?
A. B.
A. 0 B. 1
C. D.
C. ab D. 2ab
23. In the figure given below, ABC is a triangle
with AB = BC and D is an interior point of 27. If and , then
the triangle ABC such that what can be the value of ?
A. 1 B. 3
C. 5 D. 7
28. The number of prime number which are
less than 100 is
A. 24 B. 25
Consider the following statements :
C. 26 D. 27
1. Triangle ADC is an isosceles triangle.
2. D is the centroid of the triangle ABC 29. If a + b = 5 and ab = 6, then what is the value
of a3 + b3?
3. Triangle ABD is congruent to the
triangle CBD A. 35 B. 40
Which of the above statements are C. 90 D. 125
correct?
30. ABCDEF is a regular polygon. Two poles at C
A. 1 and 2 only B. 2 and 3 only and D are standing vertically and subtend
C. 1 and 3 only D. 1, 2 and 3 angles of elevation 30o and 60o at A
respectively. What is the ratio of the height
24. If one root of (a2-5a+3)x2 +(3a-1)x+2=0 is of the pole at C to that of the pole at D?
twice the other, then what is the value of
‘a’? A. 1 : 1 B.

A. B. C. D.
31. Let ABCD be a rectangle. Let P, Q, R, S be
C. D. the mid-points of sides AB, BC, CD, DA
respectively. Then the quadrilateral PQRS
25. Let P, Q, R be the mid-points of sides AB, BC, is a
CA respectively of a triangle ABC. If the area
A. Square
of the triangle ABC is 5 square units, then
the area of the triangle PQR is B. Rectangle, but need not be a square

A. square units C. Rhombus, but need not be a square


D. Parallelogram, but need not be a
B. square units rhombus

PAGE 1064
www.byjusexamprep.com

32. Three circles each of radius 3.5 cm touch If the mean of the above distribution is 50,
one another. The area subtended between what is the value of f?
them is
A. 24 B. 34
A. square units C. 56 D. 96

B. square units 38. Two parallel chords of a circle whose


diameter is 13 cm are respectively 5 cm and
C. square units 12 cm in length. If both the chords are on
the same side of the diameter, then the
distance between these chords is
D. square units
A. 5.5 cm B. 5 cm
33. If , then what C. 3.5 cm D. 3 cm
is equal to? 39. A copper wire when bent in the form of a
square encloses an area of 121 cm2. If the
same wire is bent in the form of a circle, it
A. B.
encloses an area equal to
C. D. A. 121 cm2 B. 144 cm2
C. 154 cm2 D. 168 cm2
34. What is the value of ?
40. If the surface area of a sphere is reduced to
A. 2 B. 2 2 one-ninth of the area, its radius reduces to
A. One-fourth B. One-third
C. 2 tan x D. 0
C. One-fifth D. One-ninth
35. What is equal to?
41. A field is divided into four regions as shown
in the given figure. What is the area of the
A. B. field in square metre?
C. 1 D. 2
36. (NP–1 – 1) is a multiple of p, if N is prime to
p and p is a
A. Prime number
B. Rational number
A. B.
C. Real number
D. Composite number C. D.

37. Consider the following distribution : 42. In the figure given below, D is the diameter
of each circle. What is the diameter of the
shaded circle?

PAGE 1065
www.byjusexamprep.com

A. B. 48. The mean marks obtained by 300 students


in a subject are 60. The mean of top 100
C. D. students was found to be 80 and the mean
of last 100 students was found to be 50. The
43. In the figure given below, AC is perellel to mean marks of the remaining 100 students
ED and AB = DE= 5 cm and BC = 7 cm. What are
is the area ABDE : area BDE : area BCD equal
to? A. 70 B. 65
C. 60 D. 50
49. Let a, b, c, d, e, f, g be consecutive even
numbers and j, k, l, m, n be consecutive odd
numbers. What is the average of all the
A. 10 : 5 : 7 B. 8 : 4 : 7 numbers?
C. 2 : 1 : 2 D. 8 : 4 : 5
A. B.
44. The cost of a diamond varies directly as the
square of its weight. A diamond broke into
C. D. None of these
four pieces with their weights in the ratio of
1 : 2 : 3 : 4. If the loss in total value of the
50. From the top of a building 90 m high, the
diamond was ₹70,000, what was the price
angles of depression of the top and the
of the original diamond?
bottom of a tree are 30 and 45 respectively.
A. ₹1,00,000 B. ₹1,40,000 What is the height of the tree?
C. ₹1,50,000 D. ₹1,75,000 A. B.
45. The HCF of two expressions p and q is 1.
C. D.
What is the reciprocal of their LCM?
A. B. 51. A 225 m long train is running at a speed of
30 km/hour. How much time does it take to
C. pq D. cross a man running at 3 km/hours in the
same direction?
46. What is the solution of the equation
A. 40 seconds
?
B. 30 seconds
A. 10 B. 3 C. 25 seconds
C. 1 D. 0 D. 15 seconds
47. Consider the following statements : 52. A passenger train departs from Delhi at
1. If a = bc with HCF (b, c) = 1, then HCF (c, 6 p.m. for Mumbai. At 9 p.m., an express
ab) = HCF (a, c) train, whose average speed exceeds that of
the passenger train by 15 km/hour leaves
2. If a = bc with HCF (b, c) = 1, then LCM
Mumbai for Delhi. Two trains meet each
(a, b) = LCM (a, c)
other mid-route. At what time do they
Which of the above statements is/are meet, given that the distance between the
correct? cities is 1080 km?
A. 1 only B. 2 only A. 4 pm B. 2 am
C. Both 1 and 2 D. Neither 1 nor C. 12 midnight D. 6 am

PAGE 1066
www.byjusexamprep.com

53. In a 100 m race, A runs at a speed of . If A Then which one of the following is a null
set?
gives a start of 4 m to B and still beats him
by 12 seconds, what is the speed of B? A. B.

C. D.
A. m/s B. m/s
59. If α and β are the roots of the equation
C. m/s D. m/s then what is α2 + β2 equal
to?
54. The angles of a triangle are in the ratio
2 : 4 : 3. The smallest angle of the triangle A. p2 – 2q B. q2 – 2q
is. C. p2 + 2q D. q2 – q
A. 20o B. 40o 1 1
60. If x = and y = , t > 0, t ≠ 1, then
C. 50o D. 60o t−1 t−1
t t
55. The area of a regular hexagon of side ‘a’ is what is the relation between x and y?
equal to A. yx = x1/y B. x1/y = y1/x
A. square units C. yy = yx D. xy = y1/x
61. From an aeroplane vertically over a straight
B. square units horizontal road, the angles of depression of
two consecutive kilometer-stones on the
C. square units opposite sides of the aeroplane are
observed to be α and β. The height of the
D. square units aeroplane above the road is

56. If the radius of a right circular cone is A. B.


increased by p% without increasing its
height, then what is the percentage C. D.
increase in the volume of the cone?
A. p2 B. 2p2 62. (x + 4) is a factor of which one of the
following expressions?
C. D. A.

57. Ice-cream, completely filled in a cylinder of B.


diameter 35 cm and height 32 cm, is to be C.
served by completely filling identical
disposable cones of diameter 4 cm and D.
height 7 cm. The maximum number of 63. If A : B = 3 : 4, then what is the value of the
persons that can be served in this way is
expression ?
A. 950 B. 1000
C. 1050 D. 1100 A. B.
58. If A = {x : x is a multiple of 7},
C.
B = {x : x is a multiple of 5} and
C = {x : x is a multiple of 35}, D. Cannot be determined

PAGE 1067
www.byjusexamprep.com

64. What number must be subtracted from A.


both the numerator and the denominator
of the fraction so that it becomes ? B.

A. 6 B. 8 C.
C. 9 D. 11
D.
65. Rajendra bought a mobile with 25%
discount on the selling price. If the mobile 71. Sunil wants to speed `200 on two types of
cost him ₹4,875, what is the original selling sweets, costing `7 and `10 respectively.
price of the mobile? What is the maximum number of sweets he
can get so that no money is left over?
A. ₹6,300 B. ₹6,400
A. 25 B. 26
C. ₹6,500 D. ₹6,600
C. 27 D. 28
5th
66. part of the population in a village are 72. A thief is spotted by a policeman from a
9
distance of 100 m. When the policeman
males. If 30% of the males are married, the
starts the chase, the thief also starts
percentage of unmarried females in the
running. If the speed of the thief is 8
total population is
km/hours and that of the policeman is 10
2 2 km/hour, then how far will the thief have to
A. 20 B. 27
9 9 run before he is overtaken?

7 2 A. 200 m B. 300 m
C. 27 D. 29
9 9 C. 400 m D. 500 m
67. In a class of 49 students, the ratio of girls to 73. If 5 men can do a piece of work in 10 days
boys is 4 : 3. If 4 girls leave the class, the and 12 women can do the same work in 15
ratio of girls to boys would be days, the number of days required to
A. 11 : 7 B. 8 : 7 complete the work by 5 men & 6 women is

C. 6 : 5 D. 9 : 8 A. days B. 8 days
68. The value of x which satisfy the equation
51+x + 51-x = 26 are C. days D. 12 days

A. -1, 1 B. 0, 1 74. If (a% of a)+(b% of b) = 2% of ab, then what


C. 1, 2 D. -1, 0 percent of a is b?

69. A and B working together can finish a piece A. 50%


of work in 12 days while B alone can finish B. 75%
it in 30 days. In how many days can A alone
finish the work? C. 100%
A. 18 days B. 20 days D. Cannot be determined
C. 24 days D. 25 days 75. The ratio of two numbers is 1 : 5 and their
product is 320. What is the difference
70. What is the LCM of and between the square of these two numbers?
?

PAGE 1068
www.byjusexamprep.com

A. 1024 B. 1256 80. An individual purchases three qualities of


pencils. The relevant data is given below:
C. 1536 D. 1640
76. The system of equations and
is

A. Consistent with a unique solution


B. Consistent with infinitely many
solutions It is known that the average price per pencil
C. Inconsistent is ₹1.25. Wwhat is the value of x?
A. ₹10 B. ₹30
D. None of the above
C. ₹40 D. ₹60
77. 25 kg of alloy X is mixed with 125 kg of alloy
Y. If the amount of lead and tin in the alloy 81. Consider the following frequency
X is in the ratio 1 : 2 and the amount of lead distribution :
and tin in the alloy Y is in the ratio 2 : 3, then
what is the ratio of lead to tin in the
mixture?
A. 1 : 2 B. 2 : 3
C. 3 : 5 D. 7 : 11
78. In a pie diagram, there are four slices with
angles 150o, 90o, 60o and 60o. A new pie What are the values of f1 and f2
diagram is formed by deleting one of the respectively?
slices having angle 60 in the given pie A. 10 and 17 B. 17 and 10
diagram. In the new pie diagram
C. 11 and 16 D. 16 and 11
o
A. The largest slice has angle 150 82. If D is the number of degrees and R is the
B. The smallest slice has angle 70 o number of radians in an angle θ, then which
one of the following is correct?
C. The largest slice has angle 180o
A. B.
D. The smallest slice has angle 90o
C. D.
79. Aman and Alok attempted to solve a
83. What is the minimum value of
quadratic equation. Aman made a mistake
in writing down the constant term and ?
ended up in roots (4, 3). Alok made a A. 6 B. 9
mistake in writing down the coefficient of X
to get roots (3, 2). The correct roots of the C. 12 D. 13
equation are 84. An aeroplane flying at a height of 300 m
above the ground passes vertically above
A. -4, -3
another plane at an instant when the angles
B. 6, 1 of elevation of the two planes from the
same point on the ground are 60 and 45
C. 4, 3
respectively. What is the height of the
D. -6, -1 lower plane from the ground?

PAGE 1069
www.byjusexamprep.com

A. B. 91. Which one of the following triples does not


represent the sides of a triangle?
C. D. A. (3, 4, 5) B. (4, 7, 10)

85. ABC is a triangle and D is a point on the side C. (3, 6, 8) D. (2, 3, 6)


BC. If BC = 12 cm, BD = 9 cm and
92. What is equal to?
then the length of AC is
equal to
A. B.
A. 5 cm B. 6 cm
C. 8 cm D. 9 cm C. 9 D. 99

86. What is the remainder when the number 93. In the figure given below, PQ is parallel to
(4444)4444 is divided by 9? RS and PR is parallel to QS. If
A. 4 B. 6 and then what is equal
to?
C. 7 D. 8

87. What is the value of ?

A. 0 B. 1
C. 2 D. 3
88. If 15 men take 21 days of 8 hours each to
do a piece of work, then what is the number
of days of 6 hours each that 21 women
would take, if 3 women would do as much
work as 2 men? A. 55o B. 70o
A. 18 B. 20 C. 75o D. 80o
C. 25 D. 30 94. The difference between the simple and the
compound interest on a certain sum of
89. A sum of ₹8,400 was taken as a loan. This is money at 4% per annum in 2 years is ₹10.
to be paid in two equal instalments. If the What is the sum?
rate of interest is 10% per annum,
compounded annually, then the value of A. ₹5,000 B. ₹6,000
each instalment is
C. ₹6,250 D. ₹7,500
A. ₹4,200 B. ₹4,480 95. In the figure given below, M is the mid-
C. ₹4,840 D. None of these point of AB and and
Then the triangle ADM is
12 congruent to the triangle BCM by
90. What is the value of 3 4 ?
125

A. B.

C. D.

PAGE 1070
www.byjusexamprep.com

A. SAS rule B. SSS rule What is the present age of her son?
C. ASA rule D. AAA rule A. 1 years

96. If then B. 2 years


what is the value of tan x? C. 3 years

A. 2 B. 1 D. 4 years
99. The pair of linear equations
3 1
C. D. and intersect each other, if
2 3
A.
97. If α and β are the roots of the quadratic
equation where B.

then what is the maximum value of ? C.


D.
A. 2 B. –2
100. If each of the dimensions of a rectangle is
C. 9 D. –9 increased by 200%, the area is increased by
98. Leela got married 6 years ago. Today her A. 300%
age is times her age at the time of her B. 400%
marriage. Her son’s age is times her C. 600%

age. D. 800%

PAGE 1071
www.byjusexamprep.com

ANSWERS KEY I 2017


ENGLISH

1 B 21 D 41 D 61 D 81 A 101 C
2 D 22 B 42 B 62 C 82 D 102 A
3 C 23 C 43 B 63 B 83 C 103 C
4 C 24 A 44 A 64 A 84 D 104 A
5 D 25 A 45 A 65 C 85 B 105 C
6 D 26 C 46 A 66 D 86 D 106 B
7 D 27 C 47 D 67 C 87 D 107 B
8 D 28 D 48 C 68 C 88 C 108 A
9 C 29 D 49 D 69 C 89 D 109 C
10 A 30 B 50 A 70 D 90 A 110 A
11 C 31 A 51 B 71 D 91 C 111 D
12 A 32 B 52 D 72 B 92 A 112 C
13 C 33 A 53 D 73 D 93 C 113 C
14 A 34 C 54 B 74 C 94 B 114 D
15 D 35 C 55 A 75 C 95 C 115 B
16 B 36 B 56 B 76 B 96 B 116 C
17 B 37 B 57 D 77 B 97 C 117 C
18 A 38 C 58 B 78 B 98 A 118 D
19 A 39 D 59 B 79 D 99 C 119 D
20 D 40 B 60 B 80 B 100 D 120 B

GENERAL KNOWLEDGE
1 A 21 C 41 B 61 C 81 B 101 C
2 A 22 A 42 A 62 D 82 A 102 A
3 D 23 C 43 A 63 C 83 D 103 C
4 C 24 A 44 C 64 C 84 D 104 D
5 A 25 A 45 B 65 B 85 C 105 C
6 B 26 B 46 B 66 A 86 C 106 D
7 D 27 C 47 C 67 B 87 D 107 D
8 D 28 D 48 B 68 A 88 D 108 C
9 C 29 C 49 B 69 C 89 D 109 C
10 B 30 C 50 D 70 B 90 C 110 C
11 B 31 B 51 B 71 D 91 C 111 C
12 C 32 A 52 C 72 B 92 D 112 B

PAGE 1072
www.byjusexamprep.com

13 D 33 C 53 A 73 D 93 D 113 C
14 A 34 A 54 B 74 D 94 C 114 A
15 C 35 C 55 C 75 D 95 B 115 B
16 C 36 C 56 B 76 D 96 A 116 B
17 B 37 A 57 A 77 B 97 B 117 A
18 B 38 C 58 A 78 D 98 D 118 B
19 A 39 B 59 B 79 D 99 C 119 B
20 A 40 A 60 B 80 C 100 A 120 A

Elementary Mathematics
1 B 21 B 41 C 61 B 81 B
2 D 22 B 42 A 62 C 82 C
3 C 23 C 43 C 63 C 83 B
4 C 24 D 44 B 64 C 84 A
5 C 25 C 45 D 65 B 85 D
6 B 26 C 46 A 66 B 86 C
7 C 27 B 47 C 67 C 87 B
8 C 28 C 48 A 68 A 88 D
9 A 29 B 49 D 69 B 89 D
10 A 30 D 50 D 70 D 90 C
11 C 31 D 51 D 71 A 91 C
12 B 32 B 52 C 72 A 92 A
13 B 33 C 53 A 73 B 93 C
14 C 34 A 54 C 74 C 94 A
15 A 35 D 55 A 75 C 95 B
16 B 36 C 56 B 76 D 96 A
17 B 37 A 57 C 77 D 97 C
18 C 38 A 58 A 78 C 98 B
19 D 39 A 59 D 79 A 99 C
20 C 40 B 60 A 80 B 100 B

PAGE 1073
CDS I 2017
www.byjusexamprep.com

(Solutions)
ENGLISH
1. Ans. B.
Neglect means not to give proper attention.
Deny means to declare untrue.
Refuse means to show unwillingness.
Disallow means to refuse to allow.
The sentence talks about giving training to someone which was not done previously or in other words
we can say which was neglected previously. Thus, the correct verb here would be "neglected".
2. Ans. D.
Commended, commended means appreciated. Collaborated, commanded and contrasted are out of
context. Collaborated means to work jointly with others. Commanded mean to direct authoritatively.
Contrasted means compared or appraised in respect to differences.
3. Ans. C.
Option C is the correct choice. As with verb 'knew', 'how to swim' is appropriate.
4. Ans. C.
Let's understand the meanings of the given words:
Draw = to consider the facts of a situation and make a decision about what is true, correct, likely to
happen, etc
Slip = go or move quietly or quickly, without attracting notice; lose one's footing and slide
unintentionally for a short distance.
Percolate = filter gradually through a porous surface or substance.
Creep = move slowly and carefully in order to avoid being heard or noticed.
The sentence talks about the growth going to the lowest level. Thus, the verb "percolate" would best
fit here.
5. Ans. D.
Let's understand the meanings of the given words:
rehearse = practise (a play, piece of music, or other work) for later public performance.
abuse = treat with cruelty or violence, especially regularly or repeatedly.

PAGE 1074
www.byjusexamprep.com

accuse = charge (someone) with an offence or crime.


reprimand = to express to someone your strong official disapproval of them:
The words like "rehearse, abuse and accuse" cannot fit in the context of the sentence. So, the verb
"reprimand" is the apt choice.
6. Ans. D.
Confer means grant (a title, degree, benefit). Present means a gift. Execute means to carry out fully.
Administer In legal matters, means to direct the taking of an oath.
thus, option D is the correct answer.
7. Ans. D.
Unreliable is used for someone we cannot count on or depend on.
Unjust means characterized by injustice.
Unbearable means not bearable.
Inefficient means someone who cannot bring the desired result.
The latter part of the sentence describes the person as someone who cannot be counted on. This
gives us a hint that the word "unreliable" is the correct answer.
8. Ans. D.
Simple present is used in type 1 conditional sentence. In these sentences, the if clause is in the simple
present, and the main clause is in the simple future. Hence, option D is the correct answer.
9. Ans. C.
For “Pay for” means give money to someone for goods received. So, it is the phrasal verb which will
make the sentence grammatically correct, as the sentence talks about paying money for buying
certain articles. Pay out means the act of receiving money or material gain as a compensation. Pay
off means to pay a debt. Pay of has no meaning.
10. Ans. A.
Precedent means prior arrangement; no other option is suitable. Opportunity means a favorable
juncture of circumstances. Chance is a synonym for opportunity, it also means something that
happens without intension. possibility is the condition of being possible.
11. Ans. C.
Men were attracted to childish things. Tolstoy believed it was only of unique dimensions which can
only be attractive to children.
12. Ans. A.
Tolstoy believed “Tobacco clouded his intellect and made him built castles in the air”, hence option-
a follows.
13. Ans. C.
1 and 2 only. Tolstoy believes that Eiffel Tower was built by man under the influence of tobacco,
hence 1 follows. “There is no art about the Eiffel Tower” Tolstoy says, it was only of unique

PAGE 1075
www.byjusexamprep.com

dimensions which can only be attractive to children, hence 2 follows. 3 shows the childish quality of
man and not any quality of Eiffel Tower.
14. Ans. A.
The author regrets that fields and forests have been replaced by city buildings. Because
industrialization has led to more pollution and he regrets this.
15. Ans. D.
Greater fuel consumption and pollution. The tone of the author is negative and critical, he does not
support industrial development.
16. Ans. B.
Deforestation. The author writes, “whole areas of forests being cleared to gain new land”, this means
cutting down of trees which is known as deforestation.
17. Ans. B.
The author believes that unplanned growth of industry had done more harm than good. Expressions
like, “concrete jungle”, “fumes which belch through the chimney” and “contamination of
atmosphere” confirm the belief.
18. Ans. A.
Only one is correct, which can be inferred from the line, “The sun, a mere fringe over the horizon,
immediately chased away the grey half-darkness.” The lines describe day break. Moreover, Yuri puts
the snake in a box, therefore second assumption is incorrect.
19. Ans. A.
Calm and courageous, he had caught the snake and was not afraid of its fangs showing that Yuri was
courageous, and only with calm did he wait for the snake to lose the battle.
20. Ans. D.
Absolutely shocked, ‘stuck dumb’ means that the author was so shocked that he could not say a
word.
21. Ans. D.
The author was nervous.
Option D is the correct answer.
22. Ans. B.
Can be inferred from the line, “I thought I saw a hand gripping the bars of my window”.
23. Ans. C.
The narrator says “quite unusually, sleep eluded me” negating option-a. He says he was lying down
and also the compartment was lonely, therefore option-b and option-d also do not follow. Only
option-c follows.
24. Ans. A.
Very strong, overwhelming means overpowering, very strong is the only option that is most similar.

PAGE 1076
www.byjusexamprep.com

25. Ans. A.
Writer states that while travelling on a motorbike he is in the scene; therefore, he is talking about
his own presence, his own self as part of the scene.
26. Ans. C.
The correct answer is option C 'Seeing the scenery through the window frame'
27. Ans. C.
It can be inferred from the line where the writer mentions, “You are a passive observer and it is all
moving by you boringly in a frame.”
28. Ans. D.
It can be interpreted clearly from the last line where the writer mentions’ “You are in the scene, not
just watching it anymore”.
29. Ans. D.
Refer to the last line of the passage, “he had worked about work-saving low-altitude flying”.
30. Ans. B.
Jonathan had tried to fly very high unlike other sea gulls and failed, he was tired of trying very hard.
According to context option-b follows.
31. Ans. A.
He had tried to do something that the other seagulls had not done, that is why he had learned
something new.
32. Ans. B.
‘wearily’ means ‘tiredly’. No other option match other than exhausted.
33. Ans. A.
Use of ‘is’ is incorrect, the correct usage should be ‘does not exist’.
34. Ans. C.
Accustom is followed by ‘to’. So, the correct idiomatic expression will be ‘accustom yourself to’ or
‘get accustomed to’.
35. Ans. C.
The sentence is in simple past tense. The past tense of ‘lie’ is ‘lay’. Lie mean to take a position.
36. Ans. B.
The correct sentence should be: Mahatma Gandhi is called as the father of the Nation.
Hence, option B is the correct answer.
37. Ans. B.
The main subject ‘The Chairman’ is singular, hence singular verb ‘is touring’ will be used.

PAGE 1077
www.byjusexamprep.com

38. Ans. C.
Here ‘passed each other’ means crossed each other’s path. No preposition is required by ‘passed’.
39. Ans. D.
No improvement required. Illness is a condition and not a feeling.
40. Ans. B.
‘Dispensed’ should be followed by ‘with’. Dispensed with means stop using, therefore the sentence
means that his services are no longer needed.
41. Ans. D.
The given sentence is in the simple past and no improvement is required.
42. Ans. B.
The event of living in Chennai is over long back, hence the sentence is in past perfect tense. ‘Had
lived’ will be the correct expression.
43. Ans. B.
‘Abide’ means ‘to put up with’. ‘abide’ is always followed by the preposition ‘by’.
44. Ans. A.
‘Looking for’ means ‘trying to find’, hence it’s the correct use here. Looking about means looking
here and there. Looking after means taking care of something or someone. Looking into means
looking into the depths of something.
45. Ans. A.
‘ought’ represents advice, ‘would’ represents strong possibility of an event here. ‘would have’ will
be the correct grammatical expression here. The sentence expresses that the accused was not sent
to jail because of one witness.
46. Ans. A.
As the task was completed by 8 am the sentence is in past perfect tense, ‘had written’ will be the
correct use.
47. Ans. D.
No improvement required. Grant means to permit as a favor. As the interview will be a favor the use
of Grant is correct.
48. Ans. C.
Two complete events occurred, rescue happened first hence it will be expressed in past perfect
tense, ‘had rescued’ will be the correct usage.
49. Ans. D.
The given sentence requires no improvement.
50. Ans. A.
The subject ‘The teacher’ is singular, hence ‘was taken’ will be the correct expression. Since the
teacher was herself taken to the hospital so b and c are incorrect.

PAGE 1078
www.byjusexamprep.com

51. Ans. B.
From is used to denote a point of time. The preposition ‘for’ should be used here as five days is a
period of time and the event started at the beginning of those five days.
52. Ans. D.
No improvement is required. Loan is a temporary provision of money, usage is correct. Debt is an
obligation. Demand is a claim to something owed. Advance is a supply in expectation of repayment.
53. Ans. D.
No improvement is required. The verb cope means to deal with. Cope when followed by the
preposition with, means to deal with some difficulty. Also, the sentence is in the simple present
tense. The sentence is correct.
54. Ans. B.
The use of will is redundant. The main verb is ‘Go’, till you reach is used to denote time. Further, the
event will be completed in future but both other options are in past tense.
55. Ans. A.
RQSP, S1 is a statement which is contradicted in R starting with a ‘But’, hence R follows S1. Q supports
the argument in R with an example. S draws conclusion from the example. P is directly connected to
S6 because ‘He’ mentioned in s6 is ‘The wise man’ mentioned in P.
56. Ans. B.
Option B - QPSR, is the correct sequence for the given sentences.
57. Ans. D.
RPQS, exams are taken after teaching, so R follows S1. P follows R because it mentions the end of
exam. Q follows P because assessment will be done after exam is over. And S is connected to S6,
connected by span of time.
58. Ans. B.
The correct sequence of the sentences is Q S P R (Option C)
59. Ans. B.
RSQP, R further explains S1 by mentioning that some fears are necessary, hence R follows S1. S
follows R by giving an example and supporting the proposition given in R. As we are afraid of a rapidly
coming automobile, as mentioned in S, we wait as mentioned in Q. therefore; Q follows S. P follows
Q as P states that whatever is written in Q is sensible.
60. Ans. B.
QPSR, Q tells us more about the library mentioned in S1 hence becomes the first sentence. Q also
mentions books for the first time and is followed by P, P expresses the source of these books. S states
a fact and is followed by R as R draws conclusion from S.
61. Ans. D.
QPSR, Q further describes the houses mentioned in S1 hence becomes the first sentence. Q describes
a staircase which is continued to be described in P. So, P follows Q. S goes on further describing other

PAGE 1079
www.byjusexamprep.com

aspects of the house mentioning excellent drainage systems. In R ‘The drains’ are further explained
hence R follows S.
62. Ans. C.
SQPR. S is the first sentence of the sequence. S1 states some ceaseless efforts, and a simple effort is
mentioned in S. The use of solar thermal conversion is reflected in Q as it mentions a large number
of applications. Some of these applications are mentioned In P. R mentions about the other efforts
being made which is further explained in S6 making R the last sentence in the sequence.
63. Ans. B.
SRQP, from chronological point of view S becomes the first line as reflected by the words, ‘Right from
their school days’. R takes the idea of the dream further by mentioning that it was an ‘honor’. Q
explains why this is an honor, because the best one’s lead, the idea in Q is further supported by P as
it mentions the pride of parents. P is the last line as it connects to S6 through parents and emotions.
64. Ans. A.
RQPS, R talks about ‘this fallacy’, the fallacy is the feeling mentioned in S1 hence R follows S1, R states
that the fallacy can be disproved, and Q clarifies how this can be done. Q states studying some score
and P reflects the observation of the study. So, R is followed by Q and Q is followed be P. S draws
conclusion from the observation made in P. So, S follows P becoming the last sentence.
65. Ans. C.
PRSQ, here P shows the objects of experiment and hence becomes the first line. R is directly
connected to P as it talks about the same two plants. “But” in S indicates crucial details about the
experiment, hence following R. S is connected to Q as both talk about music, Q further describes the
music mentioned in S.
66. Ans. D.
Here the sentences are to be arranged in chronological manner, i.e. the sentences are arranged in
the sequence in which they happened. So, P becomes the first part as it indicates the first immediate
response. Aster replying to the office’ message, the author rushes to the bank, indicated in R by
‘then’. On the way, the author takes care of other tasks as mentioned in Q. S takes further the ideas
mentioned in Q, stating ‘from the shop next to it’ here it means the bank. Thus, the correct sequence
is PRQS.
67. Ans. C.
R shows the first symptoms of pink eye and hence follows S1. The world ‘also’ in Q shows that R will
be followed by Q as it further describes the symptoms. Q indicates a white discharge which is further
explained in S, it can be identified by the words ‘this discharge’. P comes last as it is also related to
S6 related by the terms ‘school-children in P and ‘students’ in S6. Thus, the correct sequence is RQSP.
68. Ans. C.
PQSR, P becomes the first sentence because it describes how fire was discovered shedding light on
the idea mentioned in S1. From discovery the idea shifted towards a comparison of making fire now
and then. The idea of present use begins in Q indicated by ‘now’, saying how easy it is for us to make
fire now. S follows indication that matches are very recent and the situation was completely different
in the past. R describes the olden times, hence becomes the last sentence.

PAGE 1080
www.byjusexamprep.com

69. Ans. C.
S1 indicates a series of misconceptions. P mentions the most common misconception; hence it
becomes the first sentence to follow S1. P states that all snakes being poisonous is a misconception
and sentence S takes the idea further as it states the number of such non-poisonous and poisonous
snakes. Q introduces a fresh idea denoted by ‘Another’, so Q follows S. R clarifies the misconception
stated in Q, hence R follows Q and becomes the last sentence to be arranged before S6. Thus, PSQR
is the correct sequence.
70. Ans. D.
SRQP, only S connects with S1, as both the sentences talks about fire and flames. Also, S states the
reaction of the people on seeing the blazing rice. R takes the idea forward clarifying who the people
included on the spot. The young men and boys were followed by active women and girls as
mentioned in Q. Lastly the older people came as mentioned in P. The sentences that follow one after
the other show the sequence of people arriving to fight the fire.
71. Ans. D.
PSRQ, the sound mentioned in S1 inspired a thought which is mentioned in P making P the first line.
As a thought is inspired as stated in P, author rushes to confirm it, as stated in S. S follows P as the
author rushed out to confirm his thought. R shows what the author saw, ‘the policemen’ Q states
what ‘they’ were doing, so R is followed by Q.
72. Ans. B.
RSQP, S1 states that Chandran was angry, R relates to S as ‘R’ shows the reason for Chandran’s anger,
so R is the first sentence. Since his pen was missing he thought that it must have been lost, which is
stated in S, so S follows R. After he was convinced that the pen was lost a turn of events took place
which is denoted by ‘Just’ in Q. So, Q follows S. His sisters name Saro is mentioned in Q and the same
person is indicated by ‘she’ in P. So, P become the last sentence after Q.
73. Ans. D.
QSRP, S1 starts by saying that Ramu was in a great hurry. So, the first line following S1 will be ‘Q’ as
it shows the reason for Ramu’s hurry. Further Line ‘S’ shows the consequence of his hurry, that is he
forgets to look left or right as he always did. ‘R’ follows explaining the way he crossed without looking
left or right as mentioned in S, So, R follows S. R shows careless ness which is clearly indicated in O,
so P comes last following R.
74. Ans. C.
QRSP, only Q starts with an indefinite article, all other sentences start with a definite article or a
conjunction, so Q can become the first line. ‘A little girl went fishing’ is the beginning of a legend
which is indicated in S1, so Q follows S. R states what happen when the little girl went fishing, hence
follows Q. Instead of catching fish the girl caught the king of frogs, what the king frog said on being
caught is mentioned in S, so S follows R. The thing that happened when the girl did as the king said
has been indicated in P making it the last sentence.
75. Ans. C.
The error is in part (3) of the sentence. Every, each, either, neither are determiners that are always
followed by a singular pronoun. Therefore ‘their ideas’ should be replaced with’ his/her idea’.

PAGE 1081
www.byjusexamprep.com

76. Ans. B.
Error is in second part. Replace ‘have’ with ‘had’. Since two events are mentioned and both occurred
in the past, the even that happened first will be expressed in past perfect tense. Hence ‘had
conquered’ is the correct form.
77. Ans. B.
Error is in second part. Replace ‘has’ with ‘have’. Since ‘parents’ is in plural form it will be followed
by the plural form of has, i.e. ‘have’.
78. Ans. B.
The error is in part B of the given sentence.
Use 'swept' in place of 'sweeped'.
79. Ans. D.
The given sentence is error free.
80. Ans. B.
Error is in second part. Replace ‘grave’ with ‘graver’. Since problems are being compared in the given
sentence we will use the comparative form of ‘grave’ that is ‘graver’.
81. Ans. A.
The error is in first part. Replace ‘whom’ with ‘who’. ‘Whom’ is a pronoun which is the objective case
of the pronoun ‘who’. But who is the subject in the given sentence, and hence usage of ‘whom’ is
incorrect here.
82. Ans. D.
The given sentence is grammatically correct and there is no error.
83. Ans. C.
The error is in third part. The adjective hard has two different adverbs, and they express opposite
meanings. The adverb hard is used to modify a verb and means that the action is being done very
intensely. The adverb hardly is used to modify a verb and means that the action is not being done
very intensely. So there the correct usage will be ‘working hard’.
84. Ans. D.
The given sentence is grammatically correct and there is no error. ‘Memoranda’ is a plural noun, so
it is correctly followed by are. Memorandum is the singular form.
85. Ans. B.
The error is in second part. ‘where was he going’ is a question in itself but the given sentence is
assertive. So, in the correct form the verb ‘was’ will follow the pronoun ‘he’ as, ‘where he was going’.
86. Ans. D.
The sentence is grammatically correct.
87. Ans. D.
The given sentence is grammatically correct and there is no error.

PAGE 1082
www.byjusexamprep.com

88. Ans. C.
‘prefer’ is a verb that is always followed by the preposition ‘to’ expressing choice or preference over
something else. In this case choice of ‘this book’ over the ‘other’ one. ‘Than’ is used for comparing
to options, ‘prefer to’ expresses choice. Correct expression will be ‘to that one’.
89. Ans. D.
The given sentence is grammatically correct and there is no error.
90. Ans. A.
The error is in first part. ‘One of the’ should always be followed by a plural noun, expressing that the
speaker chooses one from many. Hence ‘features of travel’ is the correct expression, indicating that
there are many interesting features about Himachal Pradesh but the most interesting one among
them is the provision of large number of travelers’ lodges.
91. Ans. C.
The error is in third part. By using ‘not’ after ‘unless’, both the negatives cancel each other. So, the
correct usage will be ‘unless he reports for duty immediately’. Since “unless” is negative it should be
followed by affirmation, like “unless he reports for duty”. Incase “unless” was absent, we could have
written “if he does not report” and expressed the same meaning.
92. Ans. A.
The error is in first part. ‘Considering’ is a preposition that relates the facts to the principal and hence
does not need to be followed by another preposition ‘about’. A preposition like ‘considering’ should
be followed by a pronoun like ‘these’ in this case. The use of about is unnecessary and shall be
omitted. The sentence indicates that the principal offered him a seat only after considering certain
facts.
93. Ans. C.
Error is in third part. Here, “there” is an adverb that qualifies the verb ‘is’. The position of ‘there’ in
the sentence is incorrect. The correct usage should be ‘there is no trouble’. It indicates the absence
of any trouble in a country.
94. Ans. B.
Error is in second part. Returning means coming back to place from where you began. So, the use of
back after returned becomes redundant. Without using back, the meaning remains the same. We
can omit ‘back’ in part b, and use ‘returned home.’
95. Ans. C.
Error is in third part. Since “doing” was used in second part it should be followed by “doing” in third
part as well instead of using “done”. Such repetitive figure of speech is used in English grammar to
create emphasis. This sentence indicates that a thing that is worth doing is also worthy of doing well.
Here worth is the noun that must be followed by the verb doing.
96. Ans. B.
Option B has the erroneous part of the sentence. The use of passive form "he was left" is incorrect.
Here two events occur one after the other. ‘He left’ before ‘we reached’. So, the event that was
completed before the occurrence of the next event shall be expressed in past perfect tense.
Therefore, the correct form of verb will be “had left”.

PAGE 1083
www.byjusexamprep.com

97. Ans. C.
Error is in third part. By writing ‘who’ in the second part it has already been made clear that the
sentence talks about ‘a man’. The use of ‘he’ becomes redundant or repetitive. By omitting ‘he’ the
sentence becomes grammatically correct.
98. Ans. A.
The error is in first part. If we start the sentence with being, it means that we were rainy which is
incorrect. The sentence indicates that the day was Rainy, and the day will be denoted by ‘It’. So ‘It
being a rainy day’ will be the correct idiomatic expression. Otherwise the meaning of the sentence
changes.
99. Ans. C.
The error is in part C.
The correct usage 'angry with me'.
Hence, option C is the correct answer.
100. Ans. D.
The sentence is error-free.
Option D is the correct answer.
101. Ans. C.
PRSQ, the sentence starts with P which mentions the idea behind the declaration made in R, making
R the second part of the sentence. S states what the declaration was, that is ‘the International Year
of the child’, hence S follows R. Finally, Q states that the declaration was made by ‘the United Nations
Organization’, hence Q concludes the sentence.
102. Ans. A.
RPQS. R points to people who object he R is the first part, which is followed by P which states that
such people do not realize. Realize is related to ‘that’ found in Q. P is followed by Q. And S relates to
Q as it states that modern society is built around the automobile.
103. Ans. C.
PQSR, except P all other parts begin with ‘that’, ‘but’, and ‘which’, they cannot form the beginning of
a sentence, so the sentence begins with P.P is connected to Q as Q states what the school masters
believe. S begins with ‘which’ pointing to the quality mentioned in Q. So, S follows Q. R starts with
the conjunction ‘but’ forming the second part of the sentence stating that we can only measure the
quality of innate intelligence.
104. Ans. A.
SPRQ, the parts are arranged in correct order and need no rearrangement.
105. Ans. C.
PSRQ, P and S form a connection after the fixed part as they denote the place of friendship in life.
The sentence is further continued by a conjunction ‘but’ in R. Which signifies that friendship is not
everything and it cannot override truth and justice. So, ‘R’ is followed the final part Q. The sentence
ends with Q.

PAGE 1084
www.byjusexamprep.com

106. Ans. B.
PSQR, the sentence is about sports cars, so it starts with the subject ‘Sports cars’ followed by S which
describes their quality ‘with noisy exhausts’. Such cars are appealing as stated in Q. Finally, R states
who find such cars appealing so R comes at the end.
107. Ans. B.
RSPQ, R relates to the attempts of transplant that have been made ‘such as the lungs or the liver’
and hence follows the fixed first part. S states have been made pointing to R. Here the first part of
the sentence is complete, and the second part is joined by the conjunction ‘but’ found in part P.
Hence P follows S. P and Q are connected, ‘which’ mentioned in P has been clarified as ‘heart
transplant’ in Q.
108. Ans. A.
RPSQ, the time of any incident should be mentioned first and hence we start with R. After stating
when the event took place, it should be stated what happened in 1940s which is stated in P. The
allergic reactions stated in P are brought about by the liberation of small quantities of a substance
as stated in S. So, S follows P. Q comes last clarifying that the histamine is liberated in the blood
stream.
109. Ans. C.
SRQP, R may look like the first part but then S fits nowhere. By starting with S we see R easily follows.
S becomes the first part because it tells us which type of people the sentence is talking about. In part
R it is mentioned how the people with hearing problems suffer, so r follows S. Q follows R because it
states where the noises are experienced, P comes last stating that these noises are inaudible to
others.
110. Ans. A.
PQSR, is the correct sequence for the given jumbled sentences.
111. Ans. D.
SQRP, Part S points to ‘who’ which is mentioned in the beginning, therefore S becomes the first part
of the sentence. S is followed by the main verb ‘is visiting’ in part Q, describing what the lady will be
doing. She will be visiting ‘us’ which is part R, hence R follows Q. P states when the lady is visiting
making it the last part.
112. Ans. C.
The subject man will be followed by the verb ‘looked at me’ found in Q making it the first part. P
clarifies that the man looked at ‘me’ for a moment, so P follows Q. S shows the second action that
followed after looking that is tuning ‘me’ upside down. So, P is followed by S. The last action that is
emptying the pockets after turning upside down is mentioned in part R making it the last part of the
sentence.
113. Ans. C.
SRPQ, Q has the subject ‘he’ but it cannot be the first part because it also has the pronoun ‘its’ which
must have been mentioned before. S becomes the first part of the sentence because it explains the
reason for his (the subject’s) ignorance. The part R mentions what he is unaccustomed to which is

PAGE 1085
www.byjusexamprep.com

further explained in part P, so a life of rules and regulations comes together. Finally followed by Q
because it describes what he does as a consequence of being unaccustomed.
114. Ans. D.
P is the first part introducing the subject “he”, P also says that he was a tiny man, S follows P because
S describes why he is being called tiny, because he is barely five feet tall, tall being part R follows S.
After describing his height, part Q comes stating that he also had a sprightly walk.
Thus, PSRQ is the correct sequence.
115. Ans. B.
PSRQ, the subject in P “The house” is followed by its description in S as “gloomy” because the last
word in P is “was”. Then R begins to talk about the reason for gloominess, it becomes in part Q that
the liveliest member was away, that being the reason for gloominess. So, Q is the last part.
116. Ans. C.
SPQR, the subject of the sentence, the traveler, is already fixed correctly at the beginning. It should
have been followed by the verb ‘Sat down’ mentioned in P, but part S describes the reason for sitting
down so S comes first and it will be followed by P. Q shows the purpose of sitting down, so Q comes
after P. R only describes where the subject sat and comes at the end.
117. Ans. C.
PRQS, the given sentence is already arranged in correct sequence.
118. Ans. D.
Q introduces the subject, that is smallpox, and mentions that it was a dreaded disease. Q is followed
by P which states till when the disease was dreaded. P also states a reason that begins with ‘not only’,
‘not only’ is always followed by but also which is found in R, therefore P is followed by R. Finally, S
describes the real reason of the dread that is permanent disfiguring which becomes the last part.
Thus, QPRS is the correct sequence.
119. Ans. D.
PSRQ, the subject ‘rice’ appeared in part P making it the first part. Here rice is being compared to
other ‘food crops’ which is mentioned in S. Hence S follows P. R states why rice is prolific and comes
after S. Q concludes the reason by comparing the yield of rice with other crops and comes last.
120. Ans. B.
PRQS, the sentence starts with the pronoun ‘he’ because it is the subject. The Subject should be
followed by the action or verb which is present in part R ‘planned the entire strategy…’ S describes
how the work was done, i.e. ‘single-handedly’, but Q qualifies ‘single-handed’, therefore Q comes
before S.

PAGE 1086
www.byjusexamprep.com

General Knowledge
1. Ans. A.
Supreme Court inquires and decides in case of doubts and disputes arising out of elections of
President and Vice president.
Hence, option A is correct.
2. Ans. A.
The Vice-President is elected by an electoral college consisting of both elected and nominated
members of both Houses of Parliament, in accordance with the system of proportional
representation by means of the single transferable vote and the voting in such election is by secret
ballot. The Vice-President is not a member of either House of Parliament or of a House of a
Legislature of any state. Hence, option A is correct.
3. Ans. D.
• The Directive Principles of State Policy are enumerated in Part IV of the Constitution from Articles
36 to 51.
• The Directive Principles are non-justiciable in nature, that is, they are not legally enforceable by
the courts for their violation. So, the Governments cannot be compelled to implement them.
• Although the Constitution in Article 37 itself says that these principles are fundamental in the
governance of the country and it shall be the duty of the State to apply these principles in making
laws.
• These principles reflect the ideology of socialism, Gandhian and liberalism.
Hence, option D is correct.
4. Ans. C.
The Indian constitution guarantees Cultural and Educational Rights under articles 29 and 30. Under
article 29, this article seeks to protect the interests of the minority communities. This article confers
the freedom to all citizens, residing in different parts of the land, to conserve their distinct languages,
scripts or cultures state shall not impose upon it any culture other than the community’s own culture.
This article further assures that no citizen shall be denied admission into any state-run or state-aided
educational institution on grounds only of religion, race, caste, language or any of them. And Under
article 30, It provides that all minority communities—religion or linguistic, have the right to establish
and administer educational institutions of their choice. In granting aid to educational institutions, the
state shall not discriminate on the grounds of religion or language.
Hence, option C is correct.
5. Ans. A.
All are correct. Preamble is non-justiciable in nature i.e. its provisions are not enforceable in court of
law. Preamble is neither a source of power to legislature nor a prohibition upon the powers of
legislature.
Hence, option A is correct.

PAGE 1087
www.byjusexamprep.com

6. Ans. B.
The Ryotwari system was introduced by Sir Thomas Munro and Captain Alexander Read in 1820.This
system was in operation for nearly 30 years and had many features of revenue system of the
Mughals, Under the Ryotwari System every registered holder of land is recognised as its proprietor
and pays direct to Government.
Hence, option B is correct.
7. Ans. D.
The Justice Party was a political party in the Madras Presidency of British India. It was founded by T.
M. Nair and P. Theagaraya Chetty in 1916. Communal division between Brahmins and non-Brahmins
began in the presidency during the late-19th and early-20th century, mainly due to caste prejudices
and disproportionate Brahminical representation in government jobs. The Justice Party's foundation
marked the culmination of several efforts to establish an organisation to represent the non-Brahmins
in Madras. Founders of Justice Party though that Congress was mainly Brahmin party, so it started
opposing Congress.
Hence, option D is correct.
8. Ans. D.
• The All India Depressed Classes Association was founded in Nagpur in the year 1932, with M. C.
Rajah as its first President.
• The All India Depressed Classes Association opposed Ambedkar's demand for a separate
electorate for the poor.
Hence, option D is correct.
9. Ans. C.
The Poona Pact was an agreement between Dr. Babasaheb Ambedkar and Mahatma Gandhi which
was signed on 24 September 1932 at Yerwada Central Jail in Pune, India, which asserted that there
will be a single Hindu electorate, with scheduled castes having seats reserved within it, So the
representation of these classes was based on the standards of joint electorates and reserved seats.
The Communal Award was made by the British Prime Minister Ramsay MacDonald on 16 August
1932 which allowed separate electorates in India, and it was supported by Ambedkar.
Hence, option C is correct.
10. Ans. B.
The Coercive Acts included a series of laws passed by the British Parliament in 1774, including Boston
Port Act, the Massachusetts Government Act, the Administration of Justice Act, and the Quartering
Act of 1774, which were meant to punish American colonies for throwing a large tea shipment into
Boston Harbour, so, the passing of the Coercive Acts made reconciliation between Britain and her
American colonies virtually impossible . The Stamp Act of 1765 was passed by Parliament of Great
Britain to impose a direct tax on the colonies of British America. It was later repealed on March 18,
1766 after protest.
Hence, option B is correct.
11. Ans. B.
HtinKyaw is the president of Republic of Union of Myanmar.
Hence, option B is correct.

PAGE 1088
www.byjusexamprep.com

12. Ans. C.
The 2020 Summer Olympics will be held in the Japanese capital city of Tokyo. Tokyo previously
hosted the Games in 1964, and in 2020 will become the fifth city (and the first Asian city) to host the
Summer Games more than once.
Hence, option C is correct.
13. Ans. D.
Brexit is the withdrawal of the United Kingdom from the European Union. In a referendum on 23
June 2016, 51.9% of the participating UK electorate voted to leave the EU, out of a turnout of 72.2%.
Hence, option D is correct.
14. Ans. A.
• In June 2016, the Government of India's Department of Industrial Policy & Promotion (DIPP),
Ministry of Commerce & Industry, issued a new foreign direct investment (FDI) policy.
• The changes to the Foreign Direct Investment Policy have no effect on multi-brand retailing.
Hence, option A is correct.
15. Ans. C.
The Indian Navy commissioned the highly manoeuvrable fast attack craft INS Tihayu at the Eastern
Naval Command on 19 October 2016. The ship was commissioned at Visakhapatnam under the Naval
Officer-in-Charge (Andhra Pradesh) and would be deployed for coastal patrol and surveillance
operations along the Eastern Coast.
Hence, option C is correct.
16. Ans. C.
Maintenance Command is a command of the Indian Air Force. It was raised as Maintenance Group
at Chakeri in Kanpur in 1950. In 1955, it was designated as Maintenance Command.
Hence, option C is correct.
17. Ans. B.
Rank of captain of Indian Navy is equivalent to Group captain of Indian Air Force.
Hence, option B is correct.
18. Ans. B.
Amartya Sen, Indian economist who was awarded the 1998 Nobel Prize in Economic Sciences for his
contributions to welfare economics and social choice theory and for his interest in the problems of
society's poorest members.
Hence, option B is correct.
19. Ans. A.
GST is a destination-based tax, it means end users consuming any goods or services is liable to pay
the Goods and Services Tax. The tax is collected by the State in which the goods or services are
consumed and not by the state in which goods are manufactured and If there are exports, the seller
of the goods or services is exempted from paying the tax. GST is a consumption-based tax, it means

PAGE 1089
www.byjusexamprep.com

the state where the goods were consumed will receive GST and where goods were sold should not
get any taxes. GST is a value added tax as it is based on the increase in value of a product or service
at each stage of production or distribution. GST is not gross value tax.
Hence, option A is correct.
20. Ans. A.
In Most-favoured-nation countries cannot normally discriminate between their trading partners.
Hence, option A is correct.
21. Ans. C.
The Wholesale Price Index (WPI) was main index for measurement of inflation in India till April 2014
when RBI adopted new Consumer Price Index (CPI) as the key measure of inflation.
Hence, option C is correct.
22. Ans. A.
Copyright & Trade Secret are two types of Intellectual property which do not require the registration.
However, if their is any infringement of the copyright then the owner is liable to prove that it belongs
to his/her.
Hence, option A is correct.
23. Ans. C.
Every person who is arrested and detained in custody shall be produced before the nearest
magistrate within a period of twenty-four hours of such arrest excluding the time necessary for the
journey from the place of arrest to the court of the magistrate and no such person shall be detained
in custody beyond the said period without the authority of a magistrate.
Hence, option C is correct.
24. Ans. A.
All are correct except A, as every religious denomination including minor communities shall have
right to manage its own affairs.
Hence, option A is correct.
25. Ans. A.
All are correct.
Hence, option A is correct.
26. Ans. B.
All are correct except 3, as women always have the right to make reproductive choices.
Hence, option B is correct.
27. Ans. C.
Article 53: The Union's executive power.
• The Union's executive power shall be vested in the President, who shall exercise it directly or
through officer’s subordinate to him in accordance with this Constitution.

PAGE 1090
www.byjusexamprep.com

• Without limiting the generality of the preceding provision, the President shall have supreme
command of the Union's Defence Forces, and the exercise of that command shall be governed
by law.
Hence, option C is correct.
28. Ans. D.
In 2016, India and Mozambique entered a new deal by signing three Memoranda of Understanding
(MoU) in fields related to drug trafficking, pulse trading and sports. These MoUs will mutually benefit
the current issues faced by both countries. Mozambique will be focusing on the production of tur dal
(Pigeon Peas) to facilitate the long-term trade goals with India.
Hence, option D is correct.
29. Ans. C.
BRICS summit 2016 was held in Goa, India.
The 8th BRICS summit was held from 15 to 16 October 2016 at the Taj Exotica hotel in Benaulim,
Goa, India. The logo represents the map of Goa with the BRICS participants national flags (Brazil,
Russia, India, China, South Africa). Hence, option C is correct.
30. Ans. C.
Saakshar Bharat scheme was launched on 8th September 2009 during the 11th Five Year Plan, with
an aim to raise literacy rate to 80%, to bridge gender gap to 10% .The scheme is anchored with
Panchayati Raj Institutions and Local Self-government to decrease regional and social disparities and
to focus on women, SCs, STs, minorities and other disadvantaged groups. It focused mainly on adult
education, with emphasis on non-literates in the age group of 15-36 years.
Hence, option C is correct.
31. Ans. B.
• All of the options mentioned above were viewed as challenges to inclusiveness.
• Inclusiveness is the ability to include a wide range of people and treat them all fairly & equally.
Hence, option B is correct.
32. Ans. A.
Alexander Fyodorovich Kerensky was a Russian lawyer and revolutionary who was a key political
figure in the Russian Revolution of 1917. After the February Revolution of 1917 he joined the newly
formed Russian Provisional Government.
Hence, option A is correct.
33. Ans. C.
At second continental congress in 1775, George Washington was made commander in chief of
American forces.
Hence, option C is correct.

PAGE 1091
www.byjusexamprep.com

34. Ans. A.
The Industrial Revolution was the transition to new manufacturing processes in the period from
about 1760 to sometime between 1820 and 1840. Average life span of worker was 17 years.
Hence, option A is correct.
35. Ans. C.
Kepler’s laws of planetary motion state that (a) All planets move in elliptical orbits with the Sun at
one of the focal points (b) The radius vector drawn from the sun to a planet sweeps out equal areas
in equal time intervals. This follows from the fact that the force of gravitation on the planet is central
and hence angular momentum is conserved. (c) The square of the orbital period of a planet is
proportional to the cube of the semi major axis of the elliptical orbit of the planet.
Hence, option C is correct.
36. Ans. C.
Tarabai Shinde is the author. Hence, option C is correct.
37. Ans. A.
Arya Samaj was an Indian religious movement based on Vedas and was founded by Dayananda
Saraswati on April 7, 1875 and was based in Punjab and western Uttar Pradesh states like, Aligarh,
Varanasi, etc. It was against caste system and supported equality of men and women, moreover it
mainly comprised the trading castes; while Brahmo Samaj, founded by Raja Rammohan Roy, was
limited because it was popular only among elite and educated society.
Hence, option A is correct.
38. Ans. C.
• The Indian National Congress (INC) gained control of eight provinces, with the exceptions of
Bengal, Punjab, and Sindh.
• The All-India Muslim League was unable to form a government in any of the provinces.
• Provincial elections in British India (1937) under the Government of India Act 1935 were held in
eleven provinces, including Madras, Bihar, Orissa.
Hence, option C is correct.
39. Ans. B.
• In January 1931, Gandhi Ji was released from prison and began talks with Lord Irwin to end the
satyagraha campaign. Following that, a truce was declared, which was formalised in the Gandhi-
Irwin Pact, which was signed on March 5.
• It was the first nationalist activity in which a large number of women took part.
• The Salt March received extensive coverage in the American & European press.
Hence, option B is correct.
40. Ans. A.
Rowlatt act in 1919, Salt March in 1930, Chauri Chaura in 1922, Second Round Table Conference in
1931.
Hence, option A is correct.

PAGE 1092
www.byjusexamprep.com

41. Ans. B.
Headquarter are located in Kochi.
Hence, option B is correct.
42. Ans. A.
Peacetime Gallantry Awards recognize courage or self-sacrifice away from battlefield and include
Ashok Chakra, Kirti Chakra and Shaurya Chakra which is first, second and third respectively in order
of precedence ; while Param Vir Chakra and Vir Chakra are Wartime Gallantry Awards and recognizes
self-sacrifice at battlefield. Yudh Seva Medal is military decorations for service during wartime.
Hence, option A is correct.
43. Ans. A.
B, C, and D are inter-service establishment because one can go into different services through them
but through Officers Training Academy, one can go only into Indian Army.
Hence, option A is correct.
44. Ans. C.
In textile industry there is women majority, so it is impossible for government to create jobs equally
for men and women. Hence, option C is correct.
45. Ans. B.
• The second gravitational wave observation was made on 26th December 2015, and was
announced on 15th June 2016.
• Gravitational waves were produced by the merger of two black holes located approximately 1.4
billion light-years away.
Hence, option B is correct.
46. Ans. B.
He is Lionel Messi.
Hence, option B is correct.
47. Ans. C.
Tejas is an Indian single-seat, single-jet engine, multirole light fighter designed by the Aeronautical
Development Agency (ADA) and Hindustan Aeronautics Limited (HAL) for the Indian Air Force and
Indian Navy. It came from the Light Combat Aircraft (LCA) programme, which began in the 1980s to
replace India's ageing MiG-21 fighters. In 2003, the LCA was officially named "Tejas".
Hence, option C is correct.
48. Ans. B.
Bitcoin is the world's first cryptocurrency, a form of electronic cash. It is the first decentralized digital
currency: the system was designed to work without a central bank or single administrator. Bitcoins
are sent from user to user on the peer-to-peer bitcoin network directly, without the need for
intermediaries. These transactions are verified by network nodes through cryptography and
recorded in a public distributed ledger called a block chain. Bitcoin was invented by an unknown

PAGE 1093
www.byjusexamprep.com

person or group of people using the name Satoshi Nakamoto. Bitcoins are not recognised as a legal
tender by Reserve Bank of India.
Hence, option B is correct.
49. Ans. B.
3 is incorrect, as India is not a member of TPP.
Hence, option B is correct.
50. Ans. D.
The Central Statistics Office (CSO), Ministry of Statistics and Programme Implementation, has
released the provisional estimates of national income for the financial year 2015-16 and quarterly
estimates of Gross Domestic Product (GDP) for the fourth quarter (January-March) of 2015-16, both
at constant (2011-12) and current prices.
Hence, option D is correct.
51. Ans. B.
The Agreement on Trade-Related Aspects of Intellectual Property Rights (TRIPS) is an international
legal agreement between all the member nations of the World Trade Organization (WTO).
It sets down minimum standards for the regulation by national governments of many forms of
intellectual property (IP) as applied to nationals of other WTO member nations.
TRIPS was negotiated at the end of the Uruguay Round of the General Agreement on Tariffs and
Trade (GATT) in 1994 and is administered by the WTO. Hence, option B is correct.
52. Ans. C.
Revenue receipts are a regular source of income of the government, while Disinvestment receipts
are not a regular source of income of the government. All the other options are regular sources.
Hence, option C is correct.
53. Ans. A.
All are correct.
Hence, option A is correct.
54. Ans. B.
Besides some federal features, the Constitution of India also has some unitary features as well.
All States have no equal representation in the Rajya Sabha, because in India the states are given
representation in the Rajya Sabha on the basis of population i.e. proportional representation. Hence,
the number of representatives varies from state to state, for instance, Uttar Pradesh has 31 members
whereas Tripura has only 1 member. This principle is considered as a safeguard for smaller states.
States in India also have no right to territorial integrity. The Parliament can change the area,
boundaries or name of any state by unilateral action, so the consent of a State is not required for
altering its boundaries.
Moreover, despite dual Polity, the Constitution of India adopted the system of Single citizenship.
There is only Indian citizenship and no separate State Citizenship.
Hence, option B is correct.

PAGE 1094
www.byjusexamprep.com

55. Ans. C.
All are correct except 3, as Panchayats don’t have constitutional autonomy to work.
Hence, option C is correct.
56. Ans. B.
Only Indian who won consecutive two Gold medals was Devendra Jhajharia in Javelin throw. Hence,
option B is correct.
57. Ans. A.
Sarkaria commission- 1983, Punchhi commission-2007, Eleventh Financial commission 2000-2005,
Second Administrative Reform commission – 2005.
Hence, option A is correct.
58. Ans. A.
According to the Haryana Panchayati Raj (Amendment) Act, 2015, the following conditions must be
met in order to contest a Panchayat election:
• Have a working toilet at home.
• Have power bill payment slips on hand.
• Be a non-cooperative loan defaulter.
• For general candidates, a Class X pass is required, while a Class VIII pass is required for women.
Hence, option A is correct.
59. Ans. B.
Only 1 and 2 are correct, as Mahatma Gandhi’s speech in BHU on 4 February,1916 did not include
plight of untouchables and there was no issue of the mill owners on Ahmedabad in 1916.
Hence, option B is correct.
60. Ans. B.
All are correct except 1. The Shiromani Gurdwara Parbandhak Committee (or SGPC) is an
organization in India responsible for the management of gurdwaras, Sikh places of worship in three
states of Punjab, Haryana, and Himachal Pradesh and union territory of Chandigarh. SGPC also
administers Harmandir Sahib in Amritsar. In 1920, Sikhs held their scheduled meeting and elected a
bigger committee consisting of 175 members and named it Shiromani Gurdwara Parbandhak
Committee.
Hence, option B is correct.
61. Ans. C.
All are correct.
Hence, option C is correct.
62. Ans. D.
Swaraj movement was initiated by Mahatma Gandhi which focuses on self-rule or self-governance
and is intimately linked with Ahimsa (non-violence) and Satyagraha (adherence to truth), moreover

PAGE 1095
www.byjusexamprep.com

it referred to Gandhi’s concept Indian independence from British rule. The movement had two
meanings, political and non-political. It was political in the sense that it opposed being ruled by
foreigners, and non-political in the sense that every person has right to be ruled by himself or herself.
Swaraj is a virtue which need time and patience to achieve and cannot be obtained easily and quickly.
Hence, option D is correct.
63. Ans. C.
The Second Five-year plan was drafted under the leadership of P. C. Mahalanobis. It focused on heavy
industries and followed socialist pattern of society as its goal. Since savings and investment were
growing in this period, a bulk of these industries like electricity railways, steel, machineries and
communication could be developed in the public sector. However it had its problems as well, because
industry attracted more investment than agriculture, the possibility of food shortage became
adverse. So, the planners found balancing industry and agriculture really difficult; while K. N. Raj
drafted sections of India’s first Five-Year Plan.
Hence, option C is correct.
64. Ans. C.
Ilbert Bill was introduced in 1883 by Viceroy Ripon. It proposed to grant limited criminal jurisdiction
to native officials over European British subjects living in the districts under their charge. It also
allowed Indian judges and magistrates the jurisdiction to try British offenders in criminal cases. After
opposition from Europeans in Britain as well as India, the bill was withdrawn and reintroduced after
amendment saying that a jury of 50% Europeans was required if an Indian judge was to face a
European.
Hence, option C is correct.
65. Ans. B.
China, the European Union, Iran, Japan, South Korea, Mauritius, Myanmar, the United States of
America, and Australia are the nine SAARC Observer countries.
The First SAARC Summit was held in Kathmandu estb. in 1985. The members are: -
1. Afghanistan
2. Pakistan
3. India
4. Nepal
5. Bangladesh
6. Shri Lanka
7. Bhutan
China is only an observer.
Hence, option B is correct.

PAGE 1096
www.byjusexamprep.com

66. Ans. A.
India signed a historic deal to develop the strategic port of Chabahar in Iran and agreed on a three-
nation pact to build a transport-and-trade corridor through Afghanistan that could help halve the
time and cost of doing business with Central Asia and Europe. Developing the Chabahar port was
seen as crucial for India because it will not only allow New Delhi to bypass Pakistan and access global
markets but also counter China’s expanding influence in the Indian Ocean region. But the project’s
land route through Afghanistan remains a security headache.
Hence, option A is correct.
67. Ans. B.
In a boost to its non-proliferation credentials, India joined the Missile Technology Control Regime
(MTCR) at a ceremony in South Block. India’s MTCR membership would help in furtherance of
international non-proliferation objectives, India would enjoy full participation in organisational
activities, India had intensified efforts at gaining membership of the MTCR, the NSG, the Australia
Group and Wassenaar Arrangement since getting a waiver at the NSG in 2008. Membership of these
groups would help India trade more effectively in critical high-tech areas.
Hence, option B is correct.
68. Ans. A.
India competed at the 2016 Summer Olympics in Rio de Janeiro, Brazil, from 5 to 21 August 2016.
The Indian roster featured three Olympic medallists from London, including badminton star Saina
Nehwal, freestyle wrestler and four-time Olympian Yogeshwar Dutt, and rifle shooter Gagan Narang.
Medals Ranked 67th: Gold 0, Silver 1, Bronze 1, Total 2 Competitors: 117 in 15 sports.
Hence, option A is correct.
69. Ans. C.
In August 2016, The Government of India is seized of the reported leak of documents related to the
ongoing Indian Scorpene submarine programme as reported in sections of the media. The
documents that have been posted on the website by an Australian news agency have been examined
and do not pose any security compromise as the vital parameters have been blacked out.
Hence, option C is correct.
70. Ans. B.
Goods and Services Tax (GST) will replace all central as well as state taxes by a single tax. GST will
subsume various Central indirect taxes, including the Central excise duty, Countervailing duty,
Service tax, etc. It also subsumes State Value Added Tax (VAT), Octroi and entry tax, Luxury tax, etc.
Petroleum products like kerosene, naphtha and LPG will be under GST, while five items, including
crude oil, natural gas, aviation fuel, diesel and petrol have been excluded. Alcohol for human
consumption will be out of GST.
Hence, option B is correct.
71. Ans. D.
All are correct except D, as Mangifera indica is scientific name of Mango.
Hence, option D is correct.

PAGE 1097
www.byjusexamprep.com

72. Ans. B.
Kandla is Tidal port, Kolkata Haldia is a riverine port, Tuticorin is a river port, Marmagao is in Goa, is
situated on estuary on eastern coast of India. Delta formation and estuary formation take place
mostly on western coast and Goa is in western coast.
Hence, option B is correct.
73. Ans. D.
Burundi- Bur Franc, Libya- Libyan Dinar, Sudan- Sudanese Pound, South Africa- Rand/ZAR.
Hence, option D is correct.
74. Ans. D.
All are true except D, as most of the Sugar industries are located along the west coast ,not east coast.
Hence, option D is correct.
75. Ans. D.
All are correct except D. The Kuroshio is a north-flowing ocean current on the west side of the North
Pacific Ocean. It is similar to the Gulf Stream in the North Atlantic and is part of the North Pacific
ocean gyre. Like the Gulf stream, it is a strong western boundary current. Gulf stream is not present
in west Pacific coast.
Hence, option D is correct.
76. Ans. D.
the jet stream is more effective above the 12 km of the height of the surface of the earth and the
speed is 120 km/hr or more than it, the jet stream is found in the upper region of troposphere.
Hence, option D is correct.
77. Ans. B.
Normality is a way of expressing concentration of a solution and can be calculated by the formula-
Normality (N) = No. of gram equivalent of solute/ volume of solution (in litre) Gram equivalent weight
of H2 SO4 = Molecular weight/ No. of Hydrogen atom = 98/2 = 49 gm No. of gram equivalent in 49
gm of H2 SO4 = 1 So, Normality in 1 litre of solution = 1/1 = 1 N
Hence, option B is correct.
78. Ans. D.
Cathode ray is a beam of electrons emitted from the cathode of a vacuum tube. It is invisible. Cathode
ray tubes are also found in televisions and computer monitors. Since cathode ray is a beam of
electrons and all electrons are identical, charge to mass ratio is same for all the gases and does not
depend on the nature of gas.
Hence, option D is correct.
79. Ans. D.
• Chemical properties of an element depend on the number and the configuration of their valence
electrons.

PAGE 1098
www.byjusexamprep.com

• The outermost or valence electrons of an atom are the ones that take part in chemical reactions.
Hence, option D is correct.
80. Ans. C.
There are five known states of matter: solids, liquids, gases, plasma and Bose-Einstein condensates.
The main difference in the structures of each state is in the densities of the particles.
Hence, option C is correct.
81. Ans. B.
All are correct except B as Aridisol is just like sand like soil as in Arid region e.g. In Rajasthan, so there
will be very less amount of organic matter.
Hence, option B is correct.
82. Ans. A.
All are correct except 4 as a cloudburst can occur anytime and at any place which is affected by
convective weather systems.
Hence, option A is correct.
83. Ans. D.
All are correct except D, as Tsunami is a Japanese word.
Hence, option D is correct.
84. Ans. D.
Tropical rainforests are such rainforests that occur in the region of tropical rainforest climate where
there is no dry season and it remain hot and wet. Tropical evergreen rainforest biome provides
optimum environmental conditions for the growth of plants and animals because it experience heavy
rainfall and high temperature, and extend between 10 ° N and 10 ° S latitudes. The biome Tropical
rain forests are of mega thermal type because the organisms are adapted to high temperature and
wet and humid condition. The maximum development of this biome has taken place in South
America, lowland regions in Africa, and the islands off of Southeast Asia, Central America and parts
of Australia.
Hence, option D is correct.
85. Ans. C.
On December 21, the earth is in an equivalent position on the opposite points in its orbit, So the
South pole is tipped towards the sun and North pole away from it resulting in 24 hours of darkness
at the North pole, as the sun is overhead on the Tropic of Capricorn. This phenomenon is winter
solstice; while on June 21, North pole is tipped towards the Sun but the South pole is tipped away
from the sun resulting in 24 hours of darkness on the South Pole, as the sun is overhead on the Tropic
of Cancer. This phenomenon is summer solstice; while Aphelion is the position of the earth in its
orbit when it is at its distant point from the sun.
Hence, option C is correct.

PAGE 1099
www.byjusexamprep.com

86. Ans. C.
Mediterranean agriculture is practised on the either side of the Mediterranean Sea in Europe and in
north Africa from Tunisia to Atlantic coast, southern California, central Chile, south western parts of
South Africa and south and south western parts of Australia. This region is an important supplier of
citrus fruits. Viticulture or grape cultivation is a speciality of the Mediterranean region.
Hence, option C is correct.
87. Ans. D.
At present US has the largest railway network in the world. Second is China and third is India. Hence,
option D is correct
88. Ans. D.
Biosphere Reserves are established for the conservation of biodiversity for economic and social
development and maintenance of associated cultural values. There are 18 Biosphere Reserves in the
country. Agasthyamala Biosphere reserve is located at the southern-most part of the Western Ghats
and spread over two states, Kerala and Tamil Nadu. It was established in 2001; Nokrek Biosphere
Reserve is a national park situated in West Garo Hills district of Meghalaya. UNESCO made it the
Biosphere Reserves in May 2009; The Great Nicobar Biosphere Reserve has such ecosystems that
include tropical wet evergreen forests, mountain ranges and coastal plains. The region is well known
for its rich biodiversity. It was made a biosphere reserve in January 1989; while The Great Himalayan
is a National Park situated in Kullu region of Himachal Pradesh. The park was established in 1984.
Hence, option D is correct.
89. Ans. D.
There are three tiger reserves in Northeast India: Dampa Tiger Reserve, Pakul Tiger Reserve, and
Namdapha Tiger Reserve.
Note: Project Elephant was launched in 1992.
Hence, option D is correct.
90. Ans. C.
• A rainbow is a natural spectrum appearing in the sky after a rain shower. It is caused by dispersion
of sunlight by tiny water droplets, present in the atmosphere.
• A rainbow is always formed in a direction opposite to that of the Sun. The water droplets act like
small prisms.
• They refract and disperse the incident sunlight, then reflect it internally, and finally refract it again
when it comes out of the raindrops. Hence, option C is correct.
91. Ans. C.
In physics, electromagnetic radiation refers to the waves (or their quanta, photons) of the
electromagnetic field, propagating or radiating through space carrying electromagnetic radiant
energy. It includes radio waves, microwaves, infrared, (visible) light, ultraviolet, X-, and gamma
radiation. Classically, electromagnetic radiation consists of electromagnetic waves, which are
synchronised oscillations of electric and magnetic fields that propagate at the speed of light through
a vacuum, So the electromagnetic radiation consists of photons which move at speed of light within
a vacuum.
Hence, option C is correct.

PAGE 1100
www.byjusexamprep.com

92. Ans. D.
Capacitance is defined as the ability of a system to store electric charge and can be calculated by the
formula, C = Q/V, where ‘Q’ is the charge stored on each plate and ‘V’ is the applied electric potential
across the plates capacitors. Capacitance does not depend on the electrical resistivities of the
material of capacitors.
Hence, option D is correct.
93. Ans. D.
Resistance of first wire is R and other wire having radius of cross-section is twice, as R=resistivity *l/pi
r^2 ,So if we put radius value twice then resistance of that wire comes out to be R/4 now if we use
parallel rule to add resistance,we will get 4R/5 ,Hence R = PL/A ,The new resistance will be R /4 and
resultant resistance will be R/ 5 and as resistance decreases in parallel, answer will be R/5. Hence,
option D is correct.
94. Ans. C.
Visual image of human eye is 390-700 nanometer. So, approximate value is 500 nanometer only.
Hence, option C is correct.
95. Ans. B.
The plant growth regulators are small, simple molecules of diverse chemical composition:
They can be in form of indole compounds, adenine derivatives, carotenoids derivatives, terpenes or
gases like ethylene etc. The plant growth regulators are bifurcated into two groups according to their
functions in a living plant body. The first group of plant growth regulators are found in growth
activities, for instance, cell division, flowering, fruiting and seed formation. The second group of Plant
growth regulators are involved in plant responses to wounds and stresses of biotic and abiotic origin.
Hence, option B is correct.
96. Ans. A.
The mammalian heart is called myogenic (myo = muscle, genie = originating from). It means the heart
beat or the activities of the heart are auto regulated which originates from a muscle called nodal
tissue.In the human heart,heartbeat originates and contraction is initiated by a special modified
heart muscle known as sinoatrial node. It is located in the right atrium; while the QRS waves are the
depolarisation of the ventricles, which initiates the ventricular contraction.The T-wave represents
the repolarisation,that is, the return of the ventricles from excited to
normal state. Hepatic portal system is the unique vascular connection that exists between the
digestive tract and liver. This vein carries blood from intestine to the liver.
Hence, option A is correct.
97. Ans. B.
Kwashiorkar Disease is caused by protein deficiency in the diet. It results from the replacement of
mother’s milk by a high calorie-low protein diet in a child more than one year in age.
Like Marasmus, Kwashiorkor shows wasting of muscles, thinning of limbs, failure of growth and brain
development.
Hence, option B is correct.

PAGE 1101
www.byjusexamprep.com

98. Ans. D.
Stratovolcano, also known as a composite volcano, is a conical volcano built up by many layers
(strata) of hardened lava, tephra, pumice and ash. Because it is stratified, therefore it will definitely
form a volcano cone.
Hence, option D is correct.
99. Ans. C.
From south to northeast. Penneru- Andhra Pradesh, Krishna- Karnataka, Godavari- Maharashtra,
Mahanadi- Odisha, Subarnarekha- Jharkhand.
Hence, option C is correct.
100. Ans. A.
Cropping intensity refers to raising of a number of crops from the same field during one agriculture
year. It can be expressed as. Cropping intensity = (Gross cropped area / Net sown area) x 100.
Hence, option A is correct.
101. Ans. C.
Loktak Lake: the largest freshwater (sweet) lake in North -East India, also called the only Floating lake
in the world due to the floating phumdis (heterogeneous mass of vegetation, soil, and organic
matters at various stages of decomposition) on it, is located near Moirang in Manipur state, India.
The etymology of Loktak is Lok = "stream" and tak = "the end"
Hence, option C is correct.
102. Ans. A.
As per Census, a person aged seven and above who can both read and write with understanding in
any language, is treated as literate. A person, who can only read but cannot write, is illiterate. The
data provides details of State Code, District Code, Area Name, Age-group, literates, illiterates,
Literate without education level, below primary, primary, middle, Matric/Secondary, Higher
secondary/Intermediate Pre-University/Senior secondary, Non-technical diploma or certificate not
equal to degree, Technical diploma or certificate not equal to degree, Graduate & above and
Unclassified education level.
Hence, option A is correct.
103. Ans. C.
One mole of any element contains exactly 6.025x1023 atoms.
One mole of any element weighs equal to its atomic mass, thus 46 grams will mean 2 moles.
Thus, number of atoms in 46 grams of sodium-23 will be 2N.
Hence, option C is correct.
104. Ans. D.
• The first model of atom was proposed by J. J. Thomson in 1898.
• According to this model, the positive charge of the atom is uniformly distributed throughout the
volume of the atom and the negatively charged electrons are embedded in it like seeds in a
watermelon. This model was picturesquely called plum pudding model of the atom.
Hence, option D is correct.

PAGE 1102
www.byjusexamprep.com

105. Ans. C.
Isotopes- These are atoms of the elements having the same atomic number but different mass
number., Isobars-These are atoms of same mass number but different atomic number., Isotones-
These are atoms of different elements having same number of neutrons.
Hence, option C is correct.
106. Ans. D.
Atomic number is the fundamental properties of an element. Every atom is identified by its unique
atomic number. Atomic number is equal to the number of protons present in an atom of the element.
Hence, option D is correct.
107. Ans. D.
Water is neutral and has a pH value of 7 and because here it is multiple distilled water, it would be
very near to seven.
Hence, option D is correct.
108. Ans. C.
Physical changes are changes affecting the form of a chemical substance, but not it's chemical
composition. Physical changes are used to separate mixtures into their component compounds but
can not usually be used to separate compounds into chemical compounds or simpler compounds.
In the Burning of coal, burning of wood, and Heating of Potassium Chlorate there will be some sort
of chemical change, but the Heating of a Platinum crucible is a physical change.
Hence, option C is correct.
109. Ans. C.
For balancing the weight of ring, you need to put your finger or any solid object inside the ring so
that you will be able to sustain their own weight because they are not able to sustain their own
weight.so its centre of mass lies outside the body.
Hence, option C is correct.
110. Ans. C.
Capacitance = K €o A/d ,where A = Area of each plate. € = permittivity of free space, d = distance
between plate. k 1 = dielectric constant.
Since air is a bad conductor and when dielectric constant comes to the value of 7, the value of
capacitance will be around 7C. Hence, option C is correct.
111. Ans. C.
Semiconductors are the basic materials used in the present solid state electronic devices like diode,
transistor, ICs, etc. 2. Lattice structure and the atomic structure of constituent elements decide
whether a particular material will be insulator, metal or semiconductor. Semiconductors are
elemental (Si, Ge) as well as compound (GaAs, CdS, etc.). Silicon, Germanium, Gallium Arsenide are
semiconductors, but Quartz is an insulator. Hence, option C is correct.

PAGE 1103
www.byjusexamprep.com

112. Ans. B.
Power dissipated across a parallel combination = sum of each component's power (i.e. same as with
series circuits)
Power dissipated = V2/R
The voltage across each component in a parallel circuit remains constant.
As a result, the power dissipated in the second circuit will be 3P.
Hence, option B is correct.
113. Ans. C.
p type of semiconductor is obtained when Silicon is doped with a trivalent impurity like Aluminium.
Aluminium is a p-type dopant, which means that when a semiconductor i.e. silicon is doped with
aluminium, it creates a hole (positively charged carrier) by accepting an electron from the silicon
atom. Electrical conductivity of semiconductors increases and resistivity remains the same.
Hence, option C is correct.
114. Ans. A.
Deficiency of Vitamin C causes Scurvy, Swelling of gums etc.Vitamin K causes Non-clotting of blood,
Vitamin D causes Rickets in children and Osteomalasia in adults, Vitamin B1 causes Beriberi, B2
causes Cracking of skin, reddish eye, cracking of tongue, B3 causes Whitening of hair, mentally
retardness, B5 causes Pallagra or 4D syndrome,B6 causes Anemia,skin disease, B7 causes Paralysis,
hair falling, B11 causes Dysentery and B12 causes Jaundice and Teroile Glutemic.
Hence, option A is correct.
115. Ans. B.
Syngamy, also known as fertilisation, occurs between the male and female gametes. Syngamy may
occur either externally (outside the body of organisms) or internally (inside the body).
Syngamy leads to formation of a specialised cell called 'Diploid Zygote.' Hence, option B is correct.
116. Ans. B.
• Generally viruses that infect plants have single stranded RNA.
• Viruses infecting animals have double stranded RNA or DNA.
• Viruses which infect bacteria have double stranded DNA.
Hence, option B is correct.
117. Ans. A.
In cancer cells, there is breakdown of these regulatory mechanisms. Normal cells show a property
called contact inhibition by virtue of which contact with other cells inhibits their uncontrolled growth.
Cancer cells appears to have lost this property. As a result of this, cancerous cells just continue to
divide giving rise to masses of cells called tumors. Tumors are of two types: benign and malignant.
Hence, option A is correct.

PAGE 1104
www.byjusexamprep.com

118. Ans. B.
AIDS is caused by the Human Immuno deficiency Virus (HIV), a member of a group of viruses called
retrovirus, which have an envelope enclosing the RNA genome. Transmission of HIV-infection
generally occurs by (a) sexual contact with infected person, (b) by transfusion of contaminated blood
and blood products, (c) by sharing infected needles as in the case of intravenous drug abusers and
(d) from infected mother to her child through placenta.
Hence, option B is correct.
119. Ans. B.
• The process of copying genetic information from one strand of the DNA into RNA is termed as
transcription.
• In transcription, only a segment of DNA and only one of the strands is copied into RNA. This
necessitates defining the boundaries that would demarcate the region and the strand of DNA
that would be transcribed.
Hence, option B is correct.
120. Ans. A.
The two DNA strands are called polynucleotides because they are composed of simpler monomer
units called nucleotides. Each nucleotide is composed of one of four nitrogen-containing nucleobases
(cytosine, guanine, adenine or thymine), a pentose (five-carbon) sugar called deoxyribose, and a
phosphate group.
Hence, option A is correct.

PAGE 1105
www.byjusexamprep.com

Elementary Mathematics
1. Ans. C.
As,


Squaring both sides,

⇒ ………… (i)

Also,

Squaring both sides,

……….. (ii)
From (i) and (ii)


Hence option (c)
2. Ans. B.
As,
⇒ ……… (i)
Also,

⇒ ………. (ii)

PAGE 1106
www.byjusexamprep.com

Cubing both sides


……….. (iii)

From (i) and (ii) we get,


………. (iv)

From (ii) and (iv) we get,


Also,

Hence option (b)


3. Ans. C.
Statement 1:
As we know that in two consecutive integers, the one is always odd then other is even.
Hence statement 1 is correct.
Statement 2:
By Euclid’s division

a and b are positive integers.


Take

Then

Here,
Case (i), if r = 0
A = 8q

PAGE 1107
www.byjusexamprep.com

Then

Case (ii), if r = 1
[Odd integer]

⇒ [As, ]

Clearly, square of an odd integer is of the term


Hence statement 1 is correct.
Hence option (c)
4. Ans. D.
As,

Mean

⇒ ………… (i)

⇒ ………. (ii)
From (ii)
6th number + sum of 5 numbers = 102
⇒ 6th number + 75 = 102 [From (i)]
⇒ 6th number = 102 – 75 = 27
Hence option (d)
5. Ans. A.
As we know that,
Mode = 3(Median) – 2(Mean)
⇒ Mode = 3 ×220 – 2 × 270 = 660 – 540 = 120
Hence option (a)

PAGE 1108
www.byjusexamprep.com

6. Ans. C.
As,
……….. (i)

………… (ii)

Squaring and adding (i) and (ii)

Hence option (c)


7. Ans. A.

Hence option (a)


8. Ans. D.

Statement 1:
In triangle ADC and triangle BAY
∠A = ∠B = 900
AX = BY [Half of the side of squre]
AD = AB

PAGE 1109
www.byjusexamprep.com

By SAS,
Triangle ABY ≅ triangle DAX
Hence statement 1 is correct.
Statement 2:
By CPCT,
Clearly, ∠DXA = ∠AYB
Hence statement 2 is correct.
Statement 3:
We can say anything about inclination of DX with AY.
Hence statement 3 is incorrect.
Statement 4:
Clearly, DX is not perpendicular to AY.
Hence statement 4 is incorrect.
Hence only statement 1 and 2 are correct.
Hence option (d)
9. Ans. B.

As,
Taking log both sides


As base 10 the value is greater than 12 s
So, the number of digits be 13
Hence option (b)
10. Ans. B.
As,

⇒ …….. (i)

PAGE 1110
www.byjusexamprep.com

⇒ [From (i)]


⇒ x3-8-6x2+12x = 6x-6
⇒ x3-6x2+6x = 2
Hence option (b)
11. Ans. B.

As,

Arrangement in descending order be

Hence option (b)


12. Ans. A.
Given,

It is only possible when

Now,

Hence option (a)

PAGE 1111
www.byjusexamprep.com

13. Ans. A.
Given that,

We can check from the given options.


Option (a):

LHS = = RHS

Hence option (a)


14. Ans. C.
Radius of cylinder (r) = 1 cm
Height of cylinder (h) = 14 cm

Surface area of cylinder

Hence option (c)


15. Ans. D.

As,

⇒ ………… (i)

……… (ii)

Also,

⇒ ……….. (iii)

From (ii) and (iii)

PAGE 1112
www.byjusexamprep.com


Hence
Hence option (d)
16. Ans. B.
According to question,


Hence option (b)
17. Ans. A.

As we know that diagonal of trapezium intersects each other in the equal ratio.

Hence option (a)


18. Ans. B.
As,
% Change in circumference = % change in radius

PAGE 1113
www.byjusexamprep.com

⇒ 15% change in circumference = 15 % change in radius

Area of circle increased

Hence option (b)


19. Ans. B.

In

Diagonals of rectangle are equal and bisect each other.

So,

Hence option (b) is the answer.


20. Ans. C.

∠P + ∠S = 1800 [ Sum of adjacent angles of parallelogram]

………(i)

………(ii)

From (i) and (ii)


∠A = 1800 – 900 = 900
Hence option (c)

PAGE 1114
www.byjusexamprep.com

21. Ans. C.

In triangle ABC:
∠A + ∠ABC + ∠BCA = 1800
⇒ 800 + 600 + 2x0 = 1800
⇒ 2x0 = 1800 - 1400

⇒ x0 =

∠CBD = [As, BD is angle bisector of ∠ABC]

In triangle BCD:
∠CBD + y0 + x0 = 1800
=>y0 = 1800 – 300 – 200 = 1300
Hence, x0 = 200 and y0 = 1300
Hence option (c)
22. Ans. B.

Assuming right angled triangle be in a circle, where PR is diameter of circle.


PT = QT = TR [Radii of circle]
QT = TR
∠TQR = ∠TRQ
∠TQR = ∠RLM [Corresponding angles]
∠RLM = ∠LRM
Hence option (b)

PAGE 1115
www.byjusexamprep.com

23. Ans. C.

As, ∠DAC = ∠DCA


So, DA = DC
Hence triangle ADC is an isosceles triangle
Hence statement 1 is correct.
Statement 2:
As, we know that the centroid of a triangle is the point where the three medians of the triangle meet.
Hence statement 2 is incorrect.
Statement 3:
AB = CB
AD = DC
BD = BD
Thus, triangle ABD ≅ triangle CBD [By, SSS congruence criteria]
Hence statement 3 is correct.
Hence option (c)
24. Ans. A.
Given,

Having roots and (Given)

Now,

Sum of roots

⇒ ………. (i)

PAGE 1116
www.byjusexamprep.com

Products of roots

⇒ ………….. (ii)

From (i) and (ii)


(3a-1)2/9(a2-5a+3) = 1/(a2-5a+3)
⇒ 9a2+1-6a=9a2-45a+27


Hence option (a)


25. Ans. C.

Area of

Area of

Hence option (c)


26. Ans. A.
As,

PAGE 1117
www.byjusexamprep.com

Rationing,

……… (i)

Now,

Hence option (a)

PAGE 1118
www.byjusexamprep.com

27. Ans. C.
As,

So, …….. (i)


Also,

……… (ii)
From (i) and (ii) we get,

If

Hence option (c)


28. Ans. B.
There are 25 prime numbers less than 100 are
,67,89
Hence option (b)
29. Ans. A.
As,
………. (i)
……… (ii)

PAGE 1119
www.byjusexamprep.com

Squaring both sides to (i)

⇒ [As, ab = b]

Now,

Hence option (a)


30. Ans. B.
∠ABC = 1200 [Angle of regular hexagon]

∠BAC = ∠BCA =

∠DCA = 1200 – 300 = 900


Thus, ⊿ ADCA is a right triangle.
Let side DC = a

Now taking triangle ASD:

PAGE 1120
www.byjusexamprep.com

Let S is the vertex of pole

In triangle TCA:

Thus, ratio ∅

Hence CT:DS = 1:2√3


Hence option (b)
31. Ans. C.

[From Mid point theorem]

Similarly,

So, BD = AC [Diagonal of rectangle]

PAGE 1121
www.byjusexamprep.com

Thus, PQ = QR = RS = SP
Hence, PQRS is a Rhombus but need not be a square.
Hence option (c)
32. Ans. C.

Given radius of circle be 3.5 cm


Area enclosed = Area of equilateral triangle – 3 × Area of sector APQ

Hence area enclosed be

Hence option (c)


33. Ans. B.

As given

Let

Then,

PAGE 1122
www.byjusexamprep.com

Putting the value of x, y and z in 4z2(x2 + y2)

Option (b):

Here value of 4z2(x2 + y2) = value of


Hence option (b)
34. Ans. A.
As,

Hence option (a)


35. Ans. C.

PAGE 1123
www.byjusexamprep.com

Hence option (c)


36. Ans. A.
Let
Then

………. (i)
As, N is prime to p
So Let N = 3
Then from (i)

Here, is a multiple of p

Here, given condition had satisfied when p is a prime number.


Hence option (a)
37. Ans. A.

Mean =





Hence option (a)

PAGE 1124
www.byjusexamprep.com

38. Ans. C.

In

In

Distance between two chords

Hence option (c)

PAGE 1125
www.byjusexamprep.com

39. Ans. C.
Area of square be and [Given]

So,

i.e., side of square be 11 cm


Perimeter of square

So, perimeter of circle

Area of circle

Hence option (c)


40. Ans. B.
As,
Surface area of sphere

…….. (i)

……… (ii)

From (i) and (ii)

So, radius is reduced to one third.


Hence option (b)

PAGE 1126
www.byjusexamprep.com

41. Ans. C.

First of all whole part divides into 4 parts viz. I, II, III and IV
Part I:

Semi perimeter (S) =

According to Heron's formula


Area =

= sq. meter

Part II:

Area of rectangle = 2×1 = 2 sq. meter


Part III:

Area of triangle =

PAGE 1127
www.byjusexamprep.com

Part IV:

Total Area =

Hence option (c)


42. Ans. A.

Let D is diameter of each circle.


Thus, side of square = D

Diameter of shaded circle =

Hence option (a)


43. Ans. A.

Here, AC parallel to ED.


So, height of all triangles be same.
Let height of triangle be h

PAGE 1128
www.byjusexamprep.com

Required ratio =

Hence option (a)


44. Ans. A.
Ratio of weights of broken diamond
Net weight
Price

Price

Net loss
Now,

Price of original diamond

Hence option (a)


45. Ans. D.
As we know that,
The products of two numbers = LCM of two numbers × HCF of two numbers

Hence option (d)


46. Ans. D.
As,

PAGE 1129
www.byjusexamprep.com


Comparing both sides,


Hence option (d)
47. Ans. C.
We can solve this question by taking arbitrary values
As
Let
Statement 1:
HCF(c,ab) = HCF(3,12) = 3
HCF(a,c) = HCF(6,3) = 3
HCF(c,ab) = 3 = HCF(a,c)
Hence statement 1 is correct.
Statement 2:
LCM(a,b) =LCM (6,2) = 6
LCM(a,c) =LCM (6,3) = 6
i.e. LCM(a,b) = 6 = LCM(a,c)
Hence statement 2 is correct.
Hence option (a)
48. Ans. D.
As,
Mean of 300 numbers = 60

PAGE 1130
www.byjusexamprep.com

Sum of top 100 numbers + sum of last 100 numbers + sum of remaining numbers
Sum of remaining 100 numbers + 8000 + 5000 = 18000
⇒ Sum of remaining 100 numbers

Mean of remaining 100 numbers

Hence option (d)


49. Ans. D.
As, a, b, c, d, e, f, g are consecutive even numbers then numbers are
d-6, d-4, d-2, d, d+2, d+4, d+6
Total = d-6+d-4+d-2+d+d+2+d+4+d+6=7d
Also, when j, k, l, m, n are consecutive odd numbers then numbers be

Total =

Average =

Hence option (d)


50. Ans. B.

In


In :

PAGE 1131
www.byjusexamprep.com

Hence height of tree be


Hence option (b)
51. Ans. B.
As speed of train km/hr
Speed of man km/hr
Relative speed km/hr

m/sec

m/sec

Time taken by train to passes the man

Hence option (b)


52. Ans. D.
Let time taken passenger train
Time taken by express train
When distance
Then according to question,

PAGE 1132
www.byjusexamprep.com


Hence expression train will take 9 hr
i.e. 9 Pm + 9 hr = 6 AM
Hence option (d)
53. Ans. C.
Time taken by A to cover 100 m

Time taken by B to cover

Time taken by B to cover 96 meter

Speed of B

m/s

Hence option (c)


54. Ans. B.
According to question,

Hence option (b)

PAGE 1133
www.byjusexamprep.com

55. Ans. B.

Area of regular hexagon of side a be

Hence option (b)


56. Ans. D.
As,

Volume of cone [As, r be radius h be height]

Increased radius

Increased volume

% Change

Hence option (d)


57. Ans. C.

According to question

⇒ [As, ]

PAGE 1134
www.byjusexamprep.com


⇒ Persons
Hence option (c)
58. Ans. D.
According to question

Option (a):

Option (b):

Option (c):

Option (d):

Hence option (d)


59. Ans. A.
According to question,

Sum of root

So, ……….. (i)

Product of root

⇒ ………….. (ii)

From (i)

PAGE 1135
www.byjusexamprep.com

⇒ [ ]

Hence

Hence option (a)


60. Ans. C.
As, x=t(1/t-1) or x(t-1)=t
y=t(t/t-1) =[t(1/t-1)]t= xt=x(t-1).x=tx
t=y/x
now from x(t-1)=t
x(t-1)=y/x
xt=y
x(y/x) = y
xy = yx
Hence option (c)
61. Ans. B.

In triangle ABD:

In triangle ACD:

PAGE 1136
www.byjusexamprep.com

⇒ [As, x = ]

Hence height of aero plane be

Hence option (b)


62. Ans. C.
As is a factor

i.e.
⇒ is a root of equation.
Option (a)

Option (b)

Option (c)

Hence is a factor of

Hence option (c)

PAGE 1137
www.byjusexamprep.com

63. Ans. D.
As,
Let

Now,

We can’t determine the value.


Hence option (d)
64. Ans. D.
As





Hence option (d)
65. Ans. C.
Given % discount be 25%

i.e. th part

As, marked price

Then discount

Then cost of mobile


As 3 ratio cost be 4875

PAGE 1138
www.byjusexamprep.com

1 ratio costs be

Thus, original price


Hence option (c)
66. Ans. C.

Let male

Female

Unmarried females

% of unmarried females

Hence option (c)


67. Ans. B.

Number of girls

Number of boys

Number of girls left after 4 girls leaves


Ratio of girls to boys
Hence option (b)
68. Ans. A.
We can solve by using options

PAGE 1139
www.byjusexamprep.com

Option (a):
Put

As
Let

LHS

= RHS
Hence option (a)
69. Ans. B.
As, A and B together can do the work in 12 days.
B alone can do the work in 30 days.
So, A can do the work in

i.e. A can do the work in 20 days.


Hence option (b)
70. Ans. A.

As,

………. (i)

…….. (ii)

……….. (iii)

PAGE 1140
www.byjusexamprep.com

From (i), (ii) and (iii)

LCM

Hence option (a)


71. Ans. B.
Possibility 1:
Number of sweets, costing Rs. 7 be 10

Then money speed

Money left

Number of sweets costing Rs. 10

Number of sweets
In this possibility no money is left over.
Possibility 2:
Number of sweets, costing Rs. 7 be 20
Then money spend

Money left
Number of sweets costing Rs. 10

Number of sweets
Here, also no money is left over.
Hence, maximum number of sweets Sunil can get 26. So that no money is left over.
Hence option (b)
72. Ans. C.
As, relative speed of police and thief

PAGE 1141
www.byjusexamprep.com

Time taken by police to catch the thief

Distance travelled by thief before he got caught

Hence option (c)


73. Ans. A.
As,

Now,

Again,

⇒ No of days

PAGE 1142
www.byjusexamprep.com

⇒ days

Hence option (a)


74. Ans. C.
As,
a.a/100 +b.b/100 = 2ab/100



Thus a is 100bof b
Hence option (c)
75. Ans. C.
As ratio of numbers be 1 : 5
Let numbers be x, 5x
Then,



Numbers be 8 and

Now, difference between their square

Hence option (c)


76. Ans. C.
As,

⇒ ……….. (i)
And

PAGE 1143
www.byjusexamprep.com

⇒ ……….. (ii)

Here,

Then,

As we know that if

Then lines are parallel and inconsistent no solution.


Hence solution is in consistent.
Hence option (c)
77. Ans. D.
As Lead : Tin
X:- 1 : 2
Y:- 2 : 3

Lead in 25 kg

Tin in 25 kg

Now,

Lead in 125 kg

Tin in 125 kg

Lead in mixture

PAGE 1144
www.byjusexamprep.com

Tin in mixture

Ratio of Led : Tin

Hence option (d)


78. Ans. C.
Given four slices be 1500, 900, 600 and 600
When 600 be deleted
Then, remaining slices be 1500, 900, 600
Total angle = 3000
While making pie chart where 3000 is taken as 100%
Then,

Also, 50% of 3600 will be 1800


Hence, largest slice will be as angle 1800
Hence option (c)
79. Ans. B.
Roots of Aman be (4, 3)
So, equation be
…………. (i)

Here constant is 12 which is wrong


Roots of Alok be (3, 2)
So, equation be

PAGE 1145
www.byjusexamprep.com

[According to (i)]

Here coefficient of which is wrong.


So, the correct equation be

For roots,


Hence option (b)
80. Ans. B.

Number of Type A pencil =

Number of Type B pencil =

Number of Type C pencil =

Average =



Hence option (b)

PAGE 1146
www.byjusexamprep.com

81. Ans. C.

f1 = 29 – 18 = 11
f2 = 45 – 29 = 16
Hence f1 and f2 be 11 and 16.
Hence option (c)
82. Ans. A.
As, we know that
π radian = 180 degree
180° = π radians.
1° = π/180 radians
D° = Dπ/180 radians.
∴ Dπ/180 = R
∴D/180 = R/π
∴ πD = 180R
83. Ans. C.

Thus, minimum value of

Hence minimum value of be 12


Hence option (c)

PAGE 1147
www.byjusexamprep.com

84. Ans. A.

In :


⇒ …….. (i)
In :

So,
Hence option (a)
85. Ans. B.

In triangle ABC and triangle DAC


∠BAC = ∠ADC
∠ACB = ∠DCA (Common angle)
So, ⊿ABC~⊿DAC

PAGE 1148
www.byjusexamprep.com

Thus,

Hence length of AC be 6 cm
Hence option (b)
86. Ans. C.

[When 4444 is divided by 9]

[Remainder will be 7]

[Or (-2) negative remainder]

[-2 negative remainder]

Hence remainder be 7
Hence option (c)
87. Ans. C.
As,

PAGE 1149
www.byjusexamprep.com

Let

Then,

Hence option (c)


88. Ans. D.
As,

Now,


Hence number of days be 30
Hence option (d)
89. Ans. C.
As,

PAGE 1150
www.byjusexamprep.com


Hence option (c)
90. Ans. A.
As

Hence option (a)


91. Ans. D.
In a triangle, Sum of two sides must be greater than the 3rd side.
But in option (d)
As

Thus, (2,3,6) is not a triplet.


Hence option (d)
92. Ans. B.

As,

PAGE 1151
www.byjusexamprep.com

Hence option (b)


93. Ans. C.

∠UST = ∠QSR = 700 [vertically opposite angle]


∠PQS + ∠QSR = 1800 [As, PQ parallel to RS]
∠PQS = 1800 – 700 = 1100 [As, ∠QSR = 700]
Now,
∠PQS + ∠QPR = 1800 [As, PR parallel to QS]
∠QPR = 1800 – 1100 = 700
Again,
∠XPL + ∠LPR + ∠RPQ = 1800
⇒ ∠XPL = 1800 – 350 – 700 = 750 [As, ∠LPR = 350]
Hence, ∠MPQ = ∠XPL = 750 [Vertically opposite angle]
Hence option (c)
94. Ans. C.
Let the sum be x

PAGE 1152
www.byjusexamprep.com

Now,



Hence the sum be Rs. 6250
Hence option (c)
95. Ans. C.

∠AMC = ∠BMD ………(i)


∠CMD = ∠CMD (common angle)…….(ii)
Adding (i) and (ii), we get
∠AMC + ∠CMD = ∠BMD + ∠CMD
∠AMD = ∠BMC
∠DAM = ∠CBM
AM = BM
By, ASA
Triangle ADM ≅triangle BCM
Hence option (c)

PAGE 1153
www.byjusexamprep.com

96. Ans. D.
As,

⇒ sinx = 1/2

Hence option (d)


97. Ans. B.
As α and β are the roots of equation , ………. (i)

Now, ………. (ii)

From (i)

Sum of roots

Products of roots

Now,


From (ii)

[As, k < 0]

So, maximum value be -2

PAGE 1154
www.byjusexamprep.com

So, maximum value of

Hence option (b)


98. Ans. C.
Let age be x years at the time of marriage

Her present age

Her son’s age

Hence option (c)


99. Ans. B.
As,
……….. (i)

……… (ii)

Line (i) and (ii) will intersect each other

If

According to question

So,


Hence option (b)

PAGE 1155
www.byjusexamprep.com

100. Ans. D.

PAGE 1156
www.byjusexamprep.com

CDS II 2016
ENGLISH
Direction: In the following question, a passage R : The minute I came up I took a breath and
consists of six sentences. The first and the sixth plunged down again.
sentence are given in the beginning as S1 and S6.
S : I ducked down, pushed between two
The middle four sentences in each have been
men and ran for the river.
removed and jumbled up. These are labelled P,
Q, R, and S. You are required to find out the The proper sequence should be
proper sequence of the four sentences and mark
A. S Q P R B. P R Q S
your response accordingly on the Answer Sheet.
1. S1 : History shows that the growth of C. S Q R P D. Q S P R
civilization depends upon the gifts of 3. S1 : Why do the English travel?
nature, particularly the wealth yielded by
the soil. S6 : For here, in cosmopolitan England, one
is always exposed to the danger of meeting
S6 : In fact, most of the wars in the all sorts of peculiar aliens.
beginning of humanity’s history were
fought for the gain of territory. P : Besides, they are taught that travel
broadens the mind.
P : The more land they had, the more they
were satisfied. Q : They do so mainly because their
neighbor does this and they have caught
Q : The nature and the quality of the land
the bug from him.
they possessed were of great importance to
them. R : Although they have now discovered the
R : All ancient communities worked hard to sad truth that whatever travel may do to
produce food. the mind, it certainly broadens other parts
of the body.
S : There was also a great desire among
them to possess as much land as possible. S : But, and perhaps mainly, they travel to
avoid foreigners.
The proper sequence should be
The proper sequence should be
A. RQSP B. QRSP
A. R S Q P B. P R S Q
C. SQRP D. PRQS
2. S1 : There were shots as I ran. C. S Q P R D. Q P R S

S6 : Staying submerged was only too easy 4. S1 : Jumbo, the famous 3.3 metre elephant
with so much clothing and my army boots. was born in Africa over a hundred years
ago.
P : The water was icy, but I stayed until I
thought my lungs would burst. S6 : Before his death in September 1882, he
had been seen by over 20 million
Q : I tripped at the edge and went in with a
Americans.
splash.

PAGE 1157
www.byjusexamprep.com

P : After disembarkation in New York, he Q : The contemporary university stands at


was taken in a procession to the place the centre of its society.
where he was to be kept.
R : They have common historical roots yet
Q : Another admirer was the famous are deeply embedded in their societies.
American showman Barnum who bought it
S : Established in the medieval period to
for a huge sum in February 1882.
transmit established knowledge and
R : Transported from his native land to provide training for a few key professions,
London Zoo, he became a favourite of universities have become a primary creator
Queen Victoria. of new knowledge through basic research.
S : He made his transatlantic voyage aboard The proper sequence should be
SS Assyrian Monarch.
A. P S Q R B. S Q R P
The proper sequence should be
C. S P R Q D. R P S Q
A. P Q R S B. S R Q P
7. S1 : At the age of eighteen Gandhi went to
C. P S R Q D. R Q S P college, but remained for only part of the
year.
5. S1 : The bureaucrat and the social worker
are men of totally different orientations S6 : This was against his religion, and most
and styles of functioning. of his relatives were against his going.
S6 : The world will be a better place to live P : Soon after this, he was advised to go to
in if they learn a little from each other. England to study to be a lawyer.
P : The other is considered to be a man ever Q : Studies did not interest him and he did
on the move. not do well.
Q : He is portrayed as a man fond of rules R : It was difficult for him to leave India and
above all other things. go to a foreign land where he would have
to eat and drink with foreigners.
R : The one is regarded as given to
sedentary habits, doing a lot of paper work. S : This would not be easy.
S : Driven by an urge to help others he is The proper sequence should be
impatient with red tape and unnecessary
A. P S R Q B. S Q P R
delays.
C. P R Q S D. Q P S R
The proper sequence should be
8. S1 : Helen graduated in 1904 with special
A. P Q R S B. R Q P S
honours in English.
C. S R Q P D. Q P S R
S6 : Here dress was torn and roses were
6. S1 : Universities are peculiar institutions. snatched from her hat.
S6 : It is the most important institution in P : She was twenty-four years old.
the complex process of knowledge creation
Q : She was invited to the St. Louis
and distribution.
Exposition in 1904 to awaken worldwide
P : Traditionally elite institutions, the interest in the education of the deaf-blind.
modern university has provided social
R : But on Helen Keller Day the crowds got
mobility to previously disfranchised groups.
out of hand.

PAGE 1158
www.byjusexamprep.com

S : Requests were already flowing in for S : He hesitated as the sound died down and
appearances and for magazine articles. then crept on, thinking that if they could
sleep through that, they would sleep
The proper sequence should be
through any noise he could make.
A. Q P S R B. S P R Q
The proper sequence should be
C. P S Q R D. S Q R P
A. P Q S R B. R Q S P
9. S1 : One of the many young scientists who
C. P R Q S D. R Q P S
chose to throw in their lot with Rutherford
was an Oxford physical chemist, Frederick 11. S1 : Don’t you think that the housefly is a
Soddy. nuisance?
S6 : We now know that Gamma rays are a S6 : When you see a fly rubbing its legs
particularly fierce form of X-rays. together, it is just cleaning itself, and
scraping off some of the material that has
P : His association with Rutherford lasted
gathered there.
only two years, but that was long enough to
change the whole face of physics. P : For ages that’s what man considered the
fly to be – just a nuisance.
Q : He was just 23.
Q : It was discovered that these flies carried
R : They found that thorium changed into a
disease germs that cause the death of
new element, thorium X, and in the process
millions of people every year.
gave off what was apparently a gas and at
the same time a third type of ray, which R : But now we know that the innocent-
they named after the Greek letter looking housefly is one of man’s worst
‘Gamma’. enemies.
S : When he teamed up with Rutherford, S : It makes an irritating buzzing sound; it
they investigated thorium which, as Marie annoys you when it crawls on your skin; and
Curie had shown, was radioactive. so on.
The proper sequence should be The proper sequence should be
A. P Q S R B. Q P S R A. R Q P S B. P Q S R
C. Q P R S D. Q S R P C. Q P R S D. S P R Q
10. S1 : The boy felt his way up the creaking 12. S1 : Iguanodon was one of the first
stairs through thick darkness. dinosaurs to be scientifically described.
S6 : He was just telling himself he was safe S6 : This quadrupedal dinosaur lived about
when the door was flung open and the 70 million years ago and its fossils are found
gaunt old man grabbed his shoulder. in many parts of England, Europe, Asia and
North Afrcia.
P : All he had to do was just get past the
central door on the landing. P : These teeth formed a dental battery in
which replacing teeth were constantly
Q : He stopped as the great clock below
growing.
whined for a few seconds and gave out a
single, solemn stroke. Q : The fore limbs were slightly larger than
other members of its group known as
R : His eyes were raised to the faint
Ornithischia.
moonlight that shone above the landing.

PAGE 1159
www.byjusexamprep.com

R : It had numerous teeth in the sides of the Q : No one can deny the importance of
jaws, arranged in rows. pesticides in our agriculture.
S : It was about 40 feet in length. R : The proposed amendment is an attempt
at doing this, but in the process it seems to
The proper sequence should be
have gone overboard in most cases.
A. Q S R P B. S R P Q
S : Even today, nearly a fourth of our crop is
C. R Q P S D. P Q S R lost due to pests and weeds.
13. S1 : A single device can heat fluids without The proper sequence should be
requiring an electrical element.
A. R P S Q B. R Q S P
S6 : Because there is no electrical element,
C. Q S P R D. P R S Q
there is no risk of fire, so the system is
suitable for dangerous environments. Direction: Improve the underlined part of the
sentence.
P : A heat exchanger transfers heat from
the central cavity to an air heater, a water 15. Suppose if you are selected, will you give us
heater or an industrial processor. a treat?
Q : It has a container like the crank case of A. Supposing if B. If suppose
a car’s engine, which contains the fluid to
C. If D. No improvement
be heated.
16. I would rather have a noble enemy than a
R : Compression and friction at the nozzles
mean friend.
heat the fluid so that the temperature in
the central cavity rises steadily. A. would more have
S : A motor and pulley inside the container B. would have
turn a rotor wheel, which in turn forces the
C. will have
fluid again and again through narrow
nozzles into a central cavity. D. No improvement
The proper sequence should be 17. He decided to take the help of a guide lest
he may miss the way.
A. R P S Q B. S P Q R
A. he should miss
C. Q S R P D. S R P Q
B. he will miss
14. S1 : There is no doubt that the rules
governing imports, manufacture and the C. he might fail to see
use of pesticides need to be further
tightened up. D. No improvement

S6 : At the same time, better pollution 18. He wanted my permission (to taking part
control measures are needed to check the in) sports.
discharge of poisonous gases and chemicals A. to take part in
by a host of other industries which are
equally responsible for poisoning our B. for to take part in
world. C. for to taking part in
P : But a more practical and feasible D. No improvement
approach is required to tackle this problem.

PAGE 1160
www.byjusexamprep.com

19. We are doing this (in the interest of the 25. The teacher as well as his wife were invited.
poors).
A. was invited
A. in the interests of the poors
B. were also invited
B. in the interests of the poor
C. were being invited
C. for the interests of the poor
D. No improvement
D. No improvement
26. I wish I can help you.
20. He reached his destination at night.
A. may help
A. destination
B. could have helped
B. at his destination
C. could help
C. on his destination
D. No improvement
D. No improvement
27. My brother is looking forward to meeting
21. One is often pleased with himself. his employer tomorrow.
A. with one’s self A. to meet
B. with themselves B. for meeting
C. with oneself C. that he may meet
D. No improvement D. No improvement
22. Unless you are not very careful, you will run 28. My father has given his ascent for my long
into debt. tour.
A. are very A. accent
B. will be very B. approof
C. may be C. assent
D. No improvement D. No improvement
23. I am living in Bombay for the last ten years. 29. Heavy work has been thrusted on me.
A. had lived A. has been thrust
B. have been living B. has thrusted
C. lived C. has thrust down
D. No improvement D. No improvement
24. This scooter is not efficient as it used to be; 30. No sooner (had he completed) his first
instead it is still a very useful machine. novel than he fell seriously ill.
A. similarly A. he had completed
B. furthermore B. could he completed
C. nevertheless C. he completed
D. No improvement D. No improvement

PAGE 1161
www.byjusexamprep.com

31. There is many a slip between the cup and 37. Ramesh is working in this factory for the
lip. past three months.
A. cup and lip A. has been working
B. cups and lips B. has been worked
C. the cup and the lip C. had worked
D. No improvement D. No improvement
32. We can go out whenever we choose to, 38. I am waiting for three-quarters of an hour.
isn’t it?
A. I am waiting since
A. can’t we
B. I have waited since
B. are we not
C. I have been waiting for
C. don’t we
D. No improvement
D. No improvement
33. He was too conscientious in the discharge 39. This book is (the more interesting) of the
of his duties that he could not serve that three.
exploiter for long. A. the interesting
A. that he would not serve B. the most interesting
B. for serving C. most interesting
C. to serve D. No improvement
D. No improvement
Direction: Each of the following items in this
34. Decide one way or the other; you can’t be section consists of a sentence the parts of which
sitting on the fence forever. have been jumbled. These parts have been
labeled P, Q, R and S. Given below each sentence
A. be seated on the fence are four sequences namely (a), (b), (c) and (d).
B. be sitting and fencing You are required to rearrange the jumbled parts
of the sentence and mark your response
C. be dancing on the fence accordingly.
D. No improvement 40. Georgian loyalists and rebel forces
35. Raman wants (to dispose off) his house.
after a week of fighting
A. to dispose of
P
B. the disposal off
agreed to
C. the disposal off of
Q
D. No improvement
a ceasefire today
36. I (regret for) using objectionable words
against a man so mighty. R

A. repent for B. sorry for in which 51 people were killed

C. regret D. No improvement S

PAGE 1162
www.byjusexamprep.com

The correct sequence should be which suggest the need for


A. P Q R S B. Q R S P Q
C. Q P S R D. Q R P S two factors
41. The ultimate hope R
will force the nations an international language
P S
that the destructive nature of weapons The correct sequence should be
Q A. P R Q S B. R P Q S
to give up war C. R Q S P D. S Q R P
R 44. The man
has not been fulfilled is generally the one
S
P
The correct sequence should be
who can work very hard
A. P S Q R B. P Q R S
Q
C. Q P R S D. P R Q S
when he must work
42. The leader of the opposition,
R
in the manner he had planned to convince
them, can play most heartily when he has the
chance of playing
P
S
on realising that he had failed to convince
the assembly, The correct sequence should be

Q A. Q R S P B. P S Q R

who had a reputation for speech making, C. S P Q R D. Q R P S

R 45. Dear Lodger, I agree, the roof is leaking; but

was very much disappointed there would be no need

S P

The correct sequence should be while it is raining


A. P S R Q B. Q S R P Q
C. R S Q P D. S P R Q and when the Sun shines,
43. We can think of R
often confused in the public minds I can’t get it repaired
P S

PAGE 1163
www.byjusexamprep.com

The correct sequence should be at high prices


A. Q R S P B. S Q R P S
C. R S P Q D. P R Q S The correct sequence should be

46. There is no reason A. S Q P R B. Q R S P

that life may exist C. R Q P S D. Q P S R

P 49. as environmental tools


P
in great profusion
for military purpose
Q
Q
why we should not be willing to think
are finding various new uses
R
R
in other worlds
the world’s fastest computers initially
S conceived
The correct sequence should be S
A. R S P Q B. P Q S R The correct sequence should be
C. R P Q S D. R Q S P A. S Q R P B. P Q R S
47. The Regent released Voltaire C. R P S Q D. Q P S R
P 50. P) is a painful feeling

having discovered that Q) without this indirect appeal to our self-


love,
Q
R) the sense of inferiority in others,
he had imprisoned an innocent man
S) and not an exalting one
R
The correct sequence should be
and gave him a pension
A. P R S Q B. R S P Q
S C. S R Q P D. R Q S P
The correct sequence should be Direction: In the following passages, at certain
A. P Q R S B. P R S Q points you are given a choice of three words
marked (a), (b) and (c) one of which fits the
C. Q R S P D. Q R P S meaning of the passage. Mark (d) if none of the
48. which is sold options are suitable to fill the blank. Choose the
best word out of the three. Mark the letter, viz.,
P (a), (b) or (c), relating to this word on your
for its horn Answer Sheet.

Q Passage – 1
Many of us believe that science is something
the Rhinoceros is hunted by poachers
modern, ___51___ the truth is that ___52___
R has been using science for ___53___ very long

PAGE 1164
www.byjusexamprep.com

time. However, it has ___54___ a greater effect 58. modern life ___58___ and comfortable.
on human lives in the last 25 ___55___ 30 years
A. Dull B. Exciting
than in the hundreds of years ___56___ the
invention of the plough. The ___57___ gifts of C. Aimless D. None of these
science have made modern life ___58___ and
59. But science has ___59___ the same time
comfortable. But science has ___59___ the same
created new problems.
time created new problems. One of these which
may become ___60___ in the years to come, is A. At B. In
___61___ of ‘jet-lag’. With the coming of
modern jets, flying at more than 900 km an hour, C. Within D. None of these
the world ___62___ very small indeed. Today if 60. One of these which may become ___60___
you ___63___ New Delhi at 4:00 in the morning, in the years
you ___64___ eat an early breakfast in the sky
___65___ Kabul, and be in London by about 1:00 A. Bad B. Worse
p.m. C. Good D. None of these
51. Many of us believe that science is 61. to come, is ___61___ of ‘jet-lag’. With the
something modern, ___51___ the truth. coming of modern jets, flying at more than
A. If B. Though 900 km an hour,
C. Unless D. None of these A. Those B. This
52. is that ___52___ has been using science C. That D. None of these
A. Men B. People 62. the world ___62___ very small indeed.
C. Man D. None of these A. Can become B. Become
53. for ___53___ very long time. C. Has become D. None of these
A. The B. A
63. Today if you ___63___ New Delhi at 4:00 in
C. That D. None of these the morning,
54. However, it has ___54___ a greater effect A. Leave B. Will leave
on human lives in
C. Would leave D. None of these
A. Has B. Have
64. you ___64___ eat an early breakfast
C. Had D. None of these
A. Will B. Can
55. the last 25 ____55____ 30 years than
A. And B. Or C. Must D. None of these

C. Either D. None of these 65. in the sky ___65___ Kabul, and be in


London by about 1:00 p.m.
56. in the hundreds of years ___56___ the
invention of the plough. A. At B. On
A. From B. For C. Over D. None of these
C. Since D. None of these Direction: In the following passages, at certain
57. The ___57___ gifts of science have made points you are given a choice of three words
marked (a), (b) and (c) one of which fits the
A. Marvelous B. Costly meaning of the passage. Mark (d) if none of the
C. Inexpensive D. None of these options are suitable to fill the blank. Choose the

PAGE 1165
www.byjusexamprep.com

best word out of the three. Mark the letter, viz., 69. It is more easy to shape ___69___ piece of
(a), (b) or (c), relating to this word on your metal after
Answer Sheet.
A. The B. That
Passage – 2
C. A D. None of these
All of us know the kinds of substances that are
known as metals. They are commonly 70. it has just been ___70___ heated because
distinguished from other substances by their heat softens it.
bright and shiny surfaces. The majority of them A. Lightly B. Warmly
are fairly heavy. ___66___ most metals are hard
and strong, they ___67___ hammered, pulled C. Now D. None of these
and pressed into various shapes ___68___
71. Very great heat ___71___ to melt a metal
strong machines. It is more easy to shape
to
___69___ piece of metal after it has just been
___70___ heated because heat softens it. Very A. Must need B. Been needed
great heat ___71___ to melt a metal to its liquid
___72___. Metal workers always pour the liquid C. Is needed D. None of these
metal ___73___ a hollow form called a mold, 72. its liquid ___72___.
usually made of ___74___ prepared molding
sand held in a box of wood or iron. ___75___ the A. Flow B. Condition
metal ___76___ the molding boxes are broken C. Content D. None of these
open and the ___77___ metal shapes ate taken
out of the molds. ___78___ process is called 73. Metal workers always pour the liquid metal
casting. Metal objects that ___79___ by this ___73___ a hollow form called a mold,
process break more easily than those made by
A. Out of B. Into
hammering. When ___80___ metals are melted
they can be mixed together. The mixture is called C. Onto D. None of these
an alloy.
74. usually made of ___74___ prepared
66. The majority of them are fairly heavy. molding sand held in a box of wood or iron.
___66___ most metals are hard and strong,
A. Specialist B. Specified
A. Since B. Although
C. Specially D. None of these
C. When D. None of these
75. ___75___ the metal
67. they ___67___ hammered, pulled and
pressed into A. Before B. After
A. Can be B. Cannot be C. While D. None of these
C. A D. None of these 76. metal ___76___ the molding boxes are
broken open
68. various shapes ___68___ strong machines.
A. Has cooled
A. By mean of
B. By meaning of B. Have cooled

C. By means of C. Has been cooled

D. None of these D. None of these

PAGE 1166
www.byjusexamprep.com

77. and the ___77___ metal shapes ate taken a goal which supersedes them. Broadly speaking,
out of the molds. the achievements of generals, politicians, and
statesman have an importance only in their own
A. Liquid B. Solid
time. Hence these people and their acts, great as
C. Soiled D. None of these they may have been are like milestones which
mankind passes on its way to something else.
78. __78__ process is called casting.
But with works of art it is not so. The place which
A. That they occupy in the estimation of succeeding ages
and the power which they exercise over men’s
B. This spirits are as great as they were in the age which
C. Thus it produced them; indeed, their power tends to
increase with time, as they came to be better
D. None of these understood.
79. Metal objects that ___79___ by this 81. The power of art can be judged through
process break more easily than those made
by hammering. A. its influence on a few individuals.
A. Have been fashion B. its influence on the people over the
years.
B. Have been fashioned
C. the greatness of great artists.
C. Are in fashion
D. None of these D. the opinions of great thinkers.

80. When ___80___ metals are melted they 82. The statement “Newton’s theory of
can be mixed together. The mixture is gravitation has been superseded by
called an alloy. Einstein’s theory of relativity” suggests that

A. Those B. Two A. the theory of relativity has nothing to


do with theory of gravitation.
C. All D. None of these
B. the theory of relativity is new in
Direction: In this section, you have five short comparison to the theory of gravitation.
passages. After each passage, you will find some
questions based on the passage. First, read a C. the theory of relativity is an
passage and answer the questions based on it. improvement over the theory of
You are required to select your answers based on gravitation.
the contents of the passage and opinion of the
D. the theory of relativity has suppressed
author only.
the theory of gravitation.
Passage – 1
83. The achievements of generals, politicians
One of the most important things to notice and statesmen have been compared to
about the power of art is the way in which great milestones by the author because
works continue to exert their influence through
the ages. Scientific discoveries which are of A. they are inscribed on the milestones.
major importance at the time when they are B. they have contemporary relevance.
made are supersedeD. Thus, Newton’s
theory of gravitation has been superseded by C. they have topical and historical interest.
Einstein’s theory of relativity. Hence the work of
D. they are strong and lasting stones.
great scientist has value in stages on the way to

PAGE 1167
www.byjusexamprep.com

84. How is a work of art different from the work C. In some activities men are stronger than
of a scientist? women and in some others women are
stronger than men.
A. A work of art is as permanent as the
work of a scientist. D. Men and women are equally strong.
B. The influence of a work of art increases 86. The author says that any dispute about
from age to age unlike the work of a whether or not men are stronger than
scientist which diminishes in course of women is meaningless, because
time.
A. it is an already established fact that men
C. A work of art has no material value like are stronger than women.
the work of a scientist.
B. the word ‘stronger’ can be interpreted
D. A work of art is an expression of creative in various ways.
power while the work of a scientist is
C. it is difficult to assess the comparative
not.
strength of men and women.
Direction: In this section, you have five short
D. it is a dispute that might harm the man-
passages. After each passage, you will find some
woman relationship in our society.
questions based on the passage. First, read a
passage and answer the questions based on it. 87. The author says it would be legitimate to
You are required to select your answers based on argue that women are stronger than men,
the contents of the passage and opinion of the because
author only.
A. the author believes in the superior
Passage – 2 strength of women.
Most disputes about whether or not men are B. the author is not committed to any
stronger than women are meaningless because opinion.
the word ‘strong’ may mean many things. Most
C. in some of the activities women do give
men can surpass most women in lifting heavy
an impression that they are stronger
weights, in striking an object, in running, jumping
than men.
or doing heavy physical labour. But most women
live longer than most men, they have a better D. in fact women are inferior to men in
change of resisting disease, they can beat men at every respect.
operations requiring finger dexterity and the
ability to work accurately under monotonous 88. From the passage, which of the following
conditions. So it would be legitimate to argue statement is most likely to be correct?
that women are stronger than men. The truth is A. Women live longer than men because
that each gender can surpass the other in certain they can resist diseases better than
kinds of activities. To say that one is stronger men.
than the other is to indulge in futile arguments.
B. Monotonous living conditions make
85. Which one of the following statements best women stronger than men.
reflects the main contention of the author?
C. All women are incapable of running,
A. In most cases men are stronger than jumping and doing physical labour
women. because they are not strong.
B. Since women are healthier than men D. Statistically speaking, most women live
they are also stronger. longer than most men.

PAGE 1168
www.byjusexamprep.com

Direction: In this section, you have five short D. Spheres of life which are independent
passages. After each passage, you will find some and unconnected with one another.
questions based on the passage. First, read a
91. Which one of the following statements
passage and answer the questions based on it.
most correctly reflects the attitude of the
You are required to select your answers based on
author towards politicians’ opinions?
the contents of the passage and opinion of the
author only. A. The author totally disbelieves what the
politicians say.
Passage – 3
In national no less than in individual life there are B. The author believes what the politicians
no watertight compartments. No sharp lines can say.
be drawn to mark off the political from the C. The author is skeptical about the claims
moral, the social from the economic regions of of the politicians.
life. Politicians often talk as though one has only
to introduce certain political and economic D. The author thinks that the opinions of
changes for paradise to descend on earth, the politicians are contradictory.
forgetful of the fact that the efficiency of an 92. Which one of the following statements
institution depends on the way it is worked, most correctly indicates the implication of
which itself is determined by the character and the phrase ‘paradise to descend on earth’?
wisdom of the men who work it.
A. A world of perfect economic, political
89. Which one of the following statements and social well-being.
most clearly suggests the central theme of
the passage? B. A world ruled by religious persons.

A. Political and economic changes can C. A world of total liberty and equality.
solve all the problems facing the nation.
D. A world in which nobody needs to
B. There is no difference between the labour.
political, moral, social and economic
Direction: Read the passage carefully and select
regions of life.
the best answer to each question out of the four
C. It is not the institutions that are alternatives.
important but the character and
Just as some men like to play football or cricket,
wisdom of the people who manage
so some men like to climb mountains. This is
them.
often very difficult to do, for mountains are not
D. National progress depends solely on the just big hills. Paths are usually very steep. Some
efficient running of our institutions. mountainsides are straight up and down, so that
it may take many hours to climb as little as one
90. Which one of the following phrases best
hundred feet. There is always the danger that
helps to bring out the precise contextual
you may fall off and be killed or injured. Men
meaning of “watertight compartments”?
talk about conquering a mountain. It is a
A. Activities of life unaffected by public wonderful feeling to reach the top of a mountain
opinion. after climbing for hours and may be even for
days. You look down and see the whole country
B. Spheres of life where no liberty of
below you. You feel God-like. Two Italian
opinion is tolerated.
prisoners of war escaped from a prison camp in
C. Ways of life peculiar to each nation and Kenya during the war. They did not try to get
each section of society. back to their own country, for they knew that

PAGE 1169
www.byjusexamprep.com

was impossible. Instead, they climbed to the top the people who really helped civilization forward
of Mount Kenya, and then they came down again are often never mentioned at all. We do not
and gave themselves up. They had wanted to get know who first set a broken leg, or launched a
that feeling of freedom that one has, after seaworthy boat, or calculated the length of the
climbing a difficult mountain. year, or manoeuvred a field; but we know all
about the killers and destroyers. People think a
93. Some men like to climb mountains because
great deal of them, so much so that on all the
A. they do not like to play football or highest pillars in the great cities of the world you
cricket. will find the figure of a conqueror or a general or
a solider. And I think most people believed that
B. they know the trick of climbing. the greatest countries are those that have
C. they want to have a wonderful feeling. beaten in battle the greatest number of other
countries and ruled over them as conquerors.
D. they like to face danger.
97. People who are glorified often in history
94. To climb a mountain is often difficult books are those
because
A. who contributed to the public health.
A. mountains are big hills.
B. who contributed to the technical
B. it consumes more time. knowledge of man.
C. prisoners often escape from camps and C. who made calendars.
settle there.
D. who fought and won wars.
D. paths are steep and uneven.
98. The words “the people who really helped
95. It is wonderful feeling ……………’It’ refers to
civilization forward” suggest that
A. the steep path conquerors, generals, and soldiers

B. the prisoner A. contributed a great deal to civilization.


C. the mountain B. contributed only towards civilization.
D. mountaineering C. were least interested in the progress of
civilization.
96. Two Italian prisoners escaped the camp and
climbed to the top of Mount Kenya to D. contributed little to civilization.
A. escape to Italy. 99. We will find the figure of a conqueror or a
B. come down and give up. general or a soldier on all the highest pillars
in great cities because
C. gain fame as mountaineers.
A. they sacrificed their lives for the benefit
D. get the feeling of freedom. of humanity.
Direction: Read the passage carefully and select B. people have exaggerated notions about
the best answer to each question out of the four their achievements.
alternatives.
C. they had a deep concern for the welfare
Most of the people who appear most often and of humanity.
most gloriously in the history books are great
conquerors and generals and soldiers, whereas D. they built most cities.

PAGE 1170
www.byjusexamprep.com

100. The passage implies that the greatest 106. The writer(a)of this poetry(b)is
countries are those that Wordsworth.(c)No error.(d)
A. have conquered many countries and A. (a) B. (b)
ruled over them.
C. (c) D. (d)
B. are very large in their size.
107. The Jug (a) is made (b) out of China clay
C. have the largest population. (c)No error (d).
D. are civilized. A. (a) B. (b)
Direction: Each question in this section has a C. (c) D. (d)
sentence with three underlined parts labeled (a),
(b) and (c). Read each sentence to find out 108. Sita with(a)all her sisters(b)were here.(c)No
whether there is any error in any underlined part error.(d)
and indicate your response in the Answer Sheet A. (a) B. (b)
against the corresponding letter i.e., (a) or (b) or
(c). If you find no error, your response should be C. (c) D. (d)
indicated as (d).
109. As you know(a)that the ignorant(b)are
101. I am senior(a)than him(b)by two years.(c) easily duped.(c)No error.(d)
No error.(d)
A. (a) B. (b)
A. (a) B. (b)
C. (c) D. (d)
C. (c) D. (d)
110. Pay attention(a)to what(b)I am
102. When I finished writing the letter,(a)I could saying.(c)No error.(d)
not help admiring myself(b)to have
A. (a) B. (b)
achieved the impossible.(c)No error.(d)
A. (a) B. (b) C. (c) D. (d)

C. (c) D. (d) 111. One of the assistant(a)was(b)Absent.(c)No


error.(d)
103. I am(a)glad(b)that you are here.(c)No
error.(d) A. (a) B. (b)

A. (a) B. (b) C. (c) D. (d)

C. (c) D. (d) 112 Though George is a (a) honourable man (b)


his activities arouse suspicion (c) No error
104. He will be(a)cured from(b)his fever.(c)No (d)
error.(d)
A. (a) B. (b)
A. (a) B. (b)
C. (c) D. (d)
C. (c) D. (d)
113. She told her teacher that she(a)could not
105. Though he is a gifted comedian,(a)he be able to attend the class the(b) previous
prefers spend his spare time watching day because of heavy rains.(c)No error.(d)
horror(b)movies.(c)No error.(d)
A. (a) B. (b)
A. (a) B. (b)
C. (c) D. (d)
C. (c) D. (d)

PAGE 1171
www.byjusexamprep.com

114. Coleridge as well as Wordsworth were 118. The boy’s parents (a)pleaded with the
of(a)the opinion that the opposite of poetry Principal(b)that they were too poor to pay
is(b)not prose but science.(c)No error.(d) (c)his tuition fee. No error.(d)
A. (a) B. (b) A. (a) B. (b)
C. (c) D. (d) C. (c) D. (d)
115. He was courted arrest(a)in order to 119. For young Donald, (a)peace in
protest(b)against corruption among the Vietnam(b)was almost terrible as war.(c)No
government(c)servants. No error.(d) error.(d)
A. (a) B. (b) A. (a)
C. (c) D. (d) B. (b)
116. Mr. Joshi was, however, sure that the C. (c)
idea(a)would never work(b)in
D. (d)
practice.(c)No error.(d)
120. That Brutus, who was his trusted friend
A. (a) B. (b)
(a)had attacked on him caused
C. (c) D. (d) heartbreak(b)to Julius Caesar.(c)No
error.(d)
117. As I was leaving for Delhi,(a)he asked
me(b)whether I could buy a tape recorder A. (a)
for(c)him. No error.(d)
B. (b)
A. (a) B. (b)
C. (c)
C. (c) D. (d)
D. (d)

PAGE 1172
www.byjusexamprep.com

General Knowledge
1. Some Indian territory was transferred in 4. Which of the following statements with
2015 to Bangladesh by following which regard to the conduct of elections in India
procedure? is/are not correct?
A. By an agreement between the 1. The responsibility for the preparation of
Government of India and the the electoral rolls is vested in the
Government of Bangladesh Election Commission of India.
B. By a legislation passed by the 2. The Model Code of Conduct comes into
Parliament amending Schedule 1 to the existence as soon as the date of the
Constitution of India election is announced.
C. By amending Schedule 1 to the 3. The laws relating to delimitation of
Constitution of India by exercising constituencies are made by the Election
amending power of the Parliament Commission of India.
D. By amending Schedule 1 to the 4. No election shall be called in question
constitution of India by exercising except by an election petition.
amending power of the Parliament and
ratification by sixteen State Legislatures Select the correct answer using the code
given below.
2. The right to form associations and unions is
a right A. 3 only B. 1, 2 and 4
A. guaranteed to everybody C. 1, 2 and 3 D. 4 only
B. to freedom guaranteed to citizens only 5. Match List-I with List-II and select the
C. to equality before law correct answer using the code given below
the Lists:
D. to life and personal liberty
3. Which one of the following statements with
regard to the National Commission for
Scheduled Tribes is not correct?
A. The Union and every State Government
and every State Government shall
consult the Commission on all major
A. A-1 B-2 C-3 D-4
policy matters affecting Scheduled
Tribes. B. A-1 B-3 C-2 D-4
B. All the reports of the Commission and C. A-4 B-3 C-2 D-1
its recommendations shall be laid only
before Lok Sabha. D. A-4 B-2 C-3 D-1

C. The Commission while investigating any 6. Consider the following statements about
matter, has all the powers of a Civil Harappan cities:
Court. 1. Roads were not always absolutely
D. The Commission has the power to straight and did not always cross one
regulate its own procedures. another at right angles.

PAGE 1173
www.byjusexamprep.com

2. A striking feature is the uniformity in the B. The Supreme Court has advisory
average size of the bricks for houses and jurisdiction on a question of law or fact
for city walls. which may be referred to it by the
President of India.
Which of the statements given above is/are
correct? C. The Supreme Court has the power to
review its own judgment or order.
A. 1 only B. 2 only
D. Only the Supreme Court has the power
C. Both 1 and 2 D. Neither 1 nor 2
to issue writs to protect the
7. Which one of the following statements with fundamental rights of the people.
regard to the Ninth Schedule of the
10. Which of the following are constitutional
Constitution of India is not correct?
provisions and laws for the protection of
A. It was inserted by the Constitution (First the rights of the Scheduled Castes in India?
Amendment) Act, 1951.
1. Article 17 of the Constitution of India
B. The Acts and Regulations specified in
2. The Protection of Civil Rights Act, 1955
the Ninth Schedule shall become void
on the ground that it violates a 3. The Scheduled Castes and the
fundamental right in Part III of the Scheduled Tribes (Prevention of
Constitution. Atrocities) Act, 1989
C. The Supreme Court has the power of Select the correct answer using the code
judicial review of an Act included in the given below.
Ninth Schedule on the doctrine of basic
A. 1 and 3 only
structure.
B. 1 and 2 only
D. The appropriate Legislature can repeal
or amend an Act specified in the Ninth C. 1, 2 and 3
Schedule.
D. 2 and 3 only
8. The Sixth Schedule of the Constitution of
11. The members of NAFTA are
India pertains to the administration of tribal
areas in which of the following States? A. USA, Canada and Mexico
A. Assam, Meghalaya, Tripura and B. USA, Canada and India
Mizoram
C. USA, Canada and Japan
B. Meghalaya, Tripura, Manipur and
Mizoram D. USA, UK and India

C. Assam, Manipur, Meghalaya and 12. Who among the following is the Chairman
Tripura of the Fourteenth Finance Commission?

D. Manipur, Meghalaya, Tripura and A. C Rangarajan B. Vijay Kelkar


Arunachal Pradesh C. YV Reddy D. Rakesh Mohan
9. Which one of the following is not true about 13. Which among the following is/are credit
the powers of the Supreme Court? rating agency/agencies in India?
A. The Supreme Court has original and A. CRISIL B. CARE
exclusive jurisdiction in inter-
governmental disputes. C. ICRA D. All of these

PAGE 1174
www.byjusexamprep.com

14. In India, the term ‘hot money’ is used to 19. Which one of the following is correct?
refer to
A. Infantry Regiment is also known as the
A. Currency + Reserves with the RBI Mechanized Infantry Regiment.
B. Net GDR B. Crops of Electrical and Mechanical
Engineers is subsumed in the Corps of
C. Net Foreign Direct Investment
Engineers.
D. Foreign Portfolio Investment
C. Army Medical Corps and Army Dental
15. Which one of the following statements is Corps are two divisions of the Army
correct with respect to the composition of Service Corps.
national income in India?
D. Army has its own Corps of Air Defence.
A. The share of manufacturing sector has
20. Jammu and Kashmir Rifles is
declined.
A. an infantry regiment of the Indian Army
B. The share of services sector has
increased sharply. B. a battalion of the Rashtriya Rifles
C. The share of agriculture has remained C. the name of the Armed Police of the
static. State of Jammu and Kashmir
D. The share of services sector has D. a paramilitary force under the Ministry
declined. of Home Affairs
16. Which one of the following is not an Air 21. Who is the Chairman of the Chiefs of Staff
Defence Missile system? Committee?
A. Akash B. Trishul A. The chief of Army Staff
C. Tatra D. Astra B. The Chief of Naval Staff
17. Which one of the following statements is C. The Chief of Air Staff
correct?
D. The member who has been the longest
A. The rank of Second Lieutenant has been on the Committee
abolished.
22. Which one of the following is not correct in
B. The Chief of Army Staff carries the rank respect of Andaman and Nicobar
of Field Marshal. Command?
C. All officers posted to Andaman and A. It is the first integrated theatre
Nicobar Islands wear the ranks of the command in India.
Indian Navy.
B. Its headquarters is at Port Blair.
D. The rank of Major General is higher
C. It is commanded by a three-star officer.
than the rank of Lieutenant General.
D. It was set up by the British during the
18. Which one of the following is not one of the
Second World War.
Commands of the Indian Army?
23. Which one of the following statements is
A. South Western Command
not correct?
B. North Eastern Command
A. Creation of National Investment and
C. Central Command Infrastructure Fund (NIIF) was announ-
ced in the Union Budget, 2015-16.
D. Army Training Command (ARTRAC)

PAGE 1175
www.byjusexamprep.com

B. NIIF is a fund for enhancing 27. Which one of the following writs is issued
infrastructure facility in the country. by the Supreme Court to secure the
C. NIIF and NIF (National Investment Fund) freedom of a person upon unlawful arrest?
are the names of the same organization. A. Habeas Corpus
D. NIIF can have more than one alternative B. Mandamus
investment fund.
C. Certiorari
24. Which one of the following statements is
not true of the Protection of Women from D. Quo Warranto
Domestic Violence Act, 2005? 28. Which one of the following statements is
A. This Act provides civil remedies to not correct with respect to Article 32 of the
protect a woman subjected to domestic Constitution of India?
violence.
A. It provides remedies to citizens for the
B. Only women can make a complaint enforcement of fundamental rights.
under this legislation.
B. It is a part of fundamental rights.
C. Relief may be sought only against the
husband or a male live-in partner with C. The Supreme Court cannot refuse a writ
whom the woman has lived in a petition under Article 32 on the ground
domestic relationship. of delay.

D. The Act includes not just wives but also D. Protection under Article 32 also applies
women in marriage-like relationships. to the enforcement of ordinary law
which has nothing to do with the
25. Under Article 352 of the Constitution of
fundamental rights.
India, an emergency can be declared if
security of any part of India is threatened 29. Capital deepening refers to
by
A. going for more fixed capital per worker
1. war
B. emphasis on social overhead capital
2. external aggression
C. constant capital-output ratio
3. armed rebellion
D. increasing capital-output ratio
4. internal disturbance
30. According to the Classical Theory of
Select the correct answer using the code
Employment, deviations from the state of
given below.
full employment are
A. 1, 2 and 3 B. 2, 3 and 4
A. purely temporary in nature
C. 1, 3 and 4 D. 1 and 2 only
B. permanent in nature
26. According to the provisions of the
Constitution of India, which one of the C. imaginary situations
following is not a fundamental duty? D. normal situations
A. To respect the National Flag
31. Which of the following will be the outcome
B. To defend the country if an economy is under the inflationary
C. To provide education to one’s child pressure?

D. To promote village and cottage 1. Domestic currency heads for


industries depreciation.

PAGE 1176
www.byjusexamprep.com

2. Exports become less competitive with B. Members of the Missile Technology


imports getting costlier. Control Regime prevent the
proliferation of missile and unmanned
3. Cost of borrowing decreases. aerial vehicle technology capable of
4. Bondholders get benefitted. carrying above 500 kg payload for more
than 300 km.
Select the correct answer using the code
given below. C. China applied to join the Regime but
members did not offer the membership.
A. 1 and 2 B. 2 and 3
D. No other country outside the
C. 1 and 3 only D. 1, 3 and 4 membership follows the Regime rules.
32. Which of the following with regard to the 36. Which one of the following is the highest
term ‘bank run’ is correct? altitude zoo in the world?
A. The net balance of money a bank has in A. Cheyenne Mountain Zoo
its chest at the end of the day’s business
B. Pandit GB Pant High Altitude Zoo
B. The ratio of bank’s total deposits and C. Himalayan Zoological Park, Gangtok
total liabilities
D. Padmaja Naidu Himalayan Zoological
C. A panic situation when the deposit Park
holders start withdrawing cash from the
banks 37. Who among the following was defeated by
Chile to win the Copa America Football
D. The period in which a bank creates Championship, 2016?
highest credit in the market
A. Colombia B. Argentina
33. The headquarters of ‘Economic and Social
Commission for Asia and the Pacific’ is C. Ecuador D. Peru
located at 38. Rakshasa type of marriage is
A. Singapore B. Manila A. marriage by purchase
C. Bangkok D. Hong Kong B. marriage by capture
34. The first Tejas squadron of Indian Air Force C. marriage by giving dowry
consists of
D. marriage by seduction
A. 20 aircrafts with 4 in reserve 39. The epic, Silappadikaram refers to the
B. 100 aircrafts with 10 in reserve A. story of Rama
C. 20 aircrafts with no reserve B. Jaina elements in the storyline
D. 25 aircrafts with 5 in reserve C. culture of Sri Lankan Buddhists
35. Which one of the following statements D. cult of Shakti worship
about the Missile Technology Control
Regime is not correct? 40. Which one of the following was the earlier
name of Tokyo?
A. The Missile Technology Control Regime
A. Osaka B. Kyoto
is an informal and voluntary partnership
initially formed among G─7 countries. C. Samurai D. Edo

PAGE 1177
www.byjusexamprep.com

41. The ‘four great needs’ of the people as 45. Which of the following statements about
identified by the Kuomintang were the Deccan Riots of 1875 is/are true?
A. clothing, food, housing and 1. The Deccan Riots resulted in protection
transportation for peasants through the Deccan
Agriculturalists Relief Act of 1879.
B. education, food, housing and health
care 2. The riots did not spread to the whole of
Maharashtra because of prompt
C. food, housing, education and suppression by the British.
employment
3. The British were unable to contain the
D. employment, housing, education and riots and they spread throughout
health care Maharashtra.
42. The ‘water frame’ of Richard Arkwright was 4. The cotton boom in the Deccan that had
a device for been caused by the artificial demand
generated by the American Civil War
A. producing a new type of painting
caused the impoverishment of the
B. irrigating fields for rice cultivation peasants.
C. producing strong threads of yarn Select the correct answer using the code
given below.
D. the faster movement of steamships
A. 1 only B. 1 and 3
43. The term Upari refers to which one of the
following? C. 2 and 4 only D. 1, 2 and 4

A. A category of proprietary tenure under 46. Which of the following statements about
the Mughal rule the philosopher Shankara is/are true?
1. Shankara espoused a form of Vedanta
B. A category of tenancy tenure under the
called Advaita.
Maratha rule
2. He elaborated on the philosophy of
C. A soldier in the Maratha army
Gaudapada.
D. A village headman in the Mughal period 3. Shankara tried to demonstrate that the
44. Why are the Gypsies regarded as of Indian Upanishads and Brahmasutras contain a
origin? unified, systematic philosophy.

1. They follow Indian religious practices. 4. Shankara founded the Amanaya


Mathas.
2. Their language, called Romani, is an
Indo-Aryan language. Select the correct answer using the code
given below.
3. They believe that they came from India.
A. 1 and 2 only B. 1, 2 and 3
4. Epigraphic evidence locates their
C. 3 and 4 D. 4 only
original home in Uttar Pradesh.
47. Who among the following won the Men’s
Select the correct answer using the code
Singles Title in the Australian Open Tennis
given below.
Tournament, 2016?
A. 1 and 4 only B. 2 only A. Jamie Murray B. Bruno Soares
C. 1, 2 and 4 D. 2 and 3 C. Novak Djokovic D. Andy Murray

PAGE 1178
www.byjusexamprep.com

48. Recently the Government of India cleared Select the correct answer using the code
the proposal for the production of 18 given below.
indigenous ‘Dhanush’ artillery guns to be
A. 1, 2 and 3 B. 2, 3 and 4
produced in India by
C. 1 and 2 only D. 1 and 4
A. Indian Army
53. Which of the following fundamental rights
B. US Army
as enshrined in the Constitution of India
C. Indian Ordnance Factory Board belong only to the citizens?
D. Indian and US Army jointly 1. Article 19 (Protection of right to
49. Consider the following statement: freedom of speech)

“If you feel you are trapped in a black hole, 2. Article 21 (Protection of life and
don’t give up. There is a way out.” personal liberty)

Who among the following made the above 3. Article 15 (Prohibition of


statement? discrimination)

A. Albert Einstein B. Stephen Hawking 4. Article 16 (Equality of opportunity)

C. CV Raman D. Abdus Salam Select the correct answer using the code
given below.
50. Neil O’Brien, who died recently, was a
famous A. 1, 2 and 3

A. journalist B. billiards player B. 2, 3 and 4

C. dramatist D. quiz master C. 1, 3 and 4

51. Which one of the following statements is D. 1 and 4 only


not correct with respect to protection of 54. The basic structure doctrine with regard to
individuals being tried for offences? the Constitution of India relates to
A. A confession can never be used as 1. the power of judicial review
evidence against the accused.
2. the judgment in Kesavananda Bharati
B. The accused must have violated an case (1973)
existing law.
3. the constraints on Article 368 of the
C. An accused cannot be tried and Constitution of India
punished for the same offence again.
4. the judgment in Golaknath case (1967)
D. The quantum of punishment must be
provided in law as it existed on the date Select the correct answer using the code
of commission of an offence. given below.

52. A citizen of India will lose his or her A. 1, 2 and 3 B. 2, 3 and 4


citizenship if he or she C. 1 and 2 only D. 1 and 4
1. renounces Indian citizenship 55. What form of Shiva is most prominent in
2. voluntarily acquires the citizenship of the Brihadeshvara Temple built by the
another country Chola dynasty?

3. marries a citizen of another country A. Harihara B. Bhairava

4. criticizes the Government C. Rudra D. Tripurantaka

PAGE 1179
www.byjusexamprep.com

56. Match List-I with List-II and select the A. The Kitab-ul-Hind
correct answer using the code given below
B. Documents belonging to the Village
the Lists:
Panchayats of Rajasthan, Gujarat and
List-I List-II Maharashtra
(Historian) (Book) C. Sculptures from Mandor
A. Sumit 1. The Rise and D. The Ain-i-Akbari of Abul Fazl
Sarkar Growth of
Economic 59. Which one of the following statements is
Nationalism in not correct?
India A. BrahMos is a supersonic cruise missile
B. Shahid 2. A Rule of Property made by India and Russian Federation.
Amin of Bengal B. The name BrahMos came from two
C. Ranajit 3. The Swadeshi rivers, Brahmaputra of India and
Guha Movement in Moskva of Russia.
Bengal, 1903-1908 C. Indian military scientists were capable
D. Bipan 4. Event, Metaphor, of integrating BrahMos cruise missile
Chandra Memory-Chauri with Sukhoi-30 MKI fighter.
Chaura, 1922- D. Defence Research and Development
1992 Organization is the manufacturer of
Sukhoi-30 MKI fighter.

Codes: 60. Which one of the following statements


about reusable space vehicle of ISRO is not
A. A-3 B-4 C-2 D-1 correct?
B. A-3 B-2 C-4 D-1 A. In June 2016, ISRO successfully
C. A-1 B-2 C-4 D-3 launched India’s first reusable space
vehicle.
D. A-1 B-4 C-2 D-3
B. The space vehicle was launched from
57. The All India Census was Satish Dhawan Space Centre at
1. first attempted in 1872 Sriharikota in Andhra Pradesh.
2. regularly undertaken since 1881 C. The reusable launch vehicle climbed to
a height of 65 km before automatically
3. always undertaken as a five-year
steering back for landing.
exercise
D. The vehicle was initially boosted by a
Select the correct answer using the code
rocket that contained liquid fuel.
given below.
61. Which of the following voted for Brexit in
A. 1 only B. 2 and 3 only
June 2016?
C. 1 and 2 only D. 1, 2 and 3
A. England and Scotland
58. Which one of the following sources tells us
B. England and Wales
about women protesting the infidelity of
their husbands or the neglect of the wife C. Scotland and Northern Ireland
and children by the male head of the D. Wales and Northern Ireland
household?

PAGE 1180
www.byjusexamprep.com

62. Which one of the following statements is Select the correct answer using the code
not correct? given below.
A. Centre for Science and Environment in A. 2, 3 and 4 B. 1, 2 and 3
a recent study found that a number of
commonly available brands of C. 1 and 3 only D. 1 and 2 only
prepackaged breads tested positive for 65. Which among the following Acts were
potassium bromated and potassium repealed by Article 395 of the Constitution
iodate. of India?
B. Food Safety and Standards Authority of
1. The Government of India Act, 1935
India recommended to the Health
Ministry to remove potassium 2. The Indian Independence Act, 1947
bromated from the list of permissible
3. The Abolition of Privy Council
food additives.
Jurisdiction Act, 1949
C. The Union Government banned the use
of potassium bromated as a food 4. The Government of India Act, 1919
additive on the basis of the above Select the correct answer using the code
recommendation. given below.
D. Potassium bromated is a neurotoxin. A. 1 and 2 only B. 1 and 3 only
63. Which of the following statements
C. 1, 2 and 3 only D. 1, 2, 3 and 4
regarding the Constituent Assembly of
India is/are correct? 66. Which of the following Gods are also known
1. The Assembly was elected indirectly by as Lokapalas or the Guardians of the
the members of the Provincial Universe?
Legislative Assemblies. A. Yama, Indra, Varuna and Kubera
2. The elections were held on the basis of B. Indra, Varuna, Skanda and Kubera
Universal Adult Franchise.
C. Indra, Varuna, Yama and Brahma
3. The scheme of election was laid down
by the Cabinet Delegation. D. Yama, Shiva, Kubera and Indra
4. The distribution of seats was done 67. Which one of the following does not signify
based on the Mountbatten Plan. a battle tank?
Select the correct answer using the code A. T─55 B. T─155
given below.
C. T─72 D. T─90
A. 1 only B. 1, 2 and 3
C. 2 and 4 D. 1 and 3 only Directions: The following three (3) items consist
of two statements, Statement I and Statement II.
64. On 26th November, 1949, which of the Examine these two statements carefully and
following provisions of the Constitution of select the correct answer using the code given
India came into effect? below.
1. Citizenship
68. Statement I: King Ashoka abolished capital
2. Elections punishment and disbanded his army.
3. Provisional Parliament Statement II: After Kalinga War, Ashoka
4. Fundamental Rights was remorseful and become a Buddhist.

PAGE 1181
www.byjusexamprep.com

A. Both the statements are individually D. Statement I is false but Statement II is


true and Statement II is the correct true
explanation of Statement I
71. Which one of the following statements
B. Both the statements are individually concerning research centres in India is not
true but Statement II is not the correct correct?
explanation of Statement I
A. Rajiv Gandhi Centre for Biotechnology is
C. Statement I is true but Statement II is located at Kolkata.
false B. Central Arid Zone Research Institute is
D. Statement I is false but Statement II is located at Jodhpur.
true C. Centre for Social Forestry and Eco-
69. Statement I: The Zamindars were an Rehabilitation is located at Allahabad.
exploitative class in Mughal India. D. Institute of Forest Productivity is
Statement II: The Zamindars often received located at Ranchi.
the support of the peasantry in a large 72. Which one of the following statements
number of agrarian uprisings in North India concerning the natural vegetation of India
in the seventeenth century. is not correct?
A. Both the statements are individually A. Sal is found in moist deciduous forests.
true and Statement II is the correct
explanation of Statement I B. Casuarina is largely found along the
coastal region of Tamil Nadu.
B. Both the statements are individually
true but Statement II is not the correct C. Deodar is a coniferous species of tree.
explanation of Statement I D. Shola forests are found mainly on the
C. Statement I is true but Statement II is upper reaches of Himalaya.
false 73. Which one of the following is correct?
D. Statement I is false but Statement II is A. Guindy National Park is known for tiger
true reserve.
70. Statement I: The Pahi-kashta peasants were B. Namdapha National Park is meant for
non-resident cultivators cultivating lands lion conservation.
on a contractual basis.
C. Jaldapara is a biosphere reserve.
Statement II: The Pahi-kashta peasants D. Rann of Kutch is a Wild Ass Sanctuary.
worked under the temptation of favourable
terms of revenue or the compulsion of 74. Which one of the following is not related to
economic distress. wind erosion?

A. Both the statements are individually A. Wind gap B. Zeugen


true and Statement II is the correct C. Dreikanter D. Demoiselle
explanation of Statement I
75. Which one of the following statements is
B. Both the statements are individually not correct?
true but Statement II is not the correct
explanation of Statement I A. Position of the Sun, Earth and Moon in
a straight line of 180 ° angle is known as
C. Statement I is true but Statement II is syzygy.
false

PAGE 1182
www.byjusexamprep.com

B. Syzygy conjunction causes solar eclipse. 79. Match List-I with List-II and select the
correct answer using the code given below
C. Syzygy opposition causes lunar eclipse.
the List:
D. Syzygy conjunction occurs at the time of
perihelion only.
76. Which of the following statements
regarding soil is/are correct?
1. Alluvial soils are rich in nitrogen
Codes:
content.
A. A-3 B-4 C-1 D-2
2. Black soils are rich in iron and lime but
deficient in nitrogen. B. A-2 B-1 C-4 D-3

3. Laterite soils are rich in iron and C. A-3 B-1 C-4 D-2
aluminium but deficient in nitrogen and D. A-2 B-4 C-1 D-3
potassium. 80. Match List-I with List-II and select the
Select the correct answer using the code correct answer using the code given below
given below. the Lists:
A. 1 and 2 only B. 3 only
C. 2 and 3 only D. 1, 2 and 3
77. Match List-I with List-II and select the
correct answer using the code given below
the Lists:
Codes:
A. A-4 B-2 C-3 D-1
B. A-1 B-3 C-2 D-4
C. A-1 B-2 C-3 D-4
D. A-4 B-3 C-2 D-1
Codes: 81. Light travels in a straight line. This
statement does hold if the medium of
A. A-4 B-3 C-2 D-1 travel for light is _________.
B. A-1 B-3 C-2 D-4 A. of variable refractive index
C. A-1 B-2 C-3 D-4 B. made up of slabs of different refractive
indices
D. A-4 B-2 C-3 D-1
C. homogeneous and transparent
78. Which one of the following fishing banks is
located off the coast of the United D. inhomogeneous and transparent
Kingdom? 82. Liquids and gases never show
A. Great Fisher Bank A. diamagnetic property
B. Grand Banks B. paramagnetic property
C. Dogger Bank C. ferromagnetic property

D. Reed Bank D. electromagnetic property

PAGE 1183
www.byjusexamprep.com

83. The energy of a photon, whose momentum 88. Which of the following statements
is 10 MeV/c, where c is the speed of light, is concerning atmosphere of the earth are
given by correct?
A. 10 MeV B. 100 MeV 1. In stratosphere, temperature increases
C. 1 MeV D. 0.1 MeV with altitude.

84. In total internal reflection, the light travels 2. In mesosphere, temperature decreases
from with altitude.
A. rarer to denser medium and it occurs 3. The lowest temperature of the
with no loss of intensity atmosphere is recorded in the upper
part of mesosphere.
B. denser to rarer medium and it occurs
with no loss of intensity 4. Tropopause is an isothermal zone.
C. rarer to denser medium and it occurs Select the correct answer using the code
with loss of intensity given below.
D. denser to rarer medium and it occurs A. 1 and 2 only
with loss of intensity
B. 1, 2 and 3 only
85. Which one of the following pairs of lakes is
connected by Soo Canal? C. 3 and 4 only

A. Superior and Michigan D. 1, 2, 3 and 4

B. Superior and Huron 89. Match List-I with List-II and select the
correct answer using the code given below
C. Huron and Ontario the Lists:
D. Huron and Erie
86. Which one of the following is not a tributary
of the river Ganga?
A. Son B. Mahananda
C. Teesta D. Sharda Codes:
87. Match List-I with List-II and select the A. A-4 B-2 C-3 D-1
correct answer using the code given below
the Lists: B. A-1 B-3 C-2 D-4
C. A-1 B-2 C-3 D-4
D. A-4 B-3 C-2 D-1
90. If the latitude and longitudinal extent of an
Indian State/UT is 15 °48’00’’ N to 14
°53’15’’ N and 74 °20’13’’ E to 74 °40’33’’ E,
then which one of the following is that
Codes: State/UT?
A. A-4 B-2 C-3 D-1 A. Puducherry
B. A-4 B-3 C-2 D-1 B. Chandigarh
C. A-1 B-2 C-3 D-4 C. Goa
D. A-1 B-3 C-2 D-4 D. Delhi

PAGE 1184
www.byjusexamprep.com

91. With regard to water transportation in 95. Match List-I with List-II and select the
India, which of the following statements correct answer using the code given below
are correct? the Lists:
1. Headquarters of Central Inland Water
Transport Corporation is located at
Delhi.
2. Headquarters of Inland Waterways
Authority of India is located at Noida.
3. National Inland Navigation Institute is
located at Kolkata.
4. First National Inland Waterway is Codes:
between Haldia and Allahabad. A. A-2 B-4 C-1 D-3
Select the correct answer using the code B. A-2 B-1 C-4 D-3
given below.
C. A-3 B-4 C-1 D-2
A. 1 and 2 only B. 2 and 3
D. A-3 B-1 C-4 D-2
C. 2 and 4 only D. 1, 2 and 4
96. ‘Altitude sickness’ is caused at high altitude
92. Which of following is the correct order of due to ____.
elemental composition of an adult human
A. high partial pressure of oxygen
body by mass?
B. low partial pressure of oxygen
A. C > O > H > N B. O > C > H > N
C. low level of haemoglobin
C. N > C > H > O D. N > C > O > H
D. high partial pressure of carbon dioxide
93. In case of a standard hydrogen electrode
97. Biological catalysts are protein molecules
A. absolute electrode potential is not zero which
B. absolute electrode potential is zero 1. speed up a chemical reaction
C. both absolute and standard electrode 2. remain unchanged after reaction
potential values are zero
3. function optimally at 37 °C
D. electrode potential is zero only at 25 °C
4. do not have an enzymatic activity
94. Which one of the following statements is
correct? Select the correct answer using the code
given below.
A. The oxidation number for hydrogen is
always zero A. 1 only B. 1, 2 and 3

B. The oxidation number for hydrogen is C. 2 and 3 only D. 1, 3 and 4


always +1 98. Which of the following cells do not contain
C. The oxidation number for hydrogen is nucleus?
always ─1 1. Root hair cells
D. Hydrogen can have more than one 2. Red blood cells
oxidation number

PAGE 1185
www.byjusexamprep.com

3. Platelets 103. In a bipolar junction transistor


4. Monocytes A. all the three regions (the emitter, the
base and the collector) have equal
Select the correct answer using the code concentrations of impurity
given below.
B. the emitter has the least concentration
A. 1 only B. 2 and 3 only of impurity
C. 1, 2 and 4 D. 2, 3 and 4 C. the collector has the least
99. Which one of the following hormones is concentration of impurity
essential for the uptake of glucose by cells D. the base has the least concentration of
in the human body? impurity
A. GH B. TSH 104. In the gamma decay of a nucleus
C. Insulin D. Cortisol A. the mass number of the nucleus
100. Which one of the following statements is changes whereas its atomic number
not correct? does not change

A. The response of the ear to sound of B. the mass number of the nucleus does
increasing intensity is approximately not change whereas its atomic number
logarithmic. changes
C. both the mass number and the atomic
B. The sensitivity of the human ear does
number of the nucleus changes
not vary with the frequency of the
sound. D. neither the mass number nor the
atomic number of the nucleus changes
C. When two or more waves traverse the
same medium, the displacement of any E. None of the above/More than one of
element of the medium is the algebraic the above
sum of the displacements due to each
105. The spring constant of a spring depends on
wave.
its
D. Longitudinal waves can travel in all
A. length only
media-solids, liquids and gases.
B. material only
101. Which one of the following physical
quantities has the same dimensions as that C. length and its diameter
of Planck’s constant h?
D. thickness, its diameter and its material
A. Linear momentum
106. Which of the following pairs of Indian State
B. Angular momentum and tribal population are correct?
C. Force 1. Madhya Pradesh: Largest tribal
population
D. Potential energy
2. Mizoram: Highest percentage of tribal
102. The pressure exerted by a 760 mm column population out of the total population
of mercury at 0 °C is known as of the State
A. 1 pascal B. 1 atmosphere 3. Chhattisgarh: Second largest tribal
C. 1 bar D. 1 poise population after Madhya Pradesh

PAGE 1186
www.byjusexamprep.com

4. Arunachal Pradesh: Second highest 110. Excessive use of which of the following
percentage of tribal population out of fertilizers may be responsible for the
the total population of the State after presence of a toxic substance in
Mizoram groundwater?
Select the correct answer using the code A. Nitrogen
given below.
B. Phosphate only
A. 1 and 2 only B. 2 and 4 only
C. Potassium only
C. 1, 2 and 3 D. 2, 3 and 4 D. Phosphate and potassium
107. With reference to earthquakes, which of 111. Deficiency of fluoride leads to which one of
the following statements are correct? the following health problems?
1. Earthquakes largely occur along the A. Tooth caries
converging plate boundaries.
B. Mottling of tooth
2. The point of origin of the earthquake in
the lithosphere is known as C. Bending of bones
focus/hypocentre. D. Stiffening of joints
3. The intensity of earthquake decreases 112. Dolomite powder is applied in some
with distance from the epicentre. agricultural lands. The purpose of applying
4. The epicentre of earthquakes always it is to
remains over continents only. A. increase the pH of the soil
Select the correct answer using the code B. lower the pH of the soil
given below.
C. increase the phosphorus content of the
A. 1 and 2 only soil
B. 1, 2 and 3 D. increase the nitrogen content of the soil
C. 2 and 3 only 113. In paper manufacturing, degumming of the
raw material is done using ___.
D. 1 and 4
A. Sulphuric acid B. Bleaching powder
108. According to Census 2011, which one of the
following is the correct sequence of States C. Caustic soda D. Nitric acid
in decreasing order of their total size of
114. Which one of the following is not an
population?
allotrope of carbon?
A. Kerala, Jharkhand, Assam, Punjab A. Coal B. Diamond
B. Jharkhand, Kerala, Assam, Punjab C. Graphite D. Graphene
C. Kerala, Jharkhand, Punjab, Assam 115. An irregular mode of reproduction resulting
D. Jharkhand, Kerala, Punjab, Assam in the development of an embryo without
fertilization is called
109. Which one of the following elements does
not form solid hydrogen carbonate? 1. Parthenogenesis

A. Sodium B. Potassium 2. Apogamy

C. Caesium D. Lithium 3. Sporophytic budding

PAGE 1187
www.byjusexamprep.com

Select the correct answer using the code 118. Which one of the following is the largest
given below. source of electricity in India?
A. 1 only B. 2 only A. Hydropower plants

C. 2 and 3 only D. 1, 2 and 3 B. Nuclear power plants

116. Which one of the following air pollutants C. Thermal power plants
combines with the haemoglobin of human D. Wind energy
blood and reduces its oxygen-carrying
capacity, leading to suffocation and may 119. Which one of the following is most sensitive
cause even death? to environmental change?

A. Chlorofluorocarbon A. Amphibian

B. Fly ash B. Reptile

C. Carbon monoxide C. Mammal


D. Insect
D. Sulphur dioxide
120. Which one of the following gases is
117. The HIV virus weakens the immunity of a
released mostly from landfills in urban
person because it destroys
areas?
A. mast cells A. Nitrogen
B. platelets B. Hydrogen
C. erythrocytes C. Methane
D. lymphocytes D. Oxygen

PAGE 1188
www.byjusexamprep.com

Elementary Mathematics

1. If then what is equal to? 7. Consider the following statements :


1. The classes of type 15-19, 20-24, 25-29
etc. are exclusive classes.
2. The classes of type 15-20, 20-25, 25-30
A. B. etc. are inclusive classes.
Which of the above statements is/are
C. D.
correct?

2. A motorist travels to a place 150 km away A. 1 only B. 2 only


at an average speed of 50 km/hour and C. Both 1 and 2 D. Neither 1 nor 2
returns at 30 km/hour. What is the average
speed for the whole journey? 8. A pie chart depicts the classification of total
funds of an organization according to
A. 35 km/hour B. 37 km/hour different sources of funds. A particular
sector of pie chart for corporate tax has 108
C. 37.5 km/hour D. 40 km/hour
angle at the centre. What is the percentage
3. What is the difference between the sum of of income from corporate tax to total
the cubes and that of squares of first ten funds?
natural numbers?
A. 20% B. 25%
A. 2280 B. 2640 C. 30% D. 35%
C. 3820 D. 4130 9. In a class of 100 students, there are 70 boys
4. What is the remainder when whose average marks in a subject are 75. If
the average marks of the complete class is
is divided by 29?
72, then what is the average marks of the
A. 8 B. 5 girls?
C. 3 D. 0 A. 64 B. 65

5. Let S be a set of first ten natural numbers. C. 68 D. 74


What is the possible number of pairs (a, b) 10. Number of credit cards held by an
where such that the product individual can be treated as
leaves remainder 4 when divided
A. Qualitative data
by 12?
B. Discrete data
A. 4 B. 6
C. Categorical data
C. 8 D. 10 D. None of the above
6. Suppose the class interval 10-15 has 11. The mean of 20 observations is 17. On
frequency 30, then what is the frequency checking it was found that the two
density of this class interval? observation were wrongly copied as 3 and
A. 2 B. 3 6. If wrong observation are replaced by
correct values 8 and 9, then what is the
C. 5 D. 6 correct mean?

PAGE 1189
www.byjusexamprep.com

A. 17.4 B. 16.6 17. A pole stands vertically inside a triangular


park ABC. If the angle of elevation of the top
C. 15.8 D. 14.2
of the pole from each corner of the park is
12. A cricketer has a certain average of 10 same, then in the triangle ABC, the foot of
innings. In the eleventh inning he scored the pole is at the
108 runs, thereby increasing his average by
A. Centroid
6 runs. What is his new average?
B. Circumference
A. 42 B. 47
C. 48 D. 60 C. Incentre

13. The median of set of 9 distinct observations D. Orthocentre


is 20.5. If each of the largest 4 observation 18. Consider the following :
of the set is increased by 2, then the median
of the new set 1.
A. is increased by 2
B. is decreased by 2 2.

C. is two times the original median


3.
D. remains the same as that of original set
14. For x > 0, if a variable takes discrete values Which of the above are correct?
x + 4, x – 3.5, x – 2.5, x – 3, x – 2, x + 0.5, A. 1 and 2 only
x – 0.5, x + 5, then what is the median?
B. 2 and 3 only
A. x – 1.25
C. 1 and 3 only
B. x – 0.5
D. 1, 2 and 3
C. x + 0.5
19. If the length of the shadow of a tower is
D. x + 1.25
equal to its height, then what is the Sun’s
15. The average score of class X is 83. altitude at that time?
The average score of class Y is 76 A. 15o B. 30o
The average score of class Z is 85 C. 45o D. 60o
The average score of class X and Y is 79
20. If and , then
And average score of class Y and Z is 81.
what is the value of X?
What is the average score of X, Y and Z?
A. 15o B. 30o
A. 81.5 B. 80.5
C. 45o D. 60o
C. 79.0 D. 78.0
21. Consider the following
16. What is the value of
1. sin 1o > sin 1c
?
A. 0 B. 1 2. cos 1o < cos 1c
3. tan 1o > tan 1c
C. 2 D.

PAGE 1190
www.byjusexamprep.com

Which of the above are not correct?


A. B.
A. 1 and 2 only
B. 2 and 3 only C. D.

C. 1 and 3 only 28. A man from the top of a 100 m high tower
sees a car moving towards the tower at an
D. 1, 2 and 3 angle of depression 30 After some time, the
angle of depression becomes 60o. What is
22. What is equal to?
the distance travelled by the car during this
time?
A. B.
A. B.
C. D.
C. D.
23. If where then
29. If sinx + sin2x = 1; then what is the value of
which one of the following is correct?
(cos8x + 2cos6x + cos4x)?
A. B. A. 0 B. 1
C. 2 D. 4
C. D.
30. If , then what is
24. If where
equal to?
then what is equal to?
A. 0 B.
A. B.
C. 1 D.
C. D. 31. ABCDA is a con-cyclic quadrilateral of a
circle ABCD with radius r and centre at O. If
25. If and AB is the diameter and CD is parallel and
then what is equal to? half of AB and if the circle completes one
rotation about the centre O, then the locus
A. 1 B. 2 of the middle point of CD is a circle of radius
C. 4 D. 8
A. B.
26. What is the value of
?
C. D.
A. 0 B.
32. If the perimeter of a circle is equal to that
C. 1 D. 2 of a square, then what is the ratio of area of
circle to that of square?
27. Two men on either side of a tower 75 m
high observe the angle of elevation of the A. 22 : 7 B. 14 : 11
top of the tower to be 30o and 60o What is
C. 7 : 22 D. 11 : 14
the distance between the two men?

PAGE 1191
www.byjusexamprep.com

33. What is the area of a triangle with sides of 39. What is area of largest triangle inscribed in
length 12 cm, 13 cm and 5 cm? a semi-circle of radius units?
A. 30 cm2 B. 35 cm2 A. square units
C. 40 cm2 D. 42 cm2 B. square units
34. The wheels of a car are of diameter 80 cm C. square units
each. The car is travelling at a speed of 66
km/hour. What is the number of complete D. square units
revolutions each wheel makes in 10 40. The sides of a triangle are given by
minutes?
and where a, b, c
A. 4275 B. 4350 are positive. What is the area of the triangle
C. 4375 D. 4450 equal to?

Direction: A tent of a circus is made of canvas A.


and is in the form of right circular cylinder and
right circular cone above it. The height and
B.
diameter of the cylindrical part of the tent are 5
m and 126 m respectively. The total height of the
tent is 21 m. C.
35. How many square metres of canvas are
used? D.
A. 14450 B. 14480
41. What is the volume of a sphere of radius 3
C. 14580 D. 14850 cm?
36. What is the curved surface area of the A. B.
cylinder?
C. D.
A. 1980 m2 B. 2010 m2
42. If two tangents inclined at an angle 60o are
C. 2100 m2 D. 2240 m2
drawn to a circle of radius 3 cm, then what
37. What is the slant height of the cone? is the length of each tangent?

A. 60 m B. 65 m A. B.
C. 68 m D. 70 m C. 6 cm D.
38. The diameter of the front wheel of an 43. The height of a cone is 60 cm. A small cone
engine is 2x cm and that of rear wheel is 2y is cut off at the top by a plane parallel to the
cm. To cover the same distance, what is the
base and its volume is the volume of
number of times the rear wheel revolves
when the front wheel revolves n times? original cone. What is the height from the
base at which the section is made?
A. B.
A. 15 cm B. 20 cm

C. D. C. 30 cm D. 45 cm

PAGE 1192
www.byjusexamprep.com

44. Rain water from a roof 22 m 49. The cost of painting a spherical vessel of
20 m drains into a cylindrical vessel having diameter 14 cm is Rs. 8008. What is the cost
diameter of base 2 m and height 3.5 m. If of painting per square centimeter?
the vessel is just full, what is the rainfall?
A. Rs. 8 B. Rs. 9
A. 3.5 cm B. 3 cm
C. Rs. 13 D. Rs. 14
C. 2.5 cm D. 2 cm
50. The radius of the base and the height of a
45. In an equilateral triangle another
solid right circular cylinder are in the ratio
equilateral triangle is drawn inside joining
2 : 3 and its volume is 1617 cm3. What is the
the mid-points of the sides of given
total surface area of the cylinder?
equilateral triangle and the process is
continued up to 7 times. What is the ratio A. 462 cm2 B. 616 cm2
of area of fourth triangle to that of seventh
C. 770 cm2 D. 886 cm2
triangle?
A. 256 : 1 B. 128 : 1 51. A building is in the form of a cylinder
surmounted by a hemispherical dome on
C. 64 : 1 D. 16 : 1 the diameter of the cylinder. The height of
46. A drinking glass of height 24 cm is in the the building is three times the radius of the
shape of frustum of a cone and diameters base of the cylinder. The building contains
of its bottom and top circular ends are 4 cm of air. What is the height of the
and 18 cm respectively. If we take capacity
of the glass as , then what is the building?

value of x? A. 6 m B. 4 m
A. 824 B. 1236 C. 3 m D. 2 m
C. 1628 D. 2472 52. What is the number of rounds that a wheel
47. A circle and a square have the same of diameter 5/11 m will make in traversing
perimeter. Which one of the following is 7 km?
correct? A. 3300 B. 3500
A. Their areas are equal
C. 4400 D. 4900
B. The area of the circle is larger
53. In a circle of radius 3 units, a diameter AB
C. The area of the square is times area intersects a chord of length 2 units
perpendicularly at P. If AP > BP, then what
of circle is the ratio of AP to BP?
D. The area of the square is times area
of circle A. B.

48. If a quadrilateral has an inscribed circle, C. D.


then the sum of a pair of opposite sides 54. Two circle touch externally and sum of their
equals areas is 130π cm2 and the distance between
A. Half the sum of the diagonals their centres is 14 cm. What is the
difference in the radii of the circles?
B. Sum of the other pair of opposite sides
C. Sum of two adjacent sides A. 5 cm B. 6 cm

D. None of the above C. 7 cm D. 8 cm

PAGE 1193
www.byjusexamprep.com

55. Let ABC be a right angled triangle with BC = 61. If the mean age of combined group of boys
5 cm and AC = 12 cm. Let D be a point on and girls is 18 years and the mean of age of
the hypotenuse AB such that . boys is 20 and that of girls is 16, then what
What is length of CD? is the percentage of boys in the group?
A. 60 B. 50
A. 5 cm B. cm
C. 45 D. 40
60
C. cm D. cm 62. The speeds of three cars are in the ratio 2 :
13 3 : 4. What is the ratio between the times
56. Into a conical tent of radius 8.4 m and taken by these cars to travel the same
vertical height 3.5 m, how many full bags of distance?
wheat can be emptied, if space required for A. 4 : 3 : 2 B. 2 : 3 : 4
the wheat in each bag is 1.96 m3?
C. 4 : 3 : 6 D. 6 : 4 : 3
A. 264 B. 201
63. A train is travelling at 48 km/hour
C. 132 D. 105 completely crosses another train having
57. A cubic metre of copper weighing 9000 kg half its length and travelling in opposite
is rolled into a square bar 9 m long. An exact direction at 42 km/hour in 12 s. It also
cube is cut off from the bar. How much passes a railway platform in 45 s. What is
does the cube weigh? the length of the platform?

A. 1000 kg B. A. 600 m B. 400 m


C. 300 m D. 200 m
C. 300 kg D. 64. A tank can be filled by pipe X in 2 hours and
pipe Y in 6 hours. At 10 a.m. pipe X was
58. The ratio of the curved surface area to the opened. At what time will the tank be filled
total surface area of a right circular cylinder if pipe Y is opened at 11 a.m.?
is 1 : 2, If the total surface area is 616 cm2,
what is the volume of the cylinder? A. 12:45 hours B. 5:00 p.m.

A. 539 cm3 B. 616 cm3 C. 11:45 a.m. D. 11:50 a.m.

C. 1078 cm3 D. 1232 cm3 65. X and Y entered into partnership with Rs.
700 and Rs. 600 respectively. After 3
59. If a square of side x and an equilateral months X withdrew 2/7 of his stock but
triangle of side y are inscribed in a circle, after 3 months, he puts back 3/5 of what he
then what is the ratio of x to y? had withdrawn. The profit at the end of the
year is Rs. 726. How much of this should X
A. B.
receive?
A. Rs. 336 B. Rs. 366
C. D.
C. Rs. 633 D. Rs. 663
2
60. The area of four walls of a room is 120 m .
66. Incomes of Mahesh and Kamal are in the
The length of the room is twice its breadth.
ratio 1 : 2 and their expenses are in the ratio
If the height of the room is 4 m, what is area
1:3. Which one of the following statements
of the floor?
is correct ?
A. 40 m2 B. 50 m2
A. Mahesh saves more than what Kamal
C. 60 m2 D. 80 m2 saves

PAGE 1194
www.byjusexamprep.com

B. Savings of both of them are equal A. F only B. G only


C. Kamal saves more than what Mahesh C. Both F and G D. neither F nor G
saves
73. What are the roots of the equation
D. It is not possible to determine who x
x −2 −
saves more 2 x
.27 1 =9?
67. If then what is the value of 3 2
A. 2,1 − log   B. 2,1 − log  
? 2 3

 log 3   log2 
C. −2,1 −   D. −2,1 −  
A. B.  log2   log 3 
74. Consider the following statements in
C. D. respect of positive odd integers x and y ;

68. What is the unit digit of 7139? Which of the above statements is/are
correct?
A. 9 B. 7
1. is even integer
C. 6 D. 3
2. is divisible by 4
69. The number of digits in 330 is n and it is
given that . What is the A. 1 only B. 2 only
value of n? C. Both 1 and 2 D. Neither 1 nor 2
A. 13 B. 14 75. A person can row downstream 20 km in 2
C. 15 D. 16 hours and upstream 4 km in 2 hours. What
is the speed of the current?
70. If and ,
A. 2 km/hour B. 2.5 km/hour
then what is the value of ?
C. 3 km/hour D. 4 km/hour
A. 5 B.
76. What is

C. D.
equal to?

71. is divisible by
A. B.
which one of the following integers?
A. 3 B. 5 C. D.
C. 7 D. 11
77. There are two numbers P and q such that
72. There are twelve friends A, B, C, D, E, F, G, their HCF is 1. Which of the following
H, I, J, K and L who invested money in some statements are correct?
business in the ratio of 1 : 2 : 3 : 4 : 5 : 6 : 7
: 8 : 9 : 10 : 11 : 12 and the duration for 1. Both p and q may be prime.
which they invested the money is in the 2. One number may be prime and the
ratio of 12 : 11 : 10 : 9 : 8 : 7 : 6 : 5 ; 4 : 3 : 2 other composite.
: 1 respectively. Who will get the maximum
3. Both the numbers may be composite.
profit at the end of the year?

PAGE 1195
www.byjusexamprep.com

Select the correct answer using the code A. B.


given below :
A. 1 and 2 only B. 2 and 3 only C. D.
C. 1 and 3 only D. 1, 2 and 3
83. When the speed of a train is increased by
78. A shopkeeper increases the cost price of an 20%, it takes 20 minutes less to cover the
item by 20% and offers a discount of 10% same distance. What is the time taken to
on this marked price. What is his cover the same distance with the original
percentage gain? speed?
A. 15% B. 12% A. 140 minutes B. 120 minutes
C. 10% D. 8% C. 100 minutes D. 80 minutes

79. Which one of the following is correct in 84. A can do 50% more work than B in the same
respect of the number 1729? time. B alone can do a piece of work in 30
hours. B starts working and had already
A. It cannot be written as the sum of the worked for 12 hours when A joins him. How
cubes of two positive integers many hours should B and A work together
B. It can be written as the sum of the cubes to complete the remaining work?
of two positive integers in one way only A. 6 hours B. 12 hours
C. It can be written as the sum of the cubes C. 4.8 hours D. 7.2 hours
of two positive integers in two ways 85. In an office, one-third of the workers are
only women, half of the women are married and
D. It can be written as the sum of the cubes one-third of the married women have
of two positive integers in three ways children. If three-fourth of the men are
only married and one-third of the married men
have children, then what is the ratio of
80. The cost of 2.5 kg rice is Rs. 125. The cost of married women to married men?
9 kg rice is equal to that of 4 kg pulses. The
cost of 14 kg pulses is equal to that of 1.5 kg A. 1 : 2 B. 2 : 1
tea. The cost of 2 kg tea is equal to that of 5 C. 3 : 1 D. 1 : 3
kg nuts. What is the cost of 11 kg nuts?
86. What is the remainder when 2100 is divided
A. Rs. 2310 B. Rs. 3190 by 101?
C. Rs. 4070 D. Rs. 4620 A. 1 B. 11
81. If p and q are the roots of , C. 99 D. 100
then which one of the following is correct? 87. If α and β are the two zeros of the
A. or 1 B. only polynomial 25x2 + 15x + 2 then what is a
quadratic polynomial whose zeros are
C. or 0 D. only (2α)–1 and (2β)–1?
82. A candidate scoring x% marks in an A. B.
examination fails by a marks while another
candidate who scores y% marks gets b C. D.
marks more than the minimum required 88. Which one of the following rational
pass marks. What is the maximum marks numbers has non-terminating and
for the examination? repeating decimal expansion?

PAGE 1196
www.byjusexamprep.com

A. B. A. 252 B. 256
C. 258 D. 260
C. D.
95. A is a set of positive integers such that when
89. Two men, A and B run a 4 km race on a divided by 2, 3, 4, 5 and 6 leaves the
course 0.25 km round. If their speeds are in remainder 1, 2, 3, 4 and 5 respectively. How
the ratio 5 : 4, how often does the winner many integer between 0 and 100 belong to
pass the other ? the set A?

A. Once B. Twice A. No integer B. One


C. Thrice D. Four times C. Two D. Three
90. A boy saves Rs. 4.65 daily. What is the least 96. What would be the maximum value of Q in
number of days in which he will be able to the equation ?
save an exact number of rupees?
A. 9 B. 8
A. 10 B. 20
C. 5 D. 4
C. 21 D. 25
97. If and
91. Let and
then what is the value of
. What is the number
?
of elements in and
respectively? A. –1 B. 1
A. 2 and 4 B. 4 and 2
C. 2 D. 4
C. 2 and 2 D. 4 and 4
92. The sum of the squares of two positive 98. What is equal to?
integers is 208.If the square of the larger
number is 18 times the smaller number, A. 330 B. 340
then what is the difference of the larger and
C. 355 D. 366
smaller numbers?
A. 2 B. 3 99. If λ is an integer and α, β are the roots of

C. 4 D. 6 such that and

93. In the quadratic equation a then how many values can λ


take?
and b can take any value from the set
{1, 2, 3, 4}. How many pairs of values of a A. 3 B. 9
and b are possible in order that the
C. 14 D. 15
quadratic equation has real roots?
A. 6 B. 7 100. What is the solution of

C. 8 D. 16
?
94. In an examination, a student was asked to
divide a certain number by 8. By mistake he
multiplied it by 8 and got the answer 2016 A. B.
more than the correct answer, what was C. D.
the number?

PAGE 1197
www.byjusexamprep.com

ANSWERS KEY II 2016


ENGLISH

1 A 21 C 41 C 61 C 81 B 101 B
2 A 22 A 42 C 62 C 82 B 102 C
3 D 23 B 43 B 63 A 83 B 103 D
4 D 24 C 44 C 64 B 84 B 104 B
5 B 25 A 45 B 65 C 85 C 105 B
6 C 26 C 46 C 66 B 86 B 106 B
7 D 27 D 47 D 67 A 87 C 107 C
8 C 28 C 48 C 68 C 88 D 108 C
9 A 29 D 49 A 69 C 89 C 109 B
10 B 30 D 50 D 70 A 90 D 110 D
11 D 31 C 51 B 71 C 91 C 111 A
12 B 32 A 52 C 72 B 92 A 112 A
13 C 33 C 53 B 73 B 93 C 113 B
14 B 34 D 54 C 74 C 94 D 114 A
15 C 35 A 55 B 75 B 95 D 115 A
16 D 36 C 56 C 76 A 96 D 116 D
17 A 37 A 57 A 77 B 97 D 117 D
18 A 38 C 58 B 78 B 98 D 118 D
19 B 39 B 59 A 79 B 99 B 119 C
20 D 40 D 60 B 80 B 100 A 120 B

GENERAL KNOWLEDGE
1 C 21 D 41 A 61 B 81 C 101 B
2 B 22 D 42 C 62 D 82 C 102 B
3 B 23 C 43 B 63 D 83 A 103 D
4 A 24 C 44 D 64 B 84 B 104 D
5 C 25 A 45 D 65 A 85 B 105 D
6 C 26 D 46 B 66 A 86 C 106 A
7 B 27 A 47 C 67 B 87 B 107 B
8 A 28 D 48 C 68 D 88 D 108 A
9 D 29 A 49 B 69 B 89 D 109 D
10 C 30 A 50 D 70 A 90 C 110 A
11 A 31 C 51 A 71 A 91 C 111 A
12 C 32 C 52 C 72 D 92 B 112 A

PAGE 1198
www.byjusexamprep.com

13 D 33 C 53 C 73 D 93 A 113 C
14 D 34 A 54 A 74 A 94 D 114 A
15 B 35 D 55 D 75 D 95 C 115 D
16 C 36 D 56 A 76 C 96 B 116 C
17 A 37 B 57 C 77 A 97 B 117 D
18 B 38 B 58 B 78 C 98 B 118 C
19 B 39 B 59 D 79 B 99 C 119 A
20 A 40 D 60 A 80 D 100 B 120 C

Elementary Mathematics
1 B 21 D 41 A 61 B 81 A
2 C 22 A 42 A 62 D 82 C
3 B 23 B 43 D 63 B 83 B
4 D 24 D 44 C 64 C 84 D
5 C 25 C 45 C 65 B 85 D
6 D 26 D 46 A 66 D 86 A
7 D 27 A 47 B 67 C 87 B
8 C 28 B 48 B 68 D 88 D
9 B 29 B 49 C 69 C 89 C
10 B 30 B 50 C 70 D 90 B
11 A 31 D 51 A 71 D 91 B
12 C 32 B 52 D 72 C 92 C
13 D 33 A 53 B 73 C 93 B
14 A 34 C 54 D 74 A 94 B
15 A 35 D 55 D 75 D 95 B
16 B 36 A 56 C 76 A 96 A
17 B 37 B 57 B 77 D 97 B
18 D 38 C 58 C 78 D 98 A
19 C 39 A 59 A 79 C 99 D
20 C 40 C 60 B 80 D 100 B

PAGE 1199
www.byjusexamprep.com

PAGE 1200
CDS II 2016
www.byjusexamprep.com

(Solutions)
ENGLISH
1. Ans. A.
The sentence starts with the history showing how the growth of civilization depends upon nature.
So the next sentence in sequence will be statement R as it shows how the ancient communities
produced food. Also, all other option carries a pronoun. So, the preceding statement will have a
noun. R being the next statement makes all other options invalid.
Hence, the correct answer is Option A (RQSP).
2. Ans. A.
The starting statement talks about the shots. The next statement would be the reaction of the
speaker after the shots. So statement S states that he ducked down and ran for the river. This
eliminates options (B) and (D). The next statement would be Q. The continuing statement would be
P, as it describes the nature of water that he felt after a splash. R would be the concluding statement
as it correctly connects with the last statement given.
Hence, the correct answer is Option A (SQPR).
3. Ans. D.
The starting statement mentions why English travel. So the continuing statement would be talking
about the travel. Only statement Q stated the reason of travel. So, Q being the first statement
eliminates all other options.
Hence, the correct answer is Option D (QPRS).
4. Ans. D.
The starting statement is talking about the birth of a 3 – 3 metre elephant – Jumbo. So, the
connecting statement would be discussing what happened after the birth. So statement R would be
the next statement as it describes that he was transported to London; So, R being the first statement
eliminates all other options.
Hence, the correct answer is Option D (RQSP).
5. Ans. B.
The opening statement described the bureaucrat and the social worker. So the succeeding
statement would be talking about both of them. So, the statement doing so is statement R. So, R
being the first statement eliminates all other options.
Hence, the correct answer is Option B (RQPS).

PAGE 1201
www.byjusexamprep.com

6. Ans. C.
The starting statement states about the universities being the peculiar institutions. The next
statement would be talking about the establishment of such institutions. So, S would be the
succeeding statement. Thus, the option (A) and (d) get eliminated. The continuing statement would
be talking about the contribution of the modern universities. So, the next statement would be P.
This eliminates option (B) as well.
Hence, the correct answer is Option C (SPRQ).
7. Ans. D.
The starting statement describes about the education of Gandhi. The next statement would be
talking about his interest in studies. Thus, Q will be the connecting statement. So, Q being the first
statement eliminates all other options.
Hence, the correct answer is Option D (QPSR).
8. Ans. C.
The starting statement talked about Helen and her graduation. So, the continuing statement would
be talking about the age of Helen when she completed her honours. So, P would be the succeeding
statement. So, P being the first statement eliminates all other options.
Hence, the correct answer is Option C (PSQR).
9. Ans. A.
The starting statement talked about the young scientist Frederick Soddy. So the next statement
would be talking about his contribution with Rutherford and how long the association lasted. So,
the continuing statement would be P. So, P being the first statement eliminates all other options.
Hence, the correct sequence is Option A (PQSR).
10. Ans. B.
The starting statement talked about how the boy managed to climb the stairs despite of thick
darkness. So, the continuing statement would be talking about his landing and how his eyes were
raised to moonlight. So, the next statement in continuation would be the statement R. This
eliminates options (a) and (c). The next statement would be Q. The next statement would be "He
hesitated as the sound died down and then crept on, thinking that if they could sleep through that,
they would sleep through any noise he could make". So, S follows Q and P would be the concluding
statement.
Hence, the correct answer is Option B (RQSP).
11. Ans. D.
The starting statement talked about the housefly being a nuisance. The connecting statement would
be talking about the irritating sound of the housefly making it a nuisance. Thus, next statement
would be S. So, S being the first statement eliminates all other options.
Hence, the correct answer is Option D (SPRQ).
12. Ans. B.
The starting statement is talking about the scientific description of dinosaurs i.e. Iguanodon. The
connecting statement would be the statement describing the physical structure. So, S would be the
next statement as it describes the length of the dinosaurs. So, S being the first statement eliminates
all other options.
Hence, the correct answer is Option B (SRPQ).

PAGE 1202
www.byjusexamprep.com

13. Ans. C.
The starting statement is talking about a device which can heat fluid without electrical element. The
continuing statement would be talking about the shape of the device i.e. the container which
contains the fluid. Thus, Q will be the connecting statement. So, Q being the first statement
eliminates all other options.
Hence, the correct answer is Option C (QSRP).
14. Ans. B.
The most appropriate sequence for the given jumbled sentences is RQSP.
Hence, the correct answer is Option B (RQSP).
15. Ans. C.
‘Suppose’ and ‘if’ both are used to guess, or think, or imagine something and both means the same
thing. So, using them together is superfluous. So, ‘if’ will be used.
The correct formation would be, ‘If you are selected, will you give us a treat’.
Hence, option C is the correct answer.
16. Ans. D.
‘Rather’ is always followed by ‘Than’. Thus, the phrase ‘would rather have’ is grammatically correct.
The correct formation would be, ‘I would rather have a noble enemy than a mean friend’.
Hence, option D is the correct answer.
17. Ans. A.
‘Lest’ is always followed by ‘Should’, as it is used to express a purpose.
Thus, the correct formation would be, ‘He decided to take the help of a guide lest he should miss
the way’.
Hence, option A is the correct answer.
18. Ans. A.
There are some verbs which are usually followed by to-infinitive. Some of such verbs are "want,
choose, decide, agree, try" etc. "To-infinitive" is different from a verb as it uses preposition "to" plus
the main verb. The bracketed part is incorrect as present participle "taking" has been used here
instead of "base verb". According to this rule, the correct answer is option A. Options B and C are
incorrect owning to the use of wrong prepositions and wrong verb forms.
Hence, option A is the correct answer.
19. Ans. B.
The bracketed part is incorrect because the plural form of ‘poor’ is ‘poor’. ‘Poors’ is incorrect. Also,
the correct phrase is "in the interests" not "for the interest" or "in the interest". So, bracketed part
should be replaced by option B.
Hence, option B is the correct answer.

PAGE 1203
www.byjusexamprep.com

20. Ans. D.
The sentence is grammatically and contextually correct. ‘His’ is the correct form of pronoun to be
used.
Thus, the correct formation would be, ‘He reached his destination at night’.
Hence, option D is the correct answer.
21. Ans. C.
The possessive form of pronoun ‘one’ is ‘oneself’. So the phrase ‘with himself’ needs to be replaced
with ‘with oneself’ to make the sentence grammatically correct.
Thus, the correct formation would be, ‘One is often pleased with oneself’.
Hence, option C is the correct answer.
22. Ans. A.
‘Unless’ is used to denote the condition i.e. ‘If’…… ‘Not’. So, again the use of negative word ‘not’ is
superfluous. So, the phrase, ‘are not very’ needs to be replaced with ‘are very’ to make the sentence
grammatically correct.
Thus, the correct formation would be, ‘Unless, you are very careful, you will run into debt’.
Hence, option A is the correct answer.
23. Ans. B.
If some action has started in the past and is still continuing, then present perfect continuous tense
should be used instead of the present continuous tense. So, the phrase ‘am living’ needs to be
replaced with ‘have been living’ to make the sentence contextually correct.
Thus, the correct formation would be, ‘I have been living in Bombay for the last ten years’.
Hence, option B is the correct answer.
24. Ans. C.
‘Instead’ is used as an alternative or substitute and is not relevant in this context. ‘Nevertheless’ will
be used as it means ‘inspite of that’.
Thus, the correct formation would be, ‘This scooter is not as efficient as it used to be; nevertheless
it is still a very useful machine.’
Hence, option C is the correct answer.
25. Ans. A.
‘Were’ needs to be replaced with ‘was’ to make the sentence grammatically correct. This is because,
‘If the subjects are joined by as well as, with, along with, together with, and not, in addition to, but,
besides, except, rather than, accompanied by, like, unlike, no less than, nothing but, led by, headed
by, guided by, controlled by, governed by,etc.. Then the verb will agree with the first subject.
Thus, the correct formation would be, ‘The teacher as well as his wife was invited’.

PAGE 1204
www.byjusexamprep.com

26. Ans. C.
Modal ‘can’ needs to be replaced with ‘could’ to make the sentence grammatically correct.
Sentences beginning with I wish use past form of main verb. For example, I wish I were a rich man.
I wish I could mend my past.
Thus, the correct formation would be, ‘I wish I could help you’.
27. Ans. D.
The sentence is grammatically and contextually correct. This is because, the verb, ‘to look forward
to’ means to anticipate. It requires a direct object and it will be the gerund form of the verb i.e.
‘meeting’.
Thus, the correct formation would be, ‘My brother is looking forward to meeting his employer
tomorrow’.
28. Ans. C.
‘Ascent’ means upward moment and is incorrect as per the meaning of the sentence. ‘Accent’
means to emphasize a particular feature; ‘Approof’ means trial, and ‘Assent’ means to give an
expression of approval or agreement. Thus, ‘Ascent’ needs to be replaced with ‘Assent’
Thus, the correct formation would be, ‘My father has given his assent for my long tour’.
29. Ans. D.
The phrase ‘has been thrusted’ is grammatically correct. This is because the given sentence is in
passive voice and in passive voice of present perfect tense, "has/have + third form of main verb" is
used. So, the given sentence is grammatically correct.
30. Ans. D.
If the second event occurs immediately after the first, we can express that idea using the below
structures
1. No sooner... than
2. Hardly... when
3. Scarcely ... when
With "hardly/scarcely", we always use past perfect tense. See below examples:
Hardly had I reached the station when the train came.
Scarcely had I reached the station when the train arrived.
However, with "no sooner", we can use the verb "had" as well as "did". See below examples:
No sooner had I closed my eyes than I fell asleep.
No sooner did I arrive at the station than the train came.
Thus, the given sentence is grammatically correct.
31. Ans. C.
Option C is correct. The phrase 'between the cup and the lip' means a situation where things did
not happen at the last minute the way they were expected to due to unforeseen reasons.

PAGE 1205
www.byjusexamprep.com

32. Ans. A.
The question tag ‘isn’t it’ needs to be replaced with ‘can’t we’. This is because the sentence and the
question tag must be in the same tense and the same pronoun is to be used in the question tag.
Thus, the correct formation would be, ‘we can go out whenever we choose to, can’t we?’
33. Ans. C.
Option C is the correct answer. The structure "too...to" is used to denote negative sense in the
sentence.
34. Ans. D.
‘Sitting on the fence’ is the correct idiom used. It is used to describe a person’s lack of decisiveness,
neutrality or hesitance to choose between two sides in an argument or a competition, or inability
to decide due to lack of courage. Thus, the sentence is grammatically and contextually correct.
Thus, the correct formation would be, ‘Decide one way or the other; you can’t be sitting on the
fence forever’.
35. Ans. A.
‘To dispose of’ is the correct phrase to be used here. It means to get rid of something by selling or
giving. Thus, the correct formation would be, ‘Raman wants to dispose of his house’.
36. Ans. C.
The bracketed part is incorrect because "regret" is not followed by any preposition. So, ‘regret for’
needs to be replaced with ‘regret’ to make the sentence grammatically correct.
37. Ans. A.
If some action has started in the past and is still continuing, then present perfect continuous tense
should be used instead of present continuous tense. So, the phrase ‘is working’ needs to be replaced
with ‘has been working’ to make the sentence contextually correct.
Thus, the correct formation would be, ‘Ramesh has been working in this factory for the past three
months’.
38. Ans. C.
If some action has started in the past and is still continuing, then present perfect continuous tense
should be used instead of present continuous tense. So, the phrase ‘am waiting’ needs to be
replaced with ‘have been waiting’ to make the sentence contextually correct.
Thus, the correct formation would be, ‘I have been waiting for three – quarters of an hour’.
39. Ans. B.
The bracketed part is incorrect because the sentence is not of comparative degree as there is no
use of "than" in the sentence. For making any selection or showing preference, we use degrees.
Option A is incorrect as no degree of adjective has been used here. Option C is incorrect as
superlative adjectives would require the use of "the" with them. Therefore, out of the given
alternatives, only option B is the correct choice.

PAGE 1206
www.byjusexamprep.com

40. Ans. D.
Since, the sentence starts talking about the forces, the continuing statement would be the
statement talking about the consensus of the forces on a particular point i.e. Q follows. R will be
the next statement as it connects Q by stating that they agreed for a cease fire. The next statement
would be talking about the reason of the cease fire, therefore, P follows and S would be the
concluding statement which makes a coherent paragraph.
The correct formation would be, ‘Georgian loyalists and rebel forces agreed to a ceasefire today
after a week of fighting in which 51 people were killed’.
41. Ans. C.
Since, the sentence starts talking about the ultimate hope; it should be followed by the statement
that is talking about the hope. So, only statement Q connects with the given statement. Also, in the
options there is a single option with starting statement as Q.
The correct formation would be, ‘The ultimate hope that the destructive nature of weapons will
force the nations to give up war has not been fulfilled.’
42. Ans. C.
Since, the sentence starts talking about the leader of the opposition, the connecting statement will
be R as it carries a relative pronoun ‘who’ which relates the sentences. S follows as it states the
disappointment of the leader and the next statement talked about the reason of disappointment.
P would be the concluding statement as it goes with the flow describing how the leader decided to
convince the assembly and thus makes a coherent paragraph.
The correct formation would be, ‘The leader of the opposition who had a reputation for speech
making was very much disappointed on realising that he had failed to convince the assembly in the
manner he had planned to convince them’.
43. Ans. B.
Since, the sentence starts talking about the thought process, R will be the next statement as it is
talking about the two factors. P will be the continuing statement "often confused in the public
minds". Q follows S as.
The correct formation would be, ‘We can think of two factors often confused in the public minds
which suggest the need for an international language’.
Hence, option B is the correct answer.
44. Ans. C.
Since, the sentence starts with a subject, the next statement will carry a relative pronoun connecting
the statements. S follows the opening statement. P will be the next statement which is mentioning
the other feature of the man. Q will be the next statement as it describes another quality of man.
Thus, R will be the concluding statement and thus form a coherent paragraph.
The correct formation would be, ‘The man who can play most heartily when he has the chance of
playing is generally the one who can work very hard when he must work’.
45. Ans. B.
Since, the sentence starts talking about leakage of the roof; it should be followed by the statement
that is talking about the repairing. So, only statement S connects with the given statement. Also, in
the options there is a single option with starting statement as S.
The correct formation would be, ‘Dear Lodger, I agree, the roof is leaking but I can’t get it repaired
while it is raining and when the sun shines there would be no need.’

PAGE 1207
www.byjusexamprep.com

46. Ans. C.
Since, the sentence starts talking about the reason; R will be the continuing statement as it goes
with the flow of the statement. The continuing statement talks about the existing of life i.e. P. Thus,
all other options are eliminated.
The correct formation would be, ‘There is no reason why we should not be willing to think that life
may exist in great profusion in other worlds.’
47. Ans. D.
Since, the starting statement carries a subject; the next statement will carry a verb i.e. Q follows as
it mentioned the discovery of the Regent. R will be the continuing statement which states what he
actually discovered. P follows next as it talked about the reaction of the Regent after the discovery.
And S would be the concluding statement and thus form a coherent paragraph.
The correct formation would be, ‘The Regent having discovered that he had imprisoned an innocent
man released Voltaire and gave him a pension.’
48. Ans. C.
Since, the starting statement carries a subject; R will be the first statement. The next statement
describes why are they hunted, so Q follows P, as it states that what is being done with the horns.
S would be the concluding statement and thus form a coherent paragraph.
The correct formation would be, ‘The Rhinoceros is hunted by poachers for its horn which is sold at
high prices.’
49. Ans. A.
Since, the starting statement carries a subject; S will be the first statement as it talked about the
world’s fastest computers. The next statement describes the initial use of the same and the
statement that follows states the new uses of the computers as environmental tools. Thus SQRP is
the correct sequence and thus form a coherent paragraph.
The correct formation would be, ‘The world’s fastest computers initially conceived for military
purposes are finding various new uses as environmental tools.’
50. Ans. D.
The most appropriate sequence for the given jumbled parts of the sentence is - RQSP.
Hence, option D is the correct answer.
51. Ans. B.
‘If’ is used to denote condition or supposition, ‘Though’ is used to indicate a factor that qualifies or
imposes restrictions on what was said previously, ‘Unless’ is used to introduce the case in which a
statement being made is not true or valid. Thus, the correct connector would be ‘though’.
Contextually, other words don’t fit the sentence.
52. Ans. C.
The blank requires a noun that is singular as the verb following the blank is singular. Although
‘people’ can also be used both as a singular as well as plural form but here the emphasis is being
made that ‘man’ is using science. Contextually, other words don’t fit the sentence.

PAGE 1208
www.byjusexamprep.com

53. Ans. B.
The blank requires an article. ‘The’ is incorrect as it is used for definite things; ‘that’ is grammatically
incorrect. So, ‘a’ will be the correct article to fill the blank. Contextually, other words don’t fit the
sentence.
54. Ans. C.
‘Has had’ is the correct form of verb to be used in the sentence to make it grammatically correct.
This is because we use the present perfect tense to describe an event from the past that has some
connection to the present. Contextually, other words don’t fit the sentence.
55. Ans. B.
‘And’ is used to connect words of the same part of speech, clauses, or sentences, that are to be
taken jointly, ‘Or’ is used to link alternatives and ‘Either’ is used for one or the other of two people
or things. So, ‘or’ is the correct connector to be used here. Contextually, other words don’t fit the
sentence.
56. Ans. C.
‘Since is used if some action has started in the past and is still continuing. Also, to denote point of
time, since is used and to denote period of time, for is used. Contextually, other words don’t fit the
sentence.
57. Ans. A.
‘Marvellous’ means causing great wonder; extraordinary, ‘Costly’ means expensive and
‘Inexpensive’ means cheap. Here the sentence is talking about the wonders of science. So,
‘Marvellous’ will correctly fit the blank. Contextually, other words don’t fit the sentence.
58. Ans. B.
‘Dull’ means lacking interest or excitement, ‘Exciting’ means causing great enthusiasm and
eagerness and ‘Aimless’ means without purpose or direction. Since the sentence is talking about
the gift of science so it can’t be used with negative words. So, ‘Exciting’ will correctly fit the blank.
Contextually, other words don’t fit the sentence.
59. Ans. A.
‘At’ is used with a definite point of time, ‘In’ is used for unspecific time and ‘Within’ is used when a
task is being completed before a definite time. Thus, ‘At’ fits the blank correctly. Contextually, other
words don’t fit the sentence.
60. Ans. B.
As the sentence is talking about the problems, it will exaggerate in future. Thus, the comparative
degree of ‘Bad’ i.e. ‘Worse’ will be used. Contextually, other words don’t fit the sentence.
61. Ans. C.
‘That’ is used to refer to a specific thing previously mentioned, known, or understood. Thus, ‘That’
is the correct form of pronoun to be used in the sentence. Contextually, other words don’t fit the
sentence.

PAGE 1209
www.byjusexamprep.com

62. Ans. C.
Since, the sentence is not talking about possibility, ‘can become’ is incorrect. The statement is
talking about the action that is still continuing. So present perfect form of tense will be used; so,
‘has become’ fits the blank correctly. Contextually, other words don’t fit the sentence.
63. Ans. A.
If the sentence is talking about the things that will take place in near future, the simple present
tense is used. Thus, ‘Leave’ fits the blank correctly. Contextually, other words don’t fit the sentence.
64. Ans. B.
‘Can’ is used to express possibility without doubt in present tense. ‘Must’ states compulsion and
hence incorrect. ‘Will’ is used for future tense and hence doesn’t go with the context.
65. Ans. C.
‘Over’ is the correct form of preposition to be used in the sentence.
Other options are not correct grammatically and hence are incorrect.
66. Ans. B.
‘Since’ is used for a reason, ‘When’ is used for the time and ‘Although’ is used for in spite of the fact
that; even though. So, ‘Although’ is correct as per the context of the passage. Other options are not
correct grammatically and hence are incorrect.
67. Ans. A.
Here the sentence is stating that, inspite being the fact that metals are strong, still it is possible to
hammer them. Thus, ‘can be’ fits the blank as per the context. Other options are not correct
contextually and hence are incorrect.
68. Ans. C.
‘By means of’ is the correct form of phrasal verb which means with the help of; by using. Other
options are not correct contextually and hence are incorrect.
69. Ans. C.
‘A piece of’ is the correct form of phrasal verb which means a part of something. Other options are
not correct contextually and hence are incorrect.
70. Ans. A.
‘Lightly’ means in small amount, ‘Warmly’ refers to a way that gives out warmth. So, ‘Lightly’ fits
the blank correctly. Other options are not correct grammatically and hence are incorrect.
71. Ans. C.
‘Must’ is used to denote compulsion but in this sentence compulsion is not needed. ‘Been needed’
is grammatically incorrect. Thus, ‘is needed’ fits the blank appropriately.
72. Ans. B.
Here, the state is being defined i.e. solid gets converted into liquid after heating. Thus, ‘condition’
is correct while other options are not correct grammatically.

PAGE 1210
www.byjusexamprep.com

73. Ans. B.
‘Into’ is used to denote motion, hence it’s grammatically correct. ‘Out of’ is incorrect as something
is getting poured inside. ‘Onto’ is used to position on the surface of something. Thus, other options
are not correct grammatically.
74. Ans. C.
‘Specialist’ is a noun; ‘Specified’ is a verb while ‘Specially’ is an adverb. An adverb will be needed as
it describes the manner of preparation. Thus, ‘specially’ is correct while other options are not
correct grammatically.
75. Ans. B.
‘After’ is correct as one action is initiated after the previous one is being completed. So once the
liquid gets converted into solid then further things are carried out. Other options are not correct
grammatically.
76. Ans. A.
‘Has cooled’ is the correct form of tense. This is because we use the present perfect tense to
describe an event from the past that has some connection to the present. Contextually, other words
don’t fit the sentence.
Hence, option A is the correct answer.
77. Ans. B.
‘Solid’ is the correct word that fits the blank. ‘Liquid’ is incorrect as preceding sentence mentioned
that the liquid is converted into solid. ‘Soiled’ means to tarnish or make something dirty and hence
is incorrect as per the context.
78. Ans. B.
‘This’ is used to refer to a specific thing just mentioned, ‘That’ is used to refer to a specific thing
previously mentioned, ‘thus’ is used to show the result or consequence. So, ‘this’ fits the blank as
per the context. Other options are not correct contextually and hence are incorrect.
79. Ans. B.
‘Have been fashioned’ is the correct form of phrase to be used in this sentence. This is because we
use the present perfect tense to describe an event from the past that has some connection to the
present. Contextually, other words don’t fit the sentence.
80. Ans. B.
‘Two’ is used to refer to a specific thing previously mentioned. Hence fits the blank appropriately.
Other options are not correct contextually and hence are incorrect.
81. Ans. B.
From the opening line of the passage, it can be inferred that, ‘the power of art can be judged
through its influence on the people over the years’.
82. Ans. B.
It is clearly given in the 6th line of the passage that, ‘Newton’s theory of gravitation has been
superseded by Einstein’s theory of relativity’, which suggests that the theory of relativity is new in
comparison to the theory of gravitation.
Hence, option B is correct.

PAGE 1211
www.byjusexamprep.com

83. Ans. B.
It can be clearly inferred from the 10th line of the passage that, ‘The achievements of generals,
politicians and statesman have an importance only in their own time’. I.e. they have contemporary
relevance.
84. Ans. B.
From the lines given in the passage that, ‘The people and their acts, great as they may have been
are like milestones which mankind passes on its way to something else. But with works of art it is
not so.
Thus, it can be clearly inferred that the influence of a work of art increases from age to age unlike
the work of a scientist which diminishes in course of time.
85. Ans. C.
In the whole passage, the author has interpreted the meaning of ‘strong’ in many ways. He just
doesn’t talk about the physical strength but in various fields of life in which either men or women
both holds equal importance some way or the other. Thus, the main contention of the author is
that in some activities men are stronger than women and in some others women are stronger than
men.
86. Ans. B.
From the opening line of the passage, it can be clearly inferred that, ‘Most disputes about whether
or not men are stronger than women are meaningless because the word ‘strong’ may mean many
things.
87. Ans. C.
From the 5th line of the passage, it is clear that, ‘Most women live longer than most men, they have
a better chance of resisting disease, they can beat men at operations requiring finger dexterity and
the ability to work accurately under monotonous conditions. So, then the author says that it would
be legitimate to argue that women are stronger than men.
88. Ans. D.
From the 6th line, it can be clearly inferred that, ‘Most women live longer than most men’. Thus,
option (D) is most likely to be correct.
89. Ans. C.
From the last few line of the passage, it can be clearly inferred that, ‘It is not the institutions that
are important but the character and the wisdom of the people who manage them’.
90. Ans. D.
‘Watertight compartments’ signifies that the life and the activities of an individual are entirely
separated from the public opinion of which it is a part. Thus, ‘Spheres of life which are independent
and unconnected with one another.’ can be considered as the precise contextual meaning of
‘Watertight compartments’.
91. Ans. C.
According to the author, ‘Politicians often talk as though one has only to introduce certain political
and economic changes for paradise to descend on Earth, forgetful of the fact that the efficiency of
an institution depends on the way it is worked’. Thus, it can be inferred that the author is skeptical
about the claims of the politicians.
Hence, option C is the correct answer.

PAGE 1212
www.byjusexamprep.com

92. Ans. A.
The phrase ‘Paradise to descend on Earth’ means ‘A world of perfect economic, political and social
well – being. The other options are incorrect as the passage nowhere talked about the religious
persons. Also, liberty and equality is nowhere mentioned in the passage. Thus, other options are
incorrect.
93. Ans. C.
It is clearly given in the passage that, ‘It is a wonderful feeling to reach the top of a mountain after
climbing for hours and may be even for days’. So, some men like to climb mountains because they
want to have a wonderful feeling.
94. Ans. D.
To climb a mountain is often difficult because paths are steep and uneven. It can be inferred from
the second line of the passage that, ‘Mountains are not just big hills. Paths are usually very steep.
Some mountainsides are straight up and down’.
95. Ans. D.
In the whole passage the author is talking about mountaineers and mountaineering. So,
‘Mountaineering’ is being described as a wonderful feeling.
96. Ans. D.
From the last few lines of the passage, ‘Two Italian prisoners of war escaped from a prison camp in
Kenya during the war. They did not try to get back to their own country. They climbed to the top of
Mount Kenya and then came down. They wanted to get that feeling of freedom after climbing a
difficult mountain’.
97. Ans. D.
From the opening line of the passage, it can be inferred that most of the people who appear most
often and most gloriously in the history books are great conquerors and generals and soldiers’. The
same is reflected in option D. So, it is the correct answer.
98. Ans. D.
Two contradictory statements are made in the opening lines. One is those who appeared in books
and the others who helped civilization moving forward. Thus, ‘The people who really helped
civilization forward’ suggest that conquerors, generals and soldiers contributed little to civilization.
99. Ans. B.
From the passage, it is clear that, ‘People think a great deal of them, so much so that on all highest
pillars in the great cities of the world you will find the figure of a conqueror or general or a soldier’.
Thus, people have exaggerated notions about their achievements.
100. Ans. A.
As per the last sentence of the passage, ‘The greatest countries are those that have beaten in battle
the greatest number of countries and ruled over them as conquerors.
101. Ans. B.
With certain words like, be senior to, be junior to, be superior to, be inferior to, etc. is always
followed by ‘to’.
Thus, the correct sentence would be, ‘I am senior to him by two years’.

PAGE 1213
www.byjusexamprep.com

102. Ans. C.
The modal ‘Could’ is used to express possibility. It is simply used to state one or more things that
are possible or were possible in the past. In this statement, ‘to have achieved’ needs to be replaced
with ‘for achieving’ to make the sentence correct.
Thus, the correct sentence would be, ‘When I finished writing the letter, I could not help admiring
myself for achieving the impossible’.
103. Ans. D.
The sentence is grammatically and contextually correct.
104. Ans. B.
‘Cured’ is followed by the preposition ‘of’.
Thus, the correct sentence would be, ‘He will be cured of his fever’.
105. Ans. B.
‘Prefer’ is followed by the gerund form of the verb. Thus, ‘spend’ needs to be replaced with
‘spending’ to make the sentence grammatically correct.
Thus, the correct sentence would be, ‘Though he is a gifted comedian, he prefers spending his spare
time watching horror movies.
106. Ans. B.
‘Poetry’ needs to be replaced with ‘Poem’ to make the sentence contextually correct. ‘Poetry’ is a
literary work; ‘Poem’ is a piece of writing. Here, the writing is being talked about.
Thus, the correct sentence would be, ‘The writer of this poem is Wordsworth’.
107. Ans. C.
Option B is the correct part of the sentence where an error exists. ‘Out of’ is the incorrect
preposition used in the sentence. ‘Of’ needs to be used here. Some combinations of verb "make"
with different prepositions are:
Made from
We often use made from when we talk about how something is manufactured:
Paper is made from trees.
Made of
We use 'made of' when we talk about the basic material or qualities of something. It has a meaning
similar to ‘composed of’:
She wore a beautiful necklace made of silver.
Made out of
We usually use made out of when we talk about something that has been changed or transformed
from one thing into another:
They were living in tents made out of old plastic sheets.
Made with
We use made with most often to talk about the ingredients of food and drink:
This dish is made with beef, red peppers and herbs.

PAGE 1214
www.byjusexamprep.com

108. Ans. C.
‘Were’ needs to be replaced with ‘was’ to make the sentence grammatically correct. This is because,
‘If the subjects are joined by as well as, with, along with, together with, and not, in addition to, but,
besides, except, rather than, accompanied by, like, unlike, no less than, nothing but, led by, headed
by, guided by, controlled by, governed by,etc.. Then the verb will agree with the first subject’
Thus, the correct sentence would be, ‘Sita with all her sisters was here’.
109. Ans. B.
'That' must be removed from part B of the given sentence.
Thus, the correct sentence would be, ‘As you know the ignorant are easily duped’.
110. Ans. D.
The sentence is grammatically and contextually correct.
111. Ans. A.
‘One of the assistant’ needs to be replaced with ‘one of the assistants’ to make the sentence
grammatically and contextually correct. This is because, ‘One of’ is always followed by a plural noun.
Thus, the correct sentence would be, ‘One of the assistants was absent’.
112. Ans. A.
The error is in option (A). The use of article "a" is incorrect with the word "honourable". Although
the word begins with a consonant letter, it gives the sound of 'o' as 'h' is silent in this word.
Therefore, article ‘an’ should be used before honourable. Thus, the correct sentence would b:
"Though George is an honourable man his activities arouse suspicion".
113. Ans. B.
The phrase ‘could not be able’ needs to be replaced with ‘was not able’ to make the sentence
correct. As the action has already taken place so use of modal is incorrect.
Thus, the correct sentence would be, ‘She told her teacher that she was not able to attend the class
the previous day because of heavy rains’.
114. Ans. A.
‘Were’ needs to be replaced with ‘was’ to make the sentence grammatically correct. This is because,
‘If the subjects are joined by as well as, with, along with, together with, and not, in addition to, but,
besides, except, rather than, accompanied by, like, unlike, no less than, nothing but, led by, headed
by, guided by, controlled by, governed by,etc.. Then the verb will agree with the first subject’
Thus, the correct sentence would be, ‘Coleridge as well as Wordsworth was of the opinion that the
opposite of poetry os not prose but science’.
115. Ans. A.
The phrase ‘was courted’ needs to be replaced with ‘courted’ to make the sentence grammatically
and contextually correct.
Thus, the correct sentence would be, ‘He courted arrest in order to protest against corruption
among the government servants’.

PAGE 1215
www.byjusexamprep.com

116. Ans. D.
The sentence is grammatically and contextually correct.
117. Ans. D.
The sentence is grammatically and contextually correct.
118. Ans. D.
The sentence is grammatically and contextually correct.
119. Ans. C.
The phrase ‘almost terrible as’ needs to be replaced with ‘almost as terrible as’ to make the
sentence correct. This is because, ‘As…As’ is the pair of words that needs to be used together.
Thus, the correct sentence would be, ‘For young Donald, peace in Vietnam was almost as terrible
as war’.
120. Ans. B
‘Attacked on him’ is the incorrect form of phrasal verb that needs to be replaced with ‘Attacked
him’ to make the sentence correct. This is because; ‘Attack’ is not followed by any preposition.
Thus, the correct sentence would be, ‘That Brutus, who was his trusted friend had attacked
him caused heartbreak to Julius Caesar’.

PAGE 1216
www.byjusexamprep.com

General Knowledge
1. Ans. C.
The 119th constitutional amendment bill 2013 sought to ratify the land boundary agreement
between the two countries. This amendment needed special majority in the parliament to get
passed. The bill proposed to amend the 1st schedule of the constitution to exchange the disputed
territories occupied by both the nations in accordance with the 1974 bilateral LBA.
Since, the law is not related to federal matters, only the approval of the Parliament is needed.
Hence, option C is correct.
2. Ans. B.
Article 19 (1) C of the Constitution guarantees to all citizens the right “to form associations and
unions.” Under clause (4) of Article 19, however, the State may by law impose reasonable
restrictions on this right in the interest of public order, morality or sovereignty and integrity of India.
As per the Article 19 of the Constitution, the right to form associations and unions is a fundamental
right guaranteed only to citizens.
Hence, option B is correct.
3. Ans. B.
All the reports of the Commission are presented to the President, who in turn sends them to the
both the houses of the Parliament.
sends them to both the houses of the Parliament.
Hence, option B is correct.
4. Ans. A.
The laws relating to the delimitation of constituencies are made by the Parliament.
The Model Code of Conduct comes into force immediately on announcement of the election
schedule by the commission.
Hence, option A is correct.
5. Ans. C.
In Tolkappiyam, there is a reference to the five-fold division of lands - Kurinji (hilly tracks), Mullai
(pastoral), Marudam (agricultural), Neydal (coastal) and Palai (desert).
Hence, option C is correct.
6. Ans. C.
Harappa cities followed the rectangular grid pattern where roads were straight and cut each other
at right angle. But it may not be the case always.
Brick size was uniform in nearly every Harappan city from Sutkagendor in Baluchistan to Alamgirpur
in Meerut, from Manda in Jammu Kashmir to Daimabad in Maharashtra.
Hence, option C is correct.

PAGE 1217
www.byjusexamprep.com

7. Ans. B.
Ninth Schedule says that the article mentioned in this are immune from judicial review. It is for the
larger good of the society, the government cannot be challenged in the court for his actions. Ninth
Schedule was added to the Constitution by First Constitutional Amendment, 1951.
In a landmark ruling on 11th January 2007, the Supreme Court ruled that all laws (including those
in the Ninth Schedule) would be open to judicial review, if they violated the basic structure of the
Constitution.
Hence, option B is correct.
8. Ans. A.
The Constitution of India makes special provisions for the administration of the tribal dominated
areas in four states viz. Assam, Meghalaya, Tripura and Mizoram. As per article 244 and 6th
Schedule, these areas are called “Tribal Areas“, which are technically different from the Scheduled
Areas under fifth schedule. Hence, option A is correct.
9. Ans. D.
The High Courts has similar power under the Article 226 of the Constitution. The high courts can
also issue writs to protect the Fundamental Rights of the people.
Hence, option D is correct.
10. Ans. C.
Protective arrangements: Such measures as are required to enforce equality, to provide punitive
measures for transgressions, to eliminate established practices that perpetuate inequities, etc.
A number of laws were enacted to implement the provisions in the Constitution. Examples of such
laws include The Untouchability Practices Act, 1955, Scheduled Caste and Scheduled Tribe
(Prevention of Atrocities) Act, 1989, The Employment of Manual Scavengers and Construction of
Dry Latrines (Prohibition) Act, 1993, etc. Despite legislation, social discrimination and atrocities
against the backward castes continued to persist.
Hence, option C is correct.
11. Ans. A.
The North American Free Trade Agreement (NAFTA) is an agreement signed by Canada, Mexico, and
the United States and entered into force on 1 January 1994 in order to establish a trilateral trade
bloc in North America.
Hence, option A is correct.
12. Ans. C.
The 14th Finance Commission was constituted in January 2014. Commissions chairman was former
RBI governor Y V Reddy and its members were Sushma Nath, M. Govinda Rao, Abhijit Sen, Sudipto
Mundle.
Hence, option C is correct.

PAGE 1218
www.byjusexamprep.com

13. Ans. D.
Some of the Top Credit Rating Agencies in India are:
1) Credit Rating Information Services of India Limited (CRISIL)
2) ICRA Limited
3) Credit Analysis and Research limited (CARE)
4) Brickwork Ratings (BWR)
5) India Rating and Research Pvt. Ltd.
6) Small and Medium Enterprises Rating Agency of India (SMERA)
Hence, option D is correct.
14. Ans. D.
In fact, FPI is often referred to as “hot money” because of its tendency to flee at the first signs of
trouble in an economy. These massive portfolio flows can exacerbate economic problems during
periods of uncertainty.
Hence, option D is correct.
15. Ans. B.
The growth scenario in India shows that the services sector has become the most dominant in the
later part of its growth process. The share of services sector in GDP increased from 28.5 per cent in
1950-51 to 39.6 per cent in 1990-91 and then to 52.6 per cent in 2014-15 while the share of primary
sector declined from 56.4 per cent in 1950-51 to 33.4 per cent in 1990-91 and then to only 19.0 per
cent in 2014-15.
Hence, option B is correct.
16. Ans. C.
Tatra not an Air Defence Missile System. It is a Czech manufacturer producing vehicles in Koprivnice.
It is owned by the Tatra Trucks company, based in Ostrava, and is the third oldest company in the
world producing cars with an unbroken history.
Hence, option C is correct.
17. Ans. A.
The rank of 2nd Lieutenant/Act. Sub Lieutenant/Pilot Officer were removed because the salary of
the officers holding these ranks were less compared to senior Non-Commissioned Officers i.e
Subedar Major in army, Master Warrant Officer in air force and Master Chief Petty Officer - 1st Class
in navy.
Rest of the statements are incorrect.
In modern armies, lieutenant general normally ranks immediately below general and above major
general; it is equivalent to the navy rank of vice admiral, and in air forces with a separate rank
structure, it is equivalent to air marshal.
Hence, option A is correct.

PAGE 1219
www.byjusexamprep.com

18. Ans. B.
The Indian Army is divided into six operational commands and one training command each headed
by a Limited General. They are South Western, Northern, Central, Army Training Command, Eastern,
Western and Southern Commands.
Hence, option B is correct.
19. Ans. B.
The Infantry Regiment is also known as the Mechanised Infantry Regiment of the Indian Army.
Though its aggregate size, it is closer to that or a full mechanised division with 26 mechanised
battalions.
Indian Army Corps of Electronics and Mechanical Engineers are an arms and service branch of Indian
Army having varied responsibilities of design, development, trial, inspection and refit of weapon
systems and equipment. They also provide technical advice to units on systems and recovery
operations in peace and war.
Hence, option B is correct.
20. Ans. A.
Jammu and Kashmir Rifles is an infantry regiment of the Indian Army. It is a truly historic Indian
Regiment raised by Maharaja Gulab Singh, the founder of the Dogra Kingdom in 1820.
Amongst all the princely State Forces of India, it was the only one to be absorbed into the Indian
Army as a separate Regiment and did not lose its entity by merger into the existing Regiments of
the Army. Hence, option A is correct.
21. Ans. D.
The senior most member of the Chiefs of Staff Committee (COSC) is appointed as its Chairman.
Admiral Sunil Lanba is the current and the 23rd Chief of the Naval Staff of the Indian Navy. Hence,
option D is correct.
22. Ans. D.
The Andaman and Nicobar Command is the first and only Tri-service theater command of the Indian
Armed Forces, based at Port Blair.
It was created in 2001 to safeguard India's strategic interests in Southeast Asia and the Strait of
Malacca by increasing rapid deployment of military assets in the region.
Hence, option D is correct.
23. Ans. C.
The cabinet Committee on Economic Affairs (CCEA) on 27th January, 2005 had approved the
constitution of a National Investment Fund (NIF). The Purpose of the fund was to receive
disinvestment proceeds of central public sector enterprises and to invest the same to generate
earnings without depleting the corpus.
National Investment and Infrastructure Fund (NIIF) is a fund created by the Government of India for
enhancing infrastructure financing in the country.
This is different from the National Investment Fund.
Its creation was announced in the Union Budget 2015-16. The operational framework was approved
on 20 August 2015.NIIF got registered with SEBI as Category II Alternative Investment Fund (AIF) on
December 28, 2015.
Hence, option C is correct.

PAGE 1220
www.byjusexamprep.com

24. Ans. C.
The Supreme Court has struck down the words “adult male” from the pertinent provision in the DV
Act to lay down that a woman can also file a complaint against another woman, accusing her of
domestic violence.
Hence, option C is correct.
25. Ans. A.
The 44th Constitution Amendment Act, replaced the phrase ‘internal disturbance’ with the ‘armed
rebellion’ to avoid its misuse by the government in power.
Hence, option A is correct.
26. Ans. D.
Article 43 also places upon the State the responsibility of promoting cottage industries, and the
federal government has, in furtherance of this, established several Boards for the promotion of
khadi, handlooms etc., in coordination with the state governments. It is a DPSP.
Hence, option D is correct.
27. Ans. A.
Habeas Corpus: a writ requiring a person under arrest to be brought before a judge or into court,
especially to secure the person's release unless lawful grounds are shown for their detention.
Hence, option A is correct.
28. Ans. D.
Protection under Article 32 applies only to the fundamental rights as provided by the Constitution.
Hence, option D is correct.
29. Ans. A.
Capital deepening is a situation where the capital per worker is increasing in the economy. This is
also referred to as increase in the capital intensity. Capital deepening is often measured by the rate
of change in capital stock per labour hour.
Hence, option A is correct.
30. Ans. A.
The classical economists believed in the existence of full employment in the economy. To them, full
employment was a normal situation and any deviation from this was regarded as something
abnormal. According to Pigou, the tendency of the economic system is to automatically provide full
employment in the labour market, when the demand and supply of labour are equal.
Hence, option A is correct.
31. Ans. C.
Creditors and debtors:
During inflation creditors lose because they receive in effect less in goods and services than if they
had received the repayments during a period of low prices. Debtors, on other hand, as a group gain
during inflation, since they repay their debts in currency that has lost its value.

PAGE 1221
www.byjusexamprep.com

The aggregate volume of internal trade tends to increase during inflation due to higher incomes,
greater production and larger spending. But the export trade is likely to suffer on account of a rise
in the prices of domestic goods.
The same currency unit will now buy less goods and services.
But the bondholders lose as they get a fixed interest the real value of which has already fallen.
Hence, option C is correct.
32. Ans. C.
A bank run occurs when a large number of people withdraw their money from a bank, because they
believe the bank may cease to function soon.
A banking panic or bank panic is a financial crisis that occurs when many banks suffer runs at the
same time, as people suddenly try to convert their threatened deposits into cash or try to get out
of their domestic banking system altogether.
Hence, option C is correct.
33. Ans. C.
The Economic and Social Commission for Asia and the Pacific (UNESCAP or ESCAP), located in the
United Nations Building in Rajadamnern Nok Avenue in Bangkok, Thailand, is one of the five regional
commissions of the United Nations Economic and Social Council, under the administrative direction
of the United Nations.
Hence, option C is correct.
34. Ans. A.
All squadrons of Tejas will be made up of 20 aircrafts, including four in reserve.
Hence, option A is correct.
35. Ans. D.
The Missile Technology Control Regime (MTCR) is a multilateral export control regime. It is an
informal and voluntary partnership among 35 countries to prevent the proliferation of missile and
unmanned aerial vehicle technology capable of carrying above 500 kg payload for more than 300
km.
The Missile Technology Control Regime (MTCR) was established in April 1987 by the G7 countries
The People's Republic of China is not a member of the MTCR but has agreed to abide by the original
1987 Guidelines and Annex, but not the subsequent revisions.
Israel, Romania and Slovakia have also agreed to voluntarily follow MTCR export rules even though
not yet members.
Hence, option D is correct.
36. Ans. D.
Hence, option D is correct.

PAGE 1222
www.byjusexamprep.com

37. Ans. B.
The Copa América Centenario Final was an association football match that took place on 26 June
2016 at the MetLife Stadium in East Rutherford, New Jersey, United States to determine the winner
of the Copa América Centenario.
The match was contested by Argentina and Chile, making it a rematch of the 2015 final. Chile
ultimately won on penalty kicks 4–2 after a 0–0 draw.
Hence, option B is correct.
38. Ans. B.
Rakshasa marriage is the marriage of a maiden involving her forcible abduction from her home after
her kinsmen have been slain or wounded much like its practice in Khazakh and Uzbek cultures where
it is still practised as a ritual.
Hence, option B is correct.
39. Ans. B.
Silappadikaram is one of The Five Great Epics of Tamil Literature. Epic revolves around Kannagi, who
having lost her husband to a miscarriage of justice at the court of the Pandyan Dynasty, wreaks her
revenge on his kingdom.
Hence, option B is correct.
40. Ans. D.
• Tokyo was formerly named Edo.
• Tokyo ranks first in the Global Economic Power Index and third in the Global Cities Index.
• It has served as the Japanese capital since 1869.
Hence, option D is correct.
41. Ans. A.
Founded in 1912, the Kuomintang is the oldest still active political party in the Chinese-speaking
world. Its main leader was Sun Yat-Sen. He divided the livelihood into four parts: clothing, food,
housing and transportation.
Hence, option A is correct.
42. Ans. C.
In 1769 Arkwright patented the invention that made him rich, and his country an economic
powerhouse: The spinning frame. The spinning frame was a device that could produce stronger
threads for yarns.
Hence, option C is correct.
43. Ans. B.
Upari refers to a category of tenancy tenure held under the Maratha Regime.
The Government used to assign land to various cultivators for cultivation and getting maximum
revenue. The land allotted to original settlers of villages came to be known as Mirasi land whereas
the land assigned to the migrant settlers was known as the Upari land.
Hence, option B is correct.

PAGE 1223
www.byjusexamprep.com

44. Ans. D.
Gypsies are believed to have arrived in Europe from northern India in the 1400s. They were called
Gypsies because Europeans thought they came from Egypt. This ethnic minority is made up of
distinct groups called “tribes” or “nations.” Most of the Gypsies in German-occupied Europe
belonged to the Sinti and Roma tribes.
Romani (romani ćhib) Romani, or Romany, is an Indo-Aryan language spoken by about 5-6 million
Roma people throughout Europe and the USA. The largest concetrations of Roma people live in
Turkey, Spain and Romania. In English these people are often called Gypsies.
Hence, option D is correct.
45. Ans. D.
In 1861, civil war broke out in the USA. USA was the largest supplier of cotton to Britain. Once the
civil war broke out, the demand for cotton from India became high and this led to a surge in cotton
cultivation in India and there was a period of ‘boom’ then.
In May and June 1875, peasants of Maharashtra in some parts of Pune, Satara and Ahmednagar
districts revolted against increasing agrarian distress. The Deccan Riots of 1875 targeted conditions
of debt peonage (kamiuti) to moneylenders.
In 1879, the Agriculturists Relief Act was passed which ensured that the farmers could not be
arrested and imprisoned if they were unable to pay their debts.
Hence, option D is correct.
46. Ans. B.
Adi Shankara or Shankara, was an early 8th century Indian philosopher and theologian who
consolidated the doctrine of Advaita Vedanta.
He is credited with unifying and establishing the main currents of thought in Hinduism
It is with his teacher Govinda, that Shankara studied Gaudapadiya Karika, as Govinda was himself
taught by Gaudapada.
Hence, option B is correct.
47. Ans. C.
2016 Australian Open – Men's Singles. Novak Djokovic was the defending champion and successfully
defended his title, defeating Andy Murray, 6–1, 7–5, 7–6 in a rematch of the previous year's final,
and the fourth Australian Open final between the pair.
Hence, option C is correct.
48. Ans. C.
The Dhanush is a 155 mm towed howitzer used by the Indian Army.
The gun has been developed by the Ordnance Factory Board (OFB), Kolkata, after going through
design documents running into over 12,000 pages. These documents were given to India as part of
the first phase of “Transfer of Technology” (ToT) under the Bofors gun deal. Hence, option C is
correct.
49. Ans. B.
"If you feel you are trapped in a black hole, don't give up. There is a way out." — Stephen Hawking.
Hence, option B is correct.

PAGE 1224
www.byjusexamprep.com

50. Ans. D.
Neil O'Brien (10 May 1934 – 24 June 2016) was a quiz master who was often credited for conducting
the first formal well-organised quiz in Calcutta, India.
Hence, option D is correct.
51. Ans. A.
Article-20 of the Constitution grants protection against arbitrary and excessive punishment to an
accused person. It contains three provisions in that direction No Ex-post-facto Law No person shall
be convicted of any offence except for violation of a law in force at the time of the commission of
the Act nor subjected to a penalty greater than that prescribed by the law in force at the time of the
commission of the Act.
No Double Jeopardy No person shall be prosecuted and punished for the same offence more than
once.
No Self-incrimination No person accused of any offence shall be compelled to be a witness against
himself.
As per the Constitution, a person/entity can be tried retrospectively in certain instances as provided
by the law.
Hence, option A is correct.
52. Ans. C.
The Citizenship Act, 1955, prescribes three ways of losing citizenship whether acquired under the
Act or prior to it under the Constitution, viz. renunciation, termination and deprivation:
1) By Renunciation: Any citizen of India of full age and capacity can make a declaration renouncing
his Indian citizenship.
2) By Termination: When an Indian citizen voluntarily (consciously, knowingly and without duress,
undue influence or compulsion) acquires the citizenship of another country.
3) By Deprivation:
It is a compulsory termination of Indian citizenship by the Central government, if:
• the citizen has obtained the citizenship by fraud:
• the citizen has shown disloyalty to the Constitution of India:
• the citizen has unlawfully traded or communicated with the enemy during a war;
• the citizen has, within five years after registration or naturalisation, been imprisoned in any
country for two years; and
• the citizen has been ordinarily resident out of India for seven years continuously.
Hence, option C is correct.

PAGE 1225
www.byjusexamprep.com

53. Ans. C.
Fundamental Rights available to only citizens and not foreigners
• Prohibition of discrimination on grounds of religion, race, caste, sex or place of birth (Article 15)
• Equality of opportunity in matters of public employment (Article 16)
• Six basic freedoms subject to reasonable restrictions (Article 19)
• Protection of language, script and culture of minorities (Article 29)
• Right of minorities to establish and administer educational institutions (Article 30) Hence,
option C is correct.
54. Ans. A.
The Golaknath Case of 1967 relates to the power of the Parliament to curtail the Fundamental Rights
provided in the Constitution.
In 1967, the Supreme Court reversed its earlier decisions in Golaknath v. State of Punjab, It held that
Fundamental Rights included in Part III of the Constitution are given a "transcendental position" and
are beyond the reach of Parliament. It also declared any amendment that "takes away or abridges"
a Fundamental Right conferred by Part III as unconstitutional. By 1973, the basic structure doctrine
triumphed in Justice Hans Raj Khanna's judgment in the landmark decision of Kesavananda Bharati
v. State of Kerala.
Hence, option A is correct.
55. Ans. D.
The most prominent form of Shiva in Brihadeshvara Temple built by Chola dynasty is Tripuntaka.
Tripurantaka is a manifestation of the Hindu god Shiva. In this aspect, Shiva is depicted with four
arms wielding a bow and arrow. Shiva as Tripurantaka is accredited with destroying three mythical
cities of the Asuras. It is believed that the Rudraksha came from lord Shiva’s eyes, when he
destroyed Tripura.
Hence, option D is correct.
56. Ans. A.

Hence, option A is correct.


57. Ans. C.
While it has been conducted every 10 years, beginning in 1872, the first complete census was taken
in the year 1881. Post 1949, it has been conducted by the Registrar General and Census
Commissioner of India under the Ministry of Home Affairs, Government of India.
Hence, option C is correct.

PAGE 1226
www.byjusexamprep.com

58. Ans. B.
Documents from Western India – Rajasthan, Gujarat and Maharashtra – record petitions sent by
women to the village panchayat, seeking redress and justice. Wives protested against the infidelity
of their husbands or the neglect of the wife and children by the male head of the household, the
grihasthi. While male infidelity was not always punished, the state and “superior” caste groups did
intervene when it came to ensuring that the family was adequately provided for. In most cases when
women petitioned to the panchayat, their names were excluded from the record: the petitioner
was referred to as the mother, sister or wife of the male head of the household.
Hence, option B is correct.
59. Ans. D.
The Sukhoi Su-30MKI is a twinjet multirole air superiority fighter developed by Russia's Sukhoi and
built under licence by India's Hindustan Aeronautics Limited (HAL) for the Indian Air Force (IAF).
The BrahMos is a medium-range ramjet supersonic cruise missile that can be launched from
submarine, ships, aircraft, or land.
It is a joint venture between the Russian Federation's NPO Mashinostroyeniya and India's Defence
Research and Development Organisation (DRDO) who together have formed BrahMos Aerospace
The name BrahMos is a portmanteau formed from the names of two rivers, the Brahmaputra of
India and the Moskva of Russia.
Hence, option D is correct.
60. Ans. A.
May 23, 2016 ISRO successfully flight-tested India’s first winged body aerospace vehicle operating
in hypersonic flight regime.
In this experimental mission, the HS9 solid rocket booster carrying RLV-TD lifted off from the First
Launch Pad at Satish Dhawan Space Centre, Sriharikota at 07:00hr IST. After a successful flight of
91.1second, HS9 burn out occurred, following which both HS9 and RLV-TD mounted on its top
coasted to a height of about 56 km. At that height, RLV-TD separated from HS9 booster and further
ascended to a height of about 65km.
Hence, option A is correct.
61. Ans. B.
The decision by the electorate was to "Leave the European Union" which won by a majority of
1,269,501 votes (3.78%) over those who had voted in favour of "Remain a member of the European
Union" with England and Wales voting to "Leave" whilst Scotland and Northern Ireland voted to
"Remain". Hence, option B is correct.
62. Ans. D.
CSE in its recommendation asked Indian regulator FSSAI to act immediately and ban the two
chemicals.
It says that Indian bread manufacturer’ use two chemicals, banned in many countries but not in
India, being used by bread industry are potassium bromate (KbrO3) and potassium iodate (KIO3).
The government (FSSAI) banned the use of potassium bromated as a food additive on the basis of
the above recommendation.
The regulator had recommended to the health ministry removal of potassium bromate from the list
of permissible food additives after a study by the Centre for Science and Environment (CSE). Hence,
option D is correct.

PAGE 1227
www.byjusexamprep.com

63. Ans. D.
In India, the Universal Adult Franchise was introduced only in the post-independence period during
the first general elections to the Lok Sabha and the State Assemblies.
Under the Cabinet Mission Plan of 1946, elections were held for the first time for the Constituent
Assembly.
Constituent Assembly members were elected indirectly by members of the newly elected provincial
assemblies, and initially included representatives for those provinces that formed part of Pakistan
(some of which are now in Bangladesh).
Hence, option D is correct.
64. Ans. B.
Some provisions of the Constitution pertaining to citizenship, elections, provisional parliament,
temporary and transitional provisions, and short title contained in Articles 5, 6, 7, 8, 9, 60, 324, 366,
367, 379, 380, 388, 391, 392 and 393 came into force on November 26, 1949 itself.
Hence, option B is correct.
65. Ans. A.
A395. Repeals The Indian Independence Act, 1947, and the Government of India Act, 1935, together
with all enactments amending or supplementing the latter Act, but not including the Abolition of
Privy Council Jurisdiction Act, 1949.
Hence, option A is correct.
66. Ans. A.
In Hinduism, the guardians of the cardinal directions are called the Lokapālas (लोकपाल), or
Dikpalaka.
Aṣṭa-Dikpāla ("Guardians of Eight Directions")
1) Kubera (North)
2) Yama (South)
3) Indra (East)
4) Varuṇa (West)
5) Īśāna (Northeast)
6) Agni (Southeast)
7) Vayu (Northwest)
8) Nirṛti (Southwest)
Hence, option A is correct.
67. Ans. B.
T-55, T-72 and T-90 battle are USSR/Russian origin battle tanks and are respectively based on first,
second and third generation battle tank technologies.
Hence, option B is correct.

PAGE 1228
www.byjusexamprep.com

68. Ans. D.
Edict 13 on the Edicts of Ashoka Rock Inscriptions reflect the great remorse the king felt after
observing the destruction of Kalinga: The lethal war with Kalinga transformed the vengeful Emperor
Ashoka into a stable and peaceful emperor, and he became a patron of Buddhism.
He ended the military expansion of the empire. The Kalinga war became his first and last war. He
made some significant changes in the state policy of Magadh. Hence, option D is correct.
69. Ans. B.
During the Mughal Empire, zamindars belonged to the nobility and formed the ruling class. Emperor
Akbar granted them mansabs and their ancestral domains were treated as jagirs.
In some respects of zamindars and the peasants were natural allies in any struggle against the
Mughal government.
The Zamindars often received the support of the peasantry in many agrarian uprisings in North India
in the seventeenth century.
Hence, option B is correct.
70. Ans. A.
Sources of the seventeenth century refer to two kinds of peasants – khud-kashta and pahi-kashta.
The pahi-kashta were non-resident cultivators who belonged to some other village but cultivated
lands elsewhere on a contractual basis. People became pahi-kashta either out of choice – for
example, when terms of revenue in a distant village were more favourable – or out of compulsion
– for example, forced by economic distress after a famine.
Hence, option A is correct.
71. Ans. A.

Hence, option A is correct.


72. Ans. D.
The temperate forests are called Sholas in the Nilgiris, Anaimalai and Palani hills.
Moist deciduous forests: Major Species: Teak, sal, shisham, hurra, mahua, amla, semul, kusum, and
sandalwood etc. are the main species of these forests.
Hence, option D is correct.
73. Ans. D.
Indian Wild Ass Sanctuary also known as the Wild Ass Wildlife Sanctuary is located in the Little Rann
of Kutch in the Gujarat state of India.
Hence, option D is correct.

PAGE 1229
www.byjusexamprep.com

74. Ans. A.
Demoiselles
These are rock pillars which stand as resistant rocks above soft rocks as a result of differential
erosion of hard and soft rocks.
Zeugen
A table-shaped area of rock found in arid and semi-arid areas formed when more resistant rock is
reduced at a slower rate than softer rocks around it.
A Dreikanter
It is a type of ventifact that typically forms in desert or periglacial environments due to the abrasive
action of blowing sand.
A wind gap is a gap through which a waterway once flowed that is now dry as a result of stream
capture.
Hence, option A is correct.
75. Ans. D.
In astronomy, a syzygy is a straight-line configuration of three or more celestial bodies in a
gravitational system.
Solar and lunar eclipses occur at times of syzygy, as do transits and occultations.
Hence, option D is correct.
76. Ans. C.
Alluvial Soils
The proportion of nitrogen is generally low, but potash, phosphoric acid and alkalis are adequate,
while iron oxide and lime vary within a wide range.
Black Soils
Chemically, the black soils are rich in lime, iron, magnesia and alumina. They also contain potash.
But they lack in phosphorous, nitrogen and organic matter.
Laterite Soils
With rain, lime and silica are leached away, and soils rich in iron oxide and aluminium compound
are left behind. Hence, option C is correct.
77. Ans. A.

Hence, option A is correct.


78. Ans. C.
Dogger Bank (Dutch: Doggersbank, German: Doggerbank, Danish: Doggerbanke) is a large sandbank
in a shallow area of the North Sea about 100 kilometres (62 mi) off the east coast of England.
Hence, option C is correct.

PAGE 1230
www.byjusexamprep.com

79. Ans. B.

Hence, option B is correct.


80. Ans. D.

Hence, option D is correct.


81. Ans. C.
• In a vacuum and in homogeneous, transparent media, such as air, water, and glass, light moves
in a straight line.
• When light encounters a boundary between two different transparent media, some light is
reflected and some moves from the first into the second medium.
Hence, option C is correct.
82. Ans. C.
On the basis of magnetic properties, substances are classified into three groups namely
diamagnetic, paramagnetic and ferromagnetic.
As solids diffuse and reorder very slowly under ambient conditions, the net, directionally dependent
(anisotropic) magnetic moment can persist. Conversely, liquids and gases freely tumble and reorder:
water rearranges every few picoseconds (10^12 times per second). Hence liquids and gases can
never show ferromagnetic properties.
Hence, option C is correct.
83. Ans. A.
A photon’s relativistic momentum is given by the equation,
E = pc where p is the momentum.
P = 10 MeV/c
So, E = 10 MeV
Hence, option A is correct.
84. Ans. B.
According to wave theories, for Total internal reflection to happen: Light must travel from denser
medium to rarer medium. Angle of incidence should be greater than critical angle
As in case of TIR, the image is perfectly clear, it implies that there has been no loss of intensity.
Hence, option B is correct.
85. Ans. B.
They are located on the St. Marys River between Lake Superior and Lake Huron, between the Upper
Peninsula of the US state of Michigan and the Canadian province of Ontario.
Hence, option B is correct.

PAGE 1231
www.byjusexamprep.com

86. Ans. C.
Tributaries
- left: Ramganga, Gomti, Karnali, Gandaki, Koshi, Mahananda
- right: Yamuna, Tamsa, Son, Punpun, Tons, Hindon, Kunta, Gir, Rishiganga, Hanuman Ganga, Sasur
Khaderi
Sharda: flows southeast another 100 km (62 mi) in Uttar Pradesh to join the Ghaghra (Karnali) as a
right-bank tributary. Teesta is a tributary of Jamuna river.
Hence, option C is correct.
87. Ans. B.
Hence, option B is correct.

88. Ans. D.
In troposphere and mesosphere, temperature declines with increasing altitude. In stratosphere and
thermosphere, temperature increases with increasing altitude.
The coldest temperatures in Earth’s atmosphere occur at the top of mesosphere, the Mesopause,
especially in the summer near the pole.
Transitional layers of atmosphere include tropopause, stratopause, and mesopause. These layers
are relatively thin and are located between major layers of atmosphere. These have contrasting
thermal properties. Hence, in these layers there is no change in temperature with change in
altitude.
Hence, option D is correct.
89. Ans. D.

Hence, option D is correct.

PAGE 1232
www.byjusexamprep.com

90. Ans. C.
Delhi: Latitude-28.38 N, Longitude-77.12 E
Chandigarh: Latitude-30.44 N, Longitude-76.47 E
Puducherry: Latitude- 11 ° 54' N, Longitude- 79 ° 48' E
Goa: Latitude- 15 °29'N. N, Longitude- 73 °48'E.
Hence, option C is correct.
91. Ans. C.

Hence, option C is correct.


92. Ans. B.
The correct order of elemental composition of an adult human body by mass is:-
1. O = 65%
2. C = 18%
3. H = 9.5%
4. N = 3.2%
5. Ca = 1.5%
6. Ph =1.2%
Hence, option B is correct.
93. Ans. A.
The Standard hydrogen electrode (abbreviated SHE), is a redox electrode which forms the basis of
the thermodynamic scale of oxidation-reduction potentials. Its absolute electrode potential is
estimated to be 4.44 ± 0.02 V at 25 °C, but to form a basis for comparison with all other electrode
reactions, hydrogen's standard electrode potential (E0) is declared to be zero volts only at 298K.
Hence, option A is correct.
94. Ans. D.
Hydrogen has two possible oxidation numbers: +1 and -1
-1 in case of hydrides.
Hence, option D is correct.
95. Ans. C.

Hence, option C is correct.

PAGE 1233
www.byjusexamprep.com

96. Ans. B.
o Altitude sickness is the negative health effect of high altitude which is caused by rapid exposure
to low amounts of oxygen at high elevation.
o It typically occurs above 2.5k metres.
o Ascending slowly is the best way to avoid altitude sickness.
Hence, option B is correct.
97. Ans. B.
Enzymes are biological catalysts. Enzymes are the catalysts involved in biological chemical reactions.
A fundamental task of proteins is to act as enzymes—catalysts that increase the rate of virtually all
the chemical reactions within cells.
A catalyst remains chemically unchanged itself at the end of a reaction.
Each enzyme has a temperature that it works optimally in, which in humans is around 98.6 degrees
Fahrenheit, 37 degrees Celsius – the normal body temperature for humans.
Hence, option B is correct.
98. Ans. B.
The absence of a nucleus is an adaptation of the red blood cell for its role. It allows the red blood
cell to contain more haemoglobin and, therefore, carry more oxygen molecules
Like red cells, platelets (thrombocytes) have no nucleus.
Monocytes have a small spherical nucleus and has abundant dark staining condensed chromatin.
There are 5 organelles found in a root hair cell. They are the: nucleus, cytoplasm, cell membrane,
cell wall and vacuole.
Hence, option B is correct.
99. Ans. C.
• Insulin is an anabolic hormone that stimulates glucose and fatty acid uptake into adipose cells,
promoting triglyceride synthesis.
• Glucagon is a peptide hormone, produced by alpha cells of the pancreas.
• It works to raise the concentration of glucose and fatty acids in the bloodstream, and is
considered to be the main catabolic hormone of the body.
Hence, option C is correct.
100. Ans. B.
The human ear can respond to minute pressure variations in the air if they are in the audible
frequency range, roughly 20 Hz - 20 kHz. In fact, tests conducted show an increased sensitivity in
the 3000 Hz range which is the optimal amplification frequency in the human ear. So, the sensitivity
of the human ear varies with the frequency of the sound. Hence, option B is correct.

PAGE 1234
www.byjusexamprep.com

101. Ans. B.
In S.I. System unit of angular momentum is "joule. Second".
SI unit of Planck's constant is joule-seconds (j-s).
So, dimensions of linear momentum are dimensions of mass times velocity
SI unit of Potential energy is Joule (J).
1 newton of force is the force required to accelerate an object with a mass of 1 kilogram 1 meter
per second per second.
Hence, option B is correct.
102. Ans. B.
1 atm: It was originally defined as the pressure exerted by 760 mm of mercury at 0 °C and standard
gravity (g = 9.80665 m/s2).
Hence, option B is correct.
103. Ans. D.
• A transistor is a semiconductor device that is used to boost or switch electronic signals &
electrical power.
• It is typically made of semiconductor material and has at least three terminals for connecting to
an external circuit.
• Bipolar Junction Transistor constructed with the three doped Semiconductor Regions (Base,
Collector, and Emitter) are separated by two p-n Junctions in a transistor.
Hence, option D is correct.
104. Ans. D.
o The emission of gamma rays does not alter the number of protons or neutrons in the nucleus
but instead has the effect of moving the nucleus from a higher to a lower energy state (unstable
to stable).
Hence, option D is correct.
105. Ans. D.
In dealing with a coil spring the spring constant will depend on the stiffness of the spring material,
the thickness of the wire from which the spring is wound and, diameter of the turns of the coil, the
number of turns per unit length and the overall length of the spring.
Hence, option D is correct.
106. Ans. A.
Madhya Pradesh has the largest Tribal population. It has more than 1.6 crore Schedule Tribe
population whom are 21% of state population according to 2011 census.
Second highest is Odisha by numbers.
The population of Mizoram in 2001 Census has been 888,573. Of them, 839,310 are Scheduled
Tribes (STs) constituting 94.5 per cent of the total population of the state.

PAGE 1235
www.byjusexamprep.com

The population of Nagaland in 2001 Census has been 1,990,036. Of these, 1,774,026 are Scheduled
Tribes (STs) constituting 89.1 per cent of the total population of the state.
The total population of Arunachal Pradesh in 2001 Census has been 1,097,968. Of these 705,158
persons are Scheduled Tribes (STs), constituting 64.2 per cent of the total population.
Hence, option A is correct.
107. Ans. B.
The epicentre of Earthquakes can be over both oceanic and continental surfaces. Epicentres above
the oceanic surface is the cause of tsunamis of large intensity.
An earthquake's hypocentre is the position where the strain energy stored in the rock is first
released, marking the point where the fault begins to rupture.
Hence, option B is correct.
108. Ans. A.

Hence, option A is correct.


109. Ans. D.
Potassium bicarbonate (also known as potassium hydrogen carbonate or potassium acid carbonate)
is the inorganic compound with the chemical formula KHCO3. It is a white solid
Sodium bicarbonate is a white solid that is crystalline, but often appears as a fine powder.
LiHCO3 is found in solution form while rest alkali metals hydrogen carbonates are solid due to the
anomalous properties of Li and its diagonal relationship with the Mg.
Hence, option D is correct.
110. Ans. A.
Excessive use of nitrate including manure may be responsible for the presence of toxic substance in
groundwater. Phosphate can also enter the groundwater via excessive use of fertilizers. High
application rates of nitrogen containing fertilizers combined with the high-water solubility of
nitrate, leads to increased runoff into surface water as well as leaching into groundwater, thereby
causing groundwater pollution.
Only a fraction of the nitrogen-based fertilizers, is converted to produce and other plant matter.
Nitrate pollution in groundwater comes from pit latrines, it can cause blue baby syndrome.
Hence, option A is correct.
111. Ans. A.
• Fluoride deficiency is a disorder which may cause increased dental caries due to a lack of
fluoride in the diet.
• Dental caries or tooth decay is the breakdown of dental tissues by the acidic products released
by the "bacterial fermentation of dietary carbohydrates”.
Hence, option A is correct.

PAGE 1236
www.byjusexamprep.com

112. Ans. A.
Dolomite is a limestone compound that contains calcium and magnesium. It provides valuable
nutrients to plants and helps change the pH of the soil by raising it to match the plants' needs.
It even provides more nutrients than straight lime. It is often used in addition to balanced fertilizers,
particularly with seeding fruit.
Hence, option A is correct.
113. Ans. C.
Paper production is basically a two-step process in which a fibrous raw material is first converted
into pulp, and then the pulp is converted into paper. The harvested wood is first processed so that
the fibres are separated from the unusable fraction of the wood, the lignin. Pulp making can be
done mechanically or chemically. In the chemical pulping process, alkali (e.g., sodium hydroxide in
the form of a lye or caustic soda) is usually used to remove the lignin that binds the fibres. So, for
degumming of raw material, caustic soda is used. Hence, option C is correct.
114. Ans. A.
• Allotropy is the property of some chemical elements to exist in two or more different forms, or
allotropes, when found in nature.
• There are several allotropes of carbon like diamond, graphite, graphene, fullerene, etc.
• Coal is not an allotrope of carbon because it is not 100% carbon. It contains lot of silica, sulphur
and other compounds of carbon apart from carbon.
Hence, option A is correct
115. Ans. D.
In animals, parthenogenesis means development of an embryo from an unfertilized egg cell. In
plants parthenogenesis is a component process of apomixis.
Apogamy is a type of reproduction, occurring in some ferns, in which the sporophyte develops from
the gametophyte without fusion of gametes.
Adventitious embryony also called sporophytic budding, is a type of apomixis in which an embryo
develops directly from nucellus or integument tissue.
So, all three are correct.
Hence, option D is correct.
116. Ans. C.
Carbon monoxide (CO) is very dangerous because it binds with the haemoglobin more efficiently
than oxygen does. It "hogs the seats" so that oxygen can't get a ride. And those CO molecules keep
riding around, never giving their seats up to the oxygen. This means there is no way to get oxygen
to the brain, heart, or other cells and those cells start to die. And finally, it leads to suffocation and
even death of a person.
Hence, option C is correct.

PAGE 1237
www.byjusexamprep.com

117. Ans. D.
The HIV virus continuously invades and destroys T-cells (lymphocytes). Eventually the body can't
replace the T-cells fast enough, and the immune system deteriorates. When the immune system
has deteriorated to the point that it is unable to fight off diseases, a person is diagnosed with AIDS.
Hence, option D is correct.
118. Ans. C.
About 65% of the electricity consumed in India is generated by thermal power plants, 22% by
hydroelectric power plants, 3% by nuclear power plants and rest by 10% from other alternate
sources like solar, wind, biomass etc.
India is the sixth largest in terms of power generation. Hence, option C is correct.
119. Ans. A.
Amphibians are one of the most sensitive groups to change, whether that is caused by habitat loss,
invasive species, disease, trade or climate change. Their sensitivity to environmental threats has
been generally linked to the observation that amphibians are characterized by a rather permeable
skin. Given their complex life cycles and other traits, amphibians are often recognized as indicators
of ecosystem health.
Hence, option A is correct.
120. Ans. C.
• Decomposing waste in the landfills, by the action of microorganisms, produce methane and
carbon dioxide.
• Landfill gas consists of 40-60% of methane and rest is carbon dioxide. Trace amount of volatile
organic compounds (<1) are also present which mainly comprise of hydrocarbons.
Hence, option C is correct.

PAGE 1238
www.byjusexamprep.com

Elementary Mathematics
1. Ans. B.
As,

Hence option (B)


2. Ans. C.
Since distance is same in both ride.

Then, the average speed =

Hence option (C)


3. Ans. B.

As we know that,

And,

Required difference = 3025 – 385 = 2640


Hence option (B)

PAGE 1239
www.byjusexamprep.com

4. Ans. D.
As we know that
(xn + yn) is divided by (x + y)
If n be odd, then remainder be zero.
From question,
(135 + 165) + (145 + 155)
As, (135 + 165) is divided by 29 so, remainder be zero.
Also, (145 + 155) is divided by 29 so, remainder be zero.
Hence, 135 + 145 + 155 + 165 is divided by 29 then remainder will be zero.
Hence option (D)
5. Ans. C.
As, s = {1, 2, 3, 4, 5, 6, 7, 8, 9, 10}
According to question,

Hence possible number of pairs = 8


Hence option (C)
6. Ans. D.
Given, class interval be 10-15
Frequency be 30
Frequency density =

Hence option (D)


7. Ans. D.
When the lower and the upper class limit is included, then it is an inclusive class interval. For
example - 220 - 234, 235 - 249 ..... etc. are inclusive type of class intervals.
When the lower limit is included, but the upper limit is excluded, then it is an exclusive class
interval. For example - 150 - 153, 153 - 156.....etc are exclusive type of class intervals .e.g. 15-20,
20-25 etc
Hence both the statements are incorrect.
Hence option (D)

PAGE 1240
www.byjusexamprep.com

8. Ans. C.
As a particular sector of pie chart for corporate tax be 1080 at the center

Then, percentage of income from corporate tax to total funds =

Hence option (C)


9. Ans. B.
Number of total students = 100
Number of boys = 70
Number of girls = 30
Average of boys = 75
Total marks of boys = 75×70 = 5250
Average of class = 72
Total marks of class = 72×100 = 7200

Average marks of girls =

Hence option (B)


10. Ans. B.
Discrete data is information that can be categorized into a classification. Discrete data is based on
counts. Only a finite number of values is possible, and the values cannot be subdivided meaningfully.
Hence number of credit card held by an individual can be treated as discrete data.
Hence option (B)
11. Ans. A.
As mean of 20 observation = 17
Then, sum of 20 observations = 17×20 = 340
According to question,

Hence option (A)


12. Ans. C.
Let the cricketer’s average be x run per match
According to question,

=>
New average = 42 + 6 = 48
Hence option (C)

PAGE 1241
www.byjusexamprep.com

13. Ans. D.
Let data be p, q, r, s, 20.5, t, u, v, w
After increasing the largest four number by 2 then new data be
p, q, r, s, 20.5, t + 2, u + 2, v + 2, w + 2 …..(i)
Clearly, from (i),
Median remains same.
Hence option (D)
14. Ans. A.
Data in ascending order be
x -3.5, x-3, x-2.5, x-2, x-0.5, x+0.5, x+4, x+5
Here number of term is even.

Median =

Hence option (A)


15. Ans. A.
Total score of class X = 83x
Total score of class Y = 76y
Total score of class Z = 85z
According to question,

=>
=> ……..(i)
Also,

=>
=> ……..(ii)
From equation (i) and (ii), we get

= t (say)
Then, x = 3t, y = 4t, z = 5t
Now,

PAGE 1242
www.byjusexamprep.com

Hence average score of x, y and z be 81.5


Hence option (A)
16. Ans. B.
As, tan10 tan20 tan30 ………..tan870 tan880 tan890
= tan(900 - 890)tan(900 - 880)……………tan880 tan890
= cot890 cot880 …………..tan880 tan890

=
=1
Hence option (B)
17. Ans. B.

Here, ⊿POA≅⊿POB (BY AAS) then AO = BO


⊿POB≅⊿POC (BY AAS) then BO = CO
i.e. AO = BO = CO (Distance of sides from vertex are equal)
AS, we know that, distance between the circumcentre and vertices are equal.
Hence, P must be circumcentre.
Hence option (B)

PAGE 1243
www.byjusexamprep.com

18. Ans. D.
Statement 1:
As
And

Hence statement 1 is correct.


Statement 2:

Hence statement 2 is correct.


Statement 3:

Hence statement 3 is correct.


Hence option (D)

PAGE 1244
www.byjusexamprep.com

19. Ans. C.

As,

Hence altitude of sun be


Hence option (C)
20. Ans. C.
As,
……. (i)

As,

⇒ …….. (ii)
Comparing (i) and (ii) we can conclude
tan x =1

Hence option (C)
21. Ans. D.
As we know that

Statement 1:

PAGE 1245
www.byjusexamprep.com

Clearly,
sin 1o < sin 1c
Hence statement 1 is incorrect.
Statement 2:

Clearly,
cos 1o > cos 1c
Hence statement 2 is incorrect.
Statement 3:

Clearly,
tan 1o < tan 1c
Hence statement 3 is incorrect.
Hence option (D)
22. Ans. A.
As we know that
=>
=> ……… (i)

As,

[From (i)]

PAGE 1246
www.byjusexamprep.com

Hence option (A)


23. Ans. B.
As,

⇒ ….. (i)

A will be minimum when

Then
[From (i)]

:
[As, ]
Then

Hence

Hence option (B)

PAGE 1247
www.byjusexamprep.com

24. Ans. D.
As,

After comparing, we get


Now,

Hence option (D)


25. Ans. C.
As,

=>

=> …….. (i)


Also,


[From (i)]

Now,

Hence option (C)

PAGE 1248
www.byjusexamprep.com

26. Ans. D.
As,


Also,
As,

Hence option (D)


27. Ans. A.

In Triangle ACD:


In Triangle BCD:



Then,



Hence required distance be m
Hence option (A)

PAGE 1249
www.byjusexamprep.com

28. Ans. B.

As angle of elevation = angle of depression


In triangle ABC:

⇒ ………. (i)
In triangle BCD:

⇒ ……. (ii)

From (i) and (ii)

Hence distance travelled by car during time be

Hence option (B)


29. Ans. B.

we have
sinx = 1-sin2x = cos2x
now

PAGE 1250
www.byjusexamprep.com

= sin4x + 2sin3x + sin2x


=sin2x(sin2x + 2sinx + 1)
=sin2x(sinx + 1)2
=(1-sinx)(1+sinx)(1+sinx)
=(1-sin2x)(1+sinx)
=cos2x(1+sinx)
=sinx(1+sinx)
=sinx + sin2x
=1
30. Ans. B.
As,

Let
As we know that

Then,

Hence

Hence option (B)


31. Ans. D.

PAGE 1251
www.byjusexamprep.com

Here OCD be an equilateral triangle.


In right angled triangle OCM

Here, required radius of circle be

Hence option (D)


32. Ans. B.
Perimeter of circle when radius r = Perimeter of square with side a
=>

=> …………(i)

Hence option (B)


33. Ans. A.

Area of right-angled triangle ABC

Hence option (A)

PAGE 1252
www.byjusexamprep.com

34. Ans. C.
Given diameter of wheel be 80 cm.
So, radius (r) of wheel

Time

Distance covered by car

m cm

Number of revolutions of wheel

Hence option (C)


35. Ans. D.
Canvas used = Curved surface Area of cylinder + Curved surface Area of cone

Hence option (D)


36. Ans. A.
Radius of cylinder (r)
Height of cylinder (H)
Curved surface area of cylinder

Hence option (A)

PAGE 1253
www.byjusexamprep.com

37. Ans. B.

Radius of cylinder radius of cone

Height of cone (h)

Slant height (l)

m
Hence option (B)
38. Ans. C.

Given, wheel II revolves n times.


Then,

(Circumference of wheel I) m

Hence option (C)

PAGE 1254
www.byjusexamprep.com

39. Ans. A.

Radius of semi-circle be r Area of largest triangle inscribed in a semi-circle

Hence option (A)


40. Ans. C.
We can solve this question by hit and trial method.
Let
Then sides of triangle be

Area of

Option (a):

Area

Option (b):

Area

PAGE 1255
www.byjusexamprep.com

Option (c):
Area

Hence option (C)


41. Ans. A.
Given radius of sphere (r) be 3 cm

As, volume of sphere

Hence option (A)


42. Ans. A.

As, [Because both triangle is congruent]


In right angled triangle OAB:

Hence option (A)

PAGE 1256
www.byjusexamprep.com

43. Ans. D.

Given, …….. (i)

As,
So,

From (i)


Hence height from the base at which the section is made

cm
Hence option (D)
44. Ans. C.
Given diameter of base and height of cylinder vessel be 2 m and 3.5 m respectively.
Then radius of base (r)

m
Let height of roof be H m
Then volume of roof ……. (i)
Volume of cylindrical vessel

…… (ii)
As

PAGE 1257
www.byjusexamprep.com

Then,




Hence option (C)
45. Ans. C.

Here all triangles are equilateral triangle and formed after taking the mid-point of sides.
Let Area of be A then Area of nth triangle …….. (i)

According to question,

Hence,
Hence option (C)
46. Ans. A.
Given, height (h) = 24 cm

Radius of bottom circle (r)

Also, given capacity of glass


i.e. volume of glass is in shape of frustum be ……. (i)
As, volume of frustum

………….(ii)
After comparing (i) and (ii), we get

Hence option (A)

PAGE 1258
www.byjusexamprep.com

47. Ans. B.
Let side of square be x then perimeter of square = 4x
Perimeter of circle having radius
According to question

Area of square

Area of circle

Here,
Hence Area of circle is greater than area of square.
Hence option (B)
48. Ans. B.

As DX and DP are pair of tangents from D to circle and we know that length of pair of tangent be
same in this case.
So, Let DX = DP = 5 cm
Similarly,
Let CX = CY = 8 cm
BY = BZ = 9 cm
AP = AZ = 7 cm
Now, according to question
BD = 9 + 8 = 17 cm
AC = 7 + 5 = 12 cm
AC + BD = 17 + 12 = 29 cm

PAGE 1259
www.byjusexamprep.com

Also,
AB + CD = 29 cm
Hence the sum of a pair of opposite side equals to sum of the other pairs of opposite sides.
Hence option (B)
49. Ans. C.
As surface Area of spherical vessel

Cost of painting per square centimetre


Hence option (C)
50. Ans. C.
Let radius and height of base of solid circular cylinder be r and h respectively.
Given,

⇒ ………. (i)

Volume of cylinder




so, [From (i)]



Total surface Area of cylinder

Hence option (C)

PAGE 1260
www.byjusexamprep.com

51. Ans. A.

Volume of [r = radius]

Volume of [As h=2r]

Volume of building

According to question:



Hence, height of building
Hence option (A)
52. Ans. D.
Circumference of Wheel

Number of rounds

Hence option (D)

PAGE 1261
www.byjusexamprep.com

53. Ans. B.

From Pythagoras theorem,

Now,

Then

Hence option (B)


54. Ans. D.

Given, ……… (i)


And
⇒ ……… (ii)



⇒ ……. (iii)
Also,
(R-r)2 = (R+r)2 -4 Rr



Hence option (D)

PAGE 1262
www.byjusexamprep.com

55. Ans. D.
Figure 1:

Area of triangle ABC= 1/2.5.12=30 cm square


Area of triangle BCD= 1/2.BC.CD.sin30=5CD/4 cm square
Area of triangle ACD= 1/2.AC.CD.sin60 = CD cm square
Area of triangle ABC= Area of triangle BCD+ Area of triangle ACD
30 cm square = 5CD/4 cm square + CD cm square
120 = (5+ ) CD
CD= 120/(5+ )
56. Ans. C.
Given radius of cone = 8.4 m
Vertical height of cone = 3.5 m

Number of bag =

Hence option (C)


57. Ans. B.

Weight of cube = 9000 kg

PAGE 1263
www.byjusexamprep.com

Volume of cube Volume of square bar


Now, no of cube cut off × volume of cube of length m = volume of cube of length 1 m

Weight of one cube

Hence option (B)


58. Ans. C.
Let radius and height of right circular cone be r and h respectively.
According to question,


⇒ ………. (i)
Total surface area

PAGE 1264
www.byjusexamprep.com

Volume of cylinder

[r = h]

Hence option (C)


59. Ans. A.

As

As,

Also,

Then

Hence option (A)


60. Ans. B.
Let breadth of room (b) = x m.
Length of room (l) = 2x m.

PAGE 1265
www.byjusexamprep.com

Given, height of room = 4 m.


According to question,


So, length of room


Breadth of room
Area of the floor

Hence option (B)


61. Ans. B.
According to concept of mixture and allegation

Thus,
Percentage of Boys in the group

Hence option (B)


62. Ans. D.
As

Ratio of time be

Hence option (D)

PAGE 1266
www.byjusexamprep.com

63. Ans. B.
Speed of Train A

Speed of Train B

Let length of Train A = 2x m


Length of Train B = x m
Total length = 2x + x = 3x m
Combined speed

As, distance = speed × time



Let length of platform = y m


According to question,


Hence option (B)
64. Ans. C.
Total work = LCM of 2 and 6:
X will perform unit/hr

Y will perform unit/hr

X perform between 10:00 AM to 11:00 AM = 6 – 3 = 3 unit.


Now, x and y together will perform = 3 + 1 = 4 unit.
Time taken for remaining work = min.

= 45 min.
Hence tank be filled at 11:45 AM
Hence option (C)
65. Ans. B.
In case of x:
Investment in 1st 3 months = 700
Again, investment in next 3 months

Investment in next 6 months

PAGE 1267
www.byjusexamprep.com

Total investment in 1 year by x

Total investment in 1 year by y

Required amount received by x

Hence x will receive Rs. 366


Hence option (B)
66. Ans. D.
We can solve this question in different cases.
Case 1:
Let income of Mahesh = 2k
Income of Kamal = 4k
Expanses of Mahesh = k
Expanses of Kamal = 3k
Saving of Mahesh = 2k – k = k
Saving of Kamal = 4k – 3k = k
i.e. saving of Mahesh = saving of Kamal
case 2:
Let income of Mahesh = 3k
Income of Kamal = 6k
Expanses of Mahesh = k
Expanses of Kamal = 3k
Saving of Mahesh = 3k – k = 2k
Saving of Kamal = 6k – 3k = 3k
i.e. saving of Mahesh is less than saving of Kamal.
Case 3:
Let income of Mahesh = 5k
Income of Kamal = 10k
Expanses of Mahesh = 3k
Expanses of Kamal = 9k
Saving of Mahesh = 5k – 3k = 2k

PAGE 1268
www.byjusexamprep.com

Saving of Kamal = 10k – 9k = k


i.e. saving of Mahesh is more than saving of Kamal.
Clearly, it is not possible to determine who saves more.
Hence option (D)
67. Ans. C.
As,

⇒ ……… (i)

Also,

……. (ii)

And

⇒ …….. (iii)

Subtract (iii) from (ii)

……. (iv)
From (i) and (iv)

Hence option (C)


68. Ans. D.
As we know that unit digit in will be 1
Now,

……… (i)

PAGE 1269
www.byjusexamprep.com

The unit digit in will be 1 and the unit digit in will be 3.


Thus, unit digit of will be
Hence unit digit of will be 3
Hence option (D)
69. Ans. C.
Let
Taking log both sides



Now, taking antilog both sides,

Antilog of 14.313
Hence number of digits (n)

Hence option (C)


70. Ans. D.
As,
………. (i)
And



⇒ [From (i)]
⇒ ……… (ii)

From (i) and (ii)

Putting the value of q in (i)

PAGE 1270
www.byjusexamprep.com

Now,

Hence option (D)


71. Ans. D.
As,

Here 341 is divisible by 11.


Hence will be divisible by 11
Hence option (D)
72. Ans. C.
As ratio of investment of
be
Duration of investment of
be
Multiplying ratio of investment and time we get ratio
……….. (i)
i.e. F and G equal maximum profit [From (i)]
Hence option (C)
73. Ans. C.
As,


Taking log both sides

PAGE 1271
www.byjusexamprep.com

Either

Or

Hence option (C)


74. Ans. A.
Let x and y be and respectively.
Then,

i.e. is an odd number but not necessarily a multiple of 4


Hence only statement 1 is correct.
Hence option (A)
75. Ans. D.
Let speed of boat and current be x and y.
According to question,
….. (i)

……… (ii)

Adding (i) and (ii) we get


From (i)

Thus speed of current be 4 km/hr


Hence option (D)

PAGE 1272
www.byjusexamprep.com

76. Ans. A.
Given,

Hence option (A)


77. Ans. D.
Statement 1:
If we take two prime numbers viz 2 and 3 then HCF be 1
Statement 2:
If 7 and 25 be the prime and composite numbers respectively
Then the HCF of 7 and 25 be 1
Statement 3:
If 25 and 16 be the two composite numbers
Then their HCF be also 1
Hence all the three statements are correct.
Hence option (D)
78. Ans. D.
Let cost price

Market price

Selling price after distance

Profit

Hence option (D)

PAGE 1273
www.byjusexamprep.com

79. Ans. C.
Given, number be 1729
and

i.e. 1729 can be written as the sum of the cubes of two positive integers in two ways only.
Hence option (C)
80. Ans. D.
Given, cost of 2.5 kg rice

Cost of 9 kg rice

cost of 4 kg pulse
Cost of 14 kg pulses cost of 1.5 kg tea

Cost of 2 kg tea cost of 5 kg nuts

Cost of 11 kg nuts

Hence required cost be Rs. 4620


Hence option (D)
81. Ans. A.
As p and q are the roots of
Then sum of roots
⇒ …….. (i)
Also, product of roots = pq = q



If from (i)




Hence p = 0 or 1
Hence option (A)

PAGE 1274
www.byjusexamprep.com

82. Ans. C.
Let maximum marks be p
Marks obtained by A

Minimum passing marks for A …… (i)

Marks obtained by B

Minimum passing marks for B …… (ii)

As (i) and (ii), we get

Hence option (C)


83. Ans. B.
Let original speed be 100 then increased speed be 120 time taken in original speed be t
Then time taken in increase speed be
As, distance in the both cases will be same
So,




Hence option (B)
84. Ans. D.
Let efficiency of B be 100
So, efficiency of A be 150

According to question,

PAGE 1275
www.byjusexamprep.com


Now, Let A and B completed the work in x hours

Hence the required hours be


Hence option (D)
85. Ans. D.
Let k be total number of workers
According to question,
Women worker

Married women worker

Men Worker

Married Men worker

Hence, Married Women: Married man


Hence option (D)
86. Ans. A.
If any number in the term

where p = prime number


a and p are co-prime then remainder will be 1.
Given, p = 101 and a = 2

As, ……. (i)

Equation (i) satisfies the standard form so, required remainder will be 1
Hence option (A)

PAGE 1276
www.byjusexamprep.com

87. Ans. B.
As, ……. (i)
If and are the roots of (i)
Then

If and would be roots then


Sum of roots =

Product of roots

Quadratic equation be


Hence quadratic polynomial be

Hence option (B)


88. Ans. D.
Option (A):
[Terminating decimal]
Option (B):
[Terminating decimal]
Option (C):
[Terminating decimal]
Option (D):
[Non-Terminating repeating decimal]
Hence option (D)

PAGE 1277
www.byjusexamprep.com

89. Ans. C.

Total number of rounds

Speed of A : Speed of B

As we know that time

So, time taken by A : time taken by B

LCM of (4, 5) = 20

Number of Rounds completed by A =

Number of Rounds Completed by B =

When A will complete 5 rounds then B will complete 4 round and they meet at a point.
i.e. If A will complete i.e. 15 rounds
Then he will meet to B.
Hence A will trice pass the B.
Hence option (C)
90. Ans. B.
As, a boy saves Rs. 4.65 daily.

From the table,


Clearly, in 20 days the boy will be able to save an exact number of rupees.
Hence option (B)
91. Ans. B.
As

PAGE 1278
www.byjusexamprep.com

So,
Number of elements in
Also,

So,
Number of elements in
Hence option (B)
92. Ans. C.
Let two positive integers be x and y such that
According to question,
……. (i)
……. (ii)
From (i) and (ii)



As,
If
Then [Not possible]
If
Then

Then
Hence option (C)
93. Ans. B.
As we know that for real roots of quadratic equation,
Discriminant

⇒ ……….. (i)
The pairs which follows the condition will be

Hence total number of pairs


Hence option (B)

PAGE 1279
www.byjusexamprep.com

94. Ans. B.
Let the number be x
According to question,


Hence option (B)


95. Ans. B.
As,
Number = LCM of (2, 3, 4, 5, 6) – (Difference of divisors and remainder)
Differences of divisors and remainder ………… (i)
As number when divided by 2, 3, 4, 5, 6 gives remainder 1, 2, 3, 4,5 respectively
Then,

From (i)
Number
Other number
Clearly, there is one number of set A which is below 100
Hence option (B)
96. Ans. A.
As,
……… (i)
According to concept of addition
……. (ii)
………. (iii)

PAGE 1280
www.byjusexamprep.com

From (i), (ii) and (iii)


We can simply say that

………. (iv)
Q would be maximum when P and R will be minimum.
So,

From (iv)



Hence option (A)
97. Ans. B.
As,

Then

Similarly,

So,

Hence option (B)

PAGE 1281
www.byjusexamprep.com

98. Ans. A.
As,

Hence option (A)


99. Ans. D.
As,

=>

Sum of roots

Product of roots

As ……….. (i)
………… (ii)
From (i) and (ii)

[It is not possible because ]

So,

Hence total value of would be 15.


Hence option (D)

PAGE 1282
www.byjusexamprep.com

100. Ans. B.
As,

According to question

=>
Hence option (B)

PAGE 1283
www.byjusexamprep.com

CDS I 2016
ENGLISH
Direction: Look at the underlined part of the 5. It was the mother of the girl of whose voice
sentence. Below the sentence are given three I had recognized.
possible substitutions for the underlined part. If
A. whose voice
one of them (A), (B) or (C) is better than the
underlined part, indicate your response against B. the voice of who
the corresponding letter (A), (B) or (C). If none of C. voice whose
the substitutions improves the sentence,
indicate (D) as your response. Thus, a “No D. No improvement
improvement” response will be signified by the 6. The Executive Council is consisted of ten
letter (D). members.
1. The police accused him for theft. A. consists of
A. with B. comprises of
B. in C. constituted of
C. of D. No improvement
D. No improvement
7. The maid was laying the table for dinner.
2. He wanted that I left immediately.
A. setting up
A. I may leave
B. lying
B. me to leave
C. sorting out
C. I leave
D. No improvement
D. No improvement
8. We have so arranged the matters and one
3. This is to certify that I know Mr. J. Mathews
of us is always on duty.
since 1970.
A. that one of us
A. am knowing
B. had known B. so that one of us

C. have known C. such that one of us

D. No improvement D. No improvement
4. They took away everything that belonged 9. Hardly have we got into the forest when it
to him. began to rain.
A. that had been belonging A. Hardly we got
B. that belong B. We had hardly got
C. that has been belonging C. We had got hard
D. No improvement D. No improvement

PAGE 1284
www.byjusexamprep.com

10. Each time he felt tired he lied down. 16. It is an ancient, historical place and it once
belongs to the Pandavas.
A. lies
A. belonged
B. lays
B. belonging
C. lay
C. belong
D. No improvement
D. No improvement
11. Though it was raining, but I went out.
17. Since we were knowing the correct route,
A. but yet I we did not worry at all.
B. I A. knew
C. however I B. have known

D. No improvement C. know

12. There is no chance of success unless you do D. No improvement


not work hard. 18. Our country can progress when only
A. unless you work people work hard.

B. until your working A. when people only work hard


B. when people work hard only
C. until you do not work
C. only when people work hard
D. No improvement
D. No improvement
13. She has grown too old to do little work.
19. Wake me up when father will come.
A. some
A. comes
B. any
B. will have come
C. a little
C. came
D. No improvement
D. No improvement
14. No one enjoys to deceive his family.
20. Do take an umbrella with you lest you do
A. deceiving not get wet.
B. for deceiving A. lest you should get wet

C. deceive B. lest you should not get wet

D. No improvement C. lest you might not get wet

15. Have you ever saw the flower of a pumpkin D. No improvement


plant? Direction: The following question consists of a
sentence, the parts of which have been jumbled.
A. see
These parts have been labelled P, Q, R and S.
B. seeing Given below each sentence are four sequences
namely A, B, C and D. You are required to re-
C. seen
arrange the jumbled parts of the sentence and
D. No improvement mark your response accordingly.

PAGE 1285
www.byjusexamprep.com

21. P. that is would affect the investigation 24. P. my brother


process
Q. to attend his friends wedding
Q. they refused
R. is going to Chennai
R. of these raids saying
S. tomorrow
S. to divulge the venues
A. P S Q R B. QPSR
A. P Q R S B. SRPQ
C. R Q P S D. PRSQ
C. Q S R P D. RPQS
Direction: The following question consists of a
Direction: The following question consists of a sentence, the parts of which have been jumbled.
sentence, the parts of which have been jumbled. These parts have been labelled P, Q, R and S.
These parts have been labelled P, Q, R and S. Given below each sentence are four sequences
Given below each sentence are four sequences namely A, B, C and D. You are required to re-
namely A, B, C and D. You are required to re- arrange the jumbled parts of the sentence and
arrange the jumbled parts of the sentence and mark your response accordingly.
mark your response accordingly.
25. P. Quickly
22. P. that he already has
Q. he gave orders
Q. buying things
R. to catch the thief
R. that rich man
S. to his men
S. goes on
A. S P R Q B. QSRP
A. P S Q R B. RSQP
C. P S R Q D. RSPQ
C. S Q R P D. RPQS
Direction: The following question consists of a
Direction: The following question consists of a
sentence, the parts of which have been jumbled.
sentence, the parts of which have been jumbled.
These parts have been labelled P, Q, R and S.
These parts have been labelled P, Q, R and S.
Given below each sentence are four sequences
Given below each sentence are four sequences
namely A, B, C and D. You are required to re-
namely A, B, C and D. You are required to re-
arrange the jumbled parts of the sentence and
arrange the jumbled parts of the sentence and
mark your response accordingly.
mark your response accordingly.
26. P. to give a definition
23. P. the police commissioner rushed
Q. if I were
Q. the crowd
R. to control R. I would begin

S. the police force S. like this

A. Q R S P B. PQRS A. Q P R S B. PQRS

C. P S R Q D. RSPQ C. S R Q P D. RSPQ

Direction: The following question consists of a Direction: The following question consists of a
sentence, the parts of which have been jumbled. sentence, the parts of which have been jumbled.
These parts have been labelled P, Q, R and S. These parts have been labelled P, Q, R and S.
Given below each sentence are four sequences Given below each sentence are four sequences
namely A, B, C and D. You are required to re- namely A, B, C and D. You are required to re-
arrange the jumbled parts of the sentence and arrange the jumbled parts of the sentence and
mark your response accordingly. mark your response accordingly.

PAGE 1286
www.byjusexamprep.com

27. A. Q S P R B. QRSP
C. P S R Q D. P R S Q
Direction: The following question consists of a
A. R Q S P B. QRPS sentence, the parts of which have been jumbled.
These parts have been labelled P, Q, R and S.
C. P R S Q D. S P Q R
Given below each sentence are four sequences
Direction: The following question consists of a namely A, B, C and D. You are required to
sentence, the parts of which have been jumbled. re-arrange the jumbled parts of the sentence
These parts have been labelled P, Q, R and S. and mark your response accordingly.
Given below each sentence are four sequences
namely A, B, C and D. You are required to re- 31. The bird-catcher
arrange the jumbled parts of the sentence and
mark your response accordingly.
28.

A. Q R S P B. QSRP
A. P S R Q B. PRSQ
C. P Q S R D. P R Q S
C. S P Q R D. R Q S P
Direction: The following question consists of a Direction: The following question consists of a
sentence, the parts of which have been jumbled. sentence, the parts of which have been jumbled.
These parts have been labelled P, Q, R and S. These parts have been labelled P, Q, R and S.
Given below each sentence are four sequences Given below each sentence are four sequences
namely A, B, C and D. You are required to re- namely A, B, C and D. You are required to re-
arrange the jumbled parts of the sentence and arrange the jumbled parts of the sentence and
mark your response accordingly. mark your response accordingly.

29. I believed then 32. Man is a biological being (P)/ his physical
and material needs (Q)/ confined to (R)/ not
merely (S)
A. R S P Q B. SRQP
C. R P S Q D. S P R Q
A. P R Q S B. PSRQ
Direction: The following question consists of a
C. R P Q S D. R P S Q sentence, the parts of which have been jumbled.
Direction: The following question consists of a These parts have been labelled P, Q, R and S.
sentence, the parts of which have been jumbled. Given below each sentence are four sequences
These parts have been labelled P, Q, R and S. namely A, B, C and D. You are required to
Given below each sentence are four sequences re-arrange the jumbled parts of the sentence
namely A, B, C and D. You are required to and mark your response accordingly.
re-arrange the jumbled parts of the sentence 33. A gang of robbers
and mark your response accordingly.
30.

PAGE 1287
www.byjusexamprep.com

A. Q S R P B. SQPR S6 : In the trees lived a nightingale tha sang


so sweetly that all who passed by stood still
C. S P Q R D. Q P S R
and listened.
Direction: The following question consists of a P : In the middle of the garden there was a
sentence, the parts of which have been jumbled. lovely forest with tall trees and deep lakes.
These parts have been labelled P, Q, R and S.
Given below each sentence are four sequences Q : In this garden were to be seen the most
namely A, B, C and D. You are required to wonderful flowers with silver bells ties to
re-arrange the jumbled parts of the sentence them.
and mark your response accordingly. R : The garden was so large that even the
34. The opposition members gardener himself did not know where it
began and where it ended.
S : These bells always sounded so that
nobody should pass by without noticing the
flowers.

A. S P Q R B. QRPS The proper sequence should be

C. R S P Q D. S R Q P A. Q P R S B. SPQR
C. Q S R P D. Q P S R
Direction: The following question consists of a
sentence, the parts of which have been jumbled. Direction: In the given question six sentences of
These parts have been labelled P, Q, R and S. a passage are a given. The first and sixth
Given below each sentence are four sequences sentences are given in the beginning as S1 and
namely A, B, C and D. You are required to re- S6. The middle four sentences in each have been
arrange the jumbled parts of the sentence and jumbled up and labelled P, Q, R and S. You are
mark your response accordingly. required to find the proper sequence of the four
sentences and mark your response accordingly.
35. When
37. S1 : One of the first things the learning of
new language teaches you is that language
comes from the region of the unconscious.
S6 : The test of how much you know is : how
much can you say without having to think
how you are going to say it?

A. S P R Q B. RSQP P : What is often meant by “thinking in a


language” is really the ability to use it
C. S Q P R D. Q P S R without thinking about it.
Direction: In the given question six sentences of Q : We grown-up people have to filter it
a passage are a given. The first and sixth through our minds – a much more laborious
sentences are given in the beginning as S1 and process.
S6. The middle four sentences in each have been
jumbled up and labelled P, Q, R and S. You are R : That is why children learn a new
required to find the proper sequence of the four language so effortlessly : it comes straight
sentences and mark your response accordingly. from their instincts.
S : But we cannot say that we know a
36. S1 : Once upon a time there was a king who
language, or know what we have studied of
had a wonderfully nice garden.
it, until we can use it instinctively.

PAGE 1288
www.byjusexamprep.com

The proper sequence should be P : I have heard of a father who, having


offered to take the baby out in a
A. S Q R P B. RPSQ
perambulator, was tempted by the sunny
C. P Q S R D. R Q S P morning to pause on his journey and slip
into a public house for a glass of beer.
Direction: In the given question six sentences of
a passage are a given. The first and sixth Q : A little later, his wife had to do some
sentences are given in the beginning as S1 and shopping which took her past the public
S6. The middle four sentences in each have been house where to her horror, she discovered
jumbled up and labelled P, Q, R and S. You are her sleeping baby.
required to find the proper sequence of the four R : Leaving the perambulator outside, he
sentences and mark your response accordingly. disappeared into the drink shop.
38. S1 : For seventeen years she led a sheltered S : Indignant at her husband’s behaviour,
life in the convent. she decided to teach him a lesson.
S6 : Two years later she left the Loreto The proper sequence should be
Convent where she had spent many happy A. P Q R S B. PRQS
and useful years.
C. P S Q R D. P Q S R
P : Her heart went out to the people living
Direction: In the given question six sentences of
there.
a passage are a given. The first and sixth
Q : In 1946 she asked for permission to work sentences are given in the beginning as S1 and
in the slums. S6. The middle four sentences in each have been
jumbled up and labelled P, Q, R and S. You are
R : Then one day, while she was returning
required to find the proper sequence of the four
from an errand, she saw the slums of
sentences and mark your response accordingly.
Calcutta.
40. S1 : Human ways of life have steadily
S : She felt she had found her second changed.
vocation, her real calling.
S6 : Even if we try to do nothing, we cannot
The proper sequence should be prevent change.
A. P R S Q B. RPSQ P : Ancient Egypt – Greece – the Roman
Empire – the Dark Ages and the Middle
C. R P Q S D. Q R P S
Ages – the Renaissance – the age of modern
Direction: In the given question six sentences of science and of modern nations one has
a passage are a given. The first and sixth succeeded the other; the history has never
sentences are given in the beginning as S1 and stood still.
S6. The middle four sentences in each have been Q : About ten thousand years ago, man
jumbled up and labelled P, Q, R and S. You are lived entirely by hunting.
required to find the proper sequence of the four
sentences and mark your response accordingly. R : A settled civilized life only began when
agriculture was discovered.
39. S1 : Good memory is so common that we
S : From that time to this, civilization has
regard a man who does not possess it as
always been changing.
eccentric.
The proper sequence should be
S6 : She wheeled away the perambulator,
picturing to herself his terror when he A. Q R S P B. QPSR
would come out and find the baby gone. C. Q S R P D. P R S Q

PAGE 1289
www.byjusexamprep.com

Direction: In the given question six sentences of R : Then I rushed to the bank, collected all
a passage are a given. The first and sixth my money and made reservations for my
sentences are given in the beginning as S1 and return journey.
S6. The middle four sentences in each have been
S : From the shop next to it, I bought a
jumbled up and labelled P, Q, R and S. You are
couple of trunks to dump myat I books and
required to find the proper sequence of the four
other odd articles so that I could send them
sentences and mark your response accordingly.
away in advance.
41. S1 : In our youth we are apt to think that
applause and publicity constitute success. The proper sequence should be

S6 : So let us be initiated into the mysteries A. P Q R S B. PRQS


of maturity and be taught how to resist and C. P R S Q D. P S R Q
spurn the lure of hollow shows.
Direction: In the given question six sentences of
P : The man who values the applause more a passage are a given. The first and sixth
than his own effort has not outgrown his sentences are given in the beginning as S1 and
youth. S6. The middle four sentences in each have been
Q : It is our achievement or work which wins jumbled up and labelled P, Q, R and S. You are
lasting rewards. required to find the proper sequence of the four
sentences and mark your response accordingly.
R : But these are only the trappings, the
ephemeral illusions. 43. S1 : Wordsworth knew the behaviour of
owls in the night better than most of us
S : One should concentrate on one’s work
know the ways of black birds in day time.
knowing that applause will come unsought.
The proper sequence should be S6 : His great poetry owes much to the
night.
A. S R Q P B. PSRQ
P : Out of school there were no restrictions
C. Q P S R D. R Q P S on the hours he kept.
Direction: In the given question six sentences of Q : No poet ever had happier school days.
a passage are a given. The first and sixth
sentences are given in the beginning as S1 and R : He would skate by the light of the stars,
S6. The middle four sentences in each have been snare woodcocks at dead of night, watch
jumbled up and labelled P, Q, R and S. You are the sunrise after a long ramble.
required to find the proper sequence of the four
S : Throughout life he was an inveterate
sentences and mark your response accordingly.
walker by night.
42. S1 : My office sent an urgent email asking
The proper sequence should be
me to return.
S6 : It was evening before I could sit and A. Q P R S B. PSQR
write to my parents that I would be joining C. Q R P S D. S Q P R
them soon.
Direction: In the given question six sentences of
P : I immediately replied requesting a few a passage are a given. The first and sixth
days of grace as I had to book the return sentences are given in the beginning as S1 and
passage, pack and attend sundry matters S6. The middle four sentences in each have been
before winding up my establishment here. jumbled up and labelled P, Q, R and S. You are
Q : On the way, I went to the laundry and required to find the proper sequence of the four
made sure I would get my clothes in time. sentences and mark your response accordingly.

PAGE 1290
www.byjusexamprep.com

44. S1 : Science has already conferred an S: As he turned it slantwise to the light, the
immense boon on manking by the growth glint upon the rims of the copper shells
of medicine. within the drum showed that it was fully
loaded.
S6 : The general death rate in 1948 (10.8)
was the lowest ever recorded up to that A. P Q R S B. RPQS
date.
C. Q P R S D. P R Q S
P : It has continued ever since and is still
continuing. Direction: The following sentence has a blank
space and four words are given below it. Select
Q : In the eighteenth century people the word you consider most appropriate for the
expected most of their children to die blank and indicate your choice accordingly.
before they were grown up.
46. The mounting pressure was so over-
R : In 1920 the infant mortality rate in whelming that he ultimately________ to
England and Wales was 80 per thousand; in her wish.
1948 it was 34 per thousand.
A. yielded in B. gave in
S : Improvement began at the start of the
nineteenth century, chiefly owing to C. cowed in D. agreed in
vaccination. 47. Authority____________ when it is not
The proper sequence should be supported by the moral purity of its user.

A. R P Q S B. QSPR A. prevails B. entails

C. S Q R P D. P Q S R C. crumbles D. waits

Direction: In the given question six sentences of Direction: Each of the following sentences in this
a passage are a given. The first and sixth section has a blank space and four words or
sentences are given in the beginning as S1 and group of words are given after the sentence.
S6. The middle four sentences in each have been Select the word or group of words you consider
jumbled up and labelled P, Q, R and S. You are the most appropriate for the blank space and
required to find the proper sequence of the four indicate your response on the Answer Sheet
sentences and mark your response accordingly. accordingly.
45. S1: The young traveller gazed out into the 48. In a developing country like India, some
dismal country with a face of mingled industries will have to be brought within
repulsion and interest. public __________ and control, for the
other-wise rapid growth of the economy
S6: He quickly restored it to his secret
may be impossible.
pocket.
A. perspective B. hegemony
P: At intervals, he drew from his pocket a
bulky letter to which he referred, and on C. observation D. ownership
the margins of which he scribbled some
Direction: Each of the following sentences in this
notes.
section has a blank space and four words or
Q: It was a navy revolver of the largest size. group of words are given after the sentence.
Select the word or group of words you consider
R: From the back of his waist he produced
the most appropriate for the blank space and
something which one would hardly have
indicate your response on the Answer Sheet
expected to find in the possession of so
accordingly.
mild-mannered a man.

PAGE 1291
www.byjusexamprep.com

49. Gandhiji conceived of the idea of Select the word or group of words you consider
channelizing the powerful currents of the the most appropriate for the blank space and
united mass movement so as to give the indicate your response on the Answer Sheet
utmost impetus to the national accordingly.
__________ for independence.
53. A really sophisticated person would never
A. struggle B. conflict be ________ enough to think that he is
always right.
C. onslaught D. march
A. reverent B. naïve
Direction: The following sentence has a blank
space and four words are given below it. Select C. articulate D. humble
the word you consider most appropriate for the
Direction: The following sentence has a blank
blank and indicate your choice accordingly.
space and four words are given below it. Select
50. Because of his_________ habits, he could the word you consider most appropriate for the
not save much money. blank and indicate your choice accordingly.
A. extravagant B. frugal 54. Speeding and blocking are traffic offences
which lead to__________ accidents.
C. unsavory D. bad
A. troublesome B. final
Direction: Each of the following sentences in this
section has a blank space and four words or C. great D. gruesome
group of words are given after the sentence.
Direction: In the following question, a sentence
Select the word or group of words you consider
is given with a blank to be filled in with an
the most appropriate for the blank space and
appropriate word. Select the correct alternative
indicate your response on the Answer Sheet
out of the four and indicate it by selecting the
accordingly.
appropriate option.
51. Socrates was ___________ of spreading
55. Creative people are often_________ with
discontent among young men of Athens
their own uniqueness.
and of trying to destroy their faith in the old
gods. A. obsessed B. deranged
A. rebuked B. disparaged C. unbalanced D. dissatisfied
C. accused D. demonized Direction: The following question has a sentence
with three parts labelled a, b and C. Read each
Direction: In the following question, a sentence
sentence to find out whether there is any error
is given with a blank to be filled in with an
in any underlined part and indicate your
appropriate word. Select the correct alternative
response on the Answer Sheet against the
out of the four and indicate it by selecting the
corresponding letter i.e., A or B or C. If you find
appropriate option.
no error, your response should be indicated as D.
52. The robbers fell ___________ amongst
themselves over the sharing of the loot. 56.
A. out B. through
C. off D. across
Direction: Each of the following sentences in this A. (a) B. (b)
section has a blank space and four words or
C. (c) D. (d)
group of words are given after the sentence.

PAGE 1292
www.byjusexamprep.com

Direction: The following question has a sentence sentence to find out whether there is any error
with three parts labelled a, b and C. Read each in any underlined part and indicate your
sentence to find out whether there is any error response on the Answer Sheet against the
in any underlined part and indicate your corresponding letter i.e., A or B or C. If you find
response on the Answer Sheet against the no error, your response should be indicated as D.
corresponding letter i.e., A or B or C. If you find
60. Their all belongings (a) were lost (b) in the
no error, your response should be indicated as D.
fire. (c) No error (d)
57. A. (a) B. (b)
C. (c) D. (d)
Direction: The following question has a sentence
A. (a) B. (b) with three parts labelled a, b and C. Read each
sentence to find out whether there is any error
C. (c) D. (d) in any underlined part and indicate your
Direction: The following question has a sentence response on the Answer Sheet against the
with three parts labelled a, b and C. Read each corresponding letter i.e., A or B or C. If you find
sentence to find out whether there is any error no error, your response should be indicated as D.
in any underlined part and indicate your 61. He was in the temper (a)/ and refused (b)/
response on the Answer Sheet against the to discuss the matter again (c)/ No error (d).
corresponding letter i.e., A or B or C. If you find
no error, your response should be indicated as D. A. (a) B. (b)

58. C. (c) D. (d)


Direction: The following question has a sentence
with three parts labelled a, b and C. Read each
sentence to find out whether there is any error
A. (a) B. (b) in any underlined part and indicate your
response on the Answer Sheet against the
C. (c) D. (d) corresponding letter i.e., A or B or C. If you find
Direction: The following question has a sentence no error, your response should be indicated as D.
with three parts labelled a, b and C. Read each
sentence to find out whether there is any error 62.
in any underlined part and indicate your
response on the Answer Sheet against the
corresponding letter i.e., A or B or C. If you find
no error, your response should be indicated as D.
A. (a) B. (b)
59.
C. (c) D. (d)
Direction: The following question has a sentence
with three parts labelled a, b and C. Read each
A. (a) B. (b) sentence to find out whether there is any error
in any underlined part and indicate your
C. (c) D. (d) response on the Answer Sheet against the
Direction: The following question has a sentence corresponding letter i.e., A or B or C. If you find
with three parts labelled a, b and C. Read each no error, your response should be indicated as D.

PAGE 1293
www.byjusexamprep.com

63. 66.

A. (a) B. (b)
C. (c) D. (d)
A. (a) B. (b)
Direction: The following question has a sentence
C. (c) D. (d) with three parts labelled a, b and C. Read each
Direction: The following question has a sentence sentence to find out whether there is any error
with three parts labelled a, b and C. Read each in any underlined part and indicate your
sentence to find out whether there is any error response on the Answer Sheet against the
in any underlined part and indicate your corresponding letter i.e., A or B or C. If you find
response on the Answer Sheet against the no error, your response should be indicated as D.
corresponding letter i.e., A or B or C. If you find
no error, your response should be indicated as D. 67.

64.

A. (a) B. (b)
A. (a) B. (b) C. (c) D. (d)
C. (c) D. (d) Direction: The following question has a sentence
Direction: The following question has a sentence with three parts labelled a, b and C. Read each
with three parts labelled a, b and C. Read each sentence to find out whether there is any error
sentence to find out whether there is any error in any underlined part and indicate your
in any underlined part and indicate your response on the Answer Sheet against the
response on the Answer Sheet against the corresponding letter i.e., A or B or C. If you find
corresponding letter i.e., A or B or C. If you find no error, your response should be indicated as D.
no error, your response should be indicated as D.
68.
65.

A. (a) B. (b) A. (a) B. (b)

C. (c) D. (d) C. (c) D. (d)


Direction: The following question has a sentence Direction: The following question has a sentence
with three parts labelled a, b and C. Read each with three parts labelled a, b and C. Read each
sentence to find out whether there is any error sentence to find out whether there is any error
in any underlined part and indicate your in any underlined part and indicate your
response on the Answer Sheet against the response on the Answer Sheet against the
corresponding letter i.e., A or B or C. If you find corresponding letter i.e., A or B or C. If you find
no error, your response should be indicated as D. no error, your response should be indicated as D.

PAGE 1294
www.byjusexamprep.com

69. about unicorns and salamanders; not one of


them thought it necessary to avoid dogmatic
statements about them because he had never
seen one of them.
71. The writer believes that
A. (a) B. (b)
A. most people could avoid making foolish
C. (c) D. (d) mistakes if they were clever
Direction: The following question has a sentence B. through observation we could avoid
with three parts labelled a, b and C. Read each making many mistakes
sentence to find out whether there is any error
in any underlined part and indicate your C. Aristotle made many mistakes because
response on the Answer Sheet against the he was not observant
corresponding letter i.e., A or B or C. If you find D. All errors are caused by our own error
no error, your response should be indicated as D. in thinking
70. 72. With reference to the passage, which one
of the following is the correct statement?
A. Aristotle was able to avoid the mistake
of thinking that women have fewer
A. (a) B. (b) teeth than men
C. (c) D. (d) B. Aristotle thought women have fewer
teeth than men
Direction: Given below is a short passage. After
the passage, you will find some items based on C. Aristotle proved that women have
the passage. First, read a passage and answer the fewer teeth by counting his wife’s teeth
items based on it. You are required to select your D. Aristotle may have thought that women
answers based on the contents of the passage have fewer teeth because he never had
and opinion of the author only. a wife
To avoid the various foolish opinions to which 73. The writer says that if he was writing a book
mankind is prone, no superhuman brain is on hedgehogs
required. A few simple rules will keep you, not
from all errors, but from silly errors. A. he would maintain that they eat black
beetles because he had been told so
If the matter is one that can be settled by
observation, make the observation yourself. B. he would first observe their eating
Aristotle could have avoided the mistake of habits
thinking that women have fewer teeth than C. he would think it unnecessary to verify
men, by the simple device of asking Mrs Aristotle that they ate black beetles
to keep her mouth open while he counted.
D. he would make the statement that they
Thinking that you know, when in fact you do not,
ate black beetles and later verify it
is a bad mistake to which we are all prone. I
believe myself that hedgehogs eat black beetles, 74. The writer is of the opinion that
because I have been told that they do; but if I
A. unicorns and salamanders were
were writing a book on the habits of hedgehogs,
observed by ancient and medieval
I should not commit myself until I had seen one
writers but were unknown to modern
enjoying this diet. Aristotle, however, was less
writers
cautious. Ancient and medieval writers knew all

PAGE 1295
www.byjusexamprep.com

B. ancient and medieval writers wrote 77. When he tried to take out the purse, he
authoritatively about unicorns and discovered that
salamanders though they had never
A. it was not there
seen them
B. it was lost
C. unicorns and salamanders do not exist
C. it was a new purse
D. only those who had observed the habits
of unicorns and salamanders wrote D. it was his friend’s purse
about them
Direction: Given below is a short passage. After
75. A ‘dogmatic statement’ in the context the passage, you will find some items based on
means a statement which is the passage. First, read a passage and answer the
items based on it. You are required to select your
A. convincing
answers based on the contents of the passage
B. proved and opinion of the author only.
C. unquestionable A profound terror, increased still by the
darkness, the silence and his waking images,
D. doubtful
froze his heart within him. He almost felt his hair
Direction: Given below is a short passage. After stand on end, when by straining his eyes to their
the passage, you will find some items based on utmost, he perceived through the shadows two
the passage. First, read a passage and answer the faint yellow lights. At first he attributed these
items based on it. You are required to select your lights to the reflection of his own pupils, but soon
answers based on the contents of the passage the vivid brilliance of the night aided him
and opinion of the author only. gradually to distinguish the objects around him
in the cave, and he beheld a huge animal lying
Since I had nothing better to do, I decided to go
but two steps from him.
to the market to buy a few handkerchiefs, the
old ones had done vanishing trick. On the way I 78. The opening of the passage suggests that
met an old friend of mine and I took him to a
A. darkness, silence and waking images
nearby restaurant for tea and snacks. Afterwards
added to his already being in pro-found
I went to the shop and selected a dozen
terror
handkerchiefs. I pulled out my purse to make the
payment, and discovered that it was empty; I B. a profound terror increased the waking
then realized that it was not my purse, it was a images in his frozen heart
different purse altogether. How that happened
C. the person was frightened by darkness
is still a source of wonder to me and I refuse to
and silence
believe that it was the work of my good old
friend, for it was his purse that I held in my hand. D. a profound terror was caused in him by
the silence and darkness of the night
76. The man could not buy the handkerchiefs
because 79. What he felt when he perceived through
the shadows two faint lights,
A. he did not like the handkerchiefs
A. he experienced a great strain
B. his friend did not allow him to buy them
B. he felt his hair stand upright
C. the shop did not have any
handkerchiefs C. his hands felt strained to their utmost
D. he had no money in the purse D. his pupils dilated

PAGE 1296
www.byjusexamprep.com

80. The person in the story 83. From the passage one gathers that the
Eastern people must
A. imagined that he saw an animal
A. appreciate scientific achievements
B. could not recognize the animal
B. build huge industrial organizations
C. saw the animal by chance
C. avoid being controlled by machines and
D. expected to see the animal techniques of industrial production
Direction: Given below is a short passage. After D. be fascinated by machines
the passage, you will find some items based on 84. According to the author, science and
the passage. First, read a passage and answer the technology are
items based on it. You are required to select your
answers based on the contents of the passage A. totally harmless
and opinion of the author only. B. extremely dangerous
We are tempted to assume that technological C. to be treated as idols
progress is real progress and that material D. useful, if they are not worshipped
success is the criterion of civilization. If the blindly
Eastern people fascinated by machines and
techniques and use them, as Western nations Direction: Given below is a short passage. After
do, to build huge industrial organizations and the passage, you will find some items based on
large military establishments, they will get the passage. First, read a passage and answer the
involved in power politics and drift into the items based on it. You are required to select your
danger of death. Scientific and technological answers based on the contents of the passage
civilization brings great opportunities and great and opinion of the author only.
rewards but also great risks and temptations. It is not luck but labour that makes men. Luck,
Science and technology are neither good nor say an American writer, is ever waiting for
bad. They are not to be tabooed but tamed and something to turn up; labour with keen eyes and
assigned their proper place. They become strong will always turns up something. Luck lies
dangerous only if they become idols. in bed and wishes the postman would bring him
news of a legacy; labour turns out at six and with
81. According to the author, people think that
busy pen and ringing hammer lays the
real progress lies in
foundation of competence. Luck whines, labour
A. material success and technological watches. Luck relies on chance; labouron
growth character. Luck slips downwards to self-
indulgence; labour strides upwards and aspires
B. imitating Western nations
to independence. The conviction, therefore, is
C. having large industries and political extending that diligence is the mother of good
power luck; in other words, that a man’s success in life
will be proportionate to his efforts, to his
D. taking risks and facing temptations
industry, to his attention to small things.
82. According to the author, science and 85. Which one of the following statements
technology should be sums up the meaning of the passage?
A. tabooed and eliminated from life A. Luck waits without exertion but labour
B. used in a controlled and careful manner exerts without waiting

C. encouraged and liberally used B. Luck waits and complains without


working while labour achieves success
D. made compulsory in education although it complains

PAGE 1297
www.byjusexamprep.com

C. Luck often ends in defeat but labour our personal judgement. We should only have to
produces luck follow the rules faithfully. Classical mechanics
approaches this ideal so closely that it is often
D. Luck is self-indulgent but labour is
thought to have achieved it. But this leaves out
selfless
of account the element of personal judgement
86. Which one of the following statements is involved in applying the formulae of mechanics
true about the passage? to the facts of experience.

A. Luck is necessary for success 88. The purpose of the exact science is to

B. Success depends on hard work and A. form opinions about out experience
attention to details
B. formulate principles which will help us
C. Expectation of good luck always meets to exercise our personal judgement
with disappointment
C. assert our intellectual superiority
D. Success is exactly proportionate to hard
D. make formal and testable rules which
work only
can help verify experience
87. “________labour turns out at six and with
89. An exact theory of the universe is
busy pen and ringing hammer lays the
foundation of competence.” This statement A. not desirable
means
B. improbable
A. hard work of all kinds makes people
C. possible
efficient and skilled
D. yet to be made
B. thelabour lays the foundation of the
building 90. In exact sciences
C. the writer and the labourer are the true A. personal judgements are set aside in
eyes of the society favour of a mechanical theory
D. there is no worker who works so hard as B. one does not find answers to all
the labourer who begins his day at six in questions and problems
the morning
C. one reposes faith in actual experience
Direction: Given below is a short passage. After
the passage, you will find some items based on D. one interprets the universe according
the passage. First, read a passage and answer the to one’s wish
items based on it. You are required to select your 91. Classical mechanics
answers based on the contents of the passage
and opinion of the author only. A. has formulated precise rules based on
experience
The avowed purpose of the exact sciences is to
establish complete intellectual control over B. has gained intellectual control over the
experience in terms of precise rules which can be world
formally set out and empirically tested. Could C. has formulated an exact theory of the
that ideal be fully achieved, all truth and all error universe
could henceforth be ascribed to an exact theory
of the universe, while we who accept this theory D. just falls short of achieving intellectual
would be relieved of any occasion for exercising control over experience

PAGE 1298
www.byjusexamprep.com

Direction: In the following question, out of the Direction: Each item in this section consists of a
four alternatives, select the word similar in sentence with an underlined word followed by
meaning to the given word. four words/group of words. Select the option
that is nearest in meaning to the underlined
92. Ambiguous
word/words and mark your response in your
A. Contrasting Answer Sheet accordingly.
B. Connivance 96. Grand Oberoi is one of the most palatial and
luxurious hotels in India.
C. Vague
D. Willful A. Very clean

E. Regress B. Very special

Direction: Each item in this section consists of a C. Sense of taste


sentence with an underlined word followed by D. Magnificent
four words/group of words. Select the option
that is nearest in meaning to the underlined Direction: In the following question, out of
word/words and mark your response in your the four alternatives, select the word
Answer Sheet accordingly. similar in meaning to the given word.

93. The purpose of this meeting is to elucidate 97. TACTFUL


the major points of the new research A. diplomatic B. indifferent
report.
C. intelligent D. deceitful
A. Clarify B. Calculate
Direction: Each item in this section consists of a
C. Summarize D. Update sentence with an underlined word followed by
Direction: In the following question, out of the four words/group of words. Select the option
four alternatives, select the word similar in that is nearest in meaning to the underlined
meaning to the given word. word/words and mark your response in your
Answer Sheet accordingly.
94. MONOTONOUS
98. These African species of the monkeys are
A. dreary
quite dangerous and voracious.
B. dreadful
A. Very bad B. Insatiable
C. single-minded
C. Stingy D. Malicious
D. monologue
Direction: Each item in this section consists of a
Direction: Each item in this section consists of a sentence with an underlined word followed by
sentence with an underlined word followed by four words/group of words. Select the option
four words/group of words. Select the option that is nearest in meaning to the underlined
that is nearest in meaning to the underlined word/words and mark your response in your
word/words and mark your response in your Answer Sheet accordingly.
Answer Sheet accordingly.
99. The strictures of the United Nations have
95. Her words always make me hesitate and failed to have any effect on the warring
kindle with suspicion. factions.
A. Make fun of B. Excite A. Strictness B. Disapproval
C. Very kind D. Kind-hearted C. Censure D. Nonplus

PAGE 1299
www.byjusexamprep.com

Direction: In the following question, out of the 103. Of course we ___103___ act on some
given alternatives, select the word similar in impulse
meaning to the given word.
A. never B. belatedly
100. Obeisance
C. sometimes D. seldom
A. Veneration B. Excursion
104. or ___104___ urge.
C. Subjugation D. Obligation
A. peaceful
E. Gusto
B. uncontrollable
Direction: Each of the following sentences in this
section has a blank space with four words or C. indisputable
group of words given. Select whichever word or D. within limits
group of words you consider most appropriate
for the blank space and indicate your response 105. If suddenly you throw ___105___ brick at
accordingly. me
I was engaged in many activities and I wanted a A. no B. an
proper reconciliation between my activity and
C. a D. the
thought. Thought without ___101___ is
undeveloped thought. Action Without 106. and my ___106___ goes up in front
___102___ is folly. Of course we ___103___ act
on some impulse or ____104___ urge. If A. hand B. wrist
suddenly you throw ___105___ brick at me and C. finger D. ear
my ___106___ goes up in front to ___107___
myself, it is an automatic, ___108___ action and 107. to ___107___ myself, it is an automatic,
not a result ___109___ deliberate thought. Our A. stimulate B. resue
living is ___110___ by a series Of automatic
___111___ from morning till night Anything C. protect D. shelter
___112___ do outside that common range of
108. ___108___ action and not
___113___ however, has to be ___114___ by
some measure of thinking. ___115___ more A. uncontrollable B. instinctive
action and thought are ___116___ and
integrated, the More ___117___ effective C. impulsive D. precipitate
become and the happier you ___118___ There 109. a result ___109___ deliberate thought.
will then be no ___119___ between a wish to do
something and ___120___ to act. A. to B. in

101. I was engaged in many activities, and I C. of D. an


wanted a proper reconciliation between my 110. Our living is ___110___ by a series Of
activity and thought. Thought without
___101___ is undeveloped thought. A. made B. conditioned
A. wish B. action C. developed D. hardened
C. idea D. movement 111. automatic ___111___ from morning till
night
102. Action Without ___102___ is folly.
A. thoughts B. actions
A. thought B. wish
C. wishes D. ideas
C. idea D. action

PAGE 1300
www.byjusexamprep.com

112. Anything ___112___ do outside that 117. the More ___117___ effective become and
the
A. we B. I
A. they
C. they D. them
B. thoughts
113. range of ___113___ however,
C. we
A. thoughts B. ideas
D. the
C. actions D. wishes
118. happier you ___118___ There will
114. has to be ___114___ by some measure of
A. appear
thinking.
B. develop
A. proceeded
C. grow
B. preceded
D. stimulate
C. followed
119. then be no ___119___ between a wish
D. progressed
A. reconciliation
115. ___115___ more action and thought
B. conflict
A. Some B. If
C. inflict
C. The D. few
D. encounter
116. are ___116___ and integrated, 120. to do something and ___120___ to act.
A. developed A. inability
B. allied B. probability
C. hostile C. plausibility
D. unsympathetic D. possibility

PAGE 1301
www.byjusexamprep.com

General Knowledge
1. Among the currently operational Indian 3. A progressive tax takes a larger share of
Satellites, there is no: tax from poor families than it does from
rich families
A. Communication satellite
4. Indirect taxes have the advantage of
B. Navigation satellite
being cheaper and easier to collect
C. Earth observation satellite
Select the correct answer using the code
D. Jupiter orbiter satellite given below:
2. The rank of Major General in Indian Army is A. 1 and 3 only B. 2 and 4 only
equivalent to:
C. 1, 2 and 4 only D. 1, 2, 3 and 4
A. Air Marshal in Indian Air Force
5. Which of the following statements is/are
B. Rear Admiral in Indian Navy
true?
C. Air commodore in Indian Air Force
1. Faxian’s Gaoseng Faxian Zhuan was the
D. Commodore in Indian Navy earliest first-hand Chinese account of
3. Which of the following statement(s) is/are Buddhist sites and practices in India
false? 2. Faxian was only 25 years old at the time
1. Wage Boards are tripartite in nature, of writing the text
with representatives from workers, 3. Faxian’s main aim in coming to India
employers and independent members was to obtain and take back texts
2. Except for the Wage Boards for containing monastic rules
Journalists and Non-Journalists, all the
Select the correct answer using the code
other wage boards are statutory in
given below:
nature
A. 1, 2 and 3 B. 2 only
Select the correct answer using the code
given below: C. 1 and 3 only D. 3 only
A. 1 only 6. Consider the following statements:
B. 2 only 1. The province of Assam was created in
C. 1 and 2 only the year 1911

D. 1, 2 and 3 2. Eleven districts comprising Assam were


separated from the Lieutenant
4. Which of the following statements are
Governorship of Bengal and established
correct?
as an independent administration
1. Ability to pay principle of taxation holds under a Chief Commissioner in the year
that the amount of taxes people pay 1874
should relate to their income or wealth
Which of the statements given above is/are
2. The Benefit Principle of taxation states correct?
that individuals should be taxed in
proportion to the benefit they receive A. 1 only B. 2 only
from Government programmes C. Both 1 and 2 D. Neither 1 nor 2

PAGE 1302
www.byjusexamprep.com

7. Consider the following statements: C. 180 days in advance ticket booking


facility for passengers
1. The inscriptions on the pillar at
Rummindei give vivid details of D. Bio-toilets
Ashoka’s Dhamma
12. ‘Citizenfour’, the 87th Academy Award
2. The Nigalisagar inscription records the winner in the category of documentary
fact of Ashoka having visited the feature, is based on the life of:
Kongagamana stupa
A. Abraham Lincoln
Which of the statements given above is/are
correct? B. Albert Einstein

A. 1 only B. 2 only C. Edward Snowden


C. Both 1 and 2 D. Neither 1 nor 2 D. Laura Poitras
8. The Right to Education was added to the 13. Which one of the following terms is used in
Fundamental Rights in the Consitution of Economics to denote a technique for
India through the: avoiding a risk by making a counteracting
transaction?
A. Constitution (86th Amendment) Act,
2002 A. Dumping B. Hedging
B. Constitution (93rd Amendment) Act, C. Discounting D. Deflating
2005
14. Which of the following is/are example(s) of
C. Constitution (87th Amendment) Act, ‘Near Money’?
2003
1. Treasury Bill
D. Constitution (97th Amendment) Act,
2011 2. Credit Card

9. The category of ‘Overseas Citizens of India’ 3. Savings accounts and small time
was entered in the Citizenship Act of India deposits
through an amendment in the years: 4. Retail money market mutual funds
A. 1986 B. 1992
Select the correct answer using the code
C. 1996 D. 2005 given below:
10. Which one of the following services of India A. 1 only B. 2 only
Post has permanently been discontinued?
C. 1, 2 and 3 D. 1,3 and 4
A. Money Order
15. Consider the following statements:
B. Telegram
1. Abhinavagupta wrote a comprehensive
C. Postal Life Insurance treatise called the Tantraloka which
D. Inland Letter systematically presents the techings of
the Kula and Trika systems
11. Which one of the following is not a thrust
area in the Railway Budget 2015-16? 2. The Samaraichchakaha by Haribhadra
suri written in Gujarat around the
A. Online booking of disposable bed rolls
eighth century is technically not a
B. Defence Travel System to eliminate tantric work but is saturated with
Warrants tantric ideas and practices

PAGE 1303
www.byjusexamprep.com

Which of the statements given above is/are 20. Which of the following statements with
correct? regard to preventive detention in India
is/are correct?
A. 1 only B. 2 only
1. The detenue has no rights other than
C. Both 1 and 2 D. Neither 1 nor 2
those mentioned in clauses (4) and (5)
16. Consider the following statements about of Article 22 of Constitution of India
votive inscriptions in the second century
2. The detenue has a right to challenge the
BC:
detention order on the ground that he
1. They record gifts made to religious was already in jail when the detention
institutions order was passed
2. They tell us about the idea of 3. The detenue can claim bail on the
transference of the meritorious results ground that he has been in prison
of the action of one person to another beyond twenty-four hours without and
person order of the magistrate
Which of the statements given above is/are Selct the correct answer using the code
correct? given below:
A. 1 only B. 2 only A. 1 and 2 only
C. Both 1 and 2 D. Neither 1 nor 2 B. 2 only

17. Which of the following kingdom did the C. 3 only


temple of Hazara Rama belong to? D. 1, 2 and 3
A. Avadh B. Travancore 21. Which one of the following is not an
C. Vijayanagara D. Ahom objective of the Rashtriya Uchchatar
Shiksha Abhiyan (RUSA)?
18. Which one of the following languages is not
recognized in the Eighth Schedule to the A. Improve the overall quality of private
Constitution of India? educational institutions

A. English B. Sanskrit B. Ensure reforms in the affiliation,


academic and examination systems
C. Urdu D. Nepali
C. Correct regional imbalances in access to
19. A Member of Lok Sabha does not become higher education
disqualified to continue as a Member of the
D. Create an enabling atmosphere in the
House if the Member:
higher educational institutions to
A. voluntarily gives up his/her devote themselves to research and
membership of the political party from innovations
which he/she was elected
22. Which one of the following is not a
B. is expelled by the political party from monitorable target of the Beti Bacho Beti
which he/she had been elected to the Padhao Abhiyan?
House
A. Provide girls’ toilet in every school in
C. joins a political party after being elected 100 Child Sex Ratio (CSR) districts by the
as an independent candidate year 2017
D. abstains from voting contrary to the B. 100 percent girls’ enrolment in
direction by his/her political party secondary education by the year 2020

PAGE 1304
www.byjusexamprep.com

C. Promote a protective environment for 27. Iqta in medieval India meant:


girl children through implementation of
A. land assigned to religious personnel for
Protection of Children from Sexual
spiritual purposes
Offences (POCSO) Act, 2012
B. land revenue from different territorial
D. Train Elected Representatives/ units assigned to army officers
Grassroot functionaries Community
Champions to mobilize communities to C. charity for educational and cultural
improve CSR and promote girl’s activities
education D. the rights of the zamindar
23. The National Policy for Children, 2013 28. Which of the following statements about
recognizes every person as a child below the Vijayanagara Empire is/are true?
the age of:
1. The kings claimed to rule on behalf of
A. 12 years B. 14 years the God Virupaksha
C. 16 years D. 18 years 2. Rulers used the title “Hindu Suratrana”
to indicate their close links with Gods
24. Which one of the following is not among
the aims of the Second Five Year Plan 3. All royal orders were signed in Kannada,
(1956-97 to 1960-61)? Sanskrit and Tamil
A. Rapid industrialization with particular 4. Royal portrait sculpture was now
emphasis on the development of basic displayed in temples
and heavy industries Select the correct answer using the code
B. Large expansion of employment given below:
opportunities A. 4 only B. 1 and 2 only
C. Achieve self-sufficiency in food grains C. 1, 2 and 3 D. 1, 2 and 4
and increase agricultural production to
meet the requirements of industry and 29. In which one of the following cases, the
exports Constitutional validaity of the Muslim
Women (Protection of Rights of Divorce)
D. Reduction of inequalities in income and Act 1986, was upheld by the Supreme Court
wealth and a more even distribution of of India?
economic power
A. Muhammad Ahmed Khan v. Shah Bano
25. Which of the following is not a ‘Public Begum
Good’?
B. Danial Latifi v. Union of India
A. Electricity
C. Mary Roy v. State of Kerala
B. National Defence D. Shankari Prased v. Union of India
C. Light House 30. Consider the following statements about
D. Public Parks the President of India:

26. Which one of the following books was not 1. The President has the right to address
illustrated with paintings in Akbar’s court? and send message to the Council of
Ministers to elicit specific information
A. Hamzanama B. Razmnama
2. The President can call for information
C. Baburnama D. Tarikh-i-Alfi relating to proposals for legislation

PAGE 1305
www.byjusexamprep.com

3. All decisions of the Council of Ministers 2. The onus of implementation of the Act
relating to administration of the Union lies at the level of the State/UT
must be communicated to the Governments
President
3. The Act seeks to recognize and vest
Which of the statements given above are certain forest rights in the forest
correct? dwelling Scheduled Tribes and other
traditional forest dwellers
A. 1 and 3 only B. 2 and 3 only
C. 1 and 2 only D. 1, 2 and 3 Select the correct answer using the code
given below:
31. In which one of the following judgments of
the Constitutional Bench of the Supreme A. 1 and 2 only
Court o India, the ‘rarest of rare’ principle in B. 2 and 3 only
the award of death penalty was first laid
down? C. 1 and 3 only
A. Bachan Singh v. State of Punjab (1980) D. 1, 2 and 3
B. Gopalanachari v. State of Kerala (1980) 35. The agrahara in early India was:
C. Dr. Upendra Baxi v. State of UP (1983) A. the name of a village or land granted to
D. Tukaram v. State of Maharashtra (1979) Brahmins

32. Who among the following is the winner of B. the garland of flowers of Agar
the World Food Prize (year 2015)? C. the grant of land to officers and soldiers
A. Sanjaya Rajaram D. land or village settled by Vaishya
B. Baldev Singh Dhillon farmers

C. Sir Fazle Hasan Abed 36. Which one among the following statements
about the coins of the Gupta rulers is
D. Rajendra Singh Paroda correct?
33. Leander Paes won the US Open Mixed
A. The obverse and reverse, both, had only
Doubles Tennis Title (2015) partnering with:
the king’s portrait and date
A. Kristina Mladenovic
B. The obverse and reverse, both, had only
B. Flavia pennetta an image of a deity and date
C. Martina Hingis C. The obverse generally had king’s
portrait and reverse had an image of a
D. Sania Mirza
deity or a motif
34. Which of the following statements relating
to the Scehduled Tribes and Other D. The obverse generally had king’s
Traditional Forest Dwellers (Recognition of portrait and reverse always had a date
Forest Rights) Act, 2006 are correct? 37. Human Development Report for each year
1 The Act recognizes forest rights of at global level is published by:
forest dwelling Scheduled Tribes who A. WTO B. World Bank
have been occupying the forest land
before October 25, 1980 C. UNDP D. IMF

PAGE 1306
www.byjusexamprep.com

38. Which of the following statements about 41. Who among the following is the first Indian
comptroller and Auditor General of India sportsperson to reach the finals in the
(CAG) are correct? World Badminton Championship (Women)
in 2015?
1. The CAG will hold office for a period of
six years from the date he assumes the A. Jawal Gutta
office. He shall vacate office on
attaining the age of 65 years, if earlier B. Saina Nehwal
than the expiry of the 6 year term C. P.V. Sindhu
2. The powers of CAG are derived from the D. Madhumita Bisht
Constitution of India
42. The Erawan shrine, which witnessed a
3. The CAG is a multi-member body major bomb blast in August 2015, is located
appointed by the President of India in at:
consultation with the Prime Minister
and the Council of Ministers A. Singapore B. Bangkok

4. The CAG may be removed by the C. Kuala Lumpur D. Kabul


President only on an address from both 43. Which of the following is/are the
Houses of Parliament, on the grounds of example(s) of Transfer Payment(s)?
proved misbehavior or incapacity
1. Unemployment Allowance
Select the correct answer using the code
given below: 2. Payment of Salary
A. 1, 2 and 4 B. 1, 2 and 3 3. Social Security Payments
C. 3 and 4 D. 1 and 2 only 4. Old age Pension
39. Which of the following statements with Select the correct answer using the code
regard to UID/Aadhar Card are correct? given below:
1. It is a 12 digit unique form of A. 1 and 3 only
identification for all residents of India
B. 1, 2 and 3 only
2. It is an identity number along with the
biometric information of the individuals C. 1, 3 and 4 only
3. It is a national identity and citizenship D. None of the above
card
44. Which of the statements given below is/ are
Select the correct answer using the code correct?
given below:
1. The ideal of a common civil code is set
A. 2 and 3 only B. 1 and 2 only forth in Article 44 of the Constitution of
C. 1 and 3 only D. 1, 2 and 3 India

40. Which one of the following 2. In certain respects, the High Courts in
Articles/Schedules in the Constitution of India have been given more extensive
India deals with Autonomous District powers than the Supreme Court
Councils? 3. The Supreme Court of India, the first
A. Eighth Schedule B. Article 370 fully independent Court for the country,
was set up under the Constitution of
C. Sixth Schedule D. Article 250 India in 1950

PAGE 1307
www.byjusexamprep.com

Select the correct answer using the code Select the correct answer using the code
given below: given below:
A. 1, 2 and 3 B. 2 and 3 only A. 1 and 2 only B. 1, 2 and 3
C. 1 only D. 3 only C. 2 and 3 only D. 1 only
45. What were the 12 states of the Sikh 49. Who among the following Prime Ministers
confederacy called? of India were defeated by a vote of No
A. Misl B. Gurmata confidence?

C. Sardari D. Rakhi 1. Morarji Desai


46. The followers of Gorakhnath were called: 2. Viswanath Pratap Singh
A. Jogis B. Nath-Panthis 3. H.D. Deve Gowda
C. Tantriks D. Sanyasis 4. Atal Bihari Vajpayee
47. Which of the following statements relating Select the correct answer using the code
to the office of the President of India are given below:
correct?
A. 1, 2, 3 and 4
1. The President has the power to grant
pardon to a criminal in special cases B. 1, 2 and 3 only

2. The President can promulgate C. 2, 3 and 4 only


ordinances even when the Parliament is D. 1 and 4 only
in session
50. Which one of the following nations has
3. The President can dissolve the Rajya
faced severe economic crisis in the year
Sabha during emergency
2015 resulting in default in repayment of
4. The President has the power to IMF loan?
nominate two members in the Lok
A. China B. Greece
Sabha from the Anglo Indian
community C. Ireland D. Belgium
Select the correct answer using the code 51. Which one of the following island nations
given below: went for Parliamentary elections in the
A. 1 and 2 B. 1 only month of August 2015?

C. 3 and 4 only D. 1, 3 and 4 A. Maldives B. Fiji

48. Which of the following statements C. Sri Lanka D. Singapore


regarding Rajya Sabha is/are correct? 52. The first summit of the Forum for India
1. The maximum permissible strength of Pacific Islands Cooperation (FIPIC) was held
Rajya Sabha is 250 in :
2. In Rajya Sabha, 238 members are A. Jaipur B. Suva
elected indirectly from the States and
C. New Delhi D. Port Moresby
Union Territories
53. Which of the following statement(s) are
3. It shares legislative powers equally with
true with respect to the concept of
Lok Sabha in matters such as creation of
All India Services

PAGE 1308
www.byjusexamprep.com

‘EFFICIENCY’ as used in mainstream 57. As per the Consitution of India, the Write of
economics? Prohibition relates to an order :
1. Efficiency occurs when no possible 1. issued against judicial and quasi-judicial
reorganization of production can make authority.
anyone better off without making 2. to prohibit an inferior Court from
someone else worse off proceeding in a particular case where it
2. An economy is clearly inefficient if it is has no jurisdiction to try
inside the Production Possibility 3. to restrain a person from holding a
Frontier (PPF) public office to which he is not entitled
3. At a minimum, an efficient economy is Select the correct answer using the code
on its Production Possibility Frontier given below:
(PPF)
A. 1 and 2 only B. 2 and 3 only
4. The terms such as ‘Pareto Efficiency’,
‘Pareto Optimality’ and ‘Allocative C. 1 only D. 1, 2 and 3
Efficiency’ are all essentially one and 58. The Second Administrative Reforms
the same which denote ‘efficiency in Commission (2005) was concerned with :
resource allocation’
A. reforms in institutional arrangements
Select the correct answer using the code for good governance
given below:
B. reforms in the Indian Penal Code and
A. 1 and 4 only B. 1 and 3 only the Criminal Justice System
C. 2 and 3 only D. 1, 2, 3 and 4 C. creating an ombudsman mechanism for
reduction of corruption in public life
54. The Lilavati of Bhaskara is a standard text
on: D. devising new measures for urban
governance and management
A. Mathematics B. Surgery
59. Consider the following statements :
C. Poetics D. Linguistics
1. The President of India shall have the
55. Kamandaka’s Nitisara is a contribution to: power to appoint and remove the
Speaker of Lok Sabha
A. Logic and Philosophy
2. The Speaker has to discharge the
B. Mathematics
functions of his office himself
C. Political morality throughout his term and cannot
delegate his functions to the Deputy
D. Grammar
Speaker during his absence from the
56. Which one among the following was not an station or during his illness
attribute of Samudragupta described in Which of the statements given above is/are
Prayag Prashasti? correct?
A. Sharp and polished intellect A. 1 only
B. Accomplished sculptor B. 2 only
C. Fine musical performances C. Both 1 and 2
D. Poetical talent of a genius D. Neither 1 nor 2

PAGE 1309
www.byjusexamprep.com

60. Tarun, Kishore and Shishu categories are a 64. Which one of the Five Year Plans had a high
part of which scheme? priority to bring inflation under control and
to achieve stability in the economic
A. Regional Rural Banks (RRBs)
situation?
B. Micro Units Development & Refinance
A. Fourth Plan (1969-74)
Agency Ltd. (MUDRA).
B. Fifth Plan (1974-79)
C. Small Industries Development Bank of
India (SIDBI) C. Sixth Plan (1980-85)
D. Industrial Development Bank of India D. Seventh Plan (1985-90)
(IDBI)
65. Which of the following is/are not depicted
61. BRICS leaders signed the agreement to in the Rajput paintings?
establish a New Development Bank at the
1. The stories of Krishna
summit held in :
2. Ragas and Raginis
A. New Delhi, India (2012)
3. The deeds of Hamza
B. Durban, South Afrcia (2013)
4. The deeds of Babur
C. Fortaleza, Brazil (2014)
Select the correct answer using the code
D. Ufa, Russia (2015)
given below:
62. Which one of the following nations is not a
A. 1, 2 and 3 B. 2, 3 and 4
member of the Eurasian Economic Union?
C. 3 and 4 only D. 4 only
A. Belarus B. Russia
66. Which one of the following is the common
C. Kazakhstan D. Uzbekistan
element between the Kailasanatha Temple
63. Which of the following statement(s) is/are at Ellora and the Shore Temple at
true with respect to Phillips Curve? Mamallapuram?
1. It shows the trade-off between A. Both are examples of Nagara
unemployment and inflation architecture
2. The downward sloping curve of Phillips B. Both are carved out from solid rocks
Curve is generally held to be valid only
C. Both are Gupta period temples
in the short run
D. Both were built under the patronage of
3. In the long run, Phillips Curve is usually
Pallava Kings
thought to be horizontal at the non-
accelerating inflation rate of 67. Who among the following archaeologists
unemployment (NAIRU) was the first to identify similarities between
pre-Harappan culture and the mature
Select the correct answer using the code
Harappan culture?
given below:
A. Amalananada Ghosh
A. 1 only
B. Rakhaldas Banerji
B. 2 and 3 only
C. Daya Ram Sahni
C. 1 and 2 only
D. Sir John Marshall
D. 1, 2 and 3

PAGE 1310
www.byjusexamprep.com

68. Which of the following statements relating 2. Hind Swaraj was the outcome of the
to the bnadung Conference on Afro-Asian experience of Gandhi’s prolonged
Resurgence (1955) are correct? struggle against Colonial Raj in India
1. Bandung Conference was organized by Which of the statements given above is/are
Indronesia, Myanmar (Burma), Ceylon correct?
(Sri Lanka), India, and Pakistan in which A. 1 only B. 2 only
29 countries representing more than
half of the world’s population sent C. Both 1 and 2 D. Neither 1 nor 2
delegates 71. Arrange the following popular hill stations
2. The Conference reflected the five of India in terms of their height (from
sponsors’ dissatisfaction with what they highest to the lowest) from the mean sea
regarded as a reluctance by the level:
Western powers to consult with them 1. Mussoorie 2. Shimla
on decisions affecting Asia
3. Ooty 4. Darjeeling
3. The Conference was concerned over
Select the correct answer using the code
tension between the People’s Republic
given below:
of China and the United States
A. 3-2-4-1 B. 3-1-4-2
Select the correct answer using the code
given below: C. 2-3-4-1 D. 2-4-1-3
A. 1 and 2 only 72. A person rings a metallic bell near a strong
concrete wall. He hears the echo after 0.3 s.
B. 2 and 3 only If the sound moves with a speed of 340 m/s,
C. 1 and 3 only how far is the wall from him?

D. 1, 2 and 3 A. 102 m B. 11 m

69. Which of the following is/are Constitutional C. 51 m D. 30 m


Body/Bodies? 73. The synthetic rubber has replaced nautral
1. National Commission for Scheduled rubber for domestic and industrial
Tribes purposes. Which one of the following is the
main reason behind that?
2. National Commission for Women
A. Natural rubber is unable to meet the
3. National Commission for Minorities growing demand of different industries
4. National Human Rights Commission B. Natural rubber is grown in tropical
countries only
Select the correct answer using the code
given below: C. Raw material for synthetic rubber is
easily available
A. 1 only B. 1,3 and 4 only
D. Natural rubber is not durable
C. 3 and 4 only D. 1, 2,3 and 4
74. Which of the following statements about
70. Consider the following statements: India’s scientific and research mission in
1. In Hind Swaraj, Mahatma Gandhi Antarctica is/are correct?
formulates a conception of good life for 1. The first scientific base station in
the individual as well as the society Antarctica was Dakshin Gangotri

PAGE 1311
www.byjusexamprep.com

2. Dakshin Gangotri is now being used as 78. Which one of the following statements
supply base and transit camp about bar magnet is correct?
3. The Maitri station is manned A. The pole strength of the north-pole of a
throughout the year for scientific bar magnet is larger than that of the
activities south-pole
Select the correct answer using the code B. When a piece of bar magnet is bisected
given below: perpendicular to its axis, the north and
A. 1 only B. 2 and 3 only south poles get separated
C. 1 and 2 only D. 1, 2 and 3 C. When a piece of bar magnet is bisected
perpendicular to its axis, two new bar
75. Which of the following statements
magnets are formed
regarding chemical industry in India is/are
correct? D. The poles of a bar magnet are unequal
1. Chemical industry is one of the oldest in magnitude and opposite in nature
indsutries in India 79. Which of the following statements is/are
2. Dyestuff sector is one of the important correct?
segments of chemical industry 1. Acid rain reacts with buildings made
3. Textile industry accounts for the largest from limestone
consumption of dyestuffs 2. Burning of sulphur containing coal can
Select the correct answer using the code contribute to acid rain
given below:
3. Eutrophication is an effective measure
A. 1 only B. 2 and 3 only to control pollution
C. 1 and 3 only D. 1, 2 and 3 Select the correct answer using the code
76. Which one of the following statements is given below:
not correct? A. 1 and 2 only B. 2 and 3 only
A. Atoms of different elements may have C. 1 only D. 1, 2 and 3
same mass numbers
80. The ‘eye’ of the cyclone has:
B. Atoms of an element may have
different mass numbers A. Abnormally high temperature and
C. All the atoms of an element have same lowest pressure
number of protons B. Abnormally low temperature and
D. All the atoms of an element will always pressure
have same number of neutrons C. Clear sky and low pressure
77. Which one of the following statements is D. Dense cloud cover and low pressure
not correct?
81. Which one of the following is the reason
A. Hydrogen is an element
due to which the wind in the Southern
B. Hydrogen is the lightest element hemisphere is deflected towards its left?
C. Hydrogen has no isotopes A. Difference in the water masses of
D. Hydrogen and oxygen form an northern and southern hemisphere
explosive mixture B. Temperature and pressure variations

PAGE 1312
www.byjusexamprep.com

C. Inclined axis of the Earth 87. A myopic person has a power of-1.25
Dioptre. What is the focal length and nature
D. Rotation of the Earth
of his lens?
82. Consider the following tributaries of river
A. 50 cm and convex lens
Brahmaputra:
B. 80 cm and convex lens
1. Lohit 2. Tista
C. 50 cm and concave lens
3. Subansiri 4. Sankosh
D. 80 cm and concave lens
Arrange the above rivers from west to east:
88. Norman Borlaug won Nobel Peace Prize for
A. 2-4-3-1 B. 2-3-4-1
his contributions in:
C. 4-2-3-1 D. 3-1-2-4
1. development of high-yielding crops.
83. Which one of the following is the casuse of
2. modernization of irrigation
long-term sea-level change?
infrastructure
A. Atmospheric disturbance
3. introduction of synthetic fertilizers and
B. Change in marine water density pesticides.
C. Melting of icebergs Select the correct answer using the code
given below:
D. Melting of ice sheets
A. 1 only B. 2 only
84. Which one of the following is an example of
chemical change? C. 2 and 3 only D. 1, 2 and 3
A. Burning of paper 89. The living content of cell is called
protoplasm. It is composed of:
B. Magnetisation of soft iron
A. Cytoplasm only
C. Dissolution of cane sugar in water
B. Cytoplasm and nucleoplasm
D. preparation of ice cubes from water
C. Nucleoplasm only
85. Which one of the following hormones
contains peptide chain? D. Cytoplasm, nucleoplasm and other
organelles
A. Oxytocin
90. Glucose is a source of energy. Which one of
B. Corticotropin
the following type of a molecule, Glucose
C. Insulin is?
D. Cortisone A. Carbohydrate B. Protein
86. A piece of ice, 100g in mass is kept at 0 C. C. Fat D. Nucleic acid
The amount of heat it requires to melt at
91. Which one of the following islands is of
0 C is (take latent heat of melting of ice to
volcanic origin?
be 333.6 J/g):
A. Reunion island
A. 750.6 J
B. Andaman & Nicobar islands
B. 83.4 J
C. Lakshadweep islands
C. 33360 J
D. Maldives
D. 3.336 J

PAGE 1313
www.byjusexamprep.com

92. Which of the following statements is/are 95. Vitamin B12 deficiency causes pernicious
correct? anemia. Animals cannot synthesize vitamin
B12. Humans must obtain all their vitamin
1. The earth is nearest to the Sun at
B12 from their diet. The complexing metal
Perihelion, which generally occurs on
ion in vitamin B12 is :
January 3
A. Mg2+ (Magnesium ion)
2. The earth is farthest away from the Sun
at Perihelion, which generally occurs on B. Fe2+ (Iron ion)
July 4
C. Co3+ (Cobalt ion)
3. The earth is farthest away from the sun
at Aphelion, which generally occurs on D. Zn2+ (Zinc ion)
July 4 96. Which one of the following vitamins has a
4. The earth is nearest to the Sun at role in blood clotting?
Aphelion, which generally occurs on A. Vitamin A B. Vitamin B
January 3
C. Vitamin D D. Vitamin K
Select the correct answer using the code
given below: 97. Which one of the following statements is
not correct?
A. 1 only B. 2 and 4
A. Sound waves in gases are longitudinal in
C. 1 and 3 D. 1 and 2 nature
93. The Mahatma Gandhi National Marine Park B. Sound waves having frequency below
is located in ________. 20 Hz are known as ultrasonic waves
A. Pirotan Isaland C. Sound waves having higher amplitudes
B. Rameswaram and louder
C. Ganga Sagar Island D. Sound waves with high audible
frequencies are sharp
D. Port Blair
98. Which one of the following statements is
94. Consider the following statements:
correct about he velocity of sound?
1. Most of the coal and the ferrous group
A. does not depend upon the nature of
of minerals in India occur in the
media
peninsula, south of the Vindhyas
B. is maximum in gases and minimum in
2. The peninsular India once formed part
liquids
of the super-continent which included
Australia, Antarctica, Africa and South C. is maximum in solids and minimum in
America. liquids
Which of the statements given above is/are D. is maximum in solids and minimum in
correct? gases
A. 1 only 99. Genetically modified (GM) crops contain
B. 2 only modified genetic material due to:

C. Both 1 and 2 1. introduction of new DNA

D. Neither 1 nor 2 2. removal of existing DNA

PAGE 1314
www.byjusexamprep.com

3. introduction of RNA A. Pakistan B. China


4. introduction of new traits C. Bangladesh D. Nepal
Select the correct answer using the code 105. Plants contain a variety of sterols like
given below: stigmasterol, ergosterol, sitosterol etc.,
which very closely resemble cholesterol.
A. 1 and 2 only B. 1, 2 and 3
These plant sterols are referred as:
C. 3 and 4 D. 1, 2 and 4
A. Phytosterols
100. In artificial insemination (AI) process, which
B. Calciferols
of the following is/are introduced into the
uterus of the female? C. Ergocalciferols
A. Egg only B. Fertilized egg D. Lumisterols
C. Sperm Only D. Egg and sperm 106. ‘German silver’ is used to make decorative
articles, coinage metal, ornaments etc. The
101. Which one of the following is the correct
name is given because:
sequence of the given planets in increasing
order of their size(diameter)? A. it is an alloy of copper and contains
silver as one of its components
A. Mars- Venus- Earth- Mercury- Uranus
B. Germans were the first to use silver
B. Mercury- Mars- Venus- Earth- Uranus
C. its appearance is like silver
C. Mercury- Mars- Venus- Uranus- Earth
D. it is an alloy of silver
D. Venus- Mercury- Mars- Earth- Uranus
107. Two systems are said to be in thermal
102. Stalactites and stalagmites are features of:
equilibrium if and only if:
A. Glacial topography
A. there can be a heat flow between them
B. Volcanic topography even if they are at different
temperatures
C. Karst topography
B. there cannot be a heat flow between
D. Fluvial topography
them even if they are at different
103. Headquarters of which one among the temperatures
following Railway Zones in India is situated
C. there is no heat flow between them
at the highest elevation from the mean sea
level? D. their temperatures are slightly different
A. East Central Railway 108. The SI unit of mechanical power is:
B. South Eastern Railway A. Joule B. Watt
C. South Western Railway C. Newton-second D. Joule-second
D. West Central Railway 109. Dengue virus is known to cause low platelet
count in blood of patient by:
104. With which one of the following countries,
India has signed an MoU under the 1. interfering in the process of platelet
International Cooperation on Brahmaputra production in bone marrow
and Sutluj rivers?
2. infecting endothelial cells

PAGE 1315
www.byjusexamprep.com

3. binding with platelets 3. Rainy and dry seasons are found in both
the climates
4. accumulating platelets in intestine
Select the correct answer using the code
Select the correct answer using the code
given below:
given below:
A. 1 and 2 only B. 1 and 3 only A. 1 only B. 2 and 3 only

C. 3 and 4 D. 1, 2 and 3 C. 1 and 3 only D. 1, 2 and 3

110. The Germplasm is required for the 114. Consider the following statements
propagation of plants and animals regarding laterite soils of India:
Germplasm is the: 1. Laterite soils are generally red in colour
1. genetic resources 2. Laterite soils are rich in nitrogen and
2. seed or tissues for breeding potash
3. egg and sperm repository 3. Laterite soils are well developed in
Rajasthan and UP
4. a germ cell’s determining zone
Select the correct answer using the code 4. Tapioca and cashew nuts grow well in
given below: this soil

A. 1 only B. 1, 2 and 3 Which of the statements given above is/are


correct?
C. 2 and 3 only D. 2 and 4
A. 1 only B. 2, 3 and 4
111. Methyl Isocyanate gas, which was involved
in the disaster in Bhopal in December 1984, C. 1 and 4 only D. 1, 2 and 4
was used in the Union Carbide factory for 115. Pir Panjal Ranga in the Himalayas is a part
production of: of:
A. Dyes B. Detergents
A. Shiwalik
C. Explosives D. Pesticides
B. Trans Himalaya
112. The handle of pressure cookers is made of
plastic because it should be made non- C. Central Himalaya
conductor of heat. The plastic used there is D. Lesser Himalaya
the first man-made plastic, which is:
116. Consider the following map of India:
A. Polythene B. Terylene
C. Nylon D. Bakelite
113. Which of the following statements
regarding Mediterranean and Monsoon
climate is/are correct?
1. Precipitation in Mediterranean climate
is in winter while in Monsoon climate it
is mostly in summer
2. The annual range of temperature in
Mediterranean climate is higher than
the Monsoon climate

PAGE 1316
www.byjusexamprep.com

The areas marked in the map given above 119. Which of the following pairs of vitamin and
account for the production of which one of disease is/are correctly matched?
the following cash crops?
1. Vitamin A : Rickets
A. Cotton
2. Vitamin B1 : Beriberi
B. Groundnut
3. Vitamin C : Scurvy
C. Sugarcane Select the correct answer using the code
D. Tobacco given below:

117. After a hot sunny day, people sprinkle A. 2 only


water on the roof-top because______ B. 2 and 3 only
A. Water helps air around the roof-top to C. 1 and 3 only
absorb the heat instantly
D. 1, 2 and 3
B. Water has lower specific heat capacity
120. Which one of the following is considered as
C. Water is easily available ‘good cholesterol’ with reference to
D. Water has large latent heat of individuals facing the risk of cardio-vascular
vaporization diseases and hypertension?

118. The rate of change of momentum of a body A. High Density Lipoprotein (HDL)
is equal to the resultant________ applied. B. Low Density Lipoprotein (LDL)
A. Energy B. Power C. Triglyceride
C. Force D. Impulse D. Fatty acids

PAGE 1317
www.byjusexamprep.com

Elementary Mathematics
1. AD is the diameter of a circle with are 707 A. 1 B.
m2 and AB = BC = CD as shown in the figure
above. All curves inside the circle are
semicircles with their diameters on AD. C. D.
What is the cost of leveling the shaded
region at the rate of Rs. 63 per square 5. The value of
metre?
is
A. 0 B. 1
C. 2 D. 3

6. Suppose for
A. Rs. 29.700 Which one of the following is not correct?
B. Rs. 22.400 A. B.
C. Rs. 14,847
C. D. Median = HM
D. None of the above
7. Suppose for where
2. If a and b are negative real numbers and c which one of the following is
is a positive real number, then which of the correct?
following is/are correct?
A. AM < Median B. GM < Median
1. a – b < a – c
C. GM = Median D. AM = Median
a b
2. If a < b, then  .
c c 8. The age distribution of 40 children is as
follows:
1 1
3. 
b c
Select the correct answer using the code
given below. Consider the following statements in
respect of the above frequency
A. 1 B. 2 only distribution:
C. 3 only D. 2 and 3 1. The median of the age distribution is 7
3. The value of the expression years.

is equal to 2. 70% of the children are in the age group


6-9 years.
A. 0 B. 1 3. The modal age of the children is 8 years.
C. 2 D. 3
Which of the above statements are correct?
4. If where A. 1 and 2 only B. 2 and 3 only

then is equal to C. 1 and 3 only D. 1, 2 and 3

PAGE 1318
www.byjusexamprep.com

9. The mean and median of 5 observations are In the figure given above, p and q are
9 and 8 respectively. If 1 is subtracted from parallel lines. What are the values of the
each observation, then the new mean and angles x, y and z?
the new median will respectively be
A.
A. 8 and 7
B.
B. 9 and 7
C. 8 and 9 C.

D. Cannot be determined due to D.


insufficient data
13. ABC is an equilateral triangle and X, Y and Z
10. The election result in which six parties are the points on BC, CA and AB respectively
contested was depicted by a pie chart. such that Which of the
Party A had an angle 135o on this pie chart. following is/are correct?
If it secured 21960 votes, how many valid
1. XYZ is an equilateral triangle.
votes in total were cast?
2. Triangle XYZ is similar to triangle ABC.
A. 51240
Select the correct answer using the code
B. 58560
given below.
C. 78320
A. 1 only
D. 87840
B. 2 only
11.
C. Both 1 and 2
D. Neither 1 nor 2
14.

The linear inequeations, for which the


shaded area in the figure given above is the
solution set, are
A. In the figure given above,
What is the value of ?
B.
A. 32o
C.
B. 64o
D.
C. 78o
12.
D. Cannot be determined due to
insufficient data
15. ABC is a triangle right angled at C as shown
in the figure above. Which one of the
following is correct?

PAGE 1319
www.byjusexamprep.com

19. A person travels 7 km eastwards and then


turns right and travels 3 km and further
turns right and travels 13 km. What is the
shortest distance of the present position of
the person from his starting point?
A. 6 km B. km
A.
C. 7 km D. km
B.
20. If a transversal intersects four parallel
C. straight lines, then the number of distinct
values of the angles formed will be
D.
A. 2 B. 4
16. If and then the C. 8 D. 16
value of xy is equal to 21. In a triangle PQR, point X is on PQ and point
Y is on PR such that units,
A. 36 B. 24
units, units and
C. 16 D. 9 units. Which of the following are correct?
17. A circle of 3 m radius is divided into three 1.
areas by semicircles of radii 1 m and 2 m as
shown in the figure above. The ratio of the 2. QR is parallel to XY.
three areas A, B and C will be 3. Triangle PYX is similar to triangle PRQ.
Select the correct answer using the code
given below.
A. 1 and 2 only B. 2 and 3 only
C. 1 and 3 only D. 1, 2 and 3
22. Consider the following statements:
A. 2 : 3 : 2 B. 1:1:1 1. There exists a regular polygon whose
C. 4 : 3 : 4 D. 1 : 2 : 1 exterior angle is 70o.
2. Let Then the exterior angle of
18. ABC is a triangle in which D is the midpoint
any regular polygon of n sides is acute.
of BC and E is the midpoint of AD. Which of
the following statements is/are correct? Which of the above statements is/are
correct?
1. The area of triangle ABC is equal to four
times the area of triangle BED. A. 1 only B. 2 only

2. The area of triangle ADC is twice the C. Both 1 and 2 D. Neither 1 nor 2
area of triangle BED. 23. Consider the following statements:
Select the correct answer using the code 1. If and are distinct positive
given below. integers, then the sum of the exterior
A. 1 only B. 2 only angles of a regular polygon of m sides is
different from the sum of the exterior
C. Both 1 and 2 D. Neither 1 nor 2 angles of a regular polygon of n sides.

PAGE 1320
www.byjusexamprep.com

2. Let m, n be integers such that A. 10:20 a.m. B. 11:30 a.m.


Then the sum of the
C. 11:40 a.m. D. 11:50 a.m.
interior angles of a regular polygon of m
sides is greater than the sum of the 28. If m and n are distinct natural numbers,
interior angles of a regular polygon of n then which of the following is/are
integer/integers?
sides, and their sum is
1.
Which of the above statements is/are
correct?
2.
A. 1 only B. 2 only
C. Both 1 and 2 D. Neither 1 nor 2
3.
24. A circular path is made from two concentric
circular rings in such a way that the smaller Select the correct answer using the code
ring when allowed to roll over the given below.
circumference of the bigger ring, it takes
three full revolutions. If the area of the A. 1 and 2 B. 2 only
pathway is equal to n times the area of the C. 2 and 3 D. 3 only
smaller ring, then n is equal to
29. Consider a circle with centre at C. Let OP,
A. 4 B. 6 OQ denote respectively the tangents to the
C. 8 D. 10 circle drawn from a point O outside the
circle. Let R be a point on OP and S be a
25. If where x, y and z are point on OQ such that
Which of the following statements is/are
correct?
natural numbers, then what is z equal to? 1. If X is the circle with centre at O and
A. 1 B. 2 radius OR, and Y is the circle with centre
at O and radius OS, then X = Y.
C. 3 D. 4
2.
26. The expenditure of a household for a
certain month is Rs 20,000, out of which Select the correct answer using the code
Rs 8,000 is spent on education, Rs 5,900 on given below.
food, Rs 2,800 on shopping and the rest on A. 1 only B. 2 only
personal care. What percentage of
expenditure is spent on personal care? C. Both 1 and 2 D. Neither 1 nor 2

A. 12% B. 16.5% 30. Let ABC be a triangle in which Let


L be the locus of points X inside or on the
C. 18% D. 21.8% triangle such that Which of the
27. Outside a meeting room, Madhukar was following statements are correct?
told by a person that each meeting takes 1. L is straight line passing through A and
place after 13/4 hours. The last meeting has in-centre of triangle ABC is on L.
been over just 45minutes ago and the next
meeting will take place at 2 p.m. At what 2. L is straight line passing through A and
time did Madhukar receive this orthocentre of triangle ABC is a point
information? on L.

PAGE 1321
www.byjusexamprep.com

3. L is straight line passing through A and


C.
centroid of triangle ABC is a point on L.
Select the correct answer using the code D. None of the above
given below.
A. 1 and 2 only B. 2 and 3 only 34. In a triangle ABC if then

C. 1 and 3 only D. 1, 2 and 3 is equal to

31. Consider a circle with centre at O and radius A. B.


7 cm. let QR be a chord of length 2 cm and
let P be the midpoint of QR. Let CD be
another chord of this circle passing through C. π D.
P such that is acute. If M is the
midpoint of CD and then 35. If where
which of the following statements are
correct? then which of the following statements
is/are correct?
1.
1. There are two values of θ satisfying the
2. If CP = m cm and PD = n cm, then m and above equation.
n are the roots of the quadratic
equation 2. is satisfied by the above
equation.
3. The ratio of the area of triangle OPR to
the area of triangle OMP is Select the correct answer using the code
given below.
Select the correct answer using the code
given below. A. 1 only B. 2 only
A. 1 and 2 only C. Both 1 and 2 D. Neither 1 nor 2
B. 2 and 3 only 36. Two observers are staioned due north of a
C. 1 and 3 only tower (of height x metre) at a distance y
metre from each other. The angles of
D. 1, 2 and 3
elevation of the tower observerd by them
32. There are five lines in a plane, no two of are 30o and 45o respectively. Then x/y is
which are parallel. The maximum number equal to
of points in which they can intersect is
A. B.
A. 4 B. 6
C. 10 D. None of these
C. D. 1
33. Which of the following is correct in respect
of the equation 37. If and
(Given that α is a real number).
then is equal to
A.
A. 0 B. 1
B. C. 2 D. 3

PAGE 1322
www.byjusexamprep.com

38. Consider the following : 43. Four equal discs are placed such that each
one touches two others. If the area of
1. empty space enclosed by them is 150/847
square centimetre, then the radius of each
disc is equal to
2.
A. 7/6 cm B. 5/6 cm
Which of the above is/are identity/ C. 1/2 cm D. 5/11 cm
identities?
44. The base of an isosceles triangle is 300 units
A. 1 only
and each of its equal sides is 170 units. Then
B. 2 only the area of the triangle is
C. Both 1 and 2 A. 9600 square units
D. Neither 1 nor 2 B. 10000 square units
39. If and then C. 12000 square units

the value of is: D. None of the above


45. The number of rounds that wheel of
A. B. diameter 7/11 metre will make in traversing
C. D. 4 km will be

40. A square is inscribed in a right-angled A. 500 B. 1000


triangle with legs p and q, and has a C. 1700 D. 2000
common right angle with the triangle. The
diagonal of the square is given by 46. Let the triangles ABC and DEF be such
that and
A. B. Let L be the midpoint of BC
and M be the midpoint of EF. Consider the
C. D. following statements :
Statement I: Triangles ABL and DEM are
41. A tangent is drawn from an external point O similar.
to a circle of radius 3 units at P such that OP
= 4 units. If C is the centre of the circle, then Statement II: Triangle ALC is congruent to
the sine of the angle COP is triangle DMF even if
A. 4/5 B. 3/4 Which one of the following is correct in
C. 3/5 D. 1/2 respect of the above statements?

42. ABC and DEF are similar triangles. If the A. Both Statement (I) and Statement (II)
ratio of side AB to side DE is are individually true and Statement (II)
is the correct explanation of Statement
then the ratio of area of
(I)
triangle ABC to that of triangle DEF is
B. Both Statement (I) and Statement (II)
A. B. are individually true but Statement (II)
is NOT the correct explanation of
C. D. Statement (I)

PAGE 1323
www.byjusexamprep.com

C. Statement (I) is true but Statement (II) Which of the following statements are not
is false correct?
D. Statement (I) is false but Statement (II) 1. A’P’B’ and C’Q’D’ are similar triangles,
is true but need not be congruent.
47. If each interior angle of a regular polygon is 2. A’P’B’ and C’Q’D’ are congruent
140o, then the number of vertices of the triangles.
polygon is equal to
3. A’P’B’ and C’Q’D’ are triangles of same
A. 10 B. 9 area.

C. 8 D. 7 4. A’P’B’ and C’Q’D’ are triangles of same


perimeter.
48. A rhombus is formed by joining midpoints
of the sides of a rectangle in the suitable Select the correct answer wsing the code
order. If the area of the rhombus is 2 square given below.
units, then the area of the rectangle is A. 2 and 3 only
A. square units B. 1 and 3 only
B. 4 square units C. 1,2 and 4 only

C. square units D. 1,2,3 and 4

D. 8 square units 51. Two poles are placed at P and Q on either


side of a road such that the line joining P
49. Suppose ABC is a triangle with AB of unit and Q is perpendicular to the length of the
length. D and E are the points lying on AB road. A person moves x metre away from P
and AC respectively such that BC and DE are parallel to the road and places another pole
parallel. If the area of triangle ABC is twice at R. Then the person moves further x
the area of triangle ADE, then the length of metre in the same direction and turns and
AD is moves a distance y metre away from the
road perpendicularly, where he finds
A. unit B. unit himself, Q and R on the same line. The
distance between P and Q (i.e., the width of
the road) in metre is
C. unit D. unit
A. x B.
50. Suppose chords AB and CD of a circle
intersect at a point P inside the circle. Two
C. y D. 2y
right-angled triangles A’P’B’ and C’Q’D’
are formed as shown in the figures below 52. A truck moves along a circular path and
such that describes 100 m when it has traced out 36o
and at the centre. The radius of the circle is
equal to

A. m B. m

C. m D. m

PAGE 1324
www.byjusexamprep.com

53. The salary of a person is increased by 10% A. B.


of his original salary. But he received the
same amount even after increment. What C. D.
is the percentage of his salary he did not 58. A person goes to a market between 4 p.m.
receive? and 5 p.m. When he comes back, he finds
A. 11% B. 10% that the hour hand and the minute hand of
the clock have interchanged their positions.
C. (100/11)% D. (90/11)% For how much time (approximately) was he
54. In a race of 1000 m, A beats B by 100 m or out of his house?
10 seconds. If they start a race of 1000 m
A. 55.38 minutes
simulataneously from the same point and if
B gets injured after running 50 m less than B. 55.48 minutes
half the race length and due to which his
C. 55.57 minutes
speed gets halved, then by how much time
will A beat B? D. 55.67 minutes
A. 65 seconds B. 60 seconds 59. There are three brothers. The sums of ages
C. 50 seconds D. 45 seconds of two of them at a time are 4 years, 6 years
and 8 years. The age difference between
55. Two pipes A and B can fill a tank in 60 the eldest and the youngest is
minutes and 75 minutes respectively. There
is also an outlet C. If A, B and C are opened A. 3 years B. 4 years
together, the tank is full in 50 minutes. How C. 5 years D. 6 years
much time will be taken by C to empty the
full tank? 60. Consider the following in respect of natural
numbers a, b and c :
A. 100 minutes B. 110 minutes
C. 120 minutes D. 125 minutes 1.

56. A cloth merchant buys cloth from a weaver 2.


and cheats him by using a scale which is 10
cm longer than a normal metre scale. He 3.
claims to cell cloth at the cost price to his
custoomers, but while selling uses a scale 4.
which is 10 cm shorter than a normal metre
scale. What is his gain? Which of the above are correct?

A. 20% B. 21% A. 1 and 2 only


B. 3 and 4 only
C. D.
C. 1, 2 and 4 only
57. In a gathering of 100 people, 70 of them can
speak Hindi, 60 can speak English and 30 D. 1, 2, 3 and 4
can speak French. Further, 30 of them can 61. The value of k, for which the system of
speak both Hindi and English, 20 can speak equations and
both Hindi and French. If x is the number of has no solution, is
people who can speak both English and
French, then which one of the following is A. 10 B. 6
correct? (Assume that everyone can speak
at least one of the three languages). C. 5 D. 3

PAGE 1325
www.byjusexamprep.com

62. A passenger train takes 1 hour less for a 66. For what value of k, is a factor of
journey of 120 km, if its speed is increased
by 10 km/hour from its usual speed. What ?
is its usual speed? A. 4 B. 3
A. 50 km/hour C. 1 D. -2
B. 40 km/hour 67. Consider the following statements:

C. 35 km/hour 1. Every natural number is a real number.

D. 30 km/hour 2. Every real number is a rational number.

63. Which of the point 3. Every integer is a real number.


and lie in 4. Every rational number is a real number.

the solution of the system of inequations Which of the above statements are correct?
and ? A. 1, 2 and 3 B. 1, 3 and 4
A. Q and R only C. 2 and 3 only D. 3 and 4 only
B. P and R only 68. A bike consumes 20 mL of petrol per
kilometer, if it is driven at a speed in the
C. P and Q only range of 25-50 km/hour and consumes 40
D. P, Q and R mL of petrol per kilometer at any other
speed. How much petrol is consumed by
64. The solution of the inequation the bike in travelling a distance of 50 km, if
is (given that x ≠ 0) the bike is driven at a speed of 40 km/hour
for the first 10 km, at a speed of 60 km/hour
for the next 30 km and at a speed of 30
A. x > 0 km/hour for the last 10 km?
B. x < 0 A. 1 L B. 1.2 L
C. 1.4 L D. 1.6 L
C.
69. What is the maximum value of m, if the
number is divisible
D. by 3m ?

A. 8 B. 7
65. Consider the following statements:
C. 6 D. 5
1. There exists a positive real number m
such that 70. Under what condition on p and q, one of the
root of the equation is the
2. for all m, n belonging to
square of the other?
set of natural numbers.
A.
Which of the above statements is/are
correct? B.

A. 1 only B. 2 only C.

C. Both 1 and 2 D. Neither 1 nor 2 D.

PAGE 1326
www.byjusexamprep.com

71. In a circle of radius 2 units, a diameter AB Which one of the following is correct in
intersects a chord of length 2 units respect of the above statements?
perpendicularly at P. If then AP is
A. Both Statement (I) and Statement (II)
equal to are individually true and Statement (II)
is the correct explanation of Statement
A. units B. units (I)
B. Both Statement (I) and Statement (II)
C. units D. 3 units
are individually true but Statement (II)
is NOT the correct explanation of
72. A clock strikes once at 1 o’clock, twice at
Statement (I)
2 o’clock and thrice at 3 o’clock, and so on.
If it takes 8 seconds to strike at 5 o’clock, C. Statement (I) is true but Statement (II)
the time taken by it to strike at 10 o’clock is is false
A. 14 seconds B. 16 seconds D. Statement (I) is false but Statement (II)
is true
C. 18 seconds D. None of these
76. If a sum of money at a certain rate of simple
73. Let S be a set of first fourteen natural interest per year doubles in 5 years and at a
numbers. The possible number of pairs (a, different rate of simple interest per year
b), where and such that ab leaves becomes three times in 12 years, then the
remainder 1 when divided by 15, is difference in the two rates of simple
interest per year is
A. 3 B. 5
A. 2% B. 3%
C. 6 D. None of these
C. D.
74. The annual incomes of two persons are in
the ratio 9 : 7 and their expenses are in the
77. A cyclist moves non-stop from A to B, a
ratio 4 : 3. If each of them saves Rs 2,000
distance of 14 km, at a certain average
per year, what is the difference in their
speed. If his average speed reduced by 1 km
annual incomes?
per hour, he takes 20 minutes more to
A. Rs 4,000 B. Rs 4,500 cover the same distance. The original
average speed of the cyclist is
C. Rs 5,000 D. Rs 5,500
A. 5 km/hour B. 6 km/hour
75. Let ABC and A’B’C’ be two triangles in which
C. 7 km/hour D. None of these
and
Let D, E and F be the mid-points of the sides
78. is equal to
BC, CA and AB respectively. Let D’, E’ and F’
be the midpoints of the sides
A. -1 B. 0
and A’B’ respectively. Consider
the following statements : C. 1 D. 2
79. A man walking at 5 km/hour noticed that a
Statement I: and
225 m long train coming in the opposite
are always true. direction crossed him in 9 seconds. The
speed of the train is
Statement II :
A. 75 km/hour B. 80 km/hour
C. 85 km/hour D. 90 km/hour

PAGE 1327
www.byjusexamprep.com

80. Let a two-digit number be k times the sum 2. and hence for any integer
of its digits. If the number formed by m, we have two values of n for which
interchanging the digits is m times the sum
of the digits, then the value of m is
A. 9 – k B. 10 – k Which of the above statements is/are
correct?
C. 11 – k D. k – 1
A. 1 only B. 2 only
81. is divisible by
C. Both 1 and 2 D. Neither 1 nor 2
A. 5 B. 7
85. If the sum of the roots of
C. 10 D. 11 is equal to the sum of
the squares of their reciprocals, then which
82. If then which of the following one of the following relations is correct?
is/are correct? A.

1. B.

C.
2. Select the correct
D.
answer using the code given below.
86. If and then the
A. 1 only value of is (given that and
B. 2 only )

C. Both 1 and 2 A. 7 B. 9

D. Neither 1 nor 2 C. 11 D. 13
83. Consider the following statements in
87. If then
respect of two different non-zero integers p
and q: is equal to (given that )
1. For (p + q) to be less than (p – q), q must A. 0 B. 1
be negative.
C. ab D. 2ab
2. For (p + q) to be greater than (p – q),
both p and q must be positive. 88. If the linear factors of
are p and q, then
Which of the above statements is/are
correct? is equal to
A. 1 only B. 2 only
A. B.
C. Both 1 and 2 D. Neither 1 nor 2
C. D.
84. Consider the following statements in
respect of the expression 89. If and then
the value of can be equal to
where n is an integer:
A. 7 B. 5
1. There are exactly two values of n for
which C. 3 D. 1

PAGE 1328
www.byjusexamprep.com

90. Let m be a non-zero integer and n be a A. B.


positive integer. Let R be the remainder
obtained on dividing the polynomial C. D.
by Then
95. If then the value of
A. R is a non-zero even integer is equal to
B. R is odd, if m is odd A. 0 B. 1
C. for some integer s, if n is even C. 2 D. 3
D. for some integer t, if 3 divides n 96. If the equations and
91. Let p and q be non-zero integers. Consider have a common root, then
the polynomial It is which one of the following is correct?

given that and are A. B.

simple factors of where m is a non- C. D.


zero integer and k is a positive integer, 97. If and
Which one of the following is
correct? then the value of
will be
A.
A. B. 3
B.
C. 1 D. 0
C.
98. If and , then what is
D.

92. If the polynomial equal to ?

is divisible by then the value of


A. B.
is
A. 1 B. 9
C. D.
C. 10 D. 13

93. If then the 99. If 3x2 − 7x − 30 − 2x2 − 7x − 5 = x − 5 has


α and β as its roots, then the value of αβ is
value of A. –15 B. –5
Will be C. 0 D. 5
A. 3 B. 1 100. If the roots of the equation
C. 0 D. –1 are in the ratio p : q,

94. The sum and difference of two expressions then Is equal to


are and
respectively. The HCF of the two A. 0 B. 1
expressions will be C. 2 D. 3

PAGE 1329
www.byjusexamprep.com

ANSWERS KEY I 2016


ENGLISH

1 C 21 C 41 D 61 A 81 A 101 B
2 B 22 B 42 B 62 C 82 B 102 A
3 C 23 C 43 A 63 A 83 C 103 C
4 D 24 D 44 B 64 C 84 D 104 B
5 A 25 B 45 D 65 C 85 A 105 C
6 A 26 A 46 B 66 B 86 B 106 A
7 D 27 A 47 C 67 A 87 A 107 C
8 A 28 A 48 D 68 B 88 D 108 B
9 B 29 D 49 A 69 B 89 D 109 C
10 C 30 C 50 A 70 A 90 A 110 B
11 B 31 B 51 C 71 B 91 D 111 B
12 A 32 D 52 A 72 B 92 C 112 A
13 B 33 A 53 B 73 B 93 A 113 C
14 A 34 D 54 D 74 B 94 A 114 B
15 C 35 C 55 A 75 C 95 B 115 C
16 A 36 C 56 A 76 D 96 D 116 B
17 A 37 D 57 B 77 D 97 A 117 A
18 C 38 B 58 A 78 A 98 B 118 C
19 A 39 B 59 B 79 B 99 C 119 B
20 A 40 A 60 A 80 C 100 A 120 A

GENERAL KNOWLEDGE
1 D 21 A 41 B 61 C 81 D 101 B
2 B 22 B 42 B 62 D 82 A 102 C
3 B 23 D 43 C 63 C 83 D 103 C
4 C 24 C 44 A 64 B 84 A 104 B
5 C 25 A 45 A 65 C 85 A 105 A
6 C 26 C 46 A 66 B 86 C 106 C
7 B 27 B 47 B 67 A 87 D 107 C
8 A 28 D 48 A 68 D 88 D 108 B
9 D 29 B 49 A 69 A 89 D 109 D
10 B 30 D 50 B 70 A 90 A 110 B
11 C 31 A 51 C 71 A 91 A 111 D
12 C 32 C 52 B 72 C 92 C 112 D

PAGE 1330
www.byjusexamprep.com

13 B 33 C 53 D 73 A 93 D 113 C
14 D 34 D 54 A 74 D 94 B 114 C
15 C 35 A 55 C 75 D 95 C 115 D
16 A 36 C 56 B 76 D 96 D 116 A
17 C 37 C 57 A 77 C 97 B 117 D
18 A 38 A 58 A 78 C 98 D 118 C
19 B 39 B 59 D 79 A 99 D 119 B
20 A 40 C 60 B 80 A 100 C 120 A

Elementary Mathematics
1 C 21 D 41 C 61 C 81 D
2 D 22 B 42 C 62 D 82 A
3 C 23 D 43 D 63 C 83 A
4 C 24 C 44 C 64 D 84 A
5 B 25 C 45 D 65 D 85 A
6 C 26 B 46 C 66 D 86 A
7 C 27 B 47 B 67 B 87 A
8 B 28 B 48 B 68 D 88 A
9 A 29 C 49 C 69 B 89 B
10 B 30 D 50 C 70 C 90 A
11 A 31 A 51 C 71 B 91 B
12 D 32 C 52 C 72 B 92 C
13 C 33 B 53 C 73 D 93 B
14 B 34 D 54 A 74 A 94 B
15 C 35 B 55 A 75 D 95 A
16 D 36 C 56 C 76 C 96 D
17 B 37 B 57 A 77 C 97 C
18 C 38 C 58 A 78 C 98 C
19 B 39 D 59 B 79 C 99 A
20 A 40 C 60 D 80 C 100 A

PAGE 1331
CDS I 2016
www.byjusexamprep.com

(Solutions)
ENGLISH
1. Ans. C.
'Accuse' is always used with the preposition 'of'. To say accuse someone for something is just
grammatically incorrect. If you accuse someone of doing something wrong or illegal, you tell them
that you believe or suspect that they did it. Thus, option C is the correct answer.
On the other hand, you are charged with a crime.
2. Ans. B.
We don’t use want with a that-clause. The only forms of clause which want takes are those which
have an infinitive—that is, a verb in the infinitive preceded by the ‘particle’ to. Hence, instead of
"that i left", "me to leave" should be used. Thus, option B is the correct answer.
3. Ans. C.
The usage of "since" indicates that a past action has a relation with one in the present. So the present
perfect tense must be used as it is used to indicate a link between the present and the past. In a
present perfect tense, the exact time of the action is not important, and we are often more
interested in the result than in the action itself. Thus, option C is the correct answer.
4. Ans. D.
The given sentence is correct as given and needs no improvement.
5. Ans. A.
The phrase "of whose" is incorrect here. "Whose" is the possessive pronoun of "who" which indicates
the belongingness or an association. Hence, the preposition "of" is redundant. The correct
replacement is option A.
6. Ans. A.
"Is" is never followed by a past participle form of verb. Thus, the underlined part is incorrect.
"Comprise of" is an incorrect phrase as the word "comprise" itself conveys a complete meaning and
the preposition "of" is not required after it. Since the given sentence is a general and a factual one,
"consists of" is the best fit answer.
7. Ans. D.
To lay a table means to place and arrange things to be used while eating a meal, such as plates,
utensils, glasses, napkins, etc. Hence, the phrasal verb is correct as given and no change is required.
Thus, option D is the correct answer.

PAGE 1332
www.byjusexamprep.com

8. Ans. A.
Here the phrase "so arranged" conveys the meaning that the matters have been arranged in a
particular manner. So to state that particular manner, the conjunction "that" should be used instead
of "and".
9. Ans. B.
The usage of "began" in the latter part of the sentence indicates that the incident took place in the
past. Hence, "had" should be used instead of "have". Thus, the underlined part should be replaced
by option B.
10. Ans. C.
The usage of the verb "felt" indicates that the sentence is in the past tense. Also, the context makes
it clear that he reclined when he felt tired. Thus, the past tense of "lie" should be used, which is "lay"
and not "lied". Thus, option C is the correct replacement.
11. Ans. B.
"Though" itself shows a contrasting clause, hence, there is no need to use "but". Hence option B is
the correct answer.
12. Ans. A.
"Unless", which means "except if", is already a word with negative meaning, hence there is no need
to use any additional word with negative meaning. So “do not work" should be replaced by “you
work".
Unless is based on the fulfillment of a said condition, whereas until is concerned with the time frame
of the fulfillment of such a condition. The given sentence talks about a condition. Therefore, "unless"
will be used here.
13. Ans. B.
Here the usage of "too old" indicates that the she has grown so old that she cannot do work. So the
sentence carries a negative tone. "Any" is used in negative sentences to show negligible amount,
while "some" is used in a positive sentence. Thus, option B is the correct answer.
14. Ans. A.
The correct replacement is ‘deceiving’.
The word is a gerund that acts as a noun in the given sentence while making a grammatically correct
sentence.
You can use the -ing form after some verbs such as enjoy, admit, appreciate, can't stand / help /bear,
deny, avoid, mind, understand, consider, finish, imagine and resent.
15. Ans. C.
Saw is the past tense of the verb see. It forms the simple past, which is used to express an action that
has started and finished at a specific time in the past. E. G. We saw the movie yesterday.
Seen is the past participle of the verb see, and it is used to form the perfect tenses: present perfect,
past perfect, etc. Past participles seen is generally used alongside have, has, had, was or were in a
sentence to form the perfect tenses. Since "have" has been used in the sentence, the present perfect
tense should be formed by using "seen".

PAGE 1333
www.byjusexamprep.com

16. Ans. A.
The usage of "once" indicates that ancient, historical place was owned by the Pandavas in the past.
Hence, the past tense "belonged" should be used. Hence, the correct answer is option A.
17. Ans. A.
The phrase "were knowing" is incorrect here and it should be replaced by option A.
It is important to understand that not all verbs can be used in the continuous form. We will refer to
these verbs as ‘Non-continuous Verbs. Another thing which is particular about verbs and the
continuous forms is that some verbs that can be used in both the simple and perfect forms and also
in the continuous forms have two different meanings.
Verbs that are physical actions which you can see someone doing can be used in all forms and with
all tenses; run, walk, eat, read, fly, say, touch etc. can be used in all tenses.
Non-continuous Verbs that cannot be used in continuous forms are usually verbs that you cannot
see somebody doing. These verbs are rarely used in continuous forms. They are:
Abstract verbs: Be, want, cost, need, care, contain, owe, exist etc.
Possession verbs: Own, belong, possess etc.
Emotion Verbs: Like, love, hate, dislike, fear, envy etc.;
She needs help. Not She is needing help.
He wants a break. Not He is wanting a break
18. Ans. C.
Here the placement of the words is incorrect. "Only people" emphasises on the people. But in the
given sentence is a conditional one and states that our country cannot flourish if people do not work
hard. Hence, "only" should be put before "when". Thus, option C is correct.
19. Ans. A.
Here the sentence is being spoken in the present tense, hence, the same tense should be used after
"when". Thus, the singular verb "comes" should be used for the "father".
20. Ans. A.
"Lest" means "in case" and is always followed by the modal "should". Hence the part after the
underlined portion must be positive and have the modal "should", thus conveying that the umbrella
should be or one could get wet. Hence, option A is the correct replacement.
21. Ans. C.
All other parts except Q, start with a preposition or a conjunction. Thus, they cannot be the first part.
Hence Q must be the first part followed by S which states the thing that was refused. Venues would
belong to the raids, hence R follows. What was said is stated in P. Thus, the correct sequence is QSRP.
22. Ans. B.
The subject is the rich man, who is mentioned in R. S and Q state what the man does and P forms the
last part correctly. Thus, the correct sequence is RSQP.

PAGE 1334
www.byjusexamprep.com

23. Ans. C.
The subject and the action of the subject is mentioned in P. S states who was rushed, thus, it forms
the second part. R and Q state the purpose and the object respectively. Hence, the correct sequence
is PSRQ.
24. Ans. D.
P has to be the first part as it carries the subject. The continuous verb in R states the action of the
subject, hence it comes next in the sequence. Q and S state the purpose and the time respectively.
Thus, the correct sequence is PRSQ.
25. Ans. B.
The subject and the verb are mentioned in Q, hence it is the first part. S is the second part as it states
the receivers of the order. The stated order is mentioned in R and the adverb of manner describing
the verb "catch" is mentioned in P. Thus, the correct sequence is QSRP.
26. Ans. A.
The given sentence is a hypothetical one, hence must start by "if". Hence Q forms the first part. P
states the hypothetical action, hence it comes next in the sequence. R and S together form the last
part. Thus, the correct sequence is QPRS.
27. Ans. A.
The subject is mentioned in R. S states what they take pride in. Thus, it follows R. The adjective phrase
describing the nature of the nature of the job is mentioned in S, hence, it follows next. The last part
is P. Thus, the correct sequence is RQSP.
28. Ans. A.
The usage of while indicates that there are two cases prevailing in a single context. Thus, PSRQ should
be the correct sequence as the other sequence that is possible is RQPS which is not mentioned in the
option.
29. Ans. D.
The stated part ends in then indicating that something was believed in the past. Hence the part after
that must indicate the present state. Hence R should be the first part. The conjunction "that" in P
tells what is to be believed. S and Q follow logically. Thus, the correct sequence is RPSQ.
30. Ans. C.
The subject and the verb are mentioned in P. S takes the sentence forward by stating what is
wondered about. The object is stated in R. Q forms the last part. Thus, the correct sequence is PSRQ.
31. Ans. B.
The correct sequence is QSRP.
32. Ans. D.
The whole sentence conveys what the man is and is not. The positive action must come first and the
part carrying "not merely" must follow later. So option D is the correct sequence.

PAGE 1335
www.byjusexamprep.com

33. Ans. A.
Q must follow the given part as it states the action of the robbers. This must be followed by S which
states the time that cannot be placed in the end as it will make the sentence sound incorrect. The
secondary action is mentioned in R with the usage of the conjunction "and". P states the time when
the stealing occurred, thus, it is the last part. Hence, the correct sequence is QSRP.
34. Ans. D.
The action of the opposition members is mentioned is mentioned in S, thus, it should follow the fixed
part. R follows next as it states the place from where they walked out of. Q states the purpose and P
carries the party "against" which the protest was made. Thus, the correct sequence is SRQP.
35. Ans. C.
The context of the boy cannot come first as "she" mentioned in Q has not been introduced. Thus,
the first part should be S. One would be run over by a vehicle, thus, Q comes next in the sequence.
The role of the boy is described in P and R, which end the sentence. Thus, the correct sequence is
SQPR.
36. Ans. C.
The opening statement talked about the king and his nice garden. The next statement in continuation
will be talking about the flowers and the bells tied to them. The following statement would be talking
about the garden being so long and next that goes with the flow is what is present in the middle of
the garden which clearly connects the ending statement about the singing of the nightingale. Thus,
the correct sequence is QSRP.
37. Ans. D.
The opening statement talks about the teachings of the new language. The next statement in
continuation will be the statement R which talks about how easy it is for children to learn new
languages. The continuing statement would be statement Q, which talks about why it is difficult for
grown up and it will be followed by S, as it talked about learning by using. The next statement would
be taking about the thought process involved in a language which connects well with the given
closing statement. Thus, the correct sequence is RQSP.
38. Ans. B.
The opening statement talked about the convent life of a seventeen year old girl. The next statement
would be the statement talking about her life out there. So, R goes in continuation with the opening
statement and P talks about how her heart went out by seeing the slums of Calcutta which makes
her feel about her second vacation and then Q mentioned how she asked for permission to work
over there which very well connects with the concluding statement. Thus, the correct sequence is
RPSQ.
39. Ans. B.
The opening statement talks about the good memory and about a man considered as unconventional
and slightly strange who doesn’t possess it. The continuing statement talks about a father who took
her baby out i.e. P. The next statement would be talking about his unusual behaviour of leaving the
child outside and getting disappeared in a drink shop. The following statement connects how his wife
accidently got discovered her bay sleeping and how she decided to teach him a lesson. Thus, the
correct sequence is PRQS.

PAGE 1336
www.byjusexamprep.com

40. Ans. A.
The opening statement states that human life has changed gradually. Then statement Q would be
the connecting statement as it discuss how man live ten thousand years back just by hunting. Then
R describes how life started with the discovery of the agriculture. And it followed with the change in
the civilization. The next statement talked about the changed in the history from now and then which
very well connects with the closing statement that we cannot prevent change. Thus, the correct
sequence is QRSP.
41. Ans. D.
The opening statement talks about what constitute success. The next statement says that it is just a
misinterpreted perception that will last for very short time i.e. statement R. So, statement R being
the first option will eliminate other options. Thus, the correct sequence would be RQPS.
42. Ans. B.
The opening statement is talking about an email mentioning the return of an employee. The next
statement in continuation will be the reaction of an employee regarding the mail. So, P follows next
which is followed by R as it states how he arranged his return by collecting money, booking tickets,
laundry and the collection books and articles. Thus, the correct sequence would be PRQS.
43. Ans. A.
The opening statement talked about the learning of Wordsworth about owls than us about black
birds. So, the next statement in continuation would be talking about the happier school days. So, Q
will be the continuing statement and this eliminates other options. Thus, the correct sequence would
be QPRS.
44. Ans. B.
The opening statement talks about the science and its progress in the field of medicine. The next
statement would be talking about the conditions before the progress in this field i.e. about the 18th
century in statement Q. The continuing statement would be talking about its improvement in the
19th century in statement S. This eliminates other options. Thus, the correct sequence would be
QSPR.
45. Ans. D.
The opening statement talks about the reaction of the traveller while travelling and his interests. The
next statement in continuation talks about how in between he took out some letter and starts
scribbling. So, the next statement would be statement P which will be followed by statement R
talking about how he drew something unusual thing from the back of his waist. Then the next
statement i.e. statement Q talked about the revolver that he took out. And then statement S talked
about the reflection of the revolver in light showing it is loaded which very well continues with the
concluding statement. Thus, the correct sequence would be PRQS.
46. Ans. B.
‘Yield’ means give way to arguments, demands, or pressure, ‘Gave’ means freely set aside or devote
for a purpose, ‘Cowed’ means cause (someone) to submit to one's wishes by intimidation, ‘Agreed’
means discussed or negotiated and then accepted by all parties, ‘Over –whelmed’ means have a
strong emotional effect on. So, ‘gave in’ would be correct as someone under pressure will give in
something.

PAGE 1337
www.byjusexamprep.com

47. Ans. C.
‘Prevail’ means prove more powerful or superior, ‘Entail’ means involve (something) as a necessary
or inevitable part or consequence, ‘Crumble’ means break or fall apart into small fragments,
especially as part of a process of deterioration; disintegrate gradually over a period of time, ‘Waits’
means remain in readiness for a purpose. Thus, ‘Crumbles’ fits the blank contextually.
48. Ans. D.
‘Perspective’ means a particular attitude towards or way of regarding something; a point of view,
‘Hegemony’ means leadership or dominance, especially by one state or social group over others,
‘Observation’ means the action or process of closely observing or monitoring something or someone,
‘Ownership’ means the act, state, or right of possessing something. Thus, ‘Ownership’ fits the blank
logically. Other options are not correct contextually.
49. Ans. A.
‘Struggle’ means make forceful or violent efforts to get free of restraint or constriction, ‘Conflict’
means a serious disagreement or argument, typically a protracted one, ‘Onslaught’ means a fierce
or destructive attack, ‘March’ means walk in a military manner with a regular measured tread. Thus,
‘Struggle’ fits the blank correctly, as Gandhi’s struggle for independence is being talked about.
50. Ans. A.
‘Extravagant’ means lacking restraint in spending money or using resources, ‘Frugal’ means sparing
or economical as regards money or food, ‘Unsavoury’ means disagreeable and unpleasant because
morally disreputable. Thus, ‘Extravagant’ fits the blank as he couldn’t save any money, so he spends
a lot.
51. Ans. C.
‘Rebuked’ means express sharp disapproval or criticism of (someone) because of their behaviour or
actions, ‘Disparage’ means regard or represent as being of little worth, ‘Accused’ means a person or
group of people who are charged with or on trial for a crime, ‘Demonized’ means portray as wicked
and threatening. Thus, ‘Accused’ fits best as one is accused of something.
52. Ans. A.
‘Fell out’ means to quarrel, ‘Fell through’ means to fail, ‘Fall off’ means to tumble, ‘Fell across’ means
drop oneself to a lower or less erect position. Thus, ‘Fell out’ is the correct phrase as the robbers
started quarrelling amongst themselves.
53. Ans. B.
‘Reverent’ means feeling or showing deep and solemn respect, ‘Naïve’ means (of a person or action)
showing a lack of experience, wisdom, or judgement, ‘Articulate’ means having or showing the ability
to speak fluently and coherently, ‘Humble’ means having or showing a modest or low estimate of
one's importance. Thus, ‘Naïve’ fits best as the word needed should be contrasting to the former
part of the sentence.
54. Ans. D.
‘Gruesome’ means causing repulsion or horror; grisly. ‘Great’ and ‘Final’ are irrelevant as accidents
can’t be great and final. ‘Troublesome’ also doesn’t fits in the context. Thus, ‘Gruesome’ meaning
horror fits the blank contextually.

PAGE 1338
www.byjusexamprep.com

55. Ans. A.
‘Obsesses’ means preoccupy or fill the mind of (someone) continually and to a troubling extent.
‘Deranged’ means mad; insane.
‘Unbalanced’ means (of a person) emotionally or mentally disturbed.
‘Dissatisfied’ means not content or happy with something.
So, ‘Obsessed’ fits the blank correctly as one can be ‘obsessed’ with one’s uniqueness. Other options
are irrelevant.
56. Ans. A.
‘Hardly’ means scarcely (used to qualify a statement by saying that it is true to an insignificant
degree), ‘Hard’ means done with a great deal of force or strength. Thus, ‘Hardly’ needs to be replaced
with ‘Hard’ to make the sentence grammatically and contextually correct. This is because; the
sentence needs an adjective and not an adverb.
57. Ans. B.
‘Whom’ needs to be replaced with ‘Who’ to make the sentence grammatically and contextually
correct. This is because; ‘Whom’ used instead of ‘who’ as the object of a verb or preposition. But this
is not the case here. Also, ‘Who’ is used to introduce a clause giving further information about a
person or people previously mentioned.
58. Ans. A.
The phrase ‘look after’ needs to be replaced with the phrase ‘look for’ This is because, ‘Look after’
means to take care of someone or something whereas, ‘Look for’ means to look for someone or
something in a hope to get something or someone that you want or need.
59. Ans. B.
The modal ‘will’ needs to be replaced with ‘would’ to make the sentence grammatically correct. This
is because the sentence is in past tense, so the past form of the modal ‘will’ i.e. ‘would’ is to be used.
60. Ans. A.
‘All their belongings’ is the correct form of phrase to be used in the sentence. It means with all their
movable possessions.
61. Ans. A.
The phrase ‘in the temper’ needs to be replaced with ‘in a temper’ to make the sentence
grammatically and contextually correct. This is because, if someone is in a temper, the way that they
are behaving shows that they are feeling angry and impatient.
62. Ans. C.
Here, the relative pronoun ‘those’ needs to be added before ‘his house’ to make the sentence
correct. This is because the decoration of one house is being compared to the decoration of other.
But the sentence states that the decoration is being compared to the house.
63. Ans. A.
Despite is not followed by ‘of’. This is because, ‘Despite’ itself means inspite of or without being
affected by. Thus, there is a hidden preposition in despite. So it won’t be followed by any preposition.

PAGE 1339
www.byjusexamprep.com

64. Ans. C.
The modal ‘will’ needs to be replaced with ‘would’ to make the sentence grammatically correct. This
is because the sentence is in past tense, so the past form of the modal ‘will’ i.e. ‘would’ is to be used.
65. Ans. C.
The modal ‘Shall’ is incorrect. This is because if two actions take place in future one after another
and if the second action depends on the first action then the first action will be in simple present
tense and the second action will be in simple future tense. So, here the first action is ‘I shall reach
Chennai’. So, it will be in simple present tense.
66. Ans. B.
‘Support you claim’ needs to be replaced with ‘Supports your claim’ to make the sentence
grammatically correct. This is because, ‘Neither’ is singular, so, singular form of verb is needed.
67. Ans. A.
‘School teacher’ should be preceded by an article ‘a’, to make the sentence grammatically and
contextually correct.
68. Ans. B.
‘To smoke’ needs to be replaced with ‘That smoking’ to make the sentence grammatically and
contextually correct. This is because; gerund form of verb is needed. Also, ‘to smoke’ suggests that
his grandmother is asking him to smoke.
69. Ans. B.
‘Gave you’ is the correct form of phrasal verb. This is because, ‘it to’ is superfluous here and the
meaning of the sentence doesn’t change after eliminating it.
70. Ans. A.
The phrase ‘I am entirely agreeing’ needs to be replaced with ‘I entirely agree’ to make the sentence
grammatically and contextually correct. This is because the use of helping verb ‘am’ is incorrect also
gerund form of verb is incorrect. So, simple present tense is used.
71. Ans. B.
From the 3rd line of the passage, it can be clearly inferred that ‘If the matter is one that can be settled
by observation, make the observation yourself’. Thus, option B is the correct answer.
72. Ans. B.
From the 4th line of the passage, it can be concluded that, ‘the mistake of thinking that women have
fewer teeth than men’. Thus, Aristotle thought that women have fewer teeth than men.
73. Ans. B.
From the last few lines of the passage, it can be inferred that, ‘I believe myself that hedgehogs eat
black beetles, because I have been told that they do; but if I were writing a book on the habits of
hedgehogs, I should not commit myself until I had seen one enjoying this diet’. Thus, the writer would
first observe their eating habits.

PAGE 1340
www.byjusexamprep.com

74. Ans. B.
From the last three lines of the passage, it can be inferred that, ‘Ancient and medieval writers knew
all about unicorns and salamanders; not one of them thought it necessary to avoid dogmatic
statements about them because he had never seen one of them’. Hence, option B is correct.
75. Ans. C.
‘Dogmatic’ means inclined to lay down principles as undeniably true. Thus, in the context it means a
statement which is unquestionable.
76. Ans. D.
From the lines in the passage, ‘I pulled out my purse to make the payment, and discovered that it
was empty’. Thus, the man could not buy the handkerchiefs because he had no money in the purse.
77. Ans. D.
From the last three lines of the passage, ‘I then realized that it was not my purse, it was a different
purse altogether. How that happened is still a source of wonder to me and I refuse to believe that it
was the work of my good old friend, for it was his purse that I held in my hand’. Thus, it can be
concluded that it was his friend’s purse.
78. Ans. A.
From the opening line of the passage it can be inferred that, ‘A profound terror, increased still by the
darkness, the silence and his waking images, froze his heart within him’.
79. Ans. B.
From the passage, it can be concluded that, ‘He almost felt his hair stand on end, when by straining
his eyes to their utmost; he perceived through the shadows two faint yellow lights’. Thus, he felt his
hair stand upright.
80. Ans. C.
From the last sentence of the passage, ‘he beheld a huge animal lying but two steps from him’. Thus,
the person in the story saw the animal by chance.
81. Ans. A.
From the opening line of the passage, it has been mentioned that, ‘We are tempted to assume that
technological progress is real progress and that material success is the criterion of civilization’. Thus,
the real progress lies in material success and technological growth.
82. Ans. B.
From the lines of the passage, ‘They are not to be tabooed but tamed and assigned their proper
place’. Thus, science and technology should be used in a controlled and careful manner.
83. Ans. C.
From the passage, it is clear that, ‘If the Eastern people fascinated by machines and techniques and
use them, as Western nations do, to build huge industrial organizations and large military
establishments, they will get involved in power politics and drift into the danger of death’. Thus,
Eastern people must avoid being controlled by machines and techniques of industrial production.

PAGE 1341
www.byjusexamprep.com

84. Ans. D.
From the passage, it is clear that, ‘Scientific and technological civilization brings great opportunities
and great rewards but also great risks and temptations. They become dangerous only if they become
idols’. Thus, science and technology are useful, if they are not worshipped blindly.
85. Ans. A.
From the passage, ‘Luck is waiting for something to turn up; labour always turns up something. Luck
lies in bed and labour turns out at six, Luck whines, labour watches, Luck relies on chance; labour on
character. Luck slips downwards; labour strides upwards’.
Thus, the correct Answer is option A.
86. Ans. B.
From the passage it can be inferred that, ‘A man’s success in life will be proportionate to his efforts,
to his industry, to his attention to small things’. Thus, success depends on hard work and attention
to details.
87. Ans. A.
From the passage, it is clear that, ‘the conviction, therefore, is extending that diligence is the mother
of good luck; in other words, that a man’s success in life will be proportionate to his efforts, to his
industry, to his attention to small things’. Thus, hard work of all kinds makes people efficient and
skilled.
88. Ans. D.
It is clearly given in the first sentence that, ‘The avowed purpose of the exact sciences is to establish
complete intellectual control over experience in terms of precise rules which can be formally set out
and empirically tested’. Thus, the purpose of the exact science is to make formal and testable rules
which can verify experience.
89. Ans. D.
From the 4th line, it can be inferred that, ‘Could that ideal be fully achieved, all truth and all error
could henceforth be ascribed to an exact theory of the universe. Thus, an exact theory of the universe
is yet to be made.
90. Ans. A.
From the last line of the passage, ‘But this leaves out of account the element of personal judgement
involved in applying the formulae of mechanics to the facts of experience’. Thus, in exact science
personal judgements are set aside in favour of a mechanical theory.
91. Ans. D.
From the passage, it can be inferred that, ‘Classical mechanics approaches this ideal so closely that
it is often thought to have achieved it. But this leaves out of account the element of personal
judgement involved in applying the formulae of mechanics to the facts of experience’. Thus, classical
mechanics just falls short of achieving intellectual control over experience.
92. Ans. C.
‘Ambiguous’ means open to more than one interpretation; not having one obvious meaning,
‘Contrasting’ means compare in such a way as to emphasize differences, ‘Connivance’ means
willingness to allow or be secretly involved in an immoral or illegal act, ‘Vague’ means of uncertain,

PAGE 1342
www.byjusexamprep.com

indefinite, or unclear character or meaning, ‘Wilful’ means having or showing a stubborn and
determined intention to do as one wants, regardless of the consequences, ‘Regress’ means return to
a former or less developed state. Thus, ‘Ambiguous’ and ‘Vague’ are synonymous to each other.
93. Ans. A.
Let’s first learn the meanings of the given words:
Elucidate = make (something) clear; explain,
Clarify = make (a statement or situation) less confused and more comprehensible,
Calculate = intend (an action) to have a particular effect,
Summarize = give a brief statement of the main points of (something), ‘
Update = make (something) more modern or up to date.
Hence, option A is the correct answer.
94. Ans. A.
‘Monotonous’ means dull, tedious, and repetitious; lacking in variety and interest, ‘Dreary’ means
depressingly dull and bleak or repetitive, ‘Dreadful’ means used to emphasize the degree to which
something is the case, especially something regarded with sadness or disapproval, ‘Single – minded’
means concentrating on only one aim, ‘Monologue’ means a long, tedious speech by one person
during a conversation. Thus, ‘Monotonous’ and ‘Dreary’ are synonymous to each other.
In the following question, out of the four alternatives, select the word similar in meaning to the given
word.
95. Ans. B.
Let’s first learn the meanings of the given words:
Kindle = arouse or inspire (an emotion or feeling).
Make fun of = tease, laugh at, or joke about (someone) in a mocking or unkind way.
Very kind = generous, helpful, and thinking about other people's feelings
Kind-hearted = having a kind and sympathetic nature
Excite = cause (someone) to feel very enthusiastic and eager. she hesitated, suspicion kindling within
her
Hence, option B is the correct answer.
96. Ans. D.
Let’s first learn the meanings of the given words:
Luxurious =extremely comfortable or elegant, especially when involving great expense.
Very clean = free from dirt, marks, or stains.
Very special = better, greater, or otherwise different from what is usual.
Sense of taste = a sensation obtained from a substance in the mouth that is typically produced by
the stimulation.

PAGE 1343
www.byjusexamprep.com

Magnificent = extremely beautiful, elaborate, or impressive.


Hence, option D is the correct answer.
97. Ans. A.
‘Tactful’ means having or showing skill and sensitivity in dealing with others or with difficult issue,
‘Diplomatic’ means of or concerning diplomacy, ‘Indifferent’ means having no particular interest or
sympathy; unconcerned, ‘Intelligent’ means having or showing intelligence, especially of a high level,
‘Deceitful’ means guilty of or involving deceit; deceiving or misleading others. Thus, ‘Tactful’ and
‘Diplomatic’ are synonymous to each other.
In the following question, out of the four alternatives, select the word similar in meaning to the given
word.
98. Ans. B.
Let’s first learn the meanings of the given words:
Voracious = wanting or devouring great quantities of food.
Very Bad = not such as to be hoped for or desired; unpleasant or unwelcome.
Insatiable = (of an appetite or desire) impossible to satisfy.
Stingy = mean; ungenerous.
Malicious = characterized by malice; intending or intended to do harm.
Hence, option B is the correct answer.
99. Ans. C.
Let's first learn the meanings of the given words:
Stricture = a restriction on a person or activity
Strictness = the quality or condition of being strict
Disapproval = possession or expression of an unfavourable opinion
Censure = express severe disapproval of (someone or something), especially in a formal statement
Nonplus = surprise and confuse (someone) so much that they are unsure how to react.
Hence, option C is the correct answer.
100. Ans. A.
Obeisance (n.): deferential respect; a gesture expressing deferential respect, such as a bow or curtsy
Veneration (n.): great respect; reverence.
Excursion (n.): a short journey or trip, especially one taken as a leisure activity.
Subjugation (n.): the action of bringing someone or something under domination or control.
Obligation (n.): an act or course of action to which a person is morally or legally bound; a duty or
commitment.
Gusto (n.): enjoyment and enthusiasm in doing something.
So, the correct answer is option A.

PAGE 1344
www.byjusexamprep.com

101. Ans. B.
Wish = feel or express a strong desire or hope for something that cannot or probably will not happen.
Action= the fact or process of doing something, typically to achieve an aim.
Idea = a thought or suggestion as to a possible course of action.
The sentence means thought alone is nothing unless we implement it in doing something.
Hence, the correct answer is B.
102. Ans. A.
Thought = an idea or opinion produced by thinking, or occurring suddenly in the mind.
Wish = feel or express a strong desire or hope for something that cannot or probably will not happen.
Action= the fact or process of doing something, typically to achieve an aim.
Idea = a thought or suggestion as to a possible course of action.
The sentence means action without an idea is a folly.
Hence, the correct answer is A.
103. Ans. C.
Never = at no time in the past or future.
Belatedly = later than should have been the case.
Sometimes = occasionally, rather than all of the time.
Seldom = not often.
The sentence means we may occasionally act on impulse.
Hence, the correct answer is C.
104. Ans. B.
Peaceful = free from disturbance.
Uncontrollable = not controllable.
Indisputable = unable to be challenged or denied.
Within limits = up to a point
The sentence talks of acting on impulse or an urge that is beyond our control.
Hence, the correct answer is B.
105. Ans. C.
No= used to indicate that something is quite the opposite of what is being specified.
An= the form of the indefinite article used before words beginning with a vowel sound.
A= used when mentioning someone or something for the first time in a text or conversation.

PAGE 1345
www.byjusexamprep.com

The = denoting one or more people or things already mentioned or assumed to be common
knowledge.
The word brick is a singular countable noun beginning with a consonent sound.
Hence, the correct answer is C.
106. Ans. A.
Hand= the end part of a person's arm beyond the wrist, including the palm, fingers, and thumb.
Wrist= the joint connecting the hand with the forearm.
Finger= each of the four slender jointed parts attached to either hand (or five, if the thumb is
included).
Ear= the organ of hearing and balance in humans and other vertebrates, especially the external part
of this.
Hence, the correct answer is A.
107. Ans. C.
Stimulate= encourage or arouse interest or enthusiasm in.
Rescue= save someone from a dangerous or difficult situation.
Protect= keep safe from harm or injury.
Shelter= a place giving temporary protection from bad weather or danger.
One would try keeping oneself safe from getting injured by the brick.
Hence, the correct answer is C.
108. Ans. B.
Uncontrollable= not controllable.
Instinctive= relating to or prompted by instinct.
Impulsive= acting or done without forethought.
Precipitate= cause an event or situation, typically one that is undesirable to happen suddenly,
unexpectedly, or prematurely.
The action mentioned in the above sentence was done without any forethought.
Hence, the correct answer is B.
109. Ans. C.
To = expressing motion in the direction of a particular location.
In = expressing the situation of something that is or appears to be enclosed or surrounded by
something else.
Of = expressing the relationship between a part and a whole:
An= the form of the indefinite article used before words beginning with a vowel sound.
Hence, the correct answer is C.

PAGE 1346
www.byjusexamprep.com

110. Ans. B.
Made= create.
Condition= have a significant influence on or determine the manner or outcome of something.
Developed= advanced or elaborated to a specified degree.
Hardened= having become or been made hard or harder.
The sentence talks of something having influence on our living.
Hence, the correct answer is B.
111. Ans. B.
Thoughts = an idea or opinion produced by thinking, or occurring suddenly in the mind.
Wish = feel or express a strong desire or hope for something that cannot or probably will not happen.
Action= the fact or process of doing something, typically to achieve an aim.
Idea = a thought or suggestion as to a possible course of action.
Hence, the correct answer is B.
112. Ans. A.
The subject of the sentence is first person plural number. Therefore option A is the correct answer.
113. Ans. C.
Thoughts = an idea or opinion produced by thinking, or occurring suddenly in the mind.
Wish = feel or express a strong desire or hope for something that cannot or probably will not happen.
Action= the fact or process of doing something, typically to achieve an aim.
Idea = a thought or suggestion as to a possible course of action.
Hence, the correct answer is C.
114. Ans. B.
Proceeded = begin a course of action.
Proceeded = come before something in time.
Followed = move or travel behind.
Progressed = move forward or onward in space or time.
Hence, the correct answer is B.
115. Ans. C.
Some= an unspecified amount or number of.
If= introducing a conditional clause:
The= denoting one or more people or things already mentioned or assumed to be common
knowledge.
Few= a small number of.
Hence, the correct answer is C.

PAGE 1347
www.byjusexamprep.com

116. Ans. B.
Developed = advanced or elaborated to a specified degree.
Allied = joined by or relating to members of an alliance, in combination or working together with.
Hostile = showing or feeling opposition or dislike
Unsympathetic = not feeling, showing, or expressing sympathy.
Hence, the correct answer is A.
117. Ans. A.
They = used to refer to two or more people or things previously mentioned or easily identified.
Thoughts = an idea or opinion produced by thinking, or occurring suddenly in the mind.
We= used by a speaker to refer to himself or herself and one or more other people considered
together.
The= denoting one or more people or things already mentioned or assumed to be common
knowledge.
Hence, the correct answer is A.
118. Ans. C.
Appear = come into sight.
Develop= grow or cause to grow and become more mature, advanced, or elaborate.
Grow= undergo natural development by increasing in size and changing physically.
Stimulate= encourage or arouse interest or enthusiasm in.
Hence, the correct answer is C.
119. Ans. B.
Reconciliation = the restoration of friendly relations.
Conflict= a serious disagreement or argument, typically a protracted one.
Inflict= cause (something unpleasant or painful) to be suffered by someone or something.
Encounter= unexpectedly be faced with or experience (something hostile or difficult).
Hence, the correct answer is A.
120. Ans. A.
Inability= the state of being unable to do something.
Probability= the quality or state of being probable; the extent to which something is likely to happen
or be the case.
Plausibility= (of an argument or statement) seeming reasonable or probable.
Possibility= a thing that may happen or be the case.
Hence, the correct answer is A.

PAGE 1348
www.byjusexamprep.com

General Knowledge
1. Ans. D.
At present, there are 42 Indian satellites operational in orbit.
Out of these 42 satellites, 15 satellites are used for communication, 4 for meteorological
observations, 14 for earth observations, 7 for navigation and 2 for space science purposes. Hence,
option D is correct.
2. Ans. B.

Hence, option B is correct.


3. Ans. B.
The Wage Boards for journalists & non-journalists employed by newspapers and news agencies are
statutory, while all other Wage Boards are non-statutory in nature.
• According to the Second Commission, there is no need for a wage board to set wage rates for
workers in any industry.
• The Wage Boards are tripartite in character in which representatives of workers, employers and
independent members participate and finalise the recommendations.
Hence, option B is correct.
4. Ans. C.
Ability-to-pay taxation is a progressive taxation principle that maintains that taxes should be levied
according to a taxpayer's ability to pay. This progressive taxation approach places an increased tax
burden on individuals, partnerships, companies, corporations, trusts, and certain estates with higher
incomes.
Hence, option C is correct.
5. Ans. C.
Faxian was a Chinese Buddhist monk and translator who traveled by foot from China to India, visiting
many sacred Buddhist sites in what are now Xinjiang, Pakistan, India, Nepal, Bangladesh and Sri Lanka
between 399-412 BC to acquire Buddhist texts.
Hence, option C is correct.

PAGE 1349
www.byjusexamprep.com

6. Ans. C.
Assam Province was a province of British India, created in 1911 by the partition of the Eastern Bengal
and Assam Province. Its capital was in Shillong.
In 1874, the Assam region was separated from the Bengal Presidency, Sylhet was added to it and its
status was upgraded to a Chief Commissioner's Province, also known as the 'North-East Frontier'
non-regulation province. The capital was at Shillong.
Assamese, which had been replaced by Bengali as the official language in 1837, was reinstated
alongside Bengali. The new Commissionership included the four districts of Assam proper Nagaon,
Darrang, Sibsagar and Lakhimpur), The Lalung (Tiwa) Hills, Khasi-Jaintia Hills, Garo Hills, Naga Hills,
Goalpara, (Kamrup and Sylhet-Cachar of Bengal comprising about 54,100 sq miles. The people of
Sylhet, Goalpara, Kamrup and the Hills protested the inclusion in Assam.
Hence, option C is correct.
7. Ans. B.
• The dhamma of Ashoka was a code of conduct and a set of principles for living a life.
• It stated that everyone should follow the ahimsa principle, which is non-violence & non-injury
to all living beings.
Hence, option B is correct.
8. Ans. A.
The Constitution (Eighty-sixth Amendment) Act, 2002 inserted Article 21-A in the Constitution of
India to provide free and compulsory education of all children in the age group of six to fourteen
years as a Fundamental Right in such a manner as the State may, by law, determine.
Hence, option A is correct.
9. Ans. D.
The Overseas Citizenship of India (OCI) is an immigration status permitting a foreign citizen of Indian
origin to live and work in the Republic of India indefinitely. The OCI was introduced in response to
demands for dual citizenship by the Indian diaspora, particularly in developed countries. It was
introduced by The Citizenship (Amendment) Act, 2005 in August 2005. It was launched during the
Pravasi Bharatiya Divas convention held in Hyderabad in 2006.
Hence, option D is correct.
10. Ans. B.
Telegram was discontinued in 2015.
India's telegram service began in 1850, when the first telegram was sent from the eastern city of
Calcutta to Diamond Harbour, a southern suburb nearly 25 miles from the city centre.
Hence, option B is correct.
11. Ans. C.
The nine thrust areas in this Railway Budget 2015-16 are as follows:
• Indian Railways to become prime mover of economy once again
• Resource Mobilization for higher Investments

PAGE 1350
www.byjusexamprep.com

• Decongestion of heavy haul routes and speeding up of trains: emphasis on gauge conversion,
doubling, tripling and electrification
• Project delivery
• Passenger Amenities.
• Safety
• Transparency & System Improvement.
• Railways to continue to be the preferred mode of transport for the masses.
• Sustainability.
Hence, option C is correct.
12. Ans. C.
Citizenfour is a 2014 documentary film directed by Laura Poitras, concerning Edward Snowden and
the NSA spying scandal.
Hence, option C is correct.
13. Ans. B.
The Hedging is a financial technique that helps to reduce or mitigate the effects of measurable type
of risk from the future changes in the fair value of commodities, cash flows, securities, currencies,
assets and liabilities.
Hence, option B is correct.
14. Ans. D.
• Treasury Bill: A Treasury Bill is the Central Govt's short-term borrowing instrument. T-Bills are
issued at a discount to face value, and the holder receives the face value when they mature.
• Savings accounts & small time deposits: A savings account or small time deposit is a deposit
made by an individual with the intention of saving money & earning interest.
Hence, option D is correct.
15. Ans. C.
• Abhinavagupta was a scholar who practiced Kashmir Shaivism philosophy. He was a thinker.
• He wrote Tantraloka, which means "Light on Tantra." The teachings of the Kula and Trika
systems are presented.
Hence, option C is correct.
16. Ans. A.
Votive inscriptions (2nd century BC) describes about offerings given to super natural forces to get
their favour.
Hence, option A is correct.
17. Ans. C.
Hazara Rama Temple Hampi was built in the early 15th century by king Devaraya 1. He was the ruler
of Vijayanagara kingdom.
Hence, option C is correct.

PAGE 1351
www.byjusexamprep.com

18. Ans. A.
As Per Articles 344(1) and 351 of the Indian Constitution, the eighth schedule includes the recognition
of the following 22 languages:
1. Assamese
2. Bengali
3. Bodo
4. Dogri
5. Gujarati
6. Hindi
7. Kannada
8. Kashmiri
9. Konkani
10. Maithili
11. Malayalam
12. Manipuri
13. Marathi
14. Nepali
15. Odia
16. Punjabi
17. Sanskrit
18. Santali
19. Sindhi
20. Tamil
21. Telugu
22. Urdu
Hence, option A is correct.
19. Ans. B.
Apart from article 102, the Tenth Schedule to Constitution provides for disqualification of the
members on ground of defection. Defection refers to desertion of one’s party in favor of an opposing
one. As per the provisions of the Tenth Schedule, a member may be disqualified if:
• Voluntarily gives up the membership of his political party which gave him ticket to contest and
win
• Votes or abstains from voting in the House contrary to any direction issued by the political party
to which he belongs, unless such voting or abstention has been condoned by the political party
within fifteen days.

PAGE 1352
www.byjusexamprep.com

• A member elected as an independent candidate shall be disqualified if he joins any political


party after his election.
• However, a nominated member is allowed to join a political party provided he joins such
political party of his choices within a period of six months.
Hence, option B is correct.
20. Ans. A.
• "Protection against arrest and detention in certain cases," according to Article 22 of the Indian
Constitution.
• Nothing in Article 22 clauses (1) & (2) applies to a person detained under any law providing for
the preventive detention, which means he cannot claim bail on the grounds that he has been in
prison for more than twenty-four hours without a magistrate's order.
Hence, option A is correct.
21. Ans. A.
• Rashtriya Uchchattar Shiksha Abhiyan is a development scheme for higher education in India
launched by the Ministry of Human Resource Development. It was first introduced in the year
2013.
• Its goal was to provide strategic funding to higher education institutions across the country.
Hence, option A is correct.
22. Ans. B.
The target was to increase Girls’ enrollment from 76 % to 79 % by the year 2017. Hence, option B is
correct.
23. Ans. D.
The National Policy for Children, 2013 recognises that:
• a child is any person below the age of eighteen years
• childhood is an integral part of life with a value of its own
• children are not a homogenous group and their different needs need different responses,
especially the multi-dimensional vulnerabilities experienced by children in different
circumstances
• a long term, sustainable, multi-sectoral, integrated and inclusive approach is necessary for the
overall and harmonious development and protection of children
Hence, option D is correct.
24. Ans. C.
The second plan was to set India on the path of industrialisation. P.C. Mahalanobis was the moving
spirit behind the second five year plan. He gave the highest priority to strengthening the industrial
base of the economy.
Objectives:
(i) A sizeable increase in national income so as to raise the level of living.
(ii) Rapid industrialisation of the country with particular emphasis on the development of basic and
key industries.

PAGE 1353
www.byjusexamprep.com

(iii) A large expansion of employment opportunities by developing labour-intensive projects and


small scale industries.
(iv) Reduction in inequalities of income and distribution.
(v) To attain the annual growth rate of 5%.
Hence, option C is correct.
25. Ans. A.
In economics, a public good is a good that is both non-excludable and non-rivalrous in that individuals
cannot be effectively excluded from use and where use by one individual does not reduce availability
to others.
Public goods include knowledge, official statistics, national security, common language(s), flood
control systems, lighthouses, street lighting. Public goods that are available everywhere are
sometimes referred to as global public goods.
Hence, option A is correct.
26. Ans. C.
Bāburnāma is the name given to the memoirs of Ẓahīr-ud-Dīn Muhammad Bābur (1483–1530),
founder of the Mughal Empire and a great-great-great-grandson of Timur.
It is an autobiographical work, written in the Chagatai language, known to Babur as "Turki" (meaning
Turkic), the spoken language of the Andijan-Timurids. According to historian Stephen Frederic Dale,
Babur's prose is highly Persianized in its sentence structure, morphology, and vocabulary, and also
contains many phrases and smaller poems in Persian. During Emperor Akbar's reign, the work was
completely translated to Persian by a Mughal courtier, Abdul Rahīm, in AH 998 (1589–90).
Hence, option C is correct.
27. Ans. B.
Shamsa ud-din Iltutmish established the "Iqta‘ system" based on Mohammad Gori's ideas. It was very
close to the original form of Iqta' as its main function was only to collect taxes by Muqtis/Iqtedars in
India. They had no other right to the subjects apart from the taxes as long as taxes were paid. The
money was used to pay for the landowner's army, which could be called by the Sultan at any time,
making up for a relatively quick mobilisation and highly professional soldiers. A small part of the
money was to be given to the Sultan, but the percentage was usually insignificant compared to the
other expenses. Iqtas were given for exceptional military service or loyalty and were, unlike the
original, usually hereditary. Hence, option B is correct.
28. Ans. D.
The Vijayanagara Empire (also called Karnata Empire, and the Kingdom of Bisnegar by the
Portuguese) was based in the Deccan Plateau region in South India. It was established in 1336 by
Harihara I and his brother Bukka Raya I of Sangama Dynasty.
Hence, option D is correct.
29. Ans. B.
The Supreme Court in the case of Daniel Latifi v. Union of India held that reasonable and fair
provisions include provision for the future of the divorced wife (including maintenance) and it does

PAGE 1354
www.byjusexamprep.com

not confine itself to the iddat period only. The Constitutional validity of the Act was also upheld.
Hence, option B is correct.
30. Ans. D.
• According to Article 74, there shall be a Council of Ministers, led by the Prime Minister, to assist
as well as advise the President, who shall act in accordance with such advice in the exercise of
his functions.
• Article 78 requires the Prime Minister to communicate to the President all decisions of the
Council of Ministers (CoMs) relating to the administration of the union's affairs and legislative
proposals.
Hence, option D is correct.
31. Ans. A.
Indian judiciary has pointed out their view regarding death penalty by ruling out in Bacchan singh vs
state of Punjab[i] that the death penalty must be restricted to the “rarest of rare” cases.
Hence, option A is correct.
32. Ans. C.
The World Food Prize is an international award recognizing the achievements of individuals who have
advanced human development by improving the quality, quantity, or availability of food in the world.
Since 1987, the prize has been awarded annually to recognize contributions in any field involved in
the world food supply: food and agriculture science and technology, manufacturing, marketing,
nutrition, economics, poverty alleviation, political leadership, and the social sciences. The World
Food Prize Foundation is currently run by Kenneth M. Quinn, former U.S. Ambassador to Cambodia.
Hence, option C is correct.
33. Ans. C.
Leander Paes and Swiss Martina Hingis became the first team since 1969 to win three Grand Slam
mixed doubles titles in the same season at the US Open
Hence, option C is correct.
34. Ans. D.
All the mentioned statements are true, thus the correct response is option D. Hence, option D is
correct.
35. Ans. A.
An Agraharam or Agrahara was a grant of land and royal income from it, typically by a king or a noble
family, to religious purposes, particularly to Brahmins to maintain temples in that land or a pilgrimage
site and to sustain their families. Agraharams were also known as Chaturvedimangalams in ancient
times.
Hence, option A is correct.
36. Ans. C.
The Gupta coinage started with a remarkable series in Gold issued by Chandragupta I, the third ruler
of the dynasty.
Hence, option C is correct.

PAGE 1355
www.byjusexamprep.com

37. Ans. C.
The Human Development Report (HDR) is an annual milestone published by the Human
Development Report Office of the United Nations Development Programme (UNDP). As of 2013 the
last decade saw convergence in human development indicators (HDI) values globally, although
progress was uneven within and between regions.
Hence, option C is correct.
38. Ans. A.
CAG is not a multi-member body. It is a single person. The CAG of India is appointed by the President
of India following a recommendation by the Prime Minister. The CAG can be removed only on an
address from both house of parliament on the ground of proved misbehaviour or incapacity. The
CAG vacates the office on attaining the age of 65 years age even without completing the 6 years
term. The CAG is mentioned in the Constitution of India under Article 148 - 151.
Hence, option A is correct.
39. Ans. B.
Considered a proof of residence and not a proof of citizenship, Aadhaar does not itself grant any
rights to domicile in India. In June 2017, home Ministry clarified this.
Hence, option B is correct.
40. Ans. C.
• The sixth schedule makes provision for the creation of the District Councils and regional
councils. Each district / regional council is a body corporate that is empowered for the
administration of the area under its jurisdiction.
• They are named as District council of (name of district) and Regional Council of (name of region).
• These two bodies have perpetual succession and a common seal and shall by the said name sue
and be sued.
Hence, option C is correct.
41. Ans. B.
Nehwal created history in 2015 by becoming the first Indian to reach the final of World Badminton
Championships. Hence, option B is correct.
42. Ans. B.
The Erawan Shrine, formally the Thao Maha Phrom Shrine, is a shrine in Bangkok, Thailand, that
houses a statue of Phra Phrom, the Thai representation of the Hindu god of creation Lord Brahma.
Hence, option B is correct.
43. Ans. C.
One-way payment of money for which no money, good, or service is received in exchange.
Governments use such payments as means of income redistribution by giving out money under social
welfare programs such as social security, old age or disability pensions, student grants,
unemployment compensation, etc.
Hence, option C is correct.

PAGE 1356
www.byjusexamprep.com

44. Ans. A.
All the given statements are correct, thus the correct response is option A.
Hence, option A is correct.
45. Ans. A.
Misl generally refers to the sovereign states of the Sikh Confederacy, that rose during the 18th
century in the Punjab region of the Indian subcontinent after the collapse of the Mughal Empire.
Hence, option A is correct.
46. Ans. A.
The followers of Gorakhnath are called Jogis or Yogis.
Hence, option A is correct.
47. Ans. B.
Ordinances are laws that are promulgated by the President of India on the recommendation of the
Union Cabinet. They can only be issued when Parliament is not in session. They enable the Indian
government to take immediate legislative action.
Rajya Sabha is the permanent house and can’t be dissolved. 104th Constitutional Amendment Act,
2019, the Anglo-Indian reserved seats in the Parliament and State Legislatures of India were
abolished.
Hence, option B is correct.
48. Ans. A.
The Rajya Sabha (Council of States) is the upper house of the Parliament of India. Membership is
limited to 250 members, 12 of whom are nominated by the President of India. The remainder of the
body is elected by the state and territorial legislatures.
Article 312 provides for the creation of All India Services. Article 312 provides that an All India Service
can be created only if the Rajya Sabha declares, by resolution supported by not less than a two-thirds
majority, that it is necessary in the national interest.
Hence, option A is correct.
49. Ans. A.
A vote of no confidence means that a person in a position of responsibility is no longer deemed fit to
hold that position. A Vote of No Confidence can be introduced only in the Lok Sabha. If a majority of
the members of the house vote in favour of the motion, the motion is passed and the Government
is bound to vacate the office.
Vishwanath Pratap Singh (1990), H. D. Deve Gowda (1997), Atal Bihari Vajpayee (1999), and Morarji
Desai (1978) were defeated by the vote of no confidence.
Hence, option A is correct.
50. Ans. B.
Greece’s last-minute overtures to international creditors for financial aid were not enough to save
the country from becoming the first developed economy to default on a loan with the International
Monetary Fund.

PAGE 1357
www.byjusexamprep.com

The IMF confirmed that Greece had not made its scheduled 1.6 billion euro loan repayment to the
fund.
Hence, option B is correct.
51. Ans. C.
The 2015 Sri Lankan parliamentary election was held on 17 August 2015, ten months ahead of
schedule, to elect 225 members to Sri Lanka's 15th Parliament.
Hence, option C is correct.
52. Ans. B.
FIPIC was formed in November 2014, to strengthen India's relationship with the Pacific Island
Countries. The first FIPIC summit was held at the level of Heads of Government in November 2014 in
Suva, Fiji, followed by the FIPIC-II summit held in August 2015, in Jaipur, India.
Hence, option B is correct.
53. Ans. D.
• Pareto efficiency, also known as Pareto Optimality, is an economic state in which resources
cannot be reallocated to benefit one individual without harming at least one other.
• The Production Possibility Frontier depicts the maximum possible output that can be obtained
when goods & services are used efficiently.
Hence, option A is correct.
54. Ans. A.
The Līlāvatī is Indian mathematician Bhāskara II's treatise on mathematics, written in 1150. It is the
first volume of his main work, the Siddhānta Shiromani, alongside the Bijaganita, the Grahaganita
and the Golādhyāya.
Hence, option A is correct.
55. Ans. C.
The Nitisara of Kamandaka, also known as the Kamandakiya-Nitisara, was composed during the
period of Gupta Dynasty. It was based on Kautilya’s Arthashtra.
It is a collection of rules and specific ideas about political morality.
Hence, option C is correct.
56. Ans. B.
Prayag Prashasti was issued by Samudragupta and was composed by Harisena. It is written in very
simple and refined Sanskrit in Champu kavya style. It lists achievements of Samudragupta. This
Inscription is a eulogy of Samudragupta and mentions about the conquests of Samudragupta and
boundaries of the Gupta Empire.
Hence, option B is correct.
57. Ans. A.
The Writ of prohibition means to forbid or to stop and it is popularly known as 'Stay Order'. This writ
is issued when a lower court or a body tries to transgress the limits or powers vested in it. The writ
of prohibition is issued by any High Court or the Supreme Court to any inferior court, or quasi judicial

PAGE 1358
www.byjusexamprep.com

body prohibiting the latter from continuing the proceedings in a particular case, where it has no
jurisdiction to try. After the issue of this writ, proceedings in the lower court etc. come to a stop.
Hence, option A is correct.
58. Ans. A.
The Second Administrative Reforms Commission (ARC) was constituted on 31.08,2005, as a
Commission of Inquiry, under the Chairmanship of Shri Veerappa Moily for preparing a detailed
blueprint for revamping the public administrative system.
Hence, option A is correct.
59. Ans. D.
Lok Sabha Speaker is selected out of the MPs through the common consensus amongst the members.
The speaker is allowed to delegate his responsibilities in case of there is emergency making him/her
unavailable to discharge their duties.
Hence, option D is correct.
60. Ans. B.
The primary product of PM Narendra Modi’s Pradhan Mantri MUDRA Yojana will be providing
refinance for lending to micro businesses/units (medium and small entrepreneurs). The initial
products and schemes under this umbrella have already been created and the interventions have
been named ‘Shishu’, ‘Kishor’ and ‘Tarun’ to signify the stage of growth / development and funding
needs of the beneficiary micro unit / entrepreneur as also provide a reference point for the next
phase of graduation / growth for the entrepreneur to aspire for:
1. Shishu: covering loans up to Rs 50,000
2. Kishor: covering loans above Rs 50,000 and up to Rs. 5 lakh
3. Tarun: covering loans above Rs 5 lakh and up to Rs. 10 lakh
Hence, option B is correct.
61. Ans. C.
On 15 July 2014, the first day of the 6th BRICS summit held in Fortaleza, Brazil, the BRICS states signed
the Agreement on the New Development Bank, which makes provisions for the legal basis of the
bank. In a separate agreement, a reserve currency pool worth $100 bln was set up by BRICS nations.
Hence, option C is correct.
62. Ans. D.
The treaty establishing the Eurasian Economic Union was formally signed by three states which were
part of the former Soviet Union: Belarus, Kazakhstan, and Russia. Agreements to enlarge the EEU to
the other post-Soviet states of Armenia and Kyrgyzstan were signed on 9 October and 23 December
2014, respectively.
Hence, option D is correct.

PAGE 1359
www.byjusexamprep.com

63. Ans. C.
The Phillips curve is a single-equation econometric model, named after William Phillips, describing a
historical inverse relationship between rates of unemployment and corresponding rates of rises in
wages that result within an economy.

• According to the Philips curve, inflation and unemployment have a stable and inverse
relationship. It bears the name William Philip.
• It claims that with economic growth comes inflation, which should result in more jobs and lower
unemployment. However, this is only said to be true in the short run.
• Rising inflation will not reduce unemployment in the long run. Hence, option C is correct.
64. Ans. B.
• Fourth Plan (1969-74): It emphasised growth with stability & gradual attainment of self-
sufficiency. Raising the standard of living through programmes promoting equality and social
justice. To accelerate development by reducing fluctuations in agricultural output.
• Fifth Plan (1974-1979): Emphasis was placed on poverty alleviation & job creation.
Hence, option B is correct.
65. Ans. C.
The Hamzanama narrates the legendary exploits of Amir Hamza, an uncle of the Prophet
Muhammad, though most of the stories are extremely fanciful.
Most of the characters of the Hamzanama are fictitious. In the West the work is best known for the
enormous illustrated manuscript commissioned by the Mughal Emperor Akbar in about 1562.
Babur ws considered a foreigner by the Rajputs and thus was despised. No Rajput depiction was
commissioned to talk about the deeds of Babur. Hence, option C is correct.
66. Ans. B.
• Kailasanath temple is one of India's largest megalithic temples, located in Ellora, Maharashtra.
• It is said to have been built by Rashtrakuta king Krishna I.
• Shore Temple in Mamallapuram is a structural temple made of granite blocks.
Hence, option B is correct.

PAGE 1360
www.byjusexamprep.com

67. Ans. A.
• The Indus Valley Civilization was a Bronze Age civilization that existed in the northwestern
regions of South Asia from 3300 BCE to 1300 BCE, and in its mature form from 2600 BCE to 1900
BCE.
• Harappan Civilisation refers to the mature period of the Indus Valley Civilisation. Harappa has
now been relocated to Pakistan.
• Amalananda Ghosh was an Indian archaeologist who organised and directed archaeological
expeditions in the mid-1900s. He was the author and editor of numerous works on India's
ancient civilizations.
Hence, option A is correct.
68. Ans. D.
• In the year 1955, the Bandung Conference was held in Bandung, Indonesia, with the
participation of Asian and African states. The Conference was attended by 29 countries.
• The Conference was a significant step forward for the Non-Aligned Movement because it aimed
to promote Afro-Asian economic & cultural cooperation while also opposing colonialism by any
nation.
Hence, option D is correct.
69. Ans. A.
The below mentioned are the constitutional bodies which are used to govern india:
Election comission (u/a 324)
Union public service comission ( Article 315 to 323)
State public service comission (Article 315 to 323)
Finance comission (u/a 280)
National comission for scheduled castes (u/a 338)
National comission for scheduled tribes (u/a 388-A)
Special officer for linguistic minorities (u/a 350-B)
Comptroller and Auditor general of india (u/a 148)
Attorney general of india (u/a 76)
Advocate general of states (u/a 165)
Hence, option A is correct.
70. Ans. A.
Hind Swaraj or Indian Home Rule is a book written by Mohandas K. Gandhi in 1909. In it he expresses
his views on Swaraj, modern civilization, mechanisation etc
This book was conceptualized by MK Gandhi while he was staying in South Africa and was fighting
for the rights of the people suppressed under the imperial British rule.
Hence, option A is correct.

PAGE 1361
www.byjusexamprep.com

71. Ans. A.
Height of hill stations:
Mussoorie: 2006 m
Shimla: 2205 m
Ooty: 2240 m
Darjeeling: 2042 m
From the highest to the lowest,
Ooty > Shimla > Darjeeling > Mussoorie
Hence, option A is correct.
72. Ans. C.
An echo is the sound you hear when you make a noise and the sound wave reflects off a distant
object. The sound wave travels in a straight line. When the wave hits a solid wall, it is reflected.
d = v × t = 340 m/s × 0.3 s
d = 102 m
d is the distance the sound wave traveled back and forth. The distance of the wall from the person
will be half of this.
Required distance = d/2 = 102/2 = 51 m
Hence, option C is correct.
73. Ans. A.
• Natural rubber is an elastomer, which is a type of elastic hydrocarbon polymer.
• The monomer in rubber is isoprene, a carbon compound with two carbon-carbon double bonds.
• It is an elastic substance derived from the latex sap of Hevea&Ficus trees.
Hence, option A is correct.
74. Ans. D.
Dakshin Gangotri was the first scientific base station of India situated in Antarctica, part of the Indian
Antarctic Programme. It is located at a distance of 2,500 kilometres from the South Pole.
It is currently being used as a supply base and transit camp.
Maitri is India's second permanent research station in Antarctica as part of the Indian Antarctic
Programme. The name was suggested by the then PM Mrs Indira Gandhi.
Hence, option D is correct.
75. Ans. D.
Chemical industry is one of the oldest industries in India, which contributes significantly towards
industrial and economic growth of the nation. Since this industry has numerous forward and
backward linkages, it is called the backbone of the industrial and agricultural development of the
country and provides building blocks for many downstream industries.

PAGE 1362
www.byjusexamprep.com

The Chemical Industry of India ranks 6th in world and 3rd in Asia after Japan and China. In terms of
global shipments of chemicals, India ranks 10th in the world.
The most important sectors of India’s Chemical Industry are Alkali Chemicals, Inorganic Chemicals,
Organic Chemicals, Pesticides and Dyes & Dyestuffs, with their respective production quantity in
same order.
The textile industry is the major consumer of dyestuffs and about 70% of the total production is
consumed by this sector.
Hence, option D is correct.
76. Ans. D.
Atoms of a particular element must have the same number of protons but can have different
numbers of neutrons. When an element has different variants that, while all having the same number
of protons, have differing numbers of neutrons, these variants are called isotopes.
Hence, option D is correct.
77. Ans. C.
The three most stable isotopes of hydrogen: protium (A = 1), deuterium (A = 2), and tritium (A = 3).
Protium, the most common isotope of hydrogen, consists of one proton and one electron.
Hence, option C is correct.
78. Ans. C.
Since most bar magnets are actually made up of billions of very small pieces with their magnetic
dipoles aligned in the N-S direction of the magnet, then the result would typically be two smaller and
somewhat weaker dipole magnets with the magnetic field running in the same general direction as
the original.
Hence, option C is correct.
79. Ans. A.
Acid rain is caused by a chemical reaction that begins when compounds like sulfur dioxide and
nitrogen oxides are released into the air. These substances can rise very high into the atmosphere,
where they mix and react with water, oxygen, and other chemicals to form more acidic pollutants,
known as acid rain.
Acids have a corrosive effect on limestone or marble buildings or sculptures. It is well established
that either wet or dry deposition of sulfur dioxide significantly increases the rate of corrosion on
limestone, sandstone, and marble.
Eutrophication, or hypertrophication, is when a body of water becomes overly enriched with
minerals and nutrients that induce excessive growth of plants and algae. This process may result in
oxygen depletion of the water body.
Hence, option A is correct.
80. Ans. A.
• A cyclone is a broad term for a weather system in which winds rotate inward to a low-pressure
area.

PAGE 1363
www.byjusexamprep.com

• The circulation pattern of large weather systems is counterclockwise in the Northern


Hemisphere & clockwise in the Southern Hemisphere.
Hence, option A is correct.
81. Ans. D.
The Coriolis effect causes a deflection in global wind patterns. The anticlockwise rotation of the Earth
deflects winds to the right in the northern hemisphere and to the left in the southern hemisphere.
When the earth rotates one circle in 24 hours, the effect of it can be seen in the sea water and the
atmospheric air which tends to move towards Right in the northern hemisphere and towards Left in
the southern hemisphere.
Hence, option D is correct.
82. Ans. A.
In India, it flows for 916 km. The principal tributaries of the river joining from right are the Lohit, the
Dibang, the Subansiri, the Jiabharali, the Dhansiri, the Manas, the Torsa, the Sankosh and the Teesta
whereas the Burhidihing, the Desang, the Dikhow, the Dhansiri and the Kopili joins it from left.
Hence, option A is correct.
83. Ans. D.
An ice sheet is a mass of glacial land ice extending more than 50,000 square kilometers (20,000
square miles). The two ice sheets on Earth today cover most of Greenland and Antarctica.
Melting of these ice sheets is releasing a lot of water into the sea which will result in the rise of sea
levels in the long run.
Hence, option D is correct.
84. Ans. A.
Burning of paper is combustion. It represents a chemical reaction whereby the carbon compounds
in the paper are oxidized into different chemicals like carbon dioxide and water vapor. Other
examples of chemical change are iron rusting (iron oxide forms), gasoline burning (water vapor and
carbon dioxide form).
Hence, option A is correct.
85. Ans. A.
Amino acid-derived hormones also include thyroxine (produced by the thryoid gland) and melatonin
(produced by the pineal gland). Peptide hormones consist of a polypeptide chain; they include
molecules such as oxytocin (short polypeptide chain) or growth hormones (proteins ).
Hence, option A is correct.
86. Ans. C.
When ice melts, it remains at 0 °C (32 °F), and the liquid water that is formed with the latent heat of
fusion is also at 0 °C.
Latent heat of melting of ice = 333.6 J/g
1 g of ice requires 333.6 J heat to melt. So, for 100 g of ice,

PAGE 1364
www.byjusexamprep.com

Amount of heat required = 100 g × 333.6 J/g = 33360 J


Hence, option C is correct.
87. Ans. D.
P = 1/f
f = 1/P = 1/1.25 = 0.8 m or 80 cm
Convex lenses, or converging lenses, have positive values, as they magnify objects. A diverging lens
or concave lens, on the other hand, have a negative dioptre, since it makes objects appear smaller
than they are.
Hence, option D is correct.
88. Ans. D.
• Norman Borlaug was an agronomist from the United States.
• He is often referred to as the "Father of the Green Revolution."
• In 1970, he was awarded the Nobel Peace Prize for his contributions to the "green revolution."
• Norman Borlaug established the World Food Prize in the year 1986.
• In addition, he was awarded the Presidential Medal of Freedom and the Congressional Gold
Medal.
Hence, option D is correct.
89. Ans. D.
Protoplasm is the living content of a cell surrounded by a plasma membrane. Protoplasm is
composed of cytoplasm, nucleoplasm and other organelles. Protoplasm contains the cell nucleus and
inside the nucleus, is the nucleoplasm.
Hence, option D is correct.
90. Ans. A.
• Glucose is a sugar with the molecular formula C6H12O6.
• The suffix "-ose" denotes carbohydrate.
• Glucose is made during photosynthesis from water and carbon dioxide, using energy from
sunlight.
Hence, option A is correct.
91. Ans. A.
• Reunion Island is located in the Indian Ocean but is a French region.
• This island sits on top of a hotspot in the Earth's crust.
• The Piton de la Fournaise, a shield volcano on Reunion Island, is sometimes referred to as the
Hawaiian volcanoes' sister.
• The Andaman and Nicobar Islands are one of India's seven Union Territories, with approximately
572 islands. Port Blair is its capital.
Hence, option A is correct.

PAGE 1365
www.byjusexamprep.com

92. Ans. C.

Hence, option C is correct.


93. Ans. D.
• Mahatma Gandhi Marine National Park is a national park of India in Wandoor on the Andaman
Islands.
• It is situated 29 km from Port Blair, the Park is located on the South Western coast of South
Andaman, in the Bay of Bengal.
Hence, option A is correct.
94. Ans. B.
Most of the deposits of iron ore are located in the Archaen rocks of Jharkhand, Orissa, Madhya
Pradesh, Chhattisgarh, Karnataka and Tamil Nadu.
Hence, option B is correct.
95. Ans. C.
Vitamin B12 is also referred to as cobalamin, because it contains a metal ion (cobalt). This makes it
the largest and most complex vitamin of all. The vitamin can only be synthesized by bacteria and
therefore is mainly present in animal products.
Hence, option C is correct.
96. Ans. D.
Vitamin K, a fat-soluble vitamin, is important for blood clotting.
The ability to bind calcium ions (Ca2+) is required for the activation of the seven vitamin K-dependent
blood clotting ('coagulation') factors (e.g., prothrombin), or proteins, in the series of events that stop
bleeding through clot formation ('coagulation cascade').
Hence, option D is correct.
97. Ans. B.
Sound waves having frequency below 20 Hz are known as infrasonic waves and those above 20 KHz
are called ultrasonic.
Hence, option B is correct.

PAGE 1366
www.byjusexamprep.com

98. Ans. D.
Sound waves travel faster in solids than in liquids, and faster in liquids than in gasses. While the
density of a medium also affects the speed of sound, the elastic properties have a greater influence
on the wave speed. The density of a medium is the second factor that affects the speed of sound.
Hence, option D is correct.
99. Ans. D.
Genetically modified organisms (GMOs) can be defined as organisms (i.e. plants, animals or
microorganisms) in which the genetic material (DNA) has been altered in a way that does not occur
naturally by mating and/or natural recombination.
• Genetically modified crops are those in which the DNA of a plant has been altered using genetic
engineering techniques.
• It is commonly used to make crops resistant to pests & diseases.
• It is also used to increase the nutritional value of a crop.
• Genes from the soil bacterium Bacillus thuringiensis are found in insect-resistant crops.
Hence, option D is correct.
100. Ans. C.
Artificial insemination (AI) is the introduction of sperm into a female's uterus for the purpose of
achieving pregnancy. It is used as a fertility treatment for humans, and is a common practice in animal
breeding.
Hence, option C is correct.
101. Ans. B.
Planets in our Solar system size comparison. Largest to smallest are pictured left to right, top to
bottom: Jupiter, Saturn, Uranus, Neptune, Earth, Venus, Mars, Mercury.
Hence, option B is correct.
102. Ans. C.
Karst topography is a landscape formed from the dissolution of soluble rocks such as limestone,
dolomite, and gypsum. It is characterized by underground drainage systems with sinkholes and
caves.
Caves in karst areas often have stalactites (icicle-like masses of chemical limestone) that hang from
cave ceilings and stout stalagmites protruding from the cave floor.
Hence, option C is correct.
103. Ans. C.

Hence, option C is correct.

PAGE 1367
www.byjusexamprep.com

104. Ans. B.
In the year 2002, the Government of India had entered into a Memorandum of Understanding (MoU)
with China for five years upon provision of Hydrological information on Yaluzangbu /Brahmaputra
River during flood season by China to India
During the visit of Hon’ble Prime Minister of China to India in December, 2010, a MoU upon provision
of hydrological information of Sutlej/Langqen Zangbo River during flood season by China to India was
signed with China on 16th December, 2010 with a validity of five years.
Hence, option B is correct.
105. Ans. A.
Phytosterols (referred to as plant sterol and stanol esters) are a group of naturally occurring
compounds found in plant cell membranes. Because phytosterols are structurally similar to the
body's cholesterol, when they are consumed they compete with cholesterol for absorption in the
digestive system.
Hence, option A is correct.
106. Ans. C.
German Silver is an alloy of copper, zinc and nickel, sometimes also containing lead and tin. It was
originally named for its silver-white colour, but the term 'silver' is now prohibited for alloys not
containing that metal.
Hence, option C is correct.
107. Ans. C.
Two physical systems are in thermal equilibrium if no heat flows between them when they are
connected by a path permeable to heat. Thermal equilibrium obeys the zeroth law of
thermodynamics.
Hence, option C is correct.
108. Ans. B.
The dimension of power is energy divided by time. The SI unit of power is the watt (W), which is
equal to one joule per second. Other units of power include ergs per second (erg/s), horsepower
(hp), metric horsepower (Pferdestärke (PS) or cheval vapeur (CV)), and foot-pounds per minute.
Hence, option B is correct.
109. Ans. D.
• Dengue is caused by the Dengue virus (DENV), a flavivirus spread by mosquitos. DENV is a
positive-strand RNA virus with a single strand of RNA.
• Bone marrow is the centre for the production of blood cells, and Dengue fever causes bone
marrow suppression, resulting in a deficiency of blood cells, which leads to a low platelet count.
o a low platelet count
Hence, option D is correct.

PAGE 1368
www.byjusexamprep.com

110. Ans. B.
• Germplasm is a living tissue that is kept for the purpose of breeding animals & plants.
• Germplasm conservation has proven to be the most effective method for preserving the genetic
traits of endangered as well as commercially valuable species.
Hence, option B is correct.
111. Ans. D.
Methyl isocyanate is an intermediate chemical in the production of carbamate pesticides (such as
carbaryl, carbofuran, methomyl, and aldicarb). It has also been used in the production of rubbers
and adhesives. As a highly toxic and irritating material, it is extremely hazardous to human health.
Hence, option D is correct.
112. Ans. D.
Pressure cooker handles are made up Bakelite; a thermosetting plastic. Bakelite is used because it
has heat-resistant properties and is an electrical insulator.
Hence, option D is correct.
113. Ans. C.
The climate is known for warm to hot, dry summers and mild to cool, wet winters. Winter
temperatures are usually between 30 and 65 degrees. Summer months all average above 50 degrees.
Like in the tropical rain forest climate, temperatures remain high all year in the monsoon climate. As
shown in the climograph for Mangalore, India (Figure 9.4), the average annual temperature is 27.05
oC (80.7oF) but only has an annual temperature range of 3.6 oC (2oF).
Hence, option C is correct.
114. Ans. C.
Laterite is a soil and rock type rich in iron and aluminium and is commonly considered to have formed
in hot and wet tropical areas. Nearly all laterites are of rusty-red coloration, because of high iron
oxide content.
Laterite soils in India are found in the Eastern Ghat of Orissa, the Southern parts of Western Ghat,
Malabar Coastal plains and Ratnagiri of Maharashtra and some part of Andhra Pradesh, Tamil Nadu,
Karnataka, Meghalaya, western part of West Bengal.
They are good for oil palm, tea, coffee and cashew cultivation.
Hence, option C is correct.
115. Ans. D.
The Pir Panjal Range is a group of mountains in the Inner Himalayan region, running from east-
southeast (ESE) to west-northwest (WNW) across the Indian states of Himachal Pradesh and Jammu
and Kashmir where the average elevation varies from 1,400 m (4,600 ft) to 4,100 m (13,500 ft).
It is considered a part of the Lesser Himalayas.
Hence, option D is correct.

PAGE 1369
www.byjusexamprep.com

116. Ans. A.
The area shown in the map is the part of the Deccan Plateau. Irrigation is uneven in this area because
the rivers are rain-fed. Nevertheless, the volcanic soil of the north lends itself to cotton cultivation.
Other cash crops include millet, oil-seeds, and wheat, and there are tea and coffee plantations in the
south.
Hence, option A is correct.
117. Ans. D.
• We sprinkle water on the roof or open ground in hot summers.
• This water requires latent heat of vaporisation to change from liquid state to vapor state.
• It evaporates or vaporises by taking the large amount of latent heat from the roof or ground
and surrounding air. Hence, the roof or ground becomes cool after losing its heat to water.
Hence, option D is correct.
118. Ans. C.
The rate of change of momentum of an object is directly proportional to the resultant force applied
and is in the direction of the resultant force.
Hence,the resultant force is equal to the rate of change of momentum.
Hence, option C is correct.
119. Ans. B.
The following list can be used as a cumulative information for vitamins and related diseases.

Hence, option B is correct.


120. Ans. A.
HDL is termed as good cholesterol.
HDL cholesterol protects against heart disease by taking the "bad" cholesterol out of your blood and
keeping it from building up in your arteries. A statin can slightly increase your HDL, as can exercise.
Hence, option A is correct.

PAGE 1370
www.byjusexamprep.com

Elementary Mathematics
1. Ans. C.
Let diameter be “D” m

AB = BC = CD =

Area of shaded region

Cost

2. Ans. D.
1.

But c is a real positive number and b is real negative number, hence it is incorrect

2.

PAGE 1371
www.byjusexamprep.com

as c is real positive number and a<b is getting divided by a real positive number, hence will remain

smaller than

3.

as b is a negative number, hence its reciprocal will remain negative. Therefore

so option D. is correct.
3. Ans. C.
let 243 = a and 647 = b

4. Ans. C.

Multiplying and dividing by 2 on LHS

PAGE 1372
www.byjusexamprep.com

5. Ans. B.

6. Ans. C.

terms -

Median =

HM =

Hence “c” is incorrect.


7. Ans. C.

median =

PAGE 1373
www.byjusexamprep.com

GM =

Hence GM = Median
8. Ans. B.

1.

Hence median class is 7-8 (Highlighted)

median=

median =

=8
Hence 1. is incorrect
2. Children in age group 6-9=28

percentage =

2 is correct
3. Modal age =8 (max freq = 12)
3 is also correct.
9. Ans. A.
If we subtract a number from mean or median, their value gets reduced by the same amount.
Hence new Mean = 9-1 =8
new Median = 8-1 = 7
10. Ans. B.
Let total valid votes be x

PAGE 1374
www.byjusexamprep.com

Acc. to question

11. Ans. A.
As origin lies in the shaded area, hence we shall use the origin coordinates in the given options.
O(0,0)

True

true.
Hence option A. is correct answer.
12. Ans. D.

x + z = 180
z = 120
x = 40 + y
60 = 40 + y
y = 20
13. Ans. C.

1. as ABC is an equilateral triangle

PAGE 1375
www.byjusexamprep.com

AB=BC=AC …………………………………..1
and AZ=BX=CY …………………………...2
subtracting 2 from 1
AB-AZ=BC-CX=AC-CY
BZ=CX=AY…………………………………….3
In triangle BZX and triangle CXY

BX=CY (Given)
BX=CY (From 3)
thus triangle BZX and triangle CXY are congruent.
=>ZX=XY
similarly, XY=YZ, Thus
ZX=XY=YZ
XYZ is an equilateral triangle
2. All equilateral triangles are similar to each other. hence 2 is also correct.
14. Ans. B.
Let
and
As per exterior angle sum property

In triangle CDB,

y+180-2x+96 =180(Linear pair)


96-x+180-2x+96=180 (from 1)
192=3x
x = 64 =
15. Ans. C.

Adding above 2 equations

PAGE 1376
www.byjusexamprep.com

16. Ans. D.

…………………………………………..1

on comparing, we get

from 1
x = 9(x – 8)
x = 9x – 72
72 = 8x
x=9
y=1
xy = 9
17. Ans. B.
Ar A= Ar C =

Ar B =

Ar A= Ar C =Ar B

PAGE 1377
www.byjusexamprep.com

18. Ans. C.

As D and E are mid points of BC and AD respectively, hence AD and BE are medians.
As median divides a triangle into two parts of equal area.
1
Area ΔBED = Area ΔAEB = × Area Δ ABD
2
1
→ Area ΔBED = × Area ΔABD ……………………….. 1
2
1
Also, Area ΔABD = Area ΔACD = × Area ΔABC
2
1
and Area ΔABD = × Area ΔABC ………………..2
2
from 1 and 2,
1
Area Δ BED = Area ΔABD………3
2
1
= ×
2

→ 4×Area Δ BED =Area Δ ABC


Part – 2,
1
from eq.(3), Area Δ BED = Area Δ ABD
2
and from eq.(2) Area Δ ABD = Area Δ ACD
1
→ Area Δ BED = Area ΔACD
2
→ The area of triangle ADC is twice the area of triangle BED

19. Ans. B.

PAGE 1378
www.byjusexamprep.com

req dist =

20. Ans. A.

Let ∠1=x
∠1 + ∠2 = 1800 (Linear Pair)
→ ∠2 = 180 – x
Also ∠1 = ∠3 = x (Vertically Opposite Angles)
similarly, ∠2 = ∠4 = 180 – x (Vertically Opposite Angles)
as l, m, n and o are parallel, hence as per corresponding angles rule,
∠1 = ∠5 = ∠9 = ∠13 = x
∠2 = ∠6 = ∠10 = ∠14 = 180 – x
∠3 = ∠7 = ∠11 = ∠15 = x
∠4 = ∠8 = ∠12 = ∠16 = 180 – x
Thus, all angles are formed with 2 distinct values only (x and 180-x)

21. Ans. D.

PAGE 1379
www.byjusexamprep.com

As

hence,

→ XY ll QR (Converse of Basic Proportionality Theorem.)


In Triangle PYX and triangle PRQ.

Hence these triangles are similar with AAA similarity.


As triangle PYX and triangle PRQ similar, hence

22. Ans. B.

1. no. of sides =

PAGE 1380
www.byjusexamprep.com

as number of sides can only be a natural number, hence it cannot be a regular polygon
2. n ≥ 5

as n =

as n ≥ 5, hence

≤ 72

Hence point 2 is correct


23. Ans. D.
1. Sum of all ext. angles of a polygon is 360. Hence this statement is wrong.

2.

Hence neither 1 nor 2 is correct.


24. Ans. C.
Let R be the radius of bigger ring and r be the radius of smaller ring.
As the smaller ring when allowed to roll over the circumference of the bigger ring takes three full
revolutions of itself, hence the circumference of larger ring is three times the circumference of
smaller ring.

Area of path =

n=8
25. Ans. C.

on solving , we get

PAGE 1381
www.byjusexamprep.com

on comparing, we get z = 3
26. Ans. B.
Percentage of personal care

= 16.5%
27. Ans. B.

Last meeting = 2p.m. – 3h 15 min


= 10:45
Information received at: 10:45+45
= 11:30p.m.
28. Ans. B.
Let us take m=1 and n=2
we arrive at following

1.

2.

3.

PAGE 1382
www.byjusexamprep.com

Hence neither 1 nor 3 give an integer, as per options, only 2 will be correct.
29. Ans. C.

As OR  (OQ-OS) = OS (OP-OR)
OR.OP-OR.OS=OS.OP-OS.OR
OR.OP=OS.OP
OR=OS
Hence point 1 is correct.
2.
x + x + y + y = 180
x + y = 90

Pt 2 is also correct
30. Ans. D.

All the three given statements are properties of an isosceles triangle, hence they are correct.

31. Ans. A.

PAGE 1383
www.byjusexamprep.com

= 135
2. m x n =1 x 1
mn = 1

product of roots =

3.

32. Ans. C.

33. Ans. B.

PAGE 1384
www.byjusexamprep.com

as ≥0

≥ 0,

or
,

≤ 0,

=>

34. Ans. D.
if

then, in any triangle

35. Ans. B.

but as per question hence is the only solution

36. Ans. C.

PAGE 1385
www.byjusexamprep.com

tan 45 =

tan 30 =

37. Ans. B.
Let θ = 45
=1+1=2

38. Ans. C.

PAGE 1386
www.byjusexamprep.com

1.

let us rationalize the LHS inside the square root

Hence 1 is an identity

2.

Let’s multiply and divide LHS by within the square root

Hence both are identities


39. Ans. D.

squaring both sides

PAGE 1387
www.byjusexamprep.com

squaring both sides

adding (1) and (2)

putting in given options, we get

A. =4

B. =2

C. = 0.5
D. =1
40. Ans. C.

Ar ABC = Ar ABD + Ar BCD

diagonal = x side

41. Ans. C.

PAGE 1388
www.byjusexamprep.com

In Triangle CPO,
CO2 = CP2+PO2
CO2 = 32+42
CO2 = 9+16

CO =
CO = 5

SinΘ =

42. Ans. C.
As area of two similar triangles is proportional to the square of their corresponding sides, hence

43. Ans. D.

area of enclosed figure = area of sq- area of circle

PAGE 1389
www.byjusexamprep.com

44. Ans. C.

Height = =80

Area = 0.5 x 300 x 80


=12000 sq units
45. Ans. D.
Distance=4000m
diameter =7/11
→ radius = 7/22

rounds =

Radius is given 7/11 in question.


- Diameter is given 7/11
=2000

46. Ans. C.

PAGE 1390
www.byjusexamprep.com

1. ΔABC ~ ΔDEF (By AAA similarity)

Also
ΔABL ~ ΔDEM (By SAS similarity)
2. As AC≠DF, hence Triangle ALC cannot be congruent to triangle DMF
47. Ans. B.
Exterior angle = 180-140
=400

No. of sides =

= = 9 = number of vertices

48. Ans. B.
Area of rhombus formed by joining mid points of a rectangle is half the area of rectangle,
Area Rectangle = 2 x Area Rhombus
=2 x 2
=4 sq. units
49. Ans. C.

As ED ll BC

PAGE 1391
www.byjusexamprep.com

ΔADE ~ ΔABC

50. Ans. C.
A’P’B’ and C’Q’D’ are not even a triangle in given figure. -
As per question they are not triangles in the circle. Triangles A’P’B’ and C’Q’D’ are triangles formed
with lines whose measurements are equal to the measurements of parts of 2 intersecting cords (AB
and CD) of a circle. Points A’, P’, B’, C’, Q’ and D’ are shown in figure for better understanding of
lengths of the triangles.

AP x BP=CP x DP …………………………….(1)
A’P’ X B’P’=C’P’ x D’P’ …………………………….(2)
A’P’ X B’P’=C’Q’ x D’Q’ …………………………….(3)
1. Let us assume the triangles are similar.

BUT FROM (3),

Hence these triangles are not similar


2. As they are not similar, hence they cannot be congruent
3. Area Δ A’P’B’ = 0.5xP’Q’xA’P’
= 0.5 x C’Q’ x Q’D’
= Area Δ C’Q’D’

PAGE 1392
www.byjusexamprep.com

Hence 3 is right.
From the available options, 3 is not present in option C. only, hence C. is right answer.
51. Ans. C.

In ΔTSR and ΔQPR

ΔTSR is congruent to ΔQPR


⇒ PQ = TS = y
52. Ans. C.

53. Ans. C.
Let original salary = 100
New salary = 100 + 10% of 100
= 110

Percentage of salary he did not receive =

= (100/11) %
54. Ans. A.
Let speed of B=Sb and Speed of A = Sa
Let Distance travelled by B = Db
and Distance travelled by A = Da

PAGE 1393
www.byjusexamprep.com

Let the race finishes in time = t


As A beats B by 100 m or 10 seconds.

Dividing (1) by (2), we get

B falls after =

Sb till 450m = 10 m/s


Sb after 450m = 5 m/s

→ B will take 155 sec to finish the race,


Hence difference = 155-90 = 65 sec
55. Ans. A.
A = 60 min
B = 75 min
A + B – C = 50 min
Let Total work = LCM of (60,75,50)
= 300 unit

Speed of A = U/min

PAGE 1394
www.byjusexamprep.com

Speed of B = U/min

Speed of A + B – C = U/min

⇒5+4–C=6
C = 3U/min
Total = 300

Time taken by C =

56. Ans. C.
Let price of 1cm = Rs1
Price of 100 cm cloth = Rs 100
Actual length bought with Rs 100 = 110cm
Actual length sold for Rs 100 = 90cm
Cloth saved = 110-90=20cm
S.P. of 90cm cloth = Rs 100

S.P. of 20cm cloth

= Profit

Profit % =

57. Ans. A.
Let Hindi-A
English-B
French-C
and n(A Ո B Ո C) = k
n(A) = 70
n(B) = 60

PAGE 1395
www.byjusexamprep.com

n(C) = 30
n(A Ո B) = 30
n(B Ո C) = x
n(A Ո C) = 20
n(A U B U C) = 100
n(A U B U C) = n(A) + n(B) + n(C) – n(A Ո B) – n(B Ո C) – n(A Ո C) + n(A Ո B Ո C)
100 = 70 + 60 + 30 – 30 – x – 20 + k
k = x – 10
As k ≥ 0, always
⇒ x ≥ 10
⇒ Option (a)
58. Ans. A.
Let time = t
For minute hand, 1 min = 60

For hour hand, 1 min =

as both interchange positions, hence net angle covered = 3600

59. Ans. B.
Let ages be x, y, and z respectively
x+y = 4……………………………………1
y+z = 6……………………………………2
x+z = 8……………………………………3
Adding above three equations
2(x + y + z) = 18
x+y+z=9
using (1)
4+z=9

PAGE 1396
www.byjusexamprep.com

z=5
Using z = 5 in (2)
y+5=6
y=1
using y = 1 in (1)
x+1=4
x=3
Difference between eldest and youngest = 5 – 1
= 4 years
60. Ans. D.
Let a = 1, b = 2 and c = 3
1.

True
2.

True

4.

from point 3, 1 divide 2, hence it is also true


61. Ans. C.

For no solution

62. Ans. D.
Let speed = x Km/h

PAGE 1397
www.byjusexamprep.com

new speed = (x+10) Km/h

63. Ans. C.
and

Putting point P,Q and R in above 2 equations


P (5,-1)

True

True
Q (3,-2)

True

True
R (1,1)

PAGE 1398
www.byjusexamprep.com

True

False
Hence P and Q lie in the solution
64. Ans. D.

for above eq. to be always true


1. , which is always true
2.
Using quadratic formula

65. Ans. D.
1. For any positive real no. m, cos x =2m+1
cos x>2, which is not possible. Hence 1. is wrong
2. Let m=1, n=3
mn ≥ m+n
1.3 ≥ 1+3
3≥4
Which is not true, hence 2 is also false
66. Ans. D.
as (x+1) is a factor, f(x)=0 for x=-1

PAGE 1399
www.byjusexamprep.com

67. Ans. B.

Only statement 2. is wrong. For e.g., 2 is a real number but not rational number.
Natural numbers starts from 1 and proceed as 1, 2, 3, ….. All of these are real numbers as well hence
statement 1 is correct.
Integers include numbers such as 1, 2, 3, ….. and also -1, -2, -3, ….. All of these are real numbers as
well hence statement 3 is correct.

A rational number is a number in the form of , where p and q are integers and q is not equal to

zero. As p and q both are real number, hence a rational number is also real number, hence statement
4 is correct.
68. Ans. D.
Petrol consumption for first 10km= 20mL/Km
Petrol consumed = 10 x 20 = 200mL
Petrol consumption for second 30km= 40mL/Km
Petrol consumed = 30 x 40 = 1200mL
Petrol consumption for last 10km= 20mL/Km
Petrol consumed = 10 x 20 = 200mL
Total petrol consumed = 200+1200+200
=1600mL or 1.6L
69. Ans. B.

Finding the factors of 3 in all terms

Hence m can be max 7


70. Ans. C.
Let roots be k, k2

PAGE 1400
www.byjusexamprep.com

As

Cubing both sides

71. Ans. B.

AO = 2
OC = 2
CD = 2
As OP is perpendicular to CD, hence OP divides CD into 2 equal parts,
⇒ CB = BD = 1
In Triangle OPC,
OP2 + CP2 = OC2
OP2 = 4-1

OP = 3

AP = 2 + 3

72. Ans. B.
5 o’c = 8sec

PAGE 1401
www.byjusexamprep.com

73. Ans. D.
S=1, 2, 3, 4, 5…..14
as max product can be 13 x 14 = 182, hence we make all possible combinations for less than 182

Hence there are 4 possible combinations are (8,2),(2,8),(13,7),(7,13)


Hence ans is (d)
74. Ans. A.
Let incomes be 9x and 7x respectively
Let expenses be 4y and 3y respectively
Acc to question

Difference in annual income =

= 2 × 2000 = 4000
75. Ans. D.

PAGE 1402
www.byjusexamprep.com

1. and is not always true as it depends upon the corresponding


angles as well.
2. for a triangle = Hence

Hence B. is right
76. Ans. C.

for SI,

77. Ans. C.
Let average speed = xKm/h

acc to question

PAGE 1403
www.byjusexamprep.com

78. Ans. C.

using identity sinA=cos(90-A) and cosA=sin(90-A)

79. Ans. C.
Let speed of train = x Km/h
D=225m =0.225Km
relative speed = x+5 Km/h

on solving this, we get x= 85Km/h


80. Ans. C.
Let no be 10y+x
Acc. to ques

adding above 2

81. Ans. D.

PAGE 1404
www.byjusexamprep.com

19932 is divisible by 11, hence given no. is divisible by 11


82. Ans. A.
let

Putting these values in given equations

Hence 1. is right

From above 1. we have

Hence 2. is wrong
A is right answer
83. Ans. A.
1.

Hence 1 is right

PAGE 1405
www.byjusexamprep.com

2.

Hence 2 is wrong
A Is our answer
84. Ans. A.
Putting

Also, it is given that

Hence

Hence it has exactly 2 values for

For part 2,

For the equation to have real roots, D

Hence m cannot take any integer value less than

So only option 1 is correct


85. Ans. A.

PAGE 1406
www.byjusexamprep.com

As per question

86. Ans. A.

Putting a = 3+b

Solving for b, we get

As b > 0, b=2

a+b=7
87. Ans. A.

Let

PAGE 1407
www.byjusexamprep.com

Using C and D

Squaring both sides

Using C and D

88. Ans. A.

On comparing it with F(x)= p x q

89. Ans. B.
and

And

PAGE 1408
www.byjusexamprep.com

Using 1 and 2, we get the following equation

If x = 2, then y=3 and so sum = 5

If x = , then y = and so sum =

Hence sum is 5
90. Ans. A.
Considering all four options, we can take m=-3 and n=6 so that they satisfy all given conditions
divided by gives R = 1458

A. R is a non-zero even integer - Yes


B. R is odd if m is odd - No
C. for some integer s, if n is even - No

D. for some integer t, if 3 divides n – No


Hence answer is option (a)
91. Ans. B.

As and are simple factors of hence and

and

As and

hence

PAGE 1409
www.byjusexamprep.com

92. Ans. C.
As per question

Putting x = 1 and x = –1 in given eq. one by one

Adding 1 and 2

Putting them in question

93. Ans. B.

PAGE 1410
www.byjusexamprep.com

Adding above 4 equations


We get

Dividing the above eq by , we get

94. Ans. B.
As per question

Adding above 2 equations

Factoring a and b, we get

and

From equation, we get HCF as (x-1)


95. Ans. A.

PAGE 1411
www.byjusexamprep.com

Putting it in given equation

96. Ans. D.

&
Let the common root be a

Equating above 2 equations, we get

put this value in equation one


then

97. Ans. C.
,

Multiplying above three with a, b and c respectively

Putting values from eq 1

PAGE 1412
www.byjusexamprep.com

98. Ans. C.
1 1
Let us take = u and = v then the given equations reduce to
x y

pu + qv = m……(eqn. 1)
qu + pv = n……..(eqn. 2)
Multiply (eqn1) by n & (eqn. 2) by (-m), and adding the obtained eqns.
pu × n + qv × n = mn & qu × (–m) + pv × (–m) = (–m)n
On adding we get (pn – qm)u + (qn – pm)v = 0
v (qm − pn)
=
u (qn − pm)

1 1
Replacing u by and v by we get,
x y

1 1
 /  =
y x

99. Ans. A.
1. √(3x2-7x-30) - √( 2x2-7x-5)=x-5
Let √(3x2-7x-30) = a & √(2x2-7x-5) =b
a-b = x-5
a2 – b2 = (√ (3x2-7x-30)) 2 – (√ (2x2-7x-5)) 2
= (3x2-7x-30) – (2x2-7x-5)
=x2 – 25
= (x-5) (x+5)
Also a2 - b2 = (a-b)(a+b)
(x-5)(x+5) = (x-5)(a+b)
x+5 = a + b
x + 5 = √(3x2-7x-30) + √(2x2-7x-5)
On comparing the above equations, we can take x = √(3x2-7x-30) and 5=√(2x2-7x-5)
x = √(3x2-7x-30)
Squaring both sides
x2 = 3x2-7x-30

PAGE 1413
www.byjusexamprep.com

0 = 2x2 – 12x + 5x -30


0 = (x-6) (2x+5)

x = 6, –

Product of roots = 6 × –

= – 15
100. Ans. A.
1. As roots are in ratio p:q, let roots be p and q

Sum of roots =

And product of roots = p × q =

= = ……………………………………………….. 1

Using equation 1 in question

=0

PAGE 1414
www.byjusexamprep.com

PAGE 1332

You might also like